You are on page 1of 657

Chapterwise Topicwise

Solved Papers
2021-1979

IITJEE
JEE Main & Advanced

Mathematics
Amit M Agarwal

Arihant Prakashan (Series), Meerut


Arihant Prakashan (Series), Meerut
All Rights Reserved

© Author

Administrative & Production Offices


Regd. Office
‘Ramchhaya’ 4577/15, Agarwal Road, Darya Ganj, New Delhi -110002
Tele: 011- 47630600, 43518550

Head Office
Kalindi, TP Nagar, Meerut (UP) - 250002, Tel: 0121-7156203, 7156204

Sales & Support Offices


Agra, Ahmedabad, Bengaluru, Bareilly, Chennai, Delhi, Guwahati,
Hyderabad, Jaipur, Jhansi, Kolkata, Lucknow, Nagpur & Pune.

ISBN 978-93-25796-13-3
PO No : TXT-XX-XXXXXXX-X-XX
Published by Arihant Publications (India) Ltd.
For further information about the books published by Arihant, log on to
www.arihantbooks.com or e-mail at info@arihantbooks.com
Follow us on
CONTENTS
1. Complex Numbers 1-28

2. Theory of Equations 29-50

3. Sequences and Series 51-75

4. Permutations and Combinations 76-88

5. Binomial Theorem 89-102

6. Probability 103-134

7. Matrices and Determinants 135-170

8. Functions 171-187

9. Limit, Continuity and Differentiability 188-238

10. Application of Derivatives 239-277

11. Indefinite Integration 278-293

12. Definite Integration 294-328

13. Area 329-353

14. Differential Equations 354-378

15. Straight Line and Pair of Straight Lines 379-404


16. Circle 405-439

17. Parabola 440-457

18. Ellipse 458-473

19. Hyperbola 474-485

20. Trigonometrical Ratios and Identities 486-498

21. Trigonometrical Equations 499-514

22. Inverse Circular Functions 515-525

23. Properties of Triangles 526-547

24. Vectors 548-579

25. 3D Geometry 580-605

26. Miscellaneous 606-632

JEE Advanced Solved Paper 2021 1-16


SYLLABUS
JEE MAIN
UNIT I Sets, Relations and Functions UNIT V Mathematical Induction
Sets and their representation, Union, intersection and Principle of Mathematical Induction and its simple
complement of sets and their algebraic properties, Power applications.
set, Relation, Types of relations, equivalence relations,
functions, one-one, into and onto functions, composition UNIT VI Binomial Theorem and its Simple
of functions. Applications
Binomial theorem for a positive integral index, general
UNIT II Complex Numbers and term and middle term, properties of Binomial coefficients
Quadratic Equations and simple applications.
Complex numbers as ordered pairs of reals,
Representation of complex numbers in the form a+ib and UNIT VII Sequences and Series
their representation in a plane, Argand diagram, algebra Arithmetic and Geometric progressions, insertion of
of complex numbers, modulus and argument (or arithmetic, geometric means between two given
amplitude) of a complex number, square root of a numbers. Relation between AM and GM Sum upto n
complex number, triangle inequality, Quadratic terms of special series: ∑ n, ∑ n2, ∑n3. Arithmetico -
equations in real and complex number system and their Geometric progression.
solutions. Relation between roots and co-efficients,
nature of roots, formation of quadratic equations with
UNIT VIII Limit, Continuity and Differentiability
given roots. Real valued functions, algebra of functions, polynomials,
rational, trigonometric, logarithmic and exponential
UNIT III Matrices and Determinants functions, inverse functions. Graphs of simple functions.
Matrices, algebra of matrices, types of matrices, Limits, continuity and differenti-ability. Differentiation of
determinants and matrices of order two and three. the sum, difference, product and quotient of two
Properties of determinants, evaluation of deter-minants, functions. Differentiation of trigonometric, inverse
area of triangles using determinants. Adjoint and trigonometric, logarithmic, exponential, composite and
evaluation of inverse of a square matrix using implicit functions; derivatives of order upto two. Rolle's
determinants and elementary transformations, Test of and Lagrange's Mean Value Theorems. Applications of
consistency and solution of simultaneous linear derivatives: Rate of change of quantities, monotonic -
equations in two or three variables using determinants increasing and decreasing functions, Maxima and minima
and matrices. of functions of one variable, tangents and normals.

UNIT IV Permutations and Combinations UNIT IX Integral Calculus


Fundamental principle of counting, permutation as an Integral as an anti - derivative. Fundamental integrals
arrangement and combination as selection, Meaning of involving algebraic, trigonometric, exponential and
P(n,r) and C (n,r), simple applications. logarithmic functions. Integration by substitution, by
parts and by partial fractions. Integration using circle when the end points of a diameter are given, points
trigonometric identities. Evaluation of simple integrals of of intersection of a line and a circle with the centre at the
the type origin and condition for a line to be tangent to a circle,
dx dx dx
equation of the tangent. Sections of cones, equations of
, , dx , ,
a2 – x2
conic sections (parabola, ellipse and hyperbola) in
x2 ± a2 2
x ±a 2 a – x2
2
standard forms, condition for y=mx + c to be a tangent
dx dx , (px + q) dx ,
, and point (s) of tangency.
ax 2 + bx + c ax 2 + bx + c ax 2 + bx + c

(px + q) dx
UNIT XII Three Dimensional Geometry
, 2 2 2 2 Coordinates of a point in space, distance between two
a ± x dx and x – a dx
ax 2 + bx + c
points, section formula, direction ratios and direction
cosines, angle between two intersecting lines. Skew lines,
Integral as limit of a sum. Fundamental Theorem of
the shortest distance between them and its equation.
Calculus. Properties of definite integrals. Evaluation of
Equations of a line and a plane in different forms,
definite integrals, determining areas of the regions
intersection of a line and a plane, coplanar lines.
bounded by simple curves in standard form.
UNIT XIII Vector Algebra
UNIT X Differential Equations Vectors and scalars, addition of vectors, components of a
Ordinary differential equations, their order and degree. vector in two dimensions and three dimensional space,
Formation of differential equations. Solution of scalar and vector products, scalar and vector triple
differential equations by the method of separation of product.
variables, solution of homogeneous and linear differential
equations UNIT XIV Statistics and Probability
of the type Measures of Dispersion: Calculation of mean, median,
mode of grouped and ungrouped data. Calculation of
UNIT XI Coordinate Geometry standard deviation, variance and mean deviation for
Cartesian system of rectangular coordinates in a plane, grouped and ungrouped data.
distance formula, section formula, locus and its equation,
Probability: Probability of an event, addition and
translation of axes, slope of a line, parallel and
multiplication theorems of probability, Baye's theorem,
perpendicular
dy lines,
+ p(x)yintercepts
= q(x) of a line on the coordinate
dx probability distribution of a random variate, Bernoulli
axes.
trials and Binomial distribution.
Straight lines
Various forms of equations of a line, intersection of lines, UNIT XV Trigonometry
angles between two lines, conditions for concurrence of Trigonometrical identities and equations.
three lines, distance of a point from a line, equations of Trigonometrical functions. Inverse trigonometrical
internal and external bisectors of angles between two functions and their properties. Heights and Distances.
lines, coordinates of centroid, orthocentre and
circumcentre of a triangle, equation of family of lines UNIT XVI Mathematical Reasoning
passing through the point of intersection of two lines. Statements, logical operations And, or, implies, implied
by, if and only if. Understanding of tautology,
Circles, Conic sections
contradiction, converse and contra positive.
Standard form of equation of a circle, general form of the
equation of a circle, its radius and centre, equation of a
JEE ADVANCED
Algebra
Algebra of complex numbers, addition, multiplication, conjugation, polar representation, properties of modulus and
principal argument, triangle inequality, cube roots of unity, geometric interpretations.
Quadratic equations with real coefficients, relations between roots and coefficients, formation of quadratic equations
with given roots, symmetric functions of roots.
Arithmetic, geometric and harmonic progressions, arithmetic, geometric and harmonic means, sums of finite
arithmetic and geometric progressions, infinite geometric series, sums of squares and cubes of the first n natural
numbers.

Logarithms and their Properties, Permutations and combinations, Binomial theorem for a positive integral index,
properties of binomial coefficients.

Matrices
Matrices as a rectangular array of real numbers, equality of matrices, addition, multiplication by a scalar and product
of matrices, transpose of a matrix, determinant of a square matrix of order up to three, inverse of a square matrix of
order up to three, properties of these matrix operations, diagonal, symmetric and skew-symmetric matrices and their
properties, solutions of simultaneous linear equations in two or three variables.

Probability
Addition and multiplication rules of probability, conditional probability, independence of events, computation of
probability of events using permutations and combinations.

Trigonometry
Trigonometric functions, their periodicity and graphs, addition and subtraction formulae, formulae involving
multiple and sub-multiple angles, general solution of trigonometric equations.
Relations between sides and angles of a triangle, sine rule, cosine rule, half-angle formula and the area of a triangle,
inverse trigonometric functions (principal value only).

Analytical Geometry
Two Dimensions Cartesian oordinates, distance between two points, section formulae, shift of origin.
Equation of a straight line in various forms, angle between two lines, distance of a point from a line. Lines through
the point of intersection of two given lines, equation of the bisector of the angle between two lines, concurrency of
lines, centroid, orthocentre, incentre and circumcentre of a triangle.
Equation of a circle in various forms, equations of tangent, normal and chord.
Parametric equations of a circle, intersection of a circle with a straight line or a circle, equation of a circle through the
points of intersection of two circles and those of a circle and a straight line.
Equations of a parabola, ellipse and hyperbola in standard form, their foci, directrices and eccentricity, parametric
equations, equations of tangent and normal.
Locus Problems, Three Dimensions Direction cosines and direction ratios, equation of a straight line in space,
equation of a plane, distance of a point from a plane.

Differential Calculus
Real valued functions of a real variable, into, onto and one-to-one functions, sum, difference, product and
quotient of two functions, composite functions, absolute value, polynomial, rational, trigonometric, exponential
and logarithmic functions.
Limit and continuity of a function, limit and continuity of the sum, difference, product and quotient of two
functions, l'Hospital rule of evaluation of limits of functions.
Even and odd functions, inverse of a function, continuity of composite functions, intermediate value property of
continuous functions.
Derivative of a function, derivative of the sum, difference, product and quotient of two functions, chain rule,
derivatives of polynomial, rational, trigonometric, inverse trigonometric, exponential and logarithmic functions.
Derivatives of implicit functions, derivatives up to order two, geometrical interpretation of the derivative, tangents
and normals, increasing and decreasing functions, maximum and minimum values of a function, applications of
Rolle's Theorem and Lagrange's Mean Value Theorem.

Integral Calculus
Integration as the inverse process of differentiation, indefinite integrals of standard functions, definite integrals
and their properties, application of the Fundamental Theorem of Integral Calculus.
Integration by parts, integration by the methods of substitution and partial fractions, application of definite
integrals to the determination of areas involving simple curves.
Formation of ordinary differential equations, solution of homogeneous differential equations, variables separable
method, linear first order differential equations.

Vectors
Addition of vectors, scalar multiplication, scalar products, dot and cross products, scalar triple products and their
geometrical interpretations.
1
Complex Numbers
Topic 1 Complex Number in Iota Form
Objective Questions I (Only one correct option) 2 + 3i sin θ
6. A value of θ for which is purely imaginary, is
2z − n 1 − 2i sin θ (2016 Main)
1. Let z ∈ C with Im (z ) = 10 and it satisfies = 2i − 1
 3
2z + n π π −1  1 
(a) (b) (c) sin −1   (d) sin  
for some natural number n, then (2019 Main, 12 April II) 3 6  4   3
(a) n = 20 and Re(z ) = − 10 (b) n = 40 and Re(z ) = 10 6i –3 i 1
(c) n = 40 and Re(z ) = − 10 (d) n = 20 and Re(z ) = 10 7. If 4 3i –1 = x + iy, then (1998, 2M)
α + i  20 3 i
2. All the points in the set S =  : α ∈ R (i = −1 ) lie
α − i  (a) x = 3, y = 1 (b) x = 1, y = 1 (c) x = 0, y = 3 (d) x = 0, y = 0
on a (2019 Main, 9 April I) 13
(a) circle whose radius is 2. 8. The value of sum ∑ (i n + i n + 1 ), where i = −1, equals
(b) straight line whose slope is −1. n =1
(1998, 2M)
(c) circle whose radius is 1. (a) i (b) i − 1 (c) − i (d) 0
n
(d) straight line whose slope is 1.  1 + i
5 + 3z 9. The smallest positive integer n for which   = 1, is
3. Let z ∈ C be such that|z|< 1. If ω = , then 1 − i
5(1 − z ) (a) 8 (b) 16 (1980, 2M)
(2019 Main, 9 April II) (c) 12 (d) None of these
(a) 4 Im(ω) > 5 (b) 5 Re (ω) > 1
(c) 5 Im (ω) < 1 (d) 5 Re(ω) > 4 Objective Question II
3
x + iy (One or more than one correct option)
4. Let  −2 − i =
1
(i = −1 ), where x and y are real
 3  27 10. Let a , b, x and y be real numbers such that a − b = 1 and
numbers, then y − x equals (2019 Main, 11 Jan I) y ≠ 0. If the complex number z = x + iy satisfies
(a) 91 (b) 85 (c) – 85 (d) – 91  az + b
Im   = y, then which of the following is(are)
 π  3 + 2i sin θ   z+1
5. Let A = θ ∈  − , π : is purely imaginary 
  2  1 − 2i sin θ  possible value(s) of x? (2017 Adv.)

Then, the sum of the elements in A is (2019 Main, 9 Jan I) (a) 1 − 1 + y2 (b) − 1 − 1 − y2
3π 5π 2π
(a) (b) (c) π (d) (c) 1 + 1 + y2 (d) − 1 + 1 − y2
4 6 3

Topic 2 Conjugate and Modulus of a Complex Number


Objective Questions I (Only one correct option) (1 + i )2 2
2. If a > 0 and z = , has magnitude , then z is
1. The equation|z − i| = |z − 1|, i = −1, represents a−i 5
equal to (2019 Main, 10 April I)
1 (2019 Main, 12 April I)
(a) a circle of radius 1 3 1 3
2 (a) − i (b) − − i
5 5 5 5
(b) the line passing through the origin with slope 1 1 3 3 1
(c) a circle of radius 1 (c) − + i (d) − − i
5 5 5 5
(d) the line passing through the origin with slope − 1
2 Complex Numbers

3. Let z1 and z2 be two complex numbers satisfying| z1 | = 9 12. If w = α + iβ, where β ≠ 0 and z ≠ 1, satisfies the
and | z2 − 3 − 4i | = 4. Then, the minimum value of  w − wz 
condition that   is purely real, then the set of
| z1 − z2|is (2019 Main, 12 Jan II)  1−z 
(a) 1 (b) 2 (c) 2 (d) 0 values of z is (2006, 3M)
z −α (a)| z | = 1, z ≠ 2 (b)| z | = 1 and z ≠ 1
4. If (α ∈ R) is a purely imaginary number and
z+α (c) z = z (d) None of these
|z| = 2, then a value of α is (2019 Main, 12 Jan I) z −1
13. If|z| = 1 and w = (where, z ≠ − 1), then Re (w) is
(a) 2 (b)
1
(c) 1 (d) 2 z+1 (2003, 1M)
2 1  1  1 2
(a) 0 (b) (c) ⋅ (d)
5. Let z be a complex number such that | z | + z = 3 + i |z + 1|2
z +  |z + 1|
1 2
|z + 1|2
(where i = − 1).
14. For all complex numbers z1 , z2 satisfying |z1| = 12 and
Then,| z |is equal to (2019 Main, 11 Jan II)
|z2 − 3 − 4i| = 5, the minimum value of|z1 − z2|is
34 5 41 5
(a) (b) (c) (d) (a) 0 (b) 2 (2002, 1M)
3 3 4 4
(c) 7 (d) 17
6. A complex number z is said to be unimodular, if z ≠ 1. 15. If z1 , z2 and z3 are complex numbers such that
z – 2 z2
If z1 and z2 are complex numbers such that 1 is 1 1 1
2 – z1z2 |z1| = |z2| = |z3| =  + + = 1, then |z1 + z2 + z3|is
z1 z2 z3
unimodular and z2 is not unimodular.
(a) equal to 1 (b) less than 1 (2000, 2M)
Then, the point z 1 lies on a (2015 Main)
(c) greater than 3 (d) equal to 3
(a) straight line parallel to X-axis
16. For positive integers n1 , n2 the value of expression
(b) straight line parallel toY -axis
(1 + i )n 1 + (1 + i3 )n1 + (1 + i5 )n 2 + (1 + i7 )n 2 , here
(c) circle of radius 2
i = −1 is a real number, if and only if (1996, 2M)
(d) circle of radius 2
(a) n1 = n2 + 1 (b) n1 = n2 − 1
7. If z is a complex number such that |z| ≥ 2, then the (c) n1 = n2 (d) n1 > 0, n2 > 0
1 17. The sin x + i cos 2x
minimum value of z + complex numbers and
2 (2014 Main) cos x − i sin 2x are conjugate to each other, for
(a) is equal to 5/2 (a) x = nπ (b) x = 0 (1988, 2M)
(b) lies in the interval (1, 2) (c) x = (n + 1/2) π (d) no value of x
(c) is strictly greater than 5/2 18. The points z1 , z2, z3 and z4 in the complex plane are the
(d) is strictly greater than 3/2 but less than 5/2 vertices of a parallelogram taken in order, if and only if
8. Let complex numbers α and 1 /α lies on circles (a) z1 + z4 = z2 + z3 (b) z1 + z3 = z2 + z4 (1983, 1M)
(x − x0 )2 + ( y − y0 )2 = r 2 and (x − x0 )2 + ( y − y0 )2 = 4r 2, (c) z1 + z2 = z3 + z4 (d) None of these
respectively. 19. If z = x + iy and w = (1 − iz ) / (z − i ), then |w| = 1 implies
If z0 = x0 + iy0 satisfies the equation 2|z0|2 = r 2 + 2, then
that, in the complex plane (1983, 1M)
|α |is equal to (2013 Adv.)
1 1 1 1 (a) z lies on the imaginary axis (b) z lies on the real axis
(a) (b) (c) (d) (c) z lies on the unit circle (d) None of these
2 2 7 3
9. Let z be a complex number such that the imaginary part 20. The inequality |z − 4| < |z − 2| represents the region
of z is non-zero and a = z + z + 1 is real. Then, a cannot
2 given by (1982, 2M)
take the value (2012) (a) Re (z ) ≥ 0 (b) Re (z ) < 0
1 1 3 (c) Re (z ) > 0 (d) None of these
(a) − 1 (b) (c) (d)
5 5
3 2 4  3 i  3 i
10. Let z = x + iy be a complex number where, x and y are 21. If z =  +  + −  , then
 2 2   2 2 (1982, 2M)
integers. Then, the area of the rectangle whose vertices
are the root of the equation zz3 + zz3 = 350, is (2009)
(a) Re (z ) = 0 (b) Im (z ) = 0
(c) Re (z ) > 0, Im (z ) > 0 (d) Re (z ) > 0, Im (z ) < 0
(a) 48 (b) 32
(c) 40 (d) 80 22. The complex numbers z = x + iy which satisfy the
z z − 5i 
11. If|z|= 1 and z ≠ ± 1, then all the values of lie on equation  = 1, lie on
1 − z2 z + 5i  (1981, 2M)
(a) a line not passing through the origin (2007, 3M) (a) the X-axis
(b)|z|= 2 (b) the straight line y = 5
(c) the X-axis (c) a circle passing through the origin
(d) the Y-axis (d) None of the above
Complex Numbers 3

Objective Questions II 29. Let z be any point in A ∩ B ∩ C and let w be any point
(One or more than one correct option) satisfying |w − 2 − i| < 3. Then, |z | − |w| + 3 lies
between
23. Let S be the set of all complex numbers z satisfying (a) − 6 and 3 (b) − 3 and 6
| z 2 + z + 1| = 1. Then which of the following statements (c) − 6 and 6 (d) − 3 and 9
is/are TRUE? (2020 Adv.)
1 1 Passage II
(a) z + ≤ for all z ∈S (b)|z|≤ 2 for all z ∈S
2 2 Let S = S1 ∩ S 2 ∩ S3 , where
1 1  z − 1 + 3 i  
(c) z + ≥ for all z ∈S S1 = { z ∈ C :|z | < 4}, S 2 = z ∈ C : lm   > 0
2 2   1− 3i  
(d) The set S has exactly four elements and S3 : { z ∈ C : Re z > 0} (2008)
24. Let s, t, r be non-zero complex numbers and L be the set of 30. Let z be any point in A ∩ B ∩ C.
solutions z = x + iy (x, y ∈ R, i = − 1 ) of the equation The|z + 1 − i|2 + |z − 5 − i|2 lies between
sz + tz + r = 0, where z = x − iy. Then, which of the
(a) 25 and 29 (b) 30 and 34
following statement(s) is (are) TRUE? (2018 Adv.)
(c) 35 and 39 (d) 40 and 44
(a) If L has exactly one element, then| s|≠ |t |
31. The number of elements in the set A ∩ B ∩ C is
(b) If|s|=|t |, then L has infinitely many elements
(a) 0 (b) 1
(c) The number of elements in L ∩ {z :| z − 1 + i| = 5} is at most 2
(c) 2 (d) ∞
(d) If L has more than one element, then L has infinitely many
elements Match the Columns
25. Let z1 and z2 be complex numbers such that z1 ≠ z2 and 32. Match the statements of Column I with those of
|z1| = |z2|. If z1 has positive real part and z2 has negative Column II.
z + z2
imaginary part, then 1 may be (1986, 2M) Here, z takes values in the complex plane and Im (z )
z1 − z2 and Re (z ) denote respectively, the imaginary part and
(a) zero (b) real and positive the real part of z (2010)
(c) real and negative (d) purely imaginary
Column I Column II
26. If z1 = a + ib and z2 = c + id are complex numbers such
A. The set of points z satisfying p. an ellipse with
that |z1| = |z2| = 1 and Re (z1z2) = 0, then the pair of
| z − i| z|| = | z + i | z|| is eccentricity 4/5
complex numbers w1 = a + ic and w2 = b + id satisfies
contained in or equal to
(a)|w1| = 1 (b)|w2| = 1 (1985, 2M)
B. The set of points z satisfying q. the set of points z
(c) Re (w1 w2 ) = 0 (d) None of these
| z + 4| + | z − 4| = 0 is satisfying Im ( z) = 0
contained in or equal to
Passage Based Problems
C. If| w| = 2 , then the set of r. the set of points z
Read the following passages and answer the questions 1
points z = w − is contained satisfying|Im( z) |≤ 1
that follow. w
Passage I in or equal to

Let A, B, C be three sets of complex number as defined D. If| w| = 1, then the set of points s. the set of points
below
1
z = w + is contained in or satisfying|Re( z)|≤ 2
t. the set of points z
A = { z : lm (z ) ≥ 1} w
equal to satisfying| z| ≤ 3
B = { z :|z − 2 − i| = 3}
C = { z : Re((1 − i )z ) = 2 } (2008, 12M)
Fill in the Blanks
27. min|1 − 3i − z|is equal to
z ∈s 33. If α , β, γ are the cube roots of p, p < 0, then for any x, y
2− 3 2+ 3 3− 3 3+ 3 xα + yβ + zγ
(a) (b) (c) (d) and z then = ... .
2 2 2 2 xβ + yγ + zα (1990, 2M)
28. Area of S is equal to
10 π 20 π 16 π 32 π 34. For any two complex numbers z1 , z2 and any real
(a) (b) (c) (d)
3 3 3 3 numbers a and b,|az1 − bz2|2+ |bz1 + az2|2 = K .
(1988, 2M)
4 Complex Numbers

  x  x  41. If z1 and z2 are two complex numbers such that


sin  2 + cos  2 − i tan (x) 1 − z1z2
  is real,
35. If the expression |z1| < 1 < |z2|, then prove that < 1. (2003, 2M)
  x  z1 − z2
1 + 2 i sin  2 
 
42. For complex numbers z and w, prove that
then the set of all possible values of x is… . | z |2 w − |w|2 z = z − w, if and only if z = w or z w = 1.
(1987, 2M) (1999, 10M)

True/False 43. Find all non-zero complex numbers z satisfying


z = iz 2. (1996, 2M)
36. If three complex numbers are in AP. Then, they lie on a 44. If iz + z − z + i = 0,
3 2
then show that |z| = 1.
circle in the complex plane (1985 M)
(1995, 5M)
37. If the complex numbers, z1 , z2 and z3 represent the
45. A relation R on the set of complex numbers is defined
vertices of an equilateral triangle such that z1 − z2
| z1 | = | z2| = | z3 |, then z1 + z2 + z3 = 0. (1984, 1M) by z1 R z2, if and only if is real.
z1 + z2
38. For complex numbers z1 = x1 + iy1 and z2 = x2 + iy2, we Show that R is an equivalence relation. (1982, 2M)
write z1 ∩ z2, if x1 ≤ x2 and y1 ≤ y2. Then, for all complex 46. Find the real values of x and y for which the following
1−z equation is satisfied
numbers z with 1 ∩ z, we have ∩ 0. (1981, 2M)
1+ z (1 + i ) x − 2i (2 − 3i ) y + i
+ = i. ( 1980, 2M)
3+ i 3−i
Analytical & Descriptive Questions
1
39. Find the centre and radius of the circle formed by all the 47. Express in the form A + iB.
points represented by z = x + iy satisfying the relation (1 − cos θ ) + 2i sin θ (1979, 3M)
z − α 
  a + ib a 2 + b2
= k (k ≠ 1), where α and β are the constant 48. If x + iy = , prove that (x2 + y2)2 = 2
z −β c + id c + d2
complex numbers given by α = α 1 + iα 2, β = β1 + iβ 2. (1978, 2M)
(2004, 2M) Integer & Numerical Answer Type Question
40. Prove that there exists no complex number z such that 49. If z is any complex number satisfying | z − 3 − 2i | ≤ 2,
n
|z| < 1 /3 and ∑ a rz r = 1, where|a r|< 2. then the maximum value of|2z − 6 + 5i |is …… (2011)
r =1 (2003, 2M)

Topic 3 Argument of a Complex Number


Objective Questions I (Only one correct option) 4. If arg (z ) < 0 , then arg (−z ) − arg (z ) equals (2000, 2M)
1. If z1 , z2 are complex numbers such that Re(z1 ) = |z1 − 1|, (a) π (b) − π
π (c) − π/2 (d) π /2
Re(z2) = |z2 − 1| and arg(z1 − z2) = , then Im(z1 + z2) is
6 5. Let z and w be two complex numbers such that|z| ≤ 1,
equal to (2020 Main, 3 Sep II) |w| ≤ 1 and|z + i w|= |z − iw| = 2 , then z equals
3 1 2 (1995, 2M)
(a) (b) (c) (d) 2 3
2 3 3 (a) 1 or i (b) i or −i
(c) 1 or −1 (d) i or −1
2. Let z1 and z2 be any two non-zero complex numbers such
3z1 2z2
6. Let z and w be two non-zero complex numbers
that 3|z1| = 4|z2|. If z = + , then such that |z| = |w| and arg (z ) + arg (w) = π, then z
2z2 3z1 (2019 Main, 10 Jan I) equals (1995, 2M)
1 17 (a) w (b) −w
(a) |z| = (b) Im(z ) = 0
2 2 (c) w (d) −w
5 7. If z1 and z2 are two non-zero complex numbers such
(c) Re(z) = 0 (d) |z| =
2 that|z1 + z2| = |z1| + |z2|, then arg (z1 ) − arg (z2) is equal
3. If z is a complex number of unit modulus and argument to
 1 + z (1987, 2M)
θ, then arg   is equal to (2013 Main) π
1 + z (a) − π (b) −
2
π π
(a) − θ (b) −θ (c) θ (d) π − θ (c) 0 (d)
2 2
Complex Numbers 5

8. If a , b, c and u , v, w are the complex numbers 10. Let z1 and z2 be two distinct complex numbers and let
representing the vertices of two triangles such that z = (1 − t ) z1 + tz2 for some real number t with 0 < t < 1. If
c = (1 − r ) a + rb and w = (1 − r ) u + rv, where r is a arg (w) denotes the principal argument of a non-zero
complex number, then the two triangles (1985, 2M) complex number w, then (2010)
(a) have the same area (b) are similar (a) |z − z1| + |z − z2|= |z1 − z2|(b) arg (z − z1 ) = arg (z − z2 )
(c) are congruent (d) None of these z − z1 z − z1
(c) =0 (d) arg (z − z1 ) = arg (z2 − z1 )
z2 − z1 z2 − z1
Objective Questions II
(One or more than one correct option) Match the Columns
9. For a non-zero complex number z, let arg(z ) denote the 11. Match the conditions/expressions in Column I with
principal argument with − π < arg(z ) ≤ π . Then, which of statement in Column II (z ≠ 0 is a complex number)
the following statement(s) is (are) FALSE ? (2018 Adv.)
Column I Column II
π
(a) arg (−1 − i ) = , where i = −1
4 A. Re ( z) = 0 p. Re ( z2 ) = 0
(b) The function f : R → (− π, π], defined by B. π q. Im ( z2 ) = 0
arg ( z) =
f (t ) = arg (−1 + it ) for all t ∈ R, is continuous at all 4
points of R, where i = −1. r. Re ( z2 ) = Im ( z2 )
(c) For any two non-zero complex numbers z1 and z2,
z  Analytical & Descriptive Questions
arg  1  − arg (z1 ) + arg (z2 ) is an integer multiple of
 z2  12. |z| ≤ 1,|w|≤ 1, then show that
2π. |z − w|2 ≤ (|z| − |w|)2 + (arg z − arg w)2 (1995, 5M)
(d) For any three given distinct complex numbers z1 , z2 and
13. Let z1 = 10 + 6i and z2 = 4 + 6i. If z is any complex
z3 , the locus of the point z satisfying the condition
 (z − z1 ) (z2 − z3 )  number such that the argument of (z − z1 ) / (z − z2) is
arg   = π, lies on a straight line. π /4, then prove that|z − 7 − 9i| = 3 2. (1991, 4M)
 (z − z3 ) (z2 − z1 ) 

Topic 4 Rotation of a Complex Number


Objective Questions I (Only one correct option) 4. A man walks a distance of 3 units from the origin
towards the North-East (N 45° E) direction. From there,
1. Let S be the set of all complex numbers z satisfying
he walks a distance of 4 units towards the North-West
| z − 2 + i | ≥ 5. If the complex number z0 is such that
 1  (N 45° W) direction to reach a point P. Then, the
1
is the maximum of the set  : z ∈ S , then position of P in the Argand plane is (2007, 3M)
| z0 − 1 | | z − 1 |  (a) 3ei π/ 4 + 4 i
4 − z0 − z0 (b) (3 − 4 i ) ei π / 4
the principal argument of is (2019 Adv.)
z0 − z0 + 2 i (c) (4 + 3 i )ei π / 4
π 3π π π (d) (3 + 4 i ) ei π / 4
(a) (b) (c) − (d)
4 4 2 2 5. The shaded region, where P = (−1, 0), Q = (−1 + 2 , 2 )
5
 3 i  3 i
5
R = (−1 + 2 , − 2 ), S = (1, 0) is represented by (2005, 1M)
2. Let z =  +  + −  . If R(z ) and I (z )
 2 2  2 2 π Y
(a)|z + 1|> 2,| arg (z + 1)|<
respectively denote the real and imaginary parts of z, 4 Q
π
then (2019 Main, 10 Jan II) (b)|z + 1|< 2,| arg (z + 1)|<
2
(a) R (z ) > 0 and I (z ) > 0 (b) I (z ) = 0 π X′
P O S
X
(c) R (z ) < 0 and I (z ) > 0 (d) R (z ) = − 3 (c)|z + 1|> 2,| arg (z + 1)|>
4
3. A particle P starts from the point z0 = 1 + 2 i, where π R
(d)|z − 1|< 2,| arg (z + 1)|>
i = −1. It moves first horizontally away from origin by 2 Y′
π
5 units and then vertically away from origin by 3 units 6. If 0 < α < is a fixed angle. If P = (cos θ ,sin θ ) and
to reach a point z1. From z1 the particle moves 2 units 2
Q = {cos(α − θ ),sin(α − θ )}, then Q is obtained from P by
in the direction of the vector $i + $j and then it moves (2002, 2M)
π
through an angle in anti-clockwise direction on a (a) clockwise rotation around origin through an angle α
2
(b) anti-clockwise rotation around origin through an angleα
circle with centre at origin, to reach a point z2. The point
(c) reflection in the line through origin with slope tan α
z2 is given by (2008, 3M)
α
(a) 6 + 7i (b) −7 + 6i (c) 7 + 6i (d) − 6 + 7i (d) reflection in the line through origin with slope tan
2
6 Complex Numbers

7. The complex numbers z1 , z2 and z3 satisfying Analytical & Descriptive Questions


z1 − z3 1 − i 3
= are the vertices of a triangle which is 13. If one of the vertices of the square circumscribing the
z2 − z3 2 (2001, 1M) circle |z − 1| = 2 is 2 + 3i. Find the other vertices of
(a) of area zero (b) right angled isosceles square. (2005, 4M)
(c) equilateral (d) obtuse angled isosceles
14. Let bz + bz = c, b ≠ 0, be a line in the complex plane,
where b is the complex conjugate of b. If a point z1 is the
Objective Questions II reflection of the point z2 through the line, then show
(One or more than one correct option) that c = z1b + z2b. (1997C, 5M)
8. Let a , b ∈ R and a 2 + b2 ≠ 0. 15. Let z1 and z2 be the roots of the equation z + pz + q = 0,
2
 1 
Suppose S = z ∈ C : z = , t ∈ R, t ≠ 0, where where the coefficients p and q may be complex numbers.
 a + i bt  Let A and B represent z1 and z2 in the complex plane. If
i = − 1. If z = x + iy and z ∈ S, then (x, y) lies on ∠ AOB = α ≠ 0 and OA = OB, where O is the origin prove
(2016 Adv.) α
that p2 = 4q cos 2  .
 2
and centre 
1 1  (1997, 5M)
(a) the circle with radius , 0 for
2a  2a 
16. Complex numbers z1 , z2, z3 are the vertices A , B, C
a > 0, b ≠ 0
respectively of an isosceles right angled triangle with
and centre  −
1 1 
(b) the circle with radius − , 0 for right angle at C. Show that
2a  2a 
(z1 − z2)2 = 2(z1 − z3 ) (z3 − z2). (1986, 2 1 M)
a < 0, b ≠ 0 2

(c) the X-axis for a ≠ 0, b = 0 17. Show that the area of the triangle on the argand
(d) the Y-axis for a = 0, b ≠ 0 diagram formed by the complex number z , iz and z + iz
1
3+i is |z|2.
9. Let W = and P = {W n: n = 1, 2, 3,... }. 2 (1986, 2 1 M)
2
2
 1 18. Prove that the complex numbers z1 , z2 and the origin
Further H 1 = z ∈ C : Re (z ) > 
 2 form an equilateral triangle only if z12 + z22 − z1z2 = 0.
 − 1  (1983, 2M)
and H 2 = z ∈ C : Re (z ) < , where C is the set of all
 2  19. Let the complex numbers z1 , z2 and z3 be the vertices of
complex numbers. If z1 ∈ P ∩ H 1, z2 ∈ P ∩ H 2 and O an equilateral triangle. Let z0 be the circumcentre of the
represents the origin, then ∠ z1Oz2 is equal to triangle. Then, prove that z12 + z22 + z32 = 3z02. (1981, 4M)
(2013 JEE Adv.)
π π 2π 5π
(a) (b) (c) (d)
2 6 3 6 Integer & Numerical Answer Type Questions
20. For a complex number z, let Re(z ) denote the real part of
Fill in the Blanks
z. Let S be the set of all complex numbers z satisfying
10. Suppose z1 , z2, z3 are the vertices of an equilateral
z 4 − |z|4 = 4 i z 2, where i = −1. Then the minimum
triangle inscribed in the circle |z| = 2. If z1 = 1 + i 3, possible value of|z1 − z2|2, where z1 , z2 ∈ S with Re(z1 ) > 0
then z2 = K, z3 = … . (1994, 2M) and Re(z2) < 0 is …… . (2020 Adv.)
11. ABCD is a rhombus. Its diagonals AC and BD intersect kπ   kπ 
at the point M and satisfy BD = 2 AC. If the points D and 21. For any integer k, let α k = cos  + i sin   , where
 7  7
M represent the complex numbers 1 + i and 2 − i
i = −1. The value of the expression
respectively, then A represents the complex number 12
…or… (1993, 2M)
∑|α k + 1 − α k|
12. If a and b are real numbers between 0 and 1 such that k =1
is
the points z1 = a + i , z2 = 1 + bi and z3 = 0 form an 3

equilateral triangle, then a = K and b = K . (1990, 2M) ∑|α 4k − 1 − α 4k − 2|


k =1 (2016 Adv.)
Complex Numbers 7

Topic 5 De-Moivre’s Theorem, Cube Roots and nth Roots of Unity


Objective Questions I (Only one correct option) 9. Let z1 and z2 be nth roots of unity which subtend a right
 2π 2π 
3 angled at the origin, then n must be of the form
 1 + sin + i cos  (where, k is an integer) (2001, 1M)
1. The value of  9 9  is
(a) 4k + 1 (b) 4k + 2 (c) 4k + 3
 1 + sin 2π − i cos 2π 
(d) 4k
 9 9   1 i 3
334
 1 i 3
365

10. If i = −1, then 4 + 5  − +  + 3 − + 


[2020 Main, 2 Sep I]  2 2   2 2 
1 1
(a) − ( 3 − i ) (b) − (1 − i 3 ) is equal to (1999, 2M)
2 2 (a) 1 − i 3 (b) −1 + i 3 (c) i 3 (d) −i 3
1 1
(c) ( 3 − i ) (d) (1 − i 3 )
2 2 11. If ω is an imaginary cube root of unity, then
(1 + ω − ω 2)7 is equal to (1998, 2M)
2. If z and w are two complex numbers such that | zw| = 1
(a) 128 ω (b) −128 ω (c) 128 ω2 (d) −128 ω2
π
and arg(z ) − arg(w) = , then (2019 Main, 10 April II) 12. If ω (≠ 1) is a cube root of unity and (1 + ω )7 = A + Bω,
2
1− i then A and B are respectively
(a) zw = − i (b) zw = (1995, 2M)
2
− 1+ i (a) 0, 1 (b) 1, 1 (c) 1, 0 (d) −1, 1
(c) zw = i (d) zw = 6
2  2 πk 2 πk 
3 i
13. The value of ∑ sin 7
– i cos
7 
 is (1998, 2M)
k =1
3. If z = + (i = −1 ), then (1 + iz + z5 + iz 8 )9 is equal
2 2 (a) – 1 (b) 0 (c) – i (d) i
to (2019 Main, 8 April II)
(a) 1 (b) (−1 + 2i ) 9
(c) −1 (d) 0
Match the Columns
4. Let z0 be a root of the quadratic equation, x2 + x + 1 = 0, If
2 kπ   2 kπ 
z = 3 + 6iz081 − 3iz093 , then arg z is equal to 14. Let zk = cos   + i sin   ; k = 1, 2, …9.
 10   10 
(2019 Main, 9 Jan II)
π π π Column I Column II
(a) (b) (c) 0 (d)
4 6 3 P. For each zk, there exists a z j such that (i) True
15
zk ⋅ z j = 1
5. Let z = cos θ + i sin θ . Then, the value of ∑ Im (z 2m − 1
) at
Q. There exists a k ∈ { 1, 2, … , 9 } such that (ii) False
m =1
z1 ⋅ z = zk has no solution z in the set of
θ = 2° is (2009)
complex numbers
1 1
(a) (b)
sin 2° 3 sin 2° R. |1 − z1||1 − z2| … |1 − z9| (iii) 1
equal
1 1 10
(c) (d) 2 kπ 
1 − ∑ cos 
9
2 sin 2° 4 sin 2° S. (iv) 2
 equals
k =1  10 
6. The minimum value of |a + bω + cω |, where a, b and c2
(2011)
are all not equal integers and ω (≠ 1) is a cube root of Codes
unity, is P Q R S
(2005, 1M)
(a) (i) (ii) (iv) (iii)
1
(a) 3 (b) (c) 1 (d) 0 (b) (ii) (i) (iii) (iv)
2 (c) (i) (ii) (iii) (iv)
7. If ω (≠ 1) be a cube root of unity and (1 + ω 2)n = (1 + ω 4 )n, (d) (ii) (i) (iv) (iii)
then the least positive value of n is (2004, 1M)
(a) 2 (b) 3 (c) 5 (d) 6 Fill in the Blanks
2π 2π
8. Let ω = −
1
+i
3
, then value of the determinant 15. Let ω be the complex number cos + i sin . Then
2 2 3 3
1 1 1 the number of distinct complex number z satisfying
1 −1 − ω 2 ω 2 is (2002, 1M)
z+1 ω ω2
1 ω2 ω ω z+ω 2
1 = 0 is equal to ... .
(a) 3 ω (b) 3 ω (ω − 1) (c) 3 ω2 (d) 3 ω (1 − ω) ω2 1 z+ω
(2010)
8 Complex Numbers

16. The value of the expression 19. If 1, a1 , a 2, ... , a n − 1 are the n roots of unity, then show
1 ( 2 − ω ) (2 − ω 2) + 2(3 − ω ) (3 − ω 2) + ... that (1 − a1 ) (1 − a 2) (1 − a3 ) K (1 − a n − 1 ) = n (1984, 2M)
+ (n − 1) ⋅ (n − ω ) (n − ω 2) ,
where, ω is an imaginary cube root of unity, is….
20. It is given that n is an odd integer greater than 3, but n
is not a multiple of 3. Prove that x3 + x2 + x is a factor of
(1996, 2M)
(x + 1)n − xn − 1 . (1980, 3M)
True/False 21. If x = a + b, y = aα + bβ, z = aβ + bα , where α , β are
17. The cube roots of unity when represented on Argand complex cube roots of unity, then show that
diagram form the vertices of an equilateral triangle. xyz = a3 + b3 . (1979, 3M)
(1988, 1M)
Integer & Numerical Answer Type Questions
Analytical & Descriptive Questions 22. Let ω ≠ 1 be a cube root of unity. Then the minimum of
18. Let a complex number α , α ≠ 1, be a root of the equation the set {| a + bω + cω 2|2 : a , b, c distinct non-zero
z p + q − z p − zq + 1 = 0 integers} equals ............ (2019 Adv.)

where, p and q are distinct primes. Show that either 23. Let ω = eiπ /3 and a , b, c, x, y, z be non-zero complex
1 + α + α 2 + ... + α p − 1 = 0 numbers such that a + b + c = x, a + bω + cω 2 = y,
or 1 + α + α 2 + ... + α q − 1 = 0 a + bω 2 + cω = z.
but not both together. (2002, 5M) | x|2 + | y|2 + | z |2
Then, the value of is …… (2011)
| a |2 + | b|2 + | c|2

Answers
Topic 1 1 cot (θ / 2 )
47. A + iB = −i 49. 5
1. (c) 2. (c) 3. (b) 4. (a)  θ  1 + 3 cos2 (θ / 2 )
2 1 + 3 cos2 
5. (d) 6. (d) 7. (d) 8. (b)  2
9. (d) 10. (b, d) Topic 3
Topic 2 1. (d) 2. (*) 3. (c) 4. (a)
1. (b) 2. (b) 3. (d) 4. (d) 5. (c) 6. (d) 7. (c) 8. (b)
5. (b) 6. (c) 7. (b) 8. (c) 9. (a, b, d) 10. (a, c, d) 11. A → q ; B → p
9. (d) 10. (a) 11. (d) 12. (b) Topic 4
13. (a) 14. (b) 15. (a) 16. (d) 1. (c) 2. (b) 3. (d) 4. (d)
17. (d) 18. (b) 19. (b) 20. (d) 5. (a) 6. (d) 7. (c) 8. (a,c,d)
21. (b) 22. (a) 23. (b,c) 24. (a, c, d) 9. (c, d) 10. z 2 = − 2, z 3 = 1 − i 3
25. (a,d) 26. (a, b, c) 27. (c) 28. (b) i 3i
11. 3 − or 1 − 12. a = b = 2 ± 3
29. (d) 30. (c) 31. (b) 2 2
32. A → q, r ; B → p; C → p, s, t ; D → q, r, s, t 13. z 2 = − 3 i , z 3 = (1 − 3 ) + i and z 4 = (1 + 3 ) − i
33. ω 2
34. (a + b )(| z1| + | z 2| )
2 2 2 2
20. (8) 21. (4)
−1
35. x = 2nπ + 2α , α = tan k, where k ∈(1, 2 ) or x = 2nπ
Topic 5
36. False 37. True 1. (a) 2. (a) 3. (c) 3. (a)
38. True 5. (d) 6. (c) 7. (b) 8. (b)
α − k 2β k (α − β )
39. Centre = , Radius =   9. (d) 10. (c) 11. (d) 12. (b)
1 −k 2
 1 −k 
2
13. (d) 14. (c) 15. (1)
 3 i  n (n + 1 )
2
43. z = i , ± –  16.   −n 17. True 22. (3)
 2 2  2 
46. (x = 3 and y = −1) 23. (3)
Hints & Solutions
Topic 1 Complex Number in Iota Form x + iy  1   –1
3

3
4. We have, =  − 2 − i = (6 + i )
1. Let z = x + 10i, as Im (z ) = 10 (given). 27  3   3 
Since z satisfies, x + iy 1
⇒ =− (216 + 108i + 18i 2 + i3 )
2z − n 27 27
= 2i − 1, n ∈ N ,
2z + n =−
1
(198 + 107i )
∴ (2x + 20i − n ) = (2i − 1) (2x + 20i + n ) 27
⇒ (2x − n ) + 20i = (− 2x − n − 40) + (4x + 2n − 20)i [Q (a + b)3 = a3 + b3 + 3a 2b + 3ab2, i 2 = − 1, i3 = − i]
On comparing real and imaginary parts, we get On equating real and imaginary part, we get
2x − n = − 2x − n − 40 and 20 = 4x + 2n − 20 x = − 198 and y = − 107
⇒ 4x = − 40 and 4x + 2n = 40 ⇒ y − x = − 107 + 198 = 91
⇒ x = − 10 and − 40 + 2n = 40 ⇒ n = 40  3 + 2i sin θ   1 + 2 i sin θ 
5. Let z =   × 
So, n = 40 and x = Re (z ) = − 10  1 − 2i sin θ   1 + 2 i sin θ 
α+i (rationalising the denominator)
2. Let x + iy =
α −i 3 − 4 sin θ + 8i sin θ
2
(α + i )2 (α 2 − 1) + (2α )i α 2 − 1  2α  =
⇒ x + iy = 2 = = 2 + i 1 + 4 sin 2 θ
α +1 α2 + 1 α + 1  α 2 + 1
[Q a 2 − b2 = (a + b)(a − b) and i 2 = − 1]
On comparing real and imaginary parts, we get
 3 − 4 sin θ   8 sin θ 
2
α2 −1 2α =  + i
x= and y = 2  1 + 4 sin 2 θ   1 + 4 sin 2 θ 
α2 + 1 α +1
2 As z is purely imaginary, so real part of z = 0
 α 2 − 1  2α 
2
3 − 4 sin 2 θ
Now, x + y =  2
2 2
 + 2  ∴ = 0 ⇒ 3 − 4 sin 2 θ = 0
 α + 1  α + 1
1 + 4 sin 2 θ
3
α 4 + 1 − 2α 2 + 4α 2 (α 2 + 1)2 ⇒ sin 2 θ =
= = 2 =1 4
(α 2 + 1)2 (α + 1)2
3
⇒ x2 + y2 = 1 ⇒ sin θ = ±
2
Which is an equation of circle with centre (0, 0) and Y
radius 1 unit. 1 y=sin θ
α + i  √3/2
So, S =  ; α ∈ R lies on a circle with radius 1.
α − i 
–π/2 –π/3
X′ π X
3. Given complex number O π/3 2π/3
5 + 3z
ω=
5(1 − z ) –√3/2
−1
⇒ 5 ω − 5 ω z = 5 + 3z
Y′
⇒ (3 + 5 ω )z = 5 ω − 5
⇒ |3 + 5 ω||z| = |5 ω − 5| …(i)  π π 2π 
⇒ θ ∈ − , , 
[applying modulus both sides and |z1z2| = |z1||z2|]  3 3 3 
Q |z| < 1 2π
Sum of values of θ = .
∴ |3 + 5 ω| > |5 ω − 5| [from Eq. (i)] 3
 3 2 + 3i sin θ
⇒ ω + > |ω − 1| 6. Let z = is purely imaginary. Then, we have
 5 1 − 2i sin θ
2
 3 Re(z ) = 0
Let ω = x + iy, then  x +  + y2 > (x − 1)2 + y2 2 + 3i sin θ
 5 Now, consider z =
9 6 1 − 2i sin θ
⇒ x +
2
+ x > x + 1 − 2x
2
25 5 (2 + 3i sin θ) (1 + 2i sin θ )
16x 16 1 =
⇒ > ⇒ x > ⇒ 5x > 1 (1 − 2i sin θ ) (1 + 2i sin θ)
5 25 5
2 + 4i sin θ + 3i sin θ + 6i 2 sin 2 θ
⇒ 5 Re( ω ) > 1 =
12 − (2i sin θ) 2
10 Complex Numbers

2 + 7i sin θ − 6 sin 2 θ Topic 2 Conjugate and Modulus of


=
1 + 4 sin 2 θ Complex Number
2 − 6 sin 2 θ 7 sin θ 1. Let the complex number z = x + iy
= +i
1 + 4 sin 2 θ 1 + 4 sin 2 θ
Also given, | z − i | =| z − 1|
Q Re(z ) = 0
⇒ | x + iy − i | = | x + iy − 1|
2 − 6 sin 2 θ
∴ = 0 ⇒ 2 = 6 sin 2 θ
1 + 4 sin 2 θ ⇒ x2 + ( y − 1)2 = (x − 1)2 + y2
1
⇒ sin 2 θ = [Q| z | = (Re(z ))2 + (Im(z ))2 ]
3
1 On squaring both sides, we get
⇒ sin θ = ±
3 x2 + y2 − 2 y + 1 = x2 + y2 − 2x + 1
−1  1 −1  1  ⇒ y = x, which represents a line passing through the
⇒ θ = sin  ±  = ± sin  
 3   3 origin with slope 1.
 6i −3 i 1 (1 + i )2
7. Given, 4 3i −1  = x + iy 2. The given complex number z =
a−i
20 3 i (1 − 1 + 2i ) (a + i )
= [Q i 2 = − 1]
 6i 1 1 a2 + 1
⇒ −3 i  4 −1 −1  = x + i y 2i (a + i ) −2 + 2ai
 20 i i = = …(i)
a2 + 1 a2 + 1
⇒ x + iy = 0 [Q C 2 and C3 are identical] Q z = 2 /5 [given]
⇒ x = 0, y = 0
4 + 4a 2 2 2 2
13 13 13 ⇒ = ⇒ =
8. ∑ (i n
+i n+1
) = ∑ i (1 + i ) = (1 + i )
n
∑ i n
(a 2 + 1)2 5 1+ a 2 5
n=1 n=1 n =1
4 2
 i − (1 − i13 )  ⇒ = ⇒ a 2 + 1 = 10 ⇒ a 2 = 9 ⇒ a = 3 [Qa > 0]
= (1 + i ) (i + i 2 + i3 + K + i13 ) = (1 + i )  1 + a2 5

 1−i  –2 + 6i
 i (1 − i )  ∴ z= [From Eq. (i)]
= (1 + i )  10
 = (1 + i ) i = i − 1
 1−i   −2 + 6 i   1 3  1 3
So, z =   =  − + i ⇒ z = − − i
Alternate Solution  10   5 5  5 5
Since, sum of any four consecutive powers of iota is zero. [Qif z = x + iy, then z = x − iy]
13
∴ ∑ (i n + i n + 1 ) = (i + i 2 + K + i13 ) 3. Clearly|z1|= 9, represents a circle having centre C1 (0, 0)
n=1 and radius r1 = 9.
+ (i 2 + i3 + K + i14 ) = i + i 2 = i − 1
and |z2 − 3 − 4i|= 4 represents a circle having centre
n n
 1 + i  1 + i 1 + i C 2(3, 4) and radius r2 = 4.
9. Since,   =1 ⇒  ×  =1
1 − i  1 − i 1 + i The minimum value of |z1 − z2| is equals to minimum
n distance between circles|z1|= 9 and|z2 − 3 − 4i|= 4.
 2i 
⇒   =1
 2 Q C1C 2 = (3 − 0)2 + (4 − 0)2 = 9 + 16 = 25 = 5
⇒ in = 1 and |r1 − r2|=|9 − 4|= 5 ⇒ C1C 2 =|r1 − r2|
The smallest positive integer n for which i n = 1 is 4. ∴ Circles touches each other internally.
∴ n =4 Hence, |z1 − z2|min = 0
az + b ax + b + aiy (ax + b + aiy)((x + 1) − iy) z −α
10. = = 4. Since, the complex number (α ∈ R) is purely
z+1 (x + 1) + iy (x + 1)2 + y2 z+α
 az + b − (ax + b) y + ay(x + 1) imaginary number, therefore
∴ Im  =
 z+1 (x + 1)2 + y2 z −α z −α
+ =0 [Qα ∈ R]
(a − b) y z+α z+α
⇒ =y
(x + 1)2 + y2 ⇒ zz − αz + αz − α 2 + zz − αz + αz − α 2 = 0
2 2
Q a − b =1 ⇒ 2 z − 2 α2 = 0 [Qzz = z ]
∴ (x + 1)2 + y2 = 1 ⇒ α2 = z
2
=4 [| z | = 2 given]
∴ x = − 1 ± 1 − y2 ⇒ α=±2
Complex Numbers 11

5. We have,|z | + z = 3 + i 8. PLAN Intersection of circles, the basic concept is to solve the


equations simultaneously and using properties of modulus of
Let z = x + iy complex numbers.
∴ x2 + y2 + x + iy = 3 + i Formula used |z|2 = z ⋅ z
⇒ (x + x + y ) + iy = 3 + i
2 2
and |z1 − z2|2 = (z1 − z2) (z1 − z2)
⇒ x + x2 + y2 = 3 and y = 1 = |z1|2 − z1z2 − z2z1 + |z2|2
Here, (x − x0 ) + ( y − y0 ) = r 2
2 2
Now, x2 + 1 = 3 − x
⇒ x2 + 1 = 9 − 6x + x2 and (x − x0 )2 + ( y − y0 )2 = 4r 2 can be written as,

⇒ 6x = 8 ⇒ x =
4 |z − z0|2 = r 2 and |z − z0|2 = 4r 2
3 1
4 Since, α and lies on first and second respectively.
∴ z= +i α 2
3 1
∴ |α − z0|2 = r 2 and − z0 = 4 r 2
16 25 5 α
⇒ |z | = +1= ⇒ |z | =
9 9 3 ⇒ (α − z0 ) (α − z0 ) = r 2
6. PLAN If z is unimodular, then| z| = 1. Also, use property of modulus
⇒ |α|2 − z0α − z0α + |z0|2 = r 2 …(i)
i.e. z z =| z|2
2
Given, z2 is not unimodular i.e.|z2|≠ 1 and
1
− z0 = 4 r 2
z − 2 z2 α
and 1 is unimodular.
2 − z1z2 1  1 
⇒  − z0   − z0  = 4 r
2
z1 − 2z2 α  α 
⇒ = 1 ⇒ |z1 − 2z2|2 =|2 − z1z2|2
2 − z1z2 1 z z
⇒ (z1 − 2z2)(z1 − 2z2) = (2 − z1z2) (2 − z1z2) [zz = |z|2 ] ⇒ − 0 − 0 + |z0|2 = 4r 2
|α|2 α α
⇒ |z1|2+4|z2|2−2z1z2 − 2z1z2 Since, |α|2 = α ⋅ α
= 4+|z1|2|z2|2−2z1z2 − 2z1z2 ⇒ (|z2|2−1)(|z1|2−4) = 0 1 z ⋅α z
Q |z2|≠ 1 ⇒ − 0 − 0 ⋅ α + |z0|2 = 4r 2
|α|2 |α|2 |α|2
∴ |z1|= 2
Let z1 = x + iy ⇒ x2 + y2 = (2)2 ⇒ 1 − z0α − z0α + |α|2|z0|2 = 4r 2|α|2 …(ii)
∴ Point z1 lies on a circle of radius 2. On subtracting Eqs. (i) and (ii), we get
7. |z| ≥ 2 is the region on or outside circle whose centre is
(|α|2 − 1) + |z0|2 (1 − |α|2 ) = r 2 (1 − 4|α|2 )
(0, 0) and radius is 2.
1 ⇒ (|α|2 − 1) (1 − |z0|2 ) = r 2(1 − 4|α|2 )
Minimum z + is distance of z, which lie on circle
2  r 2 + 2
⇒ (|α|2 − 1) 1 −  = r 2(1 − 4|α|2 )
| z | = 2 from (−1 / 2, 0).  2 
1  1  r2 + 2
∴ Minimum z + = Distance of  − , 0 from (−2, 0)
2  2  Given, |z0|2 =
2
 1
2
3  −1  3
2
 − r 2
=  −2 +  +0= =  + 2 + 0 = ⇒ (|α|2 − 1) ⋅   = r 2(1 − 4|α|2 )
 2 2  2  2  2 
Y ⇒ |α|2 − 1 = − 2 + 8|α|2 ⇒ 7|α|2 = 1
∴ |α| = 1 / 7
9. PLAN If ax 2 + bx + c = 0 has roots α, β, then
D A − b ± b 2 − 4ac
X′ X α, β =
(–2,0) 1 , (0,0) (2,0) 2a
(– —
2 0
)
For roots to be real b 2 − 4ac ≥ 0.
Description of Situation As imaginary part of
z = x + iy is non-zero.
Y′
⇒ y ≠0
1
Geometrically Min z + = AD Method I Let z = x + iy
2
1 ∴ a = (x + iy)2 + (x + iy) + 1
Hence, minimum value of z + lies in the interval ⇒ (x2 − y2 + x + 1 − a ) + i (2xy + y) = 0
(1, 2). 2
⇒ (x2 − y2 + x + 1 − a ) + iy (2x + 1) = 0, …(i)
12 Complex Numbers

It is purely real, if y (2x + 1) = 0 On squaring both sides, we get


but imaginary part of z, i.e. y is non-zero. 1 + |w|2 − 2|w| Re (w) = 1 + |w|2 + 2|w| Re (w)
⇒ 2x + 1 = 0 or x = − 1 / 2 [using|z1 ± z2|2 = |z1|2 + |z2|2 ± 2|z1||z2| Re (z1z2)]
1 1 ⇒ 4|w|Re|w| = 0
From Eq. (i), − y2 − + 1 − a = 0
4 2 ⇒ Re (w) = 0
2 3 3
⇒ a =− y + ⇒ a< 14. We know, |z1 − z2| = |z1 − (z2 − 3 − 4i ) − (3 + 4i )|
4 4
≥ |z1| − |z2 − 3 − 4i | − |3 + 4i|
Method II Here, z 2 + z + (1 − a ) = 0
≥ 12 − 5 − 5 [using|z1 − z2| ≥ |z1| − |z2|]
− 1 ± 1 − 4 (1 − a )
∴ z= ∴ |z1 − z2| ≥ 2
2 ×1
Alternate Solution
− 1 ± 4a − 3
⇒ z= Clearly from the figure|z1 − z2|is minimum when z1 , z2
2
3 lie along the diameter.
For z do not have real roots, 4 a − 3 < 0 ⇒ a<
4 Y
10. Since, zz (z 2 + z 2) = 350 B
A Z1
⇒ 2 (x + y ) (x − y ) = 350
2 2 2 2
, 4) Z2
(3
⇒ (x2 + y2) (x2 − y2) = 175 C1
X′ X
Since, x, y ∈ I, the only possible case which gives C2 12
integral solution, is
x2 + y2 = 25 ... (i)
x2 − y2 = 7 ... (ii)
Y′
From Eqs. (i) and (ii),
x2 = 16, y2 = 9 ⇒ x = ± 4, y = ± 3 ∴ |z1 − z2| ≥ C 2B − C 2A ≥ 12 − 10 = 2
15. Given, |z1| = |z2| = |z3| = 1
∴ Area of rectangle = 8 × 6 = 48
Now, |z1| = 1
11. Let z = cos θ + i sin θ
z cos θ + i sin θ ⇒ |z1|2 = 1
⇒ =
1 − z 2 1 − (cos 2 θ + i sin 2 θ ) ⇒ z1z1 = 1
cos θ + i sin θ Similarly, z2z2 = 1, z3 z3 = 1
=
2 sin 2 θ − 2i sin θ cos θ 
Again now, 1 + 1 + 1 
= 1
cos θ + i sin θ i
= = z1 z2 z3
− 2i sin θ (cos θ + i sin θ ) 2 sin θ
z ⇒ | z1 + z2 + z3 |= 1 ⇒ |z1 + z2 + z3|= 1
Hence, lies on the imaginary axis i.e. Y-axis.
1 − z2 ⇒ |z1 + z2 + z3| = 1
Alternate Solution 16. (1 + i )n1 + (1 − i )n1 + (1 + i )n2 + (1 − i )n2
z z 1
Let E = = = which is an imaginary. = [n1 C 0 + n1
C1i + n1
C 2i 2 + n1
C3 i3 + K ]
1 − z 2 zz − z 2 z − z
+ [n1 C 0 − n1
C1 i + n1 C 2i 2 − n1 C3 i3 + ... ]
w − wz
12. Let z1 = be purely real ⇒ z1 = z1 + [n2 C 0 + n2
C1i + n2C 2i 2 + n2C3 i3 + K ]
1−z
w − wz w − wz + [n2 C 0 – n2
C1 i + n2C 2i 2 – n2C3 i3 + .. ]
∴ =
1− z 1−z = 2 [n1 C 0 + n1
C2 i2 + n1
C 4i 4 + K ]
⇒ w − wz − wz + wz ⋅ z = w − zw − wz + wz ⋅ z + 2 [n2 C 0 + n2
C 2i 2 + n2
C 4i 4 + K ]
⇒ (w − w ) + (w − w)| z |2 = 0
= 2 [ C0 −
n1 n1
C2 + n1
C4 − K ] + 2 [ C0 − n2 n2
C2
⇒ (w − w ) (1 − | z |2 ) = 0 + n2
C 4 −... ]
⇒ | z |2 = 1 [as w − w ≠ 0, since β ≠ 0] This is a real number irrespective of the values of n1 and
⇒ | z | = 1 and z ≠ 1 n2.
z −1 Alternate Solution
13. Since,|z| = 1 and w =
z+1 {(1 + i )n1 + (1 − i )n1 } + {(1 + i )n2 + (1 − i )n2 }
1+ w |1 + w| ⇒ A real number for all n1 and n2 ∈ R.
⇒ z − 1 = wz + w ⇒ z = ⇒ |z| =
1−w |1 − w| [Q z + z = 2 Re (z ) ⇒ (1 + i )n1 + (1 − i )n1
⇒ |1 − w| = |1 + w| [ Q|z| = 1] is real number for all n ∈ R]
Complex Numbers 13

17. Since, (sin x + i cos 2x) = cos x − i sin 2x Alternate Solution


We know that, z + z = 2 Re(z )
⇒ sin x − i cos 2x = cos x − i sin 2x 5 5
 3 i  3 i
⇒ sin x = cos x and cos 2x = sin 2x If z= +  + −  , then
 2 2  2 2
⇒ tan x = 1 and tan 2x = 1
z is purely real. i.e. Im (z ) = 0
⇒ x = π / 4 and x = π / 8 which is not possible at same
z − 5i 
time. 22. Given,   = 1 ⇒ |z − 5i| = |z + 5i|
Hence, no solution exists. z + 5i 
18. Since, z1 , z2, z3 , z4 are the vertices of parallelogram. [Q if|z − z1| = |z − z2|, then it is a perpendicular
bisector of z1 and z2 ]
D(z4) C(z3) Y
(0, 5)
O
X′ X

(0, –5)

Y′
A(z1) B(z2)
∴ Perpendicular bisector of (0, 5) and (0, – 5) is X-axis.
∴ Mid-point of AC = mid-point of BD
z1 + z3 z2 + z4 23. It is given that the complex number Z, satisfying
⇒ = |z 2 + z + 1| = 1
2 2
2
⇒ z1 + z3 = z2 + z4  1 3
⇒ z +  + =1
 1 − iz   2  4
19. Since,|w| = 1 ⇒  = 1
z−i  Q |z1 − z2| ≥ ||z1| − |z2||
⇒ |z − i| = |1 − iz|  1
2
 3 1 3
2

⇒ |z − i | = |z + i | [Q |1 − iz | = | − i || z + i | = | z + i |] ∴ z +  − −  ≥ z + − −
 2   4  2 4
∴It is a perpendicular bisector of (0, 1) and (0, − 1)
2
i.e. X-axis. Thus, z lies on the real axis. 1 3
⇒ z+ − ≤1
2 4
20. Given,|z − 4| < |z − 2|
2
Since, |z − z1| > |z − z2| represents the region on right 1 3
⇒ −1 ≤ z + − ≤1
side of perpendicular bisector of z1 and z2. 2 4
∴ |z − 2| > |z − 4| 1 1 7
2

⇒ Re (z ) > 3 and Im (z ) ∈ R ⇒ − ≤ z+ ≤
4 2 4
Y 2
1 7
⇒ 0≤ z+ ≤ {Q|z| ≥ 0}
2 4
X′ X 1 7
O (2, 0) (3, 0) (4, 0) ⇒ 0≤ z+ ≤ … (i)
2 2
1 7
Y′ Q z+ ≤
2 2
5 5
 3 i  3 i Q |z1 + z2| ≥ ||z1| − |z2||
21. Given, z =  +  + − 
 2 2  2 2 1 1
∴ z + ≥ |z| −
 −1 + i 3 −1 − i 3  2 2
Q ω = 2
and ω 2 =
2  1 1 7
  ⇒ |z| − ≤ z+ ≤
2 2 2
3+i  −1 + i 3 
Now, = −i   = − iω 7 1 7
2  2  ⇒ − ≤ |z| − ≤
2 2 2
3−i  −1 − i 3 
and =i  = iω 2 1− 7 7+1
2  2  ⇒ ≤ |z| ≤
2 2
∴ z = (− iω )5 + (iω 2)5 = − iω 2 + iω 1+ 7
= i(ω − ω 2) = i (i 3 ) = − 3 ⇒ |z| ≤
2
⇒ Re(z ) < 0 and lm (z ) = 0 ∴ |z| ≤ 2
14 Complex Numbers

 1 3
2
1 3
2
Also, given w1 = a + ic and w2 = b + id
Q z +  + ≤ z+ +
 2 4 2 4 Now, |w1| = a 2 + c2 = a 2 + b2 = 1
2 2
1 3 1 3 |w2| = b2 + d 2 = a 2 + b2 = 1
⇒ 1≤ z+ + ⇒ z+ + ≥1
2 4 2 4 and Re(w1 w2) = ab + cd = (bλ )b + c(− λc) [from Eq. (i)]
1 1
2
1 1 = λ (b2 − c2) = 0
⇒ z+ ≥ ⇒ z+ ≥ … (ii)
2 4 2 2 27. min|1 − 3 i − z|= perpendicular distance of point (1, − 3)
Z ∈S
From Eqs. (i) and (ii), we get
| 3 − 3| 3 − 3
1
≤ z+ ≤
1 7 from the line 3x + y = 0 ⇒ =
2 2 2 3+1 2
28. Since, S = S1 ∩ S 2 ∩ S3
24. We have,
sz + tz + r = 0 …(i) Y
On taking conjugate
sz + tz + r = 0 …(ii)
On solving Eqs. (i) and (ii), we get 150°
X′ X
rt − rs O (4, 0)
z= 2
|s| − |t|2
(a) For unique solutions of z y = –3√x
|s|2 − |t|2 ≠ 0 ⇒ |s| ≠ |t| Y′
It is true Clearly, the shaded region represents the area of sector
(b) If|s| = |t|, then rt − rs may or may not be zero. 1 2 1 5π 20π
So, z may have no solutions. ∴ S= r θ = × 42 × =
2 2 6 3
∴ L may be an empty set.
29. Since, |w − (2 + i )| < 3 ⇒ |w| − |2 + i| < 3
It is false.
(c) If elements of set L represents line, then this line ⇒ −3 + 5 < |w| < 3 + 5
and given circle intersect at maximum two point. ⇒ −3 − 5 < − |w| < 3 − 5 …(i)
Hence, it is true. Also, |z − (2 + i )| = 3
(d) In this case locus of z is a line, so L has infinite
⇒ −3 + 5 ≤ |z| ≤ 3 + 5 …(ii)
elements. Hence, it is true.
∴ −3 < |z| − |w| + 3 < 9
25. Given,|z1| = |z2|
z1 + z2 z1 − z2 z1z1 − z1z2 + z2z1 − z2 z2
30. |z + 1 − i| + |z − 5 − i|2
2

Now, × =
z1 − z2 z1 − z2 |z1 − z2|2 = (x + 1)2 + ( y − 1)2 + (x − 5)2 + ( y − 1)2
= 2(x2 + y2 − 4x − 2 y) + 28
|z |2 + (z2 z1 − z1 z2) − |z2|2
= 1 = 2(4) + 28 = 36 [Q x2 + y2 − 4x − 2 y = 4]
|z1 − z2|2
z2z1 − z1z2 31. Let z = x + iy
= [Q|z1|2 = |z2|2 ]
|z1 − z2|2 Set A corresponds to the region y ≥ 1 …(i)

As, we know z − z = 2i Im (z ) Set B consists of points lying on the circle, centre at


(2, 1) and radius 3.
∴ z2z1 − z1z2 = 2i Im (z2z1 )
i.e. x2 + y 2 − 4 x − 2 y = 4 …(ii)
z1 + z2 2i Im (z2 z1 )
∴ = Set C consists of points lying on the x + y = 2 …(iii)
z1 − z2 |z1 − z2|2
Y
which is purely imaginary or zero.
P
26. Since, z1 = a + ib and z2 = c + id
(0,√2)
⇒ |z1|2 = a 2 + b2 = 1 and |z2|2 = c2 + d 2 = 1 …(i) (2,1)
y=1
[Q|z1|=|z2| = 1] X′ X
(√2,0)
Also, Re (z1z2) = 0 ⇒ ac + bd = 0
a d
⇒ =− =λ [say]…(ii)
b c Y′
From Eqs. (i) and (ii), b2λ2 + b2 = c2 + λ2c2 Clearly, there is only one point of intersection of the line
⇒ b2 = c2 and a 2 = d 2 x + y = 2 and circle x2 + y2 − 4x − 2 y = 4.
Complex Numbers 15

32. A. Let z = x + iy Since, it is real, so imaginary part will be zero.


x x x
⇒ we get y x 2 + y2 = 0 ∴ −2 sin sin + cos  − tan x = 0
2 2 2
⇒ y = 0 ⇒ Im (z ) = 0 x x x x x
⇒ 2 sin sin + cos  cos x + 2 sin cos = 0
B. We have 2 2 2 2 2
2ae = 8, 2a = 10 ⇒ 10e = 8
x  x x  2x 2 x x
4  16 ⇒ sin sin + cos   cos − sin  + cos  = 0
⇒ e = ⇒ b2 = 25 1 −  = 9 2  2 2  2 2 2
5  25
x
x2 y2 ∴ sin =0
∴ + =1 2
25 9
⇒ x = 2 nπ ... (i)
C. Let w = 2 (cos θ + i sin θ )
 x x  2x 2 x x
∴ z = 2 (cos θ + i sin θ ) −
1 or sin + cos   cos − sin  + cos = 0
 2 2   2 2  2
2 (cos θ + i sin θ )
1 x
= 2 (cos θ + i sin θ ) − (cos θ − i sin θ ) On dividing by cos3 , we get
2 2
3 5  x   2 x  2 x
= cos θ + i sin θ  tan + 1 1 − tan  + 1 + tan  = 0
2 2  2   2  2
Let z = x + iy x x
⇒ tan3 − tan − 2 = 0
3 5 2 2
⇒ x = cos θ and y = sin θ
2 2 x
Let tan = t
 2x
2
 2 y
2 2
⇒   +   =1
 3 5 and f (t ) = t3 − t − 2
x2 y2 Then, f (1) = − 2 < 0
⇒ + =1 and f (2) = 4 > 0
9 / 4 25 / 4
9 /4 4 Thus, f (t ) changes sign from negative to positive in the
∴ e= 1− = interval (1, 2).
25 / 4 5
∴ Let t = k be the root for which
D. Let w = cos θ + i sin θ f (k) = 0 and k ∈(1, 2)
1 x
Then, z = x + iy = cos θ + i sin θ + ∴ t = k or tan = k = tan α
cos θ + i sin θ 2
= 2 cos θ ⇒ x/2 = nπ + α
x = 2nπ + 2α , α = tan −1 k, where k ∈ (1, 2)
⇒ x = 2 cos θ , y = 0 ⇒ 
 or x = 2nπ
xα + yβ + z γ x( p)1/3 + y( p)1/3 ω + z ( p)1/3 ω 2
33. =
xβ + yγ + zα x( p)1/3 ω 2 + y( p)1/3 ω3 + z ( p)1/3 ω 36. Since, z1 , z2, z3 are in AP.
ω 2 (x + yω + z ω 2) ⇒ 2z2 = z1 + z3
ω 2 (xω + yω 2 + z ) i.e. points are collinear, thus do not lie on circle. Hence,
ω 2 (x + yω + zω 2) it is a false statement.
= = ω2
x + yω + zω 2 37. Since, z1 , z2, z3 are vertices of equilateral triangle and
34. |az1 − bz2| + |bz1 + az2|
2 2 |z1| = |z2| = |z3|
⇒ z1 , z2, z3 lie on a circle with centre at origin.
= [a 2|z1|2 + b2|z2|2 − 2ab Re (z1z2)]
+ [b2|z1|2 + a 2|z2|2 + 2ab Re (z1z2)] ⇒ Circumcentre = Centroid
z + z2 + z3
= (a 2 + b2) (|z1|2 + |z2|2 ) ⇒ 0= 1
3
 x x ∴ z1 + z2 + z3 = 0
sin + cos  − i tan x
 2 2
35. ∈R 38. Let z = x + iy ⇒ 1 ∩ z gives 1 ∩ x + iy
x
1 + 2 i sin or 1 ≤ x and 0 ≤ y …(i)
2
 x x  x 1−z 1 − x − iy
sin + cos − i tan x 1 − 2i sin  Given, ∩0 ⇒ ∩0
 2 2   2 1+ z 1 + x + iy
=
x (1 − x − iy) (1 + x − iy)
1 + 4 sin 2 ⇒ ∩ 0 + 0i
2 (1 + x + iy) (1 + x − iy)
16 Complex Numbers

1 − x2 − y2 2iy ⇒ 1 − z1z2 − z1z2 + z1z1z2z2 < z1z1 − z1z2 − z2z1 + z2z2


⇒ − ∩ 0 + 0i
(1 + x) + y
2 2
(1 + x)2 + y2 ⇒ 1 + |z1|2|z2|2 <|z1|2 + |z2|2
⇒ x2 + y2 ≥ 1 and −2 y ≤ 0 ⇒ 1 − |z1|2 − |z2|2+ | z1|2|z2|2 < 0
or x2 + y2 ≥ 1 and y ≥ 0 which is true by Eq. (i). ⇒ (1 − |z1|2 )(1 − |z2|2 ) < 0 …(iii)
39. As we know,|z| = z ⋅ z 2 which is true by Eq. (i) as|z1| < 1 and|z2| > 1
∴ (1 − |z1|2 ) > 0 and (1 − |z2|2 ) < 0
|z − α|2
Given, = k2 ∴ Eq. (iii) is true whenever Eq. (ii) is true.
|z − β|2
⇒ (z − α )(z − α ) = k2(z − β ) (z − β ) ⇒ 1 − z1z2
 < 1 Hence proved.
 z1 − z2 
⇒ |z| − αz − αz + |α| = k (|z| − βz − β z + |β| )
2 2 2 2 2

⇒ |z|2 (1 − k2) − (α − k2β )z − (α − β k2) z 42. Given,|z|2 w − |w|2 z = z − w


+ (|α|2 − k2|β|2 ) = 0 ⇒ zz w − ww z = z − w [Q |z|2 = zz ] …(i)
(α − k2β ) (α − β k2) |α|2 − k2|β|2 Taking modulus of both sides, we get
⇒ |z|2 − z− z+ = 0 …(i)
(1 − k )
2
(1 − k )
2
(1 − k2) |zw||z − w| = |z − w|
On comparing with equation of circle, ⇒ |zw||z − w| = |z − w| [∴ |z| = |z| ]
|z| + az + az + b = 0
2 ⇒ | zw|| z − w | = |z − w |
whose centre is (− a ) and radius = |a|2 − b ⇒ |z − w|(|zw| − 1) = 0

α −kβ 2 ⇒ |z − w| = 0 or |zw| − 1 = 0
∴ Centre for Eq. (i) = ⇒ |z − w| = 0 or |zw| = 1
1 − k2
 α − k β  α − k β  αα − k ββ
2 2 2 ⇒ z − w=0 or |z w|= 1
and radius =     − ⇒ z=w or |zw| = 1
 1 − k2   1 − k2  1 − k2
Now, suppose z ≠ w
k(α − β ) 
=  Then,|zw| = 1 or|z||w| = 1
1−k 
2
1
⇒ |z| = =r [say]
40. Given, a1z + a 2z 2 + K + a nz n = 1 |w|
1 1 iφ
and |z| < …(i) Let z = reiθ and w = e
3 r
∴ |a1z + a 2z 2 + a3 z3 + K + a nz n| = 1 On putting these values in Eq. (i), we get
⇒ |a1z| + |a 2z 2| + |a3 z3| + K + |a nz n| ≥ 1 1  1 1
r 2 ei φ  − 2 (reiθ ) = reiθ − eiφ
[using|z1 + z2| ≤ |z1| + |z2|] r  r r
iφ 1 iθ iθ 1 iφ
⇒ 2{(|z| + |z| + |z| + K + |z|n } > 1
2 3
[using|a r| < 2] ⇒ re − e = re − e
r r
2|z|(1 − |z|n )  1  iφ  1  iθ
⇒ >1 [using sum of n terms of GP] ⇒  r +  e =  r +  e
1 − |z|  r  r
⇒ eiφ = eiθ ⇒ φ = θ
⇒ 2|z| − 2|z|n + 1 > 1 − |z|
⇒ 3|z| > 1 + 2|z|n + 1 1
Therefore, z = reiθ and w = eiθ
1 2 r
⇒ |z| > + |z|n + 1
3 3 1
1 ⇒ zw = reiθ . e−iθ = 1
⇒ |z| > , which contradicts …(ii) r
3 NOTE ‘If and only if ’ means we have to prove the relation in
∴ There exists no complex number z such that both directions.
n
Conversely
|z| < 1 / 3 and ∑ a rz r = 1 Assuming that z = w or z w = 1
r =1
If z = w, then
41. Given,|z1| < 1 and |z2| > 1 …(i) LHS = zz w − w wz = |z|2⋅z − |w|2⋅z
Then, to prove = |z|2⋅z − |z|2⋅z = 0
 
1 − z1z2
 < 1 
z1
 |z1| and RHS = z − w = 0
using z = |z |
 z1 − z2    2 2  If zw = 1, then zw = 1 and
⇒ |1 − z1z2| < |z1 − z2| …(ii) LHS = zz w − ww z = z ⋅ 1 − w ⋅ 1
On squaring both sides, we get, = z − w = z − w = 0 = RHS
(1 − z1z2)(1 − z1z2) < (z1 − z2)(z1 − z2) [using|z|2 = zz ] Hence proved.
Complex Numbers 17

Alternate Solution z1 − z2
45. Here, z1Rz2 ⇔ is real
We have, |z|2 w − |w|2 z = z − w z1 + z2
⇔ |z| w − |w|2 z − z + w = 0
2
z1 − z1
(i) Reflexive z1Rz1 ⇔ =0 [purely real]
⇔ (|z|2 + 1)w − (|w|2 + 1)z = 0 z1 + z2
⇔ (|z|2 + 1)w = (|w|2 + 1)z ∴ z1Rz1 is reflexive.
z |z|2 + 1
⇔ = z1 − z2
w |w|2 + 1 (ii) Symmetric z1Rz2 ⇔ is real
z z1 + z2
∴ is purely real.
w − (z2 − z1 )
z z ⇒ is real ⇒ z2Rz1
⇔ = ⇒ zw = zw …(i) z1 + z2
w w
Again, |z|2 w − |w|2 z = z − w ∴ z1Rz2 ⇒ z2Rz1
⇔ z ⋅ zw − w ⋅ wz = z − w Therefore, it is symmetric.
⇔ z (zw − 1) − w (zw − 1) = 0 (iii) Transitive z1Rz2
⇔ (z − w)(zw − 1) = 0 [from Eq. (i)] z1 − z2
⇒ is real
⇔ z = w or zw = 1 z1 + z2
Therefore, |z|2 w − |w|2 z = z − w if and only if z = w or and z2Rz3
zw = 1.
z2 − z3
43. Let z = x + iy. ⇒ is real
z2 + z3
Given, z = iz 2
Here, let z1 = x1 + iy1 , z2 = x2 + iy2 and z3 = x3 + iy3
⇒ (x + iy) = i (x + i y)2
z − z2 (x − x2) + i ( y1 − y2)
⇒ x − iy = i (x2 − y2 + 2i xy) ∴ 1 is real ⇒ 1 is real
z1 + z2 (x1 + x2) + i ( y1 + y2)
⇒ x − iy = − 2xy + i (x2 − y2)
{(x1 − x2) + i ( y1 − y2)}{(x1 + x2) − i ( y1 + y2)}
NOTE It is a compound equation, therefore we can generate ⇒
from it more than one primary equations. (x1 + x2)2 + ( y1 + y2)2
On equating real and imaginary parts, we get ⇒ ( y1 − y2) (x1 + x2) − (x1 − x2) ( y1 + y2) = 0
x = − 2xy and − y = x2 − y2 ⇒ 2 x2y1 − 2 y2x1 = 0
⇒ x + 2xy = 0 and x2 − y2 + y = 0 x1 x2
⇒ = ... (i)
⇒ x(1 + 2 y) = 0 y1 y2
⇒ x = 0 or y = − 1 / 2 Similarly, z2Rz3
When x = 0, x2 − y2 + y = 0 ⇒ 0 − y2 + y = 0 x2 x3
⇒ = ... (ii)
⇒ y(1 − y) = 0 ⇒ y = 0 or y = 1 y2 y3
When, y = − 1 / 2 , x2 − y2 + y = 0 x1 x3
1 1 3 From Eqs. (i) and (ii), we have = ⇒ z1Rz3
⇒ x2 − − = 0 ⇒ x2 = y1 y3
4 2 4
Thus, z1Rz2 and z2Rz3 ⇒ z1Rz3 . [transitive]
3
⇒ x=± Hence, R is an equivalence relation.
2
3 i (1 + i ) x − 2i (2 − 3i ) y + i
Therefore, z = 0 + i 0 , 0 + i ; ± − 46. + =i
2 2 3+ i 3−i
3 i
⇒ z = i, ± − [Q z ≠ 0] ⇒ (1 + i ) (3 − i ) x − 2i (3 − i ) + (3 + i ) (2 − 3i ) y
2 2
+ i (3 + i ) = 10i
44. Given, iz3 + z 2 − z + i = 0
⇒ 4x + 2ix − 6i − 2 + 9 y − 7iy + 3i − 1 = 10i
⇒ iz3 − i 2z 2 − z + i = 0 [Q i 2 = − 1]
⇒ 4x + 9 y − 3 = 0 and 2x − 7 y − 3 = 10
⇒ iz 2(z − i ) − 1(z − i ) = 0
⇒ x = 3 and y = − 1
⇒ (iz 2 − 1)(z − i ) = 0
1 1
⇒ z − i = 0 or iz 2 − 1 = 0 47. Now, =
(1 − cos θ ) + 2i sin θ 2 sin 2 + 4i sin θ cos θ
θ
1
⇒ z = i or z 2 = = − i 2 2 2
i θ θ
If z = i, then|z| = |i| = 1 sin − 2 i cos
1 2 2
= ×
If z 2 = − i, then |z 2| = |− i| = 1 θ θ θ  θ θ
2 sin sin + 2 i cos  sin − 2 i cos 
⇒ |z|2 = 1 ⇒ |z| = 1 2 2 2  2 2
18 Complex Numbers

θ θ ⇒ y1 + y2 = 2 3 ⇒ Im(z1 + z2) = 2 3
sin − 2 i cos
= 2 2 Hence, option (d) is correct.
θ  2θ θ
2 sin sin + 4 cos 2  |z | 4
2  2 2 2. (*) Given, 3|z1| = 4|z2|⇒ 1 = [Q z2 ≠ 0 ⇒|z2| ≠ 0]
|z2| 3
θ θ
sin − 2 i cos z1 z1 iθ z z
= 2 2 ∴ = e and 2 = 2 e−iθ
θ θ z2 z2 z1 z1
2 sin 1 + 3 cos 2 
2 2 [Q z =|z|(cos θ + i sin θ) = |z| eiθ ]
z1 4 iθ z 3
θ ⇒ = e and 2 = e−iθ
cot z2 3 z1 4
1 2
⇒ A + iB = −i
 θ θ 3 z1 2 z2 1 −iθ
2 1 + 3 cos 2  1 + 3 cos 2 ⇒ = 2eiθ and = e
 2 2 2 z2 3 z1 2
a + ib On adding these two, we get
48. Since, (x + iy)2 =
c + id z=
3 z1 2 z2
+
1
= 2eiθ + e−iθ
 z1 |z1| 2 z2 3 z1 2
|a + ib|
⇒ |x + iy|2 = Q z = |z | 1 1
|c + id|  2 2 = 2 cos θ + 2i sin θ + cos θ − i sin θ
2 2
a 2 + b2 [Q e± iθ = (cos θ ± i sin θ)]
⇒ (x2 + y2) = 5 3
c2 + d 2 = cos θ + i sin θ
2 2
a + b2
2
⇒ (x2 + y2)2 =  5  3 34
2
17
2
c2 + d 2 Hence proved. ⇒ |z| =   +   = =
 2  2 4 2
49. Given,|z − 3 − 2 i| ≤ 2 …(i) Note that z is neither purely imaginary and nor purely
To find minimum of|2z − 6 + 5 i| real.
5 ‘*’ None of the options is correct.
or 2 z − 3 + i , using triangle inequality
2 3. Given,|z| = 1 , arg z = θ∴z = eiθ
1
i.e. ||z1| − |z2|| ≤ |z1 + z2| ∴ z = e–iθ ⇒ z =
z
5 5
∴ z −3 + i = z −3 −2i + 2i + i  
2 2  1 + z  1 + z
∴ arg   = arg   = arg (z ) = θ
9 1 + z 1 + 1
= (z − 3 − 2 i ) + i 
2 z

≥ |z − 3 − 2 i| −
9
≥ 2−
9

5 4. Since, arg (z ) < 0 ⇒ arg (z ) = − θ
2 2 2 Y
5 5
⇒ z − 3 + i ≥ or|2z − 6 + 5 i| ≥ 5 (–z)
π−θ
2 2 r
X′ X
O –θ
Topic 3 Argument of a Complex Number r
1. Let the complex numbers (z)
Y′
z1 = x1 + iy1 and z2 = x2 + iy2 ⇒ z = r cos (−θ ) + i sin (−θ )
Now, it is given that
= r (cos θ − i sin θ )
Re (z1 ) = |z1 − 1| ⇒ x12 = (x1 − 1)2 + y12
and − z = − r [cos θ − i sin θ ]
⇒ y12 + 1 = 2x1 …(i)
= r [cos (π − θ ) + i sin (π − θ )]
and Re(z2) =|z2 − 1| ⇒ x22 = (x2 − 1)2 + y22
∴ arg (− z ) = π − θ
⇒ y22 + 1 = 2x2 …(ii)
π y − y2 1 Thus, arg (− z ) − arg (z )
and arg (z1 − z2) = ⇒ 1 = …(iii) = π − θ − (− θ ) = π
6 x1 − x2 3
Alternate Solution
From Eqs. (i) and (ii), we get
 − z
y12 − y22 = 2(x1 − x2) Reason arg (− z ) − arg z = arg   = arg (− 1) = π
 z 
y1 − y2 1
⇒ ( y1 + y2) = 2 ⇒ ( y1 + y2) = 2  z 
x1 − x2 3 and also arg z − arg (− z ) = arg   = arg (− 1) = π
 − z
Complex Numbers 19

5. Given, |z + iw| = |z − iw |= 2  a u 1
Consider b v 1 
⇒ |z − (− iw)| = |z − (iw )| = 2
c w 1 
⇒ |z − (− iw)| = |z − (− iw )|
Applying R3 → R3 – {(1 − r ) R1 + rR2}
∴ z lies on the perpendicular bisector of the line joining
− iw and − iw. Since, − iw is the mirror image of − iw in a u 1
the X-axis, the locus of z is the X-axis. = b v 1
c − (1 − r ) a − rb w − (1 − r ) u − rv 1 − (1 − r ) − r
Let z = x + iy and y = 0.
Now, |z | ≤ 1 ⇒ x2 + 02 ≤ 1 a u 1
= b v 1 = 0 [from Eq. (i)]
⇒ −1 ≤ x ≤ 1.
0 0 0 
∴ z may take values given in option (c).
9. (a) Let z = − 1 − i and arg(z) = θ
Alternate Solution
  −1 
 im (z ) 
|z + i w| ≤ |z| + |iw|= |z| + |w| Now, tan θ =  = =1
 Re(z )   −1 
≤1 +1 =2
π
∴ |z + iw| ≤ 2 ⇒ θ=
4
⇒ |z + i w| = 2 holds when Since, x < 0, y < 0
arg z − arg i w = 0  π 3π
∴ arg (z ) = −  π −  = −
z  4 4
⇒ arg =0
iw (b) We have, f (t ) = arg (−1 + it )

z
is purely real.  π − tan −1 t , t ≥ 0
iw arg (−1 + it ) =  −1
− (π + tan t ), t < 0
z
⇒ is purely imaginary. This function is discontinuous at t = 0.
w
z
Similarly, when |z − i w| = 2, then is purely (c) We have,
w
imaginary z 
arg  1  − arg (z1 ) + arg (z2)
Now, given relation  z2 
|z + iw| = |z − iw| = 2 z 
Now,arg  1  = arg (z1 ) − arg (z2) + 2nπ
Put w = i, we get  z2 
|z + i 2| = |z + i 2| = 2 z 
⇒ |z − 1| = 2 ∴ arg  1  − arg (z1 ) + arg (z2)
 z2 
⇒ z = −1 [Q|z| ≤ 1]
= arg (z1 ) − arg (z2) + 2nπ − arg (z1 ) + arg (z2) = 2nπ
Put w = − i , we get So, given expression is multiple of 2π.
|z − i 2| = |z − i 2| = 2  (z − z1 ) (z2 − z3 )
⇒ |z + 1| = 2 ⇒ z = 1 [Q|z| ≤ 1] (d) We have, arg   =π
 (z − z3 ) (z2 − z1 )
∴ z = 1 or − 1 is the correct option.
 z − z1   z2 − z3 
6. Since,|z|=|w|and arg (z ) = π − arg (w) ⇒   is purely real
 z − z3   z2 − z1 
Let w = re iθ , then w = re–iθ
Thus, the points A (z1 ), B(z2), C (z3 ) and D (z ) taken in
∴ z = rei ( π − θ ) = re iπ ⋅ e−iθ = − re−iθ = − w order would be concyclic if purely real.
7. Given,|z1 + z2| = |z1| + |z2| Hence, it is a circle.
On squaring both sides, we get C(z3)
|z1|2 + |z2|2 + 2|z1||z2| cos (arg z1 − arg z2)
= |z1|2 + |z2|2 + 2|z1||z2|
D(z)
⇒ 2|z1||z2|cos (arg z1 − arg z2) = 2|z1||z2| B(z2)
⇒ cos (arg z1 − arg z2) = 1
⇒ arg (z1 ) − arg (z2) = 0
8. Since a , b, c and u , v, w are the vertices of two triangles. A(z1)
Also, c = (1 − r ) a + rb
∴(a), (b), (d) are false statement.
and w = (1 − r ) u + rv …(i)
20 Complex Numbers

(1 − t ) z1 + t z2 θ − θ 2
2
10. Given, z =
(1 − t ) + t Therefore, |z − w |2 ≤ |r1 − r2|2 + 4 1
 2 
A P B
z1 z2
≤ |r1 − r2|2 + |θ1 − θ 2|2
z
t : (1 - t) ⇒ |z − w| ≤ (|z| − |w|)2 + (arg z − arg w)2
2

Clearly, z divides z1 and z2 in the ratio of t : (1 − t ), Alternate Solution


0 < t <1 |z − w|2 = |z|2 + |w|2 − 2|z||w|cos (arg z − arg w)
⇒ AP + BP = AB i.e. |z − z1|+ |z − z2|=|z1 − z2|
= |z|2 + |w|2 − 2|z||w| + 2|z||w|
⇒ Option (a) is true.
− 2|z||w|cos (arg z − arg w)
and arg (z − z1 ) = arg (z2 − z ) = arg (z2 − z1 )
 arg z − arg w
⇒ Option (b) is false and option (d) is true. = (|z| − |w|)2 + 2|z||w|⋅ 2 sin 2  …(i)
 2 
Also, arg (z − z1 ) = arg (z2 − z1 ) 2
 z − z1   arg z − arg w
⇒ ∴ |z − w|2 ≤ (|z| − |w|)2 + 4 ⋅ 1⋅ 1  
arg   =0  2 
 z2 − z1 
z − z1 [Q sin θ ≤ θ ]
∴ is purely real.
z2 − z1 ⇒ |z − w|2 ≤ (|z| − |w|)2 + (arg z − arg w)2
z − z1 z − z1 z − z1 z − z1 13. Since, z1 = 10 + 6i , z2 = 4 + 6i
⇒ = or =0
z2 − z1 z2 − z1 z2 − z1 z2 − z1  z − z1  π
and arg   = represents locus of z is a circle
 z − z2  4
Option (c) is correct.
shown as from the figure whose centre is (7, y) and
11. Let z = a + ib. ∠ AOB = 90°, clearly OC = 9 ⇒ OD = 6 + 3 = 9
Given, Re(z ) = 0 ⇒ a = 0 6
∴ Centre = (7, 9) and radius = =3 2
Then, z = ib ⇒ z 2 = − b2 or lm (z 2) = 0 2
Therefore, A → q z
C
π Y
Also, given, arg (z ) = .

°
4

45
O
(7, y)
 π π
Let z = r  cos + i sin 
 4 4 6 (4, 6) A B (10, 6)
z1 z2
 π π π π
Then, z 2 = r 2 cos 2 − sin 2  + 2 ir 2 cos sin
 4 4 4 4
= ir sin π /2 = ir
2 2
(4, 0) D (7, 0) (10, 0)
X
Therefore, Re (z 2) = 0 ⇒ B → p.
⇒ a = b =2− 3 [Q a , b ← (0, 1)]
⇒ Equation of circle is| z − (7 + 9i )| = 3 2
12. Let z = r1 (cos θ1 + i sin θ1 ) and w = r2(cos θ 2 + i sin θ 2)
We have,|z| = r1 ,|w| = r2, arg (z ) = θ1 and arg (w) = θ 2 Topic 4 Rotation of a Complex Number
Given,|z| ≤ 1,|w| < 1 1. The complex number z satisfying|z − 2 + i| ≥ 5, which
⇒ r1 ≤ 1 and r2 ≤ 1
represents the region outside the circle (including the
Now,
circumference) having centre (2, − 1) and radius 5 units.
z − w = (r1 cos θ1 − r2 cos θ 2) + i (r1 sin θ1 − r2 sin θ 2)
Y
⇒|z − w|2 = (r1 cos θ1 − r2 cos θ 2)2 + (r1 sin θ1 − r2 sin θ 2)2
= r12 cos 2 θ1 + r22 cos 2 θ 2 − 2r1r2 cos θ1 cos θ 2
+ r12 sin 2 θ1 + r22 sin 2 θ 2 − 2r1r2 sin θ1 sin θ 2 z0 (x,y)
(1,0)
= r12(cos 2 θ1 + sin θ1 ) +
2
r22(cos 2 θ 2 + sin θ 2)
2
¾
O Ö5
X¢ X
−2r1r2(cos θ1 cos θ 2 + sin θ1 sin θ 2) (2,-1)

= r12 + r22 − 2r1r2 cos (θ1 − θ 2)


= (r1 − r2)2 + 2r1r2[1 − cos (θ1 − θ 2)] Y¢
 θ − θ 2 1
= (r1 − r2)2 + 4r1r2 sin 2  1  Now, for z0 ∈ S is maximum.
 2  |z0 − 1|
2
  θ − θ 2  When |z0 − 1| is minimum. And for this it is required
≤ |r1 − r2|2 + 4sin  1  [Q r1 , r2 ≤ 1]
  2  that z0 ∈ S, such that z0 is collinear with the points
(2, − 1) and (1, 0) and lies on the circumference of the
and |sin θ| ≤ |θ|, ∀ θ ∈ R circle|z − 2 + i|= 5.
Complex Numbers 21

So let z0 = x + iy, and from the figure 0 < x < 1 and y > 0. 5. Since, |PQ | = |PS | = |PR| = 2
P Y
4 − z0 − z0 4 − x − iy − x + iy ∴ Shaded part represents the
So, =
z0 − z0 + 2i x + iy − x + iy + 2i external part of circle having 4
2(2 − x)  2 − x centre (−1, 0) and radius 2. 3e iπ/4
= = −i  A
 As we know equation of circle
2i ( y + 1)  y + 1 3
having centre z0 and radius r,
2−x is|z − z0| = r
Q is a positive real number, so π/4
y+1 ∴ |z − (−1 + 0i )| > 2 X′
O
X

4 − z0 − z0 ⇒ |z + 1| > 2 Y′
is purely negative imaginary number.
z0 − z0 + 2 i Also, argument of z + 1 with respect to positive direction
of X-axis is π/4.
 4 − z0 − z0  π
⇒ arg   =− π
 z0 − z0 + 2 i  2 ∴ arg (z + 1) ≤ …(i)
4
5 5
 3 i  3 i and argument of z + 1 in anticlockwise direction is −π /4.
2. Given, z =  +  + − 
 2 2   2 2 ∴ − π / 4 ≤ arg (z + 1) …(ii)
Q Euler’s form of From Eqs. (i) and (ii),
3 i  π π |arg (z + 1)|≤ π / 4
i( π / 6 )
+ =  cos + i sin  =e
2 2  6 6 6. In the Argand plane, P is represented by e i0 and Q is
represented by e i( α − θ )
3 i  −π   π
and − = cos   + i sin  −  = e−iπ / 6 Now, rotation about a line with angle α is given by
2 2  6   6
5π 5π
e θ → e (α − θ ). Therefore, Q is obtained from P by reflection
i −i in the line making an angle α /2.
So, z = (eiπ / 6 )5 + (e−iπ / 6 )5 = e 6 + e 6
 5π 5π   5π 5π  z1 − z3 1 − i 3 (1 − i 3 )(1 + i 3 ) z2
=  cos + i sin  +  cos − i sin  7. = =
 6 6   6 6 z2 − z3 2 2 (1 + i 3 )
[Q eiθ = cos θ + i sin θ] 1 − i 23
5π =
= 2 cos 2 (1 + i 3 )
6 π/3
π 4 2 z3 z1
∴ I (z ) = 0 and R(z ) = −2 cos = − 3 < 0 = =
6 2 (1 + i 3 ) (1 + i 3 )
 5π  π π z2 − z3 1 + i 3 π π
⇒ = = cos + i sin
Q cos 6 = cos  π − 6  = − cos 6  z1 − z3 2 3 3
 
z
2 − z3  z − z  π
3. Imaginary axis ⇒ = 1 and arg  2 3
 =
1
z − z3  z1 − z3  3
Hence, the triangle is an equilateral.
z2 z'2 (7,6) Alternate Solution
z1 − z3 1 − i 3
) 1 ∴ =
(1
,2 z2 − z3 2
3 1
90° z 0 (6,2) z2 − z3 2 1 +i 3 π π
5 ⇒ = = = cos + i sin
Real axis z1 − z3 1 − i 3 2 3 3
 z2 − z3  π z2 − z3
⇒ arg   = and also =1
z2′ = (6 + 2 cos 45° , 5 + 2 sin 45° ) = (7, 6) = 7 + 6i  z1 − z3  3 z1 − z3
By rotation about (0, 0),
 iπ  Therefore, triangle is equilateral.
z2
= ei π/ 2 ⇒ z2 = z2′  e 2  1 a − ibt
z2′   8. Here, x + iy = ×
 
a + ibt a − ibt
 π π
= (7 + 6i )  cos + i sin  = (7 + 6i ) (i ) = − 6 + 7i a − ibt
 2 2 ∴ x + iy =
a 2 + b2t 2
4. Let OA = 3, so that the complex number associated with
Let a ≠ 0, b ≠ 0
A is 3e iπ / 4 . If z is the complex number associated with P,
a − bt
then ∴ x= 2 and y = 2
z − 3eiπ / 4 4 − iπ / 2 4i a + b2t 2 a + b2t 2
= e =−
0 − 3e iπ / 4
3 3 y − bt ay
⇒ = ⇒t =
⇒ 3z − 9eiπ / 4 = 12 ieiπ / 4 ⇒ z = (3 + 4 i ) eiπ / 4 x a bx
22 Complex Numbers

On putting x =
a
, we get So, z1 = 2 (cos π / 3 + sin π / 3)
a 2 + b2t 2 Since, |z2| = |z3| = 2 [given]
 a 2 y2  Y-axis
x  a 2 + b2 ⋅ 2 2  = a ⇒ a (x + y ) = ax
2 2 2
 bx 
x z1
or x2 + y2 − =0 … (i)
a
z2
2
 1 1 P (2, 0)
x −  + y =
2
or X-axis
 2a  4a 2 P (–1, 0) O
∴Option (a) is correct.
For a ≠ 0 and b = 0,
1 1 z3
x + iy = ⇒ x = , y = 0
a a Now, the triangle z1 , z2 and z3 being an equilateral and
⇒ z lies on X-axis. the sides z1z2 and z1z3 make an angle 2π / 3 at the centre.
∴ Option (c) is correct. π 2π
Therefore, ∠POz2 = + =π
1 1 3 3
For a = 0 and b ≠ 0, x + iy = ⇒ x = 0, y = − π 2π 2π 5π
ibt bt and ∠POz3 = + + =
3 3 3 3
⇒ z lies on Y-axis.
Therefore, z2 = 2 (cos π + i sin π ) = 2 (− 1 + 0) = − 2
∴ Option (d) is correct.  5π 5π  1 3
and z3 = 2  cos + i sin  =2  − i  =1 − i 3
9. PLAN It is the simple representation of points on Argand plane and  3 3  2 2
to find the angle between the points.
n
 π π nπ nπ Alternate Solution
Here, P = W n =  cos + i sin  = cos + i sin
 6 6 6 6 Whenever vertices of an equilateral triangle having
 1 centroid is given its vertices are of the form z , zω , zω 2.
H 1 = z ∈ C : Re(z ) > 
 2 ∴ If one of the vertex is z1 = 1 + i 3 , then other two
nπ vertices are (z1ω ), (z1ω 2).
∴ P ∩ H 1 represents those points for which cos is + ve. (−1 + i 3 ) (−1 − i 3 )
6 ⇒ (1 + i 3 ) , (1 + i 3 )
2 2
Hence, it belongs to I or IV quadrant.
− (1 + 3) (1 + i 2( 3 )2 + 2i 3 )
π π 11π 11π ⇒ ,−
⇒ z1 = P ∩ H 1 = cos + i sin or cos + i sin 2 2
6 6 6 6
(−2 + 2i 3 )
3 i 3 i ⇒ −2 , − =1 − i 3
∴ z1 = + or − …(i) 2
2 2 2 2 ∴ z2 = − 2 and z3 = 1 − i 3
Similarly, z2 = P ∩ H 2 i.e. those points for which
11. Given, D = (1 + i ), M = (2 − i )

cos <0 and diagonals of a rhombus bisect each other.
6
Let B ≡ (a + ib), therefore
–√3 , 1 √3 , 1
—– — Z2 Z1 —– —
2 2 2 2 a+1 b+1
= 2, = − 1 ⇒ a + 1 = 4, b + 1 = − 2
2 2
(–1, 0) Z2 ⇒ a = 3, b = − 3 ⇒ B ≡ (3 − 3i )
O A D (1+i )
–√3 , –1 Z √3 –1
—– — 2 Z1 —– , —
2 2 2 2 M
(2−i )
5π 5π cos 7π 7π
∴ z2 = cos π + i sin π , cos + i sin , + i sin
6 6 6 6 B C
− 3 i − 3 i
⇒ z2 = − 1 , + , − Again, DM = (2 − 1) + (− 1 − 1)2 = 1 + 4 = 5
2
2 2 2 2
2π 5π But BD = 2DM ⇒ BD = 2 5
Thus, ∠z1Oz2 = , ,π and 2 AC = BD ⇒ 2 AC = 2 5
3 6
⇒ AC = 5 and AC = 2 AM
10. z1 = 1 + i 3 = r (cos θ + i sin θ ) [let]
5
⇒ r cos θ = 1, r sin θ = 3 ⇒ 5 = 2 AM ⇒ AM =
2
⇒ r = 2 and θ = π /3
Complex Numbers 23

Now, let coordinate of A be (x + iy). and 2 (a + b) = ab + 1


But in a rhombus AD = AB, therefore we have ⇒ (a = b or a + b = 1)
AD 2 = AB2 and 2 (a + b) = ab + 1
⇒ (x − 1) + ( y − 1)2 = (x − 3)2 + ( y + 3)2
2

⇒ x + 1 − 2x + y2 + 1 − 2 y = x2 + 9 − 6x+ y2 + 9 + 6 y
2 If a = b , 2 (2a ) = a 2 + 1
⇒ 4x − 8 y = 16 ⇒ a − 4a + 1 = 0
2

⇒ x − 2y = 4 4 ± 16 − 4
⇒ a= =2 ± 3
⇒ x = 2y + 4 …(i) 2
If a + b = 1, 2 = a (1 − a ) + 1 ⇒ a 2 − a + 1 = 0
5 5
Again, AM = ⇒ AM 2 = 1 ± 1 −4
2 4 ⇒a= , but a and b ∈ R
5 2
⇒ (x − 2) + ( y + 1) =
2 2
4 ∴ Only solution when a=b
5 ⇒ a = b =2± 3
⇒ (2 y + 2)2 + ( y + 1)2 = [from Eq. (i)] ⇒ a = b =2− 3 [Q a , b ∈ (0, 1)]
4
5 13. Here, centre of circle is (1, 0) is also the mid-point of
⇒ 5 y2 + 10 y + 5 = diagonals of square
4
⇒ 20 y2 + 40 y + 15 = 0 Y
⇒ 4 y2 + 8 y + 3 = 0
z3
⇒ (2 y + 1) (2 y + 3) = 0
z1(2, 3)
⇒ 2 y + 1 = 0,2 y + 3 = 0
1 3
⇒ y=− , y=− (1, 0)
2 2 X
O z
On putting these values in Eq. (i), we get 0
 1  3
x = 2  −  + 4, x = 2  −  + 4
 2  2 z2 z4
⇒ x = 3, x = 1
 i  3i 
Therefore, A is either 3 −  or 1 −  . z1 + z2
 2  2 ⇒ = z0
2
Alternate Solution
Since, M is the centre of rhombus. ⇒ z2 = − 3 i [where, z0 = 1 + 0 i ]
∴ By rotating D about M through an angle of ± π /2 , we z3 − 1
and = e± iπ/ 2
get possible position of A. z1 − 1
C B  π π
⇒ z3 = 1 + (1 + 3i ) ⋅  cos ± i sin  [Q z1 = 2 + 3i ]
 2 2
= 1 ± i (1 + 3i ) = (1 + 3 ) ± i = (1 − 3 ) + i
M and z4 = (1 + 3 ) − i
(2– i) z2
14. Let Q be z2 and its reflection be the point P (z1 ) in the
D z1(1+i ) A (z3)
given line. If O (z ) be any point on the given line then by
z − (2 − i ) 1 z − (2 − i ) 1 definition OR is right bisector of QP.
⇒ 3 = (± i ) ⇒ 3 = (± i ) ∴ OP = OQ or |z − z1| = |z − z2|
−1 + 2 i 2 −1 + 2 i 2
1 1 ⇒ |z − z1|2 = |z − z2|2
⇒ z3 = (2 − i ) ± i (2i − 1) = (2 − i ) ± (−2 − i )
2 2 ⇒ (z − z1 ) (z − z1 ) = (z − z2) (z − z2)
(4 − 2i − 2 − i ) 4 − 2i + 2 + i 3 i
= , = 1 − i, 3 − ⇒ z (z1 − z2) + z (z1 − z2) = z1z1 − z2z2
2 2 2 2
 3   i Comparing with given line zb + zb = c
∴ A is either 1 − i or 3 −  .
 2   2 z1 − z2 z1 − z2 z1z1 − z2z2
= = = λ, [say]
b b c
12. Since, z1 , z2 and z3 form an equilateral triangle.
z1 − z2 z − z2 z z − z2z2
⇒ z12 + z22 + z32 = z1z2 + z2z3 + z3 z1 = b, 1 = b, 1 1 =c …(i)
λ λ λ
⇒ (a + i ) + (1 + ib) + (0) = (a + i ) (1 + ib) + 0 + 0
2 2 2
 z − z2   z − z2 
⇒ a − 1 + 2ia + 1 − b2 + 2ib = a + i (ab + 1) − b
2 ∴ z1b + z2b = z1  1  + z2  1 
 λ   λ 
⇒ (a 2 − b2) + 2i (a + b) = (a − b) + i (ab + 1) zz − z2z2
= 1 =c [from Eq. (i)]
⇒ a 2 − b2 = a − b λ
24 Complex Numbers

15. Since, z1 + z2 = − p and z1z2 = q B (z2) 18. Since, z1 , z2 and origin form an equilateral triangle.
z1 |z1| Q if z1 , z2, z3 from an equilateral triangle, then 
Now, = (cos α + i sin α )
z2 |z2|  
z12 + z22 + z32 = z1z2 + z2z3 + z3 z1 
z1 cos α + i sin α
⇒ = A (z1)
z2 1 O ⇒ z12 + z22 + 02 = z1z2 + z2 ⋅ 0 + 0 ⋅ z1
[Q|z1|=|z2|] ⇒ z12 + z22 = z1z2 ⇒ z12 + z22 − z1z2 = 0
Applying componendo and dividendo, we get 19. Since, z1 , z2, z3 are the vertices of an equilateral
z1 + z2 cos α + i sin α + 1
= triangle.
z1 − z2 cos α + i sin α − 1
 z + z2 + z3 
2 cos 2(α /2) + 2i sin (α /2) cos (α /2) ∴ Circumcentre (z0 ) = Centroid  1  ...(i)
=  3 
−2 sin 2(α /2) + 2i sin (α /2) cos (α /2)
Also, for equilateral triangle
2 cos (α /2) [cos (α /2) + i sin (α /2)]
= z12 + z22 + z32 = z1z2 + z2z3 + z3 z1 ... (ii)
2i sin (α /2)[cos (α /2) + i sin (α /2)]
cot (α /2) −p On squaring Eq. (i), we get
= = − i cot α /2 ⇒ = − i cot (α /2) 9z02 = z12 + z22 + z32 + 2 (z1z2 + z2z3 + z3 z1 )
i z1 − z2
p2 ⇒ 9z02 = z12 + z22 + z32 + 2 (z12 + z22 + z32 ) [from Eq. (ii)]
On squaring both sides, we get = − cot2(α /2)
(z1 − z2)2 ⇒ 3z02 = z12 + z22 + z32

p 2 20. For a complex number z, it is given that


⇒ = − cot2(α /2) z 4 − | z |4 = 4iz 2
(z1 + z2)2 − 4z1z2
⇒ z 4 − z 2z 2 = 4 iz 2
p2
⇒ = − cot2(α /2) ⇒ z (z − z ) (z + z ) = 4iz 2
2
p − 4q
2
So, either z = 0 or (z − z ) (z + z ) = 4i
2
⇒ p2 = − p2 cot2(α /2) + 4q cot2(α /2)
Now, Case-I, if z 2 = 0 and z = x + iy
⇒ p (1 + cot α /2) = 4q cot2(α /2)
2 2
So, x2 − y2 + 2ixy = 0 ⇒ x2 − y2 = 0
⇒ p cosec2(α /2) = 4q cot2(α /2) ⇒ p2 = 4q cos 2 α /2
2
and xy = 0 ⇒ x = y = 0
16. Since, triangle is a right angled isosceles triangle.
⇒ z = 0, which is not possible according to given
∴ Rotating z2 about z3 in anti-clockwise direction conditions.
through an angle of π / 2 , we get
Case-II, if (z − z ) (z + z ) = 4i and z = x + iy
z2 − z 3 | z 2 − z 3 | iπ / 2
= e A (z1) So, (2iy) (2x) = 4i
z1 − z3 | z1 − z 3 |
⇒ xy = 1 is an equation of rectangular hyperbola and for
where,|z2 − z3|= |z1 − z3| minimum value of | z1 − z2|2, the z1 and z2 must be
⇒ (z2 − z3 ) = i (z1 − z3 ) vertices of the rectangular hyperbola.
On squaring both sides, we get B (z3) C (z2)
Therefore, z1 = 1 + i and z2 = − 1 − i
(z2 − z3 ) = − (z1 − z3 )
2 2
∴Minimum value of| z1 − z2|2 = (1 + 1)2 + (1 + 1)2
⇒ z22 + z32 − 2z2z3 = − z12 − z32 + 2z1z3 4 + 4 =8
⇒ z12 + z22 − 2z1z2 = 2z1z3 + 2z2z3 − 2z32 − 2z1z2  kπ  kπ
21. Given, α k = cos   + i sin  
⇒ (z1 − z2)2 = 2{(z1z3 − z32 ) + (z2z3 − z1z2)}  7  7
⇒ (z1 − z2)2 = 2(z1 − z3 )(z3 − z2)  2 kπ   2 kπ 
= cos   + i sin  
17. We have, iz = ze i π/2
. This implies that iz is the vector  14   14 

obtained by rotating vector z in anti-clockwise direction ∴ α k are vertices of regular polygon having 14 sides.
through 90°. Therefore, OA ⊥ AB. So, Let the side length of regular polygon be a.
Y ∴ α k + 1 − α k = length of a side of the regular polygon
B
z + iz

=a …(i)
iz and α 4k−1 − α 4k− 2 = length of a side of the regular
A polygon
z
X′
O
X =a …(ii)
12

∑ αk+1 −αk
k =1 12 (a )
Y′ ∴ 3
= =4
3 (a )
Area of ∆ OAB =
1 1 1
OA × OB = |z ||iz | = |z |2 ∑ α 4k−1 − α 4k− 2
2 2 2 k =1
Complex Numbers 25

Topic 5 De-Moivre’s Theorem, Cube Roots 3. Key Idea Use, e i θ = cos θ + i sin θ
π
and nth Roots of Unity 3  1 π π i
Given, z = +   i = cos + i sin = e 6
1. Given expression 2  2 6 6
3
 2π 2π  so, (1 + iz + z5 + iz 8 )9
 1 + sin + i cos 
 9 9  π 5π 8π 9
2 π 2 π  i i i 
 1 + sin − i cos  = 1 + ie 6 + e 6 + ie 6 
 9 9   
 
3
  2 2π 2 2π   2π 2π   π π 5π π 4π 9 π
 sin 9 − i cos 9  + sin 9 + i cos 9  
2
 i i i i i   i 
=  = 1 + e 2 ⋅ e 6 + e 6 + e 2 ⋅ e 3  Q i = e 2 
2π 2π  
     
1 + sin − i cos
 9 9   i

i

i
11 π  9

= 1 + e 3 + e 6 + e 6 
 2π 2π   2π 2π 
3
 
 
 sin 9 + i cos 9  ⋅ sin 9 − i cos 9 + 1 
=    2π 2π   5π 5π 
2π 2π = 1 +  cos + i sin  +  cos + i sin 
 1 + sin − i cos    3 3  6 6
 9 9 
9
 2 π 2 π 
3
 2 π 2 π 
3
 11π 11π  
= sin + i cos  =  − i 2 sin + i cos  +  cos + i sin 
 9 9  9 9  6 6  
3 9
 2π 2π   1 i 3 3 1 3 i
= i3 cos − i sin = 1 − + − + i+ − 
 9 9  B (z2)
 2 2 2 2 2 2
3 9
  2π   2π   1 3i   π π
9
= − i cos  −  + i sin  −   = +  =  cos + i sin 
  9   9  2 2   3 3
π
 2π 2π  2
= − i cos − i sin O A (z1) = cos 3π + i sin 3π [Q for any natural number ‘n’
 3 3 
(cos θ + i sin θ )n = cos(nθ ) + i sin(nθ )]
[according the De-Movier’s theorem]
= −1
  1 3 
= − i  −  − i
  2 2  4. Given, x2 + x + 1 = 0
− 1 ± 3i
3 i 1 ⇒ x=
=− + = − ( 3 − i) 2
2 2 2
[Q Roots of quadratic equation ax2 + bx + c = 0
2. It is given that, there are two complex numbers z and w,
such that| z w| = 1 and arg (z ) − arg (w) = π / 2 −b± b2 − 4ac
are given by x = ]
∴ |z || w| = 1 [Q| z1 z2| = | z1 || z2|] 2a
−1 + 3 i
π ⇒ z0 = ω , ω 2 [where ω = and
and arg (z ) = + arg (w) 2
2
−1 − 3 i
Let| z | = r, then| w| =
1
…(i) ω2 =
r 2
π are the cube roots of unity and ω, ω 2 ≠ 1)
and let arg(w) = θ, then arg(z ) = + θ …(ii)
2 Now consider z = 3 + 6i z081 − 3i z093
So, we can assume = 3 + 6i − 3i (Qω3 n = (ω 2)3 n = 1)
z = rei ( π / 2 + θ ) …(iii) = 3 + 3i = 3(1 + i )
[Q if z = x + iy is a complex number, then it can be If ‘θ’ is the argument of z, then
written as z = reiθ where, r =|z|and θ = arg (z )] Im(z )
1 tan θ = [Qz is in the first quadrant]
and w = ei θ …(iv) Re(z )
r 3 π
1 = =1⇒ θ=
Now, z ⋅ w = re− i( π / 2 + θ ) ⋅ eiθ 3 4
r
5. Given that, z = cos θ + i sin θ = e iθ
= ei( − π / 2 − θ + θ ) = e− i( π / 2) = − i [Qe− i θ = cos θ − i sin θ] 15 15 15

and
1
z w = re i( π / 2 + θ ) ⋅ e− iθ ∴ ∑ Ιµ(ζ 2 µ −1 ) = ∑ Ιµ( ε ιθ )2 µ −1 = ∑ Ιµ ε ι ( 2 µ −1 ) θ
r µ =1 µ =1 µ =1

= ei( π / 2 + θ − θ ) = e i ( π / 2) = i = sin θ + sin 3 θ + sin 5 θ + ... + sin 29 θ


26 Complex Numbers

 θ + 29 θ   15 × 2 θ   z2 
n
sin   sin   ⇒   =i
n
 2   2 
=  z1 
 2 θ
sin   ∴ z1 and z2 are nth roots of unity.
 2 
z1n = z2n = 1
sin (15 θ )sin (15 θ ) 1  z2 
n
= = ⇒
sin θ 4 sin 2°   =1
 z1 
6. Let z = |a + bω + cω 2| ⇒ in = 1
⇒ z 2 = |a + bω + cω 2|2 = (a 2 + b2 + c2 − ab − bc − ca ) ⇒ n = 4k, where k is an integer.
1 10. We know that,
or z 2 = {(a − b)2 + (b − c)2 + (c − a )2} …(i)
2 1 3
ω=− + i
Since, a , b, c are all integers but not all simultaneously 2 2
equal. 334 365
 1 i 3  1 i 3
⇒ If a = b then a ≠ c and b ≠ c ∴ 4 + 5 − +  + 3 − + 
 2 2   2 2 
Because difference of integers = integer
⇒ (b − c)2 ≥ 1 {as minimum difference of two consecutive = 4 + 5 ω334 + 3 ω365
integers is (± 1)} also (c − a )2 ≥ 1 = 4 + 5 ⋅ (ω3 )111 ⋅ ω + 3 ⋅ (ω3 )121 ⋅ ω 2
and we have taken a = b ⇒ (a − b)2 = 0 = 4 + 5 ω + 3 ω2 [Q ω3 = 1]
1
From Eq. (i), z 2 ≥ (0 + 1 + 1) ⇒ z 2 ≥ 1 = 1 + 3 + 2 ω + 3 ω + 3 ω2
2 = 1 + 2 ω + 3 (1 + ω + ω 2) = 1 + 2 ω + 3 × 0
Hence, minimum value of|z | is 1 .
[Q 1 + ω + ω 2 = 0]
7. Given, (1 + ω ) = (1 + ω )
2 n 4 n
= 1 + (−1 + 3i ) = 3i
⇒ (− ω ) = (− ω )
n 2 n
[Qω = 1 and 1 + ω + ω = 0]
3 2 11. (1 + ω − ω 2)7 = (− ω 2 − ω 2)7 [Q 1 + ω + ω 2 = 0]
⇒ ωn = 1 = (−2ω 2)7 = (−2)7ω14= − 128 ω 2
⇒ n = 3 is the least positive value of n.
12. (1 + ω )7 = (1 + ω ) (1 + ω )6
1 1 1 = (1 + ω ) (−ω 2)6 = 1 + ω
8. Let ∆ = 1 −1 − ω 2
ω 2
⇒ A + Bω = 1 + ω
1 ω2 ω
⇒ A = 1, B = 1
Applying R2 → R2 − R1 ; R3 → R3 − R1 6
 2 kπ 2 kπ 
1 1 1  13. ∑ sin 7
− i cos
7 

k =1
=
 0 − 2 − ω 2
ω 2
−1  6
ω 2
− ω −  2 kπ 2 kπ 
∑ − i  cos
 0 1 1  = + i sin 
k =1
7 7 
= (−2 − ω 2)(ω − 1) − (ω 2 − 1)2
= − 2ω + 2 − ω3 + ω 2 − (ω 4 − 2ω 2 + 1)  6 i2kπ 
= − i  ∑ e 7  = − i { ei2π / 7 + e i4 π / 7 + e i6π /7
= 3 ω 2 − 3 ω = 3ω (ω − 1) [Q ω 4 = ω ] k = 1 
z π + e i8π / 7 + e i10π / 7 + e i12π / 7 }
9. Since, arg 1 =
z2 2  (1− ei12 π / 7 )
z1 π π = − i  ei 2 π / 7 
⇒ = cos + i sin = i  1− ei 2 π / 7 
z2 2 2
 ei2π / 7 − ei14π / 7 
z1n = −i  [Q e i14π /7= 1]
∴ = (i )n ⇒ i n = 1 [Q|z2| = |z1| = 1] i2π / 7
z2n  1−e 
⇒ n = 4k  e i2π / 7 − 1 
= −i  i2π / 7 
=i
Alternate Solution 1 − e 
z π 14. (P) PLAN e iθ⋅ e iα
=e i( θ + α )
Since, arg 2 =
z1 2 2 kπ  2π 
π i i  ( k + j)
 10 
z2 z2 i Given zk = e 10 ⇒ zk ⋅ z j = e
∴ = e 2
z1 z1
zk is 10th root of unity.
z2 ⇒ zk will also be 10th root of unity.
⇒ =i [Q|z1| = |z2| = 1]
z1
Taking, z j as zk, we have zk ⋅ z j = 1 (True)
Complex Numbers 27

e iθ  1 ω ω 2
(Q) PLAN = e i( θ − α )

i 
2kπ 2π  π  
e
 10
− 
10 
i ( k − 1) 15. Let A =  ω ω2 1 
z = zk / z1 = e = e 5
ω 2 1 ω 
i
π  
For k = 2; z = e 5 which is in the given set (False) 0 0 0
(R) PLAN Now, A 2 = 0 0 0 and Tr ( A ) = 0, A = 0
 
(i) 1 − cos 2 θ = 2 sin2 θ 0 0 0
(ii) sin 2 θ = 2 sin θ cos θ and A3 = 0
5 −1 z+1 ω ω2
(i) cos 36° =
4
⇒ ω z+ω 2
1 = [ A + zl] = 0
5 + 1| 1 − z1 || 1 − z2 | K | 1 − z9 |
(ii) cos 108° = ω2 1 z+ω
4 10
 2 πk 2 πk  ⇒ z3 = 0
NOTE | 1 − zk | = 1 − cos − i sin
 10 10  ⇒ z = 0, the number of z satisfying the given equation
π k π k πk  πk is 1.
=  2 sin  sin − i cos = 2 sin
 10  10 10  10 16. Here, Tr = (r − 1) (r − ω ) (r − ω )2] = (r3 − 1)
Now, required product is n
π 2π 3π 8π 9π
29 sin ⋅ sin ⋅ sin K sin ⋅ sin ∴ Sn = ∑ (r3 − 1)
10 10 10 10 10 r =1
10 2
 n (n + 1) 
 π 2π 3π 4π 
2
5π = −n
29 sin

sin sin sin  sin  2 
10 10 10 10  10
=
10 17. Since, cube root of unity are 1, ω , ω 2 given by
2
 π π 2π 2π   1 3  1 3
29 sin cos ⋅ sin cos  ⋅1 A (1, 0), B − , −  ,C  − , − 
 10 10 10 10   2 2  2 2
=
10
2 ⇒ AB = BC = CA = 3
1 π 1 2π 
29  sin ⋅ sin  Hence, cube roots of unity form an equilateral triangle.
2 5 2 5
= 18. Given, z p + q − z p − zq + 1 = 0 …(i)
10
25 (sin 36°⋅ sin 72° )2 ⇒ (z p − 1)(z q − 1) = 0
=
10 Since, α is root of Eq. (i), either α p − 1 = 0 or α q − 1 = 0
25 αp −1 αq − 1
= 2 (2 sin 36° sin 72° )2 ⇒ Either = 0 or =0 [as α ≠ 1]
2 × 10 α −1 α −1
22 ⇒ Either 1 + α + α 2 + ... + α p − 1 = 0
= (cos 36° − cos 108° )2
5 or 1 + α + K + αq −1 = 0
2
2 2   5 − 1  5 + 1  22 5 But α p − 1 = 0 and α q − 1 = 0 cannot occur
=   +  = ⋅ =1
5  4   4   5 4 simultaneously as p and q are distinct primes, so
neither p divides q nor q divides p, which is the
(S) Sum of nth roots of unity = 0 requirement for 1 = α p = α q.
1 + α + α 2 + α3 + K + α 9 = 0 19. Since, 1, a1 , a 2, ... , a n − 1 are nth roots of unity.
9
1+ ∑ α =0
k
⇒ (xn − 1) = (x − 1) (x − a1 ) (x − a 2) .... (x − a n − 1 )
k =1 xn − 1
9 ⇒ = (x − a1 ) (x − a 2) ..... (x − a n − 1 )
 2 kπ 2 kπ  x−1
1+ ∑  cos
 10
+ i sin
10 
 =0
⇒ xn − 1 + xn − 2 + ..... + x2 + x + 1
k =1
9
2 kπ = (x − a1 ) (x − a 2) ..... (x − a n − 1 )
1+ ∑ cos =0  xn − 1 n −1 
+ xn− 2 + ... + x + 1
k =1
10 Q x − 1 = x
9  
2 kπ
So, 1− ∑ cos
10
=2 On putting x = 1 , we get 1 + 1 + ... n times
k =1 = (1 − a1 ) (1 − a 2) ..... (1 − a n − 1 )
(P) → (i), (Q) → (ii), (R) → (iii), (S) → (iv) ⇒ (1 − a1 ) (1 − a 2)... (1 − a n − 1 ) = n
28 Complex Numbers

20. Since, n is not a multiple of 3, but odd integers and = (a + bω + cω 2) (a + bω + 2 cω 2 )


x3 + x2 + x = 0 ⇒ x = 0, ω , ω 2 = (a + bω + cω 2) (a + bω 2 + cω ) [Q ω = ω 2 and ω 2 = ω]
Now, when x = 0 = a 2 + abω 2 + acω + abω + b2ω3 + bcω 2 + acω 2
⇒ (x + 1)n − xn − 1 = 1 − 0 − 1 = 0 + bcω 4 + c2ω3
∴ x = 0 is root of (x + 1)n − xn − 1 = a + b + c + ab(ω + ω ) + bc(ω + ω ) + ac(ω + ω 2)
2 2 2 2 2 4

Again, when x = ω [as ω3 = 1]


⇒ (x + 1)n − xn − 1 = (1 + ω )n − ω n − 1 = a + b + c + ab(−1) + bc(−1) + ac(−1)
2 2 2

= −ω 2n − ω n − 1 = 0 [as ω + ω 2 = −1, ω 4 = ω]
[as n is not a multiple of 3 and odd] = a + b + c − ab − bc − ca
2 2 2

Similarly, x = ω 2 is root of {( x + 1)n − xn − 1 } 1


= {(a − b)2 + (b − c)2 + (c − a )2}
Hence, x = 0, ω , ω 2 are the roots of (x + 1)n − xn − 1 2
Thus, x3 + x2 + x divides (x + 1)n − xn − 1 . Q a , b and c are distinct non-zero integers.
21. Since, α, β are the complex cube roots of unity. For minimum value a = 1, b = 2 and c = 3
∴ We take α = ω and β = ω 2. 1
∴|a + bω + cω 2|2min = {12 + 12 + 2 2}
Now, xyz = (a + b)(aα + bβ )(aβ + bα ) 2
6
= (a + b)[a 2αβ + ab(α 2 + β 2) + b2αβ ] = = 3.00
2
= (a + b)[a 2(ω ⋅ ω 2) + ab(ω 2 + ω 4 ) + b2(ω ⋅ ω 2)] 2π
i
= (a + b)(a 2 − ab + b2) [Q1 + ω + ω 2 = 0 and ω3 = 1] 23. Printing error = e 3

= a3 + b3 |x|2|+ | y|2 + |z|2


Then, =3
22. Given, ω ≠ 1 be a cube root of unity, then|a + bω + cω 2|2 (a )2 + (b)2 + |c|2

= (a + bω + cω 2) (a + bω + cω 2), (Q zz = |z|2 )
i
NOTE Here, w = e 3 , then only integer solution exists.
2
Theory of Equations
Topic 1 Quadratic Equations
Objective Questions I (Only one correct option) 7. If α and β are the roots of the equation x2 − 2x + 2 = 0,
n
1. Suppose a , b denote the distinct real roots of the α
then the least value of n for which   = 1 is
quadratic polynomial x2 + 20x − 2020 and suppose c, d  β
denote the distinct complex roots of the quadratic (a) 2 (b) 5 (2019 Main, 8 April I)
polynomial x2 − 20x + 2020. Then the value of (c) 4 (d) 3
ac(a − c) + ad (a − d ) + bc(b − c) + bd (b − d ) is (2020 Adv.)
(a) 0 (b) 8000
8. The number of integral values of m for which the
equation (1 + m2)x2 − 2(1 + 3m)x + (1 + 8m) = 0, has no
(c) 8080 (d) 16000
real root is (2019 Main, 8 April II)
2. Let λ ≠ 0 be in R. If α and β are the roots of the equation, (a) 3 (b) infinitely many
x2 − x + 2λ = 0 and α and γ are the roots of the equation, (c) 1 (d) 2
βγ 9. The number of integral values of m for which the
3x2 − 10x + 27λ = 0, then is equal to
λ (2020 Main, 4 Sep II) quadratic expression, (1 + 2m) x2 − 2(1 + 3m)
(a) 36 (b) 9 x + 4(1 + m), x ∈ R, is always positive, is
(c) 27 (d) 18 (2019 Main, 12 Jan II)
(a) 6 (b) 8 (c) 7 (d) 3
3. The set of all real values of λ for which the quadratic
equations, (λ + 1)x − 4λx + 2 = 0 always have exactly
2 2 10. If λ be the ratio of the roots of the quadratic equation in
one root in the interval (0, 1) is (2020 Main, 3 Sep II) x, 3m2x2 + m(m − 4)x + 2 = 0, then the least value of m for
1
(a) (0, 2) (b) (− 3, − 1) which λ + = 1, is (2019 Main, 12 Jan I)
(c) (2, 4] (d) (1, 3] λ
(a) − 2 + 2 (b) 4 − 2 3
4. If α and β are the roots of the quadratic equation, (c) 4 − 3 2 (d) 2 − 3
 π
x2 + x sin θ − 2 sin θ = 0, θ ∈ 0,  , then 11. If one real root of the quadratic equation
 2
81x2 + kx + 256 = 0 is cube of the other root, then a value
α 12 + β12 of k is (2019 Main, 11 Jan I)
is equal to
(α −12 + β −12 )(α − β )24 (2019 Main, 10 April I) (a) 100 (b) 144 (c) −81 (d) −300
212 26 12. If 5, 5r , 5r 2 are the lengths of the sides of a triangle, then
(a) (b)
(sin θ + 8)12 (sin θ + 8)12 r cannot be equal to (2019 Main, 10 Jan I)
2 12
212 5 7 3 3
(c) (d) (a) (b) (c) (d)
(sin θ − 4) 12
(sin θ − 8) 6 4 4 2 4

5. Let p, q ∈R. If 2 − 3 is a root of the quadratic equation, 13. The value of λ such that sum of the squares of the roots
of the quadratic equation, x2 + (3 − λ )x + 2 = λ has the
x + px + q = 0, then
2
(2019 Main, 9 April I) least value is (2019 Main, 10 Jan II)
(a) q2 − 4 p − 16 = 0 (b) p 2 − 4q − 12 = 0 4 15
(a) (b) 1 (c) (d) 2
(c) p 2 − 4q + 12 = 0 (d) q2 + 4 p + 14 = 0 9 8
6. If m is chosen in the quadratic equation 14. The number of all possible positive integral values of α
(m2 + 1)x2 − 3x + (m2 + 1)2 = 0 such that the sum of its for which the roots of the quadratic equation,
roots is greatest, then the absolute difference of the 6x2 − 11x + α = 0 are rational numbers is
cubes of its roots is (2019 Main, 9 April II) (2019 Main, 9 Jan II)
(a) 10 5 (b) 8 5 (c) 8 3 (d) 4 3 (a) 5 (b) 2 (c) 4 (d) 3
30 Theory of Equations

15. Let α and β be two roots of the equation x2 + 2x + 2 = 0, 25. Let p and q be real numbers such that p ≠ 0, p3 ≠ q and
then α 15

15
is equal to (2019 Main, 9 Jan I) p3 ≠ − q. If α and β are non-zero complex numbers
(a) 256 (b) 512 satisfying α + β = − p and α 3 + β3 = q, then a quadratic
α β
(c) −256 (d) −512 equation having and as its roots is (2010)
β α
16. Let S = { x ∈ R : x ≥ 0 and 2| x − 3| + x ( x − 6) + 6 = 0 .
(a) ( p3 + q) x2 − ( p3 + 2q) x + ( p3 + q) = 0
Then, S (2018 Main)
(b) ( p3 + q) x2 − ( p3 − 2q) x + ( p3 + q) = 0
(a) is an empty set
(c) ( p3 − q) x2 − (5 p3 − 2q) x + ( p3 − q) = 0
(b) contains exactly one element
(d) ( p3 − q) x2 − (5 p3 + 2q) x + ( p3 − q) = 0
(c) contains exactly two elements
(d) contains exactly four elements 26. Let α, β be the roots of the equation x2 − px + r = 0 and
α
17. If α , β ∈ C are the distinct roots of the equation , 2 β be the roots of the equation x2 − qx + r = 0 . Then,
2
x2 − x + 1 = 0, then α 101 + β107 is equal to (2018 Main) the value of r is (2007, 3M)
(a) −1 (b) 0 (c) 1 (d) 2 2 2
(a) ( p − q) (2q − p ) (b) (q − p ) (2 p − q)
18. For a positive integer n, if the quadratic equation, 9 9
x(x + 1) + (x + 1) (x + 2) + ... + (x + n − 1) (x + n ) = 10n 2 2
(c) (q − 2 p ) (2q − p ) (d) (2 p − q) (2q − p )
has two consecutive integral solutions, then n is 9 9
equal to (2017 Main) 27. If a , b, c are the sides of a triangle ABC such that
(a) 12 (b) 9 (c) 10 (d) 11 x2 − 2 (a + b + c) x + 3λ (ab + bc + ca ) = 0 has real roots,
19. The sum of all real values of x satisfying the equation then (2006, 3M)

(c) λ ∈  ,  (d) λ ∈  , 
2
+ 4 x − 60 4 5 4 5 1 5
(x2 − 5x + 5)x = 1 is (2016 Main) (a) λ < (b) λ >
3 3  3 3  3 3
(a) 3 (b) − 4 (c) 6 (d) 5
π π 28. If one root is square of the other root of the equation
20. Let − < θ < − . Suppose α 1 and β1 are the roots of the x2 + px + q = 0, then the relation between p and q is
6 12
equation x2 − 2x secθ + 1 = 0 , and α 2 and β 2 are the roots (a) p3 − q(3 p − 1) + q2 = 0 (2004, 1M)
of the equation x2 + 2x tan θ − 1 = 0. If α 1 > β1 and (b) p3 − q(3 p + 1) + q2 = 0
α 2 > β 2, then α 1 + β 2 equals (2016 Adv.) (c) p3 + q(3 p − 1) + q2 = 0
(d) p3 + q(3 p + 1) + q2 = 0
(a) 2(secθ − tan θ) (b) 2secθ
(c) −2tanθ (d) 0 29. The set of all real numbers x for which x2 − |x + 2| + x > 0
21. Let α and β be the roots of equation x2 − 6x − 2 = 0. If is (2002, 1M)
a − 2a 8 (a) (− ∞ , − 2) ∪ (2, ∞ ) (b) (− ∞ , − 2 ) ∪ ( 2 , ∞ )
a n = α n − β n, for n ≥ 1, then the value of 10 is (c) (− ∞ , − 1) ∪ (1, ∞ ) (d) ( 2 , ∞ )
2a 9
(2015 Main) 30. The number of solutions of log 4 (x − 1) = log 2(x − 3) is
(a) 6 (b) – 6 (c) 3 (d) – 3 (a) 3 (b) 1 (2001, 2M)
22. In the quadratic equation p(x) = 0 with real coefficients (c) 2 (d) 0
has purely imaginary roots. Then, the equation 31. For the equation 3x2 + px + 3 = 0, p > 0, if one of the root
p[ p(x)] = 0 has (2014 Adv.) is square of the other, then p is equal to (2000, 1M)
(a) only purely imaginary roots (a) 1 /3 (b) 1 (c) 3 (d) 2 /3
(b) all real roots
(c) two real and two purely imaginary roots
32. If α and β (α < β) are the roots of the equation
(d) neither real nor purely imaginary roots x2 + bx + c = 0, where c < 0 < b, then (2000, 1M)
(a) 0 < α < β (b) α < 0 < β < |α|
23. Let α and β be the roots of equation px2 + qx + r = 0,
1 1 (c) α < β < 0 (d) α < 0 < |α|< β
p ≠ 0. If p, q and r are in AP and + = 4, then the
α β 33. The equation x + 1 − x − 1 = 4x − 1 has (1997C, 2M)
value of|α − β|is (2014 Main) (a) no solution
61 2 17 (b) one solution
(a) (b) (c) two solutions
9 9
34 2 13 (d) more than two solutions
(c) (d) 3 5
9 9 (log 2 x )2 + log 2 x −
34. The equation x 4 4 = 2 has (1989; 2M)
24. Let α and β be the roots of x2 − 6x − 2 = 0, with α > β. If
(a) atleast one real solution
a10 − 2a 8
a n = α n − β n for n ≥ 1 , then the value of is (b) exactly three real solutions
2a 9
(c) exactly one irrational solution
(a) 1 (b) 2 (c) 3 (d) 4 (2011) (d) complex roots
Theory of Equations 31

35. If α and β are the roots of x2 + px + q = 0 and α 4 , β 4are 44. If 2 + i 3 is a root of the equation x2 + px + q = 0, where
the roots of x − rx + s = 0, then
2
the equation p and q are real, then ( p, q) = (…,…). (1982, 2M)
x2 − 4qx + 2q2 − r = 0 has always (1989, 2M) 99
45. The coefficient of x in the polynomial
(a) two real roots (b) two positive roots (x − 1)(x − 2)... (x − 100) is.... (1982, 2M)
(c) two negative roots
(d) one positive and one negative root True/False
2 2
36. The equation x − =1 − has 46. If P (x) = ax2 + bx + c and Q (x) = − ax2 + bx + c, where
x−1 x−1 (1984, 2M)
ac ≠ 0, then P (x) Q (x) has atleast two real roots.
(a) no root (b) one root (1985, 1M)
(c) two equal roots (d) infinitely many roots
47. The equation 2x + 3x + 1 = 0 has an irrational root.
2
(x − a )(x − b)
37. For real x, the function will assume all real (1983, 1M)
(x − c)
values provided (1984, 3M) Analytical & Descriptive Questions
(a) a > b > c (b) a < b < c (c) a > c < b (d) a ≤ c ≤ b
48. If x2 − 10ax − 11b = 0 have roots c and d. x2 − 10cx − 11d = 0
38. The number of real solutions of the equation have roots a and b, then find a + b + c + d. (2006, 6M)
|x|2−3|x| + 2 = 0 is (1982, 1M)
(a) 4 (b) 1 (c) 3 (d) 2 49. If α , β are the roots of ax + bx + c = 0, (a ≠ 0) and
2

α + δ , β + δ are the roots of Ax2 + Bx + C = 0, ( A ≠ 0) for


39. Both the roots of the equation
some constant δ, then prove that
(x − b) (x − c) + (x − a ) (x − c) + (x − a ) (x − b) = 0
b2 − 4ac B2 − 4 AC
are always (1980, 1M) = (2000, 4M)
a2 A2
(a) positive (b) negative
(c) real (d) None of these 50. Let f (x) = Ax2 + Bx + C where, A , B, C are real
numbers. prove that if f (x) is an integer whenever x is
40. Let a > 0, b > 0 and c > 0. Then, both the roots of the
an integer, then the numbers 2 A , A + B and C are all
equation ax + bx + c = 0
2
(1979, 1M) integers. Conversely, prove that if the numbers
(a) are real and negative (b) have negative real parts 2 A , A + B and C are all integers, then f (x) is an integer
(c) have positive real parts (d) None of the above whenever x is an integer. (1998, 3M)

51. Find the set of all solutions of the equation


Assertion and Reason
2|y| − |2y − 1 − 1| = 2y − 1 + 1 (1997 C, 3M)
For the following question, choose the correct answer
from the codes (a), (b), (c) and (d) defined as follows : 52. Find the set of all x for which
2x 1
(a) Statement I is true, Statement II is also true; > (1987, 3M)
Statement II is the correct explanation of Statement I 2x + 5x + 2
2
x +1
(b) Statement I is true, Statement II is also true; 53. Solve| x2 + 4x + 3 | + 2x + 5 = 0 (1987, 5M)
Statement II is not the correct explanation of
Statement I 54. For a ≤ 0, determine all real roots of the equation
(c) Statement I is true; Statement II is false x2 − 2a|x − a|− 3a 2 = 0 (1986, 5M)
(d) Statement I is false; Statement II is true 2 2
−3 −3
41. Let a , b, c, p, q be the real numbers. Suppose α , β are the 55. Solve for x : (5 + 2 6 )x + (5 − 2 6 )x = 10 (1985, 5M)
roots of the equation x + 2 px + q = 0.
2
56. If one root of the quadratic equation ax + bx + c = 0 is 2

1 equal to the nth power of the other, then show that


and α , are the roots of the equation ax2 + 2bx + c = 0,
β 1 1
n n +1 n +1
(ac ) + (a c) n
+ b =0 (1983, 2M)
where β 2 ∉ { −1, 0, 1}.
Statement I ( p − q) (b − ac) ≥ 0
2 2
57. If α and β are the roots of x + px + q = 0 and γ , δ are the
2

Statement II b ∉ pa or c ∉ qa . (2008, 3M) roots of x2 + rx + s = 0, then evaluate (α − γ ) (β − γ )


(α − δ ) (β − δ ) in terms of p, q, r and s. (1979, 2M)
Fill in the Blanks 58. Solve 2 log x a + log ax a + 3 logb a = 0 ,
42. The sum of all the real roots of the equation where a > 0, b = a 2 x (1978 , 3M )
|x − 2|2 + |x − 2| − 2 = 0 is…… . (1997, 2M)
59. If α and β are the roots of the equation
43. If the products of the roots of the equation
x2 + px + 1 = 0 ; γ , δ are the roots of x2 + qx + 1 = 0, then
x2 − 3kx + 2e2log k − 1 = 0 is 7, then the roots are real for
q2 − p2 = (α − γ )(β − γ )(α + δ )(β + δ ) (1978, 2M)
k =K . (1984, 2M)
32 Theory of Equations

Passage Type Questions 60. a12 =


(a) a11 + 2a10 (b) 2a11 + a10
Let p, q be integers and let α , β be the roots of the equation,
(c) a11 − a10 (d) a11 + a10
x2 − x − 1 = 0 where α ≠ β. For n = 0, 1, 2, …… , let
a n = pα n + qβ n. 61. If a 4 = 28, then p + 2q =
FACT : If a and b are rational numbers and a + b 5 = 0, then (a) 14 (b) 7
a = 0 = b. (2017 Adv.) (c) 21 (d) 12

Topic 2 Common Roots


Objective Questions I (Only one correct option) 3. A value of b for which the equations x2 + bx − 1 = 0,
1. If α , β and γ are three consecutive terms of a x2 + x + b = 0 have one root in common is (2011)

non-constant GP such that the equations (a) − 2 (b) −i 3 (c) i 5 (d) 2


αx2 + 2βx + γ = 0 and x2 + x − 1 = 0 have a common root,
then, α(β + γ) is equal to (2019 Main, 12 April II) Fill in the Blanks
(a) 0 4. If the quadratic equations x2 + ax + b = 0 and
(b) αβ x + bx + a = 0 (a ≠ b) have a common root, then the
2
(c) αγ numerical value of a + b is… . (1986, 2M)
(d) βγ
2. If the equations x2 + 2x + 3 = 0 and ax2 + bx + c = 0, True/False
a , b, c ∈ R have a common root, then a : b : c is 5. If x − r is a factor of the polynomial f (x) = a n xn,
(a) 1 : 2 : 3 (b) 3 : 2 : 1 (2013 Main) + a n − 1 x n−1 + K + a 0 repeated m times (1 < m ≤ n ), then
(c) 1 : 3 : 2 (d) 3 : 1 : 2 r is a root of f ′ (x) = 0 repeated m times. (1983, 1M)

Topic 3 Transformation of Roots


Objective Question I (Only one correct option) Analytical & Descriptive Question
1. Let α , β be the roots of the equation,(x − a )(x − b) = c, c ≠ 0. 2. Let a, b and c be real number s with a ≠ 0 and let α , β be
Then the roots of the equation (x − α ) (x − β ) + c = 0 are the roots of the equation ax2 + bx + c = 0. Express the
(a) a , c (b) b , c (c) a , b (d) a + c, b + c (1992, 2M) roots of a3 x2 + abcx + c3 = 0 in terms of α , β. (2001, 4M)

Topic 4 Graph of Quadratic Expression


Objective Questions I (Only one correct option)
1. Let P(4, − 4) and Q(9, 6) be two points on the parabola, 3. If both the roots of the quadratic equation
y2 = 4x and let X be any point on the arc POQ of this x2 − mx + 4 = 0 are real and distinct and they lie in the
parabola, where O is the vertex of this parabola, such interval [1, 5] then m lies in the interval
that the area of ∆PXQ is maximum. Then, this (2019 Main, 9 Jan II)
maximum area (in sq units) is (2019 Main, 12 Jan I) (a) (4, 5) (b) (−5, − 4)
125 75 (c) (5, 6) (d) (3, 4)
(a) (b)
2 2 4. If a ∈ R and the equation −3 (x − [x])2 + 2 (x − [x])
625 125
(c) (d) + a 2 = 0 (where, [x] denotes the greatest integer ≤ x) has
4 4 no integral solution, then all possible values of a lie in
2. Consider the quadratic equation, (c − 5) x2− 2cx + (c − 4) the interval (2014 Main)
= 0, c ≠ 5. Let S be the set of all integral values of c for (a) (−1, 0) ∪ (0, 1) (b) (1, 2)
which one root of the equation lies in the interval (0, 2) (c) (−2 , − 1) (d) (−∞ , − 2) ∪ (2 , ∞ )
and its other root lies in the interval (2, 3). Then, the 5. For all ‘x’, x2 + 2ax + (10 − 3a ) > 0, then the interval in
number of elements in S is (2019 Main, 10 Jan I)
which ‘a’ lies is (2004, 1M)
(a) 11 (b) 10
(a) a < − 5 (b) −5 < a < 2 (c) a > 5 (d) 2 < a < 5
(c) 12 (d) 18
Theory of Equations 33

6. If b > a, then the equation (x − a ) (x − b) − 1 = 0 has Analytical & Descriptive Questions


(a) both roots in (a , b) (2000, 1M)
9. If x2 + (a − b)x + (1 − a − b) = 0 where a , b ∈ R, then find
(b) both roots in ( − ∞ , a )
the values of a for which equation has unequal real
(c) both roots in (b, + ∞ )
roots for all values of b. (2003, 4M)
(d) one root in (−∞ , a ) and the other in (b, ∞ )
7. If the roots of the equation x2 − 2ax + a 2 + a − 3 = 0 are 10. Let a , b, c be real. If ax2 + bx + c = 0 has two real roots α
real and less than 3, then (1999, 2M)
and β, where α < − 1 and β > 1, then show that
c b
(a) a < 2 1 + +  < 0
(b) 2 ≤ a ≤ 3 a a  (1995, 5M)
(c) 3 < a ≤ 4 11. Find all real values of x which satisfy x2 − 3x + 2 > 0 and
(d) a> 4 x2 − 2x − 4 ≤ 0. (1983, 2M)
8. Let f (x) be a quadratic expression which is positive for
all real values of x. If g (x) = f (x) + f ′ (x) + f ′ ′ (x), then for Integer & Numerical Answer Type Question
any real x (1990, 2M)
12. The smallest value of k, for which both the roots of the
(a) g (x) < 0 (b) g (x) > 0 equation x2 − 8kx + 16 (k2 − k + 1) = 0 are real, distinct
(c) g (x) = 0 (d) g (x) ≥ 0 and have values atleast 4, is …… . (2009)

Topic 5 Some Special Forms


Objective Questions I (Only one correct option) 7. The largest interval for which
1. The number of real roots of the equation x12 − x9 + x4 − x + 1 > 0 is (1982, 2M)
5 + |2 − 1| = 2 (2 − 2) is
x x x
(2019 Main, 10 April II) (a) −4 < x ≤ 0
(a) 1 (b) 3 (b) 0 < x < 1
(c) 4 (d) 2 (c) −100 < x < 100
(d) −∞ < x < ∞
2. All the pairs (x, y) that satisfy the inequality
sin 2 x − 2 sin x + 5 1 8. Let a , b, c be non-zero real numbers such that
2 ⋅ ≤ 1 also satisfy the equation
sin 2 y 1
∫0 (1 + cos x)(ax2 + bx + c)dx
4 (2019 Main, 10 April I) 8
(1981, 2M)
(a) 2|sin x| = 3 sin y (b) sin x = |sin y| 2
(c) sin x = 2 sin y (d) 2 sin x = sin y = ∫ (1 + cos 8 x)(ax2 + bx + c)dx
0
3. The sum of the solutions of the equation Then, the quadratic equation ax2 + bx + c = 0 has
| x − 2| + x ( x − 4) + 2 = 0 (x > 0) is equal to (a) no root in (0, 2)
(2019 Main, 8 April I) (b) atleast one root in (1, 2)
(a) 9 (b) 12 (c) a double root in (0, 2)
(c) 4 (d) 10 (d) two imaginary roots
4. The real number k for which the equation,
2x3 + 3x + k = 0 has two distinct real roots in [0, 1] Objective Questions II
(2013 Main) (One or more than one correct option)
(a) lies between 1 and 2 (b) lies between 2 and 3
9. Let S be the set of all non-zero real numbers α such that
(c) lies between − 1and 0 (d) does not exist
the quadratic equation αx2 − x + α = 0 has two distinct
5. Let a , b, c be real numbers, a ≠ 0. If α is a root of real roots x1 and x2 satisfying the inequality x1 – x2 < 1.
a 2x2 + bx + c = 0, β is the root of a 2x2 − bx − c = 0 and Which of the following interval(s) is/are a subset of S?
0 < α < β, then the equation a 2x2 + 2bx + 2c = 0 has a root (2015 Adv.)
(a)  – , –
1 
γ that always satisfies (b)  – , 0
(1989, 2M) 1 1

α+β β  2 5  5 
(a) γ = (b) γ = α +
(c)  0,
1 
(d) 
2 2 1 1
 , 
(c) γ = α (d) α < γ < β  5  5 2
6. If a + b + c = 0, then the quadratic equation 10. Let a ∈ R and f : R → R be given by f (x) = x5 − 5x + a.
3ax2 + 2bx + c = 0 has (1983, 1M)
Then,
(a) at least one root in (0, 1) (a) f (x) has three real roots, if a > 4
(b) one root in (2, 3) and the other in (−2, − 1) (b) f (x) has only one real root, if a > 4
(c) imaginary roots (c) f (x) has three real roots, if a < − 4
(d) None of the above (d) f (x) has three real roots, if −4 < a < 4
34 Theory of Equations

Passage Based Problems equation f ( x ) = 0 has a root in R. Consider f ( x ) = kex − x for


all real x where k is real constant. (2007, 4M)
Read the following passage and answer the questions.
14. The line y = x meets y = kex for k ≤ 0 at
Passage I
(a) no point
Consider the polynomial f ( x ) = 1 + 2x + 3x 2+ 4x3 . Let s be (b) one point
the sum of all distinct real roots of f ( x ) and let t =| s|. (c) two points
(2010) (d) more than two points
11. The real numbers s lies in the interval 15. The positive value of k for which kex − x = 0 has only one
(a)  − , 0 (b)  − 11, −  (c)  − , −  (d)  0,
1 3 3 1 1 root is

 4   4  4 2  4 1
(a) (b) 1
12. The area bounded by the curve y = f (x) and the lines e
(c) e (d) log e 2
x = 0, y = 0 and x = t , lies in the interval
(a)  , 3
3
(b)  , 
21 11
(c) (9, 10) (d)  0, 
21 16. For k > 0, the set of all values of k for which kex − x = 0
4   64 16   64  has two distinct roots, is
(a)  0,  (b)  , 1
1 1
13. The function f ′ (x) is
 e e 
(a) increasing in  − t, −  and decreasing in  − , t 
1 1
(c)  , ∞ 
1
 4  4  (d) (0, 1)
e 
(b) decreasing in  − t, −  and increasing in  − , t 
1 1
 4  4 
(c) increasing in (−t , t )
True/False
(d) decreasing in (−t , t ) 17. If a < b < c < d, then the roots of the equation (x − a )
(x − c) + 2 (x − b) (x − d ) = 0 are real and distinct.
Passage II
(1984, 1M)
If a continuous function f defined on the real line R,
assumes positive and negative values in R, then the Analytical & Descriptive Question
equation f ( x ) = 0 has a root in R. For example, if it is 18. Let −1 ≤ p < 1. Show that the equation 4x3 − 3x − p = 0
known that a continuous function f on R is positive at some has a unique root in the interval [1/2, 1] and identify it.
point and its minimum values is negative, then the (2001, 4M)

Answers
Topic 1 Topic 2
1. (d) 2. (d) 3. (d) 4. (a) 1. (d) 2. (a) 3. (b) 4. –1
5. (b) 6. (b) 7. (c) 8. (a) 5. False
9. (c) 10. (c) 11. (d) 12. (b)
Topic 3
13. (d) 14. (d) 15. (c) 16. (c)
1. (c) 2. x = α 2β, αβ 2
17. (c) 18. (d) 19. (a) 20. (c)
21. (c) 22. (d) 23. (d) 24. (c) Topic 4
25. (b) 26. (d) 27. (a) 28. (a) 1. (d) 2. (a) 3. (a) 4. (a)
29. (b) 30. (b) 31. (c) 32. (b) 5. (b) 6. (d) 7. (a) 8. (b)
33. (a) 34. (b) 35. (a) 36. (a) 9. a > 1 11. x ∈ [1 − 5, 1 ) ∪ [1 + 5, 2 )
37. (d) 38. (a) 39. (c) 40. (b)
12. k = 2
41. (b) 42. 4 43. k =2 44. ( −4, 7 )
45. –5050 46. True 47. False 48. 1210 Topic 5
 2 1 1. (a) 2. (b) 3. (d) 4. (d)
51. y ∈ { −1 } ∪ [1, ∞ ) 52. x ∈ ( −2, − 1 ) ∪  − , − 
 3 2 5. (d) 6. (a) 7. (d) 8. (b)
53. −4 and ( − 1 − 3 ) 54. x = {a (1 − 2 ), a ( 6 − 1 )} 9. (a,d) 10. (b, d) 11. (c) 12. (a)
55. ±2, ± 2 57. (q − s ) 2 − rqp − rsp + sp 2 + qr 2 13. (b) 14. (b) 15. (a) 16. (a)
58. x = a −1/ 2 − 4 /3
59. q − p
2 2 1 
or a 60. (d) 17. True 18. x = cos cos−1 p
3 
61. (d)
Hints & Solutions
Topic 1 Quadratic Equations Q The root x =
1
∈ (0, 1) for λ = 3
1. Given quadratic polynomials, x + 20x − 2020 and
2 5
∴ λ ∈ (1 , 3].
x2 − 20x + 2020 having a , b distinct real and c, d distinct
complex roots respectively. Hence, option (d) is correct.
So, a + b = − 20, ab = − 2020 4. Given quadratic equation is
and c + d = 20, cd = 2020  π
x2 + x sin θ − 2 sin θ = 0, θ ∈ 0, 
Now, ac(a − c) + ad (a − d ) + bc(b − c) + bd (b − d )  2
= a 2(c + d ) − a (c2 + d 2) + b2(c + d ) − b(c2 + d 2) and its roots are α and β.
= (c + d ) (a 2 + b2) − (c2 + d 2)(a + b) So, sum of roots = α + β = − sin θ
= (c + d )[(a + b)2 − 2ab] − (a + b) [(c + d )2 − 2cd ] and product of roots = αβ = − 2 sin θ
= 20 [(20)2 + 4040] + 20 [(20)2 − 4040] ⇒ αβ = 2(α + β ) …(i)
= 2 × 20 × (20)2 = 40 × 400 = 16000 α 12 + β12
Now, the given expression is −12
2. It is given that α is a common roots of given quadratic (α + β −12)(α − β)24
equations α 12 + β12 α 12 + β12
x2 – x + 2λ = 0 and 3x2 – 10x + 27λ = 0 = = 12
 1 1 24  β + α 12
∴ 3 α – 10 α + 27λ = 0
2  12 + 12 (α − β)  12 12  (α − β)24
α β   α β 
3 α 2 − 3 α + 6λ = 0
12 12
– + –  αβ   αβ 
= 2
= 
0 – 7α + 21λ = 0 ⇒ α = 3λ  (α + β ) − 4αβ
 (α − β ) 
2

So, 9λ2 – 3λ + 2λ = 0
12
1 1  2(α + β) 
⇒ λ= ⇒α= [Qλ ≠ 0] =  [from Eq. (i)]
 (α + β ) − 8 (α + β) 
9 3 2

1
2×  
12
 
12
2λ 9 =2 =
2
=
2
[Q α + β = − sin θ]
As αβ = 2λ ⇒ β = =  
α 1 3  (α + β ) − 8  − sin θ − 8
3 212
1 =
9× (sin θ + 8)12
9λ 9 =3
and αγ = 9λ ⇒ γ = =
α 1 5. Given quadratic equation is x2 + px + q = 0, where
3 p, q ∈R having one root 2 − 3 , then other root is 2 + 3
2 (conjugate of 2 − 3 ) [Q irrational roots of a quadratic
×3
βγ 3 equation always occurs in pairs]
∴ = = 18
λ 1 So, sum of roots = − p = 4 ⇒ p = −4
9 and product of roots = q = 4 − 3 ⇒ q = 1
3. Given quadratic equations Now, from options p2 − 4q − 12 = 16 − 4 − 12 = 0
f (x) = (λ2 + 1)x2 − 4λx + 2 = 0 have exactly one root in 6. Given quadratic equation is
the interval (0, 1).
(m2 + 1)x2 − 3x + (m2 + 1)2 = 0 …(i)
So, D > 0 ⇒ 16λ2 − 4(λ2 + 1)2 > 0
Let the roots of quadratic Eq. (i) are α and β, so
⇒ 8λ − 8 > 0 ⇒ λ2 > 1
2
3
α+β= and αβ = m2 + 1
⇒ λ ∈ (−∞ , − 1) ∪ (1, ∞ ) …(i) m2 + 1
and f (0) f (1) < 0
According to the question, the sum of roots is greatest
⇒ 2(λ2 + 1 − 4λ + 2) < 0 ⇒ λ2 − 4λ + 3 < 0
and it is possible only when ‘‘(m2 + 1) is minimum’’ and
⇒ (λ − 3)(λ − 1) < 0 ⇒ λ ∈ (1, 3) …(ii) ‘‘minimum value of m2 + 1 = 1, when m = 0’’.
From Eqs, (i) and (ii), we get ∴α + β = 3 and αβ = 1, as m = 0
λ ∈ (1, 3) Now, the absolute difference of the cubes of roots
And if λ = 3, then the quadratic equation is = |α 3 − β3|
10x2 − 12x + 2 = 0 = |α − β||α 2 + β 2 + αβ|
⇒ 5 x2 − 6 x + 1 = 0 = (α + β )2 − 4αβ |(α + β )2 − αβ|
1 = 9 − 4 |9 − 1|= 8 5
⇒ x = 1,
5
36 Theory of Equations

7. Given, α and β are the roots of the quadratic equation, From inequalities Eqs. (iii) and (iv), the integral values
of m are 0, 1, 2, 3, 4, 5, 6
x − 2x + 2 = 0
2
Hence, the number of integral values of m is 7.
⇒ (x − 1)2 + 1 = 0
⇒ (x − 1)2 = − 1 10. Let the given quadratic equation in x,
⇒ x−1 = ± i [where i = −1] 3m2x2 + m(m − 4)x + 2 = 0, m ≠ 0 have roots α and β, then
m(m − 4) 2
⇒ x = (1 + i ) or (1 − i ) α +β = − 2
and αβ =
3m 3m2
Clearly, if α = 1 + i, then β = 1 − i α
n n Also, let =λ
α  1 + i β
According to the question   = 1 ⇒   =1
 β 1 − i 1 α β
n Then, λ + =1 ⇒ + =1 (given)
 (1 + i )(1 + i ) λ β α
⇒   =1 [by rationalization]
 (1 − i )(1 + i )  ⇒ α 2 + β 2 = αβ
 1 + i 2 + 2i 
n
 2i 
n ⇒ (α + β )2 = 3αβ
⇒   =1⇒   =1⇒ i =1
n
m (m − 4)2
2
 1 − i2   2 ⇒ =3
2
9m4 3m2
So, minimum value of n is 4. [Q i = 1]
4
⇒ (m − 4)2 = 18 [Qm ≠ 0]
Key Idea ⇒ m − 4 = ±3 2
8.
(i) First convert the given equation in quadratic equation. ⇒ m =4±3 2
(ii) Use, Discriminant, D = b 2 − 4 ac < 0 The least value of m = 4 − 3 2
Given quadratic equation is 11. Given quadratic equation is
(1 + m2)x2 − 2(1 + 3m)x + (1 + 8m) = 0 …(i) 81x2 + kx + 256 = 0
Now, discriminant Let one root be α, then other is α 3 .
D = [−2(1 + 3m)]2 − 4(1 + m2)(1 + 8m) k 256
Now, α + α 3 = − and α ⋅ α 3 =
= 4 [(1 + 3m)2 − (1 + m2)(1 + 8m)] 81 81
b
= 4 [1 + 9m2 + 6m − (1 + 8m + m2 + 8m3 )] [Q for ax2 + bx + c = 0, sum of roots = −
a
= 4 [−8m3 + 8m2 − 2m] c
and product of roots = ]
= − 8m(4m2 − 4m + 1) = − 8m(2m − 1)2 a
4
According to the question there is no solution of the  4
quadratic Eq. (i), then ⇒ α4 =  
 3
D <0 4
⇒ α=±
∴ −8m(2m − 1)2 < 0 ⇒ m > 0 3
∴ k = − 81 (α + α 3 )
So, there are infinitely many values of ‘m’ for which, = − 81 α (1 + α 2)
there is no solution of the given quadratic equation.
 4  16
= − 81  ±  1 +  = ± 300
9. The quadratic expression  3  9
ax2 + bx + c, x ∈ R is always positive,
if a > 0 and D < 0.
12. Let a = 5, b = 5r and c = 5r 2
So, the quadratic expression We know that, in a triangle sum of 2 sides is always
(1 + 2m) x2 − 2 (1 + 3m)x + 4(1 + m), x ∈ R will be greater than the third side.
always positive, if 1 + 2m > 0 …(i) ∴ a + b > c; b + c > a and c + a > b
and D = 4(1 + 3m)2 − 4(2m + 1) 4(1 + m) < 0 …(ii) Now, a+ b>c
From inequality Eq. (i), we get ⇒ 5 + 5r > 5r 2 ⇒ 5r 2 − 5r − 5 < 0
1 ⇒ r − r −1 <0
2
m>− …(iii)
2  1 − 5   1 + 5 
From inequality Eq. (ii), we get ⇒ r −    r −   <0
  2    2 
1 + 9m2 + 6m − 4 (2m2 + 3m + 1) < 0
⇒ m2 − 6m − 3 < 0 [Q roots of ax2 + bx + c = 0 are given by
⇒ [m − (3 + 12 )][m − (3 − 12 )] < 0 − b ± b2 − 4ac
x= and r 2 − r − 1 = 0
6 ± 36 + 12 2a
[Q m2 − 6m − 3 = 0 ⇒ m = = 3 ± 12]
2 1± 1+4 1± 5
⇒ 3 − 12 < m < 3 + 12 …(iv) ⇒r = = ]
2 2
Theory of Equations 37

1 − 5 1 + 5 14. For the roots of quadratic equation ax2 + bx + c = 0 to be


⇒ r ∈ ,  ...(i)
 2 2  rational D = (b2 − 4ac) should be perfect square.
+ – + In the equation 6x2 − 11x + α = 0
1– √ 5 1+ √ 5 a = 6, b = − 11 and c = α
2 2 ∴For roots to be rational
Similarly, b+ c>a D = (− 11)2 − 4(6) (α) should be a perfect square.
⇒ 5r + 5r 2 > 5 ⇒ D(α) = 121 − 24α should be a perfect square
⇒ r2 + r − 1 > 0 Now,
  − 1 − 5    −1 + 5   D(1) = 121 − 24 = 97 is not a perfect square.
r −    r −   >0 D(2) = 121 − 24 × 2 = 73 is not a perfect square.
  2    2 
D(3) = 121 − 24 × 3 = 49 is a perfect square.
 2 −1 ± 1 + 4 − 1 ± 5 
Q r + r − 1 = 0 ⇒ r = =  D(4) = 121 − 24 × 4 = 25 is a perfect square.
 2 2  D(5) = 121 − 24 × 5 = 1 is a perfect square.
 −1 − 5   −1 + 5  and for α ≥ 6, D(α) < 0, hence imaginary roots.
⇒ r ∈  − ∞,  ∪ , ∞ …(ii)
 2   2  ∴For 3 values of α (α = 3, 4, 5), the roots are rational.
+ – + 15. We have, x2 + 2x + 2 = 0
–1– √ 5 –1+ √ 5 −2 ± 4 −8
⇒ x= [Q roots of ax2 + bx + c = 0 are
2 2 2
and c+ a >b −b± b2 − 4ac
given by x = ]
⇒ 5r 2 + 5 > 5r 2a
⇒ r2 − r + 1 > 0 ⇒ x = −1 ± i
2 2 Let α = − 1 + i and β = − 1 − i.
1  1  1
⇒ r2 − 2 ⋅ r +   + 1 −   > 0
2  2  2 Then, α 15 + β15 = (−1 + i )15 + (− 1 − i )15

 1
2
3 = − [(1 − i )15 + (1 + i )15 ]
⇒ r −  + > 0   1 i  
15 15
 2 4 i    1
= −  2  −   +  2 +  
⇒ r ∈R … (iii)   2 2    2 2   
From Eqs. (i), (ii) and (iii), we get
  π π 
15 
 −1 + 5 1 + 5  2  cos − i sin   
r ∈ ,   4 
 2 2  = −  4
15

   π π  
+  2  cos + i sin   
  4 4  
–∞ –1– √ 5 1– √ 5 –1+ √ 5 1+ √ 5 ∞
 15π 15π   15π 15π  
= − ( 2 )15  cos − i sin  +  cos + i sin 
4  
2 2 2 2
 4 4   4
7
and is the only value that does not satisfy. [using De’ Moivre’s theorem
4 (cos θ ± i sin θ )n = cos nθ ± i sin nθ, n ∈ Z ]
13. Given quadratic equation is  15π   1 
= − ( 2 )15 2 cos = − ( 2 )15 2 ×
x2 + (3 − λ )x + 2 = λ  4   2 
x + (3 − λ )x + (2 − λ ) = 0
2
… (i)  15π  π π 1 
Let Eq. (i) has roots α and β, then α + β = λ − 3 and Q cos 4 = cos 4π − 4  = cos 4 = 2 
 
αβ = 2 − λ
b = − ( 2 )16 = − 28
[Q For ax2 + bx + c = 0, sum of roots = −
a = − 256.
c
and product of roots = ] Alternate Method
a
α 15 + β15 = (−1 + i )15 + (−1 − i )15
Now, α 2 + β 2 = (α + β )2 − 2αβ
= − [(1 − i )15 + (1 + i )15 ]
= (λ − 3)2 − 2(2 − λ )
 (1 − i )16 (1 + i )16 
= λ2 − 6λ + 9 − 4 + 2λ =− +
 1−i 1 + i 
= λ2 − 4λ + 5 = (λ2 − 4λ + 4) +1
= (λ − 2)2 + 1  [(1 − i )2]8 [(1 + i )2]8 
=− +
Clearly, a + β 2 will be least when λ = 2.
2
 1−i 1 + i 
38 Theory of Equations

 [1 + i 2 − 2i ]8 [1 + i 2 + 2 i ]8  or
=− + 
 1−i 1+ i  III. x2 − 5x + 5 = − 1 and x2 + 4x − 60 = Even integer.
 (−2 i )8 (2 i )8  Case I When x2 + 4x − 60 = 0
=− + ⇒ x2 + 10x − 6x − 60 = 0
 1−i 1 + i 
⇒ x(x + 10) − 6(x + 10) = 0
 1 1  ⇒ (x + 10) (x − 6) = 0
= − 28  +  [Q i 4n = 1, n ∈ Z ]
 1 − i 1 + i  ⇒ x = − 10or x = 6
 2  2  Note that, for these two values of x, x2 − 5x + 5 ≠ 0
= − 256  2
= − 256 = − 256
2  x2 − 5 x + 5 = 1
1 − (i )  Case II When
⇒ x2 − 5 x + 4 = 0
16. We have, 2| x − 3| + x ( x − 6) + 6 = 0
⇒ x − 4x − x + 4 = 0
2

Let x − 3 = y ⇒ x(x − 4) − 1 (x − 4) = 0
⇒ x = y+3
⇒ (x − 4) (x − 1) = 0
∴ 2| y| + ( y + 3)( y − 3) + 6 = 0
⇒ x = 4or x = 1
⇒ 2| y| + y2 − 3 = 0
Case III When x2 − 5 x + 5 = − 1
⇒ | y|2 + 2| y| − 3 = 0
⇒ x2 − 5 x + 6 = 0
⇒ (| y| + 3)(| y| − 1) = 0
⇒ x − 2x − 3x + 6 = 0
2
⇒ | y| ≠ − 3 ⇒ | y| = 1
⇒ x(x − 2) − 3(x − 2) = 0
⇒ y= ±1 ⇒ x −3 = ±1
⇒ (x − 2) (x − 3) = 0
⇒ x = 4, 2 ⇒ x = 16, 4 ⇒ x = 2 or x = 3
17. We have, α , β are the roots of x2 − x + 1 = 0 Now, when x = 2, x2 + 4x − 60 = 4 + 8 − 60 = − 48, which is
Q Roots of x2 − x + 1 = 0 are −ω ,−ω 2 an even integer.
∴ Let α = − ω and β = − ω 2
When x = 3, x2 + 4x − 60 = 9 + 12 − 60 = − 39, which is not
⇒ α 101 + β107 = (− ω )101 + (− ω 2)107 = − (ω101 + ω 214 ) an even integer.
= − (ω 2 + ω ) [Q ω3 = 1]
= − (−1) [Q1 + ω + ω 2 = 0] Thus, in this case, we get x = 2.
=1 Hence, the sum of all real values of
18. Given quadratic equation is x = − 10 + 6 + 4 + 1 + 2 = 3
x(x + 1) + (x + 1)(x + 2) + ... + (x + n − 1) (x + n ) =10n 20. Here, x − 2x secθ + 1 = 0 has roots α 1 and β1.
2

⇒ (x2 + x2 + ... + x2) + [(1 + 3 + 5 + ... + (2n − 1)]x


2 sec θ ± 4 sec2 θ − 4
+ [(1 ⋅ 2 + 2 ⋅ 3 + ... + (n − 1)n ] = 10n ∴ α 1 , β1 =
n (n 2 − 1) 2 ×1
⇒ nx + n x +
2 2
− 10n = 0
3 2 sec θ ± 2|tan θ|
=
n −1
2
2
⇒ x2 + nx + − 10 = 0
3  π π
Since, θ ∈  − ,−  ,
 6 12
⇒ 3x2 + 3nx + n 2 − 31 = 0
2 sec θ m 2 tan θ
Let α and β be the roots. i.e. θ ∈ IV quadrant =
2
Since, α and β are consecutive.
∴ α 1 = sec θ − tan θ and β1 = sec θ + tan θ [as α 1 > β1]
∴ |α − β| = 1 ⇒ (α − β )2 = 1
and x2 + 2x tan θ − 1 = 0 has roots α 2 and β 2 .
Again, (α − β ) = (α + β ) − 4αβ
2 2
2
 n 2 − 31 −2 tan θ ± 4 tan 2 θ + 4
 − 3n  i.e. α 2, β 2 =
⇒ 1=  − 4  2
 3   3 
∴ α 2 = − tan θ + sec θ
4
⇒ 1 = n 2 − (n 2 − 31) ⇒ 3 = 3n 2 − 4n 2 + 124 and β 2 = − tan θ − sec θ [as α 2 > β 2]
3
⇒ n 2 = 121 ⇒ n = ± 11 Thus, α 1 + β 2 = −2 tan θ
∴ n = 11 [Qn > 0] 21. Given, α and β are the roots of the equation
x 2 + 4 x − 60 x2 − 6x − 2 = 0.
19. Given, (x − 5x + 5)
2
=1
∴ a n = α n − β n for n ≥ 1
Clearly, this is possible when
a10 = α 10 − β10
I. x2 + 4x − 60 = 0 and x2 − 5x + 5 ≠ 0
a 8 = α 8 − β8
or
a 9 = α 9 − β9
II. x2 − 5x + 5 = 1
Theory of Equations 39

Now, consider −q 4r 4r 4
Now, α+β= = = =−
a10 − 2a 8 α 10
− β − 2(α − β )
10 8 8 p p −9 r 9
=
2a 9 2(α 9 − α 9 ) r r 1
and αβ = = =
p −9 r −9
α 8 (α 2 − 2) − β 8 (β 2 − 2) 16 4 16 + 36
= ∴ (α − β )2 = (α + β )2 − 4 αβ = + =
2(α 9 − β 9 ) 81 9 81
α 8 ⋅ 6 α − β 86 β 6 α 9 − 6 β 9 6 52
= = = =3 ⇒ (α − β )2 =
2(α 9 − β 9 ) 2(α 9 − 6 β 9 ) 2 81
2
Q α and β are the roots of  ⇒ |α − β| = 13
 x2 − 6x − 2 = 0 or x2 = 6x + 2  9
⇒ α = 6 α + 2 ⇒ α − 2 = 6 α
2 2
a10 − 2a 8 (α 10 − β10 ) − 2 (α 8 − β 8 )
and β 2 = 6 β + 2 ⇒ β 2 − 2 = 6 β  24. =
2a 9 2 (α 9 − β 9 )
Alternate Solution
α 8 (α 2 − 2) − β 8 (β 2 − 2)
Since, α and β are the roots of the equation =
2(α 9 − β 9 )
x2 − 6x − 2 = 0.
Qα is root of x2 − 6 x − 2 = 0 ⇒ α 2 − 2 = 6α
or x2 = 6 x + 2
[and β is root of x2 − 6 x − 2 = 0 ⇒ β 2 − 2 = 6 β]
∴ α 2 = 6α + 2
α 8 (6 α ) − β 8 (6 β ) 6 (α 9 − β 9 )
⇒ α 10 = 6 α 9 + 2 α 8 ...(i) = = =3
2 (α 9 − β 9 ) 2 (α 9 − β 9 )
Similarly, β10 = 6 β 9 + 2 β 8 …(ii)
α 2 + β2
On subtracting Eq. (ii) from Eq. (i), we get 25. Sum of roots = and product = 1
αβ
α 10 − β10 = 6(α 9 − β 9 ) + 2(α 8 − β 8 ) (Q a n = α n − β n)
⇒ a10 = 6a 9 + 2a 8 Given, α + β = − p and α 3 + β3 = q

⇒ a10 − 2a 8 = 6a 9 ⇒ (α + β ) (α 2 − αβ + β 2) = q
a − 2a 8 −q
⇒ 10 =3 ∴ α 2 + β 2 − αβ = ...(i)
2a 9 p
and (α + β )2 = p2
22. If quadratic equation has purely imaginary roots, then
coefficient of x must be equal to zero. ⇒ α 2 + β 2 + 2αβ = p2 ...(ii)
Let p(x) = ax + b with a, b of same sign and a , b ∈ R.
2 From Eqs. (i) and (ii), we get
p[ p(x)] = a (ax2 + b)2 + b p3 − 2q p3 + q
Then, α 2 + β2 = and αβ =
3p 3p
p(x) has imaginary roots say ix.
Then, also ax2 + b ∈ R and (ax2 + b)2 > 0 ( p3 − 2q) x
∴ Required equation is, x2 − +1 =0
∴ a (ax2 + b)2 + b ≠ 0, ∀ x ( p3 + q)
⇒ ( p3 + q) x2 − ( p3 − 2q) x + ( p3 + q) = 0
Thus, p [ p(x)] ≠ 0, ∀ x
26. The equation x2 − px + r = 0 has roots α, β and the
23. PLAN If ax 2 + bx + c = 0 has roots α and β, then α + β = − b / a α
c
and α β = . Find the values of α + β and αβ and then put equation x2 − qx + r = 0 has roots , 2β.
a 2
in (α − β )2 = (α + β )2 − 4αβ to get required value. ⇒ r = αβ and α + β = p,
Given, α and β are roots of px + qx + r = 0, p ≠ 0.
2 α 2q − p 2 (2 p − q)
and + 2β = q ⇒ β = and α =
−q r 2 3 3
∴ α+β= , αβ = …(i)
p p 2
⇒ αβ = r = (2q − p) (2 p − q)
9
Since, p, q and r are in AP.
∴ 2q = p + r ...(ii) 27. Since, roots are real, therefore D ≥ 0
1 1 α+β ⇒ 4 (a + b + c)2 − 12λ (ab + bc + ca ) ≥ 0
Also, + =4 ⇒ =4
α β αβ ⇒ (a + b + c)2 ≥ 3λ (ab + bc + ca )
−q 4r
⇒ α + β = 4 αβ ⇒ = [from Eq. (i)] ⇒ a 2 + b2 + c2 ≥ (ab + bc + ca ) (3λ − 2)
p p a 2 + b2+ c2
⇒ q = − 4r ⇒ 3λ − 2 ≤ …(i)
ab + bc + ca
On putting the value of q in Eq. (ii), we get
b2 + c2 − a 2
⇒ 2 (−4r ) = p + r ⇒ p = − 9r Also, cos A = <1
2bc
40 Theory of Equations

⇒ b2 + c2 − a 2< 2bc ⇒ − 1 = − p/3


Similarly, c + a − b < 2ca
2 2 2 ⇒ p=3
and a 2 + b2 − c2 < 2ab 32. Given, c<0 < b
⇒ a 2 + b2 + c2 < 2 (ab + bc + ca ) Since, α + β = −b …(i)
a 2 + b2 + c2 and αβ = c …(ii)
⇒ <2 …(ii)
ab + bc + ca From Eq. (ii), c < 0 ⇒ α β < 0
From Eqs. (i) and (ii), we get ⇒ Either α is –ve, β is −ve or α is + ve, β is – ve.
4 From Eq. (i), b > 0 ⇒ − b < 0 ⇒ α + β < 0 ⇒ the sum is
3λ − 2 < 2 ⇒ λ< negative.
3
⇒ Modulus of negative quantity is > modulus of positive
28. Let the roots of x2 + px + q = 0 be α and α 2. quantity but α < β is given.
⇒ α + α2 = − p and α 3 = q Therefore, it is clear that α is negative and β is positive
and modulus of α is greater than modulus of β
⇒ α (α + 1) = − p
⇒ α < 0 < β < |α|
⇒ α {α + 1 + 3α (α + 1)} = − p3
3 3
[cubing both sides] NOTE This question is not on the theory of interval in which root
⇒ q (q + 1 − 3 p) = − p3 lie, which appears looking at first sight. It is new type and first
time asked in the paper. It is important for future. The actual
⇒ p3 − (3 p − 1)q + q2 = 0
type is interval in which parameter lie.
29. Given, x2 − |x + 2| + x > 0 …(i)
33. Since, x + 1 − x − 1 = 4x − 1
Case I When x+ 2 ≥0
⇒ (x + 1) + (x − 1) − 2 x2 − 1 = 4x − 1
∴ x2 − x − 2 + x > 0 ⇒ x2 − 2 > 0
⇒ x < − 2 or x > 2 ⇒ 1 − 2 x = 2 x2 − 1 ⇒ 1 + 4 x2 − 4 x = 4 x 2 − 4
5
⇒ x ∈ [−2, − 2 ) ∪ ( 2 , ∞ ) …(ii) ⇒ 4x = 5 ⇒ x =
4
Case II When x + 2 < 0 But it does not satisfy the given equation.
∴ x2 + x + 2 + x > 0 Hence, no solution exists.
⇒ x2 + 2 x + 2 > 0 3
(log 2 x )2 + log 2 x −
5
34. Given, x 4 4 = 2
⇒ (x + 1)2 + 1 > 0
3 5
which is true for all x. ⇒ (log 2 x)2 + log 2 x − = log x 2
4 4
∴ x ≤ − 2 or x ∈ (−∞ , − 2) …(iii) 3 5 1
From Eqs. (ii) and (iii), we get ⇒ (log 2 x) + log 2 x − =
2
4 4 2 log 2 x
x ∈ (−∞ , − 2 ) ∪ ( 2 , ∞ )
⇒ 3(log 2 x)3 + 4(log 2 x)2 − 5(log 2 x) − 2 = 0
30. Given, log 4 (x − 1) = log 2(x − 3) = log 41/ 2 (x − 3) Put log 2 x = y
⇒ log 4 (x − 1) = 2 log 4 (x − 3) ∴ 3 y3 + 4 y2 − 5 y − 2 = 0
⇒ ( y − 1) ( y + 2) (3 y + 1) = 0
⇒ log 4 (x − 1) = log 4 (x − 3)2 ⇒ y = 1, −2 , −1 / 3
⇒ (x − 3)2 = x − 1 ⇒ log 2 x = 1, − 2, − 1 / 3
⇒ x2 + 9 − 6x = x − 1 1 1
⇒ x = 2, 1/3 ,
⇒ x2 − 7x + 10 = 0 2 4
⇒ (x − 2)(x − 5) = 0 35. Since, α , β are the roots of x2 + px + q = 0 and α 4 , β 4 are
⇒ x = 2, or x = 5 the roots of x2 − rx + s = 0.
⇒ α + β = − p ; α β = q; α 4 + β 4 = r and α 4β 4 = s
⇒ x = 5 [Q x = 2 makes log (x − 3) undefined].
Hence, one solution exists. Let roots of x2 − 4qx + (2q2 − r ) = 0 be α ′ and β′
Now, α ′ β ′ = (2q2 − r ) = 2 (αβ )2 − (α 4 + β 4 )
31. Let α , α 2 be the roots of 3x2 + px + 3 = 0
= − (α 4 + β 4 − 2α 2β 2) = − (α 2 − β 2)2 < 0
Now, S = α + α 2 = − p /3,
⇒ Roots are real and of opposite sign.
P = α3 = 1
2 2
⇒ α = 1, ω , ω 2 36. Given, x − =1 − ⇒ x=1
x−1 x−1
Now, α + α 2 = − p/3
But at x = 1, the given equation is not defined.
⇒ ω + ω 2 = − p/3
Hence, no solution exist.
Theory of Equations 41

x2 − (a + b)x + ab 41. Given, x2 + 2 px + q = 0


37. Let y =
x−c ∴ α + β = −2 p ... (i)
⇒ yx − cy = x2 − (a + b)x + ab αβ = q ... (ii)
⇒ x2 − (a + b + y)x + (ab + cy) = 0 And ax2 + 2bx + c = 0
For real roots, D ≥ 0 1 2b
∴ α+ =− ... (iii)
⇒ (a + b + y)2 − 4(ab + cy) ≥ 0 β a
⇒ (a + b)2 + y2 + 2(a + b) y − 4ab − 4cy ≥ 0 α c
and = ... (iv)
⇒ y2 + 2(a + b − 2c) y + (a − b)2 ≥ 0 β a
which is true for all real values of y. Now, ( p2 − q) (b2 − ac)
∴ D ≤0  1
2 
4 (a + b − 2c) − 4 (a − b) ≤ 0
2 2  α + β 2   α +  α 
 β
=   − αβ  −  a2
 
⇒ 4 (a + b − 2c + a − b)(a + b − 2c − a + b) ≤ 0  −2   2  β
 
 
⇒ (2a − 2c)(2b − 2c) ≤ 0  
⇒ (a − c)(b − c) ≤ 0 (α − β )2  1
2
= α −  . a ≥ 0
2
⇒ (c − a )(c − b) ≤ 0 16  β
⇒ c must lie between a and b ∴ Statement I is true.
i.e. a ≤ c ≤ b or b ≤ c ≤ a  α + β a
Again, now pa = −   a = − (α + β )
38. Since, | x|2 − 3|x| + 2 = 0  2  2
⇒ (|x| − 1) (|x| − 2) = 0 a 1
and b=− α + 
⇒ |x| = 1, 2 2  β
∴ x = 1, − 1, 2, − 2 1
Since, pa ≠ b ⇒ α+ ≠α +β
Hence, four real solutions exist. β

39. (x − a ) (x − b) + (x − b) (x − c) + (x − c) (x − a ) = 0 ⇒ β 2 ≠ 1, β ≠ { −1, 0, 1}, which is correct.


Similarly, if c ≠ qa
⇒ 3x2 − 2 (a + b + c) x + (ab + bc + ca ) = 0
α  1
Now, discriminant = 4 (a + b + c) − 12 (ab + bc + ca )
2
⇒ a ≠ a α β ⇒ α β −  ≠ 0
β  β
= 4 { a 2 + b2 + c2 − ab − bc − ca }
1
= 2 {(a − b)2 + (b − c)2 + (c − a )2} ⇒ α ≠ 0 and β − ≠0
β
which is always positive.
⇒ β ≠ { −1 , 0 , 1 }
Hence, both roots are real.
Statement II is true.
40. Since, a , b, c > 0 and ax2 + bx + c = 0
Both Statement I and Statement II are true. But
−b b2 − 4ac Statement II does not explain Statement I.
⇒ x= ±
2a 2a 42. Given,|x − 2|2 + |x − 2| − 2 = 0
Case I When b2 − 4ac > 0 Case I When x ≥ 2
−b b2 − 4ac ⇒ (x − 2)2 + (x − 2) − 2 = 0
⇒ x= − ⇒ x2 + 4 − 4 x + x − 2 − 2 = 0
2a 2a
b2 − 4ac ⇒ x2 − 3 x = 0
−b
and + both roots, are negative. ⇒ x (x − 3) = 0
2a 2a
Case II When b − 4ac = 0
2 ⇒ x = 0, 3 [0 is rejected]
−b ⇒ x=3 …(i)
⇒ x= , i.e. both roots are equal and negative
2a Case II When x < 2
Case III When b2 − 4ac < 0 ⇒ { − (x − 2)}2 − (x − 2) − 2 = 0
−b 4ac − b2 ⇒ (x − 2)2 − x + 2 − 2 = 0
⇒ x= ±i
2a 2a ⇒ x2 + 4 − 4 x − x = 0
have negative real part. ⇒ x2 − 4x − (x − 4) = 0
∴ From above discussion, both roots have negative real ⇒ x(x − 4) − 1 (x − 4) = 0
parts. ⇒ (x − 1) (x − 4) = 0
42 Theory of Equations

⇒ x = 1, 4 [4 is rejected] 49. Since, α + β = −


b
, αβ =
c
⇒ x=1 …(ii) a a
B C
Hence, the sum of the roots is 3 + 1 = 4. and α+δ+β+δ=− , (α + δ ) (β + δ ) =
A A
Alternate Solution
Now, α − β = (α + δ ) − (β + δ )
Given,| x − 2|2 + | x − 2| − 2 = 0 ⇒ (α − β )2 = [(α + δ ) − (β + δ )]2
⇒ (| x − 2| + 2) (| x − 2| − 1) = 0 ⇒ (α + β )2 − 4αβ = (α + δ + β + δ )2 − 4(α + δ ) (β + δ )
∴ | x − 2| = − 2, 1 [neglecting –2] 2 2
 b 4c  B  4C
⇒ | x − 2| = 1 ⇒ x = 3, 1 ⇒ −  − = −  −
 a a  A  A
⇒ Sum of the roots = 4 b2 4c B2 4C
⇒ − = 2−
43. Since, x − 3kx + 2e
2 2 log k
− 1 = 0 has product of roots 7. a2 a A A
⇒ 2e2 log k − 1 = 7 b − 4ac B − 4 AC
2 2

⇒ e2 log e k = 4 ⇒ =
a2 A2
⇒ k2 = 4
⇒ k =2 [neglecting −2]
50. Suppose f (x) = Ax2 + Bx + C is an integer, whenever x
is an integer.
44. If 2 + i 3 is one of the root of x2 + px + q = 0. Then,
∴ f (0), f (1), f (−1) are integers.
other root is 2 − i 3.
⇒ − p =2 + i 3 + 2 − i 3 = 4 ⇒ C , A + B + C , A − B + C are integers.
and q = (2 + i 3 ) (2 − i 3 ) = 7 ⇒ C , A + B, A − B are integers.
⇒ ( p, q) = (−4, 7) ⇒ C , A + B, ( A + B) − ( A − B) = 2 A are integers.
45. The coefficient of x99 in (x − 1)(x − 2)... (x − 100) Conversely, suppose 2 A , A + B and C are integers.
= − (1 + 2 + 3 + ... + 100) Let n be any integer. We have,
 n (n − 1) 
=−
100
(1 + 100) = −50(101) = −5050 f (n ) = An 2 + Bn + C = 2 A + ( A + B) n + C
2  2 

46. P (x) ⋅ Q (x) = (ax2 + bx + c) (−ax2 + bx + c) Since, n is an integer, n (n − 1) / 2 is an integer. Also,


2 A , A + B and C are integers.
Now, D1 = b2 − 4ac and D2 = b2 + 4ac We get f (n ) is an integer for all integer n.
Clearly, D1 + D2 = 2b2 ≥ 0
51. Given, 2| y | − |2y − 1 − 1| = 2y − 1 + 1
∴ Atleast one of D1 and D2 is (+ ve). Hence, atleast two
Case I When y ∈ (−∞ , 0]
real roots.
Hence, statement is true. ∴ 2− y + (2y − 1 − 1) = 2y − 1 + 1
⇒ 2−y = 2
47. Given, 2x2 + 3x + 1 = 0
⇒ y = − 1 ∈ (−∞ , 0] …(i)
Here, D = (3)2 − 4 ⋅ 2 ⋅ 1 = 1 which is a perfect square.
Case II When y ∈(0, 1]
∴ Roots are rational.
Hence, statement is false. ∴ 2y + (2y − 1 − 1) = 2y − 1 + 1
⇒ 2y = 2
48. Here, a + b = 10c and c + d = 10a
⇒ y = 1 ∈ (0, 1] …(ii)
⇒ (a − c) + (b − d ) = 10 (c − a )
⇒ (b − d ) = 11 (c − a ) …(i) Case III When y ∈ (1, ∞ )
Since, ‘c’ is the root of x2 − 10ax − 11b = 0 ∴ 2y − 2y − 1 + 1 = 2y − 1 + 1
⇒ c2 − 10ac − 11b = 0 …(ii) ⇒ 2y − 2 ⋅ 2y − 1 = 0
Similarly, ‘a’ is the root of ⇒ 2y − 2y = 0 true for all y > 1 …(iii)
x − 10cx − 11d = 0
2
From Eqs. (i), (ii) and (iii), we get
⇒ a 2 − 10ca − 11d = 0 …(iii) y ∈{ −1} ∪ [1, ∞ ).
On subtracting Eq. (iv) from Eq. (ii), we get 2x 1
52. Given, >
(c2 − a 2) = 11 (b − d ) …(iv) 2 x2 + 5 x + 2 x + 1
∴ (c + a ) (c − a ) = 11 × 11 (c − a ) [from Eq. (i)] 2x 1
⇒ − >0
⇒ c + a = 121 (2x + 1) (x + 2) (x + 1)
∴ a + b + c + d = 10c + 10a 2x (x + 1) − (2x + 1) (x + 2)
⇒ >0
= 10 (c + a ) = 1210 (2x + 1) (x + 2) (x + 1)
Theory of Equations 43

− (3x + 2) 56. Let α , β are roots of ax2 + bx + c = 0


⇒ > 0 ; using number line rule
(2x + 1) (x + 1) (x + 2)
Given, α = βn
– + – + – 1/( n + 1 )
c c  c
⇒ αβ = ⇒ βn + 1 = ⇒ β= 
–2 –1 – 2
3 – 1
2 a a  a
 2 1 It must satisfy ax2 + bx + c = 0
∴ x ∈ (−2, − 1) ∪  − , − 
 3 2 2 /( n + 1 ) 1 /( n + 1 )
 c  c
i.e. a  + b  + c=0
53. Given,| x + 4x + 3|+ 2x + 5 = 0
2  a  a
Case I x2 + 4x + 3 > 0 ⇒ (x < − 3 or x > − 1) a . c2/( n + 1) b. c1/( n + 1)
⇒ + 1/( n + 1) + c = 0
∴ x + 4x + 3 + 2x + 5 = 0 a 2/( n + 1)
2
a
⇒ x2 + 6x + 8 = 0 ⇒ (x + 4) (x + 2) = 0 c1/( n + 1)  a. c 1/( n + 1 )
c. a1/( n + 1) 
⇒  1/( n + 1) + b + 1/( n + 1)  = 0
⇒ x = − 4, − 2 [but x < − 3 or x > − 1] a1/( n + 1)  a c 
∴ x = − 4 is the only solution. …(i)
⇒ a n/( n + 1)c1/( n + 1) + b + cn/( n + 1)a1 / ( n + 1) = 0
Case II x2 + 4x + 3 < 0 ⇒ (−3 < x < − 1)
⇒ (a nc)1/( n + 1) + (cna )1/( n + 1) + b = 0
∴ − x2 − 4 x − 3 + 2 x + 5 = 0
⇒ x2 + 2x − 2 = 0 ⇒ (x + 1)2 = 3 57. Since, α , β are the roots of x2 + px + q = 0
⇒ | x + 1| = 3 and γ , δ are the roots of x2 + rx + s = 0
⇒ x = − 1 − 3, −1 + 3 [but x ∈ (−3, − 1)] ∴ α + β = − p, αβ = q and γ + δ = − r, γδ = s
∴ x = − 1 − 3 is the only solution. …(ii) Now, (α − γ )(α − δ )(β − γ )(β − δ )
From Eqs. (i) and (ii), we get = [α 2 − (γ + δ ) α + γδ ][β 2 − (γ + δ ) β + γδ ]
x = − 4 and (−1 − 3) are the only solutions. = (α 2 + rα + s)(β 2 + rβ + s)
54. Here, a ≤0 = (αβ )2 + r (α + β )αβ + s(α 2 + β 2) + αβr 2 + rs(α + β ) + s2
Given, x2 − 2 a | x − a | − 3 a 2 = 0
= q2 − rqp + s( p2 − 2q) + qr 2 − rsp + s2
Case I When x ≥ a
= (q − s)2 − rqp − rsp + sp2 + qr 2
⇒ x2 − 2a (x − a ) − 3a 2 = 0
58. The given equation can be rewritten as
⇒ x2 − 2ax − a 2 = 0
2 1 3
⇒ x = a ± 2a + + = 0 [Q b = a 2x, given]
log a x log a ax log a a 2x
[as a (1 + 2 ) < a and a (1 − 2 ) > a]
2 1 3
∴ Neglecting x = a (1 + 2 ) as x ≥ a ⇒ + + =0
log a x 1 + log a x 2 + log a x
⇒ x = a (1 − 2 ) …(i)
2 1 3
Case II When x < a ⇒ x + 2a (x − a ) − 3a = 0
2 2 ⇒ + + = 0, where t = log a x
t 1+ t 2+ t
⇒ x2 + 2 ax − 5a 2 = 0 ⇒ x = − a ± 6a
⇒ 2 (1 + t ) (2 + t ) + 3 t (1 + t ) + t (2 + t ) = 0
[as a ( 6 − 1) < a and a (−1 − 6 ) > a]
⇒ 6 t 2 + 11 t + 4 = 0
∴ Neglecting x = a (−1 − 6 ) ⇒ x = a ( 6 − 1) ...(ii)
⇒ (2 t + 1) (3 t + 4) = 0
From Eqs. (i) and (ii), we get
1 4
x = { a (1 − 2 ), a ( 6 − 1)} ⇒ t=− or −
2 3
2 2
−3 −3
55. Given, (5 + 2 6 )x + (5 − 2 6 )x = 10 …(i) 1 4
∴ log a x = − or log a x = −
x2 − 3 x2 − 3 1 2 3
Put y = (5 + 2 6 ) ⇒ (5 − 2 6 ) =
y ⇒ x = a −1/ 2 or x =a −4/3

1 59. Since, α + β = − p, αβ = 1 and γ + δ = − q, γδ = 1


From Eq. (i), y + = 10
y
Now, (α − γ )(β − γ )(α + δ )(β + δ )
⇒ y2 − 10 y + 1 = 0 ⇒ y=5±2 6
2
−3 = {αβ − γ (α + β ) + γ 2}{αβ + δ (α + β ) + δ 2}
⇒ (5 + 2 6 )x =5 + 2 6
2
−3
= {1 − γ (− p) + γ 2}{1 + δ ( − p) + δ 2}
or (5 + 2 6 )x =5 −2 6
= (1 + γ 2 + γp)(1 − δp + δ 2) = (− qγ + γp)(−δp − δq)
⇒ x2 − 3 = 1 or x2 − 3 = − 1
⇒ x = ± 2 or x = ± 2 [Q γ 2 + qγ + 1 = 0 and δ 2 + qδ + 1 = 0]

⇒ x = ± 2, ± 2 = (q2 − p2)(γδ ) = q2 − p2 [Q γ δ = 1]
44 Theory of Equations

60. α2 = α + 1 3. If a1x2 + b1x + c1 = 0


β =β + 1
2
and a 2x2 + b2x + c2 = 0
a n = pα + qβn n
have a common real root, then
= p(α n − 1 + α n − 2) + q(β n − 1 + β n − 2) ⇒ (a1c2 − a 2c1 )2 = (b1c2 − b2c1 ) (a1b2 − a 2b1 )
= an − 1 + an − 2 x2 + bx − 1 = 0
∴  have a common root.
∴ a12 = a11 + a10 x2 + x + b = 0 
1+ 5 1− 5
61. α= ,β = ⇒ (1 + b)2 = (b2 + 1) (1 − b)
2 2
⇒ b2 + 2b + 1 = b2 − b3 + 1 − b
a 4 = a3 + a 2
⇒ b3 + 3b = 0
= 2a 2 + a1
∴ b (b2 + 3) = 0
= 3a1 + 2a 0
⇒ b = 0, ± 3 i
28 = p(3α + 2) + q(3β + 2)
3  3 5 4. Given equations are x2 + ax + b = 0 and
28 = ( p + q) + 2 + ( p − q) 
2   2  x2 + bx + a = 0 have common root
∴ p− q =0 On subtracting above equations, we get
and
7
( p + q) × = 28 (a − b) x + (b − a ) = 0
2 ⇒ x=1
⇒ p+ q =8 ∴ x = 1 is the common root.
⇒ p= q =4 ⇒ 1 + a + b =0
∴ p + 2q = 12 ⇒ a + b = −1

5. Since, (x − r ) is a factor of the polynomial


Topic 2 Common Roots
f (x) = a nxn + a n − 1xn − 1 + ... + a 0
1 Given α, β and γ are three consecutive terms of a
Then, x = r is root of f ′ (x) = 0 repeated (m − 1) times.
non-constant GP.
Let α = α, β = αr , γ = αr 2, { r ≠ 0, 1} Hence, statement is false.
and given quadratic equation is
αx2 + 2 βx + γ = 0 …(i) Topic 3 Transformation of Roots
On putting the values of α,β, γ in Eq. (i), we get 1. Given, α , β are the roots of (x − a )(x − b) − c = 0
αx2 + 2αrx + αr 2 = 0
⇒ (x − a )(x − b) − c = (x − α ) (x − β )
⇒ x2 + 2rx + r 2 = 0
⇒ (x − a )(x − b) = (x − α )(x − β ) + c
⇒ (x + r )2 = 0
⇒ x=−r ⇒ a , b are the roots of equation (x − α )(x − β ) + c = 0

Q The quadratic equations αx2 + 2 βx + γ = 0 and 2. Since, ax2 + bx + c = 0 has roots α and β.
x2 + x − 1 = 0 have a common root, so x = − r must be root
⇒ α + β = − b /a
of equation x2 + x −1 = 0, so
r2 − r − 1 = 0 …(ii) and αβ = c / a
Now, α (β + γ ) = α (αr + αr 2) Now, a x + abcx + c3 = 0
3 2
…(i)
= α 2 (r + r 2) On dividing the equation by c2, we get
From the options, a3 2 abcx c3
x + 2 + 2 =0
βγ = αr ⋅ αr 2 = α 2r3 = α 2 (r + r 2) c2 c c
[Q r 2 − r − 1 = 0 ⇒ r3 = r + r 2]  ax
2
 ax
∴ α (β + γ ) = βγ ⇒ a   + b   + c=0
 c  c
2. Given equations are x2 + 2x + 3 = 0 …(i) ax
⇒ = α , β are the roots
and ax + bx + c = 0
2
…(ii) c
Since, Eq. (i) has imaginary roots, so Eq. (ii) will also c c
⇒ x = α , β are the roots
have both roots same as Eq. (i). a a
a b c ⇒ x = α β α , α β β are the roots
Thus, = =
1 2 3 ⇒ x = α 2β , αβ 2 are the roots
Hence, a : b : c is 1 : 2 : 3.
Theory of Equations 45

Divide the Eq. (i) by a3 , we get 1


= [4(2t − 6) + 4(t 2 − 9) + 1(6t 2 − 18t ]
3 2
b c  c
x2 + ⋅ ⋅ x+   =0 1
a a  a = |[8t − 24 + 4t 2 − 36 + 6t 2 − 18t ]|
2
⇒ x2 − (α + β ) ⋅ (αβ ) x + (αβ )3 = 0
= |5t 2 − 5t − 30| = |5(t + 2) (t − 3)|
⇒ x2 − α 2βx − αβ 2 x + (αβ )3 = 0
Now, as X is any point on the arc POQ of the parabola,
⇒ x (x − α 2β ) − αβ 2 (x − α 2β ) = 0 therefore ordinate of point X, 2t ∈ (− 4, 6) ⇒ t ∈ (− 2, 3).
⇒ (x − α 2β )(x − αβ 2) = 0 ∴ Area of ∆PXQ = − 5(t + 2) (t − 3) = − 5t 2 + 5t + 30
⇒ x = α β , αβ which is the required answer.
2 2
[Q| x − a | = − (x − a ), if x < a]
Alternate Solution The maximum area (in square units)
Since, a3 x2 + abcx + c3 = 0 25 − 4(− 5) (30)  125
=− = 4
− abc ± (abc)2 − 4 . a3 ⋅ c3  4(− 5) 
⇒ x=
2 a3 [Q Maximum value of quadratic expression
D
− (b/a )(c/a ) ± (b/a ) (c/a ) − 4(c/a )
2 2 3 ax2 + bx + c, when a < 0 is − ]
⇒ x= 4a
2
2. Let f (x) = (c − 5)x2 − 2 cx + (c − 4) = 0.
(α + β ) (α β ) ± (α + β ) (α β ) − 4(α β )
2 2 3
Then, according to problem, the graph of y = f (x) will be
⇒ x=
2 either of the two ways, shown below.
(α + β )(αβ ) ± α β (α + β )2 − 4 αβ
⇒ x=
2
 (α + β ) ± (α − β )2  O 2 3
⇒ x = αβ  
 2  Or

 (α + β ) ± (α − β ) 
⇒ x = αβ
 2  O 2 3
α + β + α − β α + β − α + β 
⇒ x = αβ ,
 2 2 
In both cases f (0). f (2) < 0 and f (2) f (3) < 0
2α 2β 
⇒ x = αβ , Now, consider f (0) f (2) < 0
 2 2 
⇒ (c − 4) [4 (c − 5) − 4c + (c − 4)] < 0
⇒ x = α 2 β , α β 2 which is the required answer.
⇒ (c − 4) (c − 24) < 0
Topic 4 Graph of Quadratic Expression ⇒ c ∈ (4, 24) … (i)
+ – +
1. Given parabola is y = 4x, 2
4 24
Since, X lies on the parabola, so let the coordinates of X
be (t 2, 2t ). Thus, the coordinates of the vertices of the Similarly, f (2) ⋅ f (3) < 0
triangle PXQ are P (4,–4), X (t 2,2t ) and Q (9, 6). ⇒ [4 (c − 5) − 4c + (c − 4)]
Y [9(c − 5) − 6c + (c − 4)] < 0
Q(9,6)
X (t 2,2t) y 2=4x ⇒ (c − 24) (4c − 49) < 0
+ – +
49/4 24
X′ X
O
 49 
⇒ c ∈ , 24 …(ii)
4 

P(4,–4)
From Eqs. (i) and (ii), we get
Y′  49 
c ∈ , 24
4 
4 −4 1
1 ∴Integral values of c are 13, 14, ……, 23.
∴ Area of ∆PXQ = t2
2t 1
2 Thus, 11 integral values of c are possible.
9 6 1
46 Theory of Equations

3. According to given information, we have the following 5. As we know, ax2 + bx + c > 0 for all x ∈ R, iff
graph a > 0 and D < 0.
Y
Given equation is x2 + 2ax + (10 − 3a ) > 0, ∀ x ∈ R
Now, D <0
⇒ 4a 2 − 4(10 − 3a ) < 0
X ⇒ 4(a 2 + 3a − 10) < 0
O 1 5
⇒ (a + 5)(a − 2) < 0 ⇒ a ∈ (−5, 2)
Now, the following conditions should satisfy
(i) D > 0 ⇒ b2 − 4ac > 0
y = (x – a)(x – b) –1
⇒ m2 − 4 × 1 × 4 > 0
⇒ m2 − 16 > 0 6.
α a b β
⇒ (m − 4) (m + 4) > 0
1
⇒ m ∈ (− ∞ , − 4) ∪ (4, ∞ )
(ii) The vertex of the parabola should lie From graph, it is clear that one of the roots of
between x = 1and x = 5 (x − a )(x − b) − 1 = 0 lies in (− ∞ , a ) and other lies in
b m (b, ∞ ).
∴ − ∈ (1, 5) ⇒1 < < 5 ⇒ m∈ (2, 10)
2a 2 7. Let f (x) = x2 − 2ax + a 2 + a − 3
(iii) f (1) > 0 ⇒1 − m + 4 > 0
Since, both root are less than 3.
⇒ m < 5 ⇒ m ∈ (−∞ , 5)
⇒ α < 3, β < 3
(iv) f (5) > 0 ⇒ 25 − 5m + 4 > 0
⇒ Sum, S = α + β < 6
 29
⇒ 5m < 29 ⇒ m ∈  − ∞,  α+β
 5 ⇒ <3
2
From the values of m obtained in (i), (ii), (iii) and (iv), we 2a
get m ∈ (4, 5). ⇒ <3
2
⇒ a <3 …(i)
–∞ ∞
–4 2 4 5 29/5 Again, product, P = αβ
⇒ P <9 ⇒ αβ < 9
⇒ a + a −3 <9
2
4. Put t = x − [x] = { X }, which is a fractional part function
and lie between 0 ≤ { X } < 1 and then solve it. ⇒ a 2 + a − 12 < 0
Given, a ∈ R and equation is ⇒ (a − 3) (a + 4) < 0
−3 { x − [x]} + 2 { x − [x]} + a = 0
2 2 ⇒ −4 < a < 3 …(ii)
α β 3
Let t = x − [x], then equation is Again, D = B − 4 AC ≥ 0
2

−3 t 2 + 2 t + a 2 = 0 ⇒ (−2a )2 − 4 ⋅ 1 (a 2 + a − 3) ≥ 0
⇒ 4a 2 − 4a 2 − 4a + 12 ≥ 0
1 ± 1 + 3a 2
⇒ t= ⇒ −4a + 12 ≥ 0 ⇒ a ≤ 3 …(iii)
3
Again, a f (3) > 0
Q t = x − [x] = { X } [fractional part]
⇒ 1 [(3) − 2a (3) + a 2 + a − 3] > 0
2
∴ 0 ≤ t ≤1
⇒ 9 − 6a + a 2 + a − 3 > 0
1 ± 1 + 3a 2
0≤ ≤1 ⇒ a 2 − 5a + 6 > 0
3
⇒ (a − 2) (a − 3) > 0
Taking positive sign, we get
∴ a ∈ (−∞ , 2) ∪ (3, ∞ ) …(iv)
1 + 1 + 3a 2
0≤ <1 [Q{ x } > 0] From Eqs. (i), (ii), (iii) and (iv), we get
3
a ∈ (−4, 2).
⇒ 1 + 3a 2 < 2 ⇒ 1 + 3a 2 < 4 NOTE There is correction in answer a < 2 should be −4 < a < 2 .
⇒ a 2 − 1 < 0 ⇒ (a + 1) (a − 1) < 0 8. Let f (x) = ax2 + bx + c > 0, ∀ x ∈ R
∴ a ∈ (−1, 1), for no integer solution of a, we consider
⇒ a >0
(−1, 0) ∪ (0, 1) and b2 − 4ac < 0 …(i)
Theory of Equations 47

∴ g (x) = f (x) + f ′ (x) + f ′ ′ (x) 11. Since, x2 − 3x + 2 > 0 and x2 − 2x − 4 ≤ 0


⇒ g (x) = ax2 + bx + c + 2ax + b + 2a ⇒ (x − 1) (x − 2) > 0 and x2 − 2 x + 1 ≤ 5
⇒ g (x) = ax2 + x (b + 2a ) + (c + b + 2a )
⇒ (x < 1 or x > 2) and (1 − 5 ≤ x ≤ 1 + 5 )
whose discriminant
∴ x ∈ [1 − 5 , 1) ∪ [1 + 5 , 2)
= (b + 2a )2 − 4a (c + b + 2a )
12. (i) Given, x2 − 8kx + 16 (k2 − k + 1) = 0
= b2 + 4a 2 + 4ab − 4ac − 4ab − 8a 2
= b2 − 4a 2 − 4ac = (b2 − 4ac) − 4a 2 < 0 [from Eq. (i)] Now, D = 64 { k2 − (k2 − k + 1)} = 64 (k − 1) > 0
k >1
∴ g (x) > 0 ∀ x, as a > 0 and discriminant < 0.
b 8k
Thus, g (x) > 0, ∀ x ∈ R. (ii) − >4 ⇒ >4 ⇒ k >1
2a 2
9. Given, (iii) f (4) ≥ 0
x2 + (a − b) x + (1 − a − b) = 0 has real and unequal roots. ⇒ 16 − 32k + 16 (k2 − k + 1) ≥ 0
⇒ D >0 ⇒ k2 − 3k + 2 ≥ 0
⇒ (a − b)2 − 4(1) (1 − a − b) > 0
⇒ (k − 2) (k − 1) ≥ 0
⇒ a 2 + b2 − 2ab − 4 + 4a + 4b > 0
⇒ k ≤1 or k ≥ 2
Now, to find the values of ‘a’ for which equation has Hence, k =2
unequal real roots for all values of b.
i.e. Above equation is true for all b.
Topic 5 Some Special Forms
or b2 + b(4 − 2a ) + (a 2 + 4a − 4) > 0, is true for all b.
∴ Discriminant, D < 0 1. Given equation 5 + |2x − 1| = 2x (2x − 2)
⇒ (4 − 2a )2 − 4 (a 2 + 4a − 4) < 0 Case I
⇒ 16 − 16a + 4a − 4a − 16a + 16 < 0
2 2 If 2x − 1 ≥ 0 ⇒ x ≥ 0 ,
then 5 + 2x − 1 = 2x (2x − 2)
⇒ −32a + 32 < 0 ⇒ a >1
Put 2x = t, then
10. Y
a<0 5 + t − 1 = t 2 − 2t ⇒ t 2 − 3t − 4 = 0
y = ax2 + bx + c ⇒ t − 4t + t − 4 = 0 ⇒ t (t − 4) + 1(t − 4) = 0
2

⇒ t = 4 or − 1 ⇒ t = 4 (Q t = 2x > 0)
–1 1
⇒ 2 =4⇒x=2 >0
x

α 0 β
X ⇒ x = 2 is the solution.
Y
Case II
a>0
If 2x − 1 < 0 ⇒ x < 0 ,
then 5 + 1 − 2x = 2x (2x − 2)
y = ax2 + bx + c
Put 2x = y, then 6 − y = y2 − 2 y
α X
–1 0 1 β ⇒ y2 − y − 6 = 0 ⇒ y2 − 3 y + 2 y − 6 = 0
⇒ ( y + 2) ( y − 3) = 0 ⇒ y = 3 or − 2
⇒ y = 3(as y = 2x > 0) ⇒ 2x = 3
⇒ x = log 2 3 > 0
So, x = log 2 3 is not a solution.
From figure, it is clear that, if a > 0, then f (−1) < 0 and Therefore, number of real roots is one.
f (1) < 0 and if a < 0, f (−1) > 0 and f (1) > 0. In both
cases, af (−1) < 0 and af (1) < 0. 2. Given, inequality is
⇒ a (a − b + c) < 0 sin 2 x − 2sin x + 5 1
2 ⋅ ≤1
sin 2 y
and a (a + b + c) < 0 4
(sin x − 1 )2 + 4 2
On dividing by a 2, we get ⇒2 ⋅ 2−2sin y
≤1
b c b c 2
(sin x − 1 ) + 4 2
1 − + < 0 and 1 + + <0 ⇒2 ≤2 2sin y
a a a a
On combining both, we get ⇒ (sin x − 1)2 + 4 ≤ 2 sin 2 y
b
1 ± + <0
c [if a > 1 and am ≤ a n ⇒ m ≤ n]
a a
Q Range of (sin x − 1)2 + 4 is [2, 2 2 ]
b c
⇒ 1 +  + < 0
a a and range of 2 sin 2 y is [0, 2].
48 Theory of Equations

∴The above inequality holds, iff ∴ By Rolle’s theorem, f ′ (α ) = 0 for 0 < α < 1
(sin x − 1)2 + 4 = 2 = 2 sin 2 y Now, f ′ (x) = 3ax2 + 2bx + c
⇒ f ′ (α ) = 3aα 2 + 2bα + c = 0
⇒ sin x = 1 and sin 2 y = 1
∴ Eq. (i) has exist atleast one root in the interval (0, 1).
⇒ sin x = |sin y| [from the options]
Thus, f ′ (x) must have root in the interval (0, 1) or
3. Key Idea Reduce the given equation into quadratic equation. 3ax2 + 2bx + c = 0 has root ∈ (0, 1).
Given equation is 7. Given, x12 − x9 + x4 − x + 1 > 0
| x − 2| + x ( x − 4) + 2 = 0 Here, three cases arises:
⇒ | x − 2| + x − 4 x + 4 = 2
Case I When x ≤ 0 ⇒ x12 > 0, − x9 > 0, x4 > 0, − x > 0
⇒ | x − 2| + ( x − 2)2 = 2
∴ x12 − x9 + x4 − x + 1 > 0, ∀ x ≤ 0 …(i)
⇒ (| x − 2|)2 + | x − 2| − 2 = 0
Case II When 0 < x ≤ 1
Let| x − 2| = y, then above equation reduced to
x9 < x4 and x < 1 ⇒ − x9 + x4 > 0 and 1 − x > 0
y2 + y − 2 = 0 ⇒ y2 + 2 y − y − 2 = 0
⇒ y( y + 2) − 1( y + 2) = 0 ⇒ ( y + 2)( y − 1) = 0 ∴ x − x + x − x + 1 > 0, ∀ 0 < x ≤ 1
12 9 4
…(ii)
⇒ y = 1, − 2 Case III When x > 1 ⇒ x > x and x > x 12 9 4

∴ y=1 [Q y = | x − 2| ≥ 0] ∴ x12 − x9 + x4 − x + 1 > 0, ∀ x > 1 …(iii)


⇒ | x − 2| = 1 From Eqs. (i), (ii) and (iii), the above equation holds for
⇒ x −2 = ±1 all x ∈ R.
⇒ x = 3 or 1
8. Consider,
⇒ x = 9 or 1 1
∴ Sum of roots = 9 + 1 = 10 f (x) = ∫ (1 + cos 8 x)(ax2 + bx + c) dx
0
4. Let f (x) = 2x3 + 3x + k Obviously, f (x) is continuous and differentiable in the
On differentiating w.r.t. x, we get interval [1, 2].
f ′ (x) = 6x2 + 3 > 0, ∀ x ∈ R Also, f (1) = f (2) [given]
⇒ f (x) is strictly increasing function. ∴ By Rolle’s theorem, there exist atleast one point
⇒ f (x) = 0 has only one real root, so two roots are not k ∈ (1, 2), such that f ′ (k) = 0.
possible. Now, f ′ (x) = (1 + cos 8 x)(ax2 + bx + c)
5. Since, α is a root of a 2x2 + bx + c = 0 f ′ (k) = 0
⇒ a α + bα + c = 0
2 2
... (i) ⇒ (1 + cos k)(ak + bk + c) = 0
8 2

and β is a root of a x − bx − c = 0
2 2 ⇒ ak2 + bk + c = 0 [as (1 + cos 8 k) ≠ 0]
⇒ a 2β 2 − bβ − c = 0 ... (ii) ∴ x = k is root of ax2 + bx + c = 0,
Let f (x) = a 2x2 + 2bx + 2c where k ∈ (1, 2)
∴ f (α ) = a 2α 2 + 2bα + 2c 9. Given, x1 and x2 are roots of αx2 − x + α = 0.
= a 2α 2 − 2a 2α 2 = − a 2α 2 1
∴ x1 + x2 = and x1x2 = 1
[from Eq. (i)] α
and f (β ) = α 2β 2 + 2bβ + 2c Also, x1 − x2 < 1
= a 2β 2 + 2a 2β 2 = 3a 2β 2 [from Eq. (ii)] ⇒ |x1 − x2|2 < 1
⇒ f (α ) f (β ) < 0 ⇒ (x1 − x2)2 < 1
or (x1 + x2)2 − 4x1x2 < 1
f (x) must have a root lying in the open interval (α , β ).
1 1
∴ α < γ <β ⇒ − 4 < 1 or <5
α2 α2
6. Let f (x) = ax3 + bx2 + cx + d …(i)
⇒ 5α 2 − 1 > 0 or ( 5 α − 1) ( 5 α + 1) > 0
∴ f (0) = d and f (1) = a + b + c + d = d + – +
[Q a + b + c = 0]
∴ f (0) = f (1) –1/√5 1/√5
f is continuous in the closed interval [0, 1] and f is  1  1 
derivable in the open interval (0, 1). ∴ α ∈  −∞ , −  ∪ , ∞ …(i)
 5  5 
Also, f (0) = f (1).
Theory of Equations 49

Also, D >0  3 1
f (x) changes its sign in  − , − 
⇒ 1 − 4α 2 > 0  4 2
 1 1  3 1
or α ∈ − ,  …(ii) Hence, f (x) = 0 has a root in  − , −  .
 2 2  4 2
From Eqs. (i) and (ii), we get 1/ 2 t 3/ 4
 1 −1   1 1 
α ∈ − ,  ∪  , 
12. ∫0 f (x) dx < ∫ f (x) dx < ∫
0 0
f (x) dx
 2 5   5 2 Now, ∫ f (x) dx = ∫ (1 + 2x + 3x2 + 4x3 ) dx
10. PLAN
(i) Concepts of curve tracing are used in this question. = x + x2 + x3 + x4
(ii) Number of roots are taken out from the curve traced.
Let y = x5 − 5x
(i) As x → ∞, y → ∞ and as x → − ∞, y → − ∞
1
(ii) Also, at x = 0, y = 0, thus the curve passes through
the origin.
dy 1 t 3
(iii) = 5x4 − 5 = 5 (x4 − 1) = 5 (x2 − 1) (x2 + 1) 2
dx 4

= 5 (x − 1) (x + 1) (x2 + 1) 1/ 2 15 3 3/ 4 530
⇒ ∫0 f (x) dx = > ,
16 4 ∫0 f (x) dx =
256
<3
+ – +
–1 1 13. As, f ′′ (x) = 2 (12x + 3)
dy 1
Now, > 0 in (− ∞ , − 1) ∪ (1, ∞ ), thus f (x) is f ′ (x) > 0, when x > − and
dx 4
increasing in these intervals. 1
f ′ (x) < 0, when x < − .
dy 4
Also, < 0 in (− 1, 1), thus decreasing in (− 1, 1).
dx ∴ It could be shown as
(iv) Also, at x = − 1, dy /dx changes its sign from + ve to
–ve. 1
∴ x = − 1 is point of local maxima.
Similarly, x = 1 is point of local minima.
S
Local maximum value, y = (− 1)5 − 5 (−1) = 4
–3 –1 1 3
Local minimum value, y = (1)5 − 5(1) = − 4 4 4
2 2
(–1, 4)
14. Let y = x intersect the curve y = kex at exactly one point
when k ≤ 0 .
–1 Y
(1, – 4)

Now, let y = − a
As evident from the graph, if − a ∈ (− 4, 4) X′ X
i.e. a ∈ (− 4, + 4)
Then, f (x) has three real roots and if − a > 4
or − a < − 4, then f (x) has one real root.
i.e. for a < − 4 or a > 4, f (x) has one real root. Y′
11. Given, f (x) = 4x3 + 3x2 + 2x + 1 15. Let f (x) = ke − x x

f ′ (x) = 2 (6x2 + 3x + 1) f ′ (x) = kex − 1 = 0


⇒ D = 9 − 24 < 0 ⇒ x = − ln k
Hence, f (x) = 0 has only one real root. f ′ ′ (x) = kex
 1 3 4 ∴ [ f ′ ′ (x)] x = − ln k = 1 > 0
f −  = 1 −1 + − > 0
 2 4 8 Hence, f (− ln k) = 1 + ln k
 3 6 27 108
f −  = 1 − + − For one root of given equation
 4 4 16 64
1 + ln k = 0
64 − 96 + 108 − 108
= <0 1
64 ⇒ k=
e
50 Theory of Equations

16. For two distinct roots, 1 + ln k < 0 (k > 0) Which is ≤ 0 , ∀ p ∈ [−1, 1].
1 1 
ln k < −1 ⇒ k < ∴ f (x) has atleast one root in ,1 .
2 
e
 1 Now, f ′ (x) = 12x2 − 3 = 3 (2x − 1) (2x + 1)
Hence, k ∈ 0, 
 e 3 1  1 1 
=  x −   x +  > 0 in ,1
4 2  2 2 
17. Let f (x) = (x − a ) (x − c) + 2 (x − b) (x − d )
⇒ f (x) is an increasing function in [1 /2, 1]
Here, f (a ) = + ve
f (b) = − ve Therefore, f (x) has exactly one root in [1 /2, 1] for any
p ∈ [−1, 1].
f (c) = − ve
Now, let x = cos θ
f (d ) = + ve
1   π
∴ There exists two real and distinct roots one in the ∴ x∈ , 1 ⇒ θ ∈ 0,
2   3 
interval (a , b) and other in (c, d ).
From Eq. (i),
Hence, statement is true.
4 cos3 θ − 3 cos θ = p
18. Let f (x) = 4x3 − 3x − p …(i)
3
⇒ cos 3θ = p
 1  1  1 4 3
⇒ 3θ = cos −1 p
Now, f   =4   −3   − p= − − p
 2  2  2 8 2 1
⇒ θ = cos −1 p
= − (1 + p) 3
f (1) = 4(1)3 − 3(1) − p = 1 − p 1 
⇒ cos θ = cos  cos −1 p
 1 3 
⇒ f   . f (1) = − (1 + p)(1 − p)
 2 1 
⇒ x = cos  cos −1 p
= ( p + 1)( p − 1) = p − 1
2 3 
3
Sequences and Series
Topic 1 Arithmetic Progression (AP)
Objective Questions I (Only one correct option) Analytical and Descriptive Question
1. If a1 , a 2, a3 , ... , a n are in AP and a1 + a 4 + a7 + ... + a16 5. If a1 , a 2 ..... , a n are in arithmetic progression, where
= 114 , then a1 + a 6 + a11 + a16 is equal to ai > 0, ∀ i, then show that
(2019 Main, 10 April I) 1 1
(a) 64 (b) 76 + + ...
a1 + a 2 a 2 + a3
(c) 98 (d) 38
1 n −1
2. If 19th term of a non-zero AP is zero, then its (49th + =
an − 1 + an a1 + a n
term) : (29th term) is (2019 Main, 11 Jan II)
(1982, 2M)
(a) 1 : 3 (b) 4 : 1
(c) 2 : 1 (d) 3 : 1
True/False
3. For any three positive real numbers a , b and c, if
9 (25a 2 + b2) + 25 (c2 − 3ac) = 15b (3a + c), then (2017 Main)
6. n1 , n2, K , n p are p positive integers, whose sum is an
(a) b, c and a are in GP even number, then the number of odd integers among
(b) b, c and a are in AP them is odd. (1985, 1M)
(c) a , b and c are in AP
(d) a , b and c are in GP Integer & Numerical Answer Type Questions
4. If Tr is the r th term of an AP, for r = 1, 2, 3, .... . If for 7. Let AP (a ; d ) denote the set of all the terms of an
1 infinite arithmetic progression with first term a and
some positive integers m and n, we have Tm = and
n common difference d > 0. If
1
Tn = , then Tmn equals AP (1 ; 3) ∩ AP (2 ; 5) ∩ AP (3 ; 7) = AP (a ; d )
m (1998, 2M)
1 1 1 then a + d equals ................ (2019 Adv.)
(a) (b) +
mn m n 8. The sides of a right angled triangle are in arithmetic
(c) 1 (d) 0 progression. If the triangle has area 24, then what is the
length of its smallest side? (2017 Adv.)

Topic 2 Sum of n Terms of an AP


Objective Questions I (Only one correct option) 3. For x ∈ R, let [x] denote the greatest integer ≤ x, then the
1. If a1 , a 2, a3 , ... are in AP such that a1 + a7 + a16 = 40, sum of the series
 1  1 1   1 2   1 99 
then the sum of the first 15 terms of this AP is − + − − + − − +…+ − − is
(2019 Main, 12 April II)  3   3 100   3 100   3 100 
(a) 200 (b) 280 (2019 Main, 12 April I)
(c) 120 (d) 150 (a) − 153 (b) − 133
2. Let S n denote the sum of the first n terms of an AP. If (c) − 131 (d) − 135
S 4 = 16 and S 6 = − 48, then S10 is equal to 4. If the sum and product of the first three terms in an AP
(2019 Main, 12 April I) are 33 and 1155, respectively, then a value of its 11th
(a) − 260 (b) − 410 term is (2019 Main, 9 April II)
(c) − 320 (d) − 380 (a) 25 (b) –36 (c) –25 (d) –35
52 Sequences and Series

5. Let the sum of the first n terms of a non-constant AP 12. The sum V1 + V 2 + ... + V n is
n (n − 7)
a1 , a 2, a3 .....be 50n + A, where A is a constant. (a)
1
n (n + 1) (3n 2 − n + 1)
2 12
If d is the common difference of this AP, then the 1
(b) n (n + 1) (3n 2 + n + 2)
ordered pair (d , a50 ) is equal to (2019 Main, 9 April I) 12
(a) (A, 50 + 46A) (b) (50, 50 + 45A) 1
(c) n (2n 2 − n + 1)
(c) (50, 50 + 46A) (d) (A, 50 + 45A) 2
1
6. The sum of all two digit positive numbers which when (d) (2n3 − 2n + 3)
divided by 7 yield 2 or 5 as remainder is 3
(2019 Main, 10 Jan I) 13. Tr is always
(a) 1256 (b) 1465 (c) 1356 (d) 1365 (a) an odd number (b) an even number
30
(c) a prime number (d) a composite number
7. Let a1 , a 2, ..... a30 be an AP, S = ∑ ai and
i =1 14. Which one of the following is a correct statement ?
15 (a) Q1 , Q2 , Q3 ,... are in an AP with common difference 5
T= ∑ a( 2 i − 1). If a5 = 27 and S − 2T = 75, (b) Q1 , Q2 , Q3 ,... are in an AP with common difference 6
i =1
(c) Q1 , Q2 , Q3 ,... are in an AP with common difference 11
then a10 is equal to (2019 Main, 9 Jan I) (d) Q1 = Q2 = Q3 = ...
(a) 42 (b) 57
(c) 52 (d) 47 Fill in the Blanks
8. Let bi > 1 for i = 1, 2, ... , 101 . Suppose log e b1 , log e b2, 15. Let p and q be the roots of the equation x2 − 2x + A = 0
... , log e b101 are in AP with the common difference log e 2 and let r and s be the roots of the equation
. Suppose a1 , a 2, ... , a101 are in AP, such that a1 = b1 and x2 − 18x + B = 0. If p < q < r < s are in arithmetic
a51 = b51. If t = b1 + b2 + ... + b51 and progression, then A = … and B = … . (1997, 2M)
s = a1 + a 2 + ... + a51, then (2016 Adv.)
(a) s > t and a101 > b101 (b) s > t and a101 < b101 16. The sum of the first n terms of the series
n (n + 1)2
(c) s < t and a101 > b101 (d) s < t and a101 < b101 12 + 2 ⋅ 22 + 32 + 2 ⋅ 42 + 52 + 2 ⋅ 62 + K is , when
2
9. If the sum of first n terms of an AP is cn 2, then the sum n is even. When n is odd, the sum is .... . (1988, 2M)
of squares of these n terms is (2009) 17. The sum of integers from 1 to 100 that are divisible by 2
n (4n 2 − 1) c2 n (4n 2 + 1) c2 or 5 is …… (1984, 2M)
(a) (b)
6 3
n (4n 2 − 1) c2 n (4n 2 + 1) c2 Analytical & Descriptive Questions
(c) (d)
3 6
18. The fourth power of the common difference of an
10. If the sum of the first 2n terms of the AP series 2,5,8,..., arithmetic progression with integer entries is added to
is equal to the sum of the first n terms of the AP series the product of any four consecutive terms of it. Prove
57, 59, 61,..., then n equals (2001, 1M) that resulting sum is the square of an integer.(2000, 4M)
(a) 10 (b) 12
19. The real numbers x1 , x2, x3 satisfying the equation
(c) 11 (d) 13
x3 − x2 + βx + γ = 0 are in AP. Find the intervals in
which β and γ lie. (1996, 3M)
Objective Question II
(One or more than one correct option) 20. The interior angles of a polygon are in arithmetic
progression. The smallest angle is 120° and the common
4n k( k + 1 )
difference is 5°. Find the number of sides of the polygon.
11. If S n = ∑ (−1) 2 k2. Then, S n can take value(s) (1980, 3M)
2 (2013 Adv.)
(a) 1056 (b) 1088 Integer & Numerical Answer Type Questions
(c) 1120 (d) 1332 21. Suppose that all the terms of an arithmetic progression
are natural numbers. If the ratio of the sum of the first
Passage Based Problems seven terms to the sum of the first eleven terms is 6 : 11
and the seventh term lies in between 130 and 140, then
Read the following passage and answer the questions.
the common difference of this AP is (2015 Adv.)
Passage 22. A pack contains n cards numbered from 1 to n. Two
Let V r denotes the sum of the first r terms of an consecutive numbered cards are removed from the pack
arithmetic progression (AP) whose first term is r and and the sum of the numbers on the remaining cards is
the common difference is (2r − 1). Let Tr = V r + 1 − V r and 1224. If the smaller of the numbers on the removed
Qr = Tr + 1 − Tr for r = 1, 2, . . . (2007, 8M) cards is k, then k − 20 is equal to (2013 Adv.)
Sequences and Series 53

23. Let a1 , a 2, a3 , K , a100 be an arithmetic progression with 24. Let a1 , a 2, a3 , ... , a11 be real numbers satisfying a1 = 15,
p
27 − 2a 2 > 0 and a k = 2a k − 1 − a k − 2 for k = 3, 4, ... , 11.
a1 = 3 and S p = ∑ ai , 1 ≤ p ≤ 100. For any integer n with
a 2 + a 22 + K + a11
2
i =1 If 1 = 90, then the value of
Sm 11
1 ≤ n ≤ 20, let m = 5n. If does not depend on n, then a 2 a1 + a 2 + K + a11
Sn is…… (2010)
11
is equal to …… (2011)

Topic 3 Geometric Progression (GP)


Objective Questions I (Only one correct option) 8. Let f (x) = ax2 + bx + c, a ≠ 0 and ∆ = b2 − 4ac. If α + β,
1. Let a , b and c be in GP with common ratio r, where a ≠ 0 α 2 + β 2 and α 3 + β3 are in GP, then (2005, 1M)
1 (a) ∆ ≠ 0 (b) b∆ = 0 (c) c∆ = 0 (d) bc ≠ 0
and 0 < r ≤ . If 3a, 7b and 15care the first three terms of
2 9. Let a , b, c be in an AP and a 2, b2, c2 be in GP. If a < b < c
an AP, then the 4th term of this AP is 3
and a + b + c = , then the value of a is (2002, 1M)
(2019 Main, 10 April II) 2
2 7 1 1 1 1 1 1
(a) 5a (b) a (c) a (d) a (a) (b) (c) − (d) −
3 3 2 2 2 3 2 3 2 2
2. If three distinct numbers a , b and c are in GP and the 10. Let α , β be the roots of x2 − x + p = 0 and γ, δ be the roots
equations ax + 2bx + c = 0 and dx + 2ex + f = 0 have a
2 2
of x2 − 4x + q = 0. If α , β , γ, δ are in GP, then the integer
common root, then which one of the following values of p and q respectively are (2001, 1M)
statements is correct? (2019 Main, 8 April II)
(a) − 2, − 32 (b) − 2,3 (c) − 6, 3 (d) − 6, − 32
d e f
(a) d , e and f are in GP (b) , and are in AP
a b c 11. If a , b, c, d and p are distinct real numbers such that
d e f (a 2 + b2 + c2) p2 − 2 (ab + bc + cd ) p
(c) d , e and f are in AP (d) , and are in GP
a b c
+ (b2 + c2 + d 2) ≤ 0, then a , b, c, d
3. The product of three consecutive terms of a GP is 512. If (a) are in AP (b) are in GP (1987, 2M)
4 is added to each of the first and the second of these
(c) are in HP (d) satisfy ab = cd
terms, the three terms now form an AP. Then, the sum
of the original three terms of the given GP is 12. If a , b, c are in GP, then the equations ax2 + 2bx + c = 0
(2019 Main, 12 Jan I) d e f
and dx2 + 2ex + f = 0 have a common root, if , , are
(a) 36 (b) 28 (c) 32 (d) 24 a b c
a3 in (1985, 2M)
4. Let a1 , a 2, .... , a10 be a GP. If = 25, then
a1 (a) AP (b) GP
a9 (c) HP (d) None of these
equals (2019 Main, 11 Jan I)
a5
13. The third term of a geometric progression is 4. The
(a) 53 (b) 2(52 ) (c) 4(52 ) (d) 54 product of the first five terms is (1982, 2M)
5. Let a , b and c be the 7th, 11th and 13th terms (a) 43 (b) 45
respectively of a non-constant AP. If these are also the (c) 44 (d) None of these
a
three consecutive terms of a GP, then is equal to
c Analytical & Descriptive Questions
(2019 Main, 9 Jan II)
14. Find three numbers a , b, c between 2 and 18 such that
7 1
(a) 2 (b) (c) 4 (d) (i) their sum is 25. (ii) the numbers 2, a , b are
13 2
consecutive terms of an AP. (iii) the numbers b, c, 18 are
6. If a , b and c be three distinct real numbers in GP and consecutive terms of a GP. (1983, 2M)
a + b + c = xb, then x cannot be (2019 Main, 9 Jan I) 15. Does there exist a geometric progression containing
(a) 4 (b) 2 (c) −2 (d) −3 27,8 and 12 as three of its term? If it exists, then how
7. If the 2nd, 5th and 9th terms of a non-constant AP are many such progressions are possible? (1982, 2M)
in GP, then the common ratio of this GP is (2016 Main) 16. If the mth, nth and pth terms of an AP and GP are equal
8 4 7
(a) (b) (c) 1 (d) and are x, y, z, then prove that xy − z ⋅ yz − x ⋅ z x − y = 1.
5 3 4 (1979, 3M)
54 Sequences and Series

Topic 4 Sum of n Terms & Infinite Terms of a GP


Objective Questions I (Only one correct option) 10. Sum of the first n terms of the series +
1 3 7 15
+ + + ...
2 4 8 16
1. If|x|< 1,| y|< 1 and x ≠ y, then the sum to infinity of the
is equal to (1988, 2M)
following series (x + y) + (x2 + xy + y2)
(a) 2n − n − 1 (b) 1 − 2−n (c) n + 2 − n − 1 (d) 2n + 1
+ (x3 + x2y + xy2 + y3 ) + … is (2020 Main, 2 Sep I)
x + y + xy x + y − xy
(a)
(1 + x) (1 + y)
(b)
(1 − x) (1 − y)
Objective Question II
x + y + xy x + y − xy (One or more than one correct option)
(c) (d)
(1 − x) (1 − y) (1 + x) (1 + y) 11. Let α and β be the roots of x2 − x − 1 = 0, with α > β. For
20 all positive integers n, define
1
2. The sum ∑ k 2k is equal to (2019 Main, 8 April II) α n − βn
k =1 an = , n ≥ 1,
11 11 α −β
(a) 2 − (b) 1 −
219
220 b1 = 1 and bn = a n − 1 + a n + 1, n ≥ 2
3 21
(c) 2 − (d) 2 − Then which of the following options is/are correct?
217 220 ∞
b 8
3. Let S n = 1 + q + q + K + q and 2 n (a) ∑ 10nn = 89 (2019 Adv.)
n =1
2 n
 q + 1  q + 1  q + 1
Tn = 1 +   +  +K+  , where q is a (b) bn = α n + β n for all n ≥ 1
 2   2   2 
(c) a1 + a2 + a3 + K + an = an + − 1for all n ≥ 1
real number and q ≠ 1. If ∞
2
a 10
101
C1 + 101C 2 ⋅ S1 + K + 101C101 ⋅ S100 = αT100, then α is (d) ∑ nn =
equal to (2019 Main, 11 Jan II) n = 1 10 89
(a) 2100 (b) 202
12. Let S1 , S 2, ... be squares such that for each n ≥ 1 the
(c) 200 (d) 299
length of a side of S n equals the length of a diagonal of
4. The sum of an infinite geometric series with positive S n + 1. If the length of a side of S1 is 10 cm, then for which
27
terms is 3 and the sum of the cubes of its terms is . of the following values of n is the area of S n less than
19 1 sq cm? (1999, 3M)
Then, the common ratio of this series is
(a) 7 (b) 8 (c) 9 (d) 10
(2019 Main, 11 Jan I)
4 2 2 1 Analytical & Descriptive Questions
(a) (b) (c) (d)
9 3 9 3 2 3 n
13. Let An =   −   +   + ... + (− 1)n − 1   ,
3 3 3 3
5. Three positive numbers form an increasing GP. If the        
middle term in this GP is doubled, then new numbers 4 4 4 4
are in AP. Then, the common ratio of the GP is Bn = 1 − An. Find a least odd natural number n0 , so that
(a) 2 + 3 (b) 3 + 2 (2014 Main) Bn > An , ∀ n ≥ n0. (2006, 6M)
(c) 2 − 3 (d) 2 + 3 14. If S1 , S 2, S3 , ... , S n are the sums of infinite geometric
6. If (10)9 + 2(11)1 (10)8 + 3(11)2(10)7 + ... + 10(11)9 = k(10)9, series, whose first terms are 1, 2, 3,..., n and whose
then k is equal to (2014 Main) 1 1 1 1
common ratios are , , ,... , respectively, then
(a)
121
(b)
441
(c) 100 (d) 110
2 3 4 n+1
10 100 find the values of S12 + S 22 + S32 + ... + S 22n − 1. (1991, 4M)
7. The sum of first 20 terms of the sequence 15. The sum of the squares of three distinct real numbers,
0.7, 0.77, 0.777,… , is (2013 Main) which are in GP, is S 2. If their sum is a S, then show that
7 7
(a) (179 − 10− 20 ) (b) (99 − 10− 20 ) 1 
81 9 a 2 ∈  , 1 ∪ (1, 3)
3  (1986, 5M)
7 7
(c) (179 + 10− 20 ) (d) (99 + 10− 20 )
81 9
Integer & Numerical Answer Type Question
8. An infinite GP has first term x and sum 5, then x
belongs to (2004, 1M) 16. Let S k , where k = 1, 2, , K , 100, denotes the sum of the
(a) x < − 10 (b) −10 < x < 0 (c) 0 < x < 10 (d) x > 10 k −1
infinite geometric series whose first term is and
k!
9. Consider an infinite geometric series with first term a 1
and common ratio r. If its sum is 4 and the second term the common ratio is . Then, the value of
is 3 /4, then (2000, 2M)
k
2 100
100
(a) a = 4/7, r = 3/7 (b) a = 2, r = 3 /8
100 !
+ ∑|(k2 − 3k + 1) Sk |is …… (2010)
(c) a = 3/2, r = 1/2 (d) a = 3, r = 1/4 k =1
Sequences and Series 55

Topic 5 Harmonic Progression (HP)


Objective Questions I (Only one correct option) 5. Suppose four distinct positive numbers a1 , a 2, a3 , a 4
1. If a1 , a 2, a3 ,… are in a harmonic progression with a1 = 5 are in GP. Let b1 = a1 , b2 = b1 + a 2, b3 = b2 + a3 and
b4 = b3 + a 4.
and a 20 = 25. Then, the least positive integer n for which
a n < 0, is (2012) Statement I The numbers b1 , b2, b3 , b4 are neither in
(a) 22 (b) 23 AP nor in GP.
(c) 24 (d) 25 Statement II The numbers b1 , b2, b3 , b4 are in HP.
(2008, 3M)
2. If the positive numbers a , b, c,d are in AP. Then,
abc, abd , acd , bcd are (2001, 1M) Fill in the Blank
(a) not in AP / GP / HP (b) in AP
6. If cos (x − y), cos x and cos (x + y)’ are in HP. Then
(c) in GP (d) in HP
 y
cos x ⋅ sec   = K . (1997C, 2M)
3. Let a1 , a 2, ..., a10 be in AP and h1 , h2, equal to ..... , h10 be  2
in HP. If a1 = h1 = 2 and a10 = h10 = 3, then a 4h7 is
(1999, 2M)
Analytical & Descriptive Questions
(a) 2 (b) 3 (c) 5 (d) 6
4. If x > 1, y > 1, z > 1 are in GP, then
1
,
1
, 7. If a , b, c are in AP, a 2, b2, c2 are in HP, then prove that
1 + ln x 1 + ln y c
1 either a = b = c or a , b, − form a GP.
are in (1998, 2M) 2 (2003, 4M)
1 + ln z
(a) AP (b) HP 8. Let a and b be positive real numbers. If a , A1 , A2, b are
(c) GP (d) None of these in arithmetic progression, a , G1 , G2, b are in geometric
progression and a , H 1 , H 2, B are in harmonic
Assertion and Reason progression, then show that
G1G2 A + A2 (2a + b)(a + 2b)
For the following question, choose the correct = 1 =
answer from the codes (a), (b), (c) and (d) defined as H 1H 2 H 1 + H 2 9ab (2002, 5M)
follows:
9. (i) The value of x + y + z is 15. If a , x, y, z , b are in AP
(a) Statement I is true, Statement II is also true; 1 1 1 5
Statement II is the correct explanation of Statement I while the value of + + is . If a , x, y, z , b are in
x y z 3
(b) Statement I is true, Statement II is also true;
Statement II is not the correct explanation of HP, then find a and b.
Statement I (ii) If x, y, z are in HP, then show that
(c) Statement I is true; Statement II is false log (x + z ) + log (x + z − 2 y) = 2 log (x − z ). (1978, 3M)
(d) Statement I is false; Statement II is true

Topic 6 Relation between AM, GM, HM and Some Special Series


Objective Questions I (Only one correct option) 4. If the sum of the first 15 terms of the series
3 3 3 3
1 +2 3
1 +2 +3
3 3 3 3
 3  1  1  3
  + 1  +  2  + 3 +  3  + ...
3
1. The sum of series 1 + + + ...
1+2 1+2+3  4  2  4  4
13 + 23 + 33 + K + 153 1 is equal to 225 k, then k is equal to
+ − (1 + 2 + 3 + K + 15) is
1 + 2 + 3 + K + 15 2 (2019 Main, 12 Jan II)

equal to (2019 Main, 10 April II) (a) 108 (b) 27 (c) 54 (d) 9
1 + 2 + 3 + ... + k 5
(a) 620 (b) 660 (c) 1240 (d) 1860 5. Let S k = . If S12 + S 22 + ... + S10
2
= A,
k 12
3 × 13 5 × (13 + 23 )
2. The sum of series + then A is equal to (2019 Main, 12 Jan I)
12 12 + 22
(a) 156 (b) 301 (c) 283 (d) 303
7 × (1 + 2 + 3 )
3 3 3
+ + .......... + upto 10th term, is 6. Let x, y be positive real numbers and m, n positive
12 + 22 + 32 (2019 Main, 10 April I) integers. The maximum value of the expression
(a) 680 (b) 600 (c) 660 (d) 620 xm yn
is
3. The sum of the series 1 + 2 × 3 + 3 × 5 + 4 × 7 +... upto (1 + x ) (1 + y2n )
2m
(2019 Main, 11 Jan II)
11th term is (2019 Main, 9 April II) 1 1 m+ n
(a) (b) 1 (c) (d)
(a) 915 (b) 946 (c) 916 (d) 945 2 4 6mn
56 Sequences and Series

7. The sum of the following series 18. If a , b and c are distinct positive numbers, then the
9 (1 + 2 + 3 ) 12 (1 + 2 + 3 + 4 )
2 2 2 2 2 2 2
expression (b + c − a ) (c + a − b) (a + b − c) − abc is
1+6+ +
7 9 (a) positive (b) negative (1991, 2M)
15 (1 + 2 + ... + 5 )
2 2 2
(c) non-positive (d) non-negative
+ + ... up to 15 terms is
11 19. If x1 , x2,... , xn are any real numbers and n is any positive
(2019 Main, 9 Jan II)
integer, then (1982, 1M)
(a) 7510 (b) 7820 (c) 7830 (d) 7520 2 2
n   n n  n 
(a) n ∑ xi2 <  ∑ xi  (b) n ∑ xi2 ≥  ∑ xi 
12
8. Let a1 , a 2, a3 , …, a 49 be in AP such that ∑ a 4k + 1 = 416 i =1

i =1 

i =1

i =1 

k=0
2
and a 9 + a 43 = 66. If a12 + a 22 + … + a17
2
= 140 m, then m n  n 
(c) n ∑ xi2 ≥ n  ∑ xi  (d) None of these
is equal to (2018 Main)  
i =1 i =1 
(a) 66 (b) 68 (c) 34 (d) 33
9. Let A be the sum of the first 20 terms and B be the sum Passage Based Problems
of the first 40 terms of the series
Passage
12 + 2 ⋅ 22 + 32 + 2 ⋅ 42 + 52 + 2 ⋅ 62 + …
Let A1 , G1 , H 1 denote the arithmetic, geometric and
If B − 2 A = 100λ, then λ is equal to (2018 Main)
harmonic means, respectively, of two distinct positive
(a) 232 (b) 248 (c) 464 (d) 496 numbers. For n ≥ 2, let An − 1 and H n − 1 has arithmetic,
10. If the sum of the first ten terms of the series geometric and harmonic means as An , Gn , H n ,
2 2 2 2
 3  2  1  4 16 respectively. (2007, 8M)
1  + 2  + 3  + 4 + 4  + K, is
2
m, then
 5  5  5  5 5 20. Which one of the following statements is correct?
m is equal to (2016 Main) (a) G1 > G2 > G3 > ...
(a) 102 (b) 101 (c) 100 (d) 99 (b) G1 < G2 < G3 < ...
(c) G1 = G2 = G3 = ...
11. If m is the AM of two distinct real numbers l and
(d) G1 < G3 < G5 < ... and G2 > G4 > G6 >...
n (l, n > 1) and G1 , G2 and G3 are three geometric means
between l and n, then G14 + 2G24 + G34 equals (2015) 21. Which of the following statements is correct?
(a) 4l2mn (b) 4lm2n (c) lmn 2 (d) l2m2n 2 (a) A1 > A2 > A3 >...
(b) A1 < A2 < A3 <...
12. The sum of first 9 terms of the series
13 13 + 23 13 + 23 + 33 (c) A1 > A3 > A5 >... and A2 < A4 < A6 <...
+ + + ... is (d) A1 < A3 < A5 <... and A2 > A4 > A6 >...
1 1+3 1+3+5 (2015)
(a) 71 (b) 96 (c) 142 (d) 192 22. Which of the following statements is correct ?
(a) H1 > H 2 > H3 >...
π tan α
2
13. If α ∈ 0,  , then x2 + x + is always greater (b) H1 < H 2 < H3 <...
 2 x2 + x (c) H1 > H3 > H5 >... and H 2 < H 4 < H 6 <...
than or equal to (2003, 2M) (d) H1 < H3 < H5 <... and H 2 > H 4 > H 6 >...
(a) 2 tan α (b) 1 (c) 2 (d) sec2 α
14. If a1 , a 2,... , a n are positive real numbers whose product
Objective Questions II
is a fixed number c, then the minimum value of (One or more than one correct option)
a1 + a 2 + ... + a n − 1 + 2a n is (2002, 1M) 23. For a positive integer n let
(a) n (2c)1/ n (b) (n + 1)c1/ n 1 1 1 1
a (n ) = 1 + + + + ... + n , then
(c) 2nc1/ n (d) (n + 1) (2c)1/ n 2 3 4 (2 ) − 1 (1999, 3M)
15. If a , b, c are positive real numbers such that (a) a (100) ≤ 100 (b) a (100) > 100
a + b + c + d = 2 , then M = (a + b) (c + d ) satisfies the (c) a (200)≤ 100 (d) a (200) > 100
relation (2000, 2M) 24. If the first and the (2n − 1)th term of an AP, GP and HP
(a) 0 < M ≤ 1 (b) 1 ≤ M ≤ 2 (c) 2 ≤ M ≤ 3 (d) 3 ≤ M ≤ 4 are equal and their nth terms are a , b and c
16. The harmonic mean of the roots of the equation respectively, then (1988, 2M)
(5 + 2 ) x2 − (4 + 5 ) x + 8 + 2 5 = 0 is (1999, 2M) (a) a = b = c (b) a ≥ b ≥ c
(a) 2 (b) 4 (c) a + c = b (d) ac − b2 = 0
(c) 6 (d) 8
Fill in the Blanks
17. The product of n positive numbers is unity, then their
sum is (1991, 2M) 25. If x be is the arithmetic mean and y, z be two geometric
(a) a positive integer (b) divisible by n means between any two positive numbers, then
1 y3 + z3
(c) equal to n + (d) never less than n = ...
n xyz (1997C, 2M)
Sequences and Series 57

26. If the harmonic mean and geometric mean of two 32. If a > 0, b > 0 and c > 0, then prove that
positive numbers are in the ratio 4 : 5. Then, the two 1 1 1
numbers are in the ratio … . (1992, 2M) (a + b + c) + + ≥9 (1984, 2M)
a b c
True/False Integer & Numerical Answer Type Questions
27. If x and y are positive real numbers and m, n are any 33. Let m be the minimum possible value of
n m log3 (3y1 + 3y2 + 3y3 ), where y1 , y2, y3 are real numbers for
x y 1
positive integers, then > . which y1 + y2 + y3 = 9. Let M be the maximum possible
(1 + x2n )(1 + y2m ) 4 value of (log3 x1 + log3 x2 + log3 x3 ), where x1 , x2, x3 are
(1989, 1M) positive real numbers for which x1 + x2 + x3 = 9. Then the
value of log 2(m3 ) + log3 (M 2) is ……… . (2020 Adv.)
28. For 0 < a < x, the minimum value of function
34. Let a1 , a 2, a3 , .... be a sequence of positive integers in
log a x + log x a is 2.
arithmetic progression with common difference 2. Also,
let b1 , b2, b3 , .... be a sequence of positive integers in
Analytical & Descriptive Questions geometric progression with common ratio 2. If a1 = b1 = c,
29. If a , b, c are positive real numbers, then prove that then the number of all possible values of c, for which the
equality
{(1 + a ) (1 + b) (1 + c)}7 > 77 a 4b4c4 (2004, 4M)
2(a1 + a 2 + K + a n ) = b1 + b2 + K + bn
30. Let a1 , a 2,.. be positive real numbers in geometric holds for some positive integer n, is …… (2020 Adv.)
progression. For each n, if An , Gn , H n are respectively, 35. If m arithmetic means (AMs) and three geometric means
the arithmetic mean, geometric mean and harmonic (GMs) are inserted between 3 and 243 such that 4th AM
mean of a1 , a 2, .... , a n. Then, find an expression for the is equal to 2nd GM, then m is equal to
geometric mean of G1 , G2, ... , Gn in terms of [2020 Main, 3 Sep II]
A1 , A2, ... , An , H 1 , H 2, ... , H n. (2001, 5M) 36. Let a,b,c be positive integers such that b /a is an integer.
31. If p is the first of the n arithmetic means between two If a,b,c are in geometric progression and the arithmetic
a 2 + a − 14
numbers and q be the first on n harmonic means mean of a,b,c is b + 2, then the value of is
a+1
between the same numbers. Then, show that q does not
2 (2014 Adv.)
 n + 1
lie between p and   p. (1991, 4M) 37. The minimum value of the sum of real numbers
 n − 1 a − 5 , a − 4 , 3a − 3 , 1, a 8 and a10 with a > 0 is …… (2011)

Answers
Topic 1 Topic 4
1. (b) 2. (d) 3. (b) 4. (c) 1. (b) 2. (a) 3. (a) 4. (b)
6. False 7. (157.00) 8. (6) 5. (d) 6. (c) 7. (c) 8. (c)
Topic 2 9. (d) 10. (c) 11. (b, c, d) 12. (b,c,d)
1
1. (a) 2. (c) 3. (b) 4. (c) 13. (7) 14. (2n )(2n + 1 )( 4n + 1 ) – 1 16. (4)
5. (a) 6. (c) 7. (c) 8. (b) 6
9. (c) 10. (c) 11. (a, d) 12. (b) Topic 5
13. (d) 14. (b) 15. A = – 3 , B = 77 1. (d) 2. (d) 3. (d) 4. (b)
n (n + 1 ) 
2
5. (c) 6. ± 2 9. (i) a = 1, b = 9
16.   17. 3050
 2 
Topic 6
 1  1 
19. β ∈ – ∞, and γ ∈ – , ∞ 20. (9) 21. (9) 1. 2. 4.
 3   27  (a) (c) 3. (b) (b)
5. (d) 6. (c) 7. (b) 8. (c)
22. (5) 23. (9) 24. (0)
9. (b) 10. (b) 11. (b) 12. (b)
Topic 3 13. (a) 14. (a) 15. (a) 16. (b)
1. (c) 2. (b) 3. (b) 4. (d) 17. (d) 18. (b) 19. (b) 20. (c)
5. (c) 6. (b) 7. (b) 8. (c) 21. (a) 22. (b) 23. (a, d) 24. (a, b, d)
9. (d) 10. (a) 11. (b) 12. (a) 25. 2 26. 4:1 27. False 28. False
13. (b) 33. (8) 34. C =12 for n =3 35. (39)
14. (a = 5 ) (b = 8 ) (c = 12 ) 15. Yes, infinite 36. (4) 37. (8)
Hints & Solutions
Topic 1 Arithmetic Progression (AP) = a + (n − 1) d + (mn − n ) d
1 1 (m − 1)
= Tn + n (m − 1) = + =1
1. Key Idea Use nth term of an AP i.e. an = a + ( n − 1) d , simplify the mn m m
given equation and use result.
5. Since, a1 , a 2, ... , a n are in an AP.
Given AP is a1 , a 2, a3 , … , a n ∴ (a 2 − a1 ) = (a3 − a 2) = ... = (a n − a n − 1 ) = d
Let the above AP has common difference ‘d’, then 1 1 1
a1 + a 4 + a7 + … + a16 Thus, + + ... +
a1 + a 2 a 2 + a3 a n −1 + a n
= a1 + (a1 + 3d ) + (a1 + 6d ) + … + (a1 + 15d )
= 6a1 + (3 + 6 + 9 + 12 + 15)d  a 2 − a1   a3 − a 2   a n − a n −1 
= +  + ... +  
∴ 6a1 + 45d = 114 (given)  d   d   d 
 
⇒ 2a1 + 15d = 38 …(i) 1 1 (a n − a1 ) (n − 1)
Now, a1 + a 6 + a11 + a16 = ( a n − a1 ) = =
d d a n + a1 a n + a1
= a1 + (a1 + 5d ) + (a1 + 10d ) + (a1 + 15d )
= 4a1 + 30d = 2(2a1 + 15d ) 6. Since, n1 , n2,... , n p are p positive integers, whose sum is
= 2 × 38 = 76 [from Eq. (i)] even and we know that, sum of any two odd integers is
even.
2. Let tn be the nth term of given AP. Then, we have t19 = 0
∴Number of odd integers must be even.
⇒ a + (19 − 1)d = 0 [Qtn = a + (n − 1)d ]
Hence, it is a false statement.
⇒ a + 18d = 0 …(i)
t49 a + 48d 7. Given that, AP (a ; d ) denote the set of all the terms of an
Now, =
t29 a + 28d infinite arithmetic progression with first term ‘a’ and
− 18d + 48d common difference d > 0.
= [using Eq. (i)]
Now, let mth term of first progression
− 18d + 28d
30d AP(1 ; 3) = 1 + (m − 1)3 = 3m − 2 …(i)
= = 3 :1 and nth term of progression
10d
AP (2; 5) = 2 + (n − 1)5 = 5n − 3 …(ii)
3. We have, and rth term of third progression AP (3; 7)
225a 2 + 9b2 + 25c2 − 75ac − 45ab − 15bc = 0 = 3 + (r − 1)7 = 7r − 4 …(iii)
⇒ (15a )2 + (3b)2 + (5c)2 − (15a )(5c) − (15a )(3b) are equal. Then
− (3b)(5c) = 0 3m − 2 = 5n − 3 = 7r − 4
1 Now, for AP ( 1 ; 3 ) ∩ AP (2 ; 5 ) ∩ AP (3; 7), the common
⇒ [(15a − 3b) + (3b − 5c) + (5c − 15a ) ] = 0
2 2 2
5n − 1
2 terms of first and second progressions, m =
3
⇒ 15a = 3b, 3b = 5c and 5c = 15a
⇒ n = 2, 5, 11, … and the common terms of second and
a b c 5n + 1
∴ 15a = 3b = 5c ⇒ = = =λ (say) the third progressions, r = ⇒ n = 4, 11,…
1 5 3 7
⇒ a = λ , b = 5λ , c = 3λ Now, the first common term of first, second and third
∴ b, c, a are in AP. progressions (when n = 11), so a = 2 + (11 − 1)5 = 52
1 and d = LCM(3, 5, 7) = 105
4. Let Tm = a + (m − 1) d = …(i)
n So, AP (1 ; 3) ∩ AP (2 ; 5) ∩ AP (3 ; 7) = AP (52 ; 105)
1 So, a = 52 and d = 105 ⇒ a + d = 157.00
and Tn = a + (n − 1) d = …(ii)
m 8. Let the sides are a − d , a and a + d. Then,
On subtracting Eq. (ii) from Eq. (i), we get a (a − d ) = 48
1 1 m−n
(m − n ) d = − = and a − 2ad + d 2 + a 2 = a 2 + 2ad + d 2
2
n m mn
⇒ a 2 = 4ad
1
⇒ d= ⇒ a = 4d
mn
Thus, a = 8, d = 2
Again, Tmn = a + (mn − 1) d Hence, a − d =6
= a + (mn − n + n − 1) d
Sequences and Series 59

Topic 2 Sum of n Terms of an AP = − 2 − 2 − 2 − 2 K 33 times


= (−2) × 33 = − 66
1. Let the common difference of given AP is ‘d’.
 1  1 1   1 2   1 99 
Since, a1 + a7 + a16 = 40 ∴ − + − − + − − +K+ − −
 3   3 100   3 100   3 100 
∴ a1 + a1 + 6d + a1 + 15d = 40 [Q a n = a1 + (n − 1) d ]
= (− 67) + (− 66) = − 133.
⇒ 3a1 + 21d = 40 …(i)
Now, sum of first 15 terms is given by Alternate Solution
15 Q [− x] = − [x] − 1, if x ∉Integer,
S15 = [2a1 + (15 − 1) d ]  1  2  n − 1
2 and [x] + x + + x+ +K+ x+ = [nx],
15  n   n   n 
= [2a1 + 14d ] = 15 [a1 + 7d ]
2 n ∈N.
From Eq. (i), we have So given series
a1 + 7d =
40  1  1 1   1 2   − 1 99 
− + − − + − − +…K+ −
3  3   3 100   3 100   3 100 
40
So, S15 = 15 ×  1    1 1  
3 = − − 1 +  − + − 1
 3    3 100  
= 5 × 40 = 200
 1 2    1 99  
2. Given S n denote the sum of the first n terms of an AP. + − + − 1 + K +  − + − 1
 3 100    3 100  
Let first term and common difference of the AP be ‘a’
and ‘d’, respectively. 1 
= (− 1) × 100 − × 100 = − 100 − 33 = − 133.
3 
∴ S 4 = 2[2a + 3d ] = 16 (given)
4. Let first three terms of an AP as a − d, a, a + d.
 n 
Q S n = [2a + (n − 1)d ] So, 3a = 33 ⇒ a = 11
 2 
[given sum of three terms = 33
⇒ 2a + 3d = 8 … (i)
and product of terms = 1155]
and S 6 = 3[2a + 5d ] = − 48 [given]
⇒ (11 − d )11(11 + d ) = 1155 [given]
⇒ 2a + 5d = − 16 … (ii)
⇒ 112 − d 2 = 105
On subtracting Eq. (i) from Eq. (ii), we get
⇒ d 2 = 121 − 105 = 16
2d = − 24
⇒ d = ±4
⇒ d = − 12
So the first three terms of the AP are either 7, 11, 15 or
So, 2a = 44 [put d = −12 in Eq. (i)]
15, 11, 7.
Now, S10 = 5[2a + 9d ]
So, the 11th term is either 7 + (10 × 4) = 47
= 5[44 + 9(− 12)] = 5[44 − 108]
or 15 + (10 × (−4)) = − 25.
= 5 × (− 64) = − 320
3. Given series is 5. Key Idea Use the formula of sum of first n terms of AP, i.e
n
Sn = [2 a + ( n − 1) d ]
 1  1 1   1 2   1 99 
− + − − + − − + K ... + − − 2
 3   3 100   3 100   3 100 
Given AP, is
[where, [x] denotes the greatest integer ≤ x]
a1 , a 2, a3 ,… having sum of first n-terms
Now, n
= [2a1 + (n − 1)d ]
 1  1 1   1 2   1 66  2
− , − − , − − , …+ − −
 3   3 100   3 100   3 100  [where, d is the common difference of AP]
all the term have value − 1 n (n − 7)
= 50n + A (given)
 1 67   1 68   1 99  2
and − − , − − , …, − − all the term 1 n−7
 3 100   3 100   3 100  ⇒ [2a1 + (n − 1)d ] = 50 + A
have value − 2. 2 2
1  7  n
 1  1 1   1 2   1 66  ⇒ [2a1 + nd − d ] = 50 − A + A
So, − + − − + − − + ... + − − 2  2  2
 3   3 100   3 100   3 100 
 d  nd  7  n
= − 1 − 1 − 1 − 1 K 67 times. ⇒  a1 −  + = 50 − A + A
 2 2  2  2
= (− 1) × 67 = − 67 On comparing corresponding term, we get
 1 67   1 68   1 99  d 7
and − − + − − +K+ − − d = A and a1 − = 50 − A
 3 100   3 100   3 100 
2 2
60 Sequences and Series
A 7 Also, a1 , a 2, ... , a101 are in AP.
⇒ a1 − = 50 − A [Qd = A]
2 2 Given, a1 = b1 and a51 = b51
⇒ a1 = 50 − 3 A ⇒ a1 + 50 D = 250 b1
So a50 = a1 + 49d ⇒ a1 + 50 D = 250 a1 [Q a1 = b1 ] …(ii)
= (50 − 3 A ) + 49 A [Q d = A] Now, t = b1 + b2 + K + b51
= 50 + 46 A
(251 − 1)
Therefore, (d , a50 ) = ( A , 50 + 46 A ) ⇒ t = b1 …(iii)
2 −1
6. Clearly, the two digit number which leaves remainder 2 and s = a1 + a 2 + K + a51
when divided by 7 is of the form N = 7k + 2 [by Division
51
Algorithm] = (2a1 + 50 D ) …(iv)
2
∴ t = a1 (251 − 1) [Q a1 = b1 ]
For, k = 2, N = 16
k = 3, N = 23 or t = 251 a1 − a1 < 251 a1 …(v)
51
M M and s= [a1 + (a1 + 50 D )] [from Eq. (ii)]
k = 13, N = 93 2
∴ 12 such numbers are possible and these numbers =
51
[a1 + 250 a1 ]
forms an AP. 2
12 51 51 50
Now, S= [16 + 93] = 654 = a1 + 2 a1
2 2 2
 n  ∴ s > 251 a1 …(vi)
QS n = ( a + l )
 2  From Eqs. (v) and (vi), we get s > t
Similarly, the two digit number which leaves remainder Also, a101 = a1 + 100 D and b101 = 2100 b1
5 when divided by 7 is of the form N = 7k + 5
 250 a1 − a1 
For k = 1, N = 12 ∴ a101 = a1 + 100   and b101 = 2100 a1
 50 
k = 2, N = 19
M ⇒ a101 = a1 + 251 a1 − 2a1 = 251 a1 − a1
k = 13, N = 96 ⇒ a101 < 251 a1 and b101 > 251 a1
∴13 such numbers are possible and these numbers also ⇒ b101 > a101
forms an AP. 9. Let S n = cn 2
13
Now, S′ = [12 + 96] = 702
2 S n − 1 = c (n − 1)2 = cn 2 + c − 2cn
 n  ∴ Tn = 2cn − c [Q Tn = S n − S n − 1 ]
QS n = ( a + l )
 2  Tn2 = (2 cn − c) = 4c n + c2 − 4c2n
2 2 2

Total sum = S + S′ = 654 + 702 = 1356


4c2 ⋅ n (n + 1) (2n + 1)
∴ Sum = Σ Tn2 = + nc2 − 2c2n (n + 1)
7. We have, S = a1 + a 2 + … + a30 6
= 15[2a1 + 29d ] …(i) 2 c2n (n + 1) (2n + 1) + 3nc2 − 6c2n (n + 1)
=
(where d is the common difference) 3
 n  nc2(4n 2 + 6n + 2 + 3 − 6n − 6) nc2(4n 2 − 1)
Q S n = [2a + (n − 1)d ] = =
 2  3 3
and T = a1 + a3 + … + a 29 10. According to given condition,
15 S 2n = S′n
= [2a1 + 14 × 2d )]
2 2n n
⇒ [2 × 2 + (2n − 1) × 3] = [2 × 57 + (n − 1) × 2]
(Q common difference is 2d) 2 2
⇒ 2T = 15[2a1 + 28d ] …(ii) 1
⇒ (4 + 6n − 3) = (114 + 2n − 2)
From Eqs. (i) and (ii), we get 2
S − 2T = 15d = 75 [QS − 2T = 75] ⇒ 6n + 1 = 57 + n − 1 ⇒ 5n = 55
⇒ d =5 ∴ n = 11
Now, a10 = a5 + 5d 11. PLAN Convert it into differences and use sum ofn terms of an AP,
= 27 + 25 = 52 i.e.
n
S n = [2a + ( n − 1 )d ]
2
8. If log b1 , log b2, ... , log b101 are in AP, with common k( k + 1)
difference log e 2 , then b1 , b2, ... , b101 are in GP, with 4n 2
common ratio 2. Now, Sn = ∑ (−1) ⋅ k2
k =1
∴ b1 = 20 b1 , b2 = 21 b1 , b3 = 22b1,…, b101 = 2100 b1 …(i)
Sequences and Series 61

= − (1)2 − 22 + 32 + 42 − 52 − 62 + 72 + 82 + K ∴ p = a − 3d = 5 − 6 = − 1
= (3 − 1 ) + (4 − 2 ) + (7 − 5 ) + (8 − 6 ) + K
2 2 2 2 2 2 2 2
q = a − d =5 −2 =3
= 2{(4 + 6 + 12 + K ) + (6 + 14 + 22 + K )} r = a + d =5 + 2 = 7
144424443 144424443
n terms n terms and s = a + 3d = 5 + 6 = 11
n n  Therefore, A = pq = − 3 and B = rs = 77
=2 {2 × 4 + (n − 1) 8} + {2 × 6 + (n − 1) 8}
 2 2 
16. Here, 12 + 2 ⋅ 22 + 32 + 2 ⋅ 42 + 52 + ... upto n terms
= 2 [n (4 + 4n − 4) + n (6 + 4n − 4)]
n (n + 1)2
= 2 [4n + 4n + 2n ] = 4n (4n + 1)
2 2 = [when n is even] …(i)
2
Here, 1056 = 32 × 33, 1088 = 32 × 34, When n is odd, 12 + 2 ⋅ 22 + 32 + 2 ⋅ 42 + 52 ... + n 2
1120 = 32 × 35, 1332 = 36 × 37 = {12 + 2⋅22 + 32 + 2 ⋅ 42 + ... + 2 (n − 1)2} + n 2
1056 and 1332 are possible answers.  (n − 1) (n )2 
= + n
2
[from Eq. (i)]
r 1 2 
12. Here, V r = [ 2r + (r − 1) ( 2r − 1)] = ( 2r3 − r 2 + r ) 
2 2
n −1  (n + 1)
1 = n2  + 1 = n 2
∴ ΣV r = [ 2 Σr3 − Σr 2 + Σr ]  2  2
2
∴ 12 + 2⋅ 22 + 32 + 2 ⋅ 42 + ... upto n terms, when n is odd
1   n (n + 1)  n (n + 1) ( 2n + 1) n (n + 1) 
2
= 2   − +  n 2 (n + 1)
2  2  6 2  =
2
n (n + 1)
⇒ = [ 3n (n + 1) − ( 2n + 1) + 3] 17. Integers divisible by 2 are {2,4,6,8,10, ...,100}.
12
Integers divisible by 5 are {5,10,15, ...,100}.
1
= n (n + 1) ( 3n 2 + n + 2) Thus, sum of integers divisible by 2
12
50
1 1 = (2 + 100) = 50 × 51 = 2550
13. V r + 1 − V r = (r + 1)3 − r3 − [(r + 1)2 − r 2] + = 3r 2 + 2r − 1 2
2 2
Sum of integers divisible by 5
∴ Tr = 3r 2 + 2r − 1 = (r + 1) ( 3r − 1)
20
which is a composite number. = (5 + 100) = 10 × 105 = 1050
2
14. Since, Tr = 3r 2 + 2r − 1 Sum of integers divisible by 10
and Tr + 1 = 3 (r + 1)2 + 2 (r + 1) − 1 10
= (10 + 100) = 5 × 110 = 550
∴ Qr = Tr+1 − Tr = 3 [ 2r + 1] + 2 [1] 2
⇒ Qr = 6 r + 5 ∴ Sum of integers from 1 to 100 divisible by 2 or 5
⇒ Qr+ 1 = 6(r + 1) + 5 = 2550 + 1050 − 550
Common difference = Qr+ 1 − Qr = 6 = 2550 + 500 = 3050
15. Given, p + q = 2, pq = A 18. Let four consecutive terms of the AP are a − 3d , a − d ,
and r + s = 18, rs = B a + d , a + 3d, which are integers.
Again, required product
and it is given that p, q, r , s are in an AP.
P = (a − 3d )(a − d )(a + d )(a + 3d ) + (2d )4
Therefore, let p = a − 3d , q = a − d , r = a + d
[by given condition]
and s = a + 3d
= (a 2 − 9d 2)(a 2 − d 2) + 16d 4
Since, p<q<r<s
= a 4 − 10a 2d 2 + 9d 4 + 16d 4 = (a 2 − 5d 2)2
We have, d >0
Now, a − 5d = a 2 − 9d 2 + 4d 2
2 2
Now, 2 = p + q = a − 3d + a − d = 2a − 4d
= (a − 3d )(a + 3d ) + (2d )2
⇒ a − 2d = 1 …(i)
= I ⋅ I + I2 [given]
Again, 18 = r + s = a + d + a + 3d
=I +I =I
2 2 2
18 = 2a + 4d
=I [where, I is any integer]
⇒ 9 = a + 2d …(ii)
Therefore, P = (I ) = Integer
2
On subtracting Eq. (i) from Eq. (ii), we get
19. Since, x1 , x2, x3 are in an AP. Let x1 = a − d , x2 = a and
8 = 4d ⇒ d = 2
x3 = a + d and x1 , x2, x3 be the roots of x3 − x2 + βx + γ = 0
On putting in Eq. (ii), we get a = 5
∴ Σα = a − d + a + a + d = 1
62 Sequences and Series

⇒ a = 1 /3 …(i) ⇒
26
<d<
28
[since, d is a natural number]
Σαβ = (a − d ) a + a (a + d ) + (a − d ) (a + d ) = β …(ii) 3 3
and αβγ = (a − d ) a (a + d ) = − γ …(iii) ∴ d =9
From Eq. (i), 22. Let number of removed cards be k and (k + 1).
3a = 1 ⇒ a = 1 / 3 n (n + 1)
∴ − k − (k + 1) = 1224
From Eq. (ii), 3a − d = β
2 2 2
⇒ n 2 + n − 4k = 2450 ⇒ n 2 + n − 2450 = 4k
⇒ 3 (1 / 3)2 − d 2 = β [from Eq. (i)]
⇒ (n + 50) (n − 49) = 4k
⇒ 1 /3 − β = d 2
∴ n > 49
NOTE In this equation, we have two variables β and γ but we have
only one equation. So, at first sight it looks that this equation Let n = 50
cannot solve but we know that d 2 ≥ 0, ∀ d ∈ R, then β can ∴ 100 = 4k ⇒ k = 25
be solved. This trick is frequently asked in IIT examples. Now k − 20 = 5
1
⇒ −β ≥0 [Q d 2 ≥ 0] 23. Given, a1 = 3,m = 5n and a1 , a 2, …, is an AP.
3
Sm S5 n
⇒ β≤
1
⇒ β ∈ [− ∞ , 1 / 3] ∴ = is independent of n.
3 Sn Sn
From Eq. (iii), a (a 2 − d 2) = − γ 5n
[2 × 3 + (5n − 1) d ]
5 {(6 − d ) + 5n }
1 1 2 1 1 2 = 2 = ,
⇒  − d  = −γ ⇒ − d = −γ n (6 − d ) + n
3 9  27 3 [2 × 3 + (n − 1) d ]
2
1 1 2 independent of n
⇒ γ+ = d
27 3 If 6 − d =0 ⇒ d =6

⇒ γ+
1
≥0 ∴ a 2 = a1 + d = 3 + 6 = 9
27 S
or If d = 0, then m is independent of n.
⇒ γ ≥ − 1 / 27 Sn
 1  ∴ a2 = 9
⇒ γ∈ − ,∞
 27 
24. a k = 2a k − 1 − a k − 2
Hence, β ∈ (− ∞ , 1 / 3] and γ ∈ [−1 / 27, ∞ )
20. Since, angles of polygon are in an AP. ⇒ a1 , a 2, . . . , a11 are in an AP.
∴ Sum of all angles a12 + a 22 + . . . + a11
2
11a 2 + 35 × 11d 2 + 10ad
∴ = = 90
n 11 11
= (n − 2) × 180° = {2 (120° ) + (n − 1) 5° }
2 ⇒ 225 + 35 d 2 + 150 d = 90
⇒ 5n 2 − 125n + 720 = 0 ⇒ n 2 − 25n + 144 = 0 ⇒ 35 d 2 + 150 d + 135 = 0
⇒ (n − 9) (n − 16) = 0 ⇒ n = 9, 16 ⇒ d = − 3, −
9
If n = 9, then largest angle = a + 8d = 160° 7
27
Again, if n = 16, the n largest angle Given, a2 <
2
= a + 15d = 120° + 75 = 195°
9
which is not possible. ∴ d = − 3 and d ≠ −
7
[since, any angle of polygon cannot be > 180°] a1 + a 2 + . . . + a11 11
⇒ = [ 30 − 10 × 3] = 0
Hence, n = 9 [neglecting n = 16] 11 2
S 6
21. Given, 7 = and 130 < t7 < 140
S11 11 Topic 3 Geometric Progression (GP)
7
[2a + 6d ]
2 6 7 (2a + 6d ) 1. Key Idea Use nth term of AP i.e., an = a + ( n − 1) d , If a, A , b are in
⇒ = ⇒ =6
11
[2a + 10d ] 11 (2a + 10d ) AP, then 2A = a + b and nth term of G.P. i.e., an = ar n − 1.
2
⇒ a = 9d …(i) It is given that, the terms a , b, c are in GP with common
1
Also, 130 < t7 < 140 ratio r, where a ≠ 0 and 0 < r ≤ .
2
⇒ 130 < a + 6d < 140 So, let, b = ar and c = ar 2
⇒ 130 < 9d + 6d < 140 [from Eq. (i)] Now, the terms 3a, 7b and 15c are the first three terms
⇒ 130 < 15d < 140 of an AP, then
Sequences and Series 63

2(7b) = 3a + 15 c ∴ d (− r )2 + 2e(− r ) + f = 0
⇒ 14ar = 3a + 15ar 2 [as b = ar, c = ar ]
2
⇒ dr 2 − 2er + f = 0
⇒ 14r = 3 + 15r 2 [as a ≠ 0]  c  c
⇒ d   − 2e  + f = 0
⇒ 15r − 14r + 3 = 0
2  a  b
⇒ 15r 2 − 5r − 9r + 3 = 0 ⇒
d 2e f
− + =0
⇒ 5r (3r − 1) − 3(3r − 1) = 0 a b c
⇒ (3r − 1) (5r − 3) = 0 d f 2e
⇒ + = [Q c ≠ 0]
1 3 a c b
⇒ r = or
3 5 a
3. Let the three consecutive terms of a GP are , a and ar.
 1 1 r
as, r ∈ 0, , so r =
 2  3 Now, according to the question, we have
a
Now, the common difference of AP = 7b − 3a ⋅ a ⋅ ar = 512
r
7  2a
= 7ar − 3a = a  − 3 = − ⇒ a3 = 512
3  3
⇒ a=8 … (i)
 −2a 
So, 4 th
term of AP = 3a + 3  =a Also, after adding 4 to first two terms, we get
 3  8
+ 4, 8 + 4, 8r are in AP
r
2. Given, three distinct numbers a , b and c are in GP. 8
⇒ 2 (12) = + 4 + 8r
∴ b2 = ac …(i) r8 2 
and the given quadratic equations ⇒ 24 = + 8r + 4 ⇒ 20 = 4  + 2r
r r 
ax2 + 2bx + c = 0 …(ii) 2
⇒ 5 = + 2r ⇒ 2r − 5r + 2 = 0
2
dx2 + 2ex + f = 0 …(iii) r
For quadratic Eq. (ii), ⇒ 2r − 4r − r + 2 = 0
2

the discriminant D = ( 2b)2 − 4ac ⇒ 2r (r − 2) − 1(r − 2) = 0


= 4(b2 − ac) = 0 [from Eq. (i)] ⇒ (r − 2) (2r − 1) = 0
1
⇒ Quadratic Eq. (ii) have equal roots, and it is equal to ⇒ r = 2,
b 2
x = − , and it is given that quadratic Eqs. (ii) and (iii)
a Thus, the terms are either 16, 8, 4 or 4, 8, 16. Hence,
have a common root, so required sum = 28.
2
 b  b
d−  + 2e  −  + f = 0 4. Let r be the common ratio of given GP, then we have the
 a   a following sequence a1 , a 2 = a1r , a3 = a1r 2, ... , a10 = a1r 9
⇒ db − 2eba + a f = 0
2 2 Now, a3 = 25 a1
⇒ d (ac) − 2eab + a 2f = 0 [Q b2 = ac] ⇒ a1r 2 = 25 a1
⇒ dc − 2eb + af = 0 [Q a ≠ 0] ⇒ r 2 = 25
⇒ 2eb = dc + af a 9 a1r 8
Consider, = = r 4 = (25)2 = 54
e dc af a5 a1r 4
⇒ 2 = 2+ 2
b b b
[dividing each term by b2] 5. Let A be the Ist term of AP and d be the common
difference.
 e d f
⇒ 2  = + [Q b2 = ac] ∴ 7th term = a = A + 6d
 b a c
[Q nth term = A + (n − 1)d]
d e f
So, , , are in AP. 11th term = b = A + 10d
a b c
Alternate Solution 13th term = c = A + 12d
Q a, b, c are also in GP
Given, three distinct numbers a , b and c are in GP. Let
a = a, b = ar, c = ar 2 are in GP, which satisfies ∴ b2 = ac
ax2 + 2bx + c = 0 ⇒ ( A + 10d )2 = ( A + 6d ) ( A + 12d )
∴ ax2 + 2(ar )x + ar 2 = 0 ⇒ A + 20 Ad + 100d 2 = A 2 + 18 Ad + 72d 2
2

⇒ x2 + 2rx + r 2 = 0 [Q a ≠ 0] ⇒ 2 Ad + 28d 2 = 0
⇒ (x + r )2 = 0 ⇒ x = − r. ⇒ 2d ( A + 14d ) = 0
According to the question, ax2 + 2bx + c = 0 and ⇒ d = 0 or A + 14d = 0
dx2 + 2ex + f = 0 have a common root. But d ≠0 [Q the series is non constant AP]
So, x = − r satisfies dx2 + 2ex + f = 0 ⇒ A = − 14d
64 Sequences and Series

∴ a = A + 6d = − 14d + 6d = − 8d 9. Since, a, b and c are in an AP.


and c = A + 12d = − 14d + 12d = − 2d Let a = A − D, b = A, c = A + D
a − 8d
⇒ = =4 3
c − 2d Given, a + b+ c=
2
6. Let b = ar and c = ar 2, where r is the common ratio. ⇒ ( A − D) + A + ( A + D) =
3
2
Then, a + b + c = xb
3 1
⇒ a + ar + ar 2 = xar ⇒ 3A = ⇒ A =
2 2
⇒ 1 + r + r 2 = xr … (i) [Q a ≠ 0] 1 1 1
∴ The number are − D , , + D.
1 + r + r2 1 2 2 2
⇒ x= =1 + r +
r r 2 2
1  1 1 
1 Also,  − D , ,  + D are in GP.
We know that, r + ≥ 2 (for r > 0) 2  4 2 
r
2 2 2 2
1
r + ≤ − 2 (for r < 0) [using AM ≥ GM]  1 1  1  1 1 
and ∴   =  − D  + D ⇒ =  − D 2
r  4 2  2  16  4 
1
∴ 1+r + ≥ 3 ⇒
1
− D2 = ±
1
⇒ D2 =
1
r 4 4 2
1
or 1 + r + ≤ −1 1 1 1
r ⇒ D=± ⇒ a= ±
2 2 2
⇒ x ≥ 3 or x ≤ −1
⇒ x ∈ ( − ∞ ,−1] ∪ [3, ∞ ) 1 1
So, out of the given values, a = − is the right
2 2
Hence, x cannot be 2.
choice.
Alternate Method
α + β =1 λ + δ = 4
From Eq. (i), we have 10.  and 
αβ = p λ δ = q
1 + r + r 2 = xr ⇒ r 2 + (1 − x)r + 1 = 0
Let r be the common ratio.
For real solution of r , D ≥ 0.
Since, α , β , γ and δ are in GP.
⇒ (1 − x )2 − 4 ≥ 0
Therefore, β = αr, γ = αr 2
⇒ x 2 − 2x − 3 ≥ 0
and δ = αr3
⇒ ( x − 3)( x + 1) ≥ 0 Then, α + αr = 1 ⇒ α (1 + r ) = 1 …(i)
+ – +
and αr 2 + αr3 = 4 ⇒ αr 2(1 + r ) = 4 …(ii)
–1 3
From Eqs. (i) and (ii), r 2 = 4 ⇒ r = ± 2
⇒ x ∈ ( −∞ , − 1] ∪ [3, ∞ )
∴ x cannot be 2. Now, α ⋅ αr = p and αr 2 ⋅ αr3 = q
On putting r = − 2, we get
7. Let a be the first term and d be the common difference.
Then, we have a + d, a + 4d, a + 8 d in GP, α = − 1, p = − 2 and q = − 32
i.e. (a + 4d ) 2 = (a + d ) (a + 8 d ) 2
Again putting r = 2, we get α = 1 / 3 and p = −
⇒ a + 16 d 2 + 8ad = a 2 + 8ad + ad + 8 d 2
2 9
⇒ 8 d 2 = ad Since, q and p are integers.
⇒ 8d = a [Q d ≠ 0] Therefore, we take p = − 2 and q = − 32.
Now, common ratio, 11. Here, (a 2 + b2 + c2) p2 − 2 (ab + bc + cd ) p
a + 4d 8 d + 4d 12 d 4
r= = = = + (b2 + c2 + d 2) ≤ 0
a+d 8d + d 9d 3
⇒ (a 2p2 − 2abp + b2) + (b2p2 − 2bcp + c2)
8. Since, (α + β), (α 2 + β 2), (α 3 + β3 ) are in GP.
+ (c2p2 − 2cdp + d 2) ≤ 0
⇒ (α 2 + β 2)2 = (α + β ) (α 3 + β3 )
⇒ (ap − b) + (bp − c) + (cp − d ) ≤ 0
2 2 2
⇒ α 4 + β 4 + 2α 2β 2 = α 4 + β 4 + αβ3 + βα 3
[since, sum of squares is never less than zero]
⇒ αβ (α 2 + β 2 − 2αβ ) = 0
Since, each of the squares is zero.
⇒ αβ (α − β )2 = 0
∴ (ap − b)2 = (bp − c)2 = (cp − d )2 = 0
⇒ α β =0 or α =β
c b c d
⇒ =0 or ∆ =0 ⇒ p= = =
a a b c
⇒ c∆ = 0 ∴ a , b, c,d are in GP.
Sequences and Series 65

12. Since, a , b, c are in GP. 1 3


and + =0
n− p m−n
⇒ b = ac
2

Given, ax + 2bx + c = 0
2 ⇒ 3 (n − p) = n − m
⇒ ax + 2 ac x + c = 0
2 and 2n = 3 p – m
c Hence, there exists infinite GP for which 27, 8 and 12 as
⇒ ( a x+ c )2 = 0 ⇒ x = − three of its terms.
a
16. Let a , d be the first term and common difference of an
Since, ax2 + 2bx + c = 0 and dx2 + 2ex + f = 0 have
common root. AP and b, r be the first term and common ratio of a GP.
Then, x = a + (m − 1) d and x = brm−1
∴ x = − c/a must satisfy.
y = a + (n − 1)d and y = br n−1
dx2 + 2ex + f = 0
z = a + ( p − 1)d and z = br p−1
c c d 2e f
⇒ d ⋅ − 2e + f =0 ⇒ − + =0 Now, x − y = (m − n )d, y − z = (n − p)d
a a a ac c
2e d f and z − x = ( p − m)d
⇒ = + [Q b2 = ac] Again now, xy − z ⋅ yz − x ⋅ z x − y
b a c
d e f = [br m − 1 ]( n − p)d ⋅ [br n − 1 ]( p − m) d ⋅ [br p − 1 ](m − n)d
Hence, , , are in an AP.
a b c = b[ n − p + p − m + m − n] d ⋅ r [(m − 1)( n − p) + ( n − 1)( p − m)+ ( p − 1)(m − n)]d
13. Here, t3 = 4 ⇒ ar 2 = 4 = b0 ⋅ r 0 = 1
∴ Product of first five terms = a ⋅ ar ⋅ ar 2 ⋅ ar3 ⋅ ar 4
= a5 r10 = (a r 2)5 = 45
Topic 4 Sum of n Terms and Infinite
Terms of a GP
14. If a , b, c ∈ (2, 18), then
1. Given series of infinite terms, if|x| < 1 ,| y| < 1 and x ≠ y is
a + b + c = 25 ....(i)
S = (x + y) + (x2 + xy + y2) + (x3 + x2y + xy2 + y3 ) + …
Since, 2, a , b are in AP. On multiplying (x − y) both sides, we get
⇒ 2a = b + 2 .... (ii)
⇒ (x − y)S = (x2 − y2) + (x3 − y3 ) + (x4 − y4 ) + ....
and b, c, 18 are in GP.
⇒ c2 = 18b ... (iii) = [x2 + x3 + x4 + ... ] − [ y2 + y3 + y4 + .... ]
From Eqs. (i), (ii) and (iii), x2 y2
= − as|x| < 1 and| y| < 1
b+2 1−x 1−y
+ b + 18b = 25
2 x2(1 − y) − y2(1 − x) (x2 − y2) − (x2y − y2x)
⇒ 3b + 2 + 6 2 b = 50 ⇒ (x − y) S = =
(1 − x)(1 − y) (1 − x)(1 − y)
⇒ 3b + 6 2 b − 48 = 0 (x − y) [(x + y) − xy] x + y − xy
⇒ b + 2 2 b − 16 = 0 ⇒ (x − y) S = ⇒S=
(1 − x) (1 − y) (1 − x) (1 − y)
⇒ b + 4 2 b − 2 2 b − 16 = 0 20
 1
⇒ b ( b + 4 2) − 2 2 ( b + 4 2) = 0 2. Let S = ∑ k 2k 
k =1
⇒ ( b − 2 2) ( b + 4 2) = 0
1 2 3 4 20
⇒ b = 8, a = 5 and c = 12 S= + + + + … + 20 …(i)
2 22 23 24 2
15. Let 27, 8, 12 be three terms of a GP.  1
On multiplying by   both sides, we get
⇒ tm = 27, tn = 8 and t p = 12  2
AR m − 1 = 27, ARn − 1 = 8 S 1 2 3 19 20
= 2 + 3 + 4 + … + 20 + 21 …(ii)
2 2 2 2 2 2
and AR p − 1 = 12
1/(m − n) 1/( n − p) On subtracting Eq. (ii) from Eq. (i), we get
 27 8
∴ R=  and R=  S 1 1 1 1 20
8  12 S− = + + + … + 20 − 21
1/(m − n) 1/( n − p)
2 2 22 23 2 2
 27  2
⇒   =  1 1 
8  3 1 − 20 
S 2 2  − 20
⇒ 33/(m − n) ⋅ 31/( n − p) = 21/( n − p) ⋅ 23/(m − n) ⇒ =
3 1
2 1−
1 221
+
m− n n− p 2
3 3 1  a (1 − r n ) 
⇒ =1 ⇒ + =0 Q sum of GP = , r < 1
1
+
3
m−n n− p  1−r 
n− p m− n
2
66 Sequences and Series

S 1 20 1 10 11 27 27 (1 − r ) (1 + r 2 − 2r )
= 1 − 20 − 21 = 1 − 20 − 20 = 1 − 20 ⇒ =
2 2 2 2 2 2 19 (1 − r ) (1 + r + r 2)
11 [Q (1 − r )3 = (1 − r ) (1 − r )2]
⇒ S =2 −
219 ⇒ r 2 + r + 1 = 19 (r 2 − 2r + 1)
⇒ 18r 2 − 39r + 18 = 0
3. We have, S n = 1 + q + q + … + q and 2 n
⇒ 6r 2 − 13r + 6 = 0
 q + 1  q + 1  q + 1
2 n ⇒ (3r − 2) (2r − 3) = 0
Tn = 1 +   +  +…+   2 3
 2   2   2  ∴ r = or r = (reject) [Q0 < r < 1]
Also, we have 3 2
101
C1 + 101
C2S1 + 101
C3S 2 + … + C101S100 = αT100
101 5. Let a , ar , ar 2 are in GP, where (r > 1).
On multiplying middle term by 2, we have
⇒ 101
C1 + 101
C2(1 + q ) + 101
C3 (1 + q + q 2 )
a , 2ar , ar 2 are in an AP.
+…+ 101
C101 (1 + q + q2 + … + q100) ⇒ 4ar = a + ar 2
= α ⋅ T100 ⇒ r − 4r + 1 = 0
2

(1 − q 2 )  1 − q3  4 ± 16 − 4
⇒ 101C1 + 101
C2 + 101
C3   ⇒ r= =2 ± 3
1− q  1− q  2
 1 − q4   1 − q101  ⇒ r =2 + 3 [since, AP is increasing]
+ 101
C4   +…+ 101
C101  
 1−q  1−q  6. Given,
 1 − rn k ⋅ 109 = 109 + 2 (11)1 (10)8 + 3(11)2(10)7 + ... + 10(11)9
= α ⋅ T100 [Qfor a GP, S n = a  ,r ≠1] 2 9
1−r  11  11  11
⇒ k = 1 + 2   + 3  + ... + 10   ...(i)
1  10  10  10
⇒ [{ 101C1 + 101C2 + … + 101C101 } 2 9 10
1− q  11  11  11  11  11
 k =1   + 2   + ... + 9   + 10   …(ii)
− { 101C1q + 101C2q 2 + … +101 C101q101 } = α ⋅ T100  10  10  10  10  10
1 On subtracting Eq. (ii) from Eq. (i), we get
⇒ [( 2101 − 1) − ((1 + q )101 − 1)] = αT100
(1 − q )  11 11  11
2
 11  11
9 10
k 1 −  = 1 + +   + ... +   − 10  
 10 10  10  10  10
[Q nC 0 + nC1 + … + nC n = 2n]
2101 − ( q + 1)101  11 10 
⇒ =α 1   − 1
1− q  10 − 11   10
⇒ k  10  − 10  11
 =  
 q + 1  q + 1
2
 q + 1 
100  10   11   10
 − 1
1 + +
 2 
 +…+ 
 2 
   10 
 2 
 a (r n − 1) 
  q + 1 
101
Q In GP,sum of n terms = r − 1 , when r > 1
 1−     
2101
− ( q + 1) 101
 2  
⇒ = α 1 ⋅   11 10  11
10 
1− q  q+1  ⇒ − k = 10 10   − 10 − 10   
1−  10  10
 2   
 
∴ k = 100
q+1
[Q q ≠ 1 ⇒ q + 1 ≠ 2 ⇒ ≠ 1]
2 7. Let S = 0.7 + 0.77 + 0.777 + …
α [2 101
− (q + 1) 101
] 7 77 777
= ⇒ α = 2100 = + + + … upto 20 terms
(1 − q) ⋅ 2100 10 102 103
1 11 111 
4. Let the GP be a , ar , ar 2, ar3 , .... ∞; where a > 0 and =7 + + + … upto 20 terms
10 102 103 
0 < r < 1. 79 99 999 
Then, according the problem, we have = + + +… upto 20 terms
a 9 10 100 1000 
3=
1−r 7  1  1   1 
= 1 − 10 + 1 − 102 + 1 − 103 
and
27
= a3 + (ar )3 + (ar 2)3 + (ar3 )3 + ...
9 
19 + … + upto 20 terms]
27 a3  a  7
⇒ = Q S∞ = = [(1 + 1 +…+ upto 20 terms)
19 1 − r 3  1 − r  9

27 (3 (1 − r )) 
3
a  1 1 1 
⇒ = Q3 = ⇒ a = 3 (1 − r ) −  + 2 + 3 + … + upto 20 terms 
 10 10 
19 1 − r3  1−r 
10
Sequences and Series 67

 1   1  
20
11. Given quadratic equation x2 − x − 1 = 0 having roots α
 1 −     and β, (α > β )
7 10  10  
= 20 −  1+ 5 1− 5
9 1 So, α = and β =
 1−  2 2
 10 
  and α + β = 1, αβ = −1
 20  α n − βn
Q ∑ = 20 and sum of n terms of  Q an =
α −β
, n ≥1
 i =1 
 a (1 − r n )  α n+ 1 − β n+ 1
GP, S n = 1 − r when (r < 1)  So, a n+ 1 = = α n + α n−1 β + α n− 2 β 2+ ....
  α −β
+ αβ n−1 + β n
7  1  1  
20
n− 2 n−3 n−2
= 20 − 1 −    = α −α
n
−α β −....−β +β n
[as αβ = −1]
9  9  10
  = α n + β n − (α n− 2 + α n−3β + ...+ β n− 2)
179 1  1  20  7 = α n + β n − a n−1
7
=  +   = [179 + (10)− 20 ]  α n−1 − β n−1 
 10 = α n− 2 + α n−3 β + ...+ β n− 2
9  9 9  81  as a n−1 = α − β
 
8. We know that, the sum of infinite terms of GP is ⇒ a n+ 1 + a n−1 = α n + β n = bn , ∀n ≥ 1
 a
, |r| < 1 So, option (b) is correct.
S ∞ = 1 − r ∞ ∞
 ∞ ,|r| ≥ 1 b α n + βn
Now, ∑ nn = ∑ n
[as, bn = α n + β n]
n=1 10 n=1 10
x
∴ S∞ = =5 [|r| < 1] ∞ ∞ n n
1−r α β  α β 
=∑  +∑  Q 10 < 1 and 10 < 1
x  10  10  
n=1 n=1
or 1−r=
5 α β
5−x 10 10 α β
⇒ r= exists only when|r| < 1. = + = +
5 α β 10 − α 10 −β
1− 1−
5−x 10 10
i.e. −1 < < 1 or −10 < − x < 0 10 α − αβ + 10 β − αβ 10(α + β ) − 2 αβ
5 = =
(10 − α ) (10 − β ) 100 − 10(α + β ) + αβ
⇒ 0 < x < 10
10(1) − 2(−1) 12
3 = = [as α + β = 1 and αβ = −1]
9. Since, sum = 4 and second term = . 100 − 10(1) − 1 89
4
So, option (a) is not correct.
It is given first term a and common ratio r.
a 3 3 Q α 2 = α + 1 and β 2 = β + 1
⇒ = 4, ar = ⇒ r = ⇒ α n+ 2
= α n+ 1 + α n and β n+ 2 = β n+ 1 + β n
1−r 4 4a
n+ 2
⇒ (α + β ) = (α n+ 1 + β n+ 1 ) + (α n + β n )
n+ 2
a 4a 2
⇒ =4 ⇒ =4 ⇒ a n+ 2 = a n+ 1 + a n
1−
3 4a − 3
Similarly, a n+ 1 = a n + a n−1
4a
a n = a n−1 + a n−2
⇒ (a − 1) (a − 3) = 0 ⇒ a = 1 or 3 ………
When a = 1, r = 3 / 4 ………
a3 = a 2 + a1
and when a = 3, r = 1 / 4 On adding, we get
10. Sum of the n terms of the series
1 3 7 15
+ + + + ... a n+ 2 = (a n + a n−1 + a n− 2+ ....+ a 2 + a1 ) + a 2
2 4 8 16  α 2 − β2 
upto n terms can be written as Q a 2 = = α + β = 1
 α −β 
 1  1  1  1
1 −  + 1 −  + 1 −  + 1 −  + ... upto n terms So, a n+ 2 − 1 = a1 + a 2 + a3 + .....+ a n
 2  4  8  16
So, option (c) is also correct.
1 1 1  ∞ ∞
= n −  + + + ... + n terms an α n − βn
2 4 8  And, now ∑ =∑
n=1 (α − β ) 10
n n
n=1 10
1 1
1 − n  1 ∞ α
n ∞
β 
n
2 
=n− 2 = n + 2−n − 1 = ∑   − ∑   
1 α − β n=1 10 n=1
10 
1−
2
68 Sequences and Series

 α β 
∴ S1 =
1
=2
1  10 10   α β  1 − 1 /2
=  − , as < 1 and < 1
α −β 1 − α β
 1 −   10 10 
S2 =
2
=3
 10 10  1 − 1 /3
1  α β  1  10 α − αβ − 10 β + αβ 
=  −  = 100 − 10 (α + β ) + αβ  M M M
α − β  10 − α 10 − β α − β   2n − 1
10(α − β ) 10 10 S 2n − 1 = = 2n
= = = 1 − 1 / 2n
(α − β ) [100 − 10 (α + β ) + αβ ] 100 − 10 − 1 89
∴ S12 + S 22 + S32 + ... + S 22n − 1
Hence, options (b), (c) and (d) are correct.
= 22 + 32 + 42 + ... + (2n )2
12. Let a n denotes the length of side of the square S n. 1
= (2n ) (2n + 1) (4n + 1) − 1
We are given, a n = length of diagonal of S n + 1. 6
a
⇒ an = 2 an + 1 ⇒ an + 1 = n 15. Let three numbers in GP be a , ar , ar 2.
2
∴ a 2 + a 2r 2 + a 2r 4 = S 2 ...(i)
This shows that a1 , a 2, a3 , K form a GP with common
ratio 1 / 2. and a + ar + ar 2 = a S ...(ii)
n −1
 1 On dividing Eq. (i) by Eq. (ii) after squaring it, we get
Therefore, a n = a1  
 2 a 2 (1 + r 2 + r 4 ) S2 (1 + r 2)2 − r 2 1
n −1 = 2 2 ⇒ =
 1 a (1 + r + r )
2 2 2
aS (1 + r + r 2)2 a 2
⇒ a n = 10   [ Q a1 = 10, given]
 2 1
r+ +1
2( n − 1) (1 + r 2 − r ) 1 r
 1 100 ⇒ = ⇒ a =
2
⇒ a n2 = 100   ⇒ ≤1 [Q a n2 ≤ 1, given] (1 + r 2 + r ) a 2 1
 2 2n − 1 r + −1
r
⇒ 100 ≤ 2n − 1 1 y+ 1
Put r+ =y ⇒ =a 2
This is possible for n ≥ 8. r y−1
Hence, (b), (c), (d) are the correct answers. ⇒ y + 1 = a 2y − a 2
13. Bn = 1 − An > An a2 + 1  1 
⇒ y= Q | y | = r + r > 2
 n a2 − 1  
1 −  −  
3
1 3  4  1 a2 + 1
⇒ An < ⇒ < ⇒ >2 [where , (a 2 − 1) ≠ 0]
2 4 3 2 a2 −1
1+
4 ⇒ |a 2 + 1| > 2|a 2 − 1|
n
 3 1 ⇒ (a + 1) − {2 (a 2 − 1)}2 > 0
2 2
⇒ −  > −
 4 6 ⇒ {(a 2 + 1) − 2 (a 2 − 1)}{(a 2 + 1) + 2 (a 2 − 1)} > 0
Obviously, it is true for all even values of n. ⇒ (− a 2 + 3) (3a 2 − 1) > 0
But for 1
3 1 ∴ < a2 < 3
n = 1, − <− 3
1 
4 6 ∴ a 2 ∈  , 1 ∪ (1, 3) [Q a 2 ≠ 1]
3 
 3 27
3
1 k −1
n = 3,  −  = − <− 1
 4 64 6 16. We have, S k = k ! =
1 (k − 1)!
 3
5
243 1 1−
n = 5,  −  = − <− k
 4 1024 6 1
Now, (k2 − 3k + 1) S k = {(k − 2) (k − 1) − 1} ×
and for n = 7, (k − 1)!
1 1
 3
7
2187 1 = −
−  = − >− (k − 3)! (k − 1)!
 4 12288 6 100
 1 1  1002
Hence, minimum odd natural number n0 = 7. ⇒ ∑|(k2 − 3k + 1) S k|= 1 + 1 + 2 −  +  =4−
 99 ! 98 ! 100 !
k =1
14. Consider an infinite GP with first term 1, 2, 3, ..., n and 100
1002
100 ! k∑
common ratios
1 1 1
, , , ... ,
1
. ⇒ + |(k2 − 3k + 1) S k|= 4
2 3 4 n+1 =1
Sequences and Series 69

1 1 1
Topic 5 Harmonic Progression (HP) and = =
1 + ln z 1 + ln x + 2 ln r A + 2D
1
1. PLAN nth term of HP, t n = 1 1 1
a + ( n − 1) n Therefore, , , are in HP.
1 + ln x 1 + ln y 1 + ln z
Here, a1 = 5, a 20 = 25 for HP
1 1 5. Let a1 = 1, a 2 = 2 , ⇒ a3 = 4 , a 4 = 8
∴ = 5 and = 25
a a + 19d ∴ b1 = 1, b2 = 3, b3 = 7, b4 = 15
1 1 1 1 4 Clearly, b1 , b2, b3 , b4 are not in HP.
⇒ + 19d = ⇒ 19d = − =−
5 25 25 5 25 Hence, Statement II is false.
−4
∴ d= Statement I is already true.
19 × 25
1 6. Since, cos (x − y), cos x and cos (x + y) are in HP.
Since, an < 0 ⇒ + (n − 1) d < 0 2 cos (x − y) cos (x + y)
5 ∴ cos x =
1 4 95 cos (x − y) + cos (x + y)
⇒ − (n − 1) < 0 ⇒ (n − 1) >
5 19 × 25 4 ⇒ cos x (2 cos x ⋅ cos y) = 2 {cos 2 x − sin 2 y}
95 ⇒ cos 2 x ⋅ cos y = cos 2 x − sin 2 y
⇒ n >1 + or n > 24.75
4 ⇒ cos x (1 − cos y) = sin 2 y
2

∴ Least positive value of n = 25 y y y


⇒ cos 2 x ⋅ 2 sin 2 = 4 sin 2 ⋅ cos 2
2 2 2
2. Since, a , b, c, d are in AP. 2 y y
⇒ cos x ⋅ sec = 2 ⇒ cos x ⋅ sec = ± 2
2
a b c d 2 2
⇒ , , , are in AP.
abcd abcd abcd abcd
7. Since, a , b, c are in an AP.
1 1 1 1
⇒ , , , are in AP. ∴ 2b = a + c
bcd cda abd abc
⇒ bcd , cda , abd , abc are in HP. and a 2, b2, c2 are in HP.
2
⇒ abc, abd , cda , bcd are in HP. 2a 2c2  a + c 2a 2c2
⇒ b2 = ⇒   = 2 2
a 2 + c2  2  a +c
3. Since, a1 , a 2, a3 , K , a10 are in AP.
⇒ (a 2 + c2)(a 2 + c2 + 2ac) = 8a 2c2
Now, a10 = a1 + 9d ⇒ 3 = 2 + 9d
⇒ (a 2 + c2) + 2ac(a 2 + c2) = 8a 2c2
⇒ d = 1 / 9 and a 4 = a1 + 3d
⇒ (a + c ) + 2ac(a 2 + c2) + a 2c2 = 9a 2c2
2 2
⇒ a 4 = 2 + 3(1 / 9) = 2 + 1 / 3 = 7 / 3
⇒ (a 2 + c2 + ac)2 = 9a 2c2
Also, h1 , h2, h3 , K , h10 are in HP.
1 1 1 1 ⇒ a 2 + c2 + ac = 3ac
⇒ , , ,K, are in AP.
h1 h2 h3 h10 ⇒ a 2 + b2 – 2ac = 0
Given, h1 = 2, h10 = 3 ⇒ (a – c)2 = 0 ⇒ a = c
1 1 1 1 1 and if a = c ⇒ b = c or a 2 + c2 + ac = – 3ac
∴ = + 9d1 ⇒ = + 9d1 ⇒ − = 9d1
h10 h1 3 2 6 ⇒ a 2 + c2 + 2ac = –2ac
1 1 1 ⇒ (a + c)2 = –2ac
⇒ d1 = − and = + 6d1
54 h7 h1 ac
⇒ 4b2 = –2ac ⇒ b2 = –
1 1 6 ×1 2
⇒ = + c
h7 2 − 54 Hence, a , b, – are in GP.
1 1 1 18 2
⇒ = − ⇒ h7 = c
h7 2 9 7 ∴ Either a = b = c or a , b, − are in GP.
2
7 18
∴ a 4h7 = × =6 8. Since, a , A1 , A2, b are in AP.
3 7
⇒ A1 + A2 = a + b
4. Let the common ratio of the GP be r. Then,
a , G1 , G2, b are in GP ⇒ G1G2 = ab
y = xr and z = xr 2

and a , H 1 , H 2, b are in HP.


⇒ ln y = ln x + ln r and ln z = ln x + 2 ln r
3ab 3ab
Let A = 1 + ln x, D = ln r ⇒ H1 = , H2 =
2b + a b + 2a
1 1 1 1 1
Then, = , = = 1 1 1 1
1 + ln x A 1 + ln y 1 + ln x + ln r A + D ∴ + = +
H1 H 2 a b
70 Sequences and Series

H 1 + H 2 A1 + A2 1 1 2
⇒ = = + …(i)  n( n + 1)
H 1H 2 G1G2 a b 15  
15
13 + 23 + K + n3  2 
= ∑ = ∑
Now,
G1G2
=
ab
n =1 1 + 2 + K + n n =1
n ( n + 1)
H 1H 2  3ab   3ab  2
   
 2b + a   b + 2a   n 2
n (n + 1) 
 n (n + 1)
n

(2a + b) (a + 2b) Q ∑ r3 =   and ∑ r = 


= …(ii)  r = 1  2  2 
r =1
9ab 15
n( n + 1) 1 15
From Eqs. (i) and (ii), we get = ∑ 2
=
2
∑ (n 2 + n )
G1G2 A + A2 ( 2a + b) (a + 2b) n =1 n =1
= 1 =
H 1H 2 H 1 + H 2 9ab 1 15 × 16 × 31 15 × 16 
= +
2  6 2 
9. (i) Now, a + b = (a + x + y + z + b) − (x + y + z )
 n n (n + 1) (2n + 1) 
5 Q ∑ r =
2
= (a + b) − 15 
2  r = 1 6 
[since, a , x, y, z are in AP] 1
= [(5 × 8 × 31) + (15 × 8)]
5 2
∴ Sum = (a + b)
2 = (5 × 4 × 31) + (15 × 4) = 620 + 60 = 680
⇒ a + b = 10 1 1 15 × 16
…(i) and S 2 = (1 + 2 + 3 + K + 15) = × = 60
1 1 1 1 1 2 2 2
Since, a , x, y, z , b are in HP, then , , , ,
are in AP. a x y z b Therefore, S = S1 − S 2 = 680 − 60 = 620.
1 1  1 1 1 1 1   1 1 1 2. Given series is
Now, + = + + + +  − + + 
a b  a x y z b  x y z  3 × 13 5 × (13 + 23 ) 7 × (13 + 23 + 33 )
+ + + ...
5  1 1 5 a + b 10 1 2
1 +2
2 2
12 + 22 + 33
=  +  − ⇒ =
2  a b 3 ab 9 So, nth term
9 × 10 (3 + (n − 1)2)(13 + 23 + 33 ... + n3 )
⇒ ab = [from Eq. (i)] Tn =
10 12 + 22 + 32 + K + n 2
⇒ ab = 9 …(ii)  n (n + 1)
2
(2n + 1) ×  
On solving Eqs. (i) and (ii), we get  2 
=
a = 1, b = 9 n (n + 1)(2n + 1)
6
(ii) LHS = log (x + z ) + log (x + z − 2 y)
 n 3  n (n + 1)  2 n n (n + 1)(2n + 1) 
  2 xz    2xz  Q rΣ= 1 r =  and Σ r 2 =
= log (x + z ) + log x + z − 2   Qy=  
 
  x + z   x + z   2 r =1 6
3n (n + 1) 3 2
(x − z )2 So, Tn = = (n + n )
= log (x + z ) + log 2 2
(x + z )
Now, sum of the given series upto n terms
= 2 log (x − z ) = RHS 3
S n = ΣTn = [Σn 2 + Σn ]
2
Topic 6 Relation between AM, GM, HM 3  n (n + 1)(2n + 1) n (n + 1) 
= +
and Some Special Series 2  6 2 
1. Given series, 3 10 × 11 × 21 10 × 11 
∴ S10 = +
13 + 23 13 + 23 + 33 2  6 2 
S =1+ + + ... + 3
1+ 2 1+ 2+ 3 = [(5 × 11 × 7) + (5 × 11)]
2
13 + 23 + 33 + K + 153 1
− (1 + 2 + 3 + K + 15) 3 3
= × 55(7 + 1) = × 55 × 8 = 3 × 55 × 4
1 + 2 + 3 + K + 15 2 2 2
= S1 − S 2 (let) = 12 × 55 = 660
where,
3. (b) Given series is
13 + 23 13 + 23 + 33 1 + (2 × 3) + (3 × 5) + (4 × 7) + …upto
S1 = 1 + + +K+
1+ 2 1+ 2+ 3 11 terms.
13 + 23 + 33 + K + 153 Now, the rth term of the series is a r = r (2r − 1)
1 + 2 + 3 + K + 15 ∴Sum of first 11-terms is
Sequences and Series 71

( x m + x − m ) ≥ 2 and ( y n + y − n ) ≥ 2
11 11 11 11
S11 = ∑ r (2r − 1) = ∑ (2r 2 − r ) = 2 ∑ r 2 − ∑ r 1
r =1 r =1 r =1 r =1 [Q If x > 0, then x + ≥ 2]
11 × (11 + 1)(2 × 11 + 1) 11 × (11 + 1) x
=2 − ⇒ ( x m + x − m )( y n + y − n ) ≥ 4
6 2
 n
1 1
n (n + 1)(2n + 1) n
n (n + 1)  ⇒ ≤
Q ∑ r = and ∑ r = ( x m + x − m )( y n + y − n ) 4
2

 r = 1 6 r =1
2  1
∴ Maximum value = .
 11 × 12 × 23  11 × 12 4
=  − 
 3   2 
7. General term of the given series is
= (11 × 4 × 23) − (11 × 6) = 11(92 − 6) = 11 × 86 = 946
3r (12 + 22 + K + r 2) 3r [r (r + 1) (2r + 1)]
Tr = =
4. Given series is 2r + 1 6(2r + 1)
3 3 3 3
 3  1  1  3 1 3
  + 1  +  2  + 3 +  3  + ...
3
= (r + r ) 2
 4  2  4  4 2
15
3 3 3 3 1 15
 3  6  9
Let S =   +   +   +  
 12 Now, required sum = ∑ Tr = ∑ (r3 + r 2)
 4  4  4  4 r =1
2 r =1

 15
3
 n (n + 1)  2 n (n + 1) (2n + 1) 
1
+   + … + upto 15 terms =  
+ 
 4 2 2 6 n = 15
3
 3 1  n (n + 1)  n 2 + n 2n + 1 
=   [13 + 23 + 33 + 43 + 53 + ... + 153 ] =  + 
 4 2 2  2 3 
3 2  n = 15
 3  15 × 16
=    1  n (n + 1) (3n 2 + 7n + 2) 
 4  2  =  
2 2 6 n = 15
 3  n ( n + 1)
2  1 15 × 16 (3 × 225 + 105 + 2)
Q1 + 2 + 3 + ... + n = 
3 3 3
 , n ∈N = × × = 7820
  2   2 2 6
8. We have, a1 , a 2, a3 , … a 49 are in AP.
27 225 × 256
= × = 27 × 225 12
64 4 ∑ a 4k + 1 = 416 and a 9 + a 43 = 66
k=0
⇒ S = 27 × 225 = 225 k [given]
⇒ k = 27. Let a1 = a and d = common difference
1 + 2 + 3 + ... + k Q a1 + a5 + a 9 + L + a 49 = 416
5. Since, S k = ∴ a + (a + 4d ) + (a + 8d ) + … (a + 48d ) = 416
k
k ( k + 1) k + 1 ⇒
13
(2a + 48d ) = 41633a + 24d = 32
= = 2
…(i)
2k 2
2 Also a 9 + a 43 = 66 ⇒ a + 8d + a + 42d = 66
 k + 1 1
So, S 2k =   = (k + 1)
2
… (i) ⇒ 2a + 50d = 66 ⇒ a + 25d = 33 …(ii)
 2  4
10 Solving Eqs. (i) and (ii), we get
5
Now, A = S 12 + S 22 + S 23 + ... S 10
2
= ∑ S 2k a = 8and d = 1
12 k =1
Now, a1 + a 2 + a3 + L + a17
2 2 2 2
= 140m
10
5 1 1 8 + 9 + 10 + … + 242 = 140m
2 2 2

12
A=
4 ∑ (k + 1)2 = 4 [22 + 32 + 42 + ... 112]
k =1 ⇒ (12 + 22 + 32 + … + 242) − (12 + 22
1 11 × (11 + 1) (2 × 11 + 1) 2 + 3 + … + 7 ) = 140m
2 2
= −1 24 × 25 × 49 7 × 8 × 15
4  6  ⇒ − = 140m
n ( n + 1) ( 2n + 1) 6 6
[Q ∑ n 2 = ] 3 × 7 ×8 ×5
6 ⇒ (7 × 5 − 1) = 140m
6
1 11 × 12 × 23  1
= − 1 = [(22 × 23) − 1] ⇒ 7 × 4 × 5 × 34 = 140m
4  6  4
1 1 5 505 ⇒ 140 × 34 = 140m ⇒ m = 34
= [506 − 1] = [505] ⇒ A= ⇒ A = 303
4 4 12 4 9. We have,
xm yn 1 12 + 2 ⋅ 22 + 32 + 2 ⋅ 42 + 52 + 2 ⋅ 62 + …
6. Consider, = m
(1 + x )(1 + y ) (x + x )( yn + y−n )
2m 2n −m A = sum of first 20 terms
By using AM ≥ GM (because x , y ∈ R + ), we get B = sum of first 40 terms
72 Sequences and Series

∴A = 12 + 2 ⋅ 22 + 32 + 2 ⋅ 42 + 52 + 2 ⋅ 62 + … + 2 ⋅ 202 12. PLAN Write the nth term of the given series and simplify it to get its
A = (12 + 22 + 32 + … + 202) + (22 + 42 + 62 + … + 202) lowest form. Then, apply, S n = ∑ Tn
A = (12 + 22 + 32 + … + 202) + 4(12 + 22 + 32 + …+102)
13 13 + 23 13 + 23 + 33
20 × 21 × 41 4 × 10 × 11 × 21 Given series is + + + ...
A= + 1 1+3 1+3+5
6 6
20 × 21 20 × 41 × 63 Let Tn be the nth term of the given series.
A= (41 + 22) =
6 6 13 + 23 + 33 + ... + n3
∴ Tn =
Similarly 1 + 3 + 5 + ... + upto n terms
2
B = (12 + 22 + 32 + … + 402) + 4(12 + 22 +… + 202)  n (n + 1) 
40 × 41 × 81 4 × 20 × 21 × 41  
 = (n + 1)
2
=
B= + 2
6 6 n 2
4
40 × 41 40 × 41 × 123 9
(n + 1)2 1 2
B= (81 + 42) = S9 = ∑ = (2 + 32 + ... + 102) + 12 – 12]
6 6 4 4
n =1
Now, B − 2 A = 100λ
40 × 41 × 123 2 × 20 × 21 × 63 1 10(10 + 1)(20 + 1)  384
∴ − = 100λ = –1 = = 96
6 6 4  6  4
40 40 π
⇒ (5043 − 1323) = 100λ ⇒ × 3720 = 100λ 13. Here, α ∈ (0, ) ⇒ tan α > 0
6 6 2
40 × 620 tan 2 α
⇒ 40 × 620 = 100λ ⇒ λ = = 248 x2 + x +
100
x2 + x tan 2 α
∴ ≥ x2 + x ⋅
10. Let S10 be the sum of first ten terms of the series. Then, 2 x2 + x
we have
2 2 2 2 [using AM ≥ GM]
 3  2  1  4 tan 2 α
S10 = 1  + 2  + 3  + 42 + 4  ⇒ x +x+
2
≥ 2 tan α
 5  5  5  5
x2 + x
+ ... to 10 terms
2 2 2 2
 8  12  16  24 14. Given, a1 a 2 a3 ... a n = c
=   +   +   + 42 +   + ... to 10 terms
 5 5 5 5
⇒ a1 a 2 a3 ... (a n − 1 )(2a n ) = 2c …(i)
1 2 a1 + a 2 + a3 + ... + 2a n
= 2 (8 + 12 + 16 + 20 + 24 + ... to 10 terms)
2 2 2 2
∴ ≥ (a1 ⋅ a 2 ⋅ a3 ... 2a n )1/ n
5 n
42 [using AM ≥ GM]
= 2 (22 + 32 + 42 + 52 + ... to 10 terms)
5 ⇒ a1 + a 2 + a3 + ... + 2a n ≥ n (2c)1/ n [from Eq. (i)]
42 ⇒ Minimum value of
= 2 (22 + 32 + 42 + 52 + ... + 112)
5 a1 + a 2 + a3 + ... + 2a n = n (2c)1/ n
16
= ((12 + 22 + ... + 112) − 12) 15. Since, AM ≥ GM, then
25
(a + b) + (c + d )
16  11 ⋅ (11 + 1) (2 ⋅ 11 + 1)  ≥ (a + b)(c + d ) ⇒ M ≤ 1
=  − 1 2
25  6 
Also, (a + b) + (c + d ) > 0 [Q a , b, c, d > 0]
16 16 16 16
= (506 − 1) = × 505 ⇒ m= × 505 = 101 ∴ 0 < M ≤1
25 25 5 25
11. Given, m is the AM of l and n. 16. Let α , β be the roots of given quadratic equation. Then,
∴ l + n = 2m ... (i) 4+ 5 8+2 5
α+β= and α β =
and G1 , G2, G3 are geometric means between l and n. 5+ 2 5+ 2
l, G1 , G2, G3 , n are in GP. Let H be the harmonic mean between α and β, then
Let r be the common ratio of this GP. 2 αβ 16 + 4 5
H = = =4
1
α+β 4+ 5
 n 4
∴ G1 = lr, G2 = lr 2, G3 = lr3 , n − lr 4 ⇒ r= 
 l 17. Since, product of n positive numbers is unity.
Now, G14 + 2G24 + G34 = (lr ) + 2(lr ) + (lr )
4 2 4 3 4 ⇒ x1 ⋅ x2 ⋅ x3 ... xn = 1 ... (i)
= l × r 4 (1 + 2r 4 + r 6 ) = l4 × r 4 (r 4 + 1)2
4 x1 + x2 + ... + xn
Using AM ≥ GM, ≥ (x1 ⋅ x2 ... xn )1/ n
2 n
n  n + l
= l4 ×   = ln × 4 m = 4lm n
2 2
⇒ x1 + x2 + ... + xn ≥ n (1)1/ n [from Eq. (i)]
l  l 
Hence, sum of n positive numbers is never less than n.
Sequences and Series 73

18. Since, AM > GM 1  1 1  1 1


Again, a (n ) = 1 + +  +  +  +K+ 
(b + c − a ) + (c + a − b) 2  3 4  5 8
∴ > (b + c − a )(c + a − b)1/ 2
2  1 1 1
+ K +  n −1 + K + n − n
⇒ c > [(b + c − a )(c + a − b)]1/ 2 …(i) 2 +1 2  2
Similarly b > [(a + b − c)(b + c − a )]1/ 2 …(ii) 1  1 1  1 1 1
>1 + +  +  +  + +K+ 
and a > [(a + b − c)(c + a − b)] 1/ 2
…(iii) 2  4 4   8 8 8
On multiplying Eqs. (i), (ii) and (iii), we get  1 1 1
+ K +  n + K + n − n
abc > (a + b − c)(b + c − a )(c + a − b) 2 2  2
Hence, (a + b − c)(b + c − a )(c + a − b) − abc < 0 1 2 4 2n − 1 1
=1 + + + +K+ n − n
2 4 8 2 2
19. Since, x1 , x2 ,…, xn are positive real numbers.
1 1 1 1 1  1 n
∴ Using nth power mean inequality = 1 + + + + K + − n = 1 − n  +
2 2 2
14442444 2
3 2  2  2
2
x12 + x22 + ... + xn2  x1 + x2 + ... + xn  n times
≥ 
n  n 
 1  200
Therefore, a (200) > 1 − 200  + > 100
2 2
 2 
n  n 2  n 
2  n   n  2
n  i∑ ∑ i 
⇒ xi ≥ x ⇒ n  ∑ xi2 ≥  ∑ xi 
    Therefore, (d) is also the answer.
=1   i =1   i =1   i =1 
24. Since, first and (2n − 1)th terms are equal.
20. Let a and b are two numbers. Then,
a+b 2ab Let first term be x and (2n − 1) th term be y,
A1 = ; G1 = ab ; H 1 = whose middle term is tn.
2 a+b
x+ y
An − 1 + H n − 1 Thus, in arithmetic progression, tn = =a
An = , G n = An − 1 H n − 1 , 2
2 In geometric progression, tn = xy = b
2 An − 1 H n − 1 2xy
Hn = In harmonic progression, tn = =c
An − 1 + H n − 1 x+ y
Clearly, G1 = G2 = G3 = ... = ab . ⇒ b2 = ac and a > b > c [using AM > GM > HM]
Here, equality holds (i. e. a = b = c) only if all terms are
21. A2 is AM of A1 and H 1 and A1 > H 1 same. Hence, options (a), (b) and (d) are correct.
⇒ A1 > A2 > H 1
25. Let the two positive numbers be a and b.
A3 is AM of A2 and H 2 and A2 > H 2 a+b
⇒ A2 > A3 > H 2 ∴ x= [since, x is AM between a and b] … (i)
2
M M M a y z
∴ A1 > A2 > A3 > ... and = = [since, y, z are GM’s between a and b]
y z b
22. As above, A1 > H 2 > H 1 , A2 > H 3 > H 2 y2 z2
∴ a= and b =
∴ H 1 < H 2 < H 3 < ... z y
1 1 1 1 On substituting the values of a and b in Eq. (i), we get
23. Given, a (n ) = 1 ++ + +K+ n
2 3 4 2 −1 y2 z 2 y3 + z3 y3 + z3
2x = + ⇒ = 2x ⇒ =2
 1 1  1 1  1 1 z y yz xyz
=1 +  +  +  + K +  +  + K+ 
 2 3  4 7   8 15
26. Let the two positive numbers be ka and a , a > 0.
 1 1 
+ K +  n −1 + K + n  Then, G = ka ⋅ a = k ⋅ a and H =
2(ka )a
=
2ka
2 2 − 1 ka + a k + 1
 1 1  1 1 1  1 1 1 H 4
<1 +  +  +  + + K+  +  + + K +  Again, = [given]
 2 2  4 4 4  8 8 8 G 5
 1 1  2ka
+ K +  n − 1 + ... + n − 1 
2 2  k+1 4 2 k 4
⇒ = ⇒ =
2 4 8 2n −1 ka 5 k+1 5
= 1 + + + + K + n −1
2 4 8 2
⇒ 5 k = 2k + 2 ⇒ 2k − 5 k + 2 = 0
= 1 + 1 + 1 + 1 + ... + 1 = n
1444 424444 3 5 ± 25 − 16 5 ± 3 1
Thus, a(100) ≤ 100 (n ) times ⇒ k= = = 2, ⇒ k = 4, 1 / 4.
4 4 2
Therefore, (a) is the answer.
Hence, the required ratio is 4 : 1.
74 Sequences and Series
n ( n − 1) n −1
27. Using AM ≥ GM,
= a12n ⋅ r1 + 2 + K + ( n − 1) = a12nr 2 = [a1r 4 ]2 n
1 + x2 n 1 + x2n
≥ 1 ⋅ x2 n ⇒ ≥ xn = [Gm ]2n [from Eq. (i)]
2 2
1/ 2 n
xn 1 xn ⋅ ym 1  n 
⇒ ≤ ⇒ ≤ Gm =  ∏ AkH k 
1+x 2n
2 (1 + x ) (1 + y2m ) 4
2n
k = 1 
Hence, it is false statement.
⇒ Gm = ( A1 A2 K AnH 1H 2 K H n )1/ 2n
1
log a x + 31. Let two numbers be a and b and A1 , A2, ... , An be n
log a x
28. Since, > 1, using AM > GM arithmetic means between a and b. Then,
2
a , A1 , A2, ... , An , b are in AP with common difference
Here, equality holds only when x = a which is not b−a b−a
possible. So, log a x + log x a is greater than 2. d= ⇒ p = A1 = a + d = a +
n+1 n+1
Hence, it is a false statement. na + b
29. Here, (1 + a )(1 + b)(1 + c) ⇒ p= …(i)
n+1
= 1 + a + b + c + ab + bc + ca + abc …(i) Let H 1 , H 2, ... , H n be n harmonic means between a and
a + b + c + ab + bc + ca + abc b.
Since, ≥ (a b c )
4 4 4 1/7
7 1 1 1 1 1
∴ , , , ... , , is an AP with common
[using AM ≥ GM] a H1 H 2 Hn b
⇒ a + b + c + ab + bc + ca + abc ≥ 7(a 4b4c4 )1/7 (a − b)
difference, D = .
(n + 1) ab
⇒ 1 + a + b + c + ca + abc > 7(a 4b4c4 )1/7 …(ii)
1 1 1 1 (a − b)
From Eqs. (i) and (ii), we get ∴ = +D ⇒ = +
q a q a (n + 1) ab
(1 + a )(1 + b)(1 + c) > 7(a 4b4c4 )1/7
1 nb + a ( n + 1) ab
or {(1 + a )(1 + b)(1 + c)}7 > 77 (a 4b4c4 ) ⇒ = ⇒ q= ... (ii)
q (n + 1) ab nb + a
30. Let Gm be the geometric mean of G1 , G2, ... , Gn.
From Eq. (i), b = (n + 1) p − na.
⇒ Gm = (G1 ⋅ G2 K Gn )1/ n
Putting it in Eq. (ii), we get
= [(a1 ) ⋅ (a1 ⋅ a1r )1/ 2 ⋅ (a1 ⋅ a1r ⋅ a1r 2)1/3
n − 1 1/ n 1/ n
q { n (n + 1) p − n 2a + a } = (n + 1) a {(n + 1) p − na }
K (a1 ⋅ a1r ⋅ a1r K a1r
2
) ]
⇒ n (n + 1) a 2 − {(n + 1)2 p + (n 2 − 1)q}a
where, r is the common ratio of GP a1 , a 2, K , a n.
+ n (n + 1) pq = 0
( n − 1 )n
⇒ na − {(n + 1) p + (n − 1)q} a + npq = 0
2
= [(a1 ⋅ a1. K n times ) (r1/ 2 ⋅ r3/3 ⋅ r 6/ 4 K r 2n
)]1/ n
1 3 n −1 Since, a is real, therefore
+ 1 + + L+
= [a1n ⋅ r 2 2 2 ]1/ n {(n + 1) p + (n − 1)q}2 − 4n 2pq > 0
1/ n
 1  ( n − 1)n   ⇒ (n + 1) p + (n − 1)2q2 + 2 (n 2 − 1) pq − 4n 2pq > 0
2 2
 n −1 
= a1 r 2  2   = a1 r 4  …(i) ⇒ (n + 1)2 p2 + (n − 1)2q2 − 2 (n 2 + 1) pq > 0
    2
  2 (n 2 + 1)  n + 1
⇒ q2 − pq +   p >0
2
a1 + a 2 + ... + a n a1 (1 − r n ) (n − 1)2  n − 1
Now, An = =
n n (1 − r )   n + 1 
2 2
 n + 1) 2
n ⇒ q2 − 1 +    pq +   p >0
and Hn =   n − 1   n −1
1 1 1 
 + +K+ 
 a1 a 2 an   n + 1 
2
⇒ (q − p) q −   p > 0
n a1n (1 − r ) r n − 1   n − 1 
= = 
1  1 1  1 − rn
1 + + K + n − 1 
2
 n + 1
a1  r r  ⇒ q < p or q >   p
 n − 1
a1 (1 − r n ) a1n (1 − r )r n − 1
∴ An ⋅ H n = × = a12r n − 1  n + 1 2 
n (1 − r ) (1 − r n ) Q   p > p
n n  n − 1  
⇒ ∏ AkH k = ∏ (a12r n − 1 )  n + 1
2
k =1 k =1
Hence, q cannot lie between p and   p.
= (a12 ⋅ a12 ⋅ a12K n times ) × r 0 ⋅ r1 ⋅ r 2K r n − 1  n − 1
Sequences and Series 75

32. Since a , b, c > 0 ⇒ 2n 2 + 1 ≥ 2n ⇒ n ≤ 6


(a + b + c) and, also C >0⇒n >2
⇒ > (abc)1/3 …(i)
3 ∴The possible values of n are 3, 4, 5, 6
(2 × 9) − 6
[using AM ≥ GM] So, at n = 3, C = = 12
1 1 1 8 − 6 −1
+ + 1/3 32 − 8 24 8
a b c ≥  1 ⋅ 1 ⋅ 1 at, n = 4, C = = = ∉N
Also,   …(ii)
16 − 8 − 1 9 3
3  a b c
50 − 10 40
[using AM ≥ GM] at, n = 5, C = = ∉N
32 − 10 − 1 21
On multiplying Eqs. (i) and (ii), we get
72 − 12 60
 1 1 1 and at, n = 6, C = = ∉N
(a + b + c)  + +  64 − 12 − 1 51
 a b c 1
≥ (abc)1/3 ∴ The required value of C = 12 for n = 3.
9 (abc)1/3
 1 1 1 35. The 4th AM out of m AMs inserted between 3 and 243 is
∴ (a + b + c)  + +  ≥ 9
 a b c 243 − 3
A4 = 3 + 4 …(i)
33. For real numbers y1 , y2, y3 , the quantities 3 , 3
and y1 y2 m+1
3y3 are positive real numbers, so according to the and the 2nd GM out of three GMs inserted between 3
AM-GM inequality, we have and 243 is
2
3y1 + 3y2 + 3y3
1
 1 
≥ (3y1 . 3y2 . 3y3 )3   243 3 + 1 
G2 = 3     = 3(81) = 27
1/ 2
3  3 
1  
⇒ 3y1 + 3y2 + 3y3 ≥ 3(3y1 . 3y2 . 3y3 )3  240 
On applying logarithm with base ‘3’, we get Q A4 = G 2 ⇒ 3 + 4   = 27
 m + 1
 1 
log3 (3y1 + 3y2 + 3y3 ) ≥ 1 + ( y1 + y2 + y3 )  240 
 3  ⇒ 4   = 24 ⇒ m + 1 = 40 ⇒ m = 39
 m + 1
=1 + 3
=4 {Q y1 + y2 + y3 = 9} Hence, answer is 39.00
∴ m =4 36. Plan
Now, for positive real numbers x1 , x2 and x3 , according to (i) If a, b, c are in GP, then they can be taken as a, ar, ar 2
AM-GM inequality, we have where r, (r ≠ 0) is the common ratio.
x1 + x2 + x3
1 x + x2 + K + xn
(ii) Arithmetic mean of x1, x2, K , xn = 1
≥ (x1x2x3 )3 n
3
On applying logarithm with base ‘3’, we get Let a , b, c be a , ar , ar 2, where r ∈ N
 x + x2 + x3  1 a+ b+ c
log3  1  ≥ (log3 x1 + log3 x2 + log3 x3 ) Also, =b+2
 3  3 3
1 ⇒ a + ar + ar 2 = 3 (ar ) + 6
⇒ 1 ≥ (log3 x1 + log3 x2 + log3 x3 )
3 ⇒ ar 2 − 2ar + a = 6
{Q x1 + x2 + x3 = 9} 6
⇒ (r − 1)2 =
∴ M =3 a
Now, log 2(m3 ) + log3 (M 2) = 3 log 2(4) + 2 log3 (3 ) Since, 6 /a must be perfect square and a ∈ N .
=6 + 2 =8 So, a can be 6 only.
34. Given arithmetic progression of positive integers terms ⇒ r −1 = ± 1 ⇒ r =2
a1 , a 2, a3 , ..... having common difference ‘2’ and
a + a − 14 36 + 6 − 14
2
geometric progression of positive integers terms and = =4
b1 , b2, b3 , .... having common ratio ‘2’ with a1 = b1 = c, a+1 7
such that 37. Using AM ≥ GM,
2 (a1 + a 2 + a3 + ... + a n ) = b1 + b2 + b3 + .... + bn
a −5 + a −4 + a −3 + a −3 + a −3 + 1 + a 8 + a10
n  2 n − 1
⇒ 2 × [2C + (n − 1)2] = C   8
2  2 −1 1

⇒ 2nC + 2n − 2n = 2 . C − C
2 n ≥ (a − 5 ⋅ a − 4 ⋅ a − 3 ⋅ a − 3 ⋅ a − 3 ⋅ 1 ⋅ a 8 ⋅ a10 ) 8
⇒ C [2n − 2n − 1] = 2n 2 − 2n ⇒ a −5 + a −4 + 3a −3 + 1 + a 8 + a10 ≥ 8 ⋅ 1
Q C ∈ N ⇒ 2n 2 − 2n ≥ 2n − 2n − 1 Hence, minimum value is 8.
4
Permutations
and Combinations
7. In a collage of 300 students, every student reads
Topic 1 General Arrangement 5 newspapers and every newspaper is read by 60
students. The number of newspapers is (1998, 2M)
Objective Questions I (Only one correct option) (a) atleast 30
1. The number of four-digit numbers strictly greater than (b) atmost 20
4321 that can be formed using the digits 0, 1, 2, 3, 4, 5 (c) exactly 25
(repetition of digits is allowed) is (2019 Main, 8 April II) (d) None of the above
(a) 306 (b) 310 8. A five digits number divisible by 3 is to be formed using
(c) 360 (d) 288 the numbers 0, 1 , 2, 3 , 4 and 5, without repetition. The
2. How many 3 × 3 matrices M with entries from {0, 1, 2} total number of ways this can be done, is (1989, 2M)
are there, for which the sum of the diagonal entries of (a) 216 (b) 240
M T M is 5 ? (2017 Adv.) (c) 600 (d) 3125
(a) 198 (b) 162 (c) 126 (d) 135 9. Eight chairs are numbered 1 to 8. Two women and
3. The number of integers greater than 6000 that can be three men wish to occupy one chair each.
formed using the digits 3, 5, 6, 7 and 8 without First the women choose the chairs from amongst the
repetition is (2015 Main) chairs marked 1 to 4 and then the men select the chairs
(a) 216 (b) 192 from amongst the remaining. The number of possible
(c) 120 (d) 72 arrangements is
(a) 6C3 × 4 C2 (b) 4 P2 × 4 P3 (1982, 2M)
4. The number of seven-digit integers, with sum of the
digits equal to 10 and formed by using the digits 1, 2 and (c) 4 C2 + 4 P3 (d) None of these
3 only, is (2009) 10. The different letters of an alphabet are given. Words
(a) 55 (b) 66 (c) 77 (d) 88 with five letters are formed from these given letters.
Then, the number of words which have at least one
5. How many different nine-digit numbers can be formed
letter repeated, is (1980, 2M)
from the number 22 33 55 888 by rearranging its digits
so that the odd digits occupy even positions? (2000, 2M) (a) 69760 (b) 30240
(c) 99748 (d) None
(a) 16 (b) 36
(c) 60 (d) 180
Analytical & Descriptive Question
6. An n-digit number is a positive number with exactly n
digits. Nine hundred distinct n-digit numbers are to be 11. Eighteen guests have to be seated half on each side of a
formed using only the three digits 2,5 and 7. The smallest long table. Four particular guests desire to sit on one
value of n for which this is possible, is (1998, 2M) particular side and three other on the other side.
(a) 6 (b) 7 (c) 8 (d) 9 Determine the number of ways in which the sitting
arrangements can be made. (1991, 4M)
Permutations and Combinations 77

Match the Columns


Match the conditions/expressions in Column I with statement in Column II.
12. Consider all possible permutations of the letters of the word ENDEANOEL. (2008, 6M)

Column I Column II
A. The number of permutations containing the word p. 5!
ENDEA, is
B. The number of permutations in which the letter E q. 2 × 5!
occurs in the first and the last positions, is
C. The number of permutations in which none of the r. 7 × 5!
letters D, L, N occurs in the last five positions, is
D. The number of permutations in which the letters A, E, s. 21 × 5!
O occur only in odd positions, is

Topic 2 Properties of Combinational and General Selections


Objective Questions I (Only one correct option) 6. If n C 4 , n C 5 and n C 6 are in AP, then n can be
1. There are 3 sections in a question paper and each (2019 Main, 12 Jan II)
section contains 5 questions. A candidate has to answer (a) 9 (b) 11 (c) 14 (d) 12
25
a total of 5 questions, choosing at least one question
from each section. Then the number of ways, in which 7. If ∑{
r = 0
50
Cr ⋅ 50 − r
C 25 − r } = K (50C 25 ),
the candidate can choose the questions, is
[2020 Main, 5 Sep II] then, K is equal to (2019 Main, 10 Jan II)

(a) 3000 (b) 1500 (a) 224 (b) 225 − 1 (c) 225 (d) (25)2
3
(c) 2255 (d) 2250 20 20
Ci − 1  k
2. The number of ways of choosing 10 objects out of 31
8. If ∑ 
i=1
20 20
 = , then k equals
Ci + Ci − 1  21 (2019 Main, 10 Jan I)
objects of which 10 are identical and the remaining 21
are distinct, is (2019 Main, 12 April I) (a) 100 (b) 400 (c) 200 (d) 50
(a) 2 20
−1 (b) 221
(c) 2 20
(d) 2 20
+1 9. A man X has 7 friends, 4 of them are ladies and 3 are
men. His wife Y also has 7 friends, 3 of them are ladies
3. Suppose that 20 pillars of the same height have been
and 4 are men. Assume X and Y have no common
erected along the boundary of a circular stadium. If the
friends. Then, the total number of ways in which X and
top of each pillar has been connected by beams with the
Y together can throw a party inviting
top of all its non-adjacent pillars, then the total number
3 ladies and 3 men, so that 3 friends of each of X and Y
of beams is (2019 Main, 10 April II)
are in this party, is (2017 Main)
(a) 180 (b) 210 (c) 170 (d) 190
(a) 485 (b) 468 (c) 469 (d) 484
4. Some identical balls are arranged in rows to form an
equilateral triangle. The first row consists of one ball,
10. Let S = {1, 2, 3, …… , 9}. For k = 1, 2 , …… 5, let N k be the
the second row consists of two balls and so on. If 99 more number of subsets of S, each containing five elements
identical balls are added to the total number of balls out of which exactly k are odd. Then
used in forming the equilateral triangle, then all these N1 + N 2 + N 3 + N 4 + N 5 = (2017 Adv.)
balls can be arranged in a square whose each side (a) 210 (b) 252 (c) 126 (d) 125
contains exactly 2 balls less than the number of balls 11. A debate club consists of 6 girls and 4 boys. A team of
each side of the triangle contains. Then, the number of 4 members is to be selected from this club including the
balls used to form the equilateral triangle is selection of a captain (from among these 4 members) for
(2019 Main, 9 April II) the team. If the team has to include atmost one boy, the
(a) 262 (b) 190 (c) 225 (d) 157 number of ways of selecting the team is (2016 Adv.)
5. There are m men and two women participating in a (a) 380 (b) 320 (c) 260 (d) 95
chess tournament. Each participant plays two games
with every other participant. If the number of games
12. Let Tn be the number of all possible triangles formed by
played by the men between themselves exceeds the joining vertices of an n-sided regular polygon. If
number of games played between the men and the Tn + 1 − Tn = 10, then the value of n is (2013 Main)
women by 84, then the value of m is (2019 Main, 12 Jan II) (a) 7 (b) 5
(a) 12 (b) 11 (c) 9 (d) 7 (c) 10 (d) 8
78 Permutations and Combinations

13. If r , s, t are prime numbers and p, q are the positive 17. Let n ≥ 2 be an integer. Take n distinct points on a circle
integers such that LCM of p, q is r 2s4 t 2 ,then the number and join each pair of points by a line segment. Colour the
of ordered pairs ( p, q) is (2006, 3M) line segment joining every pair of adjacent points by blue
(a) 252 (b) 254 and the rest by red. If the number of red and blue line
segments are equal, then the value of n is (2014 Adv.)
(c) 225 (d) 224
5
Fill in the Blanks
14. The value of the expression 47 C 4 + ∑
j =1
52− j
C 3 is
18. Let A be a set of n distinct elements. Then, the total
(1980, 2M)
47
(a) C5 number of distinct functions from A to A is…and out of
(b) 52C5 these… are onto functions. (1985, 2M)
(c) 52C4 19. In a certain test, a i students gave wrong answers to at
(d) None of these least i questions, where i = 1, 2, K , k. No student gave
more that k wrong answers. The total number of wrong
Match the Columns answers given is … . (1982, 2M)

15. In a high school, a committee has to be formed from a True/False


group of 6 boys M1 , M 2, M 3, M 4 , M 5, M 6 and 5 girls G1 ,
G2, G 3, G 4 , G 5. 20. The product of any r consecutive natural numbers is
always divisible by r !. (1985, 1M)
(i) Let α1 be the total number of ways in which the
committee can be formed such that the committee Analytical & Descriptive Questions
has 5 members, having exactly 3 boys and 2 girls.
21. A committee of 12 is to be formed from 9 women and 8
(ii) Let α 2 be the total number of ways in which the men. In how many ways this can be done if at least five
committee can be formed such that the committee women have to be included in a committee ? In how
has at least 2 members, and having an equal many of these committees
number of boys and girls. (i) the women are in majority?
(iii) Let α 3 be the total number of ways in which the (ii) the men are in majority? (1994, 4M)
committee can be formed such that the committee
22. A student is allowed to select atmost n books from n
has 5 members, at least 2 of them being girls.
collection of (2n + 1) books. If the total number of ways
(iv) Let α 4 be the total number of ways in which the in which he can select at least one books is 63, find the
committee can be formed such that the committee value of n. (1987, 3M)
has 4 members, having at least 2 girls such that both
M1 and G1 are NOT in the committee together.
23. A box contains two white balls, three black balls and
(2018 Adv.)
four red balls. In how many ways can three balls be
drawn from the box, if at least one black ball is to be
List-I List-II included in the draw ? (1986, 2 12 M)
P. The value of α1 is 1. 136 24. 7 relatives of a man comprises 4 ladies and 3
Q. The value of α 2 is 2. 189 gentlemen, his wife has also 7 relatives ; 3 of them are
ladies and 4 gentlemen. In how many ways can they
R. The value of α 3 is 3. 192 invite a dinner party of 3 ladies and 3 gentlemen so
that there are 3 of man’s relative and 3 of the wife's
S. The value of α 4 is 4. 200
relatives? (1985, 5M)
5. 381 25. m men and n women are to be seated in a row so that no
6. 461 two women sit together. If m > n, then show that the
number of ways in which they can be seated, is
The correct option is m ! (m + 1) !
.
(a) P → 4; Q → 6; R → 2; S → 1 (m − n + 1) ! (1983, 2M)
(b) P → 1; Q → 4; R → 2; S → 3
26. mn squares of equal size are arranged to form a
(c) P → 4; Q → 6; R → 5; S → 2 rectangle of dimension m by n where m and n are
(d) P → 4; Q → 2; R → 3; S → 1 natural numbers. Two squares will be called
‘neighbours’ if they have exactly one common side. A
Integer & Numerical Answer Type Questions natural number is written in each square such that the
number in written any square is the arithmetic mean of
16. Five persons A , B, C , D and E are seated in a circular the numbers written in its neighbouring squares. Show
arrangement. If each of them is given a hat of one of the that this is possible only if all the numbers used are
three colours red, blue and green, then the number of equal. (1982, 5M)
ways of distributing the hats such that the persons
27. If n C r −1 = 36, n C r = 84 and n C r +1 = 126, then find the
seated in adjacent seats get different coloured hats is
……… (2019 Adv.) values of n and r. (1979, 3M)
Permutations and Combinations 79

Topic 3 Multinomial, Repeated Arrangement and Selection


Objective Question I (Only one correct option) Integer & Numerical Answer Type Questions
1. The number of 6 digits numbers that can be formed 8. An engineer is required to visit a factory for exactly four
using the digits 0, 1, 2,5, 7 and 9 which are divisible by days during the first 15 days of every month and it is
11 and no digit is repeated, is (2019 Main, 10 April I) mandatory that no two visits take place on consecutive
(a) 60 (b) 72 days. Then the number of all possible ways in which
(c) 48 (d) 36 such visits to the factory can be made by the engineer
during 1-15 June 2021 is ……… (2020 Adv.)
2. A committee of 11 members is to be formed from 8 males
and 5 females. If m is the number of ways the committee 9. If the letters of the word ‘MOTHER’ be permuted and
is formed with at least 6 males and n is the number of all the words so formed (with or without meaning) be
ways the committee is formed with atleast 3 females, listed as in dictionary, then the position of the word
‘MOTHER’ is………… (2020 Main, 2 Sep I)
then (2019 Main, 9 April I)
(a) m = n = 68 (b) m + n = 68 10. The number of 5 digit numbers which are divisible by 4,
(c) m = n = 78 (d) n = m − 8 with digits from the set {1, 2, 3, 4, 5} and the repetition of
digits is allowed, is ..................... . (2018 Adv.)
3. Consider three boxes, each containing 10 balls labelled
1, 2, …, 10. Suppose one ball is randomly drawn from 11. Words of length 10 are formed using the letters A, B, C,
each of the boxes. Denote by ni , the label of the ball D, E, F, G, H, I, J. Let x be the number of such words
drawn from the ith box, (i = 1, 2, 3). Then, the number of where no letter is repeated; and let y be the number of
ways in which the balls can be chosen such that such words where exactly one letter is repeated twice
y
n1 < n2 < n3 is (2019 Main, 12 Jan I) and no other letter is repeated. Then, =
9x (2017 Adv.)
(a) 82 (b) 120
(c) 240 (d) 164 12. Let n be the number of ways in which 5 boys and 5 girls
can stand in a queue in such a way that all the girls
4. The number of natural numbers less than 7,000 which
stand consecutively in the queue. Let m be the number
can be formed by using the digits 0, 1, 3, 7, 9 (repitition
of ways in which 5 boys and 5 girls can stand in a queue
of digits allowed) is equal to (2019 Main, 9 Jan II)
in such a way that exactly four girls stand consecutively
(a) 374 (b) 375 m
in the queue. Then, the value of is
(c) 372 (d) 250 n (2015 Adv.)
5. Consider a class of 5 girls and 7 boys. The number of 13. Let n1 < n2 < n3 < n4 < n5 be positive integers such that
different teams consisting of 2 girls and 3 boys that can
n1 + n2 + n3 + n4 + n5 = 20. The number of such distinct
be formed from this class, if there are two specific boys
arrangements (n1 , n2 , n3 , n4 , n5 ) is (2014 Adv.)
A and B, who refuse to be the members of the same
team, is (2019 Main, 9 Jan I) Fill in the Blanks
(a) 350 (b) 500 k(k + 1)
(c) 200 (d) 300 14. Let n and k be positive integers such that n ≥ .
2
6. If all the words (with or without meaning) having five The number of solutions (x1 , x2 ,... , xk ),
letters, formed using the letters of the word SMALL and x1 ≥ 1, x2 ≥ 2, ... , xk ≥ k for all integers satisfying
arranged as in a dictionary, then the position of the x1 + x2 + ... + xk = n is … (1996, 2M)
word SMALL is (2016 Main) 15. Total number of ways in which six ‘+’ and four ‘–’ signs
(a) 46th (b) 59th can be arranged in a line such that no two ‘–’signs occur
(c) 52nd (d) 58th together is… . (1988, 2M)
7. The letters of the word COCHIN are permuted and all
the permutations are arranged in an alphabetical order Analytical & Descriptive Question
as in an English dictionary. The number of words that 16. Five balls of different colours are to be placed in three
appear before the word COCHIN, is (2007, 3M) boxes of different sizes. Each box can hold all five. In
(a) 360 (b) 192 how many different ways can we place the balls so that
(c) 96 (d) 48 no box remains empty? (1981, 4M)
80 Permutations and Combinations

Topic 4 Distribution of Object into Group


Objective Questions I (Only one correct option) 4. The total number of ways in which 5 balls of different
colours can be distributed among 3 persons so that each
1. A group of students comprises of 5 boys and n girls. If
person gets at least one ball, is (2012)
the number of ways, in which a team of 3 students can
randomly be selected from this group such that there is (a) 75 (b) 150 (c) 210 (d) 243
at least one boy and at least one girl in each team, is 5. The number of arrangements of the letters of the word
1750, then n is equal to (2019 Main, 12 April II) BANANA in which the two N’s do not appear adjacently,
(a) 28 (b) 27 is (2002, 1M)
(c) 25 (d) 24 (a) 40 (b) 60 (c) 80 (d) 100
2. Let S be the set of all triangles in the xy-plane, each Analytical & Descriptive Questions
having one vertex at the origin and the other two
n2 !
vertices lie on coordinate axes with integral 6. Using permutation or otherwise, prove that is an
coordinates. If each triangle in S has area 50 sq. units, (n !)n
then the number of elements in the set S is integer, where n is a positive integer. (2004, 2M)
(2019 Main, 9 Jan II)
7. In how many ways can a pack of 52 cards be
(a) 36 (b) 32
(i) divided equally among four players in order
(c) 18 (d) 9
(ii) divided into four groups of 13 cards each
3.. From 6 different novels and 3 different dictionaries, 4 (iii) divided in 4 sets, three of them having 17 cards each
novels and 1 dictionary are to be selected and arranged and the fourth just one card? (1979, 3M)
in a row on a shelf, so that the dictionary is always in
the middle. The number of such arrangements is Integer & Numerical Answer Type Question
(2018 Main)
(a) atleast 1000
8. In a hotel, four rooms are available. Six persons are to
be accommodated in these four rooms in such a way that
(b) less than 500
each of these rooms contains at least one person and at
(c) atleast 500 but less than 750
most two persons. Then the number of all possible ways
(d) atleast 750 but less than 1000
in which this can be done is ……… (2020 Adv.)

Topic 5 Dearrangement and Number of Divisors


Objective Question I (Only one correct option) Fill in the Blank
1. Number of divisors of the form (4n + 2), n ≥ 0 of the 2. There are four balls of different colours and four boxes of
integer 240 is (1998, 2M) colours, same as those of the balls. The number of ways in
(a) 4 (b) 8 which the balls, one each in a box, could be placed such
(c) 10 (d) 3
that a ball does not go to a box of its own colour is.... .
(1992, 2M)

Answers
Topic 1 Topic 3
1. (b) 2. (a) 3. (b) 4. (c) 1. (a) 2. (c) 3. (b) 4. (a)
5. (c) 6. (b) 7. (c) 8. (a) 5. (d) 6. (d) 7. (c) 8. (495)
9. (d) 10. (a) 11. 9
P4 × 9 P3 (11 )! 9. (309) 10. (625) 11. (5) 12. (5)
1
12. ( A → p; B → s; C → q ; D → q ) 13. (7) 14. (2n − k + k − 2 )
2
15. (35 ways)
2
Topic 2 16. (300)
1. (d) 2. (c) 3. (c) 4. (b)
Topic 4
5. (a) 6. (c) 7. (c) 8. (a)
1. (c) 2. (a) 3. (a) 4. (b)
9. (a) 10. (c) 11. (a) 12. (b)
(52 )! (52 )! (52 )!
13. (c) 14. (c) 15. (c) 16. (30) 5. (a) 7. (i) 4
(ii) 4
(iii)
n (13 !) 4 ! (13 !) 3 ! (17 ) 3
17. (5) 18. n , ∑ ( −1 )
n n −r n
Cr (r ) n
19. 2 − 1
n
8. (1080)
r =1

20. (True) 21. 6062, (i) 2702 (ii) 1008 22. n = 3 Topic 5
23. (64) 24. (485) 27. (n = 9 and r = 3 ) 1. (a) 2. (9)
Hints & Solutions
Topic 1 General Arrangement Case II Four 1’s, three 2’s
7!
1. Following are the cases in which the 4-digit numbers Number of numbers = = 35
strictly greater than 4321 can be formed using digits 0, 4!3!
1, 2, 3, 4, 5 (repetition of digits is allowed) ∴ Total number of numbers = 42 + 35 = 77
Case-I 5. X  X  X  X  X . The four digits 3, 3, 5, 5 can be
4!
4 3 2 arranged at () places in = 6 ways.
2 !2 !
2/3/4/5 4 ways 4 numbers The five digits 2, 2, 8, 8, 8 can be arranged at (X ) places
5!
Case-II in ways = 10 ways.
2 !3 !
4 3 Total number of arrangements = 6 × 10 = 60
[since, events A and B are independent, therefore
3/4/5 0/1/2/3/4/5 3×6=18 numbers A ∩ B = A × B]
3 ways 6 ways
6. Distinct n-digit numbers which can be formed using
Case-III digits 2, 5 and 7 are 3n . We have to find n, so that
4 3n ≥ 900
n− 2
⇒ 3 ≥ 100
4/5 0/1/2/3/4/5 2×6×6=72 numbers ⇒ n −2 ≥5
2 ways 6 ways
⇒ n ≥ 7, so the least value of n is 7.
Case-IV
7. Let n be the number of newspapers which are read by
5 the students.
6×6×6=216 numbers
Then, 60n = (300) × 5
0/1/2/3/4/5
6 ways ⇒ n = 25
So, required total numbers = 4 + 18 + 72 + 216 = 310 8. Since, a five-digit number is formed using the digits
{0, 1, 2, 3, 4 and 5} divisible by 3 i.e. only possible when
sum of the digits is multiple of three.
2. Sum of diagonal entries of M M is ∑ a . T 2
i
Case I Using digits 0, 1, 2, 4, 5
9
Number of ways = 4 × 4 × 3 × 2 × 1 = 96
∑a
i =1
2
i =5
Case II Using digits 1, 2, 3, 4, 5
Possibilities Number of ways = 5 × 4 × 3 × 2 × 1 = 120
I. 2, 1, 0, 0, 0, 0, 0, 0, 0, which gives
9!
matrices ∴ Total numbers formed = 120 + 96 = 216
7! 9. Since, the first 2 women select the chairs amongst 1 to 4
9! in 4 P2 ways. Now, from the remaining 6 chairs, three
II. 1, 1, 1, 1, 1, 0, 0, 0, 0, which gives matrices
4! × 5! men could be arranged in 6 P3.
Total matrices = 9 × 8 + 9 × 7 × 2 = 198 ∴ Total number of arrangements = 4 P2 × 6P3.
3. The integer greater than 6000 may be of 4 digits or 10. Total number of five letters words formed from ten
5 digits. So, here two cases arise. different letters = 10 × 10 × 10 × 10 × 10 = 105
Case I When number is of 4 digits. Number of five letters words having no repetition
Four-digit number can start from 6, 7 or 8. = 10 × 9 × 8 × 7 × 6 = 30240
Thus, total number of 4-digit numbers, which are ∴ Number of words which have at least one letter
greater than 6000 = 3 × 4 × 3 × 2 = 72 repeated = 105 − 30240 = 69760
Case II When number is of 5 digits. 11. Let the two sides be A and B. Assume that four
Total number of five-digit numbers which are greater particular guests wish to sit on side A. Four guests who
than 6000 = 5 ! = 120 wish to sit on side A can be accommodated on nine
∴ Total number of integers = 72 + 120 = 192 chairs in 9 P4 ways and three guests who wish to sit on
side B can be accommodated in 9 P3 ways. Now, the
4. There are two possible cases remaining guests are left who can sit on 11 chairs on
Case I Five 1’s, one 2’s, one 3’s both the sides of the table in (11!) ways. Hence, the total
7! number of ways in which 18 persons can be seated
Number of numbers = = 42
5! = 9P4 × 9P3 × (11)!.
82 Permutations and Combinations

12. A. If ENDEA is fixed word, then assume this as a 3. It is given that, there are 20 pillars of the same height
single letter. Total number of letters = 5 have been erected along the boundary of a circular
Total number of arrangements = 5 !. stadium.
B. If E is at first and last places, then total number of Now, the top of each pillar has been connected by beams
permutations = 7 !/ 2 ! = 21 × 5 ! with the top of all its non-adjacent pillars, then total
C. If D, L, N are not in last five positions number of beams = number of diagonals of 20-sided
← D, L, N, N → ← E, E, E, A, O → polygon.
4! 5! Q 20C 2 is selection of any two vertices of 20-sided polygon
Total number of permutations = × = 2 × 5 !
2! 3! which included the sides as well.
D. Total number of odd positions = 5 So, required number of total beams = 20C 2 − 20
5! [Q the number of diagonals in a n-sided closed
Permutations of AEEEO are . polygon = nC 2 − n]
3! 20 × 19
Total number of even positions = 4 = − 20 = 190 − 20 = 170
2
4!
∴ Number of permutations of N, N, D, L = 4. Let there are n balls used to form the sides of equilateral
2!
triangle.
5! 4!
⇒ Total number of permutations = × = 2 × 5 ! According to the question, we have
3! 2! n ( n + 1)
+ 99 = ( n − 2 )2
2
Topic 2 Properties of Combinational ⇒ n 2 + n + 198 = 2 [ n 2 − 4 n + 4 ]
and General Selections ⇒ n 2 − 9n − 190 = 0
1. As each section has 5 questions, so number of ways to ⇒ n 2 − 19n + 10n − 190 = 0
select 5 questions are
⇒ ( n − 19 )( n + 10 ) = 0
S1 S 2 S 3
⇒ n = 19, − 10
1 1 3 ⇒ n = 19 [Qnumber of balls n > 0]
1 3 1
Now, number of balls used to form an equilateral
3 1 1 n (n + 1)
triangle is
1 2 2 2
19 × 20
and 2 1 2 = = 190.
2 2 1 2
5. Since, there are m-men and 2-women and each
∴Total number of selection of 5 questions participant plays two games with every other
= 3 × ( 5C1 × 5C1 × 5C 3 ) + 3 × ( 5C1 × 5C 2 × 5C 2 ) participant.
= 3(5 × 5 × 10) + 3(5 × 10 × 10)
∴ Number of games played by the men between
= 750 + 1500 = 2250. themselves = 2 × mC 2
2. Given that, out of 31 objects 10 are identical and and the number of games played between the men and
remaining 21 are distinct, so in following ways, we can the women = 2 × mC1 × 2C1
choose 10 objects. Now, according to the question,
0 identical + 10 distincts, number of ways = 1 × 21
C10 2 mC 2 = 2 mC1 2C1 + 84
1 identical + 9 distincts, number of ways = 1 × 21
C9 m!
⇒ = m × 2 + 42
2 identicals + 8 distincts, number of ways = 1 × 21
C8 2 !(m − 2)!
. . . . . . ⇒ m(m − 1) = 4m + 84
⇒ m2 − m = 4m + 84
. . . . . .
⇒ m2 − 5m − 84 = 0
So, total number of ways in which we can choose 10 ⇒ m2 − 12m + 7m − 84 = 0
objects is ⇒ m(m − 12) + 7 (m − 12) = 0
21
C10 + 21
C9 + 21
C8 + K + 21
C 0 = x (let) … (i) ⇒ m = 12 [Q m > 0]
n n n
⇒ C11 +
21 21
C12 + 21
C13 + K + 21
C 21 = x … (ii) 6. If C 4 , C 5 and C 6 are in AP, then
[Q nC r = nC n − r ] 2 ⋅n C 5 = n C 4 + n C 6
On adding both Eqs. (i) and (ii), we get [If a , b, c are in AP , then 2b = a + c]
2x = 21C 0 + 21C1 + 21C 2 + K + 21C10 n! n! n!
⇒ 2 = +
+ 21C11 + 21C12 + K + 21
C 21 5 !(n − 5)! 4 !(n − 4)! 6 !(n − 6)!
 n n! 
⇒ 2x = 2 ⇒ x=2 Q C r = r !(n − r )! 
21 20
Permutations and Combinations 83

2 9. Given, X has 7 friends, 4 of them are ladies and 3 are



5 ⋅ 4 !(n − 5) (n − 6)! men while Y has 7 friends, 3 of them are ladies and 4 are
men.
1 1
= + ∴ Total number of required ways
4 !(n − 4) (n − 5) (n − 6)! 6 ⋅ 5 ⋅ 4 ! (n − 6)!
2 1 1 = 3C 3 × 4C 0 × 4C 0 × 3C 3 + 3C 2 × 4C1 × 4C1 × 3C 2
⇒ = +
5(n − 5) (n − 4) (n − 5) 30 + 3C1 × 4C 2 × 4C 2 × 3C1 + 3C 0 × 4C 3 × 4C 3 × 3C 0
2 30 + (n − 4) (n − 5) = 1 + 144 + 324 + 16 = 485
⇒ =
5(n − 5) 30 (n − 4) (n − 5) 10. N i = 5C k × 4C 5 − k
⇒ 12 (n − 4) = 30 + n − 9n + 20 2
N1 = 5 × 1
⇒ n 2 − 21n + 98 = 0
N 2 = 10 × 4
⇒ n − 14n − 7n + 98 = 0
2

N 3 = 10 × 6
⇒ n (n − 14) − 7(n − 14) = 0
⇒ (n − 7) (n − 14) = 0 N4 = 5 × 4
⇒ n = 7 or 14 N5 = 1
25 N 1 + N 2 + N 3 + N 4 + N 5 = 126
7. Given, Σ { 50C r .50− r C 25− r } = K 50C 25
r=0
11. We have, 6 girls and 4 boys. To select 4 members
25  50 ! (50 − r )!  (atmost one boy)
⇒ Σ  ×  = K C 25
50
r = 0  r !(50 − r )! (25 − r )! 25 ! i.e. (1 boy and 3 girls) or (4 girls) = 6 C 3 ⋅4 C1 + 6C 4 …(i)
25  25 ! 
50 ! Now, selection of captain from 4 members = 4 C1 …(ii)
⇒ Σ  ×  = K C 25
50
r = 0  25 ! 25 ! r !(25 − r )! ∴ Number of ways to select 4 members (including the
[on multiplying 25 ! in selection of a captain, from these 4 members)
numerator and denominator.] = ( 6C 3 ⋅4 C1 + 6C 4 ) 4C1
 50 = (20 × 4 + 15) × 4 = 380
25 50 ! 
⇒ 50
C 25 Σ 25
Cr = K 50
C 25 Q C 25 = 25 ! 25 !  n +1
r = 0
  12. Given, Tn = nC 3 ⇒ Tn + 1 = C3
25 n +1
⇒ K= Σ Cr = 2
25 25 ∴ Tn +1 − Tn = C 3 − C 3 = 10
n
[given]
r = 0
n +1
[Q nC 0 + nC1 + n C 2 + ....+ nC n = 2n ] ⇒ C 2 + C 3 − C 3 = 10
n n n
[Q C r + C r
n n
+1 = Cr +1 ]

⇒ K =2 25 ⇒ n
C 2 = 10 ⇒ n = 5

8. Given, 13. Since, r , s, t are prime numbers.


3 ∴ Selection of p and q are as under
20  Ci − 1 20
 k

i=1
 20  =
 Ci + Ci − 1 
20
21
p q Number of ways
3
r0 r2 1 way
20  20C i − 1  k r1 r2

⇒  21  = (Q nC r + nC r − 1 = n +1
Cr ) 1 way
i=1  C i  21 r2 r 0 , r1 , r 2 3 ways
3
  ∴ Total number of ways to select, r = 5
20  20C i − 1  k  n n 
⇒ ∑ 
21
 =
21
Q C r =
 r
n−1
Cr − 1 

Selection of s as under
i=1  20
Ci − 1  s0 s4 1 way
 i 
3 s1 s4 1 way
 i
20
k
⇒ ∑   =
 21  21
s2 s4 1 way
i=1 s3 s4 1 way
1 20
k s4 5 ways

(21)3 ∑i i=1
3
=
21 ∴ Total number of ways to select s = 9
2 Similarly, the number of ways to select t = 5
1  n (n + 1)  k
⇒ = ∴ Total number of ways = 5 × 9 × 5 = 225
(21)3  2 
 n = 20 21
5
 3  n (n + 1)  
2
14. Here, 47 C 4 + ∑ 52 − j
C3
Q 1 + 2 + K + n = 
3 3

 2   j =1
 
21  20 × 21
2 = 47C 4 + 51
C 3 + 50C 3 + 49C 3 + 48C 3 + 47
C3
⇒ k=   = 100
(21)3  2  = ( C4 +
47 47
C3 ) + C3 + C3 + C3 +
48 49 50 51
C3
∴ k = 100 [using C r + C r − 1 = n +1C r ]
n n
84 Permutations and Combinations

= (48C 4 + 48C 3 ) + 49C 3 + 50C 3 + 51


C3 18. Let A = { x1 , x2 , ... , xn }
= ( C4 + C3 ) + C3 +
49 49 50 51
C3 ∴ Number of functions from A to A is n n and out of these
= ( C4 + C3 ) + C3 = C4 + C3 = C4
50 50 51 51 51 52 n

∑ (−1) n−r n
C r (r )n are onto functions.
15. Given 6 boys M1 , M 2 , M 3 , M 4 , M 5 , M 6 and r =1

5 girls G1 , G2 , G3 , G4 , G5 19. The number of students answering exactly


(i) α 1 → Total number of ways of selecting 3 boys and 2 k (1 ≤ k ≤ n − 1) questions wrongly is 2n − k − 2n − k − 1 . The
girls from 6 boys and 5 girls. number of students answering all questions wrongly
i..e, 6C 3 × 5C 2 = 20 × 10 = 200 is 20.
∴ α 1 = 200 Thus, total number of wrong answers
(ii) α 2 → Total number of ways selecting at least 2 = 1 (2n − 1 − 2n − 2 ) + 2 (2n − 2 − 2n − 3 ) + K
member and having equal number of boys and girls + (n − 1) (21 − 20 ) + 20 ⋅ n
i.e., 6C1 C1 + 6C 2 C 2 + 6C 3 C 3 + 6C 4 C 4 + 6C 5 C 5 = 2n − 1 + 2n − 2 + 2n − 3 + K + 21 + 20 = 2n − 1
5 5 5 5 5

= 30 + 150 + 200 + 75 + 6 = 461 20. Let r consecutive integers be x + 1, x + 2, K , x + r.


⇒ α 2 = 461 (x + r )(x + r − 1) K (x + 1) x !
∴ (x + 1) (x + 2) K (x + r ) =
(iii) α 3 → Total number of ways of selecting 5 members x!
in which at least 2 of them girls (x + r )! r ! x + r
= ⋅ = C r ⋅ (r )!
i.e., 5C 2 C 3 + 5C 3 C 2+ 5C 4 C1 + 5C 5 C 0
6 6 6 6
(x)! r !
= 200 + 150 + 30 + 1 = 381 Thus, (x + 1) (x + 2) K (x + r ) = x + rC r ⋅ (r )! , which is
α 3 = 381 clearly divisible by (r )! . Hence, it is a true statement.
(iv) α 4 → Total number of ways of selecting 4 members
21. Given that, there are 9 women and 8 men, a committee
in which at least two girls such that M1 and G1 are
of 12 is to be formed including at least 5 women.
not included together.
This can be done in
G1 is included → 4 C1 ⋅ 5C 2 + 4C 2 ⋅ 5C1 + 4C 3
= (5 women and 7 men) + (6 women and 6 men)
= 40 + 30 + 4 = 74
+ (7 women and 5 men) + (8 women and 4 men)
M1 is included → 4 C 2 ⋅ 5C1 + 4C 3 = 30 + 4 = 34
+ (9 women and 3 men) ways
G1 and M1 both are not included
Total number of ways of forming committee
4
C 4 + 4 C 3 ⋅ 5C1 + 4C 2 ⋅ 5C 2
= (9C 5 . 8C7 ) + (9C 6. 8C 6 ) + (9C7 . 8C 5 )
1 + 20 + 60 = 81
∴ Total number = 74 + 34 + 81 = 189 + (9C 8 . 8C 4 ) + (9C 9. 8C 3 )
α 4 = 189 = 1008 + 2352 + 2016 + 630 + 56 = 6062
Now, P → 4; Q → 6; R → 5; S → 2 (i) The women are in majority = 2016 + 630 + 56
Hence, option (c) is correct. = 2702
16. Given that, no two persons sitting adjacent in circular (ii) The man are in majority = 1008 ways
arrangement, have hats of same colour. So, only 22. Since, student is allowed to select at most n books out of
possible combination due to circular arrangement is (2n + 1) books.
2 + 2 + 1. 2 n +1 2n +1 2 n +1
∴ C1 + C 2 + .... + C n = 63 ... (i)
So, there are following three cases of selecting hats are
2 n +1 2n +1 2n +1 2n +1
2R + 2B + 1G or 2B + 2G + 1R or 2G + 2R + 1B. We know C0 + C1 + ..... + C2 n + 1 = 2
2n +1 2n + 1 2n + 1 2n +1
To distribute these 5 hats first we will select a person ⇒ 2( C0 + C1 + C 2 + ... + C n ) = 22 n + 1
which we can done in 5C1 ways and distribute that hat ⇒ 2n +1
C1 + 2n +1
C 2 + ... + 2n +1
C n = (22 n − 1) .... (ii)
which is one of it’s colour. And, now the remaining four
hats can be distributed in two ways. From Eqs. (i) and (ii), we get
So, total ways will be 3 × 5C1 × 2 = 3 × 5 × 2 = 30 22 n − 1 = 63
17. PLAN Number of line segment joining pair of adjacent point = n ⇒ 22 n = 64
Number of line segment obtained joining n points
on a circle = nC 2 ⇒ 2n = 6
Number of red line segments = nC 2 − n ⇒ n =3
Number of blue line segments = n 23. Case I When one black and two others balls are
∴ n
C2 − n = n drawn.
n (n − 1) ⇒ Number of ways = 3C1 ⋅6 C 2 = 45
⇒ = 2n
2 Case II When two black and one other balls are drawn
⇒ n =5 ⇒ Number of ways = 3C 2 ⋅6 C1 = 18
Permutations and Combinations 85

n
Case III When all three black balls are drawn Cr 84
Also given, n
=
⇒ Number of ways = C 3 = 1 3 C r +1 126
∴ Total number of ways = 45 + 18 + 1 = 64 r+1 2
⇒ =
n−r 3
24. The possible cases are
⇒ 2n − 5r − 3 = 0 …(ii)
Case I A man invites 3 ladies and women invites 3
gentlemen. On solving Eqs. (i) and (ii), we get
Number of ways = 4C 3 ⋅4 C 3 = 16 r = 3 and n = 9
Case II A man invites (2 ladies, 1 gentleman) and
women invites (2 gentlemen, 1 lady). Topic 3 Multinomial, Repeated
Number of ways = ( C 2 ⋅ C1 ) ⋅ ( C1 ⋅ C 2 ) = 324
4 3 3 4 Arrangement and Selection
Case III A man invites (1 lady, 2 gentlemen) and 1. Key Idea Use divisibility test of 11 and consider different situation
women invites (2 ladies, 1 gentleman). according to given condition.
Number of ways = (4 C1 ⋅3 C 2 ) ⋅ (3C 2 ⋅ 4C1 ) = 144
Since, the sum of given digits
Case IV A man invites (3 gentlemen) and women 0 + 1 + 2 + 5 + 7 + 9 = 24
invites (3 ladies). Let the six-digit number be abcdef and to be divisible by
Number of ways = 3C 3 ⋅3 C 3 = 1 11, so the difference of sum of odd placed digits and sum
∴ Total number of ways, of even placed digits should be either 0 or a multiple of
11 means|(a + c + e) − (b + d + f )|should be either 0 or
= 16 + 324 + 144 + 1 = 485
a multiple of 11.
25. Since, m men and n women are to be seated in a row so Hence, possible case is a + c + e = 12 = b + d + f (only)
that no two women sit together. This could be shown as Now, Case I
× M1 × M 2 × M 3 × ... × M m × set { a , c, e} = {0, 5, 7} and set { b, d , f } = {1, 2, 9}
which shows there are (m + 1) places for n women. So, number of 6-digits numbers = (2 × 2 !) × (3 !) = 24
∴ Number of ways in which they can be arranged [Q a can be selected in ways only either 5 or 7].
(m)! ⋅ (m + 1)! Case II
= (m) ! m + 1 Pn =
(m + 1 − n )! Set { a , c, e} = {1, 2, 9} and set { b, d , f } = {0, 5, 7}
So, number of 6-digits numbers = 3 ! × 3 ! = 36
26. Let mn squares of equal size are arrange to form a
rectangle of dimension m by n. Shown as, from figure. So, total number of 6-digits numbers = 24 + 36 = 60
neighbours of x1 are {x2 , x3 , x4 , x5} x5 are {x1 , x6 , x7 } and 2. Since there are 8 males and 5 females. Out of these 13
x7 are {x5 , x4 }. members committee of 11 members is to be formed.
x + x3 + x4 + x5 x + x6 + x7
⇒ x1 = 2 , x5 = 1 According to the question, m = number of ways when
4 3 there is at least 6 males
x + x5
and x7 = 4 = (8C 6 × 5C 5 ) + (8C7 × 5 C 4 ) + (8C 8 × 5C 3 )
2
x + x6 + x7 = (28 × 1) + (8 × 5)+ (1 × 10) = 28 + 40 + 10 = 78
∴ 4x1 = x2 + x3 + x4 + 1
3 and n = number of ways when there is at least 3 females
x + x5 = ( 5C 3 × 8 C 8 ) + ( 5C 4 × 8 C7 ) + ( 5C 5 × 8 C 6 )
⇒ 12x1 = 3x2 + 3x3 + 3x4 + x1 + x6 + 4
2 = 10 × 1 + 5 × 8 + 1 × 28 = 78
⇒ 24x1 = 6x2 + 6x3 + 6x4 + 2x1 + 2x6 + x4 + x5 So, m = n = 78
⇒ 22x1 = 6x2 + 6x3 + 7x4 + x5 + 2x6 3. Given there are three boxes, each containing 10 balls
where, x1 , x2 , x3 , x4 , x5 , x6 are all the natural numbers labelled 1, 2, 3, … , 10.
and x1 is linearly expressed as the sum of x2 , x3 , x4 , x5 , x6 Now, one ball is randomly drawn from each boxes, and
where sum of coefficients are equal only if, all ni denote the label of the ball drawn from the ith box,
observations are same. (i = 1, 2, 3).
⇒ x2 = x3 = x4 = x5 = x6 Then, the number of ways in which the balls can be
⇒ All the numbers used are equal. chosen such that n1 < n2 < n3 is same as selection of 3
n
Cr n−r+1 different numbers from numbers {1, 2, 3, … , 10} = 10C 3
27. We know that, n
= = 120.
C r −1 r
84 7 n − r + 1 4. Using the digits 0, 1, 3, 7, 9
⇒ = = [given]
36 3 r number of one digit natural numbers that can be formed
= 4,
⇒ 3n − 10r + 3 = 0 …(i)
86 Permutations and Combinations

number of two digit natural numbers that can be Number of words starting with CH, CI, CN is 4! each.
formed = 20, Similarly, number of words before the first word
starting with CO = 4! + 4! + 4! + 4! = 96.
The word starting with CO found first in the dictionary
4×5 is COCHIN. There are 96 words before COCHIN.
(Q 0 can not come in Ist box) 8. Let the engineer visits the factory first time after x1 days
number of three digit natural numbers that can be to 1 June, second time after x2 days to first visit and so
formed = 100 on.
∴ x1 + x2 + x3 + x4 + x5 = 11
where x1 , x5 ≥ 0 and x2 , x3 , x4 ≥ 1 according to the
4×5× 5 requirement of the question.
and number of four digit natural numbers less than Now, let x2 = a + 1, x3 = b + 1 and x4 = c + 1 where
7000, that can be formed = 250 a , b, c ≥ 0
∴New equation will be
x1 + a + b + c + x5 = 8
2×5× 5×5 Now, the number of all possible ways in which the
engineer can made visits is equals to the non-negative
(Q only 1 or 3 can come in Ist box)
integral solution of equation
∴Total number of natural numbers formed x1 + a + b + c + x5 = 8, and it is equal to
= 4 + 20 + 100 + 250 = 374 8 + 5−1 12 × 11 × 10 × 9
C 5 − 1 = 12C 4 = = 495.
5. Number of girls in the class = 5 and number of boys in 4 ×3 ×2
the class = 7 9. Given word in MOTHER, now alphabetical order
Now, total ways of forming a team of 3 boys and 2 girls of letters is EHMORT, so number of words start with
letter.
= 7C 3 ⋅5 C 2 = 350 E ------ is 5!
But, if two specific boys are in team, then number of H ------ is 5!
ways = 5C1 ⋅5 C 2 = 50 M E ------ is 4!
Required ways, i.e. the ways in which two specific boys M H ------ is 4!
are not in the same team = 350 − 50 = 300. M O E ------ is 3!
M O H ------ is 3!
Alternate Method
M O R ------ is 3!
Number of ways when A is selected and B is not M O T E ------ is 2!
= 5C 2 ⋅5 C 2 = 100 M O T H E R is 1
So, position of the word ‘MOTHER’ is
Number of ways when B is selected and A is not
5! + 5! + 4! + 4! + 3! + 3! + 3! + 2! + 1
= 5C 2 ⋅5 C 2 = 100
= 120 + 120 + 24 + 24 + 6 + 6 + 6 + 2 + 1 = 309
Number of ways when both A and B are not selected
10. A number is divisible by 4 if last 2 digit number is
= 5C 3 ⋅5 C 2 = 100 divisible by 4.
∴ Required ways = 100 + 100 + 100 = 300. ∴ Last two digit number divisible by 4 from (1, 2, 3, 4, 5)
4! are 12, 24, 32, 44, 52
6. Clearly, number of words start with A = = 12
2! ∴ The number of 5 digit number which are divisible by
Number of words start with L = 4 ! = 24 4, from the digit (1, 2, 3, 4, 5) and digit is repeated is
Number of words start with M =
4!
= 12 5 × 5 × 5 × (5 ×1) = 625
2! 11. x = 10 !
3! 10 ! 10 ! y 10
Number of words start with SA = =3 y = 10C1 × 9C 8 × = 10 × 9 × ⇒ = =5
2! 2! 2 9x 2
Number of words start with SL = 3 ! = 6
12. Here, B1 B2 B3 B4 B5
Note that, next word will be “SMALL”.
Hence, the position of word “SMALL” is 58th. Out of 5 girls, 4 girls are together and 1 girl is
separate. Now, to select 2 positions out of 6
7. Arrange the letters of the word COCHIN as in the order positions between boys = 6C 2 …(i)
of dictionary CCHINO.
4 girls are to be selected out of 5 = 5C 4 …(ii)
Consider the words starting from C.
Now, 2 groups of girls can be arranged in 2 !ways. …(iii)
There are 5! such words. Number of words with the two
Also, the group of 4 girls and 5 boys is arranged in 4 ! × 5 !
C’s occupying first and second place = 4 ! .
ways . …(iv)
Permutations and Combinations 87

Now, total number of ways = 6C 2 × 5C 4 × 2 ! × 4 ! × 5 ! Topic 4 Distribution of Object into Group


[from Eqs. (i), (ii), (iii) and (iv)]
1. It is given that a group of students comprises of 5 boys
∴ m = 6C 2 × 5C 4 × 2 ! × 4 ! × 5 ! and n girls. The number of ways, in which a team of 3
and n = 5! × 6! students can be selected from this group such that each
m 6C 2 × 5C 4 × 2 ! × 4 ! × 5 ! 15 × 5 × 2 × 4 ! team consists of at least one boy and at least one girls, is
⇒ = = =5 = (number of ways selecting one boy and 2 girls) +
n 6! × 5! 6 × 5 × 4!
(number of ways selecting two boys and 1 girl)
13. PLAN Reducing the equation to a newer equation, where sum of
= ( C1 × nC 2 ) ( C 2 × nC1 ) = 1750 [given]
5 5
variables is less. Thus, finding the number of arrangements
becomes easier.  n (n − 1)  5 × 4 
⇒ 5 ×  + × n = 1750
As, n1 ≥ 1, n2 ≥ 2 , n3 ≥ 3, n4 ≥ 4, n5 ≥ 5  2   2 
Let n1 − 1 = x1 ≥ 0, n2 − 2 = x2 ≥ 0, ..., n5 − 5 = x5 ≥ 0 2
⇒ n (n − 1) + 4n = × 1750 ⇒ n 2 + 3n = 2 × 350
⇒ New equation will be 5
x1 + 1 + x2 + 2 + ... + x5 + 5 = 20 ⇒ n 2 + 3n − 700 = 0 ⇒ n 2 + 28n − 25n − 700 = 0
⇒ x1 + x2 + x3 + x4 + x5 = 20 − 15 = 5 ⇒ n (n + 28) − 25(n + 28) = 0 ⇒ (n + 28) (n − 25) = 0
Now, x1 ≤ x2 ≤ x3 ≤ x4 ≤ x5 ⇒ n = 25 [Q n ∈ N ]

x1 x2 x3 x4 x5 2. According to given information, we have the following


figure.
0 0 0 0 5 Y
0 0 0 1 4 (0, b)
0 0 0 2 3 B
0 0 1 1 3
0 0 1 2 2
0 1 1 1 2 X
O A (a, 0)
1 1 1 1 1
(Note that as a and b are integers so they can be
So, 7 possible cases will be there.
negative also). Here O(0, 0), A (a , 0) and B(0, b) are the
14. The number of solutions of x1 + x2 + ... + xk = n three vertices of the triangle.
= Coefficient of t n in (t + t 2 + t 3 + ... )(t 2 + t 3 + ... )... Clearly, OA =| a |and OB =| b|.
(t k + t k +1 + ... ) 1
1 + 2 + ... + k
∴Area of ∆OAB = | a ||b|.
= Coefficient of t in t
n
(1 + t + t + ... )k
2
2
k(k + 1) But area of such triangles is given as 50 sq units.
Now, 1 + 2 + ... + k = =p [say]
2 1
∴ |a || b| = 50
1 2
and 1 + t + t 2 + ... =
1−t ⇒ |a || b| = 100 = 22 ⋅ 52
Thus, the number of required solutions Number of ways of distributing two 2’s in|a |and| b| = 3
= Coefficient of t n − p in (1 − t )− k | a| | b|
= Coefficient of t n − p in [1 + k C1 t + k +1 C 2t 2 + k + 2 C 3t 3 + ... ] 0 2
= k + n − p −1 C n − p = r C n − p 1 1
1 2 0
where, r = k + n − p − 1 = k + n − 1 − k(k + 1)
2 ⇒ 3 ways
1 Similarly, number of ways of distributing two 5’s in| a |
= (2k + 2n − 2 + k2 − k)
2 and|b|= 3 ways.
1
= (2n − k2 + k − 2) ∴ Total number of ways of distributing 2’s and 5’s
2 = 3 × 3 = 9 ways
15. Since, six ‘+’ signs are + + + + + + Note that for one value of | a | , there are 2 possible
values of a and for one value of|b|, there are 2 possible
∴ 4 negative sign has seven places to be arranged in
values of b.
⇒ 7
C 4 ways = 35 ways
∴Number of such triangles possible = 2 × 2 × 9 = 36.
16. Since, each box can hold five balls. So, number of elements in S is 36.
∴ Number of ways in which balls could be distributed so 3. Given 6 different novels and 3 different dictionaries.
that none is empty, are (2, 2, 1) or (3, 1, 1).
Number of ways of selecting 4 novels from 6 novels is
( 5C 2 3C 2 1C1 + 5C 3 C1 1C1 ) × 3 !
2
i.e.
6!
= (30 + 20) × 6 = 300
6
C4 = = 15
2 !4 !
88 Permutations and Combinations

Number of ways of selecting 1 dictionary is from 3 7. (i) The number of ways in which 52 cards be divided
3! equally among four players in order
dictionaries is 3C1 = =3
1 !2 ! (52)!
= 52C13 × 39C13 × 26C13 × 13C13 =
∴ Total number of arrangement of 4 novels and 1 (13 !)4
dictionary where dictionary is always in the middle, is
(ii) The number of ways in which a pack of 52 cards
15 × 3 × 4 ! = 45 × 24 = 1080 can be divided equally into four groups of 13 cards
4. Objects Groups Objects Groups 52
C13 × 39C13 × 26C13 × 13C13 (52)!
each = =
Distinct Distinct Identical Identical 4! 4 !(13 !)4
Distinct Identical Identical Distinct (iii) The number of ways in which a pack of 52 cards be
Description of Situation Here, 5 distinct balls are divided into 4 sets, three of them having 17 cards
distributed amongst 3 persons so that each gets at least each and the fourth just one card
one ball. i.e. Distinct → Distinct 52
C17 × 35C18 × 18C17 × 1C1 (52)!
= =
So, we should make cases 3! 3 !(17)3
A B C  A B C 
Case I  Case II  8. The groups of persons can be made only in 2, 2, 1, 1
1 1 2  1 2 2 
∴ So the number of required ways is equal to number of
Number of ways to distribute 5 balls ways to distribute the 6 distinct objects in group sizes 1,
 3 !  3 ! 1, 2 and 2
=  5C1 ⋅4 C1 ⋅3 C 3 ×  +  5C1 ⋅4 C 2 ⋅2 C 2 × 
 2 !  2 ! 6!
= (4 !)
= 60 + 90 = 150 (2 !)2 (1 !)2 (2 !) (2 !)

5. Total number of arrangements of word BANANA = 360 × 3 = 1080.


6!
= = 60
3!2! Topic 5 Dearrangement and Number of
The number of arrangements of words BANANA in Divisors
5!
which two N’s appear adjacently = = 20 1. Since, 240 = 24 .3.5
3!
∴ Total number of divisors = (4 + 1)(2)(2) = 20
Required number of arrangements = 60 − 20 = 40
Out of these 2, 6, 10, and 30 are of the form 4n + 2.
6. Here, n 2 objects are distributed in n groups, each group
containing n identical objects. 2. The number of ways in which the ball does not go its
 1 1 1 1
∴ Number of arrangements own colour box = 4 ! 1 − + − + 
 1 ! 2 ! 3 ! 4 !
2
n − n
2 2
− 2n n − 3n
2
n − 2n
2
= n
Cn . Cn . n Cn . Cn . C n K nC n
1 1 1
= 4!  − + 
(n 2 )! (n 2 − n )! n! (n 2 )!  2 6 24
= . K =
n ! (n − n )! n ! (n − 2n )!
2 2
n ! ⋅ 1 (n !)n
 12 − 4 + 1
= 24   =9
⇒ Integer (as number of arrangements has to be integer).  24 
5
Binomial Theorem
Topic 1 Binomial Expansion and General Term
Objective Questions I (Only one correct option) 9. If the fourth term in the binomial expansion of
6
1. If the constant term in the binomial expansion of   1  1
10   1+ log10 x  
 k  x + x  is equal to 200, and x > 1, then the
12
 x − 2 is 405, then|k|equals (2020 Main, 6 Sep II)  
 x   
(a) 9 (b) 1 (c) 3 (d) 2 value of x is (2019 Main, 8 April II)
2. If α and β be the coefficients of x and x respectively in
4 2
(a) 100 (b) 104 (c) 10 (d) 103
the expansion of (x + x2 − 1 )6 + (x − x2 − 1 )6, then 10. The sum of the coefficients of all even degree terms is x
in the expansion of (2019 Main, 8 April I)
(2020 Main, 8 Jan II)
(a) α + β = − 30 (b) α − β = − 132 (x + x − 1 ) + (x − x − 1 ) , (x > 1) is equal to
3 6 3 6

(c) α + β = 60 (d) α − β = 60 (a) 29 (b) 32 (c) 26 (d) 24


3. The coefficient of x18 in the product 11. The total number of irrational terms in the binomial
(1 + x)(1 − x)10 (1 + x + x2)9 is (2019 Main, 12 April I) expansion of (71/5 − 31/10 )60 is (2019 Main, 12 Jan II)
(a) 84 (b) − 126 (c) − 84 (d) 126 (a) 49 (b) 48 (c) 54 (d) 55
4. If the coefficients of x2 and x3 are both zero, in the 12. The ratio of the 5th term from the beginning to the 5th
expansion of the expression (1 + ax + bx2) (1 − 3x)15 in term from the end in the binomial expansion of
powers of x, then the ordered pair (a , b) is equal to  1 
10
(2019 Main, 10 April I)  3 1 
(a) (28, 315) (b) (− 21, 714) 2 + 1
is (2019 Main, 12 Jan I)
 
(c) (28, 861) (d) (− 54, 315)  2(3)3 
1 1 1 1
5. The term independent of x in the expansion of
(a) 1 : 2(6)3 (b) 1 : 4(16)3 (c) 4(36)3 : 1 (d) 2(36)3 : 1
1 x 
8
3
6
 −  . 2x2 − 2 is equal to
 60 81  x  (2019 Main, 12 April II)
13. The sum of the real values of x for which the middle
8
 x3 3
(a) − 72 (b) 36 (c) − 36 (d) − 108 term in the binomial expansion of  +  equals
6. The smallest natural number n, such that the 3 x
 1
n 5670 is (2019 Main, 11 Jan I)
coefficient of x in the expansion of  x2 + 3  is nC 23 , is (a) 4 (b) 0 (c) 6 (d) 8
 x 
(2019 Main, 10 April II) 14. The positive value of λ for which the coefficient of x2 in
10
 λ
(a) 35 (b) 23 (c) 58 (d) 38 the expression x2  x + 2 is 720, is
 x  (2019 Main, 10 Jan II)
7. If some three consecutive coefficients in the binomial
expansion of (x + 1) in powers of x are in the ratio 2 : 15 :
n (a) 3 (b) 5 (c) 2 2 (d) 4
70, then the average of these three coefficients is 15. If the third term in the binomial expansion of
(2019 Main, 9 April II) (1 + xlog 2 x )5 equals 2560, then a possible value of x is
(a) 964 (b) 227 (2019 Main, 10 Jan I)
(c) 232 (d) 625 1 1
(a) 4 2 (b) (c) (d) 2 2
8. If the fourth term in the binomial expansion of 4 8
6 3
2 log x   1 − t6 
 + x 8  (x > 0) is 20 × 8 , then the value of x is
7
x  16. The coefficient of t 4 in the expansion of   is
(2019 Main, 9 April I) 1−t
(a) 8−2 (b) 83 (2019 Main, 9 Jan II)
(c) 8 (d) 82 (a) 12 (b) 10 (c) 15 (d) 14
90 Binomial Theorem

17. The sum of the coefficients of all odd degree terms in the 29. If the coefficients of x3 and x4 in the expansion of
5 5
(1 + ax + bx2)(1 − 2x)18 in powers of x are both zero, then
expansion of  x + x3 − 1 +  x − x3 − 1 , (x > 1) is
(2018 Main) (a , b) is equal to
(a) −1
(a)  16,
251  251
(c)  14,
272 
(d)  16,
(b) 0 (c) 1 (d) 2 272 
 (b)  14,   
18. The value of (21C1 − 10C1 ) + (21C 2 − 10C 2)  3   3   3   3 
+ ( C3 − C3 ) + ( C 4 − 10C 4 ) + ... + (21C10 − 10C10 ) is
21 10 21

(2017 Main) Fill in the Blanks


(a) 221 − 211 (b) 221 − 210 (c) 220 − 29 (d) 220 − 210
30. Let n be a positive integer. If the coefficients of 2nd, 3rd,
19. If the number of terms in the expansion of and 4th terms in the expansion of (1 + x)n are in AP,
n
 2 4 then the value of n is… . (1994, 2M)
1 − + 2 , x ≠ 0, is 28, then the sum of the
 x x  31. If (1 + a x)n = 1 + 8x + 24x2 + … , then a = … and n = K .
coefficients of all the terms in this expansion, is (1983, 2M)
(2016 Main)
(a) 64 (b) 2187 (c) 243 (d) 729 32. For any odd integer n ≥ 1, n − (n − 1) + K 3 3

20. The sum of coefficients of integral powers of x in the + (− 1)n−113 = K


binomial expansion (1 − 2 x ) 50
is (2015 Main) 33. The larger of 9950 + 10050 and 10150 is ... .
1 50 1 1 50 1 50
(a) (3 + 1) (b) (350 ) (c) (3 − 1) (d) (2 + 1)
2 2 2 2 Analytical & Descriptive Questions
11 k
21. Coefficient of x in the expansion of
34. Prove that ∑ (−3)r − 1 3 nC 2r − 1 = 0, where k = (3n )/ 2 and n
(1 + x ) (1 + x ) (1 + x ) is
2 4 3 7 4 12
(2014 Adv.) r =1
(a) 1051 (b) 1106 (c) 1113 (d) 1120 is an even positive integer. (1993, 5M)
22. The term independent of x in expansion of 2n 2n

 x+1 x−1 
10 35. If ∑ a r (x − 2)r = ∑ br (x − 3)r and a k = 1 , ∀ k ≥ n, then
 2/ 3 −  is (2013 Main) r=0 r=0
x − x + 1 x − x1 / 2
1 /3
show that bn = 2n + 1
Cn+ 1 (1992, 6M)
(a) 4 (b) 120 (c) 210 (d) 310 36. Find the sum of the series
23. Coefficient of t 24 in (1 + t 2)12 (1 + t12) (1 + t 24 ) is n  1 3r r

7r 15
(2003, 1M) ∑ (− 1 ) r n
C r  r
+ 2r
+ 3r
+ 4r
... upto m terms .
(a) 12C6 + 3 (b) 12C6 + 1 (c) 12C6 (d) 12C6 + 2 r=0 2 2 2 2

24. In the binomial expansion of (a − b)n , n ≥ 5 the sum of (1985, 5M)
the 5th and 6th terms is zero. Then, a / b equals 37. Given, sn = 1 + q + q + K + q 2 n

(2001, 1M) 2 n
n−5 n−4 q + 1  q + 1  q + 1
(a) (b) (c)
5
(d)
6 Sn = 1 + +  + ... +   ,q ≠1
6 5 n−4 n−5 2  2   2 
n+ 1 n+ 1 n+ 1
25. If in the expansion of (1 + x)m (1 − x)n, the coefficients of Prove that C1 + C 2 s1 + C3 s2
n+ 1
x and x are 3 and −6 respectively, then m is euqal to
2 + ... + C n+ 1 sn = 2nS n
(1999, 2M) (1984, 4M)
(a) 6 (b) 9 (c) 12 (d) 24
26. The expression [x + (x3 − 1) 1/ 2]5 + [x − (x3 − 1)1/ 2]5 is a Integer & Numerical Answer Type Questions
polynomial of degree (1992, 2M) 38. The natural number m, for which the coefficient of x in
22
(a) 5 (b) 6 (c) 7 (d) 8  1
the binomial expansion of  xm + 2 is 1540, is …… .
10  x 
27. The coefficient of x in  − 2
x 3
4
is (1983, 1M)
2 x  [2020 Main, 5 Sep I]

405 504 39. Let m be the smallest positive integer such that
(a) (b)
256 259 the coefficient of x2 in the expansion of
450 (1 + x)2 + (1 + x)3 + K + (1 + x)49 + (1 + mx)50is (3n + 1)
(c) (d) None of these 51
263 C3 for some positive integer n. Then, the value of n is
(2016 Adv.)
28. Given positive integers r > 1, n > 2 and the coefficient of
40. The coefficient of x9 in the expansion of
(3r )th and (r + 2)th terms in the binomial expansion of
(1 + x) (1 + x2) (1 + x3 ) ... (1 + x100 ) is (2015 Adv.)
(1 + x)2n are equal. Then, (1980, 2M)
(a) n = 2r (b) n = 2r + 1 41. The coefficients of three consecutive terms of (1 + x)n + 5
(c) n = 3r (d) None of these are in the ratio 5 : 10 : 14. Then, n is equal to (2013 Adv.)
Binomial Theorem 91

Topic 2 Properties of Binomial Coefficient


Objective Questions I (Only one correct option) 10. If C r stands for nC r, then the sum of the series
1. Let (x + 10)50 + (x − 10)50 = a 0 + a1x + a 2x2 + K + a50x50,  n  n
2  !  !
a2  2  2
for all x ∈ R; then is equal to [C 02 − 2 C12 + 3 C 22 − ... + (−1)n (n + 1) C n2 ],
(2019 Main, 11 Jan II) n!
a0
(a) 12.25 (b) 12.50 where n is an even positive integer, is (1986, 2M)
(c) 12.00 (d) 12.75 (a) (−1)n/ 2 (n + 2) (b) (−1)n (n + 1)
(c) (−1)n/ 2 (n + 1) (d) None of these
2. The value of r for which
20
Cr 20
C0 + 20
Cr−1 20
C1 + Cr− 2 20C2 + .... + 20C 020C r
20
Objective Question II
is maximum, is (2019 Main, 11 Jan I) (One or more than one correct option)
(a) 15 (b) 10
(c) 11 (d) 20 11. For non-negative integers s and r, let
 s!
2 403
k  s  if r ≤ s,
3. If the fractional part of the number is , then k is   =  r ! (s − r )!
15 15  r 
 0 if r > s
equal to (2019 Main, 9 Jan I)
(a) 14 (b) 6 For positive integers m and n, let
m+ n
(c) 4 (d) 8 f (m, n , p)
g (m, n ) = Σ
p = 0  n + p
4. For r = 0, 1, ... , 10, if Ar, Br and C r denote respectively  
 p 
the coefficient of xr in the expansions of (1 + x)10, (1 + x)20
10 where for any non-negative integer p,
and (1 + x)30. Then, ∑ Ar (B10Br − C10 Ar ) is equal to p  m  n + i  p + n
f (m, n , p) = Σ      . (2020 Adv.)
r =1 i =0  i   p   p − i 
(a) B10 − C10 (b) 2
− C10 A10 )
A10 (B10 (2010)
Then which of the following statements is/are TRUE?
(c) 0 (d) C10 − B10 (a) g (m, n ) = g (n , m) for all positive integers m, n
(b) g (m, n + 1) = g (m + 1, n ) for all positive integers m, n
5. 30  30 30 30 30 30 30 30
0  10 −  1  11 +  2  12 + K + 20 30 is
(c) g (2m, 2n ) = 2 g (m, n ) for all positive integers m, n
equal to (2005, 1M) (d) g (2m, 2n ) = ( g (m, n ))2 for all positive integers m, n
(a) 30 C11 (b) 60
C10
(c) 30 C10 (d) 65
C55 Integer & Numerical Answer Type Questions
6. If n −1
C r = (k2 − 3) nC r + 1, then k belongs to  n n 
 ∑k ∑ nC kk2 
(2004, 1M)
(a) (− ∞ , − 2] (b) [2, ∞ )
det  n  = 0 holds for some
k=0 k=0
12. Suppose
(c) [ − 3 , 3 ] (d) ( 3 , 2]  n
 ∑ C kk∑ C k3 
n n k

10 20  , where  p = 0 if p > q,


m
k = 0k=0 
7. The sum ∑  i  m − i  
 q
is n n
Ck
i=0
positive integer n. Then ∑ equals ………
maximum when m is equal to (2002, 1M) k=0
k +1
(2019 Adv.)
(a) 5 (b) 10
(c) 15 (d) 20 13. Let X = ( C1 ) + 2( C 2) + 3( C3 )
10 2 10 2 10
+ ... + 10(10C10 )2,
2

where 10C r, r ∈{1, 2,... , 10} denote binomial coefficients.


 n  n   n  1
8. For 2 ≤ r ≤ n,   + 2  +  is equal to Then, the value of X is .......... .
 r  r − 1  r − 2 (2000, 2M) 1430 (2018 Adv.)
 n + 1  n + 1 Fill in the Blank
(a)   (b) 2  
 r − 1  r + 1
14. The sum of the coefficients of the polynomial
 n + 2  n + 2 (1 + x − 3x2)2163 is …. . (1982, 2M)
(c) 2   (d)  
 r   r  Analytical & Descriptive Questions
n n
1 r 15. Prove that
9. If a n = ∑ , then ∑ nC equals
n
Cr  n  n  n  n − 1 k − 2  n  n − 2
r=0 r=0 r
2k     − 2k −1     +2     − ...
(a) (n − 1) an (b) n an (1998, 2M)
 0  k  1  k − 1  2  k − 2
1  n  n − k  n
(c) n an (d) None of these + (−1)k     =   (2003, 4 M)
2  k  0   k
92 Binomial Theorem

16. For any positive integers m, n (with n ≥ m), 18. If n is a positive integer and
 n (1 + x + x2)n = a 0 + a1x + ... + a 2n x2n.
If   = nCm. Prove that
m Then, show that, a 02 − a12 + ... + a 22n = a n. (1994, 5M)
 n   n − 1  n − 2 m  n + 1 19. Prove that C 0 − 2 2 ⋅ C1 + 3 2 ⋅ C 2 − ... + (−1)n (n + 1) 2⋅ C n
 + +   + ... +   =  
m  m   m  m m + 1 = 0 , n > 2, where C r = nC r. (1989, 5M)
or 20. If (1 + x) = C 0 + C1x + C 2 x + ... + C nx , then show that
n 2 n
Prove that
the sum of the products of the Ci’s taken two at a time
 n  n − 1  n − 2 represented by Σ Σ CiC j is equal to
  +2  +3  + ... + (n − m + 1)
m  m   m  0 ≤ i < j ≤ n 22 n −1 −
(2n !)
. (1983, 3M)
m  n + 2 2 (n !)2
  =  (IIT JEE 2000, 6M)
m m + 2 21. Prove that C12 − 2 ⋅ C 22 + 3 ⋅ C32 − ...−2n ⋅ C 22n = (−1)n n ⋅ C n
(1979, 4M)
3!
n
 nC 
17. Prove that = ∑ (−1)r  r + 3 r  . 22. Prove that ( C 0 ) − ( C1 ) + ( C 2) − ... + (2 nC 2 n )2
2n 2 2n 2 2n 2
2(n + 3) r = 0  Cr  (1997C, 5M) = (−1)n ⋅2n C n. (1978, 4M)

Answers
Topic 1 32.
1
(n + 1 ) 2 (2n − 1 )
1. (c) 2. (b) 3. (a) 4. (a) 4
5. (c) 6. (d) 7. (c) 8. (d)  2mn − 1 
33. (101 ) 50 36.  mn n 
9. (c) 10. (d) 11. (c) 12. (c) 2 (2 − 1 ) 
13. (b) 14. (d) 15. (b) 16. (c) 38. (13) 39. (5) 40. (8) 41. (n = 6 )
17. (d) 18. (d) 19. (d) 20. (a)
Topic 2
21. (c) 22. (c) 23. (d) 24. (b)
25. (c) 26. (c) 27. (a) 28. (a) 1. (a) 2. (d) 3. (d) 4. (d)
5. (c) 6. (d) 7. (c) 8. (d)
29. (d) 30. (n = 7)
9. (c) 10. (a) 11. (a,b,d) 12. (6.20)
31. (a = 2, n = 4)
13. (646) 14. ( − 1 )

Hints & Solutions


Topic 1 Binomial Expansion and = 2 [−15 − (15 × 2) − (1 × 3)] = − 96
General Term and the coefficient of x2 in the expansion
β = 2 [6C 44C 0 + 6C 63C 2] = 2 [15 + 3] = 36
1. Since, the general term in the expansion of binomial
 k
10
∴ α − β = − 96 − 36 = − 132
 x − 2 is
 x  Hence, option (b) is correct.
 10 − r  10 − 5 r
  3. Given expression is (1 + x) (1 − x)10 (1 + x + x2)9
Tr + 1 = 10
Cr x  2 
(− k)r x− 2r = 10C r (− k)r x 2

Q Term is constant, so r = 2. = (1 + x) (1 − x) [(1 − x) (1 + x + x2)]9


10 × 9 2 = (1 − x2) (1 − x3 )9
∴ 10C 2(− k)2 = 405 ⇒ k = 405
2 Now, coefficient of x18 in the product
⇒ k2 = 9 ⇒ | k| = 3
(1 + x) (1 − x)10 (1 + x + x2)9
2. Given expression is = coefficient of x18 in the product (1 − x2) (1 − x3 )9
(x + x2 − 1 )6 + (x − x2 − 1 )6 = coefficient of x18 in (1 − x3 )9
= 2 [ C 0x + C 2 x (x − 1) + C 4x (x − 1)
6 6 6 4 2 6 2 2 2
− coefficient of x16 in (1 − x3 )9
+ 6C 6 (x2 − 1)3 ] Since, (r + 1)th term in the expansion of
4
The coefficients of x in above expansion
(1 − x3 )9 is 9C r (− x3 )r = 9C r (− 1)r x3 r
α = 2 [6C 2(−1) + 6C 42C1 (−1) + 6C 63C1 (−1)]
Binomial Theorem 93

Now, for x18, 3r = 18 ⇒ r = 6 = nC rx2n − 2r − 3 r = nC rx2n − 5 r


16 For the coefficient of x ,
and for x16, 3r = 16 ⇒ r = ∉N.
3 2n − 5r = 1 ⇒ 2n = 5r + 1 …(i)
9! 9 ×8 × 7 As coefficient of x is given as nC 23 , then either r = 23 or
∴Required coefficient is 9C 6 = = = 84 n − r = 23 .
6 !3 ! 3 ×2
If r = 23, then from Eq. (i), we get
4. Given expression is (1 + ax + bx2)(1 − 3x)15 . In the 2n = 5(23) + 1
expansion of binomial (1 − 3x)15 , the (r + 1) th term is ⇒ 2n = 115 + 1 ⇒ 2n = 116 ⇒ n = 58.
Tr + 1 = 15C r (−3x)r = 15C r (−3)r xr
If n − r = 23, then from Eq. (i) on replacing the value of
Now, coefficient of x2, in the expansion of ‘r’, we get 2n = 5(n − 23) + 1
(1 + ax + bx2)(1 − 3x)15 is ⇒ 2n = 5n − 115 + 1 ⇒ 3n = 114 ⇒ n = 38
15
C 2(−3) + a C1 (−3) + b C 0 (−3) = 0 (given)
2 15 1 15 0 So, the required smallest natural number n = 38.
⇒ (105 × 9) − 45 a + b = 0 ⇒ 45a − b = 945 …(i) 7. Key Idea Use general term of Binomial expansion ( x + a) n i.e.
Similarly, the coefficient of x3 , in the expansion of T r + 1 = nC r 1 x n − r a r
(1 + ax + bx2)(1 − 3x)15 is
Given binomial is (x + 1)n, whose general term, is
C3 (−3)3 + a 15C 2(−3)2 + b 15C1 (−3)1 = 0
15
(given) Tr + 1 = nC r xr
⇒ − 12285 + 945a − 45b = 0
According to the question, we have
⇒ 63a − 3b = 819 ⇒ 21a − b = 273 …(ii) n
C r − 1 : nC r : nC r + 1 = 2 : 15 : 70
From Eqs. (i) and (ii), we get n
Cr − 1 2
24a = 672 ⇒ a = 28 Now, n
=
Cr 15
So, b = 315 ⇒ (a , b) = (28, 315)
n!
5. Key Idea Use the general term (or (r + 1)th term) in the (r − 1)!(n − r + 1)! 2
⇒ =
expansion of binomial (a + b) n n! 15
n−r
i.e. Tr + 1 = C r a
n
b r
r !(n − r )!
r 2
 3
6 ⇒ = ⇒ 15r = 2n − 2r + 2
Let a binomial 2x2 − 2 , it’s (r + 1)th term n − r + 1 15
 x 
r
⇒ 2n − 17r + 2 = 0 …(i)
2 6−r  3 n!
= Tr + 1 = C r (2x )
6
 − 2
 x  n
C 15 r !(n − r )! 3
Similarly, n r = ⇒ =
= C r (− 3) (2)
6 r
x 6−r 12 − 2r − 2r Cr + 1 70 n! 14
= C r (−3) (2)
6 r 6 − r 12 − 4 r
x …(i) (r + 1)!(n − r − 1)!
r+1 3
Now, the term independent of x in the expansion of ⇒ = ⇒ 14r + 14 = 3n − 3r
1 x8   2 3
6 n − r 14
 −   2x − 2
 60 81  x  ⇒ 3n − 17r − 14 = 0 …(ii)
= the term independent of x in the expansion of On solving Eqs. (i) and (ii), we get
6
1  2 3 n − 16 = 0 ⇒ n = 16 and r = 2
2x − 2 + the term independent of x in the
60  x  16
C1 + C 2 + 16C3
16
6 Now, the average =
x8  2 3  3
expansion of −  2x − 2
81  x  16 + 120 + 560 696
6
= = = 232
C3 3 3
= (− 3)3 (2)6 − 3 x12 − 4 (3 ) [put r = 3] 6
60
8. Given binomial is  + x
2 log 8 x 
 1 
+  −  6C5 (− 3)5 (2)6 − 5 x12 − 4 (5 ) x8 [put r = 5] x 
 81
Since, general term in the expansion of (x + a )n is
1 35 × 2(6) Tr+ 1 = n C r xn− ra r
= (− 3)3 23 + = 36 − 72 = − 36 6 −3
3 81  2
∴ T4 = T3 + 1 = 6 C3   (xlog 8 x )3 = 20 × 87 (given)
n  x
6. Given binomial is  x2 +
1 3
 , its (r + 1) term, is  2
th
 x3  ⇒ 20   x3 log 8 x = 20 × 87 [Q 6C3 = 20]
r
 x
 1 1 3 
Tr + 1 = nC r (x2)n − r  3  = nC rx2n − 2r 3 r  log 2 x −3 
x  x ⇒ 23 x [3(log 8 x )−3 ]= (23 )7 ⇒ x 3 
= (23 )6
94 Binomial Theorem

 1  (x + x3 − 1 )6 + (x − x3 − 1 )6
Q log an (x) = log a x for x > 0; a > 0, ≠ 1
 n 
= 2 [6C 0 − 6C 2 + 6 C 4 + 6C 4 − 1 − 3]
log x − 3 )
⇒ x( 2 = 218 = 2 [1 − 15 + 15 + 15 − 1 − 3] = 2(15 − 3) = 24
On taking log 2 both sides, we get
11. The general term in the binomial expansion of (a + b)n
(log 2 x − 3) log 2 x = 18 is Tr + 1 = nC r a n − rbr.
⇒ (log 2 x) − 3 log 2 x − 18 = 0
2
So, the general term in the binomial expansion of
⇒ (log 2 x)2 − 6 log 2 x + 3 log 2 x − 18 = 0 (71/5 − 31/10 )60 is
⇒ log 2 x(log 2 x − 6) + 3 (log 2 x − 6) = 0 Tr + 1 = C r (71/5 )60 − r (−31/10 )r
60
60 − r r r r
12 −
⇒ (log 2 x − 6) (log 2 x + 3) = 0 = 60
Cr 7 5 (−1 )r 310 = (−1)r 60
Cr 7 5 310
⇒ log 2 x = −3, 6
The possible non-negative integral values of ‘r’ for
1 r r
⇒ x = 2 −3 , 2 6 ⇒ x = , 8 2 which and are integer, where r ≤ 60, are
8 5 10
6 r = 0, 10, 20, 30, 40, 50, 60.
 
  1  1 ∴There are 7 rational terms in the binomial expansion
 
9. Given binomial is   1 + log10 x   and remaining 61 − 7 = 54 terms are irrational terms.
 x + x12
 
12. Since, rth term from the end in the expansion of a
Since, the fourth term in the given expansion is 200. binomial (x + a )n is same as the (n − r + 2)th term from
3
 1  2 1 3 the beginning in the expansion of same binomial.
C3  x1 + log10 x   x12 = 200 T5 T T4 + 1
∴ 6
∴Required ratio = = 5 =
   
  T10 − 5 + 2 T7 T6 + 1
 1 4
3
+ 
3
+
1  1 

 2 (1 + log10 x ) 4  2(1 + log10 x ) 4
10
C 4 (21/3 )10− 4  
⇒ 20 × x = 200 ⇒ x = 10 T5  2(3)1/3 
 ⇒ =
3 1 6
⇒ 2(1 + log x) + 4  log10 x = 1
T10 − 5 + 2  1 
 
10
C 6 (21/3 )10− 6  
10  2(3)1/3 
[applying log10 both sides]
[QTr + 1 = n Cr x n − r a r ]
⇒ [6 + (1 + log10 x)] log10 x = 4(1 + log10 x) 6/3
2 (2(3) ) 1/3 6
= [Q 10C 4 = 10C 6 ]
⇒ (7 + log10 x) log10 x = 4 + 4 log10 x 24/3 (2(3)1/3 )4
⇒ t 2 + 7t = 4 + 4t [let log10 x = t] = 26/3 − 4/3 (2(3)1/3 )6 − 4
⇒ t + 3t − 4 = 0
2
= 223
/
⋅ 22 ⋅ 323
/
⇒ t = 1 , −4 = log10 x
= 4(6) = 4(36)1/3
23/

⇒ x = 10, 10−4 So, the required ratio is 4(36)1/3 : 1.


Since, x>1 x = 10 8
 x33
10. 13. In the expansion of  +  , the middle term is T4 + 1.
Key Idea Use formula : 3 x
( a + b) n + ( a − b) n =
2 [ n C 0 a n + nC 2a n − 2b 2 + nC 4 a n − 4 b 4 + ...... ] [Q Here, n = 8, which is even, therefore middle term
 n + 2
=  th term]
 2 
Given expression is (x + x3 − 1 )6 + (x − x3 − 1 )6
4
 x3   3 4 8 ⋅ 7 ⋅ 6 ⋅ 5 8
= 2 [6C 0x6 + 6C 2x4 ( x3 − 1 )2 ∴ 5670 = 8C4     = x
 3   x 1⋅ 2⋅ 3⋅ 4
+ 6C 4x2( x3 − 1 )4 + 6C 6 ( x3 − 1 )6 ]  8− r r
 x3   3 
{Q (a + b)n + (a − b)n QTr + 1 = 8Cr    
  3  x 
= 2 [nC 0a n + nC 2a n − 2b2 + nC 4a n − 4b4 + …]}  
= 2 [ C 0x + C 2x4 (x3 − 1) + 6C 4x2(x3 − 1)2 + 6C 6 (x3 − 1)3 ]
6 6 6 ⇒ x = 3 ⇒x= ± 3
8 4

The sum of the terms with even power of x So, sum of all values of x i.e + 3 and − 3 = 0
= 2 [ C 0x + C 2(− x ) + C 4x + C 4x + C 6 (−1 − 3x )]
6 6 6 4 6 8 6 2 6 6 14. The general term in the expansion of binomial
expression (a + b)n is Tr+ 1 = nC r a n− rbr, so the general
= 2 [ C 0x − C 2x + C 4x + C 4x − 1 − 3x ]
6 6 6 4 6 8 6 2 6
term in the expansion of binomial expression
10
Now, the required sum of the coefficients of even powers  λ
x2 x + 2 is
of x in  x 
Binomial Theorem 95

10 − r
  λ 
r
=
1 21
( C1 + 21C 2 + ... + 21C 20 ) − (210 − 1)
Tr+ 1 = x2  10C r ( x )10− r  2  =10C r x2 ⋅ x 2 λr x−2r 2
 x  
10 − r 1
2+ − 2r = (21C1 + 21C 2 + ... + 21C 21 − 1) − (210 − 1)
= 10C r λr x 2
2
10 − r 1
Now, for the coefficient of x2, put 2 + − 2r = 2 = (221 − 2) − (210 − 1) = 220 − 1 − 210 + 1 = 220 − 210
2 2
10 − r
⇒ − 2r = 0
2 19. Clearly, number of terms in the expansion of
⇒ 10 − r = 4r ⇒ r = 2  2 4
n
(n + 2) (n + 1)
1 − + 2 is or n + 2C 2.
So, the coefficient of x2 is 10C 2 λ2 = 720 [given]  x x  2
1 1
10 ! 2 10 ⋅ 9 ⋅ 8 ! 2 [assuming and 2 distinct]
⇒ λ = 720 ⇒ λ = 720 x x
2!8! 2⋅ 8!
(n + 2) (n + 1)
⇒ 45 λ2 = 720 ∴ = 28
2
⇒ λ2 = 16 ⇒ λ = ± 4
⇒ (n + 2) (n + 1) = 56 = (6 + 1) (6 + 2) ⇒ n = 6
∴ λ =4 [λ > 0]
Hence, sum of coefficients = (1 − 2 + 4)6 = 36 = 729
15. The (r + 1)th term in the expansion of (a + x)n is given
1 1
by Tr + 1 = nC ra n − rxr Note As and 2 are functions of same variables, therefore
x x
∴ 3rd term in the expansion of (1 + xlog 2 x )5 is number of dissimilar terms will be 2 n + 1, i.e. odd, which is not
5
C 2(1)5 − 2(xlog 2 x )2 possible. Hence, it contains error.
⇒ 5 C 2(1)5 − 2(xlog 2 x )2 = 2560 (given) 20. Let Tt +1 be the general term in the expension of
⇒ 10 (xlog 2 x )2 = 2560
(1 − 2 x )50
⇒ x( 2log 2 x ) = 256
∴ Tr+ 1 = 50C r (1)50− r (−2x1/ 2)r = 50C r2r xr/ 2(−1)r
⇒ log 2 x2log 2 x = log 2 256
(taking log 2 on both sides) For the integral power of x, r should be even integer.
25
⇒ 2(log 2 x)(log 2 x) = 8 (Q log 2 256 = log 2 28 = 8)
∴ Sum of coefficients = ∑ 50C 2r (2)2r
(log 2 x)2 = 4 r= 0
⇒ log 2 x = ± 2 1 1
⇒ log 2 x = 2 or log 2 x = − 2 = [(1 + 2)50 + (1 − 2)50 ] = (350 + 1)
2 2
1
⇒ x = 4 or x = 2−2 = Alternate Solution
4
1 − t 6 3 We have,
16. Clearly,   = (1 − t 6 )3 (1 − t )− 3 (1 − 2 x )50 = C o − C12 x + C 2( 2x)2 + ... + C50 (2 x )50 ...(i)
1−t
∴ Coefficient of t 4 in (1 − t 6 )3 (1 − t )−3 (1 + 2 x )50 = C o + C12 x + C 2(2 x )2 + ... + C50 (2 x )50...(ii)
= Coefficient of t 4 in (1 − t18 − 3t 6 + 3t12) (1 − t )− 3 On adding Eqs. (i) and (ii), we get
= Coefficient of t 4 in (1 − t )− 3 (1 − 2 x )50 + (1 + 2 x )50
= 3 + 4 − 1C 4 = 6C 4 = 15 = 2 [C 0 + C 2(2 x )2 + ... + C50 (2 x )50 ] ...(iii)
(Q coefficient of xr in (1 − x)− n = n + r − 1C r)
(1 − 2 x )50 + (1 + 2 x )50
17. ⇒
Key Idea Use formula : 2
= ( a + b) n + ( a − b) n = C 0 + C 2(2 x )2 + ... + C50 (2 x )50
= 2 ( n C 0 a n + nC 2a n − 2b 2 + nC 4 a n − 4 b 4 + ...)
On putting x = 1, we get
We have, (x + x − 1 ) + (x − x − 1 ) , x > 1
3 5 3 5 (1 − 2 1 )50 + (1 + 2 1 )50
= C 0 + C 2 + ... + C50 (2)50
2
= 2(5 C 0x5 + 5C 2x3 ( x3 − 1 )2 + 5C 4x( x3 − 1 )4 ) (−1)50 + (3)50
= 2(x5 + 10x3 (x3 − 1) + 5x(x3 − 1)2) ⇒ = C 0 + C 2(2)2 + ... + C50 (2)50
2
= 2(x5 + 10x6 − 10x3 + 5x7 − 10x4 + 5x) 1 + 350
⇒ = C 0 + C 2(2)2 + ... + C50 (2)50
Sum of coefficients of all odd degree terms is 2
2 (1 − 10 + 5 + 5) = 2 21. Coefficient of xr in (1 + x)n is nC r.
18. ( 21C1 − 10C1 ) + (21C 2 − 10C 2) + (21C3 − 10C3 ) In this type of questions, we find different composition of
terms where product will give us x11.
+ ... + ( C10 −
21 10
C10 )
Now, consider the following cases for x11 in
=( 21
C1 + 21
C 2 + ... + 21
C10 ) − (10C1 + 10
C 2 + ... + 10
C10 )
(1 + x2)4 (1 + x3 )7 (1 + x4 )12.
96 Binomial Theorem

Coefficient of x0 x3 x8; Coefficient of x2 x9 x0 26. We know that,


Coefficient of x4 x3 x4; Coefficient of x8 x3 x0 (a + b)5 + (a − b)5 = 5C 0a5 + 5C1a 4b + 5C 2a3 b2
= 4C 0 × 7C1 × 12C 2 + 4C1 × 7C3 × 12C 0 + 4C 2 × 7C1 + 5C3 a 2b3 + 5C 4ab4 + 5C5 b5 + 5C 0a5 − 5C1a 4b
× 12C1 + 4C 4 × 7C1 × 12C 0 + 5C 2a3 b2 − 5C3 a 2b3 + 5C 4ab4 − 5C5 b5
= 462 + 140 + 504 + 7 = 1113
= 2 [a5 + 10a3 b2 + 5ab4 ]
10
 x+1 ( x − 1)  ∴ [x + (x − 1) ] + [x − (x3 − 1)1/ 2]5
3 1/ 2 5
22.  − 
x
2/3
− x1/ 3 + 1 x − x1/ 2  = 2 [x5 + 10x3 (x3 − 1) + 5x (x3 − 1)2]
10
 (x1/ 3 )3 + 13 {( x )2 − 1}  Therefore, the given expression is a polynomial of
=  23 −  degree 7.
x − x + 1
/ 1/3
x ( x − 1) 
10
27. The general term in  −
10 x 3
 (x1/ 3 + 1)(x2/ 3 + 1 − x1/3 ) {( x )2 − 1}   is
= − 2 x2 


/
x23 − x1/3 + 1 x ( x − 1)  10 − r r
3r
 x  3
tr + 1 = (−1)r 10
Cr    2 = (−1)
r 10
C r. ⋅ x10 − 3 r
 ( x + 1) 
10
− 1/ 2 10  2 x  210 − r
= (x1/3 + 1) −  = (x − x )
1/3

 x  For coefficient of x4, we put 10 − 3r = 4


∴ The general term is ⇒ 3r = 6
10 − r r

Tr + 1 = 10 1/3 10 − r
C r (x ) (− x − 1/ 2 r
) = C r (− 1)
10 r
x 3 2 ⇒ r =2
10
x 3 32
For independent of x , put ∴ Coefficient of x4 in  − 2 = (−1)2. 10C 2.
2 x  28
10 − r r
− = 0 ⇒ 20 − 2r − 3r = 0 45 × 9 405
3 2 = =
256 256
⇒ 20 = 5r ⇒ r = 4
10 × 9 × 8 × 7 28. In the expansion (1 + x)2n, t3 r = 2nC3 r − 1 (x)3 r − 1
∴ T5 = 10C 4 = = 210
4 ×3 ×2 ×1 and tr + 2 = C r + 1 (x)r + 1
2n

23. Here, Coefficient of t 24


in {(1 + t ) (1 + t )(1 + t )}
2 12 12 24
Since, binomial coefficients of t3 r and tr + 2 are equal.
= Coefficient of t in {(1 + t ) ⋅ (1 + t + t + t )}
24 2 12 12 24 36
∴ 2n
C3 r −1 = 2nC r + 1
= Coefficient of t 24 in ⇒ 3r − 1 = r + 1 or 2n = (3r − 1) + (r + 1)
{(1 + t 2)12 + t12(1 + t 2)12 + t 24 (1 + t 2)12}; ⇒ 2r = 2 or 2n = 4r
[neglecting t36 (1 + t 2)12]
⇒ r =1 or n = 2r
= Coefficient of t 24 = (12C12 + C6 +
12
C0 ) = 2 +
12 12
C6 But r >1
24. Given, T5 + T6 = 0 ∴ We take, n = 2r
⇒ n
C 4a n − 4b4 − nC5 a n − 5 b5 = 0 29. To find the coefficient of x3 and x4, use the formula of
a nC5 n − 4 coefficient of x r in (1 − x)n is (−1)r nCr and then simplify.
⇒ n
C 4a n − 4b4 = nC5 a n − 5 b5 ⇒ = =
b nC 4 5 In expansion of (1 + ax + bx2)(1 − 2x)18.
m (m − 1) 2 Coefficient of x3 = Coefficient of x3 in (1 − 2x)18
25. (1 + x)m (1 − x)n = 1 + mx + x +K


 2 + Coefficient of x2 in a (1 − 2x)18
 n (n − 1) 2  + Coefficient of x in b(1 − 2x)18
1 − nx + x − ...
 2 
= 18
C3 ⋅ 23 + a 18C 2 ⋅ 22 − b 18C1 ⋅ 2
m(m − 1) n (n − 1) 
= 1 + (m − n ) x + + − mn x2 + K Given, coefficient of x3 = 0
 2 2 
⇒ C3 ⋅ 23 + a 18C 2 ⋅ 22 − b 18C1 ⋅ 2 = 0
18
term containing power of x ≥ 3. 18 × 17 × 16 18 × 17 2
Now, m − n =3 …(i) ⇒ − ⋅8 + a ⋅ ⋅ 2 − b ⋅ 18 ⋅ 2 = 0
3 ×2 2
[Q coefficient of x = 3, given] 34 × 16
1 1 ⇒ 17a − b = ..(i)
and m(m − 1) + n (n − 1) − mn = − 6 3
2 2 Similarly, coefficient of x4 = 0
⇒ m(m − 1) + n (n − 1) − 2mn = − 12 ⇒ 18
C 4 ⋅ 24 − a ⋅18 C3 23 + b ⋅18 C 2 ⋅ 22 = 0
⇒ m2 − m + n 2 − n − 2mn = − 12
∴ 32a − 3b = 240 …(ii)
⇒ (m − n )2 − (m + n ) = − 12
On solving Eqs. (i) and (ii), we get
⇒ m + n = 9 + 12 = 21 …(ii)
272
On solving Eqs. (i) and (ii), we get m = 12 a = 16, b =
3
Binomial Theorem 97

3m
30. Let the coefficients of 2nd, 3rd and 4th terms in the 3n 3(2m)
Also, k = = = 3m ∴ S = ∑ (−3)r − 1 ⋅6mC 2r − 1
expansion of (1 + x) is C1, C 2, C3 .
n n n n
2 2 r =1
According to given condition,
i.e. S = (−3)0 6m
C1 + (−3) 6m
C3 + K
2 (nC 2) = nC1 + nC3 3 m − 1 6m
n (n − 1) n (n − 1) (n − 2) + (−3) ⋅ C3m − 1 …(i)
⇒ 2 =n+
1 ⋅2 1 ⋅2 ⋅3 From the binomial expansion, we write
(n − 1) (n − 2) (1 + x)6 m = 6m
C0 + 6m
C1x + 6m
C2 x2 + K
⇒ n −1 =1 +
6m − 1
6 6m
C 6 m − 1x + 6m
C 6 m x6 m …(ii)
n 2 − 3n + 2 (1 − x) 6m
= 6m
C0 + 6m
C1 (− x) + 6m
C 2(− x) + K2
⇒ n −1 =1 +
6
+ 6m
C 6 m − 1 (− x)6 m − 1 + 6m
C 6 m (− x)6 m …(iii)
⇒ 6n − 6 = 6 + n 2 − 3n + 2
On subtracting Eq. (iii) from Eq. (ii), we get
⇒ n 2 − 9n + 14 = 0 ⇒ (n − 2) (n − 7) = 0
⇒ n =2 ⇒ n = 7 (1 + x)6 m − (1 − x)6 m = 2 [6 mC1x + 6m
C3 x3
But C3 is true for n ≥ 3, therefore n = 7 is the answer.
n + 6m
C5 x5 + K + 6 m C 6 m − 1x6 m − 1 ]
(1 + x) 6m
− (1 − x) 6m
31. Given, ⇒ = 6mC1 + 6m
C3 x2 + 6m
C5 x4 + ...
2x
(1 + ax) = 1 + 8x + 24x + ...
n 2
+ 6 mC 6 m − 1 x6 m − 2
n (n − 1) 2 2
⇒ 1 + anx + a x + ... = 1 + 8x + 24x2 + ... Let x =y 2
2!
(1 + y )6 m − (1 − y )6 m
n (n − 1) ⇒ = 6m
C1 + 6m
C3 y
∴ an = 8 and a 2 = 24 ⇒ 8 (8 − a ) = 48 2 y
2
⇒ 8 − a =6 ⇒ a =2 + 6m
C5 y2 + K + 6 m C 6 m − 1 y 3 m − 1
Hence, a =2 and n =4 For the required sum we have to put y = − 3 in RHS.
(1 + −3 )6m − (1 − −3 )6m
32. Since, n is an odd integer, (− 1)n−1 = 1 ∴ S=
2 −3
and n − 1, n − 3, n − 5, etc., are even integers, then
(1 + i 3 )6m − (1 − i 3 )6m
n3 − (n − 1)3 + (n − 2)3 − (n − 3)3 + K + (− 1)n−1 ⋅ 13 = …(iv)
2i 3
= n + (n − 1) + (n − 2) + K + 1
3 3 3 3
Let z = 1 + i 3 = r (cos θ + i sin θ )
− 2 [(n − 1)3 + (n − 3)3 + K + 23 )]
⇒ r = |z| = 1 + 3 = 2
  n − 1 3  n − 3 3 
= Σ n3 − 2 × 23   +  +K+1 
3
and θ = π /3
 2   2 
  Now, z 6 m = [r (cos θ + i sin θ )]6 m
[Q n − 1, n − 3, ... , are even integers] = r 6 m (cos 6m θ + i sin 6m θ )
  n − 1 
3
Again, z = r (cos θ − i sin θ )
= Σ n3 − 16 Σ   
 2  and (z )6 m = r 6 m (cos 6m θ − i sin 6m θ )
 
2 2 ⇒ z 6 m − z 6 m = r 6 m (2i sin 6m θ ) …(v)
 n (n + 1)  1  n − 1  n − 1 
= − 16     + 1 
 2   2  2   2  From Eq. (i),
z 6 m − z 6 m r 6 m (2i sin 6 m θ ) 26 m sin 6 m θ
1 16 (n − 1)2(n + 1)2 S= = =
= n 2 (n + 1)2 − 2i 3 2i 3 3
4 4 ×4 ×4
1 1 = 0 as m ∈ z , and θ = π /3
= (n + 1)2 [n 2 − (n − 1)2] = (n + 1)2(2n − 1)
4 4 35. Let y = (x − a )m, where m is a positive integer, r ≤ m
33. Consider, (101)50 − (99)50 − (100)50 dy d 2y
Now, = m(x − a )m − 1 ⇒ = m(m − 1) (x − a )m − 2
= (100 + 1)50 − (100 − 1)50 − (100)50 dx dx2
d3 y
= {(100)50 (1 + 0.01)50 − (1 − 0.01)50 − 1)} ⇒ = m(m − 1)(m − 2)(m − 3)(x − a )m − 4
dx3
= (100)50 {2 ⋅ [50C1 (0.01) + 50
C3 (0.01)3 + K ] − 1} …………………………………
…………………………………
= (100)50 {2 [50C3 (0.01)3 + 50
C5 (0.01)5 + ... ]}
On differentiating r times, we get
∴ (101) 50
− {(99) 50
+ (100) } > 0
50
dr y
⇒ (101)50 > (99)50 + (100)50 = m(m − 1) ... (m − r + 1)(x − a )m − r
dxr
34. Since, n is an even positive integer, we can write m!
= (x − a )m − r = r !(mC r )(x − a )m − r
n = 2 m ,m = 1, 2, 3, K (m − r )!
98 Binomial Theorem
n +1
dr y  q + 1
and for r > m, =0 1− 
dxr  2  2n + 1 − (q + 1)n + 1
2n 2n = = ...(ii)
 q + 1 2n (1 − q)
Now, ∑ a r (x − 2)r = ∑ br (x − 3)r [given] 1−
 2 

r=0 r=0

On differentiating both sides n times w.r.t. x, we get From Eqs. (i) and (ii),
2n 2n n+ 1 n+ 1 n+ 1 n+ 1
C1 + C 2 s1 + C3 s2 + ... + C n + 1sn = 2nS n
∑ a r (n !) C n (x − 2) r r−n
= ∑ br (n !) C n (x − 3)
r r−n

r=n r=n 38. The general term (i.e. (r + 1)th term) in the expansion of
On putting x = 3, we get  1
22
2n binomial  xm + 2 is
 x 
∑ a r (n !)rC n = (bn )n ! Tr + 1 = 22C r (xm )22 − r x−2r
r=n

[since, all the terms except first on RHS become zero] Q The coefficient 22C r = 1540 [given]
⇒ bn = nC n + n + 1C n + n + 2C n + K + 2nC n ⇒ r = 3 or 19
and 22m − mr − 2r = 1
[Q a r = 1, ∀ r ≥ n ]
at r = 3, 22m − 3m − 6 = 1 ⇒ 18m = 7
= (n + 2C n + 1 + n+ 2
C n ) + K + 2n C n
7
= n+3
Cn + 1 + K + 2n
Cn = .... ⇒ m= ∉N
16
2n + 1
= 2n
Cn + 1 + Cn =
2n
Cn + 1 Now, at r = 19, 22m − 19m − 38 = 1
n
1 3 7 r
15 r  r ⇒ 3m = 39 ⇒ m = 13 ∈ N
36. ∑ (−1)r nC r  r + 2r + 3 r + 4r + ... upto m terms  So, the natural value of m = 13
r=0  2 2 2 2 
n r n r 39. Coefficient of x2 in the expansion of
 1  3
= ∑ (−1) r n
Cr   +
 2 ∑ (−1) r n
Cr   +
 4
{(1 + x)2 + (1 + x)3 + K + (1 + x)49 + (1 + mx)50}
r=0 r=0 ⇒ 2C 2 + 3C 2 + 4C 2 + K + 49C 2 + 50C 2 ⋅ m2 = (3n + 1) ⋅51 C3
n r
 7 ⇒ 50
C3 + 50C 2m2 = (3n + 1) ⋅51 C3
∑ (−1)r nC r  8  + ... upto m terms [QrC r + r + 1C r + K+ nC r = n + 1C r + 1]
r=0

 1  3 
n
7
n n
50 × 49 × 48 50 × 49 51 × 50 × 49
= 1 −  + 1 −  + 1 −  + ... upto m terms ⇒ + × m2 = (3n + 1)
 2  4  8 3 ×2 ×1 2 3 ×2 ×1
 n  ⇒ m2 = 51n + 1
using ∑ (−1)r nC rxr = (1 − x)n  ∴ Minimum value of m2 for which (51n + 1) is integer
 
r= 0
(perfect square) for n = 5.
n n n
 1  1  1 ∴ m2 = 51 × 5 + 1 ⇒ m2 = 256 ⇒ m = 16 and n = 5
=   +   +   + ... upto m terms
 2  4  8 Hence, the value of n is 5.
  1 
m
40. Coefficient of x9 in the expansion of
1 −  n  
n
 = 2 −1
mn
 1 2 (1 + x)(1 + x2)(1 + x3 ) K (1 + x100 ) =Terms having x9
=  
 2 
1 − n  2 (2 − 1)
1 mn n
= [199 ⋅ x9 , 198 ⋅ x ⋅ x8 , 198 ⋅ x2 ⋅ x7 , 198 ⋅ x3 ⋅ x6 ,
 2  198 ⋅ x4 ⋅ x5 , 197 ⋅ x ⋅ x2 ⋅ x6 , 197 ⋅ x ⋅ x3 ⋅ x5 ,197 ⋅ x2 ⋅ x3 ⋅ x4]
 
n+ 1 n+ 1 n+ 1 n +1
∴ Coefficient of x9 = 8
37. C1 + C 2 s1 + C3 s2 + ... + C n+ 1sn
41. Let the three consecutive terms in (1 + x)n + 5 be
n+ 1
tr , tr + 1 , tr + 2 having coefficients
= ∑ n + 1C rsr −1, n+ 5
C r − 1 , n + 5C r , n + 5C r + 1.
r =1
Given, n + 5 C r − 1 : n + 5C r : n + 5C r + 1 = 5 : 10 : 14
1 − qn + 1
where sn = 1 + q + q2 + ... + qn = n+ 5
Cr 10
n+ 5
C r + 1 14
1−q ∴ = and n + 5 =
n+ 5
n+ 1  n +1 n+ 1  C r −1 5 Cr 10
 1 − qr  1
∴ ∑ n + 1C r  1 − q  = 1 − q  ∑ n + 1C r − ∑ n + 1C r qr n + 5 − (r − 1) n−r+5 7
r =1  r =1 r =1  ⇒ = 2 and =
r r+1 5
1
= [(1 + 1)n + 1 − (1 + q)n + 1 ]
1 −q ⇒ n − r + 6 =2r
1 and 5n − 5 r + 25 = 7r + 7
= [2n + 1 − (1 + q)n + 1 ] …(i)
1 −q ⇒ n + 6 = 3 r and 5n + 18 = 12r
n + 6 5n + 18
 q + 1  q + 1  q + 1
2 n
∴ =
Also, S n = 1 +   +  + ... + 
 2 
3 12
 2   2 
⇒ 4n + 24 = 5n + 18 ⇒ n = 6
Binomial Theorem 99

Topic 2 Properties of Binomial Coefficient ∴ When (16)100 is divided by 15, gives remainder 1100 = 1
and when 8(16)100 is divided by 15, gives remainder 8.
1. We have,
(x + 10)50 + (x − 10)50 = a 0 + a1x + a 2x2 + … + a50x50 2403  8
∴  = .
∴a 0 + a1x + a 2x2 + … + a50x50  15  15
= [(50C 0x50 + 50
C1x49 10 + 50
C 2 x48 ⋅ 102 + … + 50
C50 1050 ) (where {⋅} is the fractional part function)
+ ( C 0x − C1x 10 + C 2 x 10 − … +
50 50 50 49 50 48 2 50
C50 10 )] 50
⇒ k =8
= 2 [50C 0 x50 + 50C 2x48 ⋅ 102 + 50C 4x46 ⋅ 104
4. A r = Coefficient of xr in (1 + x)10 = 10C r
+…+ 50
C50 ⋅ 1050 ]
B r = Coefficient of xr in (1 + x)20 = 20
Cr
By comparing coefficients, we get
C r = Coefficient of x in r
(1 + x) 30
= 30
Cr
a 2 = 2 50C 48 (10)48 ; a 0 = 2 50C50 (10)50 = 2(10)50 10 10 10

a
∴ 2 =
2(50C 2)(10)48
=2
50 ⋅ 49 (10)48 ∴ ∑ Ar (B10 Br − C10 Ar ) = ∑ Ar B10 Br − ∑ Ar C10 Ar
r =1 r =1 r =1
a0 2 (10)50
1 ⋅ 2 2 ⋅ (10)50
10 10
[Q 50
C 48 = C 2]
50

50 × 49 5 × 49 245 = ∑ 10C r 20C10 20C r − ∑ 10C r 30C10 10C r


= = = = 12 .25 r =1 r =1
2 ⋅ (10 × 10) 20 20
10 10
2. We know that, = ∑ 10C10 − r 20C10 20C r − ∑ 10C10 − r 30C10 10C r
r =1 r =1
(1 + x)20 = 20
C0 + 20
C1x + 20
C 2x2 + ... +
10 10
20
C r − 1 xr − 1 + 20
C rxr + ... + 20
C 20x20 = 20
C10 ∑ 10C10 − r ⋅ 20C r − 30C10 ∑ 10C10 − r 10C r
∴ (1 + x)20 ⋅ (1 + x)20 = (20C 0 + 20
C1x + r =1 r =1

C r − 1 xr − 1 + = C10 ( 30C10 − 1) − 30C10 (20C10 − 1)


20
20
C 2x2 + ... + 20 20
C rxr + ... + 20
c20x20 )
× (20C 0 + 20
C1x + ...+ 20
C r − 1 xr − 1 + 20
C rxr = 30C10 − 20
C10 = C10 − B10
+ ....+ C 20x )20 20
30 30 30 30 30 30 30 30
5. Let A =     −     +     − ... +    
⇒ (1 + x) 40
= ( C0 .
20 20
Cr + 20
C1 20
C r − 1 ...  0  10  1  11  2  12 20 30
20
C r20C 0 ) xr + ... ∴ A = 30C 0 ⋅30 C10 − 30C1 ⋅30 C11 + 30
C 2 ⋅30 C12
−K + C 20 ⋅30 C30
30
On comparing the coefficient of xr of both sides, we get
= Coefficient of x20 in (1 + x)30 (1 − x)30
20
C 020C r + 20
C120C r − 1 + ... + 20
Cr 20
C0 = 40
Cr
= Coefficient of x20 in (1 − x2)30
The maximum value of 40
C r is possible only when r = 20 30

∑ (−1)r
30
n = Coefficient of x20 in C r (x2)r
[Q C n/2 is maximum when n is even]
r=0
Thus, required value of r is 20.
= (−1)10 30
C10 [for coefficient of x20, put r = 10]
3. Consider,
= 30C10
2403 = 2400 + 3 = 8 ⋅ 2400 = 8 ⋅ (24 )100 = 8 (16)100= 8(1 + 15)100 n −1
6. Given , C r = (k2 − 3) nC r + 1
= 8 (1 + 100
C1 (15) + 100
C 2(15)2 + … + 100
C100 (15)100 )
n −1 n n −1 r+1
[By binomial theorem, ⇒ C r = (k2 − 3) C r ⇒ k2 − 3 =
r+1 n
(1 + x)n = nC 0 + nC1x + nC 2x2 + … nC nxn , n ∈ N ] r+1
[since, n ≥ r ⇒
≤ 1 and n , r > 0]
= 8 + 8 (100C1 (15) + 100
C 2(15)2 + … + 100
C100 (15)100 ) n
= 8 + 8 × 15λ ⇒ 0 < k2 − 3 ≤ 1 ⇒ 3 < k2 ≤ 4
where λ =C1 +......+ C100 (15) ∈ N
100 100 99
⇒ k ∈ [−2, − 3 ) ∪ ( 3 , 2]
2403 8 + 8 × 15λ 8 2403  8
∴ = = 8λ + ⇒  = m
10  20 
15 15 15  15  15 7. ∑  i  m − i is the coefficient of xm in the expansion of
i=0
(where {⋅} is the fractional part function)
(1 + x)10 (x + 1)20,
∴ k =8 m
10  20 
Alternate Method ⇒ ∑  
m
 is the coefficient of x in the
 i  m − i
i=0
2403 = 8 ⋅ 2400 = 8(16)100
expansion of (1 + x)30
Note that, when 16 is divided by 15, gives remainder 1.
100 Binomial Theorem
m
10  20  30 30 (n + p)!
p
i.e. ∑    = Cm =  
 i  m − i  m
…(i) = ∑ mCi × nC p − i =
n ! p! i = 0
n+ p
Cp m+ n
Cp
i=0

 n m+ n
f (m, n , p)
and we know that,   is maximum, when
 r
Since, g (m, n ) = ∑  n + p
p=0
 n  
 p 
 n  r=2, if n ∈ even.
  = m+ n n+ p
C p × m + nC p m+ n
 r  max  n±1
 r = 2 , if n ∈ odd
. = ∑ n+ p
Cp
= ∑ m+ n
C p = 2m + n
p=0 p=0
30
Hence,   is maximum when m = 15. ∴ g (m, n ) = g (n , m) = 2 , and m+ n
 m
g (m, n + 1) = 2m + n + 1 = g (m + 1, n ), and
 n  n   n   n  n  
8.   + 2  + =  +  g (2m; 2n ) = 22(m + n) = (2(m + n) )2 = ( g (m, n ))2.
 r  r − 1  r − 2  r   r − 1 
 n   n    n + 1  n + 1  n + 2 12. It is given that
+  + =  +  = 
 r − 1  r − 2   r   r − 1   r 
n n
Σ k Σ n
C k k2
n+1 k=0 k=0
[Q C r + C r − 1 =
n n
Cr ] n n
=0
n
r
n
n − (n − r ) Σ n
Ck . k Σ n
C k3k
9. Let b = ∑ n
Cr
= ∑ n
Cr
k=0 k=0

r=0 r=0 n (n + 1)
n (n + 1)2 n − 2
n
1 n−r
n
⇒ 2 =0
=n ∑ n
− ∑
C r r = 0 nC r n.2 n − 1 4n
r=0
 n n (n + 1) n 
Σ nC k k = n.2 n − 1 ,
n
n−r
= na n − ∑ n
[Q nC r = nC n − r ]  Q k Σ= 0 k = 2
,
k=0
r=0 Cn − r  n n 
 Σ nC k k2 = n (n + 1)2n − 2 and Σ nC k3k = 4n 
n  = = 
= na n − b ⇒ 2b = na n ⇒ b = an k 0 k 0
2 n (n + 1) n 2n − 3
10. We have, ⇒ 4 − n (n + 1) 2
2
=0
2
C 02 − 2C12 + 3C 22 − 4C32 + ... + (−1)n (n + 1) C n2
4n 4n−1
= [C 02 − C12 + C 22 − C32 + ... + (−1)nC n2 ] ⇒ −n =0⇒n =4
2 2
− [ C12 − 2 C 22 + 3 C32 − ... + (−1)n nC n2 ] n n
Ck 4 4
Ck 1 4 5 1
n!
n
−1 n n! ∴Σ = Σ = Σ C k + 1 = (25 − 1)
= (−1)n/ 2 − (−1) 2 k=0 k + 1 k=0 k + 1 5 k=0 5
 n  n 2  n  n
 !  !   !  ! 1 31
 2  2  2  2 = (32 − 1) = = 6.20
5 5
n!  n
= (−1)n/ 2 1 +  13. We have,
 n  n  2
 !  ! X = (10C1 )2 + 2(10C 2)2+ 3(10C3 )2 + ... + 10 (10C10 )2
 2  2
10 10
 n  n
2 !  !
 2  2
⇒ X= ∑ r (10C r )2 ⇒ X = ∑ r 10C r 10C r
∴ [C 02 − 2 C12 + 3 C 22 − ... + (− 1)r (n + 1) C n2 ] r =1 r =1
n! 10
 n  n 10 9  n n 
2  !  !
 2  2 n! (n + 2)
⇒ X= ∑r× r
Cr − 1 10
Cr

Q Cr =
r
n −1
Cr − 1

r =1
= (−1)n/ 2 = (−1)n/ 2(n + 2)
n!  n  n 2
 !  ! 10
 2  2 ⇒ X = 10 ∑ 9C r − 1 10C r
r =1
11. Since,
10
m  n + i  p + n
p
f (m, n , p) = ∑  i   p   p − i  ⇒ X = 10 ∑ 9C r − 1 10C10 − r [Q nC r = nC n − r ]
i=0 r =1
n−1 2n − 1
p
m! (n + i )! ( p + n )! ⇒ X = 10 × 19
C9 [Q C r − 1nC n − r = Cn − 1 ]
= ∑ i !(m − i )! × p !(n + i − p)! × ( p − i )! (n + i )! 1 10 × C 9 C9 19
C9 19 19
i=0
Now, X= = =
(n + p)!
p
1 1430 1430 143 11 × 13
= ∑ mCi ( p − i )! (n + i − p)!
p! i = 0 =
19 × 17 × 16
= 19 × 34 = 646
8
Binomial Theorem 101

14. Sum of coefficients is obtained by putting x = 1  n   n − 1  n − 2 m 


=  +  +  + ... +   
m  m   m  m
i.e. (1 + 1 − 3)2163 = − 1 
 n − 1  n − 2 m 
Thus, sum of the coefficients of the polynomial + +  + ... +   
(1 + x − 3x2)2163 is −1.  m   m  m

 n − 2 m  n − m + 1 rows
15. To show that +  + ... +  
 m  m
2k.n C 0.n C k − 2k − 1.n C1.n − 1 C k − 1 
......
+ 2k − 2.n C 2.n − 2 C k − 2 − K + (−1)k nC kn − kC 0 = nC k m 
+  
Taking LHS m 
2k.n C 0.n C k − 2k −1.n C1 ⋅n − 1 C k − 1 + K + (−1)k.n C k.n − k C 0  n + 1
k
Now, sum of the first row is  .
= ∑ (−1) r. k − r . n
2 C r.n − r C k − r m + 1
r=0
 n 
k
n! (n − r )! Sum of the second row is  .
= ∑ (−1) r
2 k − r. .
r !(n − r )! (k − r )!(n − k)!
m + 1
r=0
 n − 1
k
n! k! Sum of the third row is  ,
m + 1
= ∑ (−1) r. k − r.
2 ⋅
(n − k)!. k ! r !(k − r )! …………………………
r=0
k m m + 1
Sum of the last row is   =  .
= ∑ (−1)r. 2k − r nC k.kC r m m + 1
r=0

 k   n + 1  n   n − 1 
1 Thus, S= + + 
= 2k.n C k  ∑ (−1)r. r .k C r  m + 1 m + 1 m + 1
r = 0 2 
m + 1  n + 1 + 1  n + 2
 1
k +K+ =   = 
=2 k.n
C k 1 −  m + 1  m + 2  m + 2
 2
[from Eq. (i) replacing n by n + 1 and m by m + 1]
= n C k = RHS
n n
Cr
 n
16. Let S =   + 
 n − 1  n − 2
+
m  n + 1
 + ... +   =   …(i)
17. ∑ (−1)r r +3
Cr
m  m   m  m m + 1 r=0
n
It is obvious that, n ≥ m. n !⋅ 3 ! n
n!
[given] = ∑ (−1)r = 3 ! ∑ (−1)r
NOTE This question is based upon additive loop. r=0
(n − r ) ! (r + 3) ! r=0
(n − r ) ! (r + 3) !
m m + 1 m + 2  n 3! n
(−1)r. (n + 3)!
Now, S =   +  +  + ... +  
m  m   m  m =
(n + 1)(n + 2)(n + 3)
∑ (n − r )!(r + 3)!
r=0
m + 1 m + 1  m + 2  n
=  + +   + K  3! n

  m + 1  m    m  m = ⋅ ∑ (−1)r ⋅n + 3C r + 3
(n + 1)(n + 2)(n + 3) r = 0
 m m + 1 
Q m = 1 = m + 1  3 ! (− 1)3
n+3
  =
(n + 1)(n + 2)(n + 3)
∑ (−1)s ⋅n + 3 C3
m + 2 m + 2  n s =3
= +  + ... +  
m + 1  m  m n + 3 
−3!  ∑ (−1)s ⋅n + 3 C s
n+1 =
[Q nC r + nC r + 1 = Cr + 1 ] (n + 1)(n + 2)(n + 3)  s =` 0 

m + 3  n
=  + ... +   n+3 n+3
m + 1 m − C0 +C1 − n + 3C 2
= ............................... −3!  (n + 3)(n + 2) 
= 0 − 1 + (n + 3) − 
(n + 1)(n + 2)(n + 3)  2! 
 n   n   n + 1
=  +   =  , which is true. …(ii) −3! (n + 2)(2 − n − 3) 3!
m + 1 m m + 1 = ⋅ =
(n + 1)(n + 2)(n + 3) 2 2(n + 3)
Again, we have to prove that
 n  n − 1  n − 2 m  n + 2 18. (1 + x + x2)n = a 0 + a1x + K + a 2nx2n …(i)
  +2  +3  + ... + (n − m + 1)   =  
m  m   m  m m + 2 Replacing x by −1 / x, we get
n
 n  n − 1  n − 2 m  1 1 a a a a 2n
Let S1 =   + 2   +3  + ... + (n − m + 1)   1 − + 2 = a 0 − 1 + 22 − 33 + K + 2n …(ii)
m  m   m  m  x x  x x x x
102 Binomial Theorem

Now, a 02 − a12 + a 22 − a32 + K + a 22n = coefficient of the = 2n 2n − (2 nC n ) = 22 n − 2 nC n


term independent of x in 22 n − 2 nC n (2n )!
[a 0 + a1x + a 2x + K + a 2nx ]
2 2n
∴ ∑∑ CiC j =
2
= 22 n−1 −
2 (n !)2
0 ≤i< j≤n
 a a a 2n 
× a 0 − 1 + 22 − K + 2n  21. We know that, (1 + x)2 n = C 0 + C1x + C 2 x2 + ... + C 2 n x 2 n
 x x x 
On differentiating both sides w.r.t. x, we get
= Coefficient of the term independent of x in 2n (1 + x)2 n−1 = C1 + 2 ⋅ C 2 x + 3 ⋅ C3 x2
n
 1 1
(1 + x + x2)n 1 − + 2 + ... + 2nC 2n x2 n−1 …(i)
 x x   1
2n
1 1 1
and 1 −  = C 0 − C1 ⋅ + C 2 ⋅ 2 − C3 ⋅ 3
 1 1 
n
 x  x x x
Now, RHS = (1 + x + x2)n 1 − + 2
 x x  1
+ ... + C 2n ⋅ 2 n …(ii)
(1 + x + x2)n (x2 − x + 1)n [(x2 + 1)2 − x2]n x
= =
x2 n x2 n
(1 + 2x + x − x )
2 4 2 n
(1 + x + x )
2 4 n
On multiplying Eqs. (i) and (ii), we get
= 2n
=
x x2n  1
2n

Thus, a 0 − a1 + a 2 − a3 + K+ a 2n
2 2 2 2 2 2n (1 + x)2 n−1 1 − 
 x
= Coefficient of the term independent of x in
= [C1 + 2 ⋅ C 2x + 3 ⋅ C3 x2 + ... + 2n ⋅ C 2n x2n−1 ]
1
(1 + x2 + x4 )n   1  1  1 
x2n × C 0 − C1   + C 2 2 − ..... + C 2 n  2 n  
  x  x  x 
= Coefficient of x2n in (1 + x2 + x4 )n
 1
= Coefficient of t n in (1 + t + t 2)n = a n Coefficient of   on the LHS
 x
19. C 0 − 22 ⋅ C1 + 32 ⋅ C 2 − ... + (−1)n (n + 1)2 ⋅ C n
1  1 
n n = Coefficient of in 2n  2 n  (1 + x)2 n−1 (x − 1)2 n
x 
= ∑ (−1) r(r + 1)2 nC r = ∑ (−1)r (r 2 + 2r + 1) nC r x
r=0 r=0 = Coefficient of x2 n−1 in 2n (1 − x2)2 n−1 (1 − x)
n n n
= ∑ (−1)r r 2 ⋅ nC r + 2 ∑ (−1)r r ⋅ nC r + ∑ (−1)r. nC r = 2n (−1)n−1 ⋅ (2n − 1) C n−1 (−1)
r=0 r=0 r=0 (2n − 1)! (2n )!
= (−1)n (2n ) = (−1)n n ⋅n
n n
(n − 1)! n ! (n !)2
= ∑ (−1) r.
r (r − 1) ⋅ C r + 3 ⋅ ∑ (−1) r ⋅ nC r
n r.

r=0 r=0
n
= − (−1)n n ⋅ C n …(iii)
+ ∑ (−1) r n
Cr  1
Again, the coefficient of   on the RHS
r=0  x
n n
= − (C12 − 2 ⋅ C 22 + 3 ⋅ C32 − ... − 2n C 22n )
= ∑ (−1)r n (n − 1) n − 2C r − 2 + 3 ∑ (−1)r n . n − 1C r − 1 …(iv)
r=2 r =1 From Eqs. (iii) and (iv),
n
+ ∑ (−1)r nC r C12 − 2 ⋅ C 22 + 3 ⋅ C32 − ... − 2n ⋅ C 22n = (−1)n n ⋅ C n
r=0 2n
22. (1 + x)2 n 1 − 
1
= n (n − 1){ n − 2C 0 − n − 2C1 + n − 2C 2−... + (−1)n n − 2C n − 2}  x
+ 3n { − n−1C 0 + n − 1C1 − n −1C 2 + ...+ (−1)n n − 1C n − 1}
= [ C 0 + (2nC1 )x + (2nC 2)x2 + ...+ (2 nC 2 n )x2 n ]
2n
+ { nC 0 − nC1 + nC 2 + ... + (−1)n nC n }
 1 1 1 
× 2 nC 0 − (2 nC1 ) + (2 nC 2) 2 + ... + (2 nC 2n ) 2n
= n (n − 1) . 0 + 3n . 0 + 0, ∀n > 2 = 0, ∀n > 2  x x x 
20. We know that, Independent terms of x on RHS
n n n n
2 ∑ ∑ Ci C j = ∑ ∑ Ci C j − ∑ ∑ CiC j = (2nC 0 )2 − (2nC1 )2 + (2nC 2)2 − ...+ (2nC 2n )2
0 ≤i < j ≤ n i=0 j=0 i = 0 j=0 2n
 x − 1 1
LHS = (1 + x)2n   = (1 − x2)2n
n n n  x  x2n
= ∑ Ci ∑ C j − ∑ Ci2 Independent term of x on the LHS = (−1)n ⋅2n C n.
i=0 j=0 i=0
6
Probability
Topic 1 Classical Probability
Objective Questions I (Only one correct option) 7. If 12 identical balls are to be placed in 3 different boxes,
then the probability that one of the boxes contains excatly
1. A person throws two fair dice. He wins
3 balls, is (2015 Main)
` 15 for throwing a doublet (same numbers on the two 11 10 12 11
55  2 
(b) 55   (c) 220   (d) 22  
dice), wins ` 12 when the throw results in the sum of 2 1 1
(a)  
9, and loses ` 6 for any other outcome on the throw. 3  3  3  3  3
Then, the expected gain/loss (in `) of the person is 8. Three boys and two girls stand in a queue. The probability
(2019 Main, 12 April II)
that the number of boys ahead of every girl is atleast one
1 1 1
(a) gain (b) loss (c) loss (d) 2 gain more that the number of girls ahead of her, is (2014 Adv.)
2 4 2
(a) 1 /2 (b) 1 /3 (c) 2 /3 (d) 3 /4
2. In a random experiment, a fair die is rolled until two
fours are obtained in succession. The probability that
9. Four fair dice D1 , D2, D3 and D4 each having six faces
the experiment will end in the fifth throw of the die is numbered 1, 2, 3, 4, 5 and 6 are rolled simultaneously. The
equal to (2019 Main, 12 Jan I) probability that D4 shows a number appearing on one of
175 225 D1 , D2 and D3 , is (2012)
(a) (b)
65 65 91 108 125 127
(a) (b) (c) (d)
200 150 216 216 216 216
(c) (d)
65 65 10. Let ω be a complex cube root of unity with ω ≠ 1. A fair die is
thrown three times. If r1, r2 and r3 are the numbers obtained
3. If there of the six vertices of a regular hexagon are
on the die, then the probability that ω r 1 + ω r2 + ω r3 = 0, is
chosen at random, then the probability that the
(2010)
triangle formed with these chosen vertices is
(a) 1/18 (b) 1/9 (c) 2/9 (d) 1/36
equilateral is
(2019 Main, 12 April I) 11. If three distinct numbers are chosen randomly from the
1 1 3 3 first 100 natural numbers, then the probability that all
(a) (b) (c) (d)
10 5 10 20 three of them are divisible by both 2 and 3, is (2004, 1M)
4 4 4 4
4. Let S = {1, 2, K , 20}. A subset B of S is said to be (a) (b) (c) (d)
55 35 33 1155
“nice”, if the sum of the elements of B is 203. Then,
the probability that a randomly chosen subset of S is 12. Two numbers are selected randomly from the set
‘‘nice’’, is (2019 Main, 11 Jan II) S = {1, 2, 3, 4, 5, 6} without replacement one by one. The
6 4 7 5 probability that minimum of the two numbers is less than
(a) (b) (c) (d)
220 220 220 220 4, is (2003, 1M)
(a) 1/15 (b) 14/15 (c) 1/5 (d) 4/5
5. If two different numbers are taken from the set {0, 1,
2, 3, …, 10}, then the probability that their sum as 13. If the integers m and n are chosen at random between 1
well as absolute difference are both multiple of 4, is and 100, then the probability that a number of the form
(2017 Main) 7m + 7n is divisible by 5, equals (1999, 2M)
6 12 14 7 1 1 1 1
(a) (b) (c) (d) (a) (b) (c) (d)
55 55 45 55 4 7 8 49
6. Three randomly chosen non-negative integers x, y 14. Seven white balls and three black balls are randomly
and z are found to satisfy the equation x + y + z = 10. placed in a row. The probability that no two black balls are
Then the probability that z is even, is (2017 Adv.) placed adjacently, equals (1998, 2M)
1 36 6 5 1 7 2 1
(a) (b) (c) (d) (a) (b) (c) (d)
2 55 11 11 2 15 15 3
104 Probability

15. Three of the six vertices of a regular hexagon are chosen Fill in the Blanks
at rondom. The probability that the triangle with three
vertices is equilateral, equals (1995, 2M) 21. Three faces of a fair die are yellow, two faces red and
one face blue. The die is tossed three times. The
(a) 1/2 (b) 1/5 (c) 1/10 (d) 1/20
probability that the colours, yellow, red and blue,
16. Three identical dice are rolled. The probability that the appear in the first, second and the third tosses
same number will appear on each of them, is (1984, 2M) respectively, is…… . (1992, 2M)
1 1 1 3 1 + 3p 1 − p 1 − 2p
(a) (b) (c) (d)
6 36 18 28 22. If , and are the probabilities of three
3 4 2
17. Fifteen coupons are numbered 1, 2, ..., 15, respectively. mutually exclusive events, then the set of all values of p
Seven coupons are selected at random one at a time is… . (1986, 2M)
with replacement. The probability that the largest 23. A determinant is chosen at random from the set of all
number appearing on a selected coupon is 9, is determinants of order 2 with elements 0 or 1 only. The
6 7 7
(a)   (b)   (c)  
9 8 3 probability that the value of the determinant chosen is
(d) None of these
 16   15   5 positive, is… . (1982, 2M)

Assertion and Reason True/False


For the following questions, choose the correct answer 24. If the letters of the word ‘ASSASSIN’ are written down
from the codes (a), (b), (c) and (d) defined as follows. at random in a row, the probability that no two S’s occur
(a) Statement I is true, Statement II is also true; together is 1/35.
Statement II is the correct explanation of Statement I
(b) Statement I is true, Statement II is also true; Analytical and Descriptive Questions
Statement II is not the correct explanation of 25. An unbiased die, with faces numbered 1, 2, 3, 4, 5 and 6
Statement I is thrown n times and the list of n numbers showing up
(c) Statement I is true; Statement II is false is noted. What is the probability that among the
(d) Statement I is false; Statement II is true numbers 1, 2, 3, 4, 5 and 6 only three numbers appear in
18. Consider the system of equations this list? (2001, 5M)
ax + by = 0, cx + dy = 0, 26. If p and q are chosen randomly from the set {1, 2, 3, 4, 5,
where a , b, c, d ∈ {0, 1}. 6, 7, 8, 9 and 10} with replacement, determine the
Statement I The probability that the system of probability that the roots of the equation x2 + px + q = 0
equations has a unique solution, is 3/8. are real. (1997, 5M)
Statement II The probability that the system of 27. In how many ways three girls and nine boys can be
equations has a solution, is 1. (2008, 3M) seated in two vans, each having numbered seats, 3 in
the front and 4 at the back? How many seating
Passage Based Problems arrangements are possible if 3 girls should sit together
in a back row on adjacent seats? Now, if all the seating
Passage arrangements are equally likely, what is the probability
Box I contains three cards bearing numbers 1, 2, 3 ; box II of 3 girls sitting together in a back row on adjacent
contains five cards bearing numbers 1, 2, 3, 4, 5; and box III seats? (1996, 5M)
contains seven cards bearing numbers 1, 2, 3, 4, 5, 6, 7. A card
28. A box contains 2 fifty paise coins, 5 twenty five paise
is drawn from each of the boxes. Let xi be the number on the
coins and a certain fixed number n (≥ 2) of ten and five
card drawn from the i th box i = 1, 2, 3. (2014 Adv.)
paise coins. Five coins are taken out of the box at
19. The probability that x1 + x2 + x3 is odd, is random. Find the probability that the total value of
29 53 57 1 these 5 coins is less than one rupee and fifty paise.
(a) (b) (c) (d)
105 105 105 2 (1988, 3M)

20. The probability that x1 , x2 and x3 are in an arithmetic 29. Six boys and six girls sit in a row at random. Find the
probability that
progression, is
9 10 11 7 (i) the six girls sit together.
(a) (b) (c) (d)
105 105 105 105 (ii) the boys and girls sit alternatively. (1978, 3M)
Probability 105

Topic 2 Addition and Subtraction Law of Probability


Objective Questions I (Only one correct option) 8. Two events A and B have probabilities 0.25 and 0.50,
respectively. The probability that both A and B occur
1. The probabilities of three events A , B and C are given
simultaneously is 0.14. Then, the probability that neither
by P ( A ) = 0.6, P (B) = 0.4 and P (C ) = 0.5. If A nor B occurs, is (1980, 1M)
P ( A ∪ B) = 0.8, P ( A ∩ C ) = 0.3, P ( A ∩ B ∩ C ) = 0.2, (a) 0.39 (b) 0.25
P (B ∩ C ) = β and P(A ∪ B ∪ C ) = α , where
(c) 0.11 (d) None of these
0.85 ≤ α ≤ 0.95, then β lies in the interval
(2020 Main, 6 Sep II) Objective Questions II
(a) [ 0.35, 0.36] (b) [0.25, 0.35]
(c) [0.20, 0.25] (d) [0.36, 0.40]
(One or more than one correct option)
2. For three events A , B and C, if P (exactly one of A or B 9. For two given events A and B, P ( A ∩ B) is (1988, 2M)

occurs) = P(exactly one of B or C occurs) = P (exactly (a) not less than P (A ) + P (B ) − 1


1 (b) not greater than P (A ) + P (B )
one of C or A occurs) = and P (all the three events (c) equal to P (A ) + P (B ) − P (A ∪ B )
4
1 (d) equal to P (A ) + P (B ) + P (A ∪ B )
occur simultaneously) = , then the probability that
16 10. If M and N are any two events, then the probability that
atleast one of the events occurs, is exactly one of them occurs is (1984, 3M)
(2017 Main) (a) P (M ) + P (N ) − 2 P (M ∩ N )
7 7 7 3
(a) (b) (c) (d) (b) P (M ) + P (N ) − P (M ∪ N )
32 16 64 16
3 1 1 (c) P (M ) + P (N ) − 2 P (M ∩ N )
3. If P (B) = , P ( A ∩ B ∩ C ) = and P ( A ∩ B ∩ C ) = , (d) P (M ∩ N ) − P (M ∩ N )
4 3 3
then P (B ∩ C ) is equal to (2002, 3M)
1 1 1 1 Fill in the Blanks
(a) (b) (c) (d)
12 6 15 9 11. Three numbers are chosen at random without
4. If E and F are events with P (E ) ≤ P (F ) and replacement from {1, 2,…, 10}. The probability that the
minimum of the chosen number is 3, or their maximum is
P (E ∩ F ) > 0, then which one is not correct? (1998, 2M) 7, is … . (1997C, 2M)
(a) occurrence of E ⇒ occurrence of F
(b) occurrence of F ⇒ occurrence of E
12. P ( A ∪ B) = P ( A ∩ B) if and only if the relation between
(c) non-occurrence of E ⇒ non-occurrence of F P ( A ) and P (B) is… . (1985, 2M)

(d) None of the above True/False


5. For the three events A, B and C, P(exactly one of the 13. If the probability for A to fail in an examination is 0.2 and
events A or B occurs) = P(exactly one of the events B that of B is 0.3, then the probability that either A or B fails
or C occurs) = P(exactly one of the events C or A is 0.5. (1989, 1M)
occurs) = p and P(all the three events occurs
1
simultaneously) = p2, where 0 < p < . Then, the Analytical and Descriptive Questions
2
probability of atleast one of the three events A, B and 14. In a certain city only two newspapers A and B are
C occurring is (1996, 2M) published, it is known that 25% of the city population
3p + 2p 2
p + 3p 2 reads A and 20% reads B, while 8% reads both A and B. It
(a) (b) is also known that 30% of those who read A but not B look
2 4
into advertisements and 40% of those who read B but not
p + 3p2 3p + 2p2
(c) (d) A look into advertisements while 50% of those who read
2 4 both A and B look into advertisements. What is the
6. If 0 < P ( A ) < 1, 0 < P (B) < 1 and P ( A ∪ B) = P ( A ) percentage of the population reads an advertisement?
+ P (B) − P ( A ) P (B), then (1995, 2M) (1984, 4M)

(a) P (B / A ) = P (B ) − P (A ) 15. A, B, C are events such that


(b) P (A ′ − B ′ ) = P (A ′ ) − P (B ′ ) Pr ( A ) = 0.3, Pr (B) = 0.4, Pr (C ) = 0.8,
(c) P (A ∪ B ) ′ = P (A ) ′ P (B )′ Pr ( AB) = 0.08, Pr ( AC ) = 0.28 and Pr ( ABC ) = 0.09
(d) P (A / B ) = P (A ) − P (B ) If Pr ( A ∪ B ∪ C ) ≥ 0.75, then show that Pr (BC ) lies in the
7. The probability that at least one of the events A and B interval [ 0.23, 0.48 ]. (1983, 2M)
occurs is 0.6. If A and B occur simultaneously with 16. A and B are two candidates seeking admission in IIT. The
probability 0.2, then P ( A ) + P (B ) is equal to probability that A is selected is 0.5 and the probability
(1987, 2M)
that both A and B are selected is atmost 0.3. Is it possible
(a) 0.4 (b) 0.8 (c) 1.2 (d) 1.4 that the probability of B getting selected is 0.9? (1982, 2M)
106 Probability

Pragraph Based Questions NONE of the remaining students gets the seat
previously allotted to him/her is
There are five students S1 , S 2, S3 , S 4 and S5 in a music class 3 1 7 1
(a) (b) (c) (d)
and for them there are five seats R1 , R2, R3 , R4and R5 40 8 40 5
arranged in a row, where initially the seat Ri is allotted to the
student Si , i = 1, 2, 3, 4, 5. But, on the examination day, the
18. For i = 1, 2, 3, 4, let Ti denote the event that the students
five students are randomly allotted the five seats. Si and Si+1 do NOT sit adjacent to each other on the day
(There are two questions based on Paragraph, the question of the examination. Then, the probability of the event
given below is one of them) (2018 Adv.) T1 ∩ T2 ∩ T3 ∩ T4 is
1 1 7 1
17. The probability that, on the examination day, the (a) (b) (c) (d)
student S1 gets the previously allotted seat R1, and 15 10 60 5

Topic 3 Independent and Conditional Probability


Objective Questions I (Only one correct option) (a) E1 and E2 are independent
1. Assume that each born child is equally likely to be a boy (b) E2 and E3 are independent
or a girl. If two families have two children each, then the (c) E1 and E3 are independent
conditional probability that all children are girls given (d) E1 , E2 and E3 are independent
that at least two are girls; is (2019 Main, 10 April I) 1
1 1 1 1 7. Let A and B be two events such that P ( A ∪ B) = ,
(a) (b) (c) (d) 6
17 12 10 11 1 1
P ( A ∩ B) =and P ( A ) = , where A stands for the
2. Four persons can hit a target correctly with 4 4
1 1 1 1 complement of the event A. Then , the events A and B
probabilities , , and respectively. If all hit at the
2 3 4 8 are (2014 Main)
target independently, then the probability that the (a) independent but not equally likely
target would be hit, is (2019 Main, 9 April I) (b) independent and equally likely
1 25 7 25
(a) (b) (c) (d) (c) mutually exclusive and independent
192 32 32 192
(d) equally likely but not independent
3. Let A and B be two non-null events such that A ⊂ B. 8. Four persons independently solve a certain problem
Then, which of the following statements is always 1 3 1 1
correct. (2019 Main, 8 April I) correctly with probabilities , , , . Then, the
2 4 4 8
(a) P (A /B ) = P (B ) − P (A ) (b) P (A/B ) ≥ P (A ) probability that the problem is solved correctly by
(c) P (A/B ) ≤ P (A ) (d) P (A/B ) = 1 atleast one of them, is (2013 Adv.)
4. Two integers are selected at random from the set { 1, 2, 235 21
(a) (b)
…… , 11}. Given that the sum of selected numbers is 256 256
even, the conditional probability that both the numbers 3 253
(c) (d)
are even is (2019 Main, 11 Jan I) 256 256
2 1 7 3 9. An experiment has 10 equally likely outcomes. Let A
(a) (b) (c) (d)
5 2 10 5 and B be two non-empty events of the experiment. If A
5. An unbiased coin is tossed. If the outcome is a head, consists of 4 outcomes, then the number of outcomes
then a pair of unbiased dice is rolled and the sum of the that B must have, so that A and B are independent, is
numbers obtained on them is noted. If the toss of the (a) 2, 4 or 8 (b) 3, 6 or 9 (2008, 3M)
coin results in tail, then a card from a well-shuffled (c) 4 or 8 (d) 5 or 10
pack of nine cards numbered 1, 2, 3, …, 9 is randomly 10. Let E c denotes the complement of an event E. If E, F, G
picked and the number on the card is noted. The are pairwise independent events with P (G ) > 0 and
probability that the noted number is either 7 or 8 is P (E ∩ F ∩ G ) = 0 . Then, P (E c ∩ F c|G ) equals(2007, 3M)
(2019 Main, 10 Jan I)
(a) P (E c ) + P (F c ) (b) P (E c ) − P (F c )
15 13 19 19
(a) (b) (c) (d) (c) P (E c ) − P (F ) (d) P (E ) − P (F c )
72 36 72 36
11. One Indian and four American men and their wives are
6. Let two fair six-faced dice A and B be thrown to be seated randomly around a circular table. Then, the
simultaneously. If E1 is the event that die A shows up conditional probability that Indian man is seated
four, E 2 is the event that die B shows up two and E3 is adjacent to his wife given that each American man is
the event that the sum of numbers on both dice is odd, seated adjacent to his wife, is (2007, 3M)
then which of the following statements is not true? 1 1 2 1
(2016 Main) (a) (b) (c) (d)
2 3 5 5
Probability 107

12. A fair die is rolled. The probability that the first time Objective Questions II
1 occurs at the even throw, is (2005, 1M)
(a) 1/6 (b) 5/11 (c) 6/11 (d) 5/36
(One or more than one correct option)
1 1
13. There are four machines and it is known that exactly 21. Let X andY be two events such that P (X ) = , P (X /Y ) =
two of them are faulty. They are tested, one by one, in 3 2
2
a random order till both the faulty machines are and P (Y /X ) = . Then
identified. Then, the probability that only two tests 5 (2017 Adv.)
4 1
are needed, is (1998, 2M) (a) P (Y ) = (b) P (X ′/Y ) =
1 1 1 1 15 2
(a) (b) (c) (d) 2 1
3 6 2 4 (c) P (X ∪Y ) = (d) P (X ∩ Y ) =
5 5
14. A fair coin is tossed repeatedly. If tail appears on first
four tosses, then the probability of head appearing on 22. If X and Y are two events such that
1 1 1
fifth toss equals (1998, 2M) P (X / Y ) = , P (Y /X ) = and P (X ∩ Y ) . Then, which of
1 1 31 1 2 3 6
(a) (b) (c) (d) the following is/are correct? (2012)
2 32 32 5
(a) P (X ∪ Y ) = 2/3
15. If from each of the three boxes containing 3 white and (b) X and Y are independent
1 black, 2 white and 2 black, 1 white and 3 black balls,
(c) X and Y are not independent
one ball is drawn at random, then the probability that
(d) P (X c ∩ Y ) = 1/3
2 white and 1 black balls will be drawn, is
(1998, 2M) 23. Let E and F be two independent events. The probability
13 1 1 3 11
(a) (b) (c) (d) that exactly one of them occurs is and the probability of
32 4 32 16 25
2
16. The probability of India winning a test match against none of them occurring is . If P (T ) denotes the
25
West Indies is 1/2. Assuming independence from
probability of occurrence of the event T, then (2011)
match to match the probability that in a 5 match 4 3 1 2
series India’s second win occurs at third test, is (a) P (E ) = , P (F ) = (b) P (E ) = , P (F ) =
(1995, 2M)
5 5 5 5
2 1 3 4
(a) 1/8 (b) 1/4 (c) 1/2 (d) 2/3 (c) P (E ) = , P (F ) = (d) P (E ) = , P (F ) =
5 5 5 5
17. An unbiased die with faces marked 1, 2, 3, 4, 5 and 6 is
24. The probabilities that a student passes in Mathematics,
rolled four times. Out of four face values obtained, the
Physics and Chemistry are m, p and c, respectively. Of
probability that the minimum face value is not less
these subjects, the students has a 75% chance of passing
than 2 and the maximum face value is not greater
in atleast one, a 50% chance of passing in atleast two and
than 5, is (1993, 1M)
a 40% chance of passing in exactly two. Which of the
(a) 16/81 (b) 1/81 (c) 80/81 (d) 65/81 following relations are true? (1999, 3M) (2011)
18. A student appears for tests I, II and III. The student is 19 27
(a) p + m + c = (b) p + m + c =
successful if he passes either in tests I and II or tests I 20 20
and III. The probabilities of the student passing in 1 1
1 (c) pmc = (d) pmc =
tests I, II and III are p, q and , respectively. If the 10 4
2
1 25. If E and F are the complementary events of E and F
probability that the student is successful, is , then respectively and if 0 < P (F ) < 1, then (1998, 2M)
2
1 (a) P (E / F ) + P (E / F ) = 1 (b) P (E / F ) + P (E / F ) = 1
(a) p = q = 1 (b) p = q = (1986, 2M)
2 (c) P (E / F ) + P (E / F ) = 1 (d) P (E / F ) + P (E / F ) = 1
1 26. Let E and F be two independent events. If the probability
(c) p = 1, q = 0 (d) p = 1, q =
2 that both E and F happen is 1/12 and the probability that
19. If A and B are two independent events such that neither E nor F happen is 1/2. Then,
P ( A ) > 0, and P (B) ≠ 1, then P ( A / B ) is equal to (a) P (E ) = 1 / 3, P (F ) = 1 / 4 (1993, 2M)
(b) P (E ) = 1 / 2, P (F ) = 1 / 6
(a) 1 − P (A / B ) (b) 1 − P (A / B ) (1982, 2M)
(c) P (E ) = 1 / 6, P (F ) = 1 / 2
1 − P (A ∪ B ) P (A ) (d) P (E ) = 1 / 4, P (F ) = 1 / 3
(c) (d)
P (B ) P (B )
27. For any two events A and B in a sample space
20. The probability that an event A happens in one trial of (1991, 2M)
P (A ) + P (B ) − 1
(a) P   ≥
an experiment, is 0.4. Three independent trials of the A
, P (B ) ≠ 0 is always true
experiments are performed. The probability that the  B P (B )
event A happens atleast once, is (1980, 1M) (b) P (A ∩ B ) = P (A ) − P (A ∩ B ) does not hold
(a) 0.936 (b) 0.784 (c) P (A ∪ B ) = 1 − P (A )P (B ), if A and B are independent
(c) 0.904 (d) None of these (d) P (A ∪ B ) = 1 − P (A )P (B ), if A and B are disjoint
108 Probability

28. If E and F are independent events such that 0 < P (E ) < 1 result is a tail, a card from a well-shuffled pack of eleven
and 0 < P (F ) < 1, then (1989, 2M) cards numbered 2, 3, 4, …, 12 is picked and the number
on the card is noted. What is the probability that the
(a) E and F are mutually exclusive
noted number is either 7 or 8? (1994, 5M)
(b) E and F c (the complement of the event F) are
independent 39. A lot contains 50 defective and 50 non-defective bulbs.
(c) E c and F c are independent Two bulbs are drawn at random, one at a time, with
(d) P (E / F ) + P (E c / F ) = 1 replacement. The events A, B, C are defined as :
A = ( the first bulb is defective)
Fill in the Blanks B = (the second bulb is non-defective)
29. If two events A and B are such that P ( A c ) = 0.3, C = (the two bulbs are both defective or both
P (B) = 0.4 and P ( A ∩ Bc ) = 0.5, then P [B / ( A ∪ Bc )] = K . non-defective).
(1994, 2M) Determine whether
30. Let A and B be two events such that P ( A ) = 0.3 and (i) A, B, C are pairwise independent.
P ( A ∪ B) = 0.8. If A and B are independent events, (ii) A, B, C are independent. (1992, 6M)
then P (B) = … . (1990, 2M) 40. In a multiple-choice question there are four alternative
31. A pair of fair dice is rolled together till a sum of either 5 answers, of which one or more are correct. A candidate
or 7 is obtained. Then, the probability that 5 comes will get marks in the question only if he ticks the correct
before 7, is… . (1989, 2M) answers. The candidates decide to tick the answers at
random, if he is allowed upto three chances to answer
32. Urn A contains 6 red and 4 black balls and urn B the questions, find the probability that he will get
contains 4 red and 6 black balls. One ball is drawn at marks in the question. (1985, 5M)
random from urn A and placed in urn B. Then, one ball
is drawn at random from urn B and placed in urn A. If 41. A and B are two independent events. The probability
1
one ball is drawn at random from urn A, the probability that both A and B occur is and the probability that
that it is found to be red, is…. (1988, 2M) 6
1
neither of them occurs is . Find the probability of the
33. A box contains 100 tickets numbered 1, 2, …,100. Two 3
tickets are chosen at random. It is given that the occurrence of A. (1984, 2M)
maximum number on the two chosen tickets is not more
than 10. The maximum number on them is 5 with
42. Cards are drawn one by one at random from a well
shuffled full pack of 52 playing cards until 2 aces are
probability… . (1985, 2M)
obtained for the first time. If N is the number of cards
required to be drawn, then show that
Analytical and Descriptive Questions (n − 1)(52 − n )(51 − n )
Pr { N = n } =
34. If A and B are two independent events, prove that 50 × 49 × 17 × 13
P ( A ∪ B) ⋅ P ( A′ ∩ B ′ ) ≤ P (C ), where C is an event where, 2 < n ≤ 50. (1983, 3M)
defined that exactly one of A and B occurs. (2004, 2M)
43. An anti-aircraft gun can take a maximum of four shots
35. A is targeting to B, B and C are targeting to A. at an enemy plane moving away from it. The
2 1 probabilities of hitting the plane at the first, second,
Probability of hitting the target by A, B and C are ,
3 2 third and fourth shot are 0.4, 0.3, 0.2, and 0.1,
1 respectively. What is the probability that the gun hits
and , respectively. If A is hit, then find the probability
3 the plane? (1981, 2M)
that B hits the target and C does not. (2003, 2M) 44. A box contanis 2 black, 4 white and 3 red balls. One ball
36. For a student to qualify, he must pass atleast two out of is drawn at random from the box and kept aside. From
three exams. The probability that he will pass the 1st the remaining balls in the box, another ball is drawn at
exam is p. If he fails in one of the exams, then the random and kept beside the first. This process is
p repeated till all the balls are drawn from the box. Find
probability of his passing in the next exam, is the probability that the balls drawn are in the sequence
2
otherwise it remains the same. Find the probability of 2 black, 4 white and 3 red. (1979, 2M)

that he will qualify. (2003, 2M)


Integer & Numerical Answer Type Questions
37. A coin has probability p of showing head when tossed. It
45. Let S be the sample space of all 3 × 3 matrices with
is tossed n times. Let pn denotes the probability that no
two (or more) consecutive heads occur. Prove that p1 = 1, entries from the set {0, 1}. Let the events E1 and E 2 be
p2 = 1 − p2 and pn = (1 − p). pn − 1 + p(1 − p) pn − 2, ∀ n ≥ 3. given by
(2000, 5M) E1 = { A ∈ S : det A = 0} and
38. An unbiased coin is tossed. If the result in a head, a pair E 2 = { A ∈ S : sum of entries of A is 7}.
of unbiased dice is rolled and the number obtained by If a matrix is chosen at random from S, then the
adding the numbers on the two faces is noted. If the conditional probability P (E1 | E 2) equals ...........
Probability 109

46. Of the three independent events E1 , E 2 and E3 , the 1 1 1


losing a game against T2 are , and , respectively. Each
probability that only E1 occurs is α , only E 2 occurs is β 2 6 3
and only E3 occurs is γ. Let the probability p that none of team gets 3 points for a win, 1 point for a draw and 0 point for
a loss in a game. Let X andY denote the total points scored by
events E1 , E 2 or E3 occurs satisfy the equations (α − 2 β ),
teams T1 and T2, respectively, after two games. (2016 Adv.)
p = αβ and (β − 3γ ) p = 2 βγ. All the given probabilities
are assumed to lie in the interval (0, 1). 47. P (X > Y ) is
1 5
probability of occurrence of E1 (a) (b)
Then, is equal to 4 12
probability of occurrence of E3 1 7
(c) (d)
2 12
Passage Type Questions 48. P (X = Y ) is
Passage (a)
11
(b)
1
Football teams T1 and T2 have to play two games against each 36 3
other. It is assumed that the outcomes of the two games are 13 1
(c) (d)
independent. The probabilities of T1 winning, drawing and 36 2

Topic 4 Law of Total Probability and Baye’s Theorem


Objective Question I (Only one correct option) transmitted to station B. The probability of each station
3
1. A pot contain 5 red and 2 green balls. At random a ball receiving the signal correctly is . If the signal received
4
is drawn from this pot. If a drawn ball is green then put
at station B is green, then the probability that the
a red ball in the pot and if a drawn ball is red, then put a
original signal green is (2010)
green ball in the pot, while drawn ball is not replace in 3 6
the pot. Now we draw another ball randomnly, the (a) (b)
5 7
probability of second ball to be red is (2019 Main, 9 Jan II) 20 9
27 26 21 32 (c) (d)
(a) (b) (c) (d) 23 20
49 49 49 49
2. A bag contains 4 red and 6 black balls. A ball is drawn at Objective Question II
random from the bag, its colour is observed and this ball
along with two additional balls of the same colour are (One or more than one correct option)
returned to the bag. If now a ball is drawn at random 5. There are three bags B1, B2 and B3 . The bag B1 contains
from the bag, then the probability that this drawn ball 5 red and 5 green balls, B2 contains 3 red and 5 green
is red, is (2018 Main) balls, and B3 contains 5 red and 3 green balls. Bags B1,
3 2 3 3 4
(a) (b) B2 and B3 have probabilities , and respectively
10 5 10 10 10
1 3
(c) (d) of being chosen. A bag is selected at random and a ball is
5 4 chosen at random from the bag. Then which of the
3. A computer producing factory has only two plants T1 following options is/are correct?
and T2. Plant T1 produces 20% and plant T2 produces (a) Probability that the chosen ball is green, given that
3
80% of the total computers produced. 7% of computers the selected bag is B3 , equals .
produced in the factory turn out to be defective. It is 8
known that P(computer turns out to be defective, given (b) Probability that the selected bag is B3 , given that the
that it is produced in plant T1) = 10P (computer turns 5
chosen ball is green, equals .
out to be defective, given that it is produced in plant T2), 13
where P (E ) denotes the probability of an event E. A 39
(c) Probability that the chosen ball is green equals .
computer produced in the factory is randomly selected 80
and it does not turn out to be defective. Then, the (d) Probability that the selected bag is B3 and the chosen
probability that it is produced in plant T2, is (2016 Adv.) 3
ball is green equals .
36 47 10
(a) (b)
73 79 6. A ship is fitted with three engines E1 , E 2 and E3 . The
78 75 engines function independently of each other with
(c) (d)
93 83 respective probabilities 1/2, 1/4 and 1/4. For the ship to
4 be operational atleast two of its engines must function.
4. A signal which can be green or red with probability Let X denotes the event that the ship is operational and
5
1 let X1, X 2 and X3 denote, respectively the events that the
and respectively, is received by station A and then engines E1, E 2 and E3 are functioning.
5
110 Probability

Which of the following is/are true? (2012) 10. The probability of the drawn ball fromU 2 being white,
(a) P [X1c| X ] = 3 / 16
is
13 23 19 11
7 (a) (b) (c) (d)
(b) P [exactly two engines of the ship are functioning] = 30 30 30 30
8
(c) P [X | X 2 ] =
5
(d) P [X | X1 ] =
7 11. Given that the drawn ball from U 2 is white, the
16 16 probability that head appeared on the coin is
17 11 15 12
Assertion and Reason (a) (b) (c) (d)
23 23 23 23
For the following questions, choose the correct answer Passage III
from the codes (a), (b), (c) and (d) defined as follows. A fair die is tossed repeatedly until a six is obtained. Let X
(a) Statement I is true, Statement II is also true; denote the number of tosses required. (2009)
Statement II is the correct explanation of Statement I
(b) Statement I is true, Statement II is also true; 12. The probability that X = 3 equals
25 25 5 125
Statement II is not the correct explanation of (a) (b) (c) (d)
Statement I 216 36 36 216
(c) Statement I is true; Statement II is false 13. The probability that X ≥ 3 equals
(d) Statement I is false; Statement II is true 125 25 5 25
(a) (b) (c) (d)
7. Let H 1 , H 2,... , H n be mutually exclusive events with 216 36 36 216
P (H i ) > 0, i = 1, 2,... , n . Let E be any other event with 14. The conditional probability that X ≥ 6 given X > 3
0 < P (E ) < 1. equals
Statement I P (H i/E ) > P (E/H i ) ⋅ P (H i ) for 125 25 5 25
(a) (b) (c) (d)
216 216 36 36
i = 1, 2, . . . , n
n Passage IV
Statement II ∑ P (Hi ) = 1 (2007, 3M) There are n urns each containing (n + 1) balls such that the
i =1
ith urn contains ‘i’white balls and (n + 1 − i) red balls. Let ui
be the event of selecting ith urn, i = 1, 2, 3, ... , n and W
Passage Based Problems denotes the event of getting a white balls. (2006, 5M)
Passage I 15. If P (ui ) ∝ i, where i = 1, 2, 3,... , n , then lim P (W ) is
Let n1 and n2 be the number of red and black balls, respectively n→ ∞
equal to
in box I. Let n3 and n4 be the number of red and black balls, 2 1 3
(a) 1 (b) (c) (d)
respectively in box II. (2015 Adv.) 3 4 4
8. One of the two boxes, box I and box II was selected at 16. If P (ui ) = c , where c is a constant, then P (un / W ) is
random and a ball was drawn randomly out of this box. equal to
The ball was found to be red. If the probability that this 2 1
1 (a) (b)
red ball was drawn from box II, is , then the correct n+1 n+1
3 n 1
option(s) with the possible values of n1 , n2, n3 and n4 (c) (d)
n+1 2
is/are
(a) n1 = 3, n2 = 3, n3 = 5, n4 = 15 17. If n is even and E denotes the event of choosing even
(b) n1 = 3, n2 = 6, n3 = 10, n4 = 50  1
numbered urn P (ui )= , then the value of P ( W /E ) is
 n 
(c) n1 = 8, n2 = 6, n3 = 5, n4 = 20
(a) n + 2 n+ 2
(d) n1 = 6, n2 = 12, n3 = 5, n4 = 20 (b)
2n + 1 2 (n + 1)
9. A ball is drawn at random from box I and transferred to n 1
box II. If the probability of drawing a red ball from box I, (c) (d)
n+1 n+1
1
after this transfer, is , then the correct option(s) with
3 Analytical and Descriptive Questions
the possible values of n1 and n2 is/are
(a) n1 = 4 and n2 = 6 (b) n1 = 2 and n2 = 3 18. A person goes to office either by car, scooter, bus or
(c) n1 = 10 and n2 = 20 (d) n1 = 3 and n2 = 6 1 3 2 1
train probability of which being , , and ,
7 7 7 7
Passage II respectively. Probability that he reaches offices late, if
LetU 1 andU 2 be two urns such thatU 1 contains 3 white and 2 2 1 4 1
red balls andU 2 contains only 1 white ball. A fair coin is tossed. he takes car, scooter, bus or train is , , and ,
9 9 9 9
If head appears then 1 ball is drawn at random from U 1 and
respectively. Given that he reached office in time,
put intoU 2. However, if tail appears then 2 balls are drawn at
random from U 1 and put into U 2. Now, 1 ball is drawn at then what is the probability that he travelled by a car ?
(2005, 2M)
random from U 2. (2011)
Probability 111

19. A bag contains 12 red balls and 6 white balls. Six balls are the two players of the pair. Assume that all the players
drawn one by one without replacement of which at least are of equal strength.
4 balls are white. Find the probability that in the next two (i) Find the probability that the player S1 is among the
drawn exactly one white ball is drawn. (Leave the answer eight winners.
in nC r). (2004, 4M) (ii) Find the probability that exactly one of the two
20. A box contains N coins, m of which are fair and the rest are players S1 and S2 is among the eight winners.
(1997C, 5M)
biased. The probability of getting a head when a fair coin is
tossed, is 1/2, while it is 2/3 when a biased coin is tossed. A 25. In a test an examinee either guesses or copies of
coin is drawn from the box at random and is tossed twice. knows the answer to a multiple choice question with
The first time it shows head and the second time it shows 1
four choices. The probability that he make a guess is
tail. What is the probability that the coin drawn is fair? 3
(2002, 5M) 1
and the probability that he copies the answer is . The
21. An urn contains m white and n black balls. A ball is drawn 6
at random and is put back into the urn along with k probability that his answer is correct given that he
additional balls of the same colour as that of the ball 1
copied it, is . Find the probability that he knew the
drawn. A ball is again drawn at random. What is the 8
probability that the ball drawn now is white? (2001, 5M) answer to the question given that he correctly
22. Eight players P1 , P2, K , P8 play a knock-out tournament. It answered it. (1991, 4M)

is known that whenever the players Pi and Pj play, the 26. An urn contains 2 white and 2 blacks balls. A ball is
player Pi will win if i < j. Assuming that the players are drawn at random. If it is white it is not replaced into
paired at random in each round, what is the probability the urn. Otherwise it is replaced along with another
that the player P4 reaches the final? (1999, 10M) ball of the same colour. The process is repeated. Find
the probability that the third ball drawn is black.
23. Three players, A, B and C, toss a coin cyclically in that (1987, 4M)
order (i.e. A, B, C, A, B, C, A, B, …) till a head shows. Let p
be the probability that the coin shows a head. Let α, β and 27. A lot contains 20 articles. The probability that the lot
γ be, respectively, the probabilities that A, B and C gets contains exactly 2 defective articles is 0.4 and the
the first head. Prove that β = (1 − p) α . Determine α, β and probability that the lot contains exactly 3 defective
γ (in terms of p). (1998, 8M) articles is 0.6. Articles are drawn from the lot at
random one by one without replacement and are
24. Sixteen players S1 , S 2, K , S16 play in a tournament. They tested till all defective articles are found. What is the
are divided into eight pairs at random from each pair a probability that the testing procedure ends at the
winner is decided on the basis of a game played between twelfth testing? (1986, 5M)

Topic 5 Probability Distribution and Binomial Distribution


3
Objective Questions I (Only one correct option)  3
out of service on the same day is   k, then k is equal
1. Let C1 and C 2 be two biased coins such that the  4
2 1 to [2020 Main, 7 Jan II]
probabilities of getting head in a single toss are and , 17
3 3 (a) 4 (b) 17 17
(c) (d)
respectively. Suppose α is the number of heads that 4 8 2
appear when C1 is tossed twice, independently, and 4. For an initial screening of an admission test, a
suppose β is the number of heads that appear when C 2 is candidate is given fifty problems to solve. If the
tossed twice, independently. Then the probability that probability that the candidate can solve any problem
the roots of the quadratic polynomial x2 − ax + β are real 4
is , then the probability that he is unable to solve less
and equal, is (2020 Adv.) 5
40 20 than two problem is (2019 Main, 12 April II)
(a) (b)
81 81 201  1 
49
316  4 
48
1 1 (a)   (b)  
(c) (d) 5  5 25  5 
2 4 49 48
54  4  164  1 
2. Four fair dice are thrown independently 27 times. Then, (c)   (d)  
5  5 25  5 
the expected number of times, at least two dice show up a
three or a five, is …… . [2020 Main, 5 Sep I] 5. Let a random variable X have a binomial distribution
k
3. In a workshop, there are five machines and the with mean 8 and variance 4. If P (X ≤ 2) = , then k is
probability of any one of them to be out of service on a day 216
1 equal to (2019 Main, 12 April I)
is . If the probability that at most two machines will be (a) 17 (b) 121 (c) 1 (d) 137
4
112 Probability

6. Minimum number of times a fair coin must be tossed so Fill in the Blanks
that the probability of getting atleast one head is more
than 99% is (2019 Main 10 April II) 15. If the mean and the variance of a binomial variate X are
(a) 8 (b) 6 (c) 7 (d) 5 2 and 1 respectively, then the probability that X takes a
value greater than one is equal to… . (1991, 2M)
7. The minimum number of times one has to toss a fair
coin so that the probability of observing atleast one head 16. For a biased die the probabilities for the different faces
is atleast 90% is (2019 Main, 8 April II) to turn up are given below
(a) 2 (b) 3 (c) 5 (d) 4 Face 1 2 3 4 5 6
8. In a game, a man wins ` 100 if he gets 5 or 6 on a throw Probability 0.1 0.32 0.21 0.15 0.05 0.17
of a fair die and loses ` 50 for getting any other number
on the die. If he decides to throw the die either till he This die is tossed and you are told that either face 1 or
gets a five or a six or to a maximum of three throws, face 2 has turned up. Then, the probability that it is face
then his expected gain/loss (in rupees) is 1, is… . (1981, 2M)
(2019 Main, 12 Jan II)
(a)
400
loss (b)
400
loss (c) 0 (d)
400
gain
Analytical & Descriptive Questions
3 9 3 17. Numbers are selected at random, one at a time, from the
9. If the probability of hitting a target by a shooter in any two-digit numbers 00, 01, 02, …, 99 with replacement.
1 An event E occurs if and only if the product of the two
shot, is , then the minimum number of independent
3 digits of a selected number is 18. If four numbers are
shots at the target required by him so that the selected, find probability that the event E occurs at least
probability of hitting the target at least once is greater 3 times. (1993, 5M)
5
than , is 18. A is a set containing n elements. A subset P of A is
6 (2019 Main, 10 Jan II)
chosen at random. The set A is reconstructed by
(a) 6 (b) 3 (c) 5 (d) 4 replacing the elements of P. A subset Q of A is again
10. Two cards are drawn successively with replacement chosen at random. Find the probability that P and Q
from a well shuffled deck of 52 cards. Let X denote the have no common elements. (1991, 4M)
random variable of number of aces obtained in the two
drawn cards. Then, P (X = 1) + P (X = 2) equals 19. Suppose the probability for A to win a game against B is
(2019 Main, 9 Jan I) 0.4. If A has an option of playing either a ‘best of 3
25 52 49 24 games’ or a ‘best of 5 games’’ match against B, which
(a) (b) (c) (d) option should choose so that the probability of
169 169 169 169
his winning the match is higher? (no game ends in a
11. A box contains 15 green and 10 yellow balls. If 10 balls draw). (1989, 5M)
are randomly drawn one-by-one with replacement, then
the variance of the number of green balls drawn is 20. A man takes a step forward with probability 0.4 and
(2017 Main) backwards with probability 0.6. Find the probability
12 6 that at the end of eleven steps he is one step away from
(a) (b) 6 (c) 4 (d)
5 25 the starting point. (1987, 3M)
12. A multiple choice examination has 5 questions. Each
question has three alternative answers of which exactly Integer & Numerical Answer Type Questions
one is correct. The probability that a student will get 4
or more correct answers just by guessing is (2013 Main) 21. Two fair dice, each with faces numbered 1, 2, 3, 4, 5 and
17 13 11 10 6, are rolled together and the sum of the numbers on the
(a) 5
(b) 5
(c) 5
(d) 5 faces is observed. This process is repeated till the sum is
3 3 3 3
either a prime number or a perfect square. Suppose the
13. India plays two matches each with West Indies and sum turns out to be a perfect square before it turns out to
Australia. In any match the probabilities of India be a prime number. If p is the probability that this
getting points 0, 1 and 2 are 0.45, 0.05 and 0.50, perfect square is an odd number, then the value of 14 p is
respectively. Assuming that the outcomes are ……… .
independent. The probability of India getting at least 7
22. The probability that a missile hits a target successfully
points, is (1992, 2M)
is 0.75. In order to destroy the target completely, at
(a) 0.8750 (b) 0.0875 (c) 0.0625 (d) 0.0250 least three successful hits are required. Then the
14. One hundred identical coins, each with probability p, of minimum number of missiles that have to be fired so
showing up heads are tossed once. If 0 < p < 1 and the that the probability of completely destroying the target
probability of heads showing on 50 coins is equal to that is NOT less than 0.95, is ……… .
of heads showing on 51 coins, then the value of p is 23. The minimum number of times a fair coin needs to be
(1988, 2M) tossed, so that the probability of getting atleast two
(a) 1/2 (b) 49/101 (c) 50/101 (d) 51/101 heads is atleast 0.96, is (2015 Adv.)
Probability 113

Answers
Topic 1 39. (i) A, B and C are pairwise independent 40.
1
1. (c) 2. (a) 3. (a) 4. (d) 5
5. (a) 6. (c) 7. (a) 8. (a) 1 1 1
41. or 43. 0.6976 44. 45. 0.50
9. (a) 10. (c) 11. (d) 12. (d) 3 2 1260
13. (a) 14. (b) 15. (c) 16. (b) 46. (6) 47. (b) 48. (c)
17. (d) 18. (b) 19. (b) 20. (c)
1 1 1 3 Topic 4
21. 22. ≤ p ≤ 23. 24. False
36 3 2 16 1. (d) 2. (b) 3. (c) 4. (c)
(3n − 3. 2n + 3 ) × 6C 3 1 5. (a,c) 6. (b, d) 7. (d) 8. (a,b)
25. 26. 0.62 27.
6n 91 9. (c,d) 10. (b) 11. (d) 12. (a)
10 (n + 2 ) 1 1
28. 1 − n + 7 29. (i) (ii) 13. (b)` 14. (d) 15. (b) 16. (a)
C5 132 462 1
17. (b) 18.
Topic 2 7
1. (b) 2. (b) 3. (a) 4. (c)
12
C 2 ⋅ C 4 C1 ⋅ C1 12C1 ⋅6 C 5 11C1 ⋅1 C1
6 10 2
9m
19. ⋅ 12 + 18 ⋅ 12 20.
5. (a) 6. (c) 7. (c) 8. (a)
18
C6 C2 C6 C2 8N + m
11 m 4
9. (a, b, c) 10. (a, c) 11. 12. P ( A ∩ B ) 21. 22.
40 m+n 35
13. False 14. 13.9% 16. No 17. (a) p p (1 − p ) p − 2p 2 + p 3
23. α= , β = , γ =
18. (c) 1 − (1 − p ) 3 1 − (1 − p ) 3 1(1 − p ) 3
1 8 24 23 99
Topic 3 24. (i) (ii) 25. 26. 27.
1. (d) 2. (b) 3. (b) 4. (a) 2 15 29 30 1900
5. (c) 6. (d) 7. (a) 8. (a)
Topic 5
9. (d) 10. (c) 11. (c) 12. (b)
1. (b) 2. (11) 3. (c) 4. (c)
13. (b) 14. (a) 15. (a) 16. (b)
5. (d) 6. (c) 7. (d) 8. (c)
17. (a) 18. (c) 19. (b) 20. (b)
9. (c) 10. (a) 11. (a) 12. (c)
21. (a, b) 22. (a,b) 23. (a, d) 24. (b, c) 11 5
13. (b) 14. (d) 15. 16.
25. (a, d) 26. (a, d) 27. (a, c) 28. (b, c, d) 16 21
1 5 2 32 n
29. 30. 31. 32. 97  3
4 7 5 55 17. 18.   19. Best of 3 games
25 4  4
1 1 193
33. 35. 36. 2 p 2 – p 3 38. 20. 11
C 6( 0 . 24 ) 5 21. (8) 22. (6) 23. (8)
9 2 792

Hints & Solutions


Topic 1 Classical Probability 5 4 26 15 + 8 − 26
= + − =
1. It is given that a person wins 2 3 6 6
23 − 26 3 1 1
`15 for throwing a doublet (1, 1) (2, 2), (3, 3), = = − = − , means loss of `
(4, 4), (5, 5), (6, 6) and win `12 when the throw results in 6 6 2 2
sum of 9, i.e., when (3, 6), (4, 5), 2. Since, the experiment should be end in the fifth throw of
(5, 4), (6, 3) occurs. the die, so total number of outcomes are 65 .
Also, losses ` 6 for throwing any other outcome, i.e., Now, as the last two throws should be result in two
when any of the rest 36 − 6 − 4 = 26 outcomes occurs. 4 4
fours
Now, the expected gain/loss (i) (ii) (iii) (iv) (v)
= 15 × P (getting a doublet) + 12 × P So, the third throw can be 1, 2, 3, 5 or 6 (not 4). Also,
(getting sum 9) − 6 × P (getting any throw number (i) and (ii) can not take two fours in
of rest 26 outcome) succession, therefore number of possibililites for throw
 6  4  26 (i) and (ii) = 62 − 1 = 35
= 15 ×  + 12 ×  − 6 × 
 36  36  36 [Q when a pair of dice is thrown then
(4, 4) occur only once].
114 Probability

5 × 35 175 7. We have mentioned that boxes are different and one


Hence, the required probability = = 5
65 6 particular box has 3 balls.
11
3. Since, there is a regular hexagon, then the number of C3 × 29 55  2
12
Then, number of ways = =  
6
ways of choosing three vertices is C3 . And, there is only 312 3  3
two ways i.e. choosing vertices of a regular hexagon 8. Total number of ways to arrange 3 boys and 2 girls
alternate, here A1, A3 , A5 or A2, A4, A6 will result in an are 5!.
equilateral triangle.
According to given condition, following cases may arise.
A1
A6 B G G B B
A2
G G B B B
A5 A3 G B G B B
G B B G B
A4
B G B G B
∴Required probability So, number of favourable ways = 5 × 3 ! × 2 ! = 60
2 2 2 ×3 ×2 ×3 ×2 1
= 6 = = = ∴ Required probability =
60 1
=
C3 6 ! 6 × 5 × 4 × 3 × 2 × 1 10 120 2
3 !3 ! 9. PLAN As one of the dice shows a number appearing on one of P1, P2
4. Number of subset of S = 220 and P3.
20(21)
Sum of elements in S is 1 + 2 + .....+20 = = 210
2 Thus, three cases arise.
 n (n + 1)  (i) All show same number.
Q 1 + 2+ ...... + n =
 2  (ii) Number appearing on D 4 appears on any one of
Clearly, the sum of elements of a subset would be 203, if D1, D 2 and D 3.
we consider it as follows (iii) Number appearing on D 4 appears on any
S − { 7}, S − {1, 6} S − {2, 5}, S − {3, 4} two of D1, D 2 and D 3.
S − { 1, 2, 4 } Sample space = 6 × 6 × 6 × 6 = 64 favourable events
∴ Number of favourables cases = 5 = Case I or Case II or Case III
5
Hence, required probability = 20 Case I First we should select one number for D4
2 which appears on all i.e. 6C1 × 1.
5. Total number of ways of selecting 2 different numbers Case II For D4 there are 6C1 ways. Now, it appears
from {0, 1, 2, ..., 10} = 11C 2 = 55 on any one of D1 , D2 and D3 i.e. 3 C1 × 1.
Let two numbers selected be x and y. For other two there are 5 × 5 ways.
Then, x + y = 4m …(i)
⇒ 6
C1 × 3C1 × 1 × 5 × 5
and x − y = 4n …(ii)
⇒ 2x = 4(m + n ) and 2 y = 4(m − n ) Case III For D4 there are 6C1 ways now it appears on
⇒ x = 2(m + n )and y = 2(m − n ) any two of D1 , D2 and D3
⇒ 3
C 2 × 12
∴x and y both are even numbers.
For other one there are 5 ways.
x y
⇒ 6
C1 × 3C 2 × 12 × 5
0 4, 8 6
C1 + 6C1 × 3C1 × 52 + 6C1 × 3C 2 × 5
2 6, 10 Thus, probability =
64
4 0, 8 6 (1 + 75 + 15)
6 2, 10 =
64
8 0, 4 91
10 2, 6
=
216
6
∴Required probability = 10. Sample space A dice is thrown thrice, n (s) = 6 × 6 × 6.
55
r r
6. Sample space → 12C 2 Favorable events ω r 1 + ω 2 + ω 3 = 0
Number of possibilities for z is even. i.e. (r1 , r2, r3 ) are ordered 3 triples which can take
z = 0 ⇒ 11C1; z = 2 ⇒ 9C1 values,
(1, 2 , 3), (1, 5, 3), (4, 2 , 3), (4, 5, 3)
z = 4 ⇒ 7C1; z = 6 ⇒ 5C1  i.e. 8 ordered pairs
z = 8 ⇒ 3C1; z = 10 ⇒ 1C1 (1, 2 , 6), (1, 5, 6), (4, 2 , 6), (4, 5, 6)
Total = 36 and each can be arranged in 3 ! ways = 6
36 6 8 ×6 2
∴ Probability = = ∴ n (E ) = 8 × 6 ⇒ P (E ) = =
66 11 6 ×6 ×6 9
Probability 115

11. Since, three distinct numbers are to be selected from Only two equilateral triangles can be formed
first 100 natural numbers. ∆AEC and ∆BFD.
⇒ n (S ) = 100C3 D

E(favourable events) = All three of them are divisible by both


2 and 3 . E C
⇒ Divisible by 6 i.e. {6, 12, 18, …, 96}
Thus, out of 16 we have to select 3.
∴ n (E ) = 16C3
16 F B
C 4
∴ Required probability = 100 3 =
C3 1155 A
12. Here, two numbers are selected from {1, 2, 3, 4, 5, 6} ∴ Favourable ways = 2
⇒ n (S ) = 6 × 5 {as one by one without replacement} 2 1
So, required probability = =
Favourable events = the minimum of the two numbers 20 10
is less than 4. n (E ) = 6 × 4 {as for the minimum of the 16. Since, three dice are rolled.
two is less than 4 we can select one from (1, 2, 3, 4) and
other from (1, 2, 3, 4, 5, 6) ∴ Total number of cases S = 6 × 6 × 6 = 216
n (E ) 24 4 and the same number appear on each of them = 6C1 = 6
∴ Required probability = = = 6 1
n (S ) 30 5 ∴ Required probability = =
216 36
13. 71 = 7, 72 = 49, 73 = 343, 74 = 2401, …
17. Since, there are 15 possible cases for selecting a coupon
Therefore, for 7r, r ∈ N the number ends at unit place and seven coupons are selected, the total number of
7, 9, 3, 1, 7, … cases of selecting seven coupons = 157
∴ 7m + 7n will be divisible by 5 if it end at 5 or 0. It is given that the maximum number on the selected
But it cannot end at 5. coupon is 9, therefore the selection is to be made from
Also for end at 0. the coupons numbered 1 to 9. This can be made in 97
ways. Out of these 97 cases, 87 does not contain the
For this m and n should be as follows
number 9.
m n Thus, the favourable number of cases = 97 − 87.
1 4r 4r − 2 97 − 87
2 4r − 1 4r − 3 ∴ Required probability =
157
3 4r − 2 4r
4 4r − 3 4r − 1 18. The number of all possible determinants of the form
a b
For any given value of m, there will be 25 values of n. = 24 = 16
Hence, the probability of the required event is c d
100 × 25 1 Out of which only 10 determinants given by
=
100 × 100 4 1 1 0 0 1 1 0 0 0 1 1 0 1 0
, , , , , , ,
NOTE Power of prime numbers have cyclic numbers in their unit 1 1 0 0 0 0 1 1 0 1 1 0 0 0
place. 0 1 0 0
,
14. The number of ways of placing 3 black balls without any 0 0 1 0
restriction is 10C3 . Since, we have total 10 places of
putting 10 balls in a row. Now, the number of ways in Vanish and remaining six determinants have non-zero
6 3
which no two black balls put together is equal to the values. Hence, the required probability = =
number of ways of choosing 3 places marked ‘—’ out of 16 8
eight places. Statement I is true.
Statement II is also true as the homogeneous equations
—W—W—W—W —W—W—W—
have always a solution and Statement II is not the
This can be done in 8C3 ways. correct explanation of Statement I.
8
C3 8 × 7 ×6 7 Number of favourable outcomes
∴ Required probability = = = 19. PLAN Probability =
10
C3 10 × 9 × 8 15 Number of total outcomes
As, x1 + x2 + x3 is odd.
15. Three vertices out of 6 can be chosen in 6C3 ways.
So, all may be odd or one of them is odd and other two
So, total ways = 6C3 = 20
are even.
116 Probability

∴ Required probability 24. Total number of ways to arrange ‘ASSASSIN’ is


8!
.
2
C1 × 3C1 × 4C1 + 1C1 × 2C1 × 4C1 + 2C1 × 2C1 × 3C1 4 !⋅ 2 !
+ 1C1 × 3C1 × 3C1 First we fix the position ⊗ A ⊗ A ⊗ I ⊗ N ⊗.
= 3
C1 × C1 × C1
5 7
Number of ways in which no two S’s occur together
24 + 8 + 12 + 9 =
4! 5
× C4
=
105 2!
53 4! × 5 × 4! × 2! 1
= ∴ Required probability = =
105 2! × 8! 14

20. Since, x1 , x2, x3 are in AP. Hence, it is a false statement.


∴ x1 + x3 = 2x2 25. Let us define a onto function F from A : [ r1 , r2, K , rn] to
So, x1 + x3 should be even number. B: [1, 2, 3], where r1 , r2, K , rn are the readings of n
throws and 1, 2, 3 are the numbers that appear in the n
Either both x1 and x3 are odd or both are even.
throws.
2
C1 × 4C1 + 1C1 × 3C1 Number of such functions, M = N − [n (1) − n (2) + n (3)]
∴ Required probability = 3
C1 × 5C1 × 7C1
where, N = total number of functions
11 and n (t ) = number of function having exactly t
=
105 elements in the range.
21. According to given condition, Now, N = 3n, n (1) = 3 . 2n, n(2) = 3, n(3) = 0
3 1 ⇒ M = 3n − 3 . 2n + 3
P ( yellow at the first toss) = =
6 2 Hence, the total number of favourable cases
2 1
P (red at the second toss) = = = (3n − 3 . 2n + 3) . 6C3
6 3
1 (3n − 3 . 2n + 3) × 6C3
and P (blue at the third toss) = ∴ Required probability =
6 6n
Therefore, the probability of the required event 26. The required probability = 1 − (probability of the event
1 1 1 1 that the roots of x2 + px + q = 0 are non-real).
= × × =
2 3 6 36
The roots of x2 + px + q = 0 will be non-real if and only if
1 + 3p 1 − p 1 − 2p p2 − 4q < 0, i.e. if p2 < 4 q
22. Since, , and are the probability of
3 4 2
The possible values of p and q can be possible according
mutually exclusive events.
to the following table.
1 + 3p 1 − p 1 −2p
∴ + + ≤1 Value of q Value of p Number of pairs of p, q
3 4 2
⇒ 4 + 12 p + 3 − 3 p + 6 − 12 p ≤ 12 1 1 1
2 1, 2 2
⇒ 13 − 3 p ≤ 12
1 3 1, 2, 3 3
⇒ p≥ ...(i)
3 4 1, 2, 3 3
1 + 3p 1− p 1 −2p 5 1, 2, 3, 4 4
and 0 ≤ ≤ 1, 0 ≤ ≤ 1, 0 ≤ ≤1
3 4 2 6 1, 2, 3, 4 4
⇒ 0 ≤ 1 + 3 p ≤ 3, 0 ≤ 1 − p ≤ 4, 0 ≤ 1 − 2 p ≤ 2 7 1, 2, 3, 4, 5 5
1 2 1 1
⇒ − ≤ p ≤ , 1 ≥ p ≥ −3 , ≥ p ≥ − ...(ii) 8 1, 2, 3, 4, 5 5
3 3 2 2 9 1, 2, 3, 4, 5 5
From Eqs. (i) and (ii), 1 / 3 ≤ p ≤ 1 / 2 10 1, 2, 3, 4, 5, 6 6
23. Since, determinant is of order 2 × 2 and each element is
Therefore, the number of possible pairs = 38
0 or 1 only. Also, the total number of possible pairs is 10 × 10 = 100
∴ n (S ) = 24 = 16 38
∴ The required probability = 1 − = 1 − 0.38 = 0.62
and the determinant is positive are 100
1 0 1 1 1 0
, , 27. We have 14 seats in two vans and there are 9 boys and 3
0 1 0 1 1 1 girls. The number of ways of arranging 12 people on 14
seats without restriction is
∴ n (E ) = 3
14 !
3
14
P12 = = 7(13 !)
Thus, the required probability = 2!
16
Probability 117

Now, the number of ways of choosing back seats is 2. 2. We have, P (exactly one of A or B occurs)
and the number of ways of arranging 3 girls on adjacent = P ( A ∪ B) − P ( A ∩ B)
seats is 2(3!) and the number of ways of arranging 9 = P ( A ) + P (B) − 2P ( A ∩ B)
boys on the remaining 11 seats is 11 P9 ways.
According to the question,
Therefore, the required number of ways 1
4 ⋅ 3 ! 11 ! P ( A ) + P (B) − 2P ( A ∩ B) = …(i)
= 2. (2 .3 !).11 P9 = = 12 ! 4
2! 1
Hence, the probability of the required event P (B) + P (C ) − 2P (B ∩ C ) = …(ii)
4
12 ! 1 1
= = and P (C ) + P ( A ) − 2P (C ∩ A ) = …(iii)
7 ⋅ 13 ! 91 4
On adding Eqs. (i), (ii) and (iii), we get
28. There are (n + 7) coins in the box out of which five coins
n+7 2 [P ( A ) + P (B) + P (C ) − P ( A ∩ B) − P (B ∩ C )
can be taken out in C5 ways.
3
The total value of 5 coins can be equal to or more than − P (C ∩ A )] =
4
one rupee and fifty paise in the following ways.
⇒ P ( A ) + P (B) + P (C ) − P ( A ∩ B) − P (B ∩ C )
(i) When one 50 paise coin and four 25 paise coins are 3
chosen. − P (C ∩ A ) =
8
(ii) When two 50 paise coins and three 25 paise coins ∴P (atleast one event occurs)
are chosen. = P(A ∪ B ∪ C )
(iii) When two 50 paise coins, 2 twenty five paise coins = P ( A ) + P (B) + P (C ) − P ( A ∩ B) − P (B ∩ C )
and one from n coins of ten and five paise. − P (C ∩ A ) + P ( A ∩ B ∩ C )
∴ The total number of ways of selecting five coins so 3 1 7  1
= + = Q P(A ∩ B ∩ C ) =
that the total value of the coins is not less than one 8 16 16  16 
rupee and fifty paise is 3 1
(2C1 ⋅5 C5 ⋅n C 0 ) + (2C 2 ⋅5 C3 ⋅n C 0 ) + (2C 2 ⋅5 C 2 ⋅n C1 ) 3. Given, P (B) = , P ( A ∩ B ∩ C ) =
4 3
= 10 + 10 + 10n = 10 (n + 2) B
A
(A ∩ B ∩ C)
So, the number of ways of selecting five coins, so
that the total value of the coins is less than one (A ∩ B ∩ C)
rupee and fifty paise is n + 7C5 − 10(n + 2)
n+7 (B ∩ C)
C5 − 10(n + 2)
∴ Required probability = n+7
C5
10 (n + 2) C
=1 − n+7
C5
1
29. (i) The total number of arrangements of six boys and and P(A ∩ B ∩ C ) =
3
six girls = 12 !
6! × 7! 1 which can be shown in Venn diagram.
∴ Required probability = =
(12)! 132 ∴ P (B ∩ C ) = P (B) − { P ( A ∩ B ∩ C + P ( A ∩ B ∩ C ))}
[since, we consider six girls at one person] 3  1 1 3 2 1
= − +  = − =
2 ×6! ×6! 1 4  3 3 4 3 12
(ii) Required probability = =
(12)! 462 4. It is given that, P (E ) ≤ P (F ) ⇒ E ⊆ F …(i)

Topic 2 Addition and Subtraction Law of and P (E ∩ F ) > 0 ⇒ E ⊂ F …(ii)


Probability (a) occurrence of E ⇒ occurrence of F [from Eq. (i)]
1. As, we know that (b) occurrence of F ⇒ occurrence of E [from Eq. (ii)]
P ( A ∩ B) = P ( A ) + P (B) − P ( A ∪ B) (c) non-occurrence of E ⇒ occurrence of F
⇒ P ( A ∩ B) = 0.6 + 0.4 − 0.8 ⇒ P ( A ∩ B) = 0.2 Hence, option (c) is not correct. [from Eq. (i)]
and, as 5. We know that,
P ( A ∪ B ∪ C ) = P ( A ) + P (B) + P (C ) P (exactly one of A or B occurs)
− P ( A ∩ B) − P (B ∩ C ) − P (C ∩ A ) + P ( A ∩ B ∩ C ) = P ( A ) + P (B) − 2P ( A ∩ B)
⇒ α = 0.6 + 0.4 + 0.5 − 0.2 − β − 0.3 + 0.2
∴ P ( A ) + P (B) − 2P ( A ∩ B) = p …(i)
⇒ α = 1.2 − β
Similarly, P (B) + P (C ) − 2P (B ∩ C ) = p …(ii)
Q 0.85 ≤ α ≤ 0.95 ⇒ 0.85 ≤ 1.2 − β ≤ 0.95
⇒ 0.25 ≤ β ≤ 0.35 and P (C ) + P ( A ) − 2P (C ∩ A ) = p …(iii)
118 Probability

On adding Eqs. (i), (ii) and (iii), we get = P (M ∪ N ) − P (M ∩ N ) = P (M ) + P (N ) − 2P (M ∩ N )


2 [P ( A ) + P (B) + P (C ) − P ( A ∩ B) Hence, (a) and (c) are correct answers.
− P (B ∩ C ) − P (C ∩ A )] = 3 p 11. Let E1 be the event getting minimum number 3 and E 2
⇒ P ( A ) + P (B) + P (C ) − P ( A ∩ B) be the event getting maximum number 7.
3p
− P (B ∩ C ) − P (C ∩ A ) = …(v) Then, P (E1 ) = P (getting one number 3 and other two
2 from numbers 4 to 10)
It also given that, P ( A ∩ B ∩ C ) = p2 …(v) 1
C1 × 7C 2 7
= 10 =
∴ P(at least one of the events A, B, and C occurs) C3 40
= P ( A ) + P (B) + P (C ) − P ( A ∩ B)
P (E 2) = P(getting one number 7 and other two from
− P (B ∩ C ) − P (C ∩ A ) + P ( A ∩ B ∩ C ) numbers 1 to 6)
3p 1
C1 × 6C 2 1
= + p2 [from Eqs. (iv) and (v)] = 10 =
2 C3 8
3 p + 2 p2 and P (E1 ∩ E 2) = P(getting one number 3, second
=
2 number 7 and third from 4 to 6)
6. Since, P ( A ∩ B) = P ( A ) ⋅ P (B)
1
C1 × 1C1 × 3C1 1
= 10
=
C3 40
It means A and B are independent events, so A ′ and B ′
are also independent. ∴ P (E1 ∪ E 2) = P (E1 ) + P (E 2) − P (E1 ∩ E 2)
∴ P ( A ∪ B) ′ = P ( A ′∩ B ′ ) = P ( A )′ ⋅ P (B)′ 7 1 1 11
= + − =
Alternate Solution 40 8 40 40
P ( A ∪ B)′ = 1 − P ( A ∪ B) = 1 − { P ( A ) + P (B) − P ( A ) ⋅ P (B)} 12. P ( A ∪ B) = P ( A ) + P (B) − P ( A ∩ B)
= {1 − P ( A )}{1 − P (B)} = P ( A )′ P (B)′ If P ( A ∪ B) = P ( A ∩ B),
7. Given, P ( A ∪ B) = 0.6 , P ( A ∩ B) = 0.2 then P ( A ) and P (B) are equals.
∴ P ( A ) + P (B ) = [1 − P ( A )] + [1 − P (B)] Since, P ( A ∪ B) = P ( A ∩ B) ⇒ A and B are equals sets
= 2 − [P ( A ) + P (B)] Thus, P ( A ) and P (B) is equal to P ( A ∩ B).
= 2 − [P ( A ∪ B) + P ( A ∩ B)] = 2 − [0.6 + 0.2] = 1.2 13. Given, P (A fails in examination) = 0.2
8. Given, P ( A ) = 0.25, P (B) = 0.50, P ( A ∩ B) = 0.14 and P (B fails in examination) = 0.3
∴ P ( A ∪ B) = P ( A ) + P (B) − P ( A ∩ B) P ( A ∩ B) = P ( A )P (B) = (0.2) (0.3)
= 0.25 + 0.50 – 0.14 = 0.61 ∴ P ( A ∪ B) = P ( A ) + P (B) − P ( A ∩ B)
Now, P ( A ∪ B) = 1 − P ( A ∪ B) = 1 − 0.61 = 0.39 = 0.2 + 0.3 − 0.06 = 0.44
9. We know that, Hence, it is a false statement.
P ( A ∩ B) = P ( A ) + P (B) − P ( A ∪ B) 14. Let P ( A ) and P (B) denote respectively the percentage of
Also, P ( A ∪ B) ≤ 1 city population that reads newspapers A and B.
∴ P ( A ∩ B)min. , when P ( A ∪ B)max = 1 Then,
⇒ P ( A ∩ B) ≥ P ( A ) + P (B) − 1 25 1 20 1
P ( A) = = , P (B) = = ,
∴ Option (a) is true. 100 4 100 5
8 2
Again, P ( A ∪ B) ≥ 0 P ( A ∩ B) = = ,
100 25
∴ P ( A ∩ B)max. , when P ( A ∪ B)min. = 0
1 2 17
⇒ P ( A ∩ B) ≤ P ( A ) + P (B) P ( A ∩ B) = P ( A ) − P ( A ∩ B) = − = ,
4 25 100
∴ Option (b) is true. 1 2 3
P ( A ∩ B) = P (B) − P ( A ∩ B) = − =
Also, P ( A ∩ B) = P ( A ) + P (B) − P ( A ∪ B), Thus, (c) is 5 25 25
also correct. Let P (C ) be the probability that the population who
Hence, (a), (b), (c) are correct options. reads advertisements.
∴ P (C ) = 30% of P ( A ∩ B) + 40% of P ( A ∩ B)
10. P(exactly one of M, N occurs)
= P{(M ∩ N ) ∪ (M ∩ N )} = P (M ∩ N ) + P (M ∩ N ) + 50% of P ( A ∩ B)
= P (M ) − P (M ∩ N ) + P (N ) − P (M ∩ N ) [since, A ∩ B, A ∩ B and A ∩ B are all mutually
exclusive]
= P (M ) + P (N ) − 2P (M ∩ N )
3 17 2 3 1 2 139
Also, P(exactly one of them occurs) ⇒ P (C ) = × + × + × = = 13 . 9%
10 100 5 25 2 25 1000
= {1 − P (M ∩ N )}{1 − P (M ∪ N )}
Probability 119

4
15. We know that,  1
 
P ( A ) + P (B) + P (C ) − P ( A ∩ B) − P (B ∩ C )  2 1 1
= = =
− P (C ∩ A ) + P ( A ∩ B ∩ C ) = P ( A ∪ B ∪ C )  1
4 3
 1  1 4  1  1
2 2
1 + 4 + 6 11
  + C3     + C 2   
4
⇒ 0.3 + 0.4 + 0.8 – {0.08 + 0.28 + P (BC )} + 0.09  2  2  2  2  2
= P(A ∪ B ∪ C )
⇒ 1.23 − P (BC ) = P ( A ∪ B ∪ C ) 2. Key Idea Use P ( A) = 1 − P ( A) and condition of independent
where, 0.75 ≤ P ( A ∪ B ∪ C ) ≤ 1 events i.e P ( A ∩ B) = P ( A) ⋅ P ( B)

⇒ 0.75 ≤ 1.23 − P (BC ) ≤ 1 ⇒ − 0.48 ≤ − P (BC ) ≤ − 0.23 Given that probability of hitting a target independently
⇒ 0.23 ≤ P (BC ) ≤ 0.48 by four persons are respectively
1 1 1 1
16. Given, P ( A ) = 0.5 and P ( A ∩ B) ≤ 0.3 P1 = , P2 = , P3 = and P4 =
2 3 4 8
⇒ P ( A ) + P (B) − P ( A ∪ B) ≤ 0.3 Then, the probability of not hitting the target is
⇒ P (B) ≤ 0.3 + P ( A ∪ B) − P ( A ) ≤ P ( A ∪ B) − 0.2  1  1  1  1
= 1 −  1 −  1 −  1 − 
[since, P ( A ∪ B) ≤ 1 ⇒ P ( A ∪ B) − 0.2 ≤ 0.8 ]  2  3  4  8
∴ P (B) ≤ 0.8 [Q events are independent]
1 2 3 7 7
⇒ P (B) cannot be 0.9. =× × × =
2 3 4 8 32
17. Here, five students S1 , S 2, S3 , S 4 and S5 and five seats Therefore, the required probability of hitting the target
R1 , R2, R3 , R4 and R5 = 1 − (Probability of not hitting the target)
∴ Total number of arrangement of sitting five students =1−
7 25
=
is 5 ! = 120 32 32
Here, S1 gets previously alloted seat R1 P ( A ∩ B)
3. We know that, P( A / B) =
∴S 2, S3 , S 4 and S5 not get previously seats. P (B)
Total number of way S 2, S3 , S 4 and S5 not get previously [by the definition of conditional probability]
seats is Q A⊂B
 1 1 1 1  1 1 1 ⇒ A∩B= A
4 ! 1 − + − +  = 24 1 − 1 + − + 
 1 ! 2 ! 3 ! 4 !  2 6 24 P ( A)
∴ P( A / B) = …(i)
 12 − 4 + 1 P (B)
= 24   =9 As we know that, 0 ≤ P (B) ≤ 1
 24 
1 P ( A)
9 3 ∴ 1≤ < ∞ ⇒ P ( A) ≤ <∞
∴ Required probability = = P (B) P (B)
120 40
P ( A)
⇒ ≥ P ( A) …(ii)
18. Here, n (T1 ∩ T2 ∩ T3 ∩ T4 ) P (B)
Total = − n (T1 ∪ T2 ∪ T3 ∪ T4 ) Now, from Eqs. (i) and (ii), we get
⇒ n (T1 ∩ T2 ∩ T3 ∩ T4 ) P( A/B) ≥ P(A)
= 5 ! − [4C1 4 ! 2 ! − (3 C1 3 ! 2 ! + 3C1 3 ! 2 ! 2 !) 4. In {1, 2, 3, ...., 11} there are 5 even numbers and 6 odd
+ (2C1 2 ! 2 ! + 4C1 2 ⋅ 2 !) − 2] numbers. The sum even is possible only when both are
⇒ n (T1 ∩ T2 ∩ T3 ∩ T4 ) odd or both are even.
= 120 − [192 − (36 + 72) + (8 + 16) − 2] Let A be the event that denotes both numbers are even
= 120 − [192 − 108 + 24 − 2] = 14 and B be the event that denotes sum of numbers is even.
Then, n ( A ) = 5C 2 and n (B) = 5C 2 + 6C 2
14 7
∴ Required probability = = Required probability
120 60 P ( A ∩ B) 5
C / 11C
P ( A / B) = = 6 2 5 2
P (B) ( C 2 + C 2)
Topic 3 Independent and Conditional 11
C2
Probability 5
C2 10 2
1. Let event B is being boy while event G being girl. = 6 = =
C 2 + 5C 2 15 + 10 5
1
According to the question, P (B) = P (G ) = 1
2 5. Clearly, P (H ) = Probability of getting head =
Now, required conditional probability that all children 2
1
are girls given that at least two are girls, is and P (T ) = Probability of getting tail =
All 4 girls 2
=
(All 4 girls ) + (exactly 3 girls + 1 boy) Now, let E1 be the event of getting a sum 7 or 8, when a
+ (exactly 2 girls + 2 boys) pair of dice is rolled.
120 Probability

Then, E1 = {(6, 1), (5, 2), (4, 3), (3, 4), (2, 5), 8. PLAN It is simple application of independent event, to solve a
(1, 6), (6, 2), (5, 3), (4, 4), (3, 5), (2, 6)} certain problem or any type of compitition each event in
independent of other.
⇒ P (E1 ) = Probability of getting 7 or 8 when a pair of
11 Formula used
dice is thrown =
36 P ( A ∩ B) = P ( A ) ⋅ P (B), when A and B are independent
Also, let P (E 2) = Probability of getting 7 or 8 when a events.
2 Probability that the problem is solved correctly by
card is picked from cards numbered 1, 2, ...., 9 = atleast one of them = 1 − (problem is not solved by all)
9
∴Probability that the noted number is 7 or 8 ∴ P (problem is solved) = 1 − P (problem is not solved)
= P ((H ∩ E1 ) or (T ∩ E 2)) = 1 − P ( A ) ⋅ P (B ) ⋅ P (C ) ⋅ P (D )
= P (H ∩ E1 ) + P (T ∩ E 2)
 1 1 3 7 21 235
[Q (H ∩ E1 ) and (T ∩ E 2) are mutually exclusive] =1 −  ⋅ ⋅ ⋅  =1 − =
 2 4 4 8 256 256
= P (H ) ⋅ P (E1 ) + P (T ) ⋅ P (E 2)
2
[Q{ H , E1 } and {T , E 2} both are sets of 9. Since, P ( A ) =
5
independent events]
1 11 1 2 19 For independent events,
= × + × = P ( A ∩ B) = P ( A )P (B)
2 36 2 9 72
2 1 2 3 4
6. Clearly, E1 = {(4, 1), (4, 2), (4, 3), (4, 4), (4, 5), (4, 6)} ⇒ P ( A ∩ B) ≤ ⇒ P ( A ∩ B) = , , ,
5 10 10 10 10
E 2 = {(1, 2), (2, 2), (3, 2), (4, 2), (5, 2), (6, 2)} [maximum 4 outcomes may be in A ∩ B]
and E3 = {(1, 2), (1, 4), (1, 6), (2, 1), (2, 3), (2, 5), 1
(i) Now, P ( A ∩ B) =
(3, 2), (3, 4), (3, 6), (4, 1), (4, 3), (4, 5), 10
(5, 2), (5, 4), (5, 6), (6, 1), (6, 3), (6, 5)} 1
6 1 6 1 ⇒ P ( A ) . P (B) =
⇒ P (E1 ) = = , P (E 2) = = 10
36 6 36 6 1 5 1
18 1 ⇒ P (B) = × = , not possible
and P (E3 ) = = 10 2 4
36 2 2 2 2
(ii) Now, P ( A ∩ B) = ⇒ × P (B) =
Now, P (E1 ∩ E 2) = P (getting 4 on die A and 2 on die B) 10 5 10
1 5
= = P (E1 ) ⋅ P (E 2) ⇒ P (B) = , outcomes of B = 5
36 10
3
P (E 2 ∩ E3 ) = P (getting 2 on die B and sum of numbers (iii) Now, P ( A ∩ B) =
on both dice is odd) 10
3 3 2 3
= = P (E 2) ⋅ P (E3 ) ⇒ P ( A )P (B) = ⇒ × P (B) =
36 10 5 10
P (E1 ∩ E3 ) = P (getting 4 on die A and sum of numbers 3
P (B) = , not possible
on both dice is odd) 4
3 4 4
= = P (E1 ) ⋅ P (E3 ) (iv) Now, P ( A ∩ B) = ⇒ P ( A ) . p(B) =
36 10 10
and P (E1 ∩ E 2 ∩ E3 ) = P [getting 4 on die A, 2 on die B ⇒ P (B) = 1 , outcomes of B = 10.
and sum of numbers is odd]
E ∩F c c
P (E c ∩ F c ∩ G )
= P (impossible event) = 0 10. P   =
Hence, E1, E 2 and E3 are not independent.  G  P (G )
1 1 1 P (G ) − P (E ∩ G ) − P (G ∩ F )
7. Given, P ( A ∪ B) = , P ( A ∩ B) = , P ( A ) = =
6 4 4 P (G )
1 5 P (G ) [1 − P (E ) − P (F )]
∴ P ( A ∪ B) = 1 − P ( A ∪ B) = 1 − = = [Q P (G ) ≠ 0]
6 6 P (G )
1 3 = 1 − P (E ) − P (F ) = P (E c ) − P (F )
and P ( A) = 1 − P ( A ) = 1 − =
4 4
11. Let E = event when each American man is seated
∴ P ( A ∪ B) = P ( A ) + P (B) − P ( A ∩ B) adjacent to his wife
5 3 1
= + P (B) − and A = event when Indian man is seated adjacent
6 4 4 to his wife
1
⇒ P (B) = ⇒ A and B are not equally likely Now, n ( A ∩ E ) = (4 !) × (2 !)5
3
1 Even when each American man is seated adjacent to his
P ( A ∩ B) = P ( A ) ⋅ P (B) = wife.
4
So, events are independent. Again, n (E ) = (5 !) × (2 !)4
Probability 121

 A  n ( A ∩ E ) (4 !) × (2 !)
5
2 18. Let A, B and C denote the events of passing the tests I,
∴ P  = = =
 E n (E ) (5 !) × (2 !) 4
5 II and III, respectively.

Alternate Solution Evidently A, B and C are independent events.

Fixing four American couples and one Indian man in According to given condition,
1
between any two couples; we have 5 different ways in = P [( A ∩ B) ∪ ( A ∩ C )]
which his wife can be seated, of which 2 cases are 2
favourable. = P ( A ∩ B) + P ( A ∩ C ) − P ( A ∩ B ∩ C )
2 = P ( A ) P (B) + P ( A ) ⋅ P (C ) − P ( A ) ⋅ P (B) ⋅ P (C )
∴ Required probability =
5 1 1
= pq + p ⋅ − pq ⋅
12. Let E be the event of getting 1 on a die. 2 2
1 5 ⇒ 1 = 2 pq + p − pq ⇒ 1 = p(q + 1) …(i)
⇒ P (E ) = and P (E ) =
6 6 The values of option (c) satisfy Eq. (i).
∴ P (first time 1 occurs at the even throw) [Infact, Eq. (i) is satisfied for infinite number of values
= t2 or t4 or t6 or t8 ... and so on of p and q. If we take any values of q such that 0 ≤ q ≤ 1,
1
= { P (E )P (E )} + { P (E ) P (E ) P (E ) P (E )} + K ∞ then, p takes the value . It is evident that,
5 q+1
3 5 1
          
5 1 5 1 5 1 36 5 0< ≤ 1 i.e. 0 < p ≤ 1. But we have to choose correct
=    +     +     +K∞ = = q+1
 6  6  6  6  6  6 25 11
1−
36 answer from given ones.]

13. Probability that only two tests are needed = Probability 19. Since, P ( A / B ) + P ( A / B ) = 1
that the first machine tested is faulty × Probability that ∴ P(A / B) = 1 − P(A / B)
2 1 1 20. Given that, P ( A ) = 0.4, P ( A ) = 0.6
the second machine tested is faulty = × =
4 3 6
P(the event A happens at least once)
14. The event that the fifth toss results in a head is = 1 − P (none of the event happens)
independent of the event that the first four tosses result = 1 − (0.6) (0.6) (0.6) = 1 − 0.216 = 0.784
in tails.
∴ Probability of the required event = 1 / 2 1
21. P (X ) =
3
15. P (2 white and 1 black) = P (W1W 2B3 or W1B2W3 or  X  P (X ∩ Y ) 1
P  = =
Y  P (Y ) 2
B1W 2W3 )
= P (W1W 2B3 ) + P (W1B2W3 ) + P (B1W 2W3 )  Y  P (X ∩ Y ) 2
P  = =
 X P (X ) 5
= P (W1 )P (W 2)P (B3 ) + P (W1 )P (B2)P (W3 )
2
+ P (B1 )P (W 2)P (W3 ) P (X ∩ Y ) =
3.2.3 3.2.1 1.2.1 1 13 15
= + + = (9 + 3 + 1) = 4
4 4 4 4 4 4 4 4 4 32 32 P (Y ) =
15
16. Given, P (India wins) = 1/2 4 2

∴ P (India losses ) = 1 / 2  X ′  P (Y ) − P (X ∩ Y ) 15 15 1
P  = = =
Y  P (Y ) 4 2
Out of 5 matches India’s second win occurs at third test.
⇒ India wins third test and simultaneously it has won 15
1 4 2 7 7
one match from first two and lost the other. P (X ∪ Y ) = + − = =
∴ Required probability = P (LWW ) + P (WLW ) 3 15 15 15 15
3 3
 1  1 1 22. PLAN
=  +  = P ( A ∩ B)
 2  2 4 (i) Conditional probability, i.e. P( A / B) =
P( B)
17. Let A = getting not less than 2 and not greater than 5 (ii) P ( A ∪ B) = P( A ) + P( B) − P ( A ∩ B)
4 (iii) Independent event, then P ( A ∩ B) = P( A ) ⋅ P( B)
⇒ A ={2, 3, 4, 5} ⇒ P ( A ) =
6 1 Y  1
Here, P (X /Y ) = ,P  =
But die is rolled four times, therefore the probability in 2  X 3
getting four throws and P (X ∩ Y ) = 6
 4  4  4  4 16  X  P (X ∩ Y )
=     = ∴ P  =
 6  6  6  6 81 Y  P (Y )
122 Probability

1 1 /6 1 24. Let A, B and C respectively denote the events that the


⇒ = ⇒ P (Y ) = …(i)
2 P (Y ) 3 student passes in Maths, Physics and Chemistry.
Y  1 P (X ∩ Y ) 1 It is given,
P  = ⇒ =
 X 3 P (X ) 3 P ( A ) = m, P (B) = p and P (C ) = c and
1 1
⇒ = P (X ) P (passing atleast in one subject)
6 3
1 = P ( A ∪ B ∪ C ) = 0.75
∴ P (X ) = …(ii)
2 ⇒ 1 − P ( A′ ∩ B ′ ∩ C ′ ) = 0.75
P (X ∪ Y ) = P (X ) + P (Y ) − P (X ∩ Y ) Q [P ( A ) = 1 − P ( A )
1 1 1 2
= + − = …(iii) and [P ( A ∪ B ∪ C ] = P ( A′ ∩ B′ ∩ C′ )]
2 3 6 3
1 1 1 1 ⇒ 1 − P ( A′ ) . P (B ′ ) . P (C′ ) = 0.75
P (X ∩ Y ) = and P (X ) ⋅ P (Y ) = ⋅ =
6 2 3 6 Q A, B and C are independent events, therefore A′, B′
⇒ P (X ∩ Y ) = P (X ) ⋅ P (Y ) and C ′ are independent events.
i.e. independent events ⇒ 0.75 = 1 − (1 − m) (1 − p) (1 − c)
∴ P (X c ∩ Y ) = P (Y ) − P (X ∩ Y ) ⇒ 0 .25 = (1 − m) (1 − p) (1 − c) …(i)
1 1 1
= − = Also, P (passing exactly in two subjects)= 0.4
3 6 6
⇒ P ( A ∩ B ∩ C ∪ A ∩ B ∩ C ∪ A ∩ B ∩ C ) = 0.4
23. E F
⇒ P ( A ∩ B ∩ C ) + P ( A ∩ B ∩ C ) + P ( A ∩ B ∩ C ) = 0.4
⇒ P ( A ) P (B) P (C ) + P ( A )P (B ) P (C )
+ P ( A ) P (B) P (C ) = 0.4
⇒ pm (1 − c) + p(1 − m) c + (1 − p) mc = 0.4
⇒ pm − pmc + pc − pmc + mc − pmc = 0.4 …(ii)
11
P (E ∪ F ) − P (E ∩ F ) = …(i)
25
Again, P (passing atleast in two subjects) = 0.5
[i.e. only E or only F]
⇒ P(A ∩ B ∩ C ) + P(A ∩ B ∩ C )
+ P ( A ∩ B ∩ C ) + P ( A ∩ B ∩ C ) = 0.5
⇒ pm(1 − c) + pc(1 − m) + cm(1 − p) + pcm = 0.5
E F
⇒ pm − pcm + pc − pcm + cm − pcm + pcm = 0.5
⇒ ( pm + pc + mc) − 2 pcm = 0.5 …(iii)
From Eq. (ii),
2 pm + pc + mc − 3 pcm = 0.4 …(iv)
Neither of them occurs = From Eq. (i),
25
2 0.25 = 1 − (m + p + c) + ( pm + pc + cm) − pcm …(v)
⇒ P (E ∩ F ) = …(ii)
25 On solving Eqs. (iii), (iv) and (v), we get
11 p + m + c = 1.35 = 27 / 20
From Eq. (i), P (E ) + P (F ) − 2 P (E ∩ F ) = …(iii)
25 Therefore, option (b) is correct.
From Eq. (ii), ( 1 − P (E )) ( 1 − P (F )) =
2 Also, from Eqs. (ii) and (iii), we get pmc = 1 / 10
25 Hence, option (c) is correct.
2 P (E ∩ F ) P (E ∩ F )
⇒ 1 − P (E ) − P (F ) + P (E ) ⋅ P (F ) = …(iv) 25. (a) P (E / F ) + P (E / F ) = +
25 P (F ) P (F )
From Eqs. (iii) and (iv),
P (E ∩ F ) + P (E ∩ F ) P (F )
7 12 = = =1
P (E ) + P (F ) = and P (E ) ⋅ P (F ) = P (F ) P (F )
5 25
Therefore, option (a) is correct.
7  12 7 12
∴ P (E ) ⋅ − P (E ) = ⇒ (P (E ))2 − P (E ) + =0 P (E ∩ F ) P (E ∩ F )
 5  25 5 25 (b) P (E / F ) + P (E / F ) = +
P (F ) P (F )
 3  4
⇒ P (E ) − P (E ) − =0 P (E ∩ F ) P (E ∩ F )
 5   5  = + ≠1
3 4 4 3 P (F ) 1 − P (F )
∴ P (E ) = or ⇒ P (F ) = or
5 5 5 5 Therefore, option (b) is not correct.
Probability 123

P (E ∩ F ) P (E ∩ F ) Now, P (E ∩ F ) = P (E ) − P (E ∩ F ) = P (E ) − P (E ) ⋅ P (F )
(c) P (E / F ) + P (E / F ) = +
P (F ) P (F ) = P (E ) [1 − P (F )] = P (E ) ⋅ P (F )
P (E ∩ F ) P (E ∩ F ) and P (E ∩ F ) = P (E ∪ F ) = 1 − P (E ∪ F )
= + ≠1
P (F ) 1 − P (F ) = 1 − [1 − P (E ) ⋅ P (F )]
Therefore, option (c) is not correct. [Q E and F are independent]
P (E ∩ F ) P (E ∩ F ) = P (E ) ⋅ P (F )
(d) P (E / F ) + P (E / F ) = +
P (F ) P (F ) So, E and F as well as E and F are independent events.
P (E ∩ F ) + P (E ∩ F ) P (F ) P (E ∩ F ) + P (E ∩ F )
= = =1 Now, P (E / F ) + P (E / F ) =
P (F ) P (F ) P (F )
Therefore, option (d) is correct. P (F )
= =1
1 P (F )
26. Both E and F happen ⇒ P (E ∩ F ) =
12 29. P ( A c ) = 0.3 [given]
1
and neither E nor F happens ⇒ P (E ∩ F ) = ⇒ P ( A ) = 0.7
2
P (B) = 0.4 [given]
But for independent events, we have ⇒ P (Bc ) = 0.6 and P ( A ∩ Bc ) = 0.5 [given]
1
P (E ∩ F ) = P (E ) P (F ) = …(i) Now, P ( A ∪ Bc ) = P ( A ) + P (Bc ) − P ( A ∩ Bc )
12
= 0.7 + 0.6 − 0.5 = 0.8
and P (E ∩ F ) = P (E ) P (F ) P{ B ∩ ( A ∪ Bc )}
= {1 − P (E )}{(1 − P (F )} ∴ P [B / ( A ∪ B ] =
c
P ( A ∪ Bc )
= 1 − P (E ) − P (F ) + P (E )P (F )
P{(B ∩ A ) ∪ (B ∩ Bc )} P{(B ∩ A ) ∪ φ } P (B ∩ A )
1 1 = = =
⇒ = 1 − { P (E ) + P (F )} + 0.8 0.8 0.8
2 12 1
1 1 7 = [P ( A ) − P ( A ∩ B )]
c
⇒ P (E ) + P (F ) = 1 − + = …(ii) 0.8
2 12 12
0.7 − 0.5 0.2 1
On solving Eqs. (i) and (ii), we get = = =
0.8 0.8 4
1 1
either P (E ) = and P (F ) = 30. P ( A ∪ B) = P ( A ) + P (B) − P ( A ) P (B), as A and B are
3 4
1 1 independent events.
or P (E ) = and P (F ) = ⇒ 0.8 = (0.3) + P (B) − (0.3) P (B)
4 3
5
27. We know that, ⇒ 0.5 = (0.7) P (B) ⇒ P (B) =
7
 A  P ( A ∩ B) P ( A ) + P (B) − P ( A ∪ B)
P  = = 31. 5 can be thrown in 4 ways and 7 can be thrown in 6
 B P (B) P (B)
ways, hence number of ways of throwing neither 5 nor 7
Since, P ( A ∪ B) < 1 is 36 − (4 + 6) = 26
⇒ − P ( A ∪ B ) > −1 ∴ Probability of throwing a five in a single throw with a
4 1
⇒ P ( A ) + P (B) − P ( A ∪ B) > P ( A ) + P (B) − 1 pair of dice = = and probability of throwing neither
36 9
P ( A ) + P (B) − P ( A ∪ B) P ( A ) + P (B) − 1 26 13
⇒ > 5 nor 7 = =
P (B) P (B) 36 18
 A  P ( A ) + P (B) − 1 Hence, required probability
⇒ P  >
 B P (B) 1
2
 1  13  1  13  1 2
Hence, option (a) is correct. =   +     +     + ... = 9 =
 9  18  9  18  9 13 5
The choice (b) holds only for disjoint i.e. P ( A ∩ B) = 0 1−
18
Finally, P ( A ∪ B) = P ( A ) + P (B) − P ( A ∩ B)
32. Let R be drawing a red ball and B for drawing a black
= P ( A ) + P (B) − P ( A ) ⋅ P (B), ball, then required probability
if A , B are independent = RRR + RBR + BRR + BBR
= 1 − {1 − P ( A )} {1 − P (B)} 6 5 6 6 6 5
= × × + × × 
= 1 − P ( A ) ⋅ P (B )  10 11 10  10 11 10
Hence, option (c) is correct, but option (d) is not correct. 4 4 7  4 7 6
+ × × + × × 
28. Since, E and F are independent events. Therefore,  10 11 10  10 11 10
P (E ∩ F ) = P (E ) ⋅ P (F ) ≠ 0, so E and F are not mutually =
640 32
=
exclusive events. 1100 55
124 Probability

33. Let A be the event that the maximum number on the For n = 2, p2 = 1 − p (two heads simultaneously occur).
two chosen tickets is not more than 10, and B be the = 1 − p(HH ) = 1 − pp = 1 − p2
event that the minimum number on them is 5
5
For n ≥ 3, pn = pn − 1 (1 − p) + pn − 2(1 − p) p
C
∴ P ( A ∩ B) = 100 1 ⇒ pn = (1 − p) pn − 1 + p(1 − p) pn − 2
C2
Hence proved.
10
C
and P ( A ) = 100 2 38. Let, E1 = the event noted number is 7
C2
E 2 = the event noted number is 8
 B  P ( A ∩ B) H = getting head on coin
Then P  =
 A P ( A) T = be getting tail on coin
5
C1 1 ∴ By law of total probability,
= 10 =
C2 9 P (E1 ) = P (H ) ⋅ P (E1 / H ) + P (T ) ⋅ P (E1 / T )
34. Here, P ( A ∪ B) . P ( A′ ∩ B ′ ) and P (E 2) = P (H ) ⋅ P (E 2 / H ) + P (T ) ⋅ P (E 2 / T )
where, P (H ) = 1 / 2 = P (T )
⇒ { P ( A ) + P (B) − P ( A ∩ B)}{ P ( A′ ) . P (B ′ )}
[since A, B are independent, so A ′ , B ′ are independent] P (E1/H ) = probability of getting a sum of 7 on two dice
∴ P ( A ∪ B) . P ( A ′ ∩ B ′ ) ≤ { P ( A ) + P (B)}. { P ( A′ ) . P (B′ )} Here, favourable cases are
= P ( A ) . P ( A′ ) . P (B′ ) + P (B) . P ( A′ ) . P (B′ ) {(1, 6), (6, 1), (2, 5), (5, 2), (3, 4), (4, 3)}.
≤ P ( A ) . P (B′ ) + P (B) . P ( A′ ) 6 1
…(i) ∴ P (E1 / H ) = =
36 6
[Q P ( A′ ) ≤ 1 and P (B ′ ) ≤ 1]
Also, P (E1 / T ) = probability of getting 7 numbered
⇒ P ( A ∪ B) P ( A ′ ∩ B ′ ) ≤ P ( A ) . P (B ′ ) + P (B) . P ( A′ )
.
card out of 11 cards
⇒ P ( A ∪ B) . P ( A′ ∩ B ′ ) ≤ P (C )
1
[Q P (C ) = P ( A ) . P (B ′ ) + P (B). P ( A′ )] =
11
2
35. Given, P ( A ) = probability that A will hit B = P (E 2 / H ) = probability of getting a sum of 8 on two dice
3
1 Here, favourable cases are
P (B) = probability that B will hit A =
2 {(2, 6), (6, 2), (4, 4), (5, 3), (3, 5)}.
1 5
P (C ) = probability that C will hit A = ∴ P (E 2 / H ) =
3 36
P (E ) = probability that A will be hit P (E 2 / T ) = probability of getting ‘8’ numbered
1 2 2
⇒ P (E ) = 1 − P (B ) ⋅ P (C ) = 1 − ⋅ = card out of 11 cards
2 3 3
= 1 / 11
Probability if A is hit by B and not by C
1.2  1 1  1 1  1 1 17
∴ P (E1 ) =  ×  +  ×  = + =
P (B) . P (C ) 2 3 1  2 6  2 11 12 22 132
= P (B ∩ C / E ) = = =
P (E ) 2 2 1 5  1 1 
and P (E 2) =  ×  +  × 
3  2 36  2 11
36. Let Ei denotes the event that the students will pass the 1  91  91
ith exam, where i = 1, 2, 3 =   =
2  396 729
and E denotes the student will qualify.
Now, E1 and E 2 are mutually exclusive events.
∴ P (E ) = [P (E1 ) × P (E 2 / E1 )]
Therefore,
+ [P (E1 ) × P (E 2′ /E1 ) × P (E3 / E 2′ )]
17 91 193
+ [P (E1′ ) × P (E 2 / E ′1 ) × P (E3 / E 2)] P (E1 or E 2) = P (E1 ) + P (E 2) = + =
132 792 792
p p
= p + p(1 − p) . + (1 − p) . . p
2
39. Let D1 denotes the occurrence of a defective bulb in Ist
2 2
draw.
2 p2 + p2 − p3 + p2 − p3
⇒ P (E ) = = 2 p2 − p3 Therefore, P (D1 ) =
50 1
=
2 100 2
37. Since, pn denotes the probability that no two (or more) and let D2 denotes the occurrence of a defective bulb in
consecutive heads occur. IInd draw.
50 1
⇒ pn denotes the probability that 1 or no head occur. Therefore, P (D2) = =
100 2
For n = 1 , p1 = 1 because in both cases we get less than and let N 1 denotes the occurrence of non-defective bulb
two heads (H, T). in Ist draw.
Probability 125

Therefore, P (N 1 ) =
50 1
= ⇒ 6 x2 − 5 x + 1 = 0
100 2 ⇒ (3x − 1)(2x − 1) = 0
Again, let N 2 denotes the occurrence of non-defective 1 1
bulb in IInd draw. ⇒ x = and
50 1 3 2
Therefore, P (N 2) = = 1 1
100 2 ∴ P ( A ) = or
Now, D1 is independent with N 1and D2 is independent 3 2
with N 2 . 42. P (N th draw gives 2nd ace)
According to the given condition, = P{ 1 ace and (n − 2) other cards are drawn in (N − 1)
A = {the first bulb is defective} = { D1D2, D1N 2} draws} × P { N th draw is 2nd ace}
B = {the second bulb is non-defective} = { D1N 2, N 1N 2} 4 ⋅ (48)! ⋅ (n − 1)! (52 − n )! 3
= ⋅
and C = {the two bulbs are both defective} (52)! ⋅ (n − 2)! (50 − n )! (53 − n )
= { D1D2, N 1N 2} 4(n − 1)(52 − n )(51 − n ) ⋅ 3
Again, we know that, =
52 ⋅ 51 ⋅ 50 ⋅ 49
A ∩ B = { D1N 2}, B ∩ C = { N 1N 2}. (n − 1) (52 − n ) (51 − n )
=
C ∩ A = { D1D2} and A ∩ B ∩ C = φ 50 ⋅ 49 ⋅ 17 ⋅ 13
Also, P ( A ) = P{ D1D2} + P{ D1N 2} 43. Let P (H 1 ) = 0.4, P (H 2) = 0.3, P (H 3 ) = 0.2, P (H 4 ) = 0.1
= P (D1 )P (D2) + P (D1 )P (N 2) P (gun hits the plane)
 1  1  1  1 1 = 1 − P(gun does not hit the plane)
=   +   =
 2  2  2  2 2
= 1 − P (H 1 ) ⋅ P (H 2) ⋅ P (H 3 ) ⋅ P (H 4 )
1 1 = 1 − (0.6) (0.7) (0.8) (0.9) = 1 − 0.3024 = 0.6976
Similarly, P (B) = and P (C ) =
2 2
44. Since, the drawn balls are in the sequence black, black,
 1  1 1
Also, P ( A ∩ B) = P (D1N 2) = P (D1 )P (N 2) =     = white, white, white, white, red, red and red.
 2  2 4
Let the corresponding probabilities be
1 1
Similarly, P (B ∩ C ) = , P (C ∩ A ) = p1 , p2,... , p9
4 4
and P ( A ∩ B ∩ C ) = 0. 2 1 4 3 2
Then, p1 = , p2 = , p3 = , p4 = , p5 =
Since, P ( A ∩ B) = P ( A )P (B), P (B ∩ C ) = P (B)P (C ) 9 8 7 6 5
and P (C ∩ A ) = P (C )P ( A ). 1 3 2
p6 = , p7 = , p8 = , p9 = 1
Therefore, A, B and C are pairwise independent. 4 3 2
Also, P ( A ∩ B ∩ C ) ≠ P ( A )P (B)P (C ) therefore A, B and ∴ Required probabilitie
C cannot be independent. p1 . p2 . p3 ⋅ K ⋅ p9
40. The total number of ways to answer the question  2  1  4   3   2  1   3  2  1
=                 (1) =
 9  8  7  6   5  4   3  2  1260
= C1 + C 2 + C3 + C 4 = 2 − 1 = 15
4 4 4 4 4

P(getting marks) = P( correct answer in I chance) 45. Given sample space (S) of all 3 × 3 matrices with entries
+ P(correct answer in II chance) from the set {0, 1} and events
+ P( correct answer in III chance) E1 = { A ∈ S : det( A ) = 0} and
1  14 1   14 13 1  3 1 E 2 = { A ∈ S : sum of entries of A is 7}.
= + ⋅  + ⋅ ⋅ = =
15  15 14   15 14 13  15 5 For event E 2, means sum of entries of matrix A is 7,
1 1 then we need seven 1s and two 0s.
41. Given, P ( A ) ⋅ P (B) = , P ( A ) ⋅ P (B ) = 9!
6 3 ∴Number of different possible matrices =
1 7!2!
∴ [1 − P ( A )] [1 − P (B)] = ⇒ n (E 2) = 36
3
Let P ( A ) = x and P (B) = y For event E1 ,| A|= 0, both the zeroes must be in same
row/column.
1 1
⇒ (1 − x)(1 − y) = and xy = ∴ Number of matrices such that their determinant is
3 6
zero
1 1
⇒ 1 − x − y + xy = and xy = 3!
= 6 × = 18 = n (E1 ∩ E 2)
3 6 2!
5 1  E  n (E1 ∩ E 2)
⇒ x+ y= and xy = ∴Required probability, P  1  =
6 6  E 2 n (E 2)
5  1
⇒ x  − x = 18 1
6  6 = = = 0.50
36 2
126 Probability

46. PLAN 3. Let x = P (computer turns out to be defective, given that


Forthe events to be independent, it is produced in plant T2)
P( E1 ∩ E 2 ∩ E 3 ) = P( E1 ) ⋅ P( E 2 ) ⋅ P( E 3 )  D
P( E1 ∩ E 2 ∩ E 3 ) = P(only E1 occurs)
⇒ x= P   …(i)
 T2
= P( E1 ) ⋅ (1 − P( E 2 )) (1 − P( E 3 ))
where, D = Defective computer
Let x, y and z be probabilities of E1 , E 2 and E3 ,
respectively. ∴ P (computer turns out to be defective given that is
∴ α = x (1 − y) (1 − z ) …(i) produced in plant T1) = 10x
β = (1 − x) ⋅ y(1 − z ) …(ii)  D
i.e. P   = 10x …(ii)
γ = (1 − x) (1 − y)z …(iii)  T1 
⇒ p = (1 − x) (1 − y) (1 − z ) …(iv) 20 80
Also, P (T1 ) = and P (T2) =
Given, (α − 2 β ) p = αβ and ( β − 3γ ) p = 2 βγ …(v) 100 100
From above equations, x = 2 y and y = 3z Given, P (defective computer) =
7
∴ x = 6z 100
x 7
⇒ =6 i.e. P (D ) =
z 100
Using law of total probability,
47. Here, P (X > Y ) = P (T1win ) P (T1 win )
 D  D
+ P (T1 win ) P (draw ) + P (draw ) P (T1 win ) P (D ) = 9(T1 ) ⋅ P   + P (T2) ⋅ P  
 1 1  1 1  1 1 5  T1   T2
= ×  + ×  + ×  =
 2 2  2 6  6 2 12 7  20   80 
∴ =  ⋅ 10x +  ⋅x
48. P [X = Y ] = P (draw ) ⋅ P (draw ) 100  100  100

+ P (T1 win ) P (T2 win ) + P (T2 win ) ⋅ P (T1 win ) 1


⇒ 7 = (280)x ⇒ x = …(iii)
= (1 / 6 × 1 / 6) + (1 / 2 × 1 / 3) + (1 / 3 × 1 / 2) = 13 / 36 40
 D 1  D  10
∴ P  = and P   =
Topic 4 Law of Total Probability and  T2 40  T1  40
Baye’s Theorem  D 1 39  D 10 30
⇒ P   =1− = and P   = 1 − = …(iv)
1. Let A be the event that ball drawn is given and B be the  T2 40 40  T1  40 40
event that ball drawn is red.
Using Baye’s theorem,
2 5
∴ P ( A ) = and P (B) = T  P (T2 ∩ D )
7 7 P  2 =
 D  P (T1 ∩ D ) + P (T2 ∩ D )
Again, let C be the event that second ball drawn is red.
∴ P (C ) = P ( A ) P (C / A ) + P (B) P (C / B)  D
P (T2) ⋅ P  
2 6 5 4  T2
= × + × =
7 7 7 7  D  D
P (T1 ) ⋅ P   + P (T2) ⋅ P  
12 + 40 32  T1   T2
= =
49 49 80 39

2. Key idea Use the theorem of total probability 100 40 78
= =
Let E1 = Event that first ball drawn is red 20 30 80 39 93
⋅ + ⋅
E 2 = Event that first ball drawn is black 100 40 100 40
A = Event that second ball drawn is red 4. From the tree diagram, it follows that
4  A 6 S
P (E1 ) = , P   =
10  E1  12 4 1
5 5
6  A 4
⇒ P (E 2) = , P  = G R
10  E 2 12
3 1 3 1
By law of total probability 4 4 4 4
 A  A
P ( A ) = P (E1 ) × P   + P (E 2) × P   AG AR AR 1 AG
 E1   E 2 3 1 3 11 3 4 3
4 4 4 44 4 4
4 6 6 4 24 + 24 48 2
= × + × = = = BG BR BG BR BG
10 12 10 12 120 120 5
Probability 127

46 1 1 1
P (BG ) = Given, P (E1 ) = , P (E 2) = , P (E3 ) =
80 2 4 4
10 5 P (E1 ∩ E 2 ∩ E3 ) + P (E1 ∩ E 2 ∩ E3 ) 
P (BG|G ) = = ∴ P(X) = 
16 8 
+ P (E1 ∩ E 2 ∩ E3 ) + P (E1 ∩ E 2 ∩ E3 ) 
5 4 1
P (BG ∩ G ) = × =
8 5 2  1 1 3 1 3 1 1 1 1  1 1 1
= ⋅ ⋅ + ⋅ ⋅ + ⋅ ⋅ + ⋅ ⋅ 
1  2 4 4 2 4 4 2 4 4  2 4 4
1 80 20 = 1 /4
∴ P (G|BG ) = 2 = × =
P (BG ) 2 46 23
Now, (a) P (X1c /X)
5. Key Idea Use conditional probability, total probability and Baye’s 1 1 1
theorem.  X1c ∩ X  P (E1 ∩ E 2 ∩ E3 ) 2 ⋅ 4 ⋅ 4 1
=P = = =
It is given that there are three bags B1 , B2 and B3 and  P (X )  P (X ) 1 8
probabilities of being chosen B1 , B2 and B3 are 4
respectively
(b) P (exactly two engines of the ship are functioning)
3 3 4
∴ P (B1 ) = , P (B2) = and P (B3 ) = . P (E1 ∩ E 2 ∩ E3 ) + P (E1 ∩ E 2 ∩ E3 ) + P (E1 ∩ E 2 ∩ E3 )
10 10 10 =
P (X )
5 R 3 R 5 R
1 1 3 1 3 1 1 1 1
5 G 5 G 3 G ⋅ ⋅ + ⋅ ⋅ + ⋅ ⋅
7
B1 B2 B3
=2 4 4 2 4 4 2 4 4=
1 8
Now, probability that the chosen ball is green, given 4
G 3  X  P (X ∩ X 2)
that selected bag is B3 = P   = (c) P   =
 B3  8  X 2 P (X 2)
Now, probability that the selected bag is B3 , given that P (ship is operating with E 2 function )
=
B  P (X 2)
the chosen ball is green = P  3 
G
P (E1 ∩ E 2 ∩ E3 ) + P (E1 ∩ E 2 ∩ E3 ) + P (E1 ∩ E 2 ∩ E3 )
G =
P   P (B3 ) P (E 2)
 B3  1 1 3 1 1 1 1 1 1
= [by Baye’s ⋅ ⋅ + ⋅ ⋅ + ⋅ ⋅
G G G = 2 4 4 2 4 4 2 4 4 =
5
P   P (B1 ) + P   P (B2) + P   P (B3 ) 1
 B1   B2  B3  8
4
theorem] P (X ∩ X1 )
3 4  1 (d) P (X / X1 ) =
 ×  P (X1 )
 8 10 2 4
= = = 1 1 1 1 3 1 1 1 3
5 3  5 3   3 4  1 + 5 + 1 13 ⋅ ⋅ + ⋅ ⋅ + ⋅ ⋅
 ×  + ×  + ×  =2 4 4 2 4 4 2 4 4
 10 10  8 10  8 10 2 8 2
1 /2
Now, probability that the chosen ball is green 7
=
G G G 16
= P (G ) = P (B1 )P   + P (B2)P   + P (B3 )P  
 B1   B2  B3  7. Statement I If P (H i ∩ E ) = 0 for some i, then
[By using theorem of total probability] H   E
P  i = P   = 0
3 5   3 5  4 3  E  Hi 
= ×  + ×  + × 
 10 10  10 8  10 8 If P (H i ∩ E ) ≠ 0, ∀ i = 1, 2, K , n , then
3 3 3 12 + 15 + 12 39  H  P (H i ∩ E ) P (H i )
= + + = = P  i = ×
20 16 20 80 80  E P (H i ) P (E )
Now, probability that the selected bag is B3 and the  E
G 4 3 3 P   × P (H i )
chosen ball is green = P (B3 ) × P   = × =  Hi   E
 B3  10 8 20 = > P   ⋅ P (H i ) [Q0 < P (E ) < 1]
P (E )  Hi 
Hence, options (a) and (c) are correct.
Hence, Statement I may not always be true.
6. PLAN It is based on law of total probability and Bay’s Law. Statement II Clearly, H 1 ∪ H 2 ∪ . . . ∪ H n = S
Description of Situation It is given that ship would [sample space]
work if atleast two of engines must work. If X be event ⇒ P (H 1 ) + P (H 2) + . . . + P (H n ) = 1
that the ship works. Then, X ⇒ either any two of
E1 , E 2, E3 works or all three engines E1 , E 2, E3 works. Hence, Statement II is ture.
128 Probability

Passage I Head appears


2W 2W
8.
n1 n3 1W
Red Red 2R
U1 U2 2 Cases
or
n2 Black n4 Black 3W 1W
Box I Box II 1R
1R 1R
U1 U2
Let A = Drawing red ball
∴ P ( A ) = P (B1 ) ⋅ P ( A / B1 ) + P (B2) ⋅ P ( A / B2) Tail appears
1  n1  1  n3  1W 3W 3W 1W 2W 2W
=  +  
2  n1 + n2 2  n3 + n4  2R
2W 0R 2R 1R 1R
1 U1 U2 U1 U2 U1
Given, P (B2 / A ) = U2
3
P (B2) ⋅ P (B2 ∩ A ) 1 3 Cases
⇒ =
P ( A) 3 10. Now, probability of the drawn ball from U 2 being white
1  n3  is
   3C
2  n3 + n4  1
2
C 2
C 1
C 
⇒ = P (white / U 2) = P (H ) ⋅  5 1 × 2 1 + 5 1 × 2 1 
1  n1  1  n3  3  C1 C1 C1 C1 
  +  
2  n1 + n2 2  n3 + n4   3C 3
C 2
C 1
C 3
C1 ⋅ 2C1 C1 
2
+ P (T )  5 2 × 3 2 + 5 2 × 3 1 + × 
n3 (n1 + n2) 1  C2 C2 C2 C2 5
C2 3
C2 
⇒ =
n1 (n3 + n4 ) + n3 (n1 + n2) 3
1 3 2 1
Now, check options, then clearly options (a) and (b) =  ×1 + × 
2 5 5 2
satisfy.
13 1 1 6 2  23
9. +  ×1 + × + × =
1 Red 2 10 10 3 10 3  30
(n1 – 1) Red (n3 + 1) Red
11. P (Head appeared/white from U 2)
 3C1 2C1 2
C 1
C 
n2 Black n4 Black 5 × 2 + 5 1 × 2 1
= P (H ) .  C1 C1 C1 C1 
Box I Box II
23 / 30
1 Black 3 2 1
 ×1 + × 
= 
n1 Red n3 Red 1 5 5 2
or 2 20 / 30
12
(n2 – 1) Black (n4 + 1) Black =
23
Box I Box II
Passage III
1
∴ P (drawing red ball from B1) = 5 5 1 25
3 12. P (X = 3) = ⋅ ⋅ =
6 6 6 216
 n1 − 1   n1   n2   n1  1
⇒   +   = 5 5
13. P (X ≥ 3) = ⋅ ⋅ 1 =
25
 n1 + n2 − 1  n1 + n2  n1 + n2  n1 + n2 − 1 3 6 6 36
n12 + n1n2 − n1 1 P {(X > 3) / (X ≥ 6)} ⋅ P (X ≥ 6)
⇒ = 14. P {(X ≥ 6) / (X > 3)} =
(n1 + n2) (n1 + n2 − 1) 3 P (X > 3)
Clearly, options (c) and (d) satisfy.   5  1  5 6  1
5 
1 ⋅   ⋅   +   ⋅   + ... ∞ 
 6  6  6  6
Passage II =   = 25
3W 1W   5 3 1  5 4 1  36
Initial
2R
  ⋅ +   ⋅ + ... ∞ 
 6 6 6 6 
U1 U2
Probability 129

Passage IV  B  B
∴ P (B) = P   P ( A1 ) + P   P ( A2)
2  A1   A2 
15. Here, P (ui ) = ki, Σ P (ui ) = 1 ⇒ k =
n (n + 1) 12
C 2.6 C 4 10
C1.2 C1 C1.6 C5 . 11C1.1 C1
12
n
2i 2 = 18
.
12
+ 18 12
lim P (W ) = lim
n→ ∞ n→ ∞
∑ n (n + 1)2 C6 C2 C6 C2
i=1
20. Let E be the event that coin tossed twice, shows head at
2n (n + 1)(2n + 1) first time and tail at second time and F be the event that
= lim = 2 /3
n→∞ 6n (n + 1)2 coin drawn is fair.
P (E / F ) ⋅ P (F )
n P (F / E ) =
P (E / F ) ⋅ P (F ) + P (E / F ′ ) ⋅ P (F ′ )
+1
16. P  n  =
u n 2
= 1 1 m
W  Σi n+1 ⋅ ⋅
n+1 = 2 2 N
1 1 m 2 1 N −m
⋅ ⋅ + ⋅ ⋅
W  2 + 4 + 6 + ... n+2
17. P   = =
2 2 N 3 3 N
 E n (n + 1) 2 (n + 1) m
2 4 9m
= =
m 2 (N − m) 8N + m
18. As, the statement shows problem is to be related to +
Baye’s law. 4 9
Let C , S , B, T be the events when person is going by car, 21. Let W1 = ball drawn in the first draw is white.
scooter, bus or train, respectively. B1 = ball drawn in the first draw in black.
1 3 2 1 W 2 = ball drawn in the second draw is white.
∴ P (C ) = , P (S ) = , P (B) = , P (T ) =
7 7 7 7 Then , P (W 2) = P (W1 ) P (W 2 / W1 ) + P (B1 )P (W 2 / B1 )
Again, L be the event of the person reaching office late.  m   m+ k   n   m 
=    +   
∴ L be the event of the person reaching office in time.  m + n   m + n + k  m + n   m + n + k
 L 7  L 8  L 5 m(m + k) + mn m (m + k + n ) m
Then, P   = , P   = , P   = = = =
C 9 S 9  B 9 (m + n ) (m + n + k) (m + n ) (m + n + k) m + n

 L 8 22. The number of ways in which P1 , P2, K , P8 can be paired


and P  =
T 9 in four pairs
1 8
 L = [( C 2)(6C 2)(4C 2)(2C 2)]
P   ⋅ P (C ) 4!
C C 1 8! 6! 4!
∴ P  = = × × × ×1
 L  L  L  L 4 ! 2 !6 ! 2 !4 ! 2 !2 !
P   ⋅ P (C ) + P   ⋅ P (S ) + P   ⋅ P (B)
C S  B 1 8×7 6 ×5 4 ×3
= × × ×
 L 4 ! 2 ! ×1 2 ! × 1 2 ! × 1
+ P   ⋅ P (T )
T 8 × 7 ×6 ×5
= = 105
7 1 2 .2 .2 .2
×
9 7 1 Now, atleast two players certainly reach the second
= =
7 1 8 3 5 2 8 1 7 round between P1, P2 and P3 and P4 can reach in final if
× + × + × + ×
9 7 9 7 9 7 9 7 exactly two players play against each other between P1,
P2, P3 and remaining player will play against one of the
19. Let A1 be the event exactly 4 white balls have been
players from P5 , P6, P7, P8 and P4 plays against one of the
drawn. A2 be the event exactly 5 white balls have been remaining three from P5 …P8.
drawn.
This can be possible in
A3 be the event exactly 6 white balls have been drawn. 3
C 2 × 4C1 × 3C1 = 3 . 4 . 3 = 36 ways
B be the event exactly 1 white ball is drawn from two
draws. Then, ∴ Probability that P4 and exactly one of P5 ... P8 reach
36 12
 B  B  B second round = =
P (B) = P   P ( A1 ) + P   P ( A2) + P   P ( A3 ) 105 35
 A1   A2   A3 
If P1 , Pi , P4 and Pj , where i = 2 or 3 and j = 5 or 6 or 7
 B reach the second round, then they can be paired in 2
But P   = 0 1 4
 A3  pairs in ( C 2) (2C 2) = 3 ways. But P4 will reach the
2!
[since, there are only 6 white balls in the bag] final, if P1 plays against Pi and P4 plays against Pj .
130 Probability

Hence, the probability that P4 will reach the final round (14)! (16)!
∴ n (E ) = and n (S ) =
1 (2 !)7 ⋅ 7 ! (2 !)8 ⋅ 8 !
from the second =
3
12 1 4 ∴ Probability of S1 and S 2 being in the same pair
∴ Probability that P4 will reach the final is × = . (14)! ⋅ (2 !)8 ⋅ 8 ! 1
35 3 35 = =
(2 !)7 ⋅ 7 !⋅ (16)! 15
23. Let q = 1 − p = probability of getting the tail. We have,
The probability of any one wining in the pairs of
α = probability of A getting the head on tossing firstly
S1 , S 2 = P (certain event) = 1
= P (H 1 or T1T2T3 H 4 or T1T2T3T4T5T6H 7 or …)
∴ The pairs of S1 , S 2 being in two pairs separately
= P (H ) + P (H )P (T )3 + P (H )P (T )6 + … and S1 wins, S 2 loses + The probability of S1 , S 2 being
P (H ) p in two pairs separately and S1 loses, S 2 wins.
= =
1 − P (T )3
1 − q3  (14)!   (14)! 
Also,  (2 !) ⋅ 7 !
7  1 1  (2 !)7 ⋅ 7 !  1 1
= 1 −  × × + 1 − × ×
β = probability of B getting the head on tossing secondly 
(16)!
 2 2 
(16)!  2 2
= P (T1H 2 or T1T2T3T4H 5 or T1T2T3T4T5T6T7H 8 or …)  (2 !)8 ⋅ 8 !   (2 !)8 ⋅ 8 ! 
= P (H ) [P (T ) + P (H )P (T )4 + P (H )P (T )7 + K ] 1 14 × (14)! 7
= × =
= P (T )[P (H ) + P (H )P (T )3 + P (H )P (T )6 + ... ] 2 15 × (14)! 15
p(1 − p) 1 7 8
= q α = (1 − p) α = ∴ Required probability = + =
1 − q3 15 15 15
Again, we have 25. Let E1 , E 2, E3 and A be the events defined as
α + β + γ =1 E1 = the examinee guesses the answer
p + p(1 − p)
⇒ γ = 1 − (α + β ) = 1 − E 2 = the examinee copies the answer
1 − q3
E3 = the examinee knows the answer
p + p(1 − p)
=1 − and A = the examinee answer correctly
1 − (1 − p)3
1 1
We have, P (E1 ) = , P (E 2) =
1 − (1 − p)3 − p − p(1 − p) 3 6
=
1 − (1 − p)3 Since, E1 , E 2, E3 are mutually exclusive and exhaustive
1 − (1 − p)3 − 2 p + p2 p − 2 p2 + p3 events.
γ= = 1 1 1
1 − (1 − p)3 1 − (1 − p)3 ∴ P (E1 ) + P (E 2) + P (E3 ) = 1 ⇒ P (E3 ) = 1 − − =
3 6 2
p p(1 − p)
Also, α= , β= If E1 has already occured, then the examinee guesses.
1 − (1 − p)3 1 − (1 − p)3
Since, there are four choices out of which only one is
24. (i) Probability of S1 to be among the eight winners correct, therefore the probability that he answer
correctly given that he has made a guess is 1/4.
= (Probability of S1 being a pair ) 1
i.e. P ( A / E1 ) =
× (Probability of S1 winning in the group) 4
1 1 1
=1 × = [since, S1 is definitely in a group] It is given that, P ( A / E 2) =
2 2 8
(ii) If S1 and S 2 are in the same pair, then exactly one and P ( A / E3 ) = probability that he answer correctly
wins. given that he know the answer = 1
If S1 and S 2 are in two pairs separately, then exactly By Baye’s theorem, we have
one of S1 and S 2 will be among the eight winners. If P (E3 ) ⋅ P ( A / E3 )
P (E3 / A ) =
S1 wins and S 2 loses or S1 loses and S 2 wins.  
Now, the probability of S1 , S 2 being in the same pair P (E1 ) ⋅ P ( A / E1 ) + P (E 2) ⋅ P ( A / E 2) 
 + P (E3 ) ⋅ P ( A / E3 ) 
and one wins 
= (Probability of S1 , S 2 being the same pair) 1
×1
× (Probability of anyone winning in the pair). 2 24
∴ P (E3 / A ) = =
 1 1  1 1  1  29
and the probability of S1 , S 2 being the same pair  ×  +  ×  +  × 1
 3 4  6 8  2 
n (E )
=
n (S ) 26. Let Bi = ith ball drawn is black.
where, n (E ) = the number of ways in which 16 Wi = ith ball drawn is white, where i = 1, 2
persons can be divided in 8 pairs. and A = third ball drawn is black.
Probability 131

We observe that the black ball can be drawn in the third Topic 5 Probability Distribution and
draw in one of the following mutually exclusive ways.
(i) Both first and second balls drawn are white and Binomial Distribution
third ball drawn is black. 1. It is given that α is the number of heads that appear
i.e. (W1 ∩ W 2) ∩ A when C1 is tossed twice, the probability distribution of
random variable α is
(ii) Both first and second balls are black and third ball
drawn is black. α 0 1 2
(B1 ∩ B2) ∩ A 2 2
2    
i.e.  1 2 1 2
P(α)    3  3  
(iii) The first ball drawn is white, the second ball drawn  3  3
is black and the third ball drawn is black.
Similarly, it is given that β is the number of heads that
i.e. (W1 ∩ B2) ∩ A appear when C 2 is tossed twice, so probability
(iv) The first ball drawn is black, the second ball drawn distribution of random variable β is
is white and the third ball drawn is black.
β 0 1 2
i.e. (B1 ∩ W 2) ∩ A 2
2 4 1
∴ P ( A ) = P [{(W1 ∩ W 2) ∩ A } ∪{(B1 ∩ B2) ∩ A } P(β)  
 3 9 9
∪ {(W1 ∩ B2) ∩ A } ∪ {(B1 ∩ W 2) ∩ A }]
Now, as the roots of quadratic polynomial x2 − αx + β are
= P{(W1 ∩ W 2) ∩ A } + P{(B1 ∩ B2) ∩ A } real and equal, so D = α 2 − 4β = 0 and it is possible if
+ P{(W1 ∩ B2) ∩ A } + P{(B1 ∩ W 2) ∩ A } (α,β) = (0, 0)
= P (W1 ∩ W 2) ⋅ P ( A / (W1 ∩ W 2)) + P (B1 ∩ B2) or (2, 1)
2 2 2
∴ P ( A / (B1 ∩ B2)) + P (W1 ∩ B2) ⋅ P ( A / (W1 ∩ B2))  1  2  2  4
∴ Required probability =     +    
+ P (B1 ∩ W 2) ⋅ P ( A / (B1 ∩ W 2))  3  3  3  9
4 16 20
 2 1  2 3 4 = + =
=  ×  ×1 +  ×  × 81 81 81
 4 3  4 5 6
 2 2 3  2 2 3 2. As we know that the probability of show up a three or a
+ ×  × + ×  × five on a throw of a dice is
 4 3 4  4 5 4
2 1
1 1 1 3 23 P (E ) = =
= + + + = 6 3
6 5 4 20 30 2
∴ P (E ) =
3
27. The testing procedure may terminate at the twelfth
testing in two mutually exclusive ways. Now, as four fair dice are thrown independently 27
I : When lot contains 2 defective articles. times, so probability of getting at least two 3’s or 5’s in
one trial
II : When lot contains 3 defective articles. 2 2 3 4
 1  2  1  2  1
Let A = testing procedure ends at twelth testing p = 4C 2    + 4C3     + 4C 4  
 3  3  3  3  3
A1 = lot contains 2 defective articles  4 2 1
= 6  4 + 4 4 + 4
A2 = lot contains 3 defective articles 3  3 3
∴ Required probability 24 + 8 + 1 33 11
= = 4 =
34 3 27
= P ( A1 ) ⋅ P ( A / A1 ) + P ( A2) ⋅ P ( A / A2)
Therefore, the expected number of times in 27 trials
Here, P ( A / A1 ) = probability that first 11 draws contain = 27 p = 11
10 non-defective and one-defective and twelfth draw
contains a defective article. 3. It is given that out of the five machines in a workshop,
the probability of any one of them to be out of service on
18
C10 × 2C1 1 1
= 20
× …(i) a day is .
C11 9 4
P ( A / A2) = probability that first 11 draws contains 9 So, the probability that at most two machines will be
non-defective and 2-defective articles and twelfth draw out of service on the same day
17
C 9 × 3C 2 1 = probability that no machine is out of service
contains defective = 20
× … (ii)
C11 9 + probability that exactly one machine is out of service +
probability that exactly two of machines are out of
∴ Required probability
service
= (0.4)P ( A / A1 ) + 0.6 P ( A / A2)  1
5
1 1
4 2
 1  1
3
 3
3
1 −  + C1 1 −  + C 2  1 −  =   k
5 5
0.4 × 18C10 × 2C1 1 0.6 × 17C 9 × 3C 2 1 99  4 4 4  4  4  4
= 20
× + 20
× =
C11 9 C11 9 1900 (given)
132 Probability
5 4 3
 3 5  3 33  3 According to the question,
⇒   +   + 10 5 =   k n n
 4 4  4 4  4  1 99  1 99
1−  > ⇒  <1 −
2  2 100  2 100
 3 15 10
⇒ k=  + 2 + 2  1
n
1
 4 4 4 ⇒   < ⇒ 2n > 100
 2 100
9 + 15 + 10 34
= = ⇒ n=7 [for minimum]
16 16
17 7. The required probability of observing atleast one head
⇒ k=
8 = 1 − P (no head)
1
4. Given that, there are 50 problems to solve in an =1 − n [let number of toss are n]
admission test and probability that the candidate can 2
4  1
solve any problem is = q (say). So, probability that the Q P (Head) = P (Tail) =
5  2 
4 1 1 90
candidate cannot solve a problem is p = 1 − q = 1 − = . According to the question, 1 − n ≥
5 5 2 100
1 1
⇒ n ≤ ⇒ 2n ≥ 10 ⇒ n ≥ 4
Now, let X be a random variable which denotes the 2 10
number of problems that the candidate is unable to So, minimum number of times one has to toss a fair coin
solve. Then, X follows binomial distribution with so that the probability of observing atleast one head is
1
parameters n = 50 and p = . atleast 90% is 4.
5
Now, according to binomial probability distribution
8. Let p and q represents the probability of success and
concept failure in a trial respectively. Then,
50 − r 2 1 4 2
r
 1  4 p = P (5 or 6) = = and q = 1 − p = = .
P (X = r ) = 50C r     , r = 0, 1, ... , 50 6 3 6 3
 5  5
Now, as the man decides to throw the die either till he
∴Required probability gets a five or a six or to a maximum of three throws, so
= P (X < 2) = P (X = 0) + P (X = 1) he can get the success in first, second and third throw or
50 49 49 not get the success in any of the three throws.
 4 449  4  4 50 54  4
= 50C 0   + 50C1 =   +  =   So, the expected gain/loss (in `)
 5 (5)50  5  5 5  5  5
= ( p × 100) + qp(− 50 + 100)
5. Let for the given random variable ‘X’ the binomial + q2p(− 50 − 50 + 100) + q3 (− 50 − 50 − 50)
probability distribution have n-number of independent 1   2 1
2
 2  1  2
3
trials and probability of success and failure are p and q =  × 100 +  ×  (50) +     (0) +   (− 150)
3   3 3  3  3  3
respectively. According to the question,
Mean = np = 8 and variance = npq = 4 =
100 100
+ +0−
1200
1 1 3 9 27
∴ q = ⇒ p =1−q = 900 + 300 − 1200 1200 − 1200
2 2 = = =0
1 27 27
Now, n × = 8 ⇒ n = 16
2 9. The probability of hitting a target at least once
 1
16 = 1 − (probability of not hitting the target in any trial)
P (X = r ) = 16C r   = 1 − nC 0 p0qn
 2
where n is the number of independent trials and p and q
∴ P (X ≤ 2) = P (X = 0) + P (X = 1) + P (X = 2)
16 16 16
are the probability of success and failure respectively.
 1  1  1 [by using binomial distribution]
=16 C 0   + 16 C1   + 16 C 2 
 2  2  2 1 1 2
Here, p= and q =1− p =1− =
1 + 16 + 120 137 k 3 3 3
= = 16 = 16 (given)
216 2 2 0
 1  2
n
5
According to the question, 1 − nC 0     >
⇒ k = 137  3  3 6
n n
 2 5  2 1
6. As we know probability of getting a head on a toss of a ⇒   <1− ⇒   <
1  3 6  3 6
fair coin is P (H ) = = p (let)
2 Clearly, minimum value of n is 5.
Now, let n be the minimum numbers of toss required to 10. Let p = probability of getting an ace in a draw
get at least one head, then required probability
= probability of success
= 1 − (probability that on all ‘n’ toss we are getting tail)
 1
n
 1 and q = probability of not getting an ace in a draw
=1 −   Q P (tail) = P (Head ) = = probability of failure
 2  2 
Probability 133

Then, p=
4
=
1 ⇒ p = 1 /2
52 13 ∴ n = 4, p = q = 1 / 2
1 12 Now, P (X > 1) = 1 − { P (X = 0) + P (X = 1)}
and q =1 − p=1 − =
13 13 0 4 1 3
 1  1  1  1
Here, number of trials, n = 2 = 1 − 4C 0     − 4C1    
 2  2  2  2
Clearly, X follows binomial distribution with parameter
1 1 4 11
n = 2 and p = . =1 − − =
13 16 16 16
x 2− x
 1   12 0 .1 0 .1 5
Now, P (X = x) = 2C x     , x = 0, 1, 2 16. Probability (face 1) = = =
 13  13 0 .1 + 0 .32 0 .42 21
∴ P (X = 1) + P (X = 2) 17. Let E be the event that product of the two digits is 18,
1 2 0
 1   12  1   12 therefore required numbers are 29 , 36, 63 and 92.
= C1     + 2C 2   
2
 13  13  13  13 4
Hence, p = P (E ) =
 12  1 100
=2  +
 169 169 and probability of non-occurrence of E is
24 1 25 4 96
= + = q = 1 − P (E ) = 1 − =
169 169 169 100 100
11. Given box contains 15 green and 10 yellow balls. Out of the four numbers selected, the probability that
the event E occurs atleast 3 times, is given as
∴ Total number of balls = 15 + 10 = 25
15 3 P = 4C3 p3 q + 4C 4 p4
P(green balls) = = = p = Probability of success 3 4
25 5  4   96   4  97
=4     +  = 4
P(yellow balls) =
10 2
= = q = Probability of unsuccess  100  100  100 25
25 5
and n = 10 = Number of trials. 18. Since, set A contains n elements. So, it has 2n subsets.
3 2 12
∴Variance = npq = 10 × × = ∴ Set P can be chosen in 2n ways, similarly set Q can be
5 5 5 chosen in 2n ways.
12. Probability of guessing a correct answer, p = and
1 ∴ P and Q can be chosen in (2n )(2n ) = 4n ways.
3 Suppose, P contains r elements, where r varies from 0 to
probability of guessing a wrong answer, q = 2 /3 n. Then, P can be chosen in nC r ways, for 0 to be disjoint
∴ The probability of guessing a 4 or more correct from A, it should be chosen from the set of all subsets of
4 5
 1 2  1 2 1 11 set consisting of remaining (n − r ) elements. This can be
answers = 5C 4   ⋅ + 5C5   = 5 ⋅ 5 + 5 = 5
 3 3  3 3 3 3 done in 2n − r ways.
∴ P and Q can be chosen in nC r ⋅ 2n − r ways.
13. India play 4 matches and getting at least 7 points. It can
only be possible in WWWD or WWWW position, where W But, r can vary from 0 to n.
represents two points and D represents one point. ∴ Total number of disjoint sets P and Q
n
Therefore, the probability of the required event = ∑ nC r2n − r = (1 + 2)n = 3n
= 4C3 (0.05) (0.5)3 + 4C 4 (0.5)4 r=0

= [4(0.05) + 0.5] (0.5) 3


3 n  3
n
Hence, required probability = = 
= 0.0875 4 n  4
14. Let X be the number of coins showing heads. Let X be a 19. Case I When A plays 3 games against B.
binomial variate with parameters n = 100 and p.
In this case, we have n = 3, p = 0.4 and q = 0.6
Since, P (X = 50) = P (X = 51) Let X denote the number of wins. Then,
⇒ 100
C50 p50 (1 − p)50 = 100C51 ( p)51 (1 − p)49 P (X = r ) = 3C r (0.4)r (0.6)3 − r; r = 0, 1, 2, 3
(100) ! (51 !) × (49 !) p p 51
⇒ ⋅ = ⇒ = ∴ P1 = probability of winning the best of 3 games
(50 !) (50 !) 100 ! 1− p 1 − p 50
= P (X ≥ 2)
51
⇒ p= = P (X = 2) + P (X = 3)
101
= 3C 2(0.4)2(0.6)1 + 3C3 (0.4)3 (0.6)0
15. For Binomial distribution, mean = np = 0.288 + 0.064 = 0.352
and variance = npq Case II When A plays 5 games against B.
∴ np = 2 and npq = 1 [given] In this case, we have
⇒ q = 1 / 2 and p + q = 1 n = 5, p = 0.4 and q = 0.6
134 Probability

Let X denotes the number of wins in 5 games. and P (E ) = probability of occuring perfect square before
Then, prime
2
P (X = r ) = 5C r (0.4)r (0.6)5 − r , where r = 0, 1, 2K , 5  7   14  7   14  7 
=   +     +     +…∞
∴ P2 = probability of winning the best of 5 games  36  36  36  36  36
= P (X ≥ 3) 7
36 7
= P (X = 3) + P (X = 4) + P (X = 5) = =
14 22
1−
= 5C3 (0.4)3 (0.6)2 + 5C 4 (0.4)4 (0.6) + 5C5 (0.4)5 (0.6)0 36
= 0.2304 + 0.0768 + 0.1024 = 0.31744 2
4
Clearly, P1 > P2. Therefore, first option i.e. ‘best of 3 ∴ P (T / E ) = 11 = = p
7 7
games’ has higher probability of winning the match.
22
20. The man will be one step away from the starting point,
⇒ 14 p = 8
if
(i) either he is one step ahead or (ii) one step behind the 22. It is given that the probability, a missile hits a target
3
starting point. successfully p = , so the probability to not hits the
4
The man will be one step ahead at the end of eleven 1
steps, if he moves six steps forward and five steps target is q = .
4
backward. The probability of this event is
11 And it is also given that to destroy the target
C 6 (0.4)6 (0.6)5 .
completely, at least three successful hits are required.
The man will be one step behind at the end of eleven Now, according to the question, let the minimum
steps, if he moves six steps backward and five steps number of missiles required to fired is n, so
forward. The probability of this event is 11C 6 (0.6)6 (0.4)5 . n
C3 p3 qn − 3 + nC 4 p4qn − 4 + … + nC n pn ≥ 0.95
∴ Required probability

= 11C 6 (0.4)6 (0.6)5 + C 6 (0.6)6 (0.4)5 = 11C 6 (0.24)5
11
  1
n
 3  1
n −1 2
 3  1
n − 2
1 −  nC 0   + nC1     + nC 2     ≥ 0.95
21. Let an event E of sum of outputs are perfect square  4  4  4  4  4
 
(i.e., 4 or 9), so
95 1 3n n (n − 1) 9
E = {(1, 3), (2, 2), (3, 1), (3, 6), (4, 5), (5, 4), (6, 3)} ⇒ 1− ≥ + +
100 4n 4n 2 4n
and an event F of sum of outputs are prime numbers
4 n
2 + 6n + 9n − 9n
2
(i.e., 2, 3, 5, 7, 11) so ⇒ ≥
20 2
F = {(1, 1), (1, 2), (2, 1), (1, 4), (2, 3), (3, 2), (4, 1), (1, 6),
⇒ 10(9n 2 − 3n + 2 ) ≤ 4n
(2, 5), (3, 4), (4, 3), (5, 2), (6, 1), (5, 6), (6, 5)}
Now, at n = 3, LHS = 720, RHS = 64
and event T of sum of outputs are odd numbers
at n = 4, LHS = 1340, RHS = 256
(i.e., 3, 5, 7, 9, 11)
at n = 5, LHS = 2120, RHS = 1024
T = {(1, 2), (2, 1), (1, 4), (2, 3), (3, 2), (4, 1), (1, 6),
at n = 6, LHS = 3080, RHS = 4096
(2, 5), (3, 4), (4, 3), (5, 2), (6, 1), (3, 6), (4, 5), (5, 4),
(6, 3), (5, 6), (6, 5)} Hence, n = 6 missiles should be fired.
Now, required probability p = P (T / E ) 23. Using Binomial distribution,
P (T ∩ E ) P (X ≥ 2) = 1 − P (X = 0) − P (X = 1)
=
P (E ) n  n −1
 1  1  1
where, P (T ∩ E ) = probability of occuring perfect square = 1 −   −  nC1 ⋅   ⋅   
 2  2  2
odd number before prime  
 4   14  4 
2
 14  4  1 1  1 + n
=  +   +     +…∞ = 1 − n − nC1 ⋅ n = 1 −  n 
 36  36  36  36  36 2 2  2 
4 Given, P (X ≥ 2) ≥ 0. 96
= 36 =
4
=
2 (n + 1) 24 n+1 1
14 22 11 ∴ 1− ≥ ⇒ ≤ ⇒ n =8
1− 2n 25 2n 25
36
7
Matrices and Determinants
Topic 1 Types of Matrices, Addition, Subtraction,
Multiplication and Transpose of a Matrix
Objective Questions I (Only one correct option) 1 0 0
1. If A is a symmetric matrix and B is a skew-symmetric 6. Let P =  4 1 0 and I be the identity matrix of order
 
2 3  16 4 1
matrix such that A + B =  , then AB is equal to
5 −1 3. If Q = [qij ] is a matrix, such that P50 − Q = I, then
(2019 Main, 12 April I)
q31 + q32
equals
 −4 −2 4 −2 q21 (2016 Adv.)
(a)   (b) 
 −1 4   −1 −4 (a) 52 (b) 103
(c) 201 (d) 205
 4 −2  −4 2
(c)   (d) 
 1 −4 1 4 1 2 2
 0 2y 1  7. If A = 2 1 −2 is a matrix satisfying the equation
 
2. The total number of matrices A = 2x y −1 ,  c 2 b 
 
2x − y 1  AAT = 9 I, where, I is 3 × 3 identity matrix, then the
ordered pair (a, b) is equal to (2015 Main)
(x, y ∈ R, x ≠ y) for which AT A = 3I3 is
(2019 Main, 9 April II) (a) (2, − 1) (b) (−2, 1)
(a) 2 (b) 4 (c) (2, 1) (d) (−2, − 1)
(c) 3 (d) 6
 3 /2 1 /2  1 1
cos α − sin α  0 −1 8. If P =  , A = 0 1 and Q = PAP , then
T
3. Let A =   , (α ∈ R) such thatA32 =  .  −1 /2 3 / 2  
sin α cos α  1 0  PTQ 2005 P is
(2005, 1M)
Then, a value of α is (2019 Main, 8 April I)
π π π  1 2005  1 2005
(a)  (b) 
(a)
32
(b) 0 (c)
64
(d)
16 0 1   2005 1 
 1 0  1 0
1 0 0 (c)   (d)  
4. Let P = 3 1 0 and Q = [qij ] be two 3 × 3 matrices such  2005 1  0 1
 
9 3 1 9. If A = α 0
and B = 
1 0
, then value of α for which
q + q31 
1 1 5 1
that Q − P 5 = I3 . Then, 21 is equal to
q32 A 2 = B,is (2003, 1M)
(2019 Main, 12 Jan I)
(a) 1 (b) –1
(a) 10 (b) 135 (c) 4 (d) no real values
(c) 9 (d) 15
0 2q r  10. If A and B are square matrices of equal degree, then
5. Let A =  p q −r . If AAT = I3 , then| p|is
which one is correct among the following? (1995, 2M)
  (a) A + B = B + A
 p − q r  (2019 Main, 11 Jan I) (b) A + B = A − B
1 1 1 1 (c) A − B = B − A
(a) (b) (c) (d)
5 2 3 6 (d) AB = BA
136 Matrices and Determinants

Objective Questions II 15. If the point P (a , b, c), with reference to Eq. (i), lies on
One or more than one correct option) the plane 2x + y + z = 1, then the value of 7a + b + c is
(a) 0 (b) 12 (c) 7 (d) 6
1 0 0 1 0 0
16. Let b = 6, with a and c satisfying Eq. (i). If α and β are
11. Let P1 = I = 0 1 0, P2 = 0 0 1,
    the roots of the quadratic equation ax2 + bx + c = 0, then
0 0 1 0 1 0 ∞ n
 1 1
0 1 0 0 1 0 ∑  α + β is equal to
n= 0
P3 = 1 0 0, P4 = 0 0 1, 6
    (a) 6 (b) 7 (c) (d) ∞
0 0 1 1 0 0 7
0 0 1 0 0 1 17. Let ω be a solution of x3 − 1 = 0 with Im (ω ) > 0. If a = 2
P5 = 1 0 0, P6 = 0 1 0 with b and c satisfying Eq. (i) then the value of
    3 1 3
0 1 0 1 0 0 + b + c is
ω a
ω ω
6
2 1 3 (a) − 2 (b) 2
and X = ∑ Pk 1 0 2 PkT (c) 3 (d) − 3
k=1
 
3 2 1 Passage II
where, PkT
denotes the transpose of the matrix Pk. Then Let p be an odd prime number and T p be the following set
which of the following option is/are correct? (2019 Adv.) of 2 × 2 matrices
 
Tp =  A = 
a b
(a) X is a symmetric matrix ; a , b, c ∈ { 0, 1, 2, K , p − 1}
(b) The sum of diagonal entries of X is 18   c a  (2010)
(c) X − 30 I is an invertible matrix 18. The number of A in T p such that det (A) is not divisible
1 1
(d) If X 1 = α 1, then α = 30
by p, is
  (a) 2 p 2 (b) p3 − 5 p
1 1
(c) p3 − 3 p (d) p3 − p 2
12. Let X and Y be two arbitrary, 3 × 3, non-zero, 19. The number of A in T p such that the trace of A is not
skew-symmetric matrices and Z be an arbitrary, 3 × 3, divisible by p but det ( A ) is divisible by p is
non-zero, symmetric matrix. Then, which of the (a) ( p − 1) ( p 2 − p + 1) (b) p3 − ( p − 1)2
following matrices is/are skew-symmetric? (c) ( p − 1)2 (d) ( p − 1) ( p 2 − 2)
(2015 Adv.)
(a)Y 3 Z 4 − Z 4Y 3 (b) X 44 + Y 44
20. The number of A in T p such that A is either symmetric or
(c) X 4 Z 3 − Z 3 X 4 (d) X 23 + Y 23 skew-symmetric or both and det (A) is divisible by p is
(a) ( p − 1)2 (b) 2 ( p − 1)
13. For 3 × 3 matrices M and N , which of the following
(c) ( p − 1)2 + 1 (d) 2 p − 1
statement(s) is/are not correct ? (2013 Adv.)
NOTE The trace of a matrix is the sum of its diagonal entries.
(a) N T M N is symmetric or skew-symmetric, according as
M is symmetric or skew-symmetric
Analytical and Descriptive Questions
(b) MN − NM is symmetric for all symmetric matrices M
and N a  b c
21. If matrix A =  b  , where a , b, c are real
c a
(c) M N is symmetric for all symmetric matrices M and N  c  a b
(d) (adj M ) (adj N ) = adj (MN ) for all invertible matrices M
positive numbers, abc = 1 and AT A = I , then find the
and N
value of a3 + b3 + c3 . (2003, 2M)
14. Let ω be a complex cube root of unity with ω ≠ 0 and
P = [ pij ] be an n × n matrix with pij = ωi+ j . Then, P 2 ≠ 0 Integer & Numerical Answer Type Questions
when n is equal to (2013 Adv.)
(a) 57 (b) 55 22. The trace of a square matrix is defined to be the sum of
(c) 58 (d) 56 its diagonal entries. If A is a 2 × 2 matrix such that the
trace of A is 3 and the trace of A3 is − 18, then the value
of the determinant of A is ……… . (2020 Adv.)
Passage Based Problems
−1 + 3 i
Passage I 23. Let z = , where i = −1, and r , s ∈ {1, 2, 3}. Let
2
Let a , b and c be three real numbers satisfying (− z )r z 2s 
1 9 7 P =  2s  and I be the identity matrix of order 2.
 z zr 
[a b c] 8 2 7 = [0 0 0] ...(i)
  Then, the total number of ordered pairs (r , s) for which
7 3 7 (2011) P 2 = − I is (2016 Adv.)
Matrices and Determinants 137

Topic 2 Properties of Determinants


Objective Questions I (Only one correct option) 1 1 1 
1. If the minimum and the maximum values of the 7. Let the numbers 2, b, c be in an AP and A = 2 b c .
 
π π  2 2
4 b c 
function f : , → R,
 4 2  If det( A ) ∈ [2, 16], then c lies in the interval
− sin 2 θ −1 − sin 2 θ 1 (2019 Main, 8 April II)

defined by f (θ ) = − cos 2 θ −1 − cos 2 θ 1 are m and M (a) [3, 2 + 23 / 4 ] (b) (2 + 23 / 4 , 4)


(c) [4, 6] (d) [2, 3)
12 10 −2
 1 sin θ 1 
 3π 5π 
8. If A = − sin θ sin θ ; then for all θ ∈ 
respectively, then the ordered pair (m, M ) is equal to
1 , ,
(2020 Main, 5 Sep I)    4 4
 − 1 − sin θ 1 
(a) (0, 2 2 ) (b) (−4, 4) (c) (0, 4) (d) (−4, 0)
det( A ) lies in the interval (2019 Main, 12 Jan II)
2. A value of θ ∈ (0, π / 3), for which
(a)  , 3 (b)  , 4 (c)  0,  (d)  1, 
3 5 3 5
1 + cos 2 θ sin 2 θ 4 cos 6θ  2   2   2   2 
cos θ
2
1 + sin 2 θ 4 cos 6θ = 0, is
a−b−c 2a 2a
cos θ
2
sin θ
2
1 + 4 cos 6θ
(2019 Main, 12 April II) 9. If 2b b−c−a 2b
π π 7π 7π 2c 2c c−a −b
(a) (b) (c) (d)
9 18 24 36
= (a + b + c) (x + a + b + c)2, x ≠ 0 and a + b + c ≠ 0, then
3. The sum of the real roots of the equation x is equal to (2019 Main, 11 Jan II)
x−6 −1 (a) − (a + b + c) (b) − 2(a + b + c)
2− 3x x − 3 = 0, is equal to (c) 2(a + b + c) (d) abc
− 3 2x x + 2 (2019 Main, 10 April II) 10. Let a1 , a 2, a3 ..... , a10 be in GP with ai > 0 for
(a) 0 (b) − 4 (c) 6 (d) 1 i = 1, 2, ..... ,10 and S be the set of pairs (r , k), r , k ∈ N
x sin θ cos θ (the set of natural numbers) for which
4. If ∆1 = − sin θ − x 1 log e a1r a 2k log e a 2ra3k log e a3r a 4k
cos θ 1 x log e a 4r a5k log e a5r a 6k log e a 6r a7k = 0
x sin 2θ cos 2θ log e a7r a 8k log e a 8r a 9k log e a 9r a10
k

and ∆ 2 = − sin 2θ −x 1 , x ≠ 0, Then, the number of elements in S, is (2019 Main, 10 Jan II)
cos 2θ 1 x (a) 4 (b) 2
(c) 10 (d) infinitely many
 π
then for all θ ∈ 0,  (2019 Main, 10 April I) 2 b 1
 2

11. Let A = b b + 1 b, where b > 0. Then, the minimum
2
(a) ∆1 + ∆ 2 = − 2(x3 + x − 1)  
(b) ∆1 − ∆ 2 = − 2x3 1 b 2 
(c) ∆1 + ∆ 2 = − 2x3 det ( A )
value of is
(d) ∆1 − ∆ 2 = x(cos 2θ − cos 4θ) b (2019 Main, 10 Jan II)
1 1 1 2 1 3 1 n − 1 1 78 (a) − 3 (b) −2 3 (c) 2 3 (d) 3
5. If  . .  ...  = ,then the
0 1 0 1 0 1 0 1  0 1  12. Let d ∈ R, and
1 n   −2 4+ d (sin θ ) − 2 
inverse of   is
0 1  (2019 Main, 9 April I) A= 1  (sin θ ) + 2 d  , θ ∈ [θ , 2π ]. If
 
 1 0  1 −13  1 0  1 −12  5 (2 sin θ ) − d (− sin θ ) + 2 + 2d 
(a)   (b)   (c) 13 1 (d)  
12 1  0 1    0 1  the minimum value of det(A) is 8, then a value of d is
(2019 Main, 10 Jan I)
6. Let α and β be the roots of the equation x2 + x + 1 = 0.
(a) −5 (b) −7 (c) 2( 2 + 1) (d) 2( 2 + 2)
Then, for y ≠ 0 in R,
y+1 α β x − 4 2x 2x 
α y+β 1 is equal to 13. If  2x x − 4 2x 

 = ( A + Bx)(x − A ) ,
2
then the
β 1 y+α  2x 2x x − 4
(2019 Main, 9 April I)
(a) y( y2 − 1) (b) y ( y2 − 3)
ordered pair ( A , B) is equal to (2018 Main)
(c) y3 − 1 (d) y3 (a) (−4, − 5) (b) (−4, 3) (c) (−4, 5) (d) (4, 5)
138 Matrices and Determinants

14. Let ω be a complex number such that 2ω + 1 = z, where Objective Questions II


1 1 1
(One or more than one correct option)
z = − 3. If 1 −ω − 1 ω 2 = 3 k, then k is equal to
2
23. Which of the following is(are) NOT the square of a 3 × 3
1 ω2 ω7 (2017 Main)
matrix with real entries? (2017 Adv.)
(a) − z (b) z (c) − 1 (d) 1 1 0 0  1 0 0 
15. If α, β ≠ 0 and f (n ) = α n + β n and (a)  0 1 0  (b)  0 − 1 0 
   
3 1 + f (1) 1 + f (2)  0 0 −1  0 0 −1
1 + f (1) 1 + f (2) 1 + f (3) − 1 0 0   1 0 0
1 + f (2) 1 + f (3) 1 + f (4) (c)  0 − 1 0  (d)  0 1 0
   
 0 0 −1  0 0 1
= K (1 − α )2(1 − β )2 (α − β )2, then K is equal to
(2014 Main)
1 24. Which of the following values of α satisfy the equation
(a) αβ (b) (c) 1 (d) −1 (1 + α )2 (1 + 2α )2 (1 + 3 α )2
αβ
(2 + α )2 (2 + 2α )2 (2 + 3 α )2 = − 648 α ?
16. Let P = [aij ] be a 3 × 3 matrix and let Q = [bij ], where
bij = 2i + j aij for 1 ≤ i , j ≤ 3. If the determinant of P is 2, (3 + α )2 (3 + 2α )2 (3 + 3 α )2
(2015 Adv.)
then the determinant of the matrix Q is (2012) (a) −4 (b) 9 (c) −9 (d) 4
(a) 210 (b) 211 (c) 212 (d) 213
25. Let M and N be two 3 × 3 matrices such that MN = NM .
17. If A = α 2
2 α
and| A3| = 125, then the value of α is Further, if M ≠ N 2 and M 2 = N 4, then
  (2004, 1M) (2014 Adv.)
(a) ± 1 (b) ± 2 (c) ± 3 (d) ± 5 (a) determinant of (M 2 + MN 2 ) is 0
18. The number of distinct real roots of (2001, 1M) (b) there is a 3 × 3 non-zero matrixU such that (M 2 + MN 2 ) U
is zero matrix
 sin x cos x cos x  π π (c) determinant of (M 2 + MN 2 ) ≥ 1
 cos x sin x cos x  = 0 in the interval − ≤ x ≤ is (d) for a 3 × 3 matrix U, if (M 2 + MN 2 ) U equals the zero
 cos x cos x sin x  4 4
matrix, then U is the zero matrix
(a) 0 (b) 2
 a b aα + b
(c) 1 (d) 3 26. The determinant  b c bα + c 
 1 x x+1  aα + b bα + c 0 
19. If f (x) =  2x x (x − 1) (x + 1) x ,
 3x (x − 1) x (x − 1) (x − 2) (x + 1) x (x − 1)  is equal to zero, then (1986, 2M)
(a) a, b, c are in AP
then f (100) is equal to (1999, 2M)
(b) a, b, c are in GP
(a) 0 (b) 1 (c) a, b, c are in HP
(c) 100 (d) –100 (d) (x − α ) is a factor of ax2 + 2bx + c
20. The parameter on which the value of the determinant
 1 a a2  Integer & Numerical Answer Type Questions

 cos ( p − d ) x cos px cos ( p + d) x 
 27. Let P be a matrix of order 3 × 3 such that all the entries in
sin ( p − d ) x sin px sin ( p + d ) x  P are from the set { − 1, 0, 1}. Then, the maximum possible
does not depend upon, is (1997, 2M) value of the determinant of P is ......... .
(a) a (b) p (c) d (d) x
xp + y x y
Fill in the Blanks
21. The determinant yp + z y z = 0, if 28. For positive numbers x, y and z, the numerical value of the
1 log x y log x z
0 xp + y yp + z (1997C, 2M)
determinant log y x 1 log y z is…… .
(a) x, y, z are in AP (b) x, y, z are in GP
log z x log z y 1 (1993, 2M)
(c) x, y, z are in HP (d) xy, yz , zx are in AP
22. Consider the set A of all determinants of order 3 with 1 a a − bc
2

entries 0 or 1 only. Let B be the subset of A consisting 29. The value of the determinant 

1 b b2 − ca is … .

of all determinants with value 1. Let C be the subset 
1 c c2 − ab 
of A consisting of all determinants with value –1. (1988, 2M)
Then,
(a) C is empty (1981, 2M)
x 3 7
30. Given that x = − 9 is a root of 2 x 2  = 0, the other two
(b) B has as many elements as C 7 6 x
(c) A = B ∪ C roots are... and... . (1983, 2M)
(d) B has twice as many elements as C
Matrices and Determinants 139

1 4 20  40. For all values of A , B, C and P , Q , R, show that


1 − 2
31. The solution set of the equation  = 0 is… .
5  (1994, 4M)
1 2x 5x2 (1981, 2M)  cos ( A − P ) cos ( A − Q ) cos ( A − R) 

 cos (B − P ) cos (B − Q ) cos (B − R) = 0
 λ2 + 3λ λ − 1 λ + 3
 cos (C − P ) cos (C − Q ) cos (C − R) 
32. Let pλ + qλ + rλ + sλ + t = 
4 3 2
 λ+1 − 2λ λ − 4 
 λ −3 λ + 4 3λ  41. For a fixed positive integer n, if
be an identity in λ , where p,q,r,s and t are constants.  n! (n + 1)! (n + 2)! 
Then, the value of t is…. . (1981, 2M) D =  (n + 1)! (n + 2)! (n + 3)! ,
 (n + 2)! (n + 3)! (n + 4)! 
True/False
 D 
1 a bc 1 a a2 then show that  3
− 4 is divisible by n. (1992, 4M)
2
 (n !) 
33. The determinants 1 b ca and 1 b b are not
2  p b c
1 c ab 1 c c 42. If a ≠ p, b ≠ q, c ≠ r and a q c  = 0
identically equal. (1983, 1M) a b r
p q r
Then, find the value of + + . (1991, 4M)
Analytical and Descriptive Questions p−a q−b r−c
34. If M is a 3 × 3 matrix, where M T M = I and det (M ) = 1, 43. Let the three digit numbers A28, 3B9 and 62C, where A,
then prove that det (M − I ) = 0 (2004, 2M) B and C are integers between 0 and 9, be divisible by a
35. Let a , b, c be real numbers with a + b + c = 1 . Show
2 2 2 fixed integer k. Show that the determinant
that the equation  A 3 6
 8 9 C is divisible by k. (1990, 4M)
 ax − by − c bx + ay cx + a  2 B 2 
 bx + ay − ax + by − c cy + b  = 0 represents
 cx + a cy + b − ax − by + c   a −1 n 6 
a straight line. (2001, 6M) 44. Let ∆ a = 
 (a − 1)2 2n 2 4n − 2  
 (a − 1) 3n 2 − 3n 
3
3n3
36. Prove that for all values of θ n

sin θ cos θ sin 2θ


Show that ∑ ∆ a = c ∈ constant. (1989, 5M)
a =1

 2π   2π   4π 
sin θ +  cos θ +  sin 2θ +  =0 45. Show that
 3  3  3 x+1 x+ 2
 xCr x
Cr + 1 x
Cr + 2   xCr Cr + 1 C r + 2
 2π   2π   4π 
sin θ −  cos θ −  sin 2 θ −   yC y
Cr + 1 y
C r + 2 =  y C r y+1
Cr + 1 C r + 2
y+ 2
 3  3  3  (2000, 3M) z r  z 
C z z z +1 z+2
 r Cr + 1 Cr + 2   Cr Cr + 1 C r + 2
37. Suppose, f (x) is a function satisfying the following
(1985, 3M)
conditions
(a) f (0) = 2, f (1) = 1 46. If α be a repeated root of a quadratic equation f (x) = 0
(b) f has a minimum value at x = 5 / 2, and and A (x), B (x) and C (x) be polynomials of degree 3, 4
 2ax 2ax − 1 2ax + b + 1 and 5 respectively, then show that
(c) for all x, f ′ (x) = 
 b b+ 1 −1 
  A (x) B (x) C (x) 
 2(ax + b) 2ax + 2b + 1 2ax + b   A (α ) B (α ) C (α ) 
where a, b are some constants. Determine the constants  A′ (α ) B′ (α ) C′ (α ) 
a, b and the function f (x). (1998, 3M) is divisible by f (x), where prime denotes the
bc ca ab derivatives. (1984, 3M)
38. Find the value of the determinant p q r , where 47. Without expanding a determinant at any stage, show
1 1 1 that
a , b and c are respectively the pth , qth and rth terms of a  x2 + x x+1 x−2 
harmonic progression. (1997C, 2M)  2x2 + 3x − 1 3x 3x − 3  = xA + B
 2 
2x − 1 2x − 1
39. Let a > 0, d > 0. Find the value of the determinant  x + 2x + 3 
1 1 1 where A and B are determinants of order 3 not
a a (a + d ) (a + d ) (a + 2d ) involving x. (1982, 5M)
1 1 1 48. Let a, b, c be positive and not all equal. Show that the
(a + d ) (a + d ) (a + 2d ) (a + 2d ) (a + 3d )  a b c
1 1 1 value of the determinant  b c a is negative.
(a + 2d ) (a + 2d ) (a + 3d ) (a + 3d ) (a + 4d )  c a b (1981, 4M)
140 Matrices and Determinants

Topic 3 Adjoint and Inverse of a Matrix


Objective Questions I (Only one correct option)  2 −3 
6. If A =   , then adj (3 A + 12 A ) is equal to
2

 −4 1  (2017 Main)
 sin 4 θ − 1 − sin 2 θ  −1  72 − 84  51 63
1. Let M =   = α I + βM , (a)  (b) 
 
1 + cos θ cos 4 θ 
2
 − 63 51   84 72
where α = α (θ ) and β = β (θ ) are real numbers, and I is the  51 84  72 − 63
(c)   (d)  
2 × 2 identity matrix. If α * is the minimum of the set  63 72  − 84 51 
{α (θ ): θ ∈ [0, 2π )} and β * is the minimum of the set
{ β(θ ) : θ ∈ [0, 2π) }, then the value of α * + β * is 5a − b
7. If A =  and A adj A = AAT , then 5a + b is equal
(2019 Adv.) 3 2 
17 31
(a) − (b) − to (2016 Main)
16 16 (a) − 1 (b) 5
37 29
(c) − (d) − (c) 4 (d) 13
16 16
8. If A is a 3 × 3 non-singular matrix such that AAT = AT A
5 2α 1 
and B = A −1 AT , then BBT is equal to
2. If B = 0 2 1  is the inverse of a 3 × 3 matrix A, then
(2014 Main)
  (a) I + B (b) I
α 3 −1 (c) B −1 (d) (B −1 )T
the sum of all values of α for which det ( A ) + 1 = 0, is
(2019 Main, 12 April I) 1 α 3
(a) 0 (b) −1 9. If P = 1 3 3 is the adjoint of a 3 × 3 matrix A and
 
(c) 1 (d) 2 2 4 4
et e− t cos t e− t sin t  | A | = 4 , then α is equal to (2013 Main)
3. If A = e −e cos t − e sin t − e sin t + e cos t  then A
 t −t −t −t −t (a) 4 (b) 11
(c) 5 (d) 0
et
 2e− t sin t −2e− t cos t 

is (2019 Main, 9 Jan II) 10. If P is a 3 × 3 matrix such that PT = 2P + I, where PT is
(a) invertible only when t = π the transpose of P and I is the 3 × 3 identity matrix, then
(b) invertible for every t ∈ R  x  0
(c) not invertible for any t ∈ R there exists a column matrix, X =  y ≠ 0 such that
π    
(d) invertible only when t =  z  0
2 (2012)
 0
4. Let A and B be two invertible matrices of order 3 × 3. If (a) PX =  0 (b) PX = X
 
det( ABAT ) = 8 and det( AB− 1 ) = 8, then det(BA − 1BT ) is  0
equal to (2019 Main, 11 Jan II)
1 (c) PX = 2X (d) PX = − X
(a) 1 (b)
4 11. Let ω ≠ 1 be a cube root of unity and S be the set of all
1
(c) (d) 16  1 a b
16
non-singular matrices of the form  ω 1 c , where
cos θ − sin θ   2 
5. If A =  , then the matrix ω ω 1 
sin θ
 cos θ 
each of a , b and c is either ω or ω 2. Then, the number of
π
A −50 when θ = , is equal to distinct matrices in the set S is (2011)
12 (2019 Main, 9 Jan I)
(a) 2 (b) 6
 1 3  3 1
 −  (c) 4 (d) 8
(a)  2 2  
(b)  2 2
− 3 1   1 3 12. Let M and N be two 3 × 3 non-singular
 2 2   2 2  skew-symmetric matrices such that MN = NM . If PT
 3 1   1 3 denotes the transpose of P, then

 2   2  M 2N 2(M T N )−1 (MN −1 )T is equal to (2011)
(c)  2 (d)  2
− 1 3  3 1  (a) M 2 (b) −N 2
 2 2   2 2  (c) −M 2 (d) MN
Matrices and Determinants 141

1 0 0 k k2
identity matrix of order 3. If q23 = − and det (Q ) = ,
13. If A = 0 1 1, 6 A −1 = A 2 + cA + dI , then (c, d ) is 8 2
 
0 −2 4 (2005, 1M)
then (2016 Adv.)
(a) α = 0, k = 8 (b) 4α − k + 8 = 0
(a) (− 6, 11) (b) (− 11, 6)
(c) det (P adj (Q )) = 29 (d) det (Q adj (P )) = 213
(c) (11, 6) (d) (6, 11)
17. Let M be a 2 × 2 symmetric matrix with integer entries.
Objective Questions II Then, M is invertible, if (2014 Adv.)
(One or more than one correct option) (a) the first column of M is the transpose of the second row
of M
14. Let M be a 3 × 3 invertible matrix with real entries and (b) the second row of M is the transpose of the first column
let I denote the 3 × 3 identity matrix. If of M
M −1 = adj(adj M ), then which of the following (c) M is a diagonal matrix with non-zero entries in the
statements is/are ALWAYS TRUE? (2020 Adv.) main digonal
(d) the product of entries in the main diagonal of M is not
(a) M = I (b) det M = 1
the square of an integer
(c) M 2 = I (d) (adj M )2 = I
1 4 4 
0 1 a  − 1 1 − 1 18. If the adjoint of a 3 × 3 matrix P is 2 1 7, then the
15. Let M = 1 2 3  and adj M =  8 − 6 2   
    1 1 3 
3 b 1  − 5 3 − 1 possible value(s) of the determinant of P is/are
where a and b are real numbers. Which of the following (a) − 2 (b) − 1 (c) 1 (d) 2
options is/are correct? (2019 Adv.)
(a) det (adj M 2 ) = 81 Integer & Numerical Answer Type Question
α   1 19. Let k be a positive real number and let
(b) If M β  =  2, then α − β + γ = 3  2k − 1 2 k 2 k 
   
 γ   3  
A= 2 k 1 − 2k  and
(c) (adj M )− 1 + adj M − 1 = − M  − 2 k 2k − 1 
(d) a + b = 3
 0 2k − 1 k 
3 −1 −2  
B =  1 − 2k 0 2 k
16. Let P = 2 0 α , where α ∈ R. Suppose Q = [qij ] is a 
   − k − 2 k 0 
3 −5 0 
matrix such that PQ = kI , where k ∈ R, k ≠ 0 and I is the If det (adj A ) + det (adj B) = 106, then [k] is equal to……
(2010)

Topic 4 Solving System of Equations


Objective Questions I (Only one correct option)
1. If [x] denotes the greatest integer ≤ x , then the system of
liner equations [sin θ ]x + [− cos θ ] y = 0, [cot θ ]x + y = 0 2. Let λ be a real number for which the system of linear
(2019 Main, 12 April II) equations
π 2π  x + y + z = 6, 4x + λy − λz = λ − 2 and
(a) have infinitely many solutions if θ ∈  , 
2 3 3x + 2 y − 4z = − 5
7π 
and has a unique solution if θ ∈  π , . has infinitely many solutions. Then λ is a root of the
 6 quadratic equation (2019 Main, 10 April II)
(b) has a unique solution if (a) λ2 − 3λ − 4 = 0 (b) λ2 + 3λ − 4 = 0
π 2π   7π 
θ ∈  ,  ∪  π,  (c) λ2 − λ − 6 = 0 (d) λ2 + λ − 6 = 0
2 3  6
π 2π 
(c) has a unique solution if θ ∈  , 
3. If the system of linear equations
2 3 x+ y+ z =5
7π 
and have infinitely many solutions if θ ∈  π ,  x + 2 y + 2z = 6
 6
x + 3 y + λz = µ,(λ , µ ∈ R), has infinitely many solutions,
(d) have infinitely many solutions if then the value of λ + µ is
π 2π 7π 
θ ∈  ,  ∪  π ,  (2019 Main, 10 April I)
2 3  6 (a) 7 (b) 12 (c) 10 (d) 9
142 Matrices and Determinants

4. If the system of equations 2x + 3 y − z = 0, x + ky − 2z = 0 12. If the system of linear equations


and 2x − y + z = 0 has a non-trivial solution (x, y, z ), then x − 4 y + 7z = g,
x y z 3 y − 5z = h, − 2x + 5 y − 9z = k
+ + + k is equal to
y z x (2019 Main, 9 April II) is consistent, then (2019 Main, 9 Jan II)
1 1 3 (a) 2 g + h + k = 0 (b) g + 2h + k = 0
(a) −4 (b) (c) − (d)
2 4 4 (c) g + h + k = 0 (d) g + h + 2k = 0
5. If the system of linear equations 13. The system of linear equations
x − 2 y + kz = 1 , 2x + y + z = 2 ,3x − y − kz = 3 x + y + z = 2, 2x + 3 y + 2z = 5
has a solution (x, y, z ), z ≠ 0, then (x, y) lies on the 2x + 3 y + ( a 2 − 1)z = a + 1 (2019 Main, 9 Jan I)
straight line whose equation is (2019 Main, 8 April II)
(a) has infinitely many solutions for a = 4
(a) 3x − 4 y − 4 = 0 (b) 3x − 4 y − 1 = 0 (b) is inconsistent when a = 4
(c) 4x − 3 y − 4 = 0 (d) 4x − 3 y − 1 = 0 (c) has a unique solution for|a| = 3
6. The greatest value of c ∈ R for which the system of (d) is inconsistent when|a| = 3
linear equations x −cy − cz = 0, cx − y + cz = 0, 14. If the system of linear equations
cx + cy − z = 0 has a non-trivial solution, is x + ky + 3z = 0, 3x + ky − 2z = 0, 2x + 4 y − 3z = 0
(2019 Main, 8 April I) xz
1 has a non-zero solution ( x , y , z ), then is equal to
(a) −1 (b) (c) 2 (d) 0 y2 (2018 Main)
2
(a) −10 (b) 10 (c) −30 (d) 30
7. The set of all values of λ for which the system of linear
equations x − 2 y − 2z = λx, x + 2 y + z = λyand 15. The system of linear equations
− x − y = λz x + λy − z = 0; λx − y − z = 0; x + y − λz = 0
has a non-trivial solution (2019 Main, 12 Jan II) has a non-trivial solution for (2016 Main)
(a) contains exactly two elements. (a) infinitely many values of λ (b) exactly one value of λ
(b) contains more than two elements. (c) exactly two values of λ (d) exactly three values of λ
(c) is a singleton. 16. The set of all values of λ for which the system of linear
(d) is an empty set. equations 2x1 − 2x2 + x3 = λx1, 2x1 − 3x2 + 2x3 = λx2 and
8. An ordered pair (α , β) for which the system of linear − x1 + 2x2 = λx3 has a non-trivial solution (2015 Main)

equations (2019 Main, 12 Jan I)


(a) is an empty set
(b) is a singleton set
(1 + α )x + βy + z = 2
(c) contains two elements
αx + (1 + β ) y + z = 3
9d) contains more than two elements
ax + βy + 2z = 2
has a unique solution, is 17. The number of value of k, for which the system of
(a) (2, 4) (b) (− 4, 2)
equation
(c) (1, − 3) (d) (−3, 1) (k + 1) x + 8 y = 4 y ⇒ kx + (k + 3) y = 3k − 1
9. If the system of linear equations has no solution, is (2013 Main)
(a) infinite (b) 1 (c) 2 (d) 3
2x + 2 y + 3z = a
3x − y + 5z = b 18. The number of 3 × 3 matrices A whose entries are either
x − 3 y + 2z = c  x  1
0 or 1 and for which the system A  y = 0 has exactly
where a , b, c are non-zero real numbers, has more than    
one solution, then (2019 Main, 11 Jan I)  z  0
(a) b − c − a = 0 (b) a + b + c = 0 two distinct solutions, is (2010)
(c) b − c + a = 0 (d) b + c − a = 0 (a) 0 (b) 29 − 1 (c) 168 (d) 2
10. The number of values of θ ∈ (0, π ) for which the system 19. Given, 2x − y + 2z = 2, x − 2 y + z = − 4, x + y + λz = 4,
of linear equations then the value of λ such that the given system of
x + 3 y + 7z = 0, equations has no solution, is (2004, 1M)
− x + 4 y + 7z = 0, (a) 3 (b) 1 (c) 0 (d) –3
(sin 3θ )x + (cos 2θ ) y + 2z = 0 20. The number of values of k for which the system of
has a non-trivial solution, is (2019 Main, 10 Jan II) equations (k + 1) x + 8 y = 4k and kx + (k + 3) y = 3k − 1
(a) two (b) three (c) four (d) one has infinitely many solutions, is/are (2002, 1M)

11. If the system of equations (a) 0 (b) 1


(c) 2 (d) ∞
x+ y+z=5 x + 2 y + 3z = 9
x + 3 y + αz = β 21. If the system of equations x + ay = 0, az + y = 0 and
has infinitely many solutions, then β − α equals ax + z = 0 has infinite solutions, then the value of a is
(2019 Main, 10 Jan I) (a) –1 (b) 1
(a) 8 (b) 18 (c) 21 (d) 5 (c) 0 (d) no real values
Matrices and Determinants 143

22. If the system of equations x − ky − z = 0, kx − y − z = 0, (a) x + 2 y + 3z = b1 , 4 y + 5z = b2 and x + 2 y + 6z = b3


x + y − z = 0 has a non-zero solution, then possible values (b) x + y + 3z = b1 , 5x + 2 y + 6z = b2 and − 2x − y − 3z = b3
of k are (2000, 2M) (c) − x + 2 y − 5z = b1 , 2x − 4 y + 10z = b2 and x − 2 y + 5z = b3
(a) –1, 2 (b) 1, 2 (c) 0, 1 (d) –1, 1 (d) x + 2 y + 5z = b1 , 2x + 3z = b2 and x + 4 y − 5z = b3

Assertion and Reason Fill in the Blank


For the following questions, choose the correct answer from
26. The system of equations λx + y + z = 0 , − x + λy + z = 0
the codes (a), (b), (c) and (d) defined as follows.
and − x − y + λz = 0 will have a non-zero solution, if
(a) Statement I is true, Statement II is also true; Statement real values of λ are given by ... (1982, 2M)
II is the correct explanation of Statement I
(b) Statement I is true, Statement II is also true; Statement
II is not the correct explanation of Statement I Analytical and Descriptive Questions
(c) Statement I is true; Statement II is false. a 0 1 a 1 1 f  a 2
(d) Statement I is false; Statement II is true. 27. A = 1 c b, B = 0 d c , U =  g , V = 0 
1 b  f h  h  0 
23. Consider the system of equations x − 2 y + 3 z = −1 , d g  
x − 3 y + 4z = 1 and − x + y − 2z = k If there is a vector matrix X, such that AX = U has
Statement I The system of equations has no solution for infinitely many solutions, then prove that BX = V
k ≠ 3 and cannot have a unique solution. If a f d ≠ 0. Then, prove
1 3 −1 that BX = V has no solution. (2004, 4M)
Statement II The determinant −1 −2 k ≠ 0 , for
28. Let λ and α be real. Find the set of all values of λ for
1 4 1 which the system of linear equations
k ≠ 0. (2008, 3M)
λx + (sin α ) y + (cos α )z = 0,

Objective Questions II (One or more than one correct) x + (cos α ) y + (sin α )z = 0


and − x + (sin α ) y − (cos α )z = 0
24. Let x ∈ R and let
has a non-trivial solution.
1 1 1 2 x x
For λ = 1, find all values of α.
P = 0 2 2, Q = 0 4 0 and R = PQP −1,
(1993, 5M)
   
0 0 3 x x 6 29. Let α 1 , α 2, β1 , β 2 be the roots of ax + bx + c = 0 and
2

the which of the following options is/are correct? px2 + qx + r = 0, respectively. If the system of equations
(2019 Adv.) α 1 y + α 2z = 0 and β1 y + β 2z = 0 has a non-trivial solution,
(a) There exists a real, number x such that PQ = QP b2 ac
then prove that 2 = .
 1  1 q pr (1987, 3M)
(b) For x = 0, if R  a  = 6  a , then a + b = 5
    30. Consider the system of linear equations in x, y, z
 b  b
(sin 3θ ) x − y + z = 0, (cos 2θ ) x + 4 y + 3z = 0 and
(c) For x = 1, there exists a unit vector α $i + β$j + γk$ for 2x + 7 y + 7z = 0
α   0
Find the values of θ for which this system has
which R β  =  0
    non-trivial solution. (1986, 5M)
 γ   0
31. Show that the system of equations, 3x − y + 4z = 3,
 2 x x
x + 2 y − 3z = − 2 and 6x + 5 y + λz = − 3 has atleast one
(d) det R = det  0 4 0 + 8, for all x ∈ R
  solution for any real number λ ≠ − 5. Find the set of
 x x 5 solutions, if λ = − 5. (1983, 5M)
 b1  32. For what values of m, does the system of equations
25. Let S be the set of all column matrices  b2  such that b1, 3x + my = m and 2x − 5 y = 20 has a solution satisfying
  the conditions x > 0, y > 0? (1979, 3M )
b3 
b2, b3 ∈R and the system of equations (in real variables) 33. For what value of k, does the following system of
− x + 2 y + 5z = b1 equations possess a non-trivial solution over the set of
rationals
2x − 4 y + 3z = b2
x − 2 y + 2z = b3 x + y − 2z = 0, 2x − 3 y + z = 0, and x − 5 y + 4z = k
has at least one solution. Then, which of the following Find all the solutions. (1979, 3M )
system(s) (in real variables) has (have) at least one
 b1 
34. Given, x = cy + bz , y = az + cx, z = bx + ay, where x, y, z
solution for each  b2  ∈ S ? are not all zero, prove that a 2 + b2 + c2 + 2ab = 1.
  (1978, 2M)
 b3 
144 Matrices and Determinants

Integer & Numerical Answer Type Questions 37. For a real number α , if the system
35. If the system of equations  1 α α 2  x   1 
    
x − 2 y + 3z = 9  α 1 α   y = −1
α α 1   z   1 
2
2x + y + z = b  
x − 7 y + az = 24, has infinitely many solutions, then a − b of linear equations, has infinitely many solutions, then
is equal to ............. . (2020 Main, 4 Sep I) 1 + α + α2 = (2017 Adv.)
0  −1 
36. If the system of linear equations,
38. Let M be a 3 × 3 matrix satisfying M 1 =  2 ,
x+ y+ z =6    
0  3 
x + 2 y + 3z = 10 1 1 1  0 
3 x + 2 y + λz = µ M −1 =  1 , and M 1 =  0  ,
       
has more than two solutions, then µ − λ2 is equal to  0  −1 1 12
………… . (2020 Main, 7 Jan II) Then, the sum of the diagonal entries of M is … (2011)

Answers
Topic 1 Topic 3
1. (b) 2. (b) 3. (c) 4. (a) 1. (d) 2. (c) 3. (b) 4. (c)
5. (b) 6. (b) 7. (d) 8. (a) 5. (c) 6. (b) 7. (b) 8. (b)
9. (d) 10. (a) 11. (a,b,d) 12. (c, d) 9. (b) 10. (d) 11. (a) 12. (c)
13. (c, d) 14. (b, c, d) 15. (d) 16. (b) 13. (a) 14. (b,c,d) 15. (b,c,d) 16. (b,c)
17. (a) 18. (d) 19. (c) 20. (d) 17. (c, d) 18. (a,d) 19. (4)
21. (4) 22. (5) 23. (1)
Topic 4
Topic 2 1. (a) 2. (c) 3. (c) 4. (b)
1. (d) 2. (a) 3. (a) 4. (c) 5. (c) 6. (b) 7. (c) 8. (a)
5. (b) 6. (d) 7. (c) 8. (a) 9. (a) 10. (a) 11. (a) 12. (a)
9. (b) 10. (d) 11. (c) 12. (a) 13. (d) 14. (b) 15. (d) 16. (c)
13. (c) 14. (a) 15. (c) 16. (d) 17. (d) 18. (a) 19. (b) 20. (b)
17. (c) 18. (c) 19. (a) 20. (b) 21. (a) 22. (d) 23. (a) 24. (b,d)
21. (b) 22. (b) 23. (a, c) 24. (b, c) 25. (a,d) 26. λ=0
π
25. (a, b) 26. (b,d) 27. (4) 28. (0) 28. − 2 ≤ λ ≤ 2 , α = nπ , nπ +
4
29. (0) 30. (2 and 7) 31. {–1,2} 32. (0) π
 1 5 1 2 5  30. θ = nπ , nπ + ( −1 )n , n ∈Z
33. False 37. a = , b = − and f ( x ) = x − x + 2 6
 4 4 4 4  4 − 5k 13k − 9 15
31. x = ,y = , z = k 32. m < − or m > 30
 4d 4  7 7 2
38. (0) 39.  
 a (a + d ) (a + 2d ) (a + 3d ) (a + 4d )
2 3 2 33. (k = 0, the given system has infinitely many solutions)
42. (2) 35. (5) 36. (13) 37. (1) 38. (9)
Hints & Solutions
Topic 1 Types of Matrices, Addition, cos α − sin α 
3. Given, matrix A =  
Subtraction and Transpose of a sin α cos α 
Matrix cos α − sin α  cos α − sin α 
∴ A2 = 
1. Given matrix A is a symmetric and matrix B is a sin α cos α  sin α cos α 
skew-symmetric.  cos2 α − sin 2 α − cosα sin α − sin α cosα 
= 
∴ A = A and B = − B
T T  sin α cosα + cosα sin α − sin 2 α + cos2 α 
2 3  cos 2 α − sin 2 α 
Since, A + B =  (given) …(i) =

5 − 1 sin 2 α cos 2 α 
On taking transpose both sides, we get Similarly,
T cos(nα ) − sin(nα )
2 3  An =  , n ∈ N
( A + B)T =   sin(nα ) cos(nα ) 
5 − 1
cos(32 α ) − sin(32 α ) 0 −1
2 5  ⇒ A32 =  =  (given)
⇒ AT + BT =   … (ii) sin(32 α ) cos(32 α )  1 0 
3 − 1 So, cos(32 α ) = 0 and sin(32 α ) = 1
Given, A = A and B = − B
T T
π π
⇒ 32 α = ⇒ α =
2 5  2 64
⇒ A−B= 
3 − 1 4. Given matrix
On solving Eqs. (i) and (ii), we get 1 0 0 0 0 0 1 0 0
2 4  0 − 1 P = 3 1 0 = 3 0 0 + 0 1 0
A=  and B = 1 0       
4 − 1    9 3 1 9 3 0 0 0 1
2 4  0 − 1  4 − 2 ⇒ P=X + I (let)
So, AB =   =  Now, P5 = (I + X )5
4 − 1 1 0  − 1 − 4
= I + 5C1 (X ) + 5C 2(X 2) + 5C3 (X 3 ) + …
2. Given matrix [Q I n = I , I ⋅ A = A and (a + x)n = nC 0a n +
n
C1a n − 1x + ...+ nC nxn]
 0 2y 1  0 0 0 0 0 0 0 0 0
A = 2x y −1 , (x, y ∈ R, x ≠ y) Here, X 2 = 3 0 0 3 0 0 = 0 0 0
      
2x − y 1  9 3 0 9 3 0 9 0 0
for which 0 0 0 0 0 0 0 0 0
and X 3 = X 2 ⋅ X = 0 0 0 3 0 0 = 0 0 0
AT A = 3I3     
9 0 0 9 3 0 0 0 0
 0 2x 2x   0 2 y 1  3 0 0
2 y y − y 2x y −1 = 0 3 0 0 0 0

     ⇒ X 4 = X 5 = 0 0 0
 1 −1 1  2x − y 1  0 0 3  
0 0 0
8x2 0 0 3 0 0 0 0 0 0 0 0
   
⇒  0 6 y 0 = 0 3 0 P5 = I + 5 3 0 0 + 10 0 0 0
2
So,
   
 0 0 3 0 0 3 9 3 0 9 0 0

Here, two matrices are equal, therefore equating the  1 0 0
corresponding elements, we get 
= 15 1 0
 
8x2 = 3 and 6 y2 = 3 135 15 1
3  2 0 0
⇒ x=±
8 and Q = I + P5 =  15 2 0 = [q ij ]
 
1 135 15 2
and y=±
2 ⇒ q21 = 15, q31 = 135 and q 32 = 15
Q There are 2 different values of x and y each. q + q31 15 + 135 150
Hence, 21 = = = 10
So, 4 matrices are possible such that AT A = 3I3 . q32 15 15
146 Matrices and Determinants

5. Given, AAT = I q31 + q32 20400 + 200 20600


Thus, = = = 103
0 2q r   0 p p  1 0 0 q21 200 200
⇒ p q − r 2q q − q = 0 1 0
   1 2 2 12 a
    
 p − q r   r − r r  0 0 1 7. Given, A = 2 1 −2, AT = 2 1 2
   
0 + 4q2 + r 2 0 + 2q2 − r 2 0 − 2q2 + r 2  1 0 0 a 2 b  2 −2 b 
 
⇒ 0 + 2q2 − r 2 p2 + q2 + r 2 p2 − q2 − r 2  = 0 1 0 1 2 2  1 2 a 
 
0 − 2q + r p −q −r p + q + r  0 0 1 and AA = 2 1 −2 2 1 2 
2 2 2 2 2 2 2 2 T
   
We know that, if two matrices are equal, then a 2 b  2 −2 b 
corresponding elements are also equal, so  9 0 a + 4 + 2b 
4q2 + r 2 = 1 = p2 + q2 + r 2, …(i) = 0 9 2a + 2 − 2b
2q2 − r 2 = 0 ⇒ r 2 = 2q2 …(ii)  
a + 4 + 2b 2a + 2 − 2b a 2 + 4 + b2 
and p2 − q 2 − r 2 = 0 …(iii)
Using Eqs. (ii) and (iii), we get It is given that, AAT = 9I
p2 = 3 q 2 …(iv)  9 0 a + 4 + 2b  1 0 0
Using Eqs. (ii) and (iv) in Eq. (i), we get ⇒  0 9 2a + 2 − 2b = 9 0 1 0
   
a + 4 + 2b 2a + 2 − 2b a + 4 + b 
2 2
4q2 + 2q2 = 1 0 0 1
⇒ 6q2 = 1  9 0 a + 4 + 2b  9 0 0
⇒ 2 p2 = 1 [using Eq. (iv)] ⇒  0 9 2a + 2 − 2b = 0 9 0
1 1    
p = ⇒ | p| = a + 4 + 2b 2a + 2 − 2b a + 4 + b  0 0 9
2 2 2
2 2
On comparing, we get
 1 0 0
6. Here, P =  4 1 0 a + 4 + 2b = 0 ⇒ a + 2b = − 4 ...(i)
  2a + 2 − 2b = 0 ⇒ a − b = − 1 …(ii)
16 4 1
and a 2 + 4 + b2 = 9 …(iii)
 1 0 0  1 0 0  1 00
∴ P = 4 1 0  4 1 0 =  4 + 4
2 
1 0 On solving Eqs. (i) and (ii), we get
    
16 4 1 16 4 1 16 + 32 4 + 4 1 a = − 2, b = − 1
 1 0 0 This satisfies Eq. (iii)
=  4 ×2 1 0 …(i) Hence, (a , b) ≡ (−2,−1)
 
16 (1 + 2) 4 × 2 1  3 /2 −1 /2   3 /2 1 /2 
8. Now, PT P =   
 1 0 0  1 0 0  1 /2 3 /2  −1 /2 3 /2
and P3 =  4 × 2 1 0  4 1 0 1 0
   ⇒ PT P = 
16 (1 + 2) 4 × 2 1 16 4 1 
0 1
 1 0 0 ⇒ PT P = I
= 4 ×3 1 0 ...(ii) ⇒ PT = P −1
 
16 (1 + 2 + 3) 4 × 3 1 Since, Q = PAPT
From symmetry, ∴ P QT 2005
P = PT [(PAPT )(PAPT ) K 2005 times ] P
 1 0 0
= (PT P ) A (PT P ) A (PT P ) K (PT P ) A (PT P )
P =
50  4 × 50 1 0 1444444442444444443
  2005 times
16 (1 + 2 + 3 + ... + 50) 4 × 50 1 = IA 2005 = A 2005
Q P50 − Q = I [given] 1 1
∴ A= 
  0 1
 1 − q11 − q12 − q13  1 0 0
∴  200 − q21 1 − q22 − q23  = 0 1 0 1 1 1 1 1 2
A2 = 
 50     = 
16 × (51) − q31 200 − q32 1 − q33  0 0 1 0 1 0 1 0 1
 2  1 2 1 1 1 3
16 × 50 × 51 A3 =   = 
⇒ 200 − q21 = 0, − q31 = 0, 0 1 0 1 0 1
2
200 − q32 = 0 ……………………
∴ q21 = 200, q32 = 200, q31 = 20400 ……………………
Matrices and Determinants 147

1 2005 1
A 2005 = 
0 1  Now, Let R = 1, then
 
1 2005 1
∴ PTQ 2005 P = 
1 
6 6
0 XR = Σ (PK Q PKT )R = Σ (PK QPKT R)
K =1 K =1

9. Given, A = α 0
, B=
1 0 6
1  1 5 1 = Σ (PK Q R) [Q PKT R = R]
K =1
α 0  α 0  α 2 0 
⇒ A2 =  = 6 6 2 2 2 6
1 1 1 1 α + 1 1 6 6
= Σ PK 3 = Σ PK 3 = 2 2 2 3
K =1   K =1     
Also, given, A 2 = B 6 6 2 2 2 6
α 2 0 1 0 30 1 1
⇒ =
α + 1 1 5 1
⇒  
XR = 30 ⇒ XR = 30R ⇒ X 1 = 30 1
 
     
⇒ α 2 = 1 and α + 1 = 5 30 1 1
Which is not possible at the same time. ⇒ (X − 30 I ) R = 0 ⇒|X − 30 I| = 0
∴ No real values of α exists. So, (X − 30I ) is not invertible and value of α = 30.
10. If A and B are square matrices of equal degree, then Hence, options (a), (b) and (d) are correct.
A+ B=B+ A 12. Given, X T = − X , Y T = −Y , Z T = Z
11. Given matrices, (a) Let P = Y 3 Z 4 − Z 4Y 3
1 0 0 1 0 0 0 1 0 Then, PT = (Y 3 Z 4 )T − (Z 4 Y 3 )T
P1 = I = 0 1 0, P2 = 0 0 1, P3 = 1 0 0 = (Z T )4 (Y T )3 − (Y T )3 (Z T )4
     
0 0 1 0 1 0 0 0 1 = − Z 4Y 3 + Y 3 Z 4 = P
0 1 0 0 0 1 0 0 1 ∴ P is symmetric matrix.
P4 = 0 0 1, P5 = 1 0 0, P6 = 0 1 0 (b) Let P = X 44 + Y 44
      Then, PT = (X T )44 + (Y T )44
1 0 0 0 1 0 1 0 0
= X 44 + Y 44 = P
2 1 3
6 ∴ P is symmetric matrix.
and X = Σ Pk 1 0 2 PKT
K =1   (c) Let P = X 4Z 3 − Z 3 X 4
3 2 1
Then, PT = (X 4Z 3 )T − (Z 3 X 4 )T
Q P1T = P1 , P2T = P2, P3T = P3 , P4T = P5 , P5T = P4 and = (Z T )3 (X T )4 − (X T )4 (Z T )3
2 1 3 = Z 3 X 4 − X 4Z 3 = − P
P6T = P6 and Let Q = 1 0 2 and Q QT = Q ∴ P is skew-symmetric matrix.
 
3 2 1 (d) Let P = X 23 + Y 23
Now, X = (P1QP1T ) + (P2QP2T ) + (P3QP3T ) + (P4QP4T ) Then, PT = (X T )23 + (Y T )23 = − X 23 − Y 23 = − P
+ (P5QP5T ) + (P6QP6T ) ∴ P is skew-symmetric matrix.
So, X = (P1QP1 ) + (P2QP2 ) + (P3QP3T )T + (P4QP4T )T
T T T T T
13. (a) (N T MN )T = N T M T (N T )T = N T M T N , is symmetric
+ (P5QP5T )T + (P6QP6T )T if M is symmetric and skew-symmetric, if M is
skew-symmetric.
= P1QP1T + P2QP2T + P3QP3T + P4QP4T + P5QP5T + P6QP6T (b) (MN − NM )T = (MN )T − (NM )T
[Q ( ABC )T = CT BT AT and ( AT )T = A and QT = Q] = NM − MN = − (MN − NM )
⇒X = X
T ∴ Skew-symmetric, when M and N are symmetric.
⇒ X is a symmetric matrix. (c) (MN )T = N T M T = NM ≠ MN
The sum of diagonal entries of X = Tr (X ) ∴ Not correct.
6 (d) (adj MN ) = (adj N ) ⋅ (adj M )
= Σ Tr (Pi Q PiT )
i =1 ∴ Not correct.
P = [ pij ]n × n with pij = wi + j
6
= Σ Tr (Q PiT Pi ) [QTr ( ABC ) = Tr (BCA )] 14. Here,
i =1
∴ When n = 1
6
= Σ Tr (QI ) [QPi’s are orthogonal matrices] P = [ pij ]1 × 1 = [ω 2]
i =1
⇒ P 2 = [ω 4 ] ≠ 0
6
= Σ Tr ( Q ) = 6 Tr (Q ) = 6 × 3 = 18 ∴ When n =2
i =1
148 Matrices and Determinants

 p11 p12  ω 2 ω3  ω 2 1  a b
P = [ pij ]2× 2 =  = 3 4
=  20. Given, A =  , a, b, c ∈ {0, 1, 2 ,... , p − 1}
 p21 p22 ω ω   1 ω  c a 
ω 2 1  ω 2 1  If A is skew-symmetric matrix, then a = 0, b = − c
P2 =    ∴ | A|= − b2
 1 ω  1 ω
ω 4 + 1 ω 2 + ω  Thus, P divides| A|, only when b = 0. ...(i)
⇒ P2 =  2 2
≠0 Again, if A is symmetric matrix, then b = c and
ω + ω 1 + ω  | A|= a 2 − b2
When n = 3 Thus, p divides| A|, if either p divides (a − b) or p
ω 2 ω3 ω 4  ω 2 1 ω  divides (a + b).
    p divides (a − b), only when a = b,
P = [ pij ]3 × 3 = ω3 ω 4 ω5  =  1 ω ω 2
ω 4 ω5 ω 6   ω ω 2 1  i.e. a = b ∈ {0, 1, 2 ,... , ( p − 1)}
   
i.e. p choices ...(ii)
ω 2 1 ω  ω 2 1 ω  0 0 0
   p divides (a + b).
P =  1 ω ω 2  1 ω ω 2 = 0 0 0 = 0
2
  ⇒ p choices, including a = b = 0 included in Eq. (i).
ω ω 2
1   ω ω 2
1  0 0 0
 ∴ Total number of choices are ( p + p − 1) = 2 p − 1
∴ P 2 = 0, when n is a multiple of 3.
a b c
P 2 ≠ 0, when n is not a multiple of 3. 21. Given, A = b c a , abc = 1 and AT A = I …(i)
⇒ n = 57 is not possible. c a b 

∴ n = 55, 58, 56 is possible. Now, AT A = I


15. As (a , b, c) lies on 2x + y + z = 1 ⇒ 2a + b + c = 1 a b c  a b c  1 0 0
⇒ b c a  b c a  = 0 1 0
⇒ 2a + 6a − 7a = 1 c a b  c a b  0 0 1
⇒ a = 1, b = 6, c = − 7  a 2 + b2 + c2 ab + bc + ca ab + bc + ca 
⇒ ab + bc + ca a 2 + b2 + c2 ab + bc + ca 
∴ 7a + b + c = 7 + 6 − 7 = 6 ab + bc + ca
 ab + bc + ca a 2 + b2 + c2 
16. If b = 6 ⇒ a = 1 and c = − 7
1 0 0
∴ ax2 + bx + c = 0 ⇒ x2 + 6x − 7 = 0 = 0 1 0
⇒ (x + 7) (x − 1) = 0 0 0 1
∴ x = 1, − 7 ⇒ a 2 + b2 + c2 = 1 and ab + bc + ca = 0 …(ii)

 1 1
n
6  6 1
2
We know, a + b + c − 3abc
3 3 3
⇒∑  −  =1 + +   + L+ ∞ =
 1 7 7  7 6 = (a + b + c)(a 2 + b2 + c2 − ab − bc − ca )
n= 0 1−
7 ⇒ a3 + b3 + c3 = (a + b + c) (1 − 0) + 3
1
= =7 [from Eqs. (i) and (ii)]
1 /7
∴ a3 + b3 + c3 = (a + b + c) + 3 …(iii)
17. If a = 2, b = 12, c = − 14
Now, (a + b + c)2 = a 2 + b2 + c2 + 2(ab + bc + ca )
3 1 3
∴ + + =1 …(iv)
ω a ωb ω c
From Eq. (iii), a3 + b3 + c3 = 1 + 3 ⇒ a3 + b3 + c3 = 4
3 1 3 3
⇒ + + = + 1 + 3ω 2 = 3ω + 1 + 3ω 2 22. Let a square matrix ‘A’ of order 2 × 2 , such that
ω 2 ω12 ω −14 ω 2
tr ( A ) = 3, is
= 1 + 3 (ω + ω 2) = 1 − 3 = − 2
x y 
A= 
18. The number of matrices for which p does not divide z 3 − x
Tr ( A ) = ( p − 1) p2 of these ( p − 1)2 are such that p
x y  x y 
divides| A|. The number of matrices for which p divides So, A2 =    
Tr ( A ) and p does not divides| A|are ( p − 1)2. z 3 − x z 3 − x
∴ Required number = ( p − 1) p2 − ( p − 1)2 + ( p − 1)2  x2 + yz xy + 3 y − xy 
= 
= p3 − p2  xz + 3 z − xz yz + (3 − x)2
19. Trace of A = 2a, will be divisible by p, iff a = 0. x2 + yz 3y  x y 
∴ A3 =   z 3 − x
| A|= a 2 − bc, for (a 2 − bc) to be divisible by p. There are  + 3z yz + 9 + x − 6x 
2

exactly ( p − 1) ordered pairs (b, c) for any value of a.
Q tr ( A3 ) = x3 + xyz + 3 yz + 3 yz + 3 yz − xyz
∴ Required number is ( p − 1)2. + 27 − 9x + 3x2 − x3 − 18x + 6x2
Matrices and Determinants 149

= 9 yz + 27 − 27x + 9x2 = − 18 (given)  π


∴ f (θ ) minimum = f   = − 4 = m
⇒ yz + 3 − 3x + x2 = − 2  2
⇒ 3x − x2 − yz = 5  π
and f (θ ) maximum = f   = 0 = M
x y  4
Q | A| = = 3x − x2 − yz = 5. ∴The ordered pair (m, M ) is (−4, 0).
z 3−x
−1 + i 3 1 + cos 2 θ sin 2 θ 4 cos 6 θ
23. Here, z= =ω 2. Let ∆ = cos θ
2
1 + sin 2 θ 4 cos 6 θ =0
2
(−ω )r ω 2s  cos θ
2
sin θ
2
1 + 4 cos 6 θ
Q P =  2s 
 ω ωr  Applying C1 → C1 + C 2, we get
(− ω )r ω 2s  (− ω )r ω 2s  2 sin 2 θ 4 cos 6 θ
P 2 =  2s  
 ω ω r   ω 2s ωr  ∆ = 2 1 + sin θ 4 cos 6 θ =0
2

 ω 2r + ω 4s ω r + 2s [(− 1)r + 1] 1 sin 2 θ 1 + 4 cos 6 θ


=  r + 2s 
ω [(− 1)r + 1] ω 4s + ω 2r  Applying R1 → R1 − 2R3 and R2 → R2 − 2R3 , we get
Given, P = − I
2
0 − sin 2 θ
− 2 − 4 cos 6 θ
∴ ω 2r + ω 4s = − 1 and ω r + 2s [(− 1)r + 1] = 0 ∆ = 0 1 − sin 2 θ − 2 − 4 cos 6 θ = 0
Since, r ∈{1, 2, 3} and (− 1)r + 1 = 0 1 sin 2 θ 1 + 4 cos 6 θ
⇒ r = {1, 3}
Also, ω 2r + ω 4s = − 1 On expanding w.r.t. C1, we get
If r = 1, then ω 2 + ω 4s = − 1 ⇒ sin 2 θ (2 + 4 cos 6 θ ) + (2 + 4 cos 6 θ ) (1 − sin 2 θ ) = 0
1 2π
which is only possible, when s = 1. ⇒ 2 + 4 cos 6 θ = 0 ⇒ cos 6 θ = − = cos
2 3
As, ω2 + ω4 = − 1
2π π   π 
∴ r = 1, s = 1 ⇒ 6θ = ⇒θ = Q θ ∈ 0, 3  
Again, if r = 3, then
3 9  
ω 6 + ω 4s = − 1 x −6 −1
⇒ ω 4 s = −2 [never possible] 3. Given equation 2 − 3x x − 3 = 0
∴ r ≠3 − 3 2x x + 2
⇒ (r , s) = (1, 1) is the only solution. On expansion of determinant along R1, we get
Hence, the total number of ordered pairs is 1. x [(− 3x) (x + 2) − 2x(x − 3)] + 6 [2(x + 2) + 3(x − 3)]
− 1 [2(2x) − (− 3x) (− 3)] = 0
Topic 2 Properties of Determinants ⇒ x [− 3x2 − 6x − 2x2 + 6x] + 6[2x + 4 + 3x − 9]
1. Given function − 1 [4x − 9x] = 0
 − sin 2 θ −1 − sin 2 θ 1 
⇒ x(− 5x2) + 6(5x − 5) − 1(− 5x) = 0
f (θ ) =  − cos 2 θ −1 − cos 2 θ 1 
  ⇒ −5x3 + 30x − 30 + 5x = 0
 12 10 −2 
⇒ 5x3 − 35x + 30 = 0 ⇒ x3 − 7x + 6 = 0.
− sin 2 θ −1 − sin 2 θ 1  Since all roots are real
= 2− cos 2 θ −1 − cos 2 θ 1  ∴ Sum of roots = −
coefficient of x2
=0
 
−1  coefficient of x3
 6 5
On applying 4. Given determinants are
R1 → R1 + R3 and R2 → R2 + R3 , we get x sin θ cos θ
6 − sin θ 4 − sin θ 0 
2 2
∆1 = − sin θ − x 1
f (θ ) = 26 − cos 2 θ 4 − cos 2 θ 0  cos θ 1 x
 
 6 5 −1  = − x3 + sin θ cos θ − sin θ cos θ + x cos 2 θ − x + x sin 2 θ
= 2(−1)[(6 − sin 2 θ )(4 − cos 2 θ ) − (4 − sin 2 θ )(6 − cos 2 θ )] = − x3
= − 2 [24 − 6 cos 2 θ − 4 sin 2 θ + sin 2 θ cos 2 θ − x sin 2θ cos 2θ
24 + 4 cos θ + 6 sin θ − sin θ cos θ ]
2 2 2 2 and ∆ 2 = − sin 2θ −x 1 ,x≠0
= − 2 [−2 cos θ + 2 sin θ ] = 4 cos 2 θ
2 2 cos 2θ 1 x
π π  π  = − x3 (similarly as ∆1)
As θ ∈ , ⇒2θ ∈ ,π
 4 2   2  So, according to options, we get ∆1 + ∆ 2 = − 2x3
150 Matrices and Determinants

5. Given = y [ y2 − (α + β )2 + 2αβ + 2αβ − 1 + (α + β ) − αβ ]


1 1 1 2 1 3 ... 1 n − 1 1 78 = y[ y2 − 1 + 3 − 1 − 1] = y3 [Qα + β = −1 and αβ = 1]
0 1 0 1 0 1 0 =
     1  0 1  1 1 1 
1 1 1 2 1 2 + 1 7. Given, matrix A = 2 b c , so
 
Q 0 1 0 1 = 0 1 
, 2 2
4 b c 
   
1 2 + 1 1 3 1 3 + 2 + 1 1 1 1
0 1  0 1 = 0 , det( A ) = 2 b c
    1   
2 2
4 b c 
: : :
: : : On applying, C 2 → C 2 − C1 and C3 → C3 − C1,
1 1 1 2 1 3 1 n − 1 1 0 0 
∴     ...  b − 2 c − 2
0 1 0 1 0 1 0 1  we get det( A ) = 2 b − 2 c − 2=  2
  b − 4 c2 − 4
4 b − 4 c − 4
2 2
1 (n − 1) + (n − 2)+ ...+3 + 2 + 1
=  b −2 c−2
0 1  =
(b − 2)(b + 2) (c − 2)(c + 2)
 n (n − 1)  1 78
1    1 1 
= 2 =  = (b − 2)(c − 2) 
0 1  0 1  b + 2 c + 2
Since, both matrices are equal, so equating [taking common (b − 2) from C1 and
corresponding element, we get (c − 2) from C 2]
n (n − 1) = (b − 2)(c − 2)(c − b)
= 78 ⇒ n (n − 1) = 156 = 13 × 12 = 13(13 − 1)
2 Since, 2, b and c are in AP, if assume common difference
of AP is d, then
⇒ n = 13
b = 2 + d and c = 2 + 2d
1 13 1 −13
So, A=  = A −1 =   So, | A| = d (2d )d = 2d3 ∈ [2, 16] [given]
0 1  0 1  ⇒ d3 ∈ [1, 8] ⇒ d ∈ [1, 2]
a b −1  d − b 
[Qif|A|= 1and A =   , then A =  − c a   ∴ 2 + 2d ∈ [2 + 2, 2 + 4]
 c d  
= [4, 6] ⇒ c ∈ [4, 6]
6. Given, quadratic equation is x2 + x + 1 = 0 having roots  1 sin θ 1 
α , β. 
8. Given matrix A = − sin θ 1 sin θ 
Then, α + β = −1 and αβ = 1  
 −1 − sin θ 1 
Now, given determinant 1 sin θ 1
y+1 α β ⇒ det( A ) =| A|= − sin θ 1 sin θ
∆= α y+β 1 −1 − sin θ 1
β 1 y+α = 1(1 + sin 2 θ ) − sin θ (− sin θ + sin θ ) + 1(sin 2 θ + 1)
On applying R1 → R1 + R2 + R3 , we get ⇒| A| = 2 (1 + sin 2 θ ) …(i)
 3π 5π 
y+1+α +β y+1+α +β y+1+α +β As we know that, for θ ∈  , 
 4 4
∆= α y+β 1
 1 1  1
β y+α sin θ ∈  − ,  ⇒ sin θ ∈ 0, 
2
 2
1  2 2 
y y y  1   3
= α y+β [Qα + β = −1] ⇒ 1 + sin 2 θ ∈ 0 + 1, + 1 ⇒ 1 + sin 2 θ ∈ 1, 
1  2   2
β 1 y+α 
3
⇒ 2(1 + sin 2θ ) ∈ [2, 3) ⇒| A| ∈ [2, 3) ⊂  , 3
On applying C 2 → C 2 − C1 and C3 → C3 − C1, we get 2 
y 0 0 a−b−c 2a 2a
∆ = α y + β −α 1 −α 9. Let ∆ = 2b b−c−a 2b
β 1 −β y + α −β 2c 2c c−a −b
= y[( y + (β − α )) ( y − (β − α )) − (1 − α ) (1 − β )] Applying R1 → R1 + R2 + R3 , we get
[expanding along R1] a+ b+ c a+ b+ c a+ b+ c
∆= 2b b−c−a 2b
= y [ y2 − (β − α )2 − (1 − α − β + αβ )]
2c 2c c−a −b
= y [ y2 − β 2 − α 2 + 2αβ − 1 + (α + β ) − αβ ]
Matrices and Determinants 151

1 1 1 log e (a r + kRk ) log e Rr + k log e R2r + 2k


= (a + b + c) 2b b − c − a 2b ⇒ log e a r + kR3 r + 4k log e Rr + k log e R2r + 2k = 0
2c 2c c−a −b log e a r + kR6r + 7k log e Rr + k log e R2r + 2k
(taking common (a + b + c) from R1)
Applying C 2 → C 2 − C1 and C3 → C3 − C1 , we get log e (a r + kRk ) log e Rr + k 2 log e Rr + k
1 0 0 ⇒ log e (a r + kR3 r + 4k ) log e Rr + k 2 log e Rr + k = 0
∆ = (a + b + c) 2b − (a + b + c) 0 log e (a r + kR6r + 7k ) log e Rr + k 2 log e Rr + k
2c 0 − (a + b + c) [Q log mn = n log m and here
Now, expanding along R1, we get log e R2r + 2k = log e R2( r + k) = 2 log e Rr + k ]
∆ = (a + b + c) 1. {(a + b + c)2 − 0 } Q Column C 2 and C3 are proportional,
= (a + b + c)3 = (a + b + c)(x + a + b + c)2 (given) So, value of determinant will be zero for any value of
(r , k), r , k ∈ N .
⇒ (x + a + b + c)2 = (a + b + c)2
∴ Set ‘S’ has infinitely many elements.
⇒ x + a + b + c = ± (a + b + c)
2 b 1
⇒ x = − 2(a + b + c) [Q x ≠ 0]
11. Given matrix, A = b b2 + 1 b, b > 0
log e a1ra 2k log e a 2ra3k log e a3r a 4k  
1 b 2 
10. Given, log e a 4r a5k log e a5r a 6k log e a 6r a7k = 0 2 b 1
log e a7ra 8k log e a 8r a 9k log e a 9r a10
k
So, det ( A ) =| A| = b b2 + 1 b
On applying elementary operations 1 b 2
C 2 → C 2 − C1 and C3 → C3 − C1, we get = 2 [2(b2 + 1) − b2] − b(2b − b)
log e a1ra 2k log e a 2ra3k − log e a1ra 2k +1(b2 − b2 − 1)
log e a 4r a5k log e a5r a 6k − log e a 4r a5k = 2[2b + 2 − b ] − b − 1
2 2 2

log e a7ra 8k log e a 8r a 9k − log e a7ra 8k = 2b2 + 4 − b2 − 1 = b2 + 3


det( A ) b2 + 3 3
log e a3r a 4k − log e a1ra 2k ⇒ = = b+
b b b
log e a 6r a7k − log e a 4r a5k = 0 Now, by AM ≥ GM, we get
k
log e a 9r a10 − log e a7ra 8k b+
3
1/ 2
b ≥  b × 3  {Q b > 0 }
 a ra k   arak  2  b
log e a1ra 2k log e  2r 3k  log e  3r k4 
 a1 a 2   a1 a 2  ⇒
3
b+ ≥2 3
a a 
r k  arak  b
⇒ log e a 4r a5k log e  5r 6k  log e  6r 7k  = 0 det ( A )
 a 4a5   a 4a5  So, minimum value of =2 3
b
 a 8r a 9k  k 
 a 9r a10
log e a7ra 8k log e  r k  log e  r k  12. Given,
 a7 a 8   a7 a 8 
 −2 4+ d (sin θ ) − 2 
  m 
Q log e m − log e n = log e  n   A =  1 (sin θ ) + 2 d 
   
 5 (2 sin θ ) − d (− sin θ ) + 2 + 2d 
[Q a1 , a 2, a3 ....... , a10 are in GP, therefore put
a1 = a , a 2 = aR, a3 = aR2, ... , a10 = aR9 ] −2 4+ d (sin θ ) − 2
∴ | A| = 1 (sin θ ) + 2 d
 a r + kRr + 2k 
log e a r + kRk log e   5 (2 sin θ ) − d (− sin θ ) + 2 + 2d
 a r + k Rk 
 a r + k R4 r + 5 k  −2 4+ d (sin θ ) − 2
⇒ log e a r + kR3 r + 4k log e  r + k 3 r + 4k  = 1 (sin θ ) + 2 d
a R 
 a r + kR7r + 8k  1 0 0
log e a r + kR6r + 7k log e  r + k 6r + 7k  (R3 → R3 − 2R2 + R1 )
a R 
= 1 [(4 + d )d − (sin θ + 2) (sin θ − 2)]
 a r + kR2r + 3 k  (expanding along R3 )
log e  
 a r + k Rk  = (d 2 + 4d − sin 2 θ + 4)
 a r + kR5 r + 6k  = (d 2 + 4d + 4) − sin 2 θ
log e  r + k 3 r + 4k  = 0 = (d + 2)2 − sin 2 θ
a R 
Note that| A|will be minimum if sin 2 θ is maximum i.e. if
 a r + k R8 r + 9 k  sin 2 θ takes value 1.
log e  r + k 6r + 7k 
a R  Q | A|min = 8,
152 Matrices and Determinants

therefore (d + 2)2 − 1 = 8 15. PLAN Use the property that, two determinants can be multiplied
⇒ (d + 2)2 = 9 column-to-row or row-to-column, to write the given
determinant as the product of two determinants and then
⇒ d+2=±3 expand.
⇒ d = 1, − 5 Given, f (n ) = α n + β n, f (1) = α + β, f (2) = α 2 + β 2,
13. Given, f (3) = α 3 + β3 , f (4) = α 4 + β 4
x − 4 2x 2x  3 1+ f (1) 1 + f (2)
 2x x − 4 2x  = ( A + Bx)(x − A )2 Let ∆ = 1 + f (1) 1 + f (2) 1 + f (3)
 
 2x 2x x − 4 1 + f (2) 1 + f (3) 1 + f (4)

⇒ Apply C1 → C1 + C 2 + C3 3 1 + α + β 1 + α 2 + β2
 5x − 4 2x 2x  ⇒ ∆= 1+α +β 1 + α 2 + β 2 1 + α 3 + β3
 5x − 4 x − 4 2x  = ( A + Bx)(x − A )2 1 + α 2 + β2 1 + α 3 + β3 1 + α 4 + β 4
 
 5x − 4 2x x − 4
1 ⋅1 + 1 ⋅1 + 1 ⋅1 1 ⋅1 + 1 ⋅α + 1 ⋅β
Taking common (5x − 4) from C1, we get = 1 ⋅1 + 1 ⋅α + 1 ⋅β 1 ⋅1 + α ⋅α + β ⋅β
1 2x 2x 
1 ⋅ 1 + 1 ⋅ α 2 + 1 ⋅ β2 1 ⋅ 1 + α 2 ⋅ α + β2 ⋅ β
(5x − 4)
1 x − 4 2x  = ( A + Bx)(x − A )
2
1 ⋅ 1 + 1 ⋅ α 2 + 1 ⋅ β2
1 2x x − 4 1 ⋅ 1 + α ⋅ α 2 + β ⋅ β2
Apply R2 → R2 − R1 and R3 → R3 − R1 1 ⋅ 1 + α 2 ⋅ α 2 + β2 ⋅ β2
1 2x 0  2
1 1 1 1 1 1 1 1 1
∴ (5x − 4)
0 − x − 4 0   = ( A + Bx)(x − A )
2
=1 α β 1 α β = 1 α β
0 0 −x − 4
1 α 2 β2 1 α2 β2 1 α 2 β2
Expanding along C1, we get
(5x − 4)(x + 4)2 = ( A + Bx)(x − A )2 On expanding, we get ∆ = (1 − α )2(1 − β )2(α − β )2
Equating, we get, A = − 4 and B = 5 But given, ∆ = K (1 − α )2(1 − β )2(α − β )2
14. Given, 2ω + 1 = z Hence, K (1 − α )2(1 − β )2(α − β )2 = (1 − α )2(1 − β )2(α − β )2
⇒ 2ω + 1 = − 3 [Q z = − 3] ∴ K =1
− 1 + 3i
⇒ ω= 16. PLAN It is a simple question on scalar multiplication, i.e.
2 k a1 k a2 k a3 a1 a2 a3
Since, ω is cube root of unity. b1 b 2 b 3 = k b1 b 2 b 3
c1 c 2 c 3 c1 c 2 c 3
− 1 − 3i
∴ ω2 = and ω3 n = 1
2 Description of Situation Construction of matrix,
1 1 1
 a11 a12 a13 
Now, 1 − ω − 1 ω 2 = 3k
2
i.e. if a = [aij ]3 × 3 =  a21 a22 a23 
1 ω2 ω7  a31 a32 a33 

1 1 1  a11 a12 a13 


Here, P = [aij ]3 × 3 =  a 21 a 22 a 23 
⇒ 1 ω ω 2 = 3k  a31 a32 a33 
1 ω2 ω
 b11 b12 b13 
[Q 1 + ω + ω 2 = 0 and ω7 = (ω3 )2 ⋅ ω = ω] Q = [bij ]3 × 3 =  b21 b22 b23 
On applying R1 → R1 + R2 + R3 , we get  b31 b32 b33 
3 1 + ω + ω2 1 + ω + ω2 where, bij = 2i + j aij
1 ω ω2 = 3k 4 a11 8 a12 16 a13
1 ω2 ω ∴ Q = 8 a 21 16 a 22 32 a 23
16 a31 32 a32 64 a33
3 0 0
a11 a12 a13
⇒ 1 ω ω 2 = 3k
= 4 × 8 × 16 2 a 21 2 a 22 2 a 23
1 ω2 ω
4 a31 4 a32 4 a33
⇒ 3(ω 2 − ω 4 ) = 3k
a11 a12 a13
⇒ (ω 2 − ω ) = k = 29 × 2 × 4 a 21 a 22 a 23
 − 1 − 3i   − 1 + 3i  a31 a32 a33
∴ k=  −  = − 3i = − z
 2   2  = 212 ⋅ P = 212 ⋅ 2 = 213
Matrices and Determinants 153

17. We know, | A n| = | A|n xp + y x y

Since, | A | = 125 ⇒ | A| = 125


3 3 21. Given, yp + z y z =0
0 xp + y yp + z
⇒ α 2 = 5 ⇒ α2 − 4 = 5 ⇒ α = ± 3
2 α  Applying C1 → C1 − ( p C 2 + C3 )
 sin x cos x cos x  0 x y
18. Given,  cos x sin x cos x  = 0 ⇒ 0 y z =0
 cos x cos x sin x  − (xp + yp + yp + z ) xp + y
2
yp + z
Applying C1 → C1 + C 2 + C3
⇒ − (xp + 2 yp + z ) (xz − y2) = 0
2
 sin x + 2 cos x cos x cos x 
=  sin x + 2 cos x sin x cos x  ∴ Either xp + 2 yp + z = 0 or y2 = xz
2

 sin x + 2 cos x cos x sin x  ⇒ x, y, z are in GP.


1 cos x cos x 
22. Since, A is the determinant of order 3 with entries 0 or
= (2 cos x + sin x) 1 sin x cos x  = 0
1 only.
1 cos x sin x 
Also, B is the subset of A consisting of all
Applying R2 → R2 − R1 , R3 → R3 − R1 determinants with value 1.
1 cos x cos x  [since, if we interchange any two rows or columns,
⇒ (2 cos x + sin x) 0 sin x − cos x 0 = 0 then among themself sign changes]
0 0 sin x − cos x 
Given, C is the subset having determinant with
⇒ (2 cos x + sin x) (sin x − cos x)2 = 0 value −1.
⇒ 2 cos x + sin x = 0 or sin x − cos x = 0
∴ B has as many elements as C.
⇒ 2 cos x = − sin x or sin x = cos x
π π 23. For a matrix to be square of other matrix its
⇒ cot x = − 1 / 2 gives no solution in − ≤ x ≤ determinant should be positive.
4 4
and sin x = cos x ⇒ tan x = 1 ⇒ x = π /4 (a) and (c) → Correct, (b) and (d) → Incorrect

19. Given, 24. Given determinant could be expressed as product of two


x+1 determinants.
 1 x 
f (x) = 2x x (x − 1) (x + 1) x  (1 + α )2 (1 + 2α )2 (1 + 3α )2
 3x (x − 1) x (x − 1) (x − 2) (x + 1) x (x − 1)  i.e. (2 + α )2 (2 + 2α )2 (2 + 3α )2 = − 648 α
Applying C3 → C3 − (C1 + C 2) (3 + α )2 (3 + 2α )2 (3 + 3α )2
 1 x 0
1 + 2α + α2 1 + 4α + 4α2 1 + 6α + 9 α2
= 2x x (x − 1) 0 = 0
 3x (x − 1) x (x − 1)(x − 2) 0 ⇒ 4 + 4α + α2 4 + 8α + 4α2 4 + 12 α + 9 α 2
∴ f (x) = 0 ⇒ f (100) = 0 9 + 6 α + α 2 9 + 12 α + 4 α 2 9 + 18 α + 9 α 2

 1 a a2  = − 648 α
20. Let ∆ =
 cos ( p − d ) x cos px cos ( p + d) x 
 1 α α2 1 1 1
sin ( p − d ) x sin px sin ( p + d) x  ⇒ 4 2 α α 2 ⋅ 2 4 6 = − 648 α
Applying C1 → C1 + C3 9 3α α2 1 4 9
1 + a2 a a2 1 1 1 1 1 1
⇒ ∆ = cos ( p − d ) x + cos ( p + d ) x cos px cos ( p + d )x
⇒ α 3 4 2 1 ⋅ 2 4 6 = − 648 α
sin( p − d ) x + sin( p + d ) x sin px sin ( p + d )x
9 3 1 1 4 9
1 + a2 a a2
⇒ ∆ = 2 cos px cos dx cos px cos ( p + d ) x ⇒ −8 α 3 = − 648 α ⇒ α 3 − 81α = 0
2 sin px cos dx sin px sin ( p + d ) x ⇒ α (α − 81) = 0
2

∴ α = 0, ± 9
Applying C1 → C1 − 2 cos dx C 2
25. PLAN (i) If A and B are two non-zero matrices and AB = BA, then
1 + a 2 − 2a cos dx a a2  ( A − B)( A + B) = A 2 − B 2.
⇒ ∆ =
 0 cos px cos ( p + d ) x 
 (ii) The determinant of the product of the matrices is equal to
 0 sin px sin ( p + d ) x  product of their individual determinants, i.e. | A B | = | A || B |.
⇒ ∆ = (1 + a 2 − 2a cos dx) [sin ( p + d ) x cos px Given, M 2 = N 4 ⇒ M2 − N 4 = 0
− sin px cos ( p + d ) x] ⇒ (M − N ) (M + N 2) = 0
2
[as MN = NM ]
⇒ ∆ = (1 + a 2 − 2a cos dx) sin dx Also, M ≠N 2

which is independent of p. ⇒ M + N2 =0
154 Matrices and Determinants

⇒ det (M + N 2) = 0 1 a a 2 − bc 1 a a2 1 a bc
Also, det (M 2 + MN 2) = (det M) (det M + N 2) 29. 1 b b − ca = 1 b b − 1 b ca
2 2

= (det M) (0) = 0 1 c c2 − ab 1 c c2 1 c ab
As, det (M 2 + MN 2) = 0
1 a bc a a 2 abc
Thus, there exists a non-zero matrix Usuch that 1
Now, 1 b ca = b b2 abc
(M 2 + MN 2) U = 0 abc
1 c ab c c2 abc
 a b aα + b
26. Given,  b c bα + c  = 0 Applying R1 → aR1 , R2 → bR2, R3 → cR3
 aα + b bα + c 0 
a a2 1 1 a a2
Applying C3 → C3 − (α C1 + C 2) 1
= ⋅ abc b b 1 = 1 b b2
2

 a  abc
b 0 c c2 1 1 c c2

 b c 0 = 0

 aα + b bα + c − (aα + 2bα + c) 1 a a 2 − bc
2

⇒ − (aα 2 + 2bα + c) (ac − b2) = 0 ∴ 1 b b2 − ca = 0


⇒ aα 2 + 2bα + c = 0 or b2 = ac 1 c c2 − ab
⇒ x − α is a factor of ax2 + 2bx + c or a , b, c are in GP. x 3 7
a1 b1 c1 30. Given, 2 x 2 =0
27. Let Det (P ) = a 2 b2 c2 7 6 x
a3 b3 c3 Applying R1 → R1 + R2 + R3
= a1 (b2c3 − b3 c2) − a 2 (b1c3 − b3 c1 ) + a3 (b1c2 − b2c1 ) x+9 x+9 x+9 1 1 1
Now, maximum value of Det (P ) = 6 ⇒ 2 x 2 = 0 ⇒ (x + 9) 2 x 2 = 0
If a1 = 1, a 2 = − 1, a3 = 1, b2c3 = b1c3 = b1c2 = 1 7 6 x 7 6 x
and b3 c2 = b3 c1 = b2c1 = − 1
Applying C 2 → C 2 − C1 and C3 → C3 − C1
But it is not possible as
1 0 0
(b2c3 ) (b3 c1 ) (b1c2) = − 1 and (b1c3 ) (b3 c2) (b2c1 ) = 1
i.e., b1b2b3 c1c2c3 = 1 and − 1 ⇒ (x + 9) 2 x − 2 0 = 0 ⇒ (x + 9) (x − 2) (x − 7) = 0
Similar contradiction occurs when 7 −1 x − 7
a1 = 1, a 2 = 1, a3 = 1, b2c1 = b3 c1 = b1c2 = 1 ⇒ x = − 9, 2, 7 are the roots.
and b3 c2 = b1c3 = b1c2 = − 1
∴ Other two roots are 2 and 7.
Now, for value to be 5 one of the terms must be zero but
that will make 2 terms zero which means answer 1 4 20
cannot be 5 31. Given, 1 −2 5 =0
1 1 1 2
1 2x 5x
Now, −1 1 1 =4 ⇒ 1 (− 10 x2 − 10x) − 4 (5x2 − 5) + 20 (2x + 2) = 0
1 −1 1
⇒ −30x2 + 30x + 60 = 0
Hence, maximum value is 4. ⇒ (x − 2) (x + 1) = 0
1 log x y log x z ⇒ x = 2, − 1
28. Let ∆ = log y x 1 log y z Hence, the solution set is { −1, 2}.
log z x log z y 1
λ 2 + 3λ λ −1 λ+3
log y log z 32. Given, λ+1 −2 λ λ − 4
1
log x log x λ −3 λ + 4 3λ
log x log z
= 1
log y log y = pλ 4 + qλ 3 + rλ 2 + sλ + t
log x log y Thus, the value of t is obtained by putting λ = 0.
1
log z log z 0 −1 3
On dividing and multiplying R1 , R2, R3 by log x, ⇒ 1 0 −4 = t
log y, log z, respectively. −3 4 0
log x log y log z
1 ⇒ t =0
= log x log y log z = 0 [Q determinants of odd order skew-symmetric matrix
log x log y log z
log x log y log z is zero]
Matrices and Determinants 155

 1 a bc  a a 2 abc 1
⇒ [(x2 + y2 + 1) (acx + a 2x2 + abxy)] = 0
1
33. Let ∆ =  
 1 b ca  = abc b b abc
2 ax
1
 1 c ab 
2
c c abc ⇒ [ax(x2 + y2 + 1) (c + ax + by)] = 0
ax
Applying R1 → aR1 , R2 → bR2, R3 → cR3 ⇒ (x2 + y2 + 1)(ax + by + c) = 0
⇒ ax + by + c = 0
a a2 1 1 a a2
1 which represents a straight line.
= ⋅ abc b b2 1 = 1 b b2
abc sin θ cos θ sin 2θ
c c2 1 1 c c2
 2π   2π   4π 
sin θ +  cos θ +  sin 2θ + 
1 a bc 1 a a2  3   3   3
36. Let ∆ =
∴ 1 b ca = 1 b b2
 2π   2π   4π 
1 c ab 1 c c2 sin θ −  cos θ −  sin 2θ − 
 3  3  3
Hence, statement is false. Applying R2 → R2 + R3
34. Since, M M = I and|M | = 1
T

∴ |M − I | = |IM − M T M | [Q IM = M ] sin θ cos θ sin 2θ


 2π   2π   4π 
⇒ |M − I | = |(I − M T )M |= |(I − M )T ||M |= | I − M | sin θ +  cos θ +  sin 2θ + 
 3  3  3
= (− 1)3 |M − I |[Q I − M is a 3 × 3 matrix] =
 2π   2π   4π 
+ sin θ −  + cos θ −  + sin 2θ − 
= − |M − I |  3  3  3
⇒ 2|M − I| = 0  2π   2π   4π 
sin θ −  cos θ −  sin 2θ − 
 3  3  3
⇒ |M − I | = 0
 ax − by − c bx + ay cx + a   2π   2π 
Now, sin θ +  + sin θ − 
35. Given,  bx + ay −ax + by − c cy + b = 0  3  3
 cx + a cy + b − ax − by + c
 2π 2π   2π 2π 
θ + +θ−  θ + −θ + 
a x − aby − ac
2
bx + ay cx + a = 2 sin  3 3  cos  3 3 
1
⇒ abx + a 2y − ax + by − c cy + b =0  2   2 
a
acx + a 2 cy + b − ax − by + c    
2π  π
Applying C1 → C1 + bC 2 + cC3 = 2 sin θ cos = 2 sin θ cos  π − 
3  3
 (a 2 + b2 + c2) x ay + bx cx + a 
1 2 π
⇒ (a + b2 + c2) y by − c − ax b + cy  = 0 = − 2 sin θ cos = − sin θ
a + − ax − by
 3
 a +b +c
2 2 2 b cy c
  2 π   2π 
and cos θ +  + cos θ − 
1x ay + bx cx + a   3   3
⇒  y by − c − ax b + cy  = 0
a 1 b + cy c − ax − by   2π 2π   2π 2π 
θ + +θ−  θ + −θ + 
= 2 cos  3 3  cos  3 3 
[Q a 2 + b2 + c2 = 1]
 2   2 
Applying C 2 → C 2 − bC1 and C3 → C3 − cC1    
1x ay a   2π 
⇒  y − c − ax b = 0 = 2 cos θ cos  
a 1  3
cy − ax − by 
 1
x2 axy ax = 2 cos θ  −  = − cos θ
1  2
⇒ y − c − ax b =0
ax  4π   4π 
1 cy − ax − by and sin 2θ +  + sin 2θ − 
 3  3
Applying R1 → R1 + yR2 + R3  4π 4π   4π 4π 
   2θ + + 2θ −   2θ + − 2θ + 
1 x + y +1
2 2
0 0 = 2 sin  3 3  cos  3 3 
⇒ 
 y − c − ax b = 0
  2   2 
ax 1 cy − ax − by     

4π  π

1
[(x2 + y2 + 1) {(− c − ax)(− ax − by) − b(cy)}] = 0 = 2 sin 2θ cos = 2 sin 2θ cos  π + 
3  3
ax
1 π
⇒ [(x2 + y2 + 1) (acx + bcy + a 2x2 + abxy − bcy)] = 0 = − 2 sin 2θ cos = − sin 2θ
ax 3
156 Matrices and Determinants

  39. Given, a > 0, d > 0 and let


 sin θ cos θ sin 2 θ 
1 1 1
∴ ∆ =  − sin θ − cos θ − sin 2θ = 0
 a a (a + d ) (a + d ) (a + 2d )
sin θ − 2π  cos θ − 2π  sin 2θ − 4π  
 1 1 1
  3  3  3  ∆=
(a + d ) (a + d ) (a + 2d ) (a + 2d ) (a + 3d )
[since, R1 and R2 are proportional] 1 1 1
2ax 2ax − 1 2ax + b + 1 (a + 2d ) (a + 2d ) (a + 3d ) (a + 3d ) (a + 4d )
37. Given, f ′ (x) = b b+1 −1 Taking
1
common from R1 ,
2 (ax + b) 2ax + 2b + 1 2ax + b a (a + d ) (a + 2d )
1
Applying R3 → R3 − R1 − 2R2, we get from R2,
(a + d )(a + 2d )(a + 3d )
 2ax 2ax − 1 2ax + b + 1  1
f ′ (x) =  b b+1 −1  from R3
 0 0 1 (a + 2d ) (a + 3d )(a + 4d )
2ax − 1   2ax −1  1
= ⇒ ∆=
2ax
= [C 2 → C 2 − C1] a (a + d )2(a + 2d )3 (a + 3d )2(a + 4d )
 b b+1   b 1
⇒ f ′ (x) = 2ax + b  (a + d )(a + 2d ) (a + 2d ) a 
 (a + 2d )(a + 3d ) (a + 3d ) (a + d ) 
On integrating, we get f (x) = ax + bx + c, 2
 (a + 3d )(a + 4d ) (a + 4d ) (a + 2d ) 
where c is an arbitrary constant. 1
⇒ ∆= ∆′
Since, f has maximum at x = 5 / 2. a (a + d )2(a + 2d )3 (a + 3d )2(a + 4d )
⇒ f ′ (5 / 2) = 0 ⇒ 5a + b = 0 …(i)  (a + d )(a + 2d ) (a + 2d ) a 
Also, f (0) = 2 where, ∆′ =  (a + 2d )(a + 3d ) (a + 3d ) (a + d ) 
 (a + 3d )(a + 4d ) (a + 4d ) (a + 2d ) 
⇒ c = 2 and f (1) = 1
⇒ a + b + c=1 …(ii) Applying R2 → R2 − R1 , R3 → R3 − R2
On solving Eqs. (i) and (ii) for a , b, we get (a + d ) (a + 2d ) (a + 2d ) a
1 5 ⇒ ∆′ = (a + 2d ) (2d ) d d
a = ,b = − (a + 3d ) (2d ) d d
4 4
1 2 5 Applying R3 → R3 − R2
Thus, f (x) = x − x + 2
4 4 (a + d )(a + 2d ) (a + 2d ) a
∆′ = (a + 2d )2d d d
38. Since, a , b, c are pth , qth and rth terms of HP.
2d 2 0 0
1 1 1
⇒ , , are in an AP.
a b c Expanding along R3 , we get
a + 2d a 
1
= A + ( p − 1) D
 ∆′ = 2d 2
  d d
a
1  ∆′ = (2d 2)(d )(a + 2d − a ) = 4d 4
⇒ = A + ( q − 1) D  …(i)
b  4d 4
1 ∴ ∆=
= A + (r − 1) D  a (a + d ) (a + 2d )3 (a + 3d )2(a + 4d )
2
c 
1 1 1 cos ( A − P ) cos ( A − Q ) cos ( A − R)
bc ca ab 40. Let ∆ = cos (B − P ) cos (B − Q ) cos (B − R)
a b c cos (C − P ) cos (C − Q ) cos (C − R)
Let ∆ = p q r = abc p q r [from Eq. (i)]
cos A cos P + sin A sin P cos ( A − Q )
1 1 1 1 1 1 ⇒ ∆ = cos B cos P + sin B sin P cos (B − Q )
cos C cos P + sin C sin P cos (C − Q )
A + ( p − 1) D A + (q − 1) D A + (r − 1) D cos ( A − R)
= abc p q r cos (B − R)
1 1 1 cos (C − R)
 cos A cos P cos ( A − Q ) cos ( A − R) 
Applying R1 → R1 − ( A − D ) R3 − DR2 ⇒ ∆ =  cos B cos P cos (B − Q ) cos (B − R) 
0 0 0 bc ca ab  cos C cos P cos (C − Q ) cos (C − R) 
= abc p q r = 0 ⇒ p q r =0  sin A sin P cos ( A − Q ) cos ( A − R) 
+  sin B sin P cos (B − Q ) cos (B − R) 
1 1 1 1 1 1
 sin C sin P cos (C − Q ) cos (C − R) 
Matrices and Determinants 157

 cos A cos ( A − Q ) cos ( A − R)  p b c


⇒ ∆ = cos P  cos B cos (B − Q ) cos (B − R)  42. Let ∆ = a q c
 cos C cos (C − Q ) cos (C − R) 
a b r
 sin A cos ( A − Q ) cos ( A − R) 
+ sin P  sin B cos (B − Q ) cos (B − R)  Applying R1 → R2 − R1 and R3 → R3 − R1 , we get
 sin C cos (C − Q ) cos (C − R)  p b c
Applying C 2 → C 2 − C1 cos Q , C3 → C3 − C1 cos R in ∆= a− p q−b 0
first determinant and C 2 → C 2 − C1 sin Q and in a−p 0 r−c
second determinant a− p q−b p b
cos A sin A sin Q sin A sin R =c + (r − c)
a−p 0 a− p q−b
⇒ ∆ = cos P cos B sin B sin Q sin B sin R
= − c (a − p) (q − b) + (r − c) [ p(q − b) − b(a − p)]
cos C sin C sin Q sin C sin R
= − c (a − p) (q − b) + p(r − c) (q − b) − b(r − c)(a − p)
 sin A cos A cos Q cos A cos R  Since, ∆ = 0
+ sin P sin B cos B cos Q cos B cos R 
 sin C cos C cos Q cos C cos R  ⇒ − c (a − p) (q − b) + p(r − c) (q − b) − b(r − c) (a − p) = 0
c p b
 cos A sin A sin A  ⇒ + + =0
∆ = cos P sin Q sin R cos B sin B sin B  r−c p−a q−b
 cos C sin C sin C  [on dividing both sides by (a − p)(q − b)(r − c)]
 sin A cos A cos A  p b c
+ sin P cos Q cos R sin B cos B cos B  ⇒ + +1+ + 1 =2
p−a q−b r−c
 sin C cos C cos C 
p q r
∆ =0 + 0 =0 ⇒ + + =2
p−a q−b r−c
 n! (n + 1)! (n + 2)! 
41. Given, D =  (n + 1)! (n + 2)! (n + 3)!  43. We know, A 28 = A × 100 + 2 × 10 + 8
 (n + 2)! (n + 3)! (n + 4)!  3B9 = 3 × 100 + B × 10 + 9
Taking n !, (n + 1)! and (n + 2)! common from R1 , R2 and 62 C = 6 × 100 + 2 × 10 + C
and R3 , respectively.
Since, A 28, 3B 9 and 62 C are divisible by k, therefore
1 (n + 1) (n + 1) (n + 2)  there exist positive integers m1 , m2 and m3 such that,
∴ D = n ! (n + 1)! (n + 2) ! 1 (n + 2) (n + 2) (n + 3) 
(n + 3) (n + 3) (n + 4)  100 × A + 10 × 2 + 8 = m1k , 100 × 3 + 10 × B + 9 = m2k
1
and 100 × 6 + 10 × 2 + C = m3 k … (i)
Applying R2 → R2 − R1 and R3 → R3 − R2, we get A 3 6
1 (n + 1) (n + 1) (n + 2)  ∴ ∆= 8 9 C
D = n !(n + 1)!(n + 2)! 0 1 2n + 4 
2n + 6 2 B 2
0 1 
Expanding along C1 , we get Applying R2 → 100R1 + 10R3 + R2
A 3
D = (n !)(n + 1)!(n + 2)![(2n + 6) − (2n + 4)]
⇒ ∆ = 100 A + 2 × 10 + 8 100 × 3 + 10 × B + 9
D = (n !)(n + 1)! (n + 2)! [2]
2 B
On dividing both side by (n !)3 6
D (n !)(n !)(n + 1)(n !)(n + 1)(n + 2)2 100 × 6 + 10 × 2 + C
⇒ 3
=
(n !) (n !)3 2
D A 3 6
⇒ = 2(n + 1)(n + 1)(n + 2)
(n !)3 = A28 3B9 62C [from Eq. (i)]
D 2 B 2
⇒ = 2(n3 + 4n 2 + 5n + 2)
(n !)3 A 3 6 A 3 6
= 2n (n 2 + 4n + 5) + 4 = m1k m2k m3 k = k m1 m2 m3
D
⇒ − 4 = 2n (n 2 + 4n + 5) 2 B 2 2 B 2
(n !)3
∴ ∆ = mk
 D 
which shows that  3
− 4 is divisible by n. Hence, determinant is divisible by k.
 (n !) 
158 Matrices and Determinants

 a −1 n 6  46. Since, α is repeated root of f (x) = 0.


44. Given, ∆ a = 
 (a − 1) 2n
2 2
4n − 2   ∴ f (x) = a (x − α )2, a ∈ constant (≠ 0)
 (a − 1) 3n 2 − 3n 
3 3
3n
 A (x) B(x) C (x) 
 n  Let φ (x) =  A (α ) B(α ) C (α ) 
 ∑ (a − 1) n 6   A ′ (α ) B ′ (α ) C ′ (α ) 
n  an=1 
To show φ (x) is divisible by (x − α )2, it is sufficient to
∴ ∑ a ∑
∆ =  (a − 1 ) 2
2 n 2
4 n − 2 
 show that φ (α ) and φ ′ (α ) = 0.
a=1 a=1
 n   A (α ) B(α ) C (α ) 
 ∑ (a − 1)3 3n 3n − 3n 
3 2
∴ φ (α ) =  A (α ) B(α ) C (α ) 
 a=1   A ′ (α ) B ′ (α ) C ′ (α ) 
n (n − 1) =0 [Q R1 and R2 are identical]
n 6
2
n (n − 1)(2n − 1)  A ′ (x) B ′ (x) C ′ (x) 
= 2n 2 4n − 2 Again, φ ′ (x) =  A (α ) B(α ) C (α ) 
6  A ′ (α ) B ′ (α ) C ′ (α ) 
n 2(n − 1)2
3n3 3n 2 − 3n  A ′ (α ) B ′ (α ) C ′ (α ) 
4
φ ′ (α ) =  A (α ) B(α ) C (α ) 
 A ′ (α ) B ′ (α ) C ′ (α ) 
1 1 6
n 2(n − 1) (2n − 1) =0 [Q R1 and R3 are identical]
= 2n 4n − 2
2 3 Thus, α is a repeated root of φ (x) = 0.
n (n − 1)
3n 2 3n 2 − 3n Hence, φ (x) is divisible by f (x).
2
1 1 6  x2 + x x+1 x−2
=
n3 (n − 1)
2n − 1 6n 12n − 6 47. Let ∆ =  2x2 + 3x − 1 3x 3x − 3
 2 2x − 1 2x − 1
 x + 2x + 3
12
n − 1 6n 6n − 6
Applying C3 → C3 − 6 C1 Applying R2 → R2 − (R1 + R3 ), we get
1 1 0  x2 + x x+ 1 x−2 
n3 (n − 1) ∆ = − 0 
= 2n − 1 6n 0 = 0  4 0 
12  x + 2x + 3 2x − 1 2x − 1 
2
n − 1 6n 0
n x2
Applying R1 → R1 +
∑ ∆a = c
⇒ [c = 0, i.e. constant] R2
4
a=1
x2
 xCr x
Cr + 1 x
Cr + 2  and R3 → R3 + R2, we get
4
45. Let ∆ =  y C r y
Cr + 1 y
Cr + 2   x x+ 1 x−2
z z z 
C Cr + 1 Cr + 2  
 r ∆=
 −4
0 0
Applying C3 → C3 + C 2  2x + 3 2 x − 1 2 x −1
x+1
 xCr x
Cr + 1 Cr + 2   x + 0 x + 1 x − 2
∆ = y C r y
Cr + 1 y+1
Cr + 2  Applying R3 → R3 − 2R1 =  − 4 0 0 
z z z +1   3 −3 3 
C Cr + 1 Cr + 2 
 r
n+1  x x x  0 1 −2 
[Q nC r + nC r − 1 = Cr ] =  −4 0 0  +  −4 0 0
Applying C 2 → C 2 + C1  3 −3 3   3 −3 3
 xCr x+1
Cr + 1 x+1
Cr + 2   1 1 1   0 1 − 2 
= x − 4 0 0  +  −4 0
∆ = y C r Cr + 2 
y+1 y+1 0
Cr + 1
z z+1 z+1   3 −3 3   3 −3 3
C Cr + 1 Cr + 2 
 r ⇒ ∆ = Ax + B
Applying C3 → C3 + C 2  1 1 1
 xCr x+1
Cr + 1 x+ 2
Cr + 2  where, A =  −4 0 0
 3 −3 3 
⇒ ∆ = y C r y+1
Cr + 1 y+ 2
Cr + 2  Hence proved.
z z +1 z+ 2   0 1 −2 
C Cr + 1 Cr + 2 
 r and B =  −4 0 0
 3 −3 3
Matrices and Determinants 159

a b c 2. Given matrix B is the inverse matrix of 3 × 3 matrix A,


48. Let ∆ =  b c a
c a b 5 2α 1 
where B = 0 2 1 
Applying C1 → C1 + C 2 + C3  
α 3 − 1
 a + b + c b c 1 b c
∆ =  a + b + c c a = (a + b + c) 1 c a We know that,
 a + b + c a b 1 a b 1  1 
−1
det( A ) = Q det( A ) = det( A ) 
Applying R2 → R2 − R1 and R3 → R3 − R1 , we get det(B)
1 b c 
Since, det( A ) + 1 = 0 (given)
= (a + b + c) 0 c − b a − c 
0 a − b b − c  1
+ 1 =0
det(B)
= (a + b + c) [− (c − b)2 − (a − b) (a − c)]
⇒ det(B) = − 1
= − (a + b + c) (a 2 + b2 + c2 − ab − bc − ca )
1 ⇒ 5(− 2 − 3) − 2α (0 − α ) + 1 (0 − 2α ) = − 1
= − (a + b + c) (2a 2 + 2b2 + 2c2 − 2ab − 2bc − 2ca )
2 ⇒ − 25 + 2α 2 − 2α = − 1
1
= − (a + b + c)[(a − b)2 + (b − c)2 + (c − a )2] ⇒ 2α 2 − 2α − 24 = 0
2 ⇒ α 2 − α − 12 = 0
which is always negative. ⇒ (α − 4) (α + 3) = 0
⇒ α = − 3, 4
Topic 3 Adjoint and Inverse of a Matrix
So, required sum of all values of α is 4 − 3 = 1
1. It is given that matrix.0
 sin 4 θ et e− t cos t e− t sin t
−1 − sin 2 θ  −1
M =  = α I + βM , where 3. | A | = et −e −t
cos t − e− t sin t −e −t
sin t + e− t cos t
1 + cos 2
θ cos 4
θ  et 2e− t sin t − 2e− t cos t
α = α (θ ) and β = β (θ ) are real numbers and I is the 2 × 2
1 cos t sin t
identity matrix.
= (et ) (e− t ) (e− t ) 1 − cos t − sin t − sin t + cos t
Now, det(M ) =|M|= sin 4 θ cos 4 θ + 1 + sin 2 θ + cos 2 θ
1 2 sin t − 2 cos t
+ sin 2 θ cos 2 θ
= sin θ cos θ + sin θ cos θ + 2
4 4 2 2 (taking common from each column)
 sin 4 θ −1 − sin 2 θ  α 0  β Aplying R2 → R2 − R1 and R3 → R3 − R1, we get
and   = 0 α  + |M| (adj M ) [Qet − t = e0 = 1]
1 + cos 2
θ cos 4
θ   
1 cos t sin t
 adj M 
Q M −1 = = e− t 0 − 2 cos t − sin t − 2 sin t + cos t
 |M| 
0 2 sin t − cos t − 2 cos t − sin t
 sin 4 θ −1 − sin 2 θ  α 0 
⇒  = = e− t ((2 cos t + sin t )2 + (2 sin t − cos t )2)
1 + cos θ
2
cos 4 θ  0 α 
(expanding along column 1)
β  cos 4 θ 1 + sin 2 θ   a b   d −b
= e− t (5 cos 2 t + 5 sin 2 t )
+   Q adj  = 
|M| −1 − cos θ
2
sin θ  
4
c d  − c a  = 5e− t (Qcos 2 t + sin 2 t = 1)
−t
⇒ β = −|M|and α = sin 4 θ + cos 4 θ ⇒| A | = 5e ≠ 0 for all t ∈ R
1 ∴ A is invertible for all t ∈ R
⇒ α = α (θ) = 1 − sin 2(2 θ ), and
2 [QIf| A | ≠ 0, then A is invertible]
 1
2
7
β = β (θ ) = − sin 2 θ cos 2 θ +  +  4. Given,| ABAT| = 8
 2  4
  ⇒ | A||B|| AT| = 8 [Q|XY | = |X ||Y |]
 sin 2(2 θ ) 1 2
7  ∴ | A|2|B| = 8 …(i) [Q| AT| = | A|]

= −  +  +  Also, we have | AB−1| = 8
 4 2 4
  ⇒ | A||B−1| = 8
1 37
Now, α * = α min = and β * = β min = − | A|  1 
⇒ =8 …(ii) Q| A −1|=| A|−1 =
2 16 |B|  | A|
Q α is minimum at sin (2 θ ) = 1 and β is minimum at
2

sin 2(2 θ ) = 1 On multiplying Eqs. (i) and (ii), we get


1 37 29 | A|3 = 8 ⋅ 8 = 43
So, α * + β * = − =−
2 16 16 ⇒ | A| = 4
160 Matrices and Determinants


| A| 4 1
|B| = = =  16 − 9  2 − 3
Now, 3 A 2 + 12 A = 3   + 12 − 4 1 
8 8 2  − 12 13   
−1 T 1
Now, |BA B | = |B| |B|  48 − 27  24 − 36
| A| = +  
− 36 39  − 48 12 
 1 1  1 1
=    =  72 − 63
 2 4  2 16 = 
− 84 51 
cos θ − sin θ 
5. We have, A =   51 63 
sin θ cos θ  ∴ adj (3 A 2 + 12 A ) =  
84 72
∴ | A| = cos 2 θ + sin 2 θ = 1
5a − b
 cos θ sin θ  7. Given, A =  and A adj A = AAT
and adj A =   3 2 
− sin θ cos θ 
a b   d − b Clearly, A (adj A ) = A I 2
[Q If A =  , then adj A = − c a  ] [Q if A is square matrix of order n,
 c d    then A (adj A ) = (adj A ) ⋅ A = A I n ]
 cos θ sin θ   adj A  5a − b
⇒ A −1 =  
−1
Q A =  = I2
 − sin θ cos θ   | A|  3 2
Note that, A −50 = ( A −1 )50 = (10a + 3b) I 2
Now, A −2 = ( A −1 )( A −1 ) 1 0
= (10a + 3b)  
 cos θ sin θ   cos θ sin θ  0 1
⇒ A −2 =   
− sin θ cos θ  − sin θ cos θ  10a + 3b 0 
=  ...(i)
 cos 2 θ − sin 2 θ cos θ sin θ + sin θ cos θ   0 10 a + 3 b 
= 
 − cos θ sin θ − cos θ sin θ − sin 2 θ + cos 2 θ  5a − b  5a 3
and AAT = 
 cos 2 θ sin 2 θ  3 2  − b 2
= 
− sin 2 θ cos 2 θ  25a 2 + b2 15a − 2b
=  ...(ii)
Also, A −3 = ( A −2)( A −1 )  15a − 2b 13 
 cos 2 θ sin 2 θ   cos θ sin θ  A (adj A ) = AAT
A −3 =   
Q
− sin 2 θ cos 2 θ  − sin θ cos θ  10a + 3b 0  25a 2 + b2 15a − 2b
∴ =
 cos 3 θ sin 3 θ  
 0 10a + 3b  15a − 2b 13 

= 
− sin 3 θ cos 3 θ  [using Eqs. (i) and (ii)]
−50  cos 50 θ sin 50 θ  ⇒ 15a − 2b = 0
Similarly, A = 
− sin 50 θ cos 50 θ  ⇒ a=
2b
...(iii)
 25 25  15
cos π sin π π 10a + 3b = 13
 6 6   and ...(iv)
=  when θ = 
25 25   12  On substituting the value of ‘a ’ from Eq. (iii) in
− sin π cos π
 6 6  Eq. (iv), we get
π π   Q cos  25π  = cos 4π + π  = cos π   2b
     10 ⋅   + 3b = 13
cos sin   15 
 6 6  6   6 6 
=   20b + 45b
π π 25π  π π
− sin cos  and sin  
 = sin 4π +  = sin  ⇒ = 13
 6 6    6   6 6  15
65b
 3 1  ⇒ = 13 ⇒ b = 3
 15
= 2 2 
 Now, substituting the value of b in Eq. (iii), we get
 −1 3
5a = 2
 2 2 
Hence, 5a + b = 2 + 3 = 5
 2 − 3
6. We have, A =   8. PLAN Use the following properties of transpose
− 4 1  ( AB)T = BT AT , ( AT )T = A and A −1 A = I and simplify. If A is
 2 − 3  2 − 3 non-singular matrix, then| A | ≠ 0.
∴ A2 = A ⋅ A =   
− 4 1  − 4 1  Given, AAT = AT A and B = A −1 AT
 4 + 12 − 6 − 3  16 − 9 BBT = ( A −1 AT )( A −1 AT )T
= = 
− 8 − 4 12 + 1  − 12 13  = A −1 AT A ( A −1 )T [Q ( AB)T = BT AT ]
Matrices and Determinants 161

= A −1 AAT ( A −1 )T [Q AAT = AT A] Given, 6 A −1 = A 2 + cA + dI , multiplying both sides by


= IAT ( A −1 )T [Q A −1 A = I ] A, we get
= AT ( A −1 )T = ( A −1 A )T [Q ( AB)T = BT AT ] 6I = A3 + cA 2 + dA
= IT = I ⇒ A3 + cA 2 + dA − 6I = O …(ii)
1 α 3 On comparing Eqs. (i) and (ii), we get
9. Given, P = 1 3 3 c = − 6 and d = 11
 
2 4 4 14. It is given that matrix M be a 3 × 3 invertible matrix,
∴ | P | = 1(12 − 12) − α (4 − 6) + 3 (4 − 6) = 2 α − 6 such that
M −1 = adj(adj M )
Q P = adj ( A ) [given]
⇒ M −1 = |M| M
∴ |P| = |adj A|= | A|2 = 16 [Q|adj A| = | A|n −1 ]
(Q for a matrix A of order ‘n’ adj(adj A ) = | A|n − 2 A}
∴ 2 α − 6 = 16 ⇒ M −1M =|M|M 2 ⇒ M 2|M|= I … (i)
⇒ 2 α = 22 Q det(M 2|M|) = det(I ) = 1
⇒ α = 11 ⇒ |M|3 |M|2 = 1
10. Given, PT = 2 P + I …(i) ⇒ |M| = 1 … (ii)
∴ (PT )T = (2 P + I )T = 2 PT + I from Eqs. (i) and (ii), we get
⇒ P = 2 PT + I M2 = I
−1
As, adj M =|M|M = M
⇒ P = 2 (2 P + I ) + I
⇒ (adj M )2 = M 2
⇒ P = 4P + 3I or 3P = − 3I
⇒ (adj M )2 = I
⇒ PX = − IX = –X
0 1 a 
1 a b
15. Given square matrix M = 1 2 3 
11. | A| ≠ 0, as non-singular ω 1 c ≠0  
3 b 1 
ω2 ω 1
 −1 1 −1 
⇒ 1 (1 − cω ) − a (ω − cω 2) + b (ω 2 − ω 2) ≠ 0
and adj (M ) =  8 −6 2
⇒ 1 − cω − aω + acω 2 ≠ 0  
−5 3 −1
⇒ (1 − cω ) (1 − aω ) ≠ 0 −1 1 −1
1 1
⇒ a ≠ ,c≠ Q adj (M ) =|M|2 = 8 −6 2
ω ω
−5 3 −1
⇒ a = ω , c = ω and b ∈{ω , ω 2} ⇒ 2 solutions
⇒ |M| = −1(6 − 6) − 1(−8 + 10) − 1(24 − 30) = −2 + 6 = 4
2
12. Given, M T = − M , N T = − N and MN = NM ...(i)
⇒ |M|= ±2
∴ M 2N 2 (M T N )− 1 (MN −1 )T
∴det (adj M 2) =|M 2|2 =|M|4 = 16
⇒ M 2 N 2N −1 (M T )−1 (N −1 )T ⋅ M T
As we know A (adj A ) = | A| I
⇒ M 2 N (NN −1 )(− M )−1 (N T )−1 (− M )
⇒ M = |M|(adj M )−1 …(i)
⇒ M 2 N I (− M −1 ) ( − N )−1 (− M ) T
0 −2 −6 
⇒ − M 2 NM −1N −1 M 1  −2 −4 −2 
Q (adj M )−1 =
⇒ − M ⋅ (MN )M −1N −1 M = − M (NM )M −1N −1 M |adj M|  
−4 −6 −2
⇒ − MN (NM −1 )N −1 M = − M (N N −1 )M ⇒ − M2
 0 −2 −4 
NOTE Here, non-singular word should not be used, since there 1
is no non-singular 3 × 3 skew-symmetric matrix. = −2 −4 −6 
4 
−6 −2 −2
13. Every square matrix satisfied its characteristic
equation, 0 1 a 0 −2 −4 
1−λ 0 0 So 1 2 3 =
|M| 
−2 −4 −6 
i.e. | A − λI | = 0 ⇒ 0 1−λ 1 =0   4  
3 b 1  −6 −2 −2
0 −2 4−λ
⇒ |M|= −2, a = 2and b = 1
⇒ (1 − λ ) {(1 − λ ) (4 − λ ) + 2} = 0 0 1 2
⇒ λ3 − 6λ2 + 11λ − 6 = 0 ⇒ M = 1 2 3
 
⇒ A − 6 A 2 + 11 A − 6I = O
3
…(i) 3 1 1
162 Matrices and Determinants

α  1 0 1 2 α  1 17. PLAN A square matrix M is invertible, iff dem (M) or| M| ≠ 0.
Now, If M β  = 2 ⇒ 1 2 3 β  = 2 a b
         Let M = 
 γ  3 3 1 1  γ  3  b c
⇒ B + 2 γ = 1, α + 2 β + 3γ = 2 and 3 α + β + γ = 3 a  b
(a) Given,   =   ⇒ a = b = c = α [let]
⇒ α = 1, β = −1and γ = 1  b  c
⇒ α −β + γ = 3
α α 
And (adj M )−1 + adj (M −1 ) ⇒ M =  ⇒ |M |= 0 ⇒ M is non-invertible.
α α 
= 2(adj M )−1 [Qadj (M −1 ) = (adj M )−1]
 M M (b) Given, [b c] = [a b]
= 2 −  = −M [Q(adj M )−1 = from Eq. (i)]
 2 |M| ⇒ a = b = c=α [let]
and Q a = 2 and b = 1, so a + b = 3 Again,|M | = 0
Hence, options (b), (c) and (d) are correct. ⇒ M is non-invertible.
3 − 1 − 2 a 0
(c) As given M =   ⇒|M |= ac ≠ 0
16. Here, P = 2 0 α   0 c
 
3 − 5 0  [Q a and c are non-zero]
⇒ M is invertible.
Now, |P| = 3(5α ) + 1(− 3α ) − 2(− 10)
= 12α + 20 …(i) a b
(d) M =   ⇒|M |= ac − b2 ≠ 0
T
 5α 2α − 10  b c
∴ adj (P ) = − 10 6 12  Q ac is not equal to square of an integer.
 
 − α − (3α + 4) 2  M is invertible.
 5α − 10 −α  18. PLAN If| A n × n| = ∆, then|adj A| = ∆A − 1
=  2α 6 − 3α − 4 …(ii)
 
 − 10 12 2  1 4 4 
PQ = kI ⇒ |P||Q| = |kI| Here, adj P3 × 3 = 2 1 7
As,  
⇒ |P||Q| = k3 1 1 3 
 k2  k2  ⇒ |adj P | = | P |2
⇒ |P|  = k3  given,|Q| = 
 2  2 1 4 4
⇒ |P| = 2k …(iii) ∴ | adj P | = 2 1 7 = 1 (3 − 7) − 4 (6 − 7) + 4 (2 − 1)
Q PQ = kI ⇒ Q = kp−1I 1 1 3
adjP k(adj P ) = − 4 + 4 + 4 = 4 ⇒ |P |= ± 2
= k⋅ = [from Eq. (iii)]
|P| 2k 19. | A | = (2k + 1) ,| B| = 0
3

 5 α − 10 −α 
adj P 1  But det (adj A) + det (adj B) = 106
= = 2α 6 − 3α − 4
2 2  ⇒ (2k + 1)6 = 106
− 10 12 2  9
− 3α − 4  k ⇒ k= ⇒ [k] = 4
∴ q23 = given, q23 = − 2
2  8 
(3α + 4) k Topic 4 Solving System of Equations
⇒ − =−
2 8 1. Given system of linear equations is
⇒ (3α + 4) × 4 = k
[sin θ ] x + [− cos θ ] y = 0 …(i)
⇒ 12α + 16 = k …(iv)
From Eq. (iii), |P|= 2k and [cot θ ] x + y = 0 …(ii)
⇒ 12α + 20 = 2k [from Eq. (i)] …(v) where, [x] denotes the greatest integer ≤ x.
On solving Eqs. (iv) and (v), we get [sin θ ] [− cos θ ]
α = − 1 and k = 4 …(vi) Here, ∆=
[cot θ ] 1
∴ 4α − k + 8 = − 4 − 4 + 8 = 0
∴ Option (b) is correct. ⇒ ∆ = [sin θ ] − [− cos θ ] [cot θ ]
Now, |P adj (Q )| = |P||adj Q|  π 2π 
When θ ∈ , 
 k2
2 2 3 
k5 210
= 2k  = = = 29
 2 2 2  3 
sin θ ∈  , 1
∴ Option (c) is correct.  2 
Matrices and Determinants 163

⇒ [sin θ ] = 0 …(iii) 4. Given system of linear equations


 1 2x + 3 y − z = 0,
− cos θ ∈ 0, 
 2
x + ky − 2z = 0
⇒ [− cos θ ] = 0 …(iv)
and 2x − y + z = 0 has a non-trivial solution (x, y, z ).
 1 
and cot θ ∈  − , 0 2 3 −1
 3 
∴ ∆ = 0 ⇒1 k −2 = 0

⇒ [cot θ ] = − 1 …(v)
2 −1 1
So, ∆ = [sin θ ] − [− cos θ ] [cot θ ]
− (0 × (− 1)) = 0 [from Eqs. (iii), (iv) and (v)] 2(k − 2) − 3(1 + 4) − 1(−1 − 2k) = 0
 π 2π  ⇒ 2k − 4 − 15 + 1 + 2k = 0
Thus, for θ ∈  ,  , the given system have infinitely
2 3  9
many solutions. ⇒ 4 k = 18 ⇒ k =
2
 7π   1 
When θ ∈  π,  , sin θ ∈  − , 0 ⇒ [sin θ ] = − 1 So, system of linear equations is
 6  2 
2x + 3 y − z = 0 …(i)
 3  2x + 9 y − 4z = 0 …(ii)
− cos θ ∈  , 1 ⇒ [cos θ ] = 0
 2  and 2x − y + z = 0 …(iii)
and cot θ ∈ ( 3 , ∞ ) ⇒ [cot θ ] = n , n ∈ N . From Eqs. (i) and (ii), we get
∆ = − 1 − (0 × n ) = − 1 y 1
So, 6 y − 3z = 0, =
 7π  z 2
Thus, for θ ∈  π,  , the given system has a unique
 6 From Eqs. (i) and (iii), we get
solution. x 1
4x + 2 y = 0 ⇒ = −
2. Given, system of linear equations y 2
x+ y+ z =6 … (i) x x y 1 z  y 1 x 1
So, = × = − ⇒ = −4 Q z = 2 and y = − 2 
4 x + λy − λz = λ − 2 …(ii) z y z 4 x
and 3x + 2 y − 4z = − 5 …(iii) x y z 1 1 9 1
has infinitely many solutions, then ∆ = 0 ∴ + + + k= − + −4 + = .
y z x 2 2 2 2
1 1 1
5. Given system of linear equations
⇒ 4 λ − λ =0
x − 2 y + kz = 1 …(i)
3 2 −4
2x + y + z = 2 …(ii)
⇒ 1(− 4λ + 2λ ) − 1(− 16 + 3λ) + 1(8 − 3λ) = 0 and 3x − y − kz = 3 …(iii)
⇒ − 8λ + 24 = 0 ⇒ λ = 3 has a solution (x, y, z ), z ≠ 0.
From, the option λ = 3, satisfy the quadratic equation On adding Eqs. (i) and (iii), we get
λ2 − λ − 6 = 0. x − 2 y + kz + 3x − y − kz = 1 + 3
3. Given system of linear equations 4x − 3 y = 4 ⇒ 4x − 3 y − 4 = 0
x+ y+ z =5 …(i) This is the required equation of the straight line in
x + 2 y + 2z = 6 …(ii) which point (x, y) lies.
x + 3 y + λz = µ …(iii)
(λ, µ ∈ R) 6. Key Idea A homogeneous system of linear equations have
non-trivial solutions iff ∆ = 0
The above given system has infinitely many solutions,
then the plane represented by these equations Given system of linear equations is
intersect each other at a line, means (x + 3 y + λz − µ ) x − cy − cz = 0,
= p(x + y + z − 5) + q (x + 2 y + 2z − 6) cx − y + cz = 0
= ( p + q)x + ( p + 2q) y + ( p + 2q)z − (5 p + 6q) and cx + cy − z = 0
On comparing, we get We know that a homogeneous system of linear
p + q = 1, p + 2q = 3, p + 2q = λ equations have non-trivial solutions iff
and 5 p + 6q = µ ∆ =0
So, ( p, q) = (−1, 2) 1 − c − c
⇒ c −1 c
⇒ λ = 3 and µ = 7  = 0
⇒ λ + µ = 3 + 7 = 10  c c − 1 
164 Matrices and Determinants

⇒ 1(1 − c2) + c(− c − c2) − c(c2 + c) = 0 has more than one solution, then D =0 and
⇒ 1 − c2 − c2 − c3 − c3 − c2 = 0 D1 = D2 = D3 = 0. In the given problem,
a 2 3
⇒ −2c3 − 3c2 + 1 = 0
D1 = 0 ⇒ b − 1 5 = 0
⇒ 2c3 + 3c2 − 1 = 0
c −3 2
⇒ (c + 1)[2c2 + c − 1] = 0
⇒ (c + 1)[2c2 + 2c − c − 1] = 0 ⇒ a (− 2 + 15) − 2(2b − 5c) + 3(− 3b + c) = 0
⇒ 13a − 4b + 10c − 9b + 3c = 0
⇒ (c + 1)(2c − 1)(c + 1) = 0
⇒ 13a − 13b + 13c = 0
1
⇒ c = − 1 or ⇒ a − b + c=0⇒b − a − c=0
2
1 10. We know that,
Clearly, the greatest value of c is . the system of linear equations
2
a1x + b1 y + c1z = 0
7. The given system of linear equations is
x − 2 y − 2 z = λx a 2x + b2y + c2z = 0
x + 2 y + z = λy a3 x + b3 y + c3 z = 0
− x − y − λz = 0, has a non-trivial solution, if
which can be rewritten as a1 b1 c1
(1 − λ )x − 2 y − 2z = 0 a 2 b2 c2 = 0
⇒ x + (2 − λ ) y + z = 0
a3 b3 c3
x + y + λz = 0
Now, for non-trivial solution, we should have Now, if the given system of linear equations
1 − λ −2 −2 x + 3 y + 7z = 0
1 2 − λ 1 =0 − x + 4 y + 7z = 0,
1 1 λ and (sin 3 θ )x + (cos 2 θ ) y + 2z = 0
[Q If a1x + b1 y + c1z = 0; a 2x + b2y + c2z = 0 has non-trivial solution, then
a3 x + b3 y + c3 z = 0] 1 3 7
a1 b1  c1 −1 4 7 =0
has a non-trivial solution, then a 2 b2 c2 = 0 sin 3 θ cos 2 θ 2

a3 b3 c3  ⇒ 1(8 − 7 cos 2 θ ) − 3 (− 2 − 7 sin 3 θ )
⇒ (1 − λ ) [(2 − λ )λ − 1] + 2 [λ − 1] − 2 [1 − 2 + λ ] = 0 + 7 (− cos 2 θ − 4 sin 3 θ ) = 0
⇒ (λ − 1)[λ2 − 2λ + 1 + 2 − 2] = 0 ⇒ 8 − 7 cos 2 θ + 6 + 21 sin 3 θ
⇒ (λ −1)3 = 0 − 7 cos 2 θ − 28 sin 3 θ = 0
⇒ λ =1 ⇒ − 7 sin 3 θ − 14 cos 2 θ + 14 = 0
8. Given system of linear equations, ⇒ − 7 (3 sin θ − 4 sin3 θ ) − 14 (1 − 2 sin 2 θ ) +14 = 0
(1 + α )x + βy + z = 2 [ Q sin 3 A = 3 sin A − 4 sin3 A and
αx + (1 + β ) y + z = 3 cos 2 A = 1 − 2 sin 2 A]
αx + βy + 2z = 2
has a unique solution, if ⇒ 28 sin θ + 28 sin θ − 21 sin θ − 14 + 14 = 0
3 2

⇒ 7 sin θ [4 sin 2 θ + 4 sin θ − 3] = 0


1+α β 1
⇒ sin θ [4 sin 2 θ + 6 sin θ − 2 sin θ − 3] = 0
α (1 + β ) 1 ≠ 0
⇒ sin θ [2 sin θ (2 sin θ + 3) − 1 (2 sin θ + 3)] = 0
α β 2
⇒ (sin θ ) (2 sin θ − 1) (2 sin θ + 3) = 0
Apply R1 → R1 − R3 and R2 → R2 − R3 1
Now, either sin θ = 0 or
1 0 −1 2
0 1 −1 ≠0  3 
Q sin θ ≠ − as − 1 ≤ sin θ ≤ 1
 2 
α β 2
In given interval (0, π ),
⇒ 1(2 + β ) − 0(0 + α ) − 1(0 − α ) ≠ 0 ⇒ α + β + 2 ≠ 0 … (i)
1
Note that, only (2, 4) satisfy the Eq. (i). sin θ =
2
9. We know that, if the system of equations π 5π
⇒ θ= , [Q sin θ ≠ 0, θ ∈ (0, π )]
a1x + b1 y + c1z = d1 6 6
a 2x + b2y + c2z = d2 Hence, 2 solutions in (0, π )
a3 x + b3 y + c3 z = d3
Matrices and Determinants 165

11. Since, the system of equations has infinitely many


solution, therefore D = D1 = D2 = D3 = 0 2 0 0
1
Here, = 5 −1
1 1 1 2
5 a a2 − 4
D = 1 2 3 = 1 (2α − 9) − 1 (α − 3) + 1(3 − 2) = α − 5 a+1 −
2 2
1 3 α
1 5 a 
1 1 5 = 2  (a 2 − 4) +  −   [Expanding along R1]
 2 2 2 
and D3 = 1 2 9 = 1 (2 β − 27) − 1(β − 9) + 5 (3 − 2)
a2 5 a
1 3 β =2 − 2 + −  = a2 − 4 + 5 − a = a2 − a + 1
= β − 13 2 2 2

Now, D =0 Clearly, when a = 4, then D = 13 ≠ 0 ⇒ unique solution


and
⇒ α −5 =0 ⇒ α =5
when|a| = 3, then D = 0 and D1 ≠ 0.
and D3 = 0 ⇒ β − 13 = 0 ⇒ β = 13
∴When |a| = 3, then the system has no solution i.e.
∴ β − α = 13 − 5 = 8 system is inconsistent.
1 −4 7
14. We have,
12. Here, D= 0 3 −5 x + ky + 3z = 0; 3x + ky − 2z = 0; 2x + 4 y − 3z = 0
−2 5 −9 System of equation has non-zero solution, if
= 1(− 27 + 25) + 4(0 − 10) + 7(0 + 6) 1 k 3 
[expanding along R1]  3 k −2 = 0
 
= − 2 − 40 + 42 = 0  2 4 −3
∴The system of linear equations have infinite many ⇒ (−3k + 8) − k(−9 + 4) + 3(12 − 2k) = 0
solutions.
⇒ − 3k + 8 + 9k − 4k + 36 − 6k = 0
[Q system is consistent and does not have unique
⇒ − 4k + 44 = 0 ⇒ k = 11
solution as D = 0]
Let z = λ , then we get
⇒ D1 = D2 = D3 = 0
g −4 7 x + 11 y + 3λ = 0 …(i)
Now, D1 = 0 ⇒ h 3 − 5 = 0 3x + 11 y − 2λ = 0 …(ii)
and 2x + 4 y − 3λ = 0 …(iii)
k 5 −9
Solving Eqs. (i) and (ii), we get
⇒ g (− 27 + 25) + 4(− 9h + 5k) + 7(5h − 3k) = 0
5λ −λ xz 5λ2
⇒ − 2 g − 36h + 20k + 35h − 21k = 0 x= , y= ,z=λ ⇒ 2= 2
= 10
2 2 y  λ
⇒ − 2g − h − k = 0 ⇒2g + h + k = 0 2 × − 
 2
13 According to Cramer’s rule, here
15. Given, system of linear equation is
1 1 1 1 0 0 x + λy − z = 0; λx − y − z = 0; x + y − λz = 0
D= 2 3 2 = 2 1 0 Note that, given system will have a non-trivial solution
2 3 a2 − 1 2 1 a2 − 3 only if determinant of coefficient matrix is zero,
(Applying C 2 → C 2 − C1 and C3 → C3 − C1) 1 λ −1
= a −3
2
(Expanding along R1) i.e. λ −1 −1 = 0
2 1 1 2 1 0 1 1 −λ
and D1 = 5 3 2 = 5 3 −1
⇒ 1 (λ + 1) − λ(− λ2 + 1) − 1(λ + 1) = 0
a + 1 3 a2 − 1 a + 1 3 a2 − 1 −3 ⇒ λ + 1 + λ3 − λ − λ − 1 = 0
(Applying C3 → C3 − C 2) ⇒ λ3 − λ = 0 ⇒ λ(λ2 − 1) = 0
⇒ λ = 0or λ = ± 1
2 0 0 Hence, given system of linear equation has a
5 non-trivial solution for exactly three values of λ.
= 5 3− −1
2
16. Given system of linear equations
a + 1 3 − (a + 1) a 2 − 1 − 3
2 2x1 − 2x2 + x3 = λx1 …(i)
1
(Applying C 2 → C 2 − C1) ⇒ (2 − λ )x1 − 2x2 + x3 = 0 …(ii)
2 2x1 − 3x2 + 2x3 = λx2 …(iii)
166 Matrices and Determinants

⇒ 2x1 − ( 3 + λ )x2 + 2x3 = 0 ⇒ k2 − 4k + 3 ⇒ (k –1) (k –3) = 0


− x1 + 2x2 = λx3 ⇒ − x1 + 2x2 − λx3 = 0 k = 1, 3
Since, the system has non-trivial solution. 8 4.1
If k –1, then ≠ , false
2 − λ −2 1 1+3 2
∴  2 − (3 + λ ) 2  = 0 8 4.3
  And, if k = 3, then ≠ , true
 −1 2 λ  6 9 −1

⇒ (2 − λ )(3λ + λ2 − 4) + 2(−2λ + 2) + 1(4 − 3) − λ ) = 0 Therefore, k = 3


⇒ (2 − λ )(λ2 + 3λ − 4) + 4(1 − λ ) + (1 − λ ) = 0 Hence, only one value of k exist.
⇒ (2 − λ )(λ + 4)(λ − 1) + 5(1 − λ ) = 0  x 1
⇒ (λ − 1)[(2 − λ )(λ + 4) − 5] = 0 18. Since, A  y = 0 is linear equation in three variables
   
⇒ (λ − 1)(λ2 + 2λ − 3) = 0  z  0
⇒ (λ − 1)[(λ − 1)(λ + 8)] = 0 and that could have only unique, no solution or
⇒ (λ − 1)2(λ + 3) = 0 infinitely many solution.
⇒ λ = 1, 1, − 3 ∴It is not possible to have two solutions.
Hence, λ contains two elements. Hence, number of matrices A is zero.
17. Given equations can be written in matrix form 19. Since, given system has no solution.
AX = B ∴ ∆ = 0 and any one amongst ∆ x , ∆ y , ∆ z is non-zero.
k+1 x  2 −1 2   2 −1 2
where, A = 
8  4k
, X =   and B = Let 1 −2 1  = 0 and ∆ z =  1 −2 −4  = 6 ≠ 0
 k k + 3   y 3k − 1
1 1 λ 1 1 4
For no solution, A = 0 and (adj A) B ≠ 0
⇒ λ =1
k+1
Now, A = 
8 
= 0 ⇒ (k2 + 1)(k + 3) − 8k = 0
 k k + 3  20. For infinitely many solutions, we must have
k+1 8 4k
k2 + 4k + 3 − 8k = 0 ⇒ k2 − 4k × 3 = 0 = = ⇒ k =1
k k + 3 3k − 1
⇒ (k − 1)(k − 3) = 0 ⇒ k = 1, k = 3,
k + 3 −8  21. Given equations x + ay = 0, az + y = 0, ax + z = 0 has
Now adj A = 
 − k k + 1  infinite solutions.
k + 3 − 8   4k 1 a 0 
Now, (adj A)B =  
∴ 0 1 a = 0
 − k k + 1   3 k + 1 
a 0 1 
 (k + 3)(4k) − 8 (3k − 1)
= ⇒ 1 + a3 = 0 or a = − 1
 − 4k2 + (k + 1)(3k − 1) 
 4k2 − 12k + 8 
22. Since, the given system has non-zero solution.
=  1 −k −1 
 − k + 2k − 1 
2
∴ k −1 −1  = 0
Put k = 1 1 1 −1 
4 − 12 + 8 0
(adj A) B =  = not true Applying C1 → C1 − C 2, C 2 → C 2 + C3
−1 + 2 − 1  0
 1 + k −k − 1 −1 
Put k = 3 ⇒ 1 + k −2 −1  = 0
 0 0 −1 
36 − 36 + 8  8 
(adj A) B =  = ≠ 0 true
−9 + 6 − 1 −4 ⇒ 2(k + 1) − (k + 1)2 = 0
Hence, required value of k is 3. ⇒ (k + 1)(2 − k − 1) = 0
Alternate Solution ⇒ k=±1
Condition for the system of equations has no solution is NOTE There is a golden rule in determinant that n one’s ⇒
( n − 1) zero’s or n (constant) ⇒ ( n − 1) zero’s for all
a1 b1 c1
= ≠ constant should be in a single row or a single column.
a 2 b2 c2
23. The given system of equations can be expressed as
k+1 8 4k
∴ = ≠  1 −2 3  x   −1 
k k + 3 3k − 1  1 −3 4  y =  1
k+1 8     
Take = ⇒ k2 + 4k + 3 = 8k −1 1 −2  z   k 
k k+3
Matrices and Determinants 167

Applying R2 → R2 − R1 , R3 → R3 + R1 2 2 + 2x 2 + 5x 
1 −2 3  x   −1  = 0 8 8 
 
~ 0 −1 1  y =  2  x 3x 3x + 18
     
0 −1 1  z  k − 1 There is no common value of ‘x’, for which each
corresponding element of matrices PQ and QP is equal.
Applying R3 → R3 − R2 2 0 0
1 −2 3  x   −1  For x = 0, Q = 0 4 0
 
~ 0 −1 1  y =  2  0 0 6
     
0 0 0  z  k − 3 1  1 
When k ≠ 3 , the given system of equations has no then, if R a  = 6 a 
   
solution.  b   b 
⇒ Statement I is true. Clearly, Statement II is also
1  1 
true as it is rearrangement of rows and columns of
⇒ PQP −1 a  = 6 a  [Q R = PQP −1 ]
 1 −2 3     
 1 −3 4 .  b   b 
  1 1 1 2 0 0 6 −3 0  1  1 
−1 1 −2 1
⇒ 0 2 2 0 4 0 0 3 −2 a  = 6 a 
24. It is given, that matrices 6      
0 0 3 0 0 6 0 0 2   b   b 
1 1 1 2 x x
 2 4 6  6 − 3 0  1  
1
P = 0 2 2, Q = 0 4 0 1
    ⇒ 0 8 12 0 3 −2 a  = 6 a 
0 0 3 x x 6 6     
0 0 18 0 0 2   b   b 
adj (P )
∴ P −1 = 12 6 4  1  1 
|P|
T ⇒  0 24 8  a  = 36 a 
6 0 0     
as|P| = 6 and adj P = −3 3 0  0 0 36  b   b 
 
 0 −2 2  12 + 6 a + 4 b   36 
⇒ 0 + 24a + 8b = 36a 
6 −3 0    

1
P −1 = 0 3 −2   0 + 0 + 36b  36b 
6 
0 0 2  ⇒ 6a + 4b = 24 and 12a = 8b
−1 −1 ⇒ 3a + 2b = 12 and 3a = 2b
∴ |R| = |P Q P | [QR = PQP (given]
⇒ −1
|R| = |P||Q||P | = |Q| [Q|P||P −1| = |I| = 1] ⇒ a = 2 and b = 3
2 x x 2 x x 2 x 0 So a + b = 5.
= 0 4 0 = 0 4 0 + 0 4 0 α  0
x x 6 x x 5 x x 1 Now, R β  = 0 and αi$ + β$j + γk$ is a unit vector, so det
   
2 x x  γ  0
= 0 4 0 + 2 (4 − 0) − x (0 − 0) + 0(0 − 4x) (R) = 0
x x 5 ⇒ det(Q ) = 0 [Q R = PQP −1 So,|R|=|Q|]
2 x x
2 x x
⇒ 0 4 0 =0
= 0 4 0 + 8, for all x ∈ R
x x 6
x x 5
1 1 1 2 x x ⇒ 2 (24 − 0) − x (0 − 0) + x(0 − 4x)
Q PQ = 0 2 2 0 4 0 ⇒ 48 − 4x2 = 0
   ⇒ x2 = 12
0 0 3 x x 6
⇒ x=±2 3
2 + x 4 + 2x x + 6
So, for x = 1, there does not exist a unit vector
=  2x 2x + 8 12 
  α  0
 3x 3x 18  αi$ + β$j + γk$ , for which R β  = 0
   
2 x x 1 1 1  γ  0
and QP = 0 4 0 0 2 2
   Hence, options (b) and (d) are correct.
x x 6 0 0 3
168 Matrices and Determinants

25. We have, 26. Given system λx + y + z = 0, − x + λy + z = 0


− x + 2 y + 5z = b1 and − x − y + λz = 0
2x − 4 y + 3z = b2 will have non-zero solution, if
x − 2 y + 2z = b3 λ 1 1
has at least one solution.
−1 λ 1 = 0
−1 2 5
−1 −1 λ
∴ D= 2 −4 3
⇒ λ (λ 2 + 1) − 1 (− λ + 1) + 1(1 + λ ) = 0
1 −2 2
⇒ λ3 + λ + λ − 1 + 1 + λ = 0
and D1 = D2 = D3 = 0 ⇒ λ 3 + 3λ = 0
b1 2 5
⇒ λ (λ 2 + 3) = 0 ⇒ λ =0
⇒ D1 = b2 − 4 3
27. Since, AX = U has infinitely many solutions.
b3 − 2 2
a 0 1
= − 2b1 − 14b2 + 26b3 = 0 ⇒| A| = 0 ⇒ 1 c b =0
⇒ b1 + 7b2 = 13b3 ...(i) 1 d b
1 2 3 ⇒ a (bc − bd ) + 1(d − c) = 0 ⇒ (d − c)(ab − 1) = 0
(a) D = 0 4 5 = 1(24 − 10) + 1(10 − 12) ∴ ab = 1 or d = c
1 2 6 a 0 f
Again, | A3|= 1 c g =0 ⇒ g = h
= 14 − 2 = 12 ≠ 0
1 d h
Here, D ≠ 0 ⇒ unique solution for any b1, b2, b3 .
a f 1
1 1 3 ⇒ | A2| = 1 g b =0 ⇒ g=h
(b) D = 5 2 6 1 h b
−2 −1 −3 f 0 1
and | A1| = g c b =0 ⇒ g=h
= 1(− 6 + 6) − 1 (− 15 + 12) + 3 (− 5 + 4) = 0
h d b
For atleast one solution
∴ g = h , c = d and ab = 1 …(i)
D1 = D2 = D3 = 0
b1 1 3 Now, BX = V
Now, D1 = b2 2 6 a 1 1
|B| = 0 d c =0 [from Eq. (i)]
b3 −1 −3 f g h
= b1 (− 6 + 6) − b2(− 3 + 3) + b3 (6 − 6) = 0 [since, C 2 and C3 are equal]
1 b1 3 ∴ BX = V has no solution.
D2 = 5 b2 6 a2 1 1
− 2 b3 −3 |B1|= 0 d c =0 [from Eq. (i)]
0 g h
= − b1 (− 15 + 12) + b2(− 3 + 6) − b3 (6 − 15)
= 3b1 + 3b2 + 9b3 = 0 ⇒ b1 + b2 + 3b3 = 0 [since, c = d and g = h]
not satisfies the Eq. (i) a a2 1
|B2| = 0 0 c = a 2cf = a 2df [Q c = d ]
It has no solution. f 0 h
−1 2 −5
(c) D = 2 − 4 10 Since, adf ≠ 0 ⇒ |B2| ≠ 0
1 −2 5 |B| = 0 and |B2| ≠ 0
= − 1(− 20 + 20) − 2(10 − 10) − 5(− 4 + 4) = 0 ∴ BX = V has no solution.
Here, b2 = − 2b1 and b3 = − b1 satisfies the Eq. (i) 28. Given, λx + (sin α ) y + (cos α ) z = 0
Planes are parallel. x + (cos α ) y + (sin α ) z = 0
1 2 5
and − x + (sin α ) y − (cos α ) z = 0 has non-trivial
(d) D = 2 0 3 = 1(0 − 12) − 2 (− 10 − 3) + 5 (8 − 0) solution.
1 4 −5 ∴ ∆ =0
= 54 λ sin α cos α
D ≠0 ⇒ 1 cos α sin α = 0
It has unique solution for any b1, b2, b3 . −1 sin α − cos α
Matrices and Determinants 169

⇒ λ (− cos 2 α − sin 2 α ) − sin α (− cos α + sin α ) ⇒ sin θ (4 sin 2 θ + 4 sin θ − 3) = 0


+ cos α (sin α + cos α ) = 0 ⇒ sin θ (2 sin θ − 1) (2 sin θ + 3) = 0
1
⇒ − λ + sin α cos α + sin α cos α − sin 2 α + cos 2 α = 0 ⇒ sin θ = 0, sin θ =
2
⇒ λ = cos 2α + sin 2α
[neglecting sin θ = − 3 / 2]
Q − a 2 + b2 ≤ a sin θ + b cos θ ≤ a 2 + b2  π
  ⇒ θ = nπ , nπ + (−1)n , n ∈ Z
6
∴ − 2≤λ≤ 2 …(i)
31. The given system of equations
Again, when λ = 1, cos 2α + sin 2α = 1
3x − y + 4z = 3
1 1 1
⇒ cos 2α + sin 2α = x + 2 y − 3z = − 2
2 2 2
6 x + 5 y + λz = − 3
⇒ cos (2α − π / 4) = cos π / 4 has atleast one solution, if ∆ ≠ 0.
∴ 2α − π / 4 = 2n π ± π / 4 3 −1 4
⇒ 2α = 2nπ − π / 4 + π / 4 or 2α = 2nπ + π / 4 + π / 4 ∴ ∆ = 1 2 −3 ≠ 0
∴ α = nπ or nπ + π /4 6 5 λ
29. Since, α 1 , α 2 are the roots of ax + bx + c = 0.
2
⇒ 3 (2λ + 15) + 1 (λ + 18) + 4 (5 − 12) ≠ 0
b c ⇒ 7 (λ + 5) ≠ 0 ⇒ λ ≠ −5
⇒ α1 + α 2 = − and α 1α 2 = ...(i)
a a Let z = − k, then equations become
Also, β1 , β 2 are the roots of px2 + qx + r = 0. 3x − y = 3 − 4k and x + 2 y = 3k − 2
q r On solving, we get
⇒ β1 + β 2 = − and β1β 2 = ...(ii)
p p 4 − 5k 13k − 9
x= ,y= ,z=k
Given system of equations 7 7
α 1 y + α 2z = 0 32. Given system of equations are
and β1 y + β 2 z = 0, has non-trivial solution. 3x + my = m and 2x − 5 y = 20
α1 α 2 α 1 β1 3 m
∴ =0 ⇒ = Here, ∆= = −15 − 2m
β1 β 2 α 2 β2 2 −5
α 1 + α 2 β1 + β 2 m m
Applying componendo-dividendo, = and ∆x = = −25m
α 1 − α 2 β1 − β 2 20 −5
⇒ (α 1 + α 2) (β1 − β 2) = (α 1 − α 2) (β1 + β 2) 3 m
∆y = = 60 − 2m
⇒ (α 1 + α 2)2 {(β1 + β 2)2 − 4 β 2 β 2} 2 20
= (β1 + β 2)2{(α 1 + α 2)2 − 4 α 1α 2} If ∆ = 0, then system is inconsistent, i.e. it has no
From Eqs. (i) and (ii), we get solution.
b2  q2 4r  q2  b2 4c If ∆ ≠ 0, i.e. m ≠
15
 − =  −  , the system has a unique solution
a 2  p2 p  p2  a 2 a  2
for any fixed value of m.
b2q2 4b2r b2q2 4q2c ∆ − 25m 25m
⇒ − = − We have, x= x = =
a 2p2 a 2p a 2p2 ap2 ∆ − 15 − 2 m 15 + 2m
b2r q2c b2 ac ∆y 60 − 2m 2m − 60
⇒ = ⇒ 2= and y= = =
a p q pr ∆ − 15 − 2m 15 + 2m
30. The system of equations has non-trivial solution, if 25m
For x > 0 , >0 ⇒ m >0
∆ = 0. 15 + 2m
sin 3θ −1 1 15
⇒ cos 2θ 4 3 = 0 or m<− …(i)
2
2 7 7 2m − 60
and y > 0, >0
Expanding along C1 , we get 2m + 15
sin 3θ ⋅ (28 − 21) − cos 2 θ (−7 − 7) + 2 (−3 − 4) = 0 ⇒ m > 30 or m < −
15
…(ii)
⇒ 7 sin 3θ + 14 cos 2θ − 14 = 0 2
15
⇒ sin 3θ + 2 cos 2 θ − 2 = 0 From Eqs. (i) and (ii), we get m < − or m > 30
⇒ 3 sin θ − 4 sin3 θ + 2 (1 − 2 sin 2 θ ) − 2 = 0 2
170 Matrices and Determinants

33. Since, the given system of equations posses non-trivial x + 2 y = 10 …(ii)


0 1 −2 and 3x + 2 y = µ …(iii)
solution, if 0 −3 1 =0 ⇒ k =0 On solving Eqs. (i) and (ii), we are getting
k −5 4 y = 4 and x = 2
On solving the equations x = y = z = λ [say] From Eq. (iii), we get
∴ For k = 0, the system has infinite solutions of λ ∈R. µ = 3(2) + 2(4) = 14
Now, on putting y = 0 and µ = 14, in the given system of
34. Given systems of equations can be rewritten as linear Eq., we get
− x + cy + by = 0, cx − y + az = 0 and bx + ay − z = 0
x+ z =6 …(iv)
Above system of equations are homogeneous equation. x + 3z = 10 …(v)
Since, x, y and z are not all zero, so it has non-trivial
and 3x + λz = 14 …(vi)
solution.
On solving Eqs. (iv) and (v), we get z = 2 and x = 4
Therefore, the coefficient of determinant must be zero.
−1 c b From Eq. (vi), we get
3(4) + λ (2) = 14
∴ c −1 a = 0
⇒ λ =1
b a −1
∴ µ − λ2 = 14 − 1 = 13
⇒ − 1 (1 − a 2) − c (− c − ab) + b (ca + b) = 0
1 α α2
⇒ a 2 + b2 + c2 + 2abc − 1 = 0
37. α 1 α =0
⇒ a 2 + b2 + c2 + 2abc = 1
α2 α 1
35. System of given linear equations is
x − 2 y + 3z = 9 ⇒ α 4 − 2α 2 + 1 = 0
2x + y + z = b ⇒ α2 = 1 ⇒α = ± 1
and x − 7 y + az = 24, has infinitely many solution, so But α = 1 not possible [Not satisfying equation]
∆ = 0 = ∆1 = ∆ 2 = ∆3 ∴ α = −1
So, ∆ =0 Hence, 1 + α + α 2 = 1
1 −2 3 a1 a 2 a3 
⇒ 2 1 1= 0
  38. Let M =  b1 b2 b3 
 
1 −7 a  c1 c2 c3 
⇒ 1(a + 7) + 2(2a − 1) + 3(−14 − 1) = 0 0 − 1  1  1
⇒ 5a = 40 ⇒ a = 8 ∴ M 1 =  2 ⇒ M − 1 =  1
       
1 −2 9  0  3  0 − 1
and ∆3 = 0 ⇒2 1 b
= 0    
1 0 a 2 − 1 a1 − a 2  1
1 −7 24 M ⋅ 1 = 0 ⇒  b2  =  2,  b1 − b2  =  1,
   
           
⇒ 1(24 + 7b) + 2(48 − b) + 9(−14 − 1) = 0 1 12  c2   3  c1 − c2  − 1
⇒ 5b + 120 − 135 = 0 ⇒ b = 3
a1 + a 2 + a3   0
∴ a − b =8 −3=5  b + b + b  =  0
Hence, answer is 5.00.  1 2 3
  
 c1 + c2 + c3  12
36. The given system of linear equations
⇒ a 2 = − 1, b2 = 2, c2 = 3, a1 − a 2 = 1,
x + y + z = 6,
b1 − b2 = 1, c1 − c2 = − 1
x + 2 y + 3z = 10,
⇒ a1 + a 2 + a3 = 0, b1 + b2 + b3 = 0
and 3x + 2 y + λz = µ has more than two solutions, so
system must have infinite number of solutions. c1 + c2 + c3 = 12
Now, on putting z = 0 in above equation, we get ∴ a1 = 0, b2 = 2, c3 = 7
x+ y=6 …(i) ⇒ Sum of diagonal elements = 0 + 2 + 7 = 9
8
Functions
Topic 1 Classification of Functions, Domain and
Range and Even, Odd Functions
Objective Questions I (Only one correct option) log 2(x + 3)
7. The domain of definition of f (x) = is
x2 + 3x + 2
1. If R = {(x, y): x, y ∈ Z , x2 + 3 y2 ≤ 8} is a relation on the set
of integers Z, then the domain of R−1 is
(2001, 1M)
(a) R / {− 1, − 2}
(2020 Main, 2 Sep I)
(b) (− 2, ∞ )
(a) {−1, 0,1} (b) {− 2, − 1,1, 2} (c) R / {− 1, − 2, − 3}
(c) {−2, − 1, 0,1, 2} (d) {0,1} (d) (− 3, ∞ ) / {− 1, − 2}
2. The domain of the definition of the function 8. The domain of definition of the function y (x) is given by
1
f (x) = + log10 (x3 − x) is (2019 Main, 9 April II) the equation 2x + 2y = 2 , is (2000, 1M)
4 − x2
(a) 0 < x ≤ 1 (b) 0 ≤ x ≤ 1
(a) (−1, 0) ∪ (1, 2) ∪ (3, ∞ ) (b) (−2, − 1) ∪ (−1, 0) ∪ (2, ∞ )
(c) − ∞ < x ≤ 0 (d) −∞ < x < 1
(c) (−1, 0) ∪ (1, 2) ∪ (2, ∞ ) (d) (1, 2) ∪ (2, ∞ )
9. Let f (θ ) = sin θ (sin θ + sin 3 θ ). Then, f (θ )
3. Let f (x) = a x (a > 0) be written as f (x) = f1 (x) + f2(x), (2000, 1M)

where f1 (x) is an even function and f2(x) is an odd (a) ≥ 0, only when θ ≥ 0 (b) ≤ 0, for all real θ
function. Then f1 (x + y) + f1 (x − y) equals (c) ≥ 0, for all real θ (d) ≤ 0, only when θ ≤ 0
(2019 Main, 8 April II) 10. The domain of definition of the function
(a) 2f1 (x + y) ⋅ f2 (x − y) (b) 2f1 (x + y) ⋅ f1 (x − y) 1
y= + x + 2 is
(c) 2f1 (x) ⋅ f2 ( y) (d) 2f1 (x) ⋅ f1 ( y) log10 (1 − x) (1983, 1M)
4. Domain of definition of the function (a) (− 3, − 2) excluding − 2. 5
π (b) [0, 1] excluding 0.5
f (x) = sin −1 (2x) + for real valued x, is
6 (2003, 2M) (c) (−2, 1) excluding 0

(a)  − ,  (b)  − , 
1 1 1 1 (d) None of these
 4 2   2 2 
Match the Columns
(c)  − ,  (d)  − , 
1 1 1 1
 2 9  4 4  Match the conditions / expressions in Column I with
x2 + x + 2 statement in Column II.
5. Range of the function f (x) = 2 ; x ∈ R is
x + x+1 x2 − 6 x + 5
(2003, 2M) 11. Let f (x) = .
(a) (1, ∞) (b) (1, 11/7) x2 − 5 x + 6 (2007, 6M)
(c) (1, 7/3] (d) (1, 7/5)
Column I Column II
6. Let f (x) = (1 + b2) x2 + 2bx + 1 and let m (b) be the
A. If −1 < x < 1, then f ( x ) satisfies p. 0 < f (x ) < 1
minimum value of f (x). As b varies, the range of m (b) is
(2001, 1M) B. If 1 < x < 2 , then f ( x ) satisfies q. f (x ) < 0
(b)  0, 
1
(a) [0, 1] C. If 3 < x < 5, then f ( x ) satisfies r. f (x ) > 0
 2 

(c)  , 1 If x > 5, then f ( x ) satisfies f (x ) < 1


1 D. s.
(d) (0, 1]
 2 
172 Functions

Objective Question II True/False


(One or more than one correct option) 16. If f1 (x) and f2(x) are defined on domains D1 and D2
2x − 1 respectively, then f1 (x) + f2(x) is defined on D1 ∩ D2.
12. If S is the set of all real x such that 3 is
2 x + 3 x2 + x (1988, 1M)
positive, then S contains (1986, 2M)
Analytical & Descriptive Questions
(a)  − ∞ , −  (b)  − , − 
3 3 1
 2  2 4 17. Find the range of values of t for which
1 − 2 x + 5 x2  π π
(c)  − ,  (d)  , 3
1 1 1
2 sin t = , t∈ − , .
 4 2 2  3 x2 − 2 x − 1  2 2 
(2005, 2M)

Fill in the Blanks (x + 1) (x − 3)


18. Let y= .
 4 − x2  (x − 2)
13. If f (x) = sin log   , then the domain of Find all the real values of x for which y takes real
 1−x 
  values. (1980, 2M)
f (x) is ……… . (1985, 2M)
 x2 
Integer & Numerical Answer Type Question
14. The domain of the function f (x) = sin −1  log 2  is
 2 19. The value of the limit
given by ...
(1984, 2M) 4 2 (sin 3x + sin x)
 π2  lim
15. The values of f (x) = 3 sin  − x2 lie in the
π 3x 5x  3x

x→
2 sin 2x sin + cos  −  2 + 2 cos 2x + cos 
 16 
2
 2 2  2
interval ……… (1983, 2M)
is ……… (2020 Adv.)

Topic 2 Composite of Functions


10
Objective Questions I (Only one correct option) 4. Let ∑ f (a + k) = 16(210 − 1), where the function f
1. For a suitable chosen real constant a, let a functin k =1
a −x satisfies f (x + y) = f (x) f ( y) for all natural numbers x, y
f : R − { a } → R be defined by f (x) = . Further
a+x and f (1) = 2. Then, the natural number ‘a’ is
suppose that for any real number x ≠ − a and f (x) ≠ − a, (2019 Main, 9 April I)
 1 (a) 2 (b) 4 (c) 3 (d) 16
( fof )(x) = x. Then, f  −  is equal to
 2 [2020 Main, 6 Sep II]  1 − x  2x 
5. If f (x) = log e   ,|x| < 1, then f  is equal to
(a)
1
(b) −
1  1 + x  1 + x2 
3 3 (2019 Main, 8 April I)
(c) −3 (d) 3 (a) 2f (x) (b) 2f (x2 )
(c) (f (x))2 (d) −2f (x)
2. For x ∈ 0,  , let f (x) = x , g (x) = tan x and
3
 2 1
6. For x ∈ R − {0, 1}, let f1 (x) = , f2(x) = 1 − x and
1 − x2  π x
h (x) = . If φ(x) = ((hof )og )(x), then φ   is equal to 1
1 + x2  3 f3 (x) = be three given functions. If a function, J (x)
1 −x
(2019 Main, 12 April I)
satisfies ( f2° J ° f1 )(x) = f3 (x), then J (x) is equal to
π 11π
(a) tan (b) tan (2019 Main, 9 Jan I)
12 12 (a) f2 (x) (b) f3 (x)
7π 5π 1
(c) tan (d) tan (c) f1 (x) (d) f3 (x)
12 12 x
3. Let f (x) = x , x ∈ R.
2
For any A ⊆ R, define
7. Let a , b, c ∈ R. If f (x) = ax2+ bx + c be such that
g ( A ) = { x ∈ R : f (x) ∈ A }. If S = [0, 4], then which one of
the following statements is not true? a + b + c = 3 and f (x + y) = f (x) + f ( y) + xy, ∀ x, y ∈ R,
10
(2019 Main, 10 April I) then ∑ f (n ) is equal to
(a) f ( g (S)) = S (b) g (f (S)) ≠ S n =1 (2017 Main)
(c) g (f (S)) = g (S) (d) f(g(S)) ≠ f (S) (a) 330 (b) 165 (c) 190 (d) 255
Functions 173

8. Let f (x) = x2 and g (x) = sin x for all x ∈ R. Then, the set of 15. If f (x) = cos [π 2] x + cos [− π 2] x, where [x] stands for the
all x satisfying ( fogogof )(x) = ( gogof )(x) , where greatest integer function, then (1991, 2M)
( fog )(x) = f ( g (x)), is (2011) (a) f ( π / 2) = − 1
(a) ± nπ, n ∈ {0, 1, 2, K } (b) f ( π) = 1
(b) ± nπ, n ∈ {1, 2, K } (c) f (− π ) = 0
(d) f ( π / 4) = 1
(c) π /2 + 2nπ, n ∈ {...,− 2, − 1, 0, 1, 2, K }
(d) 2nπ, n ∈ {..., − 2, − 1, 0, 1, 2, K } 16. Let g (x) be a function defined on [− 1, 1]. If the area of
αx the equilateral triangle with two of its vertices at (0, 0)
9. Let f (x) = , x ≠ − 1. Then, for what value of α is and [x, g (x)] is 3 / 4, then the function g (x) is
x+1
(1989, 2M)
f [ f (x)] = x ? (2001, 1M)
(a) 2 (b) − 2 (c) 1 (d) −1 (a) g (x) = ± 1 − x2 (b) g (x) = 1 − x2
− 1 , x < 0 (c) g (x) = − 1 − x2 (d) g (x) = 1 + x2

10. Let g (x) = 1 + x − [x] and f (x) =  0, x = 0 , then for all x+2
 1, x > 0 17. If y = f (x) = , then
 x−1 (1984, 3M)
x, f [ g (x)] is equal to (2001, 1M) (a) x = f ( y) (b) f (1) = 3
(a) x (b) 1 (c) f (x) (d) g (x) (c) y increases with x for x < 1
(d) f is a rational function of x
11. If g { f (x) } = |sin x| and f { g (x) } = (sin x )2, then
(1998, 2M)
Fill in the Blank
(a) f (x) = sin 2 x, g (x) = x
(b) f (x) = sin x, g (x) = | x|  π  π
18. If f (x) = sin 2 x + sin 2 x +  + cos x cos  x +  and
(c) f (x) = x2 , g (x) = sin x  3  3
(d) f and g cannot be determined  5
g   = 1, then (g o f ) (x) = ... .
1   x   4 (1996, 2M)
12. If f (x) = cos(log x), then f (x) ⋅ f ( y) −  f   + f (xy)
2  y  
has the value (1983, 1M)
True/False
(a) −1 (b)
1 19. If f (x) = (a − xn )1/ n, where a > 0 and n is a positive
2 integer, then f [ f (x)] = x. (1983, 1M)
(c) − 2 (d) None of these
13. Let f (x) = | x − 1|. Then, (1983, 1M) Analytical & Descriptive Question
(a) f (x ) = {f (x) }
2 2
(b) f (x + y) = f (x) + f ( y) 20. Find the natural number a for which
(c) f (| x|) = | f (x)| (d) None of the above n

∑ f (a + k) = 16 (2n − 1),
k =1
Objective Questions II
where the function f satisfies the relation
(One or more than one correct option) f (x + y) = f (x) f ( y) for all natural numbers x , y and
π π  further f (1) = 2. (1992, 6M)
14. Let f (x) = sin  sin  sin x  for all x ∈ R and
6  2 
π Integer & Numerical Answer Type Question
g (x) = sin x for all x ∈ R. Let ( fog )(x) denotes f { g (x)}
2 21. Let the function f : [0, 1] → R be defined by
and (gof ) (x) denotes g{f (x)}. Then, which of the
following is/are true? 4x
(2015 Adv.) f (x) =
(a) Range of f is  − ,  (b) Range of fog is  − , 
1 1 1 1 4x + 2
 2 2   2 2 
Then the value of
f (x ) π
(c) lim = 1 2 3  39  1
x → 0 g (x ) 6 f   + f   + f   + ... + f   − f   is ………
 40  40  40  40  2
(d) There is an x ∈ R such that ( gof ) (x) = 1 (2020 Adv.)
174 Functions

Topic 3 Types of Functions


Objective Questions I (Only one correct option) 8. The function f : R → − ,  defined as f (x) =
1 1 x
is
1. If the function f : R → R is defined by f (x) =|x|(x − sin x),  2 2 1 + x2
(2017 Main)
then which of the following statements is TRUE?
(a) invertible
(a) f is one-one, but NOT onto (2020 Adv.)
(b) injective but not surjective
(b) f is onto, but NOT one-one
(c) surjective but not injective
(c) f is BOTH one-one and onto
(d) neither injective nor surjective
(d) f is NEITHER one-one NOR onto
9. The function f: [0, 3] → [1, 29], defined by
2. If the function f : R − {1, − 1} → A defined by
x2 f (x) = 2x3 − 15 x2 + 36x + 1, is (2012)
f (x) = , is surjective, then A is equal to (a) one-one and onto (b) onto but not one-one
1 − x2 (2019 Main, 9 April I)
(c) one-one but not onto (d) neither one-one nor onto
(a) R − {−1}
(b) [0, ∞ ) x, if x is rational 0, if x is rational
10. f (x) =  , g (x) = 
(c) R − [−1, 0) 0 , if x is irrational x, if x is irrational
(d) R − (−1, 0) Then, f − g is (2005, 1M)
3. Let a function f : (0, ∞ ) → (0, ∞ ) be defined by (a) one-one and into (b) neither one-one nor onto
1 (c) many one and onto (d) one-one and onto
f (x) = 1 − . Then, f is
x x
(2019 Main, 11 Jan II) 11. If f : [0, ∞ ) → [0, ∞ ) and f (x) = , then f is
(a) injective only 1+ x (2003, 2M)
(b) both injective as well as surjective (a) one-one and onto (b) one-one but not onto
(c) not injective but it is surjective (c) onto but not one-one (d) neither one-one nor onto
(d) neither injective nor surjective
12. Let function f : R → R be defined by f (x) = 2 x + sin x
4. The number of functions f from {1, 2, 3, … , 20} onto {1, for x ∈ R . Then, f is (2002, 1M)
2, 3, … , 20} such that f (k) is a multiple of 3, whenever k (a) one-to-one and onto (b) one-to-one but not onto
is a multiple of 4, is (2019 Main, 11 Jan II)
(c) onto but not one-to-one (d) neither one-to-one nor onto
(a) (15)! × 6! (b) 56 × 15
(c) 5! × 6! (d) 65 × (15)!
13. Let E = {1, 2, 3, 4} and F = {1, 2}. Then, the number of
x onto functions from E to F is (2001, 1M)
5. Let f : R → R be defined by f (x) = , (a) 14 (b) 16 (c) 12 (d) 8
1 + x2
x ∈ R. Then, the range of f is (2019 Main, 11 Jan I) Match the Columns
(a)  − , 
1 1
(b) (−1, 1) − {0}
 2 2 
Match the conditions/expressions in Column I with
statement in Column II.
(c) R −  − , 
1 1
(d) R − [−1, 1]
 2 2  14. Let f1 : R → R, f2 : [0, ∞ ] → R, f3 : R → R and
f4 : R → [0, ∞ ) be defined by (2014 Adv.)
6. Let N be the set of natural numbers and two functions f
and g be defined as f , g : N → N such that |x|, if x < 0 sin x, if x < 0
f1 (x) =  x ; f2(x) = x2; f3 (x) = 
n + 1  e , if x ≥ 0  x, if x ≥ 0
; if n is odd
  f [ f (x)], if x < 0
f (n ) =  2
n and f4 (x) =  2 1
 ; if n is even  f2[ f1 (x)] − 1, if x ≥ 0
 2
and g (n ) = n − (−1)n. Then, fog is (2019 Main, 10 Jan II) Column I Column II
(a) one-one but not onto (b) onto but not one-one
A. f4 is p. onto but not one-one
(c) both one-one and onto (d) neither one-one nor onto
B. f3 is q. neither continuous nor one-one
7. Let A = { x ∈ R : x is not a positive integer}. Define a
2x C. f2of1 is r. differentiable but not one-one
function f : A → R as f (x) = , then f is
x−1 (2019 Main, 9 Jan II) D. f2 is s. continuous and one-one
(a) injective but not surjective Codes
(b) not injective A B C D A B C D
(c) surjective but not injective (a) r p s q (b) p r s q
(d) neither injective nor surjective (c) r p q s (d) p r q s
Functions 175

15. Let the functions defined in Column I have domain True/False


(−π /2, π /2) and range (− ∞ , ∞ ) (1992, 2M)
x2 + 4x + 30
Column I Column II 18. The function f (x) = is not one-to-one.
x2 − 8x + 18 (1983, 1M)
A. 1 + 2 x p. onto but not one-one
B. tan x q. one-one but not onto
r. one-one and onto
s. neither one-one nor onto Analytical & Descriptive Question
19. A function f : IR → IR, where IR, is the set of real
Objective Question II αx2 + 6x − 8
numbers, is defined by f (x) = .
(One or more than one correct option) α + 6 x − 8 x2
π π
16. Let f :  − ,  → R be given by Find the interval of values of α for which is onto. Is the
 2 2 functions one-to-one for α = 3 ? Justify your answer.
f (x) = [log(sec x + tan x)]3 . Then, (1996, 5M)
(a) f (x) is an odd function (b) f (x) is a one-one function
(c) f (x) is an onto function (d) f (x) is an even function
20. Let A and B be two sets each with a finite number of
elements. Assume that there is an injective mapping
from A to B and that there is an injective mapping
Fill in the Blanks from B to A. Prove that there is a bijective mapping
17. There are exactly two distinct linear functions, …, from A to B. (1981, 2M)
and… which map {– 1, 1} onto {0, 2}. (1989, 2M)

Topic 4 Inverse and Periodic Functions


Objective Questions I (Only one correct option) 4. Suppose f (x) = (x + 1)2 for x ≥ − 1. If g (x) is the function
1. The inverse function of whose graph is reflection of the graph of f (x) with
−2x respect to the line y = x, then g (x) equals
8 −8
2x
f (x) = , x ∈ (− 1, 1) is (2002, 1M)
82x + 8−2x 1
[2020 Main 8 Jan I] (a) − x − 1, x ≥ 0 (b) ,x> −1
1 1− x (x + 1)2
(a) (log 8 e) log e  
4 1 + x (c) x + 1 , x ≥ − 1 (d) x − 1, x ≥ 0

1− x
1
(b)
1
log e   5. If f : [1, ∞ ) → [2, ∞ ) is given by f (x) = x + , then f −1(x)
x
4 1+ x
equals (2001, 1M)
1 1 + x 1 1 + x
(c) (log 8 e) log e   (d) log e   x+ x −42
x
4 1− x 4 1− x (a) (b)
2 1 + x2
2. If X and Y are two non-empty sets where f : X → Y , is x− x2 − 4
(c) (d) 1 + x2 − 4
function is defined such that 2
f (C ) = { f (x) : x ∈ C } for C ⊆ X
6. If the function f : [1, ∞ ) → [1, ∞ ) is defined by
and f −1 (D ) = { x : f (x) ∈ D } for D ⊆ Y ,
f (x) = 2x ( x − 1), then f −1 (x) is (1999, 2M)
for any A ⊆ Y and B ⊆ Y , then (2005, 1M) x ( x − 1)
(a)  
1 1
(a) f −1 {f (A )} = A (b) (1 + 1 + 4 log2 x )
 2 2
(b) f −1 {f (A )} = A, only if f (X ) = Y
1
(c) f {f −1 (B )} = B, only if B ⊆ f (x) (c) (1 − 1 + 4 log2 x ) (d) not defined
2
(d) f {f −1 (B )} = B
3. If f (x) = sin x + cos x, g (x) = x2 − 1, then g { f (x) } is 7. If f (x) = 3x − 5, then f −1 (x) (1998, 2M)
1
invertible in the domain (2004, 1M) (a) is given by
3x − 5
π π π
(a)  0,  (b)  − ,  x+ 5
 2   4 4  (b) is given by
3
π π
(c)  − ,  (d) [0 , π ] (c) does not exist because f is not one-one
 2 2  (d) does not exist because f is not onto
176 Functions

8. Which of the following functions is periodic? (1983, 1M) (b) Statement I is true, Statement II is also true;
(a) f (x) = x − [ x ], where [x] denotes the greatest integer Statement II is not the correct explanation of
less than or equal to the real number x Statement I.
(b) f (x) = sin (1 /x) for x ≠ 0, f (0) = 0 (c) Statement I is true; Statement II is false.
(c) f (x) = x cos x (d) Statement I is false; Statement II is true.
(d) None of the above
10. Let F (x) be an indefinite integral of sin 2 x.
Objective Question II Statement I The function F (x) satisfies
(One or more than one correct option) F (x + π ) = F (x) for all real x.
b−x Because
9. Let f : (0, 1) → R be defined by f (x) = , where b is a
1 − bx Statement II sin 2(x + π ) = sin 2 x, for all real x.
constant such that 0 < b < 1. Then, (2011) (2007, 3M)

(a) f is not invertible on (0, 1)


1 Analytical & Descriptive Question
(b) f ≠ f −1 on (0, 1) and f ′ (b) =
f ′ (0)
11. Let f be a one-one function with domain { x, y, z } and
1
(c) f = f −1 on (0, 1) and f ′ (b) = range {1, 2, 3}. It is given that exactly one of the
f ′ (0)
−1
following statements is true and the remaining two
(d) f is differentiable on (0, 1) are false f (x) = 1, f ( y) ≠ 1, f (z ) ≠ 2 determine f −1 (1).
(1982, 2M)
Assertion and Reason
12. If f is an even function defined on the interval (− 5, 5),
For the following questions, choose the correct answer
from the codes (a), (b), (c) and (d) defined as follows. then four real values of x satisfying the equation
 x + 1
(a) Statement I is true, Statement II is also true; f (x) = f   are ………. .
Statement II is the correct explanation of Statement I.  x + 2 (1996, 1M)

Answers
Topic 1 Topic 3
1. (a) 2. (c) 3. (d) 4. (a) 1. (c) 2. (c) 3. (d) 4. (a)
5. (c) 6. (d) 7. (d) 8. (d) 5. (a) 6. (b) 7. (a) 8. (c)
9. (c) 10. (c) 11. A→ p; B→ q; C→ q; D→ p 9. (b) 10. (d) 11. (b) 12. (a)
12. (a,d) 13. (–2,1) 14. Domain ∈ [ −2,−1 ] ∪ [1, 2 ] 13. (a) 14. (d) 15. A → q; B → r
 3   π π  3π π  16. (a, b, c)
15. 0, 16. True 17. t ∈ − , ∪ ,
 2   2 10   10 2  17. y = x + 1 and y = − x + 1
18. x ∈ [ −1, 2 ) ∪ [3, ∞ ) 19. (8) 18. True
Topic 2 19. 2 ≤ α ≤ 14, No
1. (d) 2. (b) 3. (c) 4. (c)
Topic 4
5. (a) 6. (b) 7. (a) 8. (b)
1. (c) 2. (c) 3. (b) 4. (d)
9. (d) 10. (b) 11. (a) 12. (d)
5. (a) 6. (b) 7. (b) 8. (a)
13. (d) 14. (a,b,c) 15. (a, c) 16. (b, c)
9. (b) 10. (d)
17. (a, d) 18. 1 19. True 20. (a = 3)  ± 3 ± 5
−1
11. f (1 ) = y 12.  
21. (19)  2 
Hints & Solutions
Topic 1 Classification of Functions,  – π π –1 1
sin   ≤ 2x ≤ sin ⇒ ≤ 2x ≤
Domain and Range  6  2 2 2
1. Given relation –1 1
≤x≤
R = {(x, y) : x, y ∈ Z , x2 + 3 y2 ≤ 8} 4 2
– 1 1
For, y2 = 0, x2 = 0, 1, 4 Q x∈ ,
 4 2 
For, y2 = 1, x2 = 0, 1, 4
For, y2 = 4, x2 ∈ φ x2 + x + 2
5. Let y = f (x) = , x ∈R
∴ Range of R is possible values of y = { − 1, 0, 1} x2 + x + 1
∴ Domain of R−1 = Range of R = { − 1, 0, 1} x2 + x + 2
∴ y=
1 x2 + x + 1
2. Given function f (x) = + log10 (x3 − x)
4 − x2 y=1 + 2
1
[i.e. y > 1] …(i)
For domain of f (x) x + x+1
4 − x2 ≠ 0 ⇒ x ≠ ± 2 …(i) ⇒ yx2 + yx + y = x2 + x + 2
and x3 − x > 0 ⇒ x ( y − 1) + x ( y − 1) + ( y − 2) = 0, ∀ x ∈ R
2
⇒ x(x − 1)(x + 1) > 0 Since, x is real, D ≥ 0
From Wavy curve method, ⇒ ( y − 1 )2 − 4 ( y − 1 ) ( y − 2 ) ≥ 0
⇒ ( y − 1) {( y − 1) − 4 ( y − 2)} ≥ 0
+ + ⇒ ( y − 1) (− 3 y + 7) ≥ 0
–∞ – +∞ 7
–1 0 – +1 ⇒ 1≤ y≤ …(ii)
3
 7
From Eqs. (i) and (ii), Range ∈ 1 ,
x ∈ (−1, 0) ∪ (1, ∞ ) …(ii)  3 
From Eqs. (i) and (ii), we get the domain of f (x) as
(−1, 0) ∪ (1, 2) ∪ (2, ∞ ).
6. Given, f (x) = (1 + b2) x2 + 2bx + 1
2
3. Given, function f (x) = a x , a > 0 is written as sum of an  b  b2
= (1 + b2)  x +  +1−
even and odd functions f1 (x) and f2(x) respectively.  1+ b 
2
1 + b2
a x + a −x a x − a −x 1
Clearly, f1 (x) = and f2(x) = m (b) = minimum value of f (x) = is positive
2 2 1 + b2
So, f1 (x + y) + f1 (x − y) and m (b) varies from 1 to 0, so range = (0, 1]
1 1 log (x + 3) log 2 (x + 3)
= [a x + y + a − ( x + y ) ] + [a x − y + a − ( x − y ) ] 7. Given, f (x) = 2 2 =
2 2 (x + 3x + 2) (x + 1) (x + 2)
1  1 a x
ay  For numerator, x + 3 > 0
= a x a y + x y + y + x 
2 a a a a  ⇒ x> −3 …(i)
1  1 1  1  and for denominator, (x + 1) (x + 2) ≠ 0
= a x  a y + y  + x  y + a y  
2  a  a a 
⇒ x ≠ − 1, − 2 …(ii)
1 1  1 From Eqs. (i) and (ii),
= ax + x  ay + y 
2 a  a  Domain is (− 3 , ∞ ) /{ − 1, − 2}
 a x + a −x   a y + a −y  8. Given, 2x + 2y = 2, ∀ x , y ∈ R
=2   = 2 f1 (x) ⋅ f1 ( y)
 2  2  But 2x , 2y > 0, ∀ x , y ∈ R
π Therefore, 2x = 2 − 2y < 2 ⇒ 0 < 2x < 2
4. Here, f (x) = sin −1 (2x) + , to find domain we must
6 Taking log on both sides with base 2, we get
have,
log 2 0 < log 2 2x < log 2 2 ⇒ − ∞ < x < 1
−1 π  π π
sin (2x) + ≥ 0 but − ≤ sin −1 θ ≤
6  2 2  9. It is given,
π π f (θ ) = sin θ (sin θ + sin 3 θ )
− ≤ sin −1 (2x) ≤
6 2 = (sin θ + 3 sin θ − 4 sin3 θ ) sin θ
178 Functions

= (4 sin θ − 4 sin3 θ ) sin θ = sin 2 θ (4 − 4 sin 2 θ ) 1 x2


⇒ ≤ ≤2
= 4 sin 2 θ cos 2 θ = (2 sin θ cos θ )2 2 2
= (sin 2θ )2 ≥ 0 ⇒ 1 ≤ x2 ≤ 4
which is true for all θ. ⇒ 1 ≤ |x| ≤ 2
10. For domain of y, ⇒ Domain ∈ [−2, − 1] ∪ [1, 2]
1 − x > 0, 1 − x ≠ 1 and x+ 2 >0 π2
⇒ x < 1, x ≠ 0 and x > −2
15. Given, f (x) = 3 sin − x2
16
⇒ −2 < x < 1 excluding 0  π π
⇒ Domain ∈ − ,
⇒ x ∈ (−2, 1) − {0}  4 4 
(x − 1) (x − 5)
11. Given, f (x) =  π2  1(−2x)
(x − 2) (x − 3) ∴ For range, f ′ (x) = 3 cos  − x2  ⋅ =0
 16 
  π2
The graph of f (x) is shown as : 2 − x2
16
Y
 π2 
Where, cos  − x2  = 0 or x=0
 16 
 
y=1
  π2  π2 π2
X' X neglecting cos  − x2  = 0 ⇒

− x2 = 
0 1 2 3 5   16  16 4
 3π 2 
⇒ x = −
2
, never possible 
 16 
Y'
⇒ x =0
A. If − 1 < x < 1 ⇒ 0 < f (x) < 1
π 3
B. If 1 < x < 2 ⇒ f (x) < 0 Thus, f (0) = 3 sin =
4 2
C. If 3 < x < 5 ⇒ f (x) < 0  π  π
and f −  = f   =0
D. If x > 5 ⇒ 0 < f (x) < 1  4  4
2x − 1  3 
12. Since, >0 Hence, range ∈ 0,
2 x + 3 x2 + x
3
 2 
(2x − 1)
⇒ >0 16. Since, domains of f1 (x) and f2(x) are D1 and D2 .
x(2x2 + 3x + 1)
Thus, domain of [ f1 (x) + f2(x)] is D1 ∩ D2 .
(2x − 1)
⇒ >0 Hence, given statement is true.
x (2x + 1) (x + 1)
− − 1 − 2 x + 5 x2  π π
+ + + 17. Given, 2 sin t = ,t ∈ − ,
 2 2 
−∞
−1 −1/2 0 1/2
∞ 3 x2 − 2 x − 1
Put 2 sin t = y ⇒ − 2 ≤ y ≤ 2
Hence, the solution set is,
1 − 2 x + 5 x2
x ∈ (−∞ , − 1) ∪ (−1/2, 0) ∪ (1 / 2, ∞ ) ∴ y=
3 x2 − 2 x − 1
Hence, (a) and (d) are the correct options.
 4 − x2  ⇒ (3 y − 5)x2 − 2x( y − 1) − ( y + 1) = 0
13. Given, f (x) = sin log   Since, x ∈ R − {1, − 1 / 3}
 1−x 
  [as, 3x2 − 2x − 1 ≠ 0 ⇒ (x − 1)(x + 1 / 3) ≠ 0]
4−x 2
∴ D ≥0
For domain, > 0 , 4 − x2 > 0 and 1 − x ≠ 0
1−x ⇒ 4( y − 1) + 4(3 y − 5) ( y + 1) ≥ 0
2

⇒ (1 − x) > 0 and 4 − x >02 ⇒ y2 − y − 1 ≥ 0


2
⇒ x<1 and | x| < 2 ⇒ −2 < x < 1  1 5
⇒ y−  − ≥0
 2 4
Thus, domain ∈ (−2, 1).
 x2   1 5  1 5
14. Given, f (x) = sin −1  log 2  ⇒ y− −  y− +  ≥0
 2  2 2  2 2

x2 1− 5
For domain, −1 ≤ log 2 ≤ 1 ⇒ y≤
2 2
Functions 179

1+ 5 Topic 2 Composite of Functions and


or y≥
2
Even, Odd Functions
1− 5
⇒ 2 sin t ≤ 1. For a given function f : R − { −a } → R defined by
2
a−x
1+ 5 f (x) = . Q ( fof ) (x) = x
or 2 sin t ≥ a+x
2
a−x
 π a−
⇒ sin t ≤ sin  −   a − x a+x
 10 ⇒ f  =x ⇒ =x
 a + x a−x
 3π  a+
or sin t ≥ sin   a+x
 10 
a 2 + ax − a + x
π 3π ⇒ =x
⇒ t≤− or t≥ a 2 + ax + a − x
10 10
⇒ a 2 + ax − a + x = a 2x + ax2 + ax − x2
 π π  3π π 
Hence, range of t is − , − ∪ , . ⇒ a (a − 1) = (a 2 − 1)x + x2(a − 1)
 2 10   10 2 
⇒ (a − 1) [x2 + (a + 1)x − a ] = 0 ⇒ a = 1
(x + 1) (x − 3) 1−x  1 1 + (1 /2)
18. Since, y = takes all real values only So, f (x) = ∴ f −  = =3
(x − 2) 1+ x  2 1 − (1 /2)
(x + 1) (x − 3)
when ≥0 2. Given, for x ∈ (0, 3 / 2), functions
(x − 2)
f (x) = x … (i)
− + − +
g (x) = tan x … (ii)
−∞ ∞
−1 2 3
1 − x2
and h (x) = … (iii)
⇒ −1 ≤ x < 2 or x≥3 1 + x2
∴ x ∈ [−1, 2) ∪ [3, ∞ ). Also given, φ(x) = ((hof )og )(x) = (hof ) ( g (x))
19. The limit = h ( f ( g (x))) = h ( f (tan x))
4 2 (sin 3x + sin x) 1 − ( tan x )2
lim = h ( tan x ) =
π  3x 5x  3x 1 + ( tan x )2
x→
2 sin 2x sin + cos  −  2 + 2 cos 2x + cos 
2
 2 2  2 1 − tan x π 
= = tan  − x
4 2 (2 sin 2x cos x) 1 + tan x 4 
= lim
π 3x  5x 3x  π  π π
x→ 2 sin 2x sin +  cos − cos  − 2 (1 + cos 2x) Now, φ   = tan  − 
2
2  2 2  3  4 3
 3π − 4π   π
= lim
8 2 sin 2x cos x = tan   = tan  − 
π 3x x  12   12
x→
2
2 sin 2x sin − 2 sin 2x sin − 2 (2 cos 2 x) π  π  11π 
2 2 = − tan   = tan  π −  = tan  
 12  12  12 
4 2 sin 2x cos x
= lim
π  3x x 3. Given, functions f (x) = x2, x ∈ R
x → sin 2 x sin
 − sin  − 2 cos 2 x
2
 2 2 and g ( A ) = { x ∈ R : f (x) ∈ A }; A ⊆ R
4 2 sin 2x cos x Now, for S = [0, 4]
= lim g (S ) = { x ∈ R : f (x) ∈ S = [0, 4]}
π x
x→
2
2 sin 2x cos x sin − 2 cos 2 x = { x ∈ R : x2 ∈ [0, 4]}
2
4 2 sin 2x = { x ∈ R: x ∈ [−2, 2]}
= lim ⇒ g (S ) = [−2, 2]
π x
x → 2 sin 2 x sin
2
− 2 cos x So, f ( g (S )) = [0, 4] = S
2
Now, f (S ) = { x2 : x ∈ S = [0, 4]} = [0, 16]
8 2 sin x
= lim and g ( f (S )) = { x ∈ R : f (x) ∈ f (S ) = [0, 16]}
π x
x → 4 sin x sin − 2
2
2 = { x ∈ R : f (x) ∈ [0, 16]}
8 2 16 = { x ∈ R: x2 ∈ [0, 16]}
= = = 8.
4 4 −2 = { x ∈ R : x ∈ [−4, 4]} = [−4 ,4]
− 2
2 From above, it is clear that g ( f (S )) = g (S ).
180 Functions
4. Given, f (x + y) = f (x) ⋅ f ( y) −1
 1
Let f (x) = λ x
[where λ > 0] J (X ) = X Qx=
1  X 
Q f (1) = 2 (given) 1−
X
∴ λ =2 −1 1
10 10  10  = =
So, Σ f (a + k) = Σ λa+ k = λa  Σ λk X −1 1 − X
k =1 k =1  k=1 
⇒ J (X ) = f3 (X ) or J (x) = f3 (x)
= 2a [21 + 22 + 23 + ......+210 ]
7. We have, f (x) = ax2 + bx + c
2(2 − 1) 
10
= 2a   Now, f (x + y) = f (x) + f ( y) + xy
 2 −1  Put y = 0 ⇒ f (x) = f (x) + f (0) + 0
[by using formula of sum of n-terms of a GP having ⇒ f (0) = 0
first term ‘a’ and common ratio ‘r’, is ⇒ c=0
a (r n − 1) 
Sn = , where r > 1 Again, put y = − x
r −1  ∴ f (0) = f (x) + f (− x) − x2
⇒ 2a+ 1 (210 − 1) = 16 (210 − 1) (given) ⇒ 0 = ax2 + bx + ax2 − bx − x2
⇒ 2a+ 1 = 16 = 24 ⇒ a + 1 = 4 ⇒ a = 3 ⇒ 2ax2 − x2 = 0
 1 − x 1
5. Given, f (x) = log e   ,|x| < 1, then ⇒ a=
 1 + x 2
Also, a + b + c = 3
 2x 
 2x  1 −    2x   ⇒
1
+ b + 0 =3⇒ b =
5
1 + x2 
f  = log e  Q  < 1 2 2
1 + x 
2
 1 + 2x   1 + x  
2
  x2 + 5 x
 1 + x2  ∴ f (x) =
2
 1 + x2 − 2 x 
  n 2 + 5n 1 2 5
1 + x2   (1 − x)2   1 − x
2
Now, f (n ) = = n + n
= log e  = log   = log   2 2 2
 1 + x2 + 2x  e
 (1 + x)2
e
 1 + x
  10
1 10
5 10
 1+ x 2
 ∴ ∑ f (n ) =
2 ∑ n2 + 2 ∑n
n =1 n =1 n =1
 1 − x
= 2 log e   [Q log e| A|m = m log e| A|] 1 10 × 11 × 21 5 10 × 11
 1 + x = ⋅ + ×
2 6 2 2
  1 − x 
= 2 f (x) Q f (x) = log e  1 + x  385 275 660
= + = = 330
  2 2 2
6. We have, 8. f (x) = x2, g (x) = sin x
1 1
f1 (x) = , f2(x) = 1 − x and f3 (x) = ( gof )(x) = sin x2
x 1−x
Also, we have ( f2 o J o f1 )(x) = f3 (x) go( gof ) (x) = sin (sin x2)
⇒ f2((J o f1 )(x)) = f3 (x) ( fogogof ) (x) = (sin (sin x2))2 …(i)
⇒ f2(J ( f1 (x)) = f3 (x) Again, ( gof ) (x) = sin x2
1 ( gogof ) (x) = sin (sin x2) …(ii)
⇒ 1 − J ( f1 (x)) =
1−x
Given, ( fogogof ) (x) = ( gogof ) (x)
1
[Q f2(x) = 1 − x and f3 (x) = ] ⇒ (sin (sin x2))2 = sin (sin x2)
1−x
⇒ sin (sin x2) {sin (sin x2) − 1} = 0
 1 1 1
⇒ 1 − J  = [Q f1 (x) = ]
 x 1 − x x ⇒ sin (sin x2) = 0 or sin (sin x2) = 1
 1 π
⇒ J  = 1 −
1 ⇒ sin x2 = 0 or sin x2 =
 x 1−x 2
1 − x−1 −x ∴ x2 = n π
= = π
1−x 1−x [sin x2 = is not possible as − 1 ≤ sin θ ≤ 1]
2
1
Now, put = X , then x=± nπ
x
Functions 181

αx f (|x|) = ||x| − 1|
9. Given, f (x) =
x+1 and | f (x)| = ||x − 1|| = |x − 1|
 αx  ∴ f (|x|) ≠| f (x)|, hence (c) is false.
α 
 αx   x + 1
π π 
f [ f (x) ] = f   =
 x + 1 αx 14. f (x) = sin  sin  sin x  , x ∈ R
+1 6 2 
x+1
π   π π π
α 2x = sin  sin θ , θ ∈ − , , where θ = sin x
6   2 2  2
x+1 α 2x
= = = x [given] …(i)  π π π
α x + (x + 1) (α + 1) x + 1 = sin α, α ∈ − , ,where α = sin θ
 6 6  6
x+1
 1 1
⇒ α x = (α + 1) x2 + x
2 ∴ f (x) ∈ − ,
 2 2 
⇒ x [α 2 − (α + 1) x − 1] = 0
 1 1
⇒ x(α + 1)(α − 1 − x) = 0 Hence, range of f (x) ∈ − ,
 2 2 
⇒ α − 1 = 0 and α + 1 = 0
So, option (a) is correct.
⇒ α = −1
 π π  1 1
But α = 1 does not satisfy the Eq. (i). (b) f { g (x)} = f (t ), t ∈ − , ⇒ f (t ) ∈ − ,
 2 2   2 2 
∴ Option (b) is correct.
10. g (x) = 1 + x − [x] is greater than 1 π π 
sin  sin  sin x 
f (x)  6 2 
since x − [x] > 0 (c) lim = lim
x → 0 g (x) x→ 0 π
f [ g (x)] = 1, since f (x) = 1 for all x > 0 (sin x)
2
11. Let f (x) = sin 2 x and g (x) = x π π  π π 
sin  sin  sin x  sin  sin x
Now, fog (x) = f [ g (x)] = f ( x ) = sin 2 x  6  2  6 2 
= lim ⋅
x→ 0 π π  π 
and gof (x) = g [ f (x)] = g (sin 2 x) = sin 2 x = |sin x| sin  sin x  sin x
6 2  2 
Again, let f (x) = sin x , g (x) = | x| π π
=1 × ×1 =
fog (x) = f [ g (x)] = f (| x|) 6 6
= sin| x|≠ (sin x )2 ∴Option (c) is correct.
When f (x) = x2, g (x) = sin x (d) g{ f (x)} = 1
π
fog (x) = f [ g (x)] = f (sin x ) = (sin x )2 ⇒ sin { f (x)} = 1
2
and ( gof ) (x) = g [ f (x)] = g (x2) = sin x2 ⇒ sin { f (x)} =
2
...(i)
π
= sin| x|≠|sin x|
 1 1  π π 
But f (x) ∈ − , ⊂ − ,
12. Given, f (x) = cos (log x)  2 2   6 6 
1   x   1 1
∴ f (x) ⋅ f ( y) − f   + f (xy) ∴ sin { f (x)} ∈ − , ...(ii)
 2 2 
2   y 
2
1 ⇒ sin { f (x)} ≠ , [from Eqs. (i) and (ii)]
= cos (log x) ⋅ cos(log y) − [cos (log x − log y) π
2
+ cos(log x + log y)] i.e. No solution.
1 ∴ Option (d) is not correct.
= cos (log x) ⋅ cos (log y) − [(2 cos (log x) ⋅ cos (log y)]
2 15. Since, f (x) = cos [π 2] x + cos [−π 2] x
= cos (log x) ⋅ cos (log y) − cos (log x) ⋅ cos (log y) = 0
⇒ f (x) = cos (9) x + cos (−10) x
13. Given, f (x) = |x − 1| [using [π 2] = 9 and [− π 2] = − 10]
∴ f (x ) = |x − 1|
2 2  π 9π
∴ f   = cos + cos 5π = − 1
 2 2
and { f (x)}2 = (x − 1)2
⇒ f (x2) ≠ ( f (x))2, hence (a) is false. f (π ) = cos 9π + cos 10π = − 1 + 1 = 0
Also, f (x + y) = |x + y − 1| f (− π ) = cos 9π + cos 10π = − 1 + 1 = 0
f (x) = |x − 1|,  π 9π 10π 1 1
and f   = cos + cos = +0=
f ( y) = | y − 1|  4 4 4 2 2
⇒ f (x + y) ≠ f (x) + f ( y), hence (b) is false. Hence, (a) and (c) are correct options.
182 Functions

3  1  π
16. Since, area of equilateral triangle = (BC )2 = 2 sin (2x + π / 3) ⋅ − sin 2x +  = 0
4  2   3
3 3 ⇒ f (x) = c, where c is a constant.
⇒ = ⋅ [x2 + g 2(x)] ⇒ g 2(x) = 1 − x2
4 4 But f (0) = sin 2 0 + sin 2(π / 3) + cos 0 cos π / 3
A 2
 3 1 3 1 5
=  + = + =
 2 2 4 2 4
Therefore, ( gof ) (x) = g [ f (x)] = g(5 / 4) = 1
B C 19. Given, f (x) = (a − xn )1/ n
(0,0) (x,g(x))
⇒ f [ f (x)] = [a − {(a − xn )1/ n }n ]1/ n = (xn )1/ n = x
⇒ g (x) = 1 − x2 or − 1 − x2 ∴ f [ f (x)] = x
Hence, given statement is true.
Hence, (b) and (c) are the correct options.
x+2 20. Let f (n ) = 2n for all positive integers n.
17. Given , y = f (x) =
x−1 Now, for n = 1, f (1) = 2 = 2 !
⇒ yx − y = x + 2 ⇒ x( y − 1) = y + 2 ⇒ It is true for n = 1.
y+2 Again, let f (k) is true.
⇒ x= ⇒ x = f ( y)
y−1 ⇒ f (k) = 2k, for some k ∈ N .
Here, f (1) does not exist, so domain ∈ R − {1} Again, f (k + 1) = f (k) ⋅ f (1) [by definition]
dy (x − 1) ⋅ 1 − (x + 2) ⋅ 1 3 =2 ⋅2
k
[from induction assumption]
= =−
dx (x − 1)2 (x − 1)2 = 2k + 1
⇒ f (x) is decreasing for all x ∈ R − {1}. Therefore, the result is true for n = k + 1. Hence, by
Also, f is rational function of x. principle of mathematical induction,
Hence, (a) and (d) are correct options. f (n ) = 2n , ∀ n ∈ N
n n n
18. f (x) = sin 2 x + sin 2(x + π / 3) + cos x cos (x + π / 3) Now, ∑ f (a + k) = ∑ f (a ) f (k) = f (a ) ∑ 2k
k =1 k =1 k =1
⇒ f (x) = sin 2 x + (sin x cos π / 3 + cos x sin π / 3)2
2 (2 − 1)
n
+ cos x cos (x + π / 3) = f (a ) ⋅
2 2 −1
 sin x ⋅ 1 cos x 3 
.
⇒ f (x) = sin 2 x +  +  = 2a ⋅ 2 (2n − 1) = 2a + 1 (2n − 1)
 2 2  n

+ cos x (cos x cos π / 3 − sin x sin π / 3) But ∑ f (a + k) = 16 (2n − 1) = 24 (2n − 1)


k =1
sin 2 x 3 cos 2 x 2 ⋅ 3
⇒ f (x) = sin 2 x + + + sin x cos x Therefore, a + 1 =4 ⇒ a =3
4 4 4
21. The given function f : [0, 1] → R be define by
cos 2 x 3
+ − cos x sin x ⋅ 4x 41 − x 2
2 2 f (x) = ⇒ f (1 − x) = =
1−x
sin 2 x 3 cos 2 x cos 2 x 4 +2
x
4 + 2 2 + 4x
= sin 2 x + + +
4 4 2 4x 2 4x + 2
5 5 5 ∴ f (x) + f (1 − x) = + =
= sin x + cos x =
2 2 4x + 2 2 + 4x 4x + 2
4 4 4
So, f (x) + f (1 − x) = 1 … (i)
and gof (x) = g { f (x)} = g (5 / 4) = 1
 
1  2 3  39  1
∴ f  + f  + f  +… + f  − f 
Alternate Solution  40  40  40  40  2
f (x) = sin x + sin (x + π / 3) + cos x cos (x + π / 3)
2 2
 1  39   2  38 
= f   + f    +  f  40 + f  40 
⇒ f ′ (x) = 2 sin x cos x + 2 sin (x + π / 3) cos (x + π / 3)   40   40   
− sin x cos (x + π / 3) − cos x sin (x + π / 3)   18  22    19  21 
+ … + f   + f    + f   + f   
= sin 2x + sin (2x + 2π / 3) − [sin (x + x + π / 3)]   40  40    40  40 
 2x + 2x + 2π / 3  2x − 2x − 2π / 3  
20  1
= 2 sin   ⋅ cos   + f  − f 
 2   2   40  2
 1  1
− sin (2x + π / 3) = {1 + 1 + K + 1 + 1} + f   − f   {from Eq. (i)}
 2  2
= 2 [sin (2x + π / 3) ⋅ cos π / 3] − sin (2x + π / 3)
19 − times = 19.
Functions 183

Topic 3 Types of Functions Clearly, f (x) is not injective because if f (x) < 1, then f is
many one, as shown in figure.
1. The given function f : R → R is
Also, f (x) is not surjective because range of f (x) is [0, ∞ [
f (x) = |x|(x − sin x) … (i) and but in problem co-domain is (0, ∞ ), which is wrong.
Q The function ‘f’ is a odd and continuous function ∴ f (x) is neither injective nor surjective
and as lim f (x) = ∞ and lim f (x) = − ∞, so range is R, 4. According to given information, we have if
x→ ∞ x→ −∞
therefore k ∈{4, 8, 12, 16, 20}
‘f’ is a onto function. Then, f (k) ∈ {3, 6, 9, 12, 15, 18}
 x(x − sin x), x ≥ 0 [Q Codomain ( f ) = {1, 2, 3, …, 20}]
Q f (x) = 
− x (x − sin x), x < 0 Now, we need to assign the value of f (k) for
 2x − sin x − x cos x, x > 0 k ∈{4, 8, 12, 16, 20} this can be done in 6C5 ⋅ 5 ! ways
∴ f ′ (x) = 
− 2x + sin x + x cos x, x < 0 = 6 ⋅ 5 ! = 6 ! and remaining 15 element can be associated
by 15 ! ways.
 (x − sin x) + x(1 − cos x), x > 0
(− x + sin x) − x(1 − cos x), x < 0 ∴Total number of onto functions = = 15 ! 6 !

x
Q for x > 0, x − sin x > 0 and x (1 − cos x) > 0 5. We have, f (x) = , x ∈R
1 + x2
∴ f ′ (x) > 0 ∀ x ∈ (0, ∞ )
Ist Method f (x) is an odd function and maximum
⇒ f is strictly increasing function, ∀ x ∈ (0, ∞ ).
occur at x = 1
Similarly, for x < 0, − x + sin x > 0 Y
(1, 1/2) 1
and (− x) (1 − cos x) > 0, therefore, f ′ (x) > 0 ∀ x ∈ (− ∞ , 0) y=
2
⇒ f is strictly increasing function, ∀ x ∈ (0, ∞ )
–1
X
Therefore ‘f’ is a strictly increasing function for x ∈ R O1
and it implies that f is one-one function. y=–
1
2
2. Given, function f : R – {1, − 1} → A defined as (–1, 1/2)
x2  1 1
f (x) = =y From the graph it is clear that range of f (x) is − ,
(let)  2 2 
1 − x2
⇒ x2 = y(1 − x2) [Q x2 ≠ 1] 1
y IInd Method f (x) =
⇒ x (1 + y) = y ⇒ x =
2 2
[provided y ≠ −1] 1
x+
1+ y x
y 1
Q x2 ≥ 0 ⇒ ≥ 0 ⇒ y ∈ (−∞ , − 1) ∪ [0, ∞ ) If x > 0, then by AM ≥ GM, we get x + ≥2
1+ y x
Since, for surjective function, range of f = codomain 1 1 1
⇒ ≤ ⇒ 0 < f (x) ≤
∴ Set A should be R − [−1, 0). 1 2 2
x+
 (x − 1) x
|x − 1| − x , if 0 < x ≤ 1 If x < 0, then by AM ≥ GM, we get x +
1
≤ −2
3. We have, f (x) = =
x x−1 x
 , if x > 1 1 1 1
 x ⇒ ≥− ⇒– ≤ f (x) < 0
1 2 2
1 x+
− 1, if 0 < x ≤ 1
 x
= x
1 0 1 1
1 − , if x > 1 If x = 0, then f (x) = = 0. Thus, − ≤ f (x) ≤
 x 1+0 2 2
Now, let us draw the graph of y = f (x)  1 1
Hence, f (x) ∈ − ,
Note that when x → 0, then f (x) → ∞, when x = 1, then  2 2 
f (x) = 0, and when x → ∞, then f (x) → 1 IIIrd Method
Y x
Let y = ⇒ yx2 − x + y = 0
1 + x2

x=0 Q x ∈ R, so D ≥ 0 ⇒ 1 − 4 y2 ≥ 0
 1 1
⇒ (1 − 2 y) (1 + 2 y) ≥ 0 ⇒ y ∈ − ,
y=1  2 2 
– + –
X –1/2
O 1 y=0 1/2
184 Functions

 1 1 As, x ∈R
So, range is − , .
 2 2  − 1 1
∴ (− 1) − 4 ( y)( y) ≥ 0 ⇒ 1 − 4 y2 ≥ 0 ⇒ y ∈
2
,
n + 1  2 2 
 , if n is odd
6. Given, f (n ) =  n 2 − 1 1
∴ Range = Codomain = ,
 , if n is even,  2 2 
2
So, f (x) is surjective.
 n + 1 , if n is odd
and g (n ) = n − (−1)n =  Hence, f (x) is surjective but not injective.
n − 1, if n is even PLAN To check nature of function.
+
9.
 f (n 1 ), if n is odd
Now, f ( g (n )) =  (i) One-one To check one-one, we must check whether
 f (n − 1), if n is even f ′ ( x )> 0 or f ′ ( x )< 0 in given domain.
n + 1 (ii) Onto To check onto, we must check
 , if n is odd Range = Codomain
=  n 2− 1 + 1 n
 = , if n is even Description of Situation To find range in given
 2 2 domain [a , b], put f ′ (x) = 0 and find x = α 1, α 2, …,
= f (x) α n ∈[a , b]
[Q if n is odd, then (n + 1) is even and Now, find { f (a ), f (α 1 ), f (α 2), K , f (α n ), f (b)}
if n is even, then (n − 1) is odd]
its greatest and least values gives you range.
Clearly, function is not one-one as f (2) = f (1) = 1
Now, f : [0, 3] → [1, 29]
But it is onto function.
f (x) = 2x3 − 15x2 + 36x + 1
[Q If m ∈ N (codomain) is odd, then 2m ∈ N (domain)
such that f (2m) = m and ∴ f ′ (x) = 6x2 − 30x + 36 = 6 (x2 − 5x + 6)
if m ∈ N codomain is even, then = 6 (x − 2) (x − 3)
2m − 1 ∈ N (domain) such that f (2m − 1) = m] + − +
∴Function is onto but not one-one 2 3
2x
7. We have a function f : A → R defined as, f (x) = For given domain [0, 3], f (x) is increasing as well as
x −1 decreasing ⇒ many-one
One-one Let x1, x2 ∈ A such that Now, put f ′ (x) = 0 ⇒ x = 2, 3
2x1 2x2
f (x1 ) = f (x2) ⇒ = Thus, for range f (0) = 1, f (2) = 29, f (3) = 28
x1 − 1 x2 − 1
⇒ Range ∈[1, 29]
⇒ 2x1x2 − 2x1 = 2x1x2 − 2x2 ⇒ x1 = x2
∴ Onto but not one-one.
Thus, f (x1 ) = f (x2) has only one solution, x1 = x2
 x, x ∈ Q
∴ f (x) is one-one (injective) 10. Let φ (x) = f (x) − g (x) = 
2 ×2 − x, x ∉ Q
Onto Let x = 2, then f (2) = =4 Now, to check one-one.
2 −1
Take any straight line parallel to X-axis which will
But x = 2 is not in the domain, and f (x) is one-one intersect φ(x) only at one point.
function
⇒ φ(x) is one-one.
∴f (x) can never be 4.
Similarly, f (x) can not take many values. To check onto
 x, x ∈ Q
Hence, f (x) is into (not surjective). As f (x) =  , which shows
∴f (x) is injective but not surjective. − x, x ∉ Q
x y = x and y = − x for rational and irrational values
8. We have, f (x) =
1 + x2 ⇒ y ∈ real numbers.
1 ∴ Range = Codomain ⇒ onto
 1 x Thus, f − g is one-one and onto.
∴ f  = x = = f (x)
 x 1 1 + x2
1+ 2 11. Given, f : [0, ∞ ) → [0, ∞ )
x
 1  1 Here, domain is [0, ∞ ) and codomain is [0, ∞ ). Thus, to
∴ f   = f (2)or f   = f (3)and so on.
 2  3 check one-one
x 1
So, f (x) is many-one function. Since, f (x) = ⇒ f ′ (x) = > 0, ∀ x ∈ [0, ∞ )
x 1+ x (1 + x)2
Again, let y = f (x) ⇒ y =
1 + x2 ∴ f (x) is increasing in its domain. Thus, f (x) is one-one
⇒ y + x2y = x ⇒ yx2 − x + y = 0 in its domain. To check onto (we find range)
Functions 185

x Y
Again, y = f (x) = ⇒ y + yx = x
1+ x y = 1 + 2x
y y
⇒ x= ⇒ ≥0
1− y 1− y
X′ X
−π O π
Since, x ≥ 0, therefore 0 ≤ y < 1 2 2
i.e. Range ≠ Codomain
∴ f (x) is one-one but not onto.
Y′
12. Given, f (x) = 2x + sin x It is clear from the graph that y = tan x is one-one and
⇒ f ' (x) = 2 + cos x ⇒ f ' (x) > 0 , ∀x ∈ R onto, therefore (B) → (r).
which shows f (x) is one-one, as f (x) is strictly increasing. 16. PLAN
Since, f (x) is increasing for every x ∈ R, (i) If f ′ ( x ) > 0, ∀x ∈ ( a, b ), then f( x ) is an increasing function in
( a, b ) and thus f( x ) is one-one function in ( a, b ) .
∴ f (x) takes all intermediate values between (−∞ , ∞ ).
(ii) If range of f( x ) = codomain of f( x ) , then f( x ) is an onto
Range of f (x) ∈ R. function.
Hence, f (x) is one-to-one and onto. (iii) A function f( x ) is said to be an odd function, if
f( − x ) = − f( x ), ∀x ∈ R, i.e.
13. The number of onto functions from f( − x ) + f( x ) = 0, ∀ x ∈ R
E = {1, 2, 3, 4} to F = {1, 2} f (x) = [ln (sec x + tan x)]3
= Total number of functions which map E to F 3 [ln (sec x + tan x)]2 (sec x tan x + sec 2x)
f ′ (x) =
− Number of functions for which map f (x) = 1 and (sec x + tan x)
f (x) = 2 for all x ∈ E = 24 − 2 = 14  −π π 
f ′ (x) = 3 sec x [ln (sec x + tan x)]2 > 0, ∀x ∈  , 
14. PLAN  2 2
(i) For such questions, we need to properly define the f (x) is an increasing function.
functions and then we draw their graphs.
∴ f (x) is an one-one function.
(ii) From the graphs, we can examine the function for continuity,
 π x  π π
differentiability, one-one and onto. (sec x + tan x) = tan  +  , as x ∈  − ,  , then
 4 2  2 2
− x, x < 0
f1 (x) =  x  π x
e , x ≥ 0 0 < tan  +  < ∞
 4 2
f2(x) = x2, x ≥ 0
0 < sec x + tan x < ∞ ⇒ − ∞ < ln (sec x + tan x) < ∞
sin x, x < 0
f3 (x) =  − ∞ < [ln (sec x + tan x)]3 < ∞ ⇒ −∞ < f (x) < ∞
x, x≥0
Range of f (x) is R and thus f (x) is an ont function.
 f ( f (x)), x<0 3
f4 (x) =  2 1   1 
 2 1
f ( f (x)) − 1 , x≥0 f (− x) = [ln (sec x − tan x)]3 = ln  
  sec x + tan x
 x2 , x < 0  x2 , x<0
Now, f2( f1 (x)) =  2x ⇒ f4 =  2x f (− x) = − [ln (sec x + tan x)] 3

e , x ≥ 0 e − 1 , x ≥ 0 f (x) + f (− x) = 0
2x, x<0 ⇒ f (x) is an odd function.
As f4 (x) is continuous, f ′ 4 (x) =  2x
2 e , x>0 17. Let y = ax + b and y = cx + d be two linear functions.
f4(x) When x = − 1, y = 0 and x = 1, y = 2 , then
0 = − a + b and a + b = 2 ⇒ a = b = 1
∴ y=x+1 ...(i)
x Again, when x = − 1, y = 2 and x = 1, y = 0, then
O
− c + d = 2 and c + d = 0
Graph for f4 (x)
⇒ d = 1 and c = − 1 ⇒ y = − x + 1 …(ii)
f4′ (0) is not defined. Its range is [0, ∞ ). Hence, two linear functions are y = x + 1 and y = − x + 1
Thus, range = codomain = [0, ∞ ), thus f4 is onto.
x2 + 4x + 30
Also, horizontal line (drawn parallel to X-axis) meets 18. Given, f (x) =
the curve more than once, thus function is not one-one. x2 − 8x + 18

15. y = 1 + 2x is linear function, therefore it is one-one and  (x2 − 8x + 18) (2x + 4) 


 
− (x2 + 4x + 30) (2x − 8)
its range is (− π + 1, π + 1). Therefore, (1 + 2x) is one-one ⇒ f ′ (x) = 
but not onto so (A) → (q). Again, see the figure. (x2 − 8x + 18)2
186 Functions

2 (−6x2 − 12x + 156) Topic 4 Inverse and Periodic Functions


=
(x2 − 8x + 18)2
1. Given function
−12 (x2 + 2x − 26) 82x − 8−2x 84x − 1
= f (x) = , x ∈ (− 1 , 1 ) = = y (let)
(x2 − 8x + 18)2 82x + 8−2x 84x + 1
which shows f ′ (x) is positive and negative both.
On applying componendo and dividendo law, we get
∴ f (x) is many one. 1+ y
84x =
Hence, given statement is true. 1− y
αx2 + 6x − 8 On applying logarithm having base ‘8’ both sides, we get
19. Let y =
α + 6 x − 8 x2  1 + y
4x = log 8  
⇒ αy + 6xy − 8x2y = αx2 + 6x − 8  1 − y
⇒ − αx2 − 8x2y + 6xy − 6x + αy + 8 = 0 1  1 + y 1  1 + y
⇒ x= log 8   = (log 8 e) log e  
⇒ αx2 + 8x2y − 6xy + 6x − αy − 8 = 0 4  1 − y 4  1 − y
⇒ x2 (α + 8 y) + 6x (1 − y) − (8 + αy) = 0 {by base change property of logarithm
Since, x is real. log a b = log a e ⋅ log e b}
⇒ B2 − 4 AC ≥ 0 By interchanging the variables x and y, we get the
⇒ 36 (1 − y) + 4 (α + 8 y) (8 + αy) ≥ 0
2
inverse function of f (x) and it is
⇒ 9 (1 − 2 y + y2) + [8α + (64 + α 2) y + 8 αy2] ≥ 0 1  1 + x
f −1 (x) = (log 8 e) log e  .
⇒ y2 (9 + 8α ) + y (46 + α 2) + 9 + 8α ≥ 0 …(i) 4  1 − x
⇒ A > 0, D ≤ 0, ⇒ 9 + 8 α > 0 Hence, option (c) is correct.
and (46 + α 2)2 − 4 (9 + 8α )2 ≤ 0 2. Since, only (c) satisfy given definition
⇒ α > − 9 /8 i.e. f { f −1 (B)} = B
and [46 + α − 2 (9 + 8α )][46 + α + 2 (9 + 8α )] ≤ 0
2 2
Only, if B ⊆ f (x)
⇒ α > − 9 /8 3. By definition of composition of function,
and (α 2 − 16α + 28) (α 2 + 16α + 64) ≤ 0 g ( f (x) ) = (sin x + cos x)2 − 1, is invertible (i.e. bijective)
⇒ α > − 9 /8 ⇒ g { f (x) } = sin 2x is bijective.
and [(α − 2) (α − 14)] (α + 8)2 ≤ 0 ⇒ α > − 9 /8  π π
We know, sin x is bijective, only when x ∈ − , .
and (α − 2) (α − 14) ≤ 0 [Q (α + 8)2 ≥ 0]  2 2 
⇒ α > − 9 / 8 and 2 ≤ α ≤ 14 ⇒ 2 ≤ α ≤ 14 π π
Thus, g { f (x) } is bijective, if − ≤ 2x ≤
αx2 + 6x − 8 2 2
Thus, f (x) = will be onto, if 2 ≤ α ≤ 14 π π
α + 6 x − 8 x2 ⇒ − ≤x≤
4 4
Again, when α = 3
4. It is only to find the inverse.
3 x2 + 6 x − 8
f (x) = , in this case f (x) = 0 Let y = f (x) = (x + 1)2, for x ≥ − 1
3 + 6 x − 8 x2
± y = x + 1, x≥ −1
⇒ 3 x2 + 6 x − 8 = 0 ⇒ y = x+1 ⇒ y ≥ 0, x + 1 ≥ 0
− 6 ± 36 + 96 − 6 ± 132 1 ⇒ x= y − 1 ⇒ f −1 ( y) = y −1
⇒ x= = = (− 3 ± 33 )
6 6 3 −1
⇒ f (x) = x − 1 ⇒ x≥0
This shows that 1 x2 + 1
1  1  5. Let y = x + ⇒ y= ⇒ xy = x2 + 1
f (− 3 + 33) = f (− 3 − 33) = 0 x x
3  3 
y ± y2 − 4
Therefore, f is not one-to-one. ⇒ x2 − xy + 1 = 0 ⇒ x =
2
20. Since, there is an injective mapping from A to B, each y± y2 − 4 x ± x2 − 4
element of A has unique image in B. ⇒ f −1 ( y) = ⇒ f −1 (x) =
2 2
Similarly, there is also an injective mapping from B to Since, the range of the inverse function is [1, ∞), then
A, each element of B has unique image in A or in other
words there is one to one onto mapping from A to B. x+ x2 − 4
we take f −1 (x) =
Thus, there is bijective mapping from A to B. 2
Functions 187

x − x2 − 4 1 − cos 2x
If we consider f −1 (x) = , then f −1 (x) > 1 10. Given, F (x) = ∫ sin 2 x dx = ∫ dx
2 2
1
This is possible only if (x − 2)2 > x2 − 4 F (x) =
(2x − sin 2x) + C ⇒ F (x + π ) ≠ F (x)
4
⇒ x2 + 4 − 4x > x2 − 4 ⇒ 8 > 4x
Hence, Statement I is false.
⇒ x < 2, where x > 2
But Statement II is true assin 2 x is periodic with period π.
Therefore, (a) is the answer.
11. It gives three cases
6. Let y = 2x ( x − 1), where y ≥ 1 as x ≥ 1
Case I When f (x) = 1 is true.
Taking log 2 on both sides, we get
In this case, remaining two are false.
log 2 y = log 2 2x ( x − 1) ⇒ log 2 y = x (x − 1)
∴ f ( y) = 1 and f (z ) = 2
⇒ x2 − x − log 2 y = 0
This means x and y have the same image, so f (x) is not
1 ± 1 + 4 log 2 y an injective, which is a contradiction.
⇒ x=
2
Case II When f ( y) ≠ 1 is true.
For y ≥ 1, log 2 y ≥ 0 ⇒ 4 log 2 y ≥ 0 ⇒ 1 + 4 log 2 y ≥ 1 If f ( y) ≠ 1 is true, then the remaining statements are
⇒ 1 + 4 log 2 y ≥ 1 ⇒ − 1 + 4 log 2 y ≤ − 1 false.
⇒ 1 − 1 + 4 log 2 y ≤ 0 ∴ f (x) ≠ 1 and f (z ) = 2
But x≥1 i.e. both x and y are not mapped to 1. So, either both
So, x = 1 − 1 + 4 log 2 y is not possible. associate to 2 or 3. Thus, it is not injective.
1 Case III When f (z ) ≠ 2 is true.
Therefore, we take x = (1 + 1 + 4 log 2y )
2 If f (z ) ≠ 2 is true, then remaining statements are false.
−1 1 ∴ If f (x) ≠ 1 and f ( y) = 1
⇒ f ( y) = (1 + 1 + 4 log 2 y )
2
But f is injective.
−1 1
⇒ f (x) = (1 + 1 + 4 log 2 x ) Thus, we have f (x) = 2, f ( y) = 1 and f (z ) = 3
2
Hence, f −1 (1) = y
7. Given, f (x) = 3x − 5 [given]
12. Since, f is an even function,
Let y = f (x) = 3x − 5 ⇒ y + 5 = 3x
then f (− x) = f (x), ∀ x ∈ (− 5, 5)
y+5 y+5 x+5
⇒ x= ⇒ f −1 ( y) = ⇒ f −1 (x) =  x + 1
3 3 3 Given , f (x) = f   …(i)
 x + 2
8. Clearly, f (x) = x − [x] = { x} which has period 1.
1  − x + 1
And sin , x cos x are non-periodic functions. ⇒ f (− x) = f  
x  − x + 2
b−x  − x + 1
9. Here, f (x) = , where 0 < b < 1, 0 < x < 1 ⇒ f ( x) = f   [Q f (− x) = f (x)]
1 − bx  − x + 2
For function to be invertible, it should be one-one onto. Taking f −1 on both sides, we get
∴ Check Range : −x+1
x= ⇒ − x2 + 2 x = − x + 1
b−x −x+2
Let f (x) = y ⇒ y=
1 − bx 3 ± 9 −4 3 ± 5
⇒ x2 − 3 x + 1 = 0 ⇒ x = =
⇒ y − bxy = b − x ⇒ x (1 − by) = b − y 2 2
b− y  x + 1
⇒ x= , where 0 < x < 1 Again, f (x) = f  
1 − by  x + 2
b− y b− y b− y  x + 1
∴ 0< <1 ⇒ > 0 and <1 ⇒ f (− x) = f  [Q f (− x) = f (x)]
1 − by 1 − by 1 − by 
 x + 2
1
⇒ y < b or y> …(i) Taking f −1 on both sides, we get
b x+1
(b − 1) ( y + 1) 1 −x= ⇒ x2 + 3 x + 1 = 0
<0−1 < y< …(ii) x+2
1 − by b
−3 ± 9 −4 −3 ± 5
From Eqs. (i) and (ii), we get ⇒ x= =
2 2
 1 ±3± 5
y ∈  − 1,  ⊂ Codomain Therefore, four values of x are .
 b 2
9
Limit, Continuity
and Differentiability
0 ∞ cot x − cos x
Topic 1 and Form 8. lim equals (2017 Main)
0 ∞ x → π/ 2

1
(π − 2x)3
1
(a) (b)
Objective Questions I (Only one correct option) 24 16
1 1
x + 2 sin x (c) (d)
1. lim is 8 4
x→ 0
x2 + 2 sin x + 1 − sin 2 x − x + 1 sin(π cos 2 x)
(2019 Main, 12 April II)
9. lim is equal to (2014 Main)
x→ 0 x2
(a) 6 (b) 2 (c) 3 (d) 1 π
(a) (b) 1 (c) − π (d) π
x2 − ax + b 2
2. If lim = 5, then a + b is equal to
x→1 x−1 (2019 Main, 10 April II) (1 – cos 2x)(3 + cos x)
10. lim is equal to (2013 Main)
x→ 0 x tan 4x
(a) − 4 (b) 1 (c) − 7 (d) 5
1
x4 − 1 x3 − k3 (a) 4 (b) 3 (c) 2 (d)
3. If lim = lim 2 , then k is 2
x→1 x − 1 x → k x − k2 (2019 Main, 10 April I)
 x2 + x + 1 
(a)
4
(b)
3
(c)
3
(d)
8 11. If lim  − ax − b = 4, then
3 8 2 3
x→ ∞  x+1  (2012)
2
sin x (a) a = 1, b = 4 (b) a = 1, b = − 4
4. lim equals (2019 Main, 8 April I) (c) a = 2, b = − 3 (d) a = 2, b = 3
x→ 0 2 − 1 + cos x
f (2h + 2 + h 2) − f (2)
(a) 4 2 (b) 2 12. lim , given that f ′ (2) = 6 and
h → 0 f (h − h 2 + 1 ) − f (1 )
(c) 2 2 (d) 4
f ′ (1) = 4 , (2003, 2M)
cot3 x − tan x (a) does not exist (b) is equal to −3/2
5. lim is
π  π (2019 Main, 12 Jan I) (c) is equal to 3/2 (d) is equal to 3
x→ cos  x + 
4
 4 {(a − n ) nx − tan x} sin nx
13. If lim = 0, where n is
(a) 4 2 (b) 4 (c) 8 (d) 8 2 x→ 0 x2
x cot(4x) non-zero real number, then a is equal to (2003, 2M)
6. lim is equal to (2019 Main, 11 Jan II) n+1 1
x → 0 sin 2 x cot 2 (2 x) (a) 0 (b) (c) n (d) n +
n n
(a) 0 (b) 1 (c) 4 (d) 2
(cos x − 1) (cos x − ex )
14. The integer n for which lim is a
1 + 1 + y4 − 2 x→ 0 xn
7. lim (2019 Main, 9 Jan I) finite non-zero number, is (2002, 2M)
y→ 0 y4
(a) 1 (b) 2
1
(a) exists and equals (c) 3 (d) 4
4 2
(b) does not exist x tan 2x − 2x tan x
15. lim is (1999, 2M)
(c) exists and equals
1 x→ 0 (1 − cos 2x)2
2 2 (a) 2 (b) −2
1 1 1
(d) exists and equals (c) (d) −
2 2 ( 2 + 1) 2 2
Limit, Continuity and Differentiability 189

1 − cos 2 (x − 1) x2
16. lim (1998, 2M) a − a 2 − x2 −
x→1 x −1 23. Let L = lim 4 , a > 0 . If L is finite, then
(a) exists and it equals 2 x→ 0 x4
(b) exists and it equals − 2 (a) a = 2
(c) does not exist because x − 1 → 0 (b) a = 1 (2009)
1
(d) does not exist because left hand limit is not equal to (c) L =
right hand limit 64
1
1 (d) L =
(1 − cos 2 x) 32
17. The value of lim 2 is (1991, 2M)
x→ 0 x
(a) 1 (b) −1
Fill in the Blanks
(c) 0 (d) None of these log (1 + 2h ) − 2 log (1 + h )
24. lim =K . (1997C, 2M)
sin[x] [x] ≠ 0 h→ 0 h2
 ,  
18. If f (x) =  [x]  
[x] ≠ 0 25. If f (x) = sin x, x ≠ nπ , n = 0, ± 1, ± 2, ... 
 0,
 2, other wise 
where, [x] denotes the greatest integer less than or
equal to x, then lim f (x) equals (1985, 2M)  x2 + 1 , x ≠ 0 , 2 
x→ 0  
(a) 1 (b) 0 and g (x) =  4, x = 0 , then lim g [f(x)] is ………
(c) −1 (d) None of these  5, x = 2 
x→0
 (1996, 2M)

1 2 n 
19. lim  + + ... +  is equal to (1984, 2M) 26. ABC is an isosceles triangle inscribed in a circle of radius
n → ∞  1 − n2 1 − n2 1 − n 2
r. If AB = AC and h is the altitude from A to BC, then the
1
(a) 0 (b) − ∆ABC has perimeter P = 2( 2hr − h 2 + 2hr ) and area
2
A
(c)
1
(d) None of these A = K . Also, lim 3 = K (1989, 2M)
h→ 0 P
2
20. If f (a ) = 2, f ′ (a ) = 1, g (a ) = − 1, g ′ (a ) = 2,  4  1 2 
g (x) f (a ) − g (a ) f (x)   x sin  x + x  
then the value of lim is (1983, 1M) 27. lim  = …
x→ a x−a x→ − ∞  (1 + |x|3 ) 
 
(a) − 5 (b)
1   (1987, 2M)
5
28. Let f (x) = (x + x − 16x + 20) / (x − 2) , if x ≠ 2
3 2 2
(c) 5 (d) None of these .
k , if x = 2
G (x) − G (1) If f (x) is continuous for all x, then k = … .
21. If G (x) = − 25 − x2, then lim has the value (1981, 2M)
x→1 x−1 (1983, 1M) πx
1 1 29. lim (1 − x) tan =….
(a) (b) x→1 2 (1978, 2M)
24 5
(c) − 24 (d) None of these True/False
Objective Questions II 30. If lim [ f (x) g (x)] exists, then both lim f (x) and
x→ a x→ a
(One or more than one correct option) lim g (x) exist. (1981, 2M)
x→ a
22. For any positive integer n, define fn : (0, ∞ ) → R as
n  1  Analytical & Descriptive Questions
fn (x) = Σ tan − 1   for all x ∈ (0, ∞ ).
j =1  1 + (x + j) (x + j − 1) ax −1 (1983, 3M)
(Here, the inverse trigonometric function tan − 1 x 31. Use the formula lim = log e a, to find
x→ 0 x
 π π 2x − 1
assumes values in  − ,  ). Then, which of the lim .
 2 2 x → 0 (1 + x)1/ 2 − 1
(1982, 2M)
following statement(s) is (are) TRUE? (2018 Adv.)
(a + h )2 sin (a + h ) − a 2 sin a
(a) ∑
5
tan 2 (f j (0)) = 55 32. Evaluate lim . (1980, 3M)
j =1 h→ 0 h
(b) ∑
10
j =1
(1+ f ′ j (0)) sec2 (f j (0)) = 10 x − sin x
33. Evaluate lim . (1979, 3M)
1 x→ 0 x + cos 2 x
(c) For any fixed positive integer n, lim tan(fn (x)) =
x →∞
x −1
34. Evaluate lim  
n
(d) For any fixed positive integer n, lim sec2 (fn (x)) = 1 . (1978, 3M)
x→1  2 x2 − 7 x + 5 
x →∞
190 Limit, Continuity and Differentiability

Integer & Numerical Answer Type Questions 36. Let m and n be two positive integers greater than 1. If
 ecos ( α n ) − e
x2 sin (βx) lim   = −  e , then the value of m is
35. Let α , β ∈ R be such that lim = 1 . Then, α→ 0  α m   2 n
x → 0 αx − sin x  
(2015 Adv.)
6 (α + β ) equals (2016 Adv.)

Topic 2 1∞ Form, RHL and LHL


Objective Questions I (Only one correct option) 8. Let p = lim+ (1 + tan 2 x )1/ 2x , then log p is equal to
x→ 0 (2016 Main)
1. Let f : R → R be a differentiable function satisfying
1 (a) 2 (b) 1
1 1
 1 + f (3 + x) − f (3) x (c) (d)
f ′ (3) + f ′ (2) = 0. Then lim  is equal 2 4
x→ 0  1 + f (2 − x) − f (2 ) 

to (2019 Main, 8 April II)


9. Let α (a ) and β (a ) be the roots of the equation
(a) e (b) e −1
(c) e 2
(d) 1 ( 3 1 + a − 1) x2 − ( 1 + a − 1) x + (6 1 + a − 1) = 0, where
a > − 1. Then, lim α (a ) and lim β (a ) are (2012)
π − 2 sin − 1 x a→ 0+ a→ 0 +
2. lim is equal to 5 1
x→1 − 1−x (2019 Main, 12 Jan II) (a) − and 1 (b) − and −1
2 2
π 2 1 7 9
(a) (b) (c) π (d) (c) − and 2 (d) − and 3
2 π 2π 2 2
3. Let [x] denote the greatest integer less than or equal to 1

x. Then, 10. If lim [1 + x log (1 + b2)] x = 2b sin 2 θ, b > 0


x→ 0
tan(π sin 2 x) + (|x| − sin(x[x]))2
lim and θ ∈ (− π , π ], then the value of θ is (2011)
x→ 0 x2 (2019 Main, 11 Jan I) π π
(a) ± (b) ±
(a) equals π (b) equals π + 1 4 3
(c) equals 0 (d) does not exist π π
(c) ± (d) ±
4. For each t ∈ R, let [t ] be the greatest integer less than or 6 2
 sin x 
11. For x > 0, lim (sin x)1/ x +  
equal to t. Then, 1
π   x  is (2006, 3M)
(1 − |x| + sin|1 − x|) sin  [1 − x] x→0
 
2 
lim (a) 0 (b) – 1
x → 1+ |1 − x|[1 − x] (2019 Main, 10 Jan I)
(c) 1 (d) 2
(a) equals 0 (b) does not exist
(c) equals − 1 (d) equals 1 12. Let f : R → R be such that f (1) = 3 and f ′ (1) = 6. Then,
1/ x
5. For each x ∈ R, let [x] be the greatest integer less than or  f (1 + x) 
lim equals (2002, 2M)
x→ 0
 f (1) 
equal to x. Then,
1
x([x] + |x|) sin [x]
lim is equal to (a) 1 (b) e 2
x → 0− |x| (2019 Main, 9 Jan II) (c) e2 (d) e3
(a) 0 (b) sin 1 x
x−3 
(c) − sin 1 (d) 1 13. For x ∈ R , lim   is equal to
x→ ∞ x + 2 
(2000, 2M)
6. For each t ∈R, let [t ] be the greatest integer less than or
(a) e (b) e−1
equal to t. Then,
(c) e−5 (d) e5
  1  2 15
lim x   +   + … +  
x→ 0 +  x  x  x  (2018 Main)
Fill in the Blanks
1/ x 2
(a) is equal to 0 (b) is equal to 15  1 + 5 x2 
(c) is equal to 120 (d) does not exist (in R) 14. lim   =K .
x → 0  1 + 3 x2 
(1996, 1M)
1 − x (1 + |1 − x|)  1 
7. Let f (x) = cos   x+ 4
|1 − x|  1 − x  x + 6
15. lim   = …… .
x → ∞ x + 1
for x ≠ 1. Then (1991, 2M)
(a) limx→ 1 + f (x) = 0
(b) limx→ 1 − f (x) does not exist
Analytical & Descriptive Question
1/ x
 π  
(c) limx→ 1 − f (x) = 0 16. Find lim tan  + x  .
(d) limx→ 1 + f (x) does not exist x→ 0 
 4   (1993, 2M)
Limit, Continuity and Differentiability 191

Integer & Numerical Answer Type Questions


17. Let e denote the base of the natural logarithm. The value of the real number a for which the right hand limit
1
(1 − x) x − e−1
lim is equal to a non zero real number, is ……… .
x→ 0 +
xa (2020 Adv.)
1−x
 − ax + sin(x − 1) + a 1− 1
18. The largest value of the non-negative integer a for which lim  
x
= is (2014 Adv.)
x→1  x + sin(x − 1 ) − 1  4

Topic 3 Squeeze, Newton-Leibnitz’s Theorem and Limit Based on


Converting infinite Series into Definite Integrals
Objective Questions I (Only one correct option) Objective Question II
1. If α and β are the roots of the equation (One more than one correct option)
n n x
375x2 − 25x − 2 = 0, then lim
n→ ∞
∑ α r + nlim
→∞
∑ βr is equal 
 n  n n
r =1 r =1  n n (x + n )  x +  ...  x +  
  2  n  ,
to (2019 Main, 12 April I) 4. Letf (x) = lim
n→ ∞   n 2  2 n 2 
21 29  n ! (x + n )  x +
2 2 2
 ...  x + 2 
(a) (b)
346 358   4  n  
1 7 for all x = 0. Then (2016 Adv.)
(c) (d)
(a) f   ≥ f (1) (b) f   ≤ f  
12 116 1 1 2
sec 2 x  2  3  3
2.

lim 2
f (t ) dt
equals (c) f ′ (2) ≤ 0 (d)
f ′ (3) f ′ (2)

x→
π π2 f (3) f (2)
4 x2 −
16 (2007, 3M)
Integer & Numerical Answer Type Question
8
(a) f (2)
π 5. For each positive integer n, let
1
2 1
(b) f (2) yn =((n + 1) (n + 2) ... (n + n )) n .
π n
2  1 For x ∈ R, let [x] be the greatest integer less than or
(c) f  
π  2 equal to x. If lim yn = L, then the value of [L ] is
n→ ∞
................. . (2018 Adv.)
(d) 4f (2)
1 2n
r
Fill in the Blank
3. lim ∑ equals (1999, 2M) x2
∫0
n→ ∞ n r =1 n2 + r2 cos 2t dt
(a) 1 + 5 (b) 5 − 1 (c) −1 + 2 (d) 1 + 2 6. lim =K
x→ 0 x sin x (1997C, 2M)

Topic 4 Continuity at a Point


Objective Questions I (Only one correct option) 2. The function f (x) = [x]2 − [x2] (where, [x] is the greatest
π π
1. If the function f defined on  ,  by
integer less than or equal to x), is discontinuous at
 6 3 (1999, 2M)

 2 cos x − 1 π (a) all integers (b) all integers except 0 and 1


 , x≠ (c) all integers except 0 (d) all integers except 1
f (x) =  cot x − 1 4 is continuous,
 π 3. Let [.] denotes the greatest integer function and
k, x=
 4 f (x) = [tan 2 x], then (1993, 1M)
then k is equal to (2019 Main, 9 April I) (a) lim f (x) does not exist
x→ 0
1 (b) f (x) is continuous at x = 0
(a) (b) 2
2 (c) f (x) is not differentiable at x = 0
1 (d) f ′ (0) = 1
(c) 1 (d)
2
192 Limit, Continuity and Differentiability

2x − 1
4. The function f (x) = [x] cos   π, [⋅] denotes the Analytical & Descriptive Questions
 2 
greatest integer function, is discontinuous at  π
{1 + |sin x|}a/|sin x |, < x<0
 6
(a) all x (1993, 1M) 
11. Let f (x) =  b, x=0
(b) all integer points π

(c) no x  e tan 2 x / tan 3 x
, 0<x<
6
(d) x which is not an integer
Determine a and b such that f (x) is continuous at x = 0.
5. If f (x) = x ( x + (x + 1), then (1985, 2M) (1994, 4M)
(a) f (x) is continuous but not differentiable at x = 0
(b) f (x) is differentiable at x = 0 
 1 − cos 4x , x<0
(c) f (x) is not differentiable at x = 0  x2
(d) None of the above 12. Let f (x) =  a, x=0
 x
log (1 + ax) − log (1 − bx)  , x>0
6. The function f (x) =  16 + x − 4
x
is not defined at x = 0. The value which should be Determine the value of a if possible, so that the function
assigned to f at x = 0, so that it is continuous at x = 0, is is continuous at x = 0. (1990, 4M)

(a) a − b 13. Find the values of a and b so that the function (1989)
(b) a + b (1983, 1M)  x + a 2 sin x, 0 ≤ x ≤ π /4
(c) log a + log b 
(d) None of the above
f (x) =  2x cot x + b, π /4 ≤ x ≤ π /2
a cos 2x − b sin x, π / 2 < x ≤ π

Objective Questions II
is continuous for 0 ≤ x ≤ π.
(One or more than one correct option)
14. Let g (x) be a polynomial of degree one and f (x) be
7. Let [x] be the greatest integer less than or equals to x.  g (x), x≤0
Then, at which of the following point(s) the function  1/ x
defined by f (x) =  (1 + x) 
f (x) = x cos (π (x + [x])) is discontinuous ? (2017 Adv.)
 (2 + x)  , x > 0
(a) x = − 1  
(b) x = 1 Find the continuous function f (x) satisfying
(c) x = 0 f ′ (1) = f (− 1). (1987, 6M)
(d) x = 2
15. Determine the values a, b, c, for which the function
8. For every pair of continuous function f , g : [0, 1] → R  sin (a + 1) x + sin x
such that max { f (x): x ∈ [0, 1]} = max { g (x): x ∈ [0,1]}.  , for x < 0
 x
The correct statement(s) is (are) (2014 Adv.) f (x) =  c, for x = 0
(a) [f (c)] + 3f (c) = [ g (c)] + 3 g (c) for some c ∈[0,1]
2 2  (x + bx2) 1/ 2 − x1/ 2
 , for x > 0
(b) [f (c)]2 + f (c) = [ g (c)]2 + 3 g (c) for some c ∈[0,1]  bx3/ 2
(c) [f (c)]2 + 3f (c) = [ g (c)]2 + g (c) for some c ∈[0,1]
is continuous at x = 0. (1982, 3M)
(d) [f (c)]2 = [ g (c)]2 for some c ∈[0,1]
9. For every integer n, let a n and bn be real numbers. Let Match the Columns
function f : R → R be given by π π
16. Let f1 : R → R, f2 :  −,  → R, f3 : (− 1, eπ / 2 − 2) → R and
a n + sin πx, for x ∈ [2n , 2n + 1]  2 2
f (x) =  ,
bn + cos πx, for x ∈ (2n − 1, 2n ) f4 : R → R be functions defined by
2

for all integers n. (i) f1 (x) = sin( 1− e− x ),

If f is continuous, then which of the following hold(s) for  |sin x| if x ≠ 0



(ii) f2 (x) =  tan − 1 x , where the inverse
all n ? (2012)

 1 if x = 0
(a) an−1 − bn−1 = 0
trigonometric function tan − 1 x assumes values in
(b) an − bn = 1
− π , π.
(c) an − bn + 1 = 1  
 2 2
(d) an − 1 − bn = −1
(iii) f3 (x) = [sin(log e (x + 2))], where for t ∈R, [t ] denotes the
Fill in the Blank greatest integer less than or equal to t,
x2 sin  1  if x ≠ 0
10. A discontinuous function y = f (x) satisfying x2 + y2 = 4 is   
(iv) f4 (x) =   x
given by f (x) = .... . (1982, 2M)  0 if x = 0
Limit, Continuity and Differentiability 193

List-I List-II
P. The function f1 is 1. NOT continuous at x = 0
Q. The function f2 is 2. continuous at x = 0 and NOT differentiable at x = 0
The function f3 is 3. differentiable at x = 0 and its derivative is NOT continuous at x = 0
R.

S. The function f4 is 4. differentiable at x = 0 and its derivative is continuous at x = 0


The correct option is
(a) P → 2; Q → 3; R → 1; S → 4 (b) P → 4; Q → 1; R → 2; S → 3
(c) P → 4; Q → 2; R → 1; S → 3 (d) P → 2; Q → 1; R → 4; S → 3

Topic 5 Continuity in a Domain


Objective Questions I (Only one correct option) (c) Both lim f (x) and lim f (x) exist but are not equal
x→ 4 − x→ 4 +
1. Let f : R → R be a continuously differentiable (d) lim f (x) exists but lim f (x) does not exist
x→ 4 − x→ 4 +
1
function such that f (2) = 6 and f′ (2) = . If
48 5. Let f : [−1, 3] → R be defined as
f ( x) 3
∫ 4t dt = (x − 2) g(x), then lim g (x) is equal to
6 x→ 2
(2019 Main, 12 April I)
|x| + [x], −1 ≤ x < 1

f (x) =  x + |x|, 1 ≤ x < 2
(a) 18 (b) 24 (c) 12 (d) 36  x + [x], 2 ≤ x ≤ 3 ,
 (2019 Main, 8 April II)
sin ( p + 1) x + sin x
 ,x<0 where, [t ] denotes the greatest integer less than or
 x equal to t. Then, f is discontinuous at
2. If f (x) =  q, x=0
 (a) four or more points
x+ x − x
2
x>0
 , (b) only two points
 x3/ 2 (c) only three points
is continuous at x = 0 , then the ordered pair ( p, q) is (d) only one point
equal to (2019 Main, 10 April I) 6. Let f: R → R be a function defined as
(a)  − , −  (b)  −
3 1 1 3
,   5, if x≤1
 2 2  2 2 a + bx, if 1 < x < 3
 5 1 
(d)  −
3 1 f (x) = 
(c)  ,  , 
 2 2  2 2  b + 5x, if 3 ≤ x < 5
 a|π − x|+1, x ≤ 5  30, if x≥5
3. If the function f (x) =  is continuous at
b|x − π|+3, x > 5 Then, f is (2019 Main, 9 Jan I)
x = 5, then the value of a − b is (2019 Main, 9 April II) (a) continuous if a = − 5 and b = 10
−2 2 (b) continuous if a = 5 and b = 5
(a) (b)
π+ 5 π+ 5 (c) continuous if a = 0 and b = 5
2 2 (d) not continuous for any values of a and b
(c) (d)
π −5 5− π 1
7. If f (x) = x − 1, then on the interval [0, π ] (1989, 2M)
4. If f (x) = [x] −  , x ∈ R where [x] denotes the greatest
x 2
4  (a) tan [f (x)] and 1/ f (x) are both continuous
integer function, then (2019 Main, 9 April II) (b) tan [f (x)] and 1/ f (x) are both discontinuous
(a) lim f (x) exists but lim f (x) does not exist (c) tan [f (x)] and f −1 (x) are both continuous
x→ 4 + x→ 4 −
(b) f is continuous at x = 4 (d) tan [f (x)] is continuous but 1/ f (x) is not continuous
194 Limit, Continuity and Differentiability

Objective Questions II the points of discontinuity of f in the domain are…… .


(1996, 2M)
(One or more than one correct option)
8. The following functions are continuous on (0, π) Analytical & Descriptive Question
(a) tan x
x
(b) ∫ t sin dt
1  x2
0 t (1991, 2M)  , 0 ≤ x<1
11. Let f (x) =  2
 1, 0 ≤ x≤ 3 π /4 xπ sin x, 0 < x ≤ π /2
(c)  2 3π (d)  π 2x2 − 3x + 3 , 1 ≤ x ≤ 2
2 sin x, < x< π sin ( π + x ), < x< π  2
 9 4  2 2
Discuss the continuity of f , f ′ and f ′ ′ on [0, 2].
9. Let [x] denotes the greatest integer less than or equal to (1983, 2M)
x. If f (x) = [x sin πx], then f (x) is (1986, 2M)
(a) continuous at x = 0 (b) continuous in (−1, 0) Integer & Numerical Answer Type Question
(c) differentiable at x = 1 (d) differentiable in (−1, 1)
12. If the function f defined on  − ,  by
1 1
 3 3
Fill in the Blank
1  1 + 3x

 π 
10. Let f (x) = [x] sin   , where [⋅] denotes the f (x) =  x log e  1 − 2x  , when x ≠ 0 is continuous, then k
 [x + 1] k , when x = 0
greatest integer function. The domain of f is …… and is equal to ………… . (2020 Main, 7 Jan II)

Topic 6 Continuity for Composition and Function


Objective Question II where, a and b are non-negative real numbers.
(One or more than one correct option) Determine the compositie function gof. If ( gof ) (x) is
continuous for all real x determine the values of a and b.
1
1. For the function f (x) = x cos , x ≥ 1, Further, for these values of a and b, is gof differentiable
x (2009) at x = 0 ? Justify your answer. (2002, 5M)
(a) for atleast one x in the interval
[1, ∞ ), f (x + 2) − f (x) < 2 3. Let f (x) be a continuous and g (x) be a discontinuous
(b) lim f ′(x) = 1 function. Prove that f (x) + g (x) is a discontinuous
x→ ∞ function. (1987, 2M)
(c) for all x in the interval [1, ∞ ), f (x + 2) − f (x) > 2
1 + x, 0 ≤ x ≤ 2
(d) f ′(x) is strictly decreasing in the interval [1, ∞ ) 4. Let f (x) = 
3 − x, 2 < x ≤ 3
Analytical & Descriptive Questions Determine the form of g (x) = f [ f (x)] and hence find the
 x + a , if x < 0 points of discontinuity of g, if any (1983, 2M)
2. Let f (x) = 
|x − 1|, if x ≥ 0 5. Let f (x + y) = f (x) + f ( y) for all x and y. If the function
 x + 1, if x < 0 f (x) is continuous at x = 0, then show that f (x) is
and g (x) = 
 (x − 1 ) 2
+ b , if x ≥ 0 continuous at all x. (1981, 2M)

Topic 7 Differentiability at a Point


Objective Questions I (Only one correct option) 2. If f : R → R is a differentiable function and
f ( x)
1. Let f : R → R be differentiable at c ∈ R and f (c) = 0. If 2t dt
f (2) = 6, then lim
x→ 2 ∫ (x − 2)
is (2019 Main, 9 April II)
g (x) = | f (x)|, then at x = c, g is (2019 Main, 10 April I) 6

(a) not differentiable (a) 12f ′ (2) (b) 0 (c) 24f ′ (2) (d) 2f ′ (2)
(b) differentiable if f ′ (c) ≠ 0 3. Let f (x) = 15 − x − 10 ; x ∈ R. Then, the set of all values
(c) not differentiable if f ′ (c) = 0 of x, at which the function, g (x) = f ( f (x)) is not
(d) differentiable if f ′ (c) = 0 differentiable, is (2019 Main, 9 April I)
(a) {5, 10, 15, 20} (b) {5, 10, 15}
(c) {10} (d) {10, 15}
Limit, Continuity and Differentiability 195

4. Let S be the set of all points in (− π , π ) at which the (a) 2f ′ (c) = g ′(c) (b) 2 f ′(c) = 3 g ′(c)
function, f (x) = min {sin x, cos x} is not differentiable. (c) f ′(c) = g ′(c) (d) f ′(c) = 2 g ′(c)
Then, S is a subset of which of the following?  π
 2
(2019 Main, 12 Jan I) 13. Let f (x) = x cos x , x ≠ 0, x ∈ R, then f is (2012)
π π π π π π  0,
(a) − , 0,  (b) − , − , , 
x =0
 4 4  2 4 4 2 (a) differentiable both at x = 0 and at x = 2
 3π π 3π π  3π π π 3π 
(c) − ,− , ,  (d) − ,− , ,  (b) differentiable at x = 0 but not differentiable at x = 2
 4 4 4 4  4 2 2 4 (c) not differentiable at x = 0 but differentiable at x = 2
5. Let K be the set of all real values of x, where the function (d) differentiable neither at x = 0 nor at x = 2
f (x) = sin| x| − | x| + 2(x − π ) cos| x|is not differentiable. (x − 1)n
14. Let g (x) = ; 0 < x < 2, m and n are
Then, the set K is equal to (2019 Main, 11 Jan II) log cosm (x − 1)
(a) {0} (b) φ (an empty set) integers, m ≠ 0, n > 0 and let p be the left hand
(c) { π } (d) {0, π } derivative of|x − 1| at x = 1 . If lim g (x) = p , then
 −1 , −2 ≤ x < 0 x→1 +
6. Let f (x) =  and (a) n = 1, m = 1 (b) n = 1, m = −1 (2008, 3M)
 x 2
− 1 , 0 ≤ x≤2
(c) n = 2 , m = 2 (d) n > 2, m = n
g (x) = | f (x)| + f (|x|). Then, in the interval (−2, 2), g is 15. If f is a differentiable function satisfying
(2019 Main, 11 Jan I)
 1
(a) not differentiable at one point f   = 0, ∀ n ≥ 1, n ∈ I ,then (2005, 2M)
 n
(b) not differentiable at two points
(c) differentiable at all points (a) f (x) = 0, x ∈ (0, 1]
(d) not continuous (b) f ′ (0) = 0 = f (0)
(c) f (0) = 0 but f ′ (0) not necessarily zero
7. Let f : (−1, 1) → R be a function defined by (d)|f (x)|≤ 1, x ∈ (0, 1]
f (x) = max { − x , − 1 − x2 }. If K be the set of all points at 16. Let f (x) = ||x|− 1|, then points where, f (x) is not
which f is not differentiable, then K has exactly differentiable is/are (2005, 2M)
(2019 Main, 10 Jan II)
(a) 0, ± 1 (b) ± 1
(a) three elements (b) five elements
(c) 0 (d) 1
(c) two elements (d) one element
17. The domain of the derivative of the functions
max {|x|, x2}, |x| ≤ 2  tan −1 x ,
8. Let f (x) =   if | x| ≤ 1
 8 − 2|x|, 2 < |x| ≤ 4 f (x) = 1 is (2002, 2M)
(| x| − 1), if | x| > 1
Let S be the set of points in the interval (−4, 4) at which f 2
is not differentiable. Then, S (2019 Main, 10 Jan I) (a) R − {0} (b) R − {1}
(a) equals {−2, − 1, 0, 1, 2} (c) R − {−1} (d) R − {−1, 1}
(b) equals {−2, 2}
18. Which of the following functions is differentiable
(c) is an empty set at x = 0 ? (2001, 2M)
(d) equals {−2,−1, 1, 2}
(a) cos (| x|) + | x| (b) cos (| x|) − | x|
9. Let f be a differentiable function from R to R such that (c) sin (| x|) + | x| (d) sin (| x|) − | x|
3
| f (x) − f ( y)| ≤ 2|x − y|2 , for all x, y ∈ R. If f (0) = 1, then 19. The left hand derivative of f (x) = [x] sin (π x) at x = k, k
1 is an integer, is (2001, 2M)
∫ f 2(x) dx is equal to (2019 Main, 9 Jan II) (a) (−1)k (k − 1) π (b)(−1)k − 1 (k − 1) π
0 1 (c) (−1)k kπ (d) (−1)k − 1 kπ
(a) 2 (b) (c) 1 (d) 0
2 20. Let f : R → R be a function defined by f (x) = max { x, x3 }.
10. Let S = (t ∈ R : f (x) = |x − π |(
⋅ e − 1)sin| x| is not
|x|
The set of all points, where f (x) is not differentiable, is
differentiable at t}.Then, the set S is equal to(2018 Main) (a) {−1,1 } (b) {−1, 0 } (2001, 2M)
(a) φ (an empty set) (b) {0} (c) {0,1 } (d) {−1, 0,1 }
(c) { π } (d) {0, π } 21. Let f : R → R be any function. Define g : R → R by
11. For x ∈ R, f (x) = |log 2 − sin x|and g (x) = f ( f (x)), then g (x) =| f (x)|, ∀ x. Then, g is (2000, 2M)
(a) g is not differentiable at x = 0 (2016 Main) (a) onto if f is onto
(b) g′ (0) = cos (log 2) (b) one-one if f is one-one
(c) g′ (0) = − cos (log 2) (c) continuous if f is continuous
(d) g is differentiable at x = 0 and g′ (0) = − sin (log 2) (d) differentiable if f is differentiable
12. If f and g are differentiable functions in (0, 1) satisfying 22. The function f (x) = (x2 − 1)| x2 − 3x + 2| + cos (| x|) is
f (0) = 2 = g (1), g(0) = 0 and f (1) = 6, then for some not differentiable at (1999, 2M)
c ∈] 0, 1 [ (2014 Main) (a) −1 (b) 0 (c) 1 (d) 2
196 Limit, Continuity and Differentiability

x 29. For every twice differentiable function f : R → [−2, 2]


23. The set of all points, where the function f (x) = is
1 + | x| with ( f (0))2 + ( f ′ (0))2 = 85, which of the following
differentiable, is (1987, 2M) statement(s) is (are) TRUE ? (2018 Adv.)
(a) (− ∞ , ∞ ) (b) [0, ∞ ) (a) There exist r , s ∈ R , where r < s, such that f is one-one on
(c) (− ∞ , 0) ∪ (0, ∞ ) (d) (0, ∞ ) the open interval (r , s)
24. There exists a function f (x) satisfying f (0) = 1, (b) There exists x0 ∈ (−4, 0) such that|f ′ (x0 )|≤ 1
(c) lim f (x) = 1
f ′ (0) = − 1, f (x) > 0, ∀ x and (1982, 2M) x→ ∞
(a) f ′′ (x) < 0 , ∀ x (b) − 1 < f ′′ (x) < 0 , ∀ x (d) There exists α ∈ (−4, 4) such that f (α ) + f ′′(α ) = 0 and
(c) − 2 ≤ f ′′ (x) ≤ − 1 , ∀ x (d) f ′′ (x) < − 2 , ∀ x f ′ (α ) ≠ 0

25. For a real number y, let [ y] denotes the greatest 30. Let f : (0, π)→ R be a twice differentiable function such
f (x) sin t − f (t )sin x
integer less than or equal to y. Then, the function that lim = sin 2 x for all x ∈ (0, π).
tan π [(x − π )] t→ x t−x
f (x) = is (1981, 2M)
1 + [x]2  π π
If f   = − , then which of the following statement(s)
(a) discontinuous at some x  6 12
(b) continuous at all x, but the derivative f ′ (x) does not is (are) TRUE? (2018 Adv.)
exist for some x π π
(c) f ′(x) exists for all x, but the derivative f ′ ′ (x) does not (a) f   =
 4 4 2
exist for some x
x4
(d) f ′(x) exists for all x (b) f (x)< − x2 for all x∈ (0, π)
6
Objective Questions II (c) There exists α ∈(0, π) such that f ′ (α) = 0
π π
(One or more than one correct option) (d) f ′′  + f   = 0
 2  2
26. Let f : R → R and g : R → R be functions satisfying
31. Let f : R → R, g : R → R and h : R → R be differentiable
f (x + y) = f (x) + f ( y) + f (x) f ( y)and f (x) = xg (x)
functions such that f (x) = x3 + 3x + 2, g ( f (x)) = x and
for all x, y ∈ R. If lim g (x) = 1, then which of the h ( g ( g (x))) = x for all x ∈ R. Then, (2016 Adv.)
x→ 0
1
following statements is/are TRUE? (2020 Adv.) (a) g ′(2) = (b) h ′(1) = 666
15
(a) f is differentiable at every x ∈ R (c) h(0) = 16 (d) h ( g (3)) = 36
(b) If g(0) = 1, then g is differentiable at every x∈ R
(c) The derivative f ′ (1) is equal to 1 32. Let a , b ∈ R and f : R → R be defined by
(d) The derivative f ′ (0) is equal to 1 f (x) = a cos (|x3 − x|) + b|x|sin (|x3 + x|). Then, f is
(2016 Adv.)
27. Let the function f : R → R be defined by
(a) differentiable at x = 0, if a = 0 and b = 1
f (x) = x − x + (x − 1)sin x and let g : R → R be an
3 2
(b) differentiable at x = 1, if a = 1 and b = 0
arbitrary function. Let fg : R → R be the product
(c) not differentiable at x = 0, if a = 1and b = 0
function defined by ( fg )(x) = f (x) g (x). Then which of the
following statements is/are TRUE? (2020 Adv.) (d) not differentiable at x = 1, if a = 1and b = 1

33. Let f : − , 2 → R and g : − , 2 → R be functions


(a) If g is continuous at x = 1, then fg is differentiable at 1 1
x =1  2   2 
(b) If f g is differentiable at x = 1, then g is continuous at defined by f (x) = [x2 − 3] and g (x) =|x| f (x) + |4x − 7| f (x),
x=1 where [ y] denotes the greatest integer less than or equal
(c) If g is differentiable at x = 1, then f g is differentiable to y for y ∈ R. Then, (2016 Adv.)
at x = 1
(a) f is discontinuous exactly at three points in  − , 2
1
(d) If f g is differentiable at x = 1, then g is differentiable  2 
at x = 1
(b) f is discontinuous exactly at four points in  − , 2
1
28. Let f : R be a function. We say that f has  2 
f (h ) − f (0)
(c) g is not differentiable exactly at four points in  − , 2
1
PROPERTY 1 if lim exists and is finite, and
h→ 0
|h|  2 
f (h ) − f (0) (d) g is not differentiable exactly at five points in  − , 2
1
PROPERTY 2 if lim exists and is finite.
h→ 0 h2  2 
Then which of the following options is/are correct? 34. Let g : R → R be a differentiable function with
(2019 Adv.)
g (0) = 0, g′ (0) = 0 and g′ (1) =/ 0. (2015 Adv.)
(a) f (x) = sin x has PROPERTY 2
(b) f (x) = x23
/  x
has PROPERTY 1  g (x), x =/ 0
(c) f (x) = |x|has PROPERTY 1 Let f (x) = |x|
(d) f (x) = x|x|has PROPERTY 2  0 , x=0
Limit, Continuity and Differentiability 197

and h (x) = e|x| for all x ∈ R. Let ( foh ) (x) denotes f { h (x)} (a) f (x) is differentiable only in a finite interval containing
and (hof )(x) denotes h { f (x)}. Then, which of the zero
following is/are true? (b) f (x) is continuous for all x ∈ R
(c) f ′ (x) is constant for all x ∈ R
(a) f is differentiable at x = 0
(d) f (x) is differentiable except at finitely many points
(b) h is differentiable at x = 0
(c) foh is differentiable at x = 0 39. If f (x) = min { 1, x2, x3 }, then (2006, 3M)
(d) hof is differentiable at x = 0 (a) f (x) is continuous everywhere
35. Let f, g : [−1 , 2] → R be continuous functions which are (b) f (x) is continuous and differentiable everywhere
(c) f (x) is not differentiable at two points
twice differentiable on the interval (−1, 2). Let the
(d) f (x) is not differentiable at one point
values of f and g at the points −1, 0 and 2 be as given in
the following table: 40. Let h (x) = min { x, x2} for every real number of x, then
x= −1 x=0 x=2 (a) h is continuous for all x (1998, 2M)
(b) h is differentiable for all x
f (x) 3 6 0
(c) h ′ (x) = 1, ∀ x > 1
g (x) 0 1 −1 (d) h is not differentiable at two values of x

In each of the intervals (−1, 0) and (0, 2), the function | x − 3|, x≥1
( f − 3 g )″ never vanishes. Then, the correct statement(s) 41. The function f (x) =  x2 3x 13 is (1988, 2M)
is/are (2015 Adv.)  4 − 2 + 4 , x<1

(a) f ′ (x) − 3 g ′ (x) = 0 has exactly three solutions in (a) continuous at x = 1


(−1, 0) ∪ (0, 2) (b) differentiable at x = 1
(b) f ′ (x) − 3 g ′ (x) = 0 has exactly one solution in (−1 , 0) (c) discontinuous at x = 1
(c) f ′ (x) − 3 g ′ (x) = 0 has exactly one solution in (0, 2) (d) differentiable at x = 3
(d) f ′ (x) − 3 g ′ (x) = 0 has exactly two solutions in (−1, 0) 42. The function f (x) = 1 + |sin x|is (1986, 2M)
and exactly two solutions in (0, 2)
(a) continuous no where
36. Let f : [a , b] → [1, ∞ ) be a continuous function and (b) continuous everywhere
 (c) differentiable at x = 0
0 , if x< a
 x (d) not differentiable at infinite number of points
g : R → R be defined as g (x) = ∫ f (t )dt , if a ≤ x ≤ b.
a
 b 43. If x + | y| = 2 y, then y as a function of x is (1984, 2M)
∫ f (t )dt , if x> b
 a (a) defined for all real x
Then, (2013) (b) continuous at x = 0
(a) g (x) is continuous but not differentiable at a (c) differentiable for all x
dy 1
(b) g (x) is differentiable on R (d) such that = for x < 0
dx 3
(c) g (x) is continuous but not differentiable at b
(d) g (x) is continuous and differentiable at either a or b
but not both Assertion and Reason
 π π For the following questions, choose the correct answer
−x− , x≤− from the codes (a), (b), (c) and (d) defined as follows.
 2 2
 π (a) Statement I is true, Statement II is also true;
37. If f (x) = − cos x, − < x ≤ 0, then Statement II is the correct explanation of Statement I
 2
 x − 1, 0 < x≤1 (b) Statement I is true, Statement II is also true;
 ln x, x>1 Statement II is not the correct explanation of
(2011) Statement I
π
(a) f (x) is continuous at x = − (c) Statement I is true; Statement II is false
2
(d) Statement I is false; Statement II is true
(b) f (x) is not differentiable at x = 0
(c) f (x) is differentiable at x = 1 44. Let f and g be real valued functions defined on interval
3 (−1, 1) such that g′′ (x) is continuous, g (0) ≠ 0, g′ (0) = 0,
(d) f (x) is differentiable at x = −
2 g′′ (0) ≠ 0, and f (x) = g (x)sin x .
38. Let f : R → R be a function such that Statement I lim [ g (x) cos x − g (0) cosec x] = f ′′ (0). and
x→ 0
f (x + y) = f (x) + f ( y), ∀x, y ∈ R. If f (x) is differentiable Statement II f ′ (0) = g (0). (2008, 3M)
at x = 0, then (2011)
198 Limit, Continuity and Differentiability

52. If f : [−1, 1] → R and f ′ (0) = lim nf   and f (0) = 0.


1
Match the Columns
n→ ∞  
n
45. In the following, [x] denotes the greatest integer less 2 −1  1 
than or equal to x. Find the value of lim (n + 1) cos   − n, given that
n→ ∞π  n
Column I Column II   1  π
0 <  lim cos −1    < . (2004, 2M)
A. x| x| p. continuous in (– 1, 1) n → ∞  n  2
B. | x| q. differentiable in (– 1, 1) 53. Let α ∈ R. Prove that a function f : R → R is
C. x + [x ] r. strictly increasing (– 1, 1) differentiable at α if and only if there is a function
D. | x − 1| +| x + 1|, s. not differentiable atleast at one g : R → R which is continuous at α and satisfies
in ( −1, 1) point in (– 1, 1) f (x) − f (α ) = g (x) (x − α ) , ∀ x ∈ R. (2001, 5M)

(2007, 6M) 54. Determine the values of x for which the following
46. Match the conditions/expressions in Column I with function fails to be continuous or differentiable
statement in Column II (1992, 2M)  1 − x, x<1

f (x) = (1 − x) (2 − x), 1 ≤ x ≤ 2 . Justify your answer.
Column I Column II  3 − x, x>2
 (1997, 5M)
A. sin (π [x]) p. differentiable everywhere
 −  1 + 1 
sin{ π (x − [x])} 
B. q. no where differentiable 55. Let f (x) = x e  |x | x  , x ≠ 0
r. not differentiable at 1 and −1  , x=0
 0
Test whether
Fill in the Blanks (i) f (x) is continuous at x = 0.
47. Let F (x) = f (x) g (x) h (x) for all real x, where f (x), g (x) (ii) f (x) is differentiable at x = 0. (1997C, 5M)

and h (x) are differentiable functions. At same point 56. Let f [(x + y) / 2] = { f (x) + f ( y)} / 2 for all real x and y, if
x0 , F ′ (x0 ) = 21 F (x0 ), f ′ (x0 ) = 4 f (x0 ), g ′ (x0 ) = − 7 g (x0 ) f ′ (0) exists and equals −1 and f (0) = 1 , find f (2).
and h ′ (x0 ) = kh (x0 ), then k = K . (1997C, 2M) (1995, 5M)

 x 57. A function f : R → R satisfies the equation



48. For the function f (x) = 1 + e1/ x , x ≠ 0 ; f (x + y) = f (x) f ( y) , ∀ x , y in R and f (x) ≠ 0 for any x in
 0 , x=0 R. Let the function be differentiable at x = 0 and
f ′ (0) = 2 . Show that f ′ (x) = 2 f (x) , ∀ x in R. Hence,
the derivative from the right, f ′ (0+ ) = … and the determine f (x). (1990, 4M)
derivative from the left, f ′ (0− ) = … . (1983, 2M)
58. Draw a graph of the function
 1 y = [x] + |1 − x|, − 1 ≤ x ≤ 3.
49. Let f (x) = (x − 1) sin (x − 1) − |x|, if x ≠ 1 be a real
2

Determine the points if any, where this function is not


− 1, if x = 1
differentiable. (1989, 4M)
valued function. Then, the set of points, where f (x) is
not differentiable, is …. . (1981, 2M) 59. Let R be the set of real numbers and f : R → R be such
that for all x and y in R, f (x) − f ( y)|2 ≤ (x − y)3 . Prove
True/False that f (x) is a constant. (1988, 2M)

50. The derivative of an even function is always an odd 60. Let f (x) be a function satisfying the condition
function. (1983, 1M) f (− x) = f (x), ∀ x. If f ′ (0) exists, find its value. (1987, 2M)
61. Let f (x) be defined in the interval [− 2, 2] such that
Analytical & Descriptive Questions
 −1 , −2 ≤ x ≤ 0
f (x) = 
 −1  x + c 1
b sin  2  , − 2 < x < 0 x − 1 , 0 < x ≤ 2
 1 and g (x) = f (| x|) + | f (x)|.
51. f (x) =  , x=0
2 Test the differentiability of g (x) in (− 2, 2). (1986, 5M)

 e
a x/ 2
−1 1 62. Let f (x) = x − x − x + 1
3 2
, 0<x<
 x 2
= max { f (t ) ; 0 ≤ t ≤ x}, 0 ≤ x ≤ 1
1 and g (x) 
If f (x) is differentiable at x = 0 and|c| < , then find the = 3 − x, 1 < x ≤ 2
2
Discuss the continuity and differentiability of the
value of a and prove that 64b2 = (4 − c2). (2004, 4M)
function g (x) in the interval (0, 2). (1985, 5M)
Limit, Continuity and Differentiability 199

 x −1 66. Let f : R → R be a differentiable function such that


, when x ≠ 1
2x2 − 7x + 5  π
63. Find f ′ (1) , if f (x) =  . f (0) = 0, f   = 3 and f′ (0) = 1.
 1  2
− , when x = 1
 3 (1979, 3M) π
If g (x) = ∫ 2 [ f ′ (t ) cosec t − cot t cosec t f (t )] dt
64. If f (x) = x tan −1 x , find f ′ (1) from first principle. x

(1978, 3M)  π
for x ∈ 0, , then lim g (x) =
 2  x→ 0 (2017 Adv.)
Integer & Numerical Answer Type Questions 67. Let f : R → R and g : R → R be respectively given by
65. Let the functions f : (− 1, 1) → R and g : (− 1, 1) → (− 1, 1) f (x) =|x|+ 1 and g (x) = x2 + 1. Define h : R → R by
be defined by
max{ f (x), g (x)}, if x ≤ 0 .
f (x) = |2x − 1| + |2x + 1|and g (x) = x − [x], h (x) = 
 min{ f (x), g (x)}, if x > 0 .
where [x] denotes the greatest integer less than or equal
to x. Let f o g: (− 1, 1) → R be the composite function The number of points at which h (x) is not differentiable
is (2014 Adv.)
defined by ( f o g )(x) = f ( g (x)). Suppose cis the number of
points in the interval (− 1, 1) at which f o g is NOT 68. Let p (x) be a polynomial of degree 4 having extremum
continuous, and suppose d is the number of points in the  p (x) 
at x = 1 , 2 and lim 1 + 2 = 2 . Then, the value of p(2)
interval (− 1, 1) at which f o g is NOT differentiable. x→ 0 
 x 
Then the value of c + d is ……… . (2020 Adv.) is ……… . (2010)

Topic 8 Differentiation
Objective Questions I (Only one correct option) 7. For x > 1, if (2x)2y = 4e2x − 2y , then (1 + log e 2x)2
dy
is
dx
1. If (a + 2b cos x) (a − 2b cos y) = a 2 − b2, where a > b >
equal to (2019 Main, 12 Jan I)
dx  π π 
0, the at  ,  is x log e 2x + log e 2 x log e 2x − log e 2
dy  4 4  (2020 Main, 4 Sep I)
(a)
x
(b)
x
a−b a+ b 2a + b a − 2b (c) x log e 2x (d) log e 2x
(a) (b) (c) (d)
a+ b a−b 2a − b a + 2b dy
8. If x log e (log e x) − x2 + y2 = 4( y > 0), then at x = e is
2. If C1 + (22) 20C 2 + (32) 20C3 + ..... + (202)20C 20 = A (2β )
20
dx
, then the ordered pair ( A , β) is equal to equal to (2019 Main, 11 Jan I)
(2019 Main, 12 April II) e (2e − 1)
(a) (b)
(a) (420, 19) (b) (420, 18) (c) (380, 18) (d) (380, 19) 4 + e2 2 4 + e2
 sin x − cos x  x (1+ 2e ) (1+ 2e )
3. The derivative of tan −1   , with respect to , (c) (d)
 sin x + cos x 2 4 + e2 2 4 + e2
  π
where  x ∈ 0,   is 9. Let f :R→ R be a function such that
  2 (2019 Main, 12 April II)
f (x) = x3 + x2f ′ (1) + xf ′ ′ (2) + f ′′′ (3), x ∈ R.
2 1
(a) 1 (b) (c) (d) 2 Then, f (2) equals (2019 Main, 10 Jan I)
3 2
(a) 30 (b) − 4
 dy d 2y (c) − 2
4. If e + xy = e, the ordered pair  , 2  at x = 0 is equal
y (d) 8
 dx dx  d 2y
10. If x = 3 tan t and y = 3 sec t, then the value of at
to (2019 Main, 12 April I) dx2
(a)  , − 2  (b)  − , 2  (c)  , 2 
1 1 1 1 1 1
(d)  − , − 2 
1 1 π
t= , is
e e   e e  e e   e e  4 (2019 Main, 9 Jan II)
1 1 1 3
5. If f (1) = 1, f′ (1) = 3, then the derivative of (a) (b) (c) (d)
6 6 2 3 2 2 2
f ( f ( f (x))) + ( f (x))2 at x = 1 is (2019 Main, 8 April II)
11. For x ∈ 0,  , if the derivative of
1
(a) 12 (b) 9 (c) 15 (d) 33
 4
2
  3 cos x + sin x   π dy
6. If 2 y =  cot−1    , x ∈ 0,  then is  6x x 
  cos x − 3 sin x   2 dx tan −1   is x ⋅ g (x), then g (x) equals
 1 − 9x3  (2017 Main)
equal to (2019 Main, 8 April I)
9 3x x 3x 3
π π π π (a) (b) (c) (d)
(a) −x (b) x − (c) −x (d) 2x − 1 + 9x3 1 − 9x3 1 − 9x3 1 + 9x3
6 6 3 3
200 Limit, Continuity and Differentiability

12. If g is the inverse of a function f and f ′ (x) =


1
, then (a) p (b) p + p 2
1 + x5 (c) p + p3 (d) independent of p
g′ (x) is equal to (2015) 21. If y2 = P (x) is a polynomial of degree 3, then
1
(a) 1 + x5 (b) 5 x4 (c) (d) 1 + { g (x)}5 d  3 d 2y
1 + { g (x)}5 2 y  equals (1988, 2M)
dx  dx2 
−1 dy
13. If y = sec (tan x), then at x = 1 is equal to (a) P ′ ′ ′ (x) + P ′ (x) (b) P ′ ′ (x) ⋅ P ′ ′ ′ (x)
dx (2013)
1 1 (c) P (x) P ′ ′ ′ (x) (d) a constant
(a) (b) (c) 1 (d) 2
2 2
14. Let g (x) = log f (x), where f (x) is a twice differentiable Fill in the Blanks
positive function on (0, ∞ ) such that f (x + 1) = x f (x). dy
22. If x exy = y + sin 2 x, then at x = 0, = ...… . (1996, 2M)
 1  1 dx
Then, for N = 1, 2, 3,... , g′′  N +  − g′′   is equal to
 2  2 23. Let f (x) = x|x|. The set of points, where f (x) is twice
 1 1 1  differentiable, is … . (1992, 2M)
(a) − 4 1 + + + ... +  (2008, 3M)
 9 25 (2 N − 1)2  24. If f (x) = |x − 2|and g (x) = f [ f (x)] , then g ′ (x) = K… for
 1 1 1  x > 2. (1990, 2M)
(b) 4 1 + + + ... + 2
 9 25 (2 N − 1)  −1 
1 
25. The derivative of sec  − 2  with respect to
 1 1 1   2x − 1
(c) − 4 1 + + + ... + 2 1
 9 25 (2 N + 1)  1 − x2at x = is …… . (1986, 2M)
2
 1 1 1 
(d) 4 1 + + + ... + 2
26. If f (x) = log x (log x), then f ′ (x) at x = e is ...... . (1985, 2M)
 9 25 (2 N + 1) 
27. If fr (x), gr (x), hr (x), r = 1, 2, 3 are polynomials in x such
d 2x
15. equals that fr (a ) = gr (a ) = hr (a ), r = 1, 2, 3
dy2 (2007, 3M)
−1 −1
f1 (x) f2(x) f3 (x)
−3
 d 2y   d 2y   dy  and F (x) = g1 (x) g2(x) g3 (x) ,
(a)  2  (b) −  2   
 dx   dx   dx 
h1 (x) h2(x) h3 (x)
 d 2 y  dy − 2  d 2 y  dy − 3 then F ′ (x) at x = a is …… .
(c)  2    (d) −  2    (1985, 2M)
 dx   dx   dx   dx   2x − 1 dy
16. If f ′′ (x) = − f (x), where f (x) is a continuous double 28. If y = f  2  and f ′ (x) = sin 2 x, then = ....... .
 x + 1 dx (1982, 2M)
differentiable function and g (x) = f ′ (x).
  x 
2
  x 
2 Analytical & Descriptive Questions
If F (x) =  f    +  g    and F (5) = 5,
  2    2  29. If y =
ax2
+
bx
+
c
+ 1,
then F (10) is (2006, 3M) (x − a ) (x − b) (x − c) (x − b) (x − c) (x − c)
(a) 0 (b) 5 (c) 10 (d) 25 y′ 1  a b c 
17. Let f be twice differentiable function satisfying Prove that =  + + . (1998, 8M)
y x  a − x b − x c − x
f (1) = 1, f (2) = 4, f (3) = 9, then (2005, 2M)
dy
(a) f ′ ′ (x) = 2, ∀ x ∈ (R ) 30. Find at x = − 1, when
dx
(b) f ′ (x) = 5 = f ′ ′ (x), for some x ∈ (1, 3)
π
(c) there exists atleast one x ∈ (1, 3) such that f ′ ′ (x) = 2 (sin y)
sin
2
x
+
3
sec−1 (2x) + 2x tan ln (x + 2) = 0.
(d) None of the above 2 (1991, 4M)
18. If y is a function of x and log (x + y) = 2xy, then the value 31. If x = sec θ − cos θ and y = secn θ − cos nθ, then show that
of y′ (0) is (2004, 1M)
 dy
2

(b) −1 (x2 + 4)   = n 2 ( y2 + 4). (1989, 2M)


(a) 1 (c) 2 (d) 0  dx
19. If x2 + y2 = 1 , then (2000, 1M)
32. If α be a repeated roots of a quadratic equation f (x) = 0
(a) yy′ ′ − 2 ( y ′ )2 + 1 = 0 (b) yy′ ′ + ( y ′ )2 + 1 = 0
(c) yy′ ′ + ( y ′ )2 − 1 = 0 (d) yy′ ′ + 2 ( y ′ )2 + 1 = 0 and A (x), B(x) and C (x) be polynomials of degree 3, 4 and
3
x sin x cos x A (x) B(x) C (x)
20. Let f (x) = 6 −1 0 , where p is constant. 5 respectively, then show that A (α ) B(α ) C (α ) is
p p2 p3 A ′ (α ) B ′ (α ) C ′ (α )
d3 divisible by f (x), where prime denotes the derivatives.
Then, f (x) at x = 0 is (1997, 2M) (1984, 4M)
dx3
Limit, Continuity and Differentiability 201

33. Find the derivative with respect to x of the function 3 dy


36. Let y = ex sin x + (tan x)x , find .
  2x   dx (1981, 2M)
y = (log cos x sin x) (log sin x cos x)−1 + sin −1    5x dy
  1 + x2   37. Given, y = + cos 2 (2x + 1), find .
3 (1 − x) 2 dx
π (1980)
at x = .
4 (1984, 4M)
Integer & Numerical Answer Type Question
34. If (a + bx) ey/ x = x, then prove that
2
38. Let f : R → R be a continuous odd function, which
d 2y  dy  1
x3
= x − y . vanishes exactly at one point and f (1) = .
dx2  dx  (1983, 3M) 2
x
35. Let f be a twice differentiable function such that Suppose that F (x) = ∫ f (t ) dt for all x ∈ [−1 , 2] and
–1
(1983, 3M) x
f ′ ′ (x) = − f (x) , f ′ (x) = g (x) and G (x) = ∫ t| f { f (t )}| dt for all x ∈ [−1 , 2]. If
–1
h (x) = [ f (x) ]2 + [ g (x)]2 F (x) 1  1
lim = , then the value of f   is
Find h (10), if h (5) = 11. x→1 G (x) 14  2 (2015 Adv.)

Answers
Topic 1 15. a =
–3 1
, c = and b ∈ R 16. (d)
1. (b) 2. (c) 3. (d) 4. (a) 2 2
5. (c) 6. (b) 7. (a) 8. (b) Topic 5
9. (d) 10. (c) 11. (b) 12. (d)
1. (a) 2. (d) 3. (d) 4. (b)
13. (d) 14. (c) 15. (c) 16. (c)
5. (c) 6. (d) 7. (b) 8. (b, c)
17. (d) 18. (d) 19. (b) 20. (c)
9. (a,b, d) 10. x ∈ (– ∞,– 1 ) ∪ [ 0, ∞ ),[ − 1, 0 )
21. (a) 22. (d) 23. (a, c) 24. –1
1 11. f and f ′′ are continuous and f ′ is discontinuous at x = {1, 2 }.
25. 1 26. h 2hr − h ,
2
27. − 1
128r 12. (5)
2
28. 7 29. 30. False 31. loge 4 Topic 6
π
32. a 2 cosα + 2a sin α 33. 0 1. (b, c, d)
−1 x + a + 1, if x < −a
34. 35. (7) 36. (2) ( x + a − 1 ) 2, if a ≤ x < c

3 2. g{ f ( x )} =  2
Topic 2 x + b , if 0 ≤ x ≤ 1
( x − 2 ) 2 + b, if x > 1
1. (d) 2. (b) 3. (d) 4. (a)
5. (c) 6. (c) 7. (d) 8. (c) a = 1, b = 0
gof is differentiable at x = 0
9. (b) 10. (d) 11. (c) 12. (c)
4 − x , 2 < x ≤ 3
13. (c) 14. e2 15. e5 16. e2 
4. g ( x ) = 2 + x, 0 ≤ x ≤ 1, discontinuous at x = {1, 2 }
17. (1) 18. (2)
2 − x, 1 < x ≤ 2

Topic 3
Discontinuity of g at x = {1, 2 }
1. (c) 2. (a) 3. (b) 4. (b,c)
5. (1) 6. 1 Topic 7
1. (b) 2. (a) 3. (b) 4. (c)
Topic 4
5. (b) 6. (a) 7. (a) 8. (a)
1. (a) 2. (b) 3. (b) 4. (c)
9. (c) 10. (a) 11. (b) 12. (d)
5. (c) 6. (b) 7. (a,b,d) 8. (a, d)
13. (b) 14. (c) 15. (b) 16. (a)
9. (b,d) 10. f ( x ) = 4 − x 2
17. (d) 18. (d) 19. (a) 20. (d)
2 π –π
11. a = , b = e 2/3 12. a = 8 13. a = ,b = 21. (c) 22. (d) 23. (a) 24. (a)
3 6 12 25. (d) 26. (a,b,d) 27. (b,c,d) 28. (b,c)
2   2 1
3  log  3 – 6 x, x ≤ 0
29. (a,b,d) 30. (b,c,d) 31. (b,c)
14. f ( x ) =  32. (a,b) 33. (b,c) 34. (a,d) 35. (b,c)
1/ x
  1 + x  , x>0 36. (b, c) 37. (a, b, c, d) 38. (b, c) 39. (a, d)
  2 + x  40. (a, c, d) 41. (a, b) 42. (b, d) 43. (a, b, d)
202 Limit, Continuity and Differentiability

44. (b) 9. (c) 10.(b) 11. (a) 12. (d)


45. (A) → p, q, r, s; (B) → p, s; (C) → r, s; (D) → p, s 13. (a) 14.(a) 15. (d) 16. (b)
46. (A) → p; (B) → r 47. (24) 17. (c) 18.(a) 19. (b) 20. (d)
+ −
48. f ′( 0 ) = 0, f ′( 0 ) = 1 21. (c) 22.1 23. x ∈ R − { 0} 24. 1
 2 1
49. x = 0 50. True 51. (a = 1 ) 52. 1 −  25. –4 26. 27. 0
 π e
54. (1, 2 ) 55. (i) Yes (ii) No –2 ( x 2 – x – 1 ) 2  2x – 1 
28. ⋅ sin  2 
(x 2 + 1)2  x + 1
56. ( − 1 ) 57. e 2x 58. { 0, 1, 2 ) 60. f ′( 0 ) = 0
3
61. g ( x ) is differentiable for all x ∈ ( − 2, 2 ) − { 0, 1 ) 30.
π π2 –3
62. g ( x ) is continuous for all x ∈ ( 0, 2 ) − {1 ] and g ( x ) is
−8 32
differentiable for all x ∈ ( 0, 2 ) − {1 } 33. + 34. (11)
loge 2 16 + π 2
 2 1 π
63.  −  64.  +  65. (4) 3
 9 2 4 36. e x sin x (3 x 3 cos x 3 + sin x 3 ) + (tan x ) x [2 x cosec 2 x + log (tan x )]
66. (2) 67. (3) 68. p (2 ) = 0  5
– 2 sin ( 4 x + 2 ), x < 1
3 (1 – x ) 2
37.  –5
Topic 8  – 2 sin ( 4 x + 2 ), x > 1
2
1. (b) 2. (b) 3. (d) 4. (b) 3 ( x – 1 )
5. (d) 6. (b) 7. (b) 8. (b) 38. (7)

Hints & Solutions


0 ∞ Since, limit exist and equal to 5 and denominator is
Topic 1 and Form zero at x = 1 , so numerator x2 − ax + b should be zero at
0 ∞ x = 1,
1. Let So 1 − a + b =0 ⇒ a =1 + b …(ii)
x + 2 sin x 0 
P = lim form On putting the value of ‘a’ from Eq. (ii) in
x→ 0 0 
x + 2 sin x + 1 − sin x − x + 1
2 2
Eq. (i), we get
On rationalization, we get x2 − (1 + b) x + b (x2 − x) − b(x − 1)
lim = 5 ⇒ lim =5
(x + 2 sin x) x→1 x−1 x→1 x−1
P = lim 2
x → 0 x + 2 sin x + 1 − sin 2 x + x − 1 (x − 1) (x − b)
⇒ lim = 5 ⇒ lim (x − b) = 5
x→1 x−1 x→1
× ( x2 + 2 sin x + 1 + sin 2 x − x + 1 )
⇒ 1 − b =5 ⇒ b = −4 …(iii)
= lim ( x2 + 2 sin x + 1 + sin 2 x − x + 1 )
x→ 0 On putting value of ‘b’ from Eq. (iii) to Eq. (ii), we get
x + 2 sin x a = −3 ⇒ a + b = − 7
× lim
x→ 0 x − sin 2 x + 2 sin x + x
2
3. Given,
x + 2 sin x 0  x4 − 1 x3 − k3 (x − 1)(x + 1)(x2 + 1)
= 2 × lim form = lim 2 ⇒ lim
0 
x→ 0 x2 − sin 2 x + 2 sin x + x lim
x→1 x − 1 x→ k x − k 2 x→1 x −1
Now applying the L′ Hopital’s rule, we get (x − k)(x2 + k2 + xk) 3k2 8
1 + 2 cos x = lim ⇒ 2 ×2 = ⇒ k=
P = 2 × lim x→ k (x − k)(x + k) 2k 3
x → 0 2 x − sin 2 x +2 cos x + 1

(1 + 2) sin 2 x 0 
=2 [on applying limit] 4. Given limit is lim 0
form

0 −0 + 2 + 1 x→ 0 2 − 1 + cos x
3 sin 2 x  x
=2 × =2 = lim Q 1 + cos x = 2 cos 2
3 x→ 0 x  2 
2 − 2 cos
x + 2 sin x 2
⇒ lim =2
x→ 0
x2 + 2 sin x + 1 − sin 2 x − x + 1 sin 2 x
= lim
x→ 0  x
x2 − ax + b 2 1 − cos 
2. It is given that lim =5 …(i)  2
x→1 x−1
Limit, Continuity and Differentiability 203

sin 2 x  x x y4 1
= lim Q 1 − cos = 2 sin 2 = lim =
x→ 0  x  2 4  y→ 0 2 2 ×2
2 × 2 sin 2  y ( 1 + 1 + y + 2 ) ( 1 + y + 1)
4 4 4
 4
(by cancelling y4 and then by direct substitution).
x2 16 1
= lim 2
= =4 2 [lim sin x = lim x] = .
x→ 0  x 2 2 x→ 0 x→ 0
4 2
2 2 
 4 cot x − cos x 1 cos x(1 − sin x)
8. lim = lim ⋅
cot3 x − tan x x→ π / 2 (π − 2 x)3 x → π/ 2 8 π 
3
5. Given, limit = lim sin x − x
x → π /4  π 2 
cos  x + 
 4 π  π 
cos  − h 1 − sin  − h 
1 − tan 4 x 1  1  1 2  2 
= lim × Q cot x = = lim ⋅
x → π /4 1 3  tan x  h→ 0 8 π  π π 
3
(cos x − sin x) tan x sin  − h  − + h
2 2  2 2 
(1 − tan 2 x) 2 (1 + tan 2 x)
= lim × 1 sin h (1 − cos h )
x → π / 4 cos x − sin x tan3 x = lim
8 h→ 0 cos h ⋅ h3
cos 2 x − sin 2 x 2 (sec2 x)
= lim ×  h  h
x → π / 4 cos x − sin x cos 2 x tan3 x sin h 2 sin 2  sin h ⋅ sin 2 
1  2 1  2
[Q 1 + tan 2 x = sec2 x] = lim = lim
8 h→ 0 cos h ⋅ h3 4 h→ 0 h3 cos h
(cos x − sin x) (cos x + sin x) 2 sec4 x
= lim ×  h
2
(cos x − sin x) tan3 x
 sin h  
x → π /4 sin 
1 2 ⋅ 1 ⋅ 1 = 1 × 1 = 1
[Q (a − b ) = (a − b) (a + b)]
2 2 = lim   
4 h → 0 h   h  cos h 4 4 4 16
2 sec4 x  2 
= lim (cos x + sin x)
x → π /4 tan3 x sin(π cos 2 x) sin π (1 − sin 2 x)
4 9. lim = lim
2 ( 2)  1 1 x→0 x 2 x→ 0 x2
=  +  [on applying limit]
(1)3  2 2 sin(π − π sin 2 x)
= lim
 2 x→ 0 x2
= 4 2   = 8.
 2 sin(π sin 2 x)
= lim [Q sin (π − θ ) = sin θ ]
x→ 0 x2
x cot 4x x 1 tan 2 2x
6. lim = lim . sin π sin 2 x  sin 2 x sin θ
2 2
x→0 sin x. cot 2 x x→0 tan 4 x sin 2 x 1  
= lim × (π )  2  = π Q lim =1
2 2 x→ 0 π sin x
2
 x   θ→ 0 θ 
1 4x x tan 2x
= lim .
x→0 4 (tan 4 x) sin 2 x x2 (1 – cos 2x)(3 + cos x) 2 sin 2 x(3 + cos x)
2 2 10. We have, lim = lim
1 4x  x   tan 2x 4 x→ 0 x tan 4x x→ 0

tan 4x
× 4x
= lim   .  .
x→ 0 4 (tan 4x)  sin x  2x  1 4x
2 sin 2 x (3 + cos x) 1
1 4  x tan x  = lim × lim ×
= ⋅ 1 ⋅1 ⋅1 ⋅ = 1 Q lim = 1 = lim x2 tan 4x
4 1  x→ 0 sin x x→ 0 x 
x→ 0 x→ 0 4 lim
x→ 0 4x
1 + 1 + y4 − 2 4  sin θ tan 
= 2 × ×1 Q lim = 1 and lim =1
7. Clearly, lim 4 
 θ → 0 θ θ → 0 θ 
y→ 0 y4
=2
1 + 1 + y4 − 2 1 + 1 + y4 + 2
= lim × 11.  ∞  form
4 PLAN  
y→ 0 y 1 + 1 + y4 + 2 ∞
 0, if n< m
[rationalising the numerator]  a0
a0x + a1x
n n−1
+ K + an  , if n = m
lim =  b0
(1 + 1 + y4 ) − 2 x→∞ b 0x m + b1x m − 1 + K + b m
= lim [Q (a + b) (a − b) = a − b ]
2 2
 + ∞, if n> mand a0b 0 > 0
y→ 0
y4 ( 1 + 1 + y 4 + 2 )  − ∞, if n>m and a0b 0 < 0

Description of Situation As to make degree of
1 + y4 − 1 1 + y4 + 1
= lim × numerator equal to degree of denominator.
y→ 0
y4 ( 1 + 1 + y4 + 2 ) 1 + y4 + 1  x2 + x + 1 
∴ lim  − ax − b = 4
[again, rationalising the numerator] x→ ∞  x+1 
204 Limit, Continuity and Differentiability

x2 + x + 1 − ax2 − ax − bx − b  1 
⇒ lim =4 2x tan x −1
x→ ∞ x+1 1 − tan 2 x 
= lim
x→ 0 4 sin 4 x
x2(1 − a ) + x(1 − a − b) + (1 − b)
⇒ lim =4 1 − 1 + tan 2 x 
x→ ∞ x+1 2x tan x  
 1 − tan x 
2
Here, we make degree of numerator = lim
x→ 0 4 sin 4 x
= degree of denominator
3
∴ 1−a =0 ⇒ a =1  tan x
 ⋅x
3
x
x (1 − a − b) + (1 − b) 2x tan3 x 1  x 
and lim =4 = lim = lim
x → 0 2 sin 4 x (1 − tan 2 x) x → 0 2 sin 4 x (1 − tan 2 x)
x→ ∞ x+1
3
⇒ 1−a −b =4  tan x
 
1  x  1 ⋅ (1)3 1
⇒ b = − 4 [Q (1 − a ) = 0] = lim = =
2 x → 0  sin x 4 2(1) (1 − 0) 2
4
f (2h + 2 + h 2) − f (2)   (1 − tan x)
2
12. Here, lim  x 
h→ 0 f (h − h 2 + 1) − f (1)
[Q f ′ (2) = 6 and f ′ (1) = 4, given] 1 − cos 2 (x − 1)
16. LHL = lim−
Applying L’Hospital’s rule, x→1 x −1
{ f ′ (2h + 2 + h 2)} ⋅ (2 + 2h ) − 0 f ' (2) ⋅ 2 2 sin 2 (x − 1) |sin (x − 1)|
= lim = = lim = 2 lim
h → 0 { f ′ (h − h 2 + 1 )} ⋅ (1 − 2 h ) − 0 f ' (1) ⋅ 1 x → 1− x −1 x → 1− x −1
6 .2 Put x = 1 − h , h > 0, for x → 1− , h → 0
= =3 [using f ′ (2) = 6 and f ′ (1) = 4]
4 .1 |sin (− h )| sin h
= 2 lim = 2 lim =− 2
{(a − n ) nx − tan x} sin nx h→ 0 −h h → 0 −h
13. Given, lim =0
x→ 0 x2 1 − cos 2 (x − 1)
 tan x  sin n x Again, RHL = lim
⇒ lim (a − n ) n −  ×n =0 x→1 + x−1
x→ 0 x  nx
|sin (x − 1)|
1 = lim 2
⇒ {(a − n ) n − 1} n = 0 ⇒ (a − n ) n = 1 ⇒ a = n + x →1+ x−1
n
Put x = 1 + h, h > 0
(cos x − 1) (cos x − ex ) +
14. lim For x → 1 , h → 0
x→ 0 xn
|sin h | sin h
 x 2
x 4   = lim 2 = lim 2 = 2
 1 − + − ...  h→ 0 h h→ 0 h
 2 x  2! 4 !  
 −2 sin  
 2   x2 x3  ∴ LHL ≠ RHL.
− 1 + x + + + ...  Hence, lim
f (x) does not exist.
  2! 3!   x→1
= lim
x→ 0 x n 1
(1 − cos 2 x)
 2  2 1 .|sin x|
 x 2 x x3 17. lim = lim
 − 2 sin   − x − − − ...
2
x→ 0 x x→ 0 2 x
 2  2! 3! 
= lim 2 At x=0
x→ 0  x
4   xn − 2 RHL = lim
1 . sin h
=
1
 2 h→ 0 2 h 2
x x2  1 . sin h 1
sin 2 1 + x + + K and LHL = lim =−
2 3!  h→ 0 2 −h 2
= lim
x→ 0 2
 x Here, RHL ≠ LHL
2   xn − 3 ∴ Limit does not exist.
 2
sin [x]
Above limit is finite, if n − 3 = 0, i.e. n = 3.  , [x] ≠ 0
18. Since, f (x) =  [x]
x tan 2x − 2x tan x
15. lim  0, [x] = 0
x→ 0 (1 − cos 2x)2
sin [x]
NOTE In trigonometry try to make all trigonometric functions in  , x ∈ R − [0, 1)
⇒ f (x) =  [x]
same angle. It is called 3rd Golden rule of trigonometry.
 0, 0 ≤ x<1
2 tan x
x − 2x tan x At x = 0,
1 − tan 2 x
= lim RHL = lim 0 = 0
x→ 0 (2 sin 2 x)2 x→ 0 +
Limit, Continuity and Differentiability 205

sin [x] sin [0 − h ] (d) lim sec2( fn (x)) = lim (1 + tan 2 fn (x))
and LHL = lim = lim x→ ∞ x→ ∞
x→ 0 [x] − h→ 0 [0 − h ]
= 1 + lim tan 2( fn (x)) = 1 + 0 = 1
sin (−1) x→ ∞
= lim = sin 1 ∴ (d) statement is true.
h→ 0 −1
Since, RHL ≠ LHL x2
a − a 2 − x2 −
∴ Limit does not exist. 23. L = lim 4 , a >0
x→ 0 x4
 1 2 n 
19. lim  + +K+   1 1  
n→∞  1 − n 2 1 − n2 1 − n 2  1 x2 2  2 − 1 x4  x2
a − a ⋅ 1 − ⋅ 2 + ⋅ 4 − ... −
1+2+3+K+ n n (n + 1)  2 a 2 a  4
= lim = lim
n→∞ (1 − n 2) n→∞ 2 (1 − n ) (1 + n )  
= lim
⇒ lim
n
=−
1 x→ 0 x4
n→∞ 2 (1 − n ) 2 x2 1 x4 x2
+ ⋅ 3 + ... −
20. Given, f (a ) = 2, f ′ (a ) = 1, g (a ) = − 1, g ′ (a ) = 2 = lim 2a 8 a 4 4
x→ 0 x
g (x) f (a ) − g (a ) f (x)
∴ lim Since, L is finite
x→ a x−a
⇒ 2a = 4 ⇒ a = 2
g ′ (x) f (a ) − g (a ) f ′ (x) 1 1
= lim , ∴ L = lim =
x→ a 1 −0 x → 0 8 ⋅ a3 64
[using L’ Hospital’s rule]
log (1 + 2h ) − 2 log (1 + h ) 0 
= g ′ (a ) f (a ) − g (a ) f ′ (a ) = 2 (2) − (−1) (1) = 5 24. lim 0
form

h→ 0 h2
21. Given, G (x) = − 25 − x2 Applying L’Hospital’s rule, we get
G (x) − G (1) G ′ (x) − 0 2 2
∴ lim = lim −
x−1 1 −0 1 + 2h 1 + h
x→1 x→1 = lim
h→ 0 2h
[using L’ Hospital’s rule]
2 + 2h − 2 − 4h
= G′ (1) =
1 = lim
h → 0 2 h (1 + 2 h ) (1 + h )
24
  −1
2x = lim = −1
Q G (x) = − 25 − x2 ⇒ G ′ (x) =  h→ 0 (1 + 2h ) (1 + h )
 2 25 − x2 
sin x, x ≠ nπ , n = 0, ± 1, ± 2, K
22. We have, 25. Given, f (x) = 
 2, otherwise
n
 1 
fn (x) = ∑ tan − 1   for all x ∈ (0, ∞ ) { f (x)}2 + 1 , f (x) ≠ 0, 2
j=1
 1 + (x + j) (x + j − 1) 
g [ f (x)] =  4 , f (x) = 0
n
 (x + j) − (x + j − 1)  
⇒ fn (x) = ∑ tan − 1    5 , f (x) = 2
j=1
 1 + (x + j) (x + j − 1)
g [ f (x)] = (sin x) + 1, x ≠ , nπ = 0, ± 1, K
2
n ∴
 5 , x = nπ
⇒ fn (x) = ∑ [tan − 1 (x + j) − tan − 1 (x + j − 1)]
j=1 Now, lim g [ f (x)] = lim (sin 2 x) + 1 = 1
x→ 0 x→ 0
⇒ fn (x) = (tan − 1 (x + 1) − tan − 1 x)
+ (tan − 1 (x + 2) − tan − 1 (x + 1)) 26. Given, P = 2 ( 2hr − h 2 + 2hr )
−1 −1
+ (tan (x + 3) − tan (x + 2)) A
+ ... + (tan − 1 (x + n ) − tan − 1 (x + n − 1))
⇒ fn (x) = tan − 1 (x + n ) − tan − 1 x
This statement is false as x ≠ 0. i.e., x ∈ (0, ∞ ).
(b) This statement is also false as 0 ∉ (0, ∞ ) h
−1 −1
(c) fn (x) = tan (x + n ) − tan x r
h–r

lim tan( fn (x)) = lim tan(tan − 1 (x + n ) − tan − 1 x)


x→ ∞ x→ ∞
  B D C
n
⇒ lim tan( fn (x)) = lim tan  tan − 1 
x→ ∞ x→ ∞  1 + nx + x2
n
= lim =0 Here, BD = r 2 − (h − r )2 = 2hr − h 2
x → ∞ 1 + nx + x2
1
∴ A = . 2BD. h = ( 2hr − h 2) h
∴ (c) statement is false. 2
206 Limit, Continuity and Differentiability

h 2hr − h 2  h h

A
= lim a 2 ⋅ 2 cos  a +  ⋅ sin
lim 3  2 2
h→ 0 P h→ 0
8 ( 2hr − h 2 + 2hr )3 = lim + (2a + h ) sin (a + h )
h→ 0 h
2⋅
h3/ 2 ( 2r − h ) 2
= lim = a 2 cos a + 2a sin a
h→ 0 8 h3/ 2 ( 2r − h + 2r )3
x − sin x lim(x − sin x)1/ 2
1 2r 1 = x→ 0
= ⋅ = 33. lim
8 ( 2r + 2r )3 128 r x→ 0 x + cos 2 x lim(x + cos 2 x)1/ 2
x→ 0
1/ 2
 4 1 2 1   sin x 
 x sin + x  x4 sin + x2 lim x 1 − 
27. lim  x  = lim x x→ 0   x   0 .0
= = =0
x → −∞  1 + |x |3  x → −∞ 1 − x3 lim(0 + 1)1/ 2 1
  x→ 0

On dividing by x3 , we get  x −1  1 1
34. lim   = lim =−
sin (1 / x) 1 x→1  (x − 1 )(2 x − 5 )  x→1 (2 x − 5 ) 3
+
1 x x2 sin (βx)
x 1+0 35. Here, lim =1
lim = = −1 x → 0 αx − sin x
x → −∞ 1
−1 0 −1
3  (βx)3 (βx)5 
x x2  β x − + − K
 3! 5! 
 x3 + x2 − 16x + 20 ⇒ lim =1
 , if x ≠ 2 x→ 0  x3 x5 
28. f (x) =  (x − 2)2 αx −  x − + − K
 k , if x = 2  3! 5! 

Since, continuous at x = 2.  β3 x2 β 5 x4 
x3 β − + − K
x3 + x2 − 16x + 20  3! 5! 
⇒ f (2) = lim ,[using L’Hospital’s rule] ⇒ lim =1
x→ 2 (x − 2)2 x→ 0 x3 x5
(α − 1)x ++ −K
3! 5!
3x2 + 2x − 16 6x + 2
= lim = lim =7 Limit exists only, when α − 1 = 0
x→ 2 2 (x − 2) x→ 2 2
⇒ α =1 …(i)
∴ k=7  β3 2
x β5 x4 
πx x3 β − + − K
29. lim (1 − x) tan  3! 5! 
x→1 2 ∴ lim =1
x→ 0  1 x 2 
Put x−1 = y x  −
3
− K
3! 5! 
π  π 
∴ − lim y tan ( y + 1) = − lim y − cot  y  ⇒ 6β = 1 …(ii)
y→ 0 2 y→ 0  2 
From Eqs. (i) and (ii), we get
 π 
 y  2 2 6(α + β ) = 6α + 6 β = 6 + 1 = 7
= lim  2 ⋅ =  ecos ( α n ) − e 
y→ 0
 tan π y π π 36. Given, lim  =−
e
 2  α→ 0 α m
2
 
n
30. If lim [ f (x) g (x)] exists, then both lim f (x) and lim g (x) e { ecos( α ) −1 − 1} cos(α n ) − 1 − e
x→ a x→ a x→ a ⇒ lim ⋅ =
may or may not exist. Hence, it is a false statement.
α→ 0 cos(α n ) − 1 αm 2
αn
2x − 1 1+ x+1  ecos( α n ) −1 − 1  −2 sin 2
31. lim × ⇒ lim e  2 = − e /2
x→ 0 1 + x −1 1+ x+1  ⋅ lim
α→ 0  cos(α n ) − 1  α→ 0 α m
 
(2x − 1)( 1 + x + 1)  α n
= lim
x→ 0 x sin 2 
 2  α 2n −e
= log e (2) ⋅ (2) = 2 log e 2 = log e 4 ⇒ e × 1 × (−2) lim ⋅ m=
α→ 0 α 2n
4α 2
(a + h )2 sin (a + h ) − a 2 sin a 4
32. Here, lim
h→ 0 h α 2n − m − e
⇒ e × 1 × − 2 × 1 × lim =
a 2[sin (a + h ) − sin a ] α→ 0 4 2
= lim
h→ 0 h For this to be exists, 2n − m = 0
h [2a sin (a + h ) + h sin (a + h )] ⇒
m
=2
+
h n
Limit, Continuity and Differentiability 207

Topic 2 1∞ Form, RHL and LHL 3.


Key Idea lim f ( x) exist iff
x→a
1
lim f ( x) = lim f ( x)
 1 + f (3 + x) − f (3) x ∞ x → a+ x → a−
1. Let l = lim  [1 form]
x→ 0  1 + f (2 − x) − f (2 ) 
At x = 0,
1 1 + f (3 + x ) − f (3 ) 
lim  1 − 
x→ 0 x  1 + f ( 2 − x ) − f ( 2)  tan(π sin 2 x) + (|x| − sin(x [x]))2
⇒ l=e RHL = lim
x→ 0 +
x2
 1 + f ( 2 − x ) − f ( 2) − 1 − f (3 + x ) + f (3 ) 
lim  
x→ 0  x(1 + f ( 2 − x ) − f ( 2))
=e  tan(π sin 2 x) + (x − sin(x ⋅ 0))2
= lim
 f ( 2 − x ) − f (3 + x ) + f (3 ) − f ( 2) 
lim 
x→ 0 + x2

=e x→ 0  x(1 + f ( 2 − x ) − f ( 2))   Q|x| = x for x > 0 
and [x] = 0 for 0 < x < 1
On applying L’Hopital rule, we get  
 − f ′( 2 − x )− f ′(3 + x )  tan (π sin 2 x) + x2
lim  
x→ 0  1 − xf ′( 2 − x ) + f ( 2 − x ) − f ( 2)  = lim
l=e x → 0+ x2
On applying limit, we get  tan(π sin 2 x) π sin 2 x 
 − f ′( 2) − f ′(3 )  = lim  . + 1
 
 1 − 0 + f ( 2) − f ( 2) 
x→ 0 
+
π sin x
2
x 2

l=e = e0 = 1
tan (π sin 2 x) sin 2 x
1
= π lim . lim +1
 1 + f (3 + x) − f (3) x x→ 0 +
π sin x
2
x→ 0 +
x2
So, lim  =1
x→ 0  1 + f (2 − x) − f (2 )   tan x 
Q lim =1
 x→ 0 x 
lim π − 2 sin −1
x =π+1  sin x 
2. Let L = , then and xlim =1 
x → 1− 1−x  →0 x 
lim π − 2 sin −1 x π + 2 sin −1 x and LHL
L= × tan (π sin 2 x) + (|x| − sin (x [x])2
x → 1− 1−x π + 2 sin −1 x = lim
x→ 0 −
x2
[on rationalization]
lim π − 2 sin −1 x tan (π sin 2 x) + (− x − sin(x (− 1))2
1 = lim
= × x → 0− x2
x → 1− 1−x π + 2 sin −1 x
 Q|x| = − x for x < 0 
π  and [x] = − 1 for − 1 < x < 0
π − 2 − cos −1 x  
lim 2  1
= × tan(π sin 2 x) + (x + sin(− x))2
x → 1− 1−x π + 2 sin −1 x = lim
π x → 0− x2

Q sin −1 x + cos −1 x =
 2  tan(π sin 2 x) + (x − sin x)2
= lim
lim 2 cos −1 x lim 1 x → 0− x2
= − × [Qsin (− θ ) = − sin θ]
x→ 1 1−x x → 1− π + 2 sin −1 x
 tan(π sin x) + x + sin x − 2x sin x
2 2 2
1 lim 2 cos −1 x  lim −1 π = lim  
= − Q x → 1− sin x = 2  x → 0−  x2 
2 π x→ 1 1−x  
Put x = cos θ, then as x → 1− , therefore θ → 0+  tan(π sin 2 x) sin 2 x 2x sin x
= lim  +1+ − 
x → 0−  x2 
2
1 lim 2θ x x2
Now, L = +
2 π θ→0 1 − cos θ  tan (π sin 2 x) π sin 2 x sin 2 x sin x
2θ θ = lim  . + 1+ −2 
1 lim  x → 0−  π sin x
2 2
x2 x 
= Q 1 − cos θ = 2 sin 2 x
2 π θ→0
+
θ
  
 2 
2 sin   tan(π sin 2 x) π sin 2 x
 2 = lim . lim +
 θ x→ 0 −
π sin x
2
x→ 0 −
x2
2⋅   sin 2 x sin x
1  2 1 + lim − 2 lim
= ⋅ 2 lim x→ 0 −
x2 x → 0− x
2 π θ→ 0 +  θ
sin   = π + 1 + 1 −2 = π
 2
1 2  lim θ  Q RHL ≠ LHL
= 2 2 = Q x → 0+ sin θ = 1 ∴ Limit does not exist
2 π π  
208 Limit, Continuity and Differentiability

4. Given,  n  n n 
= −
 x  x  x 
Similarly,
π 
(1 − |x| + sin|1 − x|) sin  [1 − x]
2   1 1  2 2  15 15 
lim ∴Given limit = lim x −   + −   + … −  
|1 − x|[1 − x] + x x  x 
x →1+ x→ 0 x x x
Put x = 1 + h , then  1  2  15 
= lim (1 + 2 + 3+ ...+15) − x   +   + ... +  
x → 1+ ⇒ h → 0+ x→ 0 +  x x  x 
π   n  
(1 − |x| + sin|1 − x|) sin  [1 − x]
2  Q 0 ≤  x  < 1, therefore 
∴ lim = 120 − 0 = 120  
|1 − x|[1 − x] n  n 
x →1+ 0 ≤ x  < x ⇒ lim+ x  = 0
 x x→ 0 x 
π 
(1 − |h + 1| + sin|− h|) sin  [− h ] 1 − x( 1 + |1 − x|)  1 
2  f (x) = cos 
= lim 7. 
h→ 0 + |− h|[− h ] |1 − x|  1 − x
π  1 − x(1 + 1 − x)  1 
(1 − (h + 1) + sin h ) sin  [− h ] Now, lim f (x) = lim cos  
2  x→1 − x→1 − 1−x  1 − x
= lim
h→ 0 + h [− h ]  1 
= lim (1 − x) cos   =0
(Q|− h| = h and|h + 1| = h + 1 as h > 0) x→1 −  1 − x
π  1 − x(1 − 1 + x)  1 
( − h + sin h ) sin  (− 1) and lim f (x) = lim cos  
2  x→1 +
x→1 + x−1  1 − x
= lim
h→ 0 + h (− 1)
 1 
= lim − (x + 1) ⋅ cos   , which does not exist.
(Q [x] = − 1 for − 1 < x < 0 and h → 0+ ⇒ − h → 0− ) x→1 +  x + 1
(− h + sinh)  − π 1
= lim sin   8. Given, p = lim (1 + tan 2 x ) 2x (1∞ form)
h→ 0 + −h  2 
x → 0+
2
tan 2 x  tan x 
 sin h   h 1
lim   1
= lim   − lim+  
lim
x → 0+ 2x 2x → 0+  x 
h→ 0 +  h  h → 0  h =e =e = e2
1
 sin h   h  sin h  1
= lim   − lim+   = 1 − 1 = 0 Q lim+ = 1 ∴ log p = log e2 =
+    
h→ 0 h h→ 0 h  h→ 0 h  2

x([x] + | x|) sin [x] x([x] − x) sin [x] 9. PLAN To make the quadratic into simple form we should
5. lim = lim eliminate radical sign.
x→ 0 − | x| x→ 0 − −x
Description of Situation As for given equation,
(Q| x| = − x, if x < 0) when a → 0 the equation reduces to identity in x.
x(− 1 − x) sin (− 1) i.e. ax2 + bx + c = 0, ∀ x ∈ R or a = b = c → 0
= lim (Q lim [x] = − 1)
x→ 0 − −x x → 0−
Thus, first we should make above equation independent
− x(x + 1) sin(− 1) from coefficients as 0.
= lim = lim (x + 1)sin(− 1)
x → 0− −x x → 0−
Let a + 1 = t 6. Thus, when a → 0, t → 1.
∴ (t 2 − 1) x2 + (t3 − 1)x + (t − 1) = 0
= (0 + 1) sin (− 1) (by direct substitution)
= − sin 1 (Qsin(− θ) = − sin θ) ⇒ (t − 1) {(t + 1) x2 + (t 2 + t + 1) x + 1} = 0, as t → 1
2 x2 + 3 x + 1 = 0
6. Key Idea Use property of greatest integer function [ x ] = x − { x }.
⇒ 2 x2 + 2 x + x + 1 = 0
 1  2  15  ⇒ (2x + 1) (x + 1) = 0
lim x + + …+
x→ 0 +   x
   x   x  Thus, x = − 1, − 1 / 2
or lim α (a ) = − 1 / 2
We know, [x] = x − { x} a→ 0 +
1  1 1  and lim β (a ) = − 1
∴ = −
 x  x  x  a→ 0 +
Limit, Continuity and Differentiability 209

x+4 x+ 4
10. Here, lim {1 + x log (1 + b2)}1/ x [1∞ form]  x + 6  5 
x→ 0 15. lim   = lim 1 +  [1∞ form]
1 2 x → ∞  x + 1 x→ ∞  x + 1
lim {x log (1 + b )} ⋅
x→ 0
=e x
lim
5 ( x + 4)
x→ ∞ x +1
=e log (1 + b 2 )
= (1 + b ) 2
…(i) =e = e5
1/ x
Given, lim {1 + x log (1 + b2)}1/ x = 2b sin 2 θ  π 
x→ 0 16. lim tan  + x 
⇒ (1 + b2) = 2b sin 2 θ x→ 0 4 
1/ x
1 + b2  π 
∴ sin 2 θ = …(ii) tan + tan x 1 + tan x 
1/ x
 4 
2b = lim   = lim  
x→ 0 π x → 0  1 − tan x 
1 1 − tan tan x 
b+ 1/ 2  4 
By AM ≥ GM, b ≥  b ⋅ 1 
 b [(1 + tan x)1/tan x ]tan x/ x e1
2 = lim = = e2
x → 0 [(1 − tan x)−1/tan x ]− tan x/ x e−1
b2 + 1
⇒ ≥1 …(iii)
2b 17. The right hand limit
From Eqs. (ii) and (iii), (1 − x)1/ x − e−1
lim
sin 2 θ = 1 x →0 + xa
π 1 
 log e (1 − x )
⇒ θ = ± , as θ ∈ (− π , π ]
2 e x 
− e−1
= lim a
sin x x →0 + x
11. Here, lim (sin x)1/ x + lim  
1
1  x2 x3 
x→ 0   x→ 0 x −x− − − K
x 
− e−1
2 3 
sin x e
1
log   lim
log (1/ x )
 lim (sin x)1/ x → 0  = lim
= 0 + lim e  x
=e x → 0 cosec x x →0 + xa
x → 0 ∞ 
as, (decimal) → 0
x→ 0  x x2 x3 
− − 
 2 3 − 4 − ...... 
Applying L’Hospital’s rule, we get e−1 . e  
− e−1
= lim a
 1  +
x −  x →0 x
 x2  sin x
 x x2 x3 
lim lim tan x
x → 0 − cosec x cot x x→ 0 x −  + + + ... 
e =e = e0 = 1 2 3 4 
e −1
1/ x = e−1 lim
 f (1 + x)  x →0 + xa
12. Let y = 
 f (1)  The above limit will be non-zero, if a = 1. And at a = 1,
1 the value of the limit is
⇒ log y = [log f (1 + x) − log f (1)]  1 1
x = e−1  −  = −
 2 2e
 1 
⇒ lim log y = lim  ⋅ f ′ (1 + x)
x→ 0 x → 0  f (1 + x)  sin x
18. PLAN lim =1
x→0 x
[using L’ Hospital’s rule] (1 + x ) (1 − x )

=
f (1) 6
= sin (x − 1) + a (1 − x)  1− x 1
Given, lim   =
f (1) 3 x → 1  (x − 1 ) + sin (x − 1 )  4
 
⇒ log  lim y = 2  sin (x − 1) 
1+ x
 x→ 0  −a
 (x − 1)  1
⇒ lim y = e2 lim  =
x→1 sin(x − 1)  4
x→ 0 x 1 + 
 x − 3 (1 − 3 / x)x e−3
13. For x ∈ R , lim   = lim = 2 = e−5  (x − 1) 
x→ ∞  x + 2 x→∞ (1 + 2 / x)x e 2
1 − a 1
⇒   = ⇒ (a − 1)2 = 1
2  1/ x
2
lim [(1 + 5x ) 2 1/5 x 2 5
]  2  4
 1 + 5x x→ 0 e5
14. lim   = = = e2 ⇒ a = 2 or 0
x → 0  1 + 3 x2  2 1/3 x 2 3 e3
lim [(1 + 3x ) ]
x→ 0 Hence, the maximum value of a is 2.
210 Limit, Continuity and Differentiability

Topic 3 Squeeze, Newton-Leibnitz’s Theorem and Limit Based


on Coverting infinite Serie s into Definite Integrals
1. Given α and β are roots of quadratic equation 4. Here,
x
375x2 − 25x − 2 = 0  n 
n
 n 
∴ α+β=
25
=
1  n n (x + n )  x +  … x +  
… (i)  2  n 
375 15 f (x) = lim  ,x>0
n→ ∞  n 2 
2  2
2 
n
2  n ! (x + n )  x +  K  x2 + 2  
2
and αβ = − … (ii)  4  n  
375 
n n
Now, lim
n→ ∞
∑ α r + nlim
→∞
∑ βr Taking log on both sides, we get
x
r =1 r =1  n 
n
 n 
= (α + α 2 + α 3 + K + upto infinite terms)+  n n (x + n )  x +  K  x +  
 2  n 
log e { f (x)} = lim log 
(β + β 2 + β3 + K + upto infinite terms) n →∞  2  2 n 
2  n 2 
 n !(x + n )  x +  K  x2 + 2  
2
α β  a    4  n  
= + Q S ∞ = 1 − r for GP
1 −α 1 −β    
n
 1 
 Π x +  
α (1 − β) + β (1 − α ) x r =1  r /n 
= = lim ⋅ log  
(1 − α ) (1 − β ) n→ ∞ n  n  2 1  n 
α − αβ + β − αβ  rΠ x +
=1 
Π (r / n ) 
2 r = 1

=  (r / n ) 
1 − α − β + αβ  
 n 
(α + β ) − 2αβ x+
= 1 n  
1 − (α + β ) + αβ = x lim
n→ ∞ n
∑ log
  n
r
2
r 
r =1
x + 2
2
1 −2 
On substituting the value α + β = and αβ = from   r  n 
15 375
Eqs. (i) and (ii) respectively,  r 
we get n  ⋅x + 1 
1
1 4 = x lim
n→ ∞ n
∑ log  n2
r


+ r =1
 2 ⋅ x + 1
2
29 29 1 n 
= 15 375 = = =
1−
1

2 375 − 25 − 2 348 12
Converting summation into definite integration, we get
15 375
1  xt + 1 
sec 2 x log e{ f (x)} = x ∫ log  2 2  dt
 x t + 1
∫ f (t ) dt
0

2 0  Put, tx=z
2. lim 0
form

x→
π π 2
⇒ xdt = dz
4 x2 −
16 x  1 + z  dz
f (sec2 x) 2 sec x sec x tan x ∴ log e { f (x)} = x ∫ log  
= lim 0  1 + z 2 x
x→ π /4 2x
x  1+ z
[using L’ Hospital’s rule] ⇒ log e{ f (x)} = ∫ log   dz
2 f (2) 8 0  1 + z 2
= = f (2)
π /4 π Using Newton-Leibnitz formula, we get
2n 2n
1 r 1 r 1  1 + x
3. Let I = lim
n→ ∞ n
∑ = lim
n→ ∞ n
∑ ⋅ f ′ (x) = log  
 1 + x2 
… (i)
r =1 n +r
2 2
r =1 n 1 + (r / n )2 f (x)
2n
1 r /n Here, at x = 1 ,
= lim
n→ ∞
∑ f ′ (1)
n r =1 1 + (r / n ) 2
= log (1) = 0
f (1)
2 x
=∫ dx = [ 1 + x2 ]2 = 5 − 1
0
∴ f′ (1) = 0
0
1 + x2
Limit, Continuity and Differentiability 211

Now, sign scheme of f ′ (x) is shown below x2

+ – ∫0 cos 2tdt
0 
6. lim 0
form

x→ 0 x sin x
x=1
Applying L’Hospital’s rule, we get
∴ At x = 1 , function attains maximum.
cos 2(x2) ⋅ 2x − 0 2 ⋅ cos 2(x2) 2
Since, f (x) increases on (0, 1). = lim = lim = =1
x → 0 x cos x + sin x x→ 0 sin x 1 + 1
∴ f (1) > f (1 / 2) cos x +
x
∴ Option (a) is incorrect.
f (1 / 3) < f (2 / 3)
∴Option (b) is correct.
Topic 4 Continuity at a Point
1. Given function is
Also, f ′ (x) < 0, when x > 1  2 cos x − 1 π
 ,x≠
⇒ f′ (2) < 0 f (x) =  cot x − 1 4
 π
∴ Option (c) is correct. k , x=
 1 + x  4
f ′ (x)
Also, = log  
f (x)  1 + x2 Q Function f (x) is continuous, so it is continuous at
π
f ′ (3) f ′ (2) 4  3 x= .
∴ − = log   − log   4
f (3) f (2)  10  5
 π
∴ f   = lim f (x)
= log (2 / 3) < 0  4  x→ π
f ′ (3) f ′ (2) 4
⇒ < lim
f (3) f (2) 2 cos x − 1
⇒ k= π
∴ Option (d) is incorrect. x→ cot x − 1
4
5. We have, π π
Put x = + h, when x → , then h → 0
1 4 4
yn = [(n + 1) (n + 2) … (n + n )]1/ n
n π 
2 cos  + h − 1
and lim yn = L lim 4 
n→ ∞ k=
1
⇒ L = lim [(n + 1) (n + 2) (n + 3) … (n + n )]1/ n h→0 π 
cot + h − 1
n→ ∞ n 4 
1
 1  2  3  n  n  1 1 
⇒ L = lim 1 +  1 +  1 +  ... 1 +   2 cos h − sin h −1
n→∞   n   n   n   n  lim  2 2 
=
h→0 cot h − 1
⇒ −1
1   1  2  n  cot h + 1
log L = lim log 1 + n  + log 1 + n  … log 1 + n  
n→ ∞ n   [Q cos (x + y) = cos x cos y − sin x sin y and
n cot x cot y − 1
1  r cot (x + y) = ]
⇒ log L = lim
n→ ∞ n ∑ log 1 + n  cot y + cot x
r =1
lim cos h − sin h − 1
1
⇒ log L = ∫ 1 × log (1 + x) dx =
0 II h→0 −2
I
1
1 + cot h
d 
⇒ log L = (x ⋅ log (1 + x))10 − ∫ (log(1 + x) ∫ dx dx
0
 dx  lim  (1 − cos h ) + sin h 
=  (sin h + cos h )
[by using integration by parts] h→0  2 sin h 
1 x
⇒ log L = [x log(1 + x)]0 − ∫
1
dx  2h h h 
01 + x
lim 2 sin 2 + 2 sin 2 cos 2 
= (sin h + cos h )
1 x + 1 1  h→0 h h
⇒ log L = log 2 − ∫  −  dx  4 sin cos 
0x+ 1 x + 1  2 2 
⇒ log L = log 2 − [x]10 + [log(x + 1)]10  h h 
⇒ log L = log 2 − 1 + log 2 − 0 lim sin 2 + cos 2 
= × (sin h + cos h )
⇒ log L = log 4 − log e = log
4
⇒ L=
4
⇒ h→0 h
 2 cos 
e e  2 
4 
[L ] = =1 1
 e  ⇒k=
2
212 Limit, Continuity and Differentiability

2. NOTE All integers are critical point for greatest integer function. 6. For f (x) to be continuous, we must have
Case I When x ∈ I f (0) = lim f (x)
x→ 0
f (x) = [x]2 − [x2] = x2 − x2 = 0 log (1 + ax) − log (1 − bx)
= lim
Case II When x ∉ I x→ 0 x
If 0 < x < 1, then [x] = 0 a log (1 + ax) b log (1 − bx)
= lim +
x→ 0 ax − bx
and 0 < x2 < 1, then [x2] = 0
log (1 + x)
Next, if 1 ≤ x2 < 2 ⇒ 1 ≤ x < 2 = a ⋅1 + b ⋅1 [using lim = 1]
x→ 0 x
⇒ [x] = 1 and [x2] = 1 =a+b
Therefore, f (x) = [x]2 − [x2] = 0, if 1 ≤ x < 2 ∴ f (0) = (a + b)
Therefore, f (x) = 0, if 0 ≤ x < 2 7. f (x) = x cos(π (x + [x]))
This shows that f (x) is continuous at x = 1. At x = 0
Therefore, f (x) is discontinuous in (−∞ , 0) ∪ [ 2 , ∞ ) on
lim f (x) = lim x cos(π (x + [x]) = 0
many other points. Therefore, (b) is the answer. x→ 0 x →0
and f (x) = 0
3. Given, f (x) = [tan x] 2
∴It is continuous at x = 0 and clearly discontinuous at
Now, − 45° < x < 45° other integer points.
⇒ tan (− 45° ) < tan x < tan (45° ) 8. PLAN If a continuous function has values of opposite sign inside an
⇒ − tan 45° < tan x < tan (45° ) interval , then it has a root in that interval.
f,g : [0, 1] → R
⇒ −1 < tan x < 1
⇒ 0 < tan 2 x < 1 ⇒ [tan 2 x] = 0 We take two cases.

i.e. f (x) is zero for all values of x from x = − 45° to 45°. Case I Let f and g attain their common maximum
value at p.
Thus, f (x) exists when x → 0 and also it is continuous at
x = 0 . Also, f (x) is differentiable at x = 0 and has a value ⇒ f ( p) = g ( p),
of zero. where p ∈ [0, 1]
Therefore, (b) is the answer. Case II Let f and g attain their common maximum
2x − 1 value at different points.
4. Here, f (x) = [x] cos  π ⇒ f (a ) = M and g (b) = M
 2 
  2x − 1 ⇒ f (a ) − g (a ) > 0 and f (b) − g (b) < 0
− cos  2  π , −1 ≤ x < 0
⇒ f (c) − g (c) = 0 for some c ∈ [0, 1] as f and g are
0 , 0 ≤ x<1 continuous functions.

∴ f (x) =  cos  2x − 1 π , 1 ≤ x<2 ⇒ f (c) − g (c) = 0 for some c ∈ [0, 1] for all cases.
  ...(i)
  2 
  2 x − 1  Option (a) ⇒ f 2(c) − g 2(c) + 3 [ f (c) − g (c)] = 0
2 cos  2  π , 2 ≤ x<3
 which is true from Eq. (i).
which shows RHL = LHL at x = n ∈ Integer as if x = 1 Option (d) ⇒ f 2(c) − g 2(c) = 0 which is true from Eq. (i)
 2x − 1 Now, if we take f (x) = 1 and g (x) = 1, ∀x ∈ [0, 1]
⇒ lim cos   π = 0 and lim− 0 = 0
x →1+  2  x→1 Options (b) and (c) does not hold. Hence, options (a) and
Also, f (1) = 0 (d) are correct.
∴ Continuous at x = 1. 9. f (2n ) = a n , f (2n + ) = a n
Similarly, when x = 2, f (2n − ) = bn + 1
lim f (x) = lim f (x) = 0 ⇒ a n − bn = 1
x → 2+ x → 2−
Thus, function is discontinuous at no x. f (2n + 1) = a n
Hence, option (c) is the correct answer. f {(2n + 1)− } = a n
5. Given, f (x) = x ( x + x + 1 ) f {(2n + 1)+ } = bn + 1 − 1
⇒ f (x) would exists when x ≥ 0 and x + 1 ≥ 0. ⇒ a n = bn + 1 − 1 or a n − bn + 1 = − 1
⇒ f (x) would exists when x ≥ 0. or a n − 1 − bn = − 1
∴ f (x) is not continuous at x = 0, 10. Given, x + y =4
2 2
⇒ y = 4 − x2
because LHL does not exist. or f (x) = 4 − x2
Hence, option (c) is correct.
Limit, Continuity and Differentiability 213

 {1 + |sin x|} a/|sin x | , π/6 < x < 0  1 + x


1/ x

 ⇒ f (x) =   ,x>0
11. f (x) =  b , x=0  2 + x
 e tan 2x /tan 3 x
, 0 < x < π /6 1
 ⇒ log f (x) = [log (1 + x) − log (2 + x)]
x
Since, f (x) is continuous at x = 0.
On differentiating both sides, we get
∴ RHL (at x = 0) = LHL (at x = 0) = f (0)
 1 1    1 + x 
⇒ lim etan 2 h/tan 3 h = lim {1 + |sin h |}a /|sin h| = b x − − 1 log  
h→ 0 h→ 0 f ′ (x)  1 + x 2 + x
  2 + x 
=
⇒ e 23
/
= ea = b f (x) x2
∴ a = 2 /3  x  1 + x 
1/ x  − log  
 1 + x (1 + x)(2 + x)  2 + x 
b = e 23   
/
and ∴ f ′ (x) = 
 2 + x  x2 
12. Since, f (x) is continuous at x = 0.  
 
∴ f (0) = LHL 2 1  2 
1 − cos 4h ⇒ f ′ (1) =  − log   
⇒ a = lim 3 6  3 
h→ 0 h2 and f (−1) = − a + b = − a [from Eq. (i)]
2 sin 2 2h 4 2 1  2 
⇒ a = lim × ∴ − a =  − log   
h→ 0 h2 4 3 6  3 
⇒ a =8 2    1
2
3  log  3 − 6 x, x ≤ 0
13. Since, f (x) is continuous for 0 ≤ x ≤ π 
Thus, f (x) =  1/ x
 π  π   1 + x
∴ RHL at x =  = LHL at x =    , x>0
 4  4   2 + x
 π π  π π Now, to check continuity of f (x) (at x = 0).
⇒ 2 ⋅ cot + b =  + a 2 ⋅ sin 
 4 4  4 4 1/ x
π π π 1 + x
⇒ + b= + a ⇒ a−b= …(i) RHL = lim   =0
x → 0 2 + x
2 4 4
 π  π 2   2 1 
RHL at x =  = LHL at x =  ∴ LHL = lim log  3 − 6  x = 0
Also,
 2  2 x→ 0 3  
 2π π  π π  Hence, f (x) is continuous for all x.
⇒  a cos − b sin  = 2 ⋅ ⋅ cot + b
 2 2  2 2 
sin (a + 1) x + sin x
 , x<0
⇒ −a−b=b x

⇒ a + 2b = 0 …(ii) 15. Given, f (x) =  c , x=0
On solving Eqs. (i) and (ii), we get  (x + bx2)1/ 2 − x1/ 2
 , x>0
π π  bx3/ 2
a= and b = −
6 12 is continuous at x = 0.

14. Let g (x) = ax + b be a polynomial of degree one. ⇒ (LHL at x = 0) = (RHL at x = 0) = f (0)


 sin (a + 1) x sin x 
 ax + b, x≤0 ⇒ lim +
x→ 0 
 x x 
 1/ x
⇒ f (x) =  1 + x
 2 + x , x > 0 (1 + bx)1/ 2 − 1
 = lim =c
x→ 0 bx
Since, f (x) is continuous and f ′ (1) = f (−1) bx 1
⇒ (a + 1) + 1 = lim ⋅ =c
∴ (LHL at x = 0) = (RHL at x = 0) x→ 0 bx 1 + bx + 1
1/ x 1
 x + 1 ⇒ a+2= =c
⇒ lim (ax + b) = lim   2
x→ 0 x→ 0  x + 2 3 1
∴ a =− ,c=
⇒ b =0 …(i) 2 2
Also, f ′ (1) = f (−1) and b ∈R
214 Limit, Continuity and Differentiability

2 2x sin  1  − cos  1  , x ≠ 0


16. (i) Given, f1 : R → R and f1 (x) = sin ( 1 − e−x ) 
f4 ′ (x) =     
Thus,  x  x
∴ f1 (x) is continuous at x = 0  0, x=0
 1 
Again, lim f ′ (x) = lim  2x sin   − cos   
2 1 2
Now, f1 ′ (x) = cos 1 − e− x . (2xe− x ) 1
2 1− e −x 2 x→ 0 x→ 0   x  x 
At x = 0 does not exists.
f1 ′ (x) does not exists. Since, lim cos   does not exists.
1
x→ 0  x
∴ f1 (x) is not differential at x = 0
Hence, option (2) for P. Hence, f ′ (x) is not continuous at x = 0.
 |sin x| , if x ≠ 0 ∴ Option (3) for S.

(ii) Given, f2 (x) =  tan −1 x
 1, if x = 0 Topic 5 Continuity in a Domain
 − sin x x< 0
f ( x)
 tan −1 x 1. Given ∫ 4t3 dt = (x − 2) g (x)
6
 f ( x)
 sin x
⇒ f2 (x) =  −1
x> 0


g (x) = 6
4t3 dt
[provided x ≠ 2]
 tan x (x − 2)
 1 x=0
 f ( x)

So, lim g (x) = lim
∫6 4t3 dt
Clearly, f2 (x) is not continuous at x = 0. x→ 2 x→ 2 x−2
∴ Option (1)for Q.  0 
Q form as x → 2 ⇒ f (2) = 6
 0 
(iii) Given, f3 (x) = [sin (log e (x + 2))] , where [ ] is G.I.F.
and f3 : (−1, eπ / 2 − 2) → R 4( f (x))3 f ′ (x)
lim g (x) = lim
It is given − 1 < x < eπ / 2 − 2 x→ 2 x→ 2 1
π/ 2
⇒ −1 + 2 < x+ 2 < e −2 + 2  d φ 2( x ) 
⇒ 1 < x + 2 < eπ/ 2
Q
 dx
∫φ 1 ( x)
f (t ) dt = f (φ 2(x)), φ 2′ (x) − f (φ1 (x)) ⋅ φ1′ (x)

⇒ log e 1 < log e (x + 2) < log e eπ / 2 On applying limit, we get
π
⇒ 0 < log e (x + 2) < 1 4 × 216
2 lim g (x) = 4( f (2))3 f ′ (2) = 4 × (6)3 = = 18
x→ 2 48 48
π
⇒ sin 0 < sin log e (x + 2) < sin
2  1
Q f (2) = 6 and f ′ (2) =
⇒ 0 < sin log e (x + 2) < 1  48 
∴ [sin log e (x + 2)] = 0 2. Given function
∴ f3 (x) = 0, f ′3 (x) = f3 ′ ′ (x) = 0 sin( p + 1)x + sin x
It is differentiable and continuous at x = 0.  , x<0
 x
∴Option (4) for R f (x) =  q , x =0
x2 sin  1  , if x ≠ 0 
    x+x − x
2
(iv) Given, f4 (x) =   x , x>0
 x3 / 2
 0, if x = 0
 1 is continuous at x =`0, then
Now, lim f4 (x) = lim x2 sin   = 0 f (0) = lim f (x) = lim f (x) …(i)
x→ 0 x→ 0  x
x→ 0 − x→ 0 +
 1  1 sin( p + 1)x + sin x
f4′ (x) = 2x sin   − cos   lim f (x) = lim
 x  x x→ 0 − x→ 0 − x
f (0 + h ) − f (0)  sin(ax) 
For x = 0, f4′ (x) = lim = p+1+1=p+2 Q lim =a
h→ 0 h  x→ 0 x 
 1
h 2 sin   − 0 x + x2 − x
 h and lim f (x) = lim
⇒ f4′ (x) = lim x→ 0 + x→ 0 + x3/ 2
h→ 0 h
x [(1 + x)1/ 2 − 1]
 1 = lim
⇒ f4′ (x) = lim h sin   = 0 x→ 0 + x x
h→ 0  h
Limit, Continuity and Differentiability 215

 1 1   5. Given function f : [−1, 3] → R is defined as


  − 1 
1 2 2  2
1 + x + x + .... − 1 |x| + [x], −1 ≤ x < 1
 2 2!  
  f (x) =  x + |x|, 1 ≤ x < 2
   x + [x ], 2 ≤ x ≤ 3
= lim 
+ x
x→ 0
− x − 1 , − 1 ≤ x < 0
[Q (1 + x)n  x,
n (n − 1) 2 n (n − 1(n − 2)) 3 0 ≤ x<1
= 1 + nx + x + x + ... ,|x|< 1] 
1 ⋅2 1 ⋅2 ⋅3 =  2x, 1 ≤ x<2
 x + 2, 2 ≤ x < 3
 1 1   
  − 1 
= lim  +
1 2 2 
x + ... =
1  6, x=3
+ 2 2!  2
x→ 0
  [Q if n ≤ x < n + 1, ∀ n ∈ Integer, [x] = n]
 
Q lim f (x) = − 1 ≠ f (0) [Q f (0) = 0]
From Eq. (i), we get x→ 0 −
1 1
f (0) = q = and lim f (x) = p + 2 = Q lim f (x) = 1 ≠ f (1) [Q f (1) = 2]
2 x→ 0 − 2 x→1 −
3 Q lim f (x) = 4 = f (2) = lim f (x) = 4 [Q f (2) = 4]
⇒ p=− x→ 2− x→ 2 +
2
and lim f (x) = 5 ≠ f (3) [Q f (3) = 6]
 3 1 x→ 3 −
So, ( p, q ) =  − , 
 2 2
∴ Function f (x) is discontinuous at points 0, 1 and 3.
3. Given function
6.
a|π − x| + 1, x ≤ 5 Key Idea A function is said to be continuous if it is continuous at
f (x) =  each point of the domain.
 b|x − π| + 3, x > 5
and it is also given that f (x) is continuous at x = 5. We have,
Clearly, f (5) = a (5 − π ) + 1 …(i)  5 if x≤1
a + bx if 1 < x < 3
lim f (x) = lim[a|π − (5 − h )| + 1] f (x) = 
x→5 − h→ 0
= a(5 − π ) + 1 …(ii)  b + 5x if 3 ≤ x < 5
 30 if x≥5
and lim f (x) = lim[b| (5 + h ) − π| + 3]
x→5 + h→ 0
= b(5 − π ) + 3 …(iii) Clearly, for f (x) to be continuous, it has to be continuous
at x = 1, x = 3 and x = 5
Q Function f (x) is continuous at x = 5.
[Q In rest portion it is continuous everywhere]
∴f (5) = lim f (x) = lim f (x) ∴ lim (a + bx) = a + b = 5 …(i)
x→5 + x→5 −
x →1+
⇒ a (5 − π ) + 1 = b(5 − π ) + 3
⇒ (a − b)(5 − π ) = 2 [Q lim f (x) = lim f (x) = f (1)]
x→1 − x→1 +
2
⇒ a−b= lim (b + 5x) = b + 25 = 30 …(ii)
5−π x →5−
[Q lim f (x) = lim f (x) = f (5)]
4. Given function f (x) = [x] −  , x ∈ R x →5− x→5 +
x
4  On solving Eqs. (i) and (ii), we get b = 5 and a = 0
 4 + h  Now, let us check the continuity of f (x) at x = 3.
Now, lim f (x) = lim  [4 + h ] − 
x→ 4 + h→ 0   4  Here, lim (a + bx) = a + 3b = 15
x →3−
[Q put x = 4 + h, when x → 4+ , then h → 0]
= lim(4 − 1) = 3 and lim (b + 5x) = b + 15 = 20
h→ 0 x→3 +
 4 − h 
and lim f (x) = lim [4 − h ] −
x→ 4 − h→ 0   4  Hence, for a = 0 and b = 5, f (x) is not continuous at x = 3
− ∴f (x) cannot be continuous for any values of a and b.
[Q put x = 4 − h, when x → 4 then h → 0]
1
= lim(3 − 0) = 3 7. Given, f (x) = x − 1 for 0 ≤ x ≤ π
h→ 0 2
4 
and f (4) = [4] − =4 −1 =3 − 1 , 0 ≤ x < 2
4  ∴ [ f (x)] = 
 0, 2 ≤ x ≤ π
Q lim f (x) = f (4) = lim f (x) = 3
x→ 4 − x→ 4 + tan (−1), 0 ≤ x < 2
⇒ tan [ f (x)] = 
So, function f (x) is continuous at x = 4.  tan 0, 2≤x≤π
216 Limit, Continuity and Differentiability

∴ lim tan [ f (x)] = − tan 1  π 


x→ 2− 10. f (x) = [x] sin  
 [x + 1]
and lim tan [ f (x)] = 0
x→ 2 + We know that, [x] is continuous on R ~ I, where I
So, tan f (x) is not continuous at x = 2.  π 
denotes the set of integers and sin   is
1 x−2 1 2  [x + 1]
Now, f (x) = x − 1 ⇒ f (x) = ⇒ =
2 2 f (x) x − 2 discontinuous for [x + 1] = 0.
Clearly, 1 /f (x) is not continuous at x = 2 . ⇒ 0 ≤ x+ 1 <1 ⇒ −1 ≤ x<0
 1  Thus, the function is defined in the interval.
So, tan [ f (x)] and  are both discontinuous at x = 2 .
 f (x)   x2
 , 0 ≤ x<1
π 11. Given, f (x) =  2 …(i)
8. The function f (x) = tan x is not defined at x = , so
2 2x2 − 3x + 3 , 1 ≤ x ≤ 2
f (x) is not continuous on (0, π ).  2
1 Clearly, RHL (at x = 1) = 1 / 2 and LHL (at x = 1) = 1 / 2
(b) Since, g (x) = x sin is continuous on (0, π ) and the
x Also, f (x) = 1 / 2
integral function of a continuous function is ∴ f (x) is continuous for all x ∈[0, 2].
continuous,
x 
On differentiating Eq. (i), we get
1
∴ f (x) = ∫ t sin  dt is continuous on (0, π ).  x , 0 ≤ x<1
0  t f ′ (x) =  …(ii)
 4 x − 3 , 1 ≤ x≤2
 3π
1, 0<x≤
 4 Clearly, RHL (at x = 1) for f ′ (x) = 1
(c) Also, f (x) = 
 2x 3π and LHL (at x = 1) for f ′ (x) = 1
2 sin   , <x<π
  9 4 Also, f (1) = 1
Thus, f ′ (x) is continuous for all x ∈[0, 2].
We have, lim f (x) = 1
3π− Again, differentiating Eq. (ii), we get
x→
4
 2x 1 , 0 ≤ x < 1
lim f (x) = lim 2 sin   = 1 f ′ ′ (x) = 
3π +
x→
3 π  9 4 , 1 ≤ x ≤ 2
x→ 4
4
Clearly, RHL (at x = 1) ≠ LHL (at x = 1)
So, f (x) is continuous at x = 3π / 4.
Thus, f ′ ′ (x) is not continuous at x = 1.
⇒ f (x) is continuous at all other points.
or f ′ ′ (x) is continuous for all x ∈ [0, 2] − {1}.
π  π π
(d) Finally, f (x) = sin (x + π ) ⇒ f   = −
12. The given function ‘f’ defined on  − ,  by
2  2  2 1 1
 3 3
π  π  3π  π
lim f (x) = lim f  − h = lim sin  − h = 1  1 + 3x
π
− h→ 0 2  h→ 0 2  2  2  log e   , where x ≠ 0
x→  
 2 f (x) =  x  1 − 2x 
 where x = 0
π   k,
and lim f (x) = lim f  + h
x→ ( π / 2)
+ h→ 0 2  is continuous, then
π  3π  π 1  1 + 3x
= lim sin  + h = k = lim log e  
h→ 0 2  2  2 x→ 0 x  1 − 2x 
So, f (x) is not continuous at x = π / 2.  log e (1 + 3x) log e (1 − 2x) 
= lim −
x→ 0 
 x x 
9. We have, for −1 < x < 1 ⇒ 0 ≤ x sin π x ≤ 1 / 2
= 3 − (−2) = 5 ⇒ k = 5
∴ [x sin πx] = 0
Also, x sin πx becomes negative and numerically less
than 1 when x is slightly greater than 1 and so by
Topic 6 Continuity for Composition and
definition of [x]. Function
f (x) = [x sin π x] = − 1, when 1 < x < 1 + h 1
1. Given, f (x) = x cos , x ≥ 1 ⇒ f ′ (x) = sin
1 1
+ cos
1
Thus, f (x) is constant and equal to 0 in the closed x x x x
interval [–1, 1] and so f (x) is continuous and 1  1
⇒ f ′ ′ (x) = − cos  
differentiable in the open interval (–1, 1). x3  x
At x = 1, f (x) is discontinuous, since lim (1 − h ) = 0 Now, lim f ′ (x) = 0 + 1 = 1 ⇒ Option (b) is correct.
h→ 0 x→ ∞
and lim (1 + h ) = − 1 1
h→ 0 Now, x ∈ [1 , ∞ ) ⇒ ∈ (0 , 1] ⇒ f ′ ′ (x) < 0
∴ f (x) is not differentiable at x = 1. x
Hence, (a), (b) and (d) are correct answers. Option (d) is correct.
Limit, Continuity and Differentiability 217

As f ′ (1) = sin 1 + cos 1 > 1 1 + x, 0 ≤ x ≤ 2


4. Given, f (x) = 
f ′ (x) is strictly decreasing and lim f ′ (x) = 1 3 − x, 2 < x ≤ 3
x→ ∞
So, graph of f ′ (x) is shown as below. 1 + f (x), 0 ≤ f (x) ≤ 2
∴ fof (x) = f [ f (x)] = 
Y 3 − f (x), 2 < f (x) ≤ 3
1 + f (x), 0 ≤ f (x) ≤ 1 1 + (3 − x), 2 < x ≤ 3
 
(1, sin1+cos1) ⇒ fof = 1 + f (x), 1 < f (x) ≤ 2 = 1 + (1 + x), 0 ≤ x ≤ 1
3 − f (x), 2 < f (x) ≤ 3 3 − (1 + x), 1 < x ≤ 2
 
1
0 1
X 4 − x, 2 < x ≤ 3

⇒ ( fof ) (x) = 2 + x, 0 ≤ x ≤ 1
2 − x, 1 < x ≤ 2

Now, in [x , x + 2], x ∈ [1 , ∞ ), f (x) is continuous and Now, RHL (at x = 2) = 2 and LHL (at x = 2) = 0
differentiable so by LMVT, Also, RHL (at x = 1) = 1 and LHL (at x = 1) = 3
f (x + 2) − f (x)
f ′ (x) = Therefore, f (x) is discontinuous at x = 1, 2
2
∴ f [ f (x)] is discontinuous at x = {1, 2}.
As, f ′ (x) > 1
For all x ∈ [1 , ∞ ) 5. Since, f (x) is continuous at x = 0.
f (x + 2) − f (x) ⇒ lim f (x) = f (0)
⇒ > 1 ⇒ f (x + 2) − f (x) > 2 x→ 0
2
⇒ f (0 ) = f (0− ) = f (0) = 0
+
…(i)
For all x ∈ [1 , ∞ )
To show, continuous at x = k
 f (x) + 1 , if f (x) < 0
2. gof (x) =  RHL = lim f (k + h ) = lim [ f (k) + f (h )] = f (k) + f (0+ )
{ f (x) − 1} + b , if f (x) ≥ 0
2 h→ 0 h→ 0

 x+ a +1, if x < − a = f (k) + f (0)


 LHL = lim f (k − h ) = lim [ f (k) + f (− h )]
=  (x + a − 1)2 + b, if − a ≤ x < 0 h→ 0 h→ 0
(| x − 1| − 1)2 + b, if x ≥ 0 = f (k) + f (0− ) = f (k) + f (0)

As gof (x) is continuous at x = − a ∴ lim f (x) = f (k)
x→ k
gof (− a ) = gof (− a + ) = gof (− a − ) ⇒ f (x) is continuous for all x ∈ R.
⇒ 1 + b =1 + b =1 ⇒ b =0
Also, gof (x) is continuous at x = 0 Topic 7 Differentiability at a Point
⇒ gof (0) = gof (0+ ) = gof (0− )
1. Given function, g (x) = | f (x)|
⇒ b = b = (a − 1)2 + b ⇒ a = 1
where f : R → R be differentiable at c ∈ R and f (c) = 0,
 x + 2, if x < −1 then for function ‘g’ at x = c

Hence, gof (x) =  x2 , if − 1 ≤ x < 0 g (c + h ) − g (c)
(| x − 1| − 1)2, g′ (c) = lim [where h > 0]
 if x ≥ 0 h→ 0 h
| f (c + h )| − | f (c)| | f (c + h )|
In the neighbourhood of x = 0 , gof (x) = x2, which is = lim = lim
h→ 0 h → h
differentiable at x = 0. h 0

[as f (c) = 0 (given)]


3. As, f (x) is continuous and g (x) is discontinuous.
f (c + h ) − f (c)
Case I g (x) is discontinuous as limit does not exist at = lim [Qh > 0]
h→ 0 h
x = k.
f (c + h ) − f (c)
∴ φ (x) = f (x) + g (x) = lim
h→ 0 h
⇒ lim φ (x) = lim { f (x) + g (x)} = does not exist.
x→ k x→ k = f ′ (c) [Q f is differentiable at x = c]
∴ φ (x) is discontinuous. Now, if f ′ (c) = 0, then g (x) is differentiable at x = c,
Case II g (x) is discontinuous as, lim g (x) ≠ g (k). otherwise LHD (at x = c) and RHD
x→ k
∴ φ (x) = f (x) + g (x). (at x = c) is different.
⇒ lim φ (x) = lim { f (x) + g (x)} = exists and is a finite 2.
x→ k x→ k Key Idea (i) First use L’ Hopital rule
quantity (ii) Now, use formula
but φ (k) = f (k) + g (k) ≠ lim { f (x) + g (x)} φ2 ( x )
x→ k
∫ f (t ) dt = f [ φ2( x)] ⋅ φ′2 ( x) − f [ φ1( x)] ⋅ φ′1( x)
d
∴ φ (x) = f (x) + g (x) is discontinuous, dx φ1 ( x )
whenever g (x) is discontinuous.
218 Limit, Continuity and Differentiability
f ( x)
Here, Rf ′ (0) = lim (3 cos x − 1 − 2(x − π )sin x)
f ( x) ∫ 2tdt 0 
x→ 0 +
2tdt =3 −1 −0 =2
Let l = lim
x→ 2 ∫ = lim 6
(x − 2) x→ 2 (x − 2) 0
form, as f (2) = 6

6 and Lf ′ (0) = lim (cos x + 1 − 2(x − π )sin x)
x → 0−
On applying the L’ Hopital rule, we get
=1 + 1 −0 =2
2 f (x) f ′ (x)  d 2
φ ( x)
Q Rf ′ (0) = Lf ′ (0)
l = lim
x→ 2 1
Q ∫ f (t )dt = f (φ 2(x)) ⋅ φ2′ (x)
 dx φ1 ( x ) So, f (x) is differentiable at all values of x.
− f (φ1 (x)) ⋅ φ1′ (x)] ⇒ K =φ
So, l = 2 f (2) ⋅ f ′ (2) = 12 f ′ (2) [Q f (2) = 6] 6. Key Idea This type of problem can be solved graphically.
f ( x)
2tdt
∴ lim
x→ 2 ∫ x−2
= 12 f ′ (2)  − 1, − 2 ≤ x < 0
We have, f (x) =  2
6
x − 1 , 0 ≤ x ≤ 2
3. Given function is f (x) = 15 − |x − 10|, x ∈ R and
g (x) = f ( f (x)) and g (x) = | f (x)| + f (|x|)
 1, −2 ≤ x<0
= f (15 −|x − 10|) = 15 − |15 − |x − 10| − 10| Clearly, | f (x)| =  2
= 15 − |5 − |x − 10|| | x − 1|, 0 ≤ x≤2
15 − |5 − (x − 10)| , x ≥ 10  1, −2 ≤ x<0
= 
15 − |5 + (x − 10)| , x < 10 = − (x2 − 1), 0 ≤ x < 1
 x2 − 1 , 1 ≤ x≤2
15 − |15 − x| , x ≥ 10 
=
 15 − | x − 5| , x < 10 and f (|x|) = |x|2 − 1, 0 ≤ |x| ≤ 2
 15 + (x − 5) = 10 + x , x<5 [Q f (|x|) = − 1 is not possible as|x| </ 0]
 15 − (x − 5) = 20 − x , 5 ≤ x < 10 = x2 − 1 , |x| ≤ 2 [Q |x|2 = x2]

=
= x −1, −2 ≤ x≤2
2
 15 + (x − 15) = x , 10 ≤ x < 15
15 − (x − 15) = 30 − x , x ≥ 15 ∴ g (x) = | f (x)| + f (|x|)

From the above definition it is clear that g (x) is not  1 + x2 − 1 , −2 ≤ x<0



differentiable at x = 5, 10, 15. = − (x2 − 1) + x2 − 1, 0 ≤ x < 1
 x2 − 1 + x2 − 1, 1 ≤ x≤2
4. Let us draw the graph of y = f (x), as shown below 
y= cos x
y= sin x  x2, −2 ≤ x<0
1 
–3p = 0, 0 ≤ x<1
–p 4 O p
X 2 (x2 − 1), 1 ≤ x ≤ 2
p p/4 
2
–1
Now, let us draw the graph of y = g (x), as shown in the
y= min {sin x, cos x} figure.
Clearly, the function f (x) = min {sin x, cos x} is not Y
(2,6)
− 3π π
differentiable at x = and [these are point of
4 4 (–2,4) y=x2 y=0
intersection of graphs of sin x and cos x in (− π , π ), on y=2 (x2 –1)
 −3 π π  X′ X
which function has sharp edges]. So, S =  , , –2 –1 O 1 2
 4 4
 −3 π − π 3 π π 
which is a subset of  , , , 
 4 4 4 4
Y′
5. We have,
f (x) = sin|x| − |x| + 2 (x − π ) cos|x| 1
[ Here, y = 2 (x2 − 1) or x2 = ( y + 2) represent a parabola
− sin x + x + 2(x − π ) cos x, if x < 0 2
f (x) =  with vertex (0, − 2) and it open upward]
 sin x − x + 2(x − π ) cos x, if x ≥ 0
[Q sin(−θ ) = − sin θ and cos(−θ ) = cos θ ] Note that there is a sharp edge at x = 1 only, so g (x) is
− cos x + 1 + 2 cos x − 2(x − π )sin x ; if x < 0 not differentiable at x = 1 only.
∴ f ′ (x) = 
 cos x − 1 + 2 cos x − 2(x − π ) sin x , if x > 0 7. Key Idea This type of questions can be solved graphically.
Clearly, f (x) is differentiable everywhere except
possibly at x = 0 Given, f : (−1, 1) → R, such that
[Q f ′ (x) exist for x < 0 and x > 0] f (x) = max { −|x|, − 1 − x2 }
Limit, Continuity and Differentiability 219

On drawing the graph, we get the follwong figure. 4


Y
y=x2
y=f(x)
O 1 1
–1
–1 , –1 , 1 , –1 ,
√2 √2 √2 √2
–2 –1 1 2
y=–|x|
y = |x|
[ Q graph of y = −| x|is
Y For| x| ∈ (2, 4]
8 − 2x, x ∈ (2, 4]
f (x) = 8 − 2|x|= 
X 8 + 2 x, x ∈ [− 4, − 2)
 Q 2 < |x| ≤ 4 
⇒|x| > 2 and |x| ≤ 4
 
and graph of y = − 1 − x2  
Y
Y
5
4
X
3
y=8+2x 2 y=8–2x
1
[Q x2 + y2 = 1 represent a complete circle]
 1 X′ X
− 1 − x2 , −1 < x ≤ − –5 –4 –3 –2 –1 O 1 2 3 4 5
 2
 1 1
⇒ f (x) =  −|x|, − <x≤ Y′
 2 2
1 Hence, the graph of y = f (x) is
 − 1 − x2 , < x<1
 2 4
From the figure, it is clear that function have sharp
2x

y=
8+

1 1
edges, at x = − , 0,

8–
1
y=

2 2

2x
∴ Function is not differentiable at 3 points.
–4 –2 –1 1 2 4
8. Key Idea This type of problem can be solved graphically
From the graph it is clear that at x = − 2, − 1, 0, 1, 2 the
max {|x|, x2}, |x| ≤ 2 curve has sharp edges and hence at these points f is not
We have, f (x) = 
 8 − 2|x|, 2 < |x| ≤ 4 differentiable.
3
Let us draw the graph of y = f (x)
9. Given,| f (x) − f ( y)|≤ 2| x − y|2 , ∀ x, y ∈ R
For| x| ≤ 2 f (x) = max{|x|, x2} 1
| f (x) − f ( y)|
Let us first draw the graph of y =|x|and y = x2 as shown ⇒ ≤ 2| x − y|2
in the following figure. | x − y|
y=x2 (dividing both sides by|x − y|)
y=|x| Put x = x + h and y = x, where h is very close to zero.
2 1
f (x + h ) − f (x)
⇒ lim ≤ lim 2|(x + h ) − x|2
1 h→ 0 (x + h ) − x h→ 0
1
f (x + h ) − f (x)
–2 –1 0 1 2 ⇒ lim ≤ lim 2| h |2
h→ 0 h h→ 0
–1
f (x + h ) − f (x)
⇒ lim ≤0
–2 h→ 0 h
[substituting limit directly on right hand
Clearly, y =|x|and y = x2 intersect at x = − 1, 0, 1 side and using lim | f (x)| = lim f (x) ]
x→ a x→ a
Now, the graph of y = max {|x|, x2} for|x| ≤ 2 is
220 Limit, Continuity and Differentiability

 f (x + h ) − f (x)  13. To check differentiability at a point we use RHD and LHD at


⇒| f ′ (x)| ≤ 0 Q lim = f ′ (x) PLAN
 h→ 0 h  a point and if RHD = LHD, then f( x ) is differentiable at the
point.
⇒| f ′ (x)| = 0 (Q| f ′ (x)|can not be less than zero)
Description of Situation
⇒ f ′ (x) = 0 [Q| x| = 0 ⇔ x = 0]
f (x + h ) − f (x)
⇒ f (x) is a constant function. As, R{ f ′ (x)} = lim
h→ 0 h
Since, f (0) = 1, therefore f (x) is always equal to 1. f (x − h ) − f (x)
1 1 and L{ f ′ (x)} = lim
Now, ∫ ( f (x))2dx = ∫ dx = [x]10 = (1 − 0) = 1 h→ 0 −h
0 0

10. We have, Here, students generally gets confused in defining


modulus. To check differentiable at x = 0,
f (x) = |x − π|( ⋅ e − 1)sin|x|
|x|
f (0 + h ) − f (0)
 (x − π )(e− x − 1)sin x, x<0 R{ f ′ (0)} = lim
 h→ 0 h
f (x) = − (x − π )(ex − 1)sin x, 0 ≤ x < π
 (x − π )(ex − 1)sin x, π
x≥π h 2 cos −0
 h π
= lim = lim h ⋅ cos =0
We check the differentiability at x = 0 and π. h→ 0 h h→ 0 h
We have, π
h 2 cos  −  − 0
(x − π ) (e−x − 1) cos x + (e−x − 1) sin x f (0 − h ) − f (0)  h
 + (x − π ) sin xe−x (−1), x < 0 L {f ′ (0)} = lim = lim =0
− [(x − π )(ex − 1) cos x + (ex − 1) sin x
h→ 0 −h h→ 0 −h
f ′ (x) = 
+ (x − π ) sin xex ],0 < x < π So, f (x) is differentiable at x = 0.

(x − π )(e − 1) cos x + (e − 1) sin x x
x x
To check differentiability at x = 2,
 + (x − π ) sin xe , x > π
f (2 + h ) − f (2)
R{ f ′ (2)} = lim
Clearly, h→ 0 h
lim f ′ (x) = 0 = lim f ′ (x)
x→ 0 − x→ 0 +  π   π 
(2 + h )2 cos   −0 (2 + h )2 ⋅ cos  
 2 + h  2 + h
and lim f ′ (x) = 0 = lim f ′ (x) = lim = lim
x→ π − x→ π + h→ 0 h h→ 0 h
∴ f is differentiable at x = 0 and x = π π π 
(2 + h )2 ⋅ sin  − 
Hence, f is differentiable for all x.  2 2 + h
= lim
h→ 0 h
11. We have, f (x) = log 2 − sin x and g (x) = f ( f (x)), x ∈ R
 πh 
Note that, for x → 0, log 2 > sin x (2 + h )2 ⋅ sin  
 2 (2 + h ) π
∴ f (x) = log 2 − sin x = lim ⋅ =π
h→ 0 π 2 (2 + h)
⇒ g (x) = log 2 − sin ( f (x)) h⋅
2 (2 + h )
= log 2 − sin (log 2 − sin x)
Clearly, g (x) is differentiable at x = 0 as sin x is f (2 − h ) − f (2)
and L { f ′ (2)} = lim
differentiable. h→ 0 −h
Now, g′ (x) = − cos (log 2 − sin x) (− cos x) π π
(2 − h )2 ⋅ cos − 22 ⋅ cos
= cos x ⋅ cos (log 2 − sin x) 2−h 2
⇒ g′ (0) = 1 ⋅ cos (log 2) = lim
h→ 0 −h
12. Given, f (0) = 2 = g (1), g(0) = 0 and f (1) = 6  π 
(2 − h )2 −  − cos  −0
f and g are differentiable in (0, 1).  2 − h
= lim
Let h (x) = f (x) − 2 g (x) …(i) h→ 0 −h
h (0) = f (0) − 2 g (0) = 2 − 0 = 2 π π 
− (2 − h )2 ⋅ sin  − 
and h (1) = f (1) − 2 g (1) = 6 − 2(2) = 2  2 2 − h
= lim
h (0) = h (1) = 2 h→ 0 h
Hence, using Rolle’s theorem,  πh 
(2 − h )2 ⋅ sin  − 
h′ (c) = 0, such that c ∈ (0, 1)  2 (2 − h ) −π
= lim × =−π
Differentiating Eq. (i) at c , we get h→ 0 −π 2 (2 − h)

⇒ f ′ (c) − 2 g′ (c) = 0 2 (2 − h )
⇒ f ′ (c) = 2 g′ (c) Thus, f (x) is differentiable at x = 0 but not at x = 2.
Limit, Continuity and Differentiability 221

(x − 1)n In function,
14. Given, g (x) = ; 0 < x < 2, m ≠ 0, n are
log cosm (x − 1) y = || x| − 1|we have 3 sharp edges at x = − 1, 0, 1.
x − 1 , x ≥ 1 Hence, f (x) is not differentiable at {0, ± 1}.
integers and| x − 1| = 
1 − x, x < 1 1
(– x − 1), if x < − 1
2
The left hand derivative of|x − 1|at x = 1 is p = − 1.  −1
17. Given, f (x) =  tan x, if − 1 ≤ x ≤ 1
Also, lim g (x) = p = − 1 1
x →1+
(1 + h − 1)n  2 (x − 1), if x > 1
⇒ lim = −1 
h→ 0 log cosm (1 + h − 1) f (x) is discontinuous at x = − 1 and x = 1.
hn ∴ Domain of f ′ (x) ∈ R − { −1, 1}
⇒ lim = −1
h → 0 log cosm h sin h − h
18. RHD of sin (| x|) − | x| = lim = 1 −1 = 0
h→ 0 h
hn
⇒ lim = −1 [Q f (0) = 0]
h → 0 m log cos h
LHD of sin (| x|) − | x|
n ⋅ hn − 1 sin| − h | − | − h | sin h − h
⇒ lim = −1 = lim = =0
h→ 0
m
1
(− sin h ) h→ 0 −h −h
cos h Therefore, (d) is the answer.
[using L’Hospital’s rule]
n−2
hn − 2
19. Given, f (x) = [x] sin π x
 n h  n
⇒ lim  −  ⋅ = − 1 ⇒   lim =1 If x is just less than k, [x] = k − 1
h → 0  m  tan h   m h → 0  tan h 
    ∴ f (x) = (k − 1) sin π x.
 h   h 
(k − 1) sin π x − k sin π k
n LHD of f (x) = lim
⇒ n = 2 and =1 ⇒ m = n =2 x→ k x−k
m
(k − 1) sin π x
15. Given, f (1) = f   = f   = K = lim f   = 0 = lim
1 1 1 ,
 2  3 n→ ∞  n x→ k x−k
 1 (k − 1) sin π (k − h )
as f   = 0; n ∈ integers and n ≥ 1. = lim
 n h→ 0 −h
 1 [where x = k − h ]
⇒ lim f   = 0 ⇒ f (0) = 0
n→ ∞  n (k − 1) (−1)k − 1 ⋅ sin h π
= lim = (−1) (k − 1) π
k

Since, there are infinitely many points in


h→ 0 −h
neighbourhood of x = 0. 20. Given, f (x) = max { x, x3 } considering the graph
∴ f (x) = 0 separately, y = x3 and y = x
⇒ f ′ (x) = 0
NOTE y = x 3 is odd order parabola and y = x is always
⇒ f ′ (0) = 0
intersect at (1, 1) and ( −1, − 1).
Hence, f (0) = f ′ (0) = 0 x in (− ∞ , − 1]
 3 in (−1, 0]
16. Using graphical transformation. Now, f (x) = x x in (0, 1]
x3 in (1, ∞ )
As, we know that, the function is not differentiable at 
sharp edges. 1 in (− ∞ , − 1]
 2 in (−1, 0]
Y Y ⇒ f ′ (x) = 3x
1 in (0, 1]
3x2 in (1, ∞ )
y = |x| – 1 
O Y y =x
X X
O 1 –1 1 A (1, 1)
–1 –1 y =x 3
X′ X
(i) y = x – 1 (ii) y = |x| – 1 O
(–1, –1) B
Y Y′

The point of consideration are


–1 O 1
X f ′ (−1− ) = 1 and f ' (−1+ ) = 3
f ′ (−0− ) = 0 and f ′ (0+ ) = 1

f ′ (1 ) = 1 and f ' (1+ ) = 3
(iii) y = ||x| – 1| Hence, f is not differentiable at −1, 0, 1.
222 Limit, Continuity and Differentiability

21. Let h (x) = | x|, then g (x) = | f (x)| = h { f (x)} Q Lf ′ (1) = Rf ′ (1). Therefore, function is differentiable
Since, composition of two continuous functions is at x = 1.
continuous, g is continuous if f is continuous. So, answer f (x) − f (2)
is (c). Again, Lf ′ (2) = lim
x → 2− x−2
(a) Let f (x) = x ⇒ g (x) = | x|
 cos x − cos 2 
Now, f (x) is an onto function. Since, co-domain of x = lim − (x2 − 1) (x − 1) + 
is R and range of x is R. But g (x) is into function.

x→ 2  x−2 
Since, range of g (x) is [0, ∞ ) but co-domain is given R. = − (4 − 1) (2 − 1) − sin 2 = − 3 − sin 2
Hence, (a) is wrong. f (x) − f (2)
(b) Let f (x) = x ⇒ g (x) = | x|. Now, f (x) is one-one and R f ′ (2) = lim
x → 2+ x−2
function but g (x) is many-one function. Hence, (b) is
wrong.  cos x − cos 2 
(d) Let f (x) = x ⇒ g (x) = | x|. Now, f (x) is differentiable = lim (x2 − 1) (x − 1) + 
+
x→ 2  x−2 
for all x ∈ R but g (x) =|x|is not differentiable at x = 0
Hence, (d) is wrong. = (22 − 1) (2 − 1) − sin 2 = 3 − sin 2
22. Function f (x) = (x2 − 1)| x2 − 3x + 2| + cos (|x|) …(i) So, L f ′ (2) ≠ R f ′ (2), f is not differentiable at x = 2
NOTE In differentiable of| f ( x )| we have to consider critical Therefore, (d) is the answer.
points for which f ( x ) = 0.  x
, x≥0

=  1 x+ x
x
23. Given, f (x) =
| x|is not differentiable at x = 0 1 + | x|  , x<0
cos (− x), if x < 0 1 − x
but cos| x|= 
 cos x, if x ≥ 0  (1 + x) ⋅ 1 − x ⋅ 1
, x≥0

if x < 0 f ′ (x) =  (1 −(1x)+⋅ 1x)− x (−1)
2
 cos x, ∴
⇒ cos| x| = 
 cos x, if x ≥ 0  , x<0
 (1 − x)2
Therefore, it is differentiable at x = 0 .
Now, | x2 − 3x + 2| = |(x − 1) (x − 2)|  1
,x≥0

f ′ (x) =  (1 +1 x)
2
(x − 1) (x − 2), if x < 1 ⇒
  ,x<0
=  (x − 1) (2 − x), if 1 ≤ x < 2
 (1 − x)
2
(x − 1) (x − 2), if 2 ≤ x
 1
Therefore, ∴ RHD at x = 0 ⇒ lim =1
x→ 0 (1 + x)2
 (x2 − 1) (x − 1) (x − 2) + cos x, if − ∞ < x < 1 1
 and LHD at x = 0 ⇒ lim =1
f (x) = − (x2 − 1) (x − 1) (x − 2) + cos x, if 1 ≤ x < 2 x→ 0 (1 − x)2
 (x2 − 1) (x − 1) (x − 2) + cos x, if 2 ≤ x < ∞
 Hence, f (x) is differentiable for all x.
Now, x = 1, 2 are critical point for differentiability. 24. Since, f (x) is continuous and differentiable where
Because f (x) is differentiable on other points in its f (0) = 1 and f′ (0) = − 1, f (x) > 0, ∀ x.
domain.
Thus, f (x) is decreasing for x > 0 and concave down.
Differentiability at x = 1
⇒ f ′ ′ (x) < 0
f (x) − f (1)
L f ′ (1) = lim Therefore, (a) is answer.
x → 1− x−1
tan π [(x − π )]
 cos x − cos 1  25. Here, f (x) =
= lim (x2 − 1) (x − 2) + 1 + [x]2

x→1  x −1 
 Since, we know π [(x − π )] = nπ and tan nπ = 0
= 0 − sin 1 = − sin 1 Q 1 + [x]2 ≠ 0
cos x − cos 1 d
[ Q lim = (cos x) at x = 1 − 0 ∴ f (x) = 0, ∀ x
x → 1− x−1 dx
Thus, f (x) is a constant function.
= − sin x at x = 1 − 0 = − sin x at x = 1 = − sin 1]
f (x) − f (1) ∴ f ′ (x), f ′ ′ (x) ,... all exist for every x , their value
and Rf ′ (1) = lim
x →1+ x−1 being 0.
 cos x − cos 1  ⇒ f ′ (x) exists for all x.
= lim − (x2 − 1) (x − 2) + 
+
x→1  x−1  26. The given function f : R → R is satisfying as
= 0 − sin 1 = − sin 1 [same approach] f (x + y) = f (x) + f ( y) + f (x) f ( y).
Limit, Continuity and Differentiability 223

f (x + h ) − f (x) Similarly,
So, f ′ (x) = lim
h→ 0 h ( fg )(x) − ( fg )(1) f (1 + h ) . g (1)
lim = lim
f (x) + f (h ) + f (x) f (h ) − f (x) x →1+ x −1 h→ 0 h
= lim
h→ 0 h (1 + h )2(h ) + h sin(1 + h )
f (h ) = g (1) lim
= lim (1 + f (x)) h→ 0 h
h→ 0 h
= (1 + sin 1) g (1)
Q f (x) = xg (x)
Q RHD and LHD of function fg at x = 1 is finitely exists
f (x)
⇒ g (x) = and equal, so fg is differentiable at x = 1
x
Now, let ( fg )(x) is differentiable at x = 1, so
f (x)
∴ lim = lim g (x) = 1 (given) ( fg )(x) − ( fg )(1) ( fg )(x) − ( fg )(1)
x→ 0 x x→ 0 lim = lim
x → 1− x−1 x →1+ x−1
∴ f ′ (x) = 1 + f (x) f (x) g (x) − f (1) g (1) f (x) g (x) − f (1) g (1)
f ′ (x) ⇒ lim = lim
⇒ =1 x → 1− x−1 x →1+ x−1
1 + f (x)
f (x) g (x) f (x) g (x)
⇒ log e (1 + f (x)) = x + c ⇒ lim = lim {Q f (1) = 0}
x → 1− x−1 x →1+ x−1
⇒ 1 + f (x) = ex + c f (1 − h ) g (1 − h ) f (1 + h ) g (1 + h )
⇒ f (x) = ex + c − 1 ⇒ lim = lim
h→ 0 −h h→ 0 h
Q f (0) = 0
[(1 − h )2(− h ) + (− h )sin(1 − h )] g (1 − h )
∴ c=0 ⇒ lim
h→ 0 −h
Therefore, f (x) = ex − 1 is differentiable at every x ∈ R.
[(1 + h ) (h ) + (h )sin(1 + h )] g (1 + h )
2
And f ′ (x) = ex ⇒ f′ (0) = 1 = lim
h→ 0 h
(x) ex − 1
Now, g (x) = f = and if g(0) = 1 ⇒ lim [(1 − h )2 + sin(1 − h )] g (1 − h ) = lim
x x h→ 0 h→ 0
LHD (at x = 0) of [(1 + h )2 + sin(1 + h )] g (1 + h )
g (0 − h ) − g (0) It does not mean that g (x) is continuous or
g (x) = lim
h→ 0 −h differentiable at x = 1.
But if g is differentiable at x = 1, then it must be
e− h − 1
−1 continuous at x = 1 and so fg is differentiable at x = 1.
−h
= lim 28. It is given, that f : R → R and
h→ 0 −h
f (h ) − f (0)
e− h − 1 + h 1 Property 1 : lim exists and finite, and
= lim = h→ 0 |h|
h→ 0 h2 2
f (h ) − f (0)
and, RHD (at x = 0) of Property 2 : lim exists and finite.
h→ 0 h2
g (0 + h ) − g (0) eh − 1 − h 1
g (x) = lim = lim = Option a,
h→ 0 h h→ 0 h2 2
sin h − sin 0 1  sin h 
So, if g(0) = 1, then g is differentiable at every x ∈ R. P2 : lim = lim   = doesn’t exist.
h→ 0 h 2 h → 0 h h 
27. Given functions f : R → R be defined by Option b,
f (x) = x3 − x2 + (x − 1)sin x and g : R → R be an h 23
/
−0 / − 1/ 2
arbitrary function. P1 : lim = lim h 23 = lim h1/ 6 = 0
h→ 0 |h| h→ 0 h→ 0
Now, let g is continuous at x = 1, then
( fg )(x) − ( fg )(1) ( fg )(1 − h ) − ( fg )(1) exists and finite.
lim = lim Option c,
x → 1− x −1 h→ 0 1 − h −1
|h| − 0
Q ( fg )(x) = f (x) . g (x) (given) P1 : lim = lim |h| = 0, exists and finite.
h→ 0 |h| h → 0
f (1 − h ) . g (1 − h ) − f (1) . g (1)
= lim Option d,
h→ 0 −h
h|h| − 0 |h|
f (1 − h ) . g (1) P2 : lim = lim
= lim h→ 0 h 2 h → 0 h
h→ 0 −h
 1, if h → 0+
{Q f (1) = 0 and g is continuous at x = 1, so = −
g (1 − h ) = g (1)} −1, if h → 0
(1 − h ) (− h ) + (− h )sin (1 − h )
2 f (h ) − f (0)
= g (1) lim So lim does not exist.
h→ 0 −h h→ 0 h2
= (1 + sin 1) g (1) Hence, options (b) and (c) are correct.
224 Limit, Continuity and Differentiability

29. We have, f (x) sin t − f (t ) sin x


30. Given, lim = sin 2 x
( f (0)) + ( f ′ (0)) = 85
2 2 t→ x t −x
and f : R → [− 2, 2] Using L′ Hospital rules
(a) Since, f is twice differentiable function, so f is f (x) cos t − f ′ (t ) sin x
lim = sin 2 x
continuous function. t→ x 1
∴This is true for every continuous function. ⇒ f (x) cos x − f ′ (x) sin x = sin 2 x
Hence, we can always find x ∈ (r , s), where f (x) is ⇒ f ′ (x) sin x − f (x) cos x = − sin 2 x
f ′ (x) sin x − f (x) cos x
one-one. ⇒ = −1
sin 2 x
∴This statement is true.  f (x) 
⇒ d  = −1
(b) By L.M.V.T  sin x
f (b) − f (a ) f (b) − f (a ) On integrating, we get
f ′ (c) = ⇒| f ′ (c)| = f (x)
b−a b−a = − x + C ⇒ f (x) = − x sin x + C sin x
sin x
f (0) − f (− 4) f (0) − f (− 4) π  π π
⇒ | f ′ (x0 )| = = It is given that x = , f   = −
0+4 4 6  6 12
 π π π π π π 1
Range of f is [− 2, 2] ∴ f   = − sin + C sin = − =− + C
 6 6 6 6 12 12 2
f (0) − f (− 4)

− 4 ≤ f (0) − f (−4) ≤ 4 ⇒ 0 ≤ ≤1 ⇒ C=0
4
∴ f (x) = − x sin x
Hence,| f ′ (x0 )| ≤ 1. (a) f (x) = − x sin x
π π π π
Hence, statement is true. f   = − sin = − false
 4 4 4 4 2
(c) As no function is given, then we assume
(b) f (x) = − x sin x
 85 x
f (x) = 2 sin   x3 x4
sin x > x − , ∀ x ∈ (0, π ) ⇒ − x sin x < − x2 +
 2  6 6
 85 x x4
⇒ f (x ) < − x , ∀ x ∈ (0, π )
2
∴ f ′ (x) = 85 cos   6
 2 
It is true
Now, ( f (0))2 + ( f ′ (0))2 = (2 sin 0)2 + ( 85 cos 0)2 (c) f (x) = − x sin x
( f (0))2 + ( f ′ (0))2 = 85 f ′ (x) = − sin x − x cos x
f ′ (x ) = 0
and lim f (x) does not exists. ⇒ − sin x − x cos x = 0
x→ ∞
Hence, statement is false. tan x = − x
(d) From option b,| f ′ (x0 )| ≤ 1 and x0 ∈ (− 4, 0) Y

∴ ( f ′ (x0 ))2 ≤ 1
Hence, g (x0 ) = ( f (x0 ))2 + ( f ′ (x0 )2 ≤ 4 + 1
[Q f (x0 ) ∈ [−2, 2]] X′ π2 π1 X
o
3π/2
⇒ g (x0 ) ≤ 5
Now, let p ∈ (−4, 0) for which g ( p) = 5
Y y=–x
Similarly, let q be smallest positive number q ∈ (0, 4)
such that g (q) = 5 ⇒ Their exists α ∈ (0, π ) for which f′ (α ) = 0
It is true
Hence, by Rolle’s theorem is ( p, q)
(d) f (x) = − x sin x
g′ (c) = 0 for α ∈ (−4, 4) and since g (x) is greater than
5 as we move form x = p to x = q f ′ (x) = − sin x − x cos x
f ′ ′ (x) = − 2 cos x + x sin x
and f (x))2 ≤ 4 ⇒ ( f ′ (x))2 ≥ 1in ( p, q)
 π π  π π
Thus, g′ (c) = 0 f′ ′   = , f   = −
 2 2  2 2
⇒ f′ f + f′ f′ ′ = 0  π  π
∴ f′ ′   + f   = 0
So, f (α ) + f ′ ′ (α ) = 0 and f′ (α ) ≠ 0  2  2
Hence, statement is true. It is true.
Limit, Continuity and Differentiability 225

31. As, g ( f (x)) = x and g (x) = | x| f (x) + |4x − 7| f (x)


Thus, g (x) is inverse of f (x). = (| x| + |4x − 7|) f (x)
⇒ g ( f (x)) = x ⇒ g′ ( f (x)) ⋅ f ′ (x) = 1 = (| x| + |4x − 7|) [x2 − 3]
1 (− x − 4x − 7) (− 3), − 1 / 2 ≤ x < 0
∴ g′ ( f (x)) = …(i)  (x − 4x + 7) (− 3),
f ′ (x) 0 ≤ x<1

[where, f ′ (x) = 3x2 + 3]  (x − 4x + 7) (− 2), 1≤x< 2

When f (x) = 2, then =  (x − 4x + 7) (− 1), 2 ≤x< 3
x3 + 3x + 2 = 2 ⇒ x = 0  (x − 4x + 7) (0) , 3 ≤ x < 7 /4

i.e. when x = 0, then f (x) = 2  (x + 4x − 7) (0), 7 /4 ≤ x < 2
∴ g′ ( f (x)) = 2
1
at (0, 2) ⇒ g′ (2) =
1  (x + 4x − 7) (1), x=2

3x + 3 3
15x + 21, − 1 /2 ≤ x < 0
∴ Option (a) is incorrect. 9x − 21, 0 ≤ x<1
Now, h ( g ( g (x))) = x ⇒ h ( g ( g ( f (x))) = f (x) 
⇒ h ( g (x)) = f (x) 6x − 14, 1≤x< 2
…(ii) ∴ g (x) = 
As g ( f (x)) = x 3x − 7, 2 ≤x< 3
∴ h ( g (3)) = f (3) = 33 + 3(3) + 2 = 38 0, 3 ≤ x<2

∴ Option (d) is incorrect. 5x − 7, x=2
From Eq. (ii), h ( g (x)) = f (x) Now, the graphs of f (x)and g (x)are shown below.
⇒ h ( g ( f (x))) = f ( f (x)) Graph for f ( x)
⇒ h (x) = f ( f (x)) …(iii) Y
[using g ( f (x)) = x]
⇒ h′ (x) = f ′ ( f (x)) ⋅ f ′ (x) …(iv) 1
Putting x = 1, we get
h′ (1) = f ′ ( f (1)) ⋅ f ′ (1) = (3 × 36 + 3) × (6) = 111 × 6 = 666 X′ 0 X
∴ Option (b) is correct. –1/2 1 √2 √3 2
–1
Putting x = 0 in Eq. (iii), we get
h (0) = f ( f (0)) = f (2) = 8 + 6 + 2 = 16 –2
∴ Option (c) is correct. –3
32. Here, f (x) = a cos (|x3 − x|) + b|x|sin (|x3 + x|)
Y′
If x3 − x ≥ 0
⇒ cos|x3 − x| = cos (x3 − x) Clearly, f (x) is discontinuous at 4 points.
x3 − x ≤ 0 ∴ Option (b) is correct.
⇒ cos|x3 − x| = cos (x3 − x) Graph for g ( x)
∴ cos (|x3 − x|) = cos (x3 − x), ∀ x ∈ R …(i) Y
Again, if x3 + x ≥ 0
21
⇒ |x|sin (|x3 + x|) = x sin (x3 + x)
27/2
x3 + x ≤ 0
⇒ |x|sin (|x3 + x|) = − x sin { − (x3 + x )} 3
∴ |x|sin (|x + x|) = x sin (x + x), ∀ x ∈ R
3 3
…(ii)
X′ X
⇒ f (x) = a cos (|x3 − x|) + b|x|sin (|x3 + x|) 0
–1/2 3 √ 3–71 √2 √3 2
∴ f (x) = a cos (x3 − x) + bx sin (x3 + x) …(iii) 3 √ 2–7
which is clearly sum and composition of differential
functions.
–8
Hence, f (x) is always continuous and differentiable.
–12
33. Here,
− 3 , − 1 / 2 ≤ x < 1 –21
− 2 , 1 ≤ x < 2
 Y′
f (x) = [x − 3] = [x ] − 3 = − 1,
2 2
2 ≤x< 3 Clearly, g (x) is not differentiable at 4 points, when
 0, 3 ≤ x<2
 x ∈ (− 1 / 2, 2).
 1, x=2 ∴ Option (c) is correct.
226 Limit, Continuity and Differentiability

 g (x), x>0 Given that, f : [a , b] → [1, ∞ )



34. Here, f (x) =  0, x=0 
− g (x), x < 0  x0 , x<a
 
 g′ (x), x ≥ 0 an g (x) = ∫ f (t )dt , a ≤ x ≤ b
f ′ (x) =  a
 b
− g′ (x), x < 0 ∫ f (t )dt , x > b
 a
∴ Option (a) is correct.
 ex , x ≥ 0 Now, g (a − ) = 0 = g (a + ) = g (a )
x
(b) h (x) = e =  −x x
e , x < 0 [as g (a + ) = lim ∫a f (t )dt = 0
x→ a +
 ex , x≥0
⇒ h′ (x) =  − x a
 − e , x<0 and g (a ) = ∫ f (t )dt = 0]
a
⇒ h′ (0 ) = 1and h′ (0− ) = − 1
+ b
g (b− ) = g (b+ ) = g (b) = ∫ f (t )dt
So, h (x) is not differentiable at x = 0. a

∴Option (b) is not correct. ⇒ g is continuous for all x ∈ R.


(c) ( foh )(x) = f { h (x)}, as h (x) > 0  0 , x<a
 g (ex ), x ≥ 0 
Now, g′ (x) =  f (x) , a < x < b
=  −x
 0
 g (e ), x < 0  , x>b
 ex g′ (ex ), x≥0 g′ (a − ) = 0
⇒ ( foh )′ (x) =  − x −x
− e g′ (e ), x < 0 but g′ (a + ) = f (a ) ≥ 1
⇒ ( foh )′ (0 ) = g′ (1), ( foh )′ (0− ) = − g′ (1)
+
[Q range of f (x) is [1, ∞ ), ∀ x ∈ [a , b]]
So, ( foh )(x) is not differentiable at x = 0. ⇒ g is non-differentiable at x = a
∴ Option (c) is not correct. and g′ (b+ ) = 0
 e g( x ) , x ≠ 0
(d) (hof )(x) = e
f ( x)
= 0 but g′ (b− ) = f (b) ≥ 1
e = 1 , x = 0 ⇒ g is not differentiable at x = b.
g( x )
e −1  π π
Now, (hof )′ (0) = lim −x− , x≤−
h→ 0 x  2 2
g( x )  π
e − 1 g (x) 37. f (x) =  − cos x, − < x ≤ 0
= lim ⋅ 2
h→ 0 g (x) x  x − 1, 0 < x≤1

−1e
g( x )
g (x) − 0 |x|  log x, x>1
= lim ⋅ lim ⋅ lim
h→ 0
g (x) h→ 0 |x| h→ 0 x π
Continuity at x = − ,
|x| 2
= 1 ⋅ g′ (0) ⋅ lim = 0, as g′ (0) = 0
h→ 0 x  π  π π
f −  = − −  − = 0
∴ Option (d) is correct.  2  2 2
35. Let F (x) = f (x) − 3 g (x)  π 
RHL = lim − cos  − + h = 0
∴ F (−1) = 3, F (0) = 3 and F (2) = 3 h→ 0  2 
So, F ′ (x) will vanish atleast twice in (−1, 0) ∪ (0, 2).  π
∴ Continuous at x =  −  .
Q F ′′ (x) > 0 or < 0, ∀ x ∈ (−1, 0) ∪ (0, 2)  2
Hence, f ′ (x) − 3 g′ (x) = 0 has exactly one solution in Continuity at x = 0
(−1, 0) and one solution in (0, 2). f (0) = − 1
36. A function f (x) is continuous at x = a, RHL = lim (0 + h ) − 1 = − 1
h→ 0
if lim f (x) = lim f (x) = f (a ). ∴ Continuous at x = 0.
x→ a− x→ a +
Continuity at x = 1,
Also, a function f (x) is differentiable at x = a, if
f (1) = 0
f (x) − f (a ) f (x) − f (a )
lim = lim RHL = lim log (1 + h ) = 0
x→ a− x−a x→ a + x−a h→ 0
∴ Continuous at x = 1
− +
i.e. f ′ (a ) = f ′ (a )
Limit, Continuity and Differentiability 227

 π 40. From the figure,


− 1, x≤−
 2 Y
 π y = min {x, x 2}
sin x, − < x ≤ 0
f ′ (x) =  2
 1, 0 < x≤1
 1 X′ X
 , x>1 O 1
 x
Differentiable at x = 0, LHD = 0, RHD = 1
∴ Not differentiable at x = 0 Y′
Differentiable at x = 1, LHD = 1, RHD = 1 h (x) is continuous all x, but h (x) is not differentiable at
∴ Differentiable at x = 1. two points x = 0 and x = 1. (due to sharp edges). Also
h′ (x) = 1, ∀ x > 1.
3
Also, for x = − Hence, (a), (c) and (d) is correct answers.
2
π  |x − 3| , x≥1
⇒ f (x) = − x − 
2 41. Here, f (x) =  x2 3x 13
 4 − 2 + 4 , x < 1
3
∴ Differentiable at x = − ∴ RHL at x = 1, lim |1 + h − 3| = 2
2 h→ 0

38. f (x + y) = f (x) + f ( y), as f (x) is differentiable at x = 0. LHL at x = 1,


(1 − h )2 3 (1 − h ) 13 1 3 13 14 3
⇒ f ′ (0) = k …(i) lim − + = − + = − =2
h→ 0 4 2 4 4 2 4 4 2
f (x + h ) − f (x)
Now, f ′ (x) = lim ∴ f (x) is continuous at x = 1
h→ 0 h
f (x) + f (h ) − f (x) 
= lim  − (x − 3), 1 ≤ x<3
h→ 0 h 
Again, f (x) =  (x − 3), x≥3
f (h ) 0   x2 3x 13
= lim form
h→ 0 h 0   − + , x<1
4 2 4
Given, f (x + y) = f (x) + f ( y), ∀ x, y 
∴ f (0) = f (0) + f (0),  −1 , 1 ≤ x<3

∴ f ′ (x) =  1, x≥3
when x= y=0 ⇒ f (0) = 0) x 3
Using L’Hospital’s rule, 2 − 2 , x<1

f ′ (h ) RHD at x = 1 ⇒ −1 
= lim = f ′ (0) = k …(ii)
h→ 0 1 ∴ 1 3  differentiable at x = 1.
LHD at x = 1 ⇒ − = − 1
⇒ f ′ (x) = k, integrating both sides, 2 2 
f (x) = kx + C , as f (0) = 0 RHD at x = 3 ⇒ 1 
Again, not differentiable at x = 3.
⇒ C =0 ∴ f (x) = kx LHD at x = 3 ⇒ − 1

∴ f (x) is continuous for all x ∈ R and f ′ (x) = k, i.e. 42. We know that, f (x) = 1 + |sin x|could be plotted as,
constant for all x ∈ R. (1) y = sin x …(i)
Hence, (b) and (c) are correct. Y

39. Here, f (x) = min { 1, x2, x3 } which could be graphically 1


shown as
X′ X
Y y = x3 −π O π 2π 3π y = sin x
y= x2
−1

y=1
Y′
X′ X (2) y =|sin x| …(ii)
O 1
Y
1 y=|sin x|
Y′

⇒f (x) is continuous for x ∈ R and not differentiable at X′ X


–2 π –π O π 2π 3π
x = 1 due to sharp edge.
Hence, (a) and (d) are correct answers. Y′
228 Limit, Continuity and Differentiability

(3) y = 1+|sin x| …(iii) 46. We know, [x] ∈ I , ∀ x ∈ R.


Y Therefore, sin (π [x]) = 0, ∀ x ∈ R. By theory, we know
that sin (π [x]) is differentiable everywhere, therefore
–2 (A) ↔ (p).
y=1 +|sin x|
Again, f (x) = sin{ π (x − [x])}
–1 Now, x − [x] = { x} then π (x − [x]) = π { x}
which is not differentiable at x ∈ I.
X′ X
–π O π 2π 3π Therefore, (B) ↔ (r ) is the answer.
Y′ 47. Given, F (x) = f (x) ⋅ g (x) ⋅ h (x)
Clearly, y = 1 + |sin x| is continuous for all x, but not On differentiating at x = x0, we get
differentiable at infinite number of points.. F ′ (x0 ) = f ′ (x0 ) ⋅ g (x0 ) h (x0 ) + f (x0 ) ⋅ g′ (x0 ) h (x0 )
x + y = 2 y, when y > 0 + f (x0 ) g (x0 )h′ (x0 ) …(i)
43. Since, x + | y| = 2 y ⇒ 
 x − y = 2 y, when y < 0 where, F ′ (x0 ) = 21 F (x0 ), f ′ (x0 ) = 4 f (x0 )
 y = x, when y > 0 ⇒ x > 0 g′ (x0 ) = − 7 g (x0 ) and h ′ (x0 ) = k h (x0 )
⇒ 
 y = x /3, when y < 0 ⇒ x < 0 On substituting in Eq. (i), we get
which could be plotted as, 21 F (x0 ) = 4 f (x0 ) g (x0 )h (x0 ) − 7 f (x0 ) g (x0 ) h (x0 )
Y + k f (x) g (x0 )h (x0 )
⇒ 21 = 4 − 7 + k , [using F (x0 ) = f (x0) g (x0 ) h (x0 )]
y = x, x > 0 ∴ k = 24
45°  x
X′ X  , x ≠0
O 48. Given, f (x) = 1 + e1/ x
y =x , x < 0  0, x=0
3 h
Y′
−0
1 + e1/ h
Clearly, y is continuous for all x but not differentiable at ∴ Rf ′ (0) = f ′ (0+ ) = lim
x = 0. h→ 0 h
dy  1, x > 0 1
Also, = = lim =0
dx 1 /3, x < 0 h → 0 1 + e1/ h

Thus, f (x) is defined for all x, continuous at x = 0, −h


−0
dy 1 1 + e−1/ h
differentiable for all x ∈ R − {0}, = for x < 0. and Lf ′ (0) = f ′ (0− ) = lim
dx 3 h→ 0 −h
g (x) cos x − g (0) 0  1 1
44. We have, lim form = lim = =1
x→ 0 sin x 0  h→ 0 1 1+0
1 + 1/ h
g′ (x) cos x − g (x)sin x e
= lim =0
x→ 0 cos x ∴ f ′ (0+ ) = 0 and f ′ (0− ) = 1
Since, f (x) = g (x)sin x
 1
f ′ (x) = g′ (x) sin x + g (x) cos x (x − 1) sin
2
− | x|, if x ≠ 1
49. Given, f (x) =  (x − 1)
and f ′ ′ (x) = g′ ′ (x) sin x + 2 g′ (x) cos x − g (x) sin x  −1 , if x = 1
⇒ f ′ ′ (0) = 0
Thus, lim [ g (x) cos x − g (0) cosec x] = 0 = f ′ ′ (0)  1
x→ 0 (x − 1)2 sin − x, 0 ≤ x − {1}
 (x − 1)
⇒ Statement I is true.  1
Statement II f ′ (x) = g′ (x) sin x + g (x) cos x As, f (x) = (x − 1)2 sin + x, x<0
⇒ f ′ (0) = g (0)  (x − 1)
 −1 , x=1
Statement II is not a correct explanation of Statement I. 
45. A. x|x| is continuous, differentiable and strictly
increasing in (−1, 1) . Here, f (x) is not differentiable at x = 0 due to|x|.
B. |x| is continuous in (−1, 1) and not differentiable at Thus, f (x) is not differentiable at x = 0.
x = 0. 50. It is always true that differential of even function is and
C. x + [x] is strictly increasing in (−1, 1) and odd function.
discontinuous at x = 0
⇒ not differentiable at x = 0. 51. Since, f (x) is differentiable at x = 0.
D. |x − 1| + |x + 1| = 2 in (−1, 1) ⇒ It is continuous at x = 0.
⇒ The function is continuous and differentiable in i.e. lim f (x) = lim f (x) = f (0)
(−1, 1). x → 0+ x → 0−
Limit, Continuity and Differentiability 229

f (x) − f (α )
eah/ 2 − 1 eah/ 2 − 1 a a ⇒ lim = lim g (x)
Here, lim f (x) = lim = lim ⋅ = x→ α x−α x→ α
x → 0+ h→ 0 h h→ 0 h 2 2
a
2 ⇒ f ′ (α ) = lim g (x) ⇒ f ′ (α ) = g (α )
x→ α
 c − h −1 c
Also, lim f (x) = lim b sin −1   = b sin
x→ 0 − h → 0  2  2 ⇒ f (x) is differentiable at x = α.
∴ b sin −1
c a 1
= = Conversely, suppose f is differentiable at α, then
2 2 2 f (x) − f (α )
lim exists finitely.
⇒ a =1 x→ α x−α
 f (x) − f (α )
Also, it is differentiable at x = 0  , x≠α
Let g (x) =  x−α
R f ′ (0+ ) = L f ′ (0– )  f ′ (α ) , x=α
eh/ 2 − 1 1 Clearly, lim g (x) = f ′ (α )
− x→ α
Rf ′ (0+ ) = lim h 2 [Q a = 1] ⇒ g (x) is continuous at x = α.
h→ 0 h
Hence, f (x) is differentiable at x = α, iff g (x) is
2eh/ 2 − 2 − h 1 continuous at x = α.
= lim =
h→ 0 2h 2 8
54. It is clear that the given function
− h 1−1  c (1 − x), x<1
b sin   − 
 2  2 b /2 f (x) = (1 − x)(2 − x), 1 ≤ x ≤ 2
and Lf ′ (0 ) = lim

=
h→ 0 −h c2 (3 − x), x>2
1− 
4
continuous and differentiable at all points except
b 1 possibly at x = 1 and 2.
∴ =
4 − c2 8
Continuity at x = 1,
⇒ 64b = (4 − c )
2 2
LHL = lim f (x) = lim (1 − x)
x → 1− x → 1−
⇒ a =1 and 64b2 = (4 − c2)
= lim [1 − (1 − h )] = lim h = 0
2  1
52. Here, lim (n + 1) cos −1   − n h→ 0 h→ 0
n→ ∞ π  n and RHL = lim f (x) = lim (1 − x) (2 − x)
x →1+ x →1+
2  1  1   1
= lim n  1 +  cos −1   − 1 = lim nf   = lim [1 − (1 + h )] [2 − (1 + h )]
n→ ∞ π  n  n  n→ ∞  n h→ 0
= lim − h ⋅ (1 − h ) = 0
 1 2  1  1 h→ 0
where, f   = 1 +  cos −1   − 1 = f ′ (0)
 n π  n  n ∴ LHL = RHL = f (1) = 0
  1  Therefore, f is continuous at x = 1
given, f ′ (0) = nlim nf   
 n  Differentiability at x = 1,
 → ∞

2 1 f (1 − h ) − f (1)
∴ lim (n + 1) cos −1 − n = f ′ (0) …(i) L f ′ (1) = lim
n→ ∞ π n h→ 0 −h
1 − (1 − h ) − 0  h 
where, f (x) =
2
(1 + x) cos −1 x − 1, f (0) = 0 = lim = lim   = −1
h→ 0 −h h→ 0  − h 
π
  f (1 + h ) − f (1)
2 −1 and R f ′ (1) = lim
⇒ f ′ (x) = (1 + x) + cos −1 x h→ 0 h
π  1−x 2
 [1 − (1 + h )] [(2 − (1 + h )] − 0
= lim
2  π 2 h→ 0 h
⇒ f ′ (0) = − 1 +  = 1 − …(ii) − h (1 − h )
π  2 π = lim = lim (h − 1) = − 1
h→ 0 h h→ 0
∴ From Eqs. (i) and (ii), we get
Since, L [ f ′ (1)] = Rf ′ (1), therefore f is differentiable at
2  1 2
lim (n + 1) cos −1   − n = 1 − x = 1.
n→ ∞ π  n π Continuity at x = 2,
53. Since, g (x) is continuous at x = α ⇒ lim g (x) = g (α ) LHL = lim f (x) = lim (1 − x) (2 − x)
x→ α x → 2− x → 2−
and f (x) − f (α ) = g (x) (x − α ) , ∀x ∈ R [given]
= lim [1 − (2 − h )] [(2 − (2 − h )]
f (x) − f (α ) h→ 0
⇒ = g (x)
(x − α ) = lim (− 1 + h ) h = 0
h→ 0
230 Limit, Continuity and Differentiability

and RHL = lim f (x) = lim (3 − x) f (x + h ) − f (x)


x → 2+ x → 2+
Again, f ′ (x) = lim
h→ 0 h
= lim [3 − (2 + h )]  2x + 2h 
h→ 0 f  − f (x)
 2 
= lim (1 − h ) = 1 = lim
h→ 0 h→ 0 h
f (2x) + f (2h )
Since, LHL ≠ RHL, therefore f is not continuous at x = 2 − f (x)
= lim 2
as such f cannot be differentiable at x = 2.
h→ 0 h
Hence, f is continuous and differentiable at all points
1   2x  2h  
except at x = 2 . 2 f   − 1 + 2 f   − 1 − f (x)
   
= lim  
2 2 2
 − 1 + 1  h→ 0
  h
 xe  x x  , x>0
  1 1
[from Eq. (i)]
 − − + 
55. Given, f (x) = xe  x x  , x < 0 1
[2 f (x) − 1 + 2 f (h ) − 1] − f (x)
 0 , x=0 = lim 2
 h→ 0 h

 f (x) + f (h ) − 1 − f (x)
= lim
 −2 h→ 0 h
xe x , x > 0 f (h ) − 1
 = lim = −1 [from Eq. (ii)]
= x , x<0 h→ 0 h
 0 , x=0
 ∴ f ′ (x) = −1, ∀x ∈ R

(i) To check continuity at x = 0,
⇒ ∫ f ′ (x)dx = ∫ − 1 dx

LHL (at x = 0) = lim − h = 0 ⇒ f (x) = − x + k, where, k is a constant.


h→ 0
h But f (0) = 1,
RHL = lim =0
h→ 0 e2/ h therefore f (0) = −0 + k
Also, f (0) = 0 ⇒ 1=k
⇒ f (x) = 1 − x, ∀x ∈ R ⇒ f (2) = −1
∴ f (x) is continuous at x = 0
(ii) To check differentiability at x = 0, 57. We have, f (x + y) = f (x) ⋅ f ( y), ∀ x, y ∈ R.
f (0 − h ) − f (0) ∴ f (0) = f (0) ⋅ f (0) ⇒ f (0) { f (0) − 1} = 0
L f ′ (0) = lim ⇒ f (0) = 1 [Q f (0) ≠ 0]
h→ 0 −h
f (0 + h ) − f (0)
(0 − h ) − 0 Since, f ′ (0) = 2 ⇒ lim =2
= lim =1 h→ 0 h
h→ 0 −h
f (0 + h ) − f (0) f (h ) − 1
R f ′ (0) = lim ⇒ lim =2 [Q f (0) = 1] …(i)
h→ 0 h→ 0 h
h
f (x + h ) − f (x)
he−2 / h − 0 Also, f ′ (x) = lim
= lim =0 h→ 0 h
h→ 0 h f (x) ⋅ f (h ) − f (x)
∴ f (x) is not differentiable at x = 0. = lim ,
h→ 0 h
x + y f (x) + f ( y)
56. Given, f   = , ∀x, y ∈ R [using, f (x + y) = f (x) ⋅ f ( y)]
 2  2
 f (h ) − 1 
= f (x) lim 
On putting y = 0, we get  h→ 0 h 
 x f (x) + f (0) 1 ∴ f ′ (x) = 2 f (x)
f   = = [1 + f (x)] [Q f (0) = 1] [from Eq. (i)]
 2 2 2 f ′ (x)
⇒ =2
 x f (x)
⇒ 2 f   = f (x) + 1
 2
On integrating both sides between 0 to x, we get
 x x f ′ (x)
⇒ f (x) = 2 f   − 1, ∀ x, y ∈ R
 2
…(i)
∫ 0 f (x) dx = 2x
Since, f ′ (0) = −1, we get ⇒ log e | f (x)| − log e | f (0)| = 2x
f (0 + h ) − f (0)
lim = −1 ⇒ log e| f (x)| = 2x [Q f (0) = 1]
h→ 0 h
⇒ log e | f (0)| = 0
f (h ) − 1
⇒ lim = −1 …(ii) ⇒ f (x) = e2 x
h→ 0 h
Limit, Continuity and Differentiability 231

58. y = [x] + |1 − x|, − 1 ≤ x ≤ 3  −1 , −2 ≤ x ≤ 0


61. Given that, f (x) = 
−1 + 1 − x, −1 ≤ x < 0  (x − 1 ), 0 < x≤2
 0 + 1 − x, 0 ≤ x < 1 Since, x ∈ [−2, 2]. Therefore,|x|∈ [0, 2]
⇒ y= ⇒ f (|x|) = |x| − 1, ∀ x ∈ [−2, 2]
 1 + x − 1, 1 ≤ x < 2
 2 + x − 1, 2 ≤ x ≤ 3  x − 1, 0 ≤ x ≤ 2
⇒ f (|x|) = 
− x − 1 , − 2 ≤ x ≤ 0
 − x, −1 ≤ x < 0
 1 − x, 0 ≤ x < 1  1, −2 ≤ x < 0
 
⇒ y= Also, | f (x)| = 1 − x, 0 ≤ x < 1
 + x, 1 ≤ x<2
x − 1 , 1 ≤ x ≤ 2
 x + 1, 2 ≤ x < 3 

which could be shown as, Also, g (x) = f (|x|) + | f (x)|


 − x − 1 + 1 , −2 ≤ x ≤ 0
Y 
= x − 1 + 1 − x, 0 ≤ x < 1
4 x
1+ x − 1 + x − 1 , 1 ≤ x ≤ 2
3

 − x, −2 ≤ x ≤ 0
2 x 
1
g (x) =  0, 0 ≤ x<1
– 1 – 2(x − 1), 1 ≤ x ≤ 2
x x 
X′ –1 O X  − 1 , −2 ≤ x ≤ 0
1 2 3

Y′ ∴ g ′ (x) =  0, 0 ≤ x<1
 2, 1 ≤ x≤2

Clearly, from above figure, y is not continuous and not
differentiable at x = {0, 1, 2} . ∴ RHD (at x = 1) = 2, LHD (at x = 1) = 0
59. Since, | f ( y) − f (x)| ≤ (x − y)
2 3 ⇒ g (x) is not differentiable at x = 1.
Also, RHD (at x = 0) = 0, LHD at (x = 0) = − 1
| f ( y) − f (x)|2
⇒ ≤ (x − y) ⇒ g (x) is not differentiable at x = 0.
( y − x)2
Hence, g (x) is differentiable for all x ∈ (−2, 2) − {0, 1}
2
⇒ f ( y) − f (x)
  ≤x− y …(i) 62. Given, f (x) = x3 − x2 − x + 1
 y−x  ⇒ f ′ (x) = 3x2 − 2x − 1
f ( y) − f (x)
2 = (3x + 1) (x − 1)
⇒ lim   ≤ lim (x − y) ∴ f (x) is increasing for x ∈ (−∞ , − 1 /3) ∪ (1, ∞ )
y → x y − x  y→ x
and decreasing for x ∈ (−1 /3, 1)
⇒ | f ′ (x)|2 ≤ 0 max { f (t ); 0 ≤ t ≤ x}, 0 ≤ x ≤ 1
Also, given g (x) = 
which is only possible, if| f ′ (x)| = 0  3 − x, 1 < x≤2
∴ f ′ (x) = 0  f (x), 0 ≤ x ≤ 1
⇒ g (x) = 
or f ′ (x) = Constant 3 − x, 1 < x ≤ 2
60. Since, f (−x) = f (x) x3 − x2 − x + 1, 0 ≤ x ≤ 1
⇒ g (x) = 
∴f (x) is an even function.  3 − x, 1 < x≤2
f (0 + h ) − f (0)
∴ f ′ (0) = lim At x = 1,
h→ 0 h
f (0 − h ) − f (0) RHL = lim (3 − x) = 2
= lim [Q f (− h ) = f (h )] x→1
h→ 0 −h and LHL = lim (x3 − x2 − x + 1) = 0
x→1
Since, f ′ (0) exists. ∴ It is discontinuous at x = 1.
∴ R f ′ (0) = L f ′ (0)
3x2 − 2x − 1, 0 ≤ x ≤ 1
f (h ) − f (0) f (h ) − f (0) Also, g ′ (x) = 
⇒ lim = lim  −1 , 1 < x≤2
h→ 0 h h→ 0 −h
f (h ) − f (0) ⇒ g ′ (1+ ) = − 1
⇒ 2 lim =0
h→ 0 h and g ′ (1− ) = 3 − 2 − 1 = 0
f (h ) − f (0) ∴ g (x) is continuous for all x ∈ (0, 2) − {1} and g (x) is
⇒ lim =0
h→ 0 h differentiable for all x ∈ (0, 2) − {1}.
∴ f ′ (0) = 0
232 Limit, Continuity and Differentiability

 x −1 And, since ( fog ) (x) is not differentiable at the point


, when x ≠ 1
2x2 − 7x + 5 where g (x) is not differentiable as well as at those points
63. Given that, f (x) = 
1 also where g (x) attains the values so that f ( g (x)) is
− , when x = 1
 3 non-differentiable.
f (1 + h ) − f (1) Since g (x) is not continuous at x = 0 ∈ (− 1, 1) so fog (x) is
RHD = lim not differentiable and as f (x) = |2x − 1| + |2x + 1| is not
h→ 0 h
differentiable at x = − 1 / 2 and 1/2, so ( fog ) (x) is not
 1 + h −1  1  differentiable for those x, for which g (x) = − 1 / 2 or 1/2.
2(1 + h )2 − 7(1 + h ) + 5 −  − 3 
= lim   But g (x) ≥ 0, so g (x) can be 1/2 only and for x = − 1 / 2 and
h→ 0 h 1
1/2, g (x) = .
3h + 2 (1 + h )2 − 7(1 + h ) + 5  2
= lim  
h→ 0 3 h {2 (1 + h )2 − 7 (1 + h ) + 5 } So, ( fog ) (x) is not differentiable at x = − 1 / 2, 0, 1 / 2 ,
 
therefore value of d = 3
 2h 2  2
= lim   =− ∴ c + d = 1 + 3 = 4.
h→ 0  3 h (−3 h + 2 h 2 ) 9 π

f (1 − h ) − f (1) 66. g (x) = ∫ 2 (f ′ (t ) cosec t − cot t cosec t f (t ))dt


LHD = lim x
h→ 0 −h  π π
∴ g (x) = f   cosec − f (x) cosec x
 1 − h −1  1   2 2
2(1 − h )2 − 7(1 − h ) + 5 −  − 3 
= lim   f (x)
⇒ g (x) = 3 −
h→ 0 −h sin x
−3h + 2(1 + h 2 − 2h ) − 7(1 − h ) + 5  3 sin x − f (x)
= lim lim g (x) = lim 
h→ 0 −3h [2(1 − h )2 − 7(1 − h ) + 5] x→ 0 x→ 0  sin x 
2h 2 3 cos x − f ′ (x)
= lim
2
= − ∴ LHD = RHD = lim
x→ 0 cos x
h→ 0 −3h (2h 2 + 3h ) 9
3 −1
2 = =2
Hence, required value of f ′ (1) = − . 1
9
64. Given, f (x) = x tan −1 x 67. PLAN
(i) In these type of questions, we draw the graph of the function.
Using first principle, (ii) The points at which the curve taken a sharp turn, are the
 f (1 + h ) − f (1)  points of non-differentiability.
f ′ (1) = lim

h→ 0  h  Curve of f (x) and g (x) are
 (1 + h ) tan −1 (1 + h ) − tan −1 (1) 
= lim  
h→ 0 h
 
 tan −1 (1 + h ) − tan −1 (1) h tan −1 (1 + h ) 
= lim  + 
h→ 0 h h
 
1  h  
= lim  tan −1  −1
 + tan (1 + h )
h→ 0  h  2 + h 
 −1  h  
h (x) is not differentiable at x = ± 1 and 0.
 tan  2 + h   π As, h (x) take sharp turns at x = ± 1 and 0.
= lim  + Hence, number of points of non-differentiability of h (x)
h→ 0
 (2 + h ) ⋅ h  4
 2 + h  is 3.

 68. Let p (x) = ax4 + bx3 + cx2 + dx + e


−1  h  
 tan  
1  2 + h  π 1 π ⇒ p′ (x) = 4ax3 + 3bx2 + 2cx + d
= lim + = +
h→ 0 2 + h  h  4 2 4 ∴ p′ (1) = 4a + 3b + 2c + d = 0 ... (i)
 
 (2 + h )  and p′ (2) = 32a + 12b + 4c + d = 0 ... (ii)
65. The given functions f : (− 1, 1) → R and  p (x)
Since, lim 1 + 2  = 2 [given]
x→ 0  x 
g : (− 1, 1) → (− 1, 1) be defined by
f (x) = |2x − 1| + |2x + 1|and g (x) = x − [x]. ax4 + bx3 + (c + 1) x2 + dx + e
∴ lim =2
As, we know the composite function ( fog ) (x) is x→ 0 x2
discontinuous at the points, where g (x) is discontinuous ⇒ c + 1 = 2, d = 0, e = 0
for given domain. And, since g (x) is discontinuous at
x = 0 lies in interval (− 1, 1), so value of c = 1. ⇒ c=1
Limit, Continuity and Differentiability 233

From Eqs. (i) and (ii), we get  sin x − cos x  −1  tan x − 1 


3. Let f (x) = tan −1   = tan  
4a + 3b = − 2  sin x + cos x  tan x + 1
and 32a + 12b = − 4 [dividing numerator and denominator
1  π
⇒ a = and b = − 1. by cos x > 0, x ∈ 0, ]
4  2
x4  π 
∴ p(x) = − x3 + x2  tan x − tan 
−1  4  = tan −1   π 
4 = tan tan  x − 4  
  π   
⇒ p(2) =
16
−8 + 4  1 +  tan  (tan x)
 4 
4
⇒ p(2) = 0  tan A − tan B 
Q 1 + tan A tan B = tan ( A − B)
 
Topic 8 Differentiation  π
Since, it is given that x ∈ 0,  , so
 2
1. It is given
π  π π
(a + 2b cos x)(a − 2b cos y) = a 2 − b2 x − ∈− , 
4  4 4
where a > b > 0,
 π  π π
On differentiating w.r.t. ‘y’, we get Also, for  x −  ∈  − ,  ,
 4  4 4
 dx
(a − 2b cos y)0 + 2b (− sin x)  Then,
 dy
  π π
+ (a + 2b cos x)(0 − 2b (− sin y)) = 0 f (x) = tan −1  tan  x −   = x −
  4 4
 dx
⇒ (a − 2b cos y) − 2b (sin x)    π π 
 dy −1
Q tan tan θ = θ, for θ ∈  − 2 , 2  
+ 2b(sin y)(a + 2 b cos x) = 0  
 π π x
At  ,  , we get Now, derivative of f (x) w.r.t. is
 4 4 2
d ( f (x)) df (x)
 dx =2
(a − b) − b  + b (a + b) = 0 d (x / 2) d (x)
 dy
d  π
dx a + b =2 × x −  = 2
⇒ = dx  4
dy a − b
Hence, option (b) is correct. 4. Key Idea Differentiating the given equation twice w.r.t. ‘x’.
2. We know,
Given equation is
(1 + x)n = nC 0 + nC1x + nC 2 x2 + ... + nC n xn
ey + xy = e …(i)
On differentiating both sides w.r.t. x, we get
On differentiating both sides w.r.t. x, we get
n (1 + x)n − 1 = nC1 + 2 nC 2 x + ... + n nC n xn − 1 dy dy
ey +x + y=0 …(ii)
On multiplying both sides by x, we get dx dx
n x(1 + x)n − 1 = nC1x + 2nC 2x2 + ... + n nC nxn dy  y 
⇒ =− y  …(iii)
Again on differentiating both sides w.r.t. x, dx  e + x
we get
Again differentiating Eq. (ii) w.r.t. ‘x’, we get
n [(1 + x)n − 1 + (n − 1) x (1 + x)n − 2] 2
= nC1 + 22 nC 2x + ... + n 2 nC nxn − 1 ey
d 2y
+ y  dy d 2y dy dy
+x 2+ + =0
e   …(iv)
Now putting x = 1 in both sides, we get dx 2  dx dx dx dx
n
C1 + (22) nC 2 + (32) nC3 + ... + (n 2) nC n Now, on putting x = 0 in Eq. (i), we get
= n (2n − 1 + (n − 1) 2n − 2) ey = e1
For n = 20, we get ⇒ y=1
C1 + (22) 20C 2 + (32) 20C3 + ... + (20)2
20 20
C 20 On putting x = 0, y = 1 in Eq. (iii), we get
= 20(219 + (19) 218 ) dy
=−
1
=−
1
= 20 (2 + 19) 218 = 420 (218 ) dx e+0 e
= A (2B ) (given) dy 1
Now, on putting x = 0, y = 1 and = − in
On comparing, we get dx e
( A , B) = (420, 18) Eq. (iv), we get
234 Limit, Continuity and Differentiability

d 2y 1  1
2
 d 2y  1  1 7. Given equation is
e1 + e  −  + 0  2 +  −  +  −  = 0
dx2  e  dx   e  e (2x)2y = 4 ⋅ e2x − 2y ... (i)
d y2
1 On applying ‘ log e ’ both sides, we get
⇒ = log e (2x)2y = log e 4 + log e e2x − 2y
dx2 ( 0, 1) e2
2 y log e (2x) = log e (2)2 + (2x − 2 y)
 dy d 2y  1 1
So,  ,  at (0, 1) is  − , 2 [Q log e nm = m log e n and log e ef ( x ) = f (x)]
 dx dx2   e e
⇒ (2 log e (2x) + 2) y = 2x + 2 log e (2)
5. Let y = f ( f ( f (x))) + ( f (x))2 x + log e 2
⇒ y=
1 + log e (2x)
On differentiating both sides w.r.t. x, we get
dy On differentiating ‘y’ w.r.t. ‘x’, we get
= f ′ ( f ( f (x))) ⋅ f ′ ( f (x)) ⋅ f ′ (x) + 2 f (x) f ′ (x) 2
dx (1 + log e (2x))1 − (x + log e 2)
dy 2x
[by chain rule] =
dx (1 + log e (2x))2
So, dy = f ′ ( f ( f (1))) ⋅ f ′ ( f (1)) ⋅ f ′ (1) + 2 f (1) f ′ (1)
dx at 1
x=1
1 + log e (2x) − 1 − log e 2
dy = x
∴ = f ′ ( f (1)) ⋅ f ′ (1) ⋅ (3) + 2(1)(3) (1 + log e (2x))2
dx x = 1
[Q f (1) = 1 and f′ (1) = 3] dy  x log e (2x) − log e 2
So, (1 + log e (2x))2 = 
= f ′ (1) ⋅ (3) ⋅ (3) + 6 dx  x 
= (3 × 9) + 6 = 27 + 6 = 33 8. We have, x log e (log e x) − x2 + y2 = 4, which can be
6. Given expression is written as
  3 cos x + sin x  
2
 3 cot x + 1 
2
y2 = 4 + x2 − x log e (log e x) … (i)
2 y =  cot−1    =  cot−1   Now, differentiating Eq. (i) w.r.t. x, we get
  cos x − 3 sin x     cot x − 3  
dy 1 1
[dividing each term of numerator and 2y = 2x − x . − 1 ⋅ log e (log e x)
dx log e x x
denominator by sin x]
2 [by using product rule of derivative]
  π  1

−1
 cot cot x + 1   π  2x − − log e (log e x)
=  cot  6  Q cot = 3  dy log e x
  ⇒   =
  cot x − cot π   6  dx 2y
… (ii)
  6 
2 Now, at x = e, y2 = 4 + e2 − e log e (log e e)
  π   cot A cot B + 1 
=  cot−1  cot − x   Q cot( A − B) = cot B − cot A 
[using Eq. (i)]
  6    = 4 + e2 − e log e (1) = 4 + e2 − 0
 π 
2
π = e2 + 4
  − x , 0<x<
 6  6 ⇒ y = e2 + 4 [Q y > 0]
= 2
 π π  π π
+  − x  , <x< ∴ At x = e and y = e2 + 4,
 6  6 2
dy 2e − 1 − 0 2e − 1
 π + θ , − π < θ < 0  = = [using Eq. (ii)]
Q cot−1 (cot θ ) =  θ , 0 <θ < π 
dx 2 e2 + 4 2 e2 + 4
  
 θ − π , π < θ < 2π 
  9. We have,
 π 2
π f (x) = x3 + x2f ′ (1) + xf ′ ′ (2) + f ′ ′ ′ (3)

  − x , 0<x< ⇒ f ′ (x) = 3x2 + 2xf ′ (1) + f ′ ′ (2) … (i)
 6  6
⇒ 2y =  2 ⇒ f ′ ′ (x) = 6x + 2 f ′ (1) … (ii)
  7 π  π π
− x , <x< ⇒ f ′ ′ ′ (x) = 6 … (iii)
 6  6 2
⇒ f′ ′ ′ (3) = 6
 π  π
2 − x (−1), 0 < x < Putting x = 1 in Eq. (i), we get
dy   6  6
⇒2 = f ′ (1) = 3 + 2 f ′ (1) + f ′ ′ (2) … (iv)
dx   7π  π π
and putting x = 2 in Eq. (ii), we get
2 − x (−1), <x<
  6  6 2
f ′ ′ (2) = 12 + 2 f ′ (1) …(v)
 π π
dy  x − 6 , 0 < x < 6
From Eqs. (iv) and (v), we get
⇒ = f ′ (1) = 3 + 2 f ′ (1) + (12 + 2 f ′ (1))
dx x − 7π , π < x < π
 6 6 2 ⇒ 3 f′ (1) = − 15
Limit, Continuity and Differentiability 235

⇒ f′ (1) = − 5 14. Since, f (x) = e g ( x ) ⇒ e g ( x + 1)= f (x + 1) = xf (x) = xe g ( x )


⇒ f′ ′ (2) = 12 + 2 (− 5) = 2 [using Eq. (v)]
and g (x + 1) = log x + g (x)
∴ f (x) = x + x f ′ (1) + xf ′ ′ (2) + f ′ ′ ′ (3)
3 2
i.e. g (x + 1) − g (x) = log x ... (i)
⇒ f (x) = x3 − 5x2 + 2x + 6
1
⇒ f (2) = 23 − 5(2)2 + 2(2) + 6 = 8 − 20 + 4 + 6 = − 2 Replacing x by x − , we get
2
10. We have, x = 3 tan t and y = 3 sec t  1  1  1
g  x +  − g  x −  = log  x −  = log (2x − 1) − log 2
dy d  2  2  2
(3 sec t )
dy dt dt
Clearly, = =  1  1 −4
dx dx d (3 tan t ) ∴ g′′  x +  − g′′  x −  = ... (ii)
 2  2 (2x − 1)2
dt dt
3 sec t tan t tan t On substituting, x = 1, 2, 3,... , N in Eq. (ii) and adding,
= = = sin t we get
3 sec2 t sec t
 1  1  1 1 1 
d 2y d  dy d  dy dt g′′  N +  − g′′   = − 4 1 + + + ... + 2
.
and =   =  ⋅  2  2  9 25 (2 N − 1 ) 
dx 2 
dx dx  dt  dx dx
−1
d  dy d dx 1  dy
  (sin t ) 15. Since, = = 
dt  dx cos t cos3 t dy dy / dx  dx
= = dt = 2
=
dx d 3
(3 tan t ) 3 sec t d  dx d  dy
−1
dx
dt dt ⇒   =  
π dy  dy dx  dx dy
cos3
d 2y  π 4 = 1 1  d 2y  dy − 2 dx  d 2y  dy −3
Now,  at t =  = = d 2x
dx2  4 3 3(2 2 ) 6 2 ⇒ = −       = −  2  
dy2  dx2   dx  dy  dx   dx
 6x x   2 ⋅ (3x3/ 2) 
11. Let y = tan − 1   = tan − 1  3/ 2 2  16. Since, f ′′ (x) = − f (x)
 1 − 9x 
3
1 − (3x ) 
d
 2x  ⇒ { f ′ (x)} = − f (x)
= 2 tan − 1 (3x3/ 2) Q 2 tan − 1 x = tan − 1 
dx
 1 − x2  ⇒ g′ (x) = − f (x) [Q g (x) = f ′ (x), given]…(i)
dy 1 3 9 2 2
∴ = 2⋅ ⋅ 3 × (x)1/ 2 = ⋅ x   x    x 
1 + (3x3/ 2)2 1 + 9x3 Also, F (x) =  f    +  g   
dx 2
  2     2 
9
∴ g (x) =   x   x 1
1 + 9x3 ⇒ F ′ (x) = 2  f    ⋅ f ′   ⋅
  2   2 2
12. Here, g is the inverse of f (x).   x   x 1
+ 2  g    ⋅ g′   ⋅ = 0 [from Eq.(i)]
⇒ fog (x) = x   2   2 2
On differentiating w.r.t. x, we get ∴ F (x) is constant ⇒ F (10) = F (5) = 5
1
f ′{ g (x)} × g′ (x) = 1 ⇒ g′ (x) = 17. Let, g (x) = f (x) − x2
f ′ ( g (x))
⇒ g (x) has atleast 3 real roots which are x = 1, 2, 3
1  1  [by mean value theorem]
= Q f ′ (x) = 
1  1 + x5  ⇒ g ′ (x) has atleast 2 real roots in x ∈ (1, 3)
1 + { g (x)}5 ⇒ g ′ ′ (x) has atleast 1 real roots in x ∈ (1, 3)
⇒ f ′ ′ (x) – 2 . 1 = 0. for atleast 1 real root in x ∈ (1, 3)
⇒ g′ (x) = 1 + { g (x)}5 ⇒ f ′ ′ (x) = 2, for atleast one root in x ∈ (1, 3)
13. Given, y = sec (tan− 1 x ) 18. Given that, log (x + y) = 2xy …(i)
2
−1
Let tan x=θ 1+
x ∴ At x = 0, ⇒ log ( y) = 0 ⇒ y = 1
x
⇒ x = tan θ dy
∴ To find at (0, 1)
θ dx
∴ y = sec θ = 1 + x2
1
On differentiating w.r.t. x, we get On differentiating Eq. (i) w.r.t. x, we get
dy 1 1  dy dy
= ⋅ 2x 1 +  = 2x + 2y ⋅ 1
dx 2 1 + x2 x+ y  dy dx
dy 1 dy 2 y (x + y) − 1  dy
At x = 1, = ⇒ = ⇒   =1
dx 2 dx 1 − 2 (x + y) x  dx ( 0, 1)
236 Limit, Continuity and Differentiability

19. Given, x2 + y2 = 1 22. Given, xexy = y + sin 2 x …(i)


On differentiating w.r.t. x, we get On putting x = 0, we get
2x + 2 yy′ = 0 ⇒ x + yy′ = 0. 0 . e0 = y + 0
Again, differentiating w.r.t. x, we get ⇒ y=0
1 + y ′ y ′ + yy ′′ = 0 ⇒ 1 + ( y′ )2 + yy′′ = 0 On differentiating Eq. (i) both sides w.r.t. x, we get
x3 sin x cos x  dy  dy
1 . exy + x ⋅ exy  x ⋅ + y = + 2 sin x cos x
20. Given, f (x) = 6 −1 0  dx  dx
p p2 p3 On putting x = 0, y = 0, we get
 dy 
On differentiating w.r.t. x, we get e0 + 0(0 + 0) = + 2 sin 0 cos 0
 dx 
( 0, 0)
3x2 cos x − sin x x3 sin x cos x
 dy 
f ′ (x) = 6 −1 0 + 0 0 0 ⇒ =1
 dx 
0, 0
p p2 p3 p p2 p3
23. Given, f (x) = x|x|
x3 sin x cos x
 2 if x ≥ 0
+ 6 −1 0 ⇒ f (x) = x , 2
− x , if x < 0
0 0 0 f (x) is not differentiable at x = 0 but all R − {0} .
3x2 cos x − sin x Therefore, f ′ (x) = 2x, x>0
 − 2x, x<0
⇒ f ′ (x) = 6 −1 0
p p2 p3 ⇒ f ′ ′ (x) = 2, x>0
− 2 , x<0
6x − sin x − cos x
Therefore, f (x) is twice differentiable for all x ∈ R − {0}.
⇒ f ′ ′ (x) = 6 −1 0 +0+0
24. Given, f (x) = | x − 2|
p p2 p3
∴ g (x) = f [ f (x)] =||x − 2|− 2|
6 − cos x sin x
and f ′ ′ ′ (x) = 6 −1 0 +0+0 When, x>2
p p2 p3 g (x) = |(x − 2) − 2| = | x − 4| = x − 4
∴ g′ (x) = 1 when x > 2
6 −1 0
25. Let u = sec−1  − 2  and v = 1 − x2
1
∴ f ′ ′ ′ (0) = 6 −1 0 = 0 = independent of p
 2x − 1
p p2 p3
Put x = cos θ
21. Since, y2 = P (x) ∴ u = sec−1 (− sec 2 θ ) and v = sin θ
On differentiating both sides, we get ⇒ u = π − 2θ [Q sec–1 (− x) = π − sec−1 x]
2 yy1 = P ′ (x), and v = sin θ
Again, differentiating, we get du dv
⇒ = − 2 and = cos θ
2 yy2 + 2 y12 = P ′ ′ (x) dθ dθ
⇒ 2 y3 y2 + 2 y2y12 = y2P ′ ′ (x)
du 2  du 
⇒ =− ,   = −4
dv cos θ  dv  θ = π /3
⇒ 2 y y2 = y P ′ ′ (x) − 2 ( yy1 )
3 2 2

{ P ′ (x)}2 26. Given, f (x) = log x (log x)


⇒ 2 y3 y2 = P (x) ⋅ P ′ ′ (x) − log (log x)
2 ∴ f (x) =
log x
Again, differentiating, we get
d 3 On differentiating both sides, we get
2 ( y y2) = P ′ (x) ⋅ P ′ ′ (x) + P (x) ⋅ P ′ ′ ′ (x)  1 1 1
dx (log x)  ⋅  − log (log x) ⋅
2P ′ (x) ⋅ P ′ ′ (x)  log x x x
− f ′ (x) =
2 (log x)2
d 3
⇒ 2 ( y y2) = P (x) ⋅ P ′ ′ ′ (x)  1 1 1
dx 1 ⋅  ⋅  − log (1) ⋅
 1 e e 1
d  3 d 2y ∴ f ′ (e) = ⇒ f ′ (e) =
⇒ 2  y ⋅ 2  = P (x) ⋅ P ′ ′ ′ (x) (1)2 e
dx  dx 
Limit, Continuity and Differentiability 237

f1 (x) f2 (x) f3 (x) π


3
sin x
30. Here, (sin y) 2sec−1 (2x) + 2x tan {log (x + 2)} = 0
+
27. Given, F (x) = g1 (x) g2 (x) g3 (x) 2
h1 (x) h2 (x) h3 (x) On differentiating both sides, we get
π
f1 ′ (x) f2 ′ (x) f3 ′ (x) sin x π π
(sin y) 2 ⋅ log (sin y) ⋅ cos x ⋅
∴ F ′ (x) = g1 (x) g2 (x) g3 (x) 2 2
 π 
h1 (x) h2 (x) h3 (x)  π   sin x  −1 dy
+ sin x (sin y) 2  ⋅ cos y ⋅
f1 (x) f2 (x) f3 (x) f1 (x) f2 (x) f3 (x)  2  dx
+ g1 ′ (x) g2 ′ (x) g3 ′ (x) + g1 (x) g2 (x) g3 (x) 3 2 2x ⋅ sec2 {log (x + 2)}
+ ⋅ +
h1 (x) h2 (x) h3 (x) h1 ′ (x) h2 ′ (x) h3 ′ (x) 2 (2|x|) 4x2 − 1 (x + 2)
⇒ F ′ (a ) = 0 + 0 + 0 = 0 + 2x log 2 ⋅ tan {log (x + 2)} = 0
[Q fr (a ) = gr (a ) = hr (a ) ; r = 1, 2, 3]  3
Putting  x = − 1, y = −  , we get
 2x − 1  π 
28. Given, y= f  2 
 x + 1  3
2

− 
and f ′ (x) = sin 2 x dy  π  3
 2x − 1 d  2x − 1 = =
dy dx 2
π π2 − 3
∴ = f′  2 ⋅    3
dx  x + 1 dx  x2 + 1 1− 
 π 
 2x − 1  (x2 + 1) ⋅ 2 − (2x − 1) (2x) 
= sin 2 2  ⋅  31. Given, x = sec θ − cos θ and y = secn θ − cos n θ
 x + 1  (x2 + 1)2 
On differentiating w.r.t. θ respectively, we get
 2x − 1 −2x2 + 2x + 2
= sin 2  2 ⋅ dx
= sec θ tan θ + sin θ
 x + 1 (x2 + 1)2 dθ
−2 (x2 − x − 1)  2x − 1 dy
= sin 2  2  and = n secn − 1 θ ⋅ sec θ tan θ − n cos n − 1 θ ⋅ (− sin θ )
(x + 1)
2 2
 x + 1 dθ
dx
ax2 bx c ⇒ = tan θ (sec θ + cos θ )
29. y= + + +1 dθ
(x − a ) (x − b) (x − c) (x − b) (x − c) (x − c) dy
ax2 bx x and = n tan θ (secn θ + cos n θ )
= + + dθ
(x − a ) (x − b) (x − c) (x − b) (x − c) (x − c) dy n (secnθ + cos n θ )
ax 2
x  b  ⇒ =
= +  + 1 dx sec θ + cos θ
(x − a ) (x − b) (x − c) (x − c)  x − b  2
 dy n 2 (secnθ + cos n θ )2
ax2 x x ∴   =
= + ⋅  dx (sec θ + cos θ )2
(x − a ) (x − b) (x − c) (x − c) (x − b)
n 2 {(secn θ − cos n θ )2 + 4} n 2 ( y2 + 4)
x2  a  = =
=  + 1 {(sec θ − cos θ )2 + 4} (x2 + 4)
(x − c) (x − b)  x − 1 
2
x3  dy
⇒ y= ⇒ (x2 + 4)   = n 2( y2 + 4)
(x − a ) (x − b) (x − c)  dx

⇒ log y = log x3 − log (x − a ) (x − b) (x − c) A (x) B(x) C (x)


⇒ log y = 3 log x − log (x − a ) − log (x − b) − log(x − c) 32. Let φ (x) = A (α ) B (α ) C (α ) …(i)

On differentiating, we get A ′ (α ) B ′ (α ) C ′ (α )
y′ 3 1 1 1 Given that, α is repeated root of quadratic equation
= − − −
y x x − a x−b x− c f (x) = 0.
y ′ 1 1  1 1  1 1  ∴ We must have f (x) = (x − α )2 ⋅ g (x)
⇒ = − + − + − 
y  x x − a   x x − b  x x − c A ′ (x) B ′ (x) C ′ (x)
y′ −a b c ∴ φ ′ (x) = A (α ) B (α ) C (α )
⇒ = − −
y x (x − a ) x (x − b) x (x − c) A ′ (α ) B ′ (α ) C ′ (α )
y′ a b c
⇒ = + + A ′ (α ) B ′ (α ) C ′ (α )
y x (a − x) x (b − x) x (c − x)
⇒ φ ′ (α ) = A (α ) B (α ) C (α ) = 0
y′ 1  a b c 
⇒ =  + +  A ′ (α ) B ′ (α ) C ′ (α )
y x  a − x b − x c − x
⇒ x = α is root of φ′ (x).
238 Limit, Continuity and Differentiability

⇒ (x − α ) is a factor of φ′ (x) also.


3
Here, u = ex sin x and log v = x log (tan x)
or we can say (x − α )2 is a factor of f (x).
On differentiating both sides w.r.t. x, we get
⇒ φ (x) is divisible by f (x).
du 3
 2x   = ex sin x ⋅ (3x3 cos x3 + sin x3 ) …(ii)
33. Given,y = (log cos xsin x) ⋅ (logsin x cos x)−1 + sin −1   dx
  1 + x2   1 dv x ⋅ sec2x
2 and ⋅ = + log (tan x)
 log e (sin x) −1  2 x  v dx tan x
∴ y=  + sin  
 log e (cos x)  1 + x2  dv
= (tan x)x [2x ⋅ cosec (2x) + log (tan x)] …(iii)
(log e (cos x) ⋅ cot x  dx
dy  log (sin x) + loge ( in x) ⋅ tan x )  2
⇒ =2  e ⋅ + From Eqs. (i), (ii) and (iii), wet get
dx  log e (cos x) {log e (cos x)}2  1 + x2 dy 3
 = ex sin x (3x3 ⋅ cos x3 + sin x3 ) + (tan x)x
dx
  1  [2x cosec 2x + log (tan x)]
 2 ⋅ log   
 dy   2  2 5x
⇒   = 2 1 ⋅ 2 
+ 37. Given, y = + cos 2(2x + 1)
 dx  x = π    1+ π
2 3|1 − x|
  log 1
 4
   
 2   16  5x
+ cos 2(2x + 1), x < 1
8 32 3(1 − x)
=− + ⇒ y=
log e 2 16 + π 2 5x
 + cos 2(2x + 1), x > 1
3(x − 1)
 x 
34. Given, (a + bx) ey/ x = x ⇒ y = x log   The function is not defined at x = 1.
 a + bx
5  (1 − x) − x(−1) 
⇒ y = x [log (x) − log (a + bx)] …(i)    − 2 sin (4x + 2), x < 1
dy 3  (1 − x)2 
On differentiating both sides, we get ⇒ =
dx  5  (x − 1) − x(1) 
dy 1 b  − 2 sin (4x + 2), x > 1
=x −  + 1 [log (x) − log (a + bx)]  3  (x − 1)2 
dx  x a + bx   
   5
⇒ x
dy
= x2 
a ax − 2 sin (4x + 2), x < 1
 + y ⇒ x y1 = + y …(ii) dy  3 (1 − x)2
dx  x (a + bx) a + bx ⇒ =
dx − 5
− 2 sin (4x + 2), x > 1
Again, differentiating both sides, we get  3 (x − 1)2
 (a + bx) ⋅ 1 − x ⋅ b 
x y2 + y1 = a   + y1 38. Here, lim
F (x) 1
=
 (a + bx)2  x→1
G (x) 14
a 2x2  ax 
2
F ′ (x) 1
⇒ x3 y2 = ⇒ x3 y2 =   [from Eq. (ii)] ⇒ lim = [using L’Hospital’s rule]…(i)
x→1 G′ (x) 14
(a + bx) 2
 (a + bx)
x
d 2y  dy 
2 As F (x) = ∫ f (t ) dt ⇒ F ′ (x) = f (x) …(ii)
⇒ x y2 = (xy1 − y) ⇒ x
3
= x2
− y3 –1
dx2  dx 
and
x
G (x) = ∫ t f { f (t )} dt
–1
35. Given, h (x) = [ f (x)]2 + [ g (x)]2 ⇒ G′ (x) = x f { f (x)} …(iii)
⇒ h ′ x = 2 f (x) ⋅ f ′ (x) + 2 g (x) ⋅ g ′ (x) F (x) F ′ (x) f (x)
∴ lim = lim = lim
= 2 [ f (x) ⋅ g (x) − g (x) ⋅ f (x)] x→1 G (x) x→1 G′ (x) x→1 x f { f (x)}

=0 [Q f ′ (x) = g (x) and g′ (x) = − f (x)] f (1) 1 /2


= = ...(iv)
∴ h (x) is constant. 1 f { f (1)} f (1 / 2)
⇒ h (10) = h (5) = 11 F (x) 1
Given, lim =
3 x→1 G (x) 14
36. Since, y = ex sin x + (tan x)x , then
3
1
y = u + v, where u = ex sin x and v = (tan x)x 2 = 1 ⇒ f  1 = 7

dy  du dv  1 14  2
⇒ = +  …(i) f 
dx  dx dx  2
10
Application of Derivatives
Topic 1 Equations of Tangent and Normal
Objective Questions I (Only one correct option) 8. The normal to the curve y(x − 2) (x − 3) = x + 6 at the
x point, where the curve intersects the Y -axis passes
1. If the tangent to the curve y = 2 , x ∈ R, (x ≠ ± 3 ), at through the point
x −3 (2017 Main)

(a)  − , −  (b)  , 
1 1 1 1
a point (α , β ) ≠ (0, 0) on it is parallel to the line
 2 2  2 2
2x + 6 y − 11 = 0, then (2019 Main, 10 April II)
 1
(c)  , − 
1
(d)  , 
1 1
(a) | 6α + 2β | = 19 (b) | 6α + 2β | = 9
2 3  2 3
(c) | 2α + 6β | = 19 (d) | 2α + 6β | = 11
 1 + sin x   π
2. Let S be the set of all values of x for which the tangent to 9. Consider f (x) = tan −1   , x ∈ 0,  .
the curve y = f (x) = x − x − 2x at (x, y) is parallel to the
3 2
 1 − sin x   2
line segment joining the points (1, f (1)) and (−1, f (−1)), π
then S is equal to (2019 Main, 9 April I) A normal to y = f (x) at x = also passes through the
6
(a)  , − 1 (b)  , 1 (c) − , 1 (d) − , − 1
1 1 1 1
point (2016 Main)
3  3   3   3  2π 
(a) (0, 0) (b)  0, 
3. If the tangent to the curve, y = x3 + ax − b at the point  3
π π
(1, − 5) is perpendicular to the line, − x + y + 4 = 0, then (c)  , 0 (d)  , 0
which one of the following points lies on the curve ? 6  4 
(2019 Main, 9 April I)
10. The normal to the curve x2 + 2xy − 3 y2 = 0 at (1,1)
(a) (−2, 2) (b) (2, − 2)
(c) (−2, 1) (d) (2, − 1) (a) does not meet the curve again (2015 Main)
(b) meets in the curve again the second quadrant
4. The tangent to the curve y = x − 5x + 5, parallel to the
2
(c) meets the curve again in the third quadrant
line 2 y = 4x + 1, also passes through the point (d) meets the curve again in the fourth quadrant
(2019 Main, 12 Jan II)
11. The point(s) on the curve y3 + 3x2 = 12 y, where the
(a)  ,
7
(b)  ,
1
(c)  − , 7 (d)  , − 7
1 7 1 1
 
4 2 2 4  8  8  tangent is vertical, is (are) (2002, 2M)
 11 
(a)  ± , − 2 (b)  ±
4
5. A helicopter is flying along the curve given by , 0 (c) (0, 0)
y − x3/ 2 = 7, (x ≥ 0). A soldier positioned at the point  3   3 
1  (d)  ±
4
, 2
 , 7 wants to shoot down the helicopter when it is
2   3 
nearest to him. Then, this nearest distance is 12. If the normal to the curve y = f (x) at the point
(2019 Main, 10 Jan II)

1 7 5 1 7 1 (3, 4) makes an angle with the positive X-axis, then
(a) (b) (c) (d) 4
3 3 6 6 3 2
f ′ (3) is equal to (2000, 1M)
6. If θ denotes the acute angle between the curves, (a) –1 (b) –3/4
y = 10 − x2 and y = 2 + x2 at a point of their intersection, (c) 4/3 (d) 1
then|tan θ|is equal to (2019 Main, 9 Jan I)
7 8 4 8 13. The normal to the curve x = a (cos θ + θ sin θ ),
(a) (b) (c) (d)
17 15 9 17 y = a (sin θ − θ cos θ ) at any point ‘ θ ’ is such that
(a) it makes a constant angle with the X-axis (1983, 1M)
7. If the curves y2 = 6x, 9x2 + by2 = 16 intersect each other
(b) it passes through the origin
at right angles, then the value of b is (2018 Main)
(c) it is at a constant distance from the origin
7 9
(a) 6 (b) (c) 4 (d) (d) None of the above
2 2
240 Application of Derivatives

Objective Questions II Analytical & Descriptive Questions


(One or more than one correct option) 17. If | f (x1 ) − f (x2)| ≤ (x1 − x2)2, ∀x1 , x2 ∈ R.
Find the
14. On the ellipse 4x + 9 y = 1, the point at which the
2 2
equation of tangent to the curve y = f (x) at the point
tangents are parallel to the line 8x = 9 y, are (1999, 2M) (1, 2). (2005, 4M)
(a)  , 
2 1
18. The curve y = ax3 + bx2 + cx + 5, touches the X-axis at
 5 5
P(−2, 0) and cuts the Y-axis at a point Q, where its
(b)  − , 
2 1
gradient is 3. Find a, b, c. (1994, 5M)
 5 5
(c)  − , − 
2 1 19. Tangent at a point P1 {other than (0, 0)} on the curve
 5 5 y = x3 meets the curve again at P2. The tangent at P2
 2
(d)  , − 
1 meets the curve at P3 and so on.
5 5 Show that the abscissa of P1, P2, P3 , …, Pn, form a GP.
15. If the line ax + by + c = 0 is a normal to the curve xy = 1, Also, find the ratio of
then (1986, 2M) [area (∆P1P2P3 )]/[area (∆P2P3 P4 )]. (1993, 5M)
(a) a > 0, b > 0 20. Find the equation of the normal to the curve
(b) a > 0, b < 0 y = (1 + x)y + sin −1 (sin 2 x) at x = 0. (1993, 4M)
(c) a < 0, b > 0
(d) a < 0, b < 0 21. Find all the tangents to the curve y = cos (x + y),
−2π ≤ x ≤ 2π , that are parallel to the line x + 2 y = 0.
Fill in the Blank (1985, 5M)

16. Let C be the curve y − 3xy + 2 = 0. If H is the set of


3
Integer & Numerical Answer Type Question
points on the curve C, where the tangent is horizontal
22. The slope of the tangent to the curve ( y − x5 )2 = x (1 + x2)2
and V is the set of points on the curve C, where the
tangent is vertical, then H = K and V = K . (1994, 2M) at the point (1, 3) is (2014 Adv.)

Topic 2 Rate Measure, Increasing and Decreasing Functions


Objective Questions I (Only one correct option) 4. Let f : [0, 2] → R be a twice differentiable function such
1. A spherical iron ball of radius 10 cm is coated with a that f ′ ′ (x) > 0, for all x ∈ (0, 2). If φ (x) = f (x) + f (2 − x) ,
layer of ice of uniform thickness that melts at a rate of then φ is (2019 Main, 8 April I)
50 cm3 /min. When the thickness of the ice is 5 cm, then (a) increasing on (0, 1) and decreasing on (1, 2)
the rate at which the thickness (in cm/min) of the ice (b) decreasing on (0, 2)
decreases, is (2020 Main, 9 Jan I) (c) decreasing on (0, 1) and increasing on (1, 2)
1 5 (d) increasing on (0, 2)
(a) (b)
9π 6π 5. If the function f given by
1 1
(c) (d) f (x) = x3 − 3(a − 2) x2 + 3ax + 7,
36 π 18 π
for some a ∈ R is increasing in (0, 1] and decreasing in
2. Let f (x) = ex − x and g (x) = x2 − x, ∀ x ∈ R. Then, the set f (x) − 14
[1, 5), then a root of the equation, = 0 (x ≠ 1) is
of all x ∈ R, where the function h (x) = ( fog ) (x) is (x − 1)2
increasing, is (2019 Main, 10 April II)
(2019 Main, 12 Jan II)
−1 (a) − 7 (b) 6 (c) 7 (d) 5
(a)  0,  ∪ [1, ∞ ) (b)  −1,  ∪  , ∞ 
1 1
 2   2   2  x d−x
6. Let f (x) = − , x ∈ R, where a, b
−1 a 2 + x2 b2 + (d − x)2
(c) [0, ∞ ) (d)  , 0 ∪ [1, ∞ )
 2 
and d are non-zero real constants. Then,
3. A water tank has the shape of an inverted right circular (a) f is an increasing function of x (2019 Main, 11 Jan II)
−1  1  f ′ is not a continuous function of x
cone, whose semi-vertical angle is tan   . Water is (b)
 2 (c) f is a decreasing function of x
poured into it at a constant rate of 5 cu m/min. Then, the (d) f is neither increasing nor decreasing function of x
rate (in m/min) at which the level of water is rising at π π
the instant when the depth of water in the tank is 10 m 7. If the function g : (−∞ , ∞ ) →  − ,  is given by
 2 2
is (2019 Main, 9 April II)
2 1 1 1 −1 π
(a) (b) (c) (d) g (u ) = 2 tan (e ) − . Then, g is
u
(2008, 3M)
π 5π 15π 10π 2
Application of Derivatives 241

(a) even and is strictly increasing in (0, ∞ ) 18. If f : (0, ∞ ) → R be given by (2014 Adv.)
(b) odd and is strictly decreasing in (−∞ , ∞ )  1
(c) odd and is strictly increasing in (−∞ , ∞ ) x −  t +  dt
f (x) = ∫ e t
.
(d) neither even nor odd but is strictly increasing in (−∞ , ∞ ) 1/ x t
8. If f (x) = x3 + bx2 + cx + d and 0 < b2 < c, then in (−∞ , ∞ ) Then,
(a) f (x) is monotonically increasing on [1, ∞ )
(a) f (x) is strictly increasing function (2004, 2M)
(b) f (x) is monotonically decreasing on [0,1)
(b) f (x) has a local maxima
(c) f (x) + f   = 0, ∀x ∈ (0, ∞ )
1
(c) f (x) is strictly decreasing function  x
(d) f (x) is bounded
(d) f (2x ) is an odd function of x on R
9. The length of a longest interval in which the function
3 sin x − 4 sin3 x is increasing, is (2002, 2M)
19. If h (x) = f (x) − f (x)2 + f (x)3 for every real number x.
π π 3π Then, (1998, 2M)
(a) (b) (c) (d) π
3 2 2 (a) h is increasing, whenever f is increasing
(b) h is increasing, whenever f is decreasing
10. If f (x) = xex (1 − x ) , then f (x) is (2001, 2M)
(c) h is decreasing, whenever f is decreasing
(a) increasing in [−1 / 2, 1] (b) decreasing in R (d) Nothing can be said in general
(c) increasing in R (d) decreasing in [−1 / 2, 1]
11. For all x ∈ (0, 1) (2000, 1M) Fill in the Blanks
(a) e < 1 + x
x
(b) log e (1+ x) < x 20. The set of all x for which log (1 + x) ≤ x is equal to ..... .
(c) sin x > x (d) log ε x > x (1987, 2M)
12. Let f (x) = ∫ ex (x − 1) (x − 2) dx. Then, f decreases in the 21. The function y = 2x − log|x| is monotonically
2

interval (2000, 2M) increasing for values of x ( ≠ 0), satisfying the


(a) (−∞ , − 2) (b) (−2, − 1) (c) (1, 2) (d) (2, ∞ ) inequalities… and monotonically decreasing for values
13. The function f (x) = sin x + cos x increases, if (1999, 2M)
4 4 of x satisfying the inequalities… . (1983, 2M)

π π 3π
(a) 0 < x < (b) < x< Match the Columns
8 4 8
3π 5π 5π 3π Directions (Q.Nos. 22-24) by appropriately matching
(c) < x< (d) < x<
8 8 8 4 the information given in the three columns of the
x x following table.
14. If f (x) = and g (x) = , where 0 < x ≤ 1, then in
sin x tan x Let f (x) = x + log e x − x log e x, x ∈ (0, ∞ )
this interval (1997, 2M) Column 1 contains information about zeros of f (x), f ′ (x)
(a) both f (x) and g (x) are increasing functions and f ′′ (x).
(b) both f (x) and g (x) are decreasing functions Column 2 contains information about the limiting
(c) f (x) is an increasing function behaviour of f (x), f ′ (x) and f ′′ (x) at infinity.
(d) g (x) is an increasing function Column 3 contains information about increasing/decreasing
log (π + x) nature of f (x) and f ′ (x).
15. The function f (x) = is (1995, 1M)
log (e + x)
Column-1 Column-2 Column-3
(a) increasing on (0, ∞ )
(I) f( x ) = 0 for some (i) lim f( x ) = 0 (P) f is increasing
(b) decreasing on (0, ∞ ) x→ ∞
x ∈ (1, e 2 ) in (0, 1)
(c) increasing on (0, π / e), decreasing on ( π / e, ∞ )
(d) decreasing on (0, π / e), increasing on ( π / e, ∞ ) (II) f ′ ( x ) = 0 for some (ii) lim f( x ) = − ∞ (Q) f is decreasing
x→ ∞
x ∈ (1, e ) in (e, e 2)
16. Let f and g be increasing and decreasing functions
respectively from [0, ∞ ) to [0, ∞ ) and h (x) = f { g (x)}. If (III) f ′ ( x ) = 0 for some (iii) lim f ′ ( x ) = − ∞ (R) f′ is increasing
x→ ∞
h(0) = 0, then h (x) − h (1) is (1987, 2M) x ∈ ( 0, 1) in (0, 1)
(a) always negative (b) always positive (IV) f ′′( x ) = 0 for some (iv) lim f ′′( x ) = 0 (S) f′ is decreasing
x→ ∞
(c) strictly increasing (d) None of these x ∈ (1, e ) in (e, e 2)

22. Which of the following options is the only INCORRECT


Objective Questions II combination? (2017 Adv.)
(One or more than one correct option) (a) (I) (iii) (P) (b) (II) (iv) (Q)
(c) (II) (iii) (P) (d) (III) (i) (R)
17. If f : R → R is a differentiable function such that
f ′ (x) > 2 f (x) for all x ∈ R, and f (0) = 1 then (2017 Adv.)
23. Which of the following options is the only CORRECT
combination?
(a) f (x) > e in (0, ∞ )
2x
(b) f ′ (x) < e in (0, ∞ )
2x
(a) (I) (ii) (R) (b) (III) (iv) (P)
(c) f (x) is increasing in (0, ∞ ) (d) f (x) is decreasing in (0 ∞ ) (c) (II) (iii) (S) (d) (IV) (i) (S)
242 Application of Derivatives

24. Which of the following options is the only CORRECT 27. Using the relation 2 (1 − cos x) < x2, x ≠ 0 or prove that
combination? sin (tan x) ≥ x , ∀ x ∈ [0, π / 4]. (2003, 4M)
(a) (III) (iii) (R) (b) (IV) (iv) (S)
(c) (II) (ii) (Q) (d) (I) (i) (P) 28. If −1 ≤ p ≤ 1, then show that the equation
25. Match the conditions/expressions in Column I with 4x3 − 3x − p = 0 has a unique root in the interval [1 / 2, 1]
statements in Column II. and identify it. (2001, 5M)
Let the functions defined in Column I have domain  xeax , x≤0
(−π / 2, π / 2). 29. Let f (x) = 
 x + ax 2
− x3
, x >0
Column I Column II
where, a is a positive constant. Find the interval in
A. x + sin x p. increasing which f ′ (x) is increasing. (1996, 3M)
B. sec x q. decreasing
30. Show that 2 sin x + 2 tan x ≥ 3x, where 0 ≤ x < π /2.
r. neither increasing nor decreasing (1990, 4M)

Analytical & Descriptive Questions 31. Show that 1 + x log (x + x2 + 1 ) ≥ 1 + x2 ∀x ≥ 0.


(1983, 2M)
3x (x + 1)  π π π
26. Prove that sin x + 2x ≥
π
, ∀ x ∈ 0,
 2  32. Given A = x : ≤ x ≤  and f (x) = cos x – x (1 + x). Find
 6 3
(1980, 2M)
(Justify the inequality, if any used). (2004, 4M) f ( A ).

Topic 3 Rolle’s and Lagrange’s Theorem


Objective Question I (Only one correct option) (a) Statement I is correct, Statement II is also correct;
Statement II is the correct explanation of Statement I
1. If f : R → R is a twice differentiable function such that
(b) Statement I is correct, Statement II is also correct;
 1 1
f ′′ (x) > 0 for all x ∈ R, and f   = , f (1) = 1, then Statement II is not the correct explanation of
 2 2 (2017 Adv.) Statement I
(a) f ′ (1) ≤ 0 (b) f ′ (1) > 1 (c) Statement I is correct; Statement II is incorrect
1 1
(c) 0 < f ′ (1) ≤ (d) < f ′ (1) ≤ 1 (d) Statement I is incorrect; Statement II is correct
2 2
Analytical & Descriptive Question
Assertion and Reason
3. If f (x) and g (x) are differentiable functions for 0 ≤ x ≤ 1,
2. Let f (x) = 2 + cos x, for all real x. such that f (0) = 2, g (0) = 0
Statement I For each real t, there exists a point c in f (1) = 6, g (1) = 2
[t , t + π ], such that f ' (c) = 0. Because
Then, show that there exists c satisfying 0 < c < 1 and
Statement II f (t ) = f (t + 2π ) for each real t. (2007, 3M) f ′ (c) = 2 g′ (c). (1982, 2M)

Topic 4 Maxima and Minima


Objective Questions I (Only one correct option) 2. Let f (x) = 5 −|x − 2| and g (x) = |x + 1|, x ∈ R. If f (x)
1. Consider the rectangles lying in the region attains maximum value at α and g (x) attains minimum
(x − 1) (x2 − 5x + 6)
 π  value of β, then lim is equal to
(x, y) ∈ R × R : 0 ≤ x ≤ and 0 ≤ y ≤ 2 sin(2x) x → − αβ x2 − 6x + 8
 2 
(2019 Main, 12 April II)
and having one side on the X-axis. The area of the
(a) 1/2 (b) − 3 / 2 (c) − 1 / 2 (d) 3/2
rectangle which has the maximum perimeter among all
such rectangles, is (2020 Adv.) 3. If the volume of parallelopiped formed by the vectors
3π $i + λ$j + k
$ , $j + λk
$ and λ$i + k
$ is minimum, then λ is equal
(a) (b) π
2 to (2019 Main, 12 April I)
π π 3 1 1
(c) (d) (a) − (b) (c) 3 (d) − 3
2 3 2 3 3
Application of Derivatives 243

4. If m is the minimum value of k for which the function 14. If 20 m of wire is available for fencing off a flower-bed in
f (x) = x kx − x2 is increasing in the interval [0, 3] and M the form of a circular sector, then the maximum area (in
sq. m) of the flower-bed is (2017 Main)
is the maximum value of f in the interval [0, 3] when
k = m, then the ordered pair (m, M ) is equal to (a) 12.5 (b) 10
(2019 Main, 12 April I) (c) 25 (d) 30
(a) (4, 3 2 ) (b) (4, 3 3 ) 15. A wire of length 2 units is cut into two parts which
(c) (3, 3 3 ) (d) (5, 3 6 ) are bent respectively to form a square of side = x units
5. If f (x) is a non-zero polynomial of degree four, having and a circle of radius = r units. If the sum of the
local extreme points at x = − 1, 0, 1, then the set areas of the square and the circle so formed is
S = { x ∈ R : f (x) = f (0)} contains exactly minimum, then (2016 Main)

(a) four rational numbers (2019 Main, 9 April I) (a) 2x = ( π + 4)r (b) (4 − π )x = πr
(b) two irrational and two rational numbers (c) x = 2r (d) 2x = r
(c) four irrational numbers 1
16. The least value of α ∈ R for which 4αx2 + ≥ 1 , for all
(d) two irrational and one rational number x
6. The height of a right circular cylinder of maximum x > 0, is (2016 Adv.)
volume inscribed in a sphere of radius 3 is 1 1 1 1
(a) (b) (c) (d)
(2019 Main, 8 April II) 64 32 27 25
2
(a) 6 (b) 2 3 (c) 3 (d) 3 17. Let f (x) be a polynomial of degree four having extreme
3
lim  f (x) 
values at x = 1 and x = 2. If x→ 0 1+ = 3, then f (2) is
7. If S1 and S 2 are respectively the sets of local  x2 
minimum and local maximum points of the function, equal to
f (x) = 9x4 + 12x3 − 36x2 + 25, x ∈ R, then (a) −8 (b) −4
(a) S1 = {−2} ; S2 = {0,1} (2019 Main, 8 April I) (c) 0 (d) 4
(b) S1 = {−2, 0} ; S2 = {1}
18. If x = − 1 and x = 2 are extreme points of
(c) S1 = {−2,1} ; S2 = {0}
(d) S1 = {−1} ; S2 = {0, 2}
f (x) = α log|x| + βx2 + x, then (2014 Main)
1 1
(a) α = − 6, β = (b) α = − 6, β = −
8. The shortest distance between the line y = x and the 2 2
curve y2 = x − 2 is (2019 Main, 8 April I)
(c) α = 2, β = −
1
(d) α = 2, β =
1
7 7 11 2 2
(a) 2 (b) (c) (d)
8 4 2 4 2 19. The number of points in (− ∞ , ∞ ) for which
9. The maximum area (in sq. units) of a rectangle having x2 − x sin x − cos x = 0, is (2013 Adv.)
its base on the X-axis and its other two vertices on the (a) 6 (b) 4 (c) 2 (d) 0
parabola, y = 12 − x2 such that the rectangle lies inside
the parabola, is (2019 Main, 12 Jan I) 20. Let f, g and h be real-valued functions defined on the
2 2 2 2
(a) 36 (b) 20 2 (c) 32 (d) 18 3 interval [0, 1] by f (x) = ex + e− x , g (x) = x ex + e− x and
2 2
10. The maximum value of the function h (x) = x2ex + e− x . If a, b and c denote respectively, the
f (x) = 3x3 − 18x2 + 27x − 40 absolute maximum of f, g and h on [0, 1], then (2010)
on the set S = { x ∈ R : x2 + 30 ≤ 11x} is (2019 Main, 11 Jan I) (a) a = b and c ≠ b (b) a = c and a ≠ b
(a) 122 (b) − 122 (c) − 222 (d) 222 (c) a ≠ b and c ≠ b (d) a = b = c

11. Let A (4, − 4) and B(9, 6) be points on the parabola, 21. The total number of local maxima and local minima of
y2 = 4x. Let C be chosen on the arc AOB of the parabola, (2 + x)3 , − 3 < x ≤ −1
where O is the origin, such that the area of ∆ACB is the function f (x) =  2 is (2008, 3M)
x , −1 < x < 2
3
maximum. Then, the area (in sq. units) of ∆ACB, is
(2019 Main, 9 Jan II) (a) 0 (b) 1
1 3 1 (c) 2 (d) 3
(a) 31 (b) 32 (c) 31 (d) 30
4 4 2
22. If f (x) = x2 + 2bx + 2c2 and g (x) = − x2 − 2cx + b2, such
12. The maximum volume (in cu.m) of the right circular that min f (x) > max g (x), then the relation between b
cone having slant height 3m is (2019 Main, 9 Jan I) and c, is (2003, 2M)
4
(a) π (b) 2 3π (c) 3 3π (d) 6π (a) No real value of b and c (b) 0 < c < b 2
3
(c)| c|< | b| 2 (d)| c|> | b| 2
1 1
13. Let f (x) = x2 + and g (x) = x − , x ∈ R − { −1, 0, 1}. If
x2 x 23. If f (x) = | x|, for 0 < | x| ≤ 2 . Then, at x = 0, f has
f (x) 1 , for x=0
h (x) = , then the local minimum value of h (x) is (2000, 1M)
g (x) (2018 Main) (a) a local maximum (b) no local maximum
(a) 3 (b) −3 (c) −2 2 (d) 2 2 (c) a local minimum (d) no extremum
244 Application of Derivatives

x2 − 1 32. Let f : R → R be given by


24. If f (x) = , for every real number x, then the
x2 + 1 x5 + 5x4 + 10x3 + 10x2 + 3x + 1, x < 0;
minimum value of f (1998, 2M) 
 x − x + 1,
2
0 ≤ x < 1;
(a) does not exist because f is unbounded 2 3 8
(b) is not attained even though f is bounded
f (x) =  x − 4x + 7x − ,
2
1 ≤ x < 3;
(c) is 1 (d) is –1
 3 3
 10
25. The number of values of x, where the function  (x − 2) log e (x − 2) − x + 3 , x ≥ 3;

f (x) = cos x + cos ( 2x) attains its maximum, is(1998, 2M) Then which of the following options is/are correct?
(a) 0 (b) 1 (c) 2 (d) infinite (a) f is increasing on (− ∞ , 0) (2019 Adv.)

26. On the interval [0,1], the function x 25


(1 − x)
75
takes its (b) f ′ is NOT differentiable at x = 1
(c) f is onto
maximum value at the point (1995, 1M)
(a) 0 (b) 1/4 (c) 1/2 (d) 1/3 (d) f ′ has a local maximum at x = 1

27. Find the coordinates of all the points P on the ellipse cos(2x) cos(2x) sin(2x)
x 2
y 2 33. If f (x) = 
− cos x cos x − sin x 

, then (2017 Adv.)
+ = 1, for which the area of the ∆PON is
a 2 b2  sin x sin x cos x 
maximum, where O denotes the origin and N is the foot (a) f (x) attains its minimum at x = 0
of the perpendicular from O to the tangent at P. (b) f (x) attains its maximum at x = 0
(1990, 10M) (c) f ′ (x) = 0 at more than three points in (− π , π )
 ± a2 ± b2   ± a2 ± b2  (d) f ′ (x) = 0 at exactly three points in (− π , π )
(a)  , 
 (b)  , 

 a 2 + b2 a 2 + b2   a 2 – b2 a 2 – b2  34. Let f : R → (0, ∞ ) and g : R → R be twice differentiable
 ± a2 ± b2   ± a2 ± b2  functions such that f′ ′ and g′ ′ are continuous functions
(c)  , 
 (d)  , 

 a +b
2 2
a 2 – b2   a –b
2 2
a 2 + b2  on R. Suppose f ′ (2) = g (2) = 0, f ′ ′(2) ≠ 0 and g′ (2) ≠ 0.
f (x) g (x)
28. If P (x) = a 0 + a1x2 + a 2x4 +... + a nx2n is a polynomial in If lim = 1, then (2016 Adv.)
x → 2 f ′ (x) g′ (x)
a real variable x with 0 < a 0 < a1 < a 2 < K < a n. Then,
the function P (x) has (1986, 2M) (a) f has a local minimum at x = 2
(a) neither a maximum nor a minimum (b) f has a local maximum at x = 2
(b) only one maximum (c) f ′ ′ (2) > f (2)
(c) only one minimum (d) f (x) − f ′ ′ (x) = 0, for atleast one x ∈ R
(d) only one maximum and only one minimum 35. The function f (x) = 2|x| + |x + 2| − ||x + 2| − 2|x|| has a
29. If y = a log x + bx2 + x has its extremum values at x = − 1 local minimum or a local maximum at x is equal to
and x = 2 , then (1983, 1M) (2013 Adv.)
1 −2
(a) a = 2, b = − 1 (b) a = 2, b = − (a) −2 (b) (c) 2 (d) 2/3
2 3
1
(c) a = − 2, b = (d) None of the above 36. A rectangular sheet of fixed perimeter with sides
2 having their lengths in the ratio 8 : 15 is converted into
30. If p, q and r are any real numbers, then (1982, 1M) an open rectangular box by folding after removing
(a) max ( p , q) < max ( p , q, r ) squares of equal area from all four corners. If the total
1 area of removed squares is 100, the resulting box has
(b) min ( p , q) = ( p + q − | p − q|) maximum volume. The lengths of the sides of the
2
(c) max ( p , q) < min ( p , q, r ) rectangular sheet are (2013 Adv.)
(d) None of the above (a) 24 (b) 32 (c) 45 (d) 60
 e x
, 0 ≤ x≤1
Objective Questions II  x
37. If f (x) = 2 − ex − 1 , 1 < x≤2 and g (x) = ∫ f (t ) dt ,
0
(One or more than one correct option)  x−e , 2 < x≤3

31. Let f : R → R be given by f (x) = (x − 1)(x − 2)(x − 5). x ∈ [1, 3], then (2006, 3M)
x
(a) g (x) has local maxima at x = 1 + log e 2 and local
Define F (x) = ∫ f (t )dt, x > 0
minima at x = e
0
(b) f (x) has local maxima at x = 1and local minima at x = 2
Then which of the following options is/are correct?
(c) g (x) has no local minima
(2019 Adv.)
(d) f (x) has no local maxima
(a) F (x) ≠ 0 for all x ∈ (0, 5)
(b) F has a local maximum at x = 2 38. If f (x) is a cubic polynomial which has local maximum at
(c) F has two local maxima and one local minimum in (0, ∞ ) x = − 1. If f (2) = 18, f (1) = − 1 and f ′ (x) has local
(d) F has a local minimum at x = 1 minimum at x = 0, then (2006, 3M)
Application of Derivatives 245

(a) the distance between (– 1, 2) and (a , f (a )), where x = a ex f ′ (t )


44. Let g (x) = ∫ dt. Which of the following is true?
is the point of local minima, is 2 5 0 1 + t2
(b) f (x) is increasing for x ∈[1, 2 5 ]
(a) g ′ (x) is positive on (− ∞ , 0) and negative on (0, ∞ )
(c) f (x) has local minima at x = 1 (b) g ′ (x) is negative on (− ∞ , 0) and positive on (0, ∞ )
(d) the value of f (0) = 5 (c) g ′ (x) changes sign on both (− ∞ , 0) and (0, ∞ )
39. The function (d) g ′ (x) does not change sign (− ∞ , ∞ )
x
f (x) = ∫ t (et − 1) (t − 1) (t − 2)3 (t − 3)5 dt has a local Analytical & Descriptive Questions
−1
minimum at x equals (1999, 3M) 45. If f (x) is twice differentiable function such that f (a ) = 0,
(a) 0 (b) 1 (c) 2 (d) 3 f (b) = 2, f (c) = 1, f (d ) = 2, f (e) = 0, where a < b < c < d < e,
40. If f (x) = 3x + 12x − 1, − 1 ≤ x ≤ 2, then
2
then the minimum number of zeroes of
(1993, 3M)
 − x,
37 2 < x≤3 g (x) = { f ′ (x)}2 + f ′′ (x) ⋅ f (x) in the interval [a, e] is
(a) f (x) is increasing on [–1, 2] 46. For the circle x2 + y2 = r 2, find the value of r for which
(b) f (x) is continuous on [–1, 3] the area enclosed by the tangents drawn from the point
(c) f ′(2) does not exist P(6, 8) to the circle and the chord of contact is
(d) f (x) has the maximum value at x = 2 maximum. (2003, 2M)

Match the Columns 47. Find a point on the curve x2 + 2 y2 = 6 whose distance
from the line x + y = 7, is minimum. (2003, 2M)
41. A line L : y = mx + 3 meets Y -axis at E (0, 3) and the
arc of the parabola y = 16x, 0 ≤ y ≤ 6 at the point
2
48. Let f (x) is a function satisfying the following conditions
F (x0 , y0 ). The tangent to the parabola at F (x0 , y0 ) (i) f (0) = 2, f (1) = 1
intersects the Y -axis at G (0, y1 ). The slope m of the
(ii) f (x) has a minimum value at x = 5 / 2 and
line L is chosen such that the area of the ∆EFG has a
2 ax 2ax − 1 2 ax + b + 1
local maximum
(iii) For all x, f ′ (x) = b b+ 1 −1
Match List I with List II and select the correct answer
using the codes given below the list. 2 (ax + b) 2 ax + 2b + 1 2 ax + b

Column I Column II
where, a and b are some constants. Determine the
P. m= 1. 1/2
constants a , b and the function f (x). (1998, 8M)
Q. Maximum area of ∆EFG is 2. 4
R. y0 = 3. 2
49. Let C1 and C 2 be respectively, the parabolas x2 = y – 1
and y2 = x – 1. Let P be any point on C1 and Q be any
S. y1 = 4. 1
point on C 2. If P1 and Q1 is the reflections of P and Q,
Codes respectively, with respect to the line y = x. Prove that P1
P Q R S P Q R S lies on C 2 Q1 lies on C1 and PQ ≥ min (PP1 , QQ1 ). Hence,
(a) 4 1 2 3 (b) 3 4 1 2 determine points P0 and Q0 on the parabolas C1 and C 2
(c) 1 3 2 4 (d) 1 3 4 2 respectively such that P0Q0 ≤ PQ for all pairs of points
(P , Q ) with P on C1 and Q on C 2.
Passage Based Problems
50. If S is a square of unit area. Consider any quadrilateral
Consider the function f : (−∞ , ∞ ) → (−∞ , ∞ ) defined by which has one vertex on each side of S. If a , b, c and d
x2 − ax + 1 denote the length of the sides of the quadrilateral, then
f (x) = 2 ; 0 < a < 2. (2008, 12M)
x + ax + 1 prove that 2 ≤ a 2 + b2 + d 2 ≤ 4. (1997, 5M)

42. Which of the following is true ? 51. Determine the points of maxima and minima of the
1
(a) (2 + a ) f ′ ′ (1) + (2 − a ) f ′ ′ (−1) = 0
2 2
functionf (x) = ln x − bx + x2, x > 0, where b ≥ 0 is a
(b) (2 − a )2 f ′ ′ (1) − (2 + a )2 f ′ ′ (−1) = 0
8
constant. (1996, 5M)
(c) f ′ (1) f ′ (−1) = (2 − a )2
(d) f ′ (1) f ′ (−1) = − (2 + a )2 52. Let (h , k) be a fixed point, where h > 0 , k > 0. A straight
43. Which of the following is true ? line passing through this point cuts the positive
(a) f (x) is decreasing on (−1, 1) and has a local minimum at
directions of the coordinate axes at the points P and Q.
x=1 Find the minimum area of the ∆OPQ, O being the
(b) f (x) is increasing on (−1, 1) and has a local maximum at origin. (1995, 5M)
x=1 53. The circle x2 + y2 = 1 cuts the X-axis at P and Q. Another
(c) f (x) is increasing on (−1, 1) but has neither a local circle with centre at Q and variable radius intersects
maximum nor a local minimum at x = 1 the first circle at R above the X-axis and the line
(d) f (x) is decreasing on (−1, 1) but has neither a local segment PQ at S. Find the maximum area of the ∆QSR.
maximum nor a local minimum at x = 1 (1994, 5M)
246 Application of Derivatives

 3 (b3 − b2 + b − 1) 66. For a polynomial g (x) with real coefficients, let mg


− x + , 0 ≤ x≤1
54. Let f (x) =  (b2 + 3b + 2) denote the number of distinct real roots of g (x). Suppose
 2x − 3, 1 ≤ x≤3
 S is the set of polynomials with real coefficients defined
Find all possible real values of b such that f (x) has the by
smallest value at x = 1. (1993, 5M) S = {(x2 − 1)2(a 0 + a1x + a 2x2 + a3 x3 ) : a 0 , a1 , a 2, a3 ∈R};
55. What normal to the curve y = x2 forms the shortest For a polynomial f, let f′ and f′ ′ denote its first and
chord? (1992, 6M)
second order derivatives, respectively. Then the
minimum possible value of (mf ′ + mf ′ ′ ), where f ∈ S, is
56. A window of perimeter (including the base of the arch) is ……… (2020 Adv.)
in the form of a rectangle surmounted by a semi-circle.
The semi-circular portion is fitted with coloured glass 67. Let AD and BC be two vertical poles at A and B
while the rectangular part is fitted with clear glass. The respectively on a horizontal ground. If AD = 8 m,
clear glass transmits three times as much light per BC = 11 m and AB = 10 m; then the distance (in meters)
of a point M on AB from the point A such that
square meter as the coloured glass does. What is the
MD 2 + MC 2 is minimum is ........ (2020 Main, 6 Sep I)
ratio for the sides of the rectangle so that the window
transmits the maximum light? (1991, 4M) 68. A cylindrical container is to be made from certain solid
57. A point P is given on the circumference of a circle of material with the following constraints : It has a fixed
radius r. Chord QR is parallel to the tangent at P. inner volume of V mm3 , has a 2 mm thick solid wall and
Determine the maximum possible area of the ∆PQR. is open at the top. The bottom of the container is a solid
(1990, 4M) circular disc of thickness 2 mm and is of radius equal to
the outer radius of the container.
58. Find the point on the curve 4x + a y = 4a , 4 < a 2 < 8
2 2 2 2
If the volume of the material used to make the container
that is farthest from the point (0, –2). (1987, 4M)
is minimum, when the inner radius of the container is
59. Let A ( p2, − p) B(q2, q), C (r 2, − r ) be the vertices of the V
10 mm, then the value of is (2015 Adv.)
triangle ABC. A parallelogram AFDE is drawn with 250 π
vertices D, E and F on the line segments BC, CA and AB,
respectively. Using calculus, show that maximum area 69. A vertical line passing through the point (h , 0)
1 x2 y 2
of such a parallelogram is ( p + q)(q + r )( p − r ). intersects the ellipse + = 1 at the points P and Q. If
4 4 3
(1986, 5M)
π π the tangents to the ellipse at P and Q meet at the point
60. Let f (x) = sin3 x + λ sin 2 x, − < x < ⋅ Find the
2 2 R.
intervals in which λ should lie in the order that f (x) has If ∆(h ) = area of the ∆ PQR, ∆1 = max ∆ (h ) and
exactly one minimum and exactly one maximum. 1/ 2 ≤ h ≤ 1
8
(1985, 5M) ∆ 2 = min ∆ (h ), then ∆1 − 8∆ 2 is equal to (2013 Adv.)
1/ 2 ≤ h ≤ 1 5
x
61. Find the coordinates of the point on the curve y = ,
1 + x2 70. Let f : R → R be defined as f (x) = | x| + | x2 − 1|. The total
where the tangent to the curve has the greatest slope. number of points at which f attains either a local
(1984, 4M) maximum or a local minimum is (2012)
62. A swimmer S is in the sea at a distance d km from the 71. Let p(x) be a real polynomial of least degree which has a
closest point A on a straight shore. The house of the
local maximum at x = 1 and a local minimum at x = 3. If
swimmer is on the shore at a distance L km from A. He
p(1) = 6 and p(3) = 2, then p′ (0) is equal to (2012)
can swim at a speed of u km/h and walk at a speed of
v km/h (v > u ). At what point on the shore should be land 72. The number of distinct real roots of
so that he reaches his house in the shortest possible x4 − 4x3 + 12x2 + x − 1 = 0 is……
time? (1983, 2M)
73. Let f be a function defined on R (the set of all real
63. If ax2 + b / x ≥ c for all positive x where a > 0 and b > 0, numbers) such that f ′ (x) = 2010 (x − 2009) (x − 2010)2
then show that 27ab ≥ 4c . 2 3
(1982, 2M) (x − 2011)3 (x − 2012)4, ∀x ∈ R. If g is a function defined
64. If x and y be two real variables such that x > 0 and on R with values in the interval (0, ∞ ) such that
xy = 1. Then, find the minimum value of x + y. (1981, 2M) f (x) = ln ( g (x)), ∀ x ∈ R, then the number of points in R
at which g has a local maximum is…… (2010)
Integer & Numerical Answer Type Questions 74. The maximum value of the expression
65. Let the function f : (0, π) → R be defined by 1
is …… (2010)
sin 2 θ + 3 sin θ cos θ + 5 cos 2 θ
f (θ ) = (sin θ + cos θ ) + (sin θ − cos θ )
2 4

Suppose the function f has a local minimum at θ 75. The maximum value of the function
precisely when θ ∈{ λ 1π, ... , λ rπ }, where 0 < λ 1 < ... λ r < 1. f (x) = 2x3 − 15x2 + 36x − 48 on the set
Then the value of λ 1 + ... + λ r is ....... . (2020 Adv.) A = { x | x2 + 20 ≤ 9x} is ……… . (2009)
Answers
Topic 1 9. (c) 10. (a) 11. (a) 12. (b)
1. (a) 2. (c) 3. (b) 4. (d) 13. (d) 14. (c) 15. (c) 16. (c)
5. (c) 6. (b) 7. (d) 8. (b) 17. (c) 18. (c) 19. (c) 20. (d)
9. (b) 10. (d) 11. (d) 12. (d) 21. (c) 22. (d) 23. (a) 24. (d)
13. (c) 14. (b, d) 15. (b, c)
25. (b) 26. (b) 27. (a)
16. H = φ, V = {1, 1 } 17. y − 2 = 0
28. (c) 29. (b) 30. (b) 31. (a,b,d)
1 3
18. a = – , b = – , c = 3 19. 1 : 16 32. (b,c,d) 33. (b, c) 34. (a, d) 35. (a, b)
2 4
36. (a, c) 37. (a, b) 38. (b, c) 39. (b, d)
20. y + x − 1 = 0
40. (a, b, c, d) 41. (a) P → 4 Q → 1 R → 2 S → 3
π − 3π
21. x + 2y = and x + 2y = 42. (a) 43. (a) 44. (a) 45. 6
2 2
46. 5 units 47. (2, 1)
22. (8)
1 −5 1 5
48. a = , b = ; f ( x ) = x 2 − x + 2
Topic 2 4 4 4 4
1. (d) 2. (a) 3. (b) 4. (c)  1 5  5 1
49. P0 =  ,  and Q0 =  , 
5. (c) 6. (a) 7. (c) 8. (a)  2 4  4 2
9. (a) 10. (a) 11. (b) 12. (c) (b − b 2 − 1 ) 1
51. Maxima at x = and minima at x = (b + b 2 − 1 )
13. (b) 14. (c) 15. (b) 16. (d) 4 4
17. (a, c) 18. (c, d) 19. (a, c) 20. x > − 1 4 3
52. 2hk 53.
 1  1   1  1 9
21. x ∈  − , 0 ∪  , ∞ , x ∈  − ∞, −  ∪  0, 
 2  2   2  2 54. b ∈ ( −2, − 1 ) ∪ [1, ∞ ]
22. (d) 23. (c)
24. (c)
55. 2 x − 2y + 2 = 0, 2 x + 2y − 2 = 0
 2 a
25. A → p, B → r 29. – ,
 a 3  3 3 2
56. 6 : 6 + π 57. r sq units 58. (0, 2)
1 π  4
π 3 π  π 
32.  − 1 +  , − 1 +    3 3
 2 3  3  2 6  6 60. λ ∈  − ,  61. x = 0, y = 0
 2 2
Topic 3 ud
62. 64. (2)
1. (b) 2. (b) v − u2
2

Topic 4 65. 0.50 66. (5) 67. (5)


1. (c) 2. (a) 3. (b) 4. (b) 68. (4) 69. (9) 70. (5) 71. (9)
5. (d) 6. (b) 7. (c) 8. (c) 72. (2) 73. (1) 74. (2) 75. (7)

Hints & Solutions


Topic 1 Equations of Tangent and Normal ∴ Slope of this line = −
2 dy
=
6 dx ( α , β )
1. Equation of given curve is
x α2 + 3 1
y= , x ∈ R, (x ≠ ± 3 ) …(i) ⇒ − =−
x −3
2 (α 2 − 3)2 3
On differentiating Eq. (i) w.r.t. x, we get ⇒ 3α 2 + 9 = α 4 − 6 α 2 + 9
⇒ α4 − 9 α2 = 0
dy (x2 − 3) − x(2x) (− x2 − 3)
= = 2 ⇒ α = 0, − 3, 3
dx (x2 − 3)2 (x − 3)2
⇒ α = 3 or − 3, [Q α ≠ 0]
It is given that tangent at a point (α , β ) ≠ (0, 0) on it is Now, from Eq. (i),
parallel to the line α 3 −3 1 1
β= 2 ⇒ β= or = or −
2x + 6 y − 11 = 0. α −3 9 −3 9 −3 2 2
248 Application of Derivatives

According to the options,|6 α + 2 β | = 19 Now, equation of tangent to the curve (i) at point
 1  7 1
at (α , β ) =  ± 3, ±   ,−  and having slope 2, is
 2  2 4
1  7 1
2. Given curve is y = f (x) = x3 − x2 − 2x y + = 2  x −  ⇒ y + = 2x − 7
...(i) 4  2 4
So, f (1) = 1 − 1 − 2 = −2 ⇒ y = 2x −
29
…(iii)
and f (−1) = −1 − 1 + 2 = 0 4
1 
Since, slope of a line passing through (x1 , y1 ) and On checking all the options, we get the point  , − 7
y −y 8 
(x2, y2) is given by m = tan θ = 2 1
x2 − x1 satisfy the line (iii).
∴Slope of line joining points (1, f (1)) and 5. The helicopter is nearest to the soldier, if the tangent to
f (1) − f (−1) −2 − 0 the path, y = x3/ 2 + 7, (x ≥ 0) of helicopter at point (x, y) is
(−1, f (−1)) is m = = = −1 perpendicular to the line joining (x, y) and the position
1 − (−1) 1+1
1 
dy of soldier  , 7 .
Now, = 3x2 − 2x − 2 [differentiating Eq. (i), w.r.t. ‘x’] 2 
dx
According to the question,
dy
=m
dx
⇒ 3 x2 − 2 x − 2 = − 1 ⇒ 3 x2 − 2 x − 1 = 0
1 (x, y)
⇒ (x − 1) (3x + 1) = 0 ⇒ x = 1 ,−
3 y=x3/2+7
 1 
Therefore, set S = − , 1.
 3  (1/2, 7)

3. Given curve is y = x + ax − b
3
…(i) Q Slope of tangent at point (x, y) is
passes through point P(1, − 5). dy 3 1/ 2
= x = m1 (let ) …(i)
∴ −5 =1+ a −b⇒b−a =6 …(ii) dx 2
and slope of tangent at point P(1, − 5) to the curve (i), is 1 
and slope of line joining (x, y) and  , 7 is
2 
dy
m1 = = [3x2 + a ](1, −5 ) = a + 3 y−7
dx (1, −5 ) m2 = …(ii)
1
x−
Q The tangent having slope m1 = a + 3 at point P(1, − 5) 2
is perpendicular to line − x + y + 4 = 0 , whose slope is Now, m1 ⋅ m2 = −1
m2 = 1.
3 1/ 2  y − 7 
∴ a + 3 = −1 ⇒ a = −4 [Q m1m2 = −1] ⇒ x   = −1 [from Eqs. (i) and (ii)]
2  x − (1 /2)
Now, on substituting a = −4 in Eq. (ii), we get b = 2
3 1/ 2 x3/ 2
On putting a = −2 and b = 2 in Eq. (i), we get ⇒ x = −1 [Q y = x3/ 2 + 7]
2 1
y = x3 − 4x − 2 x−
2
Now, from option (2, − 2) is the required point which lie 3 2 1
on it. ⇒ x = −x +
2 2
4. The given curve is y = x2 − 5x + 5 …(i) ⇒ 3x2 + 2x − 1 = 0 ⇒ 3x2 + 3x − x − 1 = 0
Now, slope of tangent at any point (x, y) on the curve is ⇒ 3x(x + 1) − 1(x + 1) = 0
1 1
dy ⇒ x = , −1 ⇒ x ≥ 0 ⇒ x =
= 2x − 5 …(ii) 3 3
dx 3/ 2
 1
[on differentiating Eq. (i) w.r.t. x] and So, y=  +7 [Q y = x3/ 2 + 7]
Q It is given that tangent is parallel to line  3
2 y = 4x + 1  1  1 3 / 2 
Thus, the nearest point is  ,   + 7
So,
dy
= 2 [Q slope of line 2 y = 4x + 1 is 2]  3  3  
dx
7 Now, the nearest distance
⇒ 2x − 5 = 2 ⇒ 2x = 7 ⇒ x = 2  2
2  1 1  1
3/ 2   1
2
 1
3

7 =  −  +  7 −   − 7 =   +  
On putting x = in Eq. (i), we get  2 3   3   6  3
2
49 35 69 35 1 1 1 3+4 7 1 7
y= − +5 = − =− = + = = =
4 2 4 2 4 36 27 108 108 6 3
Application of Derivatives 249

6.  dy 6 + 30 36
Key Idea Angle between two curves is the angle between the ⇒   = = =1
tangents to the curves at the point of intersection.
 dx ( 0, 1) 4 ×9 36

Given equation of curves are ∴ Equation of normal at (0, 1) is given by


y = 10 − x2 …(i) −1
y−1 = (x − 0) ⇒ x + y − 1 = 0
and y = 2 + x2 …(ii) 1
For point of intersection, consider  1 1
which passes through the point  ,  .
 2 2
10 − x2 = 2 + x2
⇒ 2 x2 = 8 1 + sin x  π
9. We have, f (x) = tan − 1 , x ∈ 0, 
⇒ x2 = 4 1 − sin x  2
⇒ x=±2 2
Clearly, when x = 2 , then y = 6 (using Eq. (i)) and when  x x
 cos + sin 
x = − 2, then y = 6  2 2
⇒ f (x) = tan − 1 2
Thus, the point of intersection are (2, 6) and  x x
 cos − sin 
(−2, 6).  2 2
Let m1 be the slope of tangent to the curve (i) and m2 be  x x
the slope of tangent to the curve (ii)
−1
 cos + sin 
dy dy = tan  2 2 
For curve (i) = −2x and for curve (ii) = 2x  cos x − sin x 
dx dx  2 2
∴ At (2, 6), slopes m1 = − 4 and m2 = 4, and in that case
 x x x π
Q cos > sin for 0 < < 
m2 − m1 4+4 8  2 2 2 4
|tan θ| = = =
1 + m1m2 1 − 16 15  x
−1
 1 + tan 
At (−2, 6), slopes m1 = 4 and m2 = − 4 and in that case = tan  2 
 1 − tan x 
m2 − m1 −4 − 4 8  2
|tan θ| = = =
1 + m1m2 1 − 16 15   π x  π x
= tan − 1 tan  +   = +
7. We have, y2 = 6x   4 2  4 2


dy
=6 ⇒
dy 3
= 1  π 1
2y ⇒ f ′ (x) = ⇒ f′   =
dx dx y 2  6 2
Slope of tangent at ( x1 , y1 ) is m1 =
3 π
Now, equation of normal at x = is given by
y1 6
Also, 9x 2 + by 2 = 16   π  π
 y − f    = − 2  x − 
dy dy −9x  6  6
⇒ 18x + 2by =0 ⇒ =
dx dx by  π  π   π π π 4π π 
⇒  y −  = − 2  x −  Q f   = + = =
−9x1  3  6   6  4 12 12 3 
Slope of tangent at ( x1 , y1 ) is m2 =
by1
 2π 
Since, these are intersection at right angle. which passes through 0,  .
 3
27x1
∴ m1m2 = − 1 ⇒ =1 10. Given equation of curve is
by12
x2 + 2xy − 3 y2 = 0 …(i)
27x1
⇒ =1 [Q y12 = 6x1] On differentiating w.r.t x, we get
6bx1
x+ y
9 2x + 2xy′ + 2 y − 6 yy′ = 0 ⇒ y′ =
⇒ b= 3y − x
2
8. Given curve is At x = 1, y = 1, y′ = 1
y(x − 2)(x − 3) = x + 6 …(i)  dy
i.e.   =1
Put x = 0 in Eq. (i), we get  dx (1, 1)
y(− 2) (− 3) = 6 ⇒ y = 1
Equation of normal at (1, 1) is
So, point of intersection is (0, 1). 1
x+6 y − 1 = − (x − 1) ⇒ y − 1 = − (x − 1)
Now, y = 1
(x − 2)(x − 3) ⇒ x+ y=2 …(ii)
dy 1 (x − 2)(x − 3) − (x + 6)(x − 3 + x − 2)
⇒ = On solving Eqs. (i) and (ii) simultaneously, we get
dx (x − 2)2(x − 3)2 ⇒ x2 + 2x(2 − x) − 33(2 − x)2 = 0
250 Application of Derivatives

⇒ x2 + 4x − 2x2 − 3(4 + x2 − 4x) = 0 13. Given, x = a (cos θ + θ sin θ )


⇒ − x2 + 4x − 12 − 3x2 + 12x = 0 and y = a (sin θ − θ cos θ )
⇒ − 4x2 + 16x − 12 = 0 dx
∴ = a (− sin θ + sin θ + θ cos θ ) = a θ cos θ
⇒ 4x2 − 16x + 12 = 0 dθ
⇒ x2 − 4 x + 3 = 0 dy
and = a (cos θ − cos θ + θ sin θ )
⇒ (1 − 1)(x − 3) = 0 ⇒ x = 1, 3 dθ
Now, when x = 1, then y = 1 dy
= a θ sin θ ⇒
dy
= tan θ
and when x = 3, then y = − 1 dθ dx
∴ P = (1, 1) and Q = (3, − 1) Thus, equation of normal is
y − a (sin θ − θ cos θ ) − cos θ
Hence, normal meets the curve again at (3, –1) in fourth =
quadrant. x − a (cos θ + θ sin θ ) sin θ
Alternate Solution ⇒ − x cos θ + a θ sin θ cos θ + a cos 2 θ
Given, x2 + 2xy − 3 y2 = 0 = y sin θ + θ a sin θ cos θ − a sin 2 θ
⇒ (x − y)(x + 3 y) = 0 ⇒ x cos θ + y sin θ = a
⇒ x − y = 0 or x + 3 y = 0 whose distance from origin is
Equation of normal at (1, 1) is |0 + 0 − a |
=a
y − 1 = − 1(x − 1) ⇒ x + y − 2 = 0 cos 2 θ + sin 2 θ
It intersects x + 3 y = 0 at (3, –1) and hence normal meets
the curve in fourth quadrant. 14. Given, 4x2 + 9 y2 = 1 …(i)
x + y = 2 Y' On differentiating w.r.t. x, we get
y=x
x + 3y = 0 dy dy 8x 4x
8x + 18 y =0 ⇒ =− =−
(1,1)
dx dx 18 y 9y
X' X The tangent at point (h , k) will be parallel to 8x = 9 y,
O then
(3,–1) 4h 8
− = ⇒ h = − 2k
9k 9
Y'
Point (h , k) also lies on the ellipse.
11. Given, y + 3x = 12 y
3 2
...(i) ∴ 4h 2 + 9k2 = 1 ...(ii)
On differentiating w.r.t. x, we get On putting value of h in Eq. (ii), we get
dy dy 4(− 2k)2 + 9 k2 = 1 ⇒ 16k2 + 9k2 = 1
⇒ 3 y2 + 6x = 12
dx dx 1
⇒ 25k2 = 1 ⇒ k2 =
dy 6x dx 12 − 3 y2 25
⇒ = ⇒ =
dx 12 − 3 y2 dy 6x ⇒ k=±
1
dx 5
For vertical tangent, =0
dy Thus, the point, where the tangents are parallel to
 2 1 2 1
⇒ 12 − 3 y2 = 0 ⇒ y=±2 8x = 9 y are  − ,  and  , −  .
4  5 5 5 5
On putting, y = 2 in Eq. (i), we get x = ± and again
3 Therefore, options (b) and (d) are the answers.
putting y = − 2 in Eq. (i), we get 3x = − 16, no real
2
1
15. Given, xy = 1 ⇒ y =
solution. x
 4  dy 1
So, the required point is  ± , 2 . ⇒ =− 2
 3  dx x
12. Slope of tangent y = f (x) is
dy
= f ′ (x)(3 , 4) Thus, slope of normal = x2 (which is always positive) and
dx a
it is given ax + by + c = 0 is normal, whose slope = − .
Therefore, slope of normal b
a a
1 1 ⇒ − > 0 or <0
=− =− b b
f ′ (x)(3 , 4) f ′ (3)
Hence, a and b are of opposite sign.
1  3π 
But − = tan   [given]
f ′ (3)  4 16. Given, y3 − 3xy + 2 = 0
−1  π π On differentiating w.r.t. x, we get
⇒ = tan  +  = − 1
f ′ (3)  2 4 dy dy
3 y2 − 3x − 3y = 0
∴ f ′ (3) = 1 dx dx
Application of Derivatives 251


dy
(3 y2 − 3x) = 3 y ⇒
dy
=
3y ⇒ 3 = 3a (0)2 + 2b (0) + c
dx dx 3 y2 − 3x ⇒ 3=c …(ii)
For the points where tangent is horizontal, the slope of Since, the curve passes through (–2, 0).
tangent is zero. 0 = a (−2)3 + b(−2)2 + c(−2) + 5
dy 3y
i.e. =0 ⇒ =0 ⇒ 0 = − 8a + 4b − 2c + 5 … (iii)
dx 3 y − 3x
2
From Eqs. (i) and (ii),
⇒ y = 0 but y = 0 does not satisfy the given equation of
12a − 4b = − 3 … (iv)
the curve, therefore y cannot lie on the curve.
So, H =φ [null set] From Eqs. (ii) and (iii),
dy −8 a + 4 b = 1 … (v)
For the points where tangent is vertical, =∞
dx On adding Eqs. (iv) and (v), we get
y
⇒ =∞ 4a = − 2 ⇒ a = − 1 / 2
y2 − x
On putting a = − 1 / 2 in Eq. (iv), we get
⇒ y2 − x = 0 ⇒ y2 = x
12(−1 / 2) − 4b = − 3 ⇒ − 6 − 4b = − 3
On putting this value in the given equation of the curve,
we get ⇒ − 3 = 4b ⇒ b = − 3 / 4
y3 − 3 ⋅ y2 ⋅ y + 2 = 0 ⇒ − 2 y3 + 2 = 0 ∴ a = − 1 / 2 , b = − 3 / 4 and c = 3
⇒ y3 − 1 = 0 ⇒ y3 = 1 19. Let any point P1 on y = x3 be (h , h3 ).
⇒ y=1, x=1
Then, tangent at P1 is
Then, V = {1, 1}
y − h3 = 3h 2(x − h ) ...(i)
17. As | f (x1) − f (x2)| ≤ (x1 − x2)2, ∀x1 , x2 ∈ R
It meets y = x at P2.
3

⇒ | f (x1 ) − f (x2)| ≤ |x1 − x2|2 [as x2 =|x|2 ] On putting the value of y in Eq. (i), we get
f (x1 ) − f (x2) x3 − h3 = 3h 2(x − h )
∴ ≤ |x1 − x2|
x1 − x2 ⇒ (x − h ) (x2 + xh + h 2) = 3h 2(x − h )
 f (x1 ) − f (x2)  ⇒ x2 + xh + h 2 = 3h 2 or x = h
⇒ lim   ≤ lim |x1 − x2|
x1 → x2  x1 − x2  x1 → x2 ⇒ x2 + xh − 2h 2 = 0
⇒ | f ′ (x1 )| ≤ 0, ∀x1 ∈ R ⇒ (x − h ) (x + 2h ) = 0
∴ | f ′ (x)| ≤ 0, which shows| f ′ (x)| = 0 ⇒ x = h or x = − 2h
[as modulus is non negative or| f ′ (x)| ≥ 0] Therefore, x = − 2h is the point P2,
∴ f ′ (x) = 0 or f (x) is constant function. which implies y = − 8h3
⇒ Equation of tangent at (1, 2) is Hence, point P2 ≡ (−2h , − 8h3 )
y−2 Again, tangent at P2 is y + 8h3 = 3 (−2h )2(x + 2h ).
= f ′ (x) or y − 2 = 0 [Q as f ′ (x) = 0]
x−1 It meets y = x3 at P3
⇒ y − 2 = 0 is required equation of tangent. ⇒ x3 + 8h3 = 12h 2(x + 2h )
18. Given, y = ax3 + bx2 + cx + 5 touches X-axis at P(−2, 0) ⇒ x − 2hx − 8h 2 = 0
2

which implies that X-axis is tangent at (–2, 0) and the ⇒ (x + 2h ) (x − 4h ) = 0 ⇒ x = 4h


curve is also passes through (–2, 0). ⇒ y = 64h3
The curve cuts Y-axis at (0, 5) and gradient at this point Therefore, P3 ≡ (4h , 64h3 )
is given 3, therefore at (0, 5) slope of the tangent is 3.
Similarly, we get P4 ≡ (− 8h , − 83 h3 )
dy
Now, = 3ax2 + 2bx + c Hence, the abscissae are h, –2h, 4h, –8h,…, which form
dx
a GP.
Since, X-axis is tangent at (–2, 0).
Let D ′ = ∆ P1 P2 P3 and D ′ ′ = ∆ P2 P3 P4
∴  dy  =0
 dx x = − 2 h h3 1
1
−2h −8h3 1
⇒ 0 = 3a (−2)2 + 2b (−2) + c 2
D ′ ∆ P1P2 P3 4h 64h3 1
⇒ 0 = 12a − 4b + c …(i) = =
D ′ ′ ∆ P2 P3 P4 −2 h −8 h 3
1
Again, slope of tangent at (0, 5) is 3. 1
dy 4h 64h3 1
∴ =3 2
dx ( 0, 5 ) −8h −512h3 1
252 Application of Derivatives

h h3 1 Since, tangent is parallel to x + 2 y = 0 ,


1 dy 1
−2h −8h3 1 then slope =−
2 dx 2
4h 64h3 1
= 1  1
From Eq. (i), − = − sin (x + y) ⋅ 1 − 
h h3 1 2  2
1
× (−2) × (−8) −2h −8h3 1 ⇒ sin (x + y) = 1, which shows cos (x + y) = 0.
2
4h 64h3 1 ∴ y=0
π 3π
D′ 1 ⇒ x+ y= or −
⇒ = = 1 : 16 which is the required ratio. 2 2
D′ ′ 16 π 3π
∴ x= or −
20. Given, y = (1 + x)y + sin −1 (sin 2 x) 2 2
π   3π 
Let y = u + v, where u = (1 + x)y , v = sin −1 (sin 2 x). Thus, required points are  , 0 and  − , 0
2   2 
On differentiating w.r.t.x, we get
dy du dv ∴ Equation of tangents are
= + …(i)
dx dx dx y−0 1 y−0 1
= − and =−
Now, u = (1 + x)y x − π /2 2 x + 3π / 2 2
On taking logarithm both sides, we get π 3π
⇒ 2y = − x + and 2 y = − x −
log e u = y log e (1 + x) 2 2
π 3π

1 du
=
y
+
dy
{log e (1 + x)} ⇒ x + 2 y = and x + 2 y = −
u dx 1 + x dx 2 2
are the required equations of tangents.
du  y dy 
⇒ = (1 + x)y  + log e (1 + x)
22. Slope of tangent at the point (x1 , y1 ) is 
…(ii) dy
dx  1 + x dx   .
 dx ( x
1 , y1 )
Again, v = sin −1 (sin 2 x) ⇒ sin v = sin 2 x
Given curve, ( y − x5 )2 = x (1 + x2)2
dv
⇒ cos v = 2 sin x cos x  dy 
dx ⇒ 2 ( y − x5 )  − 5x4 = (1 + x2)2 + 2x (1 + x2) ⋅ 2x
 dx 
dv 1
⇒ = (2 sin x cos x)
dx cos v Put x = 1 and y = 3, dy /dx = 8
dv 2 sin x cos x 2 sin x cos x
⇒ = = …(iii)
dx 1 − sin 2 v 1 − sin 4 x Topic 2 Increasing and Decreasing
Functions
From Eq. (i),
 y  2 sin x cos x
1. Let the thickness of layer of ice is x cm, the volume of
dy dy
= (1 + x)y  + log e (1 + x) + spherical ball (only ice layer) is
dx  1 + x dx  1 − sin 4 x
4
y −1 V = π [(10 + x)3 − 103 ] …(i)
dy y(1 + x) + 2 sin x cos x / 1 − sin 4 x 3
⇒ =
dx 1 − (1 + x)y log e (1 + x) On differentiating Eq. (i) w.r.t. ‘t’, we get
dV 4 dx
At x = 0, = π (3(10 + x)2) = − 50 [given]
dt 3 dt
y = (1 + 0)y + sin −1sin (0) = 1 [− ve sign indicate that volume is decreasing as
1 −1 time passes].
dy 1 (1 + 0) + 2 sin 0 ⋅ cos 0 / (1 − sin 4 0)
∴ = dx
dx 1 − (1 + 0)1 log e (1 + 0) ⇒ 4π (10 + x)2 = − 50
dt
dy At x = 5 cm
⇒ =1
dx dx
[4π (10 + 5)2] = − 50
Again, the slope of the normal is dt
dx 50 1 1
1 ⇒ =− =− =− cm /min
m=− = −1 dt 225(4π ) 9(2π ) 18π
dy / dx
Hence, the required equation of the normal is So, the thickness of the ice decreases at the rate of
1
y − 1 = (−1) (x − 0) i.e. y + x − 1 = 0 cm /min.
18π
21. Given, y = cos (x + y) 2. The given functions are
 dy  dy f (x) = ex − x,
⇒   = − sin (x + y) ⋅ 1 +  …(i)
 dx  dx
and g (x) = x2 − x, ∀ x ∈ R
Application of Derivatives 253

Then, h (x) = ( fog )(x) = f ( g (x)) Now, at h = 10 m, the rate at which height of water
Now, h′ (x) = f ′ ( g (x)) ⋅ g′ (x) dh 4 1
level is rising = = ×5= m/min
2
= (eg( x ) − 1) ⋅ (2x − 1) = (e( x − x ) − 1) (2x − 1) dt h = 10 π (10)2 5π
= (ex( x − 1) − 1) (2x − 1) 4. Given, φ (x) = f (x) + f (2 − x), ∀ x ∈ (0, 2)
Q It is given that h (x) is an increasing function, so ⇒ φ′ (x) = f ′ (x) − f ′ (2 − x) …(i)
h′ (x) ≥ 0 Also, we have f ′ ′ (x) > 0 ∀ x ∈ (0, 2)
⇒ (ex( x − 1) − 1)(2x − 1) ≥ 0 ⇒ f ′ (x) is a strictly increasing function
Case I (2x − 1) ≥ 0 and (ex( x − 1) − 1) ≥ 0 ∀ x ∈ (0, 2).
1 Now, for φ(x) to be increasing,
⇒ x ≥ and x(x − 1) ≥ 0
2 φ′ (x) ≥ 0
⇒ x ∈ [1 / 2, ∞ ) and x ∈ (− ∞ , 0] ∪ [1, ∞ ), so x ∈ [1, ∞ ) ⇒ f ′ (x) − f ′ (2 − x) ≥ 0 [using Eq. (i)]
⇒ f ′ (x) ≥ f ′ (2 − x) ⇒ x > 2 − x
Case II (2x − 1) ≤ 0 and [ex( x − 1) − 1] ≤ 0 [Q f′ is a strictly increasing function]
⇒ 2x > 2 ⇒ x > 1
1  1
⇒ x ≤ and x(x − 1) ≤ 0 ⇒ x ∈  −∞ , and x ∈ [0, 1] Thus, φ(x) is increasing on (1, 2).
2  2 
Similarly, for φ(x) to be decreasing,
 1 φ′ (x) ≤ 0
So, x ∈ 0,
 2  ⇒ f ′ (x) − f ′ (2 − x) ≤ 0
[using Eq. (i)]
 1
From, the above cases, x ∈ 0, ∪ [1, ∞ ). ⇒ f ′ (x) ≤ f ′ (2 − x)
 2 
⇒ x<2 − x [Q f′ is a strictly increasing
function]
Key Idea Use formula : ⇒ 2x < 2
3. ⇒ x < 1 Thus, φ(x) is decreasing on (0, 1).
1
Volume of cone = πr 2h, where r = radius and h = height of the
3 5. Given that function,
cone. f (x) = x3 − 3 (a − 2) x2 + 3ax + 7, for some a ∈ R is
Given, semi-vertical angle of right circular cone increasing in (0, 1] and decreasing in [1, 5).
 1 f′ (1) = 0 [Q tangent at x = 1 will be
= tan −1  
 2 parallel to X-axis]
 1  ⇒ (3x2 − 6(a − 2) x + 3a )x = 1 = 0
Let α = tan −1  
 2 ⇒ 3 − 6(a − 2) + 3a = 0
1 ⇒ 3 − 6a + 12 + 3a = 0
⇒ tan α = ⇒ 15 − 3a = 0 ⇒ a = 5
2
r 1 r So, f (x) = x3 − 9x2 + 15x + 7
⇒ = [from fig. tan α = ] ⇒ f (x) − 14 = x3 − 9x2 + 15x − 7
h 2 h
1 ⇒ f (x) − 14 = (x − 1) (x2 − 8x + 7) = (x − 1) (x − 1)(x − 7)
⇒ r= h …(i) f (x) − 14
2 ⇒ = (x − 7) …(i)
(x − 1)2
r
f (x) − 14
Now, = 0, (x ≠ 1)
(x − 1)2
⇒ x− 7 =0 [from Eq. (i)]
l h
⇒ x=7
α
6. We have,
x (d − x)
f (x) = −
1 2 (a 2 + x2)1/ 2 (b2 + (d − x)2)1/ 2
Q Volume of cone is (V ) = πr h
3 Differentiating above w.r.t. x, we get
2 1 2x
1 1  1 (a 2 + x2)1/ 2 − x
∴ V = π  h (h ) = πh3 [from Eq. (i)]
3 2  12 2 (a 2 + x2)1/ 2
f ′ (x) =
On differentiating both sides w.r.t. ‘t’, we get (a 2 + x2)
dV 1 dh dh 4 dV 2(d − x )(−1)
= π (3h 2) ⇒ = (b2 + (d − x ) 2 )1/ 2 (−1) − (d − x )
dt 12 dt dt πh 2 dt 2(b2 + (d − x ) 2 )1/ 2

dh 4 dV ( b2 + ( d − x ) 2 )
⇒ = ×5 [Q given = 5 m3 /min]
dt πh 2 dt [by using quotient rule of derivative]
254 Application of Derivatives

a 2 + x2 − x2 b2 + (d − x)2 − (d − x)2 11. PLAN Inequation based upon uncompatible function. This type
= +
(a 2 + x2)3/ 2 (b2 + (d − x)2)3/ 2 of inequation can be solved by calculus only.
a2 b2 Option (a) Let f (x) = ex − 1 − x.
= + 2 > 0, ∀ x ∈R
(a + x )
2 2 3/ 2
(b + (d − x)2)3/ 2 then f ′ (x) = ex − 1 > 0, ∀x ∈ (0, 1)
Hence, f (x) is an increasing function of x. ⇒ f (x) increase in (0, 1)
π ⇒ f (x) > f (0) for 0 < x < 1
7. Given, g (u ) = 2 tan −1 (eu ) − for u ∈ (−∞ , ∞ )
2 ⇒ ex − 1 − x > 0 or ex > 1 + x for 0 < x < 1
π
g (− u ) = 2 tan −1 (e− u ) − Option (b) Let g (x) = log e (1 + x) − x, 0 < x < 1
2
1 x
−1 u π g′ (x) = −1 = − < 0 for 0 < x < 1
= 2 (cot (e )) − 1+ x 1+ x
2
⇒ g (x) decreases for 0 < x < 1
π  π
= 2  − tan −1 (eu ) − ⇒ g (x) < g (0) for 0 < x < 1
2  2
⇒ log e (1 + x) − x < 0 for 0 < x < 1
= π / 2 − 2 tan −1 (eu ) = − g (u )
∴ g (− u ) = − g (u ) or log e (1 + x) < x for 0 < x < 1

⇒ g (u ) is an odd function. Therefore, option (b) is the answer.


We have, g (u ) = 2 tan −1 (eu ) − π 2 Option (c) sin x > x
2eu Let h (x) = sin x − x ⇒ h′ (x) = cos x − 1
g′ (u ) =
1 + e 2u For x ∈(0, 1), cos x − 1 < 0
g′ (u ) > 0, ∀ x ∈ R [Q eu > 0] ⇒ h (x) is decreasing function.
⇒ h (x) < h (0)
So, g′ (u ) is increasing for all x ∈ R.
⇒ sin x − x < 0
8. Given, f (x) = x3 + bx2 + cx + d ⇒ sin x < x,which is not true.
⇒ f ′ (x) = 3x2 + 2bx + c Option (d) p(x) = log x − x
As we know that, if ax2 + bx + c > 0, ∀x, then a > 0 and 1
D < 0. p′ (x) = − 1 > 0, ∀x ∈ (0, 1)
x
Now, D = 4b2 − 12c = 4(b2 − c) − 8c
Therefore, p′ (x) is an increasing function.
[where, b2 − c < 0 and c > 0] ⇒ p(0) < p(x) < p(1)
∴ D = (–ve) or D < 0 ⇒ − ∞ < log x − x < −1
⇒ f ′ (x) = 3x2 + 2bx + c > 0 ∀x ∈ (− ∞ , ∞ ) ⇒ log x − x < 0
[as D < 0 and a > 0]
Hence, f (x) is strictly increasing function. ⇒ log x < x
Therefore, option (d) is not the answer.
9. Let f (x) = 3 sin x − 4 sin3 x = sin 3x
The longest interval in which sin x is increasing is of
length π . 12. Let f (x) = ∫ ex (x − 1) (x − 2) dx
So, the length of largest interval in which f (x) = sin 3x is ⇒ f ′ (x) = ex (x − 1) (x − 2)
π
increasing is . + − +
3
1 2
10. Given, f (x) = xex (1 − x )
∴ f ′ (x) < 0 for 1 < x < 2
⇒ f ′ (x) = ex (1 − x ) + xex (1 − x ) (1 − 2x)
⇒ f (x) is decreasing for x ∈(1, 2).
= ex (1 − x ) [1 + x (1 − 2x)]
13. Given, f (x) = sin 4 x + cos 4 x
= ex (1 − x ) (1 + x − 2x2)
On differentiating w.r.t. x, we get
= − ex (1 − x ) (2x2 − x − 1)
f ′ (x) = 4 sin3 x cos x − 4 cos3 x sin x
= − ex (1 − x ) (x − 1) (2x + 1)
= 4 sin x cos x (sin 2 x − cos 2 x)
 1 
which is positive in  − , 1 . = 2 sin 2x (− cos 2x) = − sin 4x
 2 
 1  Now, f ′ (x) > 0 , if sin 4x < 0
Therefore, f (x) is increasing in − , 1 .
 2  ⇒ π < 4x < 2π
Application of Derivatives 255

π π ∴ log (π + x) > log (e + x) ... (ii)


⇒ <x< …(i)
4 2 1 1
and > ... (iii)
⇒ Option (a) is not proper subset of Eq. (i), so it is not e+ x π + x
correct.
On multiplying Eqs. (ii) and (iii), we get
π 3π
Now, <x< log (π + x) log (e + x)
4 8 > …(iv)
e+ x π+x
Since, option (b) is the proper subset of Eq. (i), so it is
correct. From Eqs. (i) and (iv), f ′ (x) < 0
x ∴ f (x) is decreasing for x ∈ (0, ∞ ).
14. Given, g (x) = , where 0 < x ≤ 1
tan x 16. Let F (x) = h (x) − h (1) = f { g (x)} − f { g (1)}
Now, g (x) is continuous in [0, 1] and differentiable in ∴ F ′ (x) = f ′ { g (x)} ⋅ g ′ (x)
]0, 1[.
= (+ ) (− ) = − ve
For 0 < x < 1,
[since, f (x) is an increasing function f ′ ( g (x)) is + ve and
tan x − x sec2 x
g′ (x) = g (x) is decreasing function g ′ ( f (x)) is −ve ]
tan 2 x
Since, f ′ (x) is –ve.
Again, H (x) = tan x − x sec2 x, 0 ≤ x ≤ 1
∴ f (x) is decreasing function.
Now, H (x) is continuous in [0, 1] and differentiable in When 0 ≤ x < 1 ⇒ h (x) − h (1) = + ve
]0, 1[.
When x≥1 ⇒ h (x) − h (1) = − ve
For 0 < x < 1, H (x) = tan x − x sec2x, 0 ≤ x ≤ 1
Hence, for x > 0,
⇒ H ′ (x) = sec2 x − sec2 x − 2x sec2 x tan x
h (x) − h (1) is neither always positive nor always
= − 2x sec2 x tan x < 0
negative, so it is not strictly increasing throughout.
Hence, H (x) is decreasing function in [0, 1].
Therefore, option (d) is the answer.
Thus, H (x) < H (0) for 0 < x < 1 f ( x) x
dy dy
⇒ H (x) < 0 for 0 < x<1 17. f ′ (x) > 2 f (x) ⇒
y
> 2dx ⇒ ∫ y
> 2∫ dx
⇒ g′ (x) < 0 for 0 < x<1 1 0
ln( f (x)) > 2x ⇒ f (x) > e2x
⇒ g (x) is decreasing function in (0, 1].
Also, as f ′ (x) > 2 f (x)
x
Therefore, g (x) = is a decreasing function in ⇒ f ′ (x) > 2c2x > 0
tan x
 1
− t + 
0 < x ≤ 1. x e  t

Also, g (x) < g (0) for 0 < x ≤ 1 18. Given, f (x) = ∫ 1 dt


x
t
x
⇒ < 1 for 0 < x ≤ 1  1
− x + 
1 
− + x 
tan x  x  
e  −1  e x
f ′ (x) = 1 ⋅ −  2
⇒ x < tan x for 0 < x≤1 x  x  1 /x
0 < x≤1  1  1  1
f (x) = 
x /sin x for − x +  − x +  − x + 
Now, let e  x
e x
2e  x
 1 for x=0 = + =
x x x
Now, f is continuous in [0, 1] and differentiable in ]0, 1[.
As f ′ (x) > 0, ∀x ∈ (0, ∞ )
For 0 < x < 1,
sin x − x cos x (tan x − x) cos x ∴ f (x) is monotonically increasing on (0, ∞ ).
f ′ (x) = = > 0 for 0 < x < 1
sin 2 x sin 2 x ⇒ Option (a) is correct and option (b) is wrong.
 1  1
⇒ f (x) increases in [0, 1]. − t +  − t + 
 1 x e  t 1/ x e  t
x Now, f (x) + f   = ∫ dt + ∫ dt
Thus, f (x) = increases in 0 < x ≤ 1.  x 1/x t x t
sin x
= 0, ∀x ∈ (0, ∞ )
Therefore, option (c) is the answer.  1
− t + 
log (π + x) 2x e  t
15. Given, f (x) = Now, let g (x) = f (2x ) = ∫ dt
log(e + x) 2− x t
1 1  1
− t + 
log (e + x) ⋅ − log (π + x) ⋅ 2− x  t
π+x e+ x −x e
f ′ (x) = …(i) g (− x) = f (2 ) = ∫ dt = − g (x)
2x t
[log (e + x)]2
∴ f (2x ) is an odd function.
When x > 0, π + x > e + x
256 Application of Derivatives

19. Given, h (x) = f (x) − f (x)2 + f (x)3 f ′ (x) > 0 for (0, 1)
On differentiating w.r.t. x, we get f ′ (x) < 0 for (e, ∞ )
h ′ (x) = f ′ (x) − 2 f (x) ⋅ f ′ (x) + 3 f 2(x) ⋅ f ′ (x) ∴ P and Q are correct, II is correct, III is incorrect.
= f ′ (x)[1 − 2 f (x) + 3 f 2(x)] −1 1
f ′′ (x) = 2 − ⇒ f ′′ (x) < 0 for (0, ∞ )
 2 1 x x
= 3 f ′ (x) ( f (x))2 − f (x) +
 3 3  ∴ S, is correct, R is incorrect.
 1 1 1
2 IV is incorrect.
= 3 f ′ (x)  f (x) −  + − 
 3  3 9 lim f (x) = − ∞ ⇒ lim f ′ (x) = − ∞ ⇒ lim f ′′ (x) = 0
 x→∞ x→∞ x→∞
 1 3 − 1
2
∴ ii, iii, iv are correct.
= 3 f ′ (x)  f (x) −  + 
 3  9 
 22. (d) 23. (c)
 1 2
2 24. (c)
= 3 f ′ (x)  f (x) −  +  π π
 3  9 d x
(x + sin x) = 1 + cos x = 2 cos 2 > 0 for − < x < .
 25.
dx 2 2 2
NOTE h ′ ( x ) < 0, if f ′ ( x ) < 0 and h ′ ( x ) > 0, if f ′ ( x ) > 0 Therefore, x + sin x is increasing in the given interval.
Therefore, h (x) is an increasing function, if f (x) is Therefore, (A)→ (p) is the answer.
increasing function and h (x) is decreasing function, if d
Again, (sec x) = sec x tan x which is > 0 for 0 < x < π / 2
f (x) is decreasing function. dx
Therefore, options (a) and (c) are correct answers. −π
and < 0 for < x<0
20. Let f (x) = log (1 + x) − x 2
1 x Therefore, sec x is neither increasing nor decreasing in
⇒ f ′ (x) = −1 = − the given interval. Therefore, (B)→(r) is the answer.
1+ x 1+ x
3x (x + 1)
⇒ f ′ (x) > 0 26. Let f (x) = sin x + 2x −
π
when − 1 < x < 0 and f ′ (x) < 0 On differentiating w.r.t. x, we get
when x>0 (6x + 3)
⇒ f ′ (x) = cos x + 2 −
∴ f (x) is increasing for −1 < x < 0. π
⇒ f (x) < f (0) ⇒ log (1 + x) < x 6  π
⇒ f ′ ′ (x) = − sin x − < 0, ∀x ∈ 0,
Again, f (x) is decreasing for x > 0. π  2 
⇒ f (x) < f (0) ⇒ log (1 + x) < x  π
∴ f ′ (x) is decreasing for all x ∈ 0, .
∴ log (1 + x) ≤ x, ∀x > − 1  2 

 2x2 − log x, x >0 ⇒ f ′ (x) > 0 [ Q x < π /2]


21. Here, y= 2
2 x − log (− x), x <0 ⇒ f ′ (x) > f ′ (π / 2)
 1 ∴ f (x) is increasing.
dy 4x − x , x > 0 Thus, when x ≥ 0, f (x) ≥ f (0)
⇒ =
dx 4x − 1 , x < 0 3x(x + 1) 3x (x + 1)
 x ⇒ sin x + 2x − ≥ 0 ⇒ sin x + 2x ≥
π π
4x2 − 1 (2x − 1) (2x + 1) f (x) = sin (tan x) − x
= , x ∈ R − {0} = 27. Let
x x
f ′ (x) = cos (tan x) ⋅ sec2x − 1
 1   1 
∴ Increasing when x ∈  − , 0 ∪  , ∞ = cos (tan x) (1 + tan 2 x) − 1
 2  2 
= tan 2 x {cos (tan x)} + cos (tan x) − 1
 1  1
and decreasing when x ∈  −∞ , −  ∪ 0,  . tan 2 x
 2  2 > tan 2 x cos (tan x) −
2
Solutions. (22-24)
 x2 
f (x) = x + ln x − x ln x ⇒ f (1) = 1 > 0  (1 − cos x) < x 2
, x ≠ 0 ⇒ cos x > 1 −
2
Q2
f (e2) = e2 + 2 − 2e2 = 2 − e2 < 0  tan 2 x 
 ⇒ cos (tan x) > 1 − 
⇒ f (x) = 0 for some x ∈ (1, e2)  2 
∴ I is correct  1 
f ′ (x) > tan 2 x cos (tan x) −
1 1  2 
f ′ (x) = 1 + − ln x − 1 = − ln x
x x > tan 2 x [cos (tan x) − cos (π / 3)] > 0
Application of Derivatives 257

⇒ f (x) is increasing function, for all x ∈ [0, π / 4]  eax − 1 


= lim aeax +
As f (0) = 0 ⇒ f (x) ≥ 0, for all x ∈ [0, π / 4] x → 0−  x 
⇒ sin (tan x) ≥ x eax − 1
= lim aeax + a ⋅ lim
28. Given, −1 ≤ p ≤ 1 x → 0− x → 0− ax
Let f (x) = 4x − 3x − p = 0
3 = ae0 + a (1) = 2a
1 3 f ′ (x) − f ′ (0)
Now, f (1 / 2) = − − p = − 1 − p ≤ 0 [Q p ≥ − 1] and Rf ′ ′ (0) = lim
2 2 x→ 0 + x+0
Also, f (1) = 4 − 3 − p = 1 − p ≥ 0 [Q p ≤ 1] (1 + 2ax − 3x2) − 1
= lim
+
∴ f (x) has atleast one real root between [1 /2, 1]. x→ 0 x
Also, f ′ (x) = 12x − 3 > 0 on [1 / 2, 1]
2
2ax − 3x2
= lim = lim 2a − 3x = 2a
+
⇒ f ′ (x) increasing on [1 /2, 1] x→ 0 x x → 0+

⇒ f has only one real root between [1 /2, 1]. Therefore, Lf ′ ′ (0) = Rf ′ ′ (0) = 2a
To find a root, we observe f (x) contains 4x3 − 3x, which is a (ax + 2) eax , if x < 0
multiple angle formula for cos 3θ. Hence, f ′ ′ (x) = 2a , if x = 0
2a − 6x, if x > 0
∴ Put x = cos θ
⇒ 4 cos θ − 3 cos θ − p = 0
3 Now, for x < 0, f ′ ′ (x) > 0, if ax + 2 > 0
⇒ p = cos 3 θ ⇒ θ = (1 / 3) cos ( p) −1 ⇒ For x < 0, f ′ ′ (x) > 0, if x > − 2 / a
1  2
∴ Root is cos  cos −1 ( p) . ⇒ f ′ (x) > 0, if − < x < 0
3  a
and for x > 0, f ′ ′ (x) > 0, if 2a − 6x > 0
29. NOTE This type is asked in 1983 and repeat after 13 years.
⇒ for x > 0, f ′ ′ (x) > 0, if x < a /3
At x = 0, LHL = lim f (x) = lim xeax = 0
x → 0− x → 0− Thus, f (x) increases on [–2/a, 0] and on [0, a/3].
and RHL = lim f (x) = lim (x + ax2 − x3 ) = 0  2 a
x→ 0 +
x→ 0 +
Hence, f (x) increases on − , .
Therefore, LHL = RHL = 0 = f (0)  a 3 
So, f (x) is continuous at x = 0. 30. Let y = f (x) = 2 sin x + 2 tan x − 3x
 ax
f ′ (x) = 1 ⋅ e + axe 2 , if x < 0
ax
Also, ⇒ f ′ (x) = 2 cos x + 2 sec2 x − 3
1 + 2ax − 3x , if x > 0
For 0 ≤ x < π / 2, f ′ (x) > 0
f (x) − f (0)
and Lf ′ (0) = lim Thus, f (x) is increasing.
x → 0− x−0
When x ≥ 0, f (x) ≥ f (0)
xeax − 0
= lim = lim eax = e0 = 1 ⇒ 2 sin x + 2 tan x − 3x ≥ 0 + 0 − 0

x→0 x x → 0−
⇒ 2 sin x + 2 tan x ≥ 3x
f (x) − f (0)
and Rf ′ (0) = lim 31. Let f (x) = 1 + x log (x + x2 + 1 ) − 1 + x2
x→ 0 + x+0
 x 
x + ax2 − x3 − 0 1 + 
= lim 
x →0 + x  x2 + 1 
∴ f ′ (x) = x ⋅ + log (x + x2 + 1 )
= lim 1 + ax − x = 1 2
x+ x +1
2
x → 0+
x x x
Therefore, Lf ′ (0) = Rf ′ (0) = 1 ⇒ f ′ (0) = 1 − = + log (x + x2 + 1 ) −
(ax + 1) eax , if x < 0 x +1
2
x +1
2
x +1
2

Hence, f ′ (x) =  1, if x = 0
⇒ f ′ (x) = log (x + x2 + 1 )
1 + 2ax − 3x , if x > 0
2

Now, we can say without solving that, f ′ (x) is ⇒ f ′ (x) ≥ 0 [Q log (x + x2 + 1 ) ≥ 0]


continuous at x = 0 and hence on R. We have, ∴ f (x) is increasing for x ≥ 0.
f ′ ′ (x) = ae + a (ax + 1) e , if x < 0
ax ax
⇒ f (x) ≥ f (0)
2 a − 6 x , if x > 0
⇒ 1 + x log (x + 1 + x2 ) − 1 + x2 ≥ 1 + 0 − 1
f ′ (x) − f ′ (0)
and Lf ′ ′ (0) = lim
x → 0− x−0 ⇒ 1 + x log (x + 1 + x2 ) ≥ 1 + x2 , ∀ x ≥ 0
(ax + 1)eax − 1 π π
= lim 32. Given, A = x : ≤x≤ 
x → 0− x  6 3
258 Application of Derivatives

and f (x) = cos x − x − x2 ∴ Perimeter of the rectangle


⇒ f ' (x) = − sin x − 1 − 2x = − (sin x + 1 + 2x) π 
y=4 − x + sin 2x
π π   4 
which is negative for x ∈ , ⇒ f ' (x) < 0
 6 3  For maximum,
dy
=0
dx
or f (x) is decreasing.
⇒ − 1 + 2 cos 2x = 0
  π  π 
Hence, f ( A ) =  f   , f    1
  3  6  ⇒ cos 2x =
2
1 π  π 3 π  π  π π   π 
=  − 1 +  , − 1 +   ⇒ 2x = ⇒ x= Q x ∈ 0, 
2 3  3 2 6  6  3 6   2 
d 2y
and = − 4 sin 2x| π < 0
Topic 3 Rolle’s and Lagrange’s Theorem dx2 x = π x=
6
6
1. f ′ (x) is increasing
π
1  ∴ At x = , the rectangle PQRS have maximum
For some x in  , 1 6
2  perimeter.
f ′ (x) = 1 [LMVT] So length of sides
∴ f′ (1) > 1  π π π
PS = QR = 2  −  =
2. Given, f (x) = 2 + cos x, ∀x ∈ R  4 6 6
 π
Statement I There exists a point ∈ [t , t + r ], where and PQ = RS = 2 sin   = 3
 3
f ′ (c) = 0
π π
Hence, Statement I is true. ∴ Required area = × 3=
6 2 3
Statement II f (t ) = f (t + 2π ) is true. But statement II
is not correct explanation for statement I. 2. Given functions are f (x) = 5 − |x − 2|
3. Since, f (x) and g (x) are differentiable functions for and g (x) = | x + 1|, where x ∈ R.
Clearly, maximum of f (x) occurred at x = 2, so α = 2.
0 ≤ x ≤ 1.
f (1) − f (0) and minimum of g (x) occurred at x = − 1, so β = − 1.
∴ f ′ (c) = ⇒ αβ = − 2
1 −0 (x − 1) (x2 − 5x + 6)
Now, lim
Using Lagrange’s Mean Value theorem, x → − αβ x2 − 6 x + 8
6 −2 (x − 1) (x − 3) (x − 2)
=4 = lim [Qαβ = − 2]
1 −0 x→ 2 (x − 4) (x −2)
g (1) − g (0) 2 − 0 (x − 1) (x − 3) (2 − 1) (2 − 3) 1 × (− 1) 1
and g ′ (c) = = =2 = lim = = =
1 −0 1 −0 x→ 2 (x − 4) (2 − 4) (− 2) 2
⇒ f ′ (c) = 2 g′ (c)
3. Key Idea Volume of parallelopiped formed by the vectors a,
b and c is V = [ a b c].
Topic 4 Maxima and Minima
1. Given region is Given vectors are $i + λ$j + k
$ , $j + λk
$ and λ$i + k
$ , which
π forms a parallelopiped.
{(x, y) ∈ R × R : 0 ≤ x ≤ and 0 ≤ y ≤ 2 sin(2x)
2 ∴Volume of the parallelopiped is
On drawing the diagram, 1 λ 1
Let the side PS on the X-axis, such that P (x, 0), and
V = 0 1 λ = 1 + λ3 − λ
π 
Q (x, 2 sin(2x)), so length of the sides PS = QR = 2  − x λ 0 1
4 
and PQ = RS = 2 sin 2x. ⇒ V = λ3 − λ + 1
Y On differentiating w.r.t. λ, we get
dV
2 = 3 λ2 − 1

Q R dV
For maxima or minima, =0

1
X ⇒ λ=±
O P π/4 S π/2 3
Application of Derivatives 259

 1
2 3 > 0 , for λ =
d 2V  3 Key Idea
and = 6λ =  6.
dλ2 1 (i) Use formula of volume of cylinder, V = πr 2h
2 3 < 0 , for λ = −
 3 where, r = radius and h = height
(ii) For maximum or minimum, put first
d 2V 1
Q is positive for λ = , so volume ‘V ’ is minimum derivative of V equal to zero
dλ2 3
1 Let a sphere of radius 3, which inscribed a right circular
for λ = cylinder having radius r and height is h, so
3
4. Given function f (x) = x kx − x2 … (i) h
From the figure, = 3 cos θ
the function f (x) is defined if kx − x ≥ 0 2
2
⇒ x2 − kx ≤ 0 ⇒ h = 6 cos θ
⇒ x ∈ [0, k] … (ii) and r = 3 sin θ …(i)
r
because it is given that f (x) is increasing in interval
x ∈ [0, 3], so k should be positive.
Now, on differentiating the function f (x) w.r.t. x, we get
x
f ′ (x) = kx − x2 + × (k − 2x)
2 kx − x2 h
θ 3
2(kx − x ) + kx − 2x
2 2
3kx − 4x 2
h/2
= =
2 kx − x 2
2 kx − x 2
r
as f (x) is increasing in interval x ∈ [0, 3], so
f ′ (x) ≥ 0 ∀ x ∈ (0, 3) Q Volume of cylinder V = πr 2h
= π (3 sin θ )2(6 cos θ ) = 54π sin 2 θ cos θ .
⇒ 3kx − 4x2 ≥ 0
dV
⇒ 4x2 − 3kx ≤ 0 For maxima or minima, =0

 3k  3k  ⇒ 54π [2 sin θ cos θ − sin θ ] = 0
2 3
⇒ 4x x −  ≤ 0 ⇒ x ∈ 0, (as k is positive)
 4  4  ⇒ sin θ [2 cos 2 θ − sin 2 θ ] = 0
3k   π 
So, 3≤
⇒k ≥4 ⇒ tan 2 θ = 2 Q θ ∈ 0, 2  
4  
⇒ Minimum value of k = m = 4 2
⇒ tan θ = 2 ⇒ sin θ =
and the maximum value of f in [0, 3] is f (3). 3
1
Q f is increasing function in interval x ∈ [0, 3] and cos θ = …(ii)
3
Q M = f (3) = 3 4 × 3 − 3 = 3 3
2
From Eqs. (i) and (ii), we get
Therefore, ordered pair (m, M ) = (4, 3 3 ) 1
h =6 =2 3
3
5. The non-zero four degree polynomial f (x) has
extremum points at x = −1, 0, 1, so we can assume 7. Given function is
f ′ (x) = a (x + 1)(x − 0) (x − 1) = ax(x2 − 1) f (x) = 9x4 + 12x3 − 36x2 + 25 = y (let)
dy
where, a is non-zero constant. For maxima or minima put =0
f ′ (x) = ax3 − ax dx
dy

a a
f (x) = x4 − x2 + C ⇒ = 36x3 + 36x2 − 72x = 0
4 2 dx
⇒ x3 + x2 − 2x = 0
[integrating both sides]
⇒ x[x2 + x − 2] = 0
where, C is constant of integration.
⇒ x[x + 2x − x − 2] = 0
2
Now, since f (x) = f (0)
a 4 a 2 x 4 x2 ⇒ x[x(x + 2) − 1(x + 2)] = 0
⇒ x − x +C =C ⇒ = ⇒ x(x − 1)(x + 2) = 0
4 2 4 2
⇒ x = − 2, 0, 1
⇒ x2(x2 − 2) = 0 ⇒ x = − 2 , 0, 2
By sign method, we have following
Thus, f (x) = f (0) has one rational and two irrational
– + – +
roots.
–2 0 1
260 Application of Derivatives

dy Y
Since, changes it’s sign from negative to positive at
dx (0, 12)
x = ‘−2 ’ and ‘1’, so x = − 2, 1 are points of local minima.
dy R M Q(a, 12–a2)
Also, changes it’s sign from positive to negative at
dx
x = 0, so x = 0 is point of local maxima.
XN X
∴ S1 = { −2, 1} and S 2 = {0}. S O P(a, 0)
y=12–x2
8. Given equation of curve is YN

y2 = x − 2 …(i) Then, area of rectangle PQRS


and the equation of line is = 2 × (Area of rectangle PQMO)
y=x …(ii) [due to symmetry aboutY -axis]
= 2 × [a (12 − a 2)] = 24a − 2a3 = ∆ (let).
Y y=x The area function ∆ a will be maximum, when
y2=x–2
d∆
M = 0 ⇒ 24 − 6a 2 = 0
P(t2+2, t) da
⇒ a2 = 4 ⇒ a = 2 [Q a > 0]
X So, maximum area of rectangle
O (2, 0) PQRS = (24 × 2) − 2 (2)3
= 48 − 16 = 32 sq units
10. We have,
f (x) = 3x3 − 18x2 + 27x − 40
⇒ f ′ (x) = 9x2 − 36x + 27
Consider a point P (t 2 + 2, t ) on parabola (i).
= 9(x2 − 4x + 3) = 9 (x − 1) (x − 3) …(i)
For the shortest distance between curve (i) and line
(ii), the line PM should be perpendicular to line (ii) and Also, we have S = { x ∈ R : x2 + 30 ≤ 11 x}
parabola (i), i.e. tangent at P should be parallel to Clearly, x2 + 30 ≤ 11x
y = x. ⇒ x2 − 11x + 30 ≤ 0
dy ⇒ (x − 5) (x − 6) ≤ 0 ⇒ x ∈ [5, 6]
∴ = Slope of tangent at point P to curve (i) So, S = [5, 6]
dx at point P
Note that f (x) is increasing in [5, 6]
=1 [Q tangent is parallel
[Qf ′ (x) > 0 for x ∈[5, 6]
to line y = x]
∴f (6) is maximum, where
1
⇒ =1 f (6) = 3(6)3 − 18(6)2 + 27(6) − 40 = 122
2yP
dy 11. According to given information, we have the following
[differentiating the curve (i), we get2 y =1] figure.
dx
1 1 C(t2, 2t)
⇒ =1⇒t = [Q P (x, y) = P (t 2 + 2, t )] B(9, 6)
2t 2
 9 1
So, the point P is  ,  .
 4 2
9 − 1
4 2
Now, minimum distance = PM = A(4, –4)
2
[Q distance of a point P (x1 , y1 ) from a line
For y2 = 4ax, parametric coordinates of a point is (at 2,
|ax1 + by1 + c | 7 2at).
ax + by + c = 0 is = units
a + b
2 2
 4 2 ∴For y2 = 4x, let coordinates of C be (t 2, 2t).
t2 2t 1
9. Equation of parabola is given, y = 12 − x2 1
Then, area of ∆ABC = 9 6 1
or x2 = − ( y − 12). 2
4 −4 1
Note that vertex of parabola is (0, 12) and its open 1
downward. = |t 2(6 − (− 4)) − 2t (9 − 4) + 1(− 36 − 24)|
2
Let Q be one of the vertices of rectangle which lies on 1 10 2
parabola. Then, the coordinates of Q be (a, 12− a 2) = |10t 2 − 10t − 60| = |t − t − 6|= 5| t 2 − t − 6|
2 2
Application of Derivatives 261

Let, A (t ) = 5| t 2 − t − 6| ...(i) 1  1 2
x− < 0,  x −  + ∈ ( −∞ , 2 2 ]
Clearly, A (4, − 4) ≡ A (t12, 2t1 ) ⇒ 2t1 = − 4 x  x x − 1
⇒ t1 = − 2 x
and B(9, 6) ≡ B(t22,2t2) ⇒ 2t2 = 6 ⇒ t2 = 3 ∴ Local minimum value is 2 2.
Since, C is on the arc AOB, the parameter ‘t’ for point 14. Total length = 2r + r θ = 20
C ∈ (− 2, 3).
20 − 2r θ
Let f (t ) = t 2 − t − 6 ⇒ f ′ (t ) = 2t − 1 ⇒ θ=
1 r
Now, f ′ (t ) = 0 ⇒ t = r r
2 Now, area of flower-bed,
1 1
Thus, for A (t ), critical point is at t = A = r 2θ
2 2
2 rθ
 1  1 1 125 1 1 2 20 − 2r 
Now,A   = 5   − − 6 = = 31 [Using Eq. (i)] ⇒ A= r  
 2  2 2 4 4 2  r 

12. Let h = height of the cone, r = radius of circular base ⇒ A = 10r − r 2


dA
= (3)2 − h 2 [Q l2 = h 2 + r 2] ∴ = 10 − 2r
dr
= 9−h 2
…(i) dA
For maxima or minima, put = 0.
dr
⇒ 10 − 2r = 0 ⇒ r = 5
1 20 − 2 (5) 
3

h ∴ Amax = (5)2
l=

2  5 
1
r = × 25 × 2 = 25 sq. m
2

1 2
15. According to given information, we have Perimeter of
Now, volume (V ) of cone = πr h square + Perimeter of circle = 2 units
3
1 ⇒ 4 x + 2 πr = 2
⇒ V (h ) = π (9 − h 2)h [From Eq. (i)] 1 − 2x
3 ⇒ r= ...(i)
1 π
= π [9h − h3 ] …(ii) Now, let A be the sum of the areas of the square and the
3 circle. Then,
For maximum volume V ′ (h ) = 0 and V ′′ (h ) < 0. (1 − 2x) 2
Here, V ′ (h ) = 0 ⇒ (9 − 3h 2) = 0 A = x 2 + π r 2 = x2 + π
π2
⇒ h= 3 [Q h </ 0] (1 − 2x) 2
1 ⇒ A (x) = x2 +
and V ′′ (h ) = π (−6h ) < 0 for h = 3 π
3 dA
Thus, volume is maximum when h = 3 Now, for minimum value of A (x), =0
dx
Now, maximum volume 2 (1 − 2x) 2 − 4x
1 ⇒ 2x + ⋅ (− 2) = 0 ⇒ x =
V ( 3 ) = π (9 3 − 3 3 ) [from Eq. (ii)] π π
3 2
= 2 3π ⇒ πx + 4x = 2 ⇒ x = ...(ii)
π+4
13. We have, Now, from Eq. (i), we get
1 1 f(x)
f(x) = x2 + and g( x ) = x − ⇒ h( x ) = 2
x2 x g( x ) 1 −2⋅
π + 4 π + 4 −4 1
2 r= = = ...(iii)
1 1 π π(π + 4) π + 4
x2 +  +2
x −
2 x
∴ h( x ) = x = From Eqs. (ii) and (iii), we get x = 2r
1 1
x− x− 16. Here, to find the least value of α ∈ R, for which
x x
1
 1 2 4 αx2 + ≥ 1, for all x > 0.
⇒ h( x ) =  x −  + x
 x x − 1
i.e. to find the minimum value of α when
x 1
y = 4αx2 + ; x > 0 attains minimum value of α.
1  1 2 x
x − > 0,  x −  + ∈ [2 2 , ∞ )
x  x x − 1 dy 1
∴ = 8αx − 2 …(i)
x dx x
262 Application of Derivatives

d 2y 2 19. PLAN The given equation contains algebraic and trigonometric


Now, = 8α + 3 …(ii)
dx 2
x functions called transcendental equation. To solve
dy transcendental equations we should always plot the graph for
When = 0, then 8x3α = 1 LHS and RHS.
dx
1/3 Here, x2 = x sin x + cos x
 1 d 2y
At x =   , = 8α + 16α = 24α, Thus, y attains Y x2
 8α  dx2
1/3
 1
minimum when x =   ; α > 0.
 8α  X′ X
O
1/3
 1
∴ y attains minimum when x =   . Y′
 8α 
23
/ Let f (x) = x2 and g (x) = x sin x + cos x
 1
i.e. 4α   + (8α )1/3 ≥ 1 We know that, the graph for f (x) = x2
 8α 
To plot, g (x) = x sin x + cos x
⇒ α 1/3 + 2α 1/3 ≥ 1
1 g′ (x) = x cos x + sin x − sin x
⇒ 3α 1/3 ≥ 1 ⇒ α ≥
27 g′ (x) = x cos x …(i)
1 g′′ (x) = − x sin x + cos x …(ii)
Hence, the least value of α is .
27 Put g′ (x) = 0 ⇒ x cos x = 0
17. PLAN Any function have extreme values (maximum or minimum) at π 3π 5π 7π
its critical points, where f ′ ( x ) = 2. ∴ x = 0, , , ,
2 2 2 2
Since, the function have extreme values at x = 1 and
x = 2. g (x)
∴ f ′ (x) = 0 at x = 1 and x = 2 /2
⇒ f′ (1) = 0 and f′ (2) = 0
Also, it is given that,
lim  f (x)  lim f (x) –5 π/ 2
x→ 0 1 + =3 ⇒ 1 + =3 –π –π/2 O π/2 π 5π/2
x2 
x→ 0 2
 x
f (x)
⇒ lim
x→ 0 =2
x2 Y′
⇒ f (x) will be of the form ax4 + bx3 + 2x2. 3π 7π
At x = 0, , ,... , f ′′ (x) > 0, so minimum
[Q f (x) is of four degree polynomial] 2 2
f (x) = ax4 + bx3 + 2x2 π 5π 9π
Let At x = , , ,... , f ′ (x) < 0, so maximum
⇒ f ′ (x) = 4ax3 + 3bx2 + 4x 2 2 2
⇒ f ′ (1) = 4a + 3b + 4 = 0 ...(i) So, graph of f (x) and g (x) are shown as
and f ′ (2) = 32a + 12b + 8 = 0
Y
⇒ 8a + 3b + 2 = 0 ...(ii)
f (x)
On solving Eqs. (i) and (ii), g (x)
1
we get a = , b = −2
2
X′ X
x4 –π/2 O π/2
∴ f (x) = − 2x3 + 2x2
2
⇒ f (2) = 8 − 16 + 8 = 0
Y′
18. Here, x = − 1 and x = 2 are extreme points of
f (x) = α log|x| + βx2 + x, then So, number of solutions are 2.
α 2 2 2 2
f ′ (x) = + 2 βx + 1 20. Given function, f (x) = ex + e− x , g (x) = xex + e− x and
x 2 2
f ′ (−1) = − α − 2 β + 1 = 0 …(i) h (x) = x2ex + e− x are strictly increasing on [0, 1].
[at extreme point, f ′ (x) = 0] Hence, at x = 1, the given function attains absolute
α maximum all equal to e + 1 / e .
f ′ (2) = + 4 β + 1 = 0 …(ii)
2 ⇒ a=b=c
On solving Eqs. (i) and (ii), we get (2 + x)3 , if − 3 < x ≤ −1
1 21. Given, f (x) = 
α = 2, β = −  x 2 / 3 , if − 1 < x < 2
2
Application of Derivatives 263

3 (x + 2)2 , if − 3 < x ≤ −1 26. Let f (x) = x25 (1 − x)75 , x ∈[0, 1]



⇒ f ′ (x) = 2 − 1
3 x
3 , if − 1 < x < 2 ⇒ f ′ (x) = 25 x24 (1 − x)75 − 75x25 (1 − x)74
= 25x24 (1 − x)74 [(1 − x) − 3x]
Y
= 25x24 (1 − x)74 (1 − 4x)
For maximum value of f (x), put f ′ (x) = 0
⇒ 25x24 (1 − x)74 (1 − 4x) = 0
X′ X
(–3,0) (–1,0) 1
⇒ x = 0, 1,
4
Y′ Also, at x = 0, y = 0
Clearly, f ′ (x) changes its sign at x = −1 from +ve to −ve At x = 1, y = 0
and so f (x) has local maxima at x = −1 .
and at x = 1 / 4, y > 0
Also, f ′ (0) does not exist but f ′ (0 − ) < 0 and f ′ (0 + ) < 0.
It can only be inferred that f (x) has a possibility of a ∴ f (x) attains maximum at x = 1 / 4.
minima at x = 0 . Hence, the given function has one local 27. Let the coordinates of P be (a cos θ , b sin θ )
maxima at x = −1 and one local minima at x = 0 .
Equations of tangents at P is
22. Given f (x) = x2 + 2bx + 2c2 and g (x) = − x2 − 2cx + b2
Y
Then, f (x) is minimum and g (x) is maximum at
 −b −D  N
x = and f (x) =  , respectively. P
 4a 4a 
− (4b2 − 8c2) X' X
∴ min f (x) = = (2c2 − b2) O M
4
(4c2 + 4b2)
and max g (x) = − = (b2 + c2) K
4(−1)
Y'
Now, min f (x) > max g (x)
x y
⇒ 2c2 − b2 > b2 + c2 cos θ + sin θ = 1
a b
⇒ c2 > 2b2
Again, equation of normal at point P is
⇒ | c| > 2| b|
ax secθ − by cosec θ = a 2 − b2
23. It is clear from figure that at x = 0, f (x) is not continuous.
Let M be foot of perpendicular from O to PK, the normal
Y at P.
1
y = –x
y=x Area of ∆OPN = (Area of rectangle OMPN )
2
1
X′ X = ON ⋅ OM
O 2
1 ab
Now, ON = =
Y′ cos 2 θ sin 2 θ b2 cos 2 θ + a 2 sin 2 θ
+
a2 b2
Here, f (0) > RHL at x = 0 and f (0) > LHL at x = 0
So, local maximum at x = 0. [perpendicular from O, to line NP]
a − b2
2
(a 2 – b2) ⋅ cos θ ⋅ sin θ
x2 − 1 2 and OM = =
24. Given, f (x) = 2 =1 − 2
a 2 sec2 θ + b2cosec2θ a 2 sin 2 θ + b2 cos 2 θ
x +1 x +1
2 ab(a 2 − b2) cos θ ⋅ sin θ
f (x) will be minimum, when is maximum, Thus, area of ∆OPN =
x +1
2 2(a 2 sin 2 θ + b2 cos 2 θ )
i.e. when x2 + 1 is minimum, ab(a 2 − b2) tan θ
=
i.e. at x = 0. 2(a 2 tan 2 θ + b2)
∴ Minimum value of f (x) is f (0) = − 1 tan θ
Let f (θ ) = [0 < θ < π / 2]
α 2 tan 2 θ + b2
25. The maximum value of f (x) = cos x + cos ( 2x) is 2
which occurs at x = 0. Also, there is no other value of x sec2 θ( a 2 tan 2 θ + b2) − tanθ (2a 2 tanθ sec2θ)
f ' (θ ) =
for which this value will be attained again. (a 2 tan 2 θ + b2)2
264 Application of Derivatives

sec2 θ (a 2 tan 2 θ + b2 – 2a 2 tan 2 θ ) 1


= ( p + q − p + q), if p ≥ q
( p + q −| p − q|) = 12
1
(a 2 tan 2 θ + b2)2 ∴
2  ( p + q + p − q), if p < q
sec2 θ( a tan θ + b)(b – a tan θ ) 2
=
(a 2 tan 2 θ + b2)2  q, if p ≥ q
=
For maximum or minimum, we put  p, if p < q
f′ (θ ) = 0 ⇒ b – a tan θ = 0 1
⇒ { p + q − | p − q|} = min ( p, q)
[sec2 θ ≠ 0, a tan θ + b ≠ 0, 0 < θ < π / 2 ] 2
⇒ tan θ = b / a 31. Given, f : R → R and f (x) = (x − 1) (x − 2 ) (x − 5 )
 −1
Also, f ′ (θ )> 0, if 0 < θ−1< tan (b / a )
x
Since, F (x) = ∫ f (t ) dt , x > 0
< 0, if tan (b / a ) < θ < π / 2 0

So, F ′ (x) = f (x) = (x − 1)(x − 2 )(x − 5 )


Therefore, f (θ ) has maximum, when
According to wavy curve method
 b b
θ = tan −1   ⇒ tan θ = - + - +
 a a
1 2 5
b a
Again, sin θ = , cos θ = F ′ (x) changes, it’s sign from negative to positive at x = 1
a 2 + b2 a 2 + b2
and 5, so, F (x) has minima at x = 1 and 5 and as F ′ (x)
By using symmetry, we get the required points changes, it’s sign from positive to negative at x = 2 , so
 ±a2 ± b2  F (x) has maxima at x = 2 .
 ,  2 2
 a 2 + b2 a 2 + b2  Q F (2 ) = ∫ f (t ) dt = ∫ (t3 − 8t 2 + 17t − 10)dt
0 0
28. Given, P (x) = a 0 + a1x2 + a 2 x4 + K+ a nx2n 2
t 4
t 3
t2 
where, a n > a n − 1 > a n − 2 > K > a 2 > a1 > a 0 > 0 =  −8 + 17 − 10t 
 4 3 2 0
⇒ P ′ (x) = 2a1x + 4a 2 x3 + ... + 2na nx2n − 1
64 124 10
= 2x (a1 + 2a 2 x2 + K + na nx2n − 2) …(i) =4 − + 34 − 20 = 38 − =−
3 3 3
where, (a1 + 2a 2 x2 + 3a3 x4 + K + na nx2n − 2) > 0, ∀ x ∈ R.
Q At the point of maxima x = 2 , the functional value
P ′ (x) > 0, when x > 0 10
Thus,  F (2 ) = − , is negative for the interval x ∈ (0, 5 ), so
P ′ (x) < 0, when x < 0 3
F (x) ≠ 0 for any value of x ∈ (0, 5 ),
i.e. P ′ (x) changes sign from (–ve) to (+ve) at x = 0.
Hence, options (a), (b) and (d) are correct.
∴ P (x) attains minimum at x = 0.
32. Given function f : R → R is
Hence, it has only one minimum at x = 0 .
x5 + 5x4 + 10x3 + 10x2 + 3x + 1 , x<0
29. y = a log x + bx2 + x has extremum at x = − 1 and x = 2. 
 x 2
− x + 1 , 0 ≤ x<1
dy 2 3 8
∴ = 0, at x = − 1 f (x) =  x − 4 x2 + 7 x − , 1 ≤ x<3
dx  3 3
a  (x − 2 ) log (x − 2 ) − x + 10 , x≥3
and x=2 ⇒ + 2bx + 1 = 0, at x = − 1  e
x 3
and x=2 5x4 + 20x3 + 30x2 + 20x + 3 , x<0
 , 0 < x<1
a 2 x − 1
∴ − a − 2b + 1 = 0 and + 4b + 1 = 0 So, f ′ (x) = 
2  2 x2 − 8 x + 7 , 1 < x<3
1  x>3
⇒ a = 2 and b = −  log e (x − 2 ) ,
2 − +
At x = 1, f′′ (1 ) = 2 > 0 and f′′ (1 ) = 4 − 8 = −4 < 0
 p, if p > q ∴f ′ (x) is not differentiable at x = 1 and
30. Since, max ( p, q) = 
 q, if q > p f ′ (x) has a local maximum at x = 1.
 p, if p is greatest. For x ∈ (−∞ , 0)

and max ( p, q, r ) =  q, if q is greatest. f ′ (x) = 5x4 + 20x3 + 30x2 + 20x + 3
 r , if r is greatest. and since f′ (−1) = 5 − 20 + 30 − 20 + 3 = −2 < 0

So, f (x) is not increasing on x ∈ (−∞ , 0).
∴ max ( p, q) < max ( p, q, r ) is false.
Now, as the range of function f (x) is R, so f is onto
 p − q , if p ≥ q
We know that, | p − q| =  function.
q − p , if p < q Hence, options (b), (c) and (d) are correct.
Application of Derivatives 265

 cos 2x cos 2x sin 2x  Here, f (x) = 2|x|+ |x + 2| − ||x + 2| − 2|x||


33. f (x) = 
 − cos x cos x − sin x  − 2x − (x + 2) + (x − 2), if when x ≤ − 2
 sin x sin x cos x  − 2 x + x + 2 + 3 x + 2 , if when − 2 < x ≤ −2 /3

cos 2x(cos 2 x + sin 2 x) − cos 2x  2
= − 4x, if when − < x ≤ 0
(− cos 2 x + sin 2 x) + sin 2x(− sin 2x)  3
 4x, if when 0 < x ≤ 2
= cos 2x + cos 4x
 2x + 4, if when x > 2
f ′ (x) = − 2 sin 2x − 4 sin 4x = −2 sin 2x(1 + 4 cos 2x)
At x = 0 ⇒ f ′ (x) = 0 and f (x) = 2 − 2x − 4, if x ≤ −2
Also, f ′ (x) = 0 ⇒ sin 2x = 0  2x + 4, if −2 < x ≤ − 2 /3

−1 nπ 1  2
or cos 2x = ⇒ x= or cos 2x = − =  − 4x, if − < x≤0
4 2 4  3
 4x, if 0 < x≤2
f (x) ⋅ g (x)
34. Here, lim =1  2x + 4, if x>2
x → 2 f ′ (x) ⋅ g′ (x)

f (x) g′ (x) + f ′ (x) g (x) Graph for y = f (x) is shown as


⇒ lim =1
x → 2 f ′ ′ (x) g′ (x) + f ′ (x) g′′ (x)

[using L’ Hospital’s rule]


f (2) g′ (2) + f ′ (2) g (2)
⇒ =1
f ′′ (2) g′ (2) + f ′ (2) g′′ (2)
f (2) g′ (2)
⇒ =1 [Q f ′ (2) = g (2) = 0]
f ′′ (2) g′ (2)
⇒ f (2) = f ′′ (2) … (i)
∴ f (x) − f ′′ (x) = 0, for atleast one x ∈ R.
⇒ Option (d) is correct. 36. PLAN
Also, f : R → (0, ∞ ) ⇒ f (2) > 0 The problem is based on the concept to maximise volume of
cuboid, i.e. to form a function of volume, say f( x )find f ′( x )and f ′ ′( x ).
∴ f ′′ (2) = f (2) > 0 [from Eq. (i)]
Put f ′ ( x ) = 0 and check f ′ ′( x ) to be + ve or − ve for minimum and
Since, f′ (2) = 0 and f ′′ (2) > 0
maximum, respectively.
∴ f (x) attains local minimum at x = 2.
Here, l = 15x − 2a , b = 8x − 2a and h = a
⇒ Option (a) is correct. a
35. PLAN
a
x , if x ≥
We know that,| x| = 
0
 x , if x <
− 0
8x–2a 8x
 x − a, if x ≥ a 15x – 2a
⇒ | x − a| = 
 − ( x − a ), if x < a
and for non-differentiable continuous function, the maximum or
minimum can be checked with graph as 15x
Y Y ∴ Volume = (8x − 2a ) (15x − 2a ) a
V = 2a ⋅ (4x − a ) (15x − 2a ) …(i)

a
X X
O x=a x=a 8x – 2 a
Minimum at x = a Maximum at x = a
15x – 2a
Y
On differntiating Eq. (i) w.r.t a, we get
dv
= 6a 2 − 46ax + 60x2
da
Again, differentiating,
d 2v
= 12a − 46x
O x=a
X da 2
 dv 
  = 0 ⇒ 6x − 23x + 15 = 0
Neither maximum 2
Here,
nor minimum at x = a  da 
266 Application of Derivatives

At a = 5 ⇒ x = 3,
5 For maxima or minima, put f ′ (x) = 0 ⇒ x = 1, − 1
6 1
Now f ′ ′ (x) = (114x)
 d 2v  4
⇒  2 = 2 (30 − 23x)
 da  At x = 1 , f ′ ′ (x) > 0, minima
At x = − 1, f ′ ′ (x) < 0 , maxima
 d 2v 
At x = 3,  2 = 2(30 − 69) < 0 ∴ f (x) is increasing for [1, 2 5 ] .
 da  ∴ f (x) has local maxima at x = − 1 and f (x) has local
5  d 2v  minima at x = 1.
∴ Maximum when x = 3, also at x = ⇒  2 > 0
6  da  Also, f (0) = 34 /4
Hence, (b) and (c) are the correct answers.
∴ At x = 5 /6, volume is minimum. x
Thus, sides are 8x = 24 and 15x = 45 39. f (x) = ∫ t (et − 1) (t − 1)(t − 2)3 (t − 3)5 dt
−1

37. Given, +
– – + –
 ex , if 0 ≤ x ≤ 1 –∞ 0 1 2 3 ∞

f (x) = 2 − ex − 1 , if 1 < x ≤ 2
d x
 x−e ,
 if 2 < x ≤ 3 f ′ (x) =
dx ∫ −1 t (et − 1)(t − 1)(t − 2)3 (t − 3)5 dt

and g (x) = ∫
x
f (t ) dt = x(ex − 1)(x − 1)(x − 2)3 (x − 3)5 × 1
0
 d ψ( x ) 
⇒ g′ (x) = f (x)
Q
 dx ∫ φ ( x) f (t ) dt = f { ψ (x)}ψ′ (x) − f { φ (x)} φ′ (x)

Put g′ (x) = 0 ⇒ x = 1 + log e 2 and x = e. For local minimum, f ′ (x) = 0
 ex , if 0 ≤ x ≤ 1 ⇒ x = 0, 1, 2, 3

Also, g′′ (x) =  − ex − 1 , if 1 < x ≤ 2
Let f ′ (x) = g (x) = x(ex − 1)(x − 1)(x − 2)3 (x − 3)5
 1 , if 2 < x ≤ 3
Using sign rule,
At x = 1 + log e 2, − + − +
g′′ (1 + log e 2) = − elog e 2 < 0, g (x) has a local maximum.
1 2 3
Also, at x = e,
This shows that f (x) has a local minimum at x = 1 and
g′′ (e) = 1 > 0, g (x) has a local minima.
x = 3 and maximum at x = 2 .
Q f (x) is discontinuous at x = 1, then we get local Therefore, (b) and (d) are the correct answers.
maxima at x = 1 and local minima at x = 2.
Hence, (a) and (b) are correct answers. 40. For −1 ≤ x ≤ 2 , we have
f (x) = 3x2 + 12x − 1
38. Since, f (x) has local maxima at x = − 1 and f ′ (x) has
local minima at x = 0. ⇒ f ′ (x) = 6x + 12 > 0, ∀ − 1 ≤ x ≤ 2
∴ f ′ ′ (x) = λx Hence, f (x) is increasing in [–1, 2].
On integrating, we get Again, function is an algebraic polynomial, therefore it
2 is continuous at x ∈ (−1, 2) and (2, 3).
x
f ′ (x) = λ +c [Q f ′ (− 1) = 0] For continuity at x = 2,
2
lim f (x) = lim (3x2 + 12x − 1)
λ x → 2− x → 2−
⇒ + c = 0 ⇒ λ = − 2c …(i)
2 = lim [3 (2 − h )2 + 12(2 − h ) − 1]
h→ 0
Again, integrating on both sides, we get = lim [3 (4 + h 2 − 4h ) + 24 − 12h − 1]
h→ 0
x3
f (x) = λ + cx + d = lim (12 + 3h 2 − 12h + 24 − 12h − 1)
6 h→ 0
 8 = lim (3h 2 − 24h + 35) = 35
⇒ f (2) = λ   + 2c + d = 18 …(ii) h→ 0
 6
and lim f (x) = lim (37 − x)
λ x → 2+ x → 2+
and f (1) = + c + d = − 1 …(iii) = lim [37 − (2 + h )] = 35
6 h→ 0
From Eqs. (i), (ii) and (iii), and f (2) = 3 ⋅ 22 + 12 ⋅ 2 − 1
1 = 12 + 24 − 1 = 35
f (x) = (19x3 − 57x + 34)
4
Therefore, LHL = RHL = f (2) ⇒ function is continuous
1 57
∴ f ′ (x) = (57x − 57) =
2
(x − 1) (x + 1) at x = 2 ⇒ function is continuous in −1 ≤ x ≤ 3.
4 4
Application of Derivatives 267

f (x) − f (2) ⇒ y1 = 2
Now, Rf ′ (2) = lim
x → 2+ x−2 (x0 , y0 ) = (4t , 8t ) = (1, 4)
2

f (2 + h ) − f (2) y0 = 4
= lim
h→ 0 h  3 1 1
∴ Area = 2 −  =
37 − (2 + h ) − (3 × 22 + 12 × 2 − 1)  4 2 2
= lim
h→ 0 h
(x2 + ax + 1) − 2ax 2ax
−h 42. f (x) = =1 − 2
= lim = −1 x2 + ax + 1 x + ax + 1
h→ 0 h

f (x) − f (2) f (2 − h ) − f (2)  (x2 + ax + 1) ⋅ 2a − 2ax(2x + a ) 


and Lf ′ (2) = lim = lim f ′ (x) = −  
x → 2− x−2 h→ 0 −h  (x2 + ax + a )2 
[3(2 − h )2 + 12(2 − h ) − 1]   −2ax2 + 2a   (x2 − 1) 
 = 2 2
= 2a  2 2
...(i)
− (3 × 22 + 12 × 2 − 1)
= lim  (x + ax + a )   (x + ax + 1) 
h→ 0 −h
 (x2 + ax + 1)2 (2x) − 2(x2 − 1) 
[12 + 3h 2 − 12h + 24 − 12h − 1] − 35  (x2 + ax + 1) (2x + a ) 
= lim f ′ ′ (x) = 2a  
h→ 0 −h (x2 + ax + 1)4
 
3h 2 − 24h + 35 − 35  
= lim
h→ 0 −h 2x (x2 + ax + 1) − 2 (x2 − 1) (2x + a ) 
3h − 24 = 2a   …(ii)
= lim = 24  (x2 + ax + 1)3 
h→ 0 −1
4a (a + 2) 4a
Since, Rf ′ (2) ≠ Lf ′ (2), f ′ (2) does not exist. Now, f ′′ (1) = =
(a + 2)3 (a + 2)2
Again, f (x) is an increasing in [–1, 2] and is decreasing
in (2, 3), it shows that f (x) has a maximum value at x = 2. 4a (a − 2) − 4a
and f ′′ (−1) = =
Therefore, options (a), (b), (c), (d) are all correct. (2 − a )3
(a − 2)2
41. Here, y2 = 16x,0 ≤ y ≤ 6 ∴ (2 + a )2 f ′ ′ (1) + (2 − a )2 f ′ ′ (−1) = 4a − 4a = 0
Y 43. When x ∈ (−1, 1),
(4t 2, 8t) x2 < 1 ⇒ x2 − 1 < 0
E (0, 3) ∴ f ′ (x) < 0, f (x) is decreasing.
4a
X Also, at x = 1, f ′′ (1) = >0 [Q0 < a < 2]
G O (a + 2)2
fy
So, f (x) has local minimum at x = 1.
Tangent at F , yt = x + at 2
f ′ (ex ) x
At x = 0, y = at = 4 t 44. g′ (x) = ⋅e
1 + (ex )2
Also, (4 t 2, 8 t ) satisfy y = mx + c .
 e2x − 1   ex 
⇒ 8 t = 4mt 2 + 3 = 2a  2x 2
 
 (e + ae + 1)   1 + e 
x 2x
⇒ 4mt 2 − 8t + 3 = 0
0 3 1 g′ (x) = 0, if e2x − 1 = 0, i.e. x = 0
1 1 If x < 0, e2x < 1 ⇒ g′ (x) < 0
∴ Area of ∆ = 0 4t 1 = ⋅ 4t 2(3 − 4t )
2 2 2 d
4t 8t 1 45. Let g (x) = [ f (x) ⋅ f ′ (x)]
dx
A = 2[3t 2 − 4t3 ]
To get the zero of g (x), we take function
dA
∴ = 2[6t − 12t 2] = − 12t (12t − 1) h (x) = f (x) ⋅ f ′ (x)
dt
– + – between any two roots of h (x), there lies atleast one root
0 1 of h′ (x) = 0.
2 ⇒ g (x) = 0 ⇒ h (x) = 0
1
∴ Maximum at t = and 4mt 2 − 8t + 3 = 0 ⇒ f (x) = 0 or f ′ (x) = 0
2
If f (x) = 0 has 4 minimum solutions.
⇒ m −4 + 3 =0
f ′ (x) = 0 has 3 minimum solutions.
⇒ m =1
h (x) = 0 has 7 minimum solutions.
G (0, 4t ) ⇒ G (0,2)
⇒ h′ (x) = g (x) = 0 has 6 minimum solutions.
268 Application of Derivatives

46. To maximise area of ∆ APB, we know that, OP = 10 and 2ax 2ax − 1 2ax + b + 1
sin θ = r /10 , where θ ∈ (0, π / 2) … (i) 48. Given, f ′ (x) = b b+1 −1
Y 2(ax + b) 2ax + 2b + 1 2ax + b
P(6,8)
Applying R3 → R3 − R1 − 2R2, we get
θ 2ax 2ax − 1 2ax + b + 1
A f ′ (x) = b b+1 −1
θ
r 0 0 1
X′ Q
O
X ⇒ f ′ (x) = 2ax + b
On integrating both sides, we get
B f (x) = ax2 + bx + c
Since, maximum at x = 5 /2 ⇒ f ′ (5 /2) = 0
Y′ ⇒ 5a + b = 0 …(i)
1 f (0) = 2 ⇒ c=2
∴ Area = (2 AQ ) (PQ ) Also, …(ii)
2
and f (1) = 1 ⇒ a + b + c=1 …(iii)
= AQ . PQ = (r cos θ ) (10 − OQ )
On solving Eqs. (i), (ii) and (iii), we get
= (r cos θ ) (10 − r sin θ ) 1 5
a = ,b = − ,c=2
= 10 sin θ cos θ (10 − 10 sin 2 θ ) [from Eq. (i)] 4 4
⇒ A = 100 cos3 θ sin θ 1 2 5
Thus, f (x) = x − x + 2
dA 4 4
⇒ = 100 cos 4 θ − 300 cos 2 θ ⋅ sin 2 θ
dθ 49. Let coordinates of P be (t , t 2 + 1)
dA
Put =0 Reflection of P in y = x is P1 (t 2 + 1, t )

⇒ cos 2 θ = 3 sin 2 θ which clearly lies on y2 = x − 1
⇒ tan θ = 1 / 3 Similarly, let coordinates of Q be (s2 + 1, s)
⇒ θ = π /6 Its reflection in y = x is
dA Q1 (s, s2 + 1), which lies on x2 = y − 1.
At which < 0, thus when θ = π /6, area is maximum
dθ We have, PQ12 = (t − s)2 + (t 2 − s2)2 = P1Q 2
π ⇒ PQ1 = P1Q
From Eq. (i), r = 10 sin = 5 units
6 Also PP1 || QQ1 [Q both perpendicular to y = x]
x2 y2
47. Let us take a point P( 6 cos θ , 3 sin θ ) on + =1. C1 Y
6 3 x 2 = y −1
Now, to minimise the distance from P to given straight
P1 y=x
Q1
line x + y = 7, shortest distance exists along the
(0, 1) P
common normal.
Q y2− = 1
O
Y X′ X
(1, 0)
y
N C2
x+y=7
X′
P
X Y′
O
Thus, PP1QQ1 is an isosceles trapezium.
Also, P lies on PQ1 and Q lies on P1Q , then
Y′ PQ ≥ min { PP1QQ1 }
Let us take min { PP1QQ1 } = PP1
a 2 / x1 6 sec θ
Slope of normal at P = = = 2 tan θ = 1 ∴ PQ 2 ≥ PP12 = (t 2 + 1 − t )2 + (t 2 + 1 − t 2)
b2 / y1 6 cosec θ
2 = 2(t 2 + 1 − t 2) = f (t ) [say]
So, cos θ =
3 we have, f ′ (t ) = 4(t 2 + 1 − t )(2t − 1)

and sin θ =
1 = 4[(t − 1 / 2)2 + 3 / 4][2t − 1]
3 Now, f ′ (t ) = 0
Hence, required point is P(2, 1). ⇒ t = 1 /2
Application of Derivatives 269

Also, f ′ (t ) < 0 for t < 1 / 2 On adding Eqs. (i), (ii), (iii) and (iv), we get
and f ′ (t ) > 0 for t > 1 / 2 a 2 + b2 + c2 + d 2 = { x12 + (1 − x1 )2} + { y12 + (1 − y1 )2}
Thus, f (t ) is least when t = 1 / 2. + { x22 + (1 − x2)2} + { y22 + (1 − y2)2}
Corresponding to t = 1 / 2, point P0 on C1 is (1/2, 5/4) and where x1 , y1 , x2, y2 all vary in the interval [0, 1].
P1 (which we take as Q0) on C 2 are (5 / 4, 1 / 2). Note that Now, consider the function y = x2 + (1 − x)2, 0 ≤ x ≤ 1
P0Q0 ≤ PQ for all pairs of (P , Q ) with P on C 2. dy
differentiating ⇒ = 2x − 2(1 − x). For maximum or
50. Let the square S is to be bounded by the lines x = ±1 / 2 dx
and y = ±1 / 2. dy
minimum = 0.
2 2 dx
 1 1 
We have, a 2 =  x1 −  +  − y1 ⇒ 2x − 2(1 − x) = 0 ⇒ 2x − 2 + 2x = 0
 2 2 
⇒ 4x = 2 ⇒ x = 1 /2
Y
d 2y
A(x , 1/2) 1/2 Again, =2+2 =4
dx2
d 1
D( −1/2, 1/2) Hence, y is minimum at x = and its minimum value is
1/2 2
X'
−1/2 O
X 1/4. Clearly, value is maximum when x = 1.
B ( −1/2, y)
c 1 1 1 1
b ∴Minimum value of a 2 + b2 + c2 + d 2 = + + + = 2
2 2 2 2
C(x2 −1/2) −1/2 and maximum value is 1 + 1 + 1 + 1 = 4
Y′ 51. f (x) is a differentiable function for x > 0.
1
= x12 − y12 − x1
− y1 + Therefore, for maxima or minima, f ′ (x) = 0 must satisfy.
2
1
1 Given, f (x) = ln x − bx + x2, x > 0
Similarly, b = x2 − y1 − x2 + y1 +
2 2 2
8
2
1 1
1 ⇒ f ′ (x) = . − b + 2x
c2 = x22 − y22 + x2 + y2 + 8 x
2
For f ′ (x) = 0
1
d 2 = x12 − y22 + x1 − y2 + 1
2 ⇒ − b + 2x = 0
8x
∴ a 2 + b2 + c2 + d 2 = 2(x12 + y12 + x22 + y22) + 2
⇒ 16x2 − 8bx + 1 = 0
1
Therefore, 0 ≤ x12, x22, y12, y22 ≤ ⇒ (4x − b)2 = b2 − 1 …(i)
4
0 ≤ x12 + x22 + y12 + y22 ≤ 1 ⇒ (4x − b) = (b − 1) (b + 1)
2
[b ≥ 0, given]
⇒ 0 ≤ 2(x12 + x22 + y12 + y22) ≤ 2 Case I 0 ≤ b < 1 , has no solution. Since, RHS is
negative in this domain and LHS is positive.
But 2 ≤ 2(x12 + x22 + y12 + y22) + 2 ≤ 4 1
Case II When b = 1, then x = is the only solution.
Alternate Solution When b = 1, 4
c2 = x22 + y22 ... (i) f ′ (x) =
1
− 1 + 2x
Y 8x
2
A(x , 1) 2 2 1 1 2  1
= x − x + = x − 
a x 2 16 x  4
d B (1, y1) We have to check the sign of f ′ (x) at x = 1/4.
1/2
(0, y2)D X
Interval Sign of f′(x) Nature of f(x)
b
c −∞, 0 −ve ↓

X′  0, 1 + ve ↑
X  
O C(x2 , 0)  4
Y′  1, ∞ + ve ↑
 
4 
b = (1 − x2) +
2 2
y12 ...(ii)
a 2 = (1 − y1 )2 + (1 − x1 )2 ...(iii) From sign chart, it is clear that f ′ (x) has no change of
d 2 = x12 + (1 − y2)2 ...(iv) sign in left and right of x = 1/4.
270 Application of Derivatives

Case III When b > 1, then ⇒


dA 1
= [− k2(− cosec2θ ) − h 2 sec2θ ]
1 2 1 1 dθ 2
f ′ (x) = − b + 2x =  x2 − bx + 
8x x 2 16 1
= [k2 cosec2θ − h 2 sec2θ ]
2
2  
2
b 1 2
= 
 x −  − (b − 1) dA
x 4 16 To obtain minimum value of A, =0
 dθ
2  b 1   b 1  ⇒ k2 cosec2 θ − h 2 sec2 θ = 0
=  x − − b 2 − 1  x − + b 2 − 1 
x  4 4   4 4  k2 h2 k2
⇒ = ⇒ = tan 2 θ
=
2
(x − α ) (x − β ) sin 2 θ cos 2 θ h2
x k
1 ⇒ tan θ = ±
where, α < β and α = (b − b2 − 1 ) and h
4
1 Q tan θ < 0 , k > 0, h > 0 [given]
β = (b + b2 − 1 ). From sign scheme, it is clear that
4 k
> 0, for 0 < x < α Therefore, tan θ = − (only possible value).
 h
f ′ (x) < 0, for α < x < β d 2A 1
> 0, for x > β Now, = [−2k cosec2 θ cot θ − 2h 2 sec2 θ tan θ ]
2
 dθ 2 2
By the first derivative test, f (x) has a maxima at x = α = − [k2(1 + cot2 θ ) cot θ + h 2(1 + tan 2 θ ) tan θ ]
1  d 2A   
= (b − b2 − 1 ) h 2  − h 
4 ⇒  2 = − k21 + 2   
1  dθ  tan θ = − k/ h   k   k
and f (x) has a minima at x = β = (b + b2 − 1 )
4  k 2   − k 
+ h 2 1 + 2   
52. Let equation of any line through the point (h , k) is  h   h 
  k + h   h
2 2  h + k   k 
2 2
y − k = m(x − h ) … (i) = k2    + h 2   
   h2   h 
  k  k
2
For this line to intersect the positive direction of two
axes, m = tan θ < 0 , since the angle in anti-clockwise  (k2 + h 2) h (h 2 + k2)(k) 
direction from X-axis becomes obtuse. = + 
 k h 
Y
 h k 
= (k2 + h 2) + >0 [Q h, k > 0]
Q  k h 
(h, k)
Therefore, A is least when tan θ = − k / h. Also, the least
value of A is
X′ X
O P 1  −h   − k 
A= 2hk − k2  − h 2  
Y′ 2   k  h 
1
 k  = [2hk + kh + kh ] = 2hk
The line (i) meets X-axis at P  h − , 0 and Y-axis at 2
 m  53. Since x2 + y2 = 1 a circle S1 has centre (0, 0) and cuts
Q (0, k − mh ). X-axis at P(–1, 0) and Q(1, 0) . Now, suppose the circle
1 S 2, with centre at Q(1, 0) has radius r. Since, the circle
Let A = area of ∆ OPQ = OP . OQ
2 has to meet the first circle, 0 < r < 2 .
1 k Again, equation of the circle with centre at Q(1, 0) and
=  h −  (k − mh )
2 m radius r is
1  mh − k Y
=   (k − mh )
2 m  R S2
1 S1
=− (k − mh )2 r
2m
X′ O S X
1 (–1,0)P Q(1,0)
=− (k − h tan θ )2 [Qm = tan θ]
2 tan θ
1
=− (k2 + h 2 tan 2 θ − 2hk tan θ )
2 tan θ
Y′
1
= (2kh − k2 cot θ − h 2 tan θ )
2 (x − 1)2 + y2 = r 2 …(i)
Application of Derivatives 271

To find the coordinates of point R, we have to solve it 2 2  4 × 2  4 × 2


2
⇒ f ′ ′  = 48   − 30  
with  3  3   3 
x2 + y2 = 1 … (ii)
10 × 64 640 256
On subtracting Eq. (ii) from Eq. (i), we get = 16 × 8 − = 128 − =− <0
3 3 2
(x − 1)2 − x2 = r 2 − 1 2 2
⇒ x2 + 1 − 2 x − x 2 = r 2 − 1 Therefore, f (r ) is maximum when, r =
3
⇒ 1 − 2x = r 2 − 1 Hence, maximum value of A
2 − r2
∴ x= 1 2 2
2
2 2
2
1  8 8
2 =   4−  =   . 4−
4 3  3 4  3 3
On putting the value of x in Eq. (i), we get
 2 − r 2
2
2 . 12 − 8 2 . 2 4 4 3
  + y2 = 1 = = = =
 2  3 3 3 3 3 3 9
2  3 (b3 − b2 + b − 1)
 2 − r 2 (2 − r 2)2 − x + , if 0 ≤ x ≤ 1
⇒ y =1 − 
2
 =1 − 54. Given, f (x) =  (b2 + 3b + 2)
 2  4  2x − 3 , if 1 ≤ x ≤ 3

r 4 − 4r 2 + 4 is smallest at x = 1 .
=1 −
4 So, f (x) is decreasing on [0, 1] and increasing on [1, 3].
4 − r 4 + 4r 2 − 4 Here, f (1) = − 1 is the smallest value at x = 1.
=
4 ∴ Its smallest value occur as
4r 2 − r 4 (b3 − b2 + b − 1)
= lim f (x) = lim (− x3 ) +
4 x→1 −
x→1 –
b2 + 3b + 2
r 2(4 − r 2) In order this value is not less than –1, we must have
=
4 b3 − b2 + b − 1 (b2 + 1) (b − 1)
≥ 0 ⇒ ≥0
r 4 − r2 b2 + 3b + 2 (b + 1) (b + 2)
⇒ y=
2
Again, we know that, coordinates of S are (1 − r , 0),
therefore ∴ b ∈ (−2, − 1) ∪ [1, ∞ ]
SQ = 1 − (1 − r ) = r Y
Let A denotes the area of ∆ QSR, then 55. y = x2

1  4−r 
2
A = r r 
2  2 
  B
1 2
= r 4−r 2
4 A
X′ X
1 4
⇒ A =
2
r (4 − r 2)
16
Let f (r ) = r 4 (4 − r 2) = 4r 4 − r 6 Y′
⇒ f ′ (r ) = 16r − 6r = 2r (8 − 3r )
3 5 3 2
Any point on the parabola y = x2 is of the form (t , t 2).
For maxima and minima, put f ′ (r ) = 0 dy  dy 
Now, = 2x ⇒ = 2t
⇒ 2r3 (8 − 3r 2) = 0 dx  dx  x=t

⇒ r = 0, 8 − 3r 2 = 0 Which is the slope of the tangent. So, the slope of the


⇒ r = 0, 3r 2 = 8 normal to y = x2 at A (t , t 2) is –1/2t.
⇒ r = 0, r 2 = 8/3 Therefore, the equation of the normal to
2 2 y = x2 at A (t , t 2) is
⇒ r = 0, r =  1
3 y − t 2 =  −  (x − t ) …(i)
 2t 
[ Q0 < r < 2 , so r = 2 2 / 3]
Suppose Eq. (i) meets the curve again at B(t1 , t12).
Again, f ′ ′ (r ) = 48r 2 − 30r 4
272 Application of Derivatives

Then, t12 − t 2 = −
1
(t1 − t ) and when t = − 1 / 2 , the equation of AB is
2t y −2 x− 2
1 =
− 2 −   − 2

⇒ (t1 − t ) (t1 + t ) = − (t1 − t ) 1 1
2t 2  2
1
⇒ (t1 + t ) = −  1  1 
2t ⇒ ( y − 2)  − − 2 = (x − 2 )  − 2
 2  2 
1
⇒ t1 = − t − ⇒ 2 y − 4 = 2 (x − 2 )
2t
Therefore, length of chord, ⇒ 2x − 2 y + 2 = 0
L = AB2 = (t − t1 )2 + (t 2 − t12)2 56. Let 2b be the diameter of the circular portion and a be
the lengths of the other sides of the rectangle.
= (t − t1 ) + (t − t1 ) (t + t1 )
2 2 2

Total perimeter = 2a + 4b + πb = K [say] …(i)


= (t − t1 ) [1 + (t + t1 ) ]
2 2

Now, let the light transmission rate (per square metre)


1  1 
2 2
 
=  t + t +  1 +  t − t −   of the coloured glass be L and Q be the total amount of
 2t    2t   transmitted light.
2 3
 1  1   1 
⇒ L =  2t +   1 + 2 = 4t 2 1 + 2
 2t   4t   4t  Coloured
glass
On differentiating w.r.t. t, we get 2b
3 2
dL  1   1   2 
= 8t 1 + 2 + 12t 2 1 + 2  − 3 
dt  4t   4t   4t  a Clear glass a
2
 1    1  3
= 2 1 + 2 4t 1 + 2  − 
 4t    4t  t 
2 2
2b
 1   2  1   1
= 2  1 + 2  4t −  = 4  1 + 2  2t −  1
 4t   t  4t   t Then, Q = 2ab (3L ) + πb2(L )
2
dL
For maxima or minima, we must have =0 L
dt ⇒ Q= (πb2 + 12ab)
1 1 2
⇒ 2t − = 0 ⇒ t 2 = L
t 2 ⇒ Q = [πb2 + 6b (K − 4b − πb)] [from Eq. (i)]
1 2
⇒ t=±
2 L
⇒ Q = (6Kb − 24b2 − 5πb2)
d 2L  1  1  1 2
Now, = 8  1 + 2  − 3   2t − 
dt 2  4t   2t   t On differentiating w.r.t. b, we get
2
 1   1 dQ L
= (6K − 48b − 10πb)
+ 4  1 + 2  2 + 2
 4t   t  db 2
dQ
 d 2L   1
2 For maximum, put =0
⇒  2 = 0 + 4 1 +  (2 + 2) > 0 db
 2
 dt  t = ± 1/ 2 6K
⇒ b= …(ii)
48 + 10 π
Therefore, L is minimum, when t = ± 1 / 2.For t = 1 / 2 ,
point A is (1 / 2 , 1 / 2) and point B is (− 2 , 2). When d 2Q L
Now, = (− 48 − 10π ) < 0
t = − 1 / 2 , A is (−1 / 2 , 1 / 2), B is ( 2 , 2). db2 2
Again, when t = 1 / 2 , the equation of AB is Thus, Q is maximum and from Eqs. (i) and (ii), we get
(48 + 10π ) b = 6 {2a + 4b + πb}
y−2 x+ 2 2b 6
= ∴ Ratio = = = 6 :6 + π
1 1 a 6+ π
−2 + 2
2 2
57. Since, the chord QR is parallel to the tangent at P.
 1  1 
⇒ ( y − 2 )  + 2  = (x + 2)  − 2 ∴ ON ⊥ QR
  2    2 
Consequently, N is the mid-point of chord QR.
⇒ − 2 y + 4 = 2x + 2 ∴ QR = 2QN = 2r sin θ
⇒ 2x + 2 y − 2 = 0 Also, ON = r cos θ
Application of Derivatives 273

∴ PN = r + r cos θ d2
Now, ( AP )2 = − {(8 − 2a 2) sin θ + 8} sin θ
P dθ 2
+ (8 − 2a 2) ⋅ cos 2 θ
r π d2
For θ = , we have ( AP )2 = − (16 − 2a 2) < 0
O
2 dθ 2
r r π
Thus, AP 2 i.e. AP is maximum when θ = .The point on
2
Q R
N the curve 4x2 + a 2y2 = 4a 2 that is farthest from the point
 π π
M A(0, − 2) is  a cos , 2 sin  = (0, 2)
 2 2

Let A denotes the area of ∆PQR. 59. Let AF = x and AE = y, ∆ABC and ∆EDC are similar.
1 AB AC
Then, A = ⋅ 2r sin θ (r + r cos θ ) ∴ =
2 ED CE
⇒ A = r 2(sin θ + sin θ cos θ ) C
1 2
( r , – r)
⇒ A = r 2(sin θ + sin 2θ )
2
dA
⇒ = r (cos θ + cos 2θ )
2 a
dθ b E D
d 2A
and = r 2(− sin θ − 2 sin 2θ ) y
dθ 2
dA
For maximum and minimum values of θ,we put =0 A 2 x F B
dθ (p , – p ) (q 2, –q)
⇒ cos θ + cos 2θ = 0 ⇒ cos 2θ = − cos θ c
π
⇒ cos θ = cos (π − 2θ ) ⇒ θ = c b
3 ⇒ =
x b− y
d 2A π
Clearly, < 0 for θ = ⇒ bx = c (b − y)
dθ 2 3
π c
Hence, the area of ∆PQR is maximum when θ = . ⇒ x = (b − y)
3 b
The maximum area of ∆ PQR is given by Let z denotes the area of par
 π 1 2π   3 3 allelogram AFDE.
A = r 2 sin + sin  = r 2  + 
 3 2 3  2 4 Then, z = xy sin A
c
3 3 2 ⇒ z = (b − y) y ⋅ sin A …(i)
= r sq units b
4
On differentiating w.r.t. y we get
58. Let P (a cos θ , 2 sin θ ) be a point on the ellipse dz c
= (b − 2 y) sin A
x2 y2 dy b
4x2 + a 2y2 = 4a 2, i.e. + =1
a2 4 d 2z −2c
and = sin A
Let A(0, − 2) be the given point. dy2 b
Then,
For maximum or minimum values of z, we must have
( AP )2 = a 2 cos 2 θ + 4 (1 + sin θ )2 dz
d =0
⇒ ( AP )2 = − a 2 sin 2θ + 8 (1 + sin θ ) ⋅ cos θ dy
dθ c
d ⇒ (b − 2 y) = 0
⇒ ( AP )2 = [(8 − 2a 2) sin θ + 8 ] cos θ b

b
d ⇒ y=
For maximum or minimum, we put ( AP )2 = 0 2

⇒ [(8 − 2a 2) sin θ + 8] cos θ = 0 d 2z 2c
Clearly, 2
=− < 0, ∀ y
4 dy b
⇒ cos θ = 0 or sin θ = 2
a −4 Hence, z is maximum, when y =
b
.
4 2
[Q 4 < a 2 < 8 ⇒ > 1 ⇒ sin θ > 1, which is b
a2 − 4 impossible] On putting y = in Eq. (i), we get
2
274 Application of Derivatives

the maximum value of z is For greatest or least values of m, we should have


c b b 1 g′ (x) = 0 ⇒ x = 0, x = ± 3
z =  b −  ⋅ ⋅ sin A = bc sin A
b 2 2 4 Now,
1 (1 + x2)3 (6x2 − 6) − (2x3 − 6x) ⋅ 3 (1 + x2)2 ⋅ 2x
= area of ∆ABC g′ ′ (x) =
2 (1 + x2)6
p2 − p 1
1 1 2 At x = 0, g′ ′ (x) = − 6 < 0
= × q q 1
2 2 2 ∴ g′ (x) has a maximum value at x = 0.
r −r 1
⇒ (x = 0, y = 0) is the required point at which tangent to
Applying R3 → R3 − R1 and R2 → R2 − R1 the curve has the greatest slope.
p2 −p 1 62. Let the house of the swimmer be at B.
1 ∴ AB = L km
= q − p2 q + p 0
2
4
r 2 − p2 − r + p 0 Let the swimmer land at C on the shore and let
AC = x km
p2 −p 1
1
= ( p + q) (r − p) q − p 1 0 S
4
r + p −1 0
1 x 2 + d2
=( p + q) (r − p) (− q − r ) d
4
1
= ( p + q) (q + r ) ( p − r ) A x C (L – x) B
4
π π L
60. Let y = f (x) = sin x + λ sin 2 x, − < x <
3
2 2
∴ SC = x2 + d 2 and CB = (L − x)
Let sin x = t
Distance
∴ y = t3 + λt 2, − 1 < t < 1 ∴ Time =
Speed
dy
⇒ = 3t 2 + 2tλ = t (3 t + 2λ ) Time from S to B = Time from S to C + Time from C to B
dt
For exactly one minima and exactly one maxima dy/dt x2 + d 2 L − x
∴ T= +
must have two distinct roots ∈ (−1, 1). u v
2λ 1 L x
⇒ t = 0 and t = − ∈ (−1, 1) Let f (x) = T = x + d2 +
2

3 u v v
2λ 1 1 ⋅ 2x 1
⇒ −1 < − <1 ⇒ f ′ (x) = ⋅ +0−
3 u 2 x2 + d 2 v
3 3
⇒ − <λ< For maximum or minimum, put f ′ (x) = 0
2 2
 3 3 ⇒ v2x2 = u 2 (x2 + d 2)
⇒ λ ∈− , 
 2 2 u 2d 2
⇒ x2 =
x v2 − u 2
61. Given, y=
1 + x2 ∴ f ′ (x) = 0 at x = ±
ud
, (v > u )
dy (1 + x ) ⋅ 1 − x (2x)
2
1−x 2 v − u2
2
⇒ = =
dx (1 + x2)2 (1 + x2)2 − ud
But x≠
dy v2 − u 2
Let = g (x) [i.e. slope of tangent]
dx ud
∴ We consider, x =
1 − x2 v − u2
2
∴ g (x) =
(1 + x2)2
1 d2
(1 + x2)2 ⋅ (−2x) − (1 − x2) ⋅ 2 (1 + x2) ⋅ 2x Now, f ′ ′ (x) = > 0, ∀ x
⇒ g ′ (x) = u x2 + d 2 (x2 + d 2)
(1 + x2)4
ud
−2x (1 + x2) [(1 + x2) + 2 (1 − x2)] −2x (3 − x2) Hence, f has minimum at x = .
= = v − u2
2
(1 + x2)4 (1 + x2)3
Application of Derivatives 275

b π 5π 13π
63. Given, ax2 + ≥ c, ∀ x > 0 ; a , b > 0 ⇒ θ= , , ,…
x 12 12 12
Let
b
f (x) = ax2 + − c but θ ∈ { λ 1π , λ 2π , … λ rπ }, where 0 < λ 1 < … < λ r < 1.
x π 5π
∴ θ= ,
b 2ax3 − b 12 12
∴ f ′ (x) = 2ax − =
x2 x2 1 5
So, λ 1 + … + λ r = + = 0.50
2b 12 12
⇒ f ′ ′ (x) = 2a + 3 > 0 [since, a , b are all positive]
x 66. Given set S of polynomials with real coefficients
1/3
 b S = {(x2 − 1)2(a 0 + a1x + a 2x2 + a3 x3 ) :
Now, put f ′ (x) = 0 ⇒ x =   >0 [Q a , b > 0]
 2a  a 0 , a1 , a 2, a3 ∈ R}
1/3 and for a polynomial f ∈ S, Let
 b
At x=  , f ′ ′ (x) = + ve f (x) = (x2 − 1)2 (a 0 + a1x + a 2x2 + a3 x3 )
 2a 
1/3
it have − 1 and 1 as repeated roots twice, so graph of f (x)
 b touches the X-axis at x = − 1 and x = 1, so f ′ (x) having at
⇒ f (x) has minimum at x =   .
 2a  least three roots x = − 1, 1 and α . Where α ∈ (− 1, 1) and
  b  1/3  2 /3 f ′ ′ (x) having at least two roots in interval (− 1, 1)
 b b
and f     = a   + − c≥ 0 So, mf ′ = 3 and mf ′ ′ = 2
  2 a    2 a  (b / 2 a )1/3
∴Minimum possible value of (mf ′ + mf ′ ′ ) = 5
1/3
 2a  3b 67. On drawing the diagram of given situation
=  ⋅ − c≥0
 b 2 MD 2 + MC 2 = 64 + x2 + 121 + (10 − x)2
1/3
 2a  3b = f (x) [say]
⇒   ⋅ ≥c
 b 2 ⇒ f (x) = 2x − 20x + 285
2

On cubing both sides, we get C


2a 27b3
⋅ ≥ c3 D
b 8
⇒ 27ab ≥ 4c
2 3
11
64. Let f (x) = x + y, where xy = 1 8
1
⇒ f (x) = x +
x A M B
1 x2 − 1 x 10 – x
⇒ f ′ (x) = 1 − 2 =
x x2 For minima, f ′ (x) = 0 ⇒ 4x − 20 = 0 ⇒ x = 5
Also, f ′ ′ (x) = 2 / x3 As f ′ ′ (x) = 4 > 0
On putting f ′ (x) = 0, we get ∴ AM = 5 m
x = ± 1, but x > 0 [neglecting x = − 1] 68. Here, volume of cylindrical container, V = πr 2h …(i)
f ′ ′ (x) > 0, for x = 1
and let volume of the material used be T.
Hence, f (x) attains minimum at x = 1, y = 1
r+2
⇒ (x + y) has minimum value 2.
2
65. The given function f : R → R be defined by O r

f ( θ ) = (sin θ + cos θ )2 + (sin θ − cos θ )4


= 1 + sin 2 θ + (1 − sin 2 θ )2
h
= 1 + sin 2 θ + 1 + sin 2 2 θ − 2 sin 2 θ
= sin 2 2 θ − sin 2 θ + 2
2
 1 7
= sin 2 θ −  +
 2 4
The local minimum of function ‘ f ’ occurs when ∴ T = π [(r + 2)2 − r 2] h + π (r + 2)2 × 2
1 V
sin 2 θ = ⇒ T = π [(r + 2)2 − r 2] ⋅ 2 + 2π (r + 2)2
2 πr
π 5π 13π V
⇒ 2θ = , , … [Q V = πr 2h ⇒ h = ]
6 6 6 πr 2
276 Application of Derivatives

1  2 3 ⋅ sin3 θ 
2
 r + 2
⇒ T =V   + 2π (r + 2) − V
2
∴ ∆1 = ∆ max occurs at cos θ = = 
 r  4  cos θ 
On differentiating w.r.t. r, we get 1 45 5
When cos θ = =
dT  r + 2   −2  4 8
= 2V ⋅   ⋅   + 4π (r + 2)
dr  r   r2  1
∆ 2 = ∆ min occurs at cos θ =
dT 2
At r = 10, =0
dr  2 3 sin3 θ 
= 
 V  cos θ 
Now, 0 = (r + 2) ⋅ 4  π − 3 
 r  1 9
When cos θ = =
V 2 2
⇒ =π
r3 8
∴ ∆1 − 8∆ 2 = 45 − 36 = 9
where r = 10 5
V
⇒ =π 70. PLAN
1000 (i) Local maximum and local minimum are those points at which
V f ′ ( x ) = 0, when defined for all real numbers.
or =4
250π (ii) Local maximum and local minimum for piecewise functions
are also been checked at sharp edges.
69. PLAN As to maximise or minimise area of triangle, we should find x, if x ≥ 0
area in terms of parametric coordinates and use second Description of Situation y =|x|= 
derivative test.  − x, if x < 0
Here, tangent at P(2 cos θ , 3 sin θ ) is (x2 − 1), if x ≤ − 1 or x ≥ 1
Also, y =|x2 − 1|= 
 (1 − x ), if − 1 ≤ x ≤ 1
2

P (2 cos θ, √3 sin θ)  − x + 1 − x2 , if x ≤ − 1

− x + 1 − x , if − 1 ≤ x ≤ 0
2
y =|x|+ |x2 − 1|= 
x + 1 − x , if 0 ≤ x ≤ 1
2
O (h,0) R
(2 sec θ, 0)  x + x2 − 1 , if x ≥ 1
Q
(2 sec θ, –√3 sin θ)  − x2 − x + 1 , if x ≤ − 1
 2
 − x − x + 1, if − 1 ≤ x ≤ 0
x y = 2
cos θ + sin θ = 1
2 3 − x + x + 1, if 0 ≤ x ≤ 1
 x2 + x − 1, if x≥1
∴ R(2 sec θ , 0)
⇒ ∆ = Area of ∆PQR which could be graphically shown as
1 Y
= (2 3 sin θ ) (2 sec θ − 2 cos θ )
2
= 2 3 ⋅ sin3 θ/cos θ …(i) – x 2– x + 1 – x 2– x + 1 – x 2+ x +1 x 2+x –1
1
Since, ≤ h ≤1 1
2
1 X
∴ ≤ 2 cos θ ≤ 1 –1 –1/2 O 1/2 1
2 1 −1
Thus, f (x) attains maximum at x = , and f (x)
1 1 2 2
⇒ ≤ cos θ ≤ …(ii)
4 2 attains minimum at x = − 1, 0, 1.
d∆ 2 3 {cos θ ⋅ 3 sin θ cos θ − sin θ (− sin θ )}
2 3 ⇒ Total number of points = 5
∴ =
dθ cos 2 θ 71. PLAN If f( x ) is least degree polynomial having local maximum and
local minimum at α and β.
2 3 ⋅ sin θ
2
= [3 cos 2 θ + sin 2 θ ] Then, f ′ (x) = λ (x − α ) (x − β )
cos 2 θ
Here, p′ (x) = λ (x − 1) (x − 3) = λ (x2 − 4x + 3)
2 3 sin 2 θ
= ⋅ [2 cos 2 θ + 1] On integrating both sides between 1 to 3, we get
cos 2 θ
3 3
∫1 p′ (x) dx = ∫1 λ (x − 4x + 3) dx
2
= 2 3 tan 2 θ (2 cos 2 θ + 1) > 0
3
1 1  x3 
When ≤ cos θ ≤ , ⇒ ( p(x))31 = λ  − 2x2 + 3x
4 2 3 1
Application of Derivatives 277

 1  f (θ ) =
1
⇒ p(3) − p(1) = λ  (9 − 18 + 9) −  − 2 + 3  74. Let
 3  sin 2 θ + 3 sin θ cos θ + 5 cos 2 θ
− 4 Again let, g (θ ) = sin 2 θ + 3 sin θ cos θ + 5 cos 2 θ
⇒ 2 −6 = λ  
3  1 − cos 2 θ  1 + cos 2 θ  3
= +5  + sin 2 θ
2  2  2
⇒ λ =3
⇒ p′ (x) = 3 (x − 1) (x − 3) 3
= 3 + 2 cos 2 θ + sin 2 θ
2
∴ p′ (0) = 9
9
∴ g (θ )min = 3 − 4 +
72. f (x) = x − 4x3 + 12x2 + x − 1
4
4
f ′ (x) = 4x3 − 12x2 + 24x + 1 5 1
=3− =
2 2
f ′′ (x) = 12x2 − 24x + 24 = 12 (x2 − 2x + 2)
1
= 12 {(x − 1)2 + 1} > 0 ∀x ∴ Maximum value of f (θ ) = =2
12
⇒ f ′ (x) is increasing.
75. Given, A = { x|x2 + 20 ≤ 9x} = { x|x ∈ [4, 5]}
Since, f ′ (x) is cubic and increasing.
⇒ f ′ (x) has only one real root and two imaginary roots. Y
∴ f (x) cannot have all distinct roots.
⇒ Atmost 2 real roots.
O 2 3 4 X
Now, f (− 1) = 15, f (0) = − 1, f (1) = 9 5
∴ f (x) must have one root in (− 1, 0) and other in (0, 1). –16
⇒ 2 real roots. –20
–21
73. Let g (x) = e f ( x ), ∀ x ∈ R
⇒ g′ (x) = ef ( x ) ⋅ f ′ (x)
⇒ f ′ (x) changes its sign from positive to negative in the Now, f ′ (x) = 6(x2 − 5x + 6)
neighbourhood of x = 2009 Put f ′ (x) = 0 ⇒ x = 2, 3
⇒ f (x) has local maxima at x = 2009. f (2) = − 20, f (3) = − 21, f (4) = − 16, f (5) = 7
So, the number of local maximum is one. From graph, maximum value of f (x) on set A is f (5) = 7.
11
Indefinite Integration
Topic 1 Some Standard Results
Objective Questions I (Only one correct option) 6. If ∫
dx
cos θ (x − 2x + 10)2
2
1. If ∫ dθ = A log e|B(θ )| + C , where C is a  
5 + 7 sin θ − 2 cos 2 θ  x − 1 f (x)
= A  tan −1   +  + C , where, C is a
B(θ )   3  x2 − 2x + 10
constant of integration, then can be
A constant of integration, then (2019 Main, 10 April I)
(2020 Main, 5 Sep II) 1
2 sin θ + 1 2 sin θ + 1 (a) A = and f (x) = 9 (x − 1)
(a) (b) 27
sin θ + 3 5(sin θ + 3) 1
(b) A = and f (x) = 3 (x − 1)
5(sin θ + 3) 5(2 sin θ + 1) 81
(c) (d)
2 sin θ + 1 sin θ + 3 1
(c) A = and f (x) = 3 (x − 1)
x 54
2. Let f (x) = ∫ dx (x ≥ 0). Then f (3) − f (1) is equal to
(d) A =
1
and f (x) = 9 (x − 1)2
(1 + x)2 (2020 Main, 4 Sep I) 54
π 1 3 π 1 3
(a) − + + (b) − + + dx
1
6 2 4 12 2 4 7. If ∫ = xf (x)(1 + x6 )3 + C
π 1 3 π 1 3 x (1 + x6 )23
3 /

(c) + − (d) + −
6 2 4 12 2 4 where, C is a constant of integration, then the
dθ function f (x) is equal to (2019 Main, 8 April II)
3. If ∫ = λ tan θ + 2 log e| f (θ )| + C 1 1
cos θ (tan 2 θ + sec 2 θ )
2
(a) − (b) −
where C is a constant of integration, then the ordered pair 6x3 2x3
1 3
(λ , f (θ )) is equal to (2020 Main, 9 Jan II) (c) − (d)
(a) (1, 1 + tan θ) (b) (1, 1 − tan θ) 2x2 x2
(c) (−1, 1 + tan θ) (d) (−1, 1 − tan θ) sin
5x
4. Let α ∈ (0, π / 2) be fixed. If the integral 8. ∫ 2 dx is equal to
x
tan x + tan α sin
∫ tan x − tan α
dx = A (x) cos 2α + B (x) 2
(where, C is a constant of integration)
sin 2 α + C, where C is a constant of integration, then the (2019 Main, 8 April I)
functions A (x) and B (x) are respectively (a) 2x + sin x + 2 sin 2x + C
(2019 Main, 12 April II) (b) x + 2 sin x + 2 sin 2x + C
(a) x + α and log e|sin(x + α )| (b) x − α and log e|sin(x − α )| (c) x + 2 sin x + sin 2x + C
(c) x − α and log e|cos(x − α )| (d) x + α and log e|sin(x − α )| (d) 2x + sin x + sin 2x + C
2x3 − 1 3x13 + 2x11
5. The integral ∫ dx is equal to 9. The integral ∫ dx is equal to (where C
x4 + x (2x4 + 3x2 + 1)4
(here C is a constant of integration) (2019 Main, 12 April I) is a constant of integration) (2019 Main, 12 Jan II)
1 |x + 1|
3
1 (x + 1)2
3 x4 x12
(a) log e +C (b) log e +C (a) +C (b) +C
2 x2 2 |x3| 6(2x4 + 3x2 + 1)3 6(2x4 + 3x2 + 1)3
x3 + 1 |x3 + 1| x4 x12
(c) log e +C (d) log e +C (c) +C (d) +C
x x2 (2x + 3x + 1)
4 2 3
(2x + 3x2 + 1)3
4
Indefinite Integration 279

x+1 15. The integral


10. If ∫ dx = f (x) 2x − 1 + C, where C is a constant
2x − 1 sin 2 x cos 2 x
of integration, then f (x) is equal to (2019 Main, 11 Jan II)
∫ (sin5 x + cos3 x sin 2 x + sin3 x cos2 x dx

2 1 + cos5 x)2
(a) (x + 2) (b) (x + 4)
3 3 is equal to (2018 Main)
2 1 1 −1
(c) (x − 4) (d) (x + 1) (a) +C (b) +C
3 3 3 (1 + tan3 x) 3 (1 + tan3 x)
1 − x2 1 −1
(c) +C (d) +C
11. If ∫ dx = A (x)( 1 − x2 )m + C, 1 + cot3 x 1 + cot3 x
x4
for a suitable chosen integer m and a function A (x), (where C is a constant of integration)
where C is a constant of integration, then ( A (x))m equals dx
16. The value of ∫ 2 4 is (2015 Main)
(2019 Main, 11 Jan I) x (x + 1)3/ 4
1 −1 1
(a) (b) 1
9x4 3x3  x4 + 1 4
−1 1 (a)  4  + c (b) (x4 + 1) 4 + c
(c) (d)  x  1
27x9 27x6 1
 x4 + 1 4
π (c) − (x +
4
1) 4 +c (d) −  4  + c
12. Let n ≥ 2 be a natural number and 0 < θ < . Then,  x 
2
1 sec2 x
(sin θ n
− sin θ ) n
cos θ
17. ∫ (sec x + tan x)9/ 2 dx equals to
∫ sin n + 1 θ
dθ is equal to
(for some arbitrary constant K) (2012)
(where C is a constant of integration)
−1 1 − (sec x + tan x)2  + K
(2019 Main, 10 Jan I) 1
n+1
(a) 
(sec x + tan x)11/ 2 11 7 
n  1  n
(a) 1 −  +C 1 1 − 1 2
n 2 − 1 sin n + 1 θ  (b)  (sec x + tan x)  + K
n+1
(sec x + tan x)11/ 2 11 7 
n 1 + 1  −1 1 + 1 2
(b)  
n
+C (c)  (sec x + tan x)  + K
n 2 − 1 sin n − 1 θ  (sec x + tan x)11/ 2 11 7 
n+1 1 1 + 1 2
(sec x + tan x)  + K
n 1 − 1 
(d) 
(c)  
n
+C (sec x + tan x)11/ 2 11 7 
n 2 − 1 sin n − 1 θ 
ex e− x
n+1 18. If I = ∫ dx, J = ∫ − 4x dx.
(d)
n 
1 −
1 

n
+C
4x
e +e +1
2x
e + e−2x + 1
n 2 + 1 sin n − 1 θ 
Then, for an arbitrary constant c, the value of J − I
5x8 + 7x6 equals (2008, 3M)
13. If f (x) = ∫ 2 dx, (x ≥ 0), and f (0) = 0, then
(x + 1 + 2x7 )2 1 e4 x − e2x + 1 1 e2x + ex + 1
(a) log 4 x +c (b) log 2x +c
the value of f (1) is (2019 Main, 9 Jan I) 2 e + e2x + 1 2 e − ex + 1
1 1
(a) − (b) − 1 e2x − ex + 1 1 e4 x + e2x + 1
2 4 (c) log 2x +c (d) log 4 x +c
1 1 2 e + ex + 1 2 e − e2x + 1
(c) (d)
4 2 x
19. If f (x) = for n ≥ 2 and g (x) = ( fofo ... of ) (x).
14. For x ≠ nπ + 1, n ∈ N (the set of natural numbers), the
2
(1 + xn )1/ n 14243
f occurs n times
integral Then, ∫ xn − 2g (x) dx equals (2007, 3M)
2 sin(x2 − 1) − sin 2(x2 − 1)
∫ x 2 sin(x2 − 1) + sin 2(x2 − 1) dx is equal to
1 1
1 1− 1 1−
(a) (1 + nxn ) n + c (b) (1 + nxn ) n +c
n (n − 1) n −1
(where C is a constant of integration ) (2019 Main, 9Jan I) 1 1
1 1+ 1 1+
(a)
1
log e|sec(x2 − 1)| + C (c) (1 + nxn ) n + c (d) (1 + nxn ) n + c
2 n (n + 1) n+1
 x2 − 1 (x2 − 1) dx
(b) log e sec   + C 20. The value of ∫ is (2006, 3M)
 2  x 3
2 x4 − 2 x2 + 1
1
(c) log e sec2 (x2 − 1) + C (a) 2 2 −
2
+
1
+c (b) 2 2 +
2
+
1
+ c
2
x2 x4 x2 x4
1  x2 − 1
(d) log e sec2   + C 1 2 1
(c) 2− 2 + 4 + c (d) None of these
2  2  2 x x
280 Indefinite Integration

Objective Questions II Analytical & Descriptive Questions


(One or more than one correct option) 24. For any natural number m, evaluate
∫ (x + x2m + xm ) (2 x2m + 3 xm + 6)1/m dx, x > 0. (2002, 5M)
3m
21. Let f : R → R and g : R → R be two non-constant
differentiable functions. If f ′ (x) = (e( f ( x ) − g( x )) ) g′ (x) for  1 − x
1/ 2
dx
all x ∈ R and f (1) = g (2) = 1, then which of the following 25. Evaluate ∫  
 1 + x

x
(1997C, 3M)
statement(s) is (are) TRUE? (2018 Adv.)
(a) f (2) < 1 − log e 2 1− x
26. Evaluate ∫ dx. (1985, 2 1 M)
(b) f (2) > 1 − log e 2 1+ x 2

(c) g (1) > 1 − log e 2 dx


27. Evaluate ∫ 2 4 . (1984, 2M)
(d) g (1) < 1 − log e 2 x (x + 1)3/ 4
28. Evaluate the following: (1980, 4M)
Integer & Numerical Answer Type Question 1  x2
(i) ∫ 1 + sin  x dx (ii) ∫ dx
22. Let f : R → R be a differentiable function with f (0) = 1 2  1−x
and satisfying the equation f (x + y) = f (x) f ′ ( y) x2
+ f ′ (x) f ( y) for all x, y ∈ R. 29. Integrate . (1979, 2M)
(a + bx)2
Then, the value of log e ( f (4)) is ....... . (2018 Adv.)
30. Integrate
sin x ⋅ sin 2 x ⋅ sin 3x + sec2 x ⋅ cos 2 2x + sin 4 x ⋅ cos 4 x.
Fill in the Blank (1979, 1M)
−x x
4e + 6e
x 31. Integrate the curve . (1978, 1M)
23. If ∫ dx = Ax + B log (9e2x − 4) + C, then A = K, 1 + x4
9ex − 4e− x
1 sin x
B = ... and C = K . (1989, 2M) 32. Integrate or . (1978, 2M)
1 − cot x sin x − cos x

Topic 2 Some Special Integrals


Objective Question I (Only one correct option) Analytical & Descriptive Questions
1. The integral ∫ sec 23
/ 4/3
x cosec x dx is equal to (here C is a 3. Find the indefinite integral
constant of integration)  1 ln (1 + 6 x )

(2019 Main, 9 April I)
3 +  dx. (1992, 4M)
(a) 3 tan −1/3 x + C (b) −3 tan −1/3 x + C  x+ 4
x 3
x+ x 
3
(c) −3 cot −1/3 x + C (d) − tan −4 /3 x + C
4
4. Evaluate ∫( tan x + cot x ) dx. (1988, 3M)

2. Let I n = ∫ tan n x dx (n > 1). If (cos 2x)1/ 2


5. Evaluate ∫ sin x
dx. (1987, 6M)
I 4 + I 6 = a tan5 x + bx5 + C, where C is a constant of
integration, then the ordered pair (a , b) is equal to 2 sin x − sin 2 x
6. If f (x) is the integral of , where x ≠ 0, then
(2017 Main) x3
(a)  − , 1 (b)  , 0 (c)  , − 1 (d)  − , 0
1 1 1 1 find lim f ′ (x). (1979, 3M)
 5  5  5   5  x→ 0

Topic 3 Integration by Parts


Objective Questions (Only one correct option) 2. If ∫ esec x (sec x tan x f (x) + (sec x tan x + sec2 x))
2 2
5 −x
1. If ∫xe dx = g (x)e− x + C, where C is a constant of dx = esec x f (x) + C, then a possible choice of f (x) is
(2019 Main, 9 April II)
integration, then g (− 1) is equal to (2019 Main, 10 April II)
1 1
(a) − 1 (b) 1 (a) x sec x + tan x + (b) sec x + tan x +
2 2
1 5
(c) − (d) − 1 1
2 2 (c) sec x + x tan x − (d) sec x − tan x −
2 2
Indefinite Integration 281

3. The integral ∫ cos (log e x) dx is equal to (where C is a 6. If ∫ f (x) dx = ψ (x), then ∫ x5 f (x3 ) dx is equal to
constant of integration) 1 3
(2019 Main, 12 Jan I) (a) [x ψ(x3 ) − ∫ x2ψ(x3 )dx] + c (2013 Main)
(a) [cos(log e x) + sin(log e x)] + C
x 3
1 3
2 (b) x ψ(x3 ) − 3 ∫ x3 ψ(x3 ) dx + c
(b) x [cos(log e x) + sin(log e x)] + C 3
1
(c) x3 ψ(x3 ) − ∫ x2ψ(x3 ) dx + c
(c) x [cos(log e x) − sin(log e x)] + C 3
1
(d) [sin(log e x) − cos(log e x)] + C
x (d) [x3 ψ(x3 ) − ∫ x3 ψ(x3 ) dx] + c
2 3
1 −4 x 3
3
4. If ∫ x5 e−4x dx = e f (x) + C , Analytical & Descriptive Questions
48
where C is a constant of integration, then f (x) is equal to  2x + 2 
7. Evaluate ∫ sin −1   dx. (2000, 5M)
(2019 Main, 10 Jan II)  4x2 + 8 x + 13 
(a) − 4x3 − 1 (b) 4x3 + 1  
(c) − 2x3 − 1 (d) − 2x3 + 1 8. Find the indefinite integral
1
 1 x + x  cos θ + sin θ 
5. ∫ 1 + x − 
 x
e dx is equal to (2014 Main)
∫ cos 2θ log cos θ − sin θ  dθ. (1994, 5M)

1
x+
(a) (x − 1) e x + c sin −1 x − cos −1 x
9. Evaluate ∫ dx. (1986, 2½ M)
x+
1 sin −1 x + cos −1 x
(b) x e x + c
(x − 1) ex
x+
1
10. Evaluate ∫ dx. (1983, 2M)
(c) (x + 1) e x + c (x + 1)3
1
x+
(d) −x e x + c 11. Evaluate ∫ (elog x + sin x) cos x dx. (1981, 2M)

Topic 4 Integration, Irrational Function and Partial Fraction


Objective Questions (Only one correct option) cos3 x + cos5 x
2. The value of ∫ dx is (1995, 2M)
2x12 + 5x9 sin 2 x + sin 4 x
1. The integral ∫ dx is equal to
(x5 + x3 + 1)3 (2016 Main) (a) sin x − 6 tan −1 (sin x) + c
− x5 (b) sin x − 2 (sin x)−1 + c
(a) +C
(x + x + 1)
5 3 2 (c) sin x − 2 (sin x)−1 − 6 tan −1 (sin x) + c
x 10 (d) sin x − 2 (sin x)−1 + 5 tan −1 (sin x) + c
(b) +C
2(x5 + x3 + 1)2
x5
Analytical & Descriptive Questions
(c) +C
2(x + x + 1)
5 3 2 x3 + 3x + 2
−x 10
3. ∫ (x + 1)2 (x + 1)
2
dx. (1999, 5M)
(d) +C
2(x5 + x3 + 1)2 (x + 1)
4. Evaluate ∫ dx. (1996, 2M)
where, C is an arbitrary constant. x (1 + xex )2
Answers
3 2/3 12 7/12 4 1/2 12 5/12 1 1/3
Topic 1 3. x − x + x − x + x − 4 x 1/ 4 − 7 x 1/ 6
2 7 3 5 2
1. (d) 2. (d) 3. (c) 4. (b)
− 12 x1/12 + (2 x1/2 − 3 x1/3 + 6 x1/6 + 11 ) ln (1 + x1/6 )
5. (c) 6. (c) 7. (b) 8. (c)
9. (b) 10. (b) 11. (c) 12. (c) + 12 ln (1 + x1/2 ) − 3 [ ln(1 + x1/6 )]2 + c
13. (c) 14. (b) 15. (b) 16. (d)  tan x − cot x 
4. 2 tan −1   +c
17. (c) 18. (c) 19. (a) 20. (c)  2 
3 35
21. (b,c) 22. (2) 23. A = − , B = and C ∈R 1 2 + 1 − tan 2 x
2 36 5. − log| cot x + cot 2 x − 1 | + log +c
1 2 2 − 1 − tan 2 x
24. ⋅(2 x 3m + 3 x 2m + 6 xm ) (m + 1)/m + c
6 (m + 1 ) 6. (1)
1
25. 2 [cos−1 x − log| 1 + 1 − x | − log | x| ] + c Topic 3
2
−1 1. (d) 2. (b) 3. (a) 4. (a)
26. −2 1 − x + cos x + x (1 − x ) + c
5. (b) 6. (c)
( x 4 + 1 )1/4
27. − +c  x + 2 3
2
x 7. ( x + 1 ) tan −1   − log( 4 x + 8 x + 13 ) + c
2
 3  4
x x
28. (i) 4 sin − 4 cos + c 1  cosθ + sin θ  1
4 4 8. sin 2 θ ln   + ln(cos2 θ ) + c
 2 1  2  cosθ − sin θ  2
(ii) − 2  1 − x − (1 − x ) 3/2 + (1 − x ) 5/2  + c
 3 5  9.
2
[ x − x 2 − (1 − 2 x ) sin −1 x ] − x + c
1  a2  π
29. a + bx − 2a log (a + bx ) − + c
b3  a + bx  ex cos2 x
10. + c 11. x sin x + cos x − +c
cos 4 x cos 2 x cos 6 x (x + 1)2 4
30. − − + + sin 2 x + tan x − 2 x
16 8 24
3 x sin 4 x sin 8 x Topic 4
+ − +
128 128 1024 1. (b) 2. (c)
1 −1 1 x 1 1 3 x
31. tan ( x ) + c2
32. log (sin x − cos x ) + + c 3. − log| x + 1 | + log| x 2 + 1 | + tan −1 x + 2 +c
2 2 2 2 4 2 x +1
Topic 2 xe x 1
4. log + +c
1. (b) 2. (b) 1 + xe x 1 + xe x

Hints & Solutions


Topic 1 Some Standard Results 7 5
t+ –
cos θ 1
= ×
1 4 4 + C'
1. Since, I = ∫ dθ
2 2×5
log e
7 5
5 + 7 sin θ – 2 cos 2 θ t+ +
cos θ 4 4 4
=∫ dθ
3 + 7 sin θ + 2 sin 2 θ 1 2t + 1
= log e + C'
Let, sin θ = t ⇒ cos θ dθ = dt 5 2t + 6
dt 1 dt 2 sin θ + 1
∴ I=∫ 2 = ∫ =
1
log e
1
+ C, where C = C′– log e 2
2t + 7t + 3 2 7
t2 + t +
3
5 sin θ + 3 5
2 2
1 dt 1 dt = A log e|B(θ )|+ C [given]
= ∫ 2
= ∫ 2 2 B(θ ) 5(2 sin θ + 1)
2  7  3 49 2  7  5 ∴ = .
t +  +  –  t +  –  A (sin θ + 3)
 4  2 16  4  4
Indefinite Integration 283

2. It is given that, Now, put x − α = t ⇒ dx = dt, so


x sin (t + 2α )
f (x) = ∫ dx, (x ≥ 0) I=∫ dt
(1 + x)2 sin t
Put x = tan 2 t ⇒ dx = 2 tan t sec2 tdt sin t cos 2 α + sin 2α cos t
=∫ dt
2 tan 2 t sec2 t sin t
∴ f (x) = ∫ dt = ∫ 2 sin 2 t dt
sec4 t  cos t 
= ∫  cos 2 α + sin 2 α  dt
= ∫ (1 − cos 2t )dt [Q2 sin 2 x = 1 − cos 2x]  sin t 
sin 2t = t (cos 2 α ) + (sin 2 α ) log e |sin t |+ C
=t− +C
2 = (x − α ) cos 2 α + (sin 2 α ) log e |sin (x − α )| + C
tan t x = A (x) cos 2 α + B(x) sin 2 α + C (given)
So, =t− + C = tan −1 x − +C
1 + tan t2
1+ x Now on comparing, we get
 3  A (x) = x − α and B(x) = log e |sin (x − α )|
∴f (3) − f (1) =  tan −1 3 − + C
 1+3 
5. Key Idea
 1 
−  tan −1 (1) − + C (i) Divide each term of numerator and denominator by x 2.
 1+1 
1
(ii) Let x 2 + = t
π 3 π 1 π 1 3
= − − + = + − x
3 4 4 2 12 2 4
2x3 − 1 2 x − 1 / x2
Hence, option (d) is correct. Let integral is I = ∫ dx = ∫ dx
x +x
4
x2 +
1

3. Given integral, I = ∫ x
cos 2 θ (tan 2 θ + sec 2 θ ) [dividing each term of numerator and
sec2 θ cos 2 θ
=∫ dθ denominator by x2]
(sin 2 θ + 1)
1   1 
sec2 θ (1 − tan 2 θ ) Put x2 + = t ⇒ 2x +  − 2  dx = dt
=∫ dθ x   x 
(2 tan θ + 1 + tan 2 θ )
dt
sec2 θ (1 − tan θ ) ∴ I=∫ = log e|(t )| + C
=∫ dθ t
1 + tan θ
 1
Let, tan θ = t = log e  x2 +  +C
 x
⇒ sec2 θdθ = dt
1 −t x3 + 1
Then, I=∫ dt = log e +C
1+t x
 2  dx dx
= ∫  −1 +  dt 6. Let I = ∫ =∫
 1 + t (x − 2x + 10)
2 2
((x − 1)2 + 32)2
= − t + 2 log e|1 + t| + C Now, put x − 1 = 3 tan θ ⇒ dx = 3 sec2θ dθ
= − tan θ + 2 log e |1 + tan θ| + C 3sec2θ dθ 3sec2θ dθ
So,I = ∫ =∫ 4
= λ tan θ + 2 log e| f (θ )| + C (given) (3 tan θ + 3 )
2 2 2 2
3 sec4θ
∴ (λ , f (θ )) = (−1, 1 + tan θ ) (on comparing) 1 1 1 + cos 2θ
27 ∫ 27 ∫
Hence, option (c) is correct. = cos 2θ dθ = dθ
2
tan x + tan α  π  1 + cos 2θ 
4. Let I=∫ dx, α ∈ 0,  
Q cos 2 θ =

tan x − tan α  2 2
sin x sin α 1 1  sin 2θ 
54 ∫
+ = (1 + cos 2θ ) dθ = θ +  +C
cos x cos α 54  2 
=∫ dx
sin x sin α
− 1  x − 1 1  2 tan θ 
cos x cos α = tan −1   +   +C
54  3  108  1 + tan 2 θ 
sin x cos α + sin α cos x
=∫ dx  2 tan θ 
sin x cos α − sin α cos x
Q sin 2θ = 
sin (x + α )  1 + tan 2 θ 
=∫ dx
sin (x − α )
284 Indefinite Integration

 x − 1 = ∫ [3 − 2(1 − cos 2x) + 2 cos x]dx


 
1  x − 1 1  3 
= tan −1   + +C [Q2 sin 2 x = 1 − cos 2x]
54  3  54  x − 1
2
1+   = ∫ [3 − 2 + 2 cos 2x + 2 cos x]dx
 3 
  = ∫ [1 + 2 cos 2x + 2 cos x]dx
1  x − 1 1 x−1
= tan −1   +   +C
54  3  18  (x − 1) + 3 
2 2
= x + 2 sin x + sin 2x + C
1  x − 1 1  x−1  9. Let
= tan −1   +  2  +C
54  3  18  x − 2x + 10 3 2
+ 5
3x13 + 2x11 3
1   x − 1 3(x − 1)  I=∫ dx = ∫ x x dx
= tan −1   + +C (2x4 + 3x2 + 1)4  3 1
4

54   3  x2 − 2x + 10  2 + 2 + 4 
 x x 
It is given, that [on dividing numerator and denominator by x16]
  x − 1 f (x)  3 1
I = A tan −1   + 2 +C Now, put 2 + 2 + 4 = t
  3  x − 2x + 10  x x
1 −6 4 
On comparing, we get A = and f (x) = 3(x − 1). ⇒  3 − 5  dx = dt
54 x x 
dx
7. Let I = ∫ ⇒
3 2
 3 + 5  dx = −
dt
x (1 + x6 )2/ 3
3
x x  2
=∫
dx
=∫
dx − dt 1 t− 4 + 1 1
2/ 3 2/ 3 So, I = ∫ =− × +C= 3 +C
1  1  2t 4
2 −4 + 1 6t
x3 ⋅ x4  6 + 1 x7  6 + 1
x  x  1  3 1
= +C Qt =2 + 2 + 4
1 3  
Now, put + 1 = t3  3 1  x x
x6 6 2 + 2 + 4 
 x x 
6 dx t2
⇒ − dx = 3t 2dt ⇒ 7 = − dt x12
x 7
x 2 = +C
6 (2x + 3x2 + 1)3
4
1 2
− t dt
1
So, I = ∫ 2 = − ∫ dt 10. We have,
t2 2 x+1
1 1 1 
1/3
 3 1 
∫ 2x − 1
dx = f (x) 2x − 1 + C ...(i)
=− t + C = −  6 + 1 + C Qt = 6 +1
2 2 x   x  x+1
Let I = ∫ dx
1 1 2x − 1
=− (1 + x6 )1/3 + C
2 x2 Put 2x − 1 = t 2
= x ⋅ f (x) ⋅ (1 + x6 )1/3 + C [given] ⇒ 2dx = 2tdt ⇒ dx = tdt
On comparing both sides, we get t +1
2
1 +1
f (x) = − 3 I=∫ 2
1
tdt = ∫ (t 2 + 3) dt
2x t 2
5x 5x x 
sin 2 sin cos t2 + 1 
Q 2x − 1 = t ⇒ x = 2 
2
8. Let I = ∫ 2 dx =

2 2 dx
 
x x x
sin 2 sin cos
2 2 2 1  t3  t 2
x =  + 3t + C = (t + 9) + C
[multiplying by 2 cos in numerator and 23  6
2 2x − 1
denominator] = (2x − 1 + 9) + C [Q t = 2x − 1 ]
sin 3x + sin 2x 6
=∫ dx 2x − 1
sin x = (2x + 8) + C
[Q2 sin A cos B = sin( A + B) + sin( A − B) and 6
sin 2 A = 2 sin A cos A] x+4
= 2x − 1 + C
(3 sin x − 4 sin3 x) + 2 sin x cos x 3
=∫ dx On comparing it with Eq. (i), we get
sin x
[Q sin 3x = 3 sin x − 4 sin3 x] x+4
f (x) =
= ∫ (3 − 4 sin x + 2 cos x)dx
2 3
Indefinite Integration 285

11. We have, 5 x8 + 7 x 6
13. We have, f (x) = ∫ dx
1 − x2 (x2 + 1 + 2x7 )2
∫ x4
dx = A (x) ( 1 − x2 )m + C … (i)
 x8   x6 
5 14  + 7 14 
1  x  x 
x2 2 − 1 =∫ dx
1 − x2 x   x2 1 2x7 
2
LetI = ∫ dx = ∫ dx  7 + 7 + 7
x4 x4 x x x 
1
x 2 −1 (dividing both numerator and denominator by x14)
x 1 1
=∫ dx = ∫ 3 − 1 dx 5 x − 6 + 7 x− 8
x4 x x2 = ∫ −5 dx
(x + x− 7 + 2)2
1 −2 1
Put 2 − 1 = t 2 ⇒ 3 dx = 2t dt ⇒ 3 dx = − t dt Let x− 5 + x − 7 + 2 = t
x x x
t3 ⇒ (− 5x− 6 − 7x− 8 )dx = dt
∴ I = − ∫ t dt = −
2
+C ⇒ (5x− 6 + 7x− 8 )dx = − dt
3
dt
1  1 − x2 
3/ 2
 1  
1/ 2 ∴ f (x) = ∫ − 2 = − ∫ t −2dt
=− .  + C Q t =  2 − 1  t
3  x2  x 
  t− 2 + 1 t− 1 1
=− + C =− +C= +C
1 1
=− ( 1 − x2 )3 + C …(ii) −2 + 1 −1 t
3 x3 1 x7
On comparing Eqs. (i) and (ii), we get = −5 −7
+C= 7 +C
x +x +2 2 x + x2 + 1
1
A (x) = − 3 and m = 3 Q f (0) = 0
3x 0
1 ∴ 0= + C ⇒C = 0
∴ ( A (x)) = ( A (x))3 = −
m 0+0+1
27 x9
x7
∴ f (x) =
(sin θ − sin θ ) cos θ
n 1/ n
2 x + x2 + 1
7
12. Let I = ∫ dθ
sin n + 1 θ 1 1
⇒ f (1) = =
Put sin θ = t ⇒ cos θ dθ = dt 2(1) + 1 + 1 4
7 2
1/ n
 n t 
t
   1 −  2 sin(x2 − 1) − sin 2(x2 − 1)
(t n − t )1/ n t n   14. Let I = ∫ x
dt = ∫ 
dx
∴ I=∫ n+1 n+1
dt 2 sin(x2 − 1) + sin 2(x2 − 1)
t t
t (1 − 1 / t n−1 )1/ n (1 − 1 / t n − 1 )1/ n x2 − 1
=∫ dt = ∫ dt Put =θ
n+1 2
t tn
1 ⇒ x2 − 1 = 2θ
Put 1 − n −1 = u
t ⇒ 2x dx = 2 dθ
(n − 1) ⇒ x dx = dθ
or 1 − t −( n − 1) = u ⇒ dt = du
tn 2 sin 2 θ − sin 4 θ
dt du Now, I = ∫ dθ
⇒ = 2 sin 2 θ + sin 4 θ
t n
n −1
2 sin 2 θ − 2 sin 2 θ cos 2 θ
1
+1 =∫ dθ
u1/ ndu un 2 sin 2 θ + 2 sin 2 θ cos 2 θ
⇒ I=∫ = +C
n −1 1  (Qsin 2 A = 2 sin A cos A)
(n − 1)  + 1
n  2 sin 2 θ (1 − cos 2 θ )
n+1 =∫ dθ
2 sin 2 θ (1 + cos 2 θ )
 1  n
n 1 − n−1 
 t  1 − cos 2 θ 2 sin 2 θ
= +C =∫ dθ = ∫ dθ
(n − 1) (n + 1) 1 + cos 2 θ 2 cos 2 θ
n+1
[Q1 − cos 2 A = 2 sin 2 A and 1 + cos 2 A = 2 cos 2 A]
 1  n
n 1 − 
 sin n − 1 θ  =∫ tan 2 θ d θ = ∫ tan θd θ
= +C
n2 − 1
 x2 − 1   x2 − 1 
 1  = log e|sec θ| + C = log e sec   +C Q θ = 2 
Q u = 1 − n − 1 and t = sin θ  2   
 t 
286 Indefinite Integration

15. We have, 1 1 dt
t +  ⋅
sin 2 x ⋅ cos 2 x 2 t t 1  1 1 
I=∫ dx ⇒ I=∫ = ∫  9/ 2 + 13/ 2 dt
(sin x + cos x ⋅ sin 2 x + sin3 x ⋅ cos 2 x + cos5 x)2
5 3
t 9/ 2 2 t t 

=∫
sin 2 x cos 2 x 1 2 2 
dx = −  7/ 2 + + K
{sin3 x(sin 2 x + cos 2 x) + cos3 x(sin 2 x + cos 2 x)}2 2 7 t 11 t11/ 2 
sin 2 x cos 2 x sin 2 x cos 2 x  
=∫ dx = ∫ dx = −
1
+
1
+K
(sin3 x + cos3 x)2 cos 6 x(1 + tan3 x)2 11/ 2 
 7 (sec x + tan x )7/ 2
11 (sec x + tan x) 
tan 2 x sec2 x
=∫ dx −1 1 1 
(1 + tan3 x)2 = 11/ 2 
+ (sec x + tan x)2 + K
(sec x + tan x)  11 7 
Put tan3 x = t ⇒ 3 tan 2 x sec2 xdx = dt
ex e3 x
1 dt 18. Since, I=∫ dx and J = ∫ dx
∴ I= ∫ e4x
+ e +1
2x
1 + e2x + e4x
3 (1 + t )2
−1 (e3 x − ex )
⇒ I= +C ∴ J −I=∫ dx
3 (1 + t ) 1 + e2x + e4x
−1 Put ex = u ⇒ ex dx = du
⇒ I= +C
3 (1 + tan3 x)  1
 1 − 2
(u 2 − 1)  u  du
dx dx ∴ J −I = ∫ du = ∫
16. ∫ x2(x4 + 1)3/ 4 = ∫  1
3/ 4 1+ u + u
2 4 1
1 + 2 + u2
x5 1 + 4  u
 x   1
1  1 − 2
Put 1+ = t4  u 
x4 =∫ 2
du
−4  1
⇒ dx = 4t3 dt u +  −1
 u
x5
1

dx
= − t3 dt Put u+ =t
x5 u
 1 
Hence, the integral becomes ⇒ 1 − 2 du = dt
 u 
1/ 4
− t3 dt  1
∫ t3
= − ∫ dt = − t + c = − 1 + 4 
 x 
+c
=∫
dt 1
= log
t −1
+c
t −1 2
2
t+1
17. PLAN Integration by Substitution
1 u2 − u + 1
i.e. I = ∫ f { g ( x )} ⋅ g ′ ( x )dx = log 2 +c
2 u +u+1
Put g ( x ) = t ⇒ g ′ ( x )dx = dt
∴ I = ∫ f(t )dt 1 e2x − ex + 1
= log 2x +c
Description of Situation Generally, students gets 2 e + ex + 1
confused after substitution, i.e. sec x + tan x = t. x
Now, for sec x, we should use 19. Given, f (x) = for n ≥ 2
(1 + xn )1/ n
sec2 x − tan 2 x = 1 f (x) x
∴ ff (x) = =
⇒ (sec x − tan x) (sec x + tan x) = 1 [1 + f (x) ]n 1/ n
(1 + 2 xn )1/ n
1 x
⇒ sec x − tan x = and fff (x) =
t (1 + 3xn )1/ n
sec2 dx x
Here, I=∫ ∴ g (x) = ( fofo ... of ) (x) =
(sec x + tan x)9/ 2 14243 (1 + n xn )1/ n
n times
Put sec x + tan x = t
xn − 1 dx
⇒ (sec x tan x + sec2 x) dx = dt Let I = ∫ xn − 2g (x) dx = ∫
(1 + nxn )1/ n
dt
⇒ sec x ⋅ t dx = dt ⇒ sec x dx = d
t (1 + nxn )
1 n 2 xn − 1 dx 1 dx
n2 ∫ (1 + nxn )1/ n n 2 ∫ (1 + nxn )1/ n
1 1 1 = = dx
∴ sec x − tan x = ⇒ sec x =  t + 
t 2 t 1
1−
sec x ⋅ sec x dx I=
1
(1 + nxn ) n + c
∴ I =∫
(sec x + tan x)9/ 2 n (n − 1)
Indefinite Integration 287

(x2 − 1) dx ⇒
1
f (x) =
f (x) + f ′ (x)
20. Let I = ∫
2x − 2x + 1
x3 4 2 2
1 f ′ (x) 1
[dividing numerator and enominator by x5 ] ⇒ f ′ (x) = f (x) ⇒ =
2 f (x) 2
1 1
 3 − 5  dx On integrating, we get
x x 
=∫ 1
2 1 log f (x) = x + C
2− 2+ 4 2
x x 1
x
2 1 ⇒ f (x) = Ae2 , where eC = A
Put 2− 2+ 4 =t
x x If f (0) = 1, then A = 1
4 4 1
⇒  3 − 5  dx = dt f (x) = e2
x
x x  Hence,
1 1
1 dt 1 t1/ 2 ⇒ log e f (x) = x ⇒ log e f (4) = × 4 = 2
∴ I=
4 ∫ = ⋅
t 4 1 /2
+c 2 2
4ex + 6e− x
1 2 1 23. Given, ∫ dx = Ax + B log (9e2x − 4) + c
= 2− 2+ 4 + c 9ex − 4e− x
2 x x
4e2x + 6
21. We have, f ′ (x) = e( f ( x ) − g( x )) g′ (x) ∀ x ∈ R LHS = ∫ dx
9e2x − 4
ef ( x )
⇒ f ′ (x) = g′ (x) Let 4e2x + 6 = A (9e2x − 4) + B (18 e2x )
eg( x )
f ′ (x) g′ (x) ⇒ 9 A + 18B = 4 and − 4 A = 6
⇒ = g( x ) 3 35
ef ( x ) e ⇒ A=− and B=
⇒ e− f ( x ) f ′ (x) = e− g( x ) g′ (x) 2 36
A (9e2x − 4) + B (18e2x ) 1
On integrating both side, we get ∴ ∫ 9e2x − 4
dx = A ∫ 1 dx + B ∫
t
dt
e− f ( x ) = e− g( x ) + C
At x = 1 where t = 9e2x − 4
e− f (1) = e− g(1) + C = A x + B log (9e2x − 4) + c
−1 − g(1 )
e =e +C [Q f (1) = 1] …(i) 3
=− x+
35
log (9e2x − 4) + c
At x = 2 2 36
e− f ( 2) = e− g( 2) + C ∴
3
A =− ,B=
35
⇒ e− f ( 2) = e−1 + C [Q g(2) = 1] …(ii) 2 36
From Eqs. (i) and (ii) and c = any real number
e− f ( 2) = 2e−1 − e− g(1) …(iii) 24. For any natural number m , the given integral can be
⇒ e− f ( 2) > 2e−1 written as
We know that, e− x is decreasing (2x3 m + 3x2m + 6xm )1/ m
I = ∫ (x3 m + x2m + xm ) dx
∴ − f (2) < log e 2 − 1 x
1/ m
f (2) > 1 − log e 2 ⇒ I = ∫ (2 x3 m + 3x2m + 6 xm )
⇒ e− g(1) + e− f (2) = 2e−1 [from Eq. (iii)] (x3 m − 1 + x2m − 1 + xm − 1 ) dx
− g(1 ) −1
⇒ e <2e Put 2 x3 m + 3x2m + 6xm = t
− g (1) < log e 2 − 1 ⇒ (6 mx3 m−1 + 6 mx2m−1 + 6 mxm −1 ) dx = dt
⇒ g (1) > 1 − log e 2 1
+1
dt 1 tm
22. Given, ∴ I=∫ t1/m = ⋅
6m 6m  1 
f (x + y) = f (x) f ′ ( y) + f ′ (x) f ( y), ∀x, y ∈ R  + 1
m 
and f (0) = 1
Put x = y = 0, we get =
1
⋅ (2 x3m + 3x2m + 6 xm )(m + 1)/m + c
f (0) = f (0) f ′ (0) + f ′ (0) f (0) 6 (m + 1)
1
⇒ 1 = 2 f ′ (0) ⇒ f′ (0) =  1 − x
1/ 2
dx
2 25. Let I=∫   ⋅
Put x = x and y = 0, we get  1 + x x
f (x) = f (x) f ′ (0) + f ′ (x) f (0) Put x = cos 2 θ ⇒ dx = − 2 cos θ sin θ dθ
288 Indefinite Integration
1/ 2
 1 − cos θ  − 2 cos θ ⋅ sin θ x2
∴ I=∫   ⋅ dθ (ii) Let I=∫ dx
 1 + cos θ  cos 2 θ 1−x
θ Put 1 − x = t 2 ⇒ − dx = 2 t dt
sin
− 2 sin θ (1 − t 2)2 ⋅ (−2t )
=∫ 2 ⋅ dθ ∴ I=∫
θ cos θ dt
cos t
2 = − 2 ∫ (1 − 2t + t ) dt
2 4
θ θ θ θ
2 sin ⋅ 2 sin ⋅ cos 2 sin 2  2t3 t5 
=−∫ = − 2 t − +  +c
2
θ
2 2 d θ −2
∫ cos θ 2 d θ  3 5
cos ⋅ cos θ
2  2 1 
= − 2  1 − x − (1 − x)3/ 2 + (1 − x)5/ 2 + c
1 − cos θ  3 5 
= −2∫ dθ
cos θ x2
29. Let I=
= 2∫ (1 − sec θ ) dθ = 2 [θ − log|sec θ + tan θ|] + c (a + bx)2
 1 1  Put a + bx = t ⇒ b dx = dt
⇒ I = 2 cos −1 x − log + −1 + c 2
 x x   t − a
 
 b  dt
 1  ∴ I=∫ ⋅
⇒ I = 2 cos −1 x − log|1 + 1 − x|− log|x| + c t2 b
 2 
1  t 2 − 2 at + a 2
= 3 ∫   dt
1− x b  t2 
26. Let I=∫ dx
1+ x
1  2 a a 2
Put x = cos 2 θ ⇒ dx = − 2sin θ cos θ dθ =
b3 ∫ 1 −
 t
+ 2  dt
t 
1 − cos θ
∴ I=∫ ⋅ (− 2 sin θ cos θ ) dθ 1  a 2
1 + cos θ =  t − 2 a log t −  + c
θ θ b 
3
t
= − ∫ 2 tan ⋅ sin θ cos θ dθ = − 2 ∫ 2 sin 2 ⋅ cos θ dθ
2 2 1  a2 
=  a + bx − 2 a log (a + bx) − + c
= − 2∫ (1 − cos θ ) cos θ dθ = − 2 ∫ ( cos θ − cos θ ) dθ 2
b3  a + bx 

= − 2∫ cos θ dθ + ∫ (1 + cos 2 θ ) dθ 30. Let I1 = ∫ sin x sin 2 x sin 3x dx


sin 2θ 1
4∫
= − 2 sin θ + θ + +c = (sin 4x + sin 2 x − sin 6x) dx
2
= − 2 1 − x + cos −1 x + x (1 − x) + c =−
cos 4x cos 2 x cos 6 x
− +
16 8 24
dx dx
27. Let I = ∫
x2 (x4 + 1)3/ 4 ∫
= 3/ 4 I 2 = ∫ sec2 x ⋅ cos 2 2 x dx
 1
x ⋅ x 1 + 4 
2 3
 x  = ∫ sec2 x (2 cos 2 x − 1)2dx
4
Put 1 + x− 4 = t ⇒ − dx = dt = ∫ (4 cos 2 x + sec2 x − 4) dx
x5
1 dt 1 t1/ 4  1
1/ 4
= ∫ (2 cos 2 x + sec2 x − 2) dx
∴ I=−
4 ∫ t3/ 4 = − 4 ⋅ 1 / 4 + c = − 1 + x4  +c
= sin 2 x + tan x − 2 x
(x4 + 1)1/ 4 and I3 = ∫ sin 4 x cos 4 x dx
=− +c
x
1
128 ∫
x = (3 − 4 cos 4x + cos 8x) dx
28. (i) Let I=∫ 1 + sin dx
2 3x sin 4x sin 8x
= − +
x x x x
=∫ + sin 2 + 2 sin cos dx
cos 2 128 128 1024
4 4 4 4 ∴ I = I1 + I 2 + I3
 x x cos 4x cos 2 x cos 6 x
= ∫  cos + sin  dx =− − + + sin 2 x + tan x − 2 x
 4 4 16 8 24
x x 3x sin 4x sin 8 x
= 4 sin − 4 cos + c + − +
4 4 128 128 1024
Indefinite Integration 289

I=∫
x dx 1 2x  ln (1 + 6 x )
1 + x4 2 ∫ 1 + (x2)2
= 1
31. Let dx 3. Let I = ∫ 3 +  dx
 x+ 4
x 3
x+ x 
Put x = u ⇒ 2 x dx = du
2
∴ I = I1 + I 2
1 du 1 1
∴ I= ∫ = tan −1 (u ) + c = tan −1 (x2) + c  1 
2 1+ u 2
2 2 where, I1 = ∫  3  dx,
 x+ 4
x
sin x
32. Let I = ∫ dx ln (1 + 6 x )
sin x − cos x I2 = ∫ dx
3
x+ x
Again, let sin x = A (cos x + sin x) + B(sin x − cos x),
then A + B = 1 and A − B = 0  1 
Now, I1 = ∫  3  dx
1 1  x+ 4
x
⇒ A= , B=
2 2 Put x = t12 ⇒ dx = 12 t11dt
1 1 t11
(cos x + sin x) + (sin x − cos x) ∴ I1 = 12 ∫ dt
∴ I=∫ 2 2 dx t 4 + t3
(sin x − cos x)
1 cos x + sin x t 8 dt
= ∫
1
dx + ∫ 1 dx + c = 12 ∫
2 sin x − cos x 2 t+1
1 1 = 12∫ (t7 − t 6 + t5 − t 4 + t3 − t 2 + t − 1) dt
= log (sin x − cos x) + x + c
2 2 dt
+ 12∫
t+1
Topic 2 Some Special Integrals  t8 t7 t 6 t5 t 4 t3 t 2 
= 12  − + − + − + − t
2 4
dx 8 7 6 5 4 3 2 
1. Let I = ∫ sec3 x cos ec3 x dx = ∫ 2 4
+ 12 ln (t + 1)
cos3 x sin3 x
dx  ln (1 + 6 x ) 
and I 2 = ∫
∫ 4
4 2
 3
 x+ x 
 dx
 sin x 3
  cos3 x cos3 x Put x = u6 ⇒ dx = 6 u5 du
 cos x
ln (1 + u ) 5 ln (1 + u )
[dividing and multiplying by cos 4/3 x in denominator] ∴ I2 = ∫ 6u du = ∫ 2 . 6 u5 du
u 2 + u3 u (1 + u )
dx sec2 x dx
=∫ =∫ u3
4 4 =6 ∫ ln (1 + u ) du
tan3 x cos 2 x (tan x)3 (u + 1)
Now, put tan x = t ⇒ sec x dx = dt 2  u3 − 1 + 1
=6 ∫   ln(1 + u ) du
−4
+1  u+1 
dt t3
∴I=∫ = +C  1 
t 4/3 −4 = 6 ∫  u2 − u + 1 −  ln (1 + u ) du
+1  u + 1
3
1 ln (1 + u )
1 −3 − = 6 ∫ (u 2 − u + 1) ln (1 + u ) du − 6 ∫ du
= −3 1
+C = 1
+ C = −3 tan 3 x+C II I (u + 1)
t3 (tan x)3  u3 u 2 
=6  − + u ln (1 + u )
2. We have, I n = ∫ tan n x dx  3 2 

∴ I n + I n + 2 = ∫ tan n x dx + n+ 2 2 u3 − 3u 2 + 6 u
∫ tan x dx −∫
1
du − 6 [ln (1 + u )]2
u+1 2
= ∫ tan n x(1 + tan 2 x) dx
= (2 u3 − 3 u 2 + 6 u ) ln (1 + u )
n+1
= ∫ tan n x sec2 x dx =
tan x
+C  11 u 
n+1 −∫ 2 u 2 − 5 u +  du − 3 [ln (1 + u )]
2
 u + 1
tan5 x = (2 u3 − 3 u 2 + 6 u ) ln (1 + u )
Put n = 4, we get I 4 + I 6 = +C
5  2 u3 5 2 
1 −  − u + 11u − 11 ln (u + 1) − 3 [ln (1 + u )]2
∴ a = and b = 0  3 2 
5
290 Indefinite Integration

3 23 12 7/12 12 5/12 cos θ


∴ I= x/ − x + 2x1/ 2 − x + 3x1/3 − 4x1/ 4 = − ∫ secθ dθ + 2 ∫ dθ
2 7 5 1 + cos 2 θ
− 6 x1/ 6 − 12 x1/12 + 12 ln (x1/12 + 1) cos θ
= − log|sec θ + tan θ | + 2 ∫ dθ
+ (2 x1/ 2 − 3x1/3 + 6 x1/ 6 ) ln (1 + x1/ 6 ) 2 − sin 2 θ
2 1/ 2 5 1/3  dt
− x − x 11 x1/ 6 − 11 ln (1 + x1/ 6 )
3 2 
= − log|sec θ + tan θ | + ∫ 2 − t 2 , where sin θ = t
−3 [ln (1 + x1/ 6 )]2 + c 1  2 + sin θ 
3 12 7/12 4 1/ 2 12 5/12 = − log|sec θ + tan θ |+ 2 ⋅ log  + c
= x2 / 3 − x + x − x 2 2  2 − sin θ 
2 7 3 5
1 = − log|cot x + cot2 x − 1 |
+ x1/3 − 4x1/ 4 − 7x1/ 6 − 12 x1/12
2  2 + 1 − tan 2 x 
1
+ (2 x1/ 2 − 3x1/3 + 6 x1/ 6 + 11) ln (1 + x1/ 6 ) + log  + c
 2 − 1 − tan x 
2 2
+ 12 ln (1 + x1/12) − 3 [ln (1 + x1/ 6 )]2 + c
2 sin x − sin 2 x
I = ∫ ( tan x + cot x ) dx = ∫
tan x + 1 6. Given, f (x) = ∫   dx
4. Let dx  3
x 
tan x
Put tan x = t 2 ⇒ sec2x dx = 2t dt On differentiating w.r.t. x, we get
2 sin x − sin 2 x 2 sin x  1 − cos x
⇒ dx =
2t f ′ (x) = =  
1 + t4
dt x3 x  x2 
 x
t2 + 1 t2 + 1 2 sin 2 
 sin x 
2t
∴ I=∫ ⋅ dt = 2 ∫ t 4 + 1 dt lim f ′ (x) = lim 2  2
t2 t4 + 1  
x→ 0 x→ 0  x   x2 
1 1  
1+ 1+
t2 t2 
=2 ∫
1
dt = 2 ∫ 2
dt x 
t2 + −2 + 2  1  sin 2 
t −  + ( 2)
2
t2  t = 4 ⋅ 1 ⋅ lim  2 =1
x→ 0   x 
2

1 1 4 ×   
Put t− = u ⇒ 1 + dt = du  2 
t t2
du
∴ I =2 ∫ Topic 3 Integration by Parts
u 2 + ( 2 )2
2
2  u 1. Let given integral, I = ∫ x5 e− x dx
⇒ I= tan −1   + c
2  2
Put x2 = t ⇒ 2xdx = dt
 tan x − cot x  1
= 2 tan −1   +c So, I = ∫ t 2e− t dt
 2  2
1
= [(− t 2e− t ) + ∫ e− t (2t ) dt ] [Integration by parts]
cos 2 x cos 2 x − sin 2 x 2
5. Let I = ∫ dx = ∫ dx
1
sin x sin 2 x = [− t 2e− t + 2t (− e− t ) + ∫ 2e− t dt ]
2
=∫ cot2 x − 1 dx
1
= [− t 2e− t − 2te− t − 2e− t ] + C
Put cot x = sec θ ⇒ − cosec2x dx = sec θ tan θ dθ 2
sec θ ⋅ tan θ e− t 2
∴ I=∫ sec2 θ − 1 ⋅ dθ =− (t + 2t + 2) + C
− (1 + sec2 θ ) 2
2
sec θ ⋅ tan 2 θ e− x
=−∫ dθ =− (x4 + 2x2 + 2) + C [Q t = x2] …(i)
1 + sec2 θ 2
Q It is given that,
sin 2 θ
=−∫ dθ 2 2
I = ∫ x5 e− x dx = g (x) ⋅ e− x + C
cos θ + cos3 θ
1 − cos 2 θ By Eq. (i), comparing both sides, we get
=−∫ dθ 1
cos θ (1 + cos 2 θ ) g (x) = − (x4 + 2x2 + 2)
2
(1 + cos 2 θ ) − 2 cos 2 θ 1 5
=−∫ dθ So, g(− 1) = − (1 + 2 + 2) = −
cos θ (1 + cos 2 θ ) 2 2
Indefinite Integration 291

2. Given, ∫ esec x [(sec x tan x) f (x) + (sec x tan x + sec2 x)]dx


1 1 1
x+ x+ x+
=∫e x dx + xe x −∫e x dx
= e sec x ⋅ f (x) + C
  1 x+
1
x+ 
1
On differentiating both sides w.r.t. x, we get Q ∫ 1 − 2 e x dx = e x 
esec x [(sec x tan x) f (x) + (sec x tan x + sec2 x)]   x  
= esec x f ′ (x) + esec x (sec x tan x) f (x) x+
1
x+
1
x+
1
x+
1

⇒ esec x (sec x tan x + sec2 x) = esec x f ′ (x) =∫e x dx + xe x − ∫ ex x dx = xe x +c


⇒ f ′ (x) = sec x tan x + sec2 x
6. Given, ∫ f ( x ) dx = ψ( x )
So, f (x) = ∫ f ′ (x)dx = ∫ (sec x tan x + sec2 x)dx
Let I = ∫ x5 f (x3 ) dx
= sec x + tan x + C
So, possible value of f (x) from options, is Put x3 = t
1 dt
f (x) = sec x + tan x + . ⇒ x2dx = …(i)
2 3
3. Let I = ∫ cos(log e x)dx 1
3∫
∴ I= t f (t ) dt
1
= x cos(log e x) − ∫ x(− sin(log e x)) ⋅ dx 1 d  
x = t ⋅ ∫ f (t ) dt − ∫  (t ) ∫ f (t ) dt  dt 
[using integration by parts]
3  dt  
= x cos(log e x) + ∫ sin(log e x) dx [integration by parts]
1
1 = [t ψ (t ) − ∫ ψ (t ) dt ]
= x cos(log e x ) + x sin(log e x ) − ∫ x(cos(log e x)) x dx 3
1 3
[again, using integration by parts] = [x ψ (x3 ) – 3 ∫ x2ψ (x3 ) dx] + c [from Eq. (i)]
3
⇒ I = x cos(log e x) + x sin((log e x) − I
1

x
I = [cos(log e x) + sin(log e x)] + C. = x3 ψ (x3 ) − ∫ x2ψ (x3 ) dx + c
2 3

5 −4 x 3 1 −4 x 3  2x + 2 
4. Given, ∫ x e dx = e f (x) + C 7. Let I = ∫ sin −1   dx
48  4x2 + 8 x + 13 
 
In LHS, put x3 = t
 
2x + 2
⇒ 3x2dx = dt −1   dx
1
= ∫ sin  (2x + 2)2 + 9 
 
3
So, ∫ x5 e−4x dx = ∫ t e−4t dt
3
Put 2x + 2 = 3 tan θ ⇒ 2 dx = 3 sec2θ dθ
1  e−4t e−4t 
= t −∫  3 tan θ  3
dt 
3  −4 −4  ∴ I = ∫ sin −1  ⋅ sec2 θ dθ
 9 tan 2 θ + 9  2
[using integration by parts]  
1  te−4t e−4t   3 tan θ  3 2
=  + +C = ∫ sin −1   ⋅ sec θ dθ
3  −4 −16   3 secθ  2
1 −4 t  sin θ  3 2
=− e [4t + 1] + C = ∫ sin −1   ⋅ sec θ dθ
48  cos θ ⋅ sec θ  2
3
e−4x 3
sin −1 (sin θ ) ⋅ sec2θ dθ
2∫
=− [4x3 + 1] + C [Q t = x3 ] =
48
3 3
∴ f (x) = −1 − 4x3 (comparing with given equation) = ∫ θ ⋅ sec2θ dθ = [θ ⋅ tan θ − ∫ 1 ⋅ tan θdθ]
2 2
1
 1 x+ 3
5. ∫ 1 + x − x e x dx
= [θ tan θ − log sec θ ] + c
2
x+
1
 1 x+
1
3  2 x + 2  2 x + 2
=∫e = tan −1   ⋅ 
x dx + ∫ x 1 − 2 e x dx
 x  2  3   3 
1 1 1 2 
x+ x+ d x+  2 x + 2
=∫e x dx + xe x −∫ (x)e x dx − log 1 +    + c1
dx  3  

292 Indefinite Integration

 2x + 2 3   2 x + 2 
2 θ 1
= (x + 1) tan −1   − log 1 +    + c1 =− cos 2θ + sin 2θ
 3  4  3  2 4
  1 1
 2 x + 2 3 = − θ (1 − 2 sin 2 θ ) + sin θ 1 − sin 2 θ
= (x + 1) tan −1   − log (4x + 8 x + 13) + c
2
2 2
 3  4
1 −1 1
 3  = − sin x (1 − 2 x) + x 1−x …(ii)
let log 3 + c1 = c 2 2
 2 
From Eqs. (i) and (ii),
 cos θ + sin θ  4  1 1 
8. I = ∫ cos 2 θ ln   dθ [given] I= − (1 − 2 x) sin −1 x + x − x2 − x + c
 cos θ − sin θ  π  2 2 
We integrate it by taking parts 2
= [ x − x2 − (1 − 2 x) sin −1 x ] − x + c
 cos θ + sin θ  π
ln   as first function
 cos θ − sin θ  (x − 1) ex
10. Let I = ∫ dx
sin 2 θ  cos θ + sin θ  (x + 1)3
= ln  
2  cos θ − sin θ  x + 1 − 2 x  1 2  x
I=∫ 3 
e dx = ∫  − 3
e dx
 ( x + 1 )   ( x + 1 ) 2
(x + 1 ) 
1 d   cos θ + sin θ  
2 ∫ dθ   cos θ − sin θ  
− ln   sin 2 θdθ …(i) 1 1
= ∫ ex ⋅ dx − 2 ∫ ex ⋅ dx
(x + 1)2 (x + 1)3
d   cos θ + sin θ  
But ln  
dθ   cos θ − sin θ  
Applying integration by parts,
 1 −2 
d = ⋅ ex − ∫ ex ⋅ dx
= [ln (cos θ + sin θ ) − ln (cos θ − sin θ )]  (x + 1 ) 2
(x + 1 )3


1 (− sin θ − cos θ ) 1 ex
= . (− sin θ + cos θ ) − − 2 ∫ ex ⋅ dx = +c
(cos θ + sin θ ) cos θ − sin θ (x + 1)3 (x + 1)2
(cos θ − sin θ ) (cos θ − sin θ ) − (cos θ + sin θ )
(− sin θ − cos θ ) 11. Let I = ∫ (elog x + sin x) cos x dx
=
(cos θ + sin θ ) (cos θ − sin θ ) = ∫ (x + sin x) cos x dx
cos θ − cos θ sin θ − sin θ cos θ + sin θ + cos θ sin θ
2 2
1
+ cos 2 θ + sin 2 θ + cos θ ⋅ sin θ = ∫ x cos x dx + ∫ (sin 2x) dx
= 2
cos 2 θ − sin 2 θ cos 2 x
= (x ⋅ sin x − ∫ 1 ⋅ sin x dx) − +c
2 (cos θ + sin θ )
2 2
2 4
= =
cos 2 θ cos 2θ = x sin x + cos x −
cos 2 x
+c
4
Therefore, on putting this value in Eq.(i), we get
1  cos θ + sin θ  1 2
 cos θ − sin θ  2 ∫
I = sin 2θ ln   − sin 2 θ dθ Topic 4 Integration, Irrational Function
2 cos 2 θ
and Partial Fraction
1  cos θ + sin θ  1
= sin 2 θ ln   + ln (cos 2 θ ) + c 2x12 + 5x9 2x12 + 5x9
 cos θ − sin θ  2 1. Let I = ∫
2
(x5 + x3 + 1) 3
dx = ∫ x15 (1 + x− 2 + x− 5 ) 3 dx
sin −1 x − cos −1 x
9. Let I = ∫ dx 2x− 3 + 5x− 6
sin −1 x + cos −1 x =∫ dx
(1 + x− 2 + x− 5 ) 3
π 
sin −1 x −  − sin −1 x Now, put 1 + x− 2 + x− 5 = t
2 
=∫ dx
π ⇒ (− 2x− 3 − 5x− 6 ) dx = dt
2 ⇒ (2x− 3 + 5x− 6 ) dx = − dt
2  −1 π 4 dt
= ∫ 2 sin x −  dx = ∫ sin −1 x dx − x + c …(i) ∴ I = − ∫ 3 = − ∫ t − 3 dt
π  2 π t
Now, ∫ sin −1 x dx t− 3 + 1 1
=− +C= 2+C
−3 + 1 2t
Put x = sin 2 θ ⇒ dx = sin 2θ
θ cos 2 θ 1 x10
= ∫ θ ⋅ sin 2 θ dθ = − = +C
2
+ ∫ 2
cos 2θ dθ
2 (x + x3 + 1) 2
5
Indefinite Integration 293

cos3 x + cos5 x On putting x = 0, we get


2. Let I=∫ dx
sin 2 x + sin 4 x 0=B+C
(cos 2 x + cos 4 x) ⋅ cos x dx ⇒ B = − C = 1 /2
=∫
(sin 2 x + sin 4 x) x3 + 3x + 2 x+1 1 2
= − +
Put sin x = t ⇒ cos x dx = dt (x + 1)2 (x + 1) 2 (x2 + 1) 2 (x + 1) (x2 + 1)2
2

[(1 − t 2) + (1 − t 2)2] x3 + 3x + 2
∴ I=∫ dt ∴ I=∫ dx
t2 + t4 (x + 1)2 (x + 1)
2

1 − t 2 + 1 − 2t 2 + t 4 1 dx 1 x+1 dx
⇒ I=∫ dt =−
2 ∫ x + 1 + 2 ∫ x2 + 1 dx + 2 ∫ (x2 + 1)2
t2 + t4
1 1
2 − 3t 2 + t 4 ⇒I = − log|x + 1| + log|x2 + 1|
⇒ I=∫ dt …(i) 2 4
t 2 (t 2 + 1)
1
Using partial fraction for + tan −1 x + 2 I1 …(i)
2
y2 − 3 y + 2 A B dx
=1 + + [where, y = t 2] where, I1 = ∫
y ( y + 1) y y+1 (x2 + 1)2
⇒ A = 2, B = − 6 Put x = tan θ
y − 3y + 2
2
2 6 ⇒ dx = sec2θ dθ
∴ =1 + −
y ( y + 1) y y+1 sec2θdθ 1
∴ I1 = ∫ = ∫ cos 2 θ dθ = ∫ (1 + cos 2 θ )dθ
 2 6  (tan θ + 1)
2 2
2
Now, Eq. (i) reduces to, I = ∫ 1 + 2 −  dt
 t 1 + t 2 1 1 
= θ + sin 2θ
2 2  2 
= t − − 6 tan −1 (t ) + c
t 1 1 tan θ
= θ+ ⋅
2 2 2 (1 + tan 2 θ)
= sin x − − 6 tan −1 (sin x) + c
sin x 1 1 x
= tan −1 x + ⋅
x3 + 3x + 2 x3 + 2x + x + 2 2 2 (1 + x2)
3. = 2
(x + 1) (x + 1) (x + 1)2(x + 1)
2 2
From Eq. (i),
x(x + 1) + 2(x + 1)
2 1 1 3
I = − log|x + 1| + log|x2 + 1|+ tan −1 x + 2
x
+c
=
(x2 + 1)2(x + 1) 2 4 2 x +1

x 2 (x + 1) ex (x + 1)
= + 2 4. Let I=∫ dx = ∫ dx
(x + 1)(x + 1) (x + 1)2
2 x (1 + xe )
x 2
x ex (1 + xex )2
x Ax + B C Put 1 + xex = t ⇒ (ex + x ex ) dx = dt
Again, = +
(x + 1) (x + 1) (x2 + 1) (x + 1)
2
dt  1 1 1
I=∫
(t − 1)t 2 ∫  t − 1 t t 2 
∴ = − − dt
⇒ x = ( Ax + B) (x + 1) + C (x2 + 1)
On putting x = − 1, we get 1
= log|t − 1| − log|t| + + c
t
−1 = 2 C ⇒ C = − 1 / 2
 t − 1 1
On equating coefficients of x2, we get = log + +c
 t  t
0= A+C
 xex  1
⇒ A = − C = 1 /2 = log  + +c
1 + xe  1 + xe
x x
12
Definite Integration
Topic 1 Properties of Definite Integral
Objective Questions I (Only one correct option) π/ 2 sin3 x
8. The value of ∫ dx is
π 0 sin x + cos x
1. ∫−π|π −|x||dx is equal to (2020 Main, 3 Sep I) π −1 π−2 π −1
(2019 Main, 9 April I)
π−2
(a) (b) (c) (d)
(a) 2π 2 (b) 2 π 2 2 8 4 4
π2 x
(c) π 2 (d)
2 9. Let f (x) = ∫ g (t )dt, where g is a non-zero even function.
0
2. The integral x
π /3
If f (x + 5) = g (x), then ∫ f (t )dt equals
∫π / 6 tan x ⋅ sin 3x(2 sec x ⋅ sin 3x
3 2 2 2
0 (2019 Main, 8 April II)
+ 3 tan x ⋅ sin 6x) dx is equal to 5 x+5 x+5 5

(2020 Main, 4 Sep II)


(a) 5 ∫ g (t )dt (b) ∫ g (t )dt (c) 2 ∫ g (t )dt (d) ∫ g (t )dt
1 7 1 9 x+5 5 5 x+5
(a) − (b) (c) − (d)
9 18 18 2 2 − x cos x
10. If f (x) = and g (x) = log e x, (x > 0) then the
3. A value of α such that 2 + x cos x
π/ 4
α+1
dx  9 value of the integral ∫ g ( f (x))dx is

−π/ 4
= log e   is
(x + α ) (x + α + 1)  8 (2019 Main, 8 April I)
α (2019 Main, 12 April II)
1 1 (a) log e 3 (b) log e e
(a) − 2 (b) (c) − (d) 2 (c) log e 2 (d) log e 1
2 2
π /2 cot x 11. Let f and g be continuous functions on
4. If ∫ dx = m(π + n ), then m ⋅ n is equal to [0, a] such that f (x) = f (a − x) and g (x) + g (a − x) = 4,
0 cot x + cosec x (2019 Main, 12 April I) a
1 then ∫ f (x) g (x) dx is equal to (2019 Main, 12 Jan I)
(a) − (b) 1 0
2 a a
1 (a) 4∫ f (x) dx (b) ∫ f (x) dx
(c) (d) −1 0 0
2 a a
(c) 2∫ f (x) dx (d) − 3∫ f (x) dx
π /3 0 0
5. The integral ∫ sec 2/ 3 x cosec4/3 x dx is equal to
π /6 e  e 
2x x
12. The integral ∫  
x
(2019 Main, 10 April II) −    log e x dx is
1  e  x
(a) 35 / 6 − 32/ 3 (b) 37/ 6 − 35 / 6  
(c) 35 /3 − 31/3 (d) 34/3 − 31/3 equal to (2019 Main, 12 Jan II)
2π 3 1 1 1 1
(a) −e− 2 (b) − + −
6. The value of ∫ [sin 2x (1 + cos 3x)] dx, where [t ] denotes 2 2e 2 e 2e2
0 1 1 3 1 1
(c) −e− 2 (d) − −
the greatest integer function, is (2019 Main, 10 April I) 2 e 2 e 2e2
(a) − π (b) 2π (c) − 2π (d) π π /4 dx
1 13. The integral ∫ equals
7. The value of the integral ∫ x cot (1 − x + x )dx is −1 2 4 π /6 sin 2x(tan5 x + cot5 x)
0 (2019 Main, 11 Jan II)
1 π 1 
(a)  − tan − 1  tan − 1 
(2019 Main, 9 April II) 1 1 
π 1 π 1  (b) 
(a) − log e 2 (b) − log e 2 
5 4  3 3 20  9 3
4 2 2 2
π π 1π −1 1  π
(c) − log e 2 (d) − log e 2 (c)  − tan   (d)
4 2 10  4  9 3 40
Definite Integration 295

2 sin 2 x π /2  2 π − x
14. The value of the integral ∫ dx 24. The value of the integral ∫  x + log  cos x dx
−2  x  1 − π /2  π + x
+
 π  2 is (2012)
(where, [x] denotes the greatest integer less than or equal π2 π2 π2
(a) 0 (b) −4 (c) + 4 (d)
to x) is (2019 Main, 11 Jan I) 2 2 2
(a) 4 − sin 4 (b) 4 log 3 x sin x2
(c) sin 4 (d) 0 25. The value of ∫ log 2 sin x2 + sin (log 6 − x2 )
dx is (2011)
b
15. Let I = ∫ (x4 − 2x2) dx. If I is minimum, then the ordered (a)
1
log
3
(b)
1
log
3
(c) log
3
(d)
1
log
3
a 4 2 2 2 2 6 2
pair (a , b) is (2019 Main, 10 Jan I)
26. The value of
(a) (− 2 , 0) (b) (0, 2 ) 0
∫ −2 [x + 3x2 + 3x + 3 + (x + 1) cos (x + 1)] dx is (2005, 1M)
3
(c) ( 2 , − 2 ) (d) (− 2 , 2 )
π /3 tan θ 1
16. If ∫ dθ = 1 − , (k > 0), then the value of k (a) 0 (b) 3 (c) 4 (d) 1
0 2k sec θ 2
1 1−x
is (2019 Main, 9 Jan II) 27. The value of the integral ∫ dx is (2004, 1M)
1 0 1+ x
(a) 1 (b)
2 π π
(a) +1 (b) −1 (c) −1 (d) 1
(c) 2 (d) 4 2 2
π
17. The value of ∫ |cos x|3 dx is 1/ 2   1 + x 
0 (2019 Main, 9 Jan I) 28. The integral ∫  [x] + log    dx equals (2002, 1M)
−1/ 2   1 − x 
2 4 4
(a) (b) − (c) 0 (d)
(d) log  
1 1
3 3 3 (a) − (b) 0 (c) 1
2  2
π /2 sin 2 x
18. The value of ∫ dx is cos 2 x
π
−π / 2 1 + 2x (2018 Main) 29. The value of ∫ dx, a > 0,is (2001, 1M)
−π 1 + a x
π π π
(a) (b) (c) 4π (d)
8 2 4 (a) π (b) aπ
3 π/ 4 dx π
(c) (d) 2π
19. ∫π / 4 1 + cos x
is equal to
(2017 Main)
2
ecos x sin x , for | x|≤ 2
(a) − 2 (b) 2 (c) 4 (d) − 1 30. If f (x) = 
π /2x2 cos x  2 , otherwise
20. The value of ∫ dxis equal to 3
−π / 2 1 + ex
(2016 Adv.) then ∫ f (x) dx is equal to (2000, 2M)
−2
π 2
π 2
(a) −2 (b) + 2 (a) 0 (b) 1 (c) 2 (d) 3
4 4
log x e2
(c) π 2 − e− π / 2 (d) π 2 + eπ / 2 31. The value of the integral ∫ −1  e  dx is (2000, 2M)
e  x 
4 log x2
21. The integral ∫ dx is equal to (a) 3 / 2 (b) 5 / 2 (c) 3 (d) 5
2 log x2 + log(36 − 12x + x2) (2015, Main)
(a) 2 (b) 4 (c) 1 (d) 6 32. If for a real number y, [ y] is the greatest integer less
π /2 than or equal to y, then the value of the integral
22. The integral ∫ (2 cosec x)17dx is equal to 3π/2
π /4 (2014 Adv.) ∫ π / 2 [2 sin x] dx is (1999, 2M)
log( 1 + 2) −u 16
∫0 2(e + e ) du
u
(a) (a) − π (b) 0
π π
log( 1 + 2) (c) − (d)
(b) ∫0 (eu + e−u )17 du 2 2
log( 1 + 2) 3 π /4 dx
(c) ∫0 (eu − e−u )17 du 33. ∫ π /4 1 + cos x
is equal to
(1999, 2M)
log( 1 + 2) −u 16
∫0 2(e − e ) du 1 1
u
(d) (a) 2 (b) −2 (c) (d) −
2 2
π
x x 34. Let f (x) = x − [x] , for every real number x, where [x] is
23. The integral ∫ 1 + 4 sin 2 − 4 sin dx is equal to
2 2 1
0

(2014 Main) the integral part of x. Then, ∫ −1 f (x) dx is (1998, 2M)
(a) π − 4 (b) − 4− 4 3
3 (a) 1 (b) 2
(c) 4 3 − 4 (d) 4 3 − 4 − π /3 1
(c) 0 (d) −
2
296 Definite Integration
x
35. If g (x) = ∫ cos 4 t dt , then g (x + π ) equals (1997, 2M) (a) Statement I is correct; Statement II is correct;
0 Statement II is a correct explanation for Statement I
(a) g (x) + g ( π ) (b) g (x) − g ( π ) (b) Statement I is correct; Statement II is correct;
g (x ) Statement II is not a correct explanation for
(c) g (x) g ( π ) (d)
g( π) Statement I
(c) Statement I is correct; Statement II is false
36. Let f be a positive function.
k
(d) Statement I is incorrect; Statement II is correct
If I1 = ∫ x f [x (1 − x)] dx and
1−k
k
Passage Based Questions
I2 = ∫ f [x (1 − x)] dx, where 2k − 1 > 0.
1−k Passage I
I Let F : R → R be a thrice differentiable function. Suppose that
Then, 1 is (1997C, 2M) F (1) = 0, F (3) = − 4 and F ′ (x) < 0 for all x ∈ (1, 3). Let
I2
f (x) = xF (x) for all x ∈ R. (2015 Adv.)
(a) 2 (b) k
(c) 1/2 (d) 1 44. The correct statement(s) is/are
2π (a) f ′ (1) < 0 (b) f (2) < 0
37. The value of ∫0 [ 2 sin x ] dx, where [.] represents the (c) f ′ (x) =/ 0 for any x ∈ (1, 3) (d) f ′ (x) = 0 for some x ∈ (1, 3)
greatest integral functions, is (1995, 2M) 3 3 3
5π 5π
45. If ∫ x2 F ′ (x) dx = − 12 and ∫1 x F ′ ′ (x) dx = 40, then the
(a) − (b) − π (c) (d) −2π 1
3 3 correct expression(s) is/are
3
πx
38. If f (x) = A sin   + B, (a) 9f ′ (3) + f ′ (1) − 32 = 0 (b) ∫ f (x) dx = 12
1
 2 
3
(c) 9f ′ (3) – f ′ (1) + 32 = 0 (d) ∫ f (x) dx = − 12
f ′   = 2 and ∫ f (x) dx =
1 1 2A
, then constants 1
 2 0 π
A and B are (1995, 2M) Passage II
π π 2 3 4 4 For every function f (x) which is twice differentiable, these
(a) and (b) and (c) 0 and − (d) and 0 will be good approximation of
2 2 π π π π
π /2 dx b  b − a
39. The value of ∫
0 1 + tan3 x
is (1993, 1M) ∫ a f (x) dx =  2 
 { f (a ) + f (b)},

(a) 0 (b) 1 for more acurate results for c ∈ (a , b),


(c) π / 2 (d) π / 4 c−a b−c
F (c) = [ f (a ) − f (c)] + [ f (b) − f (c)]
40. Let f : R → R and g : R → R be continuous functions. 2 2
Then, the value of the integral a+b
π/ 2 When c =
∫ −π / 2 [ f (x) + f (− x)] [ g (x) − g (− x)] dx is (1990, 2M)
b
2
b−a
(a) π (b) 1 (c) −1 (d) 0
∫ a f (x) dx = 4 { f (a ) + f (b) + 2 f (c)} dx (2006, 6M)

41. For any integer n, the integral t (t − a )


π ∫ a f (x) dx − 2
{ f (t ) + f (a )}
cos 2 x = 0,
∫0 e cos (2n + 1) x dx has the value 46. If lim
3
(1985, 2M)
t→ a (t − a )3
(a) π (b) 1 then degree of polynomial function f (x) atmost is
(c) 0 (d) None of these (a) 0 (b) 1
π /2 cot x (c) 3 (d) 2
42. The value of the integral ∫ dx is
0 cot x + tan x 47. If f ′ ′ (x) < 0, ∀x ∈ (a , b), and (c, f (c)) is point of maxima,
(a) π / 4 (b) π / 2 (1983, 1M) where c ∈ (a , b), then f ′ (c) is
(c) π (d) None of these
f (b) − f (a ) f (b) − f (a ) 
(a) (b) 3 
b− a  b − a 
Assertion and Reason  f (b) − f (a ) 
(c) 2 (d) 0
43. Statement I The value of the integral (2013 Main)  b − a 
π /3 dx π /2
∫π / 6 1+ tan x
is equal to π /6 . 48. Good approximation of ∫
0
sin x dx, is
b b (a) π / 4 (b) π ( 2 + 1) / 4
Statement II ∫a f (x) dx = ∫a f (a + b − x) dx (c) π ( 2 + 1) / 8 (d)
π
8
Definite Integration 297

Objective Questions II Integer & Numerical Answer Type Questions


(One or more than one correct option) 56. Let [t ] denote the greatest integer less than or equal to t.
2
49. Let f : R → (0, 1) be a continuous function. Then, which Then the value of ∫ |2x − [3x]|dx is ……… .
1
of the following function(s) has (have) the value zero at (2020 Main, 2 Sep II)
some point in the interval (0, 1) ? (2017 Adv.)
π 57. The value of the integral
x 2 π /2
(a) ex − ∫ f (t ) sin t dt 3 cos θ
(b) f (x) + ∫ f (t ) sin t dt ∫ ( cos θ + sin θ )5
d θ equals ………… (2019 Adv.)
0 0 0
π
−x
2 π/ 4
2 dx
(c) x − ∫ f (t ) cos t dt (d) x9 − f (x) 58. If I =
π ∫ then 27I 2 equals .........
0 − π/ 4
(1 + e sin x
) (2 − cos 2x)
(2019 Adv.)
k+1 k+1
50. If I = ∑ k = 1
98
∫k x(x + 1)
dx , then 1/ 2 1+ 3
(2017 Adv.) 59. The value of the integral ∫ dx is
0 ((x + 1)2 (1 − x)6 )1/ 4
(a) I > log e 99 (b) I < log e 99
49 49 ....... (2018 Adv.)
(c) I < (d) I >
50 50 [x], x ≤ 2
60. Let f : R → R be a function defined by f (x) =  ,
51. Let f (x) = 7 tan 8 x + 7 tan 6 x – 3 tan 4 x – 3 tan 2 x for all 0 , x > 2
 π π where [x] denotes the greatest integer less than or equal
x ∈  – ,  . Then, the correct expression(s) is/are
 2 2 2 xf (x2)
π/4 π/ 4
to x. If I = ∫ dx, then the value of (4I − 1)
1 −1 2 + f (x + 1 )
(a) ∫ x f (x) dx = (b) ∫ f (x) dx = 0
0 12 0 (2015 Adv.) is (2015 Adv.)
π/4 1 π/ 4
(c) ∫ x f (x) dx = (d) ∫ f (x) dx = 1 1 −1  12 + 9x 
2
0 6 0
61. If α = ∫ (e9x + 3 tan x )   dx,
3
0  1 + x2 
192x  1
52. Let f ′ (x) = for all x ∈ R with f   = 0. If
where tan −1 x takes only principal values, then the
2 + sin 4 πx  2
1  3π 
m≤∫ f (x) dx ≤ M , then the possible values of m and M value of  log e|1 + α | −  is (2015 Adv.)
1/ 2  4
are (2015 Adv.)
(a) m = 13, M = 24 1  d2 
1 1 62. The value of ∫ 4x3  2
(1 − x2)5  dx is (2014 Adv.)
(b) m = , M = 0
 dx 
4 2
(c) m = − 11, M = 0
(d) m = 1, M = 12
Fill in the Blanks
63. Let f : [1, ∞ ] → [2, ∞ ] be differentiable function such
53. The option(s) with the values of a and L that satisfy the x

that f (1) = 2. If 6∫ f (t ) = dt = 3x f (x) − x3 , ∀ x ≥ 1 then
∫0 e (sin at + cos at )dt = L, is/are
t 6 4
1
equation π t the value of f (2) is .... (2011)
∫0 e (sin at + cos at )dt
6 4
(2015 Adv.) d e sin x
e4 π − 1 e4 π + 1 64. Let F (x) = , x > 0.
(a) a = 2, L = (b) a = 2, L = dx x
eπ − 1 eπ + 1 4 2 e sin x
2

e4 π − 1 e4 π + 1 If ∫ dx = F (k) − F (1), then one of the possible


(c) a = 4, L = (d) a = 4, L = 1 x
eπ − 1 eπ + 1 values of k is ..… . (1997, 2M)
1 37 π π sin (π log x)
x4 (1 − x)4 65. The value of ∫ dx is …… .
54. The value(s) of ∫ dx is (are) (2010) 1 x
0
1 + x2 (1997, 2M)

71 3 π
2π x sin 2n x
(a)
22
−π (b)
2
(c) 0 (d) − 66. For n > 0, ∫ dx = …… .
7 105 15 2
0 sin x + cos 2n x
2n
(1996, 2M)

67. If for non-zero x, af (x) + bf   = − 5, where a ≠ b,


π sin nx 1 1
55. If I n = ∫ dx , n = 0 , 1 , 2 ,... , then  x x
−π (1 + π x ) sin x (2009) 2
10 then ∫ f (x) dx = …… .
(a) I n = I n + 2 (b) ∑ I2 m + 1 = 10 π 1 (1996, 2M)
m =1 3 x
10 68. The value of ∫ dx is …… .
(c) ∑ I2 m = 0 (d) I n = I n + 1 2 5−x+ x (1994, 2M)
m =1
298 Definite Integration

3 π /4
69. The value of ∫
x π/ 3 π + 4x3
π /4 1 + sin x
dx …… .
(1993, 2M)
76. Evaluate ∫ −π / 3  π
dx. (2004, 4M)
2 − cos |x|+ 
2  3
70. The value of ∫ |1 − x2| dx is … .
−2 (1989, 2M)
1.5 77. If f is an even function, then prove that
∫0
2
71. The integral [x ] dx, where [.] denotes the greatest π /2 π /4
∫0 f (cos 2x) cos x dx = 2 ∫
0
f (sin 2x) cos x dx.
(2003, 2M)
function, equals …… . (1988, 2M)
π cos x
e
Match the Columns 78. Evaluate ∫ dx. (1999, 3M)
0 ecos x + e− cos x
72. Match the conditions/expressions in Column I with 1 −1  1  1
statement in Column II. 79. Prove that ∫ 0 tan   dx = 2 ∫ tan −1 x dx.
 1 − x + x2  0
Column I Column II
Hence or otherwise, evaluate the integral
log  
1 dx 1 2
A. ∫ −1 1 + x 2 P. 1 −1
 3
∫ tan (1 − x + x ) dx.
2
2 (1998, 8M)
0
2 log  
1 dx 2
B. ∫0 Q.  3
π /4
1− x 2 80. Integrate ∫ log (1 + tan x) dx.
0
3 dx π (1997C, 2M)
C. ∫2 1− x 2 R.
3 π 2x (1 + sin x)
2 dx π
81. Determine the value of ∫ −π 1 + cos 2 x
dx. (1995, 5M)
D. ∫1 S.
x x2 −1 2
82. Evaluate the definite integral
73. Match List I with List II and select the correct answer
1/ 3  x4   2x 
using codes given below the lists. (2014)
∫ −1/   cos −1   dx. (1995, 5M)
3 1 − x 
4
 1 + x2 
List I List II
3 2x5 + x4 − 2x3 + 2x2 + 1
P. The number of polynomials f( x ) with (i) 8
83. Evaluate ∫2 (x2 + 1) (x4 − 1)
dx. (1993, 5M)
non-negative integer coefficients of degree
1
≤ 2, satisfying f( 0) = 0 and ∫ f( x ) dx = 1, is 84. A cubic f (x) vanishes at x = − 2 and has relative
0
minimum / maximum at x = − 1 and x = 1/3.
1
Q. The number of points in the interval (ii) 2 If ∫ f (x) dx = 14 / 3, find the cubic f (x). (1992, 4M)
[− 13, 13 ] at which f( x ) = sin( x 2 ) + cos( x 2 ) −1
attains its maximum value, is
π 
x sin (2x) sin  cos x
3x 2 π 2 
R. (iii) 4
85. Evaluate ∫
2
∫−2 1 + e x dx equals 0 2x − π
dx . (1991, 4M)

S.  1/ 2 1 + x   (iv) 0 86. Show that ,


 ∫ cos 2 x log   dx  π /2 π /4
 − 1/ 2 1− x  
equals ∫0 f (sin 2x) sin x dx = 2 ∫0 f (cos 2x) cos x dx.
(1990, 4M)
 1/ 2 1 + x  
 ∫ cos 2 x log   dx 
87. Prove that for any positive integer k,
 0 1− x  
sin 2kx
= 2 [cos x + cos 3x + K + cos(2k − 1) x]
Codes sin x
π /2
P Q R S
(a) (iii) (ii) (iv) (i)
Hence, prove that ∫0 sin 2kx ⋅ cot x dx = π /2.
(1990, 4M)
(b) (ii) (iii) (iv) (i)
(c) (iii) (ii) (i) (iv) 88. If f and g are continuous functions on [0, a ] satisfying
(d) (ii) (iii) (i) (iv) f (x) = f (a − x) and g (x) + g (a − x) = 2, then show that
a a
Analytical & Descriptive Questions ∫ 0 f (x) g(x) dx = ∫ 0 f (x) dx. (1989, 4M)

1
(5050) ∫ (1 − x50 )100 dx 89. Prove that the value of the integral,
0 2a
74. The value of 1
is (2006, 6M)
∫0 [ f (x) /{ f (x) + f (2a − x)}] dx is equal to a. (1988, 4M)
∫0 (1 − x50 )101 dx
1
90. Evaluate ∫ 0 log[ (1 − x) + (1 + x)] dx. (1988, 5M)
75. Evaluate
π |cos x |  1  1  π x dx
∫ 0e 91. Evaluate ∫ 1
2 sin  2 cos x + 3 cos  2 cos x  sin x dx. + cos α sin x
, 0 < α < π. (1986, 2 M)
  (2005, 2M)
01 2
Definite Integration 299

π /2 x sin x cos x 3/ 2
92. Evaluate ∫ ∫ −1 | x sin πx| dx.
1
dx. (1985, 2 M) (ii) Find the value of
0 cos 4 x + sin 4 x 2 (1982, 3M)
−1 1
∫ 0 (tx + 1 − x) dx,
n
1/ 2 x sin x 96. Evaluate
93. Evaluate ∫0 1−x 2
dx. (1984, 2M)
where n is a positive integer and t is a parameter
π /4 sin x + cos x independent of x. Hence, show that
94. Evaluate ∫ dx. (1983, 3M)
0 9 + 16 sin 2x 1 k n−k 1
π π π
∫ 0 x (1 − x) dx = nC k (n + 1), for k = 0, 1,... , n(1981,
.
4M)
95. (i) Show that ∫ x f (sin x) dx = ∫ 0 f (sin x) dx.(1982, 2M)
0 2

Topic 2 Periodicity of Integral Functions


Objective Questions I (Only one correct option) (c)
3
< g(2) ≤ 5 / 2 (d) 2 < g(2) < 4
π/ 2 dx 2
1. The value of ∫ , where [t ] denotes
− π / 2 [x] + [sin x] + 4
Analytical & Descriptive Questions
the greatest integer less than or equal to t, is nπ + v
(2019 Main, 10 Jan II) 4. Show that ∫ |sin x|dx = 2n + 1 − cos v, where n is a
1 1 0
(a) (7 π − 5) (b) (7 π + 5)
12 12 positive integer and 0 ≤ v < π . (1994, 4M)
3 3
(c) (4 π − 3) (d) (4 π − 3) 5. Given a function f (x) such that it is integrable over
10 20
every interval on the real line and f (t + x) = f (x), for
2. Let T > 0 be a fixed real number. Suppose, f is a every x and a real t, then show that the integral
a+ t
continuous function such that for all
T
∫ f (x) dx is independent of a.
a
(1984, 4M)
x ∈ R. f (x + T ) = f (x). If I = ∫ f (x) dx,
0

then the value of ∫


3 + 3T
f (2x) dx is (2002,1M)
Integer & Numerical Answer Type Question
3
6. For any real number x, let [x] denotes the largest
3
(a) I (b) I integer less than or equal to x. Let f be a real valued
2
function defined on the interval [− 10, 10] by
(c) 3I (d) 6I
 x − [x[, if f (x) is odd
x 1 f(x) = 
3. Let g (x) = ∫ f (t ) dt, where f is such that ≤ f (t ) ≤ 1 for 1 + [x[− x, if f (x) is even
0 2
π 2 10
10 ∫− 10
t ∈ [0,1] and 0 ≤ f (t ) ≤
1
for t ∈ [1, 2]. Then, g(2) satisfies Then, the value of f (x) cos πx dx is…… (2010)
2
the inequality (2000, 2M)
3 1
(a) − ≤ g(2) < (b) 0 ≤ g(2) < 2
2 2

Topic 3 Estimation, Gamma Function and


Derivative of Definite Integration
Objective Questions I (Only one correct option) Let F (x) = ∫
x2
f ( t ) dt, for x ∈ [0, 2]. If F ′ (x) = f ′ (x), ∀
0
x 1 2  1
1. If ∫ f (t ) dt = x2 + ∫x t f (t )dt , then f′  2 is x ∈ (0, 2), then F (2) equals (2014 Adv.)
0
(a) e2 − 1 (b) e4 − 1
(2019 Main, 10 Jan II)
24 18 (c) e − 1 (d) e4
(a) (b)
25 25 3. The intercepts on X-axis made by tangents to the curve,
6 4 x
(c) (d) y = ∫ | t | dt , x ∈ R, which are parallel to the line y = 2x,
25 5 0
are equal to (2013 Main)
2. Let f : [0, 2] → R be a function which is continuous on
(a) ± 1 (b) ± 2
[0, 2] and is differentiable on (0, 2) with f (0) = 1. (c) ± 3 (d) ± 4
300 Definite Integration
x 1
4. Let f be a non-negative function defined on the interval
x x
11. ∫ 0 f (t ) dt = x + ∫ x t f (t ) dt, then the value of f (1) is
[0, 1]. If ∫0 1 − ( f ′ (t ))2 dt = ∫ f (t ) dt , 0 ≤ x ≤ 1
0
and
1
(1998, 2M)

f (0) = 0 , then (2009)


(a) (b) 0
2
(a) f   < and f   >
1 1 1 1 1
(c) 1 (d) −
 2 2  3 3 2

(b) f   > and f   > 12. Let f : R → R be a differentiable function and f (1) = 4.
1 1 1 1
 2 2  3 3 f ( x) 2t
Then, the value of lim ∫ dt is
x→1 x −1
(c) f   < and f   <
1 1 1 1 4
(1990, 2M)
 2 2  3 3 (a) 8 f ′(1) (b) 4 f ′(1)
(c) 2 f ′(1) (d) f ′(1)
(d) f   > and f   <
1 1 1 1
 2 2  3 3 1 2
13. The value of the definite integral ∫ (1 + e−x ) dx is
0
1  1 (a) −1
5. If ∫ sin x t 2 f (t ) dt = 1 − sin x, ∀ x ∈ (0, π / 2), then f  
 3 (b) 2
(1981, 2M)

is (c) 1+ e−1
(a) 3 (b) 3 (2005, 1M) (d) None of the above
(c) 1/3 (d) None of these
t2 2 Objective Question II
∫ 0 x f (x) dx = 5 t
5
6. If f (x) is differentiable and , then
(One or more than one correct option)
4
f   equals (2004, 1M) 14. Which of the following inequalities is/are TRUE?
 25 (2020 Adv.)
2 5 1 3 1 3
(a) (b) − (a) ∫ x cos x dx ≥ (b) ∫ x sin x dx ≥
5 2 0 8 0 10
5 1 2 1 1 2
(c) 1 (d) (c) ∫ x cos x dx ≥ (d) ∫ x2 sin x dx ≥
2 0 2 0 9
sin( 2x )
7. If f (x) = ∫
x 2 + 1 −t 2
e dt, then f (x) increases in (2003, 1M)
15. If g (x) = ∫ sin −1 (t ) dt, then
sin x (2017 Adv.)
x2
π π
(a) (2, 2 ) (b) no value of x (a) g′  −  = 2 π (b) g′  −  = − 2 π
 2  2
(c) (0, ∞ ) (d) (−∞ ,0)
π π
1 (c) g′   = 2 π (d) g′   = − 2 π
8. If I (m, n ) = ∫ tm (1 + t )n dt, then the expression for  2  2
0
I (m, n ) in terms of I (m + 1, n − 1) is (2003, 1M) Passage Based Questions
2n n
(a) − I (m + 1, n − 1) Let f (x) = (1 − x)2 sin 2 x + x2, ∀ x ∈ R and
m+1 m+1 x  2 (t − 1 ) 
n g (x) = ∫  − ln t f (t ) dt ∀ x ∈ (1, ∞).
(b) I (m + 1, n − 1) 1  t+1 
m+1
2n n 16. Consider the statements
(c) + I (m + 1, n − 1)
m+1 m+1 P : There exists some x ∈ R such that,
m f (x) + 2x = 2 (1 + x2).
(d) I (m + 1, n − 1)
m+1 Q : There exists some x ∈ R such that,
x 2 f (x) + 1 = 2x (1 + x).
9. Let f (x) = ∫ 2 − t 2 dt. Then, the real roots of the Then,
1
equation x2 − f '(x) = 0 are (2002, 1M) (a) both P and Q are true
1 (b) P is true and Q is false
(a) ±1 (b) ±
2 (c) P is false and Q is true
1 (d) both P and Q are false
(c) ± (d) 0 and 1
2 17. Which of the following is true?
x
10. Let f : (0, ∞ ) → R and F (x) = ∫ f (t ) dt. (a) g is increasing on (1, ∞ )
0 (b) g is decreasing on (1, ∞ )
If F (x2) = x2 (1 + x), then f (4) equals (2001, 1M) (c) g is increasing on (1, 2) and decreasing on (2, ∞ )
(a)
5
(b) 7 (c) 4 (d) 2 (d) g is decreasing on (1, 2) and increasing on (2, ∞ )
4
Definite Integration 301

x
Fill in the Blank 22. If ‘ f ’ is a continuous function with ∫ 0 f (t ) dt → ∞ as
sec x cos x sec2x + cot x cosec x |x|→ ∞ , then show that every line y = mx intersects the
x
18. f (x) = cos 2 x cos 2 x cosec2x
2 2
. curve y2 + ∫ 0 f (t ) dt = 2 (1991, 2M)
1 cos x cos x
π /2
Then, ∫ f (x) dx = K . 23. Investigate for maxima and minima the function,
0 (1987, 2M) x
f (x) = ∫ [2(t − 1)(t − 2)3 + 3(t − 1)2(t − 2)2] dt. (1988, 5M)
Analytical & Descriptive Questions 1

ln t x
19. For x > 0, let f (x) = ∫ dt. Find the function
1+ t 1 Integer & Numerical Answer Type Question
f (x) + f (1 / x) and show that f (e) + f (1 / e) = 1 / 2, where 24. Letf : R → R be a differentiable function such that its
ln t = log e t. (2000, 5M)
derivative f′ is continuous and f (π) = − 6.
20. Let a + b = 4, where a < 2 and let g (x) be a x
dg If F : [0, π ] → R is defined by F (x) = ∫ f (t )dt, and if
0
differentiable function. If > 0, ∀ x prove that
dx π
a b
∫ g(x) dx + ∫ g(x) dx increases as (b − a ) increases.
0 0
∫ ( f ′ (x) + F (x)) cos x dx = 2
(1997, 5M) 0

21. Determine a positive integer n ≤ 5, such that then the value of f (0) is ……… . (2020 Adv.)
1 x
∫ 0e (x − 1)n dx = 16 − 6e (1992, 4M)

Topic 4 Limits as the Sum


Objective Question I (Only one correct option) Objective Questions II
 (n + 1)1/3 (n + 2)1/3 (2n )1/3  (One or more than one correct option)
1. lim  + + ..... +  is equal to
n→ ∞  n 4/3
n 4/3
n 4/3  4. For a ∈ R,|a| > 1, let
(2019 Main, 10 April I)  
 
4 4/3 3 4/3 4  1 + 2 +…+ n
3 3

(a) (2) (b) (2) −
lim = 54
n→∞  
3 4 3 
 n7/3  1 1 1 
 + + … + 
(an + n )2 
3 3 4 3/ 4
(c) (2)4/3 − (d) (2)   (an + 1)2 (an + 2)2
4 4 3
Then the possible value(s) of a is/are (2020 Adv.)
n n n 1 (a) −6 (b) 7 (c) 8 (d) −9
2. lim  + 2 + 2 + ... +  is equal to
n→ ∞ n + 1 2
2
n +2 2
n +3 2
5n  5. For a ∈ R (the set of all real numbers), a ≠ − 1,
(2019 Main, 12 Jan II) (1a + 2a + K + n a ) 1
(a) tan − 1 (3) lim a−1
= .
n→∞ (n + 1 ) [(na + 1) + (na + 2) + K + (na + n )] 60
(b) tan − 1 (2)
(c) π/4
Then, a is equal to
−15 −17
(d) π/2 (a) 5 (b) 7 (c) (d)
2 2
1/ n
(n + 1)(n + 2) K 3n 
3. lim 
n n−1
 n n
n→ ∞  2n
n 
is equal to 6. Let S n = ∑ and Tn = ∑ 2 , for
(2016 Main)
k=0 n + kn +
2
k 2
k=0 n + kn + k2
18
(a) n = 1, 2, 3, ... , then (2008, 4M)
e4 π π π π
27 (a) Sn < (b) Sn > (c) Tn < (d) Tn >
(b) 3 3 3 3 3 3 3 3
e2
9 Analytical & Descriptive Question
(c)
e2
 1 1 1
(d) 3 log 3 − 2 7. Show that, lim  + +K+  = log 6.
n→ ∞ n + 1 n + 2 6n  (1981, 2M)
Answers
Topic 1 π π
80. (log 2 ) 81. π 2 82. [ π + 3 log (2 + 3 ) − 4 3 ]
1. (c) 2. (c) 3. (a) 4. (d) 8 12
5. (b) 6. (a) 7. (a) 8. (c) 1 1 8
83. log 6 − 84. f ( x ) = x 3 + x 2 − x + 2 85. 2
9. (d) 10. (d) 11. (c) 12. (a) 2 10 π
13. (c) 14. (d) 15. (d) 16. (c) 1 1 π απ π2
90. log 2 − + 91. 92.
17. (d) 18. (d) 19. (b) 20. (a) 2 2 4 sin α 16
21. (c) 22. (a) 23. (d) 24. (b)  3 1 1  3π + 1 
93.  − π +  94. (log 3 ) 95. ( ii )
25. (a) 26. (c) 27. (b) 28. (a)  12 2 20   π 2 
29. (c) 30. (c) 31. (b) 32. (c)
33. (a) 34. (a) 35. (a) 36. (c) Topic 2
37. (a) 38. (d) 39. (d) 40. (d) 1. (d) 2. (c) 3. (b) 6. (4)
41. (c) 42. (a) 43. (d) 44. (a, b, c)
45. (c, d) 46. (b) 47. (a) 48. (c)
Topic 3
49. (c, d) 50. (b, d) 51. (a, b) 52. (*) 1. (a) 2. (b) 3. (a) 4. (c)
5. (a) 6. (a) 7. (d) 8. (a)
53. (a, c) 54. (a) 55. (a, b, c) 56. (1)
9. (a) 10. (c) 11. (a) 12. (a)
57. (0.5) 58. (4.00) 59. (2) 60. (0)
13. (d) 14. (a,b,d) 15. (*) 16. (c)
61. (9) 62. (2) 63. (8/3) 64. (k = 16)
 15 π + 32  1
1  7  17. (b) 18. −   19.
65. (2) 66. π 2 67. a log 2 − 5a + b  60  2
a − b2
2  2 
21. (n = 3 )
 1
68.   69. π ( 2 − 1 ) 70. (4) 71. (2 − 2 ) 7
 2 23. At x = 1 and , f ( x ) is maximum and minimum respectively.
5
72. A → S; B → S; C → P; D → R 73. (d)
24. (4)
24   1 e  1 
74. 5051 75. e cos  2 + 2 sin  2 − 1  Topic 4
5  
1. (c) 2. (b) 3. (b) 4. (c,d)
 4π  1  π
76.  tan −1    78. 79. log 2 5. (b, d) 6. (a, d)
 3  2  2

Hints & Solutions


Topic 1 Properties of Definite Integral π /3  d  tan 4 x sin 4 3x 
π =∫    dx
1. Given integral ∫ |π −|x|| dx  dx  
π/ 6 2
−π
π π π /3
= 2 ∫ |π −|x|| dx = 2∫ |π − x | dx 1 
= tan 4 x sin 4 3x
0 0 2 
π π/ 6
= 2 ∫ (π − x) dx [Q x ∈ (0, π)]
0 1 1   1
π = (9 × 0) –  × 1  = −
 x2   π2  π 2 2  9   18
= 2  πx −  = 2  π 2 −  = 2   = π2
 2 0  2  2 α+1
dx
Hence, option (c) is correct.
3. Let I = ∫ (x + α ) (x + α + 1)
α
α+1
2. Given integral (x + α + 1) − (x + α )
π /3
= ∫ (x + α ) (x + α + 1)
dx


α
I= tan3 x ⋅ sin 2 3x(2 sec2 x sin 2 3x + 3 tan x ⋅ sin 6x)dx
α+1
π/ 6  1 1 
π /3
= ∫  −  dx
 x + α x + α + 1
=∫ 3 2
[2 tan x sec x sin 3x 4
α
π/ 6 α +1
+ 3 tan x(2 sin 3x cos 3x)]dx
4 3 = [ log e (x + α ) − log e (x + α + 1)] α
Definite Integration 303

α+1 π /2
  x+ α   x π 1
= log e   = x − tan = − 1 = (π − 2)
  x + α + 1  α  
2 0 2 2
2α + 1 2α Since, I = m(π − n )
= log e − log e 1
2α + 2 2α + 1 ∴ m(π − n ) = (π − 2)
2
 2α + 1 2α + 1  9
= log e  ×  = log e   (given) On comparing both sides, we get
 2α + 2 2α   8
1
m = and n = − 2
(2 α + 1)2 9 2
⇒ = ⇒ 8 [4α 2 + 4α + 1] = 36 (α 2 + α )
4α (α + 1) 8 1
Now, mn = × − 2 = − 1
⇒ 8α 2 + 8α + 2 = 9α 2 + 9α ⇒ α 2 + α − 2 = 0 2
π /3
⇒ (α + 2) (α − 1) = 0 ⇒ α = 1, − 2
I= ∫ sec
2/3
5. Let x cosec4/3 x dx
From the options we get α = − 2 π /6
π /2 cot x
4. Let I = ∫ dx π /3
1
π /3
sec2 x
cot x + cosec x
0 = ∫ cos 2/ 3 x sin 4/3 x
dx = ∫ (tan x)4/ 3 dx
cos x π /6 π/6
π /2 π / 2 cos x
=∫ sin x dx = ∫ dx [multiplying and dividing the denominator by cos 4/3 x]
0 cos x
+
1 0 1 + cos x
Put, tan x = t, upper limit, at x = π / 3 ⇒ t = 3 and
sin x sin x
π / 2 cos x(1 − cos x)
lower limit, at x = π / 6 ⇒ t = 1 / 3
=∫ dx and sec2 x dx = dt
0 1 − cos 2 x
3
dt  t − 1/3 
3
π / 2 cos x − cos 2 x
=∫
0 sin 2 x
dx So, I= ∫ = 
t 4/3  − 1 / 3 1/
1/ 3 3
π /2
=∫ (cosec x cot x − cot2 x) dx  1 
0 = − 3  1/ 6 − 31/ 6 = 3 ⋅ 31/ 6 − 3 ⋅ 3− 1/ 6
π /2 3 
=∫ (cosec x cot x − cosec x + 1) dx
2
0
= 37/ 6 − 35/ 6
= [− cosec x + cot x + x]π0 / 2
π /2 6. Given integral
  2 x 
  − 2 sin 
2 
π /2 2π
 cos x − 1   I = ∫ [sin 2x ⋅ (1 + cos 3x)]dx
= x+ = x +  0
 sin x  0  x
2 sin cos 
x π
 2 2  0 = ∫ [sin 2x ⋅ (1 + cos 3x)]dx
0
π /2
 x  π 1 2π
= x − tan = − 1 = [π − 2] + ∫ [sin 2x ⋅ (1 + cos 3x)]dx
 2  0 2 2 π
= m[π + n ] [given] = I1 + I 2 (let) ... (i)
1 2π
On comparing, we get m = and n = − 2 Now, I 2 = ∫ [sin 2x ⋅ (1 + cos 3x)]dx
2 π

∴ m⋅ n = − 1 let 2π − x = t, upper limit t = 0 and lower limit t = π


Alternate Solution and dx = − dt
π /2 0
cot x I 2 = − ∫ [− sin 2x ⋅ (1 + cos 3x)]dx
Let I = ∫ dx So,
π
0 cot x + cosec x
π
cos x = ∫ [− sin 2x ⋅ (1 + cos 3x)]dx …(ii)
0
π /2 π / 2 cos x π
=∫ sin x dx = ∫ dx I = ∫ [sin 2x ⋅ (1 + cos 3x)]dx
0 cos x
+
1 0 cos x + 1 ∴ 0
π
sin x sin x + ∫0 [− sin 2x ⋅ (1 + cos 3x)]dx
x
π /2
2 cos 2 − 1 [from Eqs. (i) and (ii)]
=∫ 2 dx π
0 x = ∫ (−1)dx] [Q [x] + [− x] = − 1, x ∉Integer]
2 cos 2 0
2
= −π
 θ θ
Q cos θ = 2 cos 2 − 1 and cos θ + 1 = 2 cos 2 1
 2 
2 7. Let I = ∫ x cot−1 (1 − x2 + x4 )dx
π /2 1 2 x 0
=∫ 1 − sec  dx
0  2 2 Now, put x2 = t ⇒ 2xdx = dt
304 Definite Integration

Lower limit at x = 0, t = 0 x
9. Given, f (x) = ∫ g (t )dt
Upper limit at x = 1, t = 1
0
1
1
cot−1 (1 − t + t 2)dt
2 ∫0
∴I= On replacing x by (−x), we get
−x

1
1

= ∫ tan −1 
1 
 dt

Q cot−1 x = tan −1
1 f (− x) = ∫ g(t )dt
20 1 − t + t 
2  x 
0

1 Now, put t = − u, so
1  t − (t − 1) 
= ∫ tan −1   dt x x
20  1 + t (t − 1) f (− x) = − ∫ g (− u )du = − ∫ g (u )du = − f (x)
1 
1 0 0
−1 −1
= ∫ ( tan t − tan (t − 1)dt  [Q g is an even function]
2 0 
  ⇒ f (− x) = − f (x)
−1 x − y −1 −1
Q tan 1 + xy = tan x − tan y
⇒ f is an odd function.
 
1 1 1 Now, it is given that f (x + 5) = g (x)
Q ∫ tan −1 (t − 1)dt = ∫ tan −1 (1 − t − 1)dt = − ∫ tan −1 (t )dt
∴ f (5 − x) = g (− x) = g (x) = f (x + 5)
0 0 0
a a [Q g is an even function]
because ∫ f (x)dx = ∫ f (a − x)dx ⇒ f (5 − x) = f (x + 5) …(i)
0 0
x
1
1 Let I = ∫ f (t )dt
( tan −1 t + tan −1 t )dt
2 ∫0
So, I =
0

1 1
t Put t = u + 5 ⇒ t − 5 = u ⇒ dt = du
= ∫ tan −1 tdt = [t tan −1 t ]10 − ∫ dt x −5 x −5
1 + t2
0
[by integration by parts method]
0 ∴ I= ∫ f (u + 5)du = ∫ g(u )du
−5 −5
π 1 π 1 Put u = −t
= − [log e (1 + t 2)]10 = − log e 2
4 2 4 2 ⇒ du = − dt, we get
5 −x 5

8. Key Idea Use property of definite integral.


I=− ∫ g (− t )dt = ∫ g(t )dt
5 5 −x

∫a f ( x) dx = ∫a f ( a + b − x) dx
b b b a
[Q − ∫ f (x)dx = ∫ f (x)dx and g is an even function]
a b
π 3 5
sin x
Let I=∫
0
2
sin x + cos x
dx …(i) I= ∫ f ′ (t )dt [by Leibnitz rule f ′ (x) = g (x)]
5 −x
b b
On applying the property,
∫a f (x)dx = ∫a f (a + b − x) dx, = f (5) − f (5 − x) = f (5) − f (5 + x) [from Eq. (i)]
5 5
we get
π /2 cos3 x
= ∫ f ′ (t )dt = ∫ g(t )dt
I=∫ dx …(ii) 5+ x 5+ x
0 cos x + sin x
10. The given functions are
On adding integrals (i) and (ii), we get
g (x) = log e x, x > 0
π/ 2 sin3 x + cos3 x 2 − x cos x
2I = ∫ dx nd f (x) =
0 sin x + cos x 2 + x cos x
π π /4
(sin x + cos x) (sin 2 x + cos 2 x − sin x cos x) I=∫
=∫ 2 dx Let
−π / 4
g ( f (x))dx
0 sin x + cos x
π /4  2 − x cos x 
π
 1   1
π
 Then, I=∫ log e   dx …(i)
=∫ 2 1 − (2 sin x cos x) dx = ∫ 2 1 − sin 2x dx −π / 4  2 + x cos x
0  2  0  2 
π/ 2
 1  π  1 π 1
= x + cos 2x =  − 0 + (−1 − 1) = −
 4   2  4 2 2
Now, by using the property
0 b b


π 1 π −1
I= − = ∫ f (x)dx = ∫ f (a + b − x)dx, we get
4 4 4 a a
Definite Integration 305

π /4  2 + x cos x ⇒ 5 tan 4 x sec 2x dx = dt


I=∫ log e   dx …(ii)
−π / 4  2 − x cos x  π π
x
On adding Eqs. (i) and (ii), we get 6 4
5

π /4  2 − x cos x   2 + cos x    1 
2I = ∫ log e   + log e   dx t   1
−π / 4   2 + x cos x  2 x cos x   3

π /4  2 − x cos x 2 + x cos x 1 1 1 dt 1
=∫ log e  ×  dx ∴ I= ⋅ ∫(1/ = (tan −1 (t ))(11/
−π / 4  2 + x cos x 2 − x cos x  2 5 3 )5 t 2 + 1 10 3 )5

[Q log e A + log e B = log e AB] 1  −1 −1  1  


π /4 =  tan (1) − tan  
⇒ 2I = ∫ log e (1)dx = 0 ⇒ I = 0 = log e (1) 10  9 3
−π / 4
a 1 π −1  1  
11. Let I = ∫ f (x) g (x) dx … (i) =  − tan  
0 10  4 9 3
a
= ∫ f (a − x) g (a − x) dx 2 sin 2 x
0 14. Let I = ∫ dx
Q a f (x) dx = a  −2 1 x
 ∫0 ∫0 f (a − x) dx +
2  π 
a
⇒ I = ∫ f (x) [4 − g (x)] dx sin 2 x
0 Also, let f (x) =
[Q f (x) = f (a − x) and g (x) + g (a − x) = 4] 1 x
+
a
= ∫ 4 f (x) dx − ∫ f (x) g (x) dx
a
2  π 
0 0
sin 2(− x)
a Then, f (− x) = (replacing x by − x)
⇒ I = 4 ∫ f (x) dx − I [from Eq. (i)] 1  x
0 + −
a a 2  π 
⇒ 2I = 4 ∫ f (x) dx ⇒ I = 2 ∫ f (x) dx.
0 0 sin 2 x
=
e
2x x
1   x 
12. Let I = ∫    e
x
−    log e x dx + − 1 −
1  e  x 2   π 
 
 x
x
 x
  − [x], if x ∈ I
Now, put   = t ⇒ x log e   = log t Q [− x] = − 1 − [x], if x ∉ I 
 e  e  
⇒ x (log e x − log e e) = log t sin 2 x
⇒ f (− x) = − = − f (x)
1 x
  1  1 +
⇒ x  x + (log e x − log e e) dx = t dt 2  π 
 
1 1 i.e. f (x) is odd function
⇒ (1 + log e x − 1) dx = dt ⇒ (log e x) dx = dt
t t ∴ I =0
Also, upper limit x = e ⇒ t = 1 and lower limit x = 1 ⇒
1  a  0, if f (x) is odd function 
t= Q ∫ f (x) dx =  a f (x)dx, f ( 
e  −a  ∫0
2 if x) is even function 
1  2 1 1
∴ I=∫  t −  ⋅ dt b
1/ e  t t 15. We have, I = ∫ (x4 − 2x2)dx
a
1
⇒ I=∫ (t − t −2) dt Let f (x) = x4 − 2x2 = x2(x2 − 2) = x2(x − 2 ) (x + 2 )
1/ e
1 Graph of y = f (x) = x4 − 2x2 is
  t 2 1   1   1  3 1
I =  +   =  + 1 −  2 + e  = − e − 2 Y
 2 t    2   2 e   2 2 e y=f(x)
  1
e
π /4 dx
13. Let I = ∫
π /6 sin 2x(tan5 x + cot5 x) X
– √2 √2 ∴
π /4 (1 + tan x) tan x 2 5  2 tan x  O f(x) < 0 for – √2 < x < √2
=∫ dx Q sin 2x =  + – – +
π /6 2 tan x (tan10 x + 1)  1 + tan 2 x  – √2 0 √2
1 π / 4 tan 4 x sec 2x
= ∫ dx b
2 π / 6 (tan10 x + 1) Note that the definite integral ∫ (x4 − 2x2)dx represent
a
Put tan5 x = t [Qsec2 x = 1 + tan 2 x] the area bounded byy = f (x) , x = a, b and the X -axis.
306 Definite Integration

But between x = − 2 and x = 2 , f (x) lies below the Now, as cos 3x = 4 cos3 x − 3 cos x
X-axis and so value definite integral will be negative. 1
∴ cos3 x = (cos 3x + 3 cos x)
Also, as long as f (x) lie below the X-axis, the value of 4
definite integral will be minimum. π
2 2
4 ∫0
∴(a , b) = (− 2 , 2 ) for minimum of I. ∴I= ( cos 3x + 3 cos x) dx
π /3 tan θ 1
16. We have, ∫ dθ = 1 − , (k > 0) π
0 2k sec θ 2 1 sin 3x 2
= + 3 sin x
Let I = ∫
π /3 tan θ 1 π /3 tan θ
2  3  0
0 2k sec θ
dθ =
2k ∫ 0 sec θ

1  1 3π π  1 
1 π /3 (sin θ) 1 π /3 sin θ =  sin + 3 sin − sin 0 + 3 sin 0 
=
2k 0 ∫ 1
dθ =
2k 0∫ cos θ
dθ 2 3 2 2  3 
(cos θ)
cos θ 1  1  
=  (−1) + 3 − [0 + 0]
Let cos θ = t ⇒ − sin θ dθ = dt ⇒ sin θ dθ = − dt 2 3  
for lower limit, θ = 0 ⇒ t = cos 0 = 1  3π  π π 
π
for upper limit, θ = ⇒ t = cos =
π 1 Q sin 2 = sin  π + 2  = − sin 2 = − 1
 
3 3 2
1 1 1  4
1 1/ 2 − dt −1 1/ 2 − = − +3 =
⇒ I= ∫1 = ∫1 t 2dt 2  3  3
2k t 2k
1 b b
 − +1 2
1 18. Key idea Use property = ∫ f (x)dx = ∫ f (a + b − x)dx
1  t 2 
1
1 a a
=−   =− [2 t ]12
2k  − 1 + 1 2k π /2 2
sin x
 2 1 Let I= ∫ 1 + 2x
dx
−π/ 2
2  1  2  1
=−  2 − 1 = 1 −  2 π π 
2k   2k  2 π / 2 sin  − + − x
 2 2 
Q I =1 −
1
(given)
⇒ I= ∫ −
π
+
π
−x
dx
−π/ 2
2 1+2 2 2

2  1 1 2  b b 
∴ 1 −  =1 − ⇒ =1
2k  2 2 2k Q ∫ f (x)dx = ∫ f (a + b − x)dx
 a a 
⇒ 2 = 2k ⇒ 2k = 4 ⇒ k = 2 π /2
sin 2 x
17. We know, graph of y = cos x is ⇒ I= ∫ 1 + 2−x dx
−π / 2
Y π /2
2x sin 2 x
⇒ I= ∫ 2x + 1
dx
−π / 2
π /2
X′ π X  2 x + 1
O π/2 ⇒ 2I = ∫ sin 2 x x
 2 + 1
 dx
−π / 2
π /2
Y′ ⇒ 2I = ∫ sin
2
x dx
∴ The graph of y =|cos x|is −π / 2
π /2
Y
y=|cos x| ⇒ 2I = 2 ∫ sin 2 x dx [Qsin 2 x is an even function]
0
π /2
⇒ I= ∫ sin xdx
2

0
X′ X π /2
O π/2 π  a a 
Y′
⇒ I= ∫ cos 2xdx Q ∫ f (x)dx = ∫ f (a − x)dx
π
0  0 0 
π π /2
I = ∫ |cos x|3 = 2 ∫ 2|cos x|3 dx
0 0 ⇒ 2I = ∫ dx
π 0
(Q y = |cos x|is symmetric about x = )
2 ⇒ 2I = [x]0π / 2
π
  π   π
= 2∫ 2 cos3 x dx ⇒ I=
Q cos x ≥ 0 for x ∈ 0, 2  4
0  
Definite Integration 307

3 π /4 dx 3 π / 4 1 − cos x
19. Let I = ∫ =∫ dx 22. PLAN This type of question can be done using appropriate
π /4 1 + cos x π / 4 1 − cos 2 x substitution.
π /2
3 π /4 1 − cos x Given, I = ∫ (2 cosec x)17 dx
=∫ dx π /4
π /4 sin 2 x
3 π /4 π /2 217 (cosec x)16 cosec x (cosec x + cot x)
=∫ (cosec2 x − cosec x cot x)dx =∫ dx
π /4 π /4 (cosec x + cot x)
= [− cot x + cosec x]3π π/ 4/ 4 Let cosec x + cot x = t
= [(1 + 2 ) − (− 1 + 2 )] = 2 ⇒ (− cosec x ⋅ cot x − cosec2x) dx = dt
π /2 2
x cos x and cosec x − cot x = 1 /t
20. Let I = ∫ dx …(i)
− π /2 1 + ex 1
⇒ 2 cosec x = t +
Q b f (x) dx = b f (a + b − x) dx t 16

 ∫a ∫a  1
 1 t +  dt
∴ I=−∫ 217  t
x2 cos (− x)
π /2 2+1
 2  t
⇒ I=∫ dx …(ii)  
−π / 2 1 + e− x
Let t = eu ⇒ dt = eudu.
On adding Eqs. (i) and (ii), we get When t = 1, eu = 1 ⇒ u = 0
π /2 2  1 1 
2I = ∫ x cos x  +  dx and when t = 2 + 1, eu = 2 + 1
−π / 2
 1 + ex
1 + e− x 
⇒ u = ln ( 2 + 1 )
π /2 2
=∫ x cos x ⋅ (1) dx 0 eudu
−π / 2 ⇒ I=−∫ 2(eu + e− u )16
 a 
ln ( 2 + 1 ) eu
a
Q ∫ f (x) dx = 2 ∫ f (x) dx, when f (− x) = f (x) ln ( 2 + 1 )
 − a 0  =2 ∫ (eu + e− u )16 du
0
π /2 2
⇒ 2I = 2∫ x cos x dx 23. PLAN Use the formula,| x − a | = 
x − a, x ≥a
0
 − ( x − a ), x < a
Using integration by parts, we get to break given integral in two parts and then integrate separately.
2I = 2 [x2(sin x) − (2x) (− cos x) + (2) (− sin x)] π0 / 2
2
π2  π  x π x
⇒ 2I =2 − 2 ∫0 1 − 2 sin  dx = ∫ |1 − 2 sin |dx
 2 0 2
 4 
π
π2  x π  x
∴ I= −2 = ∫ 3 1 − 2 sin  dx − ∫ π 1 − 2 sin  dx
0  2   2
4 3
π
π
21.  x 3  x π
PLAN Apply the property =  x + 4 cos  −  x + 4 cos  = 4 3 − 4 −
b b  2 0  2 π 3
∫a f(x )dx = ∫a f(a + b − x )dx and then add. 3

log x2 π /2  2  π − x 
Let I=∫
4
dx 24. I = ∫ x + log  π + x  cos x dx
− π /2
2 log x2 + log(36 − 12 x + x2 )  
a
∫− a f (x) dx = 0, when f (− x) = − f (x)
4 2 log x
=∫ dx As,
2 2 log x + log(6 − x)2
π /2 π /2
4 2 log x dx ∴ I=∫ x2 cos x dx + 0 = 2∫ (x2 cos x) dx
=∫ − π /2 0
2 2 [log x + log(6 − x)]
π /2
4 log x dx = 2 {(x2 sin x)π0 / 2 − ∫ 2x ⋅ sin x dx}
⇒ I=∫ …(i) 0
2 [log x + log(6 − x)]
π2 π /2
4 log(6 − x) =2  − 2 {(− x ⋅ cos x)0π / 2 − ∫ 1 ⋅ (− cos x) dx}
⇒ I=∫ dx …(ii)  4 0 
2 log(6 − x) + log x

Q b f (x)dx = b f (a + b − x)dx π2  π2   π2 


− 2 (sin x)0π / 2 = 2 
 ∫a ∫a =2  − 2 =  − 4
 4  4   2 
On adding Eqs. (i) and (ii), we get
25. Put x2 = t
4 log x + log(6 − x)
2I = ∫ dx ⇒ x dx = dt /2
2 log x + log(6 − x)
dt

4
2I = ∫ dx = [x]42 ⇒ 2I = 2 log 3
sin t ⋅
2 ∴ I=∫ 2 ...(i)
log 2 sin t + sin (log 6 − t )
⇒ 2I = 2 ⇒ I = 1
308 Definite Integration
b b π cos 2 x
Using, ∫a f (x) dx = ∫a f (a + b − x) dx ⇒ I=∫ ax dx …(ii)
−π 1 + ax
1 sin (log 2 + log 3 − t )
log 3
=
2∫log 2 sin (log 2 + log 3 − t ) + sin dt On adding Eqs. (i) and (ii), we get
(log 6 − (log 2 + log 3 − t )) π 1 + ax 
1 log 3 sin (log 6 − t ) 2I = ∫   cos 2x dx
−π  1 + a x 
= ∫ dt
2 log 2 sin (log 6 − t ) + sin (t )
π π 1 + cos 2x
log 3 sin (log 6 − t ) =∫ cos 2 x dx = 2 ∫ dx
∴ I=∫ dt …(ii) −π 0 2
log 2 sin (log 6 − t ) + sin t π π π
= ∫ (1 + cos 2x) dx = ∫ 1 dx + ∫ 0 cos 2x dx
On adding Eqs. (i) and (ii), we get 0 0

1 log 3 sin t + sin (log 6 − t ) π /2


2 ∫log 2 sin (log 6 − t ) + sin t
2I = dt = [x]π0 + 2∫ cos 2x dx = π + 0
0

⇒ 2I =
1 log 3 1
(t )log 2 = (log 3 − log 2) ⇒ 2I = π ⇒ I = π /2
2 2 ecos x sin x, for | x| ≤ 2
1  3 
∴ I = log   30. Given, f (x) =  2 , otherwise
4  2 
0 
26. Let I = ∫ [x3 + 3x2 + 3x + 3 + (x + 1) cos (x + 1)] dx 3 2 3
−2
0
∴ ∫ −2 f (x) dx = ∫ −2 f (x) dx + ∫ 2 f (x) dx
=∫ [(x + 1)3 + 2 + (x + 1) cos (x + 1)] dx 2 3
−2 =∫ ecos x sin x dx + ∫ 2 2 dx = 0 + 2 [x]32
−2
Put x + 1 = t
[Q ecos x sin x is an odd function]
⇒ dx = dt Q 3 f (x) dx = 2
 ∫ −2
= 2 [3 − 2] = 2
∴ I=∫
1
(t3 + 2 + t cos t ) dt 
−1
e2 2
log e x  dx = 1  log e x  dx − e  log e x  dx
=∫
1
t dt + 2 ∫
3 1
dt +
1
∫ −1 t cos t dt
31. ∫e −1
 x  ∫ e  x  ∫1  x 
− 1
−1 −1
 since, 1 is turning point for 
= 0 + 2 ⋅ 2 [x]10 + 0 = 4  log e x 
  for + ve and − ve values
[since, t3 and t cos t are odd functions]
 x  
1 1−x 1 1−x
27. I = ∫ dx = ∫ dx
= −∫
log e x
1
dx + ∫ 
e 2 log x
e 
dx
0 1+ x 0
1 − x2 e −1
x 1  x 
1 1 1 x 0 t 1 1
=∫ dx − ∫ dx = [sin −1 x] 10+ ∫1
2
dt = − [(log e x)2]1e−1 + [(log e x)2] 1e
0
1−x 2 0
1−x 2 t 2 2
1 1 5
[where, t 2 = 1 − x2 ⇒ t dt = − x dx] = − {0 − (−1)2} + (22 − 0) =
−1 −1
2 2 2
= (sin 1 − sin 0) + [t ]10 = π /2 − 1
32. The graph of y = 2 sin x for π /2 ≤ x ≤ 3π / 2 is given in
1/ 2   1 + x 
28. ∫ [x] + log   dx figure. From the graph, it is clear that
−1/ 2
  1 − x  
 2, if x = π /2
1/ 2 1/ 2  1 + x  1, if π / 2 < x ≤ 5π /6
=∫ [x] dx + ∫ −1/ 2log 1 − x dx 
−1/ 2 [2 sin x] =  0, if 5π / 6 < x ≤ π
−1, if π < x ≤ 7π / 6
1/ 2   1 + x  
=∫ [x]dx + 0 Q log  1 − x  is an odd function  −2, if 7π / 6 < x ≤ 3π /2
−1/ 2
 
0 1/ 2 0 1/ 2
Y
=∫ [x] dx + ∫0 [x] dx = ∫ (−1) dx + ∫0 (0) dx
−1/ 2 −1/ 2 2
 1 1
= − [x]−01/ 2 = − 0 +  =− 1
 2 2 π
X
π 2 π/2 5π/6 7π/6 3π/2
cos x
29. Let I = ∫ dx …(i) –1
−π 1 + ax
–2
−π cos 2(− x)
=∫ d (− x)
π 1 + a −x
Definite Integration 309

3 π /2
⇒ dt = dy
Therefore, ∫ π /2 [2 sin x]dx
x
5 π /6 π 7 π /6 3 π /2 I 2 = ∫ cos 4 ( y + π ) dy
=∫ dx + ∫ 0 dx + ∫π (−1) dx + ∫ (−2) dx 0
π /2 5π / 6 7π / 6 x x
= ∫ (− cos y)4 dy = ∫ cos 4 y dy = g (x)
= [x] 5π π/ 2/ 6 + [− x] 7π
π
/6
+ [−2x] 37ππ //62 0 0

 5π π   7π   −2 ⋅ 3 π 2 ⋅ 7 π  ∴ g (x + π ) = g (π ) + g (x)
= −  + − + π +  + 
 6 2   6   2 6  k
 5 1  7 7 
= π  −  + π 1 −  + π  − 3
36. Given, I1 = ∫ x f [x (1 − x)] dx
 6 2  6 3  1−k
k
 5 − 3  1  7 − 9
=π
 6 
 + π −  + π 
 6  3 
 = −π / 2 ⇒ I1 = ∫ (1 − x) f [(1 − x) x] dx
1−k
3 π /4 dx k k
33. Let I=∫ …(i) =∫ f [(1 − x)]dx] − ∫ xf (1 − x)]dx
π /4 1 + cos x 1−k 1 −k

3 π /4 dx I1 1
⇒ I=∫ ⇒ I1 = I 2 − I1 ⇒ =
π /4 1 + cos (π − x) I2 2
3 π /4 dx
I=∫ …(ii) 37. It is a question of greatest integer function. We have,
π /4 1 − cos x subdivide the interval π to 2π as under keeping in view
On adding Eqs. (i) and (ii), we get that we have to evaluate [2 sin x ]
3 π /4   Y
1 1
2I = ∫  +  dx 1,π/2
π / 4  1 + cos x 1 − cos x
3 π /4  2 
⇒ 2I = ∫   dx X'
O
X
π /4  1 − cos x 
2
(0,π) 30° 30° (0,2π)
3 π /4
⇒ I=∫ cosec2x dx = [− cot x] 3π π/ 4/ 4
π /4
–1,3π/2 –1/2,11π/6
 3π π –1/2,7π/6
= − cot + cot = − (−1) + 1 = 2 Y'
 4 4 
π 1
1 1 1 1 We know that, sin =
34. Let ∫ −1 f (x) dx = ∫ −1 (x − [x]) dx = ∫ −1 x dx − ∫ −1 [x] dx 6 2
 π 7π 1
1 ∴ sin  π +  = sin =−
=0−∫ [x] dx [Q x is an odd function]  6 6 2
−1
11π  π π 1
−1, if −1 ≤ x < 0 ⇒ sin = sin 2π −  = − sin = −
 6  6  6 2
But [x] =  0, if 0 ≤ x<1
 1, if x=1 9π 3π
 ⇒ sin = sin = −1
1 0 1 6 6
∴ ∫ −1 [x] dx = ∫ −1 [x] dx + ∫ 0 [x] dx Hence, we divide the interval π to 2π as
0 1
=∫ 7π   7π 11 π   11π
−1
(−1) dx + ∫0 0 dx 
π ,

,  , , 

, 2π
6  6 6   6 
1
= − [x]−01 + 0 = −1; ∴ ∫ −1 f (x) dx = 1  1  1  1 
sin x = 0, −  ,  − 1, −  ,  − , 0
 2  2  2 
x
35. Given, g (x) = ∫ cos 4 t dt
0 ⇒ 2 sin x = (0, − 1), (−2, − 1), (−1, 0)
π+x
⇒ g (x + π ) = ∫ 4
cos t dt ⇒ [2 sin x] = − 1
0
7π / 6 11 π / 6
π π+x =∫ [2 sin x] dx + ∫ 7π / 6 [2 sin x] dx
= ∫ cos 4 t dt + ∫ cos 4 t dt = I1 + I 2 π
0 π

where,
π
I1 = ∫ cos t dt = g (π )
4
+ ∫ 11π / 6 [2 sin x] dx
7π / 6 11 π / 6 2π
0
=∫ (−1) dx + ∫ 7π / 6 (−2) dx + ∫ 11π / 6 (−1) dx
π+x π
and I2 = ∫ cos t dt 4
π π  4π  π 10π 5π
=− −2  − =− =−
Put t=π+ y 6  6  6 6 3
310 Definite Integration

πx
38. Given, f (x) = A sin   1
 + B, f ′   = 2
On adding Eqs. (i) and (ii), we get
2  2 π /2
2I = ∫ 1 dx
1 2A 0
and ∫ 0 f (x) dx = π π
∴ I=
Aπ πx  1  Aπ π Aπ 4
f ′ (x) = cos ⇒ f′  = cos =
2 2  2 2 4 2 2 π /3 dx
43. Let I = ∫ …(i)
 1 Aπ 4 π / 6 1 + tan x
But f′   = 2 ∴ = 2 ⇒ A=
 2 2 2 π π /3 dx
∴ I=∫
1 2A 1   π x  2A π /6
π 
Now, ∫ f (x) dx = ⇒ ∫  A sin   + B dx = 1 + tan  − x
0 π 0   2   π 2 
1
 2A πx  2A π /3 dx
⇒ − cos + Bx = =∫
 π 2  π π /6 1 +
0 cot x
2A 2A
⇒ B+ =
π π π /3 tan xdx
⇒ I=∫ …(ii)
π /6 1 +
⇒ B =0 tan x
π /2 1 π /2 1
39. Let I=∫ dx = ∫ dx On adding Eqs. (i) and (ii), we get
0 1 + tan x
3 0 sin3 x π /3
1+ 2I = ∫ dx
cos3 x π /6

cos3 x
π /2
⇒ 2I = [x]ππ //36 dx
⇒ I=∫ dx …(i)
0 cos x + sin3 x 3
1 π π  π
π  ⇒ I= − =
cos3  − x 2  3 6  12
π /2 2 
⇒ I=∫ dx Statement I is false.
0 3π  3π 
cos  − x + sin  − x b b
2  2  But ∫ a f (x)dx = ∫ a f (a + b − x)dx is a true statement by
π /2 sin3 x
⇒ I=∫ dx …(ii) property of definite integrals.
0 sin3 x + cos3 x
44. According to the given data,
On adding Eqs. (i) and (ii), we get
F ′ (x) < 0, ∀x ∈ (1, 3)
π /2
2I = ∫ 1 dx ⇒ 2I = [x]0π / 2 = π / 2 ⇒ I = π /4 We have, f (x) = x F (x)
0
π /2
⇒ f ′ (x) = F (x) + x F ′ (x) …(i)
40. Let I = ∫ [f (x) + f (− x) ] [ g (x) − g (− x)] dx ⇒ f ′ (1) = F (1) + F ′ (1 ) < 0
−π / 2

Let φ (x) = [ f (x) + f (− x)] [ g (x) − g (− x)] [given F (1) = 0 and F ′ (x) < 0]
⇒ φ (− x) = [ f (− x) + f (x)] [ g (− x) − g (x)] Also, f (2) = 2F (2) < 0 [using F (x) < 0,∀x ∈ (1, 3)]
⇒ φ (− x) = − φ (x) Now, f ′ (x) = F (x) + x F ′ (x) < 0
⇒ φ(x) is an odd function. [using F (x) < 0, ∀ x ∈ (1, 3)]
π /2 ⇒ f ′ (x) < 0
∴ ∫ −π / 2 φ (x) dx = 0 3
∫1 x F ′ (x) dx = − 12
2
45. Given,
π 2
41. Let I = ∫ e cos x
⋅ cos {(2n + 1) x} dx
3
3
0 ⇒ [x2F (x)]31 − ∫ 2x ⋅ F (x) dx = − 12
1
a  0, f (a − x) = − f (x) 3
Using∫ f (x) dx =  a /2 ⇒ 9 F (3) − F ( 1 ) − 2∫ f (x) dx = − 12
0
2 ∫ 0 f (x) dx, f (a − x) = f (x) 1

2 [QxF (x) = f (x), given]


Again, let f (x) = ecos x
⋅ cos3 {(2n + 1) x} 3
2
⇒ − 36 − 0 − 2 ∫ f (x) dx = −12
1
∴ f (π − x) = (ecos x
){ − cos3 (2n + 1) x} = − f (x)
3 3
∴ I =0 ∴ ∫1 f (x) dx = − 12 and ∫ x3 F ′ ′ (x)dx = 40
1
π /2 cot x 3
42. Let I = ∫ dx …(i) ⇒ [x3 F ′ (x)] 31 − ∫ 3x2F ′ (x) dx = 40
0 cot x + tan x 1

π /2 tan x ⇒ [x2(xF ′ (x)] 31 − 3 × (−12) = 40


⇒ I=∫ dx …(ii)
0 cot x + tan x ⇒ { x2 ⋅ [ f ′ (x) − F (x)]} 31 = 4
Definite Integration 311

⇒ 9 [ f ′ (3) − F (3)] − [ f ′ (1) − F (1)] = 4 (d) Let g (x) = x9 − f (x)


⇒ 9 [ f ′ (3) + 4] − [ f ′ (1) − 0] = 4 g (0) = − f (0) < 0
g (1) = 1 − f (1) > 0
⇒ 9 f ′ (3) − f ′ (1) = − 32
∴ Option (d) is correct.
t (t − a )
∫a f (x) dx −
2
{ f (t ) + f (a )} 98 k + 1
(k + 1)
=0
46. Given, lim
t→ a (t − a ) 3 50. I = ∑ ∫ x(x + 1)
dx
k =1 k
Using L’Hospital’s rule, put t − a = h 98 k + 1
(k + 1)
a+ h h
∫ a f (x) dx − 2 { f (a + h ) + f (a )}
Clearly, I> ∑ ∫ (x + 1)2
dx
k =1 k
⇒ lim =0
h→ 0 h3 98 k+1
1
1 h
f (a + h ) − { f (a + h ) + f (a )} − { f ′ (a + h )}
⇒ I> ∑ (k + 1) ∫ (x + 1)2
dx
2 2 k =1 k
⇒ lim =0
h→ 0 3h 2 98
 1 1  98
1
Again, using L’ Hospital’s rule,
⇒ I> ∑ (−(k + 1)) k + 2 − k + 1  ⇒ I> ∑k+2
k =1 k =1
1 1 h
f ′ (a + h ) − f ′ (a + h ) − f ′ (a + h ) − f ′ ′ (a + h ) 1 1 98 49
lim 2 2 2 =0 ⇒ I> +…+ > ⇒ I>
h→ 0 6h 3 100 100 50
98 k + 1 98
h k+1
− f ′′ (a + h )
2
Also, I< ∑ ∫ x(k + 1)
dx = ∑ [log e (k + 1) − log e k]
⇒ lim = 0 ⇒ f ′′ (a ) = 0, ∀ a ∈ R k =1 k k =1
h→ 0 6h
⇒ f (x) must have maximum degree 1. I < log e 99
47. F ′ (c) = (b − a ) f ′ (c) + f (a ) − f (b) 51. Here, f (x) = 7 tan 8 x + 7 tan 6 x − 3 tan 4 x − 3 tan 2 x
F ′ ′ (c) = f ′ ′ (c)(b − a ) < 0  −π π 
for all x ∈  , 
f (b) − f (a )  2 2
⇒ F ′ (c) = 0 ⇒ f ′ (c) =
b−a ∴ f (x) = 7 tan 6 x sec2 x − 3 tan 2 x sec2 x
π   π = (7 tan 6 x − 3 tan 2 x) sec2 x
−0  0 +  
π /2  π π /4 π /4
∫0 sin x dx = 2 2
48. sin 0 + sin   + 2 sin   Now, ∫ x f (x)dx = ∫ x (7 tan 6 x − 3 tan 2 x) sec2 x dx
4  2  2  0 0 I II
   π /4
π = [x (tan x − tan x)]
7 3
0
= (1 + 2 ) π /4
8 −∫ 1 (tan7 x − tan3 x) dx
0
x
49. π /4
Q ex ∈ (1, e) in (0, 1) and ∫ f (t )sin t dt ∈ (0, 1) in (0, 1) =0 − ∫ tan3 x (tan 4 x − 1)dx
0
0
π /4
x = –∫ tan 3 x (tan 2 x − 1) sec2 x dx
∴ ex − ∫ f (t )sin t dt cannot be zero. 0

0 Put tan x = t ⇒ sec2 x dx = dt


So, option (a) is incorrect. π /4 1
π
∴ ∫0 x f (x)dx = − ∫ t3 (t 2 − 1) dt
0
2 1
 t 4 t5  1 1 1

1
(b) f (x) + f (t )sin t dt always positive = ∫ (t − t )dt =  −  = − =
3 5

0
0
 4 5  0 4 6 12
∴Option (b) is incorrect. π /4 π /4
π
−x
Also, ∫0 f (x) dx = ∫
0
(7 tan 6 x − 3 tan 2 x) sec2 x dx
2
1
(c) Let h (x) = x − ∫ f (t ) cos t dt, = ∫ (7t 6 − 3t 2)dt
0
0
π = [t7 − t3 ]10 = 0
2
h (0) = − ∫ f (t ) cos t dt < 0 192 x3 192x3 192x3
52. Here, f ′ (x) = ∴ ≤ f ′ (x) ≤
0 2 + sin πx
4
3 2
π 1
−1
2 On integrating between the limits to x, we get
2
h (1) = 1 − ∫ f (t ) cos t dt > 0 x 192x3 x x 192 x3
∫1/ 2 dx ≤ ∫ f ′ (x)dx ≤ ∫
0
dx
∴ Option (c) is correct. 3 1/ 2 1 /2 2
312 Definite Integration

192  4 1  4 3 1 4x2 
⇒  x −  ≤ f (x) − f (0) ≤ 24x − = ∫ (x2 − 1) (1 − x)4 + (1 + x2) − 4x +  dx
12  16 2 (1 + x2) 
0

3 1
⇒ 16x4 − 1 ≤ f (x) ≤ 24x4 − 4 
2 = ∫  (x2 − 1) (1 − x)4 + (1 + x2) − 4x + 4 −  dx
0 1 + x2 
1
Again integrating between the limits to 1, we get 1  6 4 
2 =∫  x − 4x + 5x − 4x + 4 −
5 4 2
 dx
0  1 + x2 
1 1 1  4 3
∫1/ 2 (16x − 1) dx ≤ ∫1/ 2 f (x) dx ≤∫1/ 2 24x − 2 dx
4
1
 x7 4x6 5x5 4x3 
= − + − + 4x − 4 tan −1 x
16x5  1
1
24x5 3 
1
 7 6 5 3 0
⇒  − x ≤ ∫ f (x)dx ≤  − x
 5 1/ 2 1/ 2  5 2 1/ 2 1 4 5 4 π  22
= − + − + 4 − 4  − 0 = −π
7 6 5 3 4  7
 11 2 1  33 6
⇒  +  ≤ ∫ f (x)dx ≤  +  π sin nx
5 5 1/ 2  10 10 55. Given In = ∫ dx …(i)
1
−π (1 + π x ) sin x
⇒ 2 .6 ≤ ∫ f (x) dx ≤ 3 . 9 b b
∫ a f (x) dx = ∫ a f (b + a − x) dx, we get
1/ 2
Using
(*) None of the option is correct.
4π π π x sin nx
53. Let I1 = ∫ et (sin 6 at + cos 4 at )dt In = ∫ dx …(ii)
0 −π (1 + π x ) sin x
π t
= ∫ e (sin 6 at + cos 4 at ) dt On adding Eqs. (i) and (ii), we have
0
2π π sin nx π sin nx
+∫ et (sin 6 at + cos 4 at ) dt 2I n = ∫ dx = 2 ∫ dx
π − π sin x 0 sin x
3π t sin nx
+∫ e (sin 6 at + cos 4 at ) dt [Q f (x) = is an even function]

4π t
sin x
+∫ e (sin 6 at + cos 4 at ) dt π sin nx
3π ⇒ In = ∫ dx
0 sin x
∴ I1 = I 2 + I3 + I 4 + I5 …(i)
π sin (n + 2 )x − sin nx
2π Now, I n + 2 − I n = ∫ dx
Now, I3 = ∫ e (sin at + cos at ) dt
t 6 4
0 sin x
π
π 2 cos (n + 1 ) x ⋅ sin x
Put t = π + x⇒ dt = dx =∫ dx
π
0 sin x
π +x
∴ I3 = ∫ e ⋅ (sin 6 at + cos 4 at ) dt = eπ ⋅ I 2 …(ii) π
0 π sin (n + 1) x 
3π t = 2 ∫ cos (n + 1) x dx = 2   =0
Now, I4 = ∫ e (sin 6 at + cos 4 at ) dt 0
 (n + 1)  0

Put t = 2π + x ⇒ dt = dx ∴ In + 2 = In …(iii)
π sin nx

π
I 4 = ∫ ex + 2π (sin 6 at + cos 4 at ) dt = e2π ⋅ I 2 …(iii) Since, In = ∫ dx ⇒ I1 = π and I 2 = 0
0
0 sin x
4π t From Eq. (iii) I1 = I3 = I5 = .... = π
and I5 = ∫ e (sin 6 at + cos4 at ) dt

and I 2 = I 4 = I 6 = ... = 0
Put t = 3π + x 10 10

∴ I5 = ∫ e
π 3 π +x
(sin at + cos at ) dt = e
6 4 3π
⋅ I2 …(iv)
⇒ ∑ I 2m + 1 = 10 π and ∑ I 2m = 0
0 m=1 m =1

From Eqs. (i), (ii), (iii) and (iv), we get ∴ Correct options are (a), (b), (c).
I1 = I 2 + eπ ⋅ I 2 + e2π ⋅ I 2 + e3 π ⋅ I 2 = (1 + eπ + e2π + e3 π ) I 2 2

56. The integral, I = ∫ |2x − [3x]| dx
1
∫0 e (sin at + cos at )dt
t 6 4
4/3 5 /3 2
∴ L= π t = ∫ |2x − 3| dx + ∫4/3 |2x − 4|dx + ∫1|2x − 5|dx
∫0 e (sin at + cos at )dt
6 4 1
4/3 5 /3 2
=∫ (3 − 2x) dx + ∫4/3 (4 − 2x) dx + ∫5/3 (5 − 2x) dx
π 2π 1⋅ (e4π − 1)
3π 1
= (1 + e + e + e )= for a ∈ R
eπ − 1 = [3x − x2]14/3 + [4x − x2]54//33 + [5x − x2]52/3
1 x4 (1 − x)4 1 (x4 − 1 ) (1 − x)4 + (1 − x)4
   20 25 16 16
54. Let I = ∫ dx = ∫ dx = 4 −
16
− 3 + 1 +  − − + 
0 1+ x 2 0 (1 + x2)  9  3 9 3 9
1 1 (1 + x2 − 2x)2  25 25
= ∫ (x2 − 1) (1 − x)4 dx + ∫ dx + 10 − 4 − + 
0 0 (1 + x2)  3 9
Definite Integration 313

 20 16 25  16 25 16 25 Put 3 tan x = t ⇒ 3 sec2 dx = dt , and at x = 0, t = 0 and


= (2 + 6) +  − −  + − − + + 
3 3 3  9 9 9 9 at x = 3, t = 3
2 3 1 dt 2
=8 −
21
=8 − 7 =1 So, I= ∫ = [tan −1 t ]03
3 π 0 3 1 + t2 3π
2  π 2
Hence, answer 1.00 is correct. =   = ⇒ 27I 2 = 4.00
3π  3  3 3
Key Idea Use property ∫ f ( x) dx = ∫ f ( a − x) dx
a a
57. 1/ 2 1+ 3
0 0
59. Let I=∫ dx
The given integral 0 [(x + 1)2 (1 − x)6 ]1/ 4
π/ 2
3 cos θ 1+ 3
∫(
1/ 2
I= dθ … (i) ⇒ I=∫ dx
0
cos θ + sin θ )5  1 − x  6 
0 1/ 4

π/ 2 (1 − x)2   
3 sin θ  1 + x 
⇒ I= ∫( sin θ + cos θ )5
dθ …(ii)
0 1−x − 2 dx
a a Put =t ⇒ = dt
1+ x (1 + x)2
[Using the property ∫ f (x) dx = ∫ f (a − x) dx]
1 1
0 0 when x = 0, t = 1, x = ,t =
Now, on adding integrals (i) and (ii), we get 2 3
π/ 2 1/3 (1 + 3 ) dt
3 ∴ I=∫
2I = ∫( sin θ + cos θ )4
dθ 1 −2(t )6/ 4
0
π/ 2 1/3
3 sec2 θ − (1 + 3 )  −2 
⇒ I=
= ∫ (1 + tan θ )4

2  t 
1
0

Now, let tan θ = t 2 ⇒ sec2 θ dθ = 2 t dt ⇒ I = (1 + 3 ) ( 3 − 1)


π ⇒ I =3 −1 =2
and at θ = , t → ∞
2
[x], x ≤ 2
and at θ = 0, t → 0 60. Here, f (x) = 
∞ 6 t dt ∞ t + 1 −1  0, x > 2
So, 2 I = ∫ =6 ∫ dt
0 (1 + t )4 0 (t + 1 )4 2 x f (x2)
∴ I=∫ dx
⇒ −1 2 + f (x + 1)

 ∞ dt ∞ dt   1 1  0 x f (x2) 1 x f (x2)
I = 3 ∫ − ∫0 (t + 1)4   2(t + 1)2 3(t + 1)3 
 = 3  − +  =∫ dx + ∫0 dx
 0 (t + 1)
3
0
−1 2 + f (x + 1) 2 + f (x + 1)
2 x f (x2) 3 x f (x2)
⇒ I =3
1 1 

 1 1
= 3   = ⇒ I = 0.5
+ ∫1 2 + f (x + 1)
dx + ∫ 2 2 + f (x + 1)
dx
2 3   6 2
2 x f (x2)
2 π/4
58. Given, I = ∫
dx
…(i)
+ ∫3 2 + f (x + 1)
dx
π −π/4 (1 + esin x ) (2 − cos 2x)
0 1 2 x⋅1
b
On applying property ∫ f (x)dx = ∫ f (a + b − x) dx,
b
=∫ 0 dx + ∫0 0 dx + ∫1 dx
a a
−1 2+0
we get 3 2
2 π/4 esin x dx
+ ∫ 0 dx + ∫3 0 dx
∫−π/4
2
I= ....(ii)
π (1 + esin x
) (2 − cos 2x) Q − 1 < x < 0 ⇒ 0 < x2 < 1 ⇒ [x2] = 0,
On adding integrals (i) and (ii), we get 0 < x < 1 ⇒ 0 < x2 < 1 ⇒ [x2] = 0,
2 π/4 dx
2I = ∫  1 < x2 < 2 ⇒ [x2] = 1
π − π/4 2 − cos 2x 1 < x< 2 ⇒
1 π/4 dx  1 − tan 2 x  2 < x + 1 < 1 + 2 ⇒ f (x+ 1) = 0,
⇒ I= ∫  as cos 2 x = 
π − π/4 1 − tan 2 x 1 + tan 2 x  2 < x < 3 ⇒ 2 < x2 < 3 ⇒ f (x2) = 0,
2− 
1 + tan x2 and 3 < x < 2 ⇒ 3 < x2 < 4 ⇒ f (x2) = 0
2 π/4 sec2 x 2
= ∫ dx 2 x  x2  1 1
π 0 1 + 3 tan 2 x ⇒ I=∫ dx =   = (2 − 1) =
1 2 4
 1 4 4
 sec2 x 
Q is even function 
 1 + 3 tan 2
x  ∴ 4I = 1 ⇒ 4I − 1 = 0
314 Definite Integration

1 −1  12 + 9x2 16 e sin t dt
61. Here, α = ∫ e( 9x + 3 tan
0
x)

 1 + x2 
 dx ⇒ ∫1 2
t

2
= F (k) − F (1)

Put 9x + 3 tan −1 x = t 16 e sin t


 3 
⇒ ∫1 t
dt = F (k) − F (1)
⇒ 9 +  dx = dt
 1 + x2  ⇒ 1 = F (k ) − F (1 )
[F (t )]16
9+3π /4 t  d e sin x 
∴ α=∫ e dt = [et ]90 + 3 π / 4 = e9 + 3 π / 4 − 1 Q dx { F (x)} = x , given 
0
3π 3π  
⇒ log e 1 + α = 9 + ⇒ log e α + 1 − =9
4 4 ⇒ F (16) − F (1) = F (k) − F (1)

62. PLAN Integration by parts


∴ k = 16
37 π π sin (π log x)
d  65. Let I = ∫
∫ f(x ) g (x ) dx = f(x ) ∫ g (x ) dx − ∫  dx [f(x )] ∫ g (x ) dx  dx 1 x
dx

1 d2 Put π log x = t
Given, I = ∫ 4x3 (1 − x2)5 dx π
0 I dx2 II ⇒ dx = dt
1
x
 d  1 d
(1 − x2)5 − ∫ 12 x2
37π
= 4x3 (1 − x2)5 dx ∴ I=∫ sin (t ) dt = − [cos t ]37 π
= − [cos 37π − cos 0]
 dx  0 0 dx 0
0

1
= 4x3 × 5 (1 − x2)4 (− 2x) = − [(−1) − 1] = 2
 0 2π x sin x 2n
66. Let I = ∫ dx …(i)

− 12 [x (1 − x ) ]0 − ∫ 2x (1 − x2)5 dx
2 25 1 1
0 sin 2n x + cos 2n x
 0 
2π (2π − x)[sin (2π − x)]2n
I=∫
1
= 0 − 0 − 12 (0 − 0) + 12 ∫ 2x (1 − x2)5 dx dx
0 0 [sin (2π − x)]2n + [cos (2π − x)]2n
 (1 − x ) 2 6 1 a a
 1 [Q ∫ f (x) dx = ∫ f (a − x) dx]
= 12 × −  = 12 0 + 6  = 2 0 0
 6 0   2π (2π − x) ⋅ sin 2n x
I=∫ dx
63. Given, f (1) =
1 x
and 6 ∫ f (t )dt = 3x f (x) − x3 , ∀ x ≥ 1
0 sin 2n x + cos 2n x
3 1
2π 2π sin 2n x 2π x sin 2n x
Using Newton-Leibnitz formula. ⇒ I=∫ dx − ∫ dx
0 sin x + cos x
2n 2n 0 sin n x + cos 2n x
2
Differentiating both sides
⇒ 6 f (x) ⋅ 1 − 0 = 3 f (x) + 3xf ′ (x) − 3x2
2π 2π sin 2n x
⇒ I=∫ dx − I [from Eq. (i)]
1 0 sin 2n x + cos 2n x
⇒ 3xf ′ (x) − 3 f (x) = 3x2 ⇒ f ’ (x) − f (x) = x
x 2π π sin 2n x
xf ’ (x) − f ’ (x) ⇒ I=∫ dx
⇒ =1 0 sin x + cos 2n x
2n
x2
d x  π π sin 2n x
⇒  =1 ⇒ I = π ∫ dx
 0 sin x + cos x
2n 2n
dx  x 
On integrating both sides, we get π sin 2n (2π − x) 
+∫ dx
f (x)  1 0 sin (2 π − x) + cos 2n (2 π − x)
2n
⇒ = x+ c Q f (1) = 
x`  3  using property 
 2a f (x) dx = a [f (x) + f (2a − x)]dx
∫ 0 ∫0
1 2 2
= 1 + c ⇒ c = and f (x) = x2 − x 
3 3 3
4 8  π sin 2n x dx
∴ f (2) = 4 − = I = π ∫ dx
 0 sin x + cos x
3 3 2n 2n

NOTE Here, f(1) = 2, does not satisfy given function. π sin 2n x 


1
+∫ dx
∴ f(1 ) = 0 sin x + cos x 
2n 2n
3 π sin 2n x dx
2 4 8 ⇒ I = 2π ∫ dx
0 sin 2n x + cos 2n x
For that f (x) = x2 − x and f (2) = 4 − =
3 3 3
 π /2 sin 2n x dx 
I = 4π ∫
2
4 2e sin x ⇒ dx …(ii)
∫1 dx = F (k) − F (1) +
2n 2n
64. Given,  0 sin x cos x 
x
x2 = t π /2 sin 2n (π / 2 − x)
Put ⇒ I = 4π ∫ dx
⇒ 2x dx = dt 0 sin (π / 2 − x) + cos 2n (π / 2 − x)
2n
Definite Integration 315

π /2 cos 2n x 3 π /4 π−x
⇒ I = 4π ∫ dx …(iii) =∫ dx
0 cos 2n x + sin 2n x π /4 1 + sin (π − x)
3 π /4 π 3 π /4 x
On adding Eqs. (ii) and (iii), we get =∫ dx − ∫ π /4 dx
π /4 1 + sin x 1 + sin x
π / 2 sin 2n x + cos 2n x
2I = 4π ∫ dx 3 π /4 dx
x + cos 2n x = π∫
2n
0 sin −I [from Eq. (i)]
π /4 1 + sin x
π /2 π
⇒ 2I = 4π ∫ 1 dx = 4π [x]π0 / 2 = 4π ⋅ ⇒ I = π 2 π 3 π /4 dx
0 2 =
2 ∫ π /4 (1 + sin x)
1
67. Given, af (x) + bf (1 / x) = −5 …(i)
π 3 π /4 (1 − sin x)
x
Replacing x by 1 / x in Eq. (i), we get
=
2 ∫ π /4 (1 + sin x) (1 − sin x)
dx

af (1 / x) + bf (x) = x − 5 …(ii) π 3 π /4 (1 − sin x)


On multiplying Eq. (i) by a and Eq. (ii) by b, we get
=
2 ∫ π /4 1 − sin 2 x
dx

1  π 3 π /4  1 sin x 
a 2f (x) + abf (1 / x) = a  − 5
x 
…(iii) =
2 ∫ π /4  −  dx
 cos 2 x cos 2 x
abf (1 / x) + b2f (x) = b (x − 5) …(iv) π 3 π /4
2 ∫ π /4
= (sec2 x − sec x ⋅ tan x) dx
On subtracting Eq. (iv) from Eq. (iii), we get
a π
(a 2 − b2) f (x) = − bx − 5a + 5b = [tan x − sec x]3π π/ 4/ 4
x 2
1 a  π
⇒ f (x) = 2  − bx − 5a + 5b = [− 1 − 1 − (− 2 − 2 )]
(a − b2)  x  2
π
2 1 2 a  = (− 2 + 2 2 ) = π ( 2 − 1)
⇒ ∫ 1 f (x) dx = (a 2 − b2) ∫ 1  x − bx − 5a + 5b dx 2
2
∫ −2 |1 − x |dx
2
1  b 
2 70.
= 2 a log|x| − x2 − 5(a − b)x
(a − b2)  2  −1 1 2
1
=∫ (x2 − 1) dx + ∫ (1 − x2) dx + ∫ (x2 − 1) dx
1 −2 −1 1
= [a log 2 − 2b − 10 (a − b) −1 1 2
(a 2 − b2)  x3   x3   x3 
= − x + x −  + − x
b   3  −2  3  −1  3 1
− a log 1 + + 5 (a − b)
2 
 1 8   1 1  8 1 
=  − + 1 + − 2 + 1 − + 1 −  +  − 2 − + 1
1  7   3 3   3 3   3 3 
= 2 
a log 2 − 5a + b
(a − b ) 
2
2  =4
3 x 1.5 1 2 1.5
68. Let I=∫ dx …(i) 71. ∫0 [x2] dx = ∫ 0 dx + ∫ 1 dx + ∫ 2 dx
2 5−x+ x 0 1 2

3 2+3−x = 0 + [x]1 2 + 2 [x]1.52


⇒ I=∫ dx
2 (2 + 3) − (5 − x) + 2 + 3 − x = ( 2 − 1) + 2 (1.5 − 2 )
3 5−x = 2 −1 + 3 −2 2 =2− 2
⇒ I=∫ dx …(ii)
2 x + 5 −x 72. (A) Let I = ∫
1 dx
−1 1 + x2
On adding Eqs. (i) and (ii), we get
Put x = tan θ
3 x+ 5−x 3 1
2I = ∫ dx ⇒ 2I = ∫ 1 dx = 1 ⇒ I = ⇒ dx = sec2 θ dθ
2 5−x+ x 2 2 π /4 π
∴ I = 2∫ dθ =
3 π /4 x 0 2
69. Let I = ∫ dx …(i)
π /4 1 + sin x 1 dx
 π 3π 
(B) Let I= ∫0 1 − x2
 + − x
3 π /4 4 4 
⇒ I=∫ dx Put x = sinθ
π /4  π 3π 
1 + sin  + − x ⇒ dx = cosθ d θ
4 4 
π/ 2 π
b
a
b
[Q ∫ f (x) dx = ∫ f (a + b − x) dx]
a
∴ I= ∫0 1 dθ =
2
316 Definite Integration

3 a
3 dx1  1 + x  (S) PLAN ∫− a f( x ) dx = 0
(C) ∫ 2 1 − x 2 2 log  1 − x  
=
If f( − x ) = − f( x ), i.e. f( x ) is an odd function.
2
1  4  3  1   2   1 + x
= log   − log    = log    Let f ( x ) = cos 2x log  
2  −2  −1  2   3   1 − x
2 dx π π  1 − x
(D) ∫ = [sec−1 x]12 = −0 = f (− x) = cos 2x log 
1
x x −1 2 3 3  = − f (x)
 1 + x
73. (P) PLAN (i) A polynomial satisfying the given conditions is taken. Hence, f (x) is an odd function.
(ii) The other conditions are also applied and the number 1/ 2
of polynomial is taken out. So, ∫−1/ 2 f (x) dx = 0
Let f ( x ) = ax 2 + bx + c
(P) → (ii); (Q) → (iii); (R) → (i); (S) → (iv)
f (0) = 0 ⇒ c = 0 1
1 74. Let I 2 = ∫ (1 − x50)101 dx,
Now, ∫0 f (x) dx = 1 0
1
= [(1 − x50 )101 ⋅ x]10 + ∫ 0 (1 − x 50 ⋅ x49⋅x dx
50 100
1 )
 ax3 bx2 α β
⇒  +  =1 ⇒ + =1
 3 2 0 3 2 [using integration by parts]
1
⇒ 2a + 3b = 6 =0 − ∫ (50) (101) (1 − x50 )100 (− x50 ) dx
0
As a , b are non-negative integers. 1
= − (50) (101) ∫ (1 − x50 )101 dx
So, a = 0, b = 2 or a = 3, b = 0 0
1
∴ f (x) = 2x or f (x) = 3x2 + (50) (101) ∫ (1 − x50 )100 dx = 5050I 2 + 5050I1
0
(Q) PLAN Such type of questions are converted into only sine or
(5050)I1
cosine expression and then the number of points of ∴ I 2 + 5050I 2 = 5050I1 ⇒ = 5051
maxima in given interval are obtained. I2
f ( x ) = sin ( x 2 ) + cos ( x 2 )
75. Let
 1 1  π  1  1 
= 2
 2
cos (x2) + sin (x2)
 I = ∫ e |cos x | 2 sin  cos x + 3 cos  cos x  sin x dx
2 0  2  2 
 π π  π 1 
= 2 cos x2 cos + sin sin (x2)
  ⇒ I = ∫ e |cos x | ⋅ sin x ⋅ 2 sin  cos x dx
4 4 0 2 
 π π |cos x | 1 
= 2 cos  x2 − 
 4
+ ∫ 0e ⋅ 3 cos  cos x ⋅ sin x dx
2 
π π ⇒ I = I1 + I 2 …(i)
For maximum value, x2 − = 2 n π ⇒ x2 = 2 n π +
4 4 using 2 a 
π 9π  ∫ 0 f (x) dx 
⇒ x=± ,for n = 0 ⇒ x = ± , for n = 1   0, f (2a − x) = − f (x) 
4 4  = a 
So, f (x) attains maximum at 4 points in [− 13 , 13].  2 ∫ 0 f (x) dx, f (2a − x) = + f (x)
(R) PLAN where, I1 = 0 [Q f (π − x) = − f (x)] …(ii)
a a
(i) ∫− a f( x ) dx = ∫− a f( − x ) dx
and I 2 = 6∫
π / 2 cos x
e
1 
⋅ sin x ⋅ cos  cos x dx
a a 0 2 
(ii) ∫− a f( x ) dx = 2 ∫0 f( x ) dx , if f( − x ) = f( x ) , i.e. f is an even
1  t
function. Now, I 2 = 6 ∫ et ⋅ cos   dt
0  2
2 3x 2
I= ∫−2 1 + ex dx
[put cos x = t ⇒ − sin x dx = dt]
1
  t 1 t 
I=∫
3 x2
2 = 6 et cos   + ∫ et sin dt 
and
−2 1 + e− x
dx   2  2 2 0
1
2  3x2 3x2(ex )   t 1  t et t 
⇒ 2I = ∫  + x  dx = 6 et cos   +  et sin − ∫ cos dt 
−2 1 + ex
e +1  2 2  2 2 2 
 0
2 2 1
2I = ∫ 3x2 dx ⇒ 2I = 2 ∫ 3x2 dx  t 1
= 6 et cos + et sin
t I
− 2
−2 0
 2 2 2  0 4
I = [x3 ]20 = 8
Definite Integration 317

24   1 e  1  − π /4  π 
=
5 e cos  2 + 2 sin  2 − 1
 
…(iii) ∴ 2I = − 2 ∫ π /4 f cos  − 2t  cos t dt
  2 
π /4
From Eq. (i), we get ⇒ 2I = 2 ∫ −π / 4 f (sin 2t ) cos t dt
24   1 e  1 
I= e cos   + sin   − 1 π /4
5   2 2  2  ∴ I= 2 ∫0 f (sin 2t ) cos t dt
π /3 π dx π /3 x dx 3
76. Let I = ∫ + 4∫ π ecos x
− π /3  π − π /3  π 78. Let I = ∫ dx …(i)
2 − cos |x|+  2 − cos |x|+  0 ecos x + e− cos x
 3  3
f (− x) = − f (x)
π ecos ( π − x )
a  0, =∫ cos ( π − x )
dx
Using ∫ f (x) dx =  a 0 e + e− cos ( π − x )
 ∫ 0
−a 2 f (x) dx, f (− x) = f (x) a a
[Q∫ f (x) dx = ∫ f (a − x) dx]
0 0
π /3 π dx
∴ I =2 ∫ +0
 π π e− cos x
⇒ I=∫
0
2 − cos |x| +  0 e− cos x + ecos x
dx …(ii)
 3
  On adding Eqs. (i) and (ii), we get
 x3 dx 
Q is odd  π ecos x + e− cos x π
=∫ dx = ∫ 1 dx = [x] π0 = π
 2 − cos |x|+ π   0 ecos x + e− cos x 0
  3 
π /3 dx ⇒ I = π /2
I = 2π ∫0 2 − cos (x + π / 3) 1 −1  1  1  1−x+ x 
79. ∫ 0 tan   dx = ∫ tan −1   dx
1 − x + x  1 − x(1 − x) 
2 0
π
Put x+ = t ⇒ dx = dt 1
3 = ∫ [ tan −1 (1 − x) − tan −1 x] dx
t 2 0
2 π /3 2 π /3
sec dt
dt
∴ I = 2π ∫ = 2π ∫ 2 1
= ∫ tan −1 [1 − (1 − x) ] dx +
1 −1
π / 3 2 − cos t π /3
1 + 3 tan 2
t 0 ∫ 0 tan x dx
2
= 2∫ tan −1x dx Q ∫ f (x) dx = ∫ f (a − x) dx …(i)
1 a a

Put
t
tan = u ⇒ sec2 dt = 2 du
t 0  0 0 
2 2
1  1 
3 2 du 4π Now, ∫ tan −1   dx
⇒ I = 2π ∫ = [ 3 tan −1 3u ] 13 0 1 − x + x 
2
1/ 3 1 + 3u 2 3
1 π 
3
 1
4π 4π  1 = ∫  − cot−1   dx
2 
= (tan −1 3 − tan −1 1) = tan −1   0 2
 1 − x + x  
3 3  2
π 1
π /3 π + 4x3 4π  1 = − ∫0 tan −1 (1 − x + x2) dx
∴ ∫ −π /3 dx = tan −1   2
 π  3  2 1 π 1 1
2 − cos |x|+  tan −1 (1 − x + x2) dx = tan −1
 3  ∴ ∫0 2 ∫0

(1 − x + x2)
dx
π /2
77. Let I = ∫ f (cos 2x) cos x dx …(i) π
0 = − 2I1
2
π /2  π  π  1 1 x dx
∫ 0 tan ∫ 0 1 + x2
−1
⇒ I=∫ f  cos 2  − x  ⋅ cos  − x dx where, I1 = x dx = [x tan−1 x ]10 −
0  2  2 
using a f (x) dx = a f (a − x) dx π 1 π 1
 ∫0 ∫0  4 2
=
− [log(1 + x 2 )]10 = − log 2
4 2
π /2
⇒ I=∫ f (cos 2x) sin x dx …(ii) 1 π π 1 
0 ∴ ∫ tan−1(1 − x + x 2 ) dx = − 2  − log 2 = log 2
0 2  4 2 
On adding Eqs. (i) and (ii), we get π /4
π /2 80. Let I = ∫ log (1 + tan x) dx …(i)
2I = ∫ f (cos 2x) (sin x + cos x) dx 0
0 π /4 π
π /2 I=∫ log (1 + tan ( − x)) dx
= 2 ∫0 f (cos 2x) [cos (x − π / 4)] dx 0 4
π /4  1 − tan x 
π ∴ I=∫ log 1 +  dx
Put − x + = t ⇒ − dx = dt 0  1 + tan x
4
318 Definite Integration

π /4  1 + tan x + 1 − tan x Now, cos −1 (− x) = π − cos −1 x for −1 ≤ x ≤ 1.


=∫ log   dx
0  1 + tan x  1/ 3 y4  −1  2 y 

∴I=∫ π − cos   dy
2 
π /4  2  π /4 −1/ 3 1 − y4
  1 + y  
I=∫ log   dx ⇒ I = ∫ log 2 dx − I
0  1 + tan x 0
1/ 3 y4 1/ 3 y4  2y 
= π∫ dy − ∫ cos −1   dy
π π −1/ 3 1− y 4 −1/ 3 1 − y4  1 + y2 
⇒ 2I = log 2 ⇒ I = (log 2)
4 8
1/ 3 x4 1/ 3 x4  2x 
π 2 x (1 + sin x) =π∫ dx − ∫ cos −1   dx
81. Let I=∫ dx −1/ 3 1−x 4 −1/ 3 1 − x4  1 + x2
− π 1 + cos 2 x

π 2x π 2x sin x 1/ 3 x4
I=∫ dx + ∫ dx ⇒ I = π∫ dx − I [from Eq. (i)]
− π 1 + cos x
2 − π 1 + cos 2 x −1/ 3 1 − x4
⇒ I = I1 + I 2 1/ 3 x4 1/ 3  1 
π
⇒ 2I = π ∫ dx = π ∫  −1 +  dx
Now, I1 = ∫
2x
dx
−1/ 3 1−x 4 −1/ 3
 1 − x4 
− π 1 + cos 2 x
1/ 3 1/ 3 dx
2x =−π∫ 1 dx + π ∫ −1/
Let f (x) = −1/ 3 3 1 − x4
1 + cos 2 x
−2 x − 2x 1/ 3 dx
⇒ f (− x) = = = − f (x) = − π [x]1−/1/3 3 + π I1, where I1 = ∫
1 + cos (− x) 1 + cos 2 x
2 −1/ 3 1 − x4
⇒ f (− x) = − f (x) which shows that f (x) is an odd  1 1 2π
⇒ 2I = − π  +  + πI1 = − + π I1
function.  3 3 3
∴ I1 = 0 1/ 3 dx 1/ 3 dx
Again, let g (x) =
2x sin x Now, I1 = ∫ −1/ 3 1−x 4
=2 ∫
0 1 − x4
1 + cos 2 x
[since, the integral is an even function]
2 (− x)sin (− x) 2x sin x
⇒ g (− x) = = = g (x) 1/ 3 1 + 1 + x2 − x2
1 + cos 2(− x) 1 + cos 2 x = ∫0 (1 − x2) (1 + x2)
dx

⇒ g (− x) = g (x) which shows that g (x) is an even 1/ 3 1 1/ 3 1


function. = ∫0 1−x 2
dx + ∫
0 1 + x2
dx
π 2 x sin x π x sin x
∴ I2 = ∫ dx = 2 ⋅ 2 ∫ dx
− π 1 + cos x
2 0 1 + cos 2 x 1/ 3 1 1/ 3 1
π (π − x)sin (π − x) (π − x)sin x
π
= ∫0 (1 − x) (1 + x)
dx + ∫
0 (1 + x2)
dx
=4 ∫ dx = 4 ∫ dx
0 1 + [cos (π − x)]2 0 1 + cos 2x 1 1/ 3 1 1 1/ 3 1 1/ 3 1
π sin x
π π x sin x
=
2 ∫0 1−x
dx + ∫
2 0 1+ x
dx + ∫
0 1 + x2
dx
=4 ∫ dx − 4 ∫ dx
0 1 + cos 2x 0 1 + cos 2x 1/ 3
 1 1 
= − ln|1 − x| + ln|1 + x| + tan −1 x
 2  0
π
sin x
⇒ I 2 = 4π ∫ dx − I 2 2
0 1 + cos 2x
1/ 3
π 1   1 + x 
⇒ 2I 2 = 4π ∫
sin x =   + [tan −1 x]01/ 3
dx ln 1 − x 
0 1 + cos 2x 2    0
Put cos x = t 1  1 + 1 / 3
= ln   + tan −1 1
⇒ − sin x dx = dt 2 1 − 1 / 3 3
−1 dt 1 dt 1 dt
∴ I2 = − 2 π ∫ =2 π ∫ −1 1 + t 2 = 4 π ∫ 0 1 + t 2 1  3 + 1
+ π
1 1 + t2 = ln 
2  3 − 1 6
= 4π [tan −1 t ] 01 = 4π [tan −1 1 − tan −1 0]
1  ( 3 + 1 )2 
+ π
= 4π (π / 4 − 0) = π 2 = ln 
2  3 −1  6
∴ I = I1 + I 2 = 0 + π 2 = π 2
1 π
1/ 3  x4   2x  = ln (2 + 3 ) +
82. Let I = ∫   cos −1   dx …(i) 2 6
− 1/ 3 1 − x 
4
 1 + x2 − 2π π π2
∴ 2I = + ln (2 + 3 ) +
Put x = − y ⇒ dx = − dy 3 2 6
−1/ 3 y4  −2 y  π
∴ I=∫ cos −1   (−1) dy = [π + 3 ln (2 + 3 ) − 4 3 ]
1/ 3 1− y 4
 1 + y2  6
Definite Integration 319

π 3
⇒ I= [π + 3 ln (2 + 3 ) − 4 3 ]  1  
3
1
12 = log (x − 1) −  log (x + 1)
2  2
Alternate Solution
2   2
π 1 2 1 4
Since, cos −1 y = − sin −1 y = log − log
2 2 1 2 3
−1  2 x  π −1 2x π From Eq. (i), I = I1 + I 2
∴ cos   = − sin = − 2 tan −1 x
 1 + x2  2 1 + x2 2 1 1 1 4
= log 2 − + log 2 − log
π 4  4 10 2 2 3
1/ 3 x x
I=∫ 2 ⋅ − 2 tan −1 x dx −1/ 2
 4 1 1 1
 1−x 1−x
−1/ 3 4 4
 = log [2 ⋅ 21/ 2  ]− = log 6 −
 3 10 2 10
 x4 
Q 2 tan −1 x is an odd function  84. Since, f (x) is a cubic polynomial. Therefore, f ′ (x) is a
 1 − x 4

quadratic polynomial and f (x) has relative maximum
π
1
 1  1
∴ I =2⋅
2 ∫03  −1 +

 dx + 0
1 − x4 
and minimum at x =
3
and x = − 1 respectively,
therefore, –1 and 1/3 are the roots of f ′ (x) = 0.
π 1/ 3  1 1   1  1
= ∫0  −2 +

+  dx
1 − x 1 + x2 
2 ∴
1
f ′ (x) = a (x + 1)  x −  = a  x2 − x + x − 
  
2 3 3 3
π  1 1+ x −1 
1/ 3
 2 1
= = a  x2 + x − 
−2x + 2 ⋅ 1 log 1 − x + tan x  3 3
2  0
Now, integrating w.r. t. x, we get
π  2 1 3 + 1 π
= − + log +   x3 x2 x
2  3 2 3 −1 6 f (x) = a  + − +c
3 3 3
π
= [π + 3 log (2 + 3 ) − 4 3 ]
12 where, c is constant of integration.
3 2x + x − 2x + 2x + 1
5 4 3 2 Again, f (−2) = 0
83. Let I = ∫ dx  8 4 2
2 (x2 + 1) (x4 − 1) ∴ f (−2) = a  − + +  + c
 3 3 3
3 2x5 − 2x3 + x4 + 1 + 2x2
=∫ dx  −8 + 4 + 2 
2 (x2 + 1) (x2 − 1)(x2 + 1) ⇒ 0=a +c
 3 
3 2x3 (x2 − 1) + (x2 + 1)2 −2 a
=∫ dx ⇒ 0= + c ⇒ c=
2a
2 (x2 + 1)2 (x2 − 1) 3 3
3 2x3 (x2 − 1) 3 (x2 + 1)2  x3 x2 x 2a a 3
=∫ dx + ∫ 2 (x2 + 1)2(x2 − 1) dx ∴ f (x) = a  + − + = (x + x2 − x + 2)
2 (x2 + 1)2(x2 − 1) 3 3 3 3 3
3 2x3 3 1 1 14
=∫ dx + ∫ 2 (x2 − 1) dx Again, ∫ f (x) dx = [given]
2 (x + 1 )2
2 −1 3
⇒ I = I1 + I 2 1 a 14
⇒ ∫ −1 3 (x + x2 − x + 2) dx =
3
3
3 2x 3
Now, I1 = ∫ dx
2 (x2 + 1 )2 1 a 14
⇒ ∫ −1 3 (0 + x + 0 + 2) dx = 3
2

Put x 2 + 1 = t ⇒ 2x dx = dt
10 (t − 1) 10 1 10 1
[Q y = x3 and y = − x are odd functions]
∴ I1 = ∫ dt = ∫ dt − ∫ dt a 1 2
2 x dx + 4∫ 1 dx =
1 14
3  ∫ 0
5 t 2 5 t 5 t2 ⇒
10
0  3
1 
= [log t ]10
5 +  2x3
1
  14
 t  a
5 ⇒  + 4x  =
3   3  0 3
1 1 1
= log 10 − log 5 + − = log 2 −
10 5 10 a 2  14
⇒  + 4 =
3 1 3 1 3 3  3
Again, I 2 = ∫ dx = ∫ dx
2 (x2 − 1 ) 2 (x − 1 )(x + 1 ) a  14 14
⇒   = ⇒ a =3
1 3 1 1 3 1 3 3 3
=
2 ∫ 2 (x − 1) dx − 2 ∫ 2 (x + 1) dx Hence, f (x) = x3 + x2 − x + 2
320 Definite Integration

π  87. We know that,


x sin (2x) ⋅ sin  cos x
π 2 
85. Let I=∫ dx …(i) 2 sin x [cos x + cos 3x + cos 5x + K + cos (2k − 1) x]
0 (2x − π )
= 2 sin x cos x + 2 sin x cos 3x + 2 sin x cos 5x
π 
(π − x) ⋅ sin 2 (π − x) ⋅ sin cos(π − x) + K + 2 sin x cos (2k − 1) x
π  2 
Then I=∫ dx = sin 2x + (sin 4x − sin 2x) + (sin 6x − sin 4x)
0 2 (π − x) − π
…(ii) + K + {sin 2kx − sin (2k − 2) x}
π  = sin 2kx
(π − x) ⋅ sin 2x ⋅ sin  cos x
π 2 
⇒ I=∫ dx ∴ 2 [cos x + cos 3x + cos 5x + K + cos (2k − 1) x]
0 π − 2x sin 2kx
= …(i)
π  sin x
(x − π ) sin 2x ⋅ sin  cos x
π 2  sin 2kx
⇒ I=∫ dx … (iii) Now, sin 2kx ⋅ cot x = ⋅ cos x
0 (2x − π ) sin x
On adding Eqs. (i) and (iii), we get = 2 cos x [cos x + cos 3x + cos 5x + K + cos (2k − 1) x]
π π  [from Eq. (i)]
2I = ∫ sin 2x ⋅ sin  cos x dx
0 2  = [2 cos 2 x + 2 cos x cos 3x + 2 cos x cos 5x +
π π  K + 2 cos x cos (2k − 1) x ]
⇒ 2I = 2 ∫ sin x cos x ⋅ sin  cos x dx
0 2  = (1 + cos 2x) + (cos 4x + cos 2x)
π π  + (cos 6x + cos 4x) + ... + {cos 2kx + cos (2k − 2) x}
⇒ I = ∫ sin x cos x ⋅ sin  cos x dx
0 2  = 1 + 2 [cos 2x + cos 4x + cos 6x + K + cos (2k − 2) x]
 π π 2  + cos 2kx
put cos x = t ⇒ − sin x dx = dt ⇒ sin x dx = − dt π /2
 2 2 π  ∴∫ (sin 2kx) ⋅ cot x dx
0
2 − π / 2 2t
∴ I=− ∫ ⋅ sin t dt π /2 π/ 2
π π /2 π =∫ 1 ⋅ dx + 2 ∫ ( cos 2x + cos 4x K cos (2k − 2) x) dx
0 0
4 π /2 π /2
= 2∫
π −π / 2
t sin t dt + ∫0 cos (2k) x dx
4 π /2
⇒ I = 2 [− t cos t + sin t ]π− π/ 2/ 2 π sin 2x sin 4x sin (2k − 2) x 
π = +2 + +K+
2  2 4 (2k − 2)  0
4 8
= 2 ×2= 2 π /2
π π sin (2k) x  π
+ =
π /2  2k  2
86. Let I=∫ f (sin 2x) sin x dx …(i) a
0
0
88. Let I = ∫ f (x) ⋅ g (x) dx
 π  π
0
π /2 
Then, I = ∫ f sin 2  − x  sin  − x dx a
0   2    2  I = ∫ f (a − x) ⋅ g (a − x) dx
0
π /2
=∫ f [sin 2x] ⋅ cos x dx …(ii) a
0 = ∫ f (x) ⋅ {2 − g (x)} dx
0
On adding Eqs. (i) and (ii), we get
[Q f (a − x) = f (x) and g (x) + g (a − x) = 2]
π /2
2I = ∫ f (sin 2x)(sin x + cos x) dx a a
= 2 ∫ f (x) dx − ∫ f (x) g (x) dx
0
0 0
π /4
=2 ∫ f (sin 2x)(sin x + cos x) dx ⇒
a
I = 2 ∫ f (x) dx − I ⇒ 2I = 2 ∫ f (x) dx
a
0
0 0
π /4  π
=2 2 ∫
a a
f (sin 2x) sin  x +  dx
0  4
∴ ∫ 0 f (x) g(x) dx = ∫ 0 f (x) dx
π /4  π  π π f (x)
∫0
2a
=2 2 f sin 2  − x  sin  − x +  dx 89. Let I=∫ dx …(i)
 4  4 4 0 f (x) + f (2a − x)
π /4
f (2a − x)
∫0
2a
=2 2 f (cos 2x) cos x dx I=∫ dx …(ii)
0 f (2a − x) + f (x)
π /4
∴ I= 2 ∫0 f (cos 2x) cos x dx On adding Eqs. (i) and (ii), we get
2a
π /2 π /4 2I = ∫ 1 dx = 2a ⇒ I = a
Hence, ∫ f (sin 2x) ⋅ sin x dx = 2 ∫ f (cos 2x) cos x dx 0
0 0
Definite Integration 321

1 π
90. Let I = ∫ log ( 1 − x + 1 + x ) dx = [tan −1 (∞ ) − tan −1 (cot α )]
0 sin α
Put x = cos 2θ π π π  απ
=  −  − α   =
⇒ dx = − 2 sin 2θ dθ sin α 2 2  sin α
∴ I = −2 ∫
0
log [ 1 − cos 2θ + 1 + cos 2θ ] (sin 2θ ) dθ απ
π /4 ∴ I=
sin α
0
= −2 ∫ log [ 2 (sin θ + cos θ )] sin 2θ dθ π / 2 x sin x ⋅ cos x
π /4 92. Let I = ∫ dx
0
0 cos 4 x + sin 4 x
= −2 ∫ [(log 2 ) sin 2θ
π /4 π  π  π 
+ log (sin θ + cos θ ) ⋅ sin 2θ ] dθ π /2  2
 − x sin  − x ⋅ cos  − x
 2  2 
0 ⇒ I=∫ dx
 − cos 2θ  0  π   π 
= − 2 log 2 sin 4  − x + cos 4  − x
 2  2  2 
π /4
0 π 
−2 ∫ log (sin θ + cos θ ) ⋅ sin 2θ dθ  − x ⋅ sin x cos x
π /4 π /2  2 
I II ⇒ I=∫ dx
0 cos 4 x + sin 4 x
  cos 2θ 
0
π π /2 sin x cos x π / 2 x sin x ⋅ cos x
= log 2 − 2 − log (sin θ + cos θ ) ⋅  ⇒ I= ∫0 dx − ∫ dx
  2 π / 4 2 sin 4 x + cos 4 x sin 4 x + cos 4 x
0

 cos θ − sin θ − cos 2θ   π π /2 sin x ⋅ cos x


−∫
0

π / 4  cos θ + sin θ
×  dθ 
=
2 ∫0 sin 4 x + cos 4 x
dx − I
2  
  π π / 2 tanx ⋅ sec2x
= log ( 2 ) − 2 0 + ∫

1 0
( cos θ − sin θ )2 dθ

⇒ 2I=
2 ∫0 tan 4 x + 1
dx
2 π /4
π 1 π /2 1
2 2 ∫ 0 1 + (tan 2 x)2
1 0
⇒ 2I= ⋅ d (tan 2 x)
= log 2 − ∫ (1 − sin 2θ ) dθ
2 π /4
0 π π
1  cos 2θ  ⇒ 2 I = ⋅ [tan −1 t ]0∞ = (tan −1 ∞ − tan –1 0)
= log 2 − θ +
2  2  π / 4
4 4
[where, t = tan 2 x]
1 1 π 1 1 π π 2
= log 2 −  −  = log 2 − + ⇒ I=
2 2 4 2 2 4 16

91. Let I = ∫
π x 1/ 2 x sin −1 x
0 1 + cos α sin x
dx …(i) 93. Let I = ∫ dx Put sin −1 x = θ ⇒ x = sin θ
0
1−x 2
π (π − x)
⇒ I=∫ dx ⇒ dx = cos θ dθ
0 1 + cos α sin (π − x)
π /6 θ sin θ π/6
π (π − x) ∴ I=∫ ⋅ cos θ d θ = ∫ θ sin θ d θ
⇒ I=∫ dx … (ii) 0
1 − sin θ
2 0
0 1 + cos α sin x
π/6
On adding Eqs. (i) and (ii), we get = [− θ cos θ ]π0 / 6 + ∫0 cos θ d θ
π dx
2 I = π∫  π π   π  3π 1
0 1 + cos α sin x =  − cos + 0 + sin − sin 0 = − +
 6 6   6  12 2
x
π
sec2 dx π /4 (sin x + cos x)
⇒ 2I=π ∫ 2 94. Let I=∫ dx
0 x x 0 9 + 16 sin 2x
(1 + tan 2 ) + 2 cos α tan
2 2 π /4 sin x + cos x
I=∫ dx
x x
Put tan = t ⇒ sec2 dx = 2 dt
0 25 − 16 (sin x − cos x)2
2 2
Put 4 (sin x − cos x) = t ⇒ 4 (cos x + sin x) dx = dt
∞ 2 dt
∴ 2I = π ∫ 1 0 dt 1 1 5 + t 
0
0 1 + t 2 + 2 t cos α
∴ I=
4 ∫ −4 25 − t 2 = 4 ⋅ 2 (5) log 5 − t 
∞ dt −4
⇒ 2I = 2π ∫0 (t + cos α )2 + sin 2 α  5 + 0 
1   5 − 4 
 

I= log 5 − 0 − log 5 + 4 
π
∞  40   
 −1  t + cos α  
I= tan  sin α   1  1 1 1
sin α  0 =  log 1 − log  = log 9 = (log 3)
40  9 40 20
322 Definite Integration
π
95. (i) Let I = ∫ x f (sin x) dx …(i) Topic 2 Periodicity of Integral Functions
0
π
π dx
⇒ I=∫ (π − x) f (sin x) dx …(ii) 1. Let I=∫ 2
0 −π [x] + [sin x] + 4
2
On adding Eqs. (i) and (ii), we get
π −1 dx 0 dx
2I=∫ π f (sin x) dx =∫ −π + ∫−1 [x] + [sin x] + 4
0 2
[x] + [sin x] + 4
π π π π
∴ ∫0 x f (sin x) dx =
2 ∫ 0 f (sin x) dx +∫
1 dx
+∫2
dx
0 [x] + [sin x] + 4 1 [x] + [sin x] + 4
3/ 2
(ii) Let I=∫ |x sin πx| dx − 2 , − π / 2 < x < −1
−1

 x sin πx, −1 < x ≤ 1 Q [x] =  0−,1, −1 ≤ x < 0
0 ≤ x<1
Since, |x sin πx| =  3  1, 1 ≤ x < π /2
− x sin πx, 1 < x < 2
1 3/ 2  − 1 , − π / 2 < x < −1
∴ I=∫ x sin πx dx + ∫1 − x sin πx dx 
and [sin x] = −01, , − 01 << xx << 10
−1
1  0, 1 < x < π / 2
 x cos πx  1  − cos πx
=2 − − 2 ∫ 1⋅   dx
 π 
0 0  π  [Q For x < 0,−1 ≤ sin x < 0 and for x > 0, 0 < sin x ≤ 1]
−1 dx 0 dx 1 dx
 − x cos πx 3/ 2 3 / 2  − cos πx  So, I = ∫ −π +∫ +∫
− −∫   dx − 2 − 1 + 4 −1 − 1 − 1 + 4 00+0+4
 π  1  π  2
 1  π
dx
 1  2 sin πx   1  1
1
sin πx 
3/ 2 +∫ 2
=2   + ⋅ − −  − 1 1+0+4
 π  π  π  0  π  π  π 
1 π
−1 dx 0dx 1 dx dx
2 2 1 1 = ∫ −π + ∫ −1 2 + ∫0 +∫ 2
= + 2 (0 − 0) + + 2 (+ 1 − 0) 1 4 1 5
π π π π 2

3 1  3π + 1  π 1 1 1 π 
= + 2= =  −1 +  + (0 + 1) + (1 − 0) +  − 1
  2 2 4 5 2 
π π  π2 
1 1  1 1 1  π π 
96. Let I = ∫ (t x + 1 − x)n dx = ∫ {(t − 1) x + 1 }n dx =  −1 + + −  +  + 
0 0  2 4 5  2 10 
 ((t − 1) x + 1)n + 1 
1
1  t n + 1 − 1 −20 + 10 + 5 − 4 5π + π
= =   = +
 20 10
 (n + 1) (t − 1)  0 n + 1  t − 1 
9 3π 3
1 =− + = (4π − 3)
= (1 + t + t 2 + K + t n ) …(i) 20 5 20
n+1 3 + 3T 1
Again,
1
I = ∫ (t x + 1 − x) dx = ∫ n 1
[(1 − x) + t x] dxn
2. ∫3 f (2x) dx Put 2x = y ⇒ dx =
2
dy
0 0
1 6 + 6T 6I
=∫
1
[ C 0 (1 − x) + C1 (1 − x)
n n n n −1
(t x)

2 ∫6 f ( y) dy =
2
= 3I
0
x
n
C 2(1 − x)n − 2(tx)2+ ... + nC n (tx)n + ] dx 3. Given, g (x) = ∫ f (t ) dt
0
2 1 2
 n n  ⇒ g (2) = ∫ f (t ) dt = ∫ f (t ) dt + ∫ f (t )dt
rΣ
1
=∫ C r (1 − x)n − r (t x)r  dx 0 0 1
=0
0
  1
n Now, ≤ f (t ) ≤ 1 for t ∈ [0,1]
Σ C r ∫ (1 − x)n − r ⋅ xr dx t r
1
= n
…(ii) 2
r=0  0  11 1 1

From Eqs. (i) and (ii), we get


We get ∫ 0 2 dt ≤ ∫ 0 f (t ) dt ≤ ∫ 0 1 dt
n 1 1
C r ∫ (1 − x)n − r ⋅ xr dx t r= ≤ ∫ f (t ) dt ≤ 1
1
Σ
1
n
(1 + t + K + t n ) ⇒ …(i)
r=0  0  n+1 2 0

1
On equating coefficient of t k on both sides, we get Again, 0 ≤ f (t ) ≤ for t ∈ [1, 2] …(ii)
2
C k ∫ (1 − x)n − k ⋅ xk dx =
n 1 1 2 2 2
 0  n + 1 ⇒ ∫ 10 dt ≤ ∫ 1 f (t ) dt ≤ ∫ 1 dt
1 n−k k 1
⇒ ∫ 0 (1 − x) x dx = 1
2
(n + 1)nC k ⇒ 0 ≤ ∫ f (t ) dt ≤
1 2
Definite Integration 323

2
From Eqs. (i) and (ii), we get
= 10 ∫ f (x) cos π x dx
1 1 2 3
2 ∫0
≤ f (t ) dt + ∫ f (t ) dt ≤ 0
1 2 1


1 3
≤ g (2) ≤ ⇒ 0 ≤ g (2) < 2 Now, ∫ f (x) cos π x dx
2 2 0
nπ + v π 2π 1 1
4. ∫0 |sin x|dx = ∫
0
|sin x|dx + ∫
π
|sin x|dx + ... = ∫ (1 − x) cos π x dx = − ∫ u cos π u du
nπ nπ + v 0 0
+∫ |sin x|dx + ∫ |sin x|dx 2 2
( n − 1 )π nπ
n
rπ nπ + v
and ∫ f (x) cos π x dx = ∫ (x − 1) cos πx dx
=∑∫ |sin x| dx + ∫ nπ |sin x| dx 1 1
( r − 1 )π 1
r =1
rπ = − ∫ u cos π u du
Now to solve, ∫ ( r −1) π| sin x |dx , we have 0
10 1
x = ( r − 1)π + t 40
⇒ sin x = sin [(r − 1) π + t ] = (−1)r − 1 sin t
∴ ∫ f (x) cos πx dx = − 20 ∫ u cos π u du =
π2
− 10 0
and when x = (r − 1) π, t = 0 and when 2 10
π
10 −∫10
x = r π, t = π ⇒ f (x) cos π x dx = 4
rπ π r −1
∴ ∫ ( r − 1)π|sin x| dx = ∫ 0 |(−1) sin t| dt
π
= ∫ |sin t| dt = ∫
π
sin t dt Topic 3 Estimation, Gamma Function and
0 0
Derivative of Definite Integration
= [− cos t ] π0 = − cos π + cos 0 = 2 x 1
nπ + v 1. Given, ∫ f (t ) dt = x2 + ∫ t 2f (t )dt
Again, ∫ | sin x|dx, putting x = nπ + t 0 x

nπ + v v v
On differentiating both sides, w.r.t. ‘x’, we get
Then, ∫ nπ | sin x|dx = ∫ |(−1)n sin t|dt = ∫
0 0
sin t dt
f (x) = 2x + 0 − x2f (x)
= [− cos t ]v0 = − cos v + cos 0 = 1 − cos v  ψ( x )  
d d d
nπ + v
n
rπ nπ + v Q  ∫ f (t )dt  = f (ψ (x)) ψ (x) − f (φ (x)) φ(x)
∴∫ |sin x|dx = ∑ ∫ | sin x|dx + ∫ nπ |sin x|dx  dx φ ( x )  dx dx 
0 ( r − 1 )π  
r =1
2x
n
nπ + v ⇒ (1 + x2) f (x) = 2x ⇒ f (x) =
= ∑2+ ∫ nπ |sin x|dx = 2n + 1 − cos v 1 + x2
r =1
On differentiating w.r.t. ‘x’ we get
a+ t
5. Let φ (a ) = ∫ f (x) dx (1 + x2)(2) − (2x) (0 + 2x) 2 + 2x2 − 4x2 2 − 2x2
a
f ′ (x) = = =
On differentiating w.r.t. a, we get (1 + x2)2 (1 + x2)2 (1 + x2)2
2
φ ′ (a ) = f (a + t ) ⋅ 1 − f (a ) ⋅ 1 = 0 [given, f (x + t ) = f (x)]  1  1
2 −2   2 − 1
2 −2   3
∴ φ (a ) is constant.  1  2  4 2 24
∴f′   = = = = 2 =
a+ t  2  2 2  1 
2
 5 
2 25 25
∫a 1 +   
⇒ f (x) dx is independent of a. 1 1 +   
 2   4  4 16

 x − [x], if [x] is odd.
6. Given, f (x) =  2. PLAN Newton-Leibnitz’s formula
1 + [x] − x, if [x] is even. d  ψ (x )
f(t ) dt  = f { ψ ( x )}  ψ ( x ) − f { φ ( x )}  φ ( x )
d d
f (x) and cos π x both are periodic with period 2 and both dx  ∫ φ ( x )   dx   dx 
are even. x2
10 10 Given, F (x) = ∫ f ( t ) dt
0
∴ ∫ f (x) cos π x dx = 2 ∫ f (x) cos π x dx
∴ F ′ (x) = 2x f (x)
− 10 0

Y
Also, F ′ (x) = f ′ (x) ⇒ 2x f (x) = f ′ (x)
f ′ (x) f ′ (x)

f (x)
= 2x ⇒ ∫ f (x)
dx = ∫ 2x dx

X ⇒ In f (x) = x + c 2
–10 – 9 –2 –1 0 1 2 9 10 2
+ c
⇒ f (x) = ex
2
⇒ f (x) = K ex [K = ec ]
324 Definite Integration

Now, f (0) = 1 t2 2 5
∴ 1=K
6. Here, ∫0 x f (x) dx =
5
t
2
Hence, f (x) = ex Using Newton Leibnitz's formula, differentiating both
4 sides, we get
F (2) = ∫ et dt = [et ]40 = e4 − 1
d  d 
0
t 2{ f (t 2)}  (t 2) − 0 ⋅ f (0)  (0) = 2t 4
x  dt   dt 
3. Given, y = ∫ |t| dt
0
⇒ t 2f (t 2)2t = 2t 4 ⇒ f (t 2) = t
dy
∴ = | x|⋅ 1 − 0 = |x| [by Leibnitz’s rule] 4 2  2
dx ∴ f   =− putting t =
x  25 5  5 
Q Tangent to the curve y = ∫ |t| dt , x ∈ R are parallel
0
4 2
to the line y = 2x ⇒ f   =
 25 5
∴ Slope of both are equal ⇒ x = ± 2
± 2 x 2 + 1 −t 2
Points, y=∫ | t | dt = ± 2 7. Given, f (x) = ∫ e dt
0 x2

Equation of tangent is On differentiating both sides using Newton’s Leibnitz’s


formula, we get
y − 2 = 2 (x − 2) and y + 2 = 2 (x + 2)
2
+ 1 )2  d 2  − ( x 2 )2  d 
For x intercept put y = 0, we get f ′ (x) = e− ( x  (x + 1) − e  (x2)
 dx   dx 
0 − 2 = 2 (x − 2) and 0 + 2 = 2 (x + 2) ⇒ x = ± 1 2
+ 1 )2 2 2
x x = e− ( x ⋅ 2x − e− ( x )
⋅ 2x
4. Given ∫ 1 − { f ′ (t )} dt = ∫ f (t ) dt , 0 ≤ x ≤ 1
2
4 2 2
= 2xe− ( x + 2x + 1 ) +1
0 0
(1 − e2x )
Differentiating both sides w.r.t. x by using Leibnitz’s 2 4
2x + 1 + 2x 2 + 1 )
rule, we get [where, e > 1, ∀ x and e− ( x > 0, ∀ x]
1 − { f ′ (x)} = f (x) ⇒
2
f ′ (x) = ± 1 − { f (x)} 2
∴ f ′ (x) > 0
f ′ (x) which shows 2x < 0 or x < 0 ⇒ x ∈ (−∞ , 0)
⇒ ∫ 1 − { f (x)} 2
dx = ± ∫ dx 1
8. Here, I (m, n ) = ∫ t m(1 + t )n dt reduce into I (m + 1, n − 1)
0
⇒ sin −1 { f (x)} = ± x + c [we apply integration by parts taking (1 + t )n as first
Put −1
x = 0 ⇒ sin { f (0)} = c and t m as second function]
1
⇒ c = sin −1 (0) = 0 [Q f (0) = 0]  tm + 1  1 tm + 1
∴ I (m, n ) = (1 + t )n ⋅  − ∫ n (1 + t )( n − 1) ⋅ dt
∴ f (x) = ± sin x  m + 1 0 0 m+1
but f (x) ≥ 0, ∀ x ∈ [0, 1] 2n n 1
= − ∫ (1 + t )( n − 1) ⋅ tm + 1 dt
∴ f (x) = sin x m+1 m+1 0
As we know that, 2n n
∴ I (m, n ) = − ⋅ I (m + 1, n − 1)
sin x < x, ∀ x > 0 m+1 m+1
 1 1  1 1 f (x) = ∫
x
2 − t 2 dt ⇒ f ′ (x) = 2 − x2
∴ sin   < and sin   < 9. Given,
 2 2  3 3 1

 1 1  1 1 Also, x2 − f ′ (x) = 0
⇒ f   < and f   <
 2 2  3 3 ∴ x2 = 2 − x 2 ⇒ x4 = 2 − x 2
1
5. Since ∫ t 2 f (t ) dt = 1 − sin x, thus to find f (x). ⇒ x4 + x2 − 2 = 0 ⇒ x = ± 1
sin x
x
On differentiating both sides using Newton Leibnitz 10. Given, F (x) = ∫ f (t ) dt
0
formula By Leibnitz’s rule,
d 1 d
dx ∫ sin x
i.e. t 2 f (t ) dt = (1 − sin x) F ′ (x) = f (x) …(i)
dx
⇒ {12 f (1)} ⋅ (0) − (sin 2 x) ⋅ f (sin x) ⋅ cos x = − cos x But F (x2) = x2 (1 + x) = x2 + x3 [given]
1 3 1/ 2
⇒ f (sin x) = ⇒ F (x) = x + x3/ 2 ⇒ F ′ (x) = 1 + x
sin 2 x 2
 1 3 1/ 2
For f   is obtained when sin x = 1 / 3 ⇒ f (x) = F ′ (x) = 1 + x [from Eq. (i)]
 3 2
 1 3 1/ 2 3
i.e. f   = ( 3 )2 = 3 ⇒ f (4) = 1 + (4) ⇒ f (4) = 1 + × 2 = 4
 3 2 2
Definite Integration 325

x 1 1
11. Given, ∫ 0 f (t )dt = x + ∫ x t f (t ) dt and,
1 2 1 3 x5   x4 x6 
∫ 0 x sin x dx ≥ ∫ 0  x − 6  dx =  4 − 36 
On differentiating both sides w.r.t. x, we get 0

f (x) 1 = 1 − x f (x) ⋅ 1 ⇒ (1 + x) f (x) = 1 1 1


= − =
8 2
=
1 1 1 4 36 36 9
⇒ f (x) = ⇒ f (1) = = 2
1+ x 1+1 2 1 2

f ( x)
∴ ∫ 0 x sin x dx ≥ 9
f ( x) 2t ∫4 2t dt
15. g (x) = ∫
sin 2x
sin −1 (t )dt
12. lim ∫ dt = lim
x→1 4 x−1 x→1 x−1 sin x

[using L’ Hospital’s rule] g′ (x) = 2 cos 2x sin −1 (sin 2x) − cos x sin −1 (sin x)
 π  π
2 f (x) ⋅ f ′ (x) g′   = − 2 sin −1 (0) = 0 ⇒ g′  −  = − 2 sin −1 (0) = 0
= lim = 2 f (1) ⋅ f ′ (1)  2  2
x→1 1
= 8 f ′ (1) [Q f (1) = 4] No option is matching.

13. If f (x) is a continuous function defined on [a , b], then 16. Here, f (x) + 2x = (1 − x)2 ⋅ sin 2 x + x2 + 2x …(i)
b where, P : f (x) + 2x = 2 (1 + x)2 …(ii)
m (b − a ) ≤ ∫ f (x) dx ≤ M (b − a )
a ∴ 2 (1 + x2) = (1 − x)2 sin 2 x + x2 + 2x
where, M and m are maximum and minimum values ⇒ (1 − x)2 sin 2 x = x2 − 2x + 2
respectively of f (x) in [a , b]. ⇒ (1 − x)2 sin 2 x = (1 − x)2 + 1 ⇒ (1 − x)2 cos 2 x = − 1
−x 2
Here, f (x) = 1 + e is continuous in [0, 1]. which is never possible.
Now, 0 < x<1 ⇒x < x ⇒ e 2 x2
< e ⇒e
x −x 2
>e −x ∴ P is false.
x2 −x 2 Again, let Q : h (x) = 2 f (x) + 1 − 2x (1 + x)
Again, 0 < x < 1 ⇒ x > 0 ⇒ e 2
> e ⇒e0
<1
2 where, h (0) = 2 f (0) + 1 − 0 = 1
∴ e− x < e− x < 1, ∀ x ∈ [0, 1]
h (1) = 2 f (1) + 1 − 4 = − 3, as h (0) h (1) < 0
2
⇒ 1 + e− x < 1 + e− x < 2, ∀ x ∈ [0, 1] ⇒ h (x) must have a solution.
1 1 1
∴ Q is true.
2
−x
⇒ ∫ 0 (1 + e ) dx < ∫ (1 + e− x ) dx < ∫ 2 dx
0 0
17. Here, f (x) = (1 − x)2 ⋅ sin 2 x + x2 ≥ 0, ∀ x.
1 1 2
⇒ 2− < ∫ (1 + e− x ) dx < 2 x  2 (t − 1 ) 
e 0 and g (x) = ∫  − log t f (t ) dt
1 t + 1 
x2 x4 x6
14. Q cos x = 1 − + − + ... 2 (x − 1) 
2! 4! 6! ⇒ g′ (x) =  − log x ⋅ f {
(x) …(i)
 (x + 1 )  + ve
x3 x5 x7
and sin x = x − + − + ...
3! 5! 7! For g′ (x) to be increasing or decreasing,
2 (x − 1)
1 1  3
x x 2 4 1
x 1 1 3 let φ(x) = − log x
(x + 1)
∴ ∫ 0 x cos x dx ≥ ∫ 0  x − 2 ! dx =  2 −  = − =
8 0 2 8 8
4 1 − (x − 1)2
φ′ (x) = − =
1 3 (x + 1) 2
x x (x + 1)2
⇒ ∫ 0 x cos x dx ≥ 8
φ′ (x) < 0, for x > 1 ⇒ φ (x) < φ (1)
1 1 2 x4  ⇒ φ(x) < 0
and, ∫ 0 x sin dx ≥ ∫ 0  x − 6  dx …(ii)
From Eqs. (i) and (ii), we get
1
 x3 x5  1 1 9 3 g′ (x) < 0 for x ∈ (1, ∞ )
= −  = 3 − 30 = 30 = 10
 3 30 0 ∴g (x) is decreasing for x ∈ (1, ∞ ).
1 3 sec x cos x cosec x ⋅ cotx + sec2x
⇒ ∫ 0 x sin x dx ≥ 10 18. Given, f (x) = cos 2 x cos 2 x cosec2x
1 2 1 3 x5  1 cos x2
cos 2 x
and, ∫ 0 x cos x dx ≥ ∫ 0  x − 2  dx 1
Applying R3 → R3 ,
x 4 6 1 cos x
x 1 1 2 1
= −  = 4 − 12 = 12 = 6 sec x cos x cosec x ⋅ cot x + sec2 x
 4 12 0
f (x) = cos x cos x cos x2 2
cosec2 x
1 2 1
∴ ∫ 0 x cos x dx ≥ 6 sec x cos x cos x
326 Definite Integration

Applying R1 → R1 − R3 ⇒ f (x) Again, a <2 [given]


0 0 cosec x ⋅ cot x + sec2x − cos x π π
⇒ 2 − < 2 ⇒ >0 ⇒ t >0
= cos x cos 2 x cos 2 x cosec2x 2 2
a b
sec x cos x cos x Now, ∫ g (x) dx + ∫ g (x) dx
0 0

= (cosecx ⋅ cot x + sec x − cos x) ⋅ (cos x − cos x) ⋅ cos x


2 3 2 − t/ 2 2 + t/ 2
=∫ g (x) dx + ∫0 g (x) dx
sin 2 x + cos3 x − cos3 x ⋅ sin 2 x  0
=−  ⋅ cos x ⋅ sin x
2 2
2 − t/ 2 2 + t/ 2
 sin 2 x ⋅ cos 2 x  Let F (x) = ∫ g (x) dx + ∫0 g (x) dx
0

= − sin x − cos x (1 − sin x) = − sin x − cos x


2 3 2 2 5
1  t 1  t
π /2 π /2 For t > 0, F ′ (t ) = − g 2 −  + g 2 + 
 2 2  2
∴ ∫0 f (x) dx = − ∫0 (sin x + cos x) dx
2 5
2

 n +2 m+ 1  [using Leibnitz’s rule]


 π /2  1  t 1  t
2 2 = g 2 +  − g 2 − 
Q

∫0 sinm x ⋅ cos n x dx =
m +n+2


2  2 2  2
2
 2  dg
Again, > 0, ∀ x ∈ R [given]
 3 1 6 1  dx
 ⋅ ⋅
π /2 2 2 2 2  Now, 2 − t / 2 < 2 + t / 2 ∴ t > 0
∫ 0 f (x) dx = −  2 2
+
7 

We get g (2 + t / 2) − g (2 − t / 2) > 0, ∀ t > 0
 2
 2  So, F ′ (t ) > 0, ∀ t > 0
  Hence, F (t ) increases with t, therefore F (t ) increases as
1 /2 ⋅ π 2 π  π 8  15π + 32 (b − a ) increases.
=− + = −  +  = −  
 2 5 3 1
2⋅ ⋅ ⋅ ⋅ π   4 15  60  1
21. Let I n = ∫ ex (x − 1)n dx
 2 2 2  0

x ln t Put x − 1 = t ⇒ dx = dt
19. f (x) = ∫ dt for x > 0 [given]
1 1+ t 0 t+1 0 n t
∴ In = ∫ e ⋅ t ndt = e ∫ t e dt
1/ x ln t −1 −1
Now, f (1 / x) = ∫ dt
1+ t = e  [t net ] −01 − n ∫ t n − 1et dt
1 0
 −1 
Put t = 1 / u ⇒ dt = (−1 / u 2) du
= e 0 − (−1)n e−1 − n e dt
0 n −1 t
∴ f (1 / x) = ∫
x ln (1 / u ) (−1 )
⋅ 2 du  ∫ −1 t 
1 1 + 1 /u u
0
x ln u x ln t = (−1)n + 1 − ne ∫ t n − 1et dt
=∫ du = ∫ dt −1
1 u (u + 1 ) 1 t (1 + t )
⇒ I n = (−1)n + 1 − nI n − 1 …(i)
 1 x ln t x ln t
f (x) + f   = ∫ ∫ 1 t (1 + t ) dt
1 x 1
Now, dt + For n = 1, I1 = ∫ e (x − 1) dx = [e (x x
− 1)]10 − ∫ ex dx
 x 1 (1 + t ) 0 0
x (1 +
=∫
t ) ln t
dt = ∫
x ln t 1 1
dt = [(ln t )2]1x = (ln x)2 = e1 (1 − 1) − e0 (0 − 1) − [ex ] 10 = 1 − (e − 1) = 2 − e
1 t (1 + t ) 1 t 2 2 Therefore, from Eq. (i), we get
Put x = e, I 2 = (−1)2 + 1 − 2I1 = − 1 − 2(2 − e) = 2e − 5
 1 1 1 and I3 = (−1)3 + 1 − 3I 2 = 1 − 3(2e − 5) = 16 − 6e
f (e) + f   = (ln e)2 = Hence proved.
 e 2 2 Hence, n = 3 is the answer.
20. Let t = b − a and a + b = 4 [given] 22. Since, f is continuous function and ∫
x
f (t ) dt → ∞ ,
⇒ t =4 − a − a 0

⇒ t = 4 − 2a as| x|→ ∞. To show that every line y = mx intersects the


x
t curve y2 + ∫ f (t ) dt = 2
⇒ a =2 − 0
2
Y
and t = b − (4 − b) (0,√2)
A
⇒ t = 2b − 4
t O X
⇒ = b −2 (Xp,0)
2
t B (0,–√2)
⇒ b =2 +
2
Definite Integration 327

At x = 0, y = ± 2 Then, upper limit of integral is 1 and lower limit of


Hence, (0, 2 ), (0, − 2 ) are the point of intersection of integral is 0.
the curve with the Y-axis. 1  1 n
 r 1 
As x → ∞, ∫
x
f (t ) dt → ∞ for a particular x (say xn ), then
So, p = ∫ (1 + x)1/3 dx Q lim ∑ f  n  = ∫0 f (x) dx
0 0  n → ∞ n r =1 
x
∫0
1
f (t ) dt = 2 and for this value of x, y = 0 3 
= (1 + x)4/3
4  0
The curve is symmetrical about X-axis.
3 3 3
Thus, we have that there must be some x, such that = (24/3 − 1) = (2)4/3 −
f (xn ) = 2. 4 4 4
Thus, y = mx intersects this closed curve for all values of 2. Clearly,
m. lim  n n n 1
 2 + 2 + 2 + ...+ 
x
23. Given, f (x) = ∫ [2(t − 1) (t − 2) + 3 (t − 1) (t − 2) ] dt 3 2 2 
n→ ∞ n +1 2
n +2 2
n +3 2
5n 
1
lim  n n n n 
∴ f ′ (x) = [2 (x − 1) (x − 2)3 + 3 (x − 1)2(x − 2)2] ⋅ 1 − 0 =  + + + ....+ 2 
n → ∞  n 2 + 12 n 2 + 22 n 2 + 32 n + (2n )2
= (x − 1) (x − 2)2 [2 (x − 2) + 3 (x − 1)]
lim 2n n
= (x − 1) (x − 2)2 (5x − 7) = ∑
n → ∞ r=1 n 2 + r 2
+ – +
lim 2n 1 1 2 dx
1 7/5 = ∑ ⋅ =∫
n→ ∞ r =1  r
2
n 0 1 + x2
∴ f (x) attains maximum at x = 1 and f (x) attains 1+  
7  n
minimum at x = .  pn 
5 1  r p
Q lim ∑ f   = ∫ 0 f (x)dx
n→∞ n n
24. It is given that, for functions f : R → R  r =1 
= [tan −1 x]20 = tan −1 2
and F : [0, π ] → R ,
x 1
F (x) = ∫ f (t )dt , where f (π ) = − 6 (n + 1) ⋅ (n + 2) K (3n ) n
0 3. Let l = lim  
n→ ∞  n 2n 
⇒ F ′ (π ) = f (π ) = − 6 … (i) 1
π  (n + 1) ⋅ (n + 2) ... (n + 2n ) n
Now, ∫0 ( f ′ (x) + F (x)) cos x dx = lim 
n→ ∞  n 2n


π π
= ∫ f ′ (x) cos xdx + ∫ 0 F (x) cos xdx
1
0   n + 1  n + 2  n + 2n   n
= lim      K 
π π n→ ∞   n   n   n  
= [cos x f (x)]0π + ∫ 0 f (x) sin x dx + ∫ 0 F (x) cos x dx
Taking log on both sides, we get
{by integration by parts}
1   1  2  2 n  
= (− 1) (− 6) − f (0) + [(sin x) F (x)]π0 log l = lim log 1 +  1 +  ... 1 + 
π π n→ ∞ n   n  n  n  
− ∫ F (x) cos x dx + ∫ 0 F (x) cos x dx
0 1
⇒ log l = lim
= 6 − f (0) = 2 (given) n→ ∞ n
⇒ f (0) = 4   1  2  2n  
log 1 + n  + log 1 + n  + ... + log 1 + n  
 
Topic 4 Limits as the Sum 2n
1  r
1. Let p = lim 
 (n + 1)1/3
+
(n + 2)1/3
+…+
(2n )1/3 

⇒ log l = lim
n→ ∞ n ∑ log 1 + n 
r =1
n→ ∞  n 4/3
n 4/3
n 4/3 
2
n ⇒ log l = ∫ log (1 + x) dx
(n + r )1/3 0
= lim
n→ ∞
∑ n 4/3  1 
2
r =1 ⇒ log l = log (1 + x) ⋅ x − ∫ ⋅ x dx
1/3  1 + x 0
 r
n 1 +
1/3
 n 2 x+ 1 −1
 n 1 n
 r
1/3
⇒ log l = [log (1 + x) ⋅ x]0 − ∫
2
= lim ∑ 4/3
= lim ∑ 1 + n  0 1+ x
dx
n→ ∞ n n→ ∞ n
r =1 r =1
2 1 
Now, as per integration as limit of sum. ⇒ log l = 2 ⋅ log 3 − ∫ 1 −  dx
0  1 + x
r 1
Let = x and = dx [Q n → ∞] ⇒ log l = 2 ⋅ log 3 − [x − log 1 + x ]20
n n
328 Definite Integration

⇒ log l = 2 ⋅ log 3 − [2 − log 3] n

⇒ log l = 3 ⋅ log 3 − 2 ∑ ra
r =1 1
⇒ log l = log 27 − 2 ⇒ lim =
n→ ∞  n (n + 1)  60
(n + 1)a − 1 ⋅ n 2a +
∴ l = elog 27 − 2 = 27 ⋅ e− 2 =
27  2 
e2 n a
 r
2∑  
4. Since,  n 1
r =1
  ⇒ lim a −1
=
  n→ ∞
 1  60
1 + 2 + 3 + ... + n
3 3 3
1 +  ⋅ (2na + n + 1)
lim  ,  n
n→ ∞  
7/3  1 1 1
n  + + ... +  1  n  r 
a
1 1
  (an + 1)2 (an + 2)2 (an + n)2  ⇒ lim 2 ∑    ⋅ lim =
n→ ∞ n   n   n→ ∞
 
a −1
  60
a ∈ R,|a| > 1  r =1  1 1
1 +  ⋅ 2a + 1 + 
1/3  n  n
n n
 r 1
Σ (r1/3 ) Σ   1 a 1 1
r =1 r = 1  n n ⇒ 2∫ (x ) dx ⋅ =
= lim = lim n 0 1 ⋅ (2a + 1) 60
n → ∞ 7/3 n 1 n→ ∞ 1 1
n Σ Σ 2 ⋅ [xa+ 1 ] 10 1
r = 1 (an + r )2 r =1  r n
2
⇒ =
a +  (2a + 1) ⋅ (a + 1) 60
 n
1 2 1
∴ = ⇒ (2a + 1) (a + 1) = 120
∫x
1/3
dx (2a + 1) (a + 1) 60
= 1
0
= 54, (given) ⇒ 2a 2 + 3a + 1 − 120 = 0 ⇒ 2a 2 + 3a − 119 = 0
dx −17
∫ (a + x)2
⇒ (2a + 17) (a − 7) = 0 ⇒ a = 7,
2
0
3 4/3 1 n
[x ]0 n
4 3 /4 6. Given, S n = ∑
⇒ = 54 ⇒ = 54 n 2
+ kn + k2
1 1 1 k=0
 1  − +
− x + a  a+1 a    
 0 n   n  
∑ n  k k  2
1  1 1 1

3
=
1 =∑ ⋅  < lim
4 × 54 a (a + 1) k=0
n  k k2  n → ∞ k=0 
 1 + +  1 + +   
 n n2   n  n 
⇒ a 2 + a = 72
⇒ a + 9a − 8a − 72 = 0
2
1 1
=∫ dx
⇒ a (a + 9) − 8(a + 9) = 0 0 1 + x + x2
⇒ (a − 8) (a + 9) = 0 1
⇒ a = 8 or− 9  2  2  1  
= tan −1  x +  
 3  3  2   0
Hence, options (c) and (d) are correct.
5. PLAN Converting Infinite series into definite Integral 2  π π π π
h( n) = ⋅ −  = i.e. S n <
i.e. lim
n→ ∞ n
3  3 6 3 3 3 3
π
1
h ( n)
 r Similarly, Tn >
lim
n→ ∞ n
∑ f  n  = ∫ f(x )dx 3 3
r = g ( n)
g ( n)  1 1 1  5n 1
lim 7. lim  + +K+  =∑
n→ ∞ n n→ ∞  n + 1 n+2 6n  r=1 n + r
r
where, is replaced with x. 1 5n
1
n
Σ is replaced with integral.
= lim
n→ ∞ n
∑ r
r =1 1 + 
 n
Here,
1a + 2a + K + n a
5 dx
lim =
1 =∫ = [log (1 + x)] 50 = log 6 − log 1 = log 6
n→ ∞ a −1
(n + 1) {(na + 1) + (na + 2) + K + (na + n )} 60
0 1+ x
13
Area
Topic 1 Area Based on Geometrical Figures
Without Using Integration
Objective Questions I (Only one correct option)
1. The area (in sq. units) of the largest rectangle ABCD 6. The area of the quadrilateral formed by the tangents at the
whose vertices A and B lie on the X-axis and vertices x2 y2
C and D lie on the parabola, y = x2 − 1 below the end points of latusrectum to the ellipse + = 1, is
9 5 (2003, 1M)
X-axis, is (2020 Main, 4 Sep II)
4 2 1 4 (a) 27/4 sq units (b) 9 sq units
(a) (b) (c) (d) (c) 27/2 sq units (d) 27 sq units
3 3 3 3 3 3 3
7. The area (in sq units) bounded by the curves y = | x | − 1 and
2. If the area enclosed between the curves y = kx2 and
x = ky2, (k > 0), is 1 square unit. Then, k is y = − | x | + 1 is (2002, 2M)
(2019 Main, 10 Jan I) (a) 1 (b) 2
1 2 3 (c) 2 2 (d) 4
(a) 3 (b) (c) (d)
3 3 2 8. The triangle formed by the tangent to the curve
3. The area (in sq units) of the region {(x, y) : y2 ≥ 2x f (x) = x2 + bx − b at the point (1,1) and the coordinate axes,
and x2 + y2 ≤ 4x, x ≥ 0, y ≥ 0} is (2016 Main) lies in the first quadrant. If its area is 2 sq units, then the
4 8 value of b is (2001, 2M)
(a) π − (b) π −
3 3 (a) – 1 (b) 3 (c) – 3 (d) 1
4 2 π 2 2
(c) π − (d) −
3 2 3 Objective Question II
4. The common tangents to the circle x2 + y2 = 2 and (One or more than one correct option)
the parabola y2 = 8x touch the circle at the points 9. Let P and Q be distinct points on the parabola y2 = 2x such
P,Q and the parabola at the points R, S. Then, the
area (in sq units) of the quadrilateral PQRS is that a circle with PQ as diameter passes through the vertex
(2014 Adv.) O of the parabola. If P lies in the first quadrant and the
(a) 3 (b) 6 (c) 9 (d)15 area of ∆OPQ is 3 2, then which of the following is/are the
coordinates of P ? (2015 Adv.)
5. The area of the equilateral triangle, in which three
(c)  ,
1 1 
coins of radius 1 cm are placed, as shown in the (a) (4 , 2 2 ) (b) (9 , 3 2 )  (d) (1, 2 )
4 2
figure, is

Integer & Numerical Answer Type Questions


10. A farmer F1 has a land in the shape of a triangle with
vertices at P(0, 0), Q(1, 1) and R(2, 0). From this land, a
neighbouring farmer F2 takes away the region which lies
(2005, 1M) between the sides PQ and a curve of the form y = xn (n > 1).
(a) (6 + 4 3 ) sq cm (b) (4 3 − 6) sq cm If the area of the region taken away by the farmer F2 is
(c) (7 + 4 3 ) sq cm (d) 4 3 sq cm exactly 30% of the area of ∆PQR, then the value of n is
.................... . (2018 Adv.)
330 Area

Fill in the Blanks y = f (x). If x ∈ (− 2, 2), the equation implicitly defines a


unique real-valued differentiable function y = g (x),
11. The area of the triangle formed by the positive X-axis satisfying g (0) = 0. (2008, M)
and the normal and the tangent to the circle x2 + y2 = 4
at (1, 3 ) is … . (1989, 2M)
14. If f (− 10 2 ) = 2 2 , then f′′ (− 10 2 ) is equal to
4 2 4 2 4 2 4 2
12. The area enclosed within the curve|x| + | y| = 1 is ....... . (a) (b) − (c) (d) −
73 32 73 32 73 3 73 3
(1981, 2M)
Analytical & Descriptive Question 15. The area of the region bounded by the curve y = f (x),
the X-axis and the lines x = a and x = b, where
1
13. Let O (0, 0), A (2, 0) and B (1, ) be the vertices of a − ∞ < a < b < − 2, is
3 b x
(a) ∫ dx + bf (b) − af (a )
triangle. Let R be the region consisting of all those a 3[{f (x)}2 − 1]
points P inside ∆ OAB which satisfy d (P , OA ) ≥ min b x
{ d (P , OB), d (P , AB)}, where d denotes the distance (b) − ∫a 3[{f (x)}2 − 1] dx + bf (b) − af (a)
from the point to the corresponding line. Sketch the
b x
region R and find its area. (1997C, 5M) (c) ∫ dx − bf (b) + af (a )
a 3[{f (x)}2 − 1]
Passage Based Questions b x
(d) − ∫a 3[{f (x)}2 − 1] dx − bf (b) + af (a)
Consider the functions defined implicity by the
1
equation y3 − 3 y + x = 0 on various intervals in the real
line. If x ∈ (−∞ , − 2) ∪ (2, ∞ ), the equation implicitly
16. ∫− 1 g ′ (x) dx is equal to
defines a unique real-valued differentiable function (a) 2 g(− 1) (b) 0 (c) − 2 g(1) (d) 2 g(1)

Topic 2 Area Using Integration


Objective Questions I (Only one correct option) 5. If the area (in sq units) bounded by the parabola
1
1. Let the functions f : R → R and g : R → R be defined by y2 = 4λx and the line y = λx, λ > 0, is , then λ is equal to
9
1 x −1
f (x) = ex − 1 − e−|x − 1|and g (x) =
(e + e1 − x ). (2019 Main, 12 April II)
2 (a) 2 6 (b) 48 (c) 24 (d) 4 3
Then the area of the region in the first quadrant 6. If the area (in sq units) of the region
bounded by the curves y = f (x), y = g (x) and x = 0 is {(x, y): y2 ≤ 4x, x + y ≤ 1, x ≥ 0, y ≥ 0} is a 2 + b, then
(2020 Adv.) a − b is equal to (2019 Main, 12 April I)
1 1
(a) (2 − 3 ) + (e − e−1 ) (b) (2 + 3 ) + (e − e−1 ) 10 8 2
2 2 (a) (b) 6 (c) (d) −
1 1 3 3 3
(c) (2 − 3 ) + (e + e−1 ) (d) (2 + 3 ) + (e + e−1 )
2 2 7. The area (in sq units) of the region bounded by the
curves y = 2x and y = | x + 1|, in the first quadrant is
2. Consider the region R = {(x, y) ∈ R : x ≤ y ≤ 2x }. If a
2 2
(2019 Main, 10 April II)
line y = α divides the area of region R into two equal (a)
3
(b) log e 2 +
3
(c)
1
(d)
3

1
parts, then which of the following is true? 2 2 2 2 log e 2
(2020 Main, 2 Sep II)
(a) α3 − 6α 2 + 16 = 0 (b) 3α 2 − 8α3 / 2 + 8 = 0 8. The area (in sq units) of the region
(c) 3α 2 − 8α + 8 = 0 (d) α3 − 6α3 / 2 − 16 = 0  y2 
A = (x, y) : ≤ x ≤ y + 4 is
3. The area (in sq. units) of the region  2  (2019 Main, 9 April II)
A = {(x, y) : (x − 1)[x] ≤ y ≤ 2 x, 0 ≤ x ≤ 2}, where [t ] (a) 30 (b)
53
(c) 16 (d) 18
denotes the greatest integer function, is 3
(2020 Main, 5 Sep II)
9. The area (in sq units) of the region
8 1 4
(a) 2− (b) 2+1 A = {(x, y) : x2 ≤ y ≤ x + 2} is (2019 Main, 9 April I)
3 2 3 13 9 31 10
8 4 1 (a) (b) (c) (d)
(c) 2−1 (d) 2− 6 2 6 3
3 3 2
10. Let S (α ) = {(x, y) : y2 ≤ x, 0 ≤ x ≤ α} and A(α ) is area of the
4. The area of the region {(x, y) : xy ≤ 8,1 ≤ y ≤ x2} is
region S(α ). If for λ, 0 < λ < 4, A (λ ) : A (4) = 2 : 5, then λ
(2020 Adv.)
14 7 equals (2019 Main, 8 April II)
(a) 8 log e 2 − (b) 8 log e 2 − 1 1 1 1

(a) 2  (b) 4  (c) 4  (d) 2 


3 3 4 3 2 3 4 3 2 3
14  25   5  25   5
(c) 16 log e 2 − (d) 16 log e 2 − 6
3
Area 331

11. The tangent to the parabola y2 = 4x at the point where it 22. The area (in sq units) of the region described by
intersects the circle x2 + y2 = 5 in the first quadrant, A = {(x, y) : x2 + y2 ≤ 1 and y2 ≤ 1 − x } is (2014 Main)
passes through the point π 4 π 4 π 2 π 2
(a) + (b) − (c) − (d) +
(a)  ,
3
(b)  ,
7
(c)  − ,
4
(d)  − ,
1 3 1 1 1 2 3 2 3 2 3 2 3
   
4 4 4 4  3 3  4 2
23. The area enclosed by the curves y = sin x + cos x and
12. The area (in sq units) of the region  π
y = | cos x − sin x|over the interval 0, is (2014 Adv.)
A = {(x, y) ∈ R × R|0 ≤ x ≤ 3,  2 
0 ≤ y ≤ 4, y ≤ x2 + 3x} is (2019 Main, 8 April I) (a) 4( 2 − 1) (b) 2 2 ( 2 − 1)
53 59 26
(a) (b) 8 (c) (d) (c) 2 ( 2 + 1) (d) 2 2 ( 2 + 1)
6 6 3
24. The area (in sq units) bounded by the curves
13. The area (in sq units) of the region bounded by the y = x, 2 y − x + 3 = 0, X-axis and lying in the first
parabola, y = x + 2 and the lines, y = x + 1, x = 0 and
2
quadrant, is (2013 Main, 03)
x = 3, is (2019 Main, 12 Jan I) 27
15 17 21 15 (a) 9 (b) 6 (c) 18 (d)
(a) (b) (c) (d) 4
2 4 2 4
25. Letf : [−1, 2] → [0, ∞ ) be a continuous function such that
14. The area (in sq units) in the first quadrant bounded by 2
the parabola, y = x + 1, the tangent to it at the point
2 f (x) = f (1 − x), ∀x ∈ [−1, 2]. If R1 = ∫ xf (x) dx and R2 are
−1
(2, 5) and the coordinate axes is (2019 Main, 11 Jan II) the area of the region bounded by y = f (x), x = − 1, x = 2
14 187 8 37 and the X-axis. Then, (2011)
(a) (b) (c) (d)
3 24 3 24 (a) R1 = 2R2 (b) R1 = 3R2
15. The area (in sq units) of the region bounded by the curve (c) 2R1 = R2 (d) 3R1 = R2
x2 = 4 y and the straight line x = 4 y − 2 is 26. If the straight line x = b divide the area enclosed by
(2019 Main, 11 Jan I)
7 9 5 3 y = (1 − x)2, y = 0 and x = 0 into two parts R1 (0 ≤ x ≤ b)
(a) (b) (c) (d)
8 8 4 4 1
and R2(b ≤ x ≤ 1) such that R1 − R2 = . Then, b equals
16. The area of the region A = {(x, y); 0 ≤ y ≤ x| x| + 1 and 4
3 1 1 1
− 1 ≤ x ≤ 1} in sq. units, is (2019 Main, 9 Jan II) (a) (b) (c) (d) (2011)
4 1 2 4 2 3 4
(a) 2 (b) (c) (d)
3 3 3 27. The area of the region between the curves
1 + sin x 1 − sin x
17. The area (in sq units) bounded by the parabola y= and y = and bounded by the
y = x2 − 1, the tangent at the point (2, 3) to it and the cos x cos x
Y -axis is (2019 Main, 9 Jan I) π
lines x = 0 and x = is
8 56 32 14 4 (2008, 3M)
(a) (b) (c) (d)
3 3 3 3 2 −1 t 2 −1 4t
(a) ∫ dt (b) ∫ dt
18. Let g (x) = cos x2, f (x) = x and α , β (α < β) be the roots of 0
(1 + t ) 1 − t
2 2 0
(1 + t ) 1 − t 2
2

the quadratic equation 18x2 − 9πx + π 2 = 0. Then, the 2+1 4t 2+1 t


(c) ∫ dt (d) ∫ dt
area (in sq units) bounded by the curve y = ( gof )(x) and 0 0
(1 + t 2 ) 1 − t 2 (1 + t 2 ) 1 − t 2
the lines x = α, x = β and y = 0, is (2018 Main)
(a)
1
( 3 − 1) (b)
1
( 3 + 1) 28. The area bounded by the curves y = (x − 1)2, y = (x + 1)2
2 2 1
1 1 and y = is (2005, 1M)
(c) ( 3 − 2 ) (d) ( 2 − 1) 4
2 2 1 2 1 1
(a) sq unit (b) sq unit (c) sq unit (d) sq unit
19. The area (in sq units) of the region 3 3 4 5
{(x, y) : x ≥ 0, x + y ≤ 3, x2 ≤ 4 y and y ≤ 1+ x } is 29. The area enclosed between the curves y = ax2 and
(2017 Main)
59 3 7 5 x = ay2 (a > 0) is 1 sq unit. Then, the value of a is
(a) (b) (c) (d)
12 2 3 2 (2004, 1M)
1 1 1
20. Area of region {(x, y)} ∈ R2 : y ≥ |x + 3|,
the (a) (b) (c) 1 (d)
3 2 3
5 y ≤ (x + 9) ≤ 15} is equal to (2016 Adv.)
1 4 3 5 30. The area bounded by the curves y = f (x),the X-axis and
(a) (b) (c) (d)
6 3 2 3 the ordinates x = 1 and x = b is (b − 1) sin (3b + 4). Then,
f (x) is equal to (1982, 2M)
21. The area (in sq units) of region described by (x, y) y ≤ 2x 2
(a) (x − 1) cos (3x + 4)
and y ≥ 4x − 1 is (2015 JEE Main) (b) 8sin (3x + 4)
7 5 15 9
(a) (b) (c) (d) (c) sin (3x + 4) + 3(x − 1) cos (3x + 4)
32 64 64 32 (d) None of the above
332 Area

31. The slope of tanget to a curve y = f (x) at [x, f (x)] is 2x + 1. 40. Let b ≠ 0 and for j = 0,1,2..., n. If S j is the area of the
If the curve passes through the point (1, 2), then the region bounded by the Y-axis and the curve
area bounded by the curve, the X-axis and the line x = 1 jπ ( j + 1)π
is xe = sin by,
ay
≤ y≤ . Then, show that
3 4 5 1
b (b)
(a) (b) (c) (d) S 0 , S1 , S 2,... , S n are in geometric progression. Also,
2 3 6 12
find their sum for a = − 1 and b = π . (2001, 5M)

Objective Questions II 41. If f (x) is a continuous function given by


f (x) =  2 |x| ≤ 1
(One or more than one correct option) 2x,
x + ax + b, |x| > 1
32. If the line x = α divides the area of region
Then, find the area of the region in the third quadrant
R = {(x, y) ∈ R2 : x3 ≤ y ≤ x, 0 ≤ x ≤ 1} into two equal bounded by the curves x = −2 y2 and y = f (x) lying on the
parts, then (2017 Adv.)
left on the line 8x + 1 = 0. (1999, 5M)
(a) 2α 4 − 4α 2 + 1 = 0 (b) α 4 + 4α 2 − 1 = 0
1 1 42. Let C1 and C 2 be the graphs of functions y = x2 and
(c) < α < 1 (d) 0 < α ≤
2 2 y = 2x, 0 ≤ x ≤ 1, respectively. Let C3 be the graph of a
function y = f (x), 0 ≤ x ≤ 1, f (0) = 0. For a point P on C1,
33. If S be the area of the region enclosed by
2 let the lines through P, parallel to the axes, meet C 2 and
y = e− x , y = 0, x = 0 and x = 1. Then, (2012) C3 at Q and R respectively (see figure). If for every
1 1 position of P(on C1) the areas of the shaded regions OPQ
(a) S ≥ (b) S ≥ 1 −
e e and ORP are equal, then determine f (x). (1998, 8M)
1 1 1 1  1 
(c) S ≤  1 +  (d) S ≤ + 1 −  Y
4 e 2 e  2 (0,1) (1/2,1) (1,1)
34. Area of the region bounded by the curve y = e and lines x C2 C1
x = 0 and y = e is (2009)
e Q P
(a) e − 1 (b) ∫1 ln (e + 1 − y) dy
1 e
(c) e − ∫ ex dx (d) ∫1 ln y dy
0 X
(0,0) O C (1,0)
3
35. For which of the following values of m, is the area of the R
region bounded by the curve y = x − x2 and the line
9 43. Letf (x) = max { x , (1 − x) , 2x (1 − x)}, where 0 ≤ x ≤ 1.
2 2
y = mx equals ? (1999, 3M)
2 Determine the area of the region bounded by the curves
(a) – 4 (b) –2 (c) 2 (d) 4 y = f (x), X-axis, x = 0 and x = 1. (1997, 5M)

44. Find all the possible values of b > 0, so that the area of
Analytical & Descriptive Questions the bounded region enclosed between the parabolas
4a 2 4a 1  f (−1)  2 x2
3a + 3a  y = x − bx2 and y = is maximum.
36. If 4b2 4b 1  f (1)  = 3b2 + 3b ,
(1997C, 5M)
b
   2 
 4c2 4c 1  f (2)  3c + 3c  45. If An is the area bounded by the curve y = (tan x)n and
 
π
f (x) is a quadratic function and its maximum value the lines x = 0, y = 0 and x = .
occurs at a point V. A is a point of intersection of y = f (x) 4
with X-axis and point B is such that chord AB subtends 1
Then, prove that for n > 2 , An + An + 2 =
a right angle at V. Find the area enclosed by f (x) and n+1
chord AB. (2005, 5M) 1 1
and deduce < An < . (1996, 3M)
37. Find the area bounded by the curves x = y, x = − y 2 2 2n + 2 2n − 2
and y2 = 4x − 3. (2005, 4M) 46. Consider a square with vertices at (1,1), (–1, 1), (–1, –1)
38. A curve passes through (2,0) and the slope of tangent at and (1, –1). If S is the region consisting of all points
inside the square which are nearer to the origin than to
(x + 1) + y − 3
2
point P (x, y) equals . any edge. Then, sketch the region S and find its area.
(x + 1) (1995, 5M)
Find the equation of the curve and area enclosed by the 47. In what ratio, does the X-axis divide the area of the
curve and the X-axis in the fourth quadrant. (2004, 5M) region bounded by the parabolas y = 4x − x2 and
39. Find the area of the region bounded by the curves y = x2 − x ? (1994, 5M)
y = x2, y = | 2 − x2| and y = 2 , 48. Sketch the region bounded by the curves y = x2 and
which lies to the right of the line x = 1. (2002, 5M)
y = 2 / (1 + x2). Find its area. (1992, 4M)
Area 333

49. Sketch the curves and identify the region bounded by 55. Sketch the region bounded by the curves y = 5 − x2 and
x = 1 /2, x = 2, y = log x and y = 2x . Find the area of this y = |x − 1|and find its area. ( 1985, 5M)
region. (1991, 4M)
56. Find the area of the region bounded by the X-axis
50. Compute the area of the region bounded by the curves π π
and the curves defined by y = tan x, − ≤x≤ and
log x 3 3
y = ex log x and y = , where log e = 1. (1990, 4M) π π
ex y = cot x, ≤x≤ . (1984, 4M)
51. Find all maxima and minima of the function 6 3
y = x (x − 1)2, 0 ≤ x ≤ 2 . 57. Find the area bounded by the X-axis, part of the curve
Also, determine the area bounded by the curve  8
y = 1 + 2 and the ordinates at x = 2 and x = 4. If the
y = x (x − 1)2, the Y-axis and the line x = 2 . (1989, 5M)  x 
52. Find the area of the region bounded by the curve ordinate at x = a divides the area into two equal parts,
C: y = tan x, tangent drawn to C at x = π / 4 and the then find a. (1983, 3M)
X-axis. (1988, 5M) 58. Find the area bounded by the curve x2 = 4 y and the
53. Find the area bounded by the curves straight line x = 4 y − 2.
x2 + y2 = 25, 4 y = |4 − x2|and x = 0 above the X-axis. et + e– t et – e– t
(1987, 6M) 59. For any real t, x = ,y= is a point on the
2 2
54. Find the area bounded by the curves x2 + y2 = 4, hyperbola x2 – y2 = 1. Find the area bounded by this
x2 = − 2 y and x = y. (1986, 5M) hyperbola and the lines joining its centre to the points
corresponding to t1 and – t1. (1982, 3M)

Answers
Topic 1  20 − 12 2   π (1 + e ) (en + 1 − 1 ) 
39.   sq units 40.  ⋅
1. (a) 2. (b) 3. (b) 4. (d)  3   (1 + π )
2
e − 1 
5. (a) 6. (d) 7. (b) 8. (c)  761
41.   sq units 42. f ( x ) = x 3 − x 2, 0 ≤ x ≤ 1
9. (a,d) 10. (4) 11. 2 3 sq units  192
12. 2 sq units 13. (2 − 3 ) sq unit 17 1 
43. sq unit 44. b = 1 46. (16 2 − 20 ) sq units
14. (b) 15. (a) 16. (d) 27 3 
 2
Topic 2 47. 121 : 4 48.  π −  sq units
 3
1. (a) 2. (b) 3. (a) 4. (c) 4 − 2 5  e 2 − 5
3
5. (c) 6. (b) 7. (d) 8. (d) 49.  − log 2 +  sq units 50.   sq units
 log 2 2 2  4e 
9. (b) 10. (c) 11. (b) 12. (c)
 4 10 
13. (a) 14. (d) 15. (b) 16. (a) 51. y max = , y min = 0, sq units
 27 3 
17. (a) 18. (a) 19. (d) 20. (c)
 1    4 
21. (d) 22. (a) 23. (b) 24. (a) 52.  log 2 −  sq units 53. 4 + 25 sin −1    sq units
  4     5 
25. (c) 26. (b) 27. (b) 28. (a)
1   5π 1
29. (a) 30. (c) 31. (c) 32. (a, c) 54.  − π  sq units 55.  −  sq units
3   4 2
125
33. (b, d) 34. (b, c, d) 35. (b, d) 36. sq units  1  9
3 56.  loge 3 sq units 57. 2 2 58. sq units
2  8
1 4
37. sq unit 38. y = x 2 – 2 x, sq units e 2t1 − e −2t1 1 2t1
3 3 59. − (e − e −2t1 − 4t1 )
4 4
Hints & Solutions
Topic 1 Area Based on Geometrical Figures Now, the required area is the area of shaded region, i.e.
Without Using Integration Y

1. Equation of given parabola y = x2 – 1 According to


A (2, 2)
symmetry let, the coordinate of A (– a , 0), B(a , 0) x 2 + y 2 = 4x
C (a , a 2 – 1) and D (– a , a 2 – 1).
X′ X
∴Area of rectangle P (a ) = 2a (a 2 – 1) (0, 0) B (2,0)
y2 = 2x
Y

Y′
y=x2–1
Area of a circle 2
Required area = − ∫ 2x dx
4 0
2
π (2)2 2  x3/ 2 
A(–a, 0) B(a, 0)
X = − 2 ∫ x1/ 2dx = π − 2  
(–1, 0) O (1, 0) 4 0
 3 /2  0
2 2  8
C(a, a2–1) =π− [2 2 − 0] =  π −  sq unit
D
(–a, a2–1) 3  3
(0, –1)
4. PLAN (i) y = mx + a / m is an equation of tangent to the parabola
y 2 = 4ax .
Now, for maxima P ' (a ) = 0 ⇒ 2(a – 1) + 4a = 02 2
(ii) A line is a tangent to circle, if distance of line from centre is
1 equal to the radius of circle.
⇒ 3a 2 = 1 ⇒ a = units
3 (iii) Equation of chord drawn from exterior point ( x 1, y1 ) to a
circle/parabola is given by T = 0.
∴ Area of largest rectangle is
1
2 1  4 (iv) Area of trapezium = (Sum of parallel sides)
 – 1 = sq. units 2
3 3  3 3 2
Let equation of tangent to parabola be y = mx +
2. We know that, area of region bounded by the parabolas m
x2 = 4ay and y2 = 4bx is
16
(ab) sq units. It also touches the circle x2 + y2 = 2.
3  
2
On comparing y = kx2 and x = ky2 with above equations, ∴  = 2
m 1 + m 
1 1 2
we get 4a = and 4b =
k k ⇒ m4 + m2 = 2
1 1
⇒ a= and b = ⇒ m + m2 − 2 = 0
4
4k 4k
⇒ (m − 1) (m2 + 2) = 0
2
∴ Area enclosed between y = kx2 and x = ky2 is ⇒ m = ± 1,m2 = − 2 [rejected m2 = − 2]
16  1   1  1
   = 2 So, tangents are y = x + 2, y = − x − 2.

3 4k 4k   3k
They, intersect at (−2, 0).
1
⇒ = 1 [given, area = 1 sq.unit] R
3k2 Y
1 1 P
⇒ k2 = ⇒ k=± T(–2,0)
3 3
X′ X
1 O
⇒ k= [Q k > 0]
3 Q
3. Given equations of curves are y2 = 2x Y′ S
which is a parabola with vertex (0, 0) and axis parallel
Equation of chord PQ is −2x = 2 ⇒ x = − 1
to X-axis. . ..(i)
And x2 + y 2 = 4 x Equation of chord RS is O = 4(x − 2) ⇒ x = 2
which is a circle with centre (2, 0) and radius = 2 ...(ii) ∴ Coordinates of P, Q, R, S are
On substituting y2 = 2x in Eq. (ii), we get P (−1, 1), Q (−1, − 1), R(2, 4), S (2, − 4)
x2 + 2x = 4x ⇒ x2 = 2x ⇒ x = 0 or x = 2 (2 + 8) × 3
∴ Area of quadrilateral = = 15 sq units
⇒ y = 0 or y = ± 2 [using Eq. (i)] 2
Area 335

5. Since, tangents drawn from external points to the circle 7. The region is clearly square with vertices at the points
subtends equal angle at the centre. (1, 0), (0, 1), (– 1, 0) and (0, – 1).
A Y

(0,1) y = –IxI + 1
y = IxI –1
O3
X′ X
(1,0)
(–1,0)
O1 O2 (0,–1)
1cm 1cm
30° 30° Y′
B C
3 cm D 2 cm E 3 cm ∴ Area of square = 2 × 2 = 2 sq units
8. Let y = f (x) = x2 + bx − b
∴ ∠ O1BD = 30°
Y
OD
In ∆O1BD, tan 30° = 1
BD B
⇒ BD = 3 cm (1,1)
P
Also, DE = O1O2 = 2 cm and EC = 3 cm
X
Now, BC = BD + DE + EC = 2 + 2 3 O A
3
⇒ Area of ∆ABC = (BC )2 The equation of the tangent at P (1, 1)
4 to the curve 2 y = 2x2 + 2bx − 2b is
3 y + 1 = 2x ⋅ 1 + b (x + 1) − 2b ⇒ y = (2 + b) x − (1 + b)
= ⋅ 4 (1 + 3 )2 = (6 + 4 3 ) sq cm
4 Its meet the coordinate axes at
x2 y2 1+ b
6. Given, + =1 xA = and yB = − (1 + b)
9 5 2+ b
To find tangents at the end points of latusrectum, we 1
∴ Area of ∆ OAB = OA × OB
find ae. 2
i.e. ae = a 2 − b2 = 4 = 2 1 (1 + b)2
=− × =2 [given]
2 (2 + b)
 4 5
and b2(1 − e2) = 5 1 −  = ⇒ (1 + b)2 + 4(2 + b) = 0 ⇒ b2 + 6b + 9 = 0
 9 3
⇒ (b + 3)2 = 0 ⇒ b = −3
By symmetry, the quadrilateral is a rhombus.
9. Since, ∠ POQ = 90°
Y
A Y t 12
P 2 , t1
L(ae, √ b2(1 – e2))

X′ X
D O B
X′ X
L′ O
(0, 0)
C
Y′ t 22
Q 2 , t2
So, area is four times the area of the right angled triangle Y′
formed by the tangent and axes in the Ist quadrant. t1 − 0 t2 − 0
⇒ ⋅ 2 = − 1 ⇒ t1t2 = − 4 …(i)
 5 t12 t
∴ Equation of tangent at 2,  is −0 2 −0
 3 2 2
2 5 y x y Q ar (∆OPQ ) = 3 2
x+ ⋅ =1 ⇒ + =1
9 3 5 9 /2 3 0 0 1
1 2
∴ Area of quadrilateral ABCD ∴ t1 / 2 t1 1 = ± 3 2
2 2
=4 [area of ∆ AOB] t2 / 2 t2 1
1 9 
= 4  ⋅ ⋅ 3 = 27 sq units 1  t12t2 t1t22
2 2  ⇒  −  =±3 2
2 2 2 
336 Area

1 4
⇒ ( −4t1 + 4t2) = ± 3 2 ⇒ t1 + = 3 2 [Q t1 > 0 for P] 13. Let the coordinates of P be (x, y).
4 t1 Y
⇒ t12 − 3 2t 1 + 4 = 0 ⇒ (t1 − 2 2 ) (t1 − 2 ) = 0 B
⇒ t1 = 2 or 2 2
∴ P (1, 2 ) or P (4, 2 2 )
P
10. We have, y = xn , n > 1
Q P (0, 0) Q (1, 1) and R(2, 0) are vertices of ∆ PQR. X′
O (0, 0) A (2, 0)
X
y
Q(1,1) Y′
Equation of line OA be y = 0.
x

Equation of line OB be 3 y = x.
y=

F2 F1
y = xn Equation of line AB be 3 y = 2 − x.
x′ x
P(0,0) (1,0) R(2,0) d (P , OA ) = Distance of P from line OA = y
| 3 y − x|
d (P , OB) = Distance of P from line OB =
2
| 3 y + x − 2|
d (P , AB) = Distance of P from lineAB =
y′ 2
Given, d (P , OA ) ≤ min { d (P , OB), d (P , AB)}
∴ Area of shaded region = 30% of area of ∆ PQR
 | 3 y − x| | 3 y + x − 2|
1 30 1 y ≤ min  , 
⇒ ∫0 (x − x ) dx = 100 × 2 × 2 × 1
n
 2 2 
 x2 xn + 1  1
1 | 3 y − x| | 3 y + x − 2|
3 1  3 ⇒ y≤ and y ≤
⇒  2 − n + 1  = 10 ⇒  −  = 2 2
 0  2 n + 1  10
| 3 y − x|
1 1 3 2 1 Case I When y ≤ [since, 3 y − x < 0]
⇒ = − = = ⇒ n + 1 =5 ⇒ n =4 2
n + 1 2 10 10 5 x − 3y
y≤ ⇒ (2 + 3 ) y ≤ x ⇒ y ≤ x tan 15°
11. Equation of tangent at the point (1, 3 ) to the curve 2
| 3 y + x − 2|
x2 + y2 = 4 is x + 3y = 4 Case II When y ≤ ,
2
whose X-axis intercept (4, 0).
2 y ≤ 2 − x − 3 y [since, 3 y + x − 2 < 0]
Y
⇒ (2 + 3 ) y ≤ 2 − x ⇒ y ≤ tan 15°⋅ (2 − x)
P (1,√3) Y
B (1, 1/ 3)

X′ X
(0,0) O A (4,0)
P
A
X′ X
O (0, 0) C (2, 0)
Y′ (1, 0)
Thus, area of ∆ formed by (0, 0) (1, 3 ) and (4, 0)
0 0 1 Y′
1 1 From above discussion, P moves inside the triangle as
= 1 3 1 = |(0 − 4 3 )|= 2 3 sq units
2 2 shown below :
4 0 1 ⇒ Area of shaded region
12. The area formed by | x| + | y| = 1 is square shown as = Area of ∆OQA
below : 1
Y
= (Base) × (Height)
2
1
= (2) (tan 15° ) = tan 15° = (2 − 3 ) sq unit
2
−x + y = 1 x+y=1
14. Given, y3 − 3 y + x = 0
X' X
−1 O 1 dy dy
⇒ 3 y2 −3 + 1 =0 …(i)
x+y=1 x−y=1 dx dx
 d 2y  dy
2
d 2y
⇒ 3 y2 2  + 6 y   − 3 =0 …(ii)
 dx   dx dx2
Y'
∴ Area of square = ( 2 )2 = 2 sq units At x = − 10 2 , y = 2 2
Area 337

On substituting in Eq. (i) we get So, required area is


dy dy 1/ 2 log 3e + 1
3(2 2 )2 ⋅
dx
− 3⋅
dx
+ 1 =0 ∫0 ( g (x) − f (x))dx

dy 1 1/ 2 log 3e + 1 1/ 2 log 3e + 1
⇒ =− =∫ g (x)dx − ∫ f (x)dx
dx 21 0 1

Again, substituting in Eq. (ii), we get 1 1/ 2log 3e + 1 x − 1 1/ 2 log 3e + 1


+ e1 − x )dx − (ex − 1 − e1 − x ) dx
2 ∫0 ∫1
2
= (e
2 2
d y  1 d y
3(2 2 )2 + 6 (2 2 ) ⋅  −  − 3 ⋅ 2 = 0 Y y=f(x)
dx2  21  dx
d 2y 12 2 y=g(x)
⇒ 21 ⋅ 2
=−
dx (21)2
d y − 12 2 − 4 2
2
⇒ = = 3 2
dx2 (21)3 7 ⋅3
b b
15. Required area = ∫ y dx = ∫ f (x) dx
a a X
b
O 1 ½ loge3 +1
= [ f (x) ⋅ x]ba − ∫ f ′ (x)x dx
a 1 3 1 3
1 x −1 log e + 1 log e + 1
= bf (b) − af (a ) − ∫ f ′ (x)x dx
b = [e − e1 − x ]02 − [ex − 1 + e1 − x ]12
a 2
b xdx 1 1   1 
= bf (b) − af (a ) + ∫a 3[{ f (x)}2 − 1] = 3− − e−1 + e − 3 + − 1 −1
2  3   3 
 −1 −1  1 1 4 1
dy = + (e − e−1 ) − + 2 = (2 − 3 ) + (e − e−1 )
Q f ′ (x) = dx = =  3 2 3 2
 3 ( y2
− 1 ) 3 [{ f (x )} 2
− 1 ] 
1 2. According to the question,
16. Let I = ∫ g′ (x) dx = [ g (x)]1− 1 = g (1) − g (− 1) y
−1 y=x2
y=2x
Since, y3 − 3 y + x = 0 …(i) (2, 4)
and y = g (x)
∴ { g (x)}3 − 3 g (x) + x = 0 [from Eq. (i)] y=α
At x = 1, { g (1)} − 3 g (1) + 1 = 0
3
…(ii)
x
At x = − 1, { g (− 1)}3 − 3 g (− 1) − 1 = 0 …(iii)
O
On adding Eqs. (i) and (ii), we get
{ g (1)}3 + { g (− 1)}3 − 3 { g (1) + g (− 1)} = 0
α y 4
⇒ [ g (1) + g (− 1)][{ g (1)}2 + { g (− 1)}2 − g (1) g (− 1) − 3] = 0 ∫0  y −  dy = ∫ ( y − y / 2) dy
2 α
⇒ g (1) + g (− 1) = 0
α α α 2 4(2) 16 α α α 2
⇒ g (1) = − g ( − 1) ⇒ − = − − +
3 /2 4 3 /2 4 3 /2 4
∴ I = g (1) − g (− 1)
4 α 2
64 − 48 16 4
= g (1) − { − g (1)} = 2 g (1) ⇒ α α − + = =
3 2 12 12 3
⇒ 3 α 2 − 8 α3/ 2 + 8 = 0
Topic 2 Area Using Integration Hence, option (b) is correct.
1. The given functions f : R → R and g : R → R be defined
3. As we know that,
by
x −1 1 −x  0, 0 ≤ x < 1
e −e , x≥1 
f (x) = ex − 1 − e−|x − 1| = y = (x – 1)[x] =  x – 1, 1 ≤ x < 2
0, x<1 2(x – 1), x = 2
1 x −1 
1−x
and g (x) = (e +e )
2 Now, on drawing the graph of given region with the
For point of intersection of curves f (x) and g (x) put help of equation of curves y = (x – 1)[x] and y = 2 x
y
f (x) = g (x) y=2√x
1
for x ≥ 1, ex − 1 − e1 − x = (ex − 1 + e1 − x )
2
⇒ ex − 1 = 3e1 − x 1
x–
1 y=
⇒ e2x = 3e2 ⇒ x = log3e + 1 O 1 2
x
2
338 Area

∴Area of given region Now, required area is


1 2 4/ λ
= ∫ 2 x dx + ∫ (2 x – x + 1)dx
0 1
2
= ∫ (2 λx − λx) dx
4 3/ 2
1
4 x  2 0
4/ λ
= x +  x3/ 2 – + x  
3  3 2 4/ λ
0  1  x3 / 2   x2 
=2 λ  −λ 
4  8 2 4 1  8 2 1 3 
= + – 2 + 2 – + – 1 = – sq. units.    2 0
3  3 3 2 3 2  2 0

2
4 4 4 λ  4
4. The given region {(x, y) : xy ≤ 8, 1 ≤ y ≤ x2}. λ = −  
3 λ λ 2  λ
From the figure, region A and B satisfy the given region, 32 8 32 − 24 8
= − = =
but only A is bounded region, so area of bounded region 3λ λ 3λ 3λ
Y 1
It is given that area =
xy=8 9
y=x2 8 1
⇒ =
3λ 9
Q (2,4) ⇒ λ = 24
B (1,1) A R(8,1) 6. Given region is {(x, y) : y2 ≤ 4x, x + y ≤ 1, x ≥ 0, y ≥ 0}
y=1
P
X¢ X
O B(0,1)
P
y2=4x
x+y=1
xy=8 Y¢
X
O A(1,0)
2 8 8 
A = ∫ (x − 1) dx + ∫
2
 − 1 dx
1 2 x  Now, for point P, put value of y = 1 − x to y2 = 4x, we get
[Q Points P (1, 1), Q (2, 4)and
(1 − x)2 = 4x ⇒ x2 + 1 − 2x = 4x
R(8, 1)]
 x3 
2 ⇒ x2 − 6 x + 1 = 0
=  − x + [8 log|x|− x]2
8

3 6 ± 36 − 4
 1 ⇒ x=
8 1  2
=  − 2 − + 1 + 8 log 8 − 8 − 8 log 2 + 2
3 3  = 3 ± 2 2.
14 14 Since, x-coordinate of P less than x-coordinate of point
=− + 16 log 2 = 16 log 2 −
3 3 A(1, 0).
5. Given, equation of curves are ∴ x=3 −2 2
y2 = 4λx …(i) Now, required area
and y = λx …(ii) 3 −2 2 1
=∫ 2 x dx + ∫3 − 2 2 (1 − x) dx
λ> 0 0

Area bounded by above two curve is, as per figure 3 −2 2 1


x3/ 2  x2 
=2 + x − 
Y 3 /2 0  2 3 − 2 2
A y2=4λx
(3 − 2 2 ) 2
(3 − 2 2 ) 3/ 2 +  1 −  − (3 − 2 2 ) +
4 1
=
3  2  2
O
X 4 1 1
= [( 2 − 1)2]3/ 2 + − 3 + 2 2 + (9 + 8 − 12 2 )
3 2 2
4 5 17
y=λx = ( 2 − 1)3 − + 2 2 + −6 2
3 2 2
4
the intersection point A we will get on the solving Eqs. (i) = (2 2 − 3(2) + 3( 2 ) − 1) − 4 2 + 6
and (ii), we get
3
4 4 8 2 10
λ2x2 = 4λx ⇒ x = , so y = 4. = (5 2 − 7) − 4 2 + 6 = −
λ 3 3 3
A  , 4
4 =a 2+b (given)
So,
λ 
Area 339

8 10 Now, the area enclosed by the region A


So, on comparing a = and b = − 4
3 3 4
 y2   y2 y3 

8 10
a−b= + =6
= ∫  ( y + 4) − 2 dy =  2 + 4 y − 6 
−2 −2
3 3
= 
16 64   4 8
+ 16 − −
  − 8 + 
7. Given, equations of curves  2 6  2 6
 x + 1 ,x ≥ − 1 = 8 + 16 −
32 4
− 2 + 8 − = 30 − 12 = 18 sq unit.
y = 2x and y = | x + 1| = 
− x − 1 , x < − 1
3 3

Q The figure of above given curves is


9. Given region is A = {(x, y) : x2 ≤ y ≤ x + 2}

Y Now, the region is shown in the following graph


y=x+1
Y y=x+2
(1,2)
x2=y
y=–x–1 y=2x B(2,4)
(0,1)
(0,2)
A
X′ X
(–1,0) O (–2,0)
X' –1 X'
In first quadrant, the above given curves intersect each O 2
other at (1, 2). Y'
1 For intersecting points A and B
So, the required area = ∫ ((x + 1) − 2 ) dx x
0 Taking, x2 = x + 2 ⇒ x2 − x − 2 = 0
1
x 2
2  x  ax  ⇒ x2 − 2x + x − 2 = 0
= + x− Q ∫ a dx = log a + C 
x
2 log  ⇒ x(x − 2) + 1(x − 2) = 0
 e 2 0  e 
⇒ x = −1, 2 ⇒ y = 1, 4
1 2 1 
= +1− + So, A(−1, 1) and B (2, 4).

 2 log e 2 log e 2 2
Now, shaded area = ∫ [(x + 2) − x
2
3 1 ] dx
= − −1
2 log e 2 2
 x2 x3 
=  + 2x −  =  + 4 −  −  − 2 + 
4 8 1 1
 y2    
8. Given region A = (x, y) : ≤ x ≤ y + 4  2 3  −1 2 3 2 3
 2  1 9 1 1 9
y2 =8− − = 8 − − 3 = 5 − = sq units
∴ =x 2 3 2 2 2
2
⇒ y2 = 2x …(i) 10. Given, S (α ) = {(x, y) : y2 ≤ x, 0 ≤ x ≤ α } and
and x = y + 4 ⇒ y = x − 4 …(ii) A(α ) is area of the region S(α )
Y
Graphical representation of A is
y 2 =x
Y
y2
=x
Q 2

X
X' X O
O A(λ)

P
4
y+
x=

Y' x=λ
λ λ
On substituting y = x − 4 from Eq. (ii) to Eq. (i), we get  x3/ 2  4
(x − 4)2 = 2x Clearly, A (λ ) = 2∫ x dx = 2   = λ3/ 2
0 3 / 2 0 3
⇒ x − 8x + 16 = 2x
2
A (λ ) 2
⇒ x2 − 10x + 16 = 0 Since, = , (0 < λ < 4)
A (4) 5
⇒ (x − 2)(x − 8) = 0
3 2
⇒ x = 2, 8 λ3/ 2 2  λ  2
⇒ = ⇒   = 
∴ y = − 2, 4 [from Eq. (ii)] 43/ 2 5  4  5
So, the point of intersection of Eqs. (i) and 1/3 1/3
λ 4 4
(ii) are P (2, − 2) and Q(8, 4). ⇒ =  ⇒ λ =4  
4  25  25
340 Area

11. Given equations of the parabola y2 = 4x …(i) 14. Given, equation of parabola is y = x2 + 1, which can be
and circle x + y =5 2 2
…(ii) written as x2 = ( y − 1). Clearly, vertex of parabola is
So, for point of intersection of curves (i) and (ii), put (0, 1) and it will open upward.
y2 = 4x in Eq. (ii), we get y+5
Now, equation of tangent at (2, 5) is = 2x + 1
x2 + 4 x − 5 = 0 2
⇒ x + 5x − x − 5 = 0
2 [QEquation of the tangent at (x1 , y1 ) is given by
1
⇒ (x − 1)(x + 5) = 0 T = 0. Here, ( y + y1 ) = xx1 + 1]
2
⇒ x = 1, − 5
For first quadrant x = 1 , so y = 2 . y = 4x − 3
y= 4x–3
Now, equation of tangent of parabola (i) at point (1, 2)
is T = 0 Y

⇒ 2 y = 2(x + 1) ⇒ x − y + 1 = 0
 3 7 P (2, 5)
The point  ,  satisfies, the equation of line
 4 4
x− y+ 1 =0
(0, 1)
12. Given, y ≤ x + 3x 2
R
2 2 X
 3 9  3  9 O Q (2, 0)
⇒ y ≤ x +  − ⇒ x +  ≥  y + 
 2 4  2  4 3,
0
4
Since, 0 ≤ y ≤ 4 and 0 ≤ x ≤ 3
∴The diagram for the given inequalities is
Y
y=x2+3x
Required area = Area of shaded region
2
= ∫ y(parabola) dx − (Area of ∆PQR)
0
2
y=4 = ∫ (x2 + 1) dx − (Area of ∆PQR)
0
2
–3/2
X  x3  1 3
(–3, 0) O 1 3 = + x − 2 −  ⋅ 5

9  3 0 2  4
4 x=3
1
[QArea of a triangle = × base × height]
and points of intersection of curves y = x2 + 3x and y = 4 2
are (1, 4) and (−4, 4) 8  1  5
=  + 2 − 0 −   5
3  2  4
Now required area
1 14 25 112 − 75 37
1
 x3 3x2 
3
= − = =
= ∫ (x2 + 3x)dx +
∫ 4 dx =  3 + 2  + [4x]1
3
3 8 24 24
0 1   0
15. Given equation of curve is x2 = 4 y, which represent a
1 3 2+9 11 59
= + + 4(3 − 1) = +8 = +8= sq units parabola with vertex (0, 0) and it open upward.
3 2 6 6 6
Y x2
y=
13. Given equation of parabola is y = x2 + 2, and the line is 4
y = x+1 y=
x+2
y y=x2 +2 4
B
A
y=x+1 X′ X
–1 O 2
(0,2)

x Y′
1 O (3,0)
Now, let us find the points of intersection of x2 = 4 y and
The required area = area of shaded region 4y = x + 2
3 3 For this consider, x2 = x + 2 ⇒ x2 − x − 2 = 0
=∫ ((x2 + 2) − (x + 1)) dx = ∫ (x2 − x + 1) dx
0 0 ⇒ (x − 2) (x + 1) = 0 ⇒ x = − 1, x = 2
3 1
 x3 x2   27 9  When x = − 1, then y =
=  − + x =  − + 3 − 0 4
 3 2 0  3 2 
and when x = 2, then y = 1
Thus, the points of intersection are A  − 1,  and B (2, 1).
9 9 15 1
= 9 − + 3 = 12 − = sq units  4
2 2 2
Area 341

Now, required area = area of shaded region and y1 = 3.


2
y+3
= ∫−1 {y (line) − y (parabola)} dx ⇒ = 2x − 1 ⇒ y = 4x − 5
2
2
 x + 2 x2 
2 1  x2 x3  Now, required area = area of shaded region
=∫−1  4 − 4  dx = 4  2 + 2x − 3  2
−1 = ∫ (y(parabola) − y(tangent)) dx
0
1  8  1 1 
= 2 + 4 −  −  − 2 +  2
4   3  2 3  = ∫ [(x2 − 1) − (4x − 5)] dx
0
1  1  1  1 9 2 2
= 8 − −3 = 5− = sq units. = ∫ (x2 − 4x + 4) dx = ∫ (x − 2)2 dx
4  2  4  2  8 0 0
2
16. We have, (x − 2)3 (2 − 2)3 (0 − 2)3 8
= = − = sq units.
A = {(x, y) : 0 ≤ y ≤ x| x|+ 1and − 1 ≤ x ≤ 1} 3 0
3 3 3
When x ≥ 0, then 0 ≤ y ≤ x2 + 1
18. We have,
and when x < 0, then 0 ≤ y ≤ − x2 + 1
⇒ 18x2 − 9πx + π 2 = 0
Now, the required region is the shaded region. ⇒ 18x − 6πx − 3πx + π 2 = 0
2
y
(6x − π )(3x − π ) = 0
π π
2 ⇒ x= ,
6 3
y=–x2+1 1 y=x2+1 π
Now, α < β α= ,
6
x π
–1 1 β=
y=0 3
Given, g(x) = cos x and f (x) = x
2
[Q y = x2 + 1⇒ x2 = ( y − 1), parabola with vertex (0, 1) and
y = − x2 + 1⇒ x2 = − ( y − 1) , y = gof (x)
∴ y = g ( f (x)) = cos x
parabola with vertex (0,1) but open downward]
Area of region bounded by x = α,x = β, y = 0 and curve
We need to calculate the shaded area, which is equal to
y = g ( f (x)) is
0 1 π /3
∫−1 (− x + 1)dx + ∫ (x2 + 1) dx
2
0
0 1
A= ∫ cos x dx
π /6
 x 3 x  3
= − + x +  + x A = [sin x]ππ //36
 3 − 1  3 0 π π 3 1
A = sin − sin = −
  (− 1)3   1  3 6 2 2
=  0 − − + (− 1)  +   + 1 − 0


   3    3 − 1
 3  A= 
1  4 2 4  2 
= −  − 1 + = + = 2
3  3 3 3 19. Required area
17. Given, equation of parabola is y = x − 1, which can be 2 1 2 2 x2
=∫ (1 + x )dx + ∫1 (3 − x)dx − ∫ dx
rewritten as x2 = y + 1 or x2 = ( y − (−1)). 4
0 0

Y
⇒ Vertex of parabola is (0, − 1) and it is open upward. y=1+√x
(0, 3) (1, 2)
y=x2–1 4y=x 2
(2, 3)
(0, 1) (2, 1)
x+y=3
2 X′ X
(0, 0) (1, 0)(2, 0) (3, 0)

(0, –1) Y′
y=4x–5
 3/ 2  1

2 2
x x2   x3 
= x +  + 3x −  −  
 3 /2  0  2  1 12  0
 2  1  8 
= 1 +  + 6 − 2 − 3 +  −  
Equation of tangent at (2, 3) is given by T = 0  3  2  12
y + y1 5 3 2
= + −
3 5
= 1 + = sq units
⇒ = x x1 − 1, where, x1 = 2
2 3 2 3 2 2
342 Area

20. Here, {(x, y) ∈ R2 : y ≥ |x + 3|, 5 y ≤ (x + 9) ≤ 15} and y ≥ 4x − 1 represents a region to the left of the line
∴ y ≥ x+3 y = 4x − 1 …(ii)
The point of intersection of the curves (i) and (ii) is
 x + 3 , when x ≥ − 3
⇒ y≥  (4x − 1)2 = 2x
 − x − 3 , when x ≤ − 3 ⇒ 16x + 1 − 8x = 2x
2

 x + 3 , when x ≥ − 3 ⇒ 16x2 − 10x + 11 = 0


or y2 ≥  1 1
− 3 − x, when x ≤ − 3 ⇒ x= ,
2 8
Shown as 1 
So, the points where these curves intersect are  , 1
y2=–x–3
Y 2 
y2=x+3  1 1
and  ,  .
 8 2
Y

−1
4x
1,1

y=
X′ X
–3 0 2
1 y 2 = 2x

1
Y′
2
−1 1
Also, 5 y ≤ (x + 9) ≤ 15 −1 2 2 1
X′ X
⇒ (x + 9) ≥ 5 y and x ≤ 6 O
1 −1
,
Shown as −1 8 2
Y 2

−1
(0, 9/5) Y′

X
0 1  y + 1 y2 
(–9, 0) ∴ Required area = ∫  −  dy
−1/ 2  4 2
x=6
−1
1  y2  1
=  + y − ( y3 )1−1/ 2
∴ {(x, y) ∈ R2 : y ≥ |x + 3|, 5 y ≤ (x + 9) ≤ 15} 42  −1/ 2 6
Y 1  1 1  1 
 1 1
=  + 1 −  −   − 1 + 
(1,2) 4 2 8 2  6 8
) C
B (–
4,1 1 3 3  1 9 
=  + −  
(1,0) 4 2 8  6 8 
X′ X
–9 (–4,0) A 0
E (–3,0) D 6 1 15 3 9
= × − = sq units
4 8 16 32
22. Given, A = {(x, y) : x2 + y2 ≤ 1 and y2 ≤ 1 − x}
Y′
Y
∴ Required area = Area of trapezium ABCD
− Area of ABE under parabola
− Area of CDE under parabola
1 −3 1
= (1 + 2) (5) − ∫ − (x + 3) dx − ∫ (x + 3) dx X′
(–1,0) (0,1)
X
2 − 4 − 3
−3 1
   
15  (− 3 − x)3/ 2   (x + 3)3/ 2 
= −  −
2  
3 3 Y′
 −   
 2 − 4  2  –3 1 2 1
Required area = πr + 2∫ (1 − y2)dy
15 2 2 15 2 16 15 18 3 2 0
= + [0 − 1] − [8 − 0] = − − = − = 1
2 3 3 2 3 3 2 3 2 1  y3 
= π (1)2 + 2  y − 
21. Given region is {(x, y) : y2 ≤ 2x and y ≥ 4x − 1} 2  3 0
y2 ≤ 2x repressents a region inside the parabola  π 4
=  +  sq units
y2 = 2x …(i)  2 3
Area 343

23. PLAN To find the bounded area between y = f( x ) and y = g ( x ) Hence, required area
between x = a to x = b. 3 3
= ∫ (x2 − x1 ) dy = ∫ {(2 y + 3) − y2} dy
Y 0 0
f(x) 3
 y3 
=  y2 + 3 y − = 9 + 9 − 9 = 9 sq units
3  0
g(x)

g(x) 2
f(x) 25. R1 = ∫ x f (x) dx …(i)
−1
a c X
O b b b

c b
Using ∫ a f (x) dx = ∫ a f (a + b − x) dx
∴ Area bounded = ∫a[g (x ) − f(x )]dx + ∫c [f(x ) − g (x )]dx 2
b R1 = ∫ (1 − x) f (1 − x) dx
= ∫a | f(x ) − g (x )|dx −1
2
π ∴ R1 = ∫ (1 − x) f (x) dx …(ii)
Here, f (x) = y = sin x + cos x, when 0 ≤ x ≤ −1
2
[f (x) = f (1 − x), given]
and g (x) = y = | cos x − sin x|
 π Given, R2 is area bounded by f (x), x = − 1 and x = 2.
cos x − sin x, 0 ≤ x ≤
 2
= 4 ∴ R2 = ∫ f (x) dx …(iii)
π π −1
sin x − cos x, ≤x≤
 4 2 On adding Eqs. (i) and (ii), we get
could be shown as 2
2R1 = ∫ f (x) dx …(iv)
Y −1
f(x) y = sin x + cos x
√2 From Eqs. (iii) and (iv), we get
= √2 sin x + π
4 2R1 = R2
1
26. Here, area between 0 to b is R1 and b to 1 is R2.
g(x) g(x)
b 1 1
O π/4 π/2
X ∴ ∫0 (1 − x)2 dx − ∫b (1 − x)2 dx =
4
b 1
π /4  (1 − x)3   (1 − x)3  1
∴ Area bounded = ∫ {(sin x + cos x) − (cos x − sin x)} dx ⇒  −3  −  −3  = 4
0
 0  b
π /2
+ ∫π / 4 {(sin x + cos x) − (sin x − cos x)} dx ⇒−
1 1
[(1 − b)3 − 1] + [0 − (1 − b)3 ] =
1
π /4 π /2 3 3 4
=∫ 2 sin x dx + ∫π / 4 2 cos x dx
0 2 1 1 1 1
⇒ − (1 − b) = − + = −
3
⇒ (1 − b)3 =
= − 2 [cos x]π0 / 4 + 2 [sin x ⋅ n ]ππ // 24 3 3 4 12 8
= 4 − 2 2 = 2 2( 2 − 1) sq units ⇒ (1 − b) =
1
⇒ b=
1
2 2
24. Given curves are y = x …(i)
π / 4  1 + sin x 1 − sin x 
and 2y − x + 3 = 0 …(ii) 27. Required area = ∫  −  dx
Y
0  cos x cos x 
 1 + sin x 1 − sin x 
x Q > >0
y= 0  cos x cos x 
3=
x+  
2y – x x
X' X  2 tan 2 tan 
3  1+ 2 1− 2 
–3  1 + tan 2x
1 + tan 2 
x
π /4 
2
=∫ 2 − 2  dx
Y' 0  2x 2x

 1 − tan 1 − tan 
 2 2 
On solving Eqs. (i) and (ii), we get  x x 
1 + tan 2 1 + tan 2 
2 x − ( x )2 + 3 = 0  2 2 
⇒ ( x )2 − 2 x − 3 = 0  x x
 1 + tan 1 − tan 
⇒ ( x − 3) ( x + 1) = 0 ⇒ x =3 =∫
π /4
 2 − 2  dx
[since, x = − 1 is not possible] 0  1 − tan x 1 + tan 
x

∴ y=3  2 2
344 Area

1 + tan
x
− 1 + tan
x
2 tan
x ∴ Required area OABCO = Area of curve OCBDO
π /4
=∫ 2 dx = π / 4
0
2
x ∫0 2 dx
x
– Area of curve OABDO
1 − tan 2 1 − tan 2 1/a  x 
⇒ ∫ 0  a − ax  dx = 1
2
2 2 [given]
π
x 1 x tan 4t dt
Put tan = t ⇒ sec2 dx = dt = ∫ 8
 1 x3/ 2 ax3  2
1/ a
1
2 2 2 0
(1 + t 2) 1 − t 2 ⇒  ⋅ −  =1 ⇒ 2
− =1
 a 3 /2 3 0 3a 3a 2
2 −1 4t dt π
As ∫0 (1 + t 2) 1 − t 2
[Q tan
8
= 2 − 1]
⇒ a2 =
1
⇒ a=
1
[Q a > 0]
3 3
28. The curves y = (x − 1)2, y = (x + 1)2 and y = 1 /4 are b
30. Since, ∫ f (x) dx = (b − 1) sin (3b + 4)
1
shown as
Y y = (x + 1)2 y = (x – 1)
2 On differentiating both sides w.r.t. b, we get
f (b) = 3(b − 1) ⋅ cos (3b + 4) + sin (3b + 4)
∴ f (x) = sin (3x + 4) + 3(x − 1) cos (3x + 4)
dy
1/4 P 31. Given, = 2x + 1
dx
R Q
y = 1/4 On integrating both sides

–1 –1/2 O 1/2 1
X
∫ dy = ∫ (2x + 1) dx
⇒ y = x2 + x + C which passes through (1, 2)
where, points of intersection are ∴ 2 =1+1+C
1 1
(x − 1)2 = ⇒ x= ⇒ C =0
4 2 ∴ y = x2 + x
1 1
and (x + 1) = ⇒ x = −
2
Y
4 2
 1 1  1 1 y = x (x + 1)
i.e. Q  ,  and R  − , 
 2 4  2 4
1/ 2  1
∴ Required area = 2 ∫ (x − 1)2 − dx
0  4 
1/ 2 X' X
 (x − 1)3 1  −1 O x=1
=2  − x
 3 4 0
Y'
 1 1  1  8 1 Thus, the required area bounded by X-axis, the curve
= 2 − − −  − − 0  = = sq unit
 8 ⋅3 8  3   24 3 and x = 1
1
29. As from the figure, area enclosed between the curves is 1  x3 x2  1 1 5
= ∫ (x2 + x)dx =  +  = + = sq unit
OABCO. 0
 3 2 0 3 2 6
Thus, the point of intersection of 1 α
∫0 (x − x )dx = 2∫ (x − x3 )dx
3
y = ax2 and x = ay2 32.
0

y = ax2 1 α2 α4


Y = 2 − 
4  2 4

x 2α 4 − 4α 2 + 1 = 0
y 2= a
B 4 − 16 − 8
C
(1/a,1/a) ⇒ α2 = (Q α ∈ (0, 1))
4
1
A α =1 −
2
X' X 2
O D
33. PLAN (i) Area of region f( x ) bounded between x = a to x = b is
Y'

⇒ x = a (ax2)2
1 1 y = f (x)
⇒ x = 0, ⇒ y = 0,
a a a a 1 a 2 a3 D
 1 1
So, the points of intersection are (0, 0) and  ,  ⋅ b
 a a ∫a f(x )dx = Sum of areas of rectangle shown in shaded part.
Area 345

(ii) If f( x )≥ g ( x ) when defined in [a, b ], then OABCO and points of intersection are (0,0) and
b b
{1 − m, m(1 − m)}.
∫a f( x )dx ≥ ∫ g ( x )dx
a 1 −m
2 ∴ Area of curve OABCO = ∫ [x − x2 − mx] dx
Description of Situation As the given curve y = e − x 0
cannot be integrated, thus we have to bound this function by Y
using above mentioned concept.
2
Graph for y = e− x
Y
A
1
B
X

1 (0, 0) O {1 – m, m (1 – m)}
√e
X C
O 1 1 y= y = mx
— x – x2
√2
1 −m
Since, x2 ≤ x when x ∈ [0, 1]  x2 x3 
2 = (1 − m) − 
⇒ − x2 ≥ − x or e− x ≥ e− x  2 3 0
1 − x2 1
∴ ∫0 e dx ≥ ∫ e− x dx 1 1 1
= (1 − m)3 − (1 − m)3 = (1 − m)3
0
2 3 6
1
⇒ S ≥ − (e− x )10 = 1 − …(i) 1 9
e ∴ (1 − m) =
3
[given]
1 − x2
6 2
Also, ∫0 e dx ≤ Area of two rectangles ⇒ (1 − m)3 = 27
 1  1 1 ⇒ 1 −m =3
≤ 1 ×  + 1 − ×
 2  2 e ⇒ m = −2
1 1  1 Case III When m > 0
≤ + 1 −  …(ii)
2 e 2 In this case, y = mx and y = x − x2 intersect in (0,0) and
1 1  1 1 {(1 – m), m( 1 – m)} as shown in figure
∴ + 1 −  ≥S ≥1 − [from Eqs. (i) and (ii)]
2 e 2 e Y

34. Shaded area = e −  ∫ ex dx = 1


1
0
y = mx
e
Also, ∫1 ln (e + 1 − y) dy [put e + 1 − y = t ⇒ − dy = dt]
1– m
1 e e X
=∫ ln t (− dt ) = ∫ ln t dt = ∫ ln y dy = 1 A O (0,0)
e 1 1

35. Case I When m = 0


B
In this case, y = x − x2 …(i) y = x − x2
and y=0 …(ii)
are two given curves, y > 0 is total region above X-axis.
0
Therefore, area between y = x − x2 and y = 0 ∴ Area of shaded region = ∫ (x − x2 − mx) dx
1 −m
is area between y = x − x2 and above the X-axis 0
 x2 x3 
Y = (1 − m) − 
 2 3 1−m
1 1
A = − (1 − m) (1 − m)2 + (1 − m)3
2 3
1
O B
X = − (1 − m) 3
6
9 1
1 ⇒ = − (1 − m)3 [given]
1  x2 x3  1 1 1 9 2 6
∴ A = ∫ (x − x ) dx =  −  = − = ≠
2
0
2 3 0 2 3 6 2 ⇒ (1 − m)3 = − 27
Hence, no solution exists. ⇒ (1 − m) = − 3
Case II When m < 0 ⇒ m =3 + 1 =4
In this case, area between y = x − x2 and y = mx is Therefore, (b) and (d) are the answers.
346 Area

4a 2 4a 1  x3  1  (4x − 3)3/ 2 1 


f (−1) 3a 2 + 3a 
 2  = 2   −   
36. Given, 4b 4b 1  f (1)  = 3b2 + 3b   3  0  3 ⋅ 4 / 2  3/ 4 
 2 
 4c2 4c 1  f (2)  3c + 3c 
   1 1 1 1
= 2  −  = 1 ⋅ = sq unit
⇒ 4a 2 f (−1) + 4a f (1) + f (2) = 3a 2 + 3a , …(i)  3 6 6 3
4b f (−1) + 4b f (1) + f (2) = 3b + 3b
2 2
…(ii)
38. Here, slope of tangent,
and 4c2 f (−1) + 4cf (1) + f (2) = 3c2 + 3c …(iii) dy (x + 1)2 + y − 3 dy ( y − 3)
where, f (x) is quadratic expression given by, = ⇒ = (x + 1) + ,
dx (x + 1) dx (x + 1)
f (x) = ax2 + bx + c and Eqs. (i), (ii) and (iii). Put x + 1 = X and y − 3 = Y
⇒ 4x2 f (−1) + 4x f (1) + f (2) = 3x2 + 3x dy dY
⇒ =
or {4 f (−1) − 3} x2 + {4 f (1) − 3} x + f (2) = 0 …(iv) dx dX
As above equation has 3 roots a, b and c. dY Y
∴ =X +
So, above equation is identity in x. dX X
dY 1
i.e. coefficients must be zero. ⇒ − Y =X
dX X
⇒ f (−1) = 3 / 4, f (1) = 3 / 4, f (2) = 0 …(v) 1
f (x) = ax2 + bx + c ∫ − dX 1
Q IF = e X = e− log X =
∴ a = − 1 / 4, b = 0 and c = 1, using Eq. (v) X
4 − x2 1 1 Y
X ∫
Thus, f (x) = shown as, ∴ Solution is, Y ⋅ = X ⋅ dX + c ⇒ =X +c
4 X X
Let A (−2, 0), B = (2t , − t 2 + 1) Y
Since, AB subtends right angle at vertex V (0, 1).
1 −t 2
⇒ ⋅ = −1 ⇒ t =4 y = x 2 _ 2x
2 2t
∴ B (8, − 15)
− (3x + 6)
So, equation of chord AB is y = .
2
X′ X
8  4 − x2 3x + 6 O 2
∴ Required area = ∫  +  dx
−2  4 2 
Y′
8
 x3 3x2 
= x − + + 3x y − 3 = (x + 1)2 + c(x + 1), which passes through (2, 0).
 12 4  −2
⇒ − 3 = (3)2 + 3c ⇒ c = − 4
 128  2  ∴ Required curve
= 8 − + 48 + 24 −  −2 + + 3 − 6  y = (x + 1)2 − 4(x + 1) + 3 ⇒ y = x2 − 2x
 3  3 
  x3  
2

∴ Required area =  ∫ (x2 − 2x)dx=  


2
=
125 − x2 
sq units  0  3  0
3 
37. The region bounded by the curves y = x2, y = − x2 and 8 4
= − 4 = sq units
y2 = 4 x − 3 is symmetrical about X-axis, where y = 4x − 3 3 3
meets at (1, 1). 39. The points in the graph are
∴ Area of curve (OABCO )
A (1, 1), B ( 2, 0), C (2, 2), D ( 2, 2)
= 2 ∫ x2 dx − ∫ ( 4x − 3 ) dx
1 1

 0 3/ 4  Y

y=x 2
Y y = |2 – x 2 |
y = x2
D (√2,2)
C (2,2)
(1,1) A
y=2
X' B (3/4, 0)
X A
O
C X′ X
y 2 = 4x – 3 x = 1 B (√2,0)

y = –x 2 Y′ Y′
Area 347

∴ Required area a
− jπ  − aπ 
be b  e b + 1
=∫
2 2
 
1
{ x2 − (2 − x2)} dx + ∫ 2
{2 − (x2 − 2)} dx   a
− jπ
2 2 Sj a 2 + b2 e b
=∫ ∴ = = a
1
(2x2 − 2) dx + ∫ 2
(4 − x2) dx
Sj −1 a
− ( j − 1 )π  − aπ  − ( j − 1 )π
be b  e b + 1 e b
2x3 
2
 3 2  
x  
= − 2x + 4x − 
 3 1  3 2
a 2 + b2
a
4 2 2   8 2 2 − π
= − 2 2 − + 2 + 8 − − 4 2 + =e b = constant
 3 3   3 3 
⇒ S 0 , S1 , S 2, K , S j form a GP.
 20 − 12 2  For a = − 1 and b = π
=  sq units
 3  1
. πj
π ⋅ eπ  1 ⋅π 
 eπ + 1 = π ⋅ e (1 + e)
j

40. Given, x = (sin by) e − ay Sj =


(1 + π 2)   (1 + π 2)
 
Now, −1 ≤ sin by ≤ 1
n n
⇒ − e−ay ≤ e− ay sin by ≤ e− ay ⇒ − e− ay ≤ x ≤ e− ay π ⋅ (1 + e) π (1 + e)
Y
⇒ ∑ Sj = (1 + π )2
∑ e j = (1 + π 2) (e0 + e1 + ...+ en )
j=0 j=0

π (1 + e) (en + 1 − 1)
x = e – ay x = e – ay = ⋅
(1 + π 2) e−1
S3
|x| ≤ 1
S2 41. Given, f (x) =  2x2 ,
 x + ax + b,|x| > 1
S1
Y
S0
X′ X 1
x = – 2y2 x=–
O 8
y = f (x)

X′ –2 –1
Y′ O
X
− ay
In this case, if we take a and b positive, the values − e
and e− ay become left bond and right bond of the curve
and due to oscillating nature of sin by, it will oscillate
between x = e− ay and x = − e− ay
( j + 1 ) π /b Y′
Now, Sj = ∫ sin by ⋅ e− ay dy x2 + ax + b, if x < − 1
jπ /b

since, I = sin by ⋅ e− ay dy  ⇒ f (x) = 2x, if − 1 ≤ x < 1
 − ay
∫  x2 + ax + b, if x ≥ 1
I = − e  
(a sin by + b cos by)
 a 2 + b2  f is continuous on R, so f is continuous at –1 and 1.
 −1  − a ( j + 1)π lim f (x) = lim f (x) = f (−1)
∴ S j = 2 2
e b x → −1 − x → −1 +
a + b  {a sin ( j + 1)π + b cos ( j + 1) π} and lim f (x) = lim f (x) = f (1)
x → 1− x →1+
− ajπ  ⇒ 1 − a + b = − 2 and 2 = 1 + a + b
− e b (a sin jπ + b cos jπ ) 
 ⇒ a − b = 3 and a + b = 1
a
− ( j + 1 )π
Sj = −
1
[e b {0 + b(−1) j + 1 } ∴ a =2, b = −1
a +b
2 2
 x2 + 2 x − 1 , if x < −1
− e− ajπ / b{0 + b(− 1) j }] 
Hence, f (x) =  2x, if −1 ≤ x < 1
 −
a
jπ   x2 + 2x − 1, if x≥1
b (− 1) j e b  −aπ 
=   e b + 1 
 a 2 + b2   Next, we have to find the points x = − 2 y2 and y = f (x).
 
  The point of intersection is (–2, –1).
−1/ 8  − x 
[Q (−1) j + 2 = (−1)2 (−1) j = (−1) j ] ∴ Required area = ∫ − f (x) dx
a −2  2
− jπ
 − aπ   
be b  e b + 1
= −1/ 8 −x −1 −1/ 8
2 
a +b 
2  =∫ dx − ∫ (x2 + 2x − 1)dx − ∫ 2x dx
 −2 2 −2 −1
348 Area
−1
2  x3  Now, to get the point of intersection of y = x2 and
=− [(− x)3/ 2]−−12/ 8
−  + x2 − x  − [x2] −−11/ 8 y = 2x (1 − x), we get
3 2   3   −2
x2 = 2x (1 − x)
2   1   1
3/ 2
 ⇒ 3 x2 = 2 x
=−   − 2  −  − + 1 + 1
3/ 2
3 2 8 3 ⇒ x (3x − 2) = 0

⇒ x = 0, 2 / 3
 8  1 
+  − + 4 + 2 − −1
 3  64  Similarly, we can find the coordinate of the points of
2 5 63 intersection of
= [2 2 − 2−9/ 2] + + y = (1 − x2) and y = 2x (1 − x) are x = 1 / 3 and x = 1
3 3 64
63 509 761 From the figure, it is clear that,
= + = sq units
16 × 3 64 × 3 192  (1 − x)2, if 0 ≤ x ≤ 1 / 3

f (x) = 2x (1 − x), if 1 / 3 ≤ x ≤ 2 / 3
42. Refer to the figure given in the question. Let the
 2 if 2 /3 ≤ x ≤ 1
coordinates of P be (x, x2), where 0 ≤ x ≤ 1. x ,
For the area (OPRO ), ∴ The required area
Upper boundary: y = x2 and A=∫
1
f (x) dx
lower boundary : y = f (x) 0
1/3 2 /3 1
Lower limit of x : 0 =∫ (1 − x)2 dx + ∫ 1/ 3 2x (1 − x) dx + ∫ 2/ 3 x 2
dx
0
Upper limit of x : x
x x 1/ 3
 3 2 / 3 1
 1  1 3 
∴ Area (OPRO ) = ∫0 t 2 dt − ∫ f (t ) dt = − (1 − x)3
 3 
+  x2 −
2x
 + x
3 1/ 3 3  2 / 3
0
x 0 
 t3  x
=   − ∫ f (t ) dt  1  2 3 1    2 2 2  2
3
 1 2
2
 1 
3

 3 0 0 = −   +  +   −   −  +   
   3  3  3
x3 x  3 3 3  3 3 3 
= − ∫ f (t ) dt 1  2 
3
3 0 1
+  (1) −   
 3
For the area (OPQO ), 3 3 
The upper curve : x = y 19 13 19 17
= + + = sq unit
and the lower curve : x = y /2 81 81 81 27
Lower limit of y : 0 x2
and upper limit of y : x2 44. Eliminating y from y = and y = x − bx2, we get
b
x2 x2 t
∴ Area (OPQO ) = ∫ t dt − ∫ dt x2 = bx − b2x2
0 0 2 b
⇒ x = 0,
2 2 1 2 2 x4 1 + b2
= [t3/ 2]x0 − [t 2]x0 = x3 −
3 4 3 4 Y
According to the given condition, x2
y=
b
x3 x 2 x4
− ∫ f (t ) dt = x3 −
3 0 3 4
On differentiating both sides w.r.t. x, we get y = x – bx 2
x2 − f (x) ⋅ 1 = 2x2 − x3
X′ X
⇒ f (x) = x3 − x2, 0 ≤ x ≤ 1 –1 O b
2
1+ b
43. We can draw the graph of y = x2, y = (1 − x2) and
y = 2x(1 − x) in following figure
Y
(1, 1) Y′
(0, 1)
y = (1 – x) 2
Thus, the area enclosed between the parabolas
b/(1 + b )2  x2 
2
Q y=x
(1/2, 1/2) A=∫  x − bx2 −  dx
A B 0  b
b/(1 + b )2
y = 2x (1 – x)  x2 x3 1 + b2  1 b2
= − ⋅ = ⋅
X′ X 2 3 b  0 6 (1 + b2)2
O 1/3 2/3 1
Y′
Area 349

On differentiating w.r.t. b, we get 1


⇒ 2 An <
dA 1 (1 + b2)2 ⋅ 2b − 2b2 ⋅ (1 + b2) ⋅ 2b n −1
= ⋅
db 6 (1 + b2)4 ⇒ An <
1
…(ii)
1 b (1 − b2) 2n − 2
= ⋅ 1 1
3 (1 + b2)3 From Eqs. (i) and (ii), < An <
dA 2n + 2 2n − 2
For maximum value of A, put =0
db 46. The equations of the sides of the square are as follow :
⇒ b = − 1, 0, 1, since b > 0 AB : y = 1, BC : x = − 1, CD : y = − 1, DA : x = 1
∴ We consider only b = 1. Y
dA
Sign scheme for around b = 1 is as shown below : B(–1,1) A(1,1)
db
– + –
0 1 (0,1/2)

From sign scheme, it is clear that A is maximum at


b = 1. X′ (–1/2,0) (1/2,0)
X
O
π /4
45. We have, An = ∫ (tan x)n dx
0 (0,–1/2)

Since, 0 < tan x < 1, when 0 < x < π /4


We have, 0 < (tan x)n+ 1 < (tan x)n for each n ∈N C(–1,–1) D(1,–1)
π /4 π /4
⇒ ∫0 (tan x)n+ 1 dx < ∫ (tan x)n dx Y′
0

⇒ An+ 1 < An Let the region be S and (x, y) is any point inside it.
Now, for n > 2 Then, according to given conditions,
π /4 x2 + y2 < |1 − x|,|1 + x|,|1 − y|,|1 + y|
An + An + 2 = ∫ [( tan x)n + (tan x)n + 2] dx
0
⇒ x2 + y2 < (1 − x)2, (1 + x)2, (1 − y)2, (1 + y)2
π /4
=∫ (tan x) (1 + tan x) dx
n 2
⇒ x2 + y 2 < x 2 − 2 x + 1 , x 2 + 2 x + 1 ,
0
y2 − 2 y + 1 , y 2 + 2 y + 1
Y ⇒ y < 1 − 2x, y < 1 + 2x, x < 1 − 2 y and x2 < 2 y + 1
2 2 2
n
y = (tan x)
Now, in y2 = 1 − 2x and y2 = 1 + 2x, the first equation
represents a parabola with vertex at ( 1/2,0) and second
equation represents a parabola with vertex ( –1/2, 0)
and in x2 = 1 − 2 y and x2 = 1 + 2 y, the first equation
represents a parabola with vertex at (0, 1/2) and second
equation represents a parabola with vertex at (0, –1 /2) .
Therefore, the region S is lying inside the four parabolas
B
X y2 = 1 − 2x, y2 = 1 + 2x, x2 = 1 + 2 y, x2 = 1 − 2 y
O
Y
A(1,1)
(0,1)
π /4
y
2

=∫ n
(tan x) sec x dx 2
=

0
2x

π /4
–1

 1  G
= (tan x)n + 1  E x2
 (n + 1)
I
0 =
2y
1 1 –1
= (1 − 0) =
(n + 1) n+1 X
O H F (1,0)
Since, An + 2 < An + 1 < An,
then An + An + 2 < 2 An where, S is the shaded region.
1 Now, S is symmetrical in all four quadrants, therefore
⇒ < 2 An
n+1 S = 4 × Area lying in the first quadrant.
1 Now, y2 = 1 − 2x and x2 = 1 − 2 y intersect on the line
⇒ < An …(i)
2n + 2 y = x. The point of intersection is E ( 2 − 1, 2 − 1).
1 Area of the region OEFO
Also, for n > 2 An + An < An + An − 2 =
n −1 = Area of ∆ OEH + Area of HEFH
350 Area

=
1
( 2 − 1 )2 + ∫
1/ 2
1 − 2x dx Also, y = x2 − x meets X-axis at (0,0) and (1, 0).
2 2 −1 5/ 2
1/ 2
∴ Area, A1 = ∫ [(4x − x2) − (x2 − x)] dx
1  2 1  0
= ( 2 − 1)2 + (1 − 2x)3/ 2 ⋅ (− 1)
2  3 2  =∫
5/ 2
(5x − 2x2) dx
2−1
0
1 1 5/ 2 2 3
= (2 + 1 − 2 2 ) + (1 + 2 − 2 2 ) 3/ 2
5 2 2 3  5  5 2  5
2 3 = x − x =   − . 
2 3  0 2  2 3  2
1 1
= (3 − 2 2 ) + (3 − 2 2 )3/ 2
2 3 5 25 2 125
= ⋅ − ⋅
1 1 2 4 3 8
= (3 − 2 2) + [( 2 − 1)2]3/ 2
2 3 125  2 125
1 1 = 1 −  = sq units
= (3 − 2 2 ) + ( 2 − 1)3 8  3 24
2 3
1 1 This area is considering above and below X-axis both.
= (3 − 2 ) + [2 2 − 1 − 3 2 ( 2 − 1)] Now, for area below X-axis separately, we consider
2 3 1
1 1 1  x2 x3  1 1 1
= (3 − 2 2 ) + [5 2 − 7] A2 = − ∫ (x2 − x) dx =  −  = − = sq units
2 3 0
2 3 0 2 3 6
1 1
= [9 − 6 2 + 10 2 − 14] = [4 2 − 5] sq units Therefore, net area above the X-axis is
6 6 125 − 4 121
1 A1 − A2 = = sq units
Similarly, area OEGO = (4 2 − 5) sq units 24 24
6
Hence, ratio of area above the X-axis and area below
Therefore, area of S lying in first quadrant X-axis
2 1 121 1
= (4 2 − 5) = (4 2 − 5) sq units = : = 121 : 4
6 3 24 6
4 1
Hence, S = (4 2 − 5) = (16 2 − 20) sq units 48. The curve y = x2 is a parabola. It is symmetric about
3 3
Y-axis and has its vertex at (0, 0) and the curve
47. Given parabolas are y = 4x − x2 2
y= is a bell shaped curve. X-axis is its asymptote
and y = − (x − 2)2 + 4 or (x − 2)2 = − ( y − 4) 1 + x2
Therefore, it is a vertically downward parabola with and it is symmetric about Y-axis and its vertex is (0, 2).
vertex at (2,4) and its axis is x = 2 Y
2
 1 1
and y = x2 − x ⇒ y =  x −  −
 2  4 y = x2
A 2

(–1, 1) C B (1,1)
y= 2 2
1+ x
X′ X
O M

Since, y = x2 …(i)
2
and y= …(ii)
1 + x2
2
⇒ y= ⇒ y2 + y − 2 = 0
1+ y
2 ⇒ ( y − 1) ( y + 2) = 0 ⇒ y = − 2, 1
 1 1
⇒ x −  = y + But y ≥ 0, so y = 1 ⇒ x = ± 1
 2 4
Therefore, coordinates of C are (–1, 1) and coordinates
1 1
This is a parabola having its vertex at  , −  ⋅ of B are (1,1).
2 4
∴ Required area OBACO = 2 × Area of curve OBAO
1
Its axis is at x = and opening upwards.  1 2 1 
2 = 2 ∫ dx − ∫ x2 dx
0 1 + x2
The points of intersection of given curves are  0

4 x − x 2 = x2 − x ⇒ 2 x 2 = 5 x  x 
1
2π 1  
3
2
5 = 2 [2 tan −1 x]10 −    = 2 − =  π −  sq unit
⇒ x (2 − 5x) = 0 ⇒ x = 0,  
4 
3  3
2  3  0 
Area 351

49. The required area is the shaded portion in following ∴ The required area
figure 1  (log x) 
=∫  − ex log x dx
1/ e  ex 
Y y = 2x
1 1
1  (log x)2   x2   e2 − 5
= − e (2 log x − 1 ) =   sq units
e  2 1/ e 4


1/ e  4e 
dy
y = loge x 51. Given, y = x (x − 1)2 ⇒ = x ⋅ 2 (x − 1) + (x − 1)2
dx
X′ X Y y = x(x – 1)2
O 1/2 1 2

4 max
27

Y′ X′ X
O 1/3 1 min

∴ The required area


2
2  2x  Y′
=∫ (2x − log x) dx =  − (x log x − x)
1/ 2  log 2  1/ 2 = (x − 1) ⋅ (2x + x − 1)
4 − 2 5 3 = (x − 1) (3x − 1)
= − log 2 +  sq units +• − • +
 log 2 2 2
1 /3 1
50. Both the curves are defined for x > 0. ∴ Maximum at x = 1 / 3
Both are positive when x > 1 and negative when 0 < x < 1. 1  2
2
4
ymax =  −  =
We know that, lim (log x) → −∞ 3  3  27
x→ 0 +
log x Minimum at x = 1
Hence, lim → −∞. Thus, Y-axis is asymptote of
x→ 0 + ex ymin = 0
second curve. Now, to find the area bounded by the curve y = x (x − 1)2,
And lim ex log x [(0) × ∞ form] the Y-axis and line x = 2 .
x→ 0 +
e log x  ∞ 
= lim − form Y
x→ 0 + 1 / x  ∞ 
C B
 1 2
e 
 x
= lim =0 [using L’Hospital’s rule] 4
+  1
x→ 0
 − 2 27
 x 
X′ X
Thus, the first curve starts from (0, 0) but does not O 1 A
x=2
include (0, 0).
Now, the given curves intersect, therefore Y′
log x 2
ex log x = ∴ Required area = Area of square OABC − ∫ y dx
ex 0
2
i.e. (e2x2 − 1) log x = 0 = 2 × 2 − ∫ x (x − 1)2 dx
1 0
⇒ x = 1, [Qx > 0]  x (x − 1)3  2 1 2 
e
Y = 4 −   − ∫ (x − 1)3 ⋅ 1 dx
 3 0 3 0 
log x 2
y=
ex x (x − 1)4 
= 4 −  (x − 1)3 −
3 12  0
y = ex log x
X′ 2 1 1  10
X =4− − + = sq units
O 1/e 1 3 12 12  3
dy
52. Given, y = tan x ⇒ = sec2x
dx
 dy
∴   =2
Y′  dx x = π
4
352 Area

π   25  4  1  8 
Hence, equation of tangent at A  , 1 is
4  = 2 6 + sin −1    − 8 −
 2  5  4  3 
y−1
=2
x − π /4 1  64   8  
−  − 16 −  − 8 
π 4  3   3  
⇒ y − 1 = 2x −
2  25  4 4 4 4 
= 2 6 + sin −1   − − − 
Y y = tan x
 2  5 3 3 3 

  4 
= 4 + 25 sin −1    sq units
A   5 
O 1
X′ X
B L
−1
54. Given curves are x2 + y2 = 4, x2 = − 2 y and x = y.
2
Y
Y′ 2 y=x
π 
⇒ (2x − y) =  − 1
2 
x + y2 = 4
∴ Required area is OABO
π /4 −√ 2 O √2
=∫ ( tan x) dx − area of ∆ ALB X′ X
−2 2
0
1
= [log|sec x|]π0 / 4 −
⋅ BL ⋅ AL
2
1  π π − 2
= log 2 −  −  ⋅1 −2
2 4 4 
 1 Y′ x 2 − √ 2y
=  log 2 −  sq unit
 4
Thus, the required area
53. Given curves, x2 + y2 = 25, 4 y = |4 − x2| could be 2 0 2 − x2
sketched as below, whose points of intersection are
= ∫− 2
4 − x2 dx − ∫− 2
x dx − ∫0 2
dx

(4 − x2)2
x2 + = 25 2   x2 0   x3  2
16 =2 ∫ 4 − x2 dx −     − 
Y
0
  2− 2
  3 2 0
4y = 4 – x 2 2
5 x 4 x 2
4y = x 2 – 4 4y =x 2 – 4 =2  4 − x2 − sin −1  − 1 −
2 2 2 0 3
5 1 
= (2 − π ) − =  − π  sq units
3 3 
O
X′ X
–5 –4 –2 2 4 55. Given curves y = 5 − x2 and y = |x − 1|could be sketched
x 2 + y 2 = 25 as shown, whose point of intersection are
5 − x2 = (x − 1)2
Y
–5
Y′
y = –x + 1 y=x–1
⇒ (x2 + 24) (x2 − 16) = 0
⇒ x=±4
 4 2  4 − x2  X′ X
∴ Required area = 2 ∫ 25 − x2 dx − ∫   dx –1 1 2
0 0  4 

4  x2 − 4  Y′
−∫   dx
2  4   ⇒ 5 − x = x − 2 x + 1 ⇒ 2 x2 − 2 x − 4 = 0 ⇒ x = 2 , − 1
2 2

4 ∴ Required area
 x 25  x 
= 2  25 − x2 + sin −1    2 1 2
2 2  5  =∫ 5 − x2 dx − ∫ ( − x + 1) dx − ∫ (x − 1) dx
0 −1 −1 1

 
2 4
1  x3  1  x3 x 5  x   − x2 
2
 x2 
1 2
− 4 x − −
3  0 4  3
− 4 x  = 5 − x2 + sin −1    −  + x −  − x
4   2   2 2  5   −1  2  −1  2 1
Area 353

 5 2  5 −1  − 1   58. The point of intersection of the curves x2 = 4 y and


= 1 + sin −1  − −1 + sin   
 2 5   2  5  x = 4 y − 2 could be sketched are x = − 1 and x = 2.
 1 1   1  ∴ Required area
−  − + 1 + + 1 − 2 − 2 − + 1
 2 2   2  2  x + 2   x2 
= ∫   −    dx
5  −1 2 1  1 −1  4   4
= sin + sin −1  − 
2 5 5 2 2
1  x2 x3 
 2 4 1 = + 2 x −
4  2 3  −1
5 1 1
= sin −1  1− + 1−  −
2  5 5 5 5 2
5 1  5π 1 1  8  1 1 
= sin −1 (1) − =  −  sq units = 2 + 4 − 3 −  2 − 2 + 3 
2 2  4 2 4  
 π π 1 10  −7  1 9 9
tan x, − ≤ x ≤ = −   = ⋅ = sq units

56. Given, y =  3 3 4  3  6   4 2 8
π π
 cot x, ≤x≤  et1 + e– t1 et1 – e– t1 
 6 2 59. Let P =  , 
 2 2 
which could be plotted as Y-axis.
Y  e– t + et1 e– t1 – et 
and Q =  , 
 2 2 
y = cot x y = tan x
We have to find the area of the region bounded by the
curve x2 – y2 = 1 and the lines joining the centre x = 0,
y = 0 to the points (t1 ) and (– t1 ).
X′ X Y
O
π/3
P (t1)

A
Y′ X′ X
−1 C 1 N
π /4 π /3
∴ Required area = ∫ ( tan x) dx + ∫ π / 4 ( cot x) dx
0
Q (−t1)
= [− log|cos x|]π0 / 4 + [log sin x] ππ //34
 1   3 1
= −  log − 0 +  log − log  Y′
 2   2 2
Required area
3 1
= log − 2 log  e t1 + e – t1 
2 2 = 2 area of ∆PCN – ∫ 2 ydx
3 1 1   1 
= log − log =  log e 3 sq units  
2 2 2 
1  et1 + e– t1   et1 – e– t1  t1 dy 
a 8 4  8 =2     –∫ y ⋅ dt 
57. Here, ∫2 1 + 2 dx = ∫
 x  a
1 + 2 dx
 x  2  2  2  1 dt 
 8 
a
8
4  e2t1 – e–2t1 t1  et – e– t  


x− = x− =2 –∫  dt 
x  2  x  a  8  2  
0
 8  8
⇒  a −  − (2 − 4) = (4 − 2) −  a −  e2t1 – e–2t1 1 t1 2t
 a  a = – ∫ (e + e–2t – 2)dt
8 8 4 2 0
⇒ a − + 2 =2 − a +
a a e2t1 – e–2t1 1  e2t e–2t 
16 = –  – – 2t 
⇒ 2a − = 0 ⇒ 2 (a 2 − 8) = 0 4 22 2
a 
⇒ a = ±2 2 [neglecting –ve sign] e2t1 – e–2t1 1 2t1
∴ a =2 2 = – (e – e–2t1 – 4t1 )
4 4
14
Differential Equations
Topic 1 Solution of Differential Equations by Variable
Separation Method
Objective Questions I (Only one correct option) dy  π
5. If (2 + sin x) + ( y + 1) cos x = 0 and y(0) = 1, then y 
dx  2
1. The solution curve of the differential equation,
dy is equal to (2017 Main)
(1 + e− x )(1 + y2) = y2, which passes through the point 1 2 1 4
dx (a) (b) − (c) − (d)
(0, 1) is (2020 Main, 3 Sep I) 3 3 3 3
 1+ e −x   6. If y = y(x) satisfies the differential equation
(a) y2 + 1 = y  log e   + 2 −1
  2    
8 x( 9+ x ) dy =  4 + 9 + x dx, x>0 and
 
  1 + ex  
(b) y2 + 1 = y  log e   + 2 y(0) = 7, then y(256) = (2017 Adv.)
  2   (a) 16 (b) 3 (c) 9 (d) 80
 1 + ex  13
1
(c) y2 = 1 + y log e 
 2 
 7. The value of ∑  π (k − 1 )π  π kπ 
is equal
k =1 sin  +  sin  + 
 1 + e− x  4 6  4 6
(d) y2 = 1 + y log e  
 2  to (2016 Adv.)
(a) 3 − 3 (b) 2(3 − 3 )
2. Let f be a differentiable function such that f (1) = 2 and
(c) 2( 3 − 1) (d) 2(2 + 3 )
f ′ (x) = f (x) for all x ∈ R. If h (x) = f ( f (x)), then h′ (1) is
equal to (2019 Main, 12 Jan II) dy 1 − y2
2 2 8. The differential equation = determines a
(a) 4e (b) 4e (c) 2e (d) 2e dx y
dy family of circles with (2007, 3M)
3. The solution of the differential equation, = (x − y)2,
dx (a) variable radii and a fixed centre at (0, 1)
when y(1) = 1, is (2019 Main, 11 Jan II) (b) variable radii and a fixed centre at (0, – 1)
2− y (c) fixed radius 1 and variable centres along the X-axis
(a) log e = 2( y − 1) (d) fixed radius 1 and variable centres along the Y-axis
2− x
1+ x − y 2 + sin x  dy
(b) − log e =x+ y−2 9. If y = y (x) and   = − cos x , y (0) = 1, then
1− x + y y+1  dx
 π
y   equals (2004, 1M)
2− x  2
(c) log e =x− y
2− y (a) 1/3 (b) 2/3 (c) − 1 / 3 (d) 1
1− x + y 10. A solution of the differential equation
(d) − log e = 2(x − 1)
1+ x − y  dy
2
dy
  −x + y = 0 is (1999, 2M)
4. Let f : [0, 1] → R be such that f (xy) = f (x). f ( y), for all  dx dx
x, y ∈ [0, 1] and f (0) ≠ 0. If y = y (x) satisfies the (a) y = 2 (b) y = 2x (c)y = 2x − 4 (d) y = 2x2 − 4
dy
differential equation, = f (x) with y(0) = 1, then 11. The order of the differential equation whose general
dx
 1  3 solution is given by y = (c1 + c2) cos (x + c3 ) − c4 ex + c5 ,
y   + y   is equal to (2019 Main, 9 Jan II) where c1 , c2, c3 , c4 , c5 are arbitrary constants, is
 4  4
(1998, 2M)
(a) 5 (b) 3 (c) 2 (d) 4 (a) 5 (b) 4 (c) 3 (d) 2
Differential Equations 355

Objective Questions II Integer & Numerical Answer Type Questions


(One or more than one correct option) 17. Let f : R → R be a differentiable function with f (0) = 0.
12. Let b be a nonzero real number. Suppose f : R → R is a If y = f (x) satisfies the differential equation
dy
differentiable function such that f (0) = 1. If the = (2 + 5 y) (5 y − 2), then the value of lim f (x) is ...... .
dx x→− ∞
derivative f′ of f satisfies the equation (2018 Adv.)
f (x)
f ′ (x) = 2 18. Let f : R → R be a continuous function, which satisfies
b + x2 x
f (x) = ∫ f (t ) dt . Then, the value of f (ln 5) is … .
for all x ∈R , then which of the following statements 0 (2009)
is/are TRUE? (2020 Adv.)
(a) If b > 0, then f is an increasing function
Assertion and Reason
(b) If b < 0, then f is a decreasing function For the following question, choose the correct answer
(c) f (x)f (− x) =1for all x∈ R from the codes (a), (b), (c) and (d) defined as follows.
(d) f (x) − f (− x) = 0 for all x ∈ R (a) Statement I is true, Statement II is also true;
Statement II is the correct explanation of Statement I.
13. Let f : [0, ∞ ) → R be a continuous function such that
x x− t (b) Statement I is true, Statement II is also true;
f (x) = 1 − 2x + ∫0 e f (t ) dt for all x ∈ [0, ∞ ). Then, Statement II is not the correct explanation of
Statement I.
which of the following statement(s) is (are) TRUE?
(2018 Adv.) (c) Statement I is true; Statement II is false.
(a) The curve y = f (x) passes through the point (1, 2) (d) Statement I is false; Statement II is true.
(b) The curve y = f (x) passes through the point (2, − 1) 19. Let a solution y = y(x) of the differential equation
(c) The area of the region 2
π−2 x x2 − 1 dy − y y2 − 1 dx = 0 satisfy y(2) =
{(x, y) ∈ [0, 1] × R : f (x) ≤ y ≤ 1 − x2 } is 3
4
 π
(d) The area of the region
π −1
Statement I y(x) = sec sec−1 x −  and
{(x, y) ∈ [0, 1] × R : f (x) ≤ y ≤ 1 − x2 } is  6
4 1 2 3 1
Statement II y(x) is given by = − 1− 2
14. Let y(x) be a solution of the differential equation y x x
(1 + ex ) y′ + yex = 1. If y(0) = 2, then which of the (2008, 3M)
following statement(s) is/are true? (2015 Adv.)
(a) y (−4) = 0 Analytical & Descriptive Questions
(b) y (−2) = 0
dP (x)
(c) y(x) has a critical point in the interval (−1 , 0) 20. If P(1) = 0 and > P (x), ∀ x ≥ 1 , then prove that
dx
(d) y(x) has no critical point in the interval (−1 , 0) P (x) > 0, ∀ x > 1. (2003, 4M)
15. Consider the family of all circles whose centres lie on the 21. Let y = f (x) be a curve passing through (1, 1) such that
straight line y = x.If this family of circles is represented by
the differential equation Py′ ′+ Qy′ + 1 = 0, where P , Q are the triangle formed by the coordinate axes and the
dy d 2y tangent at any point of the curve lies in the first
the functions of x, y and y′ (here, y′ = , y′ ′ = ), then quadrant and has area 2 unit. Form the differential
dx dx2
equation and determine all such possible curves.
which of the following statement(s) is/are true? (2015 Adv.)
(a) P = y + x (1995, 5M)
(b) P = y − x
(c) P + Q = 1 − x + y + y′ + ( y′ )2 Passage Based Problems
(d) P – Q = x + y – y′ – ( y′ )2 Passage
16. The differential equation representing the family of Let f : [0, 1] → R (the set of all real numbers) be a function.
curves y2 = 2c (x + c), where c is a positive parameter, Suppose the function f is twice differentiable,
is of (1999, 3M)
f ( 0) = f (1) = 0 and satisfies
(a) order 1 (b) order 2
(c) degree 3 (d) degree 4 f ′ ′ (x) − 2 f ′ (x) + f (x) ≥ ex , x ∈ [0, 1] (2013 Adv.)
356 Differential Equations

22. If the function e−x f (x) assumes its minimum in the 24. Which of the following is true?
interval [0, 1] at x = 1 / 4, then which of the following is (a) g is increasing on (1, ∞ )
true? (b) g is decreasing on (1, ∞ )
1 3 1 (c) g is increasing on (1, 2) and decreasing on (2, ∞ )
(a) f ′ (x) < f (x), < x< (b) f ′ (x) > f (x), 0 < x <
4 4 4 (d) g is decreasing on (1, 2) and increasing on (2, ∞ )
1 3
(c) f ′ (x) < f (x), 0 < x < (d) f ′ (x) < f (x), < x<1 25. Consider the statements.
4 4
I. There exists some x ∈ R such that, f (x) + 2x = 2(1 + x2)
23. Which of the following is true?
1 1 II. There exists some x ∈ R such that,
(a) 0 < f (x) < ∞ (b) − < f (x ) <
2 2 2 f (x) + 1 = 2x (1 + x)
1
(c) − < f (x ) < 1 (d) − ∞ < f (x) < 0 (a) Both I and II are true (b) I is true and II is false
4 (c) I is false and II is true (d) Both I and II are false

Topic 2 Linear Differential Equation and


Exact Differential Equation
Objective Questions I (Only one correct option) 6. The solution of the differential equation
dy
1. The general solution of the differential equation x + 2 y = x (x ≠ 0) with y(1) = 1, is
2
(2019 Main, 9 April I)
dx
( y2 − x3 )dx − xydy = 0 (x ≠ 0) is (where, C is a constant of
x2 3 x3 1
integration) (2019 Main, 12 April II) (a) y = + (b) y = +
(a) y2 − 2x2 + Cx3 = 0 (b) y2 + 2x3 + Cx2 = 0 4 4x2 5 5x2
3 1 4 1
(c) y2 + 2x2 + Cx3 = 0 (d) y2 − 2x3 + Cx2 = 0 (c) y = x2 + (d) y = x3 +
4 4x2 5 5x2
1 
2. Consider the differential equation, y2dx +  x −  dy = 0. 7. Let y = y(x) be the solution of the differential equation,
  y
dy
If value of y is 1 when x = 1, then the value of x for which (x2 + 1)2 + 2x(x2 + 1) y = 1 such that y(0) = 0. If
y = 2, is (2019 Main, 12 April I)
dx
π
5 1 3 1 1 1 3 a y(1) = , then the value of ‘a’ is (2019 Main, 8 April I)
(a) + (b) − (c) + (d) − e 32
2 e 2 e 2 e 2
1 1 1
(a) (b) (c) 1 (d)
3. Let y = y(x) be the solution of the differential equation, 4 2 16
dy  π π 8. If a curve passes through the point (1, − 2) and has slope
+ y tan x = 2x + x2 tan x, x ∈− , , such that
dx  2 2
x2 − 2 y
y(0) = 1. Then (2019 Main, 10 April II) of the tangent at any point (x, y) on it as , then
x
π π
(a) y ′   − y ′  −  = π − 2 (b) the curve also passes through the point
 4  4 (2019 Main, 12 Jan II)
π π
y ′   + y ′  −  = − 2 (a) ( 3 , 0) (b) (− 1, 2) (c) (− 2 , 1) (d) (3, 0)
 4  4
9. Let y = y(x) be the solution of the differential equation,
π π π2 π π
(c) y   + y −  = + 2 (d) y   − y −  = 2 x
dy
+ y = x log e x, (x > 1). If 2 y(2) = log e 4 − 1, then y(e)
 4  4 2  4  4 dx
4. If y = y(x) is the solution of the differential equation is equal to (2019 Main, 12 Jan I)
dy  π π e e2 e e2
= (tan x − y) sec2 x, x ∈  − ,  , such that y (0) = 0, (a) − (b) − (c) (d)
dx  2 2 2 2 4 4
 π 10. If y(x) is the solution of the differential equation
then y  −  is equal to (2019 Main, 10 April I)
 4 dy  2x + 1 −2x
+  y = e , x > 0,
1
(a) − 2 (b)
1
−e (c) 2 +
1
(d) e − 2 dx  x 
e 2 e 1
where y (1) = e−2, then (2019 Main, 11 Jan I)
dy  x  π 2
5. If cos x − y sin x = 6x, 0 < x <  and y  = 0, then
 2  3 (a) y(x) is decreasing in  , 1
dx 1
2 
 π
y  is equal to (2019 Main, 9 April II) (b) y(x) is decreasing in (0, 1)
 6
(c) y(log e 2) = log e 4
π2 π2 π2 π2 log e 2
(a) (b) − (c) − (d) − (d) y(log e 2) =
2 3 2 3 4 3 2 4
Differential Equations 357

11. Let f be a differentiable function such that 19. Let f (x) be differentiable on the interval (0, ∞) such that
3 f (x)  1 t 2f (x) − x2f (t )
f ′ (x) = 7 − , (x > 0) and f (1) ≠ 4. Then, lim x f  f (1) = 1, and lim = 1 for each x > 0 . Then,
4 x x→ 0 +  x t→ x t−x
(2019 Main, 10 Jan II) f (x) is (2007, 3M)
4
(a) does not exist (b) exists and equals 1 2x2 1 4x2
7 (a) + (b) − +
(c) exists and equals 0 (d) exists and equals 4 3x 3 3x 3
1 2 1
dy 3 1  −π π   π 4 (c) − + (d)
12. If + y= ,x ∈  ,  and y  = , then x x2 x
2
dx cos x 2
cos x  3 3   4 3
 π  20. If x dy = y (dx + y dy), y (1) = 1 and y (x) > 0. Then, y (−3)
y −  equals
 4 (2019 Main, 10 Jan I) is equal to (2005, 1M)
1 4 1 1 (a) 3 (b) 2
(a) + e6 (b) − (c) + e3 (d)
3 3 3 3 (c) 1 (d) 0
13. If y = y(x) is the solution of the differential equation, dy
21. If y (t ) is a solution of (1 + t ) − ty = 1 and y (0) = − 1,
dy  1 dt
x + 2 y = x2 satisfying y(1) = 1, then y  is equal to then y (1) is equal to
dx  2 (2003, 1M)

(2019 Main, 9 Jan I) (a) −1 / 2 (b) e + 1 / 2


13 1 49 7 (c) e − 1 / 2 (d) 1 / 2
(a) (b) (c) (d)
16 4 16 64
14. Let y = y(x) be the solution of the differential equation
Objective Questions II
dy (One or more than one correct option)
sin x + y cos x = 4x, x ∈ (0, π ).
dx 22. Let f : (0, ∞ ) → R be a differentiable function such that
 π  π f (x)
If y  = 0, then y  is equal to f ′ (x) = 2 − for all x ∈ (0, ∞ ) and f (1) ≠ 1. Then
 2  6 (2018 Main) x (2016 Adv.)
4 2 −8 2 8 2 4 2
(a) lim f ′  = 1
(a) π (b) π (c) − π (d) − π 1
9 3 9 3 9 9 x → 0+  x 

(b) lim x f   = 2
15. If a curve y = f (x) passes through the point (1, − 1) and 1
x → 0+  x
satisfies the differential equation, y(1 + xy)dx = x dy,
 1 (c) lim x2f ′(x) = 0
then f  −  is equal to x → 0+
 2 (2016 Main) (d)|f (x)|≤ 2 for all x ∈ (0, 2)
2 4 2 4
(a) − (b) − (c) (d) 23. If y(x) satisfies the differential equation
5 5 5 5
y′ − y tan x = 2 x sec x and y(0), then (2012)
16. Let y(x) be the solution of the differential equation
π π2 π π2
dy (a) y   = (b) y′   =
(x log x) + y = 2x log x, (x ≥ 1). Then, y(e) is equal to  4 8 2  4  18
dx
π π2 π 4π 2π 2
(2015 Main) (c) y   = 
(d) y′   = +
 3 9  3 3 3 3
(a) e (b) 0 (c) 2 (d) 2e
17. The function y = f (x) is the solution of the differential Analytical & Descriptive Question
dy xy x4 + 2x
equation + 2 = in (−1, 1) satisfying 24. Let u (x) and v (x) satisfy the differential equations
dx x − 1 1 − x2 du dv
3 /2 + p (x) u = f (x) and + p (x) v = g (x), where
f (0) = 0. Then, ∫ 3
f (x) dx is (2014 Adv.) dx dx

2 p (x), f (x) and g (x) are continuous functions. If
π 3 π 3 π 3 π 3 u (x1 ) > v (x1 ) for some x1 and f (x) > g (x) for all x > x1,
(a) − (b) − (c) − (d) −
3 2 3 4 6 4 6 2 prove that any point (x, y) where x > x1 does not satisfy
18. Let f : [1 /2, 1] → R (the set of all real numbers) be a the equations y = u (x) and y = v (x). (1997, 5M)

positive, non-constant and differentiable function such


that f ′ (x) < 2 f (x) and f (1 / 2) = 1 . Then, the value of Integer & Numerical Answer Type Question
1
∫ f (x) dx lies in the interval 25. Let y′ (x) + y(x) g′ (x) = g (x) g′ (x), y(0) = 0, x ∈ R, where
1/ 2 (2013 Adv.) d f (x)
(a) (2e − 1, 2e) (b) (e − 1, 2e − 1) f ′ (x) denotes and g (x) is a given non-constant
dx
e−1 e − 1
(c)  , e − 1 (d)  0,  differentiable function on R with g (0) = g (2) = 0. Then,
 2   2  the value of y(2) is …… (2011)
358 Differential Equations

Topic 3 Applications of Homogeneous Differential Equations


Objective Questions I (Only one correct option) Objective Questions II
1. If a curve y = f (x), passing through the point (1, 2), is the (One or more than one correct option)
solution of thedifferential equation, 7. Let Γ denote a curve y = y(x) which is in the first
 1
2x dy = (2xy + y )dx, then f   is equal to
2 2
quadrant and let the point (1, 0) lie on it. Let the
 2 tangent to Γ at a point P intersect the y-axis at YP . If
(2020 Main, 2 Sep II) PYP has length 1 for each point P on Γ, then which of the
1 1 −1
(a) (b) (c) 1 + log e 2 (d) following options is/are correct? (2019 Adv.)
1 + log e 2 1 − log e 2 1 + log e 2
(a) xy′ + 1 − x2 = 0 (b) xy′ − 1 − x2 = 0
2. Given that the slope of the tangent to a curve y = y(x) at
 1 + 1 − x2 
2y
any point (x, y) is. If the curve passes through the (c) y = log e   − 1 − x2
x2  x 
 
centre of the circle x2 + y2 − 2x − 2 y = 0, then its  1 + 1 − x2 
equation is (2019 Main, 8 April II) (d) y = − log e   + 1 − x2
 x 
 
(a) x2 log e|y| = − 2(x − 1) (b) x log e|y|= x − 1
(c) x log e|y| = 2(x − 1) (d) x log e|y| = − 2(x − 1) 8. A solution curve of the differential equation
dy
3. The curve amongst the family of curves represented by (x + xy + 4x + 2 y + 4)
2
− y2 = 0, x > 0, passes through
dx
the differential equation, (x2 − y2)dx + 2xydy = 0, which the point (1, 3). Then, the solution curve (2016 Adv.)
passes through (1, 1), is (2019 Main, 10 Jan II) (a) intersects y = x + 2 exactly at one point
(a) a circle with centre on the Y-axis (b) intersects y = x + 2 exactly at two points
(b) a circle with centre on the X-axis (c) intersects y = (x + 2)2
(c) an ellipse with major axis along the Y-axis (d) does not intersect y = (x + 3)2
(d) a hyperbola with transverse axis along the X-axis. 9. Tangent is drawn at any point P of a curve which passes
through (1, 1) cutting X-axis and Y-axis at A and B,
4. Let the population of rabbits surviving at a time respectively. If BP : AP = 3 : 1, then (2006, 3M)
t be governed by the differential equation dy
dp(t ) 1 (a) differential equation of the curve is 3x + y =0
= p(t ) − 200. If p(0) = 100, then p(t ) is equal to dx
dt 2 dy
(2014 Main) (b) differential equation of the curve is 3x − y=0
t

t dx
(a) 400 − (b) 300 − 200 e (c) curve is passing through  , 2
300 e 2 2 1
t

t 8 
(c) 600 − 500 e 2 (d) 400 − 300 e 2
(d) normal at (1, 1) is x + 3 y = 4.
π
5. A curve passes through the point 1,  . Let the slope of
 6 Fill in the Blank
y  y
the curve at each point (x, y) be + sec   , x > 0. 10. A spherical rain drop evaporates at a rate proportional
x  x
to its surface area at any instant t. The differential
Then, the equation of the curve is (2013 Adv.) equation giving the rate of change of the rains of the
(a) sin   = log x + (b) cosec   = log x + 2
y 1 y rain drop is …. . (1997C, 2M)
 x 2  x
 2y 
(c) sec   = log x + 2 (d) cos   = log x +
2y 1
 x   x  2
Analytical & Descriptive Questions
6. At present, a firm is manufacturing 2000 items. It is 11. If length of tangent at any point on the curve y = f (x)
estimated that the rate of change of production P with intercepted between the point and the X-axis is of
respect to additional number of workers x is given by length 1. Find the equation of the curve. (2005, 4M)
dP
= 100 − 12 x. If the firm employees 25 more 12. A right circular cone with radius R and height H
dx contains a liquid which evaporates at a rate
workers, then the new level of production of items is proportional to its surface area in contact with air
(2013 Main) (proportionality constant = k > 0). Find the time after
(a) 2500 (b) 3000 (c) 3500 (d) 4500 which the cone is empty. (2003, 4M)
Differential Equations 359

13. A hemispherical tank of radius 2 m is initially full of 15. A curve passing through the point (1, 1) has the property
water and has an outlet of 12 cm2 cross-sectional area that the perpendicular distance of the origin from the
at the bottom. The outlet is opened at some instant. normal at any point P of the curve is equal to the distance
The flow through the outlet is according to the law of P from the X-axis. Determine the equation of the
v (t ) = 0.6 2 gh (t ), where v (t ) and h (t) are respectively curve. (1999, 10M)
the velocity of the flow through the outlet and the
16. A and B are two separate reservoirs of water. Capacity of
height of water level above the outlet at time t and g is
reservoir A is double the capacity of reservoir B. Both the
the acceleration due to gravity. Find the time it takes
reservoirs are filled completely with water, their inlets
to empty the tank. (2001, 10M)
are closed and then the water is released simultaneously
Hint Form a differential equation by relating the from both the reservoirs. The rate of flow of water out of
decreases of water level to the outflow. each reservoir at any instant of time is proportional to
14. A country has food deficit of 10%. Its population grows the quantity of water in the reservoir at the time.
continuously at a rate of 3% per year. Its annual food One hour after the water is released, the quantity of
production every year is 4% more than that of the last 1
water in reservoir A is 1 times the quantity of water in
year. Assuming that the average food requirement per 2
person remains constant, prove that the country reservoir B. After how many hours do both the reservoirs
will become self- sufficient in food after n years, where have the same quantity of water? (1997, 7M)
n is the smallest integer bigger than or equal to 17. Determine the equation of the curve passing through the
ln 10 − ln 9
. origin in the form y = f (x), which satisfies the
ln (1.04) − (0.03) (2000, 10M) dy
differential equation = sin (10x + 6 y)
dx (1996, 5M)

Match the Columns


18. Match the conditions/expressions in Column I with statements in Column II. (2006, 6M)

Column I Column II
π /2
A.
∫0 {cos x cot x − log (sin x ) p. 1
cos x sin x
(sin x ) }dx

B. Area bounded by − 4 y 2 = x and x − 1 = − 5 y 2 q. 0


C. The angle of intersection of curves y = 3 x − 1 log x and y = x x − 1 is r. 3e y / 2
dy 2 s. 4
D. If = passing through (1, 0), then ( x + y + 2 ) is
dx x + y 3

Answers
Topic 1 21. (a) 22. (a) 23. (a, d) 25. (0)
1. (c) 2. (b) 3. (d) 4. (b) Topic 3
5. (a) 6. (b) 7. (c) 8. (c) 1. (a) 2. (c) 3. (b) 4. (a)
9. (a) 10. (c) 11. (c) 12. (a,c) 5. (a) 6. (c) 7. (a,c)
13. (b, c) 14. (a, c) 15. (b, c)
8. (a, d) 9. (a, c)
16. (a, c) 17. (0.40) 18. (0) 19. (b)
 1 + 1 −y2 
 dr 
d 2y
21. Differential Equation: 2 = 0, x 2
dy
+1 = 0 10.  = − λ  11.  1 − y 2 − log = ± x + c
 dt   1 − 1 −y 2 
dx dx  
Curves : x + y = 2, xy = 1
 H  14 π × 10 5 
22. (c) 23. (d) 24. (b) 12. T =  13.  unit
 k  27 g 
25. (c)
 1 
Topic 2 15. ( x 2 + y 2 = 2 x ) 16.  log 3
 2 
1. (b) 2. (b) 3. (a) 4. (d) 4
5. (b) 6. (a) 7. (d) 8. (a) 1 4  3  3  5x
17. tan −1  tan  4 x + tan −1  − −
9. (c) 10. (a) 11. (d) 12. (a) 3 5  4 5  3
13. (c) 14. (c) 15. (d) 16. (c)
17. (b) 18. (d) 19. (a) 20. (a) 18. A → p; B → s; C → q; D → r
Hints & Solutions
Topic 1 Solution of Differential Equations ⇒ h′ (x) = f ′ ( f (x)) ⋅ f ′ (x)
by Variable Separation Method [on differentiating both sides w.r.t. ‘x’]
1. Given differential equation ⇒ h′ (1) = f ′ ( f (1)) ⋅ f ′ (1)
−x dy = f ′ (2) ⋅ f ′ (1) [Q f (1) = 2 (given)]
(1 + e ) (1 + y ) = y2
2
dx = 2e2−1 ⋅ 2e1−1 [Q f ′ (x) = 2ex −1 or −2ex −1]
1 + y2 ex = 4e
⇒ ∫ y2 dy = ∫ 1 + ex dx dy
3. We have, = (x − y)2 which is a differential equation of
 1  dx
⇒  − + y = log e (1 + e ) + C,
x
 y  the form
dy
which passes through (0, 1), = f (ax + by + c)
dx
so − 1 + 1 = log e 2 + C ⇒ C = − log e 2
Put x − y = t
So, equation of required curve is
dy dt dy dt
 1 + ex  ⇒ 1− = ⇒ =1 −
y2 = 1 + y log e   dx dx dx dx
 2  dt dy
⇒ 1− = t2 [ Q = (x − y)2]
Hence, option (c) is correct. dx dx
dt dt
=1−t ⇒ ∫
1 − t2 ∫
2. Given that, f ′ (x) = f (x) ⇒ 2
= dx
dx
f ′ (x)
⇒ =1 [separating the variables]
f (x)
f ′ (x) 1  1 + t
⇒ log e   =x+C
⇒ ∫ f (x)
dx = ∫ 1 ⋅ dx
2  1 − t
[by integrating both sides w.r.t. x]  dx 1 a+x 
∫ 2 = log e

+ C
⇒ Put f (x) = t ⇒ f ′ (x)dx = dt  a −x
2
2 a a x 
dt  1 + x − y
∴ ∫ t = ∫ 1 dx ⇒
1
log e   =x+C [Q t = x − y]
2  1 − x + y
 dx 
 ∫ x
⇒ ln|t|= x + C Q = ln|x|+ C Since, y = 1 when x = 1, therefore

⇒ ln| f (x)|= x + C 1  1 + 0
…(i) log e   =1 + C
[Q t = f (x)] 2  1 + 0
Q f (1) = 2 ⇒ C = −1 [Q log 1 = 0]
So, ln (2) = 1 + C [using Eq. (i)] 1  1 + x − y
∴ log e   = x −1
⇒ C = ln 2 − ln e [Q ln e = 1] 2  1 − x + y
 2  A 1−x+ y
⇒ C = ln   [Q ln A − ln B = ln   ] ⇒ − log e = 2(x − 1)
 e  B
1+ x− y
From Eq. (i), we get 1
[Q log = log x−1 = − log x]
 2 x
ln| f (x)|= x + ln  
 e
4. Given, f (xy) = f (x) ⋅ f ( y), ∀ x, y ∈ [0, 1] ...(i)
 2 Putting x = y = 0 in Eq. (i), we get
⇒ ln| f (x)|− ln   = x
 e
f (0) = f (0) ⋅ f (0)
ef (x) A
⇒ ln =x [Q ln A − ln B = ln ] ⇒ f (0) [ f (0) − 1] = 0
2 B
⇒ f (0) = 1 as f (0) ≠ 0
e
⇒ f (x) = ex [Q ln a = b ⇒ a = eb , a > 0] Now, put y = 0 in Eq. (i), we get
2
f (0) = f (x) ⋅ f (0)
 e e  ⇒ f (x) = 1
⇒ | f (x)|= 2ex −1 Q 2 f (x) = 2 | f (x)| 
  dy dy
So, = f (x) ⇒ =1
f (x) = 2ex −1 or −2ex −1 dx dx
Now, h (x) = f ( f (x)) ⇒ ∫ dy = ∫ dx
Differential Equations 361

⇒ y= x + C   π kπ  π π 
Q y(0) = 1 sin  4 + 6  cos  4 + (k − 1) 6  
 π π  π kπ  
∴ 1 =0 + C − sin  + (k − 1)  cos  + 
13
 4 6 4 6 
⇒ C =1 = 2∑
π π π π
∴ y=x+1 k =1 sin  + (k − 1)  sin  + k 
4 6 4 6
 1 1 5  3 3 7
Now, y   = + 1 = and y   = + 1 = 13
 4 4  4 4  π π π π 
4 4 = 2 ∑ cot  + (k − 1)  − cot  + k  
 1  3 5 7   4 6  4 6 
k =1
⇒ y  + y  = + =3
 4  4 4 4
  π  π π 
dy = 2 cot   − cot  +  
5. We have, (2 + sin x) + ( y + 1) cos x = 0   4   4 6 
dx
− cos x   π π  π 2π  

dy
+
cos x
y= + cot  +  − cot  + 
  4 6  4 6 
dx 2 + sin x 2 + sin x
 π π π π  
which is a linear differential equation. + K + cot  + 12  − cot  + 13  
  4 6   4 6  
cos x
∫ dx
2 + sin x
∴ IF = e = elog ( 2 + sin x ) = 2 + sin x  π π π 
= 2 cot − cot  + 13  
∴Required solution is given by  4 4 6 
− cos x
y ⋅ (2 + sin x) = ∫ ⋅ (2 + sin x)dx + C   29 π     5 π 
2 + sin x = 2 1 − cot    = 2 1 − cot 2π + 
  12    12  
⇒ y(2 + sin x) = − sin x + C  5π  5π 
= 2 1 − cot Q cot = (2 − 3 )
Also, y(0) = 1  12  
 12 
∴ 1(2 + sin 0) = − sin 0 + C = 2 (1 − 2 + 3 )
⇒ C =2 = 2 ( 3 − 1)
2 − sin x
∴ y= dy 1 − y2
2 + sin x 8. Given, =
dx y
π
2 − sin y

 π
y  = 2 =1 ⇒ ∫ 1 − y2
dy = ∫ dx
 2 π 3
2 + sin
2 ⇒ − 1 − y2 = x + c ⇒ (x + c)2 + y2 = 1
dy 1 Here, centre (– c, 0) and radius = 1
6. =
dx 8 x 9 + x 4+ 9+ x dy − cos x ( y + 1)
9. Given, =
dx 2 + sin x
⇒ y= 4 + 9+ x + c
dy − cos x
⇒ = dx
Now, y(0) = 7 + c y + 1 2 + sin x
⇒ c=0 On integrating both sides
y(256) = 4 + 9 + 16 = 4 + 5 = 3 dy cos x
13
∫ y + 1 = − ∫ 2 + sin x dx
1
7. Here, ∑  π (k − 1)π   π kπ  ⇒ log ( y + 1) = − log (2 + sin x) + log c
k =1 sin  +  sin  + 
4 6  4 6 When x = 0, y = 1 ⇒ c = 4
4
Converting into differences, by multiplying and ⇒ y+1=
 π k π   π (k − 1)π  2 + sin x
dividing by sin  +  − + , i.e.
 4 6  4 6   π 4
∴ y  = −1
 2 3
 π
sin   .
 6  π 1
⇒ y  =
 π π  π π   2 3
sin  + k  −  + (k − 1) 
13 
 4 6  4 6 
∴ ∑ π  π π π π 
10. Given differential equation is
2
k =1 sin sin  + (k − 1)  sin  + k    dy dy
6  4 6 4 6    −x + y=0 …(i)
 dx dx
362 Differential Equations

dy 1  x
(a) y = 2 ⇒ =0 ⇒ log e| f (x)| = tan − 1   + c
dx b  b
On putting in Eq. (i), Q f (0) = 1 , so c = 0
02 − x (0) + y = 0 1  x
tan − 1  
 b
⇒ y = 0 which is not satisfied. ∴ | f (x)| = e b

1  x 1  x
dy tan − 1   tan − 1  
(b) y = 2x ⇒ =2 ⇒ f (x) = eb  b
or − eb  b
dx
On putting in Eq. (i), Q f (0) = 1
1  x
(2) − x ⋅ 2 + y = 0
2 tan − 1  
 b
∴ f (x) = eb
⇒ 4 − 2x + y = 0
1  x
tan − 1  
⇒ y = 2x which is not satisfied. b  b
1 e
dy and f ′ (x) =
(c) y = 2x − 4 ⇒ =2 b2  x
2
dx 1+  
 b
On putting in Eq. (i)
1  x
tan − 1  
(2)2 − x − 2 + y eb b
=
4 − 2x + 2x − 4 = 0 [Q y = 2x − 4] b 2 + x2
y = 2x − 4 is satisfied. ∴ f ′ (x) > 0 ∀x ∈ R and b ∈ R0.
(d) y = 2 x2 − 4 Therefore, f (x) is an increasing function ∀ b ∈ R0.
dy 1  x 1  x
tan − 1   − tan − 1  
= 4x  b b
dx and f (x) f (− x) = eb ⋅e b
=1
On putting in Eq. (i), 13. We have,
(4x)2 − x ⋅ 4x + y = 0 x x−t
f (x) = 1 − 2x + ∫0 e f (t ) dt
⇒ y = 0 which is not satisfied.
On multiplying e− x both sides, we get
11. Given, y = (c1 + c2) cos (x + c3 ) − c4 ex + c5 …(i) x −t
e− x f (x) = e− x − 2xe− x + ∫0 e f (t ) dt
⇒ y = (c1 + c2) cos (x + c3 ) − c4 e ⋅ e x c5

Now, let c1 + c2 = A , c3 = B, c4ec5 = c On differentiating both side w.r.t. x, we get


e− x f ′ (x) − e− x f (x) = − e− x − 2e− x + 2xe− x + e− x f (x)
⇒ y = A cos (x + B) − cex …(ii)
⇒ f ′ (x) − 2 f (x) = 2x − 3 [dividing both sides by e− x ]
On differentiating w.r.t. x, we get
dy
dy Let f (x) = y ⇒ f ′ (x) =
= − A sin ( A + B) − cex …(iii) dx
dx
dy
Again, on differentiating w.r.t. x, we get ∴ − 2 y = 2x − 3
dx
d 2y
= − A cos (x + B) − cex …(iv) which is linear differential equation of the form
dx2 dy
+ Py = Q. Here, P = −2 and Q = 2x − 3.
d 2y dx
⇒ = − y − 2 cex …(v)
IF = e∫ = e∫
−2 dx
= e−2x
P dx
dx2 Now,
2
d y ∴ Solution of the given differential equation is
⇒ + y = − 2 cex
dx2 y ⋅ e−2x = ∫ (2x − 3) e−2x dx + C
I II
Again, on differentiating w.r.t. x, we get
−2x
d3 y dy − (2x − 3) ⋅ e e−2x
+ = − 2 cex …(vi) y ⋅ e−2x = + 2∫ dx + C
dx3 dx 2 2
[by using integration by parts]
d3 y dy d 2y
⇒ + = + y [from Eq. (v)] − (2 x − 3 ) e−2x e−2x
dx2 dx dx2 ⇒ y ⋅ e−2x = − +C
2 2
which is a differential equation of order 3.
⇒ y = (1 − x) + Ce2x
12. Given differential equation On putting x = 0 and y = 1, we get
f (x) 1 =1 + C
f ′ (x) =
b 2 + x2 ⇒ C =0
f ′ (x) dx ∴ y=1 − x
⇒ ∫ f (x)
dx = ∫ 2
b + x2 y = 1 − x passes through (2, − 1)
Differential Equations 363

Now, area of region bounded by curve y = 1 − x2 and 15. Since, centre lies on y = x.
y = 1 − x is shows as ∴ Equation of circle is
y
x2 + y2 − 2ax − 2ay + c = 0
B (0, 1)
y= 1–x2 On differentiating, we get
2x + 2 yy′ − 2a − 2ay′ = 0
A x + yy′
x′
O
x ⇒ x + yy′ − a − ay′ = 0 ⇒ a =
–1,0 (1,0) 1 + y′
y=1–x Again differentiating, we get
y′ (1 + y′ )[1 + yy′ ′+ ( y′ )2] − (x + yy′ ) ⋅ ( y′ ′ )
0=
∴ Area of shaded region (1 + y′ )2
= Area of 1st quadrant of a circle − ⇒ (1 + y′ ) [1 + ( y′ )2 + yy′ ′ ] − (x + yy′ ) ( y′′ ) = 0
Area of ∆ OAB
π 1 π 1 π −2 ⇒ 1 + y′ [( y′ )2 + y′ + 1] + y′′ ( y − x) = 0
= (1) − × 1 × 1 = − =
2
4 2 4 2 4 On comparing with Py′′ + Qy′ + 1 = 0, we get
Hence, options b and c are correct. P = y − x and Q = ( y′ )2 + y′ + 1

14. Here, (1 + ex ) y′ + y ex = 1 16. Given, y2 = 2c (x + c) …(i)


dy dy On differentiating w.r.t. x, we get
⇒ + ex ⋅ + yex = 1
dx dx dy dy
2y = 2c ⇒ c = y
⇒ dy + ex dy + yex dx = dx dx dx
⇒ dy + d (ex y) = dx On putting this value of c in Eq. (i), we get
On integrating both sides, we get dy  dy 
y2 = 2 y x + y 
y + ex y = x + C dx  dx 
Given, y (0) = 2 dy  dy
3/ 2
⇒ y=2 ⋅ x + 2 y1/ 2  
⇒ 2 + e0 ⋅ 2 = 0 + C dx  dx
⇒ C =4 dy  dy
3/ 2
⇒ y − 2x =2 y  
∴ y (1 + e ) = x + 4
x
dx  dx
x+4 2 3
⇒ y=  dy  dy
1 + ex ⇒  y − 2x  = 4 y  
 dx  dx
−4 + 4
Now at x = − 4, y = =0 Therefore, order of this differential equation is 1 and
1 + e−4 degree is 3.
∴ y(−4) = 0 …(i) 17. We have,
dy dy
For critical points, =0 = (2 + 5 y) (5 y − 2)
dx dx
dy (1 + ex ) ⋅ 1 − (x + 4)ex  
= =0
1  dy 
i.e.
dx (1 + ex )2 dy
⇒ = dx ⇒   = dx
25 y2 − 4 25  y2 − 4 
⇒ ex (x + 3) − 1 = 0  25 
or e− x = (x + 3) On integrating both sides, we get
y = e–x Y y=x+3 1 dy
25 ∫ 2  2 2 ∫
= dx
(–1, e)
y − 
 5
(–1, 2)
1 1 y − 2 /5
X′
O
X ⇒ × log =x+C
–1 25 2 × 2 y + 2 /5
5
5y − 2
⇒ log = 20(x + C )
Y′ 5y + 2
Clearly, the intersection point lies between (− 1, 0). 5y − 2
⇒ = Ae20x [Q e20C = A ]
∴ y(x) has a critical point in the interval (− 1, 0). 5y + 2
364 Differential Equations

when x = 0 ⇒ y = 0, then A = 1 dP (x)


20. Given, P(1) = 0 and − P (x) > 0, ∀ x ≥ 1 ...(i)
5 y −2 dx
∴ = e20x
5y + 2 On multiplying Eq. (i) by e− x , we get
d d −x
5 f (x) −2 e− x ⋅ P (x) ⋅ e >0
lim = lim e20x dx dx
x → −∞ 5 f (x) + 2 x → −∞
d
5 f (x) − 2 ⇒ (P (x) ⋅ e− x ) > 0
⇒ lim = 0 dx
n → −∞ 5 f (x) + 2
⇒ P (x) ⋅ e− x is an increasing function.
⇒ lim 5 f (x) −2 = 0 ⇒ P (x) ⋅ e− x > P (1) ⋅ e−1 , ∀ x ≥ 1
n→ − ∞

2 ⇒ P (x) > 0, ∀ x > 1 [Q P (1) = 0 and e− x > 0]


⇒ lim f (x) = = 0.4
n→ − ∞ 5 21. Equation of tangent to the curve y = f (x) at point
18. From given integral equation, f (0) = 0 . A (x, y) is Y − y =
dy
(X − x)
Also, differentiating the given integral equation w.r.t. x dx
f ′ (x) = f (x) y

If f (x) ≠ 0 (1, 1)
f ′ (x)
⇒ = 1 ⇒ log f (x) = x + c Q
f (x) (x, y) y = f(x)
A
⇒ f (x) = e e
c x
x ′' x
O P
Q f (0) = 0 ⇒ ec = 0 , a contradiction
∴ f (x) = 0, ∀ x ∈ R
y′
⇒ f (ln 5) = 0
 dx 
Alternate Solution whose, x-intercept  x − y ⋅ , 0
x  dy 
Given, f (x) = ∫ f (t ) dt
0  dy
y-intercept 0, y − x 
⇒ f (0) = 0 and f ′ (x) = f (x)  dx
If f (x) ≠ 0 Given, ∆OPQ = 2
f ′ (x) 1  dx  dy
⇒ = 1 ⇒ ln f (x) = x + c ⇒ ⋅ x − y   y − x  = 2
f (x) 2  dy  dx
⇒ f (x) = e ⋅ e
c x
 1 dy
⇒  x − y  ( y − xp) = 4, where p=
Q f (0) = 0  p dx
⇒ ec = 0, a contradiction ⇒ p2x2 − 2 pxy + 4 p + y2 = 0
∴ f (x) = 0, ∀ x ∈ R ⇒ f (ln 5) = 0 ⇒ ( y − px)2 + 4 p = 0
dy y y − 1
2
∴ y − px = 2 − p
19. Given, =
dx x x − 1
2 ⇒ y = px + 2 − p …(i)
dy dx On differentiating w.r.t. x, we get
∫ y y −1
2
=∫
x x2 − 1 p= p+
dp  1
⋅ x + 2 ⋅   (− p)−1/ 2 ⋅ (−1)
dp
dx  2 dx
⇒ sec−1 y = sec−1 x + c dp −1/ 2
2 π π π ⇒ { x − (− p) } = 0
At x = 2 , y = ; = + c ⇒ c=− dx
3 6 3 6 dp
⇒ = 0 or x = (− p)−1/ 2
 π  1 3 dx
Now, y = sec sec−1 x −  = cos cos −1 − cos −1  dp
 6  x 2  If =0 ⇒ p= c
dx
  3 1 3  On putting this value in Eq. (i), we get y = cx + 2 − c
= cos cos −1  + 1− 2 1 − 
  2 x x 4  This curve passes through (1, 1).
3 1 1 ⇒ 1 = c + 2 −c
y= + 1− 2 ⇒ c = −1
2x 2 x
∴ y=−x+2
Differential Equations 365

⇒ x+ y=2 25. Here, f (x) + 2x = (1 − x)2 ⋅ sin 2 x + x2 + 2x …(i)


−1/ 2
Again, if x = (− p) where, I: f (x) + 2x = 2(1 + x)2 …(ii)
1
⇒ − p = 2 putting in Eq. (i) ∴ 2(1 + x2) = (1 − x)2 sin 2 x + x2 + 2x
x
⇒ (1 − x)2 sin 2 x = x2 − 2x + 2
−x 1
y = 2 + 2⋅ ⇒ xy = 1 ⇒ (1 − x)2 sin 2 x = (1 − x)2 + 1
x x
⇒ (1 − x)2 cos 2 x = − 1
Thus, the two curves are xy = 1 and x + y = 2.
which is never possible.
22. Let φ(x) = e−x f (x)
∴I is false.
 1
Here, φ′ (x) < 0, x ∈ 0,  Again, let h (x) = 2 f (x) + 1 − 2x(1 + x)
 4
where, h (0) = 2 f (0) + 1 − 0 = 1
1 
and φ′ (x) > 0, x ∈  , 1 h(1) = 2(1) + 1 − 4 = − 3 as [h (0)h (1) < 0]
4 
⇒ h (x) must have a solution.
 1
⇒ e− x f ′ (x) − e− x f (x) < 0, x ∈ 0,  ∴ II is true.
 4
1
⇒ f ′ (x) < f (x) ,0 < x < Topic 2 Linear Differential Equation and
4
23. Here, f ′′ (x) − 2 f ′ (x) + f (x) ≥ ex Exact Differential Equation
⇒ f ′′ (x)e− x − f ′ (x)e− x − f ′ (x)e− x + f (x)e− x ≥ 0
1. Given differential equation is
( y2 − x3 ) dx − xy dy = 0, (x ≠ 0)
d d
⇒ { f ′ (x)e−x } − { f (x)e− x } ≥ 1 dy
dx dx ⇒ xy − y2 = − x3
dx
d dy dt dy 1 dt
⇒ { f ′ (x)e−x − f (x)e− x } ≥ 1 Now, put y2 = t ⇒ 2 y = ⇒y =
dx dx dx dx 2 dx
d 2 −x x dt dt 2
⇒ { e f (x)} ≥ 1, ∀x ∈ [0, 1] ∴ − t = − x3 ⇒ − t = − 2x2
dx2 2 dx dx x
which is the linear differential equation of the form
∴ φ (x) = e− x f (x) is concave function. dt
+ Pt = Q.
f (0) = f (1) = 0 dx
⇒ φ(0) = 0 = f (1) 2
Here, P = − and Q = − 2x2.
⇒ φ (x) < 0 x
2
−x −∫
⇒ e f (x) < 0 Now, IF = e x
1dx
=
∴ f (x) < 0 x2
Q Solution of the linear differential equation is
24. Here, f (x) = (1 − x)2 ⋅ sin 2 x + x2 ≥ 0, ∀ x
(IF) t = ∫ Q (IF)dx + λ [where λ is integrating constant]
x  2 (t − 1 ) 
and g (x) = ∫  − log t f (t )dt
1  t + 1   1  1
∴ t  2 = − 2 ∫  x2 × 2 dx + λ
x   x 
2(x − 1) 
⇒ g′ (x) =  − log x ⋅ f{
(x) …(i) t
 (x + 1 )  + ve ⇒ = − 2x + λ
x2
For g′ (x) to be increasing or decreasing. y2
⇒ + 2x = λ [Q t = y2]
2(x − 1) x2
Let φ(x) = − log x
x+1 ⇒ y2 + 2x3 − λx2 = 0
4 1 − (x − 1) or y2 + 2x3 + Cx2 = 0 [let C = − λ]
φ′ (x) = − =
(x + 1)2 x x (x + 1)2 2. Given differential equation is
 1
φ′ (x) < 0, ∀ x > 1 y2dx +  x −  dy = 0
 y
⇒ φ (x) < φ (1)
dx 1 1
⇒ φ(x) < 0 …(ii) ⇒ + x = 3 , which is the linear differential
dy y2 y
From Eqs. (i) and (ii), g′ (x) < 0, x ∈ (1, ∞ ) dx
equation of the form + Px = Q.
∴ g (x) is decreasing on x ∈ (1, ∞ ). dy
366 Differential Equations

 π  π π π2 1 π2
1 1 2
Here, P = and Q = 3 1
and y  + y −  = + + + = + 2
y2 y  4  4  16 2 16 2 4
1 1
 π  π π π2
∫ 2 dy 2
− 1 1
Now, IF = e y
=e y and y  − y −  = + − − =0
 4  4  16 2 16 2
∴The solution of linear differential equation is
4. Given differential equation
x ⋅ (IF) ∫ Q (IF)dy + C dy
= (tan x − y)sec2 x
1 − 1/ y dx
⇒ x e− 1/ y = ∫ e dy + C dy
y3 ⇒ + (sec2 x) y = sec 2x tan x,
dx
1 1
∴ x e− 1/ y = ∫ (− t ) et dt + C [Qlet − = t ⇒ + 2 dy = dt] which is linear differential equation of the form
y y dy
+ Py = Q,
= − tet + ∫ e dt + C
t
[integration by parts] dx
where P = sec2 x and Q = sec2 x tan x
= − te + e + C
t t

IF = e∫
sec 2 x dx
1 = etan x
⇒ x e− 1/ y = e− 1/ y + e− 1/ y + C … (i)
y So, solution of given differential equation is
Now, at y = 1, the value of x = 1, so y × IF = ∫ (Q × IF)dx + C
1
1 ⋅ e− 1 = e− 1 + e− 1 + C ⇒ C = − y(etan x ) = ∫ etan x ⋅ sec 2 x tan x dx + C
e
On putting the value of C, in Eq. (i), we get Let tan x = t ⇒ sec 2x dx = dt
1 e1/ y yetan x = ∫ et ⋅ t dt + C = tet − ∫ et dt + C
x= +1−
y e
[using integration by parts method]
1 e1/ 2 3 1
So, at y = 2, the value of x = + 1 − = − = et (t − 1) + C
2 e 2 e ⇒ y⋅ e tan x
= etan x (tan x − 1) + C [Q t = tan x]
3. Given differential equation is Q y(0) = 0
dy
+ y tan x = 2x + x2 tan x , which is linear differential ⇒ 0 = 1(0 − 1) + C ⇒ C = 1
dx ∴ y⋅ etan x
= etan x (tan x − 1) + 1
dy
equation in the form of + Py = Q . π
dx Now, at x = −
4
Here, P = tan x and Q = 2x + x2 tan x ye−1 = e−1 (−1 − 1) + 1
∴IF = e∫ tan x dx = elog e (sec x ) = sec x ⇒ ye−1 = − 2e−1 + 1 ⇒ y = e − 2
Now, solution of linear differential equation is given as
y × IF = ∫ (Q × IF)dx + C Key Idea (i) First convert the given differential equation into
5. dy
∴ y(sec x) = ∫ (2x + x tan x) sec x dx + C
2 linear differential equation of the form + Py = Q
dx
= ∫ (2x sec x) dx + ∫x
2
sec x tan x dx + C (ii) Find IF
(iii) Apply formula, y( IF) = ∫ Q( IF) dx + C
Q ∫ x2 sec x tan x dx = x2 sec x − ∫ (2x sec x) dx
Given differential equation
dy
Therefore, solution is cos x − (sin x) y = 6x
dx
y sec x = 2∫ x sec x dx + x2 sec x − 2∫ x sec x dx + C
dy 6x
⇒ − (tan x) y = , which is the linear
⇒ y sec x = x2 sec x + C …(i) dx cos x
Q y(0) = 1 ⇒ 1(1) = 0(1) + C ⇒ C = 1 differential equation of the form
dy
Now, y = x2 + cos x [from Eq. (i)] + Px = Q,
dx
and y′ = 2x − sin x 6x
According to options, where P = − tan x and Q =
cos x
 π  − π   π 1    π 1
So, IF = e ∫
− tanx dx
y′   − y′   = 2  −  − 2 −  +  =π− 2 = e− log(sec x ) = cos x
 4  4    4  2   4 2
∴Required solution of differential equation is
 π  π   π 1    π 1
and y′   + y′  −  = 2  −  + 2 −  +  =0 cos x x2
 4  4   4 2  4 2 y(cos x) = ∫ (6x) dx + C = 6 + C = 3 x2 + C
cos x 2
Differential Equations 367

 π ⇒ y(1 + x2) = ∫
dx
+C
Given, y  = 0
 3 1 + x2
⇒ y(1 + x2) = tan −1 (x) + C
2
 π π 2
So, 0 = 3  + C ⇒ C = −
 3 3 Q y(0) = 0
π2 ∴ C =0
∴ y(cos x) = 3x2 −
3 ∴ y(1 + x2) = tan −1 x [Q C = 0]
π −1
Now, at x = tan x
6 ⇒ y=
 3 1 + x2
π π 2
π π 2 2 2
y  = 3 − =− ⇒ y=−  tan −1 x
 2 36 3 4 2 3 ⇒ ay = a  
 1 + x2 
6. Given differential equation is
[multiplying both sides by a]
dy
x + 2 y = x2, (x ≠ 0) Now, at x = 1
dx
dy  2 π
⇒ +   y = x,  tan −1 (1) aπ π
dx  x a y (1 ) = a   = a 4 = = (given)
 1+1  2 8 32
which is a linear differential equation of the form
1 1
dy
+ Py = Q ∴ a = ⇒a =
dx 4 16
2 8. We know that, slope of the tangent at any point (x, y) on
Here, P = and Q = x
x the curve is
2
∫ dx dy x2 − 2 y
∴ IF = e x = e2log x = x2 = (given)
dx x
Since, solution of the given differential equation is dy 2
⇒ + y=x …(i)
y × IF = ∫ (Q × IF) dx + C dx x
x4 which is a linear differential equation of the form
∴ y(x2) = ∫ (x × x2) dx + C ⇒ yx2 = +C dy
+ P (x) ⋅ y = Q (x),
4 dx
1 3
Q y(1) = 1, so 1 = + C ⇒C = 2
4 4 where P (x) = and Q (x) = x
x
x4 3 x2 3 Now, integrating factor
∴ yx =
2
+ ⇒y= +
4 4 4 4 x2 2
∫ dx
(IF) = e∫
P ( x )dx
= e x = e2log e x
7. Given differential equation is 2
= elog e x [Q m log a = log am ]
dy
(x2 + 1)2 + 2x(x2 + 1) y = 1
dx = x2 [Q elog e f ( x ) = f (x)]
dy  2x  1 and the solution of differential Eq. (i) is
⇒ + y=
dx  1 + x2 (1 + x2)2 y(IF) = ∫ Q (x)(IF)dx + C ⇒ y(x2) = ∫ x ⋅ x2 dx + C
[dividing each term by (1 + x2)2] x4
…(i) ⇒ yx2 =
+C …(ii)
4
This is a linear differential equation of the form
Q The curve (ii) passes through the point (1, − 2),
dy
+ P⋅ y =Q therefore
dx
1 9
2x 1 −2 = + C ⇒C = −
Here, P = and Q = 4 4
(1 + x )
2
(1 + x2)2
2x
∴ Equation of required curve is 4 yx2 = x4 − 9.
∫ 2
dx
Now, checking all the option, we get
∴Integrating Factor (IF) = e 1 + x
2
only ( 3 , 0) satisfy the above equation.
= eln(1 + x )
= (1 + x2)
9. Given differential equation is
and required solution of differential Eq. (i) is given by dy
x + y = x log e x, (x > 1)
y ⋅ (IF) = ∫ Q (IF)dx + C dx
dy 1
1 ⇒ + y = log e x …(i)
⇒ y(1 + x2) = ∫ (1 + x2)dx + C dx x
(1 + x2)2
368 Differential Equations

Which is a linear differential equation. 1


if < x<1 [by using product rule of derivative]
1
∫ dx
2
So, if = e x = elog e x = x log e 2 −2log e 2 1 −2
and y(log e 2) = e = log e 2 elog e 2
Now, solution of differential Eq. (i), is 2 2
1 −2 1
y × x = ∫ (log e x) x dx + C = . log e 2 ⋅ 2 = log e 2
2 8
x2 x2 1 3 f (x)
⇒ yx = log e x − ∫ × dx + C 11. Given, f ′ (x) = 7 − , (x > 0)
2 2 x 4 x
dy
[using integration by parts] On putting f (x) = y and f ′ (x) = , then we get
2 dx
x x2
⇒ yx =
log e x − +C …(ii) dy 3 y
2 4 = 7−
dx 4 x
Given that, 2 y(2) = log e 4 − 1 …(iii) dy 3
On substituting, x = 2, in Eq. (ii), ⇒ + y=7 …(i)
dx 4x
we get
which is a linear differential equation of the form
4 4 dy 3
2 y(2) = log e 2 − + C, + Py = Q, where P = and Q = 7.
2 4 dx 4x
[where, y(2) represents value of y at x = 2] 3
∫ dx
Now, integrating factor (IF) = e 4x
⇒ 2 y(2) = log e 4 − 1 + C …(iv)
3
[Q m log a = log am ] = e4
log x
= elog x = x3/ 4
3/ 4

From Eqs. (iii) and (iv), we get and solution of differential Eq. (i) is given by
C =0
y(IF) = ∫ (Q ⋅ (IF))dx + C
So, required solution is
x2 x2 yx3/ 4 = ∫ 7x3/ 4dx + C
yx = log e x −
2 4 3
+1
e2 e2 x4
Now, at x = e, ey(e) = log e e − ⇒ yx 3/ 4
=7 +C
2 4 3
+1
[where, y(e) represents value of y at x = e] 4
7
e
⇒ y(e) = [Q log e e = 1]. ⇒ y x3/ 4 = 4x 4 + C
4
dy  2x + 1 ⇒ y = 4 x + C x− 3 / 4
− 2x
10. We have, + y=e So, y = f (x) = 4x + C ⋅ x−3/ 4
dx  x 
 1 4
dy
+ Py = Q, where Now, f   = + C ⋅ x3/ 4
which is of the form  x x
dx
2x + 1  1 4 
P= and Q = e−2x ∴ lim x f   = lim x + Cx3/ 4
x x→ 0 +  x x→ 0 +  x 
 1 + 2x  1 
∫  dx ∫  + 2 dx = lim (4 + Cx7/ 4 ) = 4
Now, IF = e∫ =e  x 
= e x 
Pdx
x→ 0 +
ln x + 2x 12. Given, differential equation is
=e = e . e = x. e
ln x 2x 2x

dy  3  1
and the solution of the given equation is + 2 
y= , which is a linear differential
dx  cos x  cos 2 x
y ⋅ (IF) = ∫ (IF) Q dx + C dy 3
equation of the form + Py = Q, where P = and
⇒ y(xe2x ) = ∫ (x e2x . e−2x ) dx + C dx cos 2 x
1
Q= .
x2 cos 2 x
= ∫ x dx + C = +C … (i)
2 Now, Integrating factor
3
1 −2 ∫ 2 dx
IF = e cos x = e∫
3 sec 2 x dx
Since, y = e when x = 1 = e3 tan x and the solution of
2
differential equation is given by
1 −2 2 1
∴ e . e = + C ⇒ C = 0 (using Eq. (i)) y(IF) = ∫ (Q. (IF)) dx
2 2
x2 x
∴ y (xe ) =
2x
⇒ y = e−2x ⇒ e3 tan x . y = ∫ e3 tan x sec2 x dx … (i)
2 2
dy 1 −2x x −2x 1 
Now, = e + e (− 2) = e−2x  − x < 0, Let I = ∫ e3 tan x sec2 x dx
dx 2 2 2 
Differential Equations 369

3 tan x = t IF = e∫ Pdx = e∫
Put cotxdx
Now, = elogsin x = sin x
⇒ 3 sec2 x dx = dt Solution of the differential equation is
et et e3 tan x
∴ I = ∫ dt = +C= +C y ⋅ sin x = ∫ 4x cosec x sin xdx + C
3 3 3
From Eq. (i) ⇒ y sin x = ∫ 4xdx + C = 2x2 + C
e3 tan x π
e3 tan x . y = +C Put x = , y = 0, we get
3 2
π2 π2
It is given that when, C=− ⇒ y sin x = 2x2 −
π 4 2 2
x = , y is π
4 3 Put x=
e3 6
3 4
⇒ e = +C  1  π 2 π 2
3 3 ∴ y  = 2  −
 2  36  2
⇒ C = e3
e3 tan x π2 8π 2
Thus, e3 tan x y = + e3 ⇒ y= − π2 ⇒ y = −
3 9 9
π e−3 Alternate Method
Now, when x = − , e−3 y = + e3 dy
4 3 We have, sin x + y cos x = 4x, which can be written as
dx
1   π  d
⇒ y = e6 + Q tan  − 4  = − 1 (sin x ⋅ y) = 4x
3   dx
On integrating both sides, we get
13. Given differential equation can be rewritten as
dy  2 d
+   ⋅ y = x, which is a linear differential equation of
dx  x
∫ dx (sin x ⋅ y) ⋅ dx = ∫ 4x ⋅ dx
dy 2 4 x2
the form + Py = Q, where P = and Q = x. ⇒ y ⋅ sin x = +C
dx x 2
Now, integrating factor ⇒ y ⋅ sin x = 2x2 + C
2
∫ dx 2 π
(IF) =
e x = e2log x = elog x = x2 Now, as y = 0 when x =
2
[Q elog f ( x )
= f (x) ]
π2
and the solution is given by ∴ C=−
2
y(IF ) = ∫ (Q × IF ) dx + C π2
⇒ y ⋅ sin x = 2x2 −
⇒ yx2 = ∫ x3 dx + C 2
π
Now, putting x = , we get
x4 6
⇒ yx2 = +C …(i)
4  1  π 2 π 2
y  = 2  −
Since, it is given that y = 1 when x = 1  2  36  2
∴ From Eq. (i), we get π2 8π 2
1 3 ⇒ y= − π2 = −
1 = + C ⇒C = …(ii) 9 9
4 4
15. Given differential equation is
∴ 4 x2 y = x4 + 3 [using Eqs. (i) and (ii)]
x4 + 3 y(1 + xy) dx = x dy
⇒ y= ⇒ y dx + xy2 dx = x dy
4 x2
x dy − y dx
1
+3 ⇒ = x dx
 1 16 49 y2
Now, y  = =
 2 1 16 ( y dx − x dy)  x
4× ⇒ − = x dx ⇒ − d   = x dx
4 y 2  y
14. We have, On integrating both sides, we get
dy dy
sin x + y cos x = 4x ⇒ + y cot x = 4xcosec x x x2
dx dx − = +C …(i)
This is a linear differential equation of form y 2
dy
+ Py = Q Q It passes through (1, − 1).
dx 1 1
∴ 1= +C ⇒ C =
where P = cot x, Q = 4x cosec x 2 2
370 Differential Equations

x x2 1 π /3 sin 2 θ
Now, from Eq. (i) − = + =2 ∫ cos θ dθ [taking x = sin θ ]
y 2 2 0 cos θ
2x π /3 π /3
⇒ x2 + 1 = − =2 ∫ sin 2 θ dθ = ∫ (1 − cos 2θ ) dθ
y 0 0
π /3
2x  sin 2θ  π sin 2π / 3 π 3
⇒ y=− 2 = θ −  = − = −
x +1  2 0 3 2 3 4
 1 4 18. PLAN Whenever we have linear differential equation containing
∴ f −  =
 2 5 inequality, we should always check for increasing or
decreasing,
16. Given differential equation is dy dy
i.e. for + Py < 0 ⇒ + Py > 0
dy dx dx
(x log x) + y = 2x log x
Multiply by integrating factor, i.e. e ∫
Pdx
dx and convert into
dy y total differential equation.
⇒ + =2 Here, f ′ (x) < 2 f (x), multiplying by e− ∫ 2dx
dx x log x
f ′ (x) ⋅ e−2x − 2e−2x f (x) < 0
This is a linear differential equation.
d
1
∫ x log x dx ⇒ ( f (x) ⋅ e−2x ) < 0
∴ IF = e = elog(log x ) = log x dx
1 
Now, the solution of given differential equation is given ∴ φ (x) = f (x)e−2x is decreasing for x ∈ , 1
2 
by
y ⋅ log x = ∫ log x ⋅ 2dx 1
Thus, when x >
2
⇒ y ⋅ log x = 2∫ log xdx  1  1
φ (x) < φ   ⇒ e−2x f (x) < e−1 ⋅ f  
⇒ y ⋅ log x = 2 [x log x − x] + c  2  2
At x=1 ⇒ c=2  1
⇒ f (x) < e2x −1 ⋅ 1, given f   = 1
⇒ y ⋅ log x = 2 [x log x − x] + 2  2
At x = e, y = 2(e − e) + 2 ⇒ 0<∫
1
f (x) dx < ∫
1
e2x − 1 dx
⇒ y=2 1/ 2 1/ 2
1
dy 1  e2x − 1 
17. PLAN (i) Solution of the differential equation + Py = Q is ⇒ 0<∫ f (x) dx <  
dx 1/ 2  2  1/ 2
y ⋅ (IF) = ∫ Q ⋅ (IF) dx + c
1 e−1
where, IF = e ∫
P dx
⇒ 0<∫ f (x) dx <
1/ 2 2
a a
(ii) ∫− a f( x ) dx = 2 ∫0 f( x ) dx , if f( − x ) = f( x ) t 2f (x) − x2f (t )
19. Given, lim =1
t→ x t−x
Given differential equation
⇒ x f ′ (x) − 2x f (x) + 1 = 0
2
dy x x4 + 2x
+ 2 y= x f ′ (x) − 2x f (x) 1
2
dx x − 1 1 − x2 ⇒ + 4 =0
(x2)2 x
This is a linear differential equation. d  f (x) 1

x 1 ⇒   =− 4
dx  x2 
dx ln | x 2 − 1 |
2
x −1 x
IF = e = e2 = 1 − x2
On integrating both sides, we get
x(x + 2)
3
⇒ Solution is y 1 − x2 = ∫ ⋅ 1 − x2 dx 1
1 − x2 f (x) = cx2 +
3x
x5
or y 1 − x2 = ∫ (x4 + 2x) dx = + x2 + c Also, f (1) = 1, c=
2
5 3
x5 2 1
f (0) = 0 ⇒ c = 0 ⇒ f (x) 1 − x2 = + x2 Hence, f (x) = x2 +
5 3 3x
3 /2 3 /2 x2
Now, ∫ f (x) dx = ∫ dx 20. Given, x dy = y(dx + y dy), y > 0
− 3 /2 − 3 /2
1 − x2
⇒ x dy − y dx = y2dy
[using property]
x2 x dy − y dx  x
=2 ∫
3 /2
dx ⇒ = dy ⇒ d   = − dy
y2  y
0
1−x 2
Differential Equations 371

On integrating both sides, we get C


(d) f (x) = x + ,C ≠0
x x
=− y+ c …(i)
y For C > 0, lim f (x) = ∞
x → 0+
Since, y (1) = 1 ∴ Function is not bounded in (0, 2).
⇒ x = 1, y = 1 ∴ Option (d) is incorrect.
∴ c=2 23. PLAN Linear differential equation under one variable.
x + Py = Q; IF = e ∫
dy Pdx
Now, Eq. (i) becomes, + y = 2
y dx
∴ Solution is, y (IF) = ∫ Q ⋅ (IF) dx + C
Again, for x = − 3
⇒ –3 + y2 = 2 y y ′ − y tan x = 2x sec x and y (0) = 0
dy
⇒ y − 2y − 3 = 0
2
⇒ − y tan x = 2x sec x
dx
⇒ ( y + 1) ( y − 3) = 0
∴ IF = ∫ e− tan x dx = elog|cos x| = cos x
As y > 0, take y = 3, neglecting y = − 1.
dy  t  1 Solution is y ⋅ cos x = ∫ 2x sec x ⋅ cos x dx + C
21. Given, −  y= and y (0) = − 1
dt  1 + t  (1 + t ) ⇒ y ⋅ cos x = x2 + C
Which represents linear differential equation of first As y(0) = 0 ⇒ C = 0
order. ∴ y = x2 sec x
 t 
∫ −  1 + t  dt  π π2
∴ IF = e  = e− t + log (1 + t ) = e− t ⋅ (1 + t ) Now, y  =
 4 8 2
Required solution is,  π π π2
1 ⇒ y′   = +
ye− t (1 + t ) = ∫ ⋅ e− t (1 + t ) dt + c = ∫ e− t dt + c  4 2 8 2
1+ t
 π  2π  π  4π 2π
2 2

⇒ −t
(1 + t ) = − e− t + c y  = ⇒ y′   = +
ye  3 9  3 3 3 3
Since, y (0) = − 1
24. Let w (x) = u (x) − v (x) …(i)
⇒ −1 ⋅ e0 (1 + 0) = − e0 + c
and h (x) = f (x) − g (x)
c=0
1 1 On differentiating Eq. (i) w.r.t. x
∴ y=− ⇒ y (1) = − dw du dv
(1 + t ) 2 = −
dx dx dx
f (x)
22. Here, f ′ (x) = 2 − = { f (x) − p (x) ⋅ u (x)} − { g (x) − p (x) v (x)} [given]
x
dy y = { f (x) − g (x)} − p (x) [u (x) − v (x)]
or + = 2 [i.e. linear differential equation in y] dw
dx x ⇒ = h (x) − p (x) ⋅ w (x) …(ii)
1 dx
∫ dx
Integrating Factor, IF = e x = elog x = x dw
⇒ + p (x) w (x) = h (x) which is linear differential
∴ Required solution is y ⋅ (IF) = ∫ Q (IF) dx + C dx
equation .
⇒ y(x) = ∫ 2(x) dx + C The integrating factor is given by
IF = e∫
p ( x ) dx
⇒ yx = x + C 2 = r (x) [let]
C On multiplying both sides of Eq. (ii) of r (x), we get
∴ y=x+ [QC ≠ 0, as f (1) ≠ 1]
x dw
r (x) ⋅ + p (x) (r (x)) w (x) = r (x) ⋅ h (x)
 1 dx
(a) lim f ′   = lim (1 − Cx2) = 1
x → 0+  x x → 0 + d  dr 
⇒ [r (x) w (x)] = r (x) ⋅ h (x) Q = p (x) ⋅ r (x)
∴ Option (a) is correct. dx  dx 
 1 r (x) = e∫
P ( x ) dx
(b) lim x f   = lim (1 + Cx2) = 1 Now, > 0, ∀ x
x → 0+  x x → 0 +
and h (x) = f (x) − g (x) > 0, for x > x1
∴ Option (b) is incorrect. d
Thus, [r (x) w (x)] > 0, ∀ x > x1
(c) lim x2f ′ (x) = lim (x2 − C ) = − C ≠ 0 dx
+ +
x→ 0 x→ 0
∴ Option (c) is incorrect. r (x) w (x) increases on the interval [x, ∞ [
372 Differential Equations

Therefore, for all x > x1  1 1 1


log e   + = 1 ⇒ = 1 + log e 2
r (x) w (x) > r (x1 ) w (x1 ) > 0  2 y y
[Q r (x1 ) > 0 and u (x1) > v (x1)]  1 1
⇒ y= f   =
 2 1 + log e 2
⇒ w (x) > 0 ∀ x > x1
⇒ u (x) > v (x) ∀ x > x1 [Q r (x) > 0] Hence, option (a) is correct.
dy 2 y
Hence, there cannot exist a point (x, y) such that x > x1 2. Given, =
and y = u (x) and y = v (x). dx x2
dy 2
25.
dy
+ y ⋅ g′ (x) = g (x) g′ (x)
⇒ ∫ y = ∫ x2dx [integrating both sides]
dx
2
IF = e∫
g ′( x ) dx
= e g( x ) ⇒ log e| y| = − + C …(i)
x
∴ Solution is y (e g( x ) ) = ∫ g (x) ⋅ g′ (x) ⋅ e g( x ) dx + C Since, curve (i) passes through centre (1, 1) of the circle
Put g (x) = t, g′ (x) dx = dt x2 + y2 − 2x − 2 y = 0
y(eg( x ) ) = ∫ t ⋅ et dt + C ∴
2
log e (1) = − + C ⇒ C = 2
1
= t ⋅ et − ∫ 1 ⋅ et dt + C = t ⋅ et − et + C ∴ Equation required curve is
y eg( x ) = ( g (x) − 1) eg( x ) + C …(i) 2
log e| y| = − + 2 [put C = 2 in Eq. (i)]
Given, y(0) = 0, g (0) = g (2) = 0 x
∴ Eq. (i) becomes, ⇒ x log e| y| = 2(x − 1)
y(0) ⋅ eg( 0) = ( g (0) − 1) ⋅ eg( 0) + C
3. Given differential equation is
⇒ 0 = (− 1) ⋅ 1 + C ⇒ C = 1
(x2 − y2)dx + 2xy dy = 0, which can be written as
∴ y(x) ⋅ eg( x ) = ( g (x) − 1) eg( x ) + 1
dy y2 − x2
⇒ y(2) ⋅ eg( 2) = ( g (2) − 1) eg( 2) + 1, where g(2) = 0 =
dx 2xy
⇒ y(2) ⋅ 1 = (− 1) ⋅ 1 + 1
Put y = vx [Q it is in homogeneous form]
y(2) = 0
dy dv
⇒ = v+ x
dx dx
Topic 3 Applications of Homogeneous Now, differential equation becomes
Differential Equations dv v2x2 − x2 dv (v2 − 1)x2
v+ x = ⇒ v+ x =
1. Given differential equation is dx 2x(vx) dx 2vx2
2x2dy = (2xy + y2) dx dv v2 − 1 v2 − 1 − 2v2
⇒ x = −v=
dy 2xy + y2 dx 2v 2v
⇒ =
dx 2 x2 dv 1 + v2 2v dv dx
⇒ x =− ⇒ ∫ = −∫
[Homogeneous differential equation] dx 2v 1+ v 2
x
dy dv
Let y = vx = =v+ x ⇒ ln (1 + v ) = − ln x − ln C
2
dx dx  f ′ (x) 
∴ The differential equation becomes Q ∫ f (x) dx ⇒ ln| f (x)|+ C 
dv 2vx2 + v2x2
v+ x = ⇒ ln|(1 + v2)Cx|= 0 [Q ln A + ln B = ln AB]
dx 2x2
dv 2v + v2 − 2v ⇒ (1 + v )Cx = 1
2
[log e x = 0 ⇒ x = e0 = 1]
⇒ x = y
dx 2 Now, putting v = , we get
x
dv v2 dv dx
⇒ x = ⇒2 2 =  y2 
dx 2 v x 1 + 2  Cx = 1 ⇒ C (x2 + y2) = x
 −1  2x  x 
⇒ 2   = log e x + C ⇒ log e x + + C =0 …(i)
 v y Q The curve passes through (1, 1), so
y 1
as v= C (1 + 1) = 1 ⇒ C =
x 2
Q The curve (i) passes through the point (1, 2). Thus, required curve is x2 + y2 − 2x = 0, which represent
2x
So, c = − 1 ∴ log e x + =1 a circle having centre (1, 0)
y
1 ∴ The solution of given differential equation represents
Now, at x= a circle with centre on the X-axis.
2
Differential Equations 373

dp 1 7. Let a point P (h , k) on the curve y = y(x), so equation of


4. Given, differential equation is − p(t ) = −200 is a
dt 2 tangent to the curve at point P is
linear differential equation.  dy
y−k =   (x − h ) …(i)
−1  dx h, k
Here, p(t ) = , Q (t ) = −200
2
Now, the tangent (i) intersect the Y -axis at Y p, so
 1
∫ −   dt −
t
 dy dy  dy
IF = e  2 = e 2 coordinates Y p is 0, k − h  , where = 
 dx dx  dx ( h, k)
Hence, solution is So, PY p = 1 (given)
p (t ) ⋅ IF = ∫ Q (t ) ⋅ IF dt 2
 dy
⇒ h 2 + h 2  = 1

t

t  dx
p(t )⋅ e 2 = ∫ −200 ⋅ e 2dt
dy 1 − x2
t t ⇒ =± [on replacing h by x]
− − dx x
p(t )⋅ e 2 = 400 e 2 +K
1 − x2
⇒ p(t ) = 400 + ke−1/ 2 ⇒ dy = ± dx
x
If p(0) = 100, then k = − 300 On putting x = sin θ, dx = cos θ dθ, we get
t
⇒ p(t ) = 400 − 300 e2 1 − sin 2 θ cos 2 θ
dy = ± cos θdθ = ± dθ
sin θ sin θ
5. PLAN To solve homogeneous differential equation, i.e. substitute
= ± (cos ecθ − sin θ )dθ
y
=v
x ⇒ y = ± [ln (cos ecθ − cot θ ) + cos θ ] + C
∴ y = vx ⇒
dy
=v +x
dv   1 − cos θ  
⇒ y = ± ln   + cos θ  + C
dx dx
  sin θ  
Here, slope of the curve at (x, y) is   1 − 1 − sin 2 θ  
dy y  y ⇒ y = ± ln   + 1 − sin 2 θ  + C
= + sec    sin θ 
dx x  x    
y  1 − 1 − x  2 
Put =v ⇒ y = ± ln   + 1 − x2  + C [Qx = sin θ]
x  
  x  
dv dv
∴ v+ x = v + sec (v) ⇒ x = sec (v)  1 + 1 − x2 
dx dx = ± − ln + 1 − x2  + C
dv dx dx  x 
⇒ ∫ sec v = ∫ x ⇒ ∫ cos v dv = ∫ x [on rationalization]
 y
⇒ sin v = log x + log c ⇒ sin   = log(cx) Q The curve is in the first quadrant so y must be
 x
positive, so
 π  π  1 + 1 − x2 
As it passes through 1,  ⇒ sin   = log c
 6  6 y = ln   − 1 − x2 + C
 
 x 
1
⇒ log c =
2 As curve passes through (1, 0), so
 y 1 0 = 0 − 0 + c ⇒ c = 0, so required curve is
∴ sin   = log x +  1 + 1 − x2 
 x 2
y = ln   − 1 − x2
 
dP  x 
6. Given, = (100 − 12 x )
dx and required differential equation is
⇒ dP = (100 − 12 x ) dx
dy 1 − x2
On integrating both sides, we get =−
dx x
∫ dP = ∫ (100 − 12 x ) dx ⇒ xy′+ 1 − x2 = 0
P = 100x − 8x3/ 2 + C Hence, options (a) and (c) are correct.
When x = 0, then P = 2000 ⇒ C = 2000 dy
8. Given, (x2 + xy + 4x + 2 y + 4) − y2 = 0
Now, when x = 25, then is dx
dy
P = 100 × 25 − 8 × (25)3/ 2 + 2000 ⇒ [(x2 + 4x + 4) + y(x + 2)] − y2 = 0
dx
= 2500 − 8 × 125 + 2000 dy
= 4500 − 1000 = 3500 ⇒ [(x + 2)2 + y(x + 2)] − y2 = 0
dx
374 Differential Equations

Put x + 2 = X and y = Y , then |x + 3|2  (x + 3)2


i.e. log  + =0
(X 2 + XY )
dY
−Y 2 = 0  3e  (x + 2)
dX
To check the number of solutions.
⇒ X 2dY + XYdY − Y 2dX = 0
(x + 3)2
⇒ X 2dY + Y (XdY − YdX ) = 0 Let g (x) = 2 log (x + 3) + − log (3e)
(x + 2)
dY XdY − YdX
⇒ − =  (x + 2) ⋅ 2 (x + 3) − (x + 3)2 ⋅ 1
Y X2 2
∴ g′ (x) = +  −0
Y  x+ 3  (x + 2)2 
⇒ − d (log|Y |) = d  
 X 2 (x + 3)(x + 1)
= +
On integrating both sides, we get x+3 (x + 2)2
Y Clearly, when x > 0, then, g′ (x) > 0
− log|Y| = + C, where x + 2 = X and y = Y
X
∴ g (x) is increasing, when x > 0.
y
⇒ − log| y| = +C …(i) Thus, when x > 0, then g (x) > g (0)
x+ 2
 3 9
g (x) > log   + > 0
Since, it passes through the point (1, 3).  e 4
∴ − log 3 = 1 + C Hence, there is no solution. Thus, option (d) is true.
⇒ C = − 1 − log 3 = − (log e + log 3)
9. Since, BP : AP = 3 : 1. Then, equation of tangent is
= − log 3e
Y − y = f ′ (x) (X − x)
∴ Eq. (i) becomes
The intercept on the coordinate axes are
y
log| y| + − log (3e) = 0  y 
x+2 A x − , 0
 f ′ (x) 
| y| y
⇒ log   + =0 …(ii)
 3e  x + 2 and B [0, y − x f ′ (x)]
Now, to check option (a), y = x + 2 intersects the curve. Since, P is internally intercepts a line AB,
|x + 2| x + 2 |x + 2|  y 
⇒ log  + = 0 ⇒ log   = −1 3 x −  + 1 ×0
 3e  x + 2  3e   f ′ (x)
∴ x=
|x + 2| 1 3+1
⇒ = e−1 =
3e e Y
⇒ |x + 2| = 3 or x + 2 = ± 3 B (1, 1)
3 (x, y)
∴ x = 1, − 5 (rejected), as x > 0 [given] P y = f (x)
∴ x = 1 only one solution.
1 X
Thus, (a) is the correct answer. A
To check option (c), we have dy y dy 1
⇒ = ⇒ =− dx
| y| y dx − 3x y 3x
y = (x + 2)2 and log   + =0
 3e  x + 2
On integrating both sides, we get
|x + 2|2  (x + 2 )2 |x + 2|2  xy3 = c
⇒ log   + = 0 ⇒ log   = − (x + 2)
 3e  x+2  3e  Since, curve passes through (1, 1), then c = 1.
(x + 2)2
3e ∴ xy3 = 1
⇒ = e−( x + 2) or (x + 2)2 ⋅ ex + 2 = 3e ⇒ex+ 2 =
3e (x + 2)2 At
1
x= ⇒ y=2
Y 8
e x +2
Hence, (a) and (c) are correct answers.

e2 10. Since, rate of change of volume ∝ surface area


dV
3e /4 ⇒ ∝ SA
3e /( x + 2)2 dt
X dr
O ⇒ 4 πr 2 ⋅ = − λ 4 πr 2
dt
Clearly, they have no solution.
dr
To check option (d), y = (x + 3)2 = − λ is required differential equation.
dt
Differential Equations 375

 dx
2 On substituting Eq. (iv) in Eq. (i), we get
11. Since, the length of tangent = y 1 +   =1 1 dr
 dy cot θ ⋅ 3r 2 = − kπ r 2
3 dt
  dx
2
0 T
⇒ y2 1 +    =1 ⇒ cot θ ∫ dr = − k ∫ dt
 dy  R 0
 
dy y ⇒ cot θ (0 − R) = − k (T − 0)
∴ =± ⇒ R cot θ = kT ⇒ H = kT [from Eq. (iii)]
dx 1 − y2
H
⇒ T=
1 − y2 k
⇒ ∫ y
dy = ± ∫ x dx
H
∴ Required time after which the cone is empty, T =
k
1− y 2
⇒ ∫ y
dy = ± x + C
13. Let O be the centre of hemispherical tank. Let at any
instant t, water level be BAB1 and at t + dt, water level
Put y = sin θ ⇒ dy = cos θ dθ is B′ A′ B1. Let ∠ O1OB1 = θ.
cos θ
∴ ∫ sin θ ⋅ cos θ dθ = ± x + C O
θ
O1

cos 2 θ A

B B1
⇒ ⋅ sin θ dθ = ± x + C B ′1
sin 2 θ B' A′
Again put cos θ = t ⇒ − sin θ dθ = dt
t2
∴ −∫ dt = ± x + C
1 − t2 O2

 1  ⇒ AB1 = r cos θ and OA = r sin θ decrease in the water


⇒ ∫ 1 −

 dt = ± x + C
1 − t 2 volume in time dt = π AB12 ⋅ d (OA )
[πr 2 is surface area of water level and d (OA ) is depth
1+ t
⇒ t − log =± x+C of water level]
1−t
= πr 2 ⋅ cos 2 θ ⋅ r cos θ dθ
1 + 1 − y2 = πr3 ⋅ cos3 θ dθ
⇒ 1 − y2 − log =±x+C
1 − 1 − y2 Also, h (t ) = O2A = r − r sin θ = r (1 − sin θ )

12. Given, liquid evaporates at a rate proportional to its Now, outflow rate Q = A ⋅ v (t ) = A ⋅ 0.6 2 gr (1 − sin θ )

surface area. Where, A is the area of the outlet.


dV Thus, volume flowing out in time dt.
⇒ ∝ −S …(i)
dt ⇒ Q dt = A ⋅ (0.6) ⋅ 2 gr ⋅ 1 − sin θ dt
1 We have, πr3 cos3 θ dθ = A (0.6) ⋅ 2 gr ⋅ 1 − sin θ dt
We know that, volume of cone = πr 2h
3
πr3 cos3 θ
R ⇒ ⋅ dθ = dt
A (0.6) 2 gr (1 − sin θ )
Let the time taken to empty the tank be T.
r π/ 2 πr3 cos3 θ
H Then, T =∫ ⋅ dθ
0 A (0.6) ⋅ 2 gr 1 − sin θ
h − πr3 π/ 2 1 − sin 2 θ (− cos θ )
=
A (0.6) 2 gr ∫0 1 − sin θ

Let t1 = 1 − sin θ
and surface area = πr 2
− cos θ
1 ⇒ dt1 = dθ
or V = πr 2h and S = πr 2 …(ii) 1 − sin θ
3
tan θ =
R r
= tan θ −2πr3 0
Where,
H
and
h
…(iii) ∴ T=
A (0.6) 2 gr ∫1 [1 − (1 − t12)2] dt1

From Eqs. (ii) and (iii), we get


−2πr3 0
1
V = πr3 cot θ and S = πr 2
…(iv)
⇒ T=
A (0.6) 2 gr ∫1 [1 − (1 + t14 − 2t12)] dt1
3
376 Differential Equations

−2πr3 0 Thus, the least integral values of the year n, when the
⇒ T=
A (0.6) 2 gr ∫1 [1 − 1 − t14 + 2t12] dt1
country becomes self-sufficient is the smallest integer
log 10 − log 9
greater than or equal to .
2πr3 0
log (1.04) − 0.03
⇒ T=
A (0.6) 2 gr ∫1 (t14 − 2t12) dt1
15. Equation of normal at point (x, y) is
0
2 πr  t15 2t13 
3 dx
⇒ T=  − 3  Y − y=− (X − x) …(i)
A (0.6) 2 gr  5 1 dy

2π ⋅ r5/ 2 Distance of perpendicular from the origin to Eq. (i)


 1 2
⇒ T= ⋅ 0 − −0 +
6  3  dx
A   2 gr
5 y+ ⋅x
 10 =
dy
2
2π ⋅ 25/ 2 (102) 5/ 2 2 1   dx
⇒ T= − 1+  
   dy
12 ⋅ ⋅ 2 ⋅ g 3 5 
3
5 Also, distance between P and X-axis is |y|.
2π × 105 ⋅ 4 ⋅ 5 10 − 3  dx
= y+ ⋅x
(12 × 3) g  15  dy
∴ = | y|
2π × 105 × 7 14π × 105  dx
2
= = unit 1+  
3⋅3⋅ g ⋅3 27 g  dy
14. Let X 0 be initial population of the country and Y 0 be its dx 2 dx   dx 
2
⇒ y2 + ⋅ x + 2xy = y2 1 +   
initial food production. Let the average consumption be dy dy   dy 

a unit. Therefore, food required initially aX 0. It is given
2
 90   dx dx
Y p = aX 0   = 0.9 aX 0 …(i) ⇒   (x − y ) + 2xy
2 2
=0
 100  dy dy
Let X be the population of the country in year t. dx  dx 2 
⇒  dy (x − y ) + 2xy = 0
2

Then,
dX
= Rate of change of population
dy  
dt dx dy y2 − x2
3 ⇒ =0 or =
= X = 0.03 X dy dx 2xy
100 dx
dX But =0
⇒ = 0.03 dt dy
X
⇒ x = c, where c is a constant.
dX
⇒ ∫ X = ∫ 0.03 dt Since, curve passes through (1, 1), we get the equation
of the curve as x = 1.
⇒ log X = 0.03 t + c dy y2 − x2
The equation = is a homogeneous equation.
⇒ X = A ⋅ e0. 03 t , where A = ec dx 2xy
At t = 0, X = X 0, thus X 0 = A dy dv
Put y = vx ⇒ =v+ x
∴ X = X 0 e0. 03 t dx dx
Let Y be the food production in year t. dv v2x2 − x2
v+ x =
 4 
t dx 2x2v
Then, Y = Y 0 1 +  = 0.9aX 0 (1.04)
t
dv v − 1
2
v2 − 1 − 2v2 v2 + 1
 100 ⇒x = −v= =−
dx 2v 2v 2v
Q Y 0 = 0.9 aX 0 [from Eq. (i)] −2 v dx
⇒ dv =
Food consumption in the year t is aX 0 e0. 03 t . v2 + 1 x
Again, Y − X ≥0 [given] ⇒ c1 − log (v2 + 1) = log|x|
⇒ 0.9 X 0 a (1.04)t > a X 0 e0. 03 t  y2 
t ⇒ log| x|(v2 + 1) = c1 ⇒ | x|  2 + 1 = ec1
(1.04) 1 10 x 
⇒ > = .
e0. 03 t 0.9 9 ⇒ x2 + y2 = ± ec1 x or x2 + y2 = ± ecx is passing through
Taking log on both sides, we get (1, 1).
t[log (1.04) − 0.03] ≥ log 10 − log 9 ∴ 1 + 1 = ± ec ⋅ 1
log 10 − log 9 ⇒ ± ec = 2
⇒ t≥
log (1.04) − 0.03 Hence, required curve is x2 + y2 = 2x .
Differential Equations 377

16.
dV
∝ V for each reservoir. Put tan t /2 = u
dt 1 2 du
dV dV A ⇒ sec2 t / 2 dt = du ⇒ dt =
∝ − VA ⇒ = − K 1V A 2 sec2 t / 2
dx dt 2 du 2 du
[K 1 is the proportional constant] ⇒ dt = ⇒ dt =
1 + tan 2 t / 2 1 + u2
V ′A dV A t
⇒ ∫V = − K 1 ∫ dt
I1 = ∫
2 (1 + u 2)du 2 du
A VA 0 ∴ =
(1 + u ) (5u + 6u + 5) 5
2 2 ∫ 6
V ′A u2 + u + 1
⇒ log = − K 1t ⇒ V A′ = V A ⋅ e−K1 t …(i) 5
VA 2 du
Similarly for B, VB′ = VB ⋅ e−K 2t …(ii)
=
5 ∫ 6 9 9
u2 + u + − +1
On dividing Eq. (i) by Eq. (ii), we get 5 25 25
V A′ V A − (K1 2 du
5∫ 
= ⋅e − K2) t = 2 2
VB′ VB 3  4
u +  +  
 5  5
It is given that at t = 0, V A = 2 VB and at
3
t = , V A′ = VB′
3 2 5  u + 3 / 5
= ⋅ tan −1  
2 2 5 4  4 /5 
3 − (K1 − K 2 )t 3
Thus, = 2⋅ e ⇒ e− (K1 − K 2 ) = …(iii) 1 5u + 3 
2 4 = tan −1
2  4 
Now, let at t = t0 both the reservoirs have some quantity
of water. Then, 1 5 tan t / 2 + 3 
V A′ = VB′ = tan −1
2  4 
From Eq. (iii), 2e−(K − K 2 ) = 1
t0
On putting this in Eq. (ii), we get
 3
⇒ 2⋅   =1  t 
 4 5 tan + 3
1 −1  2 
t0 = log3/ 4 (1 / 2) 4
tan 
4 =x+ c
 
17. Given,
dy
= sin (10x + 6 y)  
dx  t 
5 tan + 3
Let 10x + 6 y = t …(i)  
⇒ tan −1  2
 = 4x + 4c
dy  dt  4
⇒ 10 + 6 =   
dx  dx  
dy 1  dt  1
⇒ =  − 10 ⇒ [5 tan (5x + 3 y) + 3] = tan (4x + 4c)
dx 6  dx  4

Now, the given differential equation becomes ⇒ 5 tan (5x + 3 y) + 3 = 4 tan (4x + 4c)
1  dt  When x = 0, y = 0, we get
sin t =  − 10
6  dx  5 tan 0 + 3 = 4 tan (4c)
dt 3
⇒ 6 sin t = − 10 ⇒ = tan 4c
dx 4
3

dt
= 6 sin t + 10 ⇒ 4c = tan −1
dx 4
dt  3
⇒ = dx Then, 5 tan (5x + 3 y) + 3 = 4 tan 4x + tan −1 
6 sin t + 10  4
4  3 3
On integrating both sides, we get ⇒ tan (5x + 3 y) = tan 4x + tan  −
5  4 5
1 dt
2 ∫ 3 sin t + 5
=x+ c …(ii)
4   −1 3   3 
⇒ 5x + 3 y = tan −1  tan 4x + tan −
dt dt 5  4  5 
I1 = ∫
3 sin t + 5 ∫
Let =
 2 tan t / 2  4   −1 3   3
3  +5 ⇒ 3 y = tan −1  tan 4x + tan   −  − 5x
 1 + tan 2 t / 2 5  4   5
(1 + tan 2 t / 2) dt 1 4   3  3  5x
=∫ ⇒ y= tan −1  tan 4x + tan −1  − −
 t
6 tan + 5 + 5tan 
2 t 3 5   4  5  3
 2 2
378 Differential Equations
π /2
18. A. I = ∫
x
(sin x)cos {cos x ⋅ cot x − log (sin x)sin x } dx dy 3x −1
0 Hence, = + 3x −1 ⋅ log 3 ⋅ log x
dx x
π /2 d
=∫ (sin x)cos x dx = 1  dy
0 dx ∴   =1
 dx (1, 0)
B. The point of intersection of − 4 y2 = x and
x − 1 = − 5 y2 is (− 4, − 1) and (− 4, 1). For y = xx − 1
dy
Y ⇒ = xx (1 + log x)
dx
(– 4, 1)  dy
∴   =1
 dx (1, 0)
If θ is angle between the curves, then tan θ = 0.
X′ X
(1, 0) ⇒ θ = 0°
dy 2
(– 4, –1) D. =
dx x + y
dx x y
⇒ − =
dy 2 2
Y′
1
∴ Required area ⇒ xe− y/ 2 = ⋅ ∫ y ⋅ e− y/ 2dy
2
= 2 ∫ (1 − 5 y2) dy − ∫ − 4 y2 dy
1 1
1  ye− y 2 e− y/ 2 
 0  ⇒ xe− y/ 2 =  − +k
2  − 1 / 2 (1 / 2)2 
0

 3 1
1 y 4
= 2 ∫ (1 − y2)dy = 2  y −  = sq units ⇒ x + y + 2 = key/ 2
0
 3 0 3
It passing through (1, 0).
C. The point of intersection y = 3x −1 log x and y = xx − 1 ⇒ k =3
is (1, 0). ∴ x + y + 2 = 3e y/ 2
15
Straight Line and
Pair of Straight Lines

Topic 1 Various Forms of Straight Line


Objective Questions I (Only one correct option) 5. The region represented by|x − y| ≤ 2 and|x + y| ≤ 2 is
1. The set of all possible values of θ in the interval (0, π) for bounded by a (2019 Main, 10 April I)
which the points (1, 2) and(sin θ , cos θ) lie on the same (a) rhombus of side length 2 units
side of the line x + y = 1 is (2020 Main, 2 Sep II) (b) rhombus of area 8 2 sq units
π π 3π 
(a)  0,  (b)  ,  (c) square of side length 2 2 units
 2 4 4
(d) square of area 16 sq units
3π  π
(c)  0,  (d)  0, 
 4  4 6. If the two lines x + (a − 1) y = 1 and
2x + a 2y = 1, (a ∈ R − {0, 1}) are perpendicular, then the
2. A straight line L at a distance of 4 units from the origin distance of their point of intersection from the origin is
makes positive intercepts on the coordinate axes and the (2019 Main, 9 April II)
perpendicular from the origin to this line makes an angle 2 2
of 60° with the line x + y = 0. Then, an equation of the line (a) (b)
5 5
L is (2019 Main, 12 April II) 2 2
(a) x + 3 y = 8 (c) (d)
5 5
(b) ( 3 + 1) x + ( 3 − 1) y = 8 2
7. Slope of a line passing through P (2, 3) and
(c) 3x + y = 8
intersecting the line, x + y = 7 at a distance of 4 units
(d) ( 3 − 1)x + ( 3 + 1) y = 8 2 from P, is (2019 Main, 9 April I)
3. The equation y = sin x sin(x + 2) − sin 2(x + 1) represents a (a)
1− 5
(b)
7−1
straight line lying in (2019 Main, 12 April I) 1+ 5 7+1
(a) second and third quadrants only 1− 7 5−1
(c) (d)
(b) first, second and fourth quadrants 1+ 7 5+1
(c) first, third and fourth quadrants
8. If a point R(4, y, z ) lies on the line segment joining the
(d) third and fourth quadrants only
points P(2, − 3, 4) and Q(8, 0, 10), then the distance of R
4. Lines are drawn parallel to the line 4x − 3 y + 2 = 0, at a from the origin is (2019 Main, 8 April II)
3 (a) 2 21 (b) 53 (c) 2 14 (d) 6
distance from the origin. Then which one of the
5
9. Suppose that the points (h , k), (1, 2) and (−3, 4) lie on
following points lies on any of these lines?
(2019 Main, 10 April I)
the line L1. If a line L 2 passing through the points (h , k)
and (4, 3) is perpendicular to L1, then k /h equals
(a)  − , −  (b)  − , 
1 2 1 2 (2019 Main, 8 April II)
 4 3  4 3 (a) −
1
(b)
1
 1
(c)  , − 
1
(d)  , 
1 1 7 3
4 3  4 3 (c) 3 (d) 0
380 Straight Line and Pair of Straight Lines

10. A point on the straight line, 3x + 5 y = 15 which is 20. Let k be an integer such that the triangle with vertices
equidistant from the coordinate axes will lie only in (k, − 3k), (5, k) and (− k, 2) has area
(2019 Main, 8 April I) 28 sq units. Then, the orthocentre of this triangle is at
(a) IV quadrant (b) I quadrant the point (2017 Main)

(a)  2, −  (b)  1,  (c)  1, −  (d)  2, 


(c) I and II quadrants (d) I, II and IV quadrants 1 3 3 1
 2  4  4  2
11. If a straight line passing through the point P(− 3, 4) is
such that its intercepted portion between the 21. Let a , b, c and d be non-zero numbers. If the point of
coordinate axes is bisected at P, then its equation is intersection of the lines 4ax + 2ay + c = 0 and
(2019 Main, 12 Jan II) 5bx + 2by + d = 0 lies in the fourth quadrant and is
(a) x − y + 7 = 0 (b) 4x − 3 y + 24 = 0 equidistant from the two axes, then (2014 Main)
(c) 3x − 4 y + 25 = 0 (d) 4x + 3 y = 0 (a) 2bc − 3ad = 0 (b) 2bc + 3ad = 0
12. If the straight line, 2x − 3 y + 17 = 0 is perpendicular to (c) 2ad − 3bc = 0 (d) 3bc + 2ad = 0
the line passing through the points (7, 17) and (15, β), 22. If PS is the median of the triangle with vertices P(2, 2),
then β equals (2019 Main, 12 Jan I) Q(6, – 1) and R(7, 3), then equation of the line passing
35 35
(a) (b) − 5 (c) − (d) 5 through (1, – 1) and parallel to PS is (2014 Main, 2000)
3 3 (a) 4x − 7 y − 11 = 0 (b) 2x + 9 y + 7 = 0
13. If in a parallelogram ABDC, the coordinates of A , B and (c) 4x + 7 y + 3 = 0 (d) 2x − 9 y − 11 = 0
C are respectively (1, 2), (3, 4) and (2, 5), then the 23. The x-coordinate of the incentre of the triangle that has
equation of the diagonal AD is (2019 Main, 11 Jan II) the coordinates of mid-points of its sides as (0, 1), (1, 1)
(a) 3x + 5 y − 13 = 0 (b) 3x − 5 y + 7 = 0 and (1, 0) is (2013 Main)
(c) 5x − 3 y + 1 = 0 (d) 5x + 3 y − 11 = 0 (a) 2 + 2 (b) 2 − 2 (c) 1 + 2 (d) 1 − 2
2
14. The tangent to the curve, y = xex passing through 24. A straight line L through the point (3, −2) is inclined at
an angle 60° to the line 3x + y = 1. If L also intersects
the point (1, e) also passes through the point
(2019 Main, 10 Jan II)
the X-axis, then the equation of L is (2011)
(a) y + 3x + 2 − 3 3 = 0 (b) y − 3x + 2 + 3 3 = 0
(a)  , 2e (d)  , 2e
4 5
(b) (3, 6e) (c) (2, 3e)
3  3  (c) 3 y − x + 3 + 2 3 = 0 (d) 3 y + x − 3 + 2 3 = 0

15. Two sides of a parallelogram are along the lines, 25. The locus of the orthocentre of the triangle formed by
x + y = 3 and x − y + 3 = 0. If its diagonals intersect at (2, the lines (1 + p) x − py + p (1 + p) = 0 ,
4), then one of its vertex is (2019 Main, 10 Jan II) (1 + q) x − qy + q (1 + q) = 0 and y = 0, where p ≠ q , is (2009)
(a) (3, 6) (b) (2, 6) (a) a hyperbola (b) a parabola
(c) (2, 1) (d) (3, 5) (c) an ellipse (d) a straight line
26. Let O(0, 0), P(3, 4) and Q(6, 0) be the vertices of a ∆OPQ.
16. The shortest distance between the point  , 0 and the
3
2  The point R inside the ∆OPQ is such that the triangles
OPR, PQR and OQR are of equal area. The coordinates
curve y = x , (x > 0), is (2019 Main, 10 Jan I)
of R are (2007, 3M)
3 5 3 5
(a)  , 3 (b)  3,  (c)  3,  (d)  , 
(a) (b) (c) (d) 4 2 4 4 2
2 4 2 2
3   3  3  3 3
17. If the line 3x + 4 y − 24 = 0 intersects the X-axis at the 27. Orthocentre of triangle with vertices (0, 0), (3, 4) and
point A and theY -axis at the point B, then the incentre (4, 0) is (2003, 2M)
of the triangle OAB, where O is the origin, is
(a)  3,  (c)  3, 
5 3
(2019 Main, 10 Jan I) (b) (3, 12) (d) (3, 9)
 4  4
(a) (4, 3) (b) (3, 4)
(c) (4, 4) (d) (2, 2) 28. The number of integer values of m, for which the
18. A point P moves on the line 2x − 3 y + 4 = 0. If Q(1, 4) and x-coordinate of the point of intersection of the lines
3x + 4 y = 9 and y = mx + 1 is also an integer, is (2001, 1M)
R(3, − 2) are fixed points, then the locus of the centroid of
(a) 2 (b) 0 (c) 4 (d) 1
∆PQR is a line (2019 Main, 10 Jan I)
2 3 29. A straight line through the origin O meets the parallel
(a) with slope (b) with slope
3 2 lines 4x + 2 y = 9 and 2x + y + 6 = 0 at points P and Q
respectively. Then, the point O divides the segment PQ
(c) parallel to Y -axis (d) parallel to X-axis
in the ratio (2000, 1M)
19. A straight line through a fixed point (2, 3) intersects the (a) 1 : 2 (b) 3 : 4 (c) 2 : 1 (d) 4 : 3
coordinate axes at distinct points P and Q. If O is the
origin and the rectangle OPRQ is completed, then the 30. The incentre of the triangle with vertices (1, 3 ), (0, 0)
locus of R is (2018 Main) and (2, 0) is (2000, 2M)
(a) 3x + 2 y = 6 (b) 2x + 3 y = xy  3  2 3
(b)  ,
2 1 
(d)  1,
1 
(a)  1,   (c)  ,  
(c) 3x + 2 y = xy (d) 3x + 2 y = 6xy  2   3 3 3 2   3
Straight Line and Pair of Straight Lines 381

31. If A0 , A1 , A2, A3 , A4 and A5 be a regular hexagon II. Transformation through a distance 2 units along
inscribed in a circle of unit radius. Then, the product of the positive direction of X-axis.
the lengths of the line segments A0 A1 , A0 A2 and A0 A4 is π
III. Rotation through an angle about the origin in the
(1998, 2M) 4
3 3 counter clockwise direction.
(a) 3 / 4 (b) 3 3 (c) 3 (d)
2 Then, the final position of the point is given by the
32. If the vertices P , Q , R of a ∆PQR are rational points, coordinates (1980, 1M)
which of the following points of the ∆PQR is/are always
(a) 
1 7 
,  (b) (− 2, 7 2)
rational point(s) (1998, 2M)  2 2
(c)  −
(a) centroid (b) incentre 1 7 
,  (d) ( 2, 7 2)
(c) circumcentre (d) orthocentre  2 2
(A rational point is a point both of whose coordinates are
rational numbers) 41. The points (−a , − b), (0, 0), (a , b) and (a 2, a3 ) are
(a) collinear (1979, 2M)
33. If P (1, 2), Q (4, 6), R (5, 7) and S (a , b) are the vertices of
(b) vertices of a rectangle
a parallelogram PQRS, then (1998, 2M)
(c) vertices of a parallelogram
(a) a = 2, b = 4 (b) a = 3, b = 4 (d) None of the above
(c) a = 2, b = 3 (d) a = 3, b = 5
34. The diagonals of a parallelogram PQRS are along the Objective Questions II
lines x + 3 y = 4 and 6x − 2 y = 7. Then, PQRS must be a (One or more than one correct option)
(a) rectangle (b) square (1998, 2M)
(c) cyclic quadrilateral (d) rhombus 42. Let a, λ, µ ∈R. Consider the system of linear equations
35. The graph of the function cos x cos (x + 2) − cos 2 (x + 1) ax + 2 y = λ and 3x − 2 y = µ.
is (1997, 2M) Which of the following statement(s) is/are correct?
(2016 Adv.)
(a) a straight line passing through (0, − sin 1) with slope 2
2
(a) If a = − 3, then the system has infinitely many
(b) a straight line passing through (0, 0)
solutions for all values of λ and µ
(c) a parabola with vertex (1, − sin 2 1)
π (b) If a ≠ − 3, then the system has a unique solution for all
(d) a straight line passing through the point  , − sin 2 1 values of λ and µ
2 
and parallel to the X-axis (c) If λ + µ = 0, then the system has infinitely many
36. The orthocentre of the triangle formed by the lines solutions for a = − 3
xy = 0 and x + y = 1,is (1995, 2M) (d) If λ + µ ≠ 0, then the system has no solution for a = − 3

(a)  ,
1
(b)  ,
1
(d)  ,  43. For a > b > c > 0, the distance between (1, 1) and the
1 1 1 1
  (c) (0, 0)
2 2 3 3  4 4 point of intersection of the lines ax + by + c = 0 and
37. If the sum of the distance of a point from two bx + ay + c = 0 is less than 2 2. Then, (2014 Adv.)
perpendicular lines in a plane is 1, then its locus is (a) a + b − c > 0 (b) a − b + c < 0
(a) square (b) circle (1992, 2M) (c) a − b + c > 0 (d) a + b − c < 0
(c) straight line (d) two intersecting lines
44. All points lying inside the triangle formed by the points
38. Line L has intercepts a and b on the coordinate axes. (1, 3), (5, 0) and (– 1, 2) satisfy (1986, 2M)
When, the axes are rotated through a given angle, (a) 3x + 2 y ≥ 0 (b) 2x + y − 13 ≥ 0
keeping the origin fixed, the same line L has intercepts (c) 2x − 3 y − 12 ≤ 0 (d) −2x + y ≥ 0
p and q, then (1990, 2M)
1 1 1 1
(a) a 2 + b2 = p 2 + q2 (b) + = + Fill in the Blanks
a2 b2 p2 q2
1 1 1 1 45. Let the algebraic sum of the perpendicular distance
(c) a 2 + p 2 = b2 + q2 (d) + = +
a 2
p 2
b2
q2 from the points (2, 0) , (0, 2) and (1, 1) to a variable
straight line be zero, then the line passes through a
39. If P = (1, 0), Q = (−1, 0) and R = (2, 0) are three given fixed point whose coordinates are… . (1991, 2M)
points, then locus of the points satisfying the relation
46. The orthocentre of the triangle formed by the lines
SQ 2 + SR2 = 2 SP 2, is (1988, 2M)
x + y = 1, 2 x + 3 y = 6 and 4x − y + 4 = 0 lies in quadrant
(a) a straight line parallel to X-axis number… . (1985, 2M)
(b) a circle passing through the origin
(c) a circle with the centre at the origin 47. If a, b and c are in AP, then the straight line
(d) a straight line parallel to Y-axis a x + by + c = 0 will always pass through a fixed point
whose coordinates are (…) . (1984, 2M)
40. The point (4, 1) undergoes the following three
transformations successively 48. y = 10x is the reflection of y = log10 x in the line whose
I. Reflection about the line y = x. equation is .... . (1984, 2M)
382 Straight Line and Pair of Straight Lines

True/False 61. Let ABC be a triangle with AB = AC. If D is mid point of


BC, the foot of the perpendicular drawn from D to AC
49. The lines 2x + 3 y + 19 = 0 and 9x + 6 y − 17 = 0 cut the
and F the mid-point of DE. Prove that AF is
coordinate axes in concyclic points. (1988, 1M)
perpendicular to BE. (1989, 5M)
50. No tangent can be drawn from the point (5/2, 1) to the
circumcircle of the triangle with vertices (1, 3 ), 62. The equations of the perpendicular bisectors of the
(1, − 3 ) and (3, 3 ). (1985, 1M) sides AB and AC of a ∆ABC are x − y + 5 = 0 and
x + 2 y = 0, respectively. If the point A is (1, – 2), find the
51. The straight line 5x + 4 y = 0 passes through the point of equation of the line BC. (1986, 5M)
intersection of the straight lines x + 2 y − 10 = 0 and
2x + y + 5 = 0. (1983, 1M) 63. One of the diameters of the circle circumscribing the
rectangle ABCD 4 y = x + 7. If A and B are the points
Analytical & Descriptive Questions (−3, 4) and (5, 4) respectively, then find the area of
rectangle. (1985, 3M)
52. A straight line L through the origin meets the line
x + y = 1 and x + y = 3 at P and Q respectively. Through 64. Two sides of a rhombus ABCD are parallel to the lines
P and Q two straight lines L1 and L 2 are drawn, parallel y = x + 2 and y = 7x + 3. If the diagonals of the rhombus
to 2x − y = 5 and 3x + y = 5, respectively. Lines L1 and L 2 intersect at the point (1, 2) and the vertex A is on the
intersect at R, show that the locus of R as L varies, is a Y-axis, find possible coordinates of A. (1985, 5M)
straight line. (2002, 5M) 65. Two equal sides of an isosceles triangle are given by the
53. A straight line L with negative slope passes through the equations 7x − y + 3 = 0 and x + y − 3 = 0 and its third
point (8, 2) and cuts the positive coordinate axes at side passes through the point (1, −10). Determine the
points P and Q. Find the absolute minimum value of equation of the third side. (1984, 4M)
OP + OQ, as L varies, where O is the origin. (2002, 5M) 66. The vertices of a triangle are
54. For points P = (x1 , y1 ) and Q = (x2, y2) of the coordinate [at1t2, a (t1 + t2)], [at2 t3 , a (t2 + t3 )], [at3 t1 , a (t3 + t1 )].
plane, a new distance d (P , Q ) is defined by Find the orthocentre of the triangle. (1983, 3M)
d (P , Q ) = | x1 − x2| + | y1 − y2|.
67. The ends A , B of a straight line segment of constant
Let O = (0, 0) and A = (3, 2). Prove that the set of points
in the first quadrant which are equidistant (with length c slide upon the fixed rectangular axes OX , OY
respect to the new distance) from O and A consists of the respectively. If the rectangle OAPB be completed, then
union of a line segment of finite length and an infinite show that the locus of the foot of the perpendicular
ray. Sketch this set in a labelled diagram. (2000, 10M) drawn from P to AB is
x2 / 3 + y2 / 3 = c2 / 3 (1983, 2M)
55. A rectangle PQRS has its side PQ parallel to the line
y = mx and vertices PQ and S on the lines y = a , x = b 68. The points (1, 3) and (5, 1) are two opposite vertices of a
and x = − b, respectively. Find the locus of the vertex R. rectangle. The other two vertices lie on the line
(1996, 2M) y = 2x + c. Find c and the remaining vertices. (1981, 4M)
56. A line through A (−5, − 4) meets the line x + 3 y + 2 = 0, 69. Two vertices of a triangle are (5, − 1) and (−2, 3). If the
2x + y + 4 = 0 and x − y − 5 = 0 at the points B, C and D orthocentre of the triangle is the origin, find the
respectively. If (15 / AB)2 + (10 / AC )2 = (6 / AD )2, find coordinates of the third vertex. (1978, 3M)
the equation of the line. (1993, 5M)
70. One side of a rectangle lies along the line 4x + 7 y + 5 = 0.
57. Determine all values of α for which the point (α , α 2) lies Two of its vertices are (−3, 1) and (1, 1). Find the
inside the triangles formed by the lines 2x + 3 y − 1 = 0, equations of the other three sides. (1978, 3M)

x + 2 y − 3 = 0, 5x − 6 y − 1 = 0 (1992, 6M)
Integer & Numerical Answer Type Questions
71. Let A(1, 0), B(6, 2) and C  , 6 be the vertices of a
58. Find the equations of the line passing through the point 3
(2, 3) and making intercept of length 3 unit between the 2 
lines y + 2x = 2 and y + 2x = 5. (1991, 4M) triangle ABC. If P is a point inside the triangle ABC
59. Straight lines 3x + 4 y = 5 and 4x − 3 y = 15 intersect at such that the triangles APC, APB and BPC have equal
the point A. Points B and C are chosen on these two areas, then the length of the line segment PQ, where Q
 7 1
lines such that AB = AC. Determine the possible is the point  − , −  , is (2020 Main, 7 Jan I)
 6 3
equations of the line BC passing through the point
(1, 2). (1990, 4M) 72. For a point P in the plane, let d1 (P ) and d2(P ) be the
60. A line cuts the X-axis at A (7, 0) and the Y-axis at distances of the point P from the lines x − y = 0 and
x + y = 0, respectively. The area of the region R
B (0, − 5). A variable line PQ is drawn perpendicular to
consisting of all points P lying in the first quadrant of
AB cutting the X-axis in P and the Y-axis in Q. If AQ the plane and satisfying 2 ≤ d1 (P ) + d2(P ) ≤ 4, is
and BP inters at R, find the locus of R. (1990, 4M) (2014 Adv.)
Topic 2 Angle between Straight Lines and Equation of
Angle Bisector
Objective Questions I (Only one correct option) Assertion and Reason
1. A ray of light coming from the point (2, 2 3 ) is incident For the following questions choose the correct answer
at an angle 30° on the line x = 1 at the point A. The ray from the codes (a), (b), (c) and (d) defined as follows.
gets reflected on the line x = 1 and meets X-axis at the (a) Statement I is true, Statement II is also true;
point B. Then, the line AB passes through the point Statement II is the correct explanation of Statement I
(2020 Main, 6 Sep I) (b) Statement I is true, Statement II is also true;
(a)  3, −
1  Statement II is not the correct explanation of

 3 Statement I
 3 (c) Statement I is true; Statement II is false
(b)  4, −  (d) Statement I is false; Statement II is true
 2 
5. Lines L1 : y − x = 0 and L 2 : 2x + y = 0 intersect the line
(c) (3, − 3 )
(d) (4, − 3 ) L3 : y + 2 = 0 at P and Q, respectively. The bisector of the
acute angle between L1 and L 2 intersects L3 at R.
Objective Questions II Statement I The ratio PR : RQ equals 2 2 : 5 .
(One or more than one correct option) Because
Statement II In any triangle, bisector of an angle
2. A ray of light along x + 3 y = 3 gets reflected upon
divides the triangle into two similar triangles. (2007, 3M)
reaching X-axis, the equation of the reflected ray is
(2013 Main)
(a) y = x + 3 (b) 3 y = x − 3 Fill in the Blank
(c) y = 3x − 3 (d) 3 y = x − 1 6. The vertices of a triangle are A (− 1, − 7), B(5, 1) and
3. Consider three points C (1, 4). The equation of the bisector of the angle ABC
P = {– sin ( β – α ) – cos β}, Q = {cos( β – α ),sin β} is… . (1993, 2M)
and R = {cos ( β – α + θ ) sin ( β – θ )},
π Analytical & Descriptive Questions
where 0 < α , β , θ < . Then,
4 (2008, 4M) 7. The area of the triangle formed by the intersection of
(a) P lies on the line segment RQ line parallel to X-axis and passing through (h, k) with
(b) Q lies on the line segment PR the lines y = x and x + y = 2 is 4h 2. Find the locus of point
(c) R lies on the line segment QP P. (2005)
(d) P, Q, R are non-collinear 8. Find the equation of the line which bisects the obtuse
4. Let P = (–1, 0), and Q (0, 0) and R = (3, 3 3 ) be three angle between the lines x − 2 y + 4 = 0 and 4x − 3 y + 2 = 0.
(1993, 2M)
point. Then, the equation of the bisector of the angle
PQR is (2001, 1M) 9. Lines L1 ≡ ax + by + c = 0 and L 2 ≡ lx + my + n = 0
(a)
3
x+ y = 0 (b) x + 3y = 0
intersect at the point P and makes an angle θ with each
2 other. Find the equation of a line L different from L 2
3 which passes through P and makes the same angle θ
(c) 3 x + y = 0 (d) x + y=0
2 with L1. (1988, 5M)

Topic 3 Area and Family of Concurrent Lines


Objective Questions I (Only one correct option) 2. Consider the set of all lines px + qy + r = 0 such that
1. A triangle has a vertex at (1, 2) and the mid-points of 3 p + 2q + 4 r = 0. Which one of the following statements
the two sides through it are (−1, 1) and (2, 3). Then, the is true? (2019 Main, 9 Jan I)
centroid of this triangle is (2019 Main, 12 April II) (a) Each line passes through the origin.
 3 1
(a)  1,  (b)  , 2
7 1
(b) The lines are concurrent at the point  , 
 3 3   4 2
(c) The lines are all parallel
(c)  , 1 (d)  , 
1 1 5
3   3 3 (d) The lines are not concurrent
384 Straight Line and Pair of Straight Lines

3. Two sides of a rhombus are along the lines, x − y + 1 = 0 Match the Columns
and 7x − y − 5 = 0. If its diagonals intersect at (− 1, − 2),
Match the conditions/expressions in Column I with
then which one of the following is a vertex of this
statement in Column II.
rhombus? (2016 Main)
(a) (− 3, − 9) (b) (− 3, − 8) 9. Consider the lines given by
L1 : x + 3 y − 5 = 0, L 2 : 3x − ky − 1 = 0,
(c)  , −  (d)  − , − 
1 8 10 7
3 3  3 3 L3 : 5x + 2 y − 12 = 0

4. Area of the parallelogram formed by the lines Column I Column II


y = mx, y = mx + 1, y = nx and y = nx + 1 equals (A) L1, L2, L3 are concurrent, if (p) k= −9
(2001, 1M)
|m + n| 2 (B) One of L1, L2, L3 is parallel to k=−
6
(a) (b) (q)
(m − n ) 2
|m + n| at least one of the other two, if 5
(C) L1, L2, L3 form a triangle, if 5
(c)
1
(d)
1 (r) k=
|m + n| |m − n| 6
(D) L1, L2, L3 do not form a (s) k=5
5. The points 0,  , (1, 3) and (82, 30) are vertices of
8
triangle, if
 3 (1986, 2M)
(a) an obtuse angled triangle Fill in the Blank
(b) an acute angled triangle
10. The set of lines ax + by + c = 0, where 3a + 2b + 4c = 0 is
(c) a right angled triangle
(d) None of the above
concurrent at the point… . (1982, 2M)

6. The straight lines x + y = 0, 3x + y − 4 = 0, x + 3 y − 4 = 0


True/False
form a triangle which is (1983, 1M)
x1 y1 1 a1 b1 1
(a) isosceles
(b) equilateral 11. If x2 y2 1 = a 2 b2 1 ,then the two triangles
(c) right angled x3 y3 1 a3 b3 1
(d) None of the above with vertices (x1 , y1 ), (x2, y2), (x3 , y3 ) and (a1 , b1 ), (a 2, b2),
7. Given the four lines with the equations (a3 , b3 ) must be congruent. (1985, 1M)
x + 2 y − 3 = 0, 3x + 4 y − 7 = 0,
2x + 3 y − 4 = 0,4x + 5 y − 6 = 0, then (1980, 1M) Analytical & Descriptive Questions
(a) they are all concurrent
12. Using coordinate geometry, prove that the three
(b) they are the sides of a quadrilateral altitudes of any triangle are concurrent. (1998, 8M)
(c) only three lines are concurrent
(d) None of the above 13. The coordinates of A , B, C are (6, 3), (−3, 5), (4, − 2)
respectively and P is any point (x, y). Show that the
Objective Question II ratio of the areas of the triangles ∆ PBC and ∆ ABC is
x+ y−2
(One or more than one correct option) . (1983, 2M)
7
8. Three lines px + qy + r = 0, qx + ry + p = 0 and
14. A straight line L is perpendicular to the line in
rx + py + q = 0 are concurrent, if (1985, 2M)
5x − y = 1. The area of the triangle formed by the line L
(a) p + q + r = 0 (b) p 2 + q2 + r 2 = pr + rq and the coordinate axes is 5. Find the equation of the
(c) p3 + q3 + r3 = 3 pqr (d) None of these line L.
(1980, 3M)

Topic 4 Homogeneous Equation of Pair of Straight Lines


Objective Questions I (Only one correct option)
1. Let a and b be non-zero and real numbers. Then, the equation (ax2 + by2 + c) (x2 − 5xy + 6 y2) = 0 represents (2008, 3M)
(a) four straight lines, when c = 0 and a , b are of the same sign
(b) two straight lines and a circle, when a = b and c is of sign opposite to that of a
(c) two straight lines and a hyperbola, when a and b are of the same sign and c is of sign opposite to that of a
(d) a circle and an ellipse, when a and b are of the same sign and c is of sign opposite to that of a
Straight Line and Pair of Straight Lines 385

2. Area of triangle formed by the lines x + y = 3 and angle Analytical & Descriptive Question
bisectors of the pair of straight lines x2 − y2 + 2 y = 1 is
(2004, 2M) 3. Show that all chords of curve 3x2 − y2 − 2x + 4 y = 0,which
(a) 2 sq units (b) 4 sq units subtend a right angle at the origin pass through a fixed
(c) 6 sq units (d) 8 sq units point. Find the coordinates of the point. (1991, 4M)

Topic 5 General Equation of Pair of Straight Lines


Objective Question I (Only one correct option)
1. Let PQR be a right angled isosceles triangle, right (a) 3x2 − 3 y2 + 8xy + 20x + 10 y + 25 = 0
angled at P (2, 1). If the equation of the line QR is (b) 3x2 − 3 y2 + 8xy − 20x − 10 y + 25 = 0
2x + y = 3, then the equation representing the pair of (c) 3x2 − 3 y2 + 8xy + 10 x + 15 y + 20 = 0
lines PQ and PR is (1999, 2M) (d) 3x2 − 3 y2 − 8xy − 10 x − 15 y − 20 = 0

Answers
Topic 1 66. [( −a, a (t1 + t 2 + t 3 + t1t 2t 3 )] 68. c = − 4,( 4, 4 ),(2, 0 )
1. (a) 2. (d) 3. (d) 4. (b) 69. ( − 4, − 7 )
5. (c) 6. (d) 7. (c) 8. (c) 70. 7 x − 4y + 25 = 0, 4 x + 7y = 11 = 0,7 x − 4y − 3 = 0
9. (c) 10. (c) 11. (b) 12. (d)
71. (5)
13. (c) 14. (a) 15. (a) 16. (d)
72. 6 sq units
17. (d) 18. (a) 19. (c) 20. (d)
21. (c) 22. (b) 23. (b) 24. (b) Topic 2
25. (d) 26. (c) 27. (c) 28. (a) 1. (c) 2. (b) 3. (d) 4. (c)
29. (b) 30. (d) 31. (c) 32. (a) 5. (c) 6. 7y = x + 2 7. 2 x = ± (y − 1 )
33. (c) 34. (d) 35. (d) 36. (c) 8. ( 4 + 5 ) x − (2 5 + 3 ) y + ( 4 5 + 2 ) = 0
37. (a) 38. (b) 39. (d) 40. (c) 9. 2 (al + bm )(ax + by + c ) − (a 2 + b 2 ) (lx + my + n ) = 0
41. (a) 42. (b, c, d) 43. (a, c) 44. (a, c)
45. (1, 1) 46. Ist 47. (1, − 2 ) 48. (y = x ) Topic 3
1. (b) 2. (b) 3. (c) 4. (d)
49. True 50. True 51. True
5. (d) 6. (a) 7. (c) 8. (a, c)
53. (OP + OQ = 18)
9. A → s; B → p, q; C → r; D → p, q, s
55. (m 2 − 1 ) x − my + b (m 2 + 1 ) + am = 0
 3 1
10.  ,  11. False
56. 2 x + 3y + 22 = 0
3 1
57. − < α < −1 ∪ < α < 1  4 2
2 2
14. x + 5y = ± 5 2
58. x = 2 and 3 x + 4y = 18
59. x − 7y + 13 = 0 and 7 x + y − 9 = 0 Topic 4
60. x + y − 7 x + 5y = 0
2 2
62. 14 x + 23y − 40 = 0 1. (b) 2. (a)
3. (1, − 2 )
 5
63. 32 sq units 64.  0,  or ( 0, 0 )
 2
Topic 5
65. x − 3y − 31 = 0 or 3 x + y + 7 = 0 1. (b)
Hints & Solutions
Topic 1 Various Forms of Straight Line and as we know that y < 0, is in third and fourth
quadrants only.
1. According to the question, as points (1, 2) and
(sin θ , cos θ ), θ ∈ (0 , π ), lie on the same side of the line 4. Since, equation of a line parallel to line ax + by + c = 0 is
x + y = 1, so (1 + 2 − 1) (sin θ + cos θ − 1) > 0 ax + by + k = 0
⇒ sin θ + cos θ > 1 ∴ Equation of line parallel to line
1 1 1 π  1 4x − 3 y + 2 = 0 is 4x − 3 y + k = 0 …(i)
⇒ sin θ + cos θ > ⇒ sin  + θ >
2 2 2 4  2 Now, distance of line (i) from the origin is
π π 3π | k| 3
⇒ 2 nπ + < + θ < 2 nπ + , n ∈ I. =
4 4 4 4 +3
2 2 5
π
⇒ 2 nπ < θ < 2 nπ + [as per question’s requirement]
2
 π ⇒ | k| = 3
As θ ∈ (0, π ), So for n = 0, θ ∈ 0,  ⇒ k=±3
 2
So, possible lines having equation, either 4x − 3 y + 3 = 0
Hence, option (a) is correct.
or 4x − 3 y − 3 = 0
2. According to the question, we have the following figure.  1 2
Now, from the given options the point  − ,  lies on
 4 3
(0,b) the line 4x − 3 y + 3 = 0.
M
5. The given inequalities are
60°
(a,0) |x − y| ≤ 2 and|x + y| ≤ 2 .
α
O On drawing, the above inequalities, we get a square
x+
y= x y Y
0 — + — =1
a b
(0, 2)
Let θ be the inclination of the line x + y = 0. Then,
tan θ = − 1 = tan (180° − 45° )
⇒ tan θ = tan 135° ⇒ θ = 135° X′
(–2, 0) (2, 0)
X
⇒ α + 60° = 135° ⇒ α = 75° O

Since, line L having perpendicular distance OM = 4.


So, equation of the line ‘L’ is (0, –2)
x cos α + y sin α = 4
Y′
⇒ x cos 75° + y sin 75° = 4
⇒ x cos (45° + 30° ) + y sin (45° + 30° ) = 4 Now, the area of shaded region is equal to the area of a
 3 1   3 1  square having side length (2 − 0)2 + (0 − 2)2 = 2 2
⇒ x − + y + =4
2 2 2 2  2 2 2 2  units.
⇒ ( 3 − 1) x + y ( 3 + 1) = 8 2 6.
Key Idea
(i) If lines are perpendicular to each other, then
Key Idea Use formulae : product of their slopes is −1, i.e. m1m2 = − 1
3. 2sin A sin B = cos( A − B) − cos( A + B) and cos 2 θ = 1 − 2 sin2 θ (ii) Distance between two points ( x 1, y 1) and (x 2, y 2)
= (x 2 − x 1) 2 + (y 2 − y 1) 2
Given equation is y = sin x sin(x + 2) − sin 2(x + 1)
1 1
= [cos 2 − cos(2x + 2)] − [1 − cos(2x + 2)] Given, lines x + (a − 1) y = 1
2 2 and 2x + a 2y = 1, where a ∈ R − {0, 1}
[Q2 sin A sin B = cos( A − B) − cos( A + B) are perpendicular to each other
and cos 2 θ = 1 − 2 sin 2 θ ⇒ 2 sin 2 θ = 1 − cos 2 θ]  1   2
∴ −  × −  = −1
1 1 1 1  a − 1  a 2
= cos 2 − cos(2x + 2) − + cos(2x + 2)
2 2 2 2 [Q If lines are perpendicular, then
1 1 product of their slopes is −1]
= (cos(2) − 1) = − (2 sin 2(1))
2 2 ⇒ a 2(a − 1) = − 2 ⇒ a3 − a 2 + 2 = 0
= − sin (1) < 0 ⇒ y < 0
2 ⇒ (a + 1)(a 2 − 2a + 2) = 0 ⇒ a = − 1
Straight Line and Pair of Straight Lines 387

∴ Equation of lines are x − x1 y − y1 z − z1 


= =
x − 2y = 1 …(i) x2 − x1 y2 − y1 z2 − z1 
and 2x + y = 1 …(ii) x − 8 y z − 10
On solving Eq. (i) and Eq. (ii), we get ⇒ = = …(i)
2 1 2
3 1
x = and y = − Q Points P , Q and R are collinear, so
5 5
4 − 8 y z − 10
∴ Point of intersection of the lines (i) and (ii) is = =
3 1 2 1 2
P , −  . z − 10
5 5 ⇒ −2 = y =
3 1 2
Now, required distance of the point P  , −  from ⇒ y = − 2 and z = 6
5 5
9 1 10 2 So, point R is (4, − 2, 6), therefore the distance of point R
origin = + = = from origin is
25 25 25 5
OR = 16 + 4 + 36 = 56 = 2 14
7. Let the slope of line is m, which is passing through
P(2, 3). 9. Given, points (1, 2), (−3, 4) and (h , k) are lies on line L1,
Y so slope of line L1 is
4 −2 k −2
8 m1 = =
−3 − 1 h − 1
7 −1 k − 2
⇒ m1 = = …(i)
6 2 h −1
5 ⇒ 2(k − 2) = − 1(h − 1)
4 R ⇒ 2k − 4 = − h + 1
d
3 P(2,
3
⇒ h + 2k = 5 …(ii)
)
2 θ and slope of line L 2 joining points (h , k) and
4 Q
1 3−k
(4, 3), is m2 = …(iii)
X′ X 4−h
1 2 3 4 5 6 7 8 9
Since, lines L1 and L 2 are perpendicular to each other.
Y′ x+y=7 ∴ m1m2 = − 1
Since, the distance of a point (x1 , y1 ) from the line   1  3 − k 
⇒ −    = − 1[from Eqs. (i) and (iii)]
ax + by1 + c  2  4 − h 
ax + by + c = 0 is d = 1 .
a 2 + b2 ⇒ 3 − k = 8 − 2h
∴The distance of a point P(2, 3) from the line ⇒ 2h − k = 5 …(iv)
x + y − 7 = 0, is
On solving Eqs. (ii) and (iv), we get
|2 + 3 − 7| 2
d= = = 2 (h , k) = (3, 1)
1+1 2
k 3
Now, in ∆PRQ, So, = =3
h 1
QR = 16 − d = 16 − 2 = 14 2
10. Given equation of line is 3x + 5 y = 15 …(i)
d 2 1 m+1  m2 − m1 
∴ tan θ = = = = Q tan θ = 1 + m m  Clearly, a point on the line (i), which is equidistance
QR 14 7 1 −m  1 2 from X and Y -axes will lie on the line either y = x or
m+1 1 y = − x.
⇒ =±
1 −m 7 Y
m+1 1 m+1 1
⇒ = or =− B y=x
1 −m 7 1 −m 7
1− 7 −1 − 7 (0, 3)
⇒ m= or m =
1+ 7 7 −1 y=–x A

8. Given points are P(2, − 3, 4), Q(8, 0, 10) and R(4, y, z ). (5, 0)
X
Now, equation of line passing through points P and Q 0
3x+5y=15
x − 8 y − 0 z − 10
is = =
6 3 6 In the above figure, points A and B are
[Since equation of a line passing through two points on the line (i) and are equidistance from the coordinate
A (x1 , y1 , z1 ) and B(x2, y2, z2) is given by axes.
388 Straight Line and Pair of Straight Lines

 15 15 14. Given equation of curve is y = xex


2
On solving line (i) and y = x, we get A  , . …(i)
8 8
Note that (1, e) lie on the curve, so the point of contact
Similarly, on solving line (i) and y = − x, we get is (1, e).
 15 15 Now, slope of tangent, at point (1, e), to the curve (i) is
B − , .
 2 2
=  x(2x) ex + ex 
dy 2 2
= 2e + e = 3e
So, the required points lie only in I and II quadrants. dx (1 , e ) (1 , e )

11. Let the equation of required line having intercepts a Now, equation of tangent is given by
and b with the axes is ( y − y1 ) = m (x − x1 )
x y
+ =1 …(i) y − e = 3e(x − 1) ⇒ y = 3ex − 2e
a b
Y 4 
On checking all the options, the option  , 2e satisfy
B (0,b) 3 
the equation of tangent.

b P (–3,4) 15. According to given information, we have the following


figure

A (a,0) D C (x2 , y2 )
X
O a M
x–y+3=0 (2,4)
Now, according to given information,
P is the mid-point of AB B
 a b A x+y=3
∴ P =  ,  = (−3, 4) [given]
 2 2 [Note that given lines are perpendicular to each other
as m1 × m2 = −1]
⇒ (a , b) = (−6, 8)
Clearly, point A is point of intersection of lines
On putting the value of a and b in Eq. (i), we get
x y x+ y =3 …(i)
+ = 1 ⇒ 8x − 6 y = −48
−6 8 and x − y = −3 …(ii)
⇒ 4x − 3 y + 24 = 0 So, A = (0, 3[solving
) Eqs. (i) and (ii)]
Now, as point M (2, 4) is mid-point of line joining the
12. Slope of the line 2x − 3 y + 17 = 0 is
2 points A and C, so
= m1, (let) and the slope of line joining the points (7, 17)  0 + x2 3 + y2
3 (2, 4) =  , 
β − 17 β − 17  2 2 
and (15, β ) is = = m2 (let)
15 − 7 8   x1 + x2 y1 + y2 
According to the question, m1m2 = − 1 Qmid-point =  2 , 2  
 
2 β − 17 0 + x2 3 + y2
⇒ × = − 1 ⇒ β − 17 = − 12 ⇒ β = 5. ⇒ 2= ;4 =
3 8 2 2
13. According to given information, we have the following ⇒ x2 = 4 and y2 = 5
figure ∴Thus, C ≡ (4, 5)
C(2, 5) D
Now, equation of line BC is given by
P ( y − y1 ) = m (x − x1 )
y − 5 = 1(x − 4)
A(1, 2) B(3, 4) [line BC is parallel to x − y + 3 = 0 and slope
(−1)
of x − y + 3 = 0 is = 1]
We know that, diagonals of a parallelogram intersect (−1)
at mid-point.
⇒ y = x+1 …(iii)
 5 9
∴ P = Mid-point of BC and so, P ≡  ,  and equation of line DC is
 2 2
y − 5 = −1 (x − 4)
Now, equation of AD is.
9 [line DC is parallel to x + y = 3 and
−2 −1
5 slope of x + y = 3 is = −1]
( y − 2) = 2 (x − 1) ⇒ y − 2 = (x − 1) 1
5 3
−1
2 ⇒ x+ y =9 …(iv)
⇒ 3 y − 6 = 5x − 5 On solving Eqs. (i) and (iii), we get B (1, 2) and on
solving Eqs. (ii) and (iv), we get D (3, 6)
⇒ 5x − 3 y + 1 = 0
Straight Line and Pair of Straight Lines 389

16. Let P (x1 , y1 ) be any point on the curve y = x . Now, let the centroid of ∆PQR be G (h , k), then
x +1+3
Clearly, y1 = x1 ⇒ x1 = y12 [Q (x1 , y1 ) lies on y = x] h= 1
3
∴ The point is P ( y12, y1 )
⇒ x1 = 3h − 4 …(ii)
3  y1 + 4 − 2
Now, let the given point be A  , 0 , then and k=
2  3
2x1 + 4
 3
2
9 +2
PA =  y12 −  + y12 = y14 − 3 y12 + + y12 ⇒ k= 3 [from Eq. (i)]
 2 4 3
9 5 2x + 4 + 6
= y14 − 2 y12 + = ( y12 − 1)2 + ⇒ 3k = 1
4 4 3
Clearly, PA will be least when ⇒ 9k − 10 = 2x1 …(iii)
Now, from Eqs. (ii) and (iii), we get
y12 − 1 = 0
2(3h − 4) = 9k − 10
5 5
⇒ PAmin = 0 + = ⇒ 6h − 8 = 9k − 10
4 2
⇒ 6h − 9k + 2 = 0
17. Given equation of line is
Now, replace h by x and k by y.
3x + 4 y − 24 = 0
⇒ 6x − 9 y + 2 = 0, which is the required locus and slope
For intersection with X-axis put y = 0 2  a
of this line is Q slope of ax + by + c = 0 is −
⇒ 3x − 24 = 0 ⇒ x = 8 3  b 
For intersection with Y -axis, put x = 0
19.
⇒ 4 y − 24 = 0 ⇒ y = 6
∴ A(8, 0) and B (0, 6) Q(0, β) R(α, β)

B(0,6)
(2, 3)

P(α, 0)

x y
O A(8,0) Equation of line PQ is + =1
α β
Let AB = c = 82 + 62 = 10 Since this line is passes through fixed point (2, 3).
2 3
OB = a = 6 ∴ + =1
α β
and OA = b = 8
∴Locus of R is 2β + 3α = αβ
Also, let incentre is (h k), then
ax + bx2 + cx3 i.e. 2 y + 3x = xy
h= 1 (here,
a+ b+ c ⇒ 3x + 2 y = xy
x1 = 8, x2 = 0, x3 = 0) 20. Given, vertices of triangle are (k, − 3k), (5, k) and (− k, 2).
6 × 8 + 8 × 0 + 10 × 0 k − 3k 1
= 1
6 + 8 + 10 ∴ 5 k 1 = ± 28
2
48 −k 2 1
=
=2
24 k − 3k 1
ay + by2 + cy3 ⇒ 5 k 1 = ± 56
and k= 1 (here, y1 = 0, y2 = 6, y3 = 0)
a+ b+ c −k 2 1
6 × 0 + 8 × 6 + 10 × 0 48
= = =2 ⇒ k(k − 2) + 3k(5 + k) + 1(10 + k2) = ± 56
6 + 8 + 10 24 ⇒ k2 − 2k + 15k + 3k2 + 10 + k2 = ± 56
∴Incentre is (2, 2). ⇒ 5k2 + 13k + 10 = ± 56
18. Let the coordinates of point P be (x1 , y1 ) ⇒ 5k2 + 13k − 66 = 0
or 5k2 + 13k − 46 = 0
Q P lies on the line 2x − 3 y + 4 = 0
∴ 2x1 − 3 y1 + 4 = 0 ⇒ k =2 [Qk ∈ I]
2x + 4 Thus, the coordinates of vertices of triangle are
⇒ y1 = 1 …(i) A(2, − 6), B(5, 2) and C (− 2, 2).
3
390 Straight Line and Pair of Straight Lines

Now, equation of altitude from vertex A is 23. Given mid-points of a triangle are (0, 1), (1, 1) and
−1 (1, 0). Plotting these points on a graph paper and
y − (− 6) = (x − 2) ⇒ x = 2 …(i)
 2 −2  make a triangle.
 
 − 2 − 5 So, the sides of a triangle will be 2, 2 and 22 + 22
Y i.e. 2 2.
Y

C (–2, 2) D
B (5, 2) C (0,2)
(2, 1/2)
E (1, 1)
2 (0, 1)
X′ X
O
A (2, –6)
X′ X
B (1, 0) A (2, 0)
(0, 0) 2
Y′
Y′
Equation of altitude from vertex C is 2 × 0 + 2 2 ⋅0 + 2 ⋅2
−1 x-coordinate of incentre =
y−2 = [x − (− 2)] 2+2+2 2
2 − (− 6) 
 5 −2  2 2− 2
  = × =2 − 2
2+ 2 2− 2
⇒ 3x + 8 y − 10 = 0 …(ii)
1 24. A straight line passing through P and making an angle
On solving Eqs. (i) and (ii), we get x = 2 and y = . of α = 60º , is given by
2
y − y1
 1 = tan (θ ± α )
∴ Orthocentre = 2,  x − x1
 2

21. Let coordinate of the intersection point in fourth


quadrant be (α, − α).
P (3, –2)
Since, (α, − α) lies on both lines 4 ax + 2ay + c = 0 and 60°
⇒ tan α = √3
5bx + 2by + d = 0.
60°
−c
∴ 4 aα − 2aα + c = 0 ⇒ α = …(i)
2a
−d √3x + y = 1
and 5 bα − 2 bα + d = 0 ⇒ α = …(ii)
3b ⇒ 3 x+ y=1
From Eqs. (i) and (ii), we get ⇒ y = − 3 x + 1, then tan θ = − 3
−c −d y+2 tan θ ± tan α
= ⇒ 3bc = 2ad ⇒ =
2a 3b x − 3 1 m tan θ tan α

⇒ 2ad − 3bc = 0 y+2 − 3+ 3


=
x − 3 1 − (− 3)( 3 )
7 + 6 3 − 1  13 
22. Coordinate of S =  ,  = , 1 y+2 − 3− 3
 2 2  2  and =
[QS is mid-point of line QR] x − 3 1 + (− 3)( 3 )
P (2,2) ⇒ y+ 2 =0
y+ 2 −2 3
and = = 3
x−3 1 − 3
y+ 2 = 3 x−3 3
Q R Neglecting, y + 2 = 0, as it does not intersect Y-axis.
S
(6,–1) (7,3)
25. Given, lines are (1 + p) x − py + p (1 + p) = 0 ... (i)
−2
Slope of the line PS is . and (1 + q) x − qy + q (1 + q) = 0 ... (ii)
9
On solving Eqs. (i) and (ii), we get
Required equation passes through (1 , − 1) and parallel
to PS is C { pq , (1 + p) (1 + q)}
−2 ∴ Equation of altitude CM passing through C and
y+1= (x − 1) perpendicular to AB is
9
⇒ 2x + 9 y + 7 = 0 x = pq ... (iii)
Straight Line and Pair of Straight Lines 391

 1 + q ⇒ −
4
y = −1
Q Slope of line (ii) is  .
 q  3
−q 3
∴ Slope of altitude BN (as shown in figure) is . ⇒ y=
1+ q 4
Y  3
∴ Required orthocentre = (3, y) = 3, 
 4
N C 28. On solving equations 3x + 4 y = 9 and y = mx + 1, we get
1)
e( 5
Lin H(h,k) (-p, 0) x=
A O M B
X 3 + 4m
Line (2)
Now, for x to be an integer,
3 + 4m = ± 5 or ± 1
The integral values of m satisfying these conditions are
−q −2 and −1.
∴ Equation of BN is y − 0 = (x + p)
1+ q
−q
29. Now, distance of origin from 4x + 2 y − 9 = 0 is
⇒ y= (x + p) ... (iv) | − 9| 9
(1 + q) =
Let orthocentre of triangle be H(h, k), which is the point 4 +2
2 2 20
of intersection of Eqs. (iii) and (iv). and distance of origin from 2x + y + 6 = 0 is
On solving Eqs. (iii) and (iv), we get |6| 6
=
x = pq and y = − pq 2 +1
2 2 5
⇒ h = pq and k = − pq 9 / 20 3
∴ h+ k =0 Hence, the required ratio = =
6/ 5 4
∴ Locus of H (h, k) is x + y = 0 .
30. Let the vertices of triangle be A(1, 3 ), B(0, 0) and
26. Since, triangle is isosceles, hence centroid is the desired
point. C (2, 0). Here, AB = BC = CA = 2 .
Y Therefore, it is an equilateral triangle. So, the incentre
coincides with centroid.
0 + 1 + 2 0 + 0 + 3
P (3, 4) ∴ I ≡ , 
 3 3 
⇒ I ≡ (1, 1 / 3 )
R 2
 3
2
31. Now, ( A0 A1 )2 = 1 −  + 0 − 
1
X' X
(0, 0) O Q (6, 0)  2  2
2
 1  3
2
1 3
Y' =   +   = + = 1 ⇒ A0 A1 = 1
 2  2 4 4
 4
∴ Coordinates of R 3,  . 2
 3  1
2
 3
( A 0 A 2 )2 =  1 +  + 0 − 
 2   2
27. To find orthocentre of the triangle formed by (0, 0) (3, 4)
2
and (4, 0).  3
2
 3 9 3 12
=   + −  = + = =3
B(3, 4)  2  2 4 4 4
Y
⇒ A0 A2 = 3
P 2
y)  1
2
 3
(3, and ( A 0 A 4 )2 =  1 +  +  0 + 
H  2  2
2
A  3  3 9 3 12
X' X =  +  = + = =3
O Q (4, 0)  2  4 4 4 4
(0, 0)
⇒ A0 A4 = 3
Y'
Thus, ( A0 A1 ) ( A0 A2) ( A0 A4 ) = 3
Let H be the orthocentre of ∆OAB
32. Since, the coordinates of the centroid are
∴ (slope of OP i.e. OH) ⋅(slope of BA) = −1  x1 + x2 + x3 y1 + y2 + y3 
 y − 0  4 − 0  ,  , then the centroid is always
⇒   ⋅  = −1  3 3 
 3 − 0   3 − 4
a rational point.
392 Straight Line and Pair of Straight Lines

33. PQRS is a parallelogram if and only if the mid point of 41. The point O(0, 0) is the mid-point of A (− a , − b) and
the diagonals PR is same as that of the mid-point of QS. B(a , b). Therefore, A , O , B are collinear and equation of
That is, if and only if line AOB is
1+5 4+ a 2+ 7 6+ b
= and = y= x
b
2 2 2 2 a
⇒ a = 2 and b = 3.
Since, the fourth point D (a 2, ab) satisfies the above
34. Slope of line x + 3 y = 4 is − 1 / 3 equation.
and slope of line 6x − 2 y = 7 is 3. Hence, the four points are collinear.
 −1  42. Here, ax + 2 y = λ
Here, 3 ×   = −1
 3
and 3x − 2 y = µ
Therefore, these two lines are perpendicular which
show that both diagonals are perpendicular. For a = − 3, above equations will be parallel or
coincident, i.e. parallel for λ + µ ≠ 0 and coincident,
Hence, PQRS must be a rhombus.
if λ + µ = 0 and if a ≠ − 3, equations are intersecting, i.e.
35. Let y = cos x cos (x + 2) − cos 2 (x + 1) unique solution.
= cos (x + 1 − 1) cos (x + 1 + 1) − cos 2 (x + 1) 43. PLAN Application of inequality sum and differences, along with
= cos 2 (x + 1) − sin 2 1 − cos 2 (x + 1) ⇒ y = − sin 2 1 lengths of perpendicular. For this type of questions involving
This is a straight line which is parallel to X-axis. inequality we should always ckeck all options.
It passes through (π / 2, − sin 2 1). Situation analysis Check all the inequalities
36. Orthocentre of right angled triangle is at the vertex of according to options and use length of perpendicular
right angle. Therefore, orthocentre of the triangle is at from the point (x1 , y1) to ax + by + c = 0
(0, 0). ax1 + by1 + c
i.e.
37. By the given conditions, we can take two perpendicular a 2 + b2
lines as x and y axes. If (h , k) is any point on the locus, As a > b > c>0
then |h| + |k| = 1.Therefore, the locus is |x| + | y| = 1.
This consist of a square of side 1. a − c > 0 and b > 0
Hence, the required locus is a square. ⇒ a + b−c>0 …(i)
38. Since, the origin remains the same. So, length of the a − b > 0 and c > 0 ...(ii)
perpendicular from the origin on the line in its position a + c− b > 0
x y x y
+ = 1 and + = 1 are equal. Therefore, ∴ Option (c) are correct.
a b p q
1 1 1 1 1 1 Also, the point of intersection for ax + by + c = 0 and
= ⇒ + = + bx + ay + c = 0
1 1 1 1 a 2 b 2 p2 q 2
+ +  −c –c 
a 2 b2 p2 q 2 i.e.  , 
 a + b a + b
39. Let the coordinate of S be (x, y).  −c −c 
The distance between (1, 1) and  , 
Q SQ 2 + SR2 = 2SP 2  a + b a + b
⇒ (x + 1) + y2 + (x − 2)2 + y2 = 2 [(x − 1)2 + y2]
2
i.e. less than 2 2.
⇒ x2 + 2x + 1 + y2 + x2 − 4x + 4 + y2 = 2(x2 − 2x + 1 + y2) 2 2
3  c   c 
⇒ 2x + 3 = 0 ⇒ x = − ⇒ 1 +  + 1 +  <2 2
2  a + b  a + b
Hence, it is a straight line parallel to Y-axis.  a + b + c
⇒   2 <2 2
40. Let B, C , D be the position of the point A(4, 1) after the  a+b 
three operations I, II and III, respectively. Then, B is ⇒ a + b + c < 2a + 2b
(1, 4), C (1 + 2, 4) i.e. (3, 4). The point D is obtained from
C by rotating the coordinate axes through an angle π/4 ⇒ a + b−c>0
in anti-clockwise direction. From Eqs. (i) and (ii), option (a) is correct.
Therefore, the coordinates of D are given by 44. Since, 3x + 2 y ≥ 0 …(i)
π π 1
X = 3 cos − 4 sin = − where (1, 3) (5, 0) and (− 1, 2) satisfy Eq. (i).
4 4 2
π π 7 ∴ Option (a) is true.
and Y = 3 sin + 4 cos =
4 4 2 Again, 2x + y − 13 ≥ 0
 1 7 is not satisfied by (1, 3),
∴ Coordinates of D are  − , .
 2 2 ∴ Option (b) is false.
Straight Line and Pair of Straight Lines 393

2x − 3 y − 12 ≤ 0 48. y = 10x is reflection of y = log10 x about y = x.


is satisfied for all points,
49. Since, a1x + b1 y + c1 = 0 and a 2 x + b2y + c2 = 0 cuts the
∴ Option (c) is true.
coordinate axes at concyclic points.
and − 2x + y ≥ 0 ⇒ a1a 2 = b1b2
is not satisfied by (5, 0), or a1b2 + b1a 2 = 0
∴ Option (d) is false. Given lines are, 2x + 3 y + 19 = 0
Thus, (a) and (c) are correct answers. and 9x + 6 y − 17 = 0
45. Let the variable straight line be ax + by + c = 0 ...(i) Here, a1 = 2, b1 = 3, c1 = 19
and a 2 = 9, b2 = 6, c2 = − 17
where, algebraic sum of perpendiculars from (2, 0), (0, 2)
∴ a1a 2 = 18
and (1, 1) is zero.
2a + 0 + c 0 + 2b + c a + b + c and b1b2 = 18
∴ + + =0 ⇒ a1a 2 = b1b2. Thus, points are concyclic.
a 2 + b2 a 2 + b2 a 2 + b2
Hence, given statement is true.
⇒ 3a + 3b + 3c = 0
50. Since, (1, 3 ), (1, − 3 ) and (3, 3 ) form a right angled
⇒ a + b + c=0 ... (ii)
triangle at (1, 3 )
From Eqs. (i) and (ii) ax + by + c = 0 always passes
∴ Equation of circumcircle taking (3, 3 ) and (1, − 3 ) as
through a fixed point (1, 1).
end points of diameter.
46. Let H (h , k) be orthocentre. ∴ (x − 3) (x − 1) + ( y − 3 ) ( y + 3 ) = 0
⇒ (slope of AH ) ⋅ (slope of BC ) = − 1 ⇒ x2 − 4 x + 3 + y2 − 3 = 0
A (–3/7, 16/7) ⇒ x2 + y2 − 4x = 0
5  25
At point  , 1 , S1 = + 1 − 10 < 0
2  4
0
4=

2x

∴ Point (5 / 2, 1) lies inside the circle.


+
+

3y

Hence, no tangent can be drawn.


–y

H (h,k)
4x

Hence, given statement is true.


6

51. The point of intersection of x + 2 y = 10 and 2x + y + 5 = 0


B x+y=1 C
 20 25
(–3/5, 8/5) (–3, 4) is  − ,  which clearly satisfy 5x + 4 y = 0.
 3 3
 16  Hence, given statement is true.
k− 
⇒  7  ⋅ (−1) = −1 52. Let the equation of straight line L be
 h+ 3  y = mx
 7
 1 m   3 3m 
⇒ k−
16
=h+
3 P≡ , , Q ≡  , 
7 7  m + 1 m + 1  m + 1 m + 1
19 Y
⇒ h−k=− ...(i)
7
Also, (slope of CH) ⋅(slope of AB ) = − 1 L1
k −4
⇒ ⋅ (4) = − 1 R
h+3 L2 Q
L
⇒ 4k − 16 = − h − 3 P
⇒ h + 4k = 13 …(ii) X
3 22 O
On solving Eqs. (i) and (ii), we get h = , k =
7 7 x+y=1 x+y=3
 3 22
∴ Orthocentre  ,  Now, equation of
7 7 
m −2
Hence, this coordinate lies in the first quadrant. L1 : y − 2x = …(i)
m+1
47. Since, a , b, c are in AP. and equation of
∴ 2b = a + c 3m + 9
L 2 : y + 3x = …(ii)
or a − 2b + c = 0which satisfy ax + by + c = 0 m+1
∴ ax + by + c = 0 always pass through a fixed point By eliminating m from Eqs. (i) and (ii), we get locus of R
(1, − 2). as x − 3 y + 5 = 0, which represents a straight line.
394 Straight Line and Pair of Straight Lines

53. Let L : ( y − 2) = m(x − 8), m < 0 Hence, the solution set is


{(x, y)| x = 12, y ≥ 2 } ∪ {(x, y)}|
 2 
The ponts P and Q are 8 – , 0 and (0, 2 – 8 m), x + y = 5 / 2, 0 < x < 3, 0 < y > 2}
 m 
respectively. The graph is given in adjoining figure.
 2  2  55. Let the coordinates of Q be (b, α ) and that of S be (−b, β).
Then,OP + OQ = 8 –  + (2 − 8 m) = 10 + – + (–8 m)
 m  m  Suppose, PR and SQ meet in G. Since, G is mid point of
[using AM ≥ GM] SQ, its x-coordinate must be 0. Let the coordinates of R
be (h, k).
 2  2
⇒   + (– 8 m) ≥ 2 16 [Q and −8m are positive] Since, G is mid point of PR, the x-coordinate of P must
 – m m be −h and as P lies on the line y = a, the coordinates of P
2  are (−h , a ) . Since, PQ is parallel to y = mx, slope of
⇒ –  + 8 m ≥ 8 PQ = m
m 
α−a
⇒ =m …(i)
2  b+ h
⇒ 10 –  + 8 m ≥ 10 + 8 ⇒ OP + OQ ≥ 18
m  Again, RQ ⊥ PQ
54. NOTE d : (P, Q ) = | x1 − x 2 | + | y1 − y2 |. Y
P(–h,a)
It is new method of representing distance between two
points P and Q and in future very important in coordinate
geometry. S(–b,β)
Now, let P (x, y) be any pont in the first quadrant. We O
have X′ X
G
d (P , 0) = | x − 0| + | y − 0| = | x| + | y| = x + y
[Q x, y > 0]
d (P , A ) = | X − 3| + |Y − 2| [given] Q (– b,α)
d (P , 0) = d (P , A ) [given]
Y′
⇒ x + y =| x − 3| + | y − 2| …(i) R(h,k)
Case I When 0 < x < 3, 0 < y < 2
In this case, Eq. (i) becomes 1 k −α 1
Slope of RQ = − ⇒ =− …(ii)
x+ y=3 − x+ 2 − y m h−b m
⇒ 2x + 2 y = 5 From Eq. (i), we get
or x + y = 5 /2 α − a = m (b + h ) ⇒ α = a + m (b + h ) …(iii)
Case II When 0 < x < 3, y ≥ 2 and from Eq. (ii), we get
Now, Eq. (i) becomes 1
k −α = − (h − b)
x+ y=3 − x+ y−2 m
⇒ 2x = 1 1
⇒ α =k+ (h − b) …(iv)
⇒ x = 1 /2 m
Case III When x ≥ 3, 0 < y < 2 From Eqs. (iii) and (iv), we get
Infinite segment 1
Y a + m (b + h ) = k + (h − b)
x = 1/2 m
(1/2,2)

2 y=2 ⇒ am + m2 (b + h ) = km + (h − b)
x + ite s

⇒ (m2 − 1) h − mk + b (m2 + 1) + am = 0
Fin
y = egm

1 Hence, the locus of vertex is


5/2 ent

(m2 − 1) x − my + b (m2 + 1) + am = 0
,

X 56. Let equation of line AC is


O 1/2 (5/2,0) 3
y+4 x+5
= =r
Now, Eq. (i) becomes sin θ cos θ
x+ y= x−3 + 2 − y Let line AE make angle θ with X-axis and intersects
⇒ 2 y = − 1 or y = − 1 / 2 x + 3 y + 2 = 0 at B at a distance r1 and line 2x + y + 4 = 0
Hence, no solution. at C at a distance r2 and line x − y − 5 = 0 at D at a
Case IV When x ≥ 3, y ≥ 2 distance r3 .
In this case, case I changes to ∴ AB = r1 , AC = r2, AD = r3 .
x+ y= x−3 + y−2 ⇒ 0 = −5 −5 − 3 × 4 + 2  I′ 
r1 = − Q r = − (a cos θ + b sin θ ) 
which is not possible. 1 ⋅ cos θ + 3 ⋅ sin θ  
Straight Line and Pair of Straight Lines 395

15 On solving Eqs. (i), (ii) and (iii), we get the vertices of a


⇒ r1 = …(i)
cos θ + 3 sin θ  1 1  5 7
triangle are A(−7, 5), B  ,  and C  ,  .
2 × (−5) + 1 (−4) + 4  3 9  4 8
Similarly, r2 = −
2 cos θ + 1 ⋅ sin θ Let P (α , α 2) be a point inside the ∆ABC. Since, A and P
Dx–y are on the same side of 5x − 6 y − 1 = 0, both
–5=
0 5 (−7) − 6 (5) − 1 and 5α − 6α 2 − 1 must have the same
r3 sign, therefore
=0
y+4 5α − 6α 2 − 1 < 0
C 2x +
⇒ 6 α 2 − 5α + 1 > 0
r2
B x+ 3 y + 2 = 0 ⇒ (3α − 1) (2α − 1) > 0
1 1
r1 ⇒ α < or α > …(iv)
3 2
A (–5,–4)
 5 7
Also, since P (α , α 2) and C  ,  lie on the same side of
 4 8
E  5  7
2x + 3 y − 1 = 0, therefore both 2   + 3   − 1 and
10  4  8
⇒ r2 = …(ii)
2 cos θ + sin θ 2 α + 3α 2 − 1 must have the same sign.
−5 × 1 − 4 (−1) − 5 Therefore, 2α + 3α 2 − 1 > 0
and r3 = −  1
cos θ − sin θ ⇒ (α + 1) α −  > 0
 3
6
⇒ r3 = …(iii) ⇒ α < − 1 ∪ α > 1 /3 …(v)
cos θ − sin θ
 1 1
But it is given that, and lastly  ,  and P (α , α ) lie on the same side of the
2

2 2 2
 3 9
2 2 2
 15   10   6   15  10  6 1  1
  +  =  ⇒   +  =  line therefore, + 2   − 3 and α + 2α 2 − 3 must have
 AB  AC   AD   r1   r2   r3  3  9
⇒ (cos θ + 3 sin θ )2 + (2 cos + sin θ )2 = (cos θ − sin θ )2 the same sign.
[from Eqs. (i), (ii) and (iii)] Therefore, 2α 2 + α − 3 < 0
⇒ cos 2 θ + 9 sin 2 θ + 6 cos θ sin θ + 4 cos 2 θ ⇒ 2α (α − 1) + 3 (α − 1) < 0
+ sin 2 θ + 4 cos θ sin θ = cos 2 θ + sin 2 θ − 2 cos θ sin θ 2
⇒ (2α + 3) (α − 1) < 0 ⇒ − < α < 1
⇒ 4 cos 2 θ + 9 sin 2 θ + 12 sin θ cos θ = 0 3
⇒ (2 cos θ + 3 sin θ )2 = 0 On solving Eqs. (i), (ii) and (iii), we get the common
3 1
⇒ 2 cos θ + 3 sin θ = 0 answer is − < α < − 1 ∪ < α < 1.
2 2
⇒ cos θ = − (3 / 2) sin θ
58. Let l makes an angle α with the given parallel lines and
On substituting this in equation of AC, we get intercept AB is of 3 units.
y+4 x+5
= l
sin θ − sin θ
3
2 B
y + 2x = 2
⇒ −3 ( y + 4) = 2 (x + 5) 3 3/√5
⇒ −3 y − 12 = 2x + 10
A α
⇒ 2x + 3 y + 22 = 0 y + 2x = 5
C
which is the equation of required straight line. (2,3)
57. Given lines are 2x + 3 y − 1 = 0 …(i) Now, distance between parallel lines
x + 2y − 3 = 0 …(ii) |5 − 2| 3
= =
5x − 6 y − 1 = 0 …(iii) 1 +2
2 2 5
1 2
A(–7,5) ∴ sin α = , cos α =
5 5
0

1
1=

, and tan α =
x+

2
y–

2y
+3

⇒ Equation of straight line passing through (2, 3 ) and


–3

P (a, a2)
2x

making an angle α with y + 2x = 5 is


=
0

y−3
= tan (θ + α )
B(1/3,1/9) 5x – 6y – 1 = 0 C(5/4,7/8) x−2
396 Straight Line and Pair of Straight Lines

y−3 tan θ + tan α Let R(h , k) be their point of intersection of lines


⇒ =
x − 2 1 − tan θ tan α AQ and BP.
y−3 tan θ − tan α h 5k
and = Then, + =1
x − 2 1 + tan θ tan α 7 λ
7h k
y−3 3 y−3 1 and − =1
⇒ =− and = λ 5
x−2 4 x−2 0
1  h 1  k
⇒ 3x + 4 y = 18 and x=2 ⇒ 1 −  = 1 +  [on eliminating λ]
5k  7  7h  5
59. Let m1 and m2 be the slopes of the lines 3x + 4 y = 5 and
4x − 3 y = 15, respectively. ⇒ h (7 − h ) = k(5 + k)
3 4 ⇒ h + k − 7h + 5k = 0
2 2
Then, m1 = − and m2 =
4 3 Hence, the locus of a point is
Clearly, m1m2 = − 1. So, lines AB and AC are at right
angle. Thus, the ∆ABC is a right angled isosceles x2 + y2 − 7x + 5 y = 0.
triangle. 61. Let BC be taken as X-axis with origin at D, the
Y
mid-point of BC and DA will be Y-axis.
C Given, AB = AC
4x − 3y − 15 = 0

Let BC = 2a, then the coordinates of B and C are (– a, 0)


45°
and (a , 0) let A (0, h ).
Y

A
45°
X
A (3,−1) 3x+ 4y – 5 = 0 B
E
Hence, the line BC through (1, 2) will make an angle of X' F X
45° with the given lines. So, the possible equations of B D C
BC are Y'
m ± tan 45°
( y − 2) = (x − 1) Then, equation of AC is
1 m m tan 45° x y
3 + =1 …(i)
where, m = slope of AB = − a h
4 and equation of DE ⊥ AC and passing through origin is
3
− ±1 x y
− =0
⇒ ( y − 2) = 4 (x − 1) h a
 3
1 m −  hy
 4 ⇒ x= …(ii)
a
−3 ± 4
⇒ ( y − 2) = (x − 1) On solving, Eqs. (i) and (ii), we get the coordinates of
4±3 point E as follows
1 hy y a 2h
⇒ ( y − 2) = (x − 1) + = 1 ⇒ y =
7 a2 h a 2 + h2
and ( y − 2) = − 7 (x − 1)
 ah 2 a 2h 
⇒ x − 7 y + 13 = 0 ∴Coordinate of E =  2 , 2 
 a + h a + h 2
2
and 7x + y − 9 = 0
60. The equation of the line AB is Since, F is mid-point of DE.
x y  ah 2 a 2h 
+ =1 … (i) ∴Coordinate of F  , 2 
7 −5 2 (a + h ) 2 (a + h ) 
2 2 2

⇒ 5x − 7 y = 35 ∴ Slope of AF ,
Equation of line perpendicular to AB is a 2h
h−
7x + 5 y = λ …(ii) 2 (a 2 + h 2) 2h (a 2 + h 2) − a 2h
m1 = =
It meets X-axis at P(λ/7, 0) and Y-axis at Q(0, λ /5). ah 2 − ah 2
0−
x 5y 2 (a + h )
2 2
The equations of lines AQ and BP are + = 1 and
7 λ
− (a 2 + 2h 2)
7x y
− = 1, respectively. ⇒ m1 = …(iii)
λ 5 ah
Straight Line and Pair of Straight Lines 397

a 2h  11 2
−0 So, the coordinates of C are  , .
a 2 + h2 a 2h  5 5
and slope of BE , m2 = =
ah 2 ah 2 + a3 + ah 2 Thus, the equation of BC is
+a 2 /5 − 6
a +h
2 2
y −6 = (x + 7)
ah 11 / 5 + 7
⇒ m2 = 2 …(iv)
a + 2h 2 ⇒ − 23 ( y − 6) = 14 (x + 7)
From Eqs. (iii) and (iv), m1m2 = − 1 ⇒ AF ⊥ BE ⇒ 14x + 23 y − 40 = 0

62. Let the coordinates of B and C be (x1 , y1 ) and (x2, y2) 63. Let O be the centre of circle and M be mid-point of AB.
respectively. Let m1 and m2 be the slopes of AB and AC,
respectively. Then, A M B
A(1,–2) (– 3, 4) (5, 4)

x_y+5=0 x + 2y = 0 O

F E
D C
(a, b)

B C Then, OM ⊥ AB ⇒ M (1, 4)
(x1, y1) (x2, y2)
Since, slope of AB = 0
y1 + 2
m1 = slope of AB = Equation of straight line MO is x = 1 and equation of
x1 − 1 diameter is 4 y = x + 7.
y +2 ⇒ Centre is (1, 2). Also, O is mid-point of BD
and m2 = slope of AC = 2
x2 − 1  α + 5 β + 4
⇒ ,  = (1, 2)
Let F and E be the mid point of AB and AC, respectively.  2 2 
Then, the coordinates of E and F are ⇒ α = − 3, β = 0
 x + 1 y2 − 2  x + 1 y1 − 2 ∴ AD = (−3 + 3)2 + (4 − 0)2 = 4
E 2 ,  and F  1 ,  , respectively.
 2 2   2 2  and AB = 64 + 0 = 8
Now, F lies on x − y + 5 = 0. Thus, area of rectangle = 8 × 4 = 32 sq units
x1 + 1 y1 − 2
⇒ − = −5 64. Let the coordinates of A be (0, α ). Since, the sides AB
2 2
⇒ x1 − y1 + 13 = 0 …(i) and AD are parallel to the lines y = x + 2 and y = 7x + 3,
respectively.
Since, AB is perpendicular to x − y + 5 = 0.
Y
∴ (slope of AB) ⋅ (slope of x − y + 5 = 0) = − 1.
y1 + 2 D C
⇒ ⋅ (1) = − 1
x1 − 1
⇒ y1 + 2 = − x1 + 1
E(
1,

⇒ x1 + y1 + 1 = 0 …(ii)
2)

A (0, α) B
On solving Eqs. (i) and (ii), we get
x1 = − 7, y1 = 6. X′ X
O
So, the coordinates of B are (−7, 6).
Now, E lies on x + 2 y = 0.
Y′
x2 + 1  y − 2
∴ +2 2  =0 ∴ The diagonal AC is parallel to the bisector of the angle
2  2 
between these two lines. The equation of the bisectors
⇒ x2 + 2 y2 − 3 = 0. …(iii) are given by
Since, AC is perpendicular to x + 2 y = 0 x− y+2 7x − y + 3

∴ (slope of AC) ⋅ (slope of x + 2 y = 0) = − 1 2 50
y2 + 2  1 ⇒ 5 (x − y + 2) = ± (7x − y + 3)
⇒ ⋅ −  = − 1
x2 − 1  2 ⇒ 2x + 4 y − 7 = 0 and 12x − 6 y + 13 = 0.
⇒ 2x2 − y2 = 4 … (iv) Thus, the diagonals of the rhombus are parallel to the
lines 2x + 4 y − 7 = 0 and 12x − 6 y + 13 = 0.
On solving Eqs. (iii) and (iv), we get
11 2 2 12
x2 = and y2 = ∴ Slope of AE = − or
5 5 4 6
398 Straight Line and Pair of Straight Lines

2 −α 1 2 −α ⇒ bk − b2 = ah − a 2
⇒ =− or =2
1 −0 2 1 −0 ⇒ ah − bk = a 2 − b2 …(ii)
5 x y
⇒ α= or α = 0. Equation of line AB is + = 1.
2 a b
Hence, the coordinates are (0, 5 /2) or (0, 0). Since, Q lies on AB, therefore
65. The equation of any line passing through (1, − 10) is h k
+ =1
y + 10 = m (x − 1). a b
Since, it makes equal angles, say θ, with the given lines, ⇒ bh + ak = ab …(iii)
therefore On solving Eqs. (ii) and (iii), we get
m−7 m − (−1) 1 h k 1
tan θ = = ⇒ m = or –3 = =
1 + 7m 1 + m(−1) 3 ab2 + a (a 2 − b2) − b(a 2 − b2) + a 2b a 2 + b2
Hence, the equations of third side are h k 1
⇒ 3
= 3 = 2 [from Eq. (i)]
1 a b c
y + 10 = (x − 1) or y + 10 = − 3 (x − 1)
3 ⇒ a = (hc2)1/3 and b = (kc2)1/3
i.e. x − 3 y − 31 = 0 or 3x + y + 7 = 0 On substituting the values of a and b in a 2 + b2 = c2,
66. Let ABC be a triangle whose vertices are we get h 2 / 3 + k2 / 3 = c2 / 3
A [at1t2, a (t1 + t2)], B [at2t3 , a (t2 + t3 )] and Hence, locus of a point is x2 / 3 + y2 / 3 = c2 / 3 .
C [at1t3 , a (t1 + t3 )]. 68. Since, diagonals of rectangle bisect each other, so
a (t 2 + t3 ) − a (t1 + t3 ) 1 mid point of (1, 3) and (5, 1) must satisfy y = 2x + c, i.e.
Then, Slope of BC = =
at 2t3 − at1t3 t3 (3, 2) lies on it.
a (t1 + t3 ) − a (t1 + t2) 1
Slope of AC = = D C(5,1)
at1t3 − at1t2 t1
So, the equation of a line through A perpendicular to BC
is y − a (t1 + t2) = − t3 (x − at1t2) … (i)
and the equation of a line through B perpendicular to y=
AC is 2x
A(1,3) B +
y − a (t2 + t3 ) = − t1 (x − at2t3 ) …(ii) c
The point of intersection of Eqs. (i) and (ii), is the ⇒ 2 =6 + c
orthocentre. ⇒ c = −4
On subtracting Eq. (ii) from Eq. (i), we get x = − a. ∴ Other two vertices lies on y = 2x − 4
On putting x = − a in Eq. (i), we get Let the coordinate of B be (x, 2x − 4).
y = a (t1 + t2 + t3 + t1t2t3 ) ∴ Slope of AB ⋅ Slope of BC = − 1
Hence, the coordinates of the orthocentre are  2 x − 4 − 3  2 x − 4 − 1
[− a , a (t1 + t2 + t3 + t1t2t3 )]. ⇒   ⋅  = −1
 x−1   x−5 
67. Let OA = a and OB = b. Then, the coordinates of A and B
⇒ (x2 − 6x + 8) = 0
are (a, 0) and (0, b) respectively and also, coordinates of
P are (a , b). Let θ be the foot of perpendicular from P on ⇒ x = 4, 2 ⇒ y = 4, 0
AB and let the coordinates of Q (h , k). Here, a and b are Hence, required points are (4, 4), (2, 0).
the variable and we have to find locus of Q. 69. Let the coordinates of third vertex be C (a , b).
Given, AB = c ⇒ AB2 = c2
C (a,b)
⇒ OA + OB2 = c2 ⇒ a 2 + b2 = c2
2
…(i)
Since, PQ is perpendicular to AB.
Y

B )
(0,b) P(a,b) 0, 0
H(
Q
(h
,k

A (5,–1) B (–2,3)
)

X′ X
O A(a,0) Since, CH is ⊥ AB,
Y′
⇒ Slope of AB ⋅ Slope of PQ = − 1  b  4 
∴     = −1
0−b k−b  a   −7 
⇒ ⋅ = −1
a −0 h − a ⇒ 4b = 7a ...(i)
Straight Line and Pair of Straight Lines 399

Also, AH ⊥ BC 72. PLAN Distance of a point ( x 1, y1 ) from ax + by + c = 0 is given by


 1  3 − b  ax 1 + by1 + c
∴ −    = −1 .
 5   −2 − a 
a2 + b 2
⇒ 3 − b = −10 − 5a ...(ii)
y=x
On solving Eqs. (i) and (ii), we get x<y
2√2
a = −4 , b = −7
70. Since, the side AB is perpendicular to AD.
D C (1,1) √2 x≥y
4 x + 7y + 5 = 0

√2 2√2

Let P (x, y) is the point in first quadrant.


x− y x+ y
Now, 2 ≤ + ≤4
2 2
A (–3,1) B
2 2 ≤ |x − y| + | x + y| ≤ 4 2
∴ Its equation is of the form 7x − 4 y + λ = 0
Case I x ≥ y
Since, it passes through (−3, 1).
2 2 ≤ (x − y) + (x + y) ≤ 4 2
∴ 7(−3) − 4(1) + λ = 0.
⇒ x ∈[ 2, 2 2]
⇒ λ = 25
∴ Equation of AB is Case II x < y
7x − 4 y + 25 = 0 2 2 ≤ y − x + (x + y) ≤ 4 2
Now, BC is parallel to AD. Therefore, its equation is y ∈[ 2, 2 2]
4x + 7 y + λ = 0 ⇒ A = (2 2 )2 − ( 2 )2 = 6 sq units
Since, it passes through (1, 1).
∴ 4(1) + 7(1) + λ = 0 Topic 2 Angle between Straight Lines and
⇒ λ = − 11 Equation of Angle Bisector
∴ Equation of BC is 4x + 7 y − 11 = 0
1. According to the given situation, if we draw the
Now, equation of DC is 7x − 4 y + λ = 0 diagram.
⇒ 7(1) − 4(1) + λ = 0
y
⇒ λ = −3
P′ (0, 2 √3) P (2, 2 √3)
∴ 7x − 4 y − 3 = 0
71. Given vertices of triangle ABC are A(1, 0), B(6, 2) and 30º 30º
3 
C  , 6 and the centroid of triangle G divides the A
2  120º
x
triangle such that area of triangles AGB, BGC and CGA O B
x=1
are equal, so point P is the centroid of triangle ‘G’.
 1 + 6 + 3 / 2 0 + 2 + 6
∴Coordinate of P  , 
 3 3 
Q The line x = 1 is equidistance, from point P(2, 2 3 )
 17 8
= P ,  and the Y -axis.
 6 3
So, P′ (0, 2 3 ) and according to the concept of reflection
 7 1
and given point Q  − , −  . line AB passes through P′ (0, 2 3 ).
 6 3 As the angle ∠P′ BX = ∠ABX = 120°
So length of the line segment PQ ∴Slope of line AB = tan 120° = − 3
2 2
 17 7  8 1 ∴Equation of line AB is
=  +  + + 
6 6  3 3
y − 2 3 = − 3 (x − 0)
2 2
 24  9 ⇒ 3x + y = 2 3 …(i)
=   + 
6  3 From the options, the line (i) passes through the point
(3, − 3 ).
= 42 + 32 = 16 + 9 = 25 = 5
400 Straight Line and Pair of Straight Lines

2. Take any point B (0, 1) on given line. Equation of the line BC is


4 −1
y−4 = (x − 1)
B (0, 1) √ 3y =x – 1 1 −5
3
⇒ y − 4 = − (x − 1)
4
A (√ 3, 0) ⇒ 4 y − 16 = − 3x + 3
⇒ 3x + 4 y − 19 = 0
B′ (0, –1)
Again, equation of the bisectors of the angles between
Equation of AB′ is two given lines AB and BC are
−1 −0 3 y − 4x + 17 4 y + 3x − 19
y−0 = (x − 3 ) =±
0− 3 3 +4
2 2
42 + 32
⇒ − 3y = − x + 3
⇒ 3 y − 4x + 17 = ± (4 y + 3x − 19)
⇒ x − 3y = 3
⇒ 3 y − 4x + 17 = 4 y + 3x − 19
⇒ 3y = x − 3
and 3 y − 4x + 17 = − (4 y + 3x − 19)
3. For collinear points ⇒ 36 = y + 7x and 7 y − x = 2
− sin( β − α ) − cos β 1 Out of these two, equation of the bisector of angle ABC is
∆= cos( β − α ) sin β 1 7 y = x + 2.
cos( β − α + θ ) sin( β – θ ) 1 7. Here, the triangle formed by a line parallel to X-axis
passing through P (h , k) and the straight line y = x and
Clearly, ∆ ≠ 0 for any value of α , β , θ.
y = 2 − x could be as shown below:
Hence, points are non-collinear.
Y
4. The line segment QR makes an angle of 60° with the y=x
positive direction of X-axis. (1, 1)
So, the bisector of the angle PQR will make an angle A
of 60° with the negative direction of X-axis it will (k, k)
therefore have angle of inclination of 120° and so, its B C (2, −k, k)
Y=K
equation is p (h, k)
X′
y − 0 = tan 120° (x − 0) y=x
O
y=2−x
Y
⇒ y = − 3x
⇒ y + 3x = 0
Since, area of ∆ABC = 4h 2
5. It is not necessary that the bisector of an angle will 1
divide the triangle into two similar triangles, therefore, ∴ AB ⋅ AC = 4h 2
2
statement II is false.
Now, we verify Statement I. where, AB = 2 |k – 1| and AC = 2 (|k – 1|)
∆ OPQ, OR is the internal bisector of ∠POQ. 1
⇒ ⋅ 2(k − 1)2 = 4h 2
PR OP 2
∴ = ⇒ 4h 2 = (k − 1)2
RQ OQ
⇒ 2h = ± (k − 1)
PR 22 + 22 2 2
⇒ = = The locus of a point is 2x = ± ( y − 1).
RQ 12 + 22 5
8. Given equations of lines are
1 − (− 7)
6. Equation of the line AB is y − 1 = (x − 5) x − 2 y + 4 = 0 and 4x − 3 y + 2 = 0
5 − (− 1)
Here, a1a 2 + b1b2 = 1(4) + (−2)(−3) = 10 > 0
8
⇒ y−1 = (x − 5) For obtuse angle bisector, we take negative sign.
6
x −2y + 4 4x − 3 y + 2
⇒ y−1 =
4
(x − 5) ∴ =−
3 5 5
⇒ 3 y − 3 = 4x − 20 ⇒ 5 (x − 2 y + 4) = − (4x − 3 y + 2)
⇒ 3 y − 4x + 17 = 0 ⇒ (4 + 5 )x − (2 5 + 3) y + (4 5 + 2) = 0
Straight Line and Pair of Straight Lines 401

9. Since, the required line L passes through the ⇒ x1 = − 3 and y1 = 0


intersection of L1 = 0 and L 2 = 0. So, point B is (− 3, 0)
L2 Also, it is given that mid-point of side AC is
(2, 3), so
x2 + 1 y +2
A P = 2 and 2 =3
L 2 2
(x1,y1)
⇒ x2 = 3 and y2 = 4
So, point C is (3, 4).
L1 Now, centroid of ∆ABC is
So, the equation of the required line L is  1 + (− 3) + 3 2 + 0 + 4 1 
G ,  =G  , 2
L1 + λL 2 = 0.  3 3  3 
i.e. (ax + by + c) + λ (lx + my + n ) = 0 …(i) 2 Given, px + qy + r = 0 is the equation of line such that
where, λ is a parameter. 3 p + 2q + 4r = 0
Since, L1 is the angle bisector of L = 0 and L 2 = 0. Consider, 3 p + 2q + 4r = 0
∴ Any point A (x1 , y1 ) on L1 is equidistant from L1 = 0 3 p 2q
⇒ + + r =0
and L 2 = 0. 4 4
| lx1 + my1 + n |
⇒ (dividing the equation by 4)
l2 + m 2  3  1
⇒ p  + q   + r = 0
 4  2
|(ax1 + by1 + c) + λ (lx1 + my1 + n )|
= …(ii)  3 1
(a + λl)2 + (b + λm)2 ⇒  ,  satisfy px + qy + r = 0
 4 2
But, A (x1 , y1 ) lies on L1. So, it must satisfy the equation  3 1
So, the lines always passes through the point  ,  .
of L1 , ie, ax1 + by1 + c1 = 0.  4 2
On substituting ax1 + by1 + c = 0 in Eq. (ii), we get 3. As the given lines x − y + 1 = 0 and 7x − y − 5 = 0 are not
| lx1 + my1 + n | |0 + λ (lx1 + my1 + n )| parallel, therefore they represent the adjacent sides of
= the rhombus.
l2 + m 2 (a + λl)2 + (b + λm)2
On solving x − y + 1 = 0 and 7x − y − 5 = 0, we get x = 1
⇒ λ2(l2 + m2) = (a + λl)2 + (b + λm)2 and y = 2. Thus, one of the vertex is A(1, 2).
(a 2 + b2)
∴ λ=− D C (x, y)
=0

2 (al + bm)
y–5

On substituting the value of λ in Eq. (i), we get (–1, –2)


(a 2 + b2)
7x –

(ax + by + c) − (lx + my + n ) = 0
2 (al + bm) A x − y + 1=0 B
⇒ 2 (al + bm) (ax + by + c) − (a 2 + b2) (lx + my + n ) = 0 (1, 2)
which is the required equation of line L. Let the coordinate of point C be (x, y).
x+1 y+2
Then, −1 = and − 2 =
Topic 3 Area and Family of 2 2
Concurrent Lines ⇒ x + 1 = − 2 and y = − 4 − 2
1. Let a ∆ABC is such that vertices ⇒ x= −3
and y= −6
A (1, 2), B(x1 y1 ) and C (x2, y2).
Hence, coordinates of C = (− 3, − 6)
A(1,2)
Note that, vertices B and D will satisfy x − y + 1 = 0 and
7x − y − 5 = 0, respectively.
Since, option (c) satisfies 7x − y − 5 = 0, therefore
 1 − 8
B(x1,y1) C(x2,y2) coordinate of vertex D is  , .
3 3 
It is given that mid-point of side AB is (− 1, 1).
4. Let lines OB : y = mx
x1 + 1
So, = −1 CA : y = mx + 1
2
y1 + 2 BA : y = nx + 1
and =1
2 and OC : y = nx
402 Straight Line and Pair of Straight Lines

The point of intersection B of OB and AB has x ⇒ ( p + q + r ) ( p2 + q2 + r 2 − pq − qr − pr ) = 0


coordinate
1
⋅ ⇒ p3 + q3 + r3 − 3 pqr = 0
m−n Therefore, (a) and (c) are the answers.
Y
9. (A) Solving equations L1 and L3 ,
A x y 1
= =
B
−36 + 10 −25 + 12 2 − 15
C D
∴ x = 2, y = 1
X′ X
O L1 , L 2, L3 are concurrent, if point (2, 1) lies on L 2
∴ 6 − k −1 =0 ⇒ k =5
Y′ (B) Either L1 is parallel to L 2,or L3 is parallel to L 2,then
Now, area of a parallelogram OBAC 1 3 3 −k
= or =
= 2 × area of ∆ OBA 3 −k 5 2
1 1 1 ⇒ k = −9
= 2 × × OA × DB = 2 × ×
2 2 m−n −6
or k=
1 1 5
= = (C) L1 , L 2, L3 form a triangle, if they are not concurrent,
m − n |m − n |
or not parallel.
depending upon whether m > n or m < n. 6
∴ k ≠ 5, − 9, −
5. Since, vertices of a triangle are (0, 8 / 3), (1, 3) and (82, 30) 5
5
0 8 /3 1 ⇒ k=
1 1  8  6
Now, 1 3 1 = − (1 − 82) + 1(30 − 246)
2 2  3  (D) L1 , L 2, L3 do not form a triangle, if
82 30 1 6
k = 5, − 9, − ⋅
1 5
= [216 − 216] = 0
2 10. The set of lines ax + by + c = 0, where 3a + 2b + 4c = 0 or
∴ Points are collinear. 3 1  3 1
a + b + c = 0 are concurrent at  x = , y =  i.e.
6. The points of intersection of three lines are 4 2  4 2
A (1, 1), B(2, − 2), C (−2, 2). comparing the coefficients of x and y.
 3 1
Now, | AB| = 1 + 9 = 10 , Thus, point of concurrency is  ,  .
 4 2
|BC| = 16 + 16 = 4 2,
and |CA| = 9 + 1 = 10 Alternate Solution
∴ Triangle is an isosceles. As, ax + by + c = 0, satisfy 3a + 2b + 4c = 0 which
represents system of concurrent lines whose point of
7. Given lines, x + 2 y − 3 = 0 and 3x + 4 y − 7 = 0 intersect concurrency could be obtained by comparison as,
at (1,1), which does not satisfy 2x + 3 y − 4 = 0 and 3a 2
4x + 5 y − 6 = 0. ax + by + c ≡ + b+ c
4 4
Also, 3x + 4 y − 7 = 0 and 2x + 3 y − 4 = 0 intersect at 3 1
⇒ x = , y = is point of concurrency.
(5, –2) which does not satisfy x + 2 y − 3 = 0 and 4 2
4x + 5 y − 6 = 0 .  3 1
∴  ,  is the required point.
Lastly, intersection point of x + 2 y − 3 = 0 and  4 2
2x + 3 y − 4 = 0 is (–1, 2) which satisfy 4x + 5 y − 6 = 0.
Hence, only three lines are concurrent. x1 y1 1 a1 b1 1
8. Given lines px + qy + r = 0, qx + ry + p = 0 11. Since, x2 y2 1 = a 2 b2 1
x3 y3 1 a3 b3 1
and rx + py + q = 0 are concurrent.
p q r represents area of triangles are equal, which does not
∴ q r p =0 impies triangles are congrvent. Hence, given statement
r p q is false.

Applying R1 → R1 + R2 + R3 and taking common from 12. Let the vertices of a triangle be, O (0, 0) A (a , 0) and
R1 B (b, c) equation of altitude BD is x = b.
1 1 1 c
Slope of OB is .
( p + q + r) q r p = 0 b
b
r p q Slope of AF is − .
c
Straight Line and Pair of Straight Lines 403

Now, the equation of altitude AF is ax2 + by2 + c = 0,


b
y − 0 = − (x − a ) x=0
c and y = 0.
Suppose, BD and OE intersect at P. which is a point specified as the origin.
  (a − b) 
Coordinates of P are b, b   When, a = b and c is of sign opposite to that of a,
  c  
ax2 + by2 + c = 0 represents a circle.
a−b
Let m1 be the slope of OP = Hence, the given equation,
c
(ax2 + by2 + c)(x2 − 5xy + 6 y2) = 0
c
and m2 be the slope of AB = may represent two straight lines and a circle.
b−a
2. Given, x2 − y 2 + 2 y = 1
 a − b  c 
Now, m1m2 =     = −1
 c   b − a ⇒ x2 = ( y − 1 ) 2
⇒ x = y −1
We get, that the line through O and P is perpendicular
and x=−y+1
to AB.
1 Y
|x (5 + 2) + (−3) (−2 − y) + 4 ( y − 5)|
Area of ∆PBC 2
13. = (0,3)
Area of ∆ABC 1 |6 (5 + 2) + (−3) (−2 − 3) + 4 (3 − 5)|

x+
2

y=
|7x + 7 y − 14| 7|x + y − 2|

3
= = Angle bisector
|42 + 15 − 8| 49 (0, 1) (2,1)
x + y −2
= X′ X
 7  −1 O 1 3
y=x+1 y=
14. A straight line perpendicular to 5x − y = 1 is x + 5 y = λ. –x
+1
Since, area of triangle = 5 Y′
Angle bisector
Y
From the graph, it is clear that equation of angle
bisectors are
λ/5
X′ X y=1
O λ
and x=0
∴ Area of region bounded by x + y = 3, x = 0
Y′ 1
and y = 1 is ∆ = × 2 × 2 = 2 sq units
1 λ  2
⇒ λ⋅ =5
2 5  3. The given curve is
⇒ λ2 = 50 3 x 2 − y2 − 2 x + 4 y = 0 …(i)
⇒ |λ | = 5 2 Let y = mx + c be the chord of curve (i) which subtend
∴ Equation of the line L is, x + 5 y = ± 5 2 right angle at origin. Then, the combined equation of
lines joining points of intersection of curve (i) and chord
Topic 4 Homogeneous Equation of y = mx + c to the origin, can be obtained by the equation
of the curve homogeneous, i.e.
Pair of Straight Lines
 y − mx  y − mx
1. Let a and b be non-zero real numbers. 3x2 − y2 − 2x   + 4y   =0
 c   c 
Therefore the given equation ⇒ 3cx2 − cy2 − 2xy + 2mx2 + 4 y2 − 4mxy = 0
(ax + by + c) (x − 5xy + 6 y ) = 0 implies either
2 2 2 2
⇒ (3c + 2m) x2 − 2 (1 + 2m) y + (4 − c) y2 = 0
x2 − 5xy + 6 y2 = 0 Since, the lines represented are perpendicular to each
⇒ (x − 2 y)(x − 3 y) = 0 other.
⇒ x = 2y ∴ Coefficient of x2 + Coefficient of y2 = 0
and x = 3y ⇒ 3c + 2m + 4 − c = 0
⇒ c+ m + 2 =0
represent two straight lines passing through origin or
On comparing with y = mx + c
ax2 + by2 + c = 0 when c = 0 and a and b are of same
⇒ y = mx + c passes through (1, – 2).
signs, then
404 Straight Line and Pair of Straight Lines

Topic 5 General Equation of Pair of Now, slope of QR is −2 . [given]


Straight Lines But QR ⊥ PS.
∴ Slope of PS is 1/2.
1. Let S be the mid-point of QR and given ∆PQR is an
isosceles. Let m be the slope of PQ .
m − 1 /2
Therefore, PS ⊥ QR and S is mid-point of hypotenuse, ∴ tan (± 45° ) =
therefore S is equidistant from P , Q , R. 1 − m (−1 / 2)
∴ PS = QS = RS 2m − 1
⇒ ±1=
Since, ∠ P = 90° and ∠ Q = ∠ R 2+m
But ∠ P + ∠ Q + ∠ R = 180° ⇒ m = 3, − 1 / 3
∴ 90° + ∠ Q + ∠ R = 180° ∴ Equations of PQ and PR are
⇒ ∠ Q = ∠ R = 45° y − 1 = 3 (x − 2)
Y 1
and y − 1 = − (x − 2)
3
Q
P(2,1) or 3 ( y − 1) + (x − 2) = 0
90°
Therefore, joint equation of PQ and PR is
S [3 (x − 2) − ( y − 1)] [(x − 2) + 3 ( y − 1)] = 0
⇒ 3 (x − 2)2 − 3 ( y − 1)2 + 8 (x − 2) ( y − 1) = 0
X' X ⇒ 3x2 − 3 y2 + 8xy − 20x − 10 y + 25 = 0
O R
Y'
16
Circle
Topic 1 Equation of Circle
Objective Questions I (Only one correct option) 8. Let AB be a chord of the circle x2 + y2 = r 2 subtending a
1. A circle touching the X-axis at (3, 0) and making a right angle at the centre. Then, the locus of the centroid
intercept of length 8 on the Y -axis passes through the of the ∆ PAB as P moves on the circle, is (2001, 1M)
point (2019 Main, 12 April II) (a) a parabola (b) a circle
(a) (3, 10) (b) (3, 5) (c) an ellipse (d) a pair of straight lines
(c) (2, 3) (d) (1, 5) 9. The lines 2x − 3 y = 5 and 3x − 4 y = 7 are diameters of a
2. Let O(0, 0) and A(0, 1) be two fixed points, then the locus circle of area 154 sq units. Then, the equation of this
of a point P such that the perimeter of ∆AOP is 4, is circle is (1989, 2M)
(2019 Main, 8 April I) (a) x2 + y2 + 2x − 2 y = 62 (b) x2 + y2 + 2x − 2 y = 47
(a) 8x2 − 9 y2 + 9 y = 18 (b) 9x2 − 8 y2 + 8 y = 16 (c) x2 + y2 − 2x + 2 y = 47 (d) x2 + y2 − 2x + 2 y = 62
(c) 9x2 + 8 y2 − 8 y = 16 (d) 8x2 + 9 y2 − 9 y = 18 10. AB is a diameter of a circle and C is any point on the
3. If a circle of radius R passes through the origin O and circumference of the circle. Then, (1983, 1M)
intersects the coordinate axes at A and B, then the locus (a) the area of ∆ ABC is maximum when it is isosceles
of the foot of perpendicular from O on AB is (b) the area of ∆ ABC is minimum when it is isosceles
(2019 Main, 12 Jan II) (c) the perimeter of ∆ ABC is minimum when it is
(a) (x2 + y2 )2 = 4R 2x2 y2 isosceles
(b) (x2 + y2 )3 = 4R 2x2 y2 (d) None of the above
(c) (x2 + y2 )(x + y) = R 2xy
11. The centre of the circle passing through the point (0, 1)
(d) (x2 + y2 )2 = 4Rx2 y2
and touching the curve y = x2 at (2, 4) is (1983, 1M)
4. A square is inscribed in the circle
(a)  − ,
16 27 
(b)  − ,
16 53 
x2 + y2 − 6x + 8 y − 103 = 0 with its sides parallel to the  
 5 10   7 10 
coordinate axes. Then, the distance of the vertex of this
(c)  −
16 53 
square which is nearest to the origin is ,  (d) None of these
(2019 Main, 11 Jan II)  5 10 
(a) 6 (b) 13 (c) 41 (d) 137
5. If the area of an equilateral triangle inscribed in the
Objective Questions II
circle, x + y + 10x + 12 y + c = 0
2 2 (One or more than one correct option)
is 27 3 sq units, then c is equal to (2019 Main, 10 Jan II) 12. Circle(s) touching X-axis at a distance 3 from the origin
(a) 20 (b) −25 (c) 13 (d) 25 and having an intercept of length 2 7 on Y-axis is/are
6. Let the orthocentre and centroid of a triangle be A(−3, 5) (a) x2 + y2 − 6x + 8 y + 9 = 0 (2013 Adv.)
and B(3, 3), respectively. If C is the circumcentre of this (b) x2 + y2 − 6x + 7 y + 9 = 0
triangle, then the radius of the circle having line (c) x2 + y2 − 6x − 8 y + 9 = 0
segment AC as diameter, is (2018 Main) (d) x2 + y2 − 6x − 7 y + 9 = 0
5 3 5 13. Let L1 be a straight line passing through the origin and
(a) 10 (b) 2 10 (c) 3 (d)
2 2 L 2 be the straight line x + y = 1. If the intercepts made
7. The centre of circle inscribed in square formed by the by the circle x2 + y2 − x + 3 y = 0 on L1 and L 2 are equal,
lines x2 − 8x + 12 = 0 and y2 − 14 y + 45 = 0, is (2003, 1M) then which of the following equation can represent L1?
(a) (4, 7) (b) (7, 4) (a) x + y = 0 (b) x − y = 0 (1999, 1M)
(c) (9, 4) (d) (4, 9) (c) x + 7 y = 0 (d) x − 7 y = 0
406 Circle

Fill in the Blanks Integer & Numerical Answer Type Questions


14. The lines 3x − 4 y + 4 = 0 and 6x − 8 y − 7 = 0 tangents to 22. For how many values of p, the circle
the same circle. The radius of this circle is… . (1984, 2M) x2 + y2 + 2x + 4 y − p = 0 and the coordinate axes have
15. If A and B are points in the plane such that PA / PB = k exactly three common points? (2017 Adv.)
(constant) for all P on a given circle, then the value of k 23. The straight line 2x − 3 y = 1 divide the circular region
cannot be equal to ……… . (1982, 2M)
x2 + y 2 ≤ 6 into two parts. If
 3   5 3   1 1   1 1  
True/False S = 2,  ,  ,  ,  ,−  ,  ,  , then the number of
 4   2 4   4 4   8 4  
16. The line x + 3 y = 0 is a diameter of the circle point (s) in S lying inside the smaller part is (2016 Adv.)
x2 + y2 − 6x + 2 y = 0. (1989, 1M)
Paragraph Based Questions
Analytical & Descriptive Questions Let S be the circle in the XY -plane defined by the equation
17. Let C be any circle with centre (0, 2 ). Prove that x2 + y2 = 4. (2018 Adv.)
at most two rational points can be there on C.
(There are two questions based on above Paragraph, the
(A rational point is a point both of whose coordinates are
question given below is one of them)
rational numbers.) (1997, 5M)
18. Consider a curve ax2 + 2hxy + by2 = 1 and a point P not 24. Let E1E 2 and F1F2 be the chords of S passing through
on the curve. A line drawn from the point P intersect the the point P0 (1, 1) and parallel to the X-axis and the
curve at points Q and R. If the product PQ ⋅ QR is Y -axis, respectively. Let G1G2 be the chord of S passing
independent of the slope of the line, then show that the through P0 and having slope −1. Let the tangents to S at
curve is a circle. (1997, 5M) E1 and E 2 meet at E3 , then tangents to S at F1 and F2
meet at F3 , and the tangents to S at G1 and G2 meet at
19. A circle passes through three points A, B and C with the G3 . Then, the points E3 , F3 and G3 lie on the curve
line segment AC as its diameter. A line passing through
(a) x + y = 4
A intersects the chord BC at a point D inside the circle. If
angles DAB and CAB are α and β respectively and the (b) (x − 4)2 + ( y − 4)2 = 16
distance between the point A and the mid-point of the (c) (x − 4) ( y − 4) = 4
line segment DC is d, prove that the area of the circle is (d) xy = 4
πd 2 cos 2 α 25. Let P be a point on the circle S with both coordinates
(1996, 5M) being positive. Let the tangent to S at P intersect the
cos α + cos β + 2 cos α cos β cos (β − α )
2 2
coordinate axes at the points M and N . Then, the
20. If (mi , 1 / mi ), mi > 0, i = 1, 2, 3, 4 are four distinct points mid-point of the line segment MN must lie on the
on a circle , then show that m1 m2 m3 m4 = 1. (1989, 2M) curve
(a) (x + y)2 = 3xy
21. The abscissae of the two points A and B are the roots of
the equation x2 + 2ax − b2 = 0 and their ordinates are the (b) x23
/
+ y23/
= 24/3
roots of the equation y2 + 2 py − q2 = 0. Find the equation (c) x + y = 2xy
2 2

and the radius of the circle with AB as diameter. (d) x2 + y2 = x2 y2


(1984, 4M)

Topic 2 Relation between Two Circles


Objective Questions I (Only one correct option) 3. If a variable line, 3x + 4 y − λ = 0 is such that the two
1. If the circles x + y + 5Kx + 2 y + K = 0
2 2
and circles x2 + y2 − 2x − 2 y + 1 = 0
2 (x2 + y2) + 2Kx + 3 y −1 = 0, (K ∈ R), intersect at the and x2 + y2 − 18x − 2 y + 78 = 0
points P and Q, then the line 4x + 5 y − K = 0 passes are on its opposite sides, then the set of all values of λ is
through P and Q, for (2019 Main, 10 April I) the interval (2019 Main, 12 Jan I)
(a) no values of K (a) [13, 23] (b) (2, 17)
(b) exactly one value of K (c) [12, 21] (d) (23, 31)
(c) exactly two values of K
4 Let C1 and C 2 be the centres of the circles
(d) infinitely many values of K
x2 + y2 − 2x − 2 y − 2 = 0 and x2 + y2 − 6x − 6 y + 14 = 0
2. If a tangent to the circle x2 + y2 = 1 intersects the respectively. If P and Q are the points of intersection of
coordinate axes at distinct points P and Q, then the locus these circles, then the area (in sq units) of the
of the mid-point of PQ is (2019 Main, 9 April I) quadrilateral PC1QC 2 is (2019 Main, 12 Jan I)

(a) x2 + y2 − 2x2 y2 = 0 (b) x2 + y2 − 2xy = 0 (a) 8 (b) 4


(c) 6 (d) 9
(c) x2 + y2 − 4x2 y2 = 0 (d) x2 + y2 − 16x2 y2 = 0
Circle 407

5. If the circles x2 + y2 −16x − 20 y + 164 = r 2 and Objective Questions II


(x − 4)2 + ( y − 7)2 = 36 intersect at two distinct points, (One or more than one correct option)
then (2019 Main, 9 Jan II)
(a) 0 < r < 1 (b) r > 11 16. Let T be the line passing through the points P(− 2, 7)
(c) 1 < r < 11 (d) r = 11 and Q(2, − 5). Let F1 be the set of all pairs of circles
6. If one of the diameters of the circle, given by the (S1 , S 2) such that T is tangent to S1 at P and tangent to
equation, x2 + y2 − 4x + 6 y − 12 = 0, is a chord of a circle S 2 at Q, and also such that S1 and S 2 touch each other at
S, whose centre is at (−3, 2), then the radius of S is a point, say M. Let E1 be the set representing the locus
(2016 Main) of M as the pair (S1 , S 2) varies in F1. Let the set of all
(a) 5 2 (b) 5 3 (c) 5 (d) 10 straight line segments joining a pair of distinct points of
7. If one of the diameters of the circle E1 and passing through the point R(1, 1) be F2. Let E 2 be
x2 + y2 − 2x − 6 y + 6 = 0 is a chord to the circle with the set of the mid-points of the line segments in the set
centre (2, 1), then the radius of the circle is (2004, 1M) F2. Then, which of the following statement(s) is (are)
(a) 3 (b) 2 (c) 3 (d) 2 TRUE? (2018 Adv.)

8. The number of common tangents to the circles (a) The point (− 2, 7) lies in E1
x + y − 4x − 6 y − 12 = 0 and x + y + 6x + 18 y + 26 = 0 (b) The point  ,  does NOT lie in E2
2 2 2 2 4 7
is (2015)  5 5

(c) The point  ,1 lies in E2


(a) 1 (b) 2 (c) 3 (d) 4 1
9. Let C be the circle with centre at (1, 1) and radius 1. If T 2 

(d) The point  0,  does NOT lie in E1


is the circle centred at (0, y) passing through origin and 3
touching the circle C externally, then the radius of T is  2
equal to (2014 Main)
3 3 1 1
17. A circle S passes through the point (0, 1) and is
(a) (b) (c) (d) orthogonal to the circles (x − 1)2 + y2 = 16 and x2 + y2 = 1.
2 2 2 4
Then, (2014 Adv.)
10. If the circle x2 + y2 + 2x + 2ky + 6 = 0 and (a) radius of S is 8
x2 + y2 + 2ky + k = 0 intersect orthogonally, then k is (b) radius of S is 7
(a) 2 or − 3 / 2 (b) −2 or − 3 / 2 (2000, 2M) (c) centre of S is (−7,1)
(c) 2 or 3 / 2 (d) −2 or 3 / 2 (d) centre of S is (−8,1)
11. The ∆ PQR is inscribed in the circle x2 + y2 = 25. If Q and
R have coordinates (3, 4) and (− 4, 3) respectively, then Passage Based Problems
∠ QPR is equal to (2000, 2M) Passage
(a) π / 2 (b) π / 3 (c) π / 4 (d) π / 6
Let ABCD be a square of side length 2 unit. C 2 is the
12. The number of common tangents to the circles circle through vertices A , B, C , D and C1 is the circle
x2 + y2 = 4 and x2 + y2 − 6x − 8 y = 24 is (1998, 2M) touching all the sides of square ABCD. L is the line
(a) 0 (b) 1 (c) 3 (d) 4 through A. (2006, 5M)
13. The angle between a pair of tangents drawn from a 18. If P is a point of C1 and Q is a point on C 2 , then
point P to the circle
PA 2 + PB2 + PC 2 + PD 2
x2 + y2 + 4 x − 6 y + 9 sin 2 α + 13 cos2 α = 0 is equal to
QA 2 + QB2 + QC 2 + QD 2
is 2α. The equation of the locus of the point P is (a) 0.75 (b) 1.25
(1996, 1M)
(c) 1 (d) 0.5
(a) x + y + 4x − 6 y + 4 = 0 (b) x + y + 4x − 6 y − 9 = 0
2 2 2 2

(c) x2 + y2 + 4x − 6 y − 4 = 0 (d) x2 + y2 + 4x − 6 y + 9 = 0 19. A circle touches the line L and the circle C1 externally
14. If the two circles (x − 1) + ( y − 3) = r 2
and 2 2 such that both the circles are on the same side of the
x2 + y2 − 8x + 2 y + 8 = 0 intersect in two distinct points, line, then the locus of centre of the circle is
then (1989, 2M) (a) ellipse (b) hyperbola
(a) 2 < r < 8 (b) r < 2 (c) r = 2 (d) r > 2 (c) parabola (d) parts of straight line

15. If a circle passes through the point (a, b) and cuts the 20. A line M through A is drawn parallel to BD. Point S
circle x2 + y2 = k2 orthogonally, then the equation of the moves such that its distances from the line BD and the
locus of its centre is (1988, 2M) vertex A are equal. If locus of S cuts M at T2 and T3 and
(a) 2ax + 2by − (a 2 + b2 + k 2 ) = 0 AC at T1, then area of ∆T1T2T3 is
(b) 2ax + 2by − (a 2 − b2 + k 2 ) = 0 1 2
(a) sq unit (b) sq unit
(c) x2 + y2 − 3ax − 4by + a 2 + b2 − k 2 = 0 2 3
(d) x2 + y2 − 2ax − 3by + (a 2 − b2 − k 2 ) = 0 (c) 1sq unit (d) 2 sq units
408 Circle

Match the Columns Analytical & Descriptive Questions


21. Match the conditions/expressions in Column I with 22. Let C1 and C 2 be two circles with C 2 lying inside C1. A
statement in Column II. circle C lying inside C1 touches C1 internally and C 2
Column I Column II externally. Identify the locus of the centre of C.
(2001, 5M)
A. Two intersecting circles p. have a common tangent
23. Three circles touch one another externally. The
B. Two mutually external circles q. have a common normal tangents at their points of contact meet at a point whose
C. Two circles, one strictly inside r. do not have a common distance from a point of contact is 4. Find the ratio of the
the other tangent product of the radii to the sum of the radii of the circles.
D. Two branches of a hyperbola s. do not have a common (1992, 5M)
normal

Topic 3 Equation of Tangent, Normal and Length of Tangents


Objective Questions I (Only one correct option)
1. A circle touches the Y -axis at the point 8. Let ABCD be a quadrilateral with area 18, with side AB
(0, 4) and passes through the point (2, 0). Which of the parallel to the side CD and AB = 2 CD . Let AD be
following lines is not a tangent to this circle? perpendicular to AB and CD. If a circle is drawn inside
(2020 Main, 9 Jan I) the quadrilateral ABCD touching all the sides, then its
(a) 4x − 3 y + 17 = 0 (b) 3x + 4 y − 6 = 0 radius is (2007, 3M)
(c) 4x + 3 y − 8 = 0 (d) 3x − 4 y − 24 = 0 3
(a) 3 (b) 2 (c) (d) 1
2. The common tangent to the circles x + y = 4 and 2 2 2
x2 + y2 + 6x + 8 y − 24 = 0 also passes through the point 9. If the tangent at the point P on the circle
(2019 Main, 9 April II ) x2 + y 2 + 6 x + 6 y = 2 meets the straight line
(a) (6, − 2) (b) (4, − 2) 5x − 2 y + 6 = 0 at a point Q on the Y-axis, then the length
(c) (−6, 4) (d) (−4, 6) of PQ is (2002, 1M)
(a) 4 (b) 2 5 (c) 5 (d) 3 5
3. A rectangle is inscribed in a circle with a diameter lying
along the line 3 y = x + 7. If the two adjacent vertices of 10. Let PQ and RS be tangents at the extremities of the
the rectangle are (–8, 5) and (6, 5), then the area of the diameter PR of a circle of radius r. If PS and RQ
rectangle (in sq units) is (2019 Main, 9 April II ) intersect at a point X on the circumference of the circle,
(a) 72 (b) 84 (c) 98 (d) 56 then 2r equals (2001, 1M)

4. The tangent and the normal lines at the point ( 3 , 1) to (a) PQ ⋅ RS


PQ + RS
the circle x + y = 4 and the X-axis form a triangle. The
2 2
(b)
area of this triangle (in square units) is 2
(2019 Main, 8 April II ) 2PQ ⋅ RS
(c)
(a)
1
(b)
4
(c)
2
(d)
1 PQ + RS
3 3 3 3
PQ 2 + RS 2
(d)
5. The straight line x + 2 y = 1 meets the coordinate axes at 2
A and B. A circle is drawn through A , B and the origin.
Then, the sum of perpendicular distances from A and B Objective Questions II
on the tangent to the circle at the origin is
(2019 Main, 11 Jan I )
(One or more than one correct option)
5 5 11. Let RS be the diameter of the circle x2 + y2 = 1, where S
(a) 2 5 (b) (c) 4 5 (d)
4 2 is the point (1, 0) . Let P be a variable point (other than R
6. If a circle C passing through the point and S) on the circle and tangents to the circle at S and P
(4, 0) touches the circle x2 + y2 + 4x − 6 y = 12 externally meet at the point Q. The normal to the circle at P
at the point (1, − 1), then the radius of C is intersects a line drawn through Q parallel to RS at point
(2019 Main,10 Jan I ) E. Then, the locus of E passes through the point(s)
(a) 5 (b) 2 5 (c) 57 (d) 4 (2016 Adv.)
(a)  ,
1 1 
(b)  , 
1 1
7. If the tangent at (1, 7) to the curve x = y − 6 touches the
2

 3 3  4 2
circle x2 + y2 + 16x + 12 y + c = 0, then the value of c is
(c)  , −
1 
(d)  , − 
1 1 1
(2018 Main) 
(a) 195 (b) 185 (c) 85 (d) 95 3 3 4 2
Circle 409

12. The circle C1 : x2 + y2 = 3 with centre at O intersects the 17. Points E and F are given by
parabola x = 2 y at the point P in the first quadrant. Let
2  3 3  3 1
(a)  ,  , ( 3 , 0) (b)  ,  , ( 3 , 0)
the tangent to the circle C1 at P touches other two circles  2 2  2 2
C 2 and C3 at R2 and R3 , respectively. Suppose C 2 and C3  3 3  3 1  3 3   3 1
have equal radii 2 3 and centres Q2 and Q3 , (c)  , ,  ,  (d)  , ,  , 
 2 2  2 2  2 2   2 2
respectively. If Q2 and Q3 lie on the Y-axis, then
(a) Q2Q3 = 12 (2016 Adv.) 18. Equations of the sides QR, RP are
(b) R2R3 = 4 6 2 2
(a) y = x + 1, y = − x−1
(c) area of the ∆OR2R3 is 6 2 3 3
(d) area of the ∆PQ2Q3 is 4 2 1
(b) y = x, y = 0
3
Assertion and Reason (c) y =
3
x + 1, y = −
3
x−1
2 2
13. Tangents are drawn from the point (17, 7) to the circle (d) y = 3x, y = 0
x2 + y2 = 169.
Statement I The tangents are mutually perpendicular. Fill in the Blanks
because
19. A circle is inscribed in an equilateral triangle of side a.
Statement II The locus of the points from which a The area of any square inscribed in this circle is… .
mutually perpendicular tangents can be drawn to the (1994, 2M)
given circle is x2 + y2 = 338. (2007, 3M)
20. If a circle passes through the points of intersection of
(a) Statement I is true, Statement II is true; Statement II
the coordinate axes with the lines λx − y + 1 = 0 and
is correct explanation of Statement I
x − 2 y + 3 = 0, then the value of λ is ... . (1991,2M)
(b) Statement I is true, Statement II is true, Statement II
is not correct explanation of Statement I.
(c) Statement I is true, Statement II is false.
Analytical & Descriptive Questions
(d) Statement I is false, Statement II is true. 21. Find the equation of circle touching the line
2x + 3 y + 1 = 0 at the point (1, –1) and is orthogonal to
Passage Based Problems the circle which has the line segment having end points
(0, –1) and (–2, 3) as the diameter. (2004, 4M)
Passage 1
22. Find the coordinates of the point at which the circles
A tangent PT is drawn to the circle x2 + y2 = 4 at the
x2 − y2 − 4x − 2 y + 4 = 0 and x2 + y2 − 12x − 8 y + 36 = 0
point P ( 3 , 1). A straight line L, perpendicular to PT is
a tangent to the circle (x − 3)2 + y2 = 1. (2012) touch each other. Also, find equations of common
tangents touching the circles the distinct points.
14. A possible equation of L is (1993, 5M)
(a) x − 3 y = 1 (b) x + 3y = 1
23. Two circles, each of radius 5 units, touch each other at
(c) x − 3 y = − 1 (d) x − 3y = 5
(1, 2). If the equation of their common tangent is
15. A common tangent of the two circles is 4x + 3 y = 10, find the equations of the circles. (1991, 4M)
(a) x = 4 (b) y = 2
(c) x + 3 y = 4 (d) x + 2 2 y = 6 Integer & Numerical Answer Type Questions
Passage 2 24. Let the point B be the reflection of the point A( 2, 3) with
respect to the line 8x − 6 y − 23 = 0. Let ΓA and ΓB be
A circle C of radius 1 is inscribed in an equilateral circles of radii 2 and 1 with centres A and B
∆PQR. The points of contact of C with the sides PQ, QR, respectively. Let T be a common tangent to the circles
RP are D, E, F respectively. The line PQ is given by the ΓA and ΓB such that both the circles are on the same side
 3 3 3 of T. If C is the point of intersection of T and the line
equation 3 x + y − 6 = 0 and the point D is  , .
 2 2 passing through A and B, then the length of the line
Further, it is given that the origin and the centre of C segment AC is ........... (2019 Adv.)
are on the same side of the line PQ. (2008, 12M) 25. The centres of two circles C1 and C 2 each of unit radius
16. The equation of circle C is are at a distance of 6 units from each other. Let P be the
(a) (x − 2 3 )2 + ( y − 1)2 = 1 mid-point of the line segment joining the centres of C1
2 and C 2 and C be a circle touching circles C1 and C 2
(b) (x − 2 3 )2 +  y +  = 1
1
 2 externally. If a common tangents to C1 and C passing
through P is also a common tangent to C 2 and C , then the
(c) (x − 3 )2 + ( y + 1)2 = 1
radius of the circle C is … (2009)
(d) (x − 3 )2 + ( y − 1)2 = 1
410 Circle

Topic 4 Radical Axis and Family of Circle


Objective Questions I (Only one correct option) 11. The equation of the circle passing through (1, 1) and
the points of intersection of x2 + y2 + 13x − 3 y = 0 and
1. The locus of the centres of the circles, which touch the
2x2 + 2 y2 + 4x − 7 y − 25 = 0 is (1983, 1M)
circle, x2 + y2 = 1 externally, also touch the Y-axis and
lie in the first quadrant, is (2019 Main, 10 April II)
(a) 4x2 + 4 y2 − 30x − 10 y = 25
(b) 4x2 + 4 y2 + 30x − 13 y − 25 = 0
(a) y = 1 + 2x, x ≥ 0 (b) y = 1 + 4x, x ≥ 0 (c) 4 x2 + 4 y2 − 17x − 10 y + 25 = 0
(c) x = 1 + 2 y, y ≥ 0 (d) x = 1 + 4 y, y ≥ 0 (d) None of the above
2. The line x = y touches a circle at the point (1, 1). If the 12. Two circles x2 + y2 = 6 and x2 + y2 − 6x + 8 = 0 are given.
circle also passes through the point (1, − 3), then its Then the equation of the circle through their points of
radius is (2019 Main, 10 April I) intersection and the point (1, 1) is (1980, 1M)
(a) 3 2 (b) 2 2 (c) 2 (d) 3 (a) x2 + y2 − 6 x + 4 = 0 (b) x2 + y2 − 3x + 1 = 0
(c) x2 + y2 − 4 y + 2 = 0 (d) None of these
3. Two circles with equal radii are intersecting at the
points (0, 1) and (0, −1). The tangent at the point (0,1) to
one of the circles passes through the centre of the other
Fill in the Blanks
circle. Then, the distance between the centres of these 13. For each natural number k. Let C k denotes the circle
circles is (2019 Main, 11 Jan I) with radius k centimetres and centre at origin. On the
(a) 2 (b) 2 2 (c) 1 (d) 2 circle C k a particle moves k centimetres in the
4. Three circles of radii a , b, c(a < b < c) touch each other counter-clockwise direction. After completing its motion
on C k the particle moves to C k + 1 in the radial direction.
externally. If they have X-axis as a common tangent,
The motion of the particle continue in this manner. The
then (2019 Main, 9 Jan I)
1 1 1 particle starts at (1, 0). If the particle crosses the
(a) a , b, c are in AP (b) = + positive direction of the X-axis for the first time on the
a b c
1 1 1
circle C n, then n = … (1997, 2M)
(c) a , b , c are in AP (d) = +
b a c 14. The intercept on the line y = x by the circle x2 + y2 − 2x = 0
5. The circle passing through (1, − 2) and touching the axis is AB. Equation of the circle with AB as a diameter is… .
(1996, 1M)
of x at (3, 0) also passes through the point (2013 Main)
(a) (− 5, 2) (b) (2, − 5) (c) (5, − 2) (d) (− 2, 5) 15. If the circle C1 : x + y = 16 intersects another circle C 2
2 2

6. The circle passing through the point (− 1, 0) and of radius 5 in such a manner that the common chord is
of maximum length and has a slope equal to 3/4, then
touching the Y-axis at (0, 2) also passes through the the coordinates of the centre of C 2 are… . (1988, 2M)
point (2011)
16. The points of intersection of the line 4x − 3 y − 10 = 0 and
(a)  − , 0 (b)  − , 2 (c)  − , 
3 5 3 5
(d) (− 4, 0)
 2   2   2 2 the circle x2 + y2 − 2x + 4 y − 20 = 0 are…and….(1983, 2M)
7. The locus of the centre of circle which touches Analytical & Descriptive Questions
( y − 1)2 + x2 = 1 externally and also touches X-axis, is
(a) {x = 4 y, y ≥ 0} ∪ {(0, y), y< 0}
2
(2005, 2M) 17. Consider the family of circles x2 + y2 = r 2 , 2 < r < 5. If in
(b) x2 = y the first quadrant, the common tangent to a circle of
(c) y = 4x2 this family and the ellipse 4x2 + 25 y2 = 100 meets the
(d) y2 = 4x ∪ (0, y), y∈R coordinate axes at A and B, then find the equation of the
8. If two distinct chords, drawn from the point ( p, q) on the locus of the mid-points of AB. (1999, 5M)

circle x2 + y2 = px + qy (where, pq ≠ 0) are bisected by 18. Consider a family of circles passing through two fixed
points A (3, 7) and B (6, 5). Show that the chords in
the X-axis, then (1999, 2M)
which the circle x2 + y2 − 4x − 6 y − 3 = 0 cuts the
(a) p 2 = q2 (b) p 2 = 8q2 members of the family are concurrent at a point. Find
(c) p 2 < 8q2 (d) p 2 > 8q2 the coordinates of this point. (1993, 5M)
9. The locus of the centre of a circle, which touches 19. A circle touches the line y = x at a point P such that
externally the circle x2 + y2 − 6x − 6 y + 14 = 0 and also OP = 4 2, where O is the origin. The circle contains the
touches the Y-axis, is given by the equation (1993, 1M) point (− 10, 2) in its interior and the length of its chord
(a) x2 − 6x − 10 y + 14 = 0 (b) x2 − 10x − 6 y + 14 = 0 on the line x + y = 0 is 6 2. Determine the equation of
(c) y2 − 6x − 10 y + 14 = 0 (d) y2 − 10x − 6 y + 14 = 0 the circle. (1990, 5M)

10. The centre of a circle passing through the points (0, 0), 20. Let S ≡ x2 + y2 + 2 gx + 2 fy + c = 0 be a given circle. Find
(1, 0) and touching the circle x + y = 9 is
2 2
(1992, 2M) the locus of the foot of the perpendicular drawn from the
(a) (3/2, 1/2) (b) (1/2, 3/2) origin upon any chord of S which subtends a right angle
(c) (1/2, 1/2) (d) (1 / 2, − 21/ 2 ) at the origin. (1988, 5M)
Circle 411

21. Let a given line L1 intersect the X and Y-axes at P and Q 22. Find the equations of the circles passing through (−4, 3)
respectively. Let another line L 2, perpendicular to L1, and touching the lines x + y = 2 and x − y = 2. (1982, 3M)
cut the X and Y-axes at R and S, respectively. Show that
23. Find the equation of the circle which passes through the
the locus of the point of intersection of the line PS and
point (2, 0) and whose centre is the limit of the point of
QR is a circle passing through the origin. (1987, 3M)
intersection of the lines 3x + 5 y = 1, (2 + c)x + 5c2y = 1 as c
tends to 1. (1979, 3M)

Topic 5 Equation of Chord Bisected at a Point, Product of Pair of


Tangents, Chord of Contact of Tangents, Pole and
Equation of Polar
Objective Questions I (Only one correct option) 7. The locus of the mid-point of a chord of the circle
x2 + y2 = 4 which subtends a right angle at the origin, is
1. A line y = mx + 1 intersects the circle
(a) x + y = 2
(x − 3)2 + ( y + 2)2 = 25 at the points P and Q. If the
3 (b) x2 + y2 = 1 (1984, 2M)
midpoint of the line segment PQ has x-coordinate − , (c) x2 + y2 = 2
5
(d) x + y = 1
then which one of the following options is correct?
(a) 6 ≤ m < 8 (b) − 3 ≤ m < − 1 (2019 Adv.)
(c) 4 ≤ m < 6 (d) 2 ≤ m < 4
Objective Question II
2. If the angle of intersection at a point where the two (One or more than one correct option)
circles with radii 5 cm and 12 cm intersect is 90°, then 8. The equations of the tangents drawn from the origin to
the length (in cm) of their common chord is the circle x2 + y2 + 2rx + 2hy + h 2 = 0, are (1988, 2M)
(2019 Main, 12 April I) (a) x = 0
13 120 60 13
(a) (b) (c) (d) (b) y = 0
5 13 13 2
(c) (h 2 − r 2 ) x − 2rhy = 0
3. The sum of the squares of the lengths of the chords (d) (h 2 − r 2 ) x + 2rhy = 0
intercepted on the circle, x2 + y2 = 16, by the lines,
x + y = n, n ∈ N , where N is the set of all natural Assertion and Reason
numbers, is (2019, Main, 8 April I)
(a) 320 (b) 105
For the following questions, choose the correct answer
from the codes (a), (b), (c) and (d) defined as follows.
(c) 160 (d) 210
(a) Statement I is true, Statement II is also true;
4. The centres of those circles which touch the circle, Statement II is the correct explanation of Statement I.
x + y − 8x − 8 y − 4 = 0, externally and also touch the
2 2
(b) Statement I is true, Statement II is also true;
X-axis, lie on (2016 Main)
Statement II is not the correct explanation of
(a) a circle Statement I
(b) an ellipse which is not a circle (c) Statement I is true; Statement II is false
(c) a hyperbola
(d) Statement I is false; Statement II is true
(d) a parabola
9. Consider L1 : 2x + 3 y + p − 3 = 0
5. The locus of the mid-point of the chord of contact of
tangents drawn from points lying on the straight line L2 : 2x + 3 y + p + 3 = 0
4x − 5 y = 20 to the circle x2 + y2 = 9 is (2012)
where, p is a real number and
(a) 20 (x +
2
y ) − 36x + 45 y = 0
2 C : x2 + y2 − 6x + 10 y + 30 = 0
(b) 20 (x2 + y2 ) + 36x − 45 y = 0 Statement I If line L1 is a chord of circle C, then line L 2
(c) 36 (x2 + y2 ) − 20 y + 45 y = 0 is not always a diameter of circle C.
(d) 36 (x2 + y2 ) + 20x − 45 y = 0 Statement II If line L1 is a diameter of circle C, then
6. Tangents drawn from the point P (1, 8) to the circle line L 2 is not a chord of circle C. (2008, 3M)
x2 + y2 − 6x − 4 y − 11 = 0 touch the circle at the points A
and B. The equation of the circumcircle of the ∆PAB is Fill in the Blanks
(a) x2 + y2 + 4x − 6 y + 19 = 0 (2009)
10. The chords of contact of the pair of tangents drawn from
(b) x2 + y2 − 4x − 10 y + 19 = 0 each point on the line 2x + y = 4 to the circle x2 + y2 = 1
(c) x2 + y2 − 2x + 6 y − 29 = 0 pass through the point… . (1997, 2M)
(d) x2 + y2 − 6x − 4 y + 19 = 0
412 Circle

11. The equation of the locus of the mid-points of the 20. Find the intervals of values of a for which the line
chords of the circle 4x2 + 4 y2 − 12x + 4 y + 1 = 0 that y + x = 0 bisects two chords drawn from a point
subtend an angle of 2π / 3 at its centre is … .(1993, 2M)  1 + 2a 1 − 2a 
 ,  to the circle
12. The area of the triangle formed by the tangents from  2 2 
the point (4, 3) to the circle x2 + y2 = 9 and the line
2x2 + 2 y2 − (1 + 2a ) x − (1 − 2a ) y = 0. (1996, 6M)
joining their points of contact is… . (1987, 2M)

13. From the point A (0, 3) on the circle


21. Let a circle be given by
x + 4x + ( y − 3) = 0, a chord AB is drawn and
2 2 2x (x − a ) + y (2 y − b) = 0, (a ≠ 0, b ≠ 0)
extended to a point M such that AM = 2 AB. The Find the condition on a and b if two chords, each bisected by
equation of the locus of M is…. the X-axis can be drawn to the circle from (a , b /2).
(1986, 2M) (1992, 6M)
14. The equation of the line passing through the points of 22. Lines 5x + 12 y − 10 = 0 and 5x − 12 y − 40 = 0 touch a circle
intersection of the circles 3x2 + 3 y2 − 2x + 12 y − 9 = 0 C1 of diameter 6. If the centre of C1 lies in the first
and x2 + y2 + 6x + 2 y − 15 = 0 is… . (1986, 2M)
quadrant, find the equation of the circle C 2 which is
15. Let x2 + y2 − 4x − 2 y − 11 = 0 be a circle. A pair of concentric with C1 and cuts intercepts of length 8 on these
tangents from the point (4, 5) with a pair of radii form lines. (1986, 5M)
a quadrilateral of area... . (1985, 2M) 23. Through a fixed point (h , k) secants are drawn to the
16. From the origin chords are drawn to the circle circle x2 + y2 = r 2. Show that the locus of the mid-points of
(x − 1)2 + y2 = 1. The equation of the locus of the mid the secants intercepted by the circle is x2 + y2 = hx + ky.
(1983, 5M)
points of these chords is… . (1985, 2M)
24. Let A be the centre of the circle x2 + y2 − 2x − 4 y − 20 = 0.
Analytical & Descriptive Questions Suppose that, the tangents at the points B (1, 7) and D
17. Let 2x + y − 3xy = 0 be the equation of a pair of
2 2 (4, −2 ) on the circle meet at the point C. Find the area of
the quadrilateral ABCD. (1981, 4M)
tangents drawn from the origin O to a circle of radius 3
with centre in the first quadrant. If A is one of the Integer & Numerical Answer Type Questions
points of contact, find the length of OA. (2001, 5M)
5
18. Let T1 , T2 and be two tangents drawn from (−2, 0) onto 25. Let O be the centre of the circle x2 + y2 = r 2, where r > .
2
the circle C : x2 + y2 = 1. Determine the circles Suppose PQ is a chord of this circle and the equation of
touching C and having T1 , T2 as their pair of tangents. the line passing through P and Q is 2x + 4 y = 5. If the
Further, find the equations of all possible common centre of the circumcircle of the triangle OPQ lies on the
tangents to these circles when taken two at a time. line x + 2 y = 4, then the value of r is ……… . (2020 Adv.)
(1999, 10M)
26. Two parallel chords of a circle of radius 2 are at a distance
19. C1 and C 2 are two concentric circles, the radius of C 2 3 + 1 apart. If the chords subtend at the centre, angles of
being twice that of C1. From a point P on C 2, tangents 2π
π /k and , where k > 0, then the value of [k] is……
PA and PB are drawn to C1. Prove that the centroid of k (2010)
the ∆PAB lies on C1. (1998, 8M) NOTE [k ] denotes the largest integer less than or equal to k]

Answers
Topic 1 13. (d) 14. (a) 15. (a) 16. (a,d)
1. (a) 2. (c) 3. (b) 4. (c) 17. (b, c) 18. (a) 19. (c) 20. (c)
5. (d) 6. (c) 7. (a) 8. (b) 21. A→p, q; B→p, q; C→q, r; D→q, r
9. (c) 10. (d) 11. (c) 12. (a, c) 22. Ellipse having foci are (a, b) and (0, 0)
3
13. (b, c) 14. 15. k ≠ 1 16. True 23. 16 : 1
4
21. x 2 + y 2 + 2ax + 2 py − (b 2 + q 2 ) = 0, Topic 3
radius = a + p + b + q
2 2 2 2 1. (c) 2. (a) 3. (b) 4. (c)
5. (d) 6. (a) 7. (d) 8. (b)
22. (2) 23. (2) 24. (a) 25. (d)
9. (c) 10. (a) 11. (b, d) 12. (a, b, c)
Topic 2 13. (c) 14. (a) 15. (d) 16. (d)
1. (a) 2. (c) 3. (c) 4. (b) a2 1
17. (a) 18. (a) 19. sq unit 20. λ = 2 or −
5. (c) 6. (b) 7. (c) 8. (c) 6 2
9. (d) 10. (a) 11. (c) 12. (b) 21. 2 x 2 + 2y 2 − 10 x − 5y + 1 = 0
Circle 413

22. y = 0 and 7y − 24 x + 16 = 0 Topic 5


23. ( x − 5 ) 2 + (y − 5 ) 2 = 5 2 and ( x + 3 ) 2 + (y + 1 ) 2 = 5 2 1. (d) 2. (b) 3. (d) 4. (d)
24. (10) 25. (8) 5. (a) 6. (b) 7. (c) 8. (a,c)
 1 1
Topic 4 9. (c) 10.  , 
 2 4
1. (a) 2. (b) 3. (d) 4. (b)
192
5. (c) 6. (d) 7. (a) 8. (d) 11. 16 x 2 + 16y 2 − 48 x + 16y + 31 = 0 12. sq units
25
9. (d) 10. (b) 11. (b) 12. (b)
13. n = 7 13. x 2 + y 2 + 8 x − 6y + 9 = 0 14. 10 x − 3y − 18 = 0
 9 12  9 12 15. 8 sq units 16. x 2 + y 2 − x = 0
14. x 2 + y 2 − x − y = 0 15.  − ,  and  ,− 
 5 5 5 5  17. 3 (3 + 10 )
16. ( −2, − 6 ) and ( 4, 2 ) 17. 4 x + 25y = 4 x y
2 2 2 2
 4
2
 1
2
5  4
18.  x +  + y 2 =   ; y = ± x + 
18. x = 2 and y = 23 / 3 19. x 2 + y 2 + 18 x − 2y + 32 = 0  3  3 39  5

20. x 2 + y 2 + gx + f y +
c
=0 20. a ∈ ( −∞, − 2 ) ∪ (2, ∞ ) 21. a 2 > 2b 2
2
22. ( x − 5 ) 2 + (y − 2 ) 2 = 5 2 23. x 2 + y 2 = hx + ky
22. x 2 + y 2 + 2 (10 ± 3 6 ) x + (55 ± 24 6 ) = 0
24. 75 sq units 25. (2) 26. (3)
23. 25 ( x 2 + y 2 ) − 20 x + 2y − 60 = 0

Hints & Solutions


Topic 1 Equation of Circle ⇒ (0 − 0)2 + (0 − 1)2 + (0 − x)2 + (1 − y)2 + x2 + y2 = 4
1. It is given that the circle touches the X-axis at (3, 0) and
⇒1 + x2 + ( y − 1 ) 2 + x2 + y2 = 4
making an intercept of 8 on the Y -axis.
Y ⇒ x2 + y2 − 2 y + 1 + x2 + y2 = 3

⇒ x2 + y2 − 2 y + 1 = 3 − x2 + y2
B
⇒ x2 + y2 − 2 y + 1 = 9 + x2 + y2 − 6 x2 + y2
M [squaring both sides]
8 C
⇒ 1 − 2 y = 9 − 6 x2 + y2
r r
A ⇒ 6 x2 + y2 = 2 y + 8
X
O (3,0)
⇒ 3 x2 + y2 = y + 4
Let the radius of the circle is ‘r’, then the coordinates of ⇒ 9(x2 + y2) = ( y + 4)2 [squaring both sides]
centre of circle are (3, r ).
⇒ 9x + 9 y = y + 8 y + 16
2 2 2
From the figure, we have
CM = 3 ⇒ CA = radius = r ⇒ 9x2 + 8 y2 − 8 y = 16
AB Thus, the locus of point P (x, y) is
and AM = BM = =4
2 9x2 + 8 y2 − 8 y = 16
Then, r 2 = CM 2 + AM 2
3. Let the foot of perpendicular be P (h , k). Then, the slope
= 9 + 16 = 25
k
⇒ r=±5 of line OP =
Now, the equation of circle having centre (3, ± 5) and h
radius = 5 is Y
(x − 3)2 + ( y ± 5)2 = 25

Now, from the options (3, 10) satisfy the equation of B


circle P(h, k)
(x − 3)2 + ( y − 5)2 = 25
2. Given vertices of ∆AOP are O(0, 0) and A(0, 1) A
X
O
Let the coordinates of point P are (x, y).
Clearly, perimeter = OA + AP + OP = 4 (given)
414 Circle

Q Line AB is perpendicular to line OP, so slope of line ∴ B (11, − 12) and D (− 5, 4)


h
AB = − [Q product of slopes of two Now, OA = 25 + 144 = 169 = 13;
k
perpendicular lines is (−1)] OB = 121 + 144 = 265
Now, the equation of line AB is OC = 121 + 16 = 137
h
y − k = − (x − h ) ⇒ hx + ky = h 2 + k2 and OD = 25 + 16 = 41
k
x y
or + =1 5. Clearly, centre of the circumscribed circle is the
 h 2 + k2  h 2 + k2 centroid (G) of the equilateral triangle ABC.
   
 h   k  [Q in an equilateral triangle circumcentre and centroid
coincide]
 h 2 + k2   h 2 + k2
So, point A  , 0 and B0,  C
 h   k 
Q ∆AOB is a right angled triangle, so AB is one of the 60°
diameter of the circle having radius R (given).
(–5,–6)
⇒ AB = 2R
G
2 2 r r
 h 2 + k2  h 2 + k2 120°
⇒   +  = 2R
 h   k  A B
 1 1
⇒ (h + k )  2 + 2 = 4R2
2 2 2
h k 
⇒ (h + k2)3 = 4R2h 2k2
2 Also, we know that
On replacing h by x and k by y, we get ∆AGB ≅ ∆BGC ≅ ∆CGA [by SAS congruence rule]
∴ ar (∆ABC ) = 3 ar (∆AGB)
(x2 + y2)3 = 4R2x2y2,
1 
which is the required locus. = 3  r 2 sin 120°
2 
4. Given equation of circle is x2 + y2 − 6x + 8 y − 103 = 0, 1
[Q area of triangle = ab sin (∠C )]
which can be written as (x − 3)2 + ( y + 4)2 = 128 = (8 2 )2 2
∴ Centre = (3, − 4) and radius = 8 2 Q ar (∆ABC ) = 27 3 [given]
Now, according to given information, we have the 3 2 3
∴ r = 27 3
following figure. 2 2
3
Y [sin 120° = sin (180° − 60° ) = sin 60° = ]
2
⇒ r2 = 4 × 9 ⇒ r = 6
D C
X Now, radius of circle,
O
45° r= g2 + f 2 − c
G
(3,–4) ⇒ 6 = 25 + 36 − c
[Q in the given equation of circle 2 g = 10
A B and 2 f = 12 ⇒ g = 5 and f = 6]
⇒ 36 = 25 + 36 − c
For the coordinates of A and C. ⇒ c = 25
x−3 y+ 4 6. Key idea Orthocentre, centroid and circumcentre are
Consider, = =±8 2
1 1 collinear and centroid divide orthocentre and
2 2 circumcentre in 2 : 1 ratio.
We have orthocentre and centroid of a triangle be
[using distance (parametric) form of line,
x − x1 y − y1 A(−3, 5) and B(3, 3) respectively and C circumcentre.
= = r]
cos θ sin θ
⇒ x = 3 ± 8, y = − 4 ± 8
∴ A(− 5, − 12) and C (11, 4) Clearly, AB = (3 + 3)2 + (3 − 5)2 = 36 + 4 = 2 10
Similarly, for the coordinates of B and D, consider
x−3 y+ 4 We know that, AB : BC = 2 : 1
= = ±8 2 [in this case, θ = 135°] ⇒ BC = 10
1 1

2 2 Now, AC = AB + BC = 2 10 + 10 = 3 10
⇒ x = 3 m 8, y = − 4 ± 8 Since, AC is a diameter of circle.
Circle 415

AC 3 10 5 11. Let centre of circle be (h , k) .


∴ r= ⇒ r= =3
2 2 2 so that OA 2 = OB2
7. Given, circle is inscribed in square formed by the lines
B (0,1)
x2 − 8x + 12 = 0 and y2 − 14 y + 45 = 0
⇒ x = 6 and x = 2, y = 5 and y = 9
which could be plotted as
A (2,4)
Y D (2,9) C (6,9) O (h,k )
y=9

y = x2
T

y=5 ⇒ h 2 + (k − 1)2 = (h − 2)2 + (k − 4)2


A (2,5) B (6,5)
⇒ 4h + 6k − 19 = 0 …(i)
X X k −4
O Y Also, slope of OA = and slope of tangent at (2, 4) to
h −2
where, ABCD clearly forms a square. y = x2 is 4.
∴ Centre of inscribed circle and (slope of OA) ⋅ (slope of tangent at A) = − 1
= Point of intersection of diagonals k −4
= Mid-point of AC or BD ∴ ⋅4 = − 1
h −2
 2 + 6  5 + 9
=  ,  = (4, 7) ⇒ 4k − 16 = − h + 2
 2   2 
h + 4k = 18 …(ii)
⇒ Centre of inscribed circle is (4, 7). On solving Eqs. (i) and (ii), we get
8. Choosing OA as X-axis, A = (r , 0), B = (0, r ) and any k=
53
and h = −
16
point P on the circle is (r cos θ , r sin θ ). If (x, y) is the 10 5
centroid of ∆ PAB, then  16 53
∴ Centre coordinates are  − , .
Y B (0,r )  5 10
P 12. PLAN

X′ O X (–g, –f )
A (r,0)

Y′
3x = r cos θ + r + 0 and 3 y = r sin θ + 0 + r
∴ (3x − r )2 + (3 y − r )2 = r 2 Here, the length of intercept on Y-axis is ⇒ 2 f 2 − c
Hence, locus of P is a circle. and if circle touches X-axis
9. Since, 2x − 3 y = 5 and 3x − 4 y = 7 are diameters of a ⇒ g2 = c
circle. for x + y + 2 gx + 2 fy + c = 0
2 2

Their point of intersection is centre (1, − 1) . Here, x2 + y2 + 2 gx + 2 fy + c = 0


7
Also given, πr 2 = 154 ⇒ r 2 = 154 × ⇒ r=7
22
∴ Required equation of circle is A

(x − 1) + ( y + 1) = 7
2 2 2
2√7
⇒ x + y − 2x + 2 y = 47
2 2
B
10. Clearly, ∠C = 90° as angle in semi-circle is right angled. A′ (3, 0)
Now, area of triangle is maximum when AC = BC.
2√7
i.e. Triangle is right angled isosceles.
B′
C

passes through (3, 0).


A B ⇒ 9 + 6g + c = 0 …(i)
g2 = c …(ii)
and 2 f2 − c =2 7
416 Circle

f2 − c = 7 …(iii) On subtracting Eq. (ii) from Eq. (iii), we get


From Eqs. (i) and (ii), we get p1 − 2 q1 = 0
g 2 + 6 g + 9 = 0 ⇒ ( g + 3 )2 = 0 where, p1 = x22 + y22 − x12 − y12,
⇒ g = − 3 and c = 9 q1 = y2 − y1
∴ f 2 = 16 ⇒ f =±4 On subtracting Eq. (ii) from Eq. (iv), we get
∴ x2 + y2 − 6x ± 8 y + 9 = 0 p2 − 2q2 = 0
13. Let equation of line L1 be y = mx. Intercepts made by L1 where p2 = x32 + y32 − x12 − y12, q2 = y3 − y1
and L 2 on the circle will be equal i.e. L1 and L 2 are at the Now, p1 , p2 , q1 , q2 are rational numbers. Also, either
same distance from the centre of the circle; q1 ≠ 0 or q2 ≠ 0. If q1 ≠ 0, then 2 = p1 / q1 and if q2 ≠ 0,
Centre of the given circle is (1/2, – 3/2). Therefore, then 2 = p2 / q2. In any case 2 is a rational number.
m 3 This is a contradiction.
1 /2 – 3 /2 – 1 + m+3
2 18. The given circle is ax2 + 2hxy + by2 = 1
= 2 2 ⇒ = …(i)
1+1 2
m –1 2 2 m2 + 1 Let the point P not lying on Eq. (i) be (x1 , y1 ), let θ be
the inclination of line through P which intersects the
⇒ 8m + 8 = m + 6m + 9
2 2
given curve at Q and R.
⇒ 7m – 6 m – 1 = 0 ⇒ (7m + 1)(m – 1) = 0
2
Then, equation of line through P is
1
⇒ m = – , m =1 x − x1 y − y1
7 = =r
cos θ sin θ
Thus, two chords are x + 7 y = 0
⇒ x = x1 + r cos θ , y = y1 + r sin θ
and x – y = 0.
For points Q and R, above point must lie on Eq. (i).
Therefore, (b) and (c) are correct answers.
⇒ a (x1 + r cos θ )2 + 2h (x1 + r cos θ ) ( y1 + r sin θ )
7
14. Since, 3x − 4 y + 4 = 0 and 3x − 4 y − = 0 are two parallel + b ( y1 + r sin θ )2 = 1
2
tangents. Thus, distance between them is diameter of ⇒ (a cos 2 θ + 2h sin θ cos θ + b sin 2 θ )r 2
circle + 2 (ax1 cos θ + hx1 sin θ + hy1 cos θ + by1 sin θ )r
7
4+ + (ax12 + 2hx1 y1 + by12 − 1) = 0
2 15 3
⇒ Diameter = = = It is quadratic in r, giving two values of r as PQ and PR.
32 + 42 2 . 5 2
a x12 + 2hx1 y1 + by12 − 1
3 ∴ PQ ⋅ PR =
and radius = a cos θ + 2h sin θ cos θ + b sin 2 θ
2
4
Here, ax12 + 2hx1 y1 + by12 − 1 ≠ 0, as (x1 , y1 ) does not lie on
15. Since, P lies on circle and A and B are points in plane Eq. (i),
PA
such that, = k, then the locus of P is perpendiular Also, a cos 2 θ + 2h sin θ cos θ + b sin 2 θ
PB
bisector of AB. Thus, the value of k ≠ 1. = a + 2 h sin θ cos θ + (b − a )sin 2 θ
16. Since, centre of circle is (3, − 1) which lies on x + 3 y = 0 = a + sin θ {2h cos θ + (b − a ) sin θ }
⇒ x + 3 y = 0 is diameter of x2 + y2 − 6x + 2 y = 0 = a + sin θ ⋅ 4h 2 + (b − a )2 ⋅ (cos θ sin φ + sin θ cos φ )
Hence, given statement is true. b−a
where, tan θ =
17. Equations of any circle C with centre at (0, 2 ) is given 2h
by = a + 4h 2 + (b − a )2 sin θ sin (θ + φ )
(x − 0) + ( y − 2 ) = r
2 2 2
which will be independent of θ, if
or x2 + y2 − 2 2 y + 2 = r 2 …(i) 4h 2 + (b − a )2 = 0
where, r > 0. ⇒ h = 0 and b = a
Let (x1 , y1 ), (x2, y2), (x3 , y3 ) be three distinct rational 1
points on circle. Since, a straight line parallel to X-axis ∴ Eq. (i) reduces to x2 + y2 =
a
meets a circle in at most two points, either
y1 , y2 or y1 , y3 . which is a equation of circle.
On putting these in Eq. (i), we get 19. Let the radius of the circle be r. Take X-axis along AC
x12 + y12 − 2 2 y1 = r − 2
2
…(ii) and the O (0, 0) as centre of the circle. Therefore,
coordinate of A and C are (− r , 0) and (r , 0), respectively.
x22 + y22 − 2 2 y2 = r 2 − 2 …(iii) Now, ∠ BAC = β , ∠ BOC = 2 β
x32 + y32 − 2 2 y3 = r 2 − 2 …(iv) Therefore, coordinates of B are (r cos 2 β , r sin 2 β ).
Circle 417

And slope of AD is tan (β − α ). Therefore, coordinates of D are


Let (x, y) be the coordinates of the point D. Equation of  r cos (2 β − α ) 2r cos β sin (α − β )
 ,− 
AD is  cos α cos α 
Y
B (r cos 2 β, r sin 2 β) Thus, coordinates of E are
P  r cos (2 β − α ) + r cos α cos β sin (α − β )
 ,−r 
D E  2 cos α cos α 
X′ X
A(−r, 0) O C (r,0)  2β − α + α   2β − α − α 
2 cos   ⋅ cos  
 2   2 
⇒ r ,
2 cos α
cos β sin (β − α )
Y′ r
cos α
y = tan (β − α ) (x + r ) …(i) cos β ⋅ cos (β − α ) cos β sin (β − α )
[Q slope = tan (β − α ) and point is (− r, 0)] ⇒ r ,r
cos α cos α
Now, equation of BC is
Since, AE = d, we get
r sin 2 β − 0 2 2
y= (x − r )  cos β cos (β − α )   cos β sin (β − α ) 
r cos 2 β − r d2 = r2  + 1 + r 2  
 cos α   cos α 
r ⋅ 2 sin β cos β
⇒ y= (x − r ) r2
r (− 2 sin 2 β ) = [cos 2 β cos 2 (β − α ) + cos 2 α
cos 2 α
2 sin β cos β
⇒ y= (x − r ) + 2 cos β cos (β − α ) cos α + cos 2 β sin 2 (β − α )]
−2 sin 2 β
r2
⇒ y = − cot β (x − r ) …(ii) = [cos 2 β {cos 2 (β − α ) + sin 2 (β − α ) } + cos 2 α
cos 2 α
To obtain the coordinate of D, solve Eqs. (i) and (ii) + 2cos β cosα cos ( β − α )]
simultaneously r2
= [cos β + cos α + 2 cos α cos β cos (β − α )]
2 2

⇒ tan (β − α ) (x + r ) = − cot β (x − r ) cos 2 α


⇒ x tan (β − α ) + r tan (β − α ) = − x cot β + r cot β d 2 cos 2 α
⇒ r2 =
⇒ x [tan (β − α ) + cot β ] = r [cot β − tan (β − α )] cos β + cos α + 2 cos α cos β cos (β − α )
2 2

Therefore, area of the circle


sin (β − α ) cos β   cos β sin(β − α ) 
⇒ x +  =r −  π d 2 cos 2 α
 cos (β − α ) sin β  sin β cos (β − α )  πr 2 =
cos β + cos α + 2 cos α cos β cos (β − α )
2 2

sin (β − α ) sin β + cos (β − α ) cos β 


⇒ x   1
 cos (β − α ) sin β 
20. Let the points mi ,  ; i = 1, 2, 3, 4
 mi 
 cos β cos (β − α ) − sin β sin (β − α )  lie on a circle x2 + y2 + 2 gx + 2 fy + c = 0.
=r 
 sin β cos (β − α )  1
Then, mi2 + 2 + 2 gmi +
2f
+ c=0;
mi mi
⇒ x [cos (β − α − β )] = r [cos (β − α + β )]
r cos (2 β − α ) Since, mi4 + 2 gmi3 + cmi2 + 2 fmi + 1 = 0 ; i = 1, 2, 3, 4
⇒ x= ⇒ m1 , m2, m3 and m4 are the roots of the equation
cos α
m4 + 2 gm3 + cm2 + 2 fm + 1 = 0
On putting this value in Eq. (ii), we get
1
 r cos (2 β − α )  ⇒ m1m2 m3m4 = = 1
y = − cot β  − r 1
 cos α 
21. Let (x1 , y1 ) and (x2, y2) be the coordinates of points A and
cos β ⋅ r  cos (2β − α ) − cos α 
⇒ y=−   B, respectively.
sin β  cos α  It is given that x1 , x2 are the roots of x2 + 2ax − b2 = 0
 2β − α + α α − 2β + α  ⇒ x1 + x2 = − 2a and x1x2 = − b2 …(i)
2 sin   sin  
r cos β   2   2  Also, y1 and y2 are the roots of y2 + 2 py − q2 = 0
⇒ y=−   ⇒ y1 + y2 = − 2 p and y1 y2 = − q2 …(ii)
sin β  cos α  ∴ The equation of circle with AB as diameter is,
 
(x − x1 ) (x − x2) + ( y − y1 ) ( y − y2) = 0
r cos β 2 sin β ⋅ sin (α − β )  ⇒ x2 + y2 − (x1 + x2) x − ( y1 + y2) y + (x1x2 + y1 y2) = 0
⇒ y=−  
sin β  cos α  ⇒ x2 + y2 + 2ax + 2 py − (b2 + q2) = 0
= − 2r cos β sin (α − β ) / cos α and radius = a 2 + p2 + b2 + q2
418 Circle

22. The circle and coordinate axes can have 3 common Intersection point of tangent at E1 and E 2 is (0, 4).
points, if it passes through origin. [ p = 0] ∴ Coordinates of E3 is (0, 4)
If circle is cutting one axis and touching other axis. Similarly, equation of tangent at F1 (1, − 3 ) and
Only possibility is of touching X-axis and cutting F2(1, 3 ) are x − 3 y = 4 and x + 3 y = 4, respectively
Y -axis. [ p = − 1] and intersection point is (4, 0), i.e., F3 (4, 0) and
equation of tangent at G1 (0, 2) and G2(2, 0) are 2 y = 4
23. x2 + y2 ≤ 6 and 2x − 3 y = 1 is shown as and 2x = 4, respectively and intersection point is (2, 2)
L i.e., G3 (2, 2).
Point E3 (0, 4), F3 (4, 0) and G3 (2, 2) satisfies the line
x + y = 4.
1/3 1/2
25. We have,
x2 + y 2 = 4
For the point to lie in the shade part, origin and the Let P(2 cos θ , 2 sin θ ) be a point on a circle.
point lie on opposite side of straight line L. ∴ Tangent at P is
∴ For any point in shaded part L > 0 and for any point 2 cos θ x + 2 sin θ y = 4
inside the circle S < 0. ⇒ x cos θ + y sin θ = 2
 3
Now, for 2,  L : 2x − 3 y − 1 Y
 4 N
9 3
L :4 − − 1 = > 0 P(2 cos θ, 2 sin θ)
4 4
9
and S : x + y − 6, S : 4 +
2 2
−6 <0 X′ X
16 O M
 3
⇒ 2,  lies in shaded part.
 4 x2 + y2 = 4
 5 3
For  ,  , L : 5 − 9 − 1 < 0 [neglect]
 2 4 Y′
 1 1 1 3 
 2   2 
For  ,−  , L: + − 1 > 0 ∴ The coordinates at M  , 0 and N 0,
 4 4 2 4  
 cos θ   sin θ 
 1 1 Let (h , k) is mid-point of MN
∴  ,−  lies in the shaded part.
 4 4 1 1
∴ h= and k =
 1 1 1 3 cos θ sin θ
For  ,  , L : − − 1 < 0 [neglect]
 8 4 4 4 1 1
⇒ cos θ = and sin θ =
⇒ Only 2 points lie in the shaded part. h k
24. Y 1 1 h 2 + k2
⇒ cos 2 θ + sin 2 θ = + 2 ⇒ 1=
E3(0,4) h 2
k h 2 ⋅ k2
⇒ h 2 + k2 = h 2 k2
G1 (0,2) ∴ Mid-point of MN lie on the curve x2 + y2 = x2 y2
F2(1, 3)

E1(– 3,1) E2( 3,1) Topic 2 Relation between Two Circles


Po(1,1)
1. Equation of given circles
x′ x
(–2,0) O G2(2, 0) x2 + y2 + 5Kx + 2 y + K = 0 …(i)
and 2(x2 + y2) + 2Kx + 3 y − 1 = 0
3 1
F1(1, – 3) ⇒ x2 + y2 + Kx + y − = 0 …(ii)
2 2
(0, –2)
On subtracting Eq. (ii) from Eq. (i), we get
Y′ 1 1
4Kx + y + K + = 0
2 2
Equation of tangent at E1 (− 3 , 1) is ⇒ 8Kx + y + (2K + 1) = 0 …(iii)
− 3x + y = 4 and at E 2( 3 , 1) is
[Q if S1 = 0 and S 2 = 0 be two circles, then
3x + y = 4 their common chord is given by S1 − S 2 = 0.]
Circle 419

Eq. (iii) represents equation of common chord as it is 3(9) + 4(1) − λ


and ≥ 81 + 1 − 78
given that circles (i) and (ii) intersects each other at 5
points P and Q.
⇒ |λ − 31 | ≥ 10
Since, line 4x + 5 y − K = 0 passes through point P and Q.
8K 1 2K + 1 ⇒ λ ∈ (− ∞ , 21] ∪ [41, ∞ ) ... (v)
∴ = = From Eqs. (iii), (iv) and (v), we get
4 5 −K
1 λ ∈ [ 12, 21 ]
⇒ K = [equating first and second terms]
10 4. Given circles,
and − K = 10K + 5 x2 + y 2 − 2 x − 2 y − 2 = 0 … (i)
[equating second and third terms] and x + y2 − 6x − 6 y + 14 = 0
2
… (ii)
5
⇒ 11K + 5 = 0 ⇒ K = − are intersecting each other orthogonally, because
11 2(1)(3) + 2(1)(3) = 14 − 2
1 5
Q ≠ − , so there is no such value of K, for which [Q two circles are intersected
10 11 orthogonally if 2 g1 g2 + 2 f1 f2 = c1 + c2]
line 4x + 5 y − K = 0 passes through points P and Q.
2. Equation of given circle is x2 + y2 = 1, then equation of P
tangent at the point (cos θ ,sin θ ) on the given circle is 2 2
x cos θ + y sin θ = 1 …(i) C1 (3,3)
(1,1) C2
[QEquation of tangent at the point P(cos θ , sin θ ) to
the circle x2 + y2 = r 2 is x cos θ + y sin θ = r]
Q
Now, the point of intersection with coordinate axes are
P(sec θ , 0) and Q(0, cos ec θ ). So, area of quadrilateral
Q Mid-point of line joining points P and Q is PC1QC 2 = 2 × ar (∆PC1C 2).
 sec θ cos ecθ  1 
M ,  = (h , k) (let) = 2 ×  × 2 × 2 = 4 sq units
 2 2  2 
1 1 5. Circle I is x2 + y2 − 16x − 20 y + 164 = r 2
So, cos θ = and sin θ =
2h 2k
⇒ (x − 8)2 + ( y − 10)2 = r 2
Q sin 2 θ + cos 2 θ = 1
⇒C1 (8, 10) is the centre of Istcircle and r1 = r is its radius
1 1 1 1
∴ + =1⇒ 2 + 2 =4 Circle II is (x − 4)2 + ( y − 7)2 = 36
4h 2 4k2 h k
⇒ C 2(4, 7) is the centre of 2nd circle and r2 = 6 is its
Now, locus of mid-point M is radius.
1 1
+ =4 Two circles intersect if|r1 − r2| < C1C 2 < r1 + r2
x2 y 2
⇒| r − 6|< (8 − 4)2 + (10 − 7)2 < r + 6
⇒ x2 + y2 − 4x2y2 = 0
⇒| r − 6| < 16 + 9 < r + 6
3. The given circles,
x + y − 2x − 2 y + 1 = 0
2 2
... (i) ⇒| r − 6| < 5 < r + 6
and x + y2 − 18x − 2 y + 78 = 0,
2
... (ii) Now as, 5 < r + 6 always, we have to solve only
are on the opposite sides of the variable line | r − 6| < 5 ⇒ − 5 < r − 6 < 5
3x + 4 y − λ = 0. So, their centres also lie on the opposite ⇒ 6 − 5 < r < 5 + 6 ⇒ 1 < r < 11
sides of the variable line.
6. Given equation of a circle is x2 + y2 − 4x + 6 y − 12 = 0,
⇒ [3(1) + 4(1) − λ ] [3(9) + 4(1) − λ ] < 0
whose centre is (2, − 3) and radius
[Q The points P (x1 , y1 ) and Q (x2, y2) lie on the
opposite sides of the line ax + by + c = 0, = 2 2 + (− 3) 2 + 12 = 4 + 9 + 12 = 5
if (ax1 + by1 + c)(ax2 + by2 + c) < 0] Now, according to given information, we have the
⇒ (λ − 7)(λ − 31) < 0 following figure.
⇒ λ ∈ (7, 31) ... (iii)
S
3(1) + 4(1) − λ
Also, we have ≥ 1 + 1 −1
5 A (–3, 2) C
QDistance of centre from the given line is 
 ax1 + by1 + c  O
 greater than the radius,i.e. ≥ r (2,–3)
 a2 + b2  B

⇒ |7 − λ|≥ 5 ⇒ λ ∈ (− ∞, 2] ∪ [12 , ∞ ) ... (iv)


420 Circle

x2 + y2 − 4x + 6 y − 12 = 0 Y
Clearly, AO ⊥ BC, as O is mid-point of the chord. C
Now, in ∆AOB, we have C1 (1,1)
OA = (− 3 − 2) 2 + (2 + 3) 2 T (1– k)
(0,k)
= 25 + 25 = 50 = 5 2 X′
1
X
O (1,0)
and OB = 5 C2
∴ AB = OA 2 + OB2
Y′
= 50 + 25 = 75 = 5 3
⇒ k + 1 = k2 + 2 − 2k
7. Here, radius of smaller circle, AC = 12 + 32 − 6 = 2
Clearly, from the figure the radius of bigger circle ⇒ k2 + 1 + 2k = k2 + 2 − 2k
1
r 2 = 22 + [(2 − 1)2 + (1 − 3)2] ⇒ k=
4
r2 = 9 ⇒ r =3
1
So, the radius of circle T is k, i. e. .
4
A
10. Since, the given circles intersect orthogonally.
r
2 ∴ 2 (1) (0) + 2 (k) (k) = 6 + k
C
(1,3)
C1 (2,1) [Q 2 g1 g2 + 2 f1 f2 = c1 + c2]
2 3
⇒ 2k2 − k − 6 = 0 ⇒ k = − , 2
2
B
11. Let O is the point at centre and P is the point at
circumference. Therefore, angle QOR is double the
8. PLAN Number of common tangents depend on the position of the angle QPR. So, it is sufficient to find the angle QOR.
circle with respect to each other. Y
(i) If circles touch externally ⇒ C1C 2 = r1 + r2, 3 common
tangents.
(ii) If circles touch internally ⇒ C1C 2 = r2 − r1,1 common tangent. Q (3,4)
4 ,3 )

(iii) If circles do not touch each other, 4 common tangents.


R (−

Given equations of circles are


x2 + y2 − 4x − 6 y − 12 = 0 ...(i) X′ X
O (0,0)
x + y2 + 6x + 18 y + 26 = 0
2
...(ii)
Centre of circle (i) is C1 (2, 3) and radius
= 4 + 9 + 12 = 5(r1 ) [say]
P
Centre of circle (ii) is C 2(–3, – 9) and radius
= 9 + 81 – 26 = 8(r2) [say] Y′

Now, C1C 2 = (2 + 3)2 + (3 + 9)2 Now, slope of OQ , m1 = 4 / 3 ,


slope of OR, m2 = − 3 / 4
⇒ C1C 2 = 5 + 12 2 2
Here, m1 m2 = − 1
⇒ C1C 2 = 25 + 144 = 13 Therefore, ∠ QOR = π /2
∴ r1 + r2 = 5 + 8 = 13 which implies that ∠ QPR = π /4
Also, C1C 2 = r1 + r2 12. Given, x2 + y2 = 4
Thus, both circles touch each other externally. Hence, Centre ≡ C1 ≡ (0, 0) and R1 = 2
there are three common tangents.
Again, x2 + y2 − 6x − 8 y − 24 = 0, then C 2 ≡ (3, 4)
9. PLAN Use the property, when two circles touch each other and R2 = 7
externally, then distance between the centre is equal to sum
of their radii, to get required radius. Again, C1C 2 = 5 = R2 − R1
Let the coordinate of the centre of T be (0, k). Therefore, the given circles touch internally such that,
they can have just one common tangent at the point of
Distance between their centre contact.
k + 1 = 1 + (k − 1)2 [Q C1C 2 = k + 1] 13. Centre of the circle
⇒ k + 1 = 1 + k + 1 − 2k2 x2 + y2 + 4x − 6 y + 9 sin 2 α + 13 cos2 α = 0
Circle 421

is C (−2, 3) and its radius is ∴ Locus of M is a circle having PQ as its diameter of


circle.
(−2) + (3) − 9 sin α − 13 cos α
2 2 2 2
∴ Equation of circle
= 13 − 13 cos2 α − 9 sin 2 α (x − 2) (x + 2) + ( y + 5) ( y − 7) = 0
= 13 sin 2 α − 9 sin 2 α = 4 sin 2 α = 2 sin α ⇒ x2 + y2 − 2 y − 39 = 0
Hence, E1 : x2 + y2 − 2 y − 39 = 0, x ≠ ± 2
A Locus of mid-point of chord (h , k) of the circle E1 is
xh + yk − ( y + k) − 39 = h 2 + k2 − 2k − 39
, 3) ⇒ xh + yk − y − k = h 2 + k2− 2k
−2
P (h, k) C( Since, chord is passing through (1, 1).
∴ Locus of mid-point of chord (h , k) is
B h + k − 1 − k = h 2 + k2 − 2k
⇒ h 2 + k2 − 2k − h + 1 = 0
Let (h , k) be any point P and
Locus is E 2 : x2 + y2 − x − 2 y + 1 = 0
∠ APC = α , ∠ PAC = π /2
Now, after checking options, (a) and (d) are correct.
That is, triangle APC is a right angled triangle.
AC 2 sin α 17. PLAN
∴ sin α = = (i) The general equation of a circle is
PC (h + 2)2 + (k − 3)2
x 2 + y 2 + 2 gx + 2 f y + c = 0
⇒ (h + 2)2 + (k − 3)2 = 4 where, centre and radius are given by ( − g , − f ) and
⇒ h + 4 + 4h + k2 + 9 − 6k = 4
2
g 2 + f 2 − c , respectively.
⇒ h 2 + k2 + 4h − 6k + 9 = 0 (ii) If the two circles x 2 + y 2 + 2 g 1x + 2 f1 y + c1 = 0 and
Thus, required equation of the locus is x 2 + y 2 + 2 g 2x + 2 f2 y + c 2 = 0 are orthogonal, then
2 g 1g 2 + 2 f1f2 = c1 + c 2.
x2 + y 2 + 4 x − 6 y + 9 = 0
14. As, the two circles intersect in two distinct points. Let circle be x2 + y2 + 2 gx + 2 fy + c = 0
It passes through (0, 1).
⇒ Distance between centres lies between | r1 − r2| and
| r1 + r2|. ∴ 1 + 2f + c = 0 …(i)
i.e. | r − 3|< (4 − 1) + (−1 − 3) < | r + 3|
2 2 Orthogonal with x + y − 2x − 15 = 0
2 2

⇒ | r − 3|< 5 < | r + 3| ⇒ r < 8 or r > 2 2 g (− 1) = c − 15

∴ 2 < r <8 ⇒ c = 15 − 2 g …(ii)


Orthogonal with x2 + y2 − 1 = 0
15. Let x2 + y2 + 2 gx + 2 fy + c = 0, cuts x2 + y2 = k2
c=1 …(iii)
orthogonally.
⇒ 2 g1 g2 + 2 f1 f2 = c1 + c2 ⇒ g = 7 and f = − 1

⇒ −2 g ⋅ 0 − 2 f ⋅ 0 = c − k 2 Centre is (− g , − f ) ≡ (− 7, 1)

⇒ c=k 2
…(i) ∴ Radius = g2 + f 2 − c
Also, x2 + y2 + 2 gx + 2 fy + k2 = 0 passes through (a , b). = 49 + 1 − 1 = 7
∴ a 2 + b2 + 2 ga + 2 fb + k2 = 0 …(ii) 18. Let the, equation of circles are
⇒ Required equation of locus of centre is C1 : (x − 1)2 + ( y − 1)2 = (1)2
−2ax − 2by + a + b + k = 0
2 2 2
and C 2 : (x − 1)2 + ( y − 1)2 = ( 2 )2
or 2ax + 2by − (a 2 + b2 + k2) = 0
D(0, 2) C(2, 2)
16. It is given that T is tangents to S1 at P and S 2 at Q and S1
C1 P
and S 2 touch externally at M. x2 + x2 – 2y = 0
(1, 1)

s1 M s2 A(0, 0) B(2, 0)

T ∴ Coordinates of P(1 + cos θ , 1 + sin θ )


P(–2,7) N Q(2,–5)
and Q(1 + 2 cos θ , 1 + 2 sin θ )
∴ MN = NP = NQ ∴ PA 2 + PB2 + PC 2 + PD 2
422 Circle

= {(1 + cos θ )2 + (1 + sin θ )2} + {(cos θ − 1)2 + (1 + sin θ )2}


+ {(cos θ − 1)2 + (sin θ − 1)2} C2 C1
+ {(1 + cos θ )2 + (sin θ − 1)2} = 12 O2
r2
Similarly, QA 2 + QB2 + QC 2 + QD 2 = 16 r
Σ PA 2 12
∴ = = 0.75 O1 O
ΣQA 2 16 C
19. Let C be the centre of the required circle. r1

C1

Now, OO2 = r + r2
and OO1 = r1 − r
C
⇒ OO1 + OO2 = r1 + r2
L which is greater than O1O2 as O1O2 < r1 + r2
[Q C 2 lies inside C1 ]
A ⇒ Locus of O is an ellipse with foci O1 and O2.
Alternate Solution
Now, draw a line parallel to L at a distance of r1
(radius of C1) from it. Let equations of C1 be x2 + y2 = r12 and of C 2 be
Now, CC1 = AC ⇒ C lies on a parabola. (x − a )2 + ( y − b)2 = r22

20. Since, AG = 2 Let cetnre C be (h , k) and radius r, then by the given


condition
1
∴ AT1 = T1G = (h − a )2 + (k − b)2 = r + r2 and h 2 + k2 = r1 − r
2
As, A is the focus, T1 is the vertex and BD is the directrix ⇒ (h − a )2 + (k − b)2 + h 2 + k2 = r1 + r2
of parabola.
Required locus is
(x − a )2 + ( y − b2) + x2 + y2 = r1 + r2
D C which represents an ellipse whose foci are (a , b)
and (0, 0).
G 23. Suppose the circles have centres at C1 , C 2 and C3 with
T1 radius R1 , R2 and R3 , respectively. Let the circles touch
T2 at A , B and C. Let the common tangents at A, B and C
B meet at O. We have, OA = OB = OC = 4 [given]. Now, the
A
circle with centre at O and passing through A, B and C
is the incircle of the triangle C1 C 2 C3 (because
T3
OA ⊥ C1 C 2).
Also, T2T3 is latusrectum. Therefore, the inradius of ∆ C1 C 2 C3 is 4.
∴ T2T3 = 4 ⋅
1 ∆
and r= …(i)
2 s
1 1 4
∴ Area of ∆T1T2T3 = × × = 1 sq unit
2 2 2 A
R1 R2
21. (A) When two circles are intersecting they have a C1 C2
common normal and common tangent.
R1 R2
(B) Two mutually external circles have a common
C B
normal and common tangent. R3 R3
(C) When one circle lies inside of other, then they have a
common normal but no common tangent. C3
(D) Two branches of a hyperbola have a common normal
but no common tangent.
22. Let the given circles C1 and C 2 have centres O1 and O2 Now, perimeter of a triangle
and radii r1 and r2, respectively. 2s = R1 + R2 + R2 + R3 + R3 + R1
Let the variable circle C touching C1 internally, C 2 ⇒ 2s = 2 (R1 + R2 + R3 )
externally have a radius r and centre at O. ⇒ s = R1 + R2 + R3
Circle 423

and ∆ = s (s − a ) (s − b) (s − c) and x2 + y2 + 6x + 8 y − 24 = 0, centre c2(−3, − 4) and


= (R1 + R2 + R3 ) (R3 ) (R2) (R1 ) radius r2 = 7
Q c1c2 = 9 + 16 = 5
R1 R2 R3 (R1 + R2 + R3 )
From Eq. (i), 4= and |r1 − r2| = 5
R1 + R2 + R3 Q c1c2 = |r1 − r2| = 5
R1 R2 R3 (R1 + R2 + R3 ) ∴ circle x2 + y2 = 4 touches the circle
⇒ 16 =
(R1 + R2 + R3 )2 x2 + y2 + 6x + 8 y − 24 = 0 internally.
So, equation of common tangent is
R1 R2 R3
⇒ 16 = S1 − S 2 = 0
R1 + R2 + R3 ⇒ 6x + 8 y − 20 = 0
⇒ 3x + 4 y = 10 …(i)
Topic 3 Equation of Tangent, Normal The common tangent passes through the point (6, − 2),
and Length of Tangents from the given options.
1. Since, equation of a circle which touches the Y-axis at 3. Given points are (−8, 5) and (6, 5) in which y-coordinate
point (0, 4) is is same, i.e. these points lie on horizontal line y = 5.
x2 + ( y − 4 ) 2 + λ x = 0 … (i) 3y=x+7
Q Circle (i) passes through point (2, 0), so
4 + 16 + 2λ = 0 (–8, β) (6, β)
⇒ λ = − 10
Therefore, equation of the circle is
x2 + ( y − 4)2 − 10x = 0
⇒ (x − 5)2 + ( y − 4)2 = 25 … (ii) (–8, 5) (6, 5)
Now, equation of tangent to the circle (ii) having slope ‘
m’ is
y − 4 = m(x − 5) ± 5 1 + m2
Let (−8, β) and (6, β ) are the coordinates of the other
⇒ y = mx + (4 − 5m ± 5 1 + m2 ) … (iii) vertices of rectangle as shown in the figure.
Since, the mid-point of line joining points (−8, 5) and
From the option, on taking slope of line, we have (6, β ) lies on the line 3 y = x + 7.
4  5 + β  −8 + 6
when m = , equation of tangent by Eq. (iii), we get ∴ 3  = +7
3  2  2
4  20 16  ⇒ 15 + 3 β = − 2 + 14
y= x + 4 − ±5 1+  ⇒ 3β = −3
3  3 9
⇒ β = −1
⇒ 3 y = 4x + (12 − 20 ± 25) Now, area of rectangle = |−8 − 6| × |β − 5|
= 14 × 6 = 84
⇒ 4x − 3 y − 8 ± 25 = 0
⇒ 4x − 3 y + 17 = 0 and 4x − 3 y − 33 = 0
Similarly, when m = − 3 / 4, equation of tangent is 4. Let T = 0 and N = 0 represents the tangent and normal
3  15 9 lines at the point P( 3 , 1) to the circle x2 + y2 = 4.
y = − x + 4 + ± 5 1+ 
4  4 16  Y
⇒ 4 y = − 3x + 31 ± 25 N=0
⇒ 3x + 4 y − 56 = 0 and 3x + 4 y − 6 = 0 P
4
Similarly, when m = − , equation of tangent is T=0
3
4  20 25 X
y = − x + 4 + ±  O M A
3  3 3
⇒ 4x + 3 y − 57 = 0 and 4x + 3 y − 7 = 0
∴The line 4x + 3 y − 8 = 0 is not a tangent to the circle. x2+y2=4
Hence, option (c) is correct.
2. Given circles are
So, equation of tangent (T = 0) is
x2 + y2 = 4, centre c1 (0, 0) and radius r1 = 2
3x + y = 4 …(i)
424 Circle

For point A, put y = 0, we get (x − 1)2 + ( y + 1)2 + λ (3x − 4 y − 7) = 0


4 So, the equation of required circle will be
x=
3 (x − 1)2 + ( y + 1)2 + λ (3x − 4 y − 7) = 0, for some λ ∈ R
1 1 4 …(i)
Q Area of required ∆OPA = (OA )(PM ) = × ×1
2 2 3 Q The required circle passes through (4, 0)
[QPM = y-coordinate of P] ∴(4 − 1)2 + (0 + 1)2 + λ (3 × 4 − 4 × 0 − 7) = 0
2 ⇒ 9 + 1 + λ (5) = 0
= sq unit
3 ⇒ λ = −2
5. According to given information, we have the following Substituting λ = − 2 in Eq. (i), we get
figure. (x − 1)2 + ( y + 1)2 − 2 (3x − 4 y − 7) = 0
(0,1/2)
Y ⇒ x2 + y2 − 8x + 10 y + 16 = 0
B On comparing it with
x2 + y2 + 2 gx + 2 fy + c = 0, we get
g = − 4, f = 5, c = 16
N ∴ Radius = g2 + f 2 − c
A
x = 16 + 25 − 16 = 5
O
(1, 0) x+2y=1 7. Key Idea Equation of tangent to the curve
y + y1
x 2 = 4 ay at ( x 1, y 1) is xx 1 = 4 a  
M  2 

From figure, equation of circle (diameter form) is Tangent to the curve x2 = y − 6 at (1, 7) is
 1 y+ 7
(x − 1) (x − 0) + ( y − 0) y −  = 0 x= −6
 2 2

y
x2 + y2 − x − = 0 ⇒ 2x − y + 5 = 0 …(i)
2 Equation of circle is x2 + y2 + 16x + 12 y + c = 0
Equation of tangent at (0, 0) is x + = 0
y Centre (−8, − 6)
2 r = 82 + 62 − c = 100 − c
[Q equation of tangent at (x1 , y1 ) is given by T = 0.
Since, line 2x − y + 5 = 0 also touches the circle.
Here, T = 0 2(−8) − (−6) + 5
1 1 ∴ 100 − c = 
⇒ xx1 + yy1 − (x + x1 ) − ( y + y1 ) = 0] ⇒ 2x + y = 0 
2 4  22 + 12 
|2 ⋅ 1 + 1 ⋅ 0| 2 −16 + 6 + 5
Now, AM = = ⇒ 100 − c =  
5 5  5 
[Q distance of a point P (x1 , y1 ) from a line ⇒ 100 − c = |− 5|
|ax1 + by1 + c | ⇒ 100 − c = 5
ax + by + c = 0 is ]
a 2 + b2 ⇒ c = 95
1
 1 8. 18 = (3α ) (2r )
2 ⋅0 + 1   2
1  2
and BN = = ⇒αr = 6
5 2 5 2r
2 1 4+1 5 Line, y = − (x − 2α ) is tangent to circle
∴ AM + BN = + = = α
5 2 5 2 5 2 (x − r )2 + ( y − r )2 = r 2
6. Equation of tangent to the circle 2α = 3r , αr = 6 and r = 2
x2 + y2 + 4x − 6 y − 12 = 0 at (1, − 1) is given by Y
xx1 + yy1 + 2 (x + x1 ) − 3 ( y + y1 ) − 12 = 0, where x1 = 1
and y1 = − 1 D C (a, 2r)
⇒ x − y + 2 (x + 1) − 3( y − 1) − 12 = 0 (0, 2r)
⇒ 3x − 4 y − 7 = 0
(r, r)
This will also a tangent to the required circle.
X′ X
Now, equation of family of circles touching the line (0, 0) A B (2a, 0)
3x − 4 y − 7 = 0 at point (1, − 1) is given by
Y′
Circle 425

Alternate Solution Intersecting with x = 1,


1 1 − cos θ
(x + 2x) × 2r = 18 y=
2 x sin θ
r x−r
xr = 6 ... (i) B A  1 − cos θ 
x−r ∴ Q 1, 
In ∆ AOB, tan θ =  sin θ 
r
θ r ∴ Equation of the line through Q parallel to RS is
and in ∆ DOC , 2r θ
2x − r O 2 sin 2
tan (90°− θ ) = 90 1 − cos θ 2 θ
r °−
θ y= = = tan …(i)
r sin θ θ θ 2
x−r r 2 sin cos
∴ = 2 2
r 2x − r C 2x–r D sin θ
⇒ x (2x − 3r ) = 0
2x Normal at P : y = ⋅x
cos θ
3r
⇒ x= ... (ii) ⇒ y = x tan θ …(ii)
2
Let their point of intersection be (h , k).
From Eqs. (i) and (ii), we get
θ
r =2 Then, k = tan and k = h tan θ
2
9. The line 5x − 2 y + 6 = 0 meets the Y-axis at the point  θ 
 2 tan  2h ⋅ k
(0, 3) and therefore the tangent has to pass through the ∴ k=h 2  ⇒ k=
point (0, 3) and required length  1 − tan 
2 θ 1 − k2
 2
= x12 + y12 + 6x1 + 6 y1 − 2
⇒ k(1 − k2) = 2hk
= 02 + 32 + 6 (0) + 6 (3) − 2 = 25 = 5 ∴ Locus for point E : 2x = (1 − y2) …(iii)
1
When x = , then
10. From figure, it is clear that ∆ PRQ and ∆ RSP are 3
similar. 2 2 1
1 − y2 = ⇒ y2 = 1 − ⇒ y = ±
3 3 3
1 1
∴ ,±  satisfy 2x = 1 − y .
2
3 3
r r 1
P R When x = , then
90 − θ θ 4
°

2 1 1
1 − y2 = ⇒ y2 = 1 − ⇒ y=±
4 2 2
X  1 1 
∴  , ±  does not satisfy 1 − y2 = 2x.
θ

4 2
90 °

Q S 12. Given, C1 : x2 + y2 = 3 intersects the parabola x2 = 2 y.

PR PQ
∴ = ⇒ PR2 = PQ ⋅ RS 3
P
RS RP
⇒ PR = PQ ⋅ RS ⇒ 2r = PQ ⋅ RS
− 3 O 3
11. Given, RS is the diameter of x2 + y2 = 1.
Here, equation of the tangent at P(cos θ , sin θ ) is
− 3
x cos θ + y sin θ = 1.

P (cos θ, sin θ) On solving x2 + y2 = 3 and x2 = 2 y, we get


y2 + 2 y = 3
Q ⇒ y + 2y − 3 = 0
2

E ⇒ ( y + 3)( y − 1) = 0
(h , k )
∴ y = 1, − 3 [neglecting y = − 3, as − 3 ≤ y ≤ 3]
R O S(1,0)
(–1,0) ∴ y=1 ⇒x= ± 2
x cos θ + y sin θ = 1
⇒ P( 2 , 1) ∈ I quadrant
Equation of tangent at P( 2 , 1)to C1 : x2 + y2 = 3 is
2x + 1 ⋅ y = 3 …(i)
426 Circle

Now, let the centres of C 2 and C3 be Q2 and Q3 , and ∴ Option (c) is correct.
tangent at P touches C 2 and C3 at R2 and R3 shown as 1 2
below Also, area of ∆PQ2Q3 = Q2Q3 × 2 = × 12 = 6 2
2 2
Y ∴ Option (d) is incorrect.
13. As locus of point of intersection for perpendicular
Q2 tangents is directors circle.
i.e. x2 + y2 = 2r 2
R2 C2
Here, (17, 7) lie on directors circle x2 + y2 = 338
⇒ Tangents are perpendicular.
14. Here, tangent to x2 + y2 = 4 at ( 3 , 1) is 3x + y = 4 …(i)
As, L is perpendicular to 3x + y = 4
O P( 2,1)
X ⇒ x − 3 y = λ which is tangent to
C1
|3 − 0 − λ |
Q3
(x − 3)2 + y2 = 1 ⇒ =1
1+3
C3
⇒ |3 − λ | = 2 ⇒ 3 − λ = 2 , − 2
2x + y = 3
∴ λ = 1, 5
Let Q2 be (0, k) and radius is 2 3. ⇒ L : x − 3 y = 1, x − 3 y = 5
|0 + k − 3| 15. Here, equation of common tangent be
∴ =2 3
2+1 Y
⇒ |k − 3| = 6
⇒ k = 9, − 3
∴ Q2(0, 9)and Q3 (0, − 3) 2
X
O 3
Hence, Q2Q3 = 12
∴ Option (a) is correct.
Also, R2R3 is common internal tangent to C 2 and C3 ,
and r2 = r3 = 2 3 y = mx ± 2 1 + m2
∴ R2R3 = d − (r1 + r2)
2 2
which is also the tangent to
= 12 − (4 3 ) 2 2 (x − 3)2 + y2 = 1
= 144 − 48 |3m − 0 + 2 1 + m2|
⇒ =1
= 96 = 4 6 m2 + 1

⇒ 3m + 2 1 + m2 = ± 1 + m2
Q2 (0,9)
⇒ 3m = − 3 1 + m2
R2 or 3m = − 1 + m2
C2
⇒ m2 = 1 + m2 or 9m2 = 1 + m2
1
⇒ m ∈ φ or m = ±
2 2
d
1 1
∴ y=± x±2 1+
R3 2 2 8
x 6
⇒ y=± ±
Q3 (0, −3) 2 2 2 2
⇒ 2 2 y = ± (x + 6)
C3 ∴ x + 2 2y = 6
∴ Option (b) is correct. 16. Let centre of circle C be (h , k).
Q Length of perpendicular from O(0, 0) to R2R3 is equal
to radius of C1 = 3. 3h + k − 6
Then, =1
1 1 3+1
∴ Area of ∆OR2R3 = × R2R3 × 3 = × 4 6 × 3 = 6 2
2 2 ⇒ 3h + k − 6 = 2, − 2
Circle 427

⇒ 3h + k = 4 …(i) 21. The equation of circle having tangent 2x + 3 y + 1 = 0 at


[rejecting 2 because origin and centre of (1, − 1)
C are on the same side of PQ] ⇒ (x − 1)2 + ( y + 1)2 + λ (2x + 3 y + 1) = 0
The point ( 3 , 1) satisfies Eq. (i). x + y2 + 2x(λ − 1) + y(3λ + 2) + (λ + 2) = 0
2
…(i)
∴ Equation of circle C is (x − 3 )2 + ( y − 1)2 = 1. which is orthogonal to the circle having end point of
Clearly, points E and F satisfy the equations given in diameter (0, − 1) and (−2, 3).
option (d). ⇒ x(x + 2) + ( y + 1)( y − 3) = 0
17. Slope of line joining centre of circle to point D is or x2 + y 2 + 2 x − 2 y − 3 = 0 …(ii)
2(2λ − 2) 2(3λ + 2)
3 ∴ ⋅1 + (−1) = λ + 2 − 3
−1 2 2
1
tan θ = 2 = ⇒ 2λ − 2 − 3λ − 2 = λ − 1
3 3 3
− 3 ⇒ 2λ = − 3 ⇒ λ = − 3 / 2
2
From Eq. (i) equation of circle,
It makes an angle 30° with X-axis.
2x2 + 2 y2 − 10x − 5 y + 1 = 0
∴ Points E and F will make angle 150° and −90° with
X-axis. 22. Two circles touch each other externally, if C1 C 2 = r1 + r2
E D and internally if C1 C 2 = r1 ~ r2.
Given circles are x2 + y2 − 4x − 2 y + 4 = 0,
whose centre C1 (2, 1) and radius r1 = 1
30°
and x2 + y2 − 12x − 8 y + 36 = 0
(√3,1)
whose centre C 2 (6, 4) and radius r2 = 4
The distance between the centres is
F (6 − 2)2 + (4 − 1)2 = 16 + 9 = 5
∴ E and F are given by ⇒ C1 C 2 = r1 + r2
x− 3 y−1 Therefore, the circles touch each other externally and at
= =1
cos 150° sin 150° the point of touching the point divides the line joining
x− 3 y −1 the two centres internally in the ratio of their radii, 1 : 4.
and = =1 1 × 6 + 4 × 2 14
cos (−90° ) sin (−90° ) Therefore, x1 = =
1+4 5
 3 3
∴ E= ,  and F = ( 3 , 0) 1 ×4 + 4 ×1 8
 2 2 y1 = =
1+4 5
2 2
18. Equation of QR, RP are y = x + 1 and y = – x − 1. Again, to determine the equations of common tangents
3 3 touching the circles in distinct points, we know that, the
19. In an equilateral triangle, the radius of incircle tangents pass through a point which divides the line
1 joining the two centres externally in the ratio of their
= × median of the triangle radii, i.e. 1 : 4.
3
1 × 6 − 4 × 2 −2 2
1 a2 1 4a 2 − a 2 a Therefore, x2 = = =
= a2 − = = 1 −4 −3 3
3 4 3 4 2 3
1 ×4 −4 ×1
and y2 = =0
Therefore, area of the square inscribed in this circle 1 −4
2a 2 a 2 Now, let m be the slope of the tangent and this line
= 2 (radius of circle)2 = = sq unit
4 ⋅3 6 passing through (2 / 3, 0) is
2
20. Since, the point of intersection of the coordinate axes y − 0 = m (x − 2 / 3) ⇒ y − mx + m = 0
with the line λx − y + 1 = 0 and x − 2 y + 3 = 0 forms the 3
circle. This is tangent to the Ist circle, if perpendicular
∴ (λx − y + 1) (x − 2 y + 3) = 0 distance from centre = radius.
represents a circle, if coefficient of x2 = coefficient of y2 1 − 2m + (2 / 3)m
∴ =1 [Q C1 ≡ (2, 1) and r1 = 1]
and coefficient of xy = 0 1 + m2
⇒ λ =2 or −2 λ − 1 = 0
⇒ 1 − 2m + (2 / 3) m = 1 + m2
1
⇒ λ =2 or λ=− 4
2 ⇒ 1− m = 1 + m2
3
428 Circle

16 2 8 7 8 2 2
⇒ 1+ m − m = 1 + m2 ⇒ m2 − m = 0 and sin θ = = (from ∆CQA) ...(iii)
9 3 9 3 AC CB + AB
7 8 24 Q AB = AM + MB = 2 AM [Q AM = MB]
⇒ m  m −  = 0 ⇒ m = 0, m =
9 3 7 |(8 × 2 ) − (6 × 3) − 23| 2 × 25
=2 = = 5.00
64 + 36 10
Hence, the equations of the two tangents are
From Eqs. (ii) and (iii), we get
24  2
y = 0 and y = x −  sin θ =
1
=
2

1
=
2
[Q AB = 5]
7  3 CB CB + AB CB CB + 5
⇒ y = 0 and 7 y − 24x + 16 = 0 ⇒ CB + 5 = 2CB ⇒ CB = 5 =
1
sin θ
23. We have, From the Eq. (i),
4
Slope of the common tangent = − 2
3 AC = = 2 × 5 = 10.00
3 sin θ
∴ Slope of C1C 2 =
4 25. (r + 1)2 = α 2 + 9
4
If C1C 2 makes an angle θ with X-axis, then cos θ = and
5 3 C
3 P 2
sin θ = . 1
5 C1
Y
α r

C
5
5 C2
C1 r2 + 8 = α 2
X′ θ X
⇒ r 2 + 2r + 1 = r 2 + 8 + 9
4x + 3y = 10 ⇒ 2r = 16 ⇒ r = 8
Y′
So, the equation of C1 C 2 in parametric form is
x−1 y−2 Topic 4 Radical Axis and Family of Circle
= …(i)
4 /5 3 /5 1. Let (h , k) be the centre of the circle and radius r = h, as
Since, C1 and C 2 are points on Eq. (i) at a distance of circle touch theY -axis and other circle x2 + y2 = 1 whose
5 units from P. centre (0, 0) and radius is 1.
So, the coordinates of C1 and C 2 are given by Y
x−1 y−2
= = ± 5 ⇒ x=1 ± 4
4 /5 3 /5 (h,k)
and y = 2 ± 3. C r=h>O
Thus, the coordinates of C1 and C 2 are (5, 5) and (−3, − 1), r
k >O
respectively. for first
Hence, the equations of the two circles are quadrant
X
(x − 5)2 + ( y − 5)2 = 52 and (x + 3)2 + ( y + 1)2 = 52 O
24. According to given informations the figure is as x2+y2=1
following

Q ∴ OC = r + 1
P 90° [Q if circles touch each other externally,
2
q 1 then C1C 2 = r1 + r2]
C B M A (2,3)
⇒ h 2 + k2 = h + 1, h > 0

8x-6y-23=0 and k > 0, for first quadrant.

2 ⇒ h 2 + k2 = h 2 + 2h + 1 ⇒ k2 = 2h + 1
From the figure, AC = …(i) ⇒ k = 1 + 2h , as k > 0
sin θ
1 Now, on taking locus of centre (h , k), we get
Qsin θ = (from ∆CPB) …(ii)
CB y = 1 + 2x, x ≥ 0
Circle 429

2. Since, the equation of a family of circles touching line ∴In ∆BCF, BF = (b + c)2 − (c − b)2 = 2 bc
L =0 at their point of contact(x1 , y1 ) is
Q AD + AE = BF
(x − x1 )2 + ( y − y1 )2 + λ L = 0, where λ ∈ R.
1 1 1
∴Equation of circle, touches the x = y at point (1, 1) is ∴ 2 ab + 2 ac = 2 bc ⇒ + =
c b a
(x − 1)2 + ( y − 1)2 + λ (x − y) = 0
⇒ x + y + (λ − 2)x + (− λ − 2) y + 2 = 0
2 2
…(i) 5. Let the equation of circle be
Q Circle (i) passes through point (1, − 3). (x − 3)2 + ( y − 0)2 + λy = 0
∴ 1 + 9 + (λ − 2) + 3(λ + 2) + 2 = 0
Y
⇒ 4λ + 16 = 0
⇒ λ = −4 A (3, 0)
So, equation of circle (i) at λ = − 4 , is X X
x2 + y 2 − 6 x + 2 y + 2 = 0 P
(1, –2)
Now, radius of the circle = 9 + 1 − 2 = 2 2. Y
3. Clearly, circles are orthogonal because tangent at one As it passes through (1, − 2)
point of intersection is passing through centre of the ∴ (1 − 3)2 + (− 2)2 + λ (− 2) = 0 ⇒ 4 + 4 − 2λ = 0 ⇒ λ = 4
other.
∴ Equation of circle is
Y (x − 3)2 + y2 + 4 y = 0
S2 S1
(0, 1) By hit and trial method, we see that point (5, − 2)
r r satisfies equation of circle.
X 6. Equation of circle passing through a point (x1 , y1 ) and
(α, 0)
(–α, 0) touching the straight line L, is given by
(0,–1)
(x − x1 )2 + ( y − y1 )2 + λ L = 0
∴ Equation of circle passing through (0, 2) and touching
Let C1 (α, 0) and C 2(− α , 0) are the centres. x=0
Then, S1 ≡ (x − α )2 + y2 = α 2 + 1 ⇒ (x − 0)2 + ( y − 2)2 + λ x = 0 …(i)
⇒ S1 ≡ x2 + y2 − 2 αx − 1 = 0 Also, it passes through (− 1, 0) ⇒ 1 + 4 − λ = 0
[Q radius, r = (α − 0) + (0 − 1) ]
2 2
∴ λ =5
and S 2 ≡ (x + α ) + y = α + 1
2 2 2 Eq. (i) becomes,
⇒ S 2 ≡ x2 + y2 + 2αx − 1 = 0 x2 + y 2 − 4 y + 4 + 5 x = 0
Now, 2(α ) (− α ) + 2 ⋅ 0 ⋅ 0 = (− 1) + (− 1) ⇒ α = ± 1 ⇒ x2 + y2 + 5x − 4 y + 4 = 0,
[Q condition of orthogonality is 2 g1 g2 + 2 f1 f2 = c1 + c2] For x-intercept put y = 0 ⇒ x2 + 5x + 4 = 0,
∴ C1 (1, 0) and C 2(− 1, 0) ⇒ C1C 2 = 2 (x + 1) (x + 4) = 0
4. According to given information, we have the following ∴ x = − 1, − 4
figure. Hence, (d) option (–4, 0).
7. Let the locus of centre of circle be (h , k) touching
( y − 1)2 + x2 = 1 and X-axis shown as
X
C
B
F
c
b E
D a
A (0, 1) |k| (h, k )
O 1 A
where A , B, C are the centres of the circles
|k|
Clearly, AB = a + b (sum of radii) and BD = b − a X′ X
O B
∴ AD = (a + b)2 − (b − a )2
Y′
(using Pythagoras theorem in ∆ABD)
= 2 ab Clearly, from figure,
Similarly, AC = a + c and CE = c − a Distance between C and A is always 1 + | k|,

∴In ∆ACE, AE = (a + c)2 − (c − a )2 = 2 ac i.e. (h − 0)2 + (k − 1)2 = 1 + | k|,

Similarly, BC = b + c and CF = c − b ⇒ h 2 + k2 − 2k + 1 = 1 + k2 + 2| k|
430 Circle

⇒ h 2 = 2 | k | + 2 k ⇒ x2 = 2 | y | + 2 y h 2 + 9 − 6h + k2 + 9 − 6k = 4 + h 2 + 4h
 y , y≥0 i.e. k2 − 10h − 6k + 14 = 0
where | y| = 
− y , y < 0 Thus, the locus of (h, k) is
y2 − 10x − 6 y + 14 = 0
∴ x2 = 2 y + 2 y, y ≥ 0
and x2 = − 2 y + 2 y, y < 0 10. Let C1 (h , k) be the centre of the required circle. Then,
⇒ x = 4 y, when y ≥ 0
2 (h − 0)2 + (k − 0)2 = (h − 1)2 + (k − 0)2
and x = 0, when y < 0
2
⇒ h 2 + k2 = h 2 − 2h + 1 + k2
∴ {(x, y): x2 = 4 y, when y ≥ 0} ∪ {(0, y): y < 0} ⇒ −2 h + 1 = 0 ⇒ h = 1 / 2
8. NOTE In solving a line and a circle there oftengenerate a Since, (0, 0) and (1, 0) lie inside the circle x2 + y2 = 9.
quadratic equation and further we have to apply Therefore, the required circle can touch the given circle
condition of Discriminant so question convert from internally.
coordinate to quadratic equation. i.e. C1 ⋅ C 2 = r1 ~ r2
From equation of circle it is clear that circle passes ⇒ h 2 + k2 = 3 − h 2 + k2
through origin. Let AB is chord of the circle.
1
⇒ 2 h 2 + k2 = 3 ⇒ 2 + k2 = 3
Y 4
1 3 1 9
⇒ + k2 = ⇒ + k2 =
4 2 4 4
⇒ k2 = 2 ⇒ k = ± 2
11. The required equation of circle is
A (p,q)  1 25
(x2 + y2 + 13x − 3 y) + λ  11x + y +  =0 …(i)
 2 2
C (h,0)
X′ X Its passing through (1, 1),
O
B ⇒ 12 + λ (24) = 0
1
Y′ ⇒ λ=−
2
A ≡ ( p, q) ⋅ C is mid-point and coordinate of C is (h , 0) On putting in Eq. (i), we get
11 1 25
Then, coordinates of B are (− p + 2h , − q) and B lies on x2 + y2 + 13x − 3 y − x− y− =0
the circle x2 + y2 = px + qy, we have 2 4 4
(− p + 2h )2 + (− q)2 = p (− p + 2h ) + q (− q) ⇒ 4x2 + 4 y2 + 52x − 12 y − 22x − y − 25 = 0
⇒ p2 + 4h 2 − 4 ph + q2 = − p2 + 2 ph − q2 ⇒ 4x2 + 4 y2 + 30x − 13 y − 25 = 0
⇒ 2 p2 + 2q2 − 6 ph + 4h 2 = 0 12. The required equation of circle is, S1 + λ (S 2 − S1 ) = 0.
⇒ 2h − 3 ph + p + q = 0
2 2 2
…(i) ∴ (x2 + y2 − 6) + λ (−6x + 14) = 0
There are given two distinct chords which are bisected Also, passing through (1, 1).
at X-axis then, there will be two distinct values of h ⇒ − 4 + λ (8) = 0
satisfying Eq. (i). 1
⇒ λ=
So, discriminant of this quadratic equation must be > 0. 2
⇒ D >0 ∴ Required equation of circle is
⇒ (− 3 p)2 − 4 ⋅ 2 ( p2 + q2) > 0 x2 + y2 − 6 − 3x + 7 = 0
⇒ 9 p2 − 8 p2 − 8 q 2 > 0 or x2 + y 2 − 3 x + 1 = 0
⇒ p2 − 8 q 2 > 0 ⇒ p2 > 8 q 2 13. It is given that, C1has centre (0, 0) and radius 1.
9. Let (h , k) be the centre of the circle which touches the Similarly, C 2 has centre (0, 0) and radius 2 and C k has
circle x2 + y2 − 6x − 6 y + 14 = 0 and Y-axis. centre (0, 0) and radius k.
The centre of given circle is (3, 3) and radius is Now, particle starts it motion from (1, 0) and moves 1
radian on first circle then particle shifts from C1 to C 2.
32 + 32 − 14 = 9 + 9 − 14 = 2
After that, particle moves 1 radian on C 2 and then
Since, the circle touches Y-axis, the distance from its
particle shifts from C 2 to C3 . Similarly, particle move on
centre to Y-axis must be equal to its radius, therefore its
n circles.
radius is h. Again, the circles meet externally, therefore
the distance between two centres = sum of the radii of Now, n ≥ 2π because particle crosses the X-axis for the
the two circles. first time on C n, then n is least positive integer.
Hence, (h − 3)2 + (k − 3)2 = (2 + h )2 Therefore, n = 7.
Circle 431

14. Equation of any circle passing through the point of xx1 yy


Then, Eq. (i) and + 1 = 1 are identical
intersection of x2 + y2 − 2x = 0 and y = x is 25 4
(x2 + y2 − 2x) + λ ( y − x) = 0 y1
x1 / 25 1
⇒ x2 + y2 − (2 + λ )x + λy = 0 ∴ = 4 =
cos θ sin θ r
2 + λ − λ
Its centre is  , . 25 cos θ 4 sin θ
 2 2  ⇒ x1 = , y1 =
r r
⇒ For AB to be the diameter of the required circle the
centre must be on AB, i.e. The line (i) meet the coordinates axes in A (r sec θ , 0)
2 + λ = − λ ⇒ λ = −1 [Q y = x] and β (0, r cosec θ ). Let (h , k) be mid-point of AB.
Therefore, equation of the required circle is r sec θ r cosecθ
Then, h= and k =
x2 + y2 − (2 − 1) x − 1 ⋅ y = 0 2 2
⇒ x2 + y 2 − x − y = 0 Therefore, 2h =
r
and 2k =
r
15. Given, C1: x + y = 16
2 2 cos θ sin θ

and let C 2 : (x − h ) + ( y − k)2 = 25


2 25 4
∴ x1 = and y1 =
2h 2k
∴ Equation of common chords is S1 − S 2 = 0
∴ 2hx + 2ky = (h 2 + k2 − 9) x2 y2
As (x1 , y1 ) lies on the ellipse + = 1, we get
h 3 25 4
∴ Its slope = − = [given] 1  625 1  4  25 1
k 4   +   =1 ⇒ + 2 =1
25  4h 2  4  k2 4h 2
k
If p be the length of perpendicular on it from the centre
h 2 + k2 − 9 or 25k2 + 4h 2 = 4h 2 k2
(0, 0) of C1 of radius 4, then p = .
4h 2 + 4k2 Therefore, required locus is 4x2 + 25 y2 = 4x2y2
Also, the length of the chord is 18. The equation of the circle on the line joining the points
A(3, 7) and B (6, 5) as diameter is
2 r − p =2 4 − p
2 2 2 2
(x − 3) (x − 6) + ( y − 7) ( y − 5) = 0 …(i)
The chord will be of maximum length, if φ = 0 or
16 2 and the equation of the line joining the points A (3, 7)
h 2 + k2 − 9 = 0 ⇒ h 2 + h =9 7 −5
9 and B (6, 5) is y − 7 = (x − 3)
3 −6
9
⇒ h=± ⇒ 2x + 3 y − 27 = 0 …(ii)
5
12 Now, the equation of family of circles passing through
∴ k= m
5 the point of intersection of Eqs. (i) and (ii) is
9 12  9 12 S + λP = 0
Hence, centres are  , −  and  − , 
5 5  5 5
⇒ (x − 3)(x − 6) + ( y − 7)( y − 5) + λ (2x + 3 y − 27) = 0
16. For point of intersection, we put ⇒ x2 − 6x − 3x + 18 + y2 − 5 y − 7 y + 35
3 y + 10 + 2λx + 3λy − 27λ = 0
x= in x2 + y2 − 2x + 4 y − 20 = 0
4 ⇒ S1 ≡ x2 + y2 + x (2λ − 9) + y (3λ − 12)
2
 3 y + 10  3 y + 10 + (53 − 27λ ) = 0 …(iii)
⇒   + y −2 
2
 + 4 y − 20 = 0
 4   4  Again, the circle,which cuts the members of family of
⇒ 25 y2 + 100 y − 300 = 0 circles, is
⇒ y2 + 4 y − 12 = 0
S 2 ≡ x 2 + y2 − 4 x − 6 y − 3 = 0 …(iv)
⇒ ( y − 2) ( y + 6) = 0
and the equation of common chord to circles S1 and S 2 is
⇒ y = − 6, 2
When y = −6 ⇒ x = − 2 S1 − S 2 = 0
When y=2 ⇒ x (2λ − 9 + 4) + y (3λ − 12 + 6) + (53 − 27λ + 3) = 0
⇒ x=4 ⇒ 2λx − 5x + 3λy − 6 y + 56 − 27λ = 0
∴ Point of intersection are (−2, − 6) and (4, 2) .
⇒ (− 5x − 6 y + 56) + λ (2x + 3 y − 27) = 0
17. Equation of any tangent to circle x2 + y2 = r 2 is
which represents equations of two straight lines
x cos θ + y sin θ = r … (i) passing through the fixed point whose coordinates are
obtained by solving the two equations
Suppose Eq. (i) is tangent to 4x2 + 25 y2 = 100
5x + 6 y − 56 = 0 and 2x + 3 y − 27 = 0,
x2 y2
or + = 1 at (x1 , y1 ) we get x = 2 and y = 23 /3
25 4
432 Circle

19. The parametric form of OP is 20. Let P (h , k) be the foot of perpendicular drawn from
x−0 y−0 origin O(0, 0) on the chord AB of the given circle such
=
cos 45° sin 45° that the chord AB subtends a right angle at the origin.
Since, OP = 4 2 x2 + y2 + 2g x + 2fy + c = 0
So, the coordinates of P are given by
x−0 y−0 B
= = −4 2
cos 45° sin 45°
So, P(−4, − 4)
Let C (h , k) be the centre of circle and r be its radius. P (h, k)
Now, CP ⊥ OP
A
Y
O
y=x
B (x2, y2)
The equation of chord AB is
h
y − k = − (x − h ) ⇒ hx + ky = h 2 + k2
M k
3 3√2 A(x ,y )
(h,k)C 1 1 The combined equation of OA and OB is homogeneous
X equation of second degree obtained by the help of the
P O given circle and the chord AB and is given by,
4√2 2
 hx + ky  hx + ky
x2 + y2 + (2 gx + 2 fy)  2  + c   =0
4)

 h + k2   h 2 + k2 
4 ,−

y = −x
(−

Since, the lines OA and OB are at right angle.


k+4 ∴Coefficient of x2 + Coefficient of y2 = 0
⇒ ⋅ (1) = − 1
h+4  2 gh ch 2 
⇒ 1 + 2 + 2 2
⇒ k + 4 = − h −4  h +k 2
(h + k ) 
2

⇒ h + k = −8 …(i)  2f k ck2 
+ 1 + 2 + 2 2 2
=0
Also, CP = (h + 4) + (k + 4)
2 2 2  h +k 2
(h + k ) 

⇒ (h + 4)2 + (k + 4)2 = r 2 …(ii) ⇒ 2 (h 2 + k2) + 2 ( g h + f k) + c = 0


c
In ∆ACM , we have ⇒ h 2 + k2 + gh + fk + = 0
2 2
 h + k
AC 2 = (3 2 )2 +   ∴ Required equation of locus is
 2 
c
x2 + y2 + gx + fy + =0
⇒ r 2 = 18 + 32 2
⇒ r =5 2 …(iii) 21. Let the equation of L1 be x cos α + y sin α = p1.
Also, CP = r Then, any line perpendicular to L1 is

h − 
k Y
⇒ =r
 2  L2
⇒ h − k = ± 10 …(iv) Q

From Eqs. (i) and (iv), we get


(h = − 9, k = 1) S
X′ X
or (h = 1, k = − 9) R O P
Thus, the equation of the circles are L1
Y′
(x + 9) + ( y − 1) = (5 2 )
2 2 2
x sin α − y cos α = p2
or (x − 1)2 + ( y + 9)2 = (5 2 )2 where, p2 is a variable.
⇒ x2 + y2 + 18x − 2 y + 32 = 0 Then, L1 meets X-axis at P ( p1 sec α ,0) and Y-axis at
or x2 + y2 − 2x + 18 y + 32 = 0 Q (0, p1cosec α).
Similarly, L 2 meets X-axis at R ( p2 cosec α , 0) and
Clearly, (−10, 2) lies interior of Y-axis at S (0, − p2 sec α ).
x2 + y2 + 18x − 2 y + 32 = 0 Now, equation of PS is,
Hence, the required equation of circle, is x y x y
+ =1 ⇒ − = secα …(i)
x2 + y2 + 18x − 2 y + 32 = 0 p1 sec α − p2 sec α p1 p2
Circle 433

Similarly, equation of QR is Hence, the required equations of the circles are


x
+
y
=1 ⇒
x
+
y
= cosec α …(ii) x2 + y2 + 2 (10 ± 3 6 ) x + (55 ± 24 6 ) = 0
p2cosec α p1cosec α p2 p1 23. Given lines are
Locus of point of intersection of PS and QR can be 3x + 5 y − 1 = 0 ...(i)
obtained by eliminating the variable p2 from Eqs. (i) and (2 + c)x + 5c2y − 1 = 0 ...(ii)
and (ii). x y 1
∴ = =
 x  x y −5 + 5 c2
− (2 + c) + 3 15c − 5c − 10
2
∴  − secα  + = cosec α
 p1  y p1 5(c2 − 1) 1−c
⇒ x= and y =
⇒ (x − p1 secα ) x + y = p1 y cosec α
2
5(3c − c − 2)
2
15c − 5c − 10
2

⇒ x + y − p1x sec α − p1 y cosec α = 0


2 2
2c
⇒ lim x = lim
which is a circle through origin. c→1 c→1 6 c − 1
−1 2 1
22. Let the equation of the required circle be and lim y = lim ⇒ lim x = and lim y = −
c→1 c→1 30 c − 5 c→1 5 c→1 25
x2 + y2 + 2 gx + 2 fy + c = 0 …(i)
2 1
It passes through (−4, 3). ∴ Centre =  lim x, lim y =  , − 
c→1 c→1  5 25
∴ 25 − 8 g + 6 f + c = 0 …(ii) 2 2
 2  1 64 1 1601
Since, circle touches the line x + y − 2 = 0 and ∴ Radius = 2 −  + 0 +  = + =
x − y − 2 = 0.  5  25 25 625 25
 −g − f − 2
   −g + f − 2
  2  1
2 2
∴  = = g2 + f 2 − c …(iii) ∴ Equation of the circle is  x −  +  y +  =
1601
 2   2   5  25 625
 −g − f − 2
  −g + f − 2
 =  4x 2 y 4 1 1601
Now, ⇒ x2 + y 2 − + + + − =0
 2   2  5 25 25 625 625
⇒ − g − f − 2 = ± (− g + f − 2) ⇒ 25(x2 + y2) − 20x + 2 y − 60 = 0
⇒ − g − f −2 = − g + f −2
Topic 5 Equation of Chord Bisected at a
or − g − f −2 = g − f + 2
Point, Product of Pair of Tangents,
⇒ f = 0 or g = − 2
Chord of Contact of Tangent, Pole
Case I When f = 0 and Equations of Polar
From Eq. (iii), we get
1. Key Idea Firstly find the centre of the given circle and write
 − g − 2
  the coordinates of mid point ( A ), of line segment PQ, since
= g −c
2
 2  AC ⊥ PQ therefore use (slope of AC) × (slope of PQ) = − 1

⇒ ( g + 2)2 = 2 ( g 2 − c) It is given that points P and Q are intersecting points of


⇒ g − 4 g − 4 − 2c = 0
2
…(iv) circle
(x − 3)2 + ( y + 2 )2 = 25 …(i)
On putting f = 0 in Eq. (ii). we get
Line y = mx + 1 …(ii)
25 − 8 g + c = 0 …(v) −3
And, the mid-point of PQ is A having x-coordinate so
Eliminating c between Eqs. (iv) and (v), we get 5
g 2 − 20 g + 46 = 0 y-coordinate is 1 − m.
3
⇒ g = 10 ± 3 6 and c = 55 ± 24 6 5
On substituting the values of g , f and c in Eq. (i), we get  3 3 
So, A  − , 1 − m
 5 5 
x2 + y2 + 2 (10 ± 3 6 ) x + (55 ± 24 6 ) = 0
Case II When g = − 2 From the figure,
Q AC ⊥ PQ
From Eq. (iii), we get
Y
⇒ f 2 = 2 (4 + f 2 − c)
⇒ f 2 − 2c + 8 = 0 …(vi) Q
On putting g = − 2 in Eq. (ii), we get
c = − 6 f − 41 A

On substituting c in Eq. (vi), we get O X


f 2 + 12 f + 90 = 0 P C
(3,-2)
This equation gives imaginary values of f.
Thus, there is no circle in this case.
434 Circle

⇒ (slope of AC) × (slope of PQ) = −1 Also, area of ∆C1 AC 2 =


1
C1C 2 × AM
 3  2
 −2 − 1 + m  1 AB  AB 
5  × m = −1 = × 13 ×
⇒ 
3 2 2 Q AM = 2 
 3+ 
 5  1
∴ × 13 × AM = 30 cm
(3 /5 ) m − 3  3m − 15 4
⇒ m = −1 ⇒   m = −1 120
18 / 5  18  AM = cm
13
⇒ 3m2 − 15m + 18 = 0 ⇒ m2 − 5m + 6 = 0
⇒ m = 2 or 3 3. Given equation of line is x + y = n,n ∈ N …(i)
and equation of circle is x2 + y2 = 16 …(ii)
2. Let the length of common chord = AB = 2 AM = 2x
Now, for intercept, made by circle (ii) with line (i)
A
12 5 x2+y2=16
(0, 0)
C1 C2
M
12 5 4
d
B
A B x+y=n, n 0 N
Now, C1C 2 = AC12 + AC 22 … (i) d <4
n
[Qcircles intersect each other at 90º] ⇒ <4
2
and C1C 2 = C1M + MC 2
[Q d = perpendicular distance from (0, 0) to the line
⇒ C1C 2 = 122 − AM 2 + 52 − AM 2 … (ii) |0 + 0 − n| n 
x + y = n and it equal to = 
From Eqs. (i) and (ii), we get 12 + 12 2
AC12 + AC 22 = 144 − AM 2 + 25 − AM 2 ⇒ n <4 2 …(iii)
Q n ∈ N , so n = 1, 2, 3, 4, 5
⇒ 144 + 25 = 144 − x2 + 25 − x2
Clearly, length of chord AB = 2 42 − d 2
⇒ 13 = 144 − x + 25 − x
2 2

n2  n 
On squaring both sides, we get = 2 16 − Q d = 2 
2  
169 = 144 − x2 + 25 − x2 + 2 144 − x2 25 − x2 ∴ Sum of square of all possible lengths of chords (for
n = 1, 2, 3, 4, 5)
⇒ x = 144 − x
2 2
25 − x 2
 1 
Again, on squaring both sides, we get = 4 (16 × 5) − (12 + 22 + 32 + 42 + 52)
 2 
x4 = (144 − x2) (25 − x2) = (144 × 25) − (25 + 144)x2 + x4 5(6)(11)
= 320 − 2
144 × 25 6
⇒ x2 =
169 = 320 − 110 = 210
12 × 5 60
⇒ x= = cm 4. Given equation of circle is
13 13
x2 + y2 − 8x − 8 y − 4 = 0, whose centre is C (4, 4) and
120
Now, length of common chord 2x = cm
13 radius = 42 + 42 + 4 = 36 = 6
Alternate Solution
Let the centre of required circle be C1 (x, y). Now, as it
Given, AC1 = 12 cm and AC 2 = 5 cm touch the X-axis, therefore its radius = y . Also, it
In ∆C1 AC 2, touch the circle
C1C 2 = (C1 A )2 + ( AC 2)2 [Q∠C1 AC 2 = 90º, x2 + y2 − 8x − 8 y − 4 = 0, therefore CC1 = 6 + y
because circles intersects each other at 90º] ⇒ (x − 4)2 + ( y − 4)2 = 6 + y

= (12)2 + (5)2 = 144 + 25 ⇒ x2 + 16 − 8x + y2 + 16 − 8 y = 36 + y2 + 12 y


= 169 = 13 cm ⇒ x − 8x − 8 y + 32 = 36 + 12 y
2

1 ⇒ x2 − 8x − 8 y − 4 = 12 y
Now, area of ∆C1 AC 2 = AC1 × AC 2
2 Case I If y > 0, then we have
1
= × 12 × 5 = 30 cm2 x2 − 8x − 8 y − 4 = 12 y
2
Circle 435

⇒ x2 − 8x − 20 y − 4 = 0 A
⇒ x2 − 8x − 4 = 20 y
(1, 8)P O(3 ,2)
⇒ (x − 4)2 − 20 = 20 y
⇒ (x − 4) = 20 ( y + 1,) which is a parabola.
2
B
Case II If y < 0, then we have
7. We have to find locus of mid-point of chord and we know
x2 − 8x − 8 y − 4 = − 12 y perpendicular from centre bisects the chord.
⇒ x − 8x − 8 y − 4 + 12 y = 0
2
∴ ∠OAC = 45°
⇒ x2 − 8x + 4 y − 4 = 0
⇒ x2 − 8x − 4 = − 4 y A
⇒ (x − 4) 2 = 20 − 4 y 2 45°
⇒ (x − 4)2 = − 4( y − 5), which is again a parabola.

(h , k )
O
C
5. PLAN If S : ax 2 + 2 hxy + by 2 + 2 gx + 2 fy + C
2
then equation of chord bisected at P ( x 1, y1 ) is T = S1
or a xx 1 + h( xy1 + yx 1 ) + b yy1 + g ( x + x 1 ) + f ( y + y1 ) + C B
2 2
= ax 1 + 2 hx 1 y1 + by1 + 2 gx 1 + 2 fy1 + C
OC 2
Description of Situation As equation of chord of In ∆ OAC , = sin 45° ⇒ OC = = 2
OA 2
contact is T = 0
Equation of polar
Also, h 2 + k2 = OC 2

Hence, x2 + y2 = 2 is required equation of locus of


P mid-point of chord subtending right angle at the centre.
8. Since, tangents are drawn from origin. So, the equation
of tangent be y = mx.
P'
⇒ Length of perpendicular from origin = radius
Equation of chord of contact A
ius
Here, equation of chord of contact w.r.t. P is (0, 0) Rad
 4λ − 20 = r
xλ + y.   =9 O
 5  (– r, h)
5λx + (4λ − 20) y = 45 …(i)
B
A
|mr + h |
Q ⇒ =r
l, 4l–20 P m2 + 1
9
y 2=

5 (h,k) (0,0)
x 2+

B
⇒ m2r 2 + h 2 + 2mrh = r 2 (m2 + 1)
S1=T
r 2 − h 2
and equation of chord bisected at the point Q (h , k) is ⇒ m = , m = ∞
 2rh 
xh + yk − 9 = h 2 + k2 − 9
r 2 − h 2
⇒ xh + ky = h 2 + k2 …(ii) ∴ Equation of tangents are y =   x, x = 0
From Eqs. (i) and (ii), we get  2rh 
5λ 4λ − 20 45 Therefore (a) and (c) are the correct answers.
= = 2 9. Equation of given circle C is
h k h + k2
20h 9h (x − 3)2 + ( y + 5)2 = 9 + 25 − 30
∴ λ= and λ = 2 i.e. (x − 3)2 + ( y + 5)2 = 22
4h − 5k h + k2
20h 9h Centre = (3, − 5)
⇒ = 2
4h − 5k h + k2 If L1 is diameter, then 2(3) + 3(−5) + p − 3 = 0 ⇒ p = 12
or 20 (h 2 + k2) = 9 (4h − 5k) ∴ L1 is 2x + 3 y + 9 = 0
or 20 (x2 + y2) = 36x − 45 y L 2 is 2x + 3 y + 15 = 0
6. For required circle, P (1, 8) and O (3, 2) will be the end Distance of centre of circle from L 2 equals
point of its diameter.
2(3) + 3(−5) + 15 6
∴ (x − 1) (x − 3) + ( y − 8) ( y − 2) = 0 = <2 [radius of circle]
2 +3
2 2 13
⇒ x2 + y2 − 4x − 10 y + 19 = 0
436 Circle

∴ L 2 is a chord of circle C. 13. Given, (x + 2)2 + ( y − 3)2 = 4


Statement II is false. Let the coordinate be M (h , k), where B is mid-point of A
10. A point on the line 2x + y = 4 is of the form (h , 4 − 2h ). and M.
Equation of the chord of contact is T = 0 i.e.  h k + 3
⇒ B , 
hx + (4 − 2h ) y = 1 ⇒ (4 y − 1) + h (x − 2 y) = 0 2 2 
This line passes through the point of intersection of But AB is the chord of circle
 1 1 x2 + 4 x + ( y − 3 ) 2 = 0
4 y − 1 = 0 and x − 2 y = 0 i.e. through the point  ,  .
 2 4
Thus, B must satisfy above equation.
11. Given, 4x2 + 4 y2 − 12x + 4 y + 1 = 0 h 2 4h 1 
2
∴ + +  (k + 3) − 3 = 0
⇒ x + y − 3x + y + 1 / 4 = 0
2 2
4 2  2 
3 1 ⇒ h 2 + k2 + 8h − 6k + 9 = 0
whose centre is  , −  and radius
2 2
∴ Locus of M is the circle
2 2
 3  1 1 9 1 1 3 x2 + y 2 + 8 x − 6 y + 9 = 0
=   + −  − = + − =
 2  2 4 4 4 4 2 14. Equation of straight line passing through intersection
A of two circles C1 and C 2 is (S1 − S 2) = 0.
2
∴ x2 + y2 − x + 4 y − 3 − (x2 + y2 + 6x + 2 y − 15) = 0
π/3
3
S C D 20
⇒ x − 2 y − 12 = 0 ⇒ 10x − 3 y − 18 = 0
3
B 15. Here, length of tangent AB
Again, let S be a circle with centre at C and AB is given = 42 + 52 − 4(4) − 2(5) − 11 = 2
chord and AD subtend angle 2π / 3 at the centre and D be
the mid point of AB and let its coordinates are (h, k).
B
1 1 2π π
Now, ∠DCA = (∠BCA ) = ⋅ =
2 2 3 3 2 4
(4, 5)
Using sine rule in ∆ADC, O
A
(2, 1)
DA CA
=
sin π / 3 sin π / 2
C
3 3
⇒ DA = CA sin π / 3 = ⋅
2 2
∴ Area of quadrilateral ABOC = 2 area of ∆ABO
Now, in ∆ACD 1
9 27 9 = 2 ⋅ ( AB) (OB) = 2 ⋅ 4 = 8 sq units
CD 2 = CA 2 − AD 2 = − = 2
4 16 16
16. For the equation of circle x2 + y2 − 2x = 0. Let the
But CD 2 = (h − 3 / 2)2 + (k + 1 / 2)2
9 mid-point of chords be (h , k).
⇒ (h − 3 / 2)2 + (k + 1 / 2)2 = ∴ Equation of chord bisected at the point is S1 = T .
16
∴ h 2 + k2 − 2h = xh + yk − (x + h ) which passes through
Hence, locus of a point is (0, 0).
2 2
 3  1 9 ⇒ h 2 + k2 − 2h = − h
x −  +  y +  =
 2  2 16
⇒ 16x2 + 16 y2 − 48x + 16 y + 31 = 0
M
12. Area of triangle formed by the tangents from the point (0, 0) O
(h , k) to the circle x2 + y2 = a 2 and their chord of contact (1, 0)

(h 2 + k2 − a 2)3/ 2
=a
h 2 + k2
∴ The required locus of a chord is x2 + y2 − x = 0
Thus, area of triangle formed by tangents from (4, 3) to
the circle x2 + y2 = 9 and their chord of contact 17. 2x2 + y2 − 3xy = 0 [given]
3 (42 + 32 − 9)3/ 2 3 (16 + 9 − 9)3/ 2 ⇒ 2x − 2xy − xy + y = 0
2 2
= =
42 + 32 25 ⇒ 2x (x − y) − y (x − y) = 0
=
3 (64) 192
= sq units ⇒ (2x − y) (x − y) = 0
25 25 ⇒ y = 2x, y = x
Circle 437

are the equations of straight lines passing through From figure it is clear that, ∆OLS is a right triangle
origin. with right angle at L.
Now, let the angle between the lines be 2 θ and the line Also, OL = 1 and OS = 2
y = x makes angle of 45° with X-axis. 1
∴ 1 sin (∠ LSO ) = ⇒ ∠ LSO = 30°
Therefore, tan (45° + 2 θ ) = 2 (slope of the line y = 2x) 2
Since, SA1 = SA2, ∆ SA1 A2 is an equilateral triangle.
Y y = 2x
The circle with centre at C1 is a circle inscribed in the
y=x
∆ SA1 A2. Therefore, centre C1 is centroid of ∆ SA1 A2.
3 C
3 This, C1 divides SM in the ratio 2 : 1. Therefore,
A
coordinates of C1 are (−4 / 3, 0) and its radius
= C1M = 1 / 3
∴ Its equation is (x + 4 / 3)2 + y2 = ( 1 / 3)2 …(i)
45°
The other circle touches the equilateral triangle SB1B2
X′ X ∆
O externally. Its radius r is given by r = ,
s−a
Y′ 1 3
where B1 B2 = a. But ∆ = (a ) (SN ) = a
tan 45° + tan 2 θ 1 + tan 2 θ 2 2
⇒ =2 ⇒ =2
1 − tan 45° × tan 2 θ 1 − tan 2 θ 3 a
and s− a = a −a =
(1 + tan 2 θ ) − (1 − tan 2 θ ) 2 − 1 1 2 2
⇒ = =
(1 + tan 2 θ ) + (1 − tan 2 θ ) (2 + 1) 3 Thus, r =3
2 tan 2 θ 1 1 2 tan θ 1 ⇒ Coordinates of C 2 are (4, 0).
⇒ = ⇒ tan 2 θ = ⇒ =
2 3 3 1 − tan 2 θ 3 ∴ Equation of circle with centre at C 2 is
⇒ (2 tan θ ) ⋅ 3 = 1 − tan 2 θ (x − 4)2 + y2 = 32 …(ii)
⇒ tan 2 θ + 6 tan θ − 1 = 0 Equations of common tangents to circle (i) and circle C
− 6 ± 36 + 4 × 1 × 1 − 6 ± 40 are
⇒ tan θ = =
2 2 1
x = − 1 and y = ± (x + 2) [T1 and T2]
⇒ tan θ = − 3 ± 10 3
 π
⇒ tan θ = − 3 + 10 Q0 < θ < 4  Equation of common tangents to circle (ii) and circle C
Again, in ∆ OCA are
3 1
tan θ = x = 1 and y = ± (x + 2) [T1 and T2]
OA 3
3 3 3 (3 + 10 )
∴ OA = = = Two tangents common to (i) and (ii) are T1 and T2 at O.
tan θ (− 3 + 10 ) (− 3 + 10 ) (3 + 10 ) To find the remaining two transverse tangents to (i) and
3 (3 + 10 ) (ii), we find a point I which divides the joint of C1 C 2 in
= = 3 (3 + 10 )
(10 − 9) the ratio r1 : r2 = 1 / 3 : 3 = 1 : 9
18. Y Therefore, coordinates of I are (−4 / 5, 0)
Equation of any line through I is y = m (x + 4 / 5). It will
T1
touch (i) if
B1
  − 4 4
 
m +  −0 
A1 L   3 5  1 8m 1
30° = ⇒ − = 1 + m2
X′ X 1 + m2 3  15  3
S C1 M O N C2
A2
⇒ 64 m2 = 25 (1 + m2)
B2 ⇒ 39 m2 = 25
5
T2 ⇒ m=±
39
Y′
5  4
Therefore, these tangents are y = ± x + 
39  5
l
C1 C2 19. Let the coordinate of point P be (2r cos θ , 2r sin θ )
We have, OA = r , OP = 2r
Since, ∆ OAP is a right angled triangle.
438 Circle

Equation of the chord having (α , − α ) as mid points is


C1 T = S1
A C2
 1 + 2a   1 − 2a 
θ−φ ⇒ xα + y (− α ) −   (x + α ) −   (y − α )
θ  4   4 
P X
φ O  1 + 2a   1 − 2a 
= α 2 + (− α )2 −  α −  (− α )
B  2   2 

⇒ 4xα − 4 yα − (1 + 2a ) x − (1 + 2a ) α
− (1 − 2a ) y + (1 − 2a ) α
cos φ = 1 / 2 ⇒ φ = π /3
∴Coordinates of A are { r cos (θ − π / 3), r sin (θ − π / 3)} = 4α + 4α − (1 + 2a ) ⋅ 2α + (1 − 2a ) ⋅ 2α
2 2

and that of B are [r cos (θ + π / 3), r sin (θ + π / 3)] ⇒ 4αx − 4αy − (1 + 2a ) x − (1 − 2a ) y


If p, q is the centroid of ∆ PAB, then = 8 α 2 − (1 + 2a ) α + (1 − 2a ) α
1
p = [r cos (θ − π / 3) + r cos (θ + π / 3) + 2r cos θ ] But this chord will pass through the point
3
 1 + 2a 1 − 2a 
1  , .
= [r {cos (θ − π / 3) + cos (θ + π / 3)} + 2r cos θ ]  2 2 
3
  π π π π   1 + 2a   1 − 2a  (1 + 2a ) (1 + 2a )
θ− +θ+ θ − −θ−  4α   −4α   −
1  3 3 3 3 
= r 2 cos . cos  + 2r cos θ   2   2  2
3  2 2   (1 − 2a ) (1 − 2a )
    −
2
1
= [r {2 cos θ cos π / 3} + 2r cos θ ] = 8 α 2 − 2 2 aα
3
1 ⇒ 2 α [(1 + 2a − 1 + 2a )] = 8 α 2 − 2 2 aα
= [r ⋅ cos θ + 2r cos θ ] = r cos θ 1
3 ⇒ 4 2 aα − [2 + 2 ( 2a )2] = 8 α 2 − 2 2aα
2
1  π  π
and q = [r sin θ −  + r sin θ +  + 2r sin θ ] [Q (a + b)2 + (a − b)2 = 2a 2 + 2b2]
3  3  3
⇒ 8 α 2 − 6 2 aα + 1 + 2a 2 = 0
1  π  π
= [r {sin θ −  + sin θ +  } + 2r sin θ ]
3  3  3 But this quadratic equation will have two distinct
roots, if
  π π π π 
θ− +θ+ θ − − θ− 
1  3 3 3 3  (6 2a )2 − 4 (8) (1 + 2a 2) > 0 ⇒ 72a 2 − 32 (1 + 2a 2) > 0
= r 2 sin . cos  + 2 r sin θ 
3  2 2   ⇒ 8a 2 − 32 > 0 ⇒ a 2 − 4 > 0 ⇒ a < − 2 ∪ a > 2
   
1 Therefore, a ∈ (− ∞ , − 2) ∪ (2, ∞ ) .
= [r (2 sin θ cos π / 3) + 2r sin θ ]
3 21. The given circle can be rewritten as
1 by
= [r (sin θ ) + 2r sin θ ] = r sin θ x2 + y2 − ax − =0 …(i)
3 2
Now, ( p, q) = (r cos θ , r sin θ) lies on x2 + y2 = r 2 which is
Let one of the chord through (a , b / 2) be bisected at (h , 0).
called C1.
Then, the equation of the chord having (h , 0) as
20. Given, 2x2 + 2 y2 − (1 + 2a ) x − (1 − 2a ) y = 0 mid-point is
 1 + 2a   1 − 2a  T = S1
⇒ x2 + y 2 −   x−  y=0 a b
 2   2  ⇒ h ⋅ x + 0 ⋅ y − (x + h ) − ( y + 0) = h 2 + 0 − ah − 0
2 4
Since, y + x = 0 bisects two chords of this circle, mid  a by a
points of the chords must be of the form (α , − α ). ⇒ h −  x − − h = h 2 − ah …(ii)
 2 4 2
Y It passes through (a , b / 2), then
y+

 a b b a
 h −  a − ⋅ − h = h − ah
2
x=

 2
0

O 4 2 2
X 3 a2 b2
(α,−α) ⇒ h 2 − ah + + =0 …(iii)
2 2 8
According to the given condition, Eq. (iii) must have two
distinct real roots. This is possible, if the discriminant of
(α,−α) Eq. (iii) is greater than 0.
Circle 439

9 2  a 2 b2 a 2 b2 y1 k − y1
i.e. a −4  +  >0 ⇒ − >0 ⇒ ⋅ = −1 ⇒ ky1 − y12 = − hx1 + x12
4  2 8 4 2 x1 h − x1
⇒ a 2 > 2b2 Hence, required locus is x2 + y2 = hx + ky

22. Since, 5x + 12 y − 10 = 0 and 5x − 12 y − 40 = 0 are both 24. Equation of tangents at (1, 7) and (4, − 2) are
perpendicular tangents to the circle C1. B (1,7) C
∴ OABC forms a square.
Let the centre coordinates be (h , k), where,
OC = OA = 3 and OB = 6 2
D (4,–2)
|5h + 12k − 10| |5h + 12k − 40|
⇒ = 3 and =3 A (1,2)
13 13

Q C1

x + 7 y − (x + 1) − 2 ( y + 7) − 20 = 0
O (h,k)
⇒ 5 y = 35 ⇒ y = 7
3
and 4x − 2 y − (x + 4) − 2 ( y − 2) − 20 = 0
40

C A
y=
5x

3 ⇒ 3x − 4 y = 20
+

12
12

∴ Point C is (16, 7).


5x
y=
10

–15
B 5,
12 ∴ Vertices of a quadrilateral are
A (1, 2), B(1, 7), C (16, 7), D (4, − 2)
⇒ 5h + 12k − 10 = ± 39 and 5h − 12k − 40 = ± 39 ∴ Area of quadrilateral ABCD
on solving above equations. The coordinates which lie in = Area of ∆ ABC + Area of ∆ ACD
I quadrant are (5, 2).
1 1
= × 15 × 5 + × 15 × 5 = 75 sq units
C2 2 2
A 25. As we know that the equation of family of circles passes
5
4
through the points of intersection of given circle
(5,2)
3 O x2 + y2 = r 2 and line PQ : 2x + 4 y = 5 is,
C1 (x2 + y2 − r 2) + λ (2x + 4 y − 5 ) = 0 …(i)
4
Since, the circle (i) passes through the centre of circle
B x2 + y2 = r 2,
5x − 12y = 10 So, − r − 5λ = 0 ⇒ 5λ + r 2 = 0
2
… (ii)
∴ Centre for C1 (5, 2) and the centre of circle (i) lies on the line x + 2 y = 4, so
centre (− λ , − 2λ ) satisfy the line x + 2 y = 4.
To obtain equation of circle concentric withC1 and making
Therefore, − λ − 4λ = 4
an intercept of length 8 on5x + 12 y = 10 and5x − 12 y = 40
⇒ − 5λ = 4 ⇒ r 2 = 4 {from Eq. (ii)}
∴ Required equation of circle C 2 has centre (5, 2) and
radius 5 is (x − 5)2 + ( y − 2)2 = 52 ⇒ r =2
π x
23. Given,circle is x2 + y2 = r 2 26. Let θ= ⇒ cos θ =
2k 2
Equation of chord whose mid point is given, is
3 +1−x
T = S1 ⇒ xx1 + yy1 − r 2 = x12 + y12 − r 2 ⇒ cos 2 θ =
It also passes through (h, k) hx1 + ky1 = x12 + y12 2 x
3 +1−x 2 θ
∴ Locus of (x1 , y1 ) is ⇒ 2 cos θ − 1 =
2
C
2
 x2  2
3 +1−x 2θ √3 + 1 – x
⇒ 2   −1 =
 4 2
C (0, 0)
− 1 ± 1 + 12 + 4 3
P (h, k)
⇒ x2 + x − 3 − 3 = 0 ⇒ x =
B 2
M
A (x1, y1)
− 1 ± 13 + 4 3 −1 + 2 3 + 1
= = = 3
x2 + y2 = hx + ky 2 2
Alternate Solution 3 π
∴ cos θ =⇒θ =
Let M be the mid-point of chord AB. 2 6
⇒ CM ⊥ MP π π
∴ Required angle = = 2 θ = ⇒ k =3
⇒ (slope of CM) ⋅ (slope of MP) = − 1 k 3
17
Parabola

Topic 1 Equation of Parabola and Focal Chord


Objective Questions I (Only one correct option) 7. Let( x , y ) be any point on the parabola y 2 = 4x . Let
1. The locus of a point which divides the line segment joining P be the point that divides the line segment from
the point ( 0, − 1) and a point on the parabola, x = 4 y, 2 (0, 0) to ( x , y ) in the ratio 1 : 3. Then, the locus of P
is (2011)
internally in the ratio 1 : 2, is (2020 Main, 8 Jan I)
(a) x2 = y (b) y2 = 2x
(a) 4x2 − 3 y = 2 (b) x2 − 3 y = 2
(c) y2 = x (d) x2 = 2 y
(c) 9x2 − 12 y = 8 (d) 9x2 − 3 y = 2
8. Axis of a parabola is y = x and vertex and focus
2. If the area of the triangle whose one vertex is at the vertex
of the parabola, y + 4( x − a ) = 0 and the other two
2 2 are at a distance 2 and 2 2 respectively
vertices are the points of intersection of the parabola and from the origin. Then, equation of the parabola
Y -axis, is 250 sq units, then a value of ‘a’ is is (2006, 3M)
(2019 Main, 11 Jan, II) (a) (x − y)2 = 8 (x + y − 2)
(b) (x + y)2 = 2 (x + y − 2)
(a) 5 5 (b) 5 (c) 5(21/3 ) (d) (10)2/3
(c) (x − y)2 = 4 (x + y − 2)
3. A circle cuts a chord of length 4a on the X-axis and passes (d) (x + y)2 = 2 (x − y + 2)
through a point on the Y-axis, distant 2b from the origin. 9. The locus of the mid-point of the line segment
Then, the locus of the centre of this circle, is joining the focus to a moving point on the
(2019 Main, 11 Jan, II)
parabola y 2 = 4ax is another parabola with
(a) a parabola (b) an ellipse
(c) a straight line (d) a hyperbola directrix (2002, 1M)
a
4. Axis of a parabola lies along X-axis. If its vertex and focus (a) x = − a (b) x = −
2
are at distances 2 and 4 respectively from the origin, on a
(c) x = 0 (d) x =
the positive X-axis, then which of the following points 2
does not lie on it? (2019 Main, 9 Jan, I)
10. The equation of the directrix of the parabola
(a) (4, −4) (b) (6, 4 2) (c) (8, 6) (d) (5, 2 6) y 2 + 4 y + 4x + 2 = 0 is (2001, 1M)
5. Let P be the point on the parabola, y = 8x, which is at a
2
(a) x = − 1 (b) x = 1
minimum distance from the centre C of the circle, (c) x = − 3 / 2 (d) x = 3 / 2
x 2 + ( y + 6)2 = 1. Then, the equation of the circle, passing 11. If the line x − 1 = 0 is the directrix of the parabola
through C and having its centre at P is (2016 Main) y 2 − kx + 8 = 0, then one of the values of k is
(a) x2 + y2 − 4x + 8 y + 12 = 0 (2000, 2M)
(b) x2 + y2 − x + 4 y − 12 = 0 1
(a) (b) 8
x 8
(c) x2 + y2 − + 2 y − 24 = 0
4 1
(c) 4 (d)
(d) x2 + y2 − 4x + 9 y + 18 = 0 4
6. Let O be the vertex and Q be any point on the parabola 12. The curve described parametrically by
x 2 = 8 y. If the point P divides the line segment OQ x = t 2 + t + 1, y = t 2 − t + 1 represents (1999, 2M)
internally in the ratio 1 : 3, then the locus of P is (2015) (a) a pair of straight lines (b) an ellipse
(a) x2 = y (b) y2 = x (c) y2 = 2x (d) x2 = 2 y (c) a parabola (d) a hyperbola
Parabola 441

Assertion and Reason Integer & Numerical Answer Type Questions


2
x 14. Let the curve C be the mirror image of the
13. Statement I The curve y = − + x + 1 is symmetric
2 parabola y 2 = 4x with respect to the line
with respect to the line x = 1. because (2007, 3M) x + y + 4 = 0. If A and B are the points of
Statement II A parabola is symmetric about its axis. intersection of C with the line y = − 5, then the
(a) Statement I is correct, Statement II is correct, Statement distance between A and B is (2015 Adv.)
II is a correct explanation for Statement I 15. Let S be the focus of the parabola y 2 = 8x and PQ
(b) Statement I is correct, Statement II is correct, Statement II be the common chord of the circle
is not a correct explanation for Statement I
x 2 + y 2 − 2x − 4 y = 0 and the given parabola. The
(c) Statement I is correct, Statement II is incorrect
(d) Statement I is incorrect, Statement II is correct
area of ∆PQS is (2012)

Topic 2 Equation of Tangents and Properties


Objective Questions I (Only one correct option) parabola at Aand B, respectively. IfC is the centre
1. Let L1 be a tangent to the parabola y = 4( x + 1)and L2 be
2 of the circle through the points P, A and B and
∠CPB = θ, then a value of tanθ is (2018 Main)
a tangent to the parabola y 2 = 8( x + 2) such that L1 and 1 4
(a) (b) 2 (c) 3 (d)
L2 intersect at right angles. Then, L1 and L2 meet on the 2 3
straight line (2020 Main, 6 Sep I) 8. The radius of a circle having minimum area,
(a) x + 3 = 0 (b) 2x + 1 = 0 which touches the curve y = 4 − x 2 and the lines
(c) x + 2 = 0 (d) x + 2 y = 0
y =|x|, is (2017 Main)
2. If the line ax + y = c, touches both the curves x 2 + y 2 = 1 (a) 2 ( 2 + 1) (b) 2 ( 2 − 1)
and y 2 = 4 2x, then| c|is equal to (c) 4 ( 2 − 1) (d) 4 ( 2 + 1)
(2019 Main, 10 April, II) 9. The slope of the line touching both the parabolas
1 1
(a) (b) 2 (c) 2 (d) y 2 = 4x and x 2 = − 32 y is (2014 Main)
2 2 1 3 1 2
(a) (b) (c) (d)
3. The tangents to the curve y = ( x − 2)2 − 1 at its points of 2 2 8 3
intersection with the line x − y = 3, intersect at the point 10. The tangent at (1, 7) to the curves x 2 = y − 6x
(2019 Main, 12 April II)
touches the circle x 2 + y 2 + 16x + 12 y + c = 0 at
(a)  , 1 (b)  − , − 1 (c)  , − 1 (d)  − , 1
5 5 5 5
(2005 2M)
2   2  2   2 
(a) (6, 7) (b) (–6, 7) (c) (6, –7) (d) (–6, –7)
4. The area (in sq units) of the smaller of the two circles 11. The angle between the tangents drawn from the
that touch the parabola, y 2 = 4x at the point (1, 2) and
point (1,4) to the parabola y 2 = 4x is (2004, 1M)
the X-axis is (2019 Main, 9 April, II) π π π π
(a) (b) (c) (d)
(a) 8 π( 3 − 2 2 ) (b) 4 π(3 + 2) 6 4 3 2
(c) 8 π(2 − 2 ) (d) 4 π(2 − 2 ) 12. The focal chord to y 2 = 16 x is tangent to
5. The equation of a tangent to the parabola, x = 8 y, 2
( x − 6)2 + y 2 = 2, then the possible values of the
which makes an angle θ with the positive direction of slope of this chord are (2003, 1M)
X-axis, is (2019 Main, 12 Jan, II) (a) {−1 , 1 } (b) {−2, 2}
(a) y = x tan θ − 2 cot θ (b) x = y cot θ + 2 tan θ (c) {−2, 1 / 2} (d) {2, − 1 / 2}
(c) y = x tan θ + 2 cot θ (d) x = y cot θ − 2 tan θ
13. The equation of the common tangent to the curves
6. Equation of a common tangent to the circle, y 2 = 8x and xy = − 1 is (2002, 1M)
x 2 + y 2 − 6x = 0 and the parabola, y 2 = 4x, is
(a) 3 y = 9x + 2 (b) y = 2x + 1
(2019 Main, 9 Jan, I) (c) 2 y = x + 8 (d) y = x + 2
(a) 3 y = 3x + 1 (b) 2 3 y = 12x + 1
14. The equation of the common tangent touching the
(c) 3y = x + 3 (d) 2 3 y = − x − 12 circle ( x − 3)2 + y 2 = 9 and the parabola y 2 = 4x
7. Tangent and normal are drawn at P(16, 16) on the above the X-axis is (2001, 1M)
parabola y 2 = 16x, which intersect the axis of the
(a) 3 y = 3x + 1 (b) 3 y = − (x + 3)
442 Parabola

(c) 3 y = x + 3 (d) 3 y = − (3x + 1)


Assertion and Reason 17. The value of r is
1 t2 + 1 1 t2 − 1
15. Given A circle, 2x + 2 y = 5 and a parabola,
2 2 (a) − (b) (c) (d)
t t t t
y 2 = 4 5x.
18. If st = 1, then the tangent at P and the normal at S to
Statement I An equation of a common tangent to the parabola meet at a point whose ordinate is
these curves is y = x + 5. (t 2 + 1)2 a (t 2 + 1)2
(a) 3
(b)
5 2t 2t3
Statement II If the line, y = mx + ( m ≠ 0)is the
m a (t 2 + 1)2 a (t 2 + 2)2
(c) (d)
common tangent, then msatisfies m 4 − 3m 2 + 2 = 0. t3 t3
(2013 Main)
Fill in the Blank
(a) Statement I is correct, Statement II is correct,
Statement II is a correct explanation for Statement I 19. The point of intersection of the tangents at the ends of
(b) Statement I is correct, Statement II is correct, the latusrectum of the parabola y 2 = 4x is … .
Statement II is not a correct explanation for Statement I (1994, 2M)
(c) Statement I is correct, Statement II is incorrect Analytical & Descriptive Questions
(d) Statement I is incorrect, Statement II is correct
20. At any point P on the parabola y 2 − 2 y − 4x + 5 = 0 a
Objective Questions II tangent is drawn which meets the directrix at Q. Find
(One or more than correct option) the locus of point R, which divides QP externally in the
1
16. Equation of common tangent of ratio : 1 . (2004, 4M)
2
y = x 2 , y = − x 2 + 4x − 4 is (2006, 5M)
21. Find the shortest distance of the point ( 0, c) from the
(a) y = 4 (x − 1)
parabola y = x 2, where 0 ≤ c ≤ 5. (1982, 2M)
(b) y = 0
(c) y = − 4 (x − 1)
(d) y = − 30x − 50 Integer & Numerical Answer Type Question
22. Consider the parabola y 2 = 8x.Let ∆1 be the area of the
Passage Based Problems triangle formed by the end points of its latusrectum
Passage 1 
and the point P  , 2 on the parabola and ∆ 2 be the
Let a, r, s, t be non-zero real numbers. Let 2 
P ( at 2 , 2at ),Q , R ( ar 2 , 2ar ) andS ( as2 , 2as) be distinct area of the triangle formed by drawing tangents at P
point on the parabola y 2 = 4ax. Suppose that PQ is ∆
and at the end points of the latusrectum. Then, 1 is
the focal chord and lines QR and PK are parallel, ∆2
where K is point ( 2a , 0). (2014, Adv.) (2011)

Topic 3 Equation of Normal and Properties


Objective Questions I (Only one correct option) Match the Columns
1. If the parabolas y = 4b( x − c) and y = 8ax have a
2 2
3. Match the conditions/expressions in Column I with
common normal, then which one of the following is a statement in Column II. Normals at P , Q , R are drawn
valid choice for the ordered triad ( a , b, c) ? to y 2 = 4x which intersect at ( 3, 0). Then,
(2019 Main, 10 Jan I)
(a)  , 2, 0
1 Column I Column II
(b) (1, 1, 0)
2 
A. Area of ∆PQR p. 2
(d)  , 2, 3
1
(c) (1, 1, 3)
2  B. Radius of circumcircle of ∆PQR 5
q.
2
2. If x + y = k is normal to y 2
= 12x, then k is
(a) 3 (2000, 2M) C. Centroid of ∆PQR 5 
r.  , 0
(b) 9 2 
(c) −9 D. Circumcentre of ∆PQR 2 
(d) −3 s.  , 0
3 
Parabola 443

(a) vertex is 
2a 
Objective Questions II , 0 (b) directrix is x = 0
 3 
(One or more than one correct option) 2a
(c) latusrectum is (d) focus is (a , 0)
4. Let P be the point on the parabola y 2 = 4x, which is at 3
the shortest distance from the centre S of the circle
x 2 + y 2 − 4x − 16 y + 64 = 0. Let Q be the point on the
Integer & Numerical Answer Type Question
circle dividing the line segment SP internally. Then, 8. If the normals of the parabola y 2 = 4x drawn at the
(a) SP = 2 5 (2016 Adv.) end points of its latusrectum are tangents to the
(b) SQ : QP = ( 5 + 1) : 2 circle ( x − 3)2 + ( y + 2)2 = r 2, then the value of r 2 is
(c) the x-intercept of the normal to the parabola at P is 6 (2015 Adv.)
1
(d) the slope of the tangent to the circle at Q is Analytical & Descriptive Questions
2
5. A solution curve of the differential equation 9. Normals are drawn from the point P with slopes
dy m1 , m2 , m3 to the parabola y 2 = 4x. If locus of P with
( x + xy + 4x + 2 y + 4)
2
− y 2 = 0, x > 0, passes
dx m1m2 = α is a part of the parabola itself, then findα.
(2003, 4M)
through the point (1, 3). Then, the solution curve
(a) intersects y = x + 2 exactly at one point (2016 Adv.) 10. Three normals are drawn from the point ( c, 0) to the
(b) intersects y = x + 2 exactly at two points 1
curve y 2 = x. Show that c must be greater than .
(c) intersects y = (x + 2)2 2
(d) does not intersect y = (x + 3)2 One normal is always the X-axis. Find c for which
6. Let L be a normal to the parabola y 2 = 4x. If L passes the other two normals are perpendicular to each
through the point (9, 6), then L is given by (2011) other. (1991, 4M)
(a) y − x + 3 = 0 (b) y + 3x − 33 = 0 11. Find the equation of the normal to the curve x 2 = 4 y
(c) y + x − 15 = 0 (d) y − 2x + 12 = 0
which passes through the point (1, 2). (1984, 4M)
7. The tangent PT and the normal PN to the parabola
12. Suppose that the normals drawn at three different
y 2 = 4ax at a point P on it meet its axis at points T and
points on the parabola y 2 = 4x pass through the
N, respectively. The locus of the centroid of the triangle
PTN is a parabola, whose (2009)
point ( h , 0). Show that h > 2. (1981, 4M)

Topic 4 Diameter, Chord of Contact, Chord Bisected and


Product of Pair of Tangents
Objective Questions II radius r having AB as its diameter, then the slope of
(One or more than one correct option) the line joining A and B can be (2010)
1 1 2 2
(a) − (b) (c) (d) −
1. If one end of a focal chord of the parabola, y 2 = 16x is at r r r r
(1, 4), then the length of this focal chord is
(2019 Main, 9 April, I) Passage Based Problems
(a) 22 (b) 25 (c) 24 (d) 20
Passage
2. The length of the chord of the parabola x 2 = 4 y having
Let PQ be a focal chord of the parabola y 2 = 4ax. The
equation x − 2 y + 4 2 = 0 is (2019 Main, 10 Jan II) tangents to the parabola at P and Q meet at a point
(a) 8 2 (b) 2 11 (c) 3 2 (d) 6 3 lying on the line y = 2x + a , a > 0. (2013 Adv.)
3. If a chord, which is not a tangent, of the parabola 5. Length of chord PQ is
y 2 = 16x has the equation 2x + y = p, and mid-point (a) 7a (b) 5a
( h , k), then which of the following is(are) possible (c) 2a (d) 3a
value(s) of p, h and k? (2017 Adv.) 6. If chord PQ subtends an angle θ at the vertex of
y 2 = 4ax , then tanθ is equal to
(a) p = − 1, h = 1, k = − 3 (b) p = 2, h = 3, k = − 4 2 −2
(c) p = − 2, h = 2, k = − 4 (d) p = 5, h = 4, k = − 3 (a) 7 (b) 7
3 3
4. Let A and B be two distinct points on the parabola 2 −2
(c) 5 (d) 5
y 2 = 4x. If the axis of the parabola touches a circle of 3 3
444 Parabola

Analytical & Descriptive Questions intersect at points P , Q and R. Determine the ratio
of the areas of the triangles ABC and PQR.(1996, 3M)
7. The angle between a pair of tangents drawn from a
point P to the parabola y 2 = 4ax is 45°. Show that the 10. Show that the locus of a point that divides a chord of
locus of the point P is a hyperbola. (1998, 8M)
slope 2 of the parabola y 2 = 4ax internally in the
8. From a point A common tangents are drawn to the ratio 1 : 2 is a parabola. Find the vertex of this
a2 parabola. (1995, 5M)
circle x 2 + y 2 = and parabola y 2 = 4ax. Find the
2 11. Through the vertex O of parabola y 2 = 4x , chords OP
area of the quadrilateral formed by the common and OQ are drawn at right angles to one another.
tangents, the chord of contact of the circle and the Show that for all positions of P, PQ cuts the axis of
chord of contact of the parabola. (1996, 2M) the parabola at a fixed point. Also, find the locus of
9. Points A, B and C lie on the parabola y = 4ax. The
2
the middle point of PQ. (1994, 4M)
tangents to the parabola at A, B and C, taken in pairs,

Answers
Topic 1 Topic 3
1. (c) 2. (b) 3. (a) 4. (c) 1. (c) 2. (b) 3. A→ p; B→ q; C→ s; D→ r
5. (a) 6. (d) 7. (c) 8. (a) 4. (a, c, d) 5. (a, d) 6. (a, b, d) 7. (a, d)
9. (c) 10. (d) 11. (c) 12. (c)  3
13. (a) 14. (4) 15. (4) 8. (2) 9. (2) 10.  
 4
Topic 2 11. x + y = 3
1. (a) 2. (c) 3. (c) 4. (a) Topic 4
5. (b) 6. (c) 7. (b) 8. (c) 1. (b) 2. (d) 3. (b) 4. (c, d)
9. (a) 10. (d) 11. (c) 12. (a)  15a 2 
13. (d) 14. (c) 15. (a) 16. (a, b) 5. (b) 6. (d) 8.   9. (2)
 4 
17. (d) 18. (b) 19. (− 1, 0)
 2 8
1 1 10.  ,  11. y 2 = 2 ( x – 4 )
20. ( x + 1 ) (y − 1 ) 2 + 4 = 0 21. c − , ≤ c ≤ 5  9 9
4 2
22. (2)

Hints & Solutions


Topic 1 Equation of Parabola and Focal Chord
⇒ 9x2 − 12 y = 8
1. Let a point on parabola x2 = 4 y is A (x1 , y1 ), Hence, option (c) is correct.
so x12 = 4 y1 …(i) 2. Vertex of parabola y2 = − 4(x − a 2) is (a 2, 0).
and an another point P (h , k) which divides the line For point of intersection with Y-axis, put x = 0 in the
segment joining the point B(0, − 1) and A (x1 , y1 ) given equation of parabola.
internally in the ratio 1 : 2, so This gives, y2 = 4a 2 ⇒ y = ± 2a
(2 × 0) + (1 × x1 ) Thus, the point of intersection are (0, 2a ) and (0, − 2a ).
h=
2+1
Y
2 × (−1) + (1 × y1 )
and k= B (0, 2a)
2+1
⇒ x1 = 3h and y1 = 3k + 2 …(ii)
Now, on the elimination of (x1 , y1 ) with the help of Eq. (i) O A
X
and relation (ii), we get (a2, 0)

9h 2 = 4(3k + 2)
C(0, –2a)
On taking locus of point P (h , k), we have
9x2 = 12 y + 8
Parabola 445

From the given condition, we have 5. Centre of circle x2 + ( y + 6)2 = 1 is C (0, − 6).
Area of ∆ABC = 250 Let the coordinates of point P be (2t 2, 4 t ).
1 1
∴ (BC )(OA ) = 250 [Q Area = × base × height] Now, let D = CP = (2t 2)2 + (4 t + 6)2
2 2
1 ⇒ D = 4 t 4 + 16 t 2 + 36 + 48 t
⇒ (4a )a = 250 ⇒ a = 125 = 53
2 3
2 Squaring on both sides
∴ a =5
⇒ D 2(t ) = 4 t 4 + 16 t 2 + 48 t + 36
3. According to given information, we have the following
Let F (t ) = 4 t 4 + 16 t 2 + 48 t + 36
figure.
For minimum, F ′ (t ) = 0
(0, 2b) ⇒ 16 t3 + 32t + 48 = 0 ⇒ t3 + 2t + 3 = 0
P ⇒ (t + 1) (t 2 − t + 3) = 0 ⇒ t = − 1
Thus, coordinate of point P are (2, − 4).
4a Now , CP = 22 + (− 4 + 6)2 = 4 + 4 = 2 2
A O B Hence, the required equation of circle is
(x − 2)2 + ( y + 4)2 = (2 2 )2
Let the equation of circle be ⇒ x + 4 − 4x + y2 + 16 + 8 y = 8
2

x2 + y2 + 2 gx + 2 fy + c = 0 …(i) ⇒ x2 + y2 − 4x + 8 y + 12 = 0
According the problem, 6. PLAN Any point on the parabola x 2 = 8 y is ( 4t ,2t 2 ). Point P divides
the line segment joining of O( 0, 0) and Q( 4t ,2t 2 ) in the ratio
4a = 2 g 2 − c …(ii) 1 : 3. Apply the section formula for internal division.
Equation of parabola is x2 = 8 y ...(i)
[Q The length of intercepts made by the
circle x2 + y2 + 2 gx + 2 fy + c = 0 Let any point Q on the parabola (i) is (4t , 2t 2).
with X-axis is 2 g 2 − c] Let P (h , k) be the point which divides the line segment
joining (0, 0) and (4t , 2t 2) in the ratio 1 : 3.
Also, as the circle is passing through P (0, 2b) Y
∴ 0 + 4b2 + 0 + 4bf + c = 0 [using Eq. (i)] )
,k
⇒ 4b2 + 4bf + c = 0 …(iii) p (h
3 Q(4t, 2t 2)
Eliminating ‘c’ from Eqs. (ii) and (iii), we get 1:
4b2 + 4bf + g 2 − 4a 2 = 0 X′ X
(0, 0) O
[Q4a = 2 g 2 − c ⇒ c = g 2 − 4a 2]
So, locus of (− g , − f ) is
Y′
4b2 − 4by + x2 − 4a 2 = 0 ⇒ x2 = 4by + 4a 2 − 4b2 1 × 4t + 3 × 0
∴ h= ⇒ h=t
which is a parabola. 4
4. According to given information, we have the following 1 × 2t 2 + 3 × 0 t2
figure. and k= ⇒ k=
4 2
Y 1 2
⇒ k = h ⇒ 2k = h 2
[Q t = h]
2
⇒ 2 y = x2, which is required locus.
X
(2, 0) (4, 0) 7. By section formula,
x+0 y+0
h= ,k=
4 4
∴ x = 4 h, y = 4 k
Now, if the origin is shifted to (2, 0) and (X , Y ) are the
coordinates with respect to new origin, then equation Y (x, y)Q
of parabola is Y 2 = 4aX , 3
where, X = x − 2 and Y = y and a = 4 − 2 = 2 P(h, k)
1
∴ y2 = 8(x − 2) X
(0, 0) O
Note that (8, 6) is the only point which does not satisfy
the equation. y 2 = 4x
446 Parabola

Substituting in y2 = 4 x, 11. Given, y2 = kx − 8


(4 k)2 = 4 (4 h )  8
⇒ k2 = h ⇒ y2 = k  x − 
 k
or y = x is required locus.
2
Shifting the origin Y 2 = kX , where Y = y, X = x − 8 / k.
8. Since, distance of vertex from origin is 2 and focus is k
2 2. Directrix of standard parabola is X = −
4
∴ V (1, 1) and F (2, 2) (i.e. lying on y = x) 8 k
Directrix of original parabola is x = −
where, length of latusrectum k 4
= 4a = 4 2 [Q a = 2 ] 8 k
Now, x = 1 also coincides with x = −
∴ By definition of parabola k 4
Y On solving, we get k = 4
y=x 12. Given curves are x = t 2 + t + 1 …(i)
P F
and y=t −t+1 2
…(ii)
(2, 2) On subtracting Eq. (ii) from Eq. (i),
N
M x − y = 2t
(1, 1) V Thus, x = t2 + t + 1
2
x+y–2=0  x − y  x − y
⇒ x=  +  +1
X′ X  2   2 
O
⇒ 4x = (x − y)2 + 2x − 2 y + 4
⇒ (x − y)2 = 2 (x + y − 2)
Y′ ⇒ x + y − 2xy − 2x − 2 y + 4 = 0
2 2

PM 2 = (4a ) (PN ) Now, ∆ = 1⋅ 1⋅ 4 + 2 ⋅ (−1)(−1)(−1)


where, PN is length of perpendicular upon −1 × (−1)2 − 1 × (−1)2 − 4 (−1)2
x + y − 2 = 0, i.e. tangent at vertex = 4 − 2 − 1 − 1 −4 = − 4
(x − y)2  x + y − 2 ∴ ∆ ≠0
⇒ =4 2   ab − h 2 = 1 ⋅ 1 − (−1)2 = 1 − 1 = 0
2  2  and
Hence, it represents a equation of parabola.
⇒ (x − y)2 = 8 (x + y − 2)
x2 3 1
9. Let P (h , k) be the mid-point of the line segment joining 13. y = − + x + 1 ⇒ y − = − (x − 1 )2
2 2 2
the focus (a , 0) and a general point Q (x, y) on the
⇒ It is symmetric about x = 1.
parabola. Then,
x+ a y Hence, option (a) is correct.
h= ,k= ⇒ x = 2h − a , y = 2k.
2 2 14. Let P (t 2 , 2t ) be a point on the curve y2 = 4x, whose
Put these values of x and y in y2 = 4ax, we get image is Q(x , y) on x + y + 4 = 0, then
4k2 = 4a (2h − a ) x − t 2 y − 2t −2(t 2 + 2t + 4)
= =
⇒ 4k = 8ah − 4a 2 ⇒ k2 = 2ah − a 2
2
1 1 12 + 12
So, locus of P (h , k) is y2 = 2ax − a 2 ⇒ x = − 2t − 4
 a
⇒ y2 = 2 a  x −  and y = − t2 − 4
 2
a a Y
Its directrix is x − = − ⇒ x = 0.
2 2
y 2 = 4x
10. Given, y2 + 4 y + 4 x + 2 = 0
(– 4, 0) O
⇒ ( y + 2 )2 + 4 x − 2 = 0 X′ X
 1
⇒ ( y + 2 )2 = − 4  x − 
 2 (0, – 4)
1
Replace y + 2 =Y , x − = X x+y+4=0
2
We have, Y 2 = − 4X Mirror image Y′
This is a parabola with directrix at X = 1

1
x− =1 ⇒ x=
3 Now, the straight line y = − 5 meets the mirror image.
2 2 ∴ − t2 − 4 = − 5
Parabola 447

⇒ t2 = 1
⇒ t=±1 2. Key Idea Use the equation of tangent of slope
Thus, points of intersection of A and B are (− 6, − 5) and a
‘m’ to the parabola y 2 = 4 ax is y = mx + and a line
(−2, − 5). m
ax + by + c = 0 touches the circle
∴ Distance, AB = (−2 + 6)2 + (−5 + 5)2 = 4 |c|
x 2 + y 2 = r 2, if = r.
15. PLAN Parametric coordinates for y 2 = 4 ax are ( at 2,2 at ). a2 + b2

P
Since, equation of given parabola is y2 = 4 2x and
equation of tangent line is ax + y = c or y = − ax + c,
2 2
then c = = [Qm = slope of line = − a]
m −a
Q
[Q line y = mx + c touches the parabola
Description of Situation As the circle intersects the y2 = 4ax iff c = a / m].
parabola at P and Q. Thus, points P and Q should satisfy Then, equation of tangent line becomes
circle. 2
y = − ax − …(i)
P ( 2 t 2, 4 t ) should lie on x2 + y2 − 2x − 4 y = 0 a
⇒ 4 t 4 + 16 t 2 − 4 t 2 − 16 t = 0 Q Line (i) is also tangent to the circle x2 + y2 = 1 .
⇒ 4 t 4 + 12 t 2 − 16 t = 0 2
⇒ 4 t (t3 + 3 t − 4) = 0 −
a 2
⇒ 4t (t − 1) (t 2 + t + 4) = 0 ∴ Radius = 1 = ⇒ 1 + a2 = −
1+ a 2 a
∴ t = 0, 1
2
⇒ P (2, 4) and PQ is the diameter of circle. ⇒ 1 + a2 = [squaring both sides]
1 1 a2
Thus, area of ∆PQS = ⋅ OS × PQ = ⋅ (2) ⋅ (4) = 4 ⇒ a4 + a2 − 2 = 0
2 2
⇒ (a + 2) (a 2 − 1) = 0
2

⇒ a2 = 1 [Q a 2 > 0, ∀ a ∈ R]
Topic 2 Equation of Tangents and
2
Properties ∴ | c| = = 2
|a |
1. Since equation of tangent to the parabola y2 = 4(x + 1)
3. Given equation of parabola is
having slope ‘m’ is
1 y = (x − 2)2 − 1
y = m(x + 1) + ⇒ y = x2 − 4 x + 3 …(i)
m
 1 Now, let (x1 , y1) be the point of intersection of tangents
⇒ y = mx + m +  …(i) of parabola (i) and line x − y = 3, then
 m
Equation of chord of contact of point (x1 , y1 ) w.r.t.
Similarly, equation of tangent to the parabola parabola (i) is
y2 = 8(x + 2) having slope m′ is T =0
2 1
y = m′ (x + 2) + ⇒ ( y + y1 ) = xx1 − 2(x + x1 ) + 3
m′ 2
 2 ⇒ y + y1 = 2x (x1 −2) − 4x1 + 6
⇒ y = m′ x + 2m′ +  …(ii)
 m′  ⇒ 2x(x1 − 2) − y = 4 x1 + y1 − 6, this equation represent
Q It is given that tangents (i) and (ii) are perpendicular the line x − y = 3 only, so on comparing, we get
so mm′ = − 1 2(x1 − 2) − 1 4x1 + y1 − 6
= =
∴Eq. (ii) becomes as 1 −1 3
1 2 5
y = − x − 2m − …(iii) ⇒ x1 = and y1 = − 1
m m 2
Let tangents (i) and (iii) intersects each other at point 5 
So, the required point is  , − 1 .
P (h , k), so 2 
(h + 1)m2 − km + 1 = 0 4. Given parabola y2 = 4x …(i)
and 2m2 + km + (h + 2) = 0
So, equation of tangent to parabola (i) at point (1, 2) is
h + 1 −k 1
∴ = = ⇒ h + 3 =0 2 y = 2(x + 1)
2 k h+2 [Qequation of the tangent to the parabolay2 = 4ax at
On taking locus of point P (h , k), we get a point (x1 , y1 ) is given by yy1 = 2a (x + x1 )]
x+ 3 =0 ⇒ y=x+1 …(ii)
448 Parabola

Now, equation of circle, touch the parabola at point (1, 2) |m2 ⋅ 3 − m ⋅ 0 + 1|


⇒ =3
is (m2)2 + (−m)2
(x − 1)2 + ( y − 2)2 + λ (x − y + 1) = 0
The length of perpendicular from a point (x1 , y1 ) to the
⇒ x2 + y2 + (λ − 2)x + (−4 − λ ) y + (5 + λ ) = 0 …(iii)
Also, Circle (iii) touches the x-axis, so g 2 = c ax1 + by1 + c
line ax + by + c = 0 is .
 λ − 2
2
a 2 + b2
⇒   =5 + λ
 2  3m2 + 1
⇒ λ2 − 4λ + 4 = 4λ + 20 ⇒ =3
m4 + m2
⇒ λ2 − 8λ − 16 = 0
8 ± 64 + 64 ⇒ 9m4 + 6m2 + 1 = 9(m4 + m2)
⇒ λ= 1
2 ⇒ m ≈ ∞ or m = ±
⇒ λ = 4 ± 32 = 4 ± 4 2 3
Now, radius of circle is r = g 2 + f 2 − c  1 
 3+ 2 
⇒ r =| f| [Q g 2 = c] 3m2 + 1 m
Q lim = lim = 3
λ + 4
=  = 8 + 4 2 or 8 − 4 2  m → ∞ m4 + m2 m → ∞ 1
1+ 2

 2  2 2  m 
8 −4 2
For least area r = = 4 − 2 2 units ∴ Equation of common tangents are x = 0,
2 x −x  1
So, area = πr = π (16 + 8 − 16 2 ) = 8π (3 − 2 2 ) sq unit
2 y= + 3 and y = − 3  using y = mx + 
3 3  m
5. Given parabola is x2 = 8 y …(i)
i.e. x = 0, 3 y = x + 3 and 3 y = − x − 3
Now, slope of tangent at any point (x, y) on the parabola 7. Equation of tangent and normal to the curve y2 = 16x
(i) is
at (16, 16) is x − 2 y + 16 = 0 and 2x + y − 48 = 0,
dy x
= = tan θ respectively.
dx 4
[Q tangent is making an angle θ with the positive Y
direction of X-axis]
P
So, x = 4 tan θ (16, 16)
2x
⇒ 8 y = (4 tan θ )2 6=
0 θ +
y–
[on putting x = 4 tan θ in Eq. (i)] y +1 4
x–2 8
=
⇒ y = 2 tan 2 θ 0
X′ X
Now, equation of required tangent is A(–16, 0) C(4, 0) B(24, 0)
y − 2 tan 2 θ = tan θ (x − 4 tan θ )
⇒ y = x tan θ − 2 tan 2 θ ⇒ x = y cot θ + 2 tan θ
6. We know that, equation of tangent to parabola y2 = 4ax
is Y′
a A = (−16, 0) ; B = (24, 0)
y = mx +
m Q C is the centre of circle passing through PAB
∴Equation of tangent to the parabola y2 = 4x is i.e. C = (4, 0)
1 16 − 0 16 4
y = mx + (Qa = 1) Slope of PC = = = = m1
m 16 − 4 12 3
⇒ m2x − my + 1 = 0 …(i) 16 − 0 16
Slope of PB = = = − 2 = m2
Now, let line (i) is also a tangent to the circle. 16 − 24 −8
Equation of circle x2 + y2 − 6x = 0  m − m2 
tan θ =  1 
Clearly, centre of given circle is (3, 0) and radius = 3 1 + m1m2
[Q for the circle x2 + y2 + 2 gx + 2 fy + c = 0, centre =  4 
(− g , − f ) and radius = g 2 + f 2 − c]  3+2 
⇒ tan θ =  
1 −   (2)
∴ The perpendicular distance of (3, 0) from the line (i) 4
is 3.   3 
[Q Radius is perpendicular to the ⇒ tan θ = 2
tangent of circle]
Parabola 449

8. Let the radius of circle with least area be r. a


11. We know, tangent to y2 = 4a x is y = mx + .
Then, then coordinates of centre = (0, 4 − r ). m
1
∴ Tangent to y2 = 4x is y = mx +
m
Since, tangent passes through (1, 4).
1
∴ 4 =m+
m
⇒ m2 – 4m + 1 = 0 (whose roots are m1 and m2)
∴ m1 + m2 = 4 and m1m2 = 1
and m1 – m2 = (m1 + m2)2 – 4m1m2
Since, circle touches the line y = x in first quadrant = 12 = 2 3
0 − (4 − r ) Thus, angle between tangents
∴ = r ⇒ r −4 = ± r 2
2 m2 – m1 2 3 π
4 4 tan θ = = = 3 ⇒ θ=
⇒ r= or 1 + m1m2 1 + 1 3
2+1 1− 2
12. Here, the focal chord of y2 = 16 x is tangent to circle
4  4 
But r ≠ Q 1 − 2 < 0 (x − 6)2 + y2 = 2.
1− 2
4 ⇒ Focus of parabola as (a, 0) i.e. (4, 0)
∴ r= = 4 ( 2 − 1)
2 +1 Now, tangents are drawn from (4, 0) to (x − 6)2 + y2 = 2.

9. Let the tangent to parabola be y = mx + a / m, if it Since, PA is tangent to circle.


touches the other curve, then D = 0, to get the value of m. AC 2
∴ tan θ = slope of tangent = = =1
AP 2
For parabola, y2 = 4x
BC
1
Let y = mx + be tangent line and it touches the or = −1
m BP
parabola x2 = –32 y Y y2=16x
 1
∴ x2 = –32mx +  Tangent
 m as focal chord
32 A
⇒ x2 + 32 mx + =0
m √2
P θ C(6,0)
D =0 X′
θ
X
(4,0)
 32  2
Q (32 m)2 – 4 ⋅   = 0 ⇒ m3 = 1 / 8
 m B

∴ m = 1 /2
10. The tangent at (1, 7) to the parabola x2 = y – 6x is
Y′
1
x (1) = ( y + 7) – 6
2 ∴ Slope of focal chord as tangent to circle = ± 1
[replacing x2 → xx1 and 2 y → y + y1] 2
13. Tangent to the curve y2 = 8x is y = mx + . So, it must
⇒ 2x = y + 7 – 12 m
⇒ y = 2x + 5 ...(i) satisfy xy = − 1
 2
which is also tangent to the circle ⇒ x mx +  = − 1
 m
x2 + y2 + 16 x + 12 y + c = 0
2
i.e. x2 + (2x + 5)2 + 16x + 12(2x + 5) + C = 0 must have ⇒ mx2 + x+ 1 =0
m
equal rools i.e., α = β
Since, it has equal roots.
⇒ 5x2 + 60x + 85 + c = 0
∴ D =0
– 60
⇒ α +β = ⇒
4
− 4m = 0
5 m2
⇒ α = –6 ⇒ m3 = 1
∴ x = –6 and y = 2x + 5 = –7 ⇒ m=1
∴ Point of contact is (– 6, –7). Hence, equation of common tangent is y = x + 2.
450 Parabola

14. Any tangent to y2 = 4x is of the form y = mx +


1
, Now, (m − 4)2 − (16 − m2) = 0
m ⇒ 2m (m − 4) = 0 ⇒ m = 0, 4
(Q a = 1) and this touches the circle (x − 3)2 + y2 = 9.
∴ y = 0 and y = 4 (x − 1) are the required tangents.
1
m (3) + − 0 Hence, (a) and (b) are correct answers.
If m =3
m2 + 1 17. PLAN (i) If P( at 2, 2 at ) is one end point of focal chord of parabola
 a 2a
y 2 = 4ax , then other end point is  2 ,−  .
[Q centre of the circle is (3, 0) and radius is 3]. t t 
(ii) Slope of line joining two points ( x 1, y1 ) and ( x 2, y2 ) is given by
3m2 + 1 y2 − y1
⇒ = ± 3 m2 + 1 .
m x 2 − x1
⇒ 3m2 + 1 = ± 3m m2 + 1 If PQ is focal chord, then coordinates of Q will be
⇒ 9m4 + 1 + 6m2 = 9m2 (m2 + 1) a 2a 
 2 , − .
t t 
⇒ 9m4 + 1 + 6m2 = 9m4 + 9m2
⇒ 3m2 = 1 Now, slope of QR = slope of PK
1 2a
⇒ m =± 2ar +
3 t = 2at ⇒ r + 1 /t = t
ar − 2
2 a at 2 − 2a r2 − 1 / t2 t2 − 2
If the tangent touches the parabola and circle above
t
the X-axis, then slope m should be positive.
1 t 1 t2 − 2 2
∴ m=
1
and the equation is y =
1
x+ 3 ⇒ = 2 ⇒ r− = =t−
1 t −2 t t t
3 3 r−
t
or 3 y = x + 3.
1 t2 − 1
5 ⇒ r =t− =
15. Equation of circle can be rewritten as x2 + y2 = . t t
2
18. PLAN Equation of tangent and normal at ( at 2, 2 at ) are given by
5 ty = x + at 2 and y + tx = 2 at + at 3, respectively.
Centre → (0, 0) and radius →
2 x
Tangent at P : ty = x + at 2 or y = + at
Let common tangent be t
5 x 2a a
y = mx + ⇒ m2x – my + 5 = 0 Normal at S : y + = + 3
m t t t
The perpendicular from centre to the tangent is equal 2a a a (t 2 + 1)2
Solving, 2 y = at + + 3 ⇒ y=
to radius of the circle. t t 2t3
5 /m 5 19. The coordinates of extremities of the latusrectum of
∴ =
1+m 2 2 y2 = 4x are (1, 2 ) and (1, − 2).

⇒ m 1 + m2 = 2 Equations of tangents at these points are


4(x + 1)
⇒ m2(1 + m2) = 2 y⋅2 = ⇒ 2 y = 2(x + 1) …(i)
2
⇒ m +m –2 =0
4 2
4 (x + 1)
and y (− 2) =
⇒ (m + 2)(m2 – 1) = 0
2 2
⇒ m = ±1 [Q m2 + 2 ≠ 0, as m ∈ R ] ⇒ − 2 y = 2(x + 1) …(ii)
∴ y = ± (x + 5 ), both statements are correct as m ± 1 The point of intersection of these tangents can be
satisfies the given equation of Statement II. obtained by solving Eqs. (i) and (ii) simultaneously.
m2 ∴ − 2 (x + 1) = 2 (x + 1)
16. The equation of tangent to y = x2, be y = mx − .
4 ⇒ 0 = 4 (x + 1)
Putting in y = − x + 4x − 4,we should only get one value
2
⇒ −1 = x ⇒ y=0
of x i.e. Discriminant must be zero.
Therefore, the required point is (− 1, 0).
m2
∴ mx − = − x2 + 4 x − 4 20. Given equation can be rewritten as
4
2 ( y − 1)2 = 4 (x − 1), whose parametric coordinates are
m
⇒ x + x (m − 4) + 4 −
2
=0 x − 1 = t 2 and y − 1 = 2t
4
i.e. P (1 + t 2, 1 + 2t )
D =0
Parabola 451

∴ Equation of tangent at P is, ∆1


∴ ∆1 = 2∆ 2 or =2
t ( y − 1) = x − 1 + t , which meets the directrix x = 0 at Q.
2 ∆2
1  1 Y
⇒ y=1 + t − or Q 0, 1 + t − 
t  t
Let R (h , k) which divides QP externally in the ratio T1
1 P
: 1 or Q is mid-point of RP. T2
2
Q
h + t2 + 1
⇒ 0= or t 2 = − (h + 1) …(i) X
2
1 k + 2t + 1 2
and 1+ t− = or t = …(ii) T3
t 2 1−k R
4
From Eqs. (i) and (ii), + (h + 1) = 0
(1 − k)2
or (k − 1)2 (h + 1) + 4 = 0 Topic 3 Equation of Normal and Properties
∴ Locus of a point is (x + 1) ( y − 1)2 + 4 = 0 1. Normal to parabola y2 = 4ax is given by
21. Let the point Q ( x , x 2 ) on x 2 = y whose distance from y = mx − 2am − am3
( 0, c) is minimum. ∴ Normal to parabola
Now, PQ 2 = x 2 + ( x 2 − c)2 y2 = 4b(x − c) is
Let f ( x ) = x 2 + ( x 2 − c)2 ... (i) y = m(x − c) − 2bm − bm3
f ′ ( x ) = 2x + 2 ( x 2 − c) ⋅ 2x [replacing a by b and x by x − c]
 1 = mx − (2b + c)m − bm3 … (i)
= 2x (1 + 2x2 − 2c) = 4x  x2 − c + 
 2 and normal to parabola y2 = 8 ax is
Y y = mx − 4am − 2am3 …(ii)
x2 = y
[replacing a by 2a]
(0,c)P For common normal, we should have
Q(x,x 2) mx − 4am − 2am3 = mx − (2b + c)m − bm3
[using Eqs. (i) and (ii)]
X′ X
O 4am + 2am3 = (2b + c)m + bm3
Y′ ⇒ (2a − b)m3 + (4a − 2b − c)m = 0
⇒ m((2a − b)m2 + (4a − 2b − c)) = 0
 1   1 1 ⇒ m =0
= 4x  x − c −   x + c − , when c >
 2   2 2 2b + c − 4a c
or m2 = = −2
For maxima, put f ′ (x) = 0 2a − b 2a − b
 1 1 c
4 x  x2 − c +  = 0 ⇒ x = 0 , x = ± c − As, m2 > 0, therefore >2
 2 2 2a − b
 1  Note that if m = 0, then all options satisfy
Now, f ′ ′ (x) = 4 x2 − c +  + 4x [2x]
 2 (Q y = 0 is a common normal) and if common normal is
1 other than the axis, then only option (c) satisfies
At x = ± c − ⇒ f ′ ′ (x) ≥ 0. 3
2 [Q for option (c), 2a − b = = 3 > 2]
2 −1
∴ f (x) is minimum.
Hence, minimum value of f (x) =|PQ|
2. If y = mx + c is normal to the parabola y2 = 4ax, then
c = − 2am − am3 .
2
 1  2
1  2
From given condition, y2 = 12x
=  c −  +   c −  − c
 2  2  ⇒ y2 = 4 ⋅ 3 ⋅ x
 
⇒ a =3
2
1  1  1 1 And x+ y=k
= c− +  c − − c = c − , ≤ c≤5
2  2  4 2 ⇒ y = (−1) x + k
22. As, we know area of ∆ formed by three points on ⇒ m = −1
parabola is twice the area of ∆ formed by corresponding and c=k
tangents i.e. area of ∆ PQR = 2 area of ∆ T1T2T3 . ∴ c = k = − 2 (3) (−1) − 3 (−1)3 = 9
452 Parabola
3. Since, equation of normal to the parabola y2 = 4ax is dy
5. Given, (x2 + xy + 4x + 2 y + 4) − y2 = 0
y + xt = 2at + at passes through (3, 0).
3 dx
dy
⇒ 3 t = 2 t + t3 [Q a = 1] ⇒ [(x2 + 4x + 4) + y(x + 2)] − y2 = 0
dx
⇒ t = 0, 1, − 1
dy
∴ Coordinates of the normals areP (1, 2), Q (0, 0), R(1, − 2). ⇒ [(x + 2)2 + y(x + 2)] − y2 = 0
dx
Thus, Put x + 2 = X and y = Y , then
1 dY
A. Area of ∆PQR = ×1 ×4 = 2 (X 2 + XY ) −Y 2 = 0
2 dX
2  ⇒ X 2dY + XYdY − Y 2dX = 0
C. Centroid of ∆PQR =  , 0
3  ⇒ X 2dY + Y (XdY − YdX ) = 0
Equation of circle passing through P , Q , R is dY XdY − YdX
⇒ − =
(x − 1)(x − 1) + ( y − 2)( y + 2) + λ (x − 1) = 0 Y X2
⇒ 1 −4 − λ = 0 Y 
⇒ − d (log|Y |) = d  
⇒ λ = −3  X
∴ Required equation of circle is On integrating both sides, we get
x2 + y2 − 5x = 0 Y
− log|Y| = + C, where x + 2 = X
5  5 X
∴ Centre  , 0 and radius .
2  2 and y = Y
y
4. Tangent to y 2 = 4x at (t 2, 2 t) is ⇒ − log| y| = +C …(i)
x+ 2
Y
S(2, 8) Since, it passes through the point (1, 3).
Q ∴ − log 3 = 1 + C
P (t2, 2t)
⇒ C = − 1 − log 3 = − (log e + log 3) = − log 3e
O
X ∴ Eq. (i) becomes
y
y2=4x log| y| + − log (3e) = 0
x+2
| y| y
y(2 t ) = 2(x + t 2) ⇒ log   + =0 …(ii)
 3e  x + 2
⇒ yt = x + t 2 …(i)
Equation of normal at P (t 2, 2 t) is Now, to check option (a), y = x + 2 intersects the
curve.
y + tx = 2t + t3
|x + 2| x + 2
Since, normal at P passes through centre of circle S(2, 8). ⇒ log  + =0
 3e  x + 2
∴ 8 + 2 t = 2 t + t3
|x + 2|
⇒ t = 2, i.e. P(4, 4) ⇒ log   = −1
 3e 
[since, shortest distance between two curves lie along
|x + 2| 1
their common normal and the common normal will pass ⇒ = e−1 =
through the centre of circle] 3e e
⇒ |x + 2| = 3 or x + 2 = ± 3
∴ SP = (4 − 2)2 + (4 − 8)2 = 2 5
∴ x = 1, − 5 (rejected), as x > 0 [given]
∴Option (a) is correct. ∴ x = 1 only one solution.
Thus, (a) is the correct answer.
Also, SQ = 2 To check option (c), we have
∴ PQ = SP − SQ = 2 5 − 2 | y| y
5+1 y = (x + 2)2 and log   + =0
Thus,
SQ
=
1
=  3e  x + 2
QP 5 −1 4 |x + 2|2  (x + 2 )2
∴Option (b) is wrong. ⇒ log  + =0
 3e  x+2
Now, x-intercept of normal is x = 2 + 22 = 6
|x + 2|2 
∴Option (c) is correct. ⇒ log   = − (x + 2)
1 1  3e 
Slope of tangent = =
t 2 (x + 2)2
⇒ = e−( x + 2) or (x + 2)2 ⋅ ex + 2 = 3e
∴Option (d) is correct. 3e
Parabola 453

3e 2
⇒ ex+ 2 =  3k 
⇒ 3h = 2a + a ⋅  
(x + 2)2  2a 
Y
9k2
e x+ 2 ⇒ 3h = 2a +
4a
e2 ⇒ 9k = 4a (3h − 2a )
2

∴ Locus of centroid is
3e /4
3e /( x + 2)2 4a  2a 
y2 = x − 
O
X 3  3
 2a 
Clearly, they have no solution. ∴ Vertex  , 0 ; directrix
 3 
To check option (d), y = (x + 3)2
2a a
|x + 3|2  (x + 3)2 x− =−
i.e. log  + =0 3 3
 3e  (x + 2) a
⇒ x=
To check the number of solutions. 3
(x + 3)2 4a
Let g (x) = 2 log (x + 3) + − log (3e) and latusrectum =
(x + 2) 3
 a 2a 
2  (x + 2) ⋅ 2 (x + 3) − (x + 3)2 ⋅ 1 ∴ Focus  + , 0 , i.e. (a , 0).
∴ g′ (x) = +   − 0 3 3 
x+ 3  (x + 2)2 
8. End points of latusrectum are (a , ± 2a) i. e. (1, ± 2).
2 (x + 3)(x + 1)
= + Equation of normal at (x1 , y1) is
x+3 (x + 2)2
y − y1 y
Clearly, when x > 0, then, g′ (x) > 0 =− 1
x − x1 2a
∴ g (x) is increasing, when x > 0.
y−2 2 y+2 2
Thus, when x > 0, then g (x) > g (0) i. e. = − and =
x−1 2 x−1 2
 3 9
g (x) > log   + > 0 ⇒ x+ y=3
 e 4
and x− y=3
Hence, there is no solution.
which is tangent to (x − 3)2 + ( y + 2)2 = r 2
Thus, option (d) is true.
6. Normal to y 2 = 4x , is Y

y = mx − 2m − m3 which passes through (9, 6). L(1, 2)

⇒ 6 = 9m − 2m − m3
Normal
⇒ m3 − 7m + 6 = 0
F
⇒ m = 1, 2, − 3 X′
O (1, 0)
X
∴ Equation of normals are, Normal
y − x + 3 = 0, y + 3x − 33 = 0
and y − 2x + 12 = 0 L′(1, –2)
2
7. Equation of tangent and normal at point P (at , 2at ) is Y′
ty = x = at and y = − tx + 2at + at
2 2
∴ Length of perpendicular from centre = Radius
Let centroid of ∆ PTN is R (h , k). |3 − 2 − 3|
⇒ =r
at 2 + (− at 2) + 2a + at 2 12 + 12
∴ h=
3
∴ r2 = 2
Y 2
(at , 2at) 9. We know equation of normal to y2 = 4ax is
P
y = mx − 2am − am3
2
(− at , 0) 2
N (2a + at ,0)
X Thus, equation of normal to y2 = 4x is,
3
T y = – tx + 2at + at
ty = x + at
2 y = mx − 2m − m3 , let it passes through (h , k) .
⇒ k = mh − 2m − m3
or m + m (2 − h ) + k = 0
3
…(i)
and k=
2at Here, m1 + m2 + m3 = 0,
3 m1m2 + m2m3 + m3m1 = 2 − h ,
454 Parabola

m1m2 m3 = − k where m1m2 = α Topic 4 Diameter, Chord of Contact, Chord


k
⇒ m3 = − , it must satisfy Eq. (i) Bisected and Product of Pair of
α
k3 k
Tangents
⇒ − 3 − (2 − h ) + k = 0
α α 1. Key Idea (i) First find the focus of the given parabola
⇒ k2 = α 2h − 2α 2 + α 3 y2 − y1
(ii) Then, find the slope of the focal chord by using m =
⇒ y2 = α 2x − 2α 2 + α 3 x2 − x1
On comparing with y2 = 4x (iii) Now, find the length of the focal chord by using the
⇒ α 2 = 4 and − 2α 2 + α 3 = 0 formula 4a cosec 2α.

⇒ α =2
Equation of given parabola is y2 = 16x, its focus is (4, 0).
10. We know that, normal for y2 = 4ax is given by,
y = mx − 2am − am3 . Since, slope of line passing through (x1 , y1 ) and (x2, y2)
y −y
∴ Equation of normal for y2 = x is is given by m = tan θ = 2 1 .
x2 − x1
m m3  1
y = mx − − Q a = 4  ∴ Slope of focal chord having one end point is (1, 4) is
2 4
4 −0 4
Since, normal passes through (c, 0). m = tan α = =−
1 −4 3
m m3
∴ mc − − =0 [where, ‘α’ is the inclination of focal chord with X-axis.]
2 4
Since, the length of focal chord = 4a cos ec2α
 1 m2
⇒ m  c − −  =0⇒ m =0 ∴ The required length of the focal chord
 2 4 
= 16 [1 + cot2 α ] [Q a = 4 and cosec2α = 1 + cot2 α ]
 1  9  3
or m2 = 4  c −  = 16 1 + = 25 units Q cot α =
1
=− 
 2   tan α
 16   4
⇒ m = 0, the equation of normal is y = 0
2. Given, equation of parabola is x2 = 4 y … (i)
Also, m2 ≥ 0
and the chord is x − 2 y + 4 2 = 0 … (ii)
⇒ c − 1 /2 ≥ 0 ⇒ c ≥ 1 /2
From Eqs. (i) and (ii), we have
At c = 1 /2 ⇒ m =0
[ 2 ( y − 4)]2 = 4 y ⇒ 2( y − 4)2 = 4 y
Now, for other normals to be perpendicular to each
other, we must have m1 ⋅ m2 = − 1 ⇒ ( y − 4)2 = 2 y ⇒ y2 − 8 y + 16 = 2 y
⇒ y − 10 y +16 = 0
2
…(iii)
m2  1 
or +  − c = 0, has m1 m2 = − 1 Y
4 2 
1  A x– √ 2 y+4 √ 2 =0
 − c (x1 , y1 )
2  1 1
⇒ = −1 ⇒ −c=− B (x2 , y2 )
1 /4 2 4 X
3 O
⇒ c=
4
11. Equation of normal to x 2 = 4 y is x = my − 2 m − m3 Let the roots of Eq. (iii) be y1 and y2
and passing through (1, 2). Then, y1 + y2 = 10 and y1 y2 = 16 … (iv)
∴ 1 = 2 m − 2 m − m3 Again from Eqs. (i) and (ii), we have
⇒ m = − 1 or
3
m = −1  x 
Thus, the required equation of normal is , x2 = 4  + 4
 2 
x = − y + 2 + 1 or x + y = 3 is required equation.
⇒ x2 − 2 2x − 16 = 0 … (v)
12. If three different normals are drawn from (h , 0) to Let the roots of Eq. (v) be x1 and x2
y2 = 4x. Then, x1 + x2 = 2 2 and x1x2 = −16 … (vi)
Then, equation of normals are y = mx − 2m − m3 Clearly, length of the chord AB
which passes through (h , 0).
= (x1 − x2)2 + ( y1 − y2)2
⇒ mh − 2m − m3 = 0
⇒ h = 2 + m2 = (x1 + x2)2 − 4x1x2 + ( y1 + y2)2 − 4 y1 y2
where, 2 + m2 ≥ 2 [Q (a − b)2 = (a + b)2 − 4ab]
∴ h > 2 [neglect equality as if 2 + m2 = 2 ⇒ m = 0] = 8 + 64 + 100 − 64
Therefore, three normals are coincident.
= 108 = 6 3 [from Eqs. (iv) and (vi)]
∴ h >2
Parabola 455

3. Equation of chord with mid-point (h , k). −2a/t


mOQ = = −2 t
a /t 2
T = S1 ⇒ yk − 8x − 8h = k − 16h 2

k2 2  1
2x −
yk
= 2h − + 2t 2 t + 
t  t – 2 5
4 4 ∴ tan θ = = =
Q 2x + y = p
2
1 − ⋅2 t 1 − 4 3
t
∴ k = − 4 and p = 2h − 4
1
where h = 3 ⇒ p = 2 ×3 −4 = 2 where t+ = 5
t
 t12 + t22 
4. Here, coordinate M =   , t1 + t2 i.e. mid-point of 7. Let P (α,β) be any point on the locus. Equation of pair of
 2 
chord AB. tangents from P (α , β ) to the parabola y2 = 4ax is
2
(t , 2t2)
Y
2 [ βy − 2a (x + α )]2 = (β 2 − 4aα ) ( y2 − 4a x)
B
2
M [Q T 2 = S ⋅ S1 ]
(t , 2t1)
1
A
r ⇒ β y + 4a (x + α + 2x ⋅ α ) −4a βy (x + α )
2 2 2 2 2

A′ P
X = β 2y2 − 4 β 2ax − 4aαy2 + 16a 2α x
r ⇒ β 2y2 + 4a 2x2 + 4a 2 α 2 + 8x α a 2
M′
= β 2y2 − 4β 2ax − 4aα y2 + 16a 2α x − 4a βxy − 4a βαy…(i)
B′ Now, coefficient of x2 = 4a 2
MP = t1 + t2 = r ...(i) coefficient of xy = − 4a β
2 t2 − 2 t1 2
Also, mAB = 2 2 = [when AB is chord] coefficient of y2 = 4aα
t2 − t1 t2 + t1
Again, angle between the two of Eq. (i) is given as 45°
2
⇒ mAB = [from Eq. (i)] 2 h 2 − ab
r ∴ tan 45° =
2 a+b
Also, mA′ B ′ = − [when A′ B ′ is chord]
r 2 h 2 − ab
⇒ 1=
Hence, (c, d) are the correct options. a+ b
  1  ⇒ a + b = 2 h 2 − ab
5. Since, R  – a , a  t −   lies on y = 2x + a.
  t 
⇒ (a + b)2 = 4 (h 2 − ab)
P (at2, 2at) ⇒ (4a 2 + 4aα )2 = 4 [4a 2 β 2 − (4a 2) (4aα )]
1
R – a, a t – — ⇒ 16a 2 (a + α )2 = 4 ⋅ 4a 2 [ β 2 − 4aα ]
t O
⇒ α 2 + 6aα + a 2 − β 2 = 0
⇒ (α + 3a )2 − β 2 = 8a 2
Thus, the required equation of the locus is
a , – 2a y2 = 4ax (x + 3a )2 − y2 = 8a 2 which is a hyperbola.
t2 t
8. Equation of any tangent to the parabola, y2 = 4ax is
 1 1 a
⇒ a ⋅  t −  = − 2a + a ⇒ t − = − 1 y = mx + .
 t  t m
Thus, length of focal chord a2
This line will touch the circle x2 + y2 =
 1
2
 1
2
 2
= a  t +  = a  t −  + 4 = 5a
 t  t  Y
E
2at − 0 2
6. mOP = 2 =
at − 0 t x = − a/2 B
x=a
Y

P (at2, 2at) X' X


A(−a,0) K O L
q
X
O C

Q a , –2a D
t2 t Y'
456 Parabola

 a
2
a2 2 Applying R3 → R3 − R2 and R2 → R2 − R1 , we get
If   = (m + 1)
 m 2 at12 2at1 1
1

1 1
= (m2 + 1) ∆1 = a (t22 − t12) 2a (t2 − t1 ) 0
2
m2 2 a (t32 − t22) 2a (t3 − t2) 0
⇒ 2 = m4 + m2
1 a (t22 − t12) 2a (t2 − t1 )
⇒ m + m −2 =0
4 2
=
2 a (t32 − t22) 2a (t3 − t 2)
⇒ (m − 1) (m2 + 2) = 0
2

⇒ m2 − 1 = 0, m2 = − 2 1 (t2 − t1 ) (t 2 + t1 ) (t2 − t1 )
= . a . 2a
⇒ m = ± 1 [m2 = −2 is not possible] 2 (t3 − t 2) (t3 + t 2) (t3 − t 2)
Therefore, two common tangents are t2 + t1 1
= a 2 (t2 − t1 ) (t3 − t2)
y = x + a and y = − x − a t3 + t2 1
These two intersect at A (− a , 0). = a 2|(t2 − t1 ) (t3 − t2) (t1 − t3 )|
The chord of contact of A (− a , 0) for the circle Again, let ∆ 2 = area of the ∆PQR
x2 + y2 = a 2 / 2 is (− a ) x + 0 ⋅ y = a 2/2 at1t2 a (t1 + t2) 1
1
⇒ x = − a /2 = at2t3 a (t2 + t3 ) 1
2
and chord of contact of A (− a , 0) for the parabola at3 t1 a (t3 + t1 ) 1
y2 = 4ax is 0 ⋅ y = 2a (x − a ) ⇒ x = a t1t2 (t1 + t2) 1
Again, length of BC = 2 BK 1
= a⋅a t2t3 (t2 + t3 ) 1
2
= 2 OB2 − OK 2 t3 t1 (t3 + t1 ) 1
a2 a2 a2 Applying R3 → R3 − R2, R2 → R2 − R1 , we get
=2 − =2 =a
2 4 4 t1t2 t1 + t2 1
a2
and we know that, DE is the latusrectum of the = t2 (t3 − t1 ) t3 − t1 0
2
parabola, so its length is 4a. t3 (t1 − t2) t1 − t2 0
Thus, area of the quadrilateral BCDE t1t2 t1 + t2 1
1 a2
= (BC + DE ) (KL ) = (t3 − t1 ) (t1 − t2) t2 1 0
2 2
t3 1 0
2
1  3a  15a
= (a + 4a )   = a2 t2 1
2  2 4 = (t3 − t1 ) (t1 − t2)
2 t3 1
9. Let the three points on the parabola be
a2
A ( at12 , 2at1 ), B( at22 , 2at2 ) and C( at32 , 2at3 ). = |(t3 − t1 ) (t1 − t2) (t2 − t3 )|
2
Equation of the tangent to the parabola at (at 2, 2at ) is ∆1 a 2|(t2 − t1 ) (t3 − t2) (t1 − t3 )|
ty = x + at 2 Therefore, = =2
∆ 2 1 a 2|(t − t ) (t − t ) (t − t )|
Therefore, equations of tangents at A and B are 3 1 1 2 2 3
2
t1 y = x + at12 …(i) 10. Let A (t12, 2t1 ) and B(t22, 2t2) be coordinates of the end
and t2y = x + at22 …(ii) points of a chord of the parabola y2 = 4x having slope 2.
From Eqs. (i) and (ii)
Now, slope of AB is
t1 y = t2y − at22 + at12 2t − 2t1 2(t2 − t1 ) 2
⇒ t1 y − t2y = at12 − at22 m = 22 = =
t 2 − t1
2
(t2 − t1 )(t2 + t1 ) t2 + t1
⇒ y = a (t1 + t2) [Q t1 ≠ t2]
Y
and t1 a (t1 + t2) = x + at12 [from Eq. (i)] 2 2t 1)
,
⇒ x = at1t2 A (t 1

Therefore, coordinates of P are (at1t2, a (t1 + t2)). 1


Similarly, the coordinates of Q and R are respectively, P (h, k)
[at2 t3 , a (t2 + t3 )] and [at1t3 , a (t1 + t3 )]. X′ X
O
2
Let ∆1 = Area of the ∆ABC
at 12 2at1 1 B( 2
t2 ,
1 2t
= at22 2at2 1 2 )
2 Y′
at32 2at3 1
Parabola 457

But m =2 [given] P is point of intersection of y = mx and y2 = 4x is


⇒ 2=
2  4 4 1
 2 ,  and Q is point intersection of y = − x and
t2 + t1  m m m
⇒ t1 + t2 = 1 …(i) y2 = 4x is (4m2, − 4m).
Let P (h , k) be a point on AB such that, it divides AB Now, equation of PQ is
internally in the ratio 1 : 2. 4
+ 4m
2t12 + t22 2(2t1 ) + 2t2 m
Then, h= and k = y + 4m = (x − 4 m2)
2+1 2+1 4
− 4m 2
m2
⇒ 3h = 2t12 + t22 …(ii)
and 3k = 4t1 + 2t2 …(iii) Y

On substituting value of t1 from Eq. (i) in Eq. (iii) P


3k = 4 (1 − t2) + 2t2
⇒ 3k = 4 − 2t2
X′ X
3k O L
⇒ t2 = 2 − …(iv)
2 Q
On substituting t1 = 1 − t2 in Eq. (ii), we get
3h = 2 (1 − t2 )2 + t22 y 2 = 4x
Y′
= 2 (1 − 2t2 + t22) + t22
 4 2 m
= 3t22 − 4t2 + 2 = 3  t22 − t2 +  ⇒ y + 4m = (x − 4 m2)
 3 3 1 − m2
 2
2
2 4
= 3  t2 −  + −  ⇒ (1 − m2) y + 4 m − 4m3 = mx − 4m3
 3 3 9 
⇒ mx − (1 − m2) y − 4 m = 0
2
 2 2 This line meets X-axis, where y = 0
= 3  t2 −  +
 3 3 i.e. x = 4 ⇒ OL = 4 which is constant as independent of
2  2
2 m.
⇒ 3h − = 3  t2 − 
3  3 Again, let (h , k) be the mid-point of PQ. Then,
4
 2  3k 2
2
4m2 + 2
⇒ 3  h −  = 3 2 − −  [from Eq. (iv)] h= m
 9  2 3 2
2
 2  4 3k 4
⇒ 3 h −  =3  −  − 4m
 9 3 2  and k= m
2 2
 2 9  8
⇒ h −  = k −   2 1
 9 4  9 ⇒ h = 2  m + 2
 m 
2
 8 4 2 1 
⇒ k −  = h −  and k = 2  − m
 9 9 9 m 
On generalising, we get the required locus  2 
1
 8
2
4 2 ⇒ h = 2 m −  + 2
 y −  = x −   m 
 9 9 9
1 
2 8 and k = 2  − m
This represents a parabola with vertex at  , . m 
9 9
Eliminating m, we get
11. Let the equation of chord OP be y = mx.
2h = k2 + 8
1
Then, equation of chord will be y = − x and or y = 2 (x − 4) is required equation of locus.
2
m
18
Ellipse

Topic 1 Equation of Ellipse and Focal Chord


Objective Questions I (Only one correct option) x2 y 2
7. The ellipse E1 : + = 1 is inscribed in a rectangle R
1. If e1 and e2 are the eccentricities of the ellipse, 9 4
whose sides are parallel to the coordinate axes.
x2 y 2 x 2 y2
+ = 1 and the hyperbola, − = 1 respectively Another ellipse E 2 passing through the point (0, 4)
18 4 9 4 circumscribes the rectangle R. The eccentricity of the
and (e1 , e2) is a point on the ellipse, 15x2 + 3 y2 = k, then k ellipse E 2 is (2012)
is equal to (2020 Main, 9 Jan II) 2 3 1 3
(a) 14 (b) 15 (c) 17 (d) 16 (a) (b) (c) (d)
2 2 2 4
2. An ellipse, with foci at (0, 2) and (0, − 2) and minor axis 8. If P = (x, y), F1 = (3, 0), F2 = (− 3, 0)
of length 4, passes through which of the following and 16x2 + 25 y2 = 400, then PF1 + PF2 equals (1998, 2M)
points? (2019 Main, 12 April II)
(a) 8 (b) 6 (c) 10 (d) 12
(a) ( 2 , 2) (b) (2, 2 )
(c) (2, 2 2 ) (d) (1, 2 2 ) Objective Questions II
3. In an ellipse, with centre at the origin, if the difference (One or more than one correct option)
of the lengths of major axis and minor axis is 10 and one 9. Define the collections { E1 , E 2, E3 , K } of ellipses and
of the foci is at (0, 5 3 ), then the length of its latus { R1 , R2, R3 , K } of rectangles as follows :
rectum is (2019 Main, 8 April I)
x2 y2
(a) 5 (b) 10 (c) 8 (d) 6 E1 : + =1
9 4
4. Let S and S′ be the foci of an ellipse and B be any one of R1 : rectangle of largest area, with sides parallel to the
the extremities of its minor axis. If ∆S′ BS is a right axes, inscribed in E1;
angled triangle with right angle at B and area x2 y2
(∆S′ BS ) = 8 sq units, then the length of a latus rectum E n : ellipse 2 + 2 = 1 of the largest area inscribed in
a n bn
of the ellipse is (2019 Main, 12 Jan II)
Rn − 1, n > 1;
(a) 2 2 (b) 4 2
(c) 2 (d) 4 Rn : rectangle of largest area, with sides parallel to the
axes, inscribed in E n, n > 1.
5. Let the length of the latus rectum of an ellipse with its Then which of the following options is/are correct?
major axis along X-axis and centre at the origin, be 8. If (2019 Adv.)
the distance between the foci of this ellipse is equal to (a) The eccentricities of E18 and E19 are NOT equal.
the length of its minor axis, then which one of the 5
following points lies on it? (2019 Main, 11 Jan II) (b) The distance of a focus from the centre in E9 is .
32
(a) (4 2 , 2 3 ) (b) (4 3 , 2 2 ) N
(c) (4 2 , 2 2 ) (d) (4 3 , 2 3 ) (c) ∑ (area of Rn ) < 24, for each positive integer N.
n =1
6. The equation of the circle passing through the foci of 1
x2 y2 (d) The length of latusrectum of E9 is
the ellipse + = 1 and having centre at (0, 3) is 6
16 9 (2013 Main) 10. Consider two straight lines, each of which is tangent to
(a) x2 + y2 − 6 y − 7 = 0 both the circle x2 + y2 = (1 /2) and the parabola y2 = 4x.
(b) x2 + y2 − 6 y + 7 = 0 Let these lines intersect at the point Q. Consider the
(c) x2 + y2 − 6 y − 5 = 0 ellipse whose centre is at the origin O(0, 0) and whose
(d) x2 + y2 − 6 y + 5 = 0 semi-major axis is OQ. If the length of the minor axis of
Ellipse 459

this ellipse is 2, then which of the following statement(s) Analytical & Descriptive Question
is (are) TRUE? (2018 Adv.)
x2 y2
(a) For the ellipse, the eccentricity is 1 / 2 and the length 14. Let P be a point on the ellipse 2
+ 2 = 1 , 0 < b < a.
of the latus rectum is 1 a b
(b) For the ellipse, the eccentricity is 1/2 and the length of Let the line parallel to Y-axis passing through P meet
the latus rectum is 1/2 the circle x2 + y2 = a 2 at the point Q such that P and Q
are on the same side of X-axis. For two positive real
(c) The area of the region bounded by the ellipse between
1 1 numbers r and s, find the locus of the point R on PQ
the lines x = and x =1is ( π − 2) such that PR : RQ = r : s as P varies over the ellipse.
2 4 2
(2001, 4M)
(d) The area of the region bounded by the ellipse between
the lines x =
1
and x =1is
1
( π − 2)
Passage Type Questions
2 16
Passage
Let F1 (x1 , 0) and F2 (x2, 0), for x1 < 0 and x2 > 0, be the foci of
Fill in the Blanks
x2 y2
11. An ellipse has OB as a semi-minor axis. F and F ′ are its the ellipse + = 1. Suppose a parabola having vertex at
9 8
foci and the angle FBF′ is a right angle. Then, the the origin and focus at F2 intersects the ellipse at point M in
eccentricity of the ellipse is …… . (1997, 2M) the first quadrant and at point N in the fourth quadrant.
1 (2016 Adv.)
12. An ellipse has eccentricity and one focus at the point
2 15. The orthocentre of ∆F1MN is
1 
(a)  −
9 
(b)  , 0
P  , 1 . Its one directrix is the common tangent, nearer 2
, 0
2   10  3 
to the point P, to the circle x2 + y2 = 1 and the hyperbola (c)  , 0
9
(d)  , 6 
2
x2 − y2 = 1. The equation of the ellipse, in the standard  10  3 
form is…… . (1996, 2M)
16. If the tangents to the ellipse at M and N meet at R and
x2 y2 the normal to the parabola at M meets the X-axis at Q,
13. Let P be a variable point on the ellipse 2 + 2 = 1 with then the ratio of area of ∆MQR to area of the
a b
foci F1 and F2. If A is the area of the ∆ PF1F2, then the quadrilateral MF1NF2 is
maximum value of A is… . (1994, 2M) (a) 3 : 4 (b) 4 : 5 (c) 5 : 8 (d) 2 : 3

Topic 2Questions
Objective Equation
I (Onlyof
oneTangent
correct option) and Normal
1. Let a , b and λ be positive real numbers. Suppose P is an 4. The tangent and normal to the ellipse 3x2 + 5 y2 = 32 at
end point of the latus rectum of the parabola y2 = 4λx, the point P(2, 2) meets the X-axis at Q and R,
x2 y2 respectively. Then, the area (in sq units) of the ∆PQR is
and suppose the ellipse 2 + 2 = 1 passes through the
a b (2019 Main, 10 April II)
point P. If the tangents to the parabola and the ellipse 16 14 34 68
(a) (b) (c) (d)
at the point P are perpendicular to each other, then the 3 3 15 15
eccentricity of the ellipse is (2020 Adv.)
x2 y2
(a)
1
(b)
1
(c)
1
(d)
2 5. If the line x − 2 y = 12 is tangent to the ellipse 2
+ 2 =1
2 2 3 5 a b
 −9 
2. Which of the following points lies on the locus of the foot at the point 3,  , then the length of the latusrectum
 2
of perpendicular drawn upon any tangent to the ellipse,
of the ellipse is (2019 Main, 10 April I)
x2 y 2
+ = 1 from any of its foci? (a) 8 3 (b) 9 (c) 5
4 2 (2020 Main, 6 Sep I)
(d) 12 2
(a) (−2, 3 ) (b) (−1, 2 )
(c) (−1, 3 ) (d) (1, 2) 6. If the tangent to the parabola y2 = x at a point
(α , β ), (β > 0) is also a tangent to the ellipse, x2 + 2 y2 = 1,
3. If the normal to the ellipse 3x2 + 4 y2 = 12 at a point P on
then α is equal to (2019 Main, 9 April II)
it is parallel to the line, 2x + y = 4 and the tangent to the
ellipse at P passes through Q(4, 4) then PQ is equal to (a) 2+1 (b) 2 −1 (c) 2 2 + 1 (d) 2 2 − 1
(2019 Main, 12 April I) 7. If the tangents on the ellipse 4x2 + y2 = 8 at the points
5 5 61 221 157 (1, 2) and (a , b) are perpendicular to each other, then a 2
(a) (b) (c) (d)
2 2 2 2 is equal to (2019 Main, 8 April I)
128 64 4 2
(a) (b) (c) (d)
17 17 17 17
460 Ellipse

8. If tangents are drawn to the ellipse x2 + 2 y2 = 2 at all x2


16. Tangent is drawn to ellipse + y2 = 1 at
points on the ellipse other than its four vertices, then 27
the mid-points of the tangents intercepted between the (3 3 cos θ ,sin θ ) (where, θ ∈(0, π / 2).
coordinate axes lie on the curve (2019 Main, 11 Jan I) Then, the value of θ such that the sum of intercepts on
x2 y2 1 1 axes made by this tangent is minimum, is (2003, 1M)
(a) + =1 (b) + =1
4 2 4x2 2 y2 π π π π
(a) (b) (c) (d)
2
x y 2
1 1 3 6 8 4
(c) + =1 (d) + =1
2 4 2x2 4 y2 17. If a > 2b > 0, then positive value of m for which
9. Equation of a common tangent to the parabola y = 4x 2 y = mx − b 1 + m2 is a common tangent to x2 + y2 = b2
and the hyperbola xy = 2 is (2019 Main, 11 Jan I)
and (x − a )2 + y2 = b2 is (2002, 1M)

(a) x + 2 y + 4 = 0 (b) x − 2 y + 4 = 0 2b a 2 − 4b2


(a) (b)
(c) 4x + 2 y + 1 = 0 (d) x + y + 1 = 0 a 2 − 4b2 2b
10. The eccentricity of an ellipse whose centre is at the 2b b
(c) (d)
origin is 1/2. If one of its directrices is x = − 4, then the a − 2b a − 2b
 3
equation of the normal to it at 1,  is
 2 (2017 Main)
18. The number of values of c such that the straight line
x2
(a) 2 y − x = 2 (b) 4x − 2 y = 1 y = 4 x + c touches the curve + y2 = 1 is
(c) 4x + 2 y = 7 (d) x + 2 y = 4 4 (1998, 2M)
(a) 0 (b) 2 (c) 1 (d) ∞
11. The area (in sq units) of the quadrilateral formed by the
tangents at the end points of the latusrectum to the
x2 y2
Objective Question II
ellipse + = 1 is (One or more than one correct option)
9 5 (2015 Main)
27 27 19. Let E1 and E 2 be two ellipses whose centres are at the
(a) (b) 18 (c) (d) 27
4 2 origin. The major axes of E1 and E 2 lie along the X-axis
12. The locus of the foot of perpendicular drawn from the and Y-axis, respectively. Let S be the circle
centre of the ellipse x2 + 3 y2 = 6 on any tangent to it is x2 + ( y − 1)2 = 2. The straight line x + y = 3 touches the
(2014 Main) curves S, E1 and E 2 at P, Q and R, respectively.
(a) (x2 − y2 )2 = 6x2 + 2 y2 2 2
(b) (x2 − y2 )2 = 6x2 − 2 y2 Suppose that PQ = PR = . If e1 and e2 are the
3
(c) (x2 + y2 )2 = 6x2 + 2 y2
eccentricities of E1 and E 2 respectively, then the correct
(d) (x2 + y2 )2 = 6x2 − 2 y2 expression(s) is/are (2015 Adv.)
13. The normal at a point P on the ellipse x2 + 4 y2 = 16 43 7
meets the X-axis at Q. If M is the mid-point of the line (a) e12 + e22 = (b) e1 e2 =
40 2 10
segment PQ, then the locus of M intersects the 5 3
latusrectum of the given ellipse at the points (2009) (c)|e1 – e2| =
2 2
(d) e1 e2 =
8 4
 3 5 2  3 5 19 
(a)  ± ,±  (b)  ± ,± 
 2 7  2 4  Analytical & Descriptive Questions
 4 3
(c)  ± 2 3 , ±
1
 (d)  ± 2 3 , ±  20. Find the equation of the common tangent in 1st
 7  7  quadrant to the circle x2 + y2 = 16 and the ellipse
14. The line passing through the extremity A of the major x2 y2
+ = 1. Also, find the length of the intercept of the
axis and extremity B of the minor axis of the ellipse 25 4
x2 + 9 y2 = 9 meets its auxiliary circle at the point M. tangent between the coordinate axes. (2005, 4M)
Then, the area (insqunits) of the triangle with vertices 21. Prove that, in an ellipse, the perpendicular from a focus
at A, M and the origin O is (2009)
upon any tangent and the line joining the centre of the
31 29
(a) (b) ellipse of the point of contact meet on the corresponding
10 10 directrix. (2002, 5M)
21 27
(c) (d)
10 10 22. Let ABC be an equilateral triangle inscribed in the
circle x2 + y2 = a 2. Suppose perpendiculars from A, B, C
15. Tangents are drawn to the ellipse x + 2 y = 2, then the2 2
x2 y2
locus of the mid-point of the intercept made by the to the major axis of the ellipse 2 + 2 = 1, (a > b) meets
tangents between the coordinate axes is (2004, 1M) a b
1 1 1 1 the ellipse respectively at P , Q , R so that P , Q , R lie on
(a) + =1 (b) +
=1
2x 2
4y 2
4x 2
2 y2 the same side of the major axis as A, B, C respectively.
x2 y2 x2 y2 Prove that, the normals to the ellipse drawn at the
(c) + =1 (d) + =1 points P, Q and R are concurrent. (2000, 7M)
2 4 4 2
Ellipse 461

23. A tangent to the ellipse x2 + 4 y2 = 4 meets the ellipse Integer & Numerical Answer Type Question
x2 + 2 y2 = 6 at P and Q. Prove that the tangents at P and x2 y2
Q of the ellipse x2 + 2 y2 = 6 are at right angles. 25. Suppose that the foci of the ellipse + = 1 are (f1 , 0)
9 5
(1997, 5M)
and ( f2, 0), where f1 > 0 and f2 < 0. Let P1 and P2 be two
24. Let d be the perpendicular distance from the centre of parabolas with a common vertex at (0, 0) with foci at
the ellipse x2 / a 2 + y2 / b2 = 1 to the tangent drawn at a ( f1 , 0) and (2 f2, 0), respectively. Let T1 be a tangent to P1
point P on the ellipse. If F1 and F2 are the two foci of the which passes through (2 f2, 0) and T2 be a tangent to P2
ellipse, then show that which passes through ( f1 , 0). If m1 is the slope of T1 and
 1 
 b2  m2 is the slope of T2, then the value of  2 + m22 is
(PF1 − PF2)2 = 4a 2 1 − 2  m1 
 d  (1995, 5M) (2015 Adv.)

Topic 3 Equation of Chord of Contact, Chord Bisected


at a Given Point and Diameter
Passage Based Questions 2. The orthocentre of the ∆ PAB is
(a)  5,  (b)  , 
8 7 25
Passage  7 5 8 
Tangents are drawn from the point P( 3, 4) to the ellipse
(c)  ,  (d)  , 
11 8 8 7
x2 y2
+ = 1 touching the ellipse at points A and B.  5 5  25 5 
9 4 (2010)
1. The equation of the locus of the point whose distance 3. The coordinates of A and B are
from the point P and the line AB are equal, is  8 2 161  9 8
(a) (3, 0) and (0, 2) (b)  − ,  and  − , 
 5 15   5 5
(a) 9x2 + y2 − 6 xy − 54 x − 62 y +241 = 0
(b) x2 + 9 y2 + 6 xy − 54 x + 62 y −241 = 0  8 2 161  9 8
(c)  − ,  and (0, 2) (d) (3, 0) and  − , 
(c) 9x2 + 9 y2 − 6 xy − 54 x − 62 y −241 = 0  5 15   5 5
(d) x2 + y2 − 2 xy + 27 x + 31 y −120 = 0

Answers
Topic 1 Topic 2
1. (d) 1. (a) 2. (c) 3. (a) 4. (d)
2. (a) 3. (a) 4. (d) 5. (b) 5. (b) 6. (a) 7. (d) 8. (d)
6. (a) 7. (c) 8. (c) 9. (c,d) 9. (a) 10. (b) 11. (d) 12. (c)
10. (a, c) 13. (c) 14. (d) 15. (a) 16. (b)
2
 1 17. (a)
x −  18. (b) 19. (a, b)
 3 (y − 1 ) 2
11. e = 1 / 2 12. + =1 13. b a 2 − b 2 20. y = −
2x 7 14 3
+4 , 25. (4)
1/9 1 / 12 3 3 3
x y (r + s )
2 2 2
Topic 3
14. + =1 15. (a) 16. (c)
a 2 (ar + bs ) 2 1. (a) 2. (c) 3. (d)
Hints & Solutions
Topic 1 Equation of Ellipse and On squaring Eq. (i) both sides, we get
Focal Chord b2e2 = 75
1. It is given that e1 is the eccentricity of the ellipse,  a 2  2 a2
⇒ b21 − 2  = 75 Q e = 1 − 2 
x2 y2  b   b 
+ = 1, so
18 4 ⇒ b2 − a 2 = 75
4 14 7 ⇒ (b + a )(b − a ) = 75
e1 = 1 – = =
18 18 9 ⇒ b + a = 15 [from Eq. (ii)] …(iii)
and e2 is the eccentricity of the hyperbola, On solving Eqs. (ii) and (iii), we get
x 2 y2 4 13
− = 1, so e2 = 1 + = b = 10 and a = 5
9 4 9 9
2a 2 2 × 25
 7 13  So, length of latusrectum is = = 5 units
Now, as (e1 , e2) =  ,  is a point on the ellipse, b 10
 9 9
x2 y2
15x2 + 3 y2 = k, then 4. Let the ellipse be 2
+ 2 = 1.
a b
 7  13
15   + 3   = k
 9 9 Then, according to given information, we have the
following figure.
105 + 39
⇒ =k Y
9
144
⇒ = k ⇒ k = 16 B (0,b)
9
Hence, option (d) is correct.
X
2. Let the equation of ellipse be S′(–ae,o)O S(ae,0)
x2 y2
2
+ 2 =1 …(i)
a b
Since, foci are at (0, 2) and (0, − 2), major axis is along
b b
the Y -axis. Clearly, slope of line SB = and slope of line S′B =
− ae ae
So, be = 2 …(ii)
[where e is the eccentricity of ellipse] Q Lines SB and S′ B are perpendicular, so
and 2a = length of minor axis = 4 [given]  b   b
  .   = −1
⇒ a =2 …(iii)  − ae  ae
a2 [Q product of slopes of two
Q e =1 − 2
2
b perpendicular lines is (−1)]
2
 2 4  2 ⇒ b2 = a 2e2 …(i)
∴   =1 − 2 Q e = b 
 b b Also, it is given that area of ∆S′ BS = 8
8
⇒ = 1 ⇒ b2 = 8 1 2
b2 ∴ a =8
2
x2 y2
Thus, equation of required ellipse is + =1 [QS′ B = SB = a because S′ B + SB = 2a and S′ B = SB]
4 8
x2 y2 ⇒ a 2 = 16 ⇒ a = 4 …(ii)
Now, from the option the ellipse + = 1 passes 2
4 8 b
Q e2 = 1 − = 1 − e2 [from Eq. (i)]
through the point ( 2 , 2). a2
x2 y2 ⇒ 2e2 = 1 ⇒ e2 =
1
3. One of the focus of ellipse + = 1 is on Y-axis …(iii)
a 2 b2 2
(0, 5 3 ) From Eqs. (i) and (iii), we get
∴ be = 5 3 …(i)  1  1
b2 = a 2  = 16  [using Eq. (ii)]
[where e is eccentricity of ellipse]  2  2
According to the question, ⇒ b2 = 8
2b − 2a = 10 2b2 2 ×8
⇒ b − a =5 …(ii) Now, length of latus rectum = = = 4 units
a 4
Ellipse 463

x2 y2 x2 y2
5. Let the equation of ellipse be + =1 7. PLAN Equation of an ellipse is 2
+ =1 [Qa > b]
a 2 b2 a b2
b2
Then, according the problem, we have Eccentricity, e 2 = 1− [Qa > b]
a2
2
2b
= 8 and 2ae = 2b Y
a
2b2
[Length of latusrectum = and
a
A' A
length of minor axis = 2b] X'
(– a, 0) (a, 0)
X
 b b
⇒ b   = 4 and = e
 a a
Y'
⇒ b(e) = 4
1 Description Situation As ellipse circumscribes the
⇒ b = 4.
e rectangle, then it must pass through all four vertices.
…(i) Let the equation of an ellipse E 2 be
Also, we know that b2 = a 2(1 − e2) x2 y2
b2  b  + = 1, where a < b and b = 4.
⇒ = 1 − e2 ⇒ e2 = 1 − e2 Q a = e a 2 b2
a2 Y
⇒ 2e2 = 1 (0, 4)
1
⇒ e=
2
(–3, 2) (0, 2) (3, 2)
…(ii)
From Eqs. (i) and (ii), we get
b =4 2 X' X
(–3, 0) O (3, 0)
E1
b2 32
Now, a2 = = = 64 (–3, –2) (0, –2) (3, –2)
1 − e2 1 − 1
2 E2
x2 y2
∴Equation of ellipse be + =1 (0, – 4)
64 32 Y'
Now, check all the options.
Also, it passes through (3, 2).
Only (4 3 , 2 2 ), satisfy the above equation.
9 4
x2 y2 ⇒ 2
+ 2 =1 [Q b = 4]
6. Given equation of ellipse is + =1 a b
16 9 9 1
Y ⇒ + = 1 or a 2 = 12
a2 4
a2 12 1
Eccentricity of E 2 , e2 = 1 − =1− = [Q a < b]
b2 16 4
r 1
∴ e=
X' X 2
8. Given, 16 x2 + 25 y2 = 400 [given]
Y' 2 2
x y
⇒ + =1
9 25 16
Here, a = 4, b = 3, e = 1 −
16 Now, PF1 + PF2 = Major axis = 2a [where, a = 5]


7 = 2 × 5 = 10
4
9. Given equation of ellipse
 7  x2 y2
∴ Foci = (± ae, 0) =  ± 4 × , 0 = (± 7 ,0) + =1
E1 : …(i)
 4  9 4
Radius of the circle, r = (ae)2 + b2 = 7 + 9 = 16 = 4 Now, let a vertex of rectangle of largest area with sides
parallel to the axes, inscribed in E1 be (3 cos θ , 2 sin θ ).
Now, equation of circle is So, area of rectangle R1 = 2 (3 cos θ ) × 2 (2 sin θ )
(x − 0)2 + ( y − 3)2 = 16 = 12 sin(2 θ )
∴ x2 + y2 − 6 y − 7 = 0
464 Ellipse

Y Option (d),
2b92 2b2
Length of latusrectum E 9 = =
R1 E1 a9 a ( 2 )8
E2
2 ×4 1
R2 = = units
O X 3 × 16 6
Hence, options (c) and (d) are correct.
10. We have,
1
Equation of circle x2 + y2 =
π 2
The area of R1 will be maximum, if θ = and maximum
4 and Equation of parabola y2 = 4x
area is 12 square units and length of sides of rectangle
Y
R1 are 2 a cos θ = 2 a = 3 2 = length of major axis of
ellipse E 2 and 2 b sin θ = 2 b = 2 2 = length of minor y2=4x
axis of ellipse E 2.
x2 y2
So, E 2 : 2
+ 2
= 1 and maximum area of X′ X
 a  b Q O  x2 + y2 = 1 
   

 2  2 2
 a  b
rectangle R2 = 2     and so on.
 2  2 Y′
x2 y2
So, E n = 2
+ 2
= 1, and maximum Let the equation of common tangent of parabola and
 a   b  circle is
  
 ( 2 )n−1 
 ( 2 )n−1 
 a   b  1
area of rectangle Rn = 2     y = mx +
 ( 2 )n−1   ( 2 )n−1  m
1
Now option (a), Since, radius of circle =
2
(bn )2
Since, eccentricity of ellipse E n = en′ = 1 − 1
(a n )2 0+0+
1 m
2
∴ =
 b  2 1 + m2
 
 ( 2 )n−1  b2 4 5
= 1− 2
= 1− = 1− = ⇒ m4 + m2 − 2 = 0 ⇒ m = ± 1
 a  a2 9 3
  ∴Equation of common tangents are
 ( 2 )n−1 
y = x + 1and y = − x − 1
is independent of ‘n’, so eccentricity of E18 and E19 are Intesection point of common tangent at Q (−1, 0)
equal.
x2 y2
Option (b), ∴ Equation of ellipse + =1
Distance between focus and centre of E 9 = e ⋅ a 9 1 1 /2
where, a = 1, b = 1 / 2
2 2
a 3 5 5
= (e) = 4 × = unit b21 1
( 2 )8 2 3 16 Now, eccentricity (e) = 1 − == 1−
Option (c), a22 2
N  
1
Q ∑ (area of Rn ) < (area of R1 ) + (area of R2) + ... ∞ 2 
2b2  2
n=1 and length of latusrectum = = =1
ab ab a 1
< 2ab + 2 + 2 2 + .......
2 2 Y 0, –1 

 1 1  √2
< 2ab1 + + 2 + .....
 2 2 
 1  X′ X
< 12   (–1,0) O 1  (1,0)
 1 − 1 /2 ,0

√2
N x= 1
⇒ ∑ (area of Rn ) < 24, for each positive integer N . 0, –1 
 √2
Y′ √2
n=1
Ellipse 465

∴ Area of shaded region x2 y2


13. Given, + =1
1 1 a 2 b2
=2 ∫ 1 − x2 dx
1/ 2 2 Foci F1 and F2 are ( − ae, 0) and (ae, 0), respectively. Let
1
x 1  P (x, y) be any variable point on the ellipse.
= 2 1 − x2 + sin −1 x
2 2 1/ 2 The area A of the triangle PF1F2 is given by
 π 1 π   π 1 π − 2 Y
= 2 0 +  −  +   = 2  −  =
 4 4 8   8 4 4 2
P (X, Y )
11. Since, angle FBF′ is right angled.
∴ (Slope of FB) ⋅ (Slope of F ′ B) = − 1
X' X
Y F1 O (ae, 0) F2
(–ae, 0)

B (0, b)

Y'
X' X
(– ae,0) F' O F (ae,0) x y 1
1
A= − ae 0 1
2
Y' ae 0 1
1
 0− b  0− b  = (− y) (− ae × 1 − ae × 1)
⇒   ⋅  = −1 2
 ae − 0  − ae − 0
1 x2
b2 =− y (− 2ae) = a ey = ae ⋅ b 1 − 2
⇒ = − 1 ⇒ b2 = a 2e2 2 a
− a 2e2
So, A is maximum when x = 0.
⇒ a 2(1 − e2) = a 2 e2 ⇒ e2 = 1 / 2 ⇒ e = 1 / 2
b2 a 2 − b2
12. There are two common tangents to the circle x2 + y2 = 1 ∴ Maximum of A = abe = ab 1 − 2
= ab
a a2
and the hyperbola x2 − y2 = 1. These are x = 1 and x = − 1.
= b a 2 − b2
But x = 1 is nearer to the point P(1 / 2, 1).
Therefore, directrix of the required ellipse is x = 1. 14. Given,
PR r
=
Now, if Q (x, y) is any point on the ellipse, then its RQ s
distance from the focus is Y
QP = (x − 1 / 2)2 + ( y − 1)2
and its distance from the directrix is| x − 1|. Q(a cos θ, a sin θ)
R (a cos θ,α)
By definition of ellipse,
P (a cos θ,b sin θ)
2
 1 1
QP = e|x − 1|⇒  x −  + ( y − 1)2 = |x − 1| X' X
 2 2 O (0,0)
2
 1 1
⇒  x −  + ( y − 1) = (x − 1)
2 2
 2 4
1 1
⇒ x2 − x + + y2 − 2 y + 1 = (x2 − 2x + 1)
4 4 Y'

⇒ 4x − 4x + 1 + 4 y − 8 y + 4 = x − 2x + 1
2 2 2
α − b sin θ r
⇒ =
⇒ 3x2 − 2x + 4 y2 − 8 y + 4 = 0 a sin θ − α s
 1
2
1 ⇒ α s − b sin θ ⋅ s = ra sin θ − α r
⇒ 3   x −  −  + 4 ( y − 1 )2 = 0
 3 9  ⇒ α s + α r = ra sin θ + b sin θ ⋅ s
 1
2
1 ⇒ α (s + r ) = sin θ (ra + bs)
⇒ 3  x −  + 4 ( y − 1 )2 = sin θ (ra + bs)
 3 3 ⇒ α=
2 r+s
 1
x − 
 3 ( y − 1 )2 Let the coordinates of R be (h, k).
⇒ + =1 h
1 /9 1 / 12 ∴ h = a cos θ ⇒ cos θ = …(i)
a
466 Ellipse

(ar + bs) sin θ Also, equation of tangent at N (3 / 2, − 6 ) is


and k =α =
r+s 3 x y
⋅ − 6 =1 …(ii)
k (r + s) 2 9 8
⇒ sin θ = …(ii)
ar + bs Eqs. (i) and (ii) intersect on X-axis at R(6, 0) . …(iii)
− 6  3
On squaring and adding Eqs. (i) and (ii), we get Also, normal at M (3 / 2, 6 ) is y − 6 = x − 
2  2
h 2 k2 (r + s)2
sin 2 θ + cos 2 θ = + On solving with y = 0, we get Q(7 / 2, 0) …(iv)
a 2 (ar + bs)2
Y
h 2 k2 (r + s)2
⇒ 1= +
a 2 (ar + bs)2 M (3/2, √ 6 )
x2
y (r + s)
2 2
Q (7/2,0)
Hence, locus of R is + = 1.
a 2 (ar + bs)2 (–1, 0) (1, 0) R(6, 0)
X′ X
(–3, 0) F2 O F1 (3/2,0)
x2 y 2
15. Here, + =1 …(i) (3, 0)
9 8
has foci (± ae, 0) N (3/2, – √ 6 )
where, a 2e2 = a 2 − b2
⇒ a 2e2 = 9 − 8
⇒ ae = ± 1 Y′

F1 , F2 = (± 1, 0) 1 7
5 6
i.e. ∴ Area of ∆MQR = 6 −  6=
sq units
Y
2 42
1
and area of quadrilateral MF1NF2 = 2 × {1 − (− 1)} 6
 3 , √6 
 2
2 = 2 6 sq units
y 2=4x
M Area of ∆MQR 5
∴ =
3 Area of quadrilateral MF1NF2 8
F1 F2 2 P
X′ O X
(–3, 0) (–1,0) (1,0) (3, 0)
Topic 2 Equation of Tangent and Normal
 x 2+ y 2=1   3 , – √6 
 1. Equation of given parabola is
 2
9 8 N y2 = 4 λ x … (i)
So the end point of the latus rectum of the parabola (i),
Y′ x2 y2
P (λ, 2λ ) and the given ellipse 2 + 2 = 1, passes
a b
Equation of parabola having vertex O(0, 0) and F2(1, 0) through point P(λ , 2λ ).
(as, x2 > 0)
On differentiating the equation of parabola, w.r.t. ‘x’, we
y2 = 4 x …(ii) get
2
x y2 dy 2λ
On solving + = 1 and y2 = 4x, we get =
9 8 dx y
x = 3 / 2 and y = ± 6
∴Slope of tangent to the parabola at point P is m1 = 1
Equation of altitude through M on NF1 is
Similarly, on differentiating the equation of given
y− 6 5
= x2 y2
x − 3 /2 2 6 ellipse, 2 + 2 = 1, w.r.t x, we get
a b
5
⇒ (y − 6) = (x − 3 / 2) …(iii) 2x 2 y dy dy b2x
2 6 2
+ 2 =0 ⇒ =− 2
a b dx dx a y
and equation of altitude through F1 is y = 0 …(iv)
b2
 9  ∴ Slope of tangent to the ellipse at point P is m2 = −
On solving Eqs. (iii) and (iv), we get  − , 0 as 2a 2
 10 
orthocentre. Q It is given that the tangents are perpendicular to
each other. So,
x2 y2
16. Equation of tangent at M (3 / 2, 6 ) to + = 1 is  b2 
9 8 m1m2 = − 1 ⇒ (1) −  = −1
3 x y  2a 2
⋅ + 6 ⋅ =1 …(i)
2 9 8 b2
⇒ = 2 ⇒ b = 2a
which intersect X-axis at (6, 0). a2
Ellipse 467

x2 y2 On differentiating ellipse (i), w.r.t. x, we get


∴ Eccentricity of ellipse + = 1 will be
a 2 b2 dy dy 3x
6x + 10 y =0 ⇒ =−
a2 1 1 dx dx 5y
e= 1− = 1− = {Q b > a } 3x 3 5y 5
b2 2 2 So, mT = − = − and mN = =
5 y ( 2, 2) 5 3 y ( 2, 2) 3
2. As we know that locus of the foot of perpendicular
x2 y2 Now, equation of tangent and normal to the given
drawn upon any tangent to the ellipse + = 1 from
4 2 ellipse (i) at point P(2, 2) are
any of its foci is the auxilliary circle and its equation is 3
( y − 2) = − (x − 2)
x2 + y2 = 4 …(i) 5
5
So, from the option the auxilliary circle (i) passes and ( y − 2) = (x − 2) respectively.
through the point (−1, 3 ). 3
It is given that point of intersection of tangent and
3. Key Idea Equation of tangent and normal to the ellipse normal are Q and R at X-axis respectively.
x2 y2 xx yy 1  16  4 
2
+ 2 = 1 at point p(x 1, y 1) is T = 0 ⇒ 21 + =1 So, Q , 0 and R , 0
a b a b2 3  5 
a2x b2 y 1
and − = a 2 − b 2 respectively. ∴ Area of ∆PQR = (QR) × height
x1 y1 2
1 68 68
Equation of given ellipse is 3x2 + 4 y2 = 12 = × ×2= sq units
2 15 15
x2 y2
⇒ + =1 … (i) 2 2
4 3  16 4  68 68
[QQR =  −  =   = and height = 2]
Now, let point P(2 cos θ , 3 sin θ ) , so equation of  3 5  15 15
tangent to ellipse (i) at point P is
x cos θ y sin θ Key Idea Write equation of the tangent to the ellipse at any
+ =1 … (ii) 5.
2 3 point and use formula for latusrectum of ellipse.

Since, tangent (ii) passes through point Q(4, 4) Equation of given ellipse is
4 x2 y2
∴2 cos θ + sin θ = 1 … (iii) + 2 =1 …(i)
3 a 2
b
and equation of normal to ellipse (i) at point P is  9
Now, equation of tangent at the point 3, −  on the
4x 3y  2
− =4 −3
2 cos θ 3 sin θ ellipse (i) is
⇒ 2x sin θ − 3cosθy = sin θ cos θ … (iv) 3x 9 y
⇒ − =1 …(ii)
Since, normal (iv) is parallel to line, 2x + y = 4 a 2 2b2
∴ Slope of normal (iv) = slope of line, 2x + y = 4 x2 y2
[Q the equation of the tangent to the ellipse 2
+ 2 =1
2 a b
⇒ tan θ = − 2 ⇒ tan θ = − 3 ⇒ θ = 120º xx1 yy
3 at the point (x1 , y1 ) is
2
+ 21 = 1 ]
a b
 3 1
⇒ (sin θ , cos θ ) =  ,−  Q Tangent (ii) represent the line x − 2 y = 12, so
 2 2
1 2 12
= =
 3 3 9 1
Hence, point P  − 1, 
 2 a 2 2b2
2
 3 ⇒ a 2 = 36 and b2 = 27
Now, PQ = (4 + 1)2 + 4 − 
 2 2b2 2 × 27
Now, Length of latusrectum = = = 9 units
[given cordinates of Q ≡≡ (4, 4)] a 6
25 5 5 6. Since the point (α , β) is on the parabola y2 = x, so
= 25 + =
4 2
α = β2 …(i)
4. Equation of given ellipse is Now, equation of tangent at point (α , β ) to the parabola
3x2 + 5 y2 = 32 …(i) y2 = x, is T = 0
1
Now, the slope of tangent and normal at point P(2, 2) to ⇒ yβ = (x + α )
the ellipse (i) are respectively 2
dy dx [Q equation of the tangent to the parabola y2 = 4ax at a
mT = and mN = − point (x1 , y1 ) is given by yy1 = 2a (x + x1 )]
dx ( 2, 2) dy ( 2, 2)
468 Ellipse

⇒ 2 yβ = x + β 2 [from Eq. (i)] Y


x β
⇒ y= + …(ii) B
2β 2
Since, line (ii) is also a tangent of the ellipse P (√2 cos θ, sin θ)
x2 + 2 y2 = 1
2
2 1 
2
 β 1 X
∴   = (1)   + A
 2  2 β 2
[Q condition of tangency of line y = mx + c to ellipse Now, the equation of tangent at P is
x2 y2 x 2 cos θ y sin θ
2
+ 2 = 1 is c2 = a 2m2 + b2, + =1
a b 2 1
1 1 β [Q equation of tangent at ( x1 , y1 ) is given by T = 0
here m= , a = 1, b = and c = 
2β 2 2 xx yy 
⇒ 21 + 21 = 1
β2 1 1 a b 
⇒ = +
4 4 β2 2 x y
⇒ + =1
⇒ β4 = 1 + 2 β2 2 sec θ cosec θ
⇒ β4 − 2 β2 − 1 = 0 ∴ A ( 2 sec θ, 0) and B ( 0, cosec θ)
2± 4+4 2±2 2 Let mid-point of AB be R( h , k), then
⇒ β2 = = =1 ± 2
2 2 2 sec θ cosec θ
h= and k =
⇒ β2 = 1 + 2 [Qβ 2 > 0] 2 2
Q α =β =1 + 2
2
2h = 2 sec θ and 2k = cosec θ
7. Equation of given ellipse is 1 1
⇒ cosθ = and sinθ =
4x2 + y2 = 8 …(i) 2h 2k
x2 y2 x2 y2 We know that, cos2 θ + sin2 θ = 1
⇒ + =1⇒ + =1
2 8 ( 2 )2 (2 2 )2 1 1
∴ + =1
Now, equation of tangent at point (1, 2) is 2h 2
4k2
2x + y = 4 ...(ii) 1 1
x2 y2 So, locus of (h , k) is + =1
[Q equation of tangent to the ellipse 2 + 2 = 1 at (x1 , y1 ) 2 x2 4 y2
a b
xx yy a
is 21 + 21 = 1] 9. We know that, y = mx + is the equation of tangent to
a b m
and equation of another tangent at point (a , b) is the parabola y2 = 4ax.
4ax + by = 8
1
…(iii) ∴ y = mx + is a tangent to the parabola
Since, lines (ii) and (iii) are perpendicular to each other. m
 2  4a  y 2 = 4x. [Qa = 1]
∴ −  × −  = −1
 1  b 
Let, this tangent is also a tangent to the hyperbola
[if lines a1x + b1 y + c1 = 0 and a 2x + b2y + c2 = 0 xy = 2
 a  a  Now, on substituting y = mx +
1
in xy = 2, we get
are perpendicular, then  − 1   − 2 = − 1]
 b1   b2  m
⇒ b = − 8a  1
…(iv) x mx +  = 2.
Also, the point (a , b) lies on the ellipse (i), so  m
4a 2 + b2 = 8 ⇒ m 2x 2 + x − 2m = 0
⇒ 4a 2 + 64a 2 = 8 [from Eq.(iv)] Note that tangent touch the curve exactly at one point,
8 therefore both roots of above equations are equal.
⇒ 68a = 8 ⇒ a =
2 2
3
68  1
2 ⇒ D = 0 ⇒ 1 – 4( m 2 ) ( − 2m ) ⇒ m3 =  − 
⇒ a2 =  2
17
1
⇒ m=−
8. Given equation of ellipse is x2 + 2 y2 = 2 , which can be 2
x2 y2 ∴ Required equation of tangent is
written as + =1 x
2 1 y=− −2
2
Let P be a point on the ellipse, other than its four
⇒ 2y = − x − 4
vertices. Then, the parametric coordinates of P be
( 2 cos θ , sin θ )
⇒ x + 2y + 4 = 0
Ellipse 469

10. We have, e =
1 a
and = 4 x2 y2
+
12. Equation of ellipse is x2 + 3 y2 = 6 or = 1.
2 e 6 2
∴ a =2 x cos θ y sin θ
Equation of the tangent is + =1
  1 
2
 1 a b
Now, b2 = a 2(1 − e2) = (2)21 −    = 41 −  = 3
  2   4 Let (h , k) be any point on the locus.

⇒ b= 3 h k
∴ cos θ + sin θ = 1 ...(i)
x2 y2 a b
∴ Equation of the ellipse is 2
+ =1 −b
(2) ( 3 )2 Slope of the tangent line is cot θ.
a
x2 y2
⇒ + =1 Slope of perpendicular drawn from centre (0,0) to (h , k)
4 3
is k / h.
 3
Now, the equation of normal at 1,  is Since, both the lines are perpendicular.
 2
 k  b 
a 2x b2y ∴   ×  − cot θ = − 1
− = a 2 − b2  h  a 
x1 y1
cos θ sin θ
4x 3y ⇒ = =α [say]
⇒ − =4 −3 ha kb
1 (3 /2)
⇒ cos θ = αha
⇒ 4x − 2 y = 1
sin θ = αkb
11. Given equation of ellipse is h k
x2 y 2 From Eq. (i), (αha ) + (αkb) = 1
+ =1 a b
9 5 ⇒ h 2α + k2α = 1
∴ a = 9, b = 5 ⇒ a = 3, b = 5
2 2
1
⇒ α=
2
b 5 2 h + k2
2
Now, e= 1+ = 1− =
a2 9 3 Also, sin 2 θ + cos 2 θ = 1
2
Foci = (± a e, 0) = (± 2, 0) and
b
=
5 ⇒ (αkb)2 + (αha )2 = 1
a 3 ⇒ α 2k2b2 + α 2h 2a 2 = 1
Y 2 2
kb h 2a 2
P (0,3) ⇒ + =1
(h 2 + k2)2 (h 2 + k2)2
)
, 5 /3 2k2 6 h2
−2 ⇒ + 2 =1 [Q a 2 = 6, b2 = 2]
M ( L (2,
5 /3) (h + k )
2 2 2
(h + k2)2
X'
(−2,0) (2,0)
X ⇒ 6x2 + 2 y2 = (x2 + y2)2
O Q
(9/2, 0) [replacing k by y and h by x ]
M'(−
2, L' (2, − 5/3) x2 y2
−5 13. Given, + =1
/3) 16 4
Here, a = 4, b = 2
Y' Equation of normal
4x sec θ − 2 y cosec θ = 12
∴Extremities of one of latusrectum are  7 cos θ 
M , sin θ = (h , k) [say]
 5  − 5  2 
2,  and 2, 
 3  3 7 cos θ
∴ h=
 5 2
∴Equation of tangent at 2,  is
 3 2h
⇒ = = cos θ …(i)
x(2) y(5 / 3) 7
+ = 1 or 2x + 3 y = 9
9 5 and k = sin θ …(ii)
9 
Since, Eq. (ii) intersects X and Y -axes at  , 0 On squaring and adding Eqs. (i) and (ii), we get
2 
and (0, 3), respectively. 4h 2
+ k2 = 1 [Q cos 2 θ + sin 2 θ = 1]
∴ Area of quadrilateral = 4 × Area of ∆POQ 49
1 9  4 x2
= 4 ×  × × 3 = 27 sq units Hence, locus is + y2 = 1 ... (iii)
2 2  49
470 Ellipse

Y x 2
(0,2) Equation of tangent is, cos θ + y sin θ = 1
2
P (4 cosθ,2 sinθ)
whose intercept on coordinate axes are
(− 4,0) (4,0)
X′
Q(3cosθ,0)
X A( 2 sec θ , 0) and B (0, cosec θ)
∴ Mid-point of its intercept between axes
(0,−2)  2 1 
 sec θ , cos ec θ = (h , k)
Y′  2 2 
1 1
4 3 ⇒ cos θ = and sin θ =
For given ellipse, e2 = 1 − = 2h 2k
16 4
3 Thus, focus of mid-point M is
∴ e= 1 1
2 (cos 2θ + sin 2 θ ) = 2
+ 2
3 2h 4k
∴ x= ±4× = ± 2 3 [Q x = ± ae] ...(iv) 1 1
2 ⇒ + = 1, is required locus.
2x2 4 y2
On solving Eqs. (iii) and (iv), we get
4 48 1 16. Given, tangent is drawn at (3 3 cos θ ,sin θ ) to
× 12 + y2 = 1 ⇒ y2 = 1 − =
49 49 49 x2 y2
+ = 1.
1 27 1
y=±
7 x cos θ y sin θ
∴ Equation of tangent is + = 1.
 1 3 3 1
∴ Required points  ± 2 3 , ±  .
 7 3 3 1 
Thus, sum of intercepts =  +  = f (θ ) [say]
14. Equation of auxiliary circle is  cos θ sin θ 
x2 + y2 = 9 ... (i) 3 3 sin3 θ − cos3 θ
x y ⇒ f ′ (θ ) = , put f ′ (θ ) = 0
Equation of AM is + =1 ... (ii) sin 2 θ cos 2 θ
3 1 1
⇒ sin3 θ = cos3 θ
Y 33/ 2
M 1 π π
( 125 , 95 ( ⇒ tan θ = , i.e. θ = and at θ= , f ′ ′ (0) > 0
B(0,1) 3 6 6
π
Hence, tangent is minimum at θ = .
X' X 6
N O A(3,0)
17. Given, y = mx − b 1 + m2 touches both the circles, so
distance from centre = radius of both the circles.
|ma − 0 − b 1 + m2 | |− b 1 + m2|
= b and =b
Y' m2 + 1 m2 + 1
 12 9 ⇒ |ma − b 1 + m2 | =|− b 1 + m2 |
On solving Eqs. (i) and (ii), we get M  − , .
 5 5
1 27 ⇒ m2a 2 − 2abm 1 + m2 + b2 (1 + m2) = b2 (1 + m2)
Now, area of ∆ AOM = ⋅ OA × MN = sq units
2 10 ⇒ ma − 2b 1 + m2 = 0
2 2
x y
15. Let the point P ( 2 cos θ, sin θ) on + = 1. ⇒ m2a 2 = 4b2 (1 + m2)
2 1 2b
Y ∴ m=
(mid-point of AB) a − 4b2
2

B(θ, cosec θ)
M 18. For ellipse, condition of tangency is c2 = a 2m2 + b2
P(√2 cos θ, sin θ)
x2
X′ X Given line is y = 4x + c and curve + y2 = 1
A 4
(√2 sec θ, 0) ⇒ c2 = 4 × 42 + 1 = 65
⇒ c = ± 65
Y′ So, there are two different values of c.
Ellipse 471

x2 y2 x2 y2
19. Here, E1: + = 1, (a > b) 20. Let the common tangent to x2 + y2 = 16 and + =1
a 2 b2 25 4
x2 y2 be
E 2 : 2 + 2 = 1, (c < d ) and S : x2 + ( y − 1)2 = 2
c d y = mx + 4 1 + m2 …(i)
as tangent to E1 , E 2 and S is x + y = 3.
and y = mx + 25m + 4
2
…(ii)
Y
2

Since, Eqs. (i) and (ii) are same tangent.


=
)2

R
∴ 4 1 + m2 = 25m2 + 4 ⇒ 16 (1 + m2) = 25 m2 + 4
–1

P
(y

⇒ 9m2 = 12 ⇒ m = ± 2 / 3
+

(0, 1)
x2
S:

Q Since, tangent is in Ist quadrant.


X′
O
X ∴ m < 0 ⇒ m = − 2/ 3
E1
x+y=3 So, the equation of the common tangent is
2x 7
y=− +4
3 3
E2
which meets coordinate axes at A(2 7 , 0) and
Y′  7
Let the point of contact of tangent be (x1 , y1 ) to S. 0, 4 .
 3
∴ x ⋅ x1 + y ⋅ y1 − ( y + y1 ) + 1 = 2
2
or x x1 + y y1 − y = (1 + y1 ), same as x + y = 3.  7
x1 y1 − 1 1 + y1 ∴ AB = (2 7 − 0) + 0 − 4 
2

⇒ = =  3 
1 1 3
i.e. x1 = 1 and y1 = 2 ⇒ P = (1, 2) 11 196 14 3 14 3
= 28 + = = × =
2 2 3 3 3 3 3
Since, PR = PQ = . Thus, by parametric form,
3 21. Any point on the ellipse
x −1 y −2 2 2 x2 y2
= =± + 2 = 1 be P (a cos θ , b sin θ )
−1 / 2 1 / 2 3 a 2
b
 5 4   1 8
⇒  x = , y =  and  x = , y =  The equation of tangent at point P is given by
 3 3   3 3 x cos θ y sin θ
 5 4  1 8 + =1
∴ Q =  ,  and R =  ,  a b
 3 3  3 3
The equation of line perpendicular to tangent is
Now, equation of tangent at Q on ellipse E1 is x sin θ y cos θ
x⋅5 y⋅4 − =λ
+ =1 b a
a 2 ⋅ 3 b2 ⋅ 3
Since, it passes through the focus (ae, 0), then
On comparing with x + y = 3, we get
ae sin θ
b2 4 1 −0 = λ
a 2 = 5 and b2 = 4 ⇒ e12 = 1 − 2 = 1 − = …(i) b
a 5 5
ae sin θ
Also, equation of tangent at R on ellipse E 2 is ⇒ λ=
x⋅1 y⋅8 b
+ 2 =1 x sin θ y cos θ ae sin θ
a ⋅3 b ⋅3
2
∴ Equation is − = …(i)
b a b
On comparing with x + y = 3, we get
a2 1 7 Equation of line joining centre and point of contact
a 2 = 1, b2 = 8 ⇒ e22 = 1 − 2 = 1 − = …(ii) P (a cos θ , b sin θ ) is
b 8 8 b
7 7 y = (tan θ ) x …(ii)
Now, e1 ⋅ e2 =
2 2
⇒ e1 e2 = a
40 2 10
Point of intersection Q of Eqs. (i) and (ii) has x
1 7 43 a
and e12 + e22 = + = coordinate, . Hence, Q lies on the corresponding
5 8 40 e
1 7 27 a
Also, e12 − e22 = − = directrix x = .
5 8 40 e
472 Ellipse

22. Let the coordinates of A ≡ (a cos θ , b sin θ), so that the x2 y2


23. Given, x2 + 4 y2 = 4 or + =1 …(i)
coordinates of 4 1
B = { a cos (θ + 2π / 3), a sin (θ + 2π / 3)} Equation of any tangent to the ellipse on (i) can be
and C = { a cos (θ + 4π / 3), a sin (θ + 4π / 3)} written as
According to the given condition, coordinates of P are x
cos θ + y sin θ = 1 …(ii)
(a cos θ b sin θ ) and that of Q are { a cos (θ + 2π / 3), 2
b sin (θ + 2π / 3)} and that of R are Equation of second ellipse is
a cos (θ + 4π / 3), b sin (θ + 4π / 3) Y
Y
A (a cos θ, b sin θ) A
B
P
Q Q
X'
O
X √3
R P
1
X′ X
C −2 O 2 √6
Y' −√6
[Q it is given that P, Q, R are on the same side −1
of X-axis as A, B and C]
Equation of the normal to the ellipse at P is −√3
ax by Y′
− = a 2 − b2
cos θ sin θ x2 + 2 y 2 = 6
1 x2 y2
or ax sin θ − by cos θ = (a 2 − b2) sin 2θ …(i) ⇒ + =1 …(iii)
2 6 3
Equation of normal to the ellipse at Q is
2π  2π  Suppose the tangents at P and Q meets at A (h , k).
 
ax sin θ +  − by cos θ +  Equation of the chord of contact of the tangents through
 3   3 A (h , k) is
1  4π  hx ky
= (a 2 − b2)sin 2θ +  …(ii) + =1 …(iv)
2  3 6 3
Equation of normal to the ellipse at R is
But Eqs. (iv) and (ii) represent the same straight line, so
a x sin (θ + 4π / 3) − by cos (θ + 4π / 3) comparing Eqs. (iv) and (ii), we get
1
= (a 2 − b2) sin (2θ + 8π / 3) …(iii) h /6 k /3 1
2 = =
cos θ / 2 sin θ 1
But sin (θ + 4π / 3) = sin (2π + θ − 2π / 3)
= sin (θ − 2π / 3) ⇒ h = 3 cos θ and k = 3 sin θ
and cos (θ + 4π / 3) = cos (2π + θ − 2π / 3) Therefore, coordinates of A are (3 cos θ ,3 sin θ ).
= cos (θ − 2π / 3) Now, the joint equation of the tangents at A is given by
and sin (2θ + 8π / 3) = sin (4π + 2θ − 4π / 3 ) T 2 = SS1,
= sin (2θ − 4π / 3)  hx ky 
2
 x2 y2   h 2 k2 
i.e.  + − 1 =  + − 1  + − 1 …(v)
Now, Eq. (iii) can be written as  6 3   6 3  6 3 
ax sin (θ − 2π / 3) − by cos (θ − 2π / 3)
1 h 2 1  h 2 k2 
= (a 2 − b2) sin (2θ − 4π / 3) …(iv) In Eq. (v), coefficient of x2 = −  + − 1
2 36 6  6 3 
For the lines (i), (ii) and (iv) to be concurrent, we must h 2 h 2 k2 1 1 k2
have the determinant − − = + = −
36 36 18 6 6 18
a sin θ − b cos θ k2 1  h 2 k2 
and coefficient of y2 = −  + − 1
 2π   2π  9 3 6 3 
∆1 = a sin θ +  − b cos θ + 
 3   3 k2 h 2 k2 1 h2 1
 2π   2π  = − − + =− +
a sin θ −  − b cos θ −  9 18 9 3 18 3
 3   3
1 2 Again, coefficient of x2 + coefficient of y2
(a − b2) sin 2 θ 1 1 1
2 =− (h 2 + k2) + +
1 2 18 6 3
(a − b2) sin (2 θ + 4π / 3) = 0
2 1 1
1 2 =− (9 cos 2 θ + 9 sin 2 θ ) +
(a − b2) sin (2 θ − 4π / 3) 18 2
2 9 1
=− + =0
Thus, lines (i), (ii) and (iv) are concurrent. 18 2
Ellipse 473

which shows that two lines represent by Eq. (v) are at 2 2


∴ T1 : y = m1x + ⇒ 0 = −4m1 +
right angles to each other. m1 m1
24. Let the coordinates of point P be (a cos θ , b sin θ ). ⇒ m12 = 1 / 2 …(i)
Then, equation of tangent at P is Also, tangent to P2 passes through (f1, 0) i.e. (2, 0).
x y (−4)
cos θ + sin θ = 1 …(i) ⇒ T2 : y = m2x + ⇒ 0 = 2m2 −
4
a b m2 m2
We have, d = length of perpendicular from O to the
⇒ m22 = 2 …(ii)
tangent at P
1
Y ∴ + m22 = 2 + 2 = 4
m12
P (a cosθ, b sin θ)

d Topic 3 Equation of Chord of Contact,


X' X
F1(−ae,0) O F2(ae,0) Chord Bisected at a Given Point
and Diameter
1
1. Equation of AB is y − 0 = − (x − 3)
3
Y'
x + 3y −3 = 0
|0 + 0 − 1|
1 cos 2 θ sin 2 θ ⇒ |x + 3 y − 3|2 = 10 [(x − 3)2 + ( y − 4)2]
d= ⇒ = +
cos θ sin θ
2 2 d a2 b2 On solving, we are getting
+
a2 b2 9x2 + y2 − 6xy − 54x − 62 y + 241 = 0
1 cos 2 θ sin 2 θ 2. Equation of AB is
⇒ 2
= +
d a2 b2 P (3, 4)
 b2 
We have to prove (PF1 − PF2)2 = 4a 2 1 − 2
 d 
 b2  4a 2b2 (− 95 , 85(
Now, RHS = 4a 1 − 2 = 4a 2 −
2
B A
 d  d2 (3, 0)
 cos 2 θ sin 2 θ 
= 4a 2 − 4a 2b2  +  8
 a2 b2  8 1
y − 0 = 5 (x − 3) = (x − 3) ⇒ y = − (x − 3)
= 4a 2 − 4b2 cos 2 θ − 4a 2 sin 2 θ 9 − 24 3
− −3
= 4a 2(1 − sin 2 θ ) − 4b2 cos 2 θ 5
= 4a 2 cos 2 θ − 4b2 cos 2 θ ⇒ x + 3y = 3 ...(i)
= 4 cos θ (a − b2) = 4 cos 2 θ ⋅ a 2e2 Q e = 1 − (b / a )2 
2 2
Equation of the straight line perpendicular to AB
 
through P is
Again, PF1 = e|a cos θ + a / e|= a | e cos θ + 1|
3x − y = 5
= a (e cos θ + 1) [Q − 1 ≤ cos θ ≤ 1 and 0 < e < 1]
Equation of PA is 2x − 3 = 0
Similarly, PF2 = a (1 − e cos θ )
Therefore, LHS = (PF1 − PF2)2 The equation of straight line perpendicular to PA
 − 9 8 8
= [a (e cos θ + 1) − a (1 − e cos θ )]2 through B  ,  is y = .
 5 5 5
= (ae cos θ + a − a + ae cos θ )2
= (2ae cos θ )2 = 4a 2e2 cos 2 θ  11 8
Hence, the orthocentre is  , .
Hence, LHS = RHS 5 5

25. T2 3. Figure is self-explanatory.


Y P
T1 (3, 4)
D

–9 , 8 B
(– 4, 0) 5 5
X′ O F1(2, 0) X
F2 F2(–2, 0)
F
A
2
(3, 0)
y = 8x
y2 = –16x Y′
Tangent to P1 passes through (2 f2, 0) i. e. (−4, 0).
19
Hyperbola
Topic 1 Equation of Hyperbola and Focal Chord
Objective Questions I (Only one correct option)  y2 x2 
7. Let S = (x, y) ∈ R2 : − = 1,
1. A hyperbola having the transverse axis of length 2 has  1+ r 1−r 
the same foci as that of the ellipse 3x2 + 4 y2 = 12, then where r ≠ ± 1. Then, S represents (2019 Main, 10 Jan II)
this hyperbola does not pass through which of the 2
following points? (2020 Main, 3 Sep I) (a) a hyperbola whose eccentricity is , when
0 < r < 1. 1−r
 3 
(a) 
1 
, 0 (b)  − ,1 2
 2   2  (b) a hyperbola whose eccentricity is , when
0 < r < 1. r +1
 3 1 
(c)  1, −
1 
 (d)  ,  2
 2  2 2 (c) an ellipse whose eccentricity is , when r > 1.
r+1
2. Let P be the point of intersection of the common tangents 1
(d) an ellipse whose eccentricity is , when r > 1.
to the parabola y2 = 12x and the hyperbola 8x2 − y2 = 8. If r+1
S and S′ denotes the foci of the hyperbola where S lies on
the positive X-axis then P divides SS′ in a ratio 8. A hyperbola has its centre at the origin, passes through
(2019 Main, 12 April I) the point (4, 2) and has transverse axis of length 4 along
(a) 13 : 11 (b) 14 : 13 the X-axis. Then the eccentricity of the hyperbola is
(2019 Main, 9 Jan II)
(c) 5 : 4 (d) 2 : 1
2 3
3. If 5x + 9 = 0 is the directrix of the hyperbola (a) 2 (b)
3
(c)
2
(d) 3
16x − 9 y = 144, then its corresponding focus is
2 2
π
(2019 Main, 10 April II) 9. Let 0 < θ < . If the eccentricity of the hyperbola
2
(a)  − , 0 (b) (− 5, 0) (c)  , 0
5 5
(d) (5, 0) x2 y2
 3  3  − = 1 is greater than 2, then the length of its
cos θ sin 2 θ
2
4. If a directrix of a hyperbola centred at the origin and latus rectum lies in the interval (2019 Main, 9 Jan I)
passing through the point (4, − 2 3 ) is 5x = 4 5 and its 3 3 
eccentricity is e, then (a)  1, (b) (3,∞) (c)  , 2 (d) (2, 3]
(2019 Main, 10 April I)
 2   2 
(a) 4e4 − 12e2 − 27 = 0 (b) 4e4 − 24e2 + 27 = 0
(c) 4e4 + 8e2 − 35 = 0 (d) 4e4 − 24e2 + 35 = 0 10. The eccentricity of the hyperbola whose length of the
latusrectum is equal to 8 and the length of its conjugate
5. If the vertices of a hyperbola be at (−2, 0) and (2, 0) and axis is equal to half of the distance between its foci, is
one of its foci be at (−3, 0), then which one of the (2017 Main)
following points does not lie on this hyperbola? 4 4 2
(2019 Main, 12 Jan I) (a) (b) (c) (d) 3
3 3 3
(a) (2 6 , 5) (b) (6, 5 2 )
(c) (4, 15 ) (d) (− 6, 2 10 ) 11. Consider a branch of the hyperbola
6. If a hyperbola has length of its conjugate axis equal to 5 x2 − 2 y2 − 2 2x − 4 2 y − 6 = 0
and the distance between its foci is 13, then the with vertex at the point A. Let B be one of the end points
eccentricity of the hyperbola is (2019 Main, 11 Jan II) of its latusrectum. If C is the focus of the hyperbola
13 nearest to the point A, then the area of the ∆ ABC is
(a) (b) 2 (2008, 3M)
12
13 13 (a) 1 − 2 / 3 sq unit (b) 3 / 2 − 1 sq unit
(c) (d)
8 6 (c) 1 + 2 /3 sq unit (d) 3 /2 + 1 sq unit
Hyperbola 475

12. A hyperbola, having the transverse axis of length 2 sin θ , 17. An ellipse intersects the hyperbola 2x2 − 2 y2 = 1
is confocal with the ellipse 3x + 4 y = 12 . Then, its
2 2
orthogonally. The eccentricity of the ellipse is reciprocal
equation is (2007, 3M) to that of the hyperbola. If the axes of the ellipse are
(a) x2cosec2 θ − y2sec2 θ = 1 along the coordinate axes, then (2009)
(b) x2 sec2 θ − y2cosec2 θ = 1 (a) equation of ellipse is x2 + 2 y2 = 2
(c) x2 sin 2θ − y2cos2θ = 1 (b) the foci of ellipse are (±1, 0)
(d) x2cos2 θ − y2sin 2 θ = 1 (c) equation of ellipse is x2 + 2 y2 = 4
x2 y2 (d) the foci of ellipse are (± 2 , 0)
13. If e1 is the eccentricity of the ellipse + = 1 and e2 is
16 25
the eccentricity of the hyperbola passing through the Analytical & Descriptive Question
foci of the ellipse and e1 e2 = 1, then equation of the 18. A variable straight line of slope 4 intersects the
hyperbola is (2006, 3M) hyperbola xy = 1 at two points. Find the locus of the
x2 y2 x2 y2 point which divides the line segment between these two
(a) − =1 (b) − = −1
9 16 16 9 points in the ratio 1 : 2. (1997, 5M)
x2 y2
(c) − =1 (d) None of these
9 25 Match the List
x2 y2 x2 y2
14. For hyperbola − = 1, which of the 19. Let H : − = 1, where a > b > 0, be a hyperbola in the
cos α sin 2 α
2
a 2 b2
following remains constant with change in ‘ α ’ ? (2003, 1M) XY -plane whose conjugate axis LM subtends an angle
(a) Abscissae of vertices (b) Abscissae of foci of 60° at one of its vertices N . Let the area of the ∆ LMN
(c) Eccentricity (d) Directrix be 4 3.
x2 y2
15. The equation − = 1,|r| < 1 represents List-I List-II
1− r 1 + r (1981, 2M)
(a) an ellipse (b) a hyperbola The length of the
P. 1. 8
(c) a circle (d) None of these conjugate axis of H is
4
Objective Questions II Q. The eccentricity of H is 2.
3
(One or more than one correct option) 2
The distance between the
x2 y2 R. 3.
16. Let the eccentricity of the hyperbola 2 − 2 = 1 be foci of H is 3
a b
The length of the latus
reciprocal to that of the ellipse x2 + 4 y2 = 4. If the S. 4. 4
rectum of H is
hyperbola passes through a focus of the ellipse, then
x2 y2 The correct option is (2018 Adv.)
(a) the equation of the hyperbola is − =1 (2011)
3 2 (a) P → 4; Q → 2; R → 1; S → 3
(b) a focus of the hyperbola is (2, 0) (b) P → 4; Q → 3; R → 1; S → 2
5 (c) P → 4; Q → 1; R → 3; S → 2
(c) the eccentricity of the hyperbola is
3 (d) P → 3; Q → 4; R → 2; S → 1
(d) the equation of the hyperbola is x2 − 3 y2 = 3

Topic 2 Equation of Tangent and Normal


Objective Questions I (Only one correct option) 3. If the line y = mx + 7 3 is normal to the hyperbola
1. If a hyperbola passes through the point P(10, 16) and it x2 y2
− = 1, then a value of m is
has vertices at (± 6, 0), then the equation of the normal 24 18 (2019 Main, 9 April I)
to it at P is (2020 Main, 8 Jan II) 3 15 2 5
(a) (b) (c) (d)
(a) 3x + 4 y = 94 (b) x + 2 y = 42 5 2 5 2
(c) 2x + 5 y = 100 (d) x + 3 y = 58 4. If the eccentricity of the standard hyperbola passing
2. The equation of a common tangent to the curves, through the point (4, 6) is 2, then the equation of the
y2 = 16x and xy = − 4, is (2019 Main, 12 April II) tangent to the hyperbola at (4, 6) is (2019 Main, 8 April II)
(a) x − y + 4 = 0 (b) x + y + 4 = 0 (a) 3x − 2 y = 0 (b) x − 2 y + 8 = 0
(c) x − 2 y + 16 = 0 (d) 2x − y + 2 = 0 (c) 2x − y − 2 = 0 (d) 2x − 3 y + 10 = 0
476 Hyperbola
5. The equation of a tangent to the hyperbola x 2 y2
12. If 2x − y + 1 = 0 is a tangent to the hyperbola
− =1
4x − 5 y = 20 parallel to the line x − y = 2 is
2 2
a 2 16
(2019 Main, 10 Jan I) then which of the following CANNOT be sides of a right
(a) x − y − 3 = 0 (b) x − y + 9 = 0 angled triangle? (2017 Adv.)
(c) x − y + 1 = 0 (d) x − y + 7 = 0 (a) a, 4, 1 (b) 2a, 4, 1 (c) a, 4, 2 (d) 2a, 8, 1
6. Tangents are drawn to the hyperbola 4x2 − y2 = 36 at the 13. Consider the hyperbola H: x2 − y2 = 1 and a circle S with
points P and Q. If these tangents intersect at the centre N (x2 , 0). Suppose that H and S touch each other
point T(0, 3), then the area (in sq units) of ∆PTQ is at a point P(x1 , y1 ) with x1 > 1 and y1 > 0. The common
(a) 45 5 (b) 54 3 (2018 Main) tangent to H and S at P intersects the X-axis at point M.
(c) 60 3 (d) 36 5 If (l , m) is the centroid of ∆PMN , then the correct
7. If a hyperbola passes through the point P( 2 , 3 ) and expression(s) is/are (2015 Adv.)
dl 1
has foci at (± 2, 0), then the tangent to this hyperbola at (a) = 1 − 2 for x1 > 1
dx1 3x1
P also passes through the point (2017 Main)
dm x1
(a) (3 2 , 2 3 ) (b) (2 2 , 3 3 ) (b) = for x1 > 1
(c) ( 3 , 2 ) (d) (− 2 , − 3 ) dx1 3( x2 − 1)
1

x2 y2 dl 1
8. Let P(6, 3) be a point on the hyperbola − = 1. If the (c) = 1+ for x1 > 1
a 2 b2 dx1 3x12
normal at the point P intersects the X-axis at (9, 0), then dm 1
(d) = for y1 > 0
the eccentricity of the hyperbola is (2011) dy1 3
5 3 x2 y2
(a) (b) (c) 2 (d) 3
2 2 14. Tangents are drawn to the hyperbola − = 1,
9 4
9. If the line 2x + 6 y = 2 touches the hyperbola parallel to the straight line 2x − y = 1. The points of
x2 − 2 y2 = 4, then the point of contact is (2004, 1M) contacts of the tangents on the hyperbola are (2012)

(a) 
1 
(b)  −
1 
(a) ( − 2, 6) (b) ( − 5, 2 6 ) (c)  ,
1 1  9 9
 (d) (4, − 6) ,  ,− 
 2 6  2 2 2  2 2 2
10. Let P (a sec θ , b tan θ ) and Q (a sec φ , b tan φ ), where (c) (3 3 , − 2 2) (d) (− 3 3 , 2 2 )
π x y 2 2
θ+φ= , be two points on the hyperbola 2 − 2 = 1. Passage Based Problems
2 a b
If (h , k) is the point of the intersection of the normals at x2 y2
The circle x 2 + y 2 − 8x = 0 and hyperbola − =1
P and Q, then k is equal to (1999, 2M) 9 4
a 2 + b2  a 2 + b2  a 2 + b2  a 2 + b2  intersect at the points A and B. (2010)
(a) (b) −   (c) (d) −  
a  a  b  b  15. Equation of the circle with AB as its diameter is
(a) x2 + y2 − 12 x + 24 = 0
Objective Questions II (b) x2 + y2 + 12 x + 24 = 0
(c) x2 + y2 + 24 x − 12 = 0
(One or more than one correct option) (d) x2 + y2 − 24 x − 12 = 0
11. Let a and b be positive real numbers such that a > 1 and 16. Equation of a common tangent with positive slope to the
b < a. Let P be a point in the first quadrant that lies on circle as well as to the hyperbola is
x2 y 2 (a) 2 x − 5 y − 20 = 0
the hyperbola 2 − 2 = 1. Suppose the tangent to the
a b (b) 2x − 5y + 4 = 0
hyperbola at P passes through the point (1, 0), and (c) 3x − 4y + 8 = 0
suppose the normal to the hyperbola at P cuts off equal (d) 4x − 3y + 4 = 0
intercepts on the coordinate axes. Let ∆ denote the area
of the triangle formed by the tangent at P, the normal at Integer & Numerical Answer Type Question
P and the X-axis. If e denotes the eccentricity of the 17. The line 2x + y = 1 is tangent to the hyperbola
hyperbola, then which of the following statements x2 y 2
is/are TRUE? (2020 Adv.) − = 1. If this line passes through the point of
a 2 b2
(a) 1 < e < 2 (b) 2 < e < 2
intersection of the nearest directrix and the X-axis,
(c) ∆ = a 4 (d) ∆ = b4
then the eccentricity of the hyperbola is…… (2010)
Hyperbola 477

Topic 3 Equation of Chord of Contact, Chord Bisected Diameter,


Asymptote and Rectangular Hyperbola
Objective Question I (Only one correct option)
1. If x = 9 is the chord of contact of the hyperbola (a) x1 + x2 + x3 + x4 = 0
x − y = 9, then the equation of the corresponding pair
2 2 (b) y1 + y2 + y3 + y4 = 0
of tangents is (1999, 2M) (c) x1 x2 x3 x4 = c4
(a) 9 x2 − 8 y2 + 18 x − 9 = 0 (b) 9 x2 − 8 y2 − 18 x + 9 = 0 (d) y1 y2 y3 y4 = c4
(c) 9 x2 − 8 y2 − 18 x − 9 = 0 (d) 9 x2 − 8 y2 + 18 x + 9 = 0
Analytical & Descriptive Question
Objective Question II
3. Tangents are drawn from any point on the hyperbola
(Only one or more than one correct option) x2 y2
− = 1 to the circle x2 + y2 = 9. Find the locus of
2. If the circle x + y = a intersects the hyperbola xy = c
2 2 2 2
9 4
in four points P (x1 , y1 ), Q (x2 , y2 ), R (x3 , y3 ), S (x4 , y4 ), mid-point of the chord of contact. (2005, 4M)
then (1998, 2M)

Answers
Topic 1 5. (c) 6. (a) 7. (b) 8. (b)
1. (d) 2. (c) 3. (b) 4. (d) 9. (d) 10. (d) 11. (a,d) 12. (a, c, d)
5. (b) 6. (a) 7. (c) 8. (b) 13. (a, b, d) 14. (a, b) 15. (a) 16. (b)
9. (b) 10. (c) 11. (b) 12. (a) 17. (2)
13. (b) 14. (b) 15. (b) 16. (b, d)
17. (a, b) 18. 16x + y + 10 xy = 2
2 2
19. (b) Topic 3
x 2 y 2 (x 2 + y 2 )2
Topic 2 1. (b) 2. (a,b,c,d) 3. − =
9 4 81
1. (c) 2. (a) 3. (c) 4. (c)

Hints & Solutions


Topic 1 Equation of Hyperbola and  3 1
So, from the given options point  ,  does not pass
Focal Chord  2 2
1. Equation of given ellipse is through the hyperbola (ii).
x2 y2 Hence, option (d) is correct.
3x + 4 y = 12 ⇒
2 2
+ =1 … (i)
4 3 2. Equation of given parabola y2 = 12x … (i)
3 1
∴Eccentricity of ellipse (i) is e1 = 1 − = and hyperbola 8x − y = 8
2 2
… (ii)
4 2
∴Coordinate of foci is (± 1, 0). Now, equation of tangent to parabola y = 12x having
2
3
Now, it is given that length of the transverse axis of slope ‘m’ is y = mx + … (iii)
m
2
hyperbola is 2, so e2 = 1, where e2 is the eccentricity and equation of tangent to hyperbola
2
of the hyperbola. x2 y2
− = 1 having slope ‘m’ is
So, e2 = 2 1 8
∴ Given hyperbola is rectangular hyperbola and it’s y = mx ± 12m2 − 8 …(iv)
equation is Since, tangents (iii) and (iv) represent the same line
2
x2 y2  3
− =1 ∴ m2 − 8 =  
 1
2
 1
2  m
    ⇒ m − 8m2 − 9 = 0
4
 2  2
1 ⇒ (m − 9) (m2 + 1) = 0
2

⇒ x2 − y2 = … (ii) ⇒ m = ± 3.
2
478 Hyperbola

Now, equation of common tangents to the parabola (i) 5. The vertices of hyperbola are given as (± 2, 0) and one of
and hyperbola (ii) are y = 3x + 1 and y = − 3x − 1 its foci is at (− 3, 0).
Q Point ‘P’ is point of intersection of above common ∴ (a , 0) = (2, 0) and (− ae, 0), = (− 3, 0)
tangents, On comparing x-coordinates both sides, we get
∴ P(− 1 / 3, 0) ⇒ a = 2 and − ae = − 3
and focus of hyperbola S(3, 0) and S′ (− 3, 0). 3
PS 3 + 1 / 3 10 5 ⇒ 2e = 3 ⇒ e =
Thus, the required ratio = = = = 2
PS′ 3 − 1 / 3 8 4  2
9 b2 b2 
Also, =1 + ⇒ b2 = 5 Q e = 1 + 2
3. Equation of given hyperbola is 4 4  a 
16x2 − 9 y2 = 144 So, equation of the hyperbola is
x2 y2 x2 y2
⇒ − =1 …(i) − =1 …(i)
9 16 4 5
So, the eccentricity of Eq. (i)
The point (6, 5 2 ) from the given options does not
16 5
e= 1+ = satisfy the above equation of hyperbola.
9 3
x2 y 2
x2 y 2 6. We know that in − = 1, where b2 = a 2 (e2 − 1), the
[Q the eccentricity (e) of the hyperbola 2 − 2 = 1 is a 2 b2
a b length of conjugate axis is 2b and distance between the
1 + (b / a )2 ] foci is 2ae.
and given directrix is 5x + 9 = 0 ⇒ x = − 9 / 5 ∴According the problem, 2b = 5 and 2ae = 13
  5  Now, b2 = a 2 (e2 − 1)
So, corresponding focus is  − 3   , 0 = (− 5, 0) 2
  3   5
⇒   =a e −a
2 2 2
 2
4. Let the equation of hyperbola is
25 (2ae)2
x2 y 2 ⇒ = − a2
− =1 …(i) 4 4
a 2 b2
25 169
Since, equation of given directrix is 5x = 4 5 ⇒ = − a2 [Q 2ae = 13]
4 4
 a a
169 − 25 144
so 5  = 4 5 [Q equation of directrix is x = ]
 e e ⇒ a2 = = = 36
4 4
a 4
⇒ = …(ii) ⇒ a =6
e 5
and hyperbola (i) passes through point (4, − 2 3 ) Now, 2ae = 13
16 12 ⇒ 2 × 6 × e = 13
so, − =1 …(iii) 13
a 2 b2 ⇒ e=
2 2 12
b b
The eccentricity e = 1 + ⇒ e2 = 1 + 2  y2 x2 
a2 a 7. Given, S = (x, y) ∈ R2: − = 1
 1+ r 1−r 
⇒ a 2e2 − a 2 = b2 …(iv)
 y2 x2 
From Eqs. (ii) and (iv), we get = (x, y) ∈ R2 : + = 1
16 4 16 2  1 + r r − 1 
e − e = b2 …(v)
5 5 y2 x2
From Eqs. (ii) and (iii), we get For r > 1, + = 1, represents a vertical ellipse.
1 + r r −1
16 12 5 12
− =1 ⇒ 2 − 2 =1 [Q for r > 1, r − 1 < r + 1 and r − 1 > 0]
16 2 b2 e b
e r −1
5 Now, eccentricity (e) = 1 −
12 5 12 5 − e2 12e2 r+1
⇒ = 2 −1 ⇒ 2 = ⇒ b2 = …(vi)
b 2
e b e2
5 − e2  x2 y2 a2 
Q For 2 + 2 = 1, a < b, e = 1 − 2 
From Eqs. (v) and (vi), we get  a b b 
 12e2  (r + 1) − (r − 1)
16e4 − 16e2 = 5  ⇒ 16(e2 − 1)(5 − e2 ) = 60 =
 5 − e2 r+1
⇒ 4(5e2 − e4 − 5 + e2 ) = 15 2
=
⇒ 4e − 24e + 35 = 0
4 2 r+1
Hyperbola 479

8. Equation of hyperbola is given by 11. Given equation can be rewritten as focal chord
x2 y 2 (x − 2 )2 ( y + 2 )2
− =1 − =1
a 2 b2 4 2
Q Length of transverse axis = 2a = 4 2 3
∴ a=2 For point A (x, y), e = 1 + =
4 2
x2 y2
Thus, − = 1 is the equation of hyperbola ⇒ x− 2 =2 ⇒ x=2 + 2
4 b2
Q It passes through (4, 2). Y
16 4 4 4 2 Conjugate
∴ − 2 = 1 ⇒ 4 − 2 = 1 ⇒ b2 = ⇒ b = axis
4 b b 3 3 B transverse
axis
Now, eccentricity,
X′ X
4 A C
b2 1 2
e= 1+ 2 = 1+ 3 = 1+ = Y′
a 4 3 3
x2 y2 For point C (x, y), x − 2 = ae = 6 ⇒ x = 6 + 2
9. For the hyperbola − = 1, Now, AC = 6 + 2 − 2 − 2 = 6 − 2
a 2 b2
b2 b2 2
e= 1+ 2 and BC =
= =1
a a 2
1 3
∴ For the given hyperbola, ∴ Area of ∆ ABC = × ( 6 − 2) × 1 = − 1 sq unit
2 2
sin 2 θ
e= 1+ >2
cos 2 θ 12. The given ellipse is
(Q a 2 = cos 2 θ and b2 = sin 2 θ) x2 y2
+ = 1 ⇒ a = 2, b = 3
⇒ 1 + tan 2 θ > 4 4 3
⇒ tan 2 θ > 3 1
∴ 3 = 4 (1 − e2 ) ⇒ e =
⇒tan θ ∈ (− ∞ , − 3 ) ∪ ( 3 , ∞ ) 2
1
[x2 > 3 ⇒|x| > 3 ⇒ x ∈ (−∞ , − 3 ) ∪ ( 3 , ∞ )] So, ae=2 × =1
2
 π
But θ ∈ 0,  ⇒ tan θ ∈ ( 3 , ∞ ) Hence, the eccentricity e1 of the hyperbola is given by
 2
e1 = cosec θ [Q ae = e sin θ ]
 π π
⇒ θ ∈ , 
 3 2 ⇒ b = sin θ (cosec θ − 1) = cos 2 θ
2 2 2

Now, length of latusrectum Hence, equation of hyperbola is


2b2 sin 2 θ x2 y2
= =2 = 2 sin θ tan θ − = 1 or x2 cosec2 θ − y2 sec2 θ = 1.
a cos θ sin θ cos 2 θ
2

Since, both sin θ and tan θ are increasing functions in x2 y2


 π π 13. The eccentricity of + = 1 is
 , . 16 25
 3 2
16 3
∴ Least value of latusrectum is e1 = 1 − =
25 5
π π 3  π 5
= 2 sin ⋅ tan = 2 ⋅ ⋅ 3 =3 at θ =  ∴ e2 = [Q e1 e2 = 1]
3 3 2  3 3
and greatest value of latusrectum is < ∞ ⇒ Foci of ellipse (0, ± 3)
x2 y 2
Hence, latusrectum length ∈ (3, ∞ ) ⇒ Equation of hyperbola is − = − 1.
16 9
2b2
10. We have, = 8 and 2b = ae x2 y2
a 14. Given equation of hyperbola is − =1
cos α sin 2 α
2
⇒ b2 = 4a and 2b = ae
Here, a 2 = cos 2 α and b2 = sin 2 α
Consider, 2b = ae
x2 y 2
⇒ 4b2 = a 2e2 [i.e. comparing with standard equation − = 1]
a 2 b2
⇒ 4a 2 (e2 − 1) = a 2e2
We know that, foci = (± ae, 0)
⇒ 4e2 − 4 = e2 [Q a ≠ 0]
where, ae = a 2 + b2 = cos 2 α + sin 2 α = 1
⇒ 3 e2 = 4
2 ⇒ Foci = (± 1, 0)
⇒ e= [Q e > 0]
3 where, vertices are (± cos α , 0).
480 Hyperbola

1 dy sec θ
Eccentricity, ae = 1 or e = = = cosec θ
cos α dx at A tan θ
Hence, foci remains constant with change in α. and on differentiating Eq. (ii), we get
x2 y2 dy dy x 1
15. Given equation is − = 1 , where| r | < 1 2x + 4 y =0 ⇒ =− = − cosec θ
1−r 1+ r dx dx at A 2y 2
⇒ 1 − r is (+ ve) and 1 + r is (+ ve) Since, ellipse and hyperbola are orthogonal.
x2 y 2 π
∴Given equation is of the form 2 − 2 = 1. 1
∴ − cosec2 θ = − 1 ⇒ cosec2 θ = 2 ⇒ θ = ±
a b 2 4
Hence, it represents a hyperbola when| r | < 1.  1  1
∴ A 1 ,  or 1 , − 
x2 y2  2  2
16. Here, equation of ellipse is + =1
4 1 2
 1
b2 1 3 Form Eq. (ii), 1 + 2   = 2b2
⇒ e2 = 1 − 2 = 1 − =  2
a 4 4
⇒ b2 = 1
3
∴ e= and focus (± a e, 0) ⇒ (± 3 , 0) Equation of ellipse is x + 2 y2 = 2.
2
2
 1 
x2 y2 b2 Coordinates of foci (± ae , 0) =  ± 2 ⋅ , 0 = (± 1 , 0)
For hyperbola − 2 = 1, e12 = 1 + 2  2 
2
a b a
If major axis is along Y-axis, then
1 4 b2 4
where, e12 = 2= ⇒ 1+ 2 = 1 a2
e 3 a 3 = 1 − 2 ⇒ b2 = 2a 2
2 b
b2 1
∴ = …(i) ∴ 2 x2 + y2 = 2 a 2
a2 3
2x
and hyperbola passes through (± 3, 0) ⇒ Y′ = −
y
3
⇒ = 1 ⇒ a2 = 3 …(ii) −2
a2 ⇒ y′ 
=

1
sec θ ,
1
tan θ  sin θ
 
From Eqs. (i) and (ii), b = 1
2
…(iii) 2 2

x 2
y 2 As ellipse and hyperbola are orthogonal
∴ Equation of hyperbola is − =1 2
3 1 ∴ − ⋅ cosec θ = − 1
sin θ
 2  π
Focus is (± a e1 , 0) ⇒  ± 3 ⋅ , 0 ⇒ (± 2, 0) ⇒ cosec2 θ =
1
⇒ θ=±
 3  2 4
Hence, (b) and (d) are correct answers. ∴ 2 x2 + y2 = 2 a 2
1 5
17. Given, 2 x 2 − 2 y2 = 1 ⇒ 2 + = 2a 2 ⇒ a 2 =
2 4
x2 y2 5
⇒ − =1 ... (i) ∴ 2x + y = , corresponding foci are (0, ± 1).
2 2
 1  1 2
   
 2  2 Hence, option (a) and (b) are correct.
Eccentricity of hyperbola = 2 18. Let y = 4x + c meets xy = 1 at two points A and B.
So, eccentricity of ellipse = 1 / 2 Y
y = 4x + c
Let equation of ellipse be
2:1 A
x2 y2 P
+ =1 [where a > b] xy = 1
a 2 b2 X′ X
B
1 b2 b2 1
∴ = 1− 2 ⇒ 2 =
2 a a 2
⇒ a 2 = 2b2 ⇒ x2 + 2 y2 = 2b2 ... (ii) Y′
Let ellipse and hyperbola intersect at i.e. A (t1 , 1 t1 ), B (t2 , 1 t2 )
 1 1  ∴ Coordinates of P are
A sec θ , tan θ
 2 2 
 1 1
 2⋅ + 1⋅ 
On differentiating Eq. (i), we get 2
 1 t + t t t 2
dy dy x
2
, 1
= (h , k) [say]
4x − 4 y =0 ⇒ =  2+1 2+1 
 
dx dx y  
Hyperbola 481

2t1 + t2 2t + t
∴ h= and k = 2 1 ...(i) Topic 2 Equation of Tangent and Normal
3 3t1 t2
1. Since, the vertices of hyperbola on X-axis at (± 6, 0), so
 1  1
Also,  t1 ,  and  t2 ,  lie on y = 4 x + c. x2 y2
 t1   t2 equation of hyperbola we can assume as − = 1 and
a 2 b2
1 1 |a| = 6

t2 t1 1 and hyperbola passes through point P(10, 16) so
⇒ = =4 or t1 t 2 = − 1 4 ... (ii)
t2 − t1 t1 t2 (10)2 (16)2
− =1
k 62 b2
From Eq. (i), t1 = 2h +
4 (16)2 100 − 36 64
⇒ = =
and t1 = h −
k
...(iii) b2 36 36
2 36 × 256
⇒ b2 = = 144
 k  k 1 64
From Eqs. (ii) and (iii), − h −  2h +  = −
 2   4  4 x2 y2
So, equation of hyperbola is − = 0, and the
 2h + k  8h + k 1 36 144
⇒ −    =−
 2   4  4 equation of normal to hyperbola at point P is
36x 144 y
⇒ ( 2h + k) ( 8h + k) = 2 + = 36 + 144
10 16
⇒ 16h 2 + k2 + 10hk = 2 18
Hence, required locus is 16x2 + y2 + 10xy = 2. ⇒ x + 9 y = 180
5
19. We have, ⇒ 2x + 5 y = 100
Equation of hyperbola Hence, option (c) is correct.
x2 y 2 2.
− =1 Key Idea An equation of tangent having slope
a 2 b2 ‘m’ to parabola y 2 = 4 ax is y = mx +
a
.
Y m
L (0,–b) x2 y2
– 2 =1 Given equation of curves are
a2 b
b y2 = 16x (parabola) …(i)
X′ 60°
O N(a,0)
X and xy = − 4 (rectangular hyperbola) …(ii)
b Clearly, equation of tangent having slope ‘m’ to parabola
M (0,–b) 4
(i) is y = mx + …(iii)
Y′ m
Now, eliminating y from Eqs. (ii) and (iii), we get
It is given,
 4 4
∠ LNM = 60° x mx +  = − 4 ⇒ mx2 + x + 4 = 0,
 m  m
and Area of ∆LMN = 4 3
which will give the points of intersection of tangent and
Now, ∆LNM is an equilateral triangle whose sides is 2b rectangular hyperbola.
[Q ∆LON ~ = ∆MOL ; ∴ ∠NLO = ∠NMO = 60°] 4
Since, line y = mx + is also a tangent to the
3 m
∴ Area of ∆LMN = (2b)2 rectangular hyperbola.
4
4
⇒ 4 3 = 3b2 ⇒ b = 2 ∴ Discriminant of quadratic equation mx2 + x + 4 = 0,
m
1 should be zero.
Also, area of ∆LMN = a (2b) = ab
2 [Q there will be only one point of intersection]
⇒ 4 3 = a(2) ⇒ a = 2 3  4
2

(P) Length of conjugate axis = 2b = 2(2)= 4 ⇒ D =   − 4 (m) (4) = 0


 m
b2 4 4 2 ⇒ m3 = 1 ⇒ m = 1
(Q) Eccentricity (e) = 1 + = 1+ = =
a2 12 2 3 3 So, equation of required tangent is y = x + 4.
2
(R) Distance between the foci = 2ae = 2 × 2 3 × =8 x2 y2
3 3. Given equation of hyperbola, is − =1 …(i)
24 18
2b2 2(4) 4 Since, the equation of the normals of slope m to the
(S) The length of latusrectum = = =
a 2 3 3 x2 y 2 m(a 2 + b2 )
hyperbola 2 − 2 = 1, are given by y = mx m
P → 4; Q → 3; R → 1; S → 2 a b a 2 − b2m2
482 Hyperbola

∴ Equation of normals of slope m, to the hyperbola (i), ∴ Required equation of tangent is


are ⇒ y= x± 5 −4
m(24 + 18) ⇒ y = x ± 1 ⇒ x − y ± 1 =0
y = mx ± …(ii)
24 − m2 (18) 6. Tangents are drawn to the hyperbola 4x2 − y2 = 36 at
Q Line y = mx + 7 3 is normal to hyperbola (i) the point P and Q.
∴On comparing with Eq. (ii), we get Tangent intersects at point T (0, 3)
m(42)
± =7 3 Y
24 − 18m2 T (0, 3)
6m
⇒ ± = 3
24 − 18m2
36m2 X
⇒ =3 [squaring both sides] O
24 − 18m2
⇒ 12m2 = 24 − 18m2
⇒ 30m2 = 24 (–3√5, –12)Q P(3√5, –12)
S(0, –12)
2
⇒ 5m2 = 4 ⇒ m = ±
5 Clearly, P Q is chord of contact.
∴Equation of PQ is −3 y = 36
4. Let the equation of standard hyperbola is
x2 y 2 ⇒ y = − 12
− =1 …(i) Solving the curve 4x2 − y2 = 36 and y = − 12 ,
a 2 b2
Now, eccentricity of hyperbola is we get x= ±3 5
1 1
b2 Area of ∆PQT = × PQ × ST = (6 5 × 15) = 45 5
1+ =2 (given) 2 2
a2
x2 y 2
⇒ a 2 + b2 = 4a 2 7. Let the equation of hyperbola be − = 1.
a 2 b2
⇒ b2 = 3a 2 …(ii)
∴ ae = 2 ⇒ a 2e2 = 4
Since, hyperbola (i) passes through the point (4, 6)
⇒ a + b2 = 4 ⇒ b2 = 4 − a 2
2
16 36
∴ − =1 …(iii) x2 y2
a 2 b2 ∴ − =1
a2 4 − a2
On solving Eqs. (ii) and (iii), we get
a 2 = 4 and b2 = 12 …(iv) Since, ( 2 , 3 ) lie on hyperbola.
2 3
Now, equation of tangent to hyperbola (i) at point (4, 6), ∴ − =1
a2 4 − a2
is
4x 6 y ⇒ 8 − 2a 2 − 3a 2 = a 2 (4 − a 2 )
− =1
a 2 b2 ⇒ 8 − 5a 2 = 4a 2 − a 4
4x 6 y ⇒ a − 9a 2 + 8 = 0
4
⇒ − =1 [from Eq. (iv)]
4 12 ⇒ (a − 8)(a 4 − 1) = 0 ⇒ a 4 = 8, a 4 = 1
4

y ∴ a =1
⇒ x − = 1 ⇒ 2x − y − 2 = 0
2 x2 y2
Now, equation of hyperbola is − = 1.
1 3
5. Given equation of hyperbola is
∴ Equation of tangent at ( 2 , 3 ) is given by
4x2 − 5 y2 = 20
3y y
which can be rewritten as 2x − = 1 ⇒ 2x − =1
3 3
x2 y2
⇒ − =1 which passes through the point (2 2 , 3 3 ).
5 4
The line x − y = 2 has slope, m = 1 8. Equation of normal to hyperbola at (x1 , y1 ) is
∴ Slope of tangent parallel to this line = 1 a 2x b2 y
2 2 + = (a 2 + b2 )
x y x1 y1
We know equation of tangent to hyperbola 2
− 2 =1
a b a 2x b2 y
having slope m is given by ∴ At (6, 3) = + = (a 2 + b2 )
6 3
y = mx ± a 2m2 − b2
a2⋅ 9
Q It passes through (9, 0). ⇒ = a 2 + b2
Here, a = 5, b = 4 and m = 1
2 2
6
Hyperbola 483

3a 2 a2 and equation of normal to the hyperbola at point P


⇒ − a 2 = b2 ⇒ 2 = 2 b sec θ
2 b having slope is , as normal cuts off equal
b 2
1 3 a tan θ
∴ e2 = 1 + 2 = 1 + ⇒ e= intercepts on the coordinate axes, so slope must be − 1.
a 2 2
b sec θ
Therefore, = −1
9. The equation of tangent at (x1 , y1 ) is x x1 − 2 y y1 = 4, a tan θ
which is same as 2x + 6 y = 2. ⇒ b = − tan θ {from Eq. (ii)}
x1 2y 4 Q The eccentricity of hyperbola
∴ =− 1 =
2 6 2 b2
e= 1+ = 1 + sin 2 θ
⇒ x1 = 4 and y1 = − 6 a2
Thus, the point of contact is (4, − 6 ).  π
∴ 1 < e < 2 , as θ ∈ 0, 
x2 y 2  2
10. Firstly, we obtain the slope of normal to − = 1 at
a 2 b2 And the area of required triangle is, which is isosceles is
(a sec θ , b tan θ ). On differentiating w.r.t. x, we get 1 1
∆ = ( AP )2 = [(1 − sec2 θ )2 + tan 4 θ ]
2x 2 y dy dy b2 x 2 2
2
− 2 × =0 ⇒ =
a b dx dx a 2 y Y

Slope for normal at the point (a sec θ , b tan θ ) will be


a 2b tan θ a P (a secθ, b tanθ)
− = − sin θ
b2a sec θ b A 135º
X
∴ Equation of normal at (a sec θ , b tan θ ) is O (1, 0) B
a
y − b tan θ = − sin θ (x − a sec θ )
b
⇒ (a sin θ ) x + by = (a 2 + b2 ) tan θ 1
⇒ a x + b cosecθ = (a 2 + b2 ) sec θ …(i) = (2 tan 4 θ ) = tan 4 θ = b4 .
2
x2 y2 12. Tangent ≡ 2x − y + 1 = 0
Similarly, equation of normal to − = 1 at
a 2 b2
x2 y2
(a sec φ, b tan φ ) is ax + b y cosec φ = (a 2 + b2 ) sec φ …(ii) Hyperbola ≡ − =1
a 2 16
On subtracting Eq. (ii) from Eq. (i), we get
It point ≡ (a sec θ , 4 tan θ ),
b (cosec θ − cosec φ ) y = (a 2 + b2 ) (sec θ − sec φ ) x sec θ y tan θ
tangent ≡ − =1
a 2 + b2 sec θ − sec φ a 4
⇒ y= ⋅
b cosec θ − cosec φ On comparing, we get secθ = − 2a
sec θ − sec φ sec θ − sec (π / 2 − θ ) tan θ = − 4 ⇒ 4a 2 − 16 = 1
But =
cosec θ − cosec φ cosec θ − cosec (π / 2 − θ ) 17
∴ a=
[Q φ + θ = π / 2] 2
sec θ − cosec θ Substitute the value of a in option (a), (b), (c) and (d).
= = −1 13.
sec θ − sec θ Y
)
1
,y

 a 2 + b2  a 2 + b2
1
x

y=−  , i.e. k = −  
P(

Thus,
 b   b 
M
11. Equation of given hyperbola is X′ X
(–1, 0) N(x2, 0)
2 2 (1, 0)
x y
− = 1 , a > b and a > 1
a 2 b2 x2 – y2 = 1
Let point P (a sec θ, b tan θ) on the hyperbola in first Y′
 π
quadrant i.e., Q ∈ 0,  . Equation of family of circles touching hyperbola at
 2
(x1 , y1 ) is (x − x1 )2 + ( y − y1 )2 + λ( x x1 − y y1 −1) = 0
Now equation of tangent to the hyperbola at point P is Now, its centre is (x2 , 0).
x sec θ y tan θ
− =1 …(i)  − (λx1 − 2x1 ) – (−2 y1 − λy1 ) 
∴ , = (x2 , 0)
a b  2 2 
Q The tangent (i) passes through point A(1, 0)
⇒ 2 y1 + λy1 = 0 ⇒ λ = − 2
So, a = secθ > 1 . … (ii)
and 2x1 − λx1 = 2x2 ⇒ x2 = 2x1
484 Hyperbola

∴ P (x1 , x12 − 1 ) and N (x2 , 0) = (2x1 , 0) x2 y2


15. The equation of the hyperbola is − = 1 and that of
1  9 4
As tangent intersect X-axis at M  ,0 . circle is x2 + y2 − 8x = 0
 x1 
x2 x2 − 8 x
Centroid of ∆PMN = (l, m) For their points of intersection, + =1
9 4
 1 
 3x1 +  ⇒ 4x2 + 9x2 − 72x = 36
 x y + 0 + 0  = (l, m)
⇒ 1
, 1
 3 3  ⇒ 13x2 − 72x − 36 = 0
 
  ⇒ 13x2 − 78x + 6x − 36 = 0
1
3x1 + ⇒ 13x (x − 6) + 6 (x − 6) = 0
x1
⇒ l= 13
3 ⇒ x=6, x= −
6
1
3− 2 13
dl x1 x=− not acceptable.
On differentiating w.r.t. x1 , we get = 6
dx1 3
Now, for x = 6, y = ± 2 3
dl 1 x12 − 1
⇒ = 1 − 2 , for x1 > 1 and m = Required equation is, (x − 6)2 + ( y + 2 3 ) ( y − 2 3 ) = 0
dx1 3x1 3
⇒ x2 − 12x + y2 + 24 = 0
On differentiating w.r.t. x1 , we get
⇒ x2 + y2 − 12x + 24 = 0
dm 2x1 x1
= = , for x1 > 1
dx1 2 × 3 x12 − 1 3 x12 − 1 16. Equation of tangent to hyperbola having slope m is
y y = mx + 9m2 − 4 ...(i)
Also, m= 1
3 Equation of tangent to circle is
dm 1
On differentiating w.r.t. y1 , we get = , for y1 > 0 y = m (x − 4) + 16m2 + 16 ...(ii)
dy1 3
2
2 2
Eqs. (i) and (ii) will be identical for m = satisfy.
14. PLAN Equation of tangent to x 2 − y 2 = 1 is y = mx ± a 2m2 − b 2 5
a b
∴ Equation of common tangent is 2x − 5 y + 4 = 0.
Description of Situation If two straight lines
17. On substituting  , 0 in y = − 2x + 1, we get
a1 x + b1 y + c1 = 0 a
e 
a1 b1 c1
and a 2x + b2 y + c2 = 0 are identical. Then, = = 2a
a 2 b2 c2 0=− +1
e
Equation of tangent, parallel to y = 2x − 1 is
a 1
y = 2x ± 9 (4) − 4 ⇒ =
e 2
∴ y = 2x ± 32 …(i) Also, y = − 2x + 1 is tangent to hyperbola.
The equation of tangent at (x1 , y1 ) is ∴ 1 = 4a 2 − b2
xx1 yy1
− =1 …(ii) ⇒
1
= 4 − (e2 − 1)
9 4
a2
From Eqs. (i) and (ii),
4
2x – y = 1 Tangent ⇒ = 5 − e2
e2
Y
⇒ e4 − 5 e2 + 4 = 0
P (x1, y1) ⇒ (e2 − 4) (e2 − 1) = 0
x x X
(–3, 0) O 12 (3, 0) ⇒ e=2, e=1
–1
Y
x2 – y2
=1
9 4

2 −1 ± 32 F
= = X
x1 − y1 1 F1
9 4 (
– 1
2
, 0 )
9 1
⇒ x1 = − and y1 = −
2 2 2
9 1 e = 1 gives the conic as parabola. But conic is given as
or x1 = , y1 = hyperbola, hence e = 2.
2 2 2
Hyperbola 485

Topic 3 Equation of Chord of Contact, and product of the roots x1 x2 x3 x4 = c4


Chord Bisected Diameter, Similarly, y1 + y2 + y3 + y4 = 0
Asymptote and Rectangular and y1 y2 y3 y4 = c4
Hyperbola Hence, all options are correct.
1. Let (h , k) be a point whose chord of contact with respect 3. Let any point on the hyperbola is (3 sec θ , 2 tan θ ).
to hyperbola x2 − y2 = 9 is x = 9. ∴ Chord of contact of the circle x2 + y2 = 9 with respect
We know that, chord of contact of (h , k) with respect to to the point (3 sec θ , 2 tanθ ) is,
hyperbola x2 − y2 = 9 is T = 0. (3 sec θ ) x + (2 tan θ ) y = 9 …(i)
⇒ h ⋅ x + k (− y) − 9 = 0 Let (x1 , y1 ) be the mid-point of the chord of contact.
∴ hx − ky − 9 = 0 ⇒ Equation of chord in mid-point form is
But it is the equation of the line x = 9. x x1 + yy1 = x12 + y12 …(ii)
This is possible when h = 1, k = 0 (by comparing both Since, Eqs. (i) and (ii) are identically equal.
equations). 3 sec θ 2 tanθ
Again equation of pair of tangents is ∴ =
x1 y1
T 2 = SS1 9
⇒ (x − 9)2 = (x2 − y2 − 9) (12 − 02 − 9) =
x12 + y12
⇒ x2 − 18x + 81 = (x2 − y2 − 9) (−8) 9x1
⇒ sec θ =
⇒ x2 − 18x + 81 = − 8x2 + 8 y2 + 72 3 (x12 + y12 )
⇒ 9x2 − 8 y2 − 18x + 9 = 0 9 y1
and tanθ =
2. It is given that, 2 (x12 + y12 )
x 2 + y2 = a 2 …(i) Thus, eliminating ‘ θ ’ from above equation, we get
and xy = c2
…(ii) 81 x12 81 y12
− =1
We obtain x + c /x = a
2 4 2 2
9 (x12 + y12 )2 4 (x12 + y12 )2
⇒ x − a 2x2 + c4 = 0
4
…(iii) [Q sec2 θ − tan 2 θ = 1]
Now, x1 , x2 , x3 , x4 will be roots of Eq. (iii). x2
y 2
(x + y2 )2
2

Therefore, Σ x1 = x1 + x2 + x3 + x4 = 0 ∴ Required locus is − = .


9 4 81
20
Trigonometrical Ratios
and Identities

Topic 1 Based on Trigonometric Formulae


π π
Objective Questions I (Only one correct option) 8. For any θ ∈  ,  , the expression
π 3π π 3π  
4 2
1. The value of cos3   ⋅ cos   + sin3   ⋅ sin   is 3 (sin θ − cos θ )4 + 6 (sin θ + cos θ )2 + 4 sin 6 θ equals
 8  8  8  8
(2019 Main, 9 Jan I)
(2020 Main, 9 Jan I) (a) 13 − 4 cos4 θ + 2 sin 2 θ cos2 θ
1 1 1 1 (b) 13 − 4 cos2 θ + 6 cos4 θ
(a) (b) (c) (d)
4 2 2 2 2 (c) 13 − 4 cos2 θ + 6 sin 2 θ cos2 θ
2. The value of sin 10º sin 30º sin 50º sin 70º is (d) 13 − 4 cos6 θ
(2019 Main, 9 April II) tan A cot A
1 1 1 1 9. The expression + can be written as
(a) (b) (c) (d) 1 − cot A 1 − tan A (2013 Main)
36 32 16 18
(a) sin A cos A + 1 (b) sec A cosec A + 1
3. The value of cos 2 10° − cos 10° cos 50° + cos 2 50° is (c) tan A + cot A (d) sec A + cosec A
(2019 Main, 9 April I)
3 3 10. The number of ordered pairs (α , β), where α , β ∈ (−π , π )
(a) (1 + cos 20° ) (b) + cos 20° 1
2 4 satisfying cos (α − β ) = 1 and cos (α + β ) = is
(c) 3 / 2 (d) 3 / 4 e (2005, 1M)
(a) 0 (b) 1 (c) 2 (d) 4
4. If the lengths of the sides of a triangle are in AP and the
greatest angle is double the smallest, then a ratio of 11. Given both θ and φ are acute angles and
lengths of the sides of this triangle is (2019 Main, 8 April II) 1 1
sin θ = , cos φ = , then the value of θ + φ belongs to
(a) 3 : 4 : 5 (b) 4 : 5 : 6 (c) 5 : 9 : 13 (d) 5 : 6 : 7 2 3 (2004, 1M)
3 5 π π π π 2π
5. If cos(α + β) = , sin(α − β) = and 0 < α , β < , then (a)  ,  (b)  , 
5 13 4  3 6  2 3
tan(2α ) is equal to (2019 Main, 8 April I)
2π 5π  5π 
63 63 21 33 (c)  , (d)  ,π
(a) (b) (c) (d)  3 6   6 
52 16 16 52
1 12. Which of the following numbers is rational? (1998, 2M)
6. Let fk (x) = (sin k x + cos k x) for k = 1, 2, 3 ... . Then, for
k (a) sin 15° (b) cos 15°
all x ∈ R, the value of f4 (x) − f6 (x) is equal to (c) sin 15° cos 15° (d) sin 15° cos 75°
(2019 Main, 11 Jan I)
1 5 −1 1
13. 3 (sin x − cos x)4 + 6 (sin x + cos x)2 + 4 (sin 6 x + cos 6 x)
(a) (b) (c) (d) equals (1995, 2M)
12 12 12 4
(a) 11 (b) 12 (c) 13 (d) 14
7. The value of
π π π π 14. The value of the expression 3 cosec 20° − sec 20° is
cos ⋅ cos 3 ....... cos 10 ⋅ sin 10 is
22 2 2 2 (2019 Main, 10 Jan II) equal to (1988, 2M)
1 1 1 1 (a) 2 (b) 2 sin 20° /sin 40°
(a) (b) (c) (d)
1024 2 512 256 (c) 4 (d) 4 sin 20° /sin 40°
Trigonometrical Ratios and Identities 487

π π
15. The expression (a) f2   = 1 (b) f3   = 1
  3π    16   32 
3 sin 4  − α  + sin 4 (3π + α ) π π 
  2   (c) f4   = 1 (d) f5   =1
 64   128 
 6 π  
−2 sin  + α  + sin 6 (5π − α )
 2   Match the Column
is equal to (1986, 2M)
Match the conditions/expressions in Column I with values
(a) 0 (b) 1
in Column II.
(c) 3 (d) sin 4α + cos 6 α
π 3π 5π 7π
23. (sin 3 α ) / (cos 2 α ) is
16. 1 + cos  1 + cos  1 + cos  1 + cos  is
(1992, 2M)
 8  8  8 8
Column I Column II
equal to (1984, 3M)
1 π 1 1+ 2 A. positive p. (13 π / 48, 14 π / 48)
(a) (b) cos (c) (d)
2 8 8 2 2
B. negative q. (14 π / 48, 18 π / 48)
17. Given A = sin 2 θ + cos 4 θ , then for all real values of θ
(1980, 1M) r. (18 π / 48, 23 π / 48)
3 s. (0, π / 2)
(a) 1 ≤ A ≤ 2 (b) ≤ A ≤ 1
4
13 3 13
(c) ≤ A≤1 (d) ≤ A ≤
16 4 16 Fill in the Blanks
π  5π   7π 
24. If k = sin 
4
18. If tan θ = − , then sin θ is (1978, 2M)  sin   sin   , then the numerical
3  18  18   18 
4 4 4 4 value of k is ……
(a) − but not (b) − or (1993, 2M)
5 5 5 5
4 4 25. The value of
(c) but not − (d) None of the above π 3π 5π 7π 9π 11π 13π
5 5 sin ⋅ sin ⋅ sin ⋅ sin ⋅ sin ⋅ sin ⋅ sin
14 14 14 14 14 14 14
is equal to …… . (1991, 2M)
Objective Questions II
(One or more than one correct option) Analytical & Descriptive Questions
2
19. Let f : (−1, 1) → R be such that f (cos 4 θ ) = for 26. Prove that
2 − sec2 θ tan α + 2 tan 2α + 4 tan 4α + 8 cot 8α = cot α (1988, 2M)
 π  π π  1  2π   4π   8π   16π 
θ ∈  0 ,  ∪  ,  . Then, the value(s) of f   is/are
 4  4 2  3 27. Show that 16 cos   cos   cos   cos   =1
(2012)  15   15   15   15 
3 3 2 2 (1983, 2M)
(a) 1 − (b) 1 + (c) 1 − (d) 1 +
2 2 3 3 28. Without using tables, prove that
π 1
20. For 0 < θ < , the solution(s) of (sin 12° ) (sin 48° ) (sin 54° ) = .
2 8 (1982, 2M)
6
 (m − 1) π   mπ 
∑ cosec θ + 4  cosec θ + 4  = 4 2 is/are 29. Prove that sin 2 α + sin 2 β – sin 2 γ = 2 sinα sin β sin γ ,
m =1 (2009) where α + β + γ = π . (1978, 4M)
π π
(a) (b)
4 6 Integer & Numerical Answer Type Questions
π 5π
(c) (d) 2 sin α 1 1 − cos 2β 1  π
12 12 30. If = and = , α , β ∈ 0,  ,
1 + cos 2α 7 2 10  2
sin 4 x cos 4 x 1
21. If + = , then then tan(α + 2β ) is equal to .............. .
2 3 5 (2009)
2 sin 8 x cos8 x 1 (2020 Main, 8 Jan II)
(a) tan x =2
(b) + =
3 8 27 125 31. The number of all possible values of θ, where 0 < θ < π,
1 sin 8 x cos8 x 2 for which the system of equations
(c) tan x =
2
(d) + =
3 8 27 125 ( y + z ) cos 3 θ = ( xyz ) sin 3 θ
22. For a positive integer n, let 2 cos 3 θ 2 sin 3 θ
x sin 3 θ = + `
 θ y z
fn (θ ) =  tan  (1 + sec θ )(1 + sec 2 θ )
 2 and (xyz )sin 3 θ = ( y + 2z ) cos 3 θ + ysin 3 θ have a
(1 + sec 22 θ )... (1 + sec 2nθ ), then (1999, 3M) solution (x0 , y0 , z0 ) with y0z0 ≠ 0, is …… (2010)
488 Trigonometrical Ratios and Identities

Topic 2 Graph and Conditional Identities


Objective Questions I (Only one correct option) True/False
π 4. If tan A = (1 − cos B) / sin B, then tan 2 A = tan B.
1. If α + β = and β + γ = α , then tan α equals (2001, 1M)
2 (1993, 1M)
(a) 2 (tan β + tan γ ) (b) tan β + tan γ
(c) tan β + 2 tan γ (d) 2 tan β + tan γ Analytical & Descriptive Questions
2. If α + β + γ = 2π ,then (1979, 2M) 5. In any triangle, prove that
α β γ α β γ A B C A B C
(a) tan + tan + tan = tan tan tan cot + cot + cot = cot cot cot
2 2 2 2 2 2 2 2 2 2 2 2 (2000, 3M)
α β β γ γ α
(b) tan tan + tan tan + tan tan = 1 6. ABC is a triangle such that
2 2 2 2 2 2
α β γ α β γ
sin( 2 A + B) = sin(C − A) = − sin( B + 2C ) = 1/ 2.
(c) tan + tan + tan = − tan tan tan
2 2 2 2 2 2 If A, B and C are in arithmetic progression, determine
the values of A , B and C. (1990, 5M)
(d) None of the above
7. Given α + β + γ = π, prove that
Fill in the Blank sin 2 α + sin 2 β − sin 2 γ = 2 sin α sin β cos γ. (1980, 3M)
n
8. If A + B + C = 180°, then prove that
3. Suppose sin3 x sin 3 x = ∑ Cm cos nx is an identity in x, tan A + tan B + tan C = tan A tan B tan C. (1979, 3M)
m= 0
where C 0 , C1 , K , C n are constants and C n ≠ 0. Then, the 4 5
9. If cos (α + β) = , sin (α − β) = and α , β lie between 0
value of n is… . (1981, 2M) 5 13
and π /4, find tan 2 α. (1979, 4M)

Topic 3 Maxima and Minima


Objective Questions I (Only one correct option) (a) t1 > t2 > t3 > t4 (b) t4 > t3 > t1 > t2
(c) t3 > t1 > t2 > t4 (d) t2 > t3 > t1 > t4
1 The maximum value of
 π
3 cos θ + 5 sin θ −  Fill in the Blank
 6
for any real value of θ is (2019 Main, 12 Jan I) 3. If A > 0, B > 0 and A + B = π /3, then the maximum value
79 of tan A tan B is ........... (1993)
(a) (b) 34
2
(c) 31 (d) 19 Analytical & Descriptive Question
π sin x cos 3x
2. Let θ ∈ 0,  and t1 = (tan θ )tan θ , t2 = (tan θ )cot θ , 4. Prove that the values of the function do not
 4 sin 3x cos x
t3 = (cot θ )tan θ and t4 = (cot θ )cot θ , then (2006, 3M) lie between 1 / 3 and 3 for any real x. (1997, 5M)

Topic 4 Height & Distance


1. The angle of elevation of the top of a vertical tower ground when the top of the ladder is 1 m above the
standing on a horizontal plane is observed to be 45° ground is (2019 Main, 12 April I)
from a point A on the plane. Let B be the point 30 m 25 25
(a) 25 3 (b) (c) (d) 25
vertically above the point A. If the angle of elevation of 3 3
the top of the tower from B be 30°, then the distance (in
3. ABC is a triangular park with AB = AC = 100 m. A
m) of the foot of the tower from the point A is
(2019 Main, 12 April II) vertical tower is situated at the mid-point of BC. If the
(a) 15 (3 + 3 ) (b) 15 (5 − 3 ) angles of elevation of the top of the tower at A and B are
(c) 15 (3 − 3 ) (d) 15 (1 + 3 ) cot−1 (3 2 ) and cosec−1 (2 2 ) respectively, then the
2. A 2 m ladder leans against a vertical wall. If the top of height of the tower (in m) is (2019 Main, 10 April I)
the ladder begins to slide down the wall at the rate 25 (a) 25 (b) 20
cm/s, then the rate (in cm/s) at which the bottom of the 100
(c) 10 5 (d)
ladder slides away from the wall on the horizontal 3 3
Trigonometrical Ratios and Identities 489

4. Two poles standing on a horizontal ground are of 8. PQR is a triangular park with PQ = PR = 200 m. A TV
heights 5 m and 10 m, respectively. The line joining tower stands at the mid-point of QR. If the angles of
their tops makes an angle of 15º with the ground. elevation of the top of the tower at P , Q and R are
Then, the distance (in m) between the poles, is respectively 45°, 30° and 30°, then the height of the
(2019 Main, 9 April II) tower (in m) is (2018 Main)
5
(a) 5( 3 + 1) (b) (2 + 3 ) (a) 100 (b) 50
2
(c) 100 3 (d) 50 2
(c) 10( 3 − 1) (d) 5(2 + 3 )
5. Two vertical poles of heights, 20 m and 80 m stand apart 9. Let a vertical tower AB have its end A on the level
ground. Let C be the mid-point of AB and P be a point on
on a horizontal plane. The height (in m) of the point of
the ground such that AP = 2 AB. If ∠BPC = β, then tan β
intersection of the lines joining the top of each pole to
is equal to (2017 Main)
the foot of the other, from this horizontal plane is
6 1
(2019 Main, 8 April II) (a) (b)
(a) 15 (b) 16 (c) 12 (d) 18 7 4
2 4
6. If the angle of elevation of a cloud from a point P which (c) (d)
9 9
is 25 m above a lake be 30º and the angle of depression of
reflection of the cloud in the lake from P be 60º, then the 10. A man is walking towards a vertical pillar in a straight
height of the cloud (in meters) from the surface of the path, at a uniform speed. At a certain point A on the
lake is (2019 Main, 12 Jan II) path, he observes that the angle of elevation of the top of
(a) 50 (b) 60 (c) 45 (d) 42 the pillar is 30°. After walking for 10 min from A in the
same direction, at a point B, he observes that the
7. Consider a triangular plot ABC with sides AB = 7 m, angle of elevation of the top of the pillar is 60°. Then, the
BC = 5 m and CA = 6 m. A vertical lamp-post at the time taken (in minutes) by him, from B to reach the
mid-point D of AC subtends an angle 30° at B. The pillar, is (2016 Main)
height (in m) of the lamp-post is (2019 Main, 10 Jan I)
2 3 (a) 6 (b) 10
(a) 21 (b) 2 21 (c) 7 3 (d) 21 (c) 20 (d) 5
3 2

Answers
Topic 1 6. A = 45 °, B = 60 °, C = 75 °
1. (b) 2. (c) 3. (d) 4. (b) 56
9.
5. (b) 6. (a) 7. (c) 8. (d) 33
9. (b) 10. (b) 11. (b) 12. (c)
Topic 3
13. (c) 14. (c) 15. (b) 16. (c) 1
17. (b) 18. (b) 19. (a, b) 20. (c, d) 1. (d) 2. (b) 3.
3
21. (a, b) 22. (a, b, c, d) 23. A → r; B → p
1 1 π 5π 9π Topic 4
24. 25. 30. (1) 31. θ = , ,
8 64 12 12 12 1. (a) 2. (b) 3. (b) 4. (d)
Topic 2 5. (b) 6. (a) 7. (a) 8. (a)
1. (c) 2. (a) 3. 6 4. True 9. (c) 10. (d)

Hints & Solutions


Topic 1 Based on Trigonometric Formulae  π  π  π  π
= cos3   sin   + sin3   cos  
 8  8  8  8
1. Given trigonometric expression
 π  3π   π 3π  π  π  2 π  2 π  
cos3   cos   + sin3   sin = sin   cos   cos  8  + sin  8  
 8  8  8 8  8  8  
 π  π π  π  π π 1  π  π 1 π 1
cos3   cos  −  + sin3   sin  −  = × 2 sin   cos   = sin =
 8  2 8  8  2 8 2  8  8 2 4 2 2
 π π 3π 
Q − = Hence, option (b) is correct.
 2 8 8 
490 Trigonometrical Ratios and Identities

2. We have, sin 10° sin 30° sin 50° sin 70° 1


or cos θ = − (rejected).
= sin(30° )[sin(10° )sin(50° )sin(70° )] 2
Clearly, the ratio of sides is a : b : c
1
= [sin(10° )sin(60° − 10° )sin(60° + 10° )] = sin θ : sin 3 θ : sin 2 θ
2
1 1  = sin θ : (3 sin θ − 4 sin3 θ ) : 2 sin θ cos θ
= sin(3(10° ))
2 4  = 1 : (3 − 4 sin 2 θ ) : 2 cos θ
1 = 1 : (4 cos 2 θ − 1) : 2 cos θ
[Qsin θ sin(60° − θ )sin(60° + θ ) = sin 3 θ]
4 5 6
1 1 1 1 = 1 : : = 4 :5 :6
= sin 30° = × = 4 4
8 8 2 16
5
3. We have, cos 2 10º − cos 10º cos 50º + cos 2 50º 5. Given, sin(α − β) =
13
1 3  π
= [2 cos 2 10º −2 cos 10º cos 50º +2 cos 2 50º ] and cos(α + β ) = , where α , β ∈ 0, 
5  4
2
1 π π
= [1 + cos 20º − (cos 60º + cos 40º ) + 1 + cos 100º ] Since, 0 < α < and 0 < β <
2 4 4
[Q 2 cos 2 A = 1 + cos 2 A and π π π
2 cos A cos B = cos( A + B) + cos( A − B)] ∴ 0 <α +β < + =
4 4 2
1 1   1 π
= 2 + cos 20º + cos 100º − − cos 40º Q cos 60º = ⇒ 0 <α +β <
2  2   2  2
1 3  π
= + (cos 20º − cos 40º ) + cos 100º Also, − < −β < 0
2 2  4
1 3 20º + 40º 20°− 40°  π π π π
= − 2 sin sin + cos 100° ∴ 0 − < α −β < + 0 ⇒ − < α −β <
2 2 2 2  4 4 4 4
 C + D C − D  π  π π
Q cos C − cos D = −2 sin sin ∴ α + β ∈ 0,  and α − β ∈  − , 
 2 2   2  4 4
1 3   π
= − 2 sin 30º sin(−10º ) + cos(90º +10º ) But sin(α − β ) > 0, therefore α − β ∈ 0,  .
2 2   4
1 3  5
= + sin 10º − sin 10º [Q cos (90º + θ ) = − sin θ ] Now, sin(α − β ) =
2 2  13
5
1 3 3 ⇒ tan(α − β ) = …(i)
= × = 12
2 2 4
3
and cos(α + β ) =
4. Let a , b and c be the lengths of sides of a ∆ABC such 5
that a < b < c. 4
⇒ tan (α + β ) = …(ii)
Since, sides are in AP. 3
∴ 2b = a + c …(i) Now, tan(2α ) = tan[(α + β ) + (α − β )]
Let ∠ A =θ 4 5
+
Then, ∠ C = 2θ [according to the question] tan(α + β ) + tan(α − β ) 3 12
= =
So, ∠ B = π − 3θ …(ii) 1 − tan(α + β ) tan(α − β ) 1 − 4 × 5
On applying sine rule in Eq. (i), we get 3 12
2 sin B = sin A + sin C [from Eqs. (i) and (ii)]
⇒ 2 sin(π − 3 θ ) = sin θ + sin 2 θ [from Eq. (ii)] 48 + 15 63
= =
⇒ 2 sin 3 θ = sin θ + sin 2 θ 36 − 20 16
⇒ 2 [3 sin θ − 4 sin3 θ ] = sin θ + 2 sin θ cos θ 6. We have,
⇒ 6 − 8 sin 2 θ = 1 + 2 cos θ [Qsin θ can not be zero] 1
fk (x) = (sin k x + cos k x), k = 1, 2, 3, …
⇒ 6 − 8(1 − cos 2 θ ) = 1 + 2 cos θ k
⇒ 8 cos 2 θ − 2 cos θ − 3 = 0 1
∴ f4 (x) = (sin 4 x + cos 4 x)
⇒ (2 cos θ + 1)(4 cos θ − 3) = 0 4
3 1
⇒ cos θ = = ((sin 2 x + cos 2 x)2 − 2 sin 2 x cos 2 x)
4 4
Trigonometrical Ratios and Identities 491

1 1 2 1 1 10. Since, cos (α − β ) = 1


= 1 − (sin 2x)  = − sin 2x
2
4 2  4 8
⇒ α − β = 2nπ
1
and f6 (x) = (sin 6 x + cos 6 x) But −2 π < α − β < 2 π [as α , β ∈ (− π , π )]
6 ∴ α −β =0 …(i)
1 1
= {(sin 2 x + cos 2 x)3 − 3 sin 2 x cos 2 x Given, cos (α + β ) =
6 e
(sin2 x + cos2 x )} ⇒ cos 2α =
1
< 1, which is true for four values of α .
1 e
3 2 1 1
= 1 − (2 sin x cos x)  = − sin 2x
2
[as −2π < 2α < 2π]
6 4  6 8
1
1 1 3 −2 1 11. Since, sin θ =
Now, f4 (x) − f6 (x) = − = = 2
4 6 12 12
1 π
and cos φ = ⇒ θ =
7. We know that, 3 6
sin (2nα )  1 1  1 1
cos α ⋅ cos (2α ) cos(22α )…cos (2n−1α ) = and 0 <  cos φ =  < as 0 < <
2n sin α  3 2  3 2 
π π π π π  1
∴ cos 2
⋅ cos 3 …cos 10 ⋅ sin 10 ⇒ θ= and cos −1 (0) > φ > cos −1  
2 2 2 2 6  2
  π 9    π 
sin  210 2   π π the sign changed as cos x is decreasing between 0, 2  
=  sin 10 [Qhere, α = 10 and n = 9 ]  

2 sin    π  2 2
9
π π π π 2π
  210   ⇒ θ= and <φ < ⇒ <θ + φ <
6 3 2 2 3
1  π 1 1  π 2 π 
= sin   = 9 = ∴ θ∈ , 
29  2 2 512 2 3 
8. Given expression 1 1
12. Since, sin 15° = 2 − 3 and cos 15° = 2+ 3
= 3(sin θ − cos θ ) + 6(sin θ + cos θ ) + 4 sin θ
4 2 6
2 2
= 3((sin θ − cos θ )2)2 + 6(sin θ + cos θ )2 + 4(sin 2 θ )3 1
and sin 15° cos 75° = sin 15°⋅ sin 15° = (2 − 3 )
= 3(1 − sin 2 θ )2 + 6(1 + sin 2 θ ) + 4(1 − cos 2 θ )3 4
[Q1 + sin 2 θ = (cos θ + sin θ )2 Therefore, all these values are irrational and
and 1 − sin 2 θ = (cos θ − sin θ )2] 1
sin 15° cos 15° = ⋅ 2 sin 15° cos 15°
= 3(1 + sin 2 θ − 2 sin 2 θ ) + 6(1 + sin 2 θ )
2 2
2
+ 4(1 − cos 6 θ − 3 cos 2 θ + 3 cos 4 θ ) 1 1
= ⋅ sin 30° = , which is rational.
[Q (a − b)2 = a 2 + b2 − 2ab 2 4
and (a − b) = a3 − b3 − 3a 2b + 3ab2]
3
13. Given expression =
= 3 + 3 sin 2 θ − 6 sin 2 θ + 6 + 6 sin 2 θ + 4
2
3 (sin x − cos x)4 + 6 (sin x + cos x)2 + 4(sin 6 x + cos 6 x)
− 4 cos 6 θ − 12 cos 2 θ + 12 cos 4 θ = 3 (1 − sin 2x)2 + 6 (1 + sin 2x) + 4 {(sin 2 x + cos 2 x)3
= 13 + 3 sin 2 2 θ − 4 cos 6 θ − 12 cos 2 θ + 12 cos 4 θ − 3 sin 2 x cos 2 x (sin 2 x + cos 2 x)}
= 13 + 3(2 sin θ cos θ )2 − 4 cos 6 θ − 12 cos 2 θ (1 − cos 2 θ ) = 3 (1 − 2 sin 2x + sin 2 2x) + 6 + 6 sin 2x
= 13 + 12 sin 2 θ cos 2 θ − 4 cos 6 θ − 12 cos 2 θ sin 2 θ + 4 (1 − 3 sin 2 x cos 2 x)
= 13 − 4 cos 6 θ  3 
= 3 (1 − 2 sin 2x + sin 2 2x + 2 + 2 sin 2x) + 4 1 − ⋅ sin 2 2x
 4 
9. Given expression is
tan A cot A = 13 + 3 sin 2 2x − 3 sin 2 2x = 13
+
1 − cot A 1 − tan A 14. Given expression =
=
sin A
×
sin A
+
cos A
×
cos A 3 cosec 20° − sec 20° = tan 60° cosec 20° − sec 20°
cos A sin A − cos A sin A cos A − sin A sin 60° cos 20° − cos 60°⋅ sin 20°
=
sin3 A − cos3 A  cos 60°⋅ sin 20°⋅ cos 20°
1
=   sin (60° − 20° ) sin 40°
sin A − cos A  cos A sin A  = =
cos 60°⋅ sin 20°⋅ cos 20° 1 ⋅ sin 20° cos 20°
sin 2 A + sin A cos A + cos 2 A
= 2
sin A cos A 2 sin 20° cos 20°
= =4
1 + sin A cos A 1
= = 1 + sec A cosec A sin 20° cos 20°
sin A cos A 2
492 Trigonometrical Ratios and Identities

15. Given expression = π


20. For 0 < θ <
2
  3π    π 
3 sin 4  − α  + sin 4 (3π + α ) −2 sin 6  + α  6
 (m − 1) π   mπ 
  2     2  ∑ cosec θ + 4
 cosec θ +
 
 =4 2
4 
m =1
+ sin (5π − α )]
6
6
= 3 (cos 4 α + sin 4 α ) − 2(cos 6 α + sin 6 α ) 1
= 3 (1 − 2 sin α cos α ) − 2 (1 − 3 sin α cos α )
2 2 2 2
⇒ ∑  (m − 1 )π  mπ 
=4 2
m = 1 sin  θ +  sin θ + 
 4   4 
= 3 − 6 sin 2 α cos 2 α − 2 + 6 sin 2 α cos 2 α = 1
 mπ  (m − 1) π  
sin θ + − θ + 
16. Given expression = 6
 4  4 
 π  3π   5π   7π 
⇒ ∑ π  (m − 1 ) π   mπ  
=4 2
1 + cos  1 + cos  1 + cos  1 + cos  m =1 sin sin θ +  sin θ + 
 8   8   8   8 4  4   4 
π  3π   3π   π  (m − 1) π   mπ 
 6 cot  θ +  − cot θ + 
= 1 + cos  1 + cos  1 − cos  1 − cos   4   4 
 8  8  8  8 ⇒ ∑ 1/ 2
=4 2
m =1
 π  3π 
= 1 − cos 2  1 − cos 2  6
  (m − 1) π   mπ  
 8  8 ⇒ ∑ cot θ +
 4
 − cot θ +
 
 =4
4  
1 π  π m =1
= 2 − 1 − cos  2 − 1 − cos 3   π  π  2π 
4 4  4 ⇒ cot (θ ) − cot θ +  + cot θ +  − cot θ + 
 4   4   4
1 π  π
= 1 − cos  1 − cos 3   5π   6π 
4 4  4 + ... + cot θ +  − cot θ +  =4
 4  4
1 1 1 1 1 1  3π 
= 1 −  1 +  = 1 −  = ⇒ cot θ − cot  + θ = 4
4 2   2  4  2 8  2 
17. Given, A = sin 2 θ + (1 − sin 2 θ )2 ⇒ cot θ + tan θ = 4
⇒ tan 2 θ − 4 tan θ + 1 = 0
⇒ A = sin 4 θ − sin 2 θ + 1
⇒ (tan θ − 2)2 − 3 = 0
2
 1 3 ⇒ (tan θ − 2 + 3 ) (tan θ − 2 − 3 ) = 0
⇒ A = sin 2 θ −  +
 2 4 ⇒ tan θ = 2 − 3 or tan θ = 2 + 3
2 π 5π   π 
 1 ⇒ θ= ;θ= Q θ ∈ 0, 2  
1
⇒ 0 ≤ sin 2 θ −  ≤ [Q 0 ≤ sin 2 θ ≤ 1] 12 12  
 2 4

3
≤ A ≤1 sin 4 x cos 4 x 1 sin 4 x (1 − sin 2 x)2 1
21. + = ⇒ + =
4 2 3 5 2 3 5
18. Since, tan θ < 0. sin x 1 + sin x − 2 sin x 1
4 4 2
⇒ + =
∴ Angle θ is either in the second or fourth quadrant. 2 3 5
6
Then, sin θ > 0 or < 0 ⇒ 5 sin 4 x − 4 sin 2 x + 2 =
5
∴ sin θ may be
4
or −
4 ⇒ 25 sin 4 x − 20 sin 2 x + 4 = 0
5 5 ⇒ (5 sin 2 x − 2)2 = 0
2 2
19. f (cos 4θ ) = …(i) ⇒ sin 2 x =
2 − sec2 θ 5
3 2
1 cos x = , tan 2 x =
2

At cos 4θ = 5 3
3
sin 8 x cos 8 x 1
1 ∴ + =
⇒ 2 cos 2 θ − 1 =
2
8 27 125
3
2 2 22. NOTE Multiplicative loop is very important approach in IIT
⇒ cos 2 θ = ⇒ cos 2 θ = ±
2
…(ii)
3 3 Mathematics.
2 ⋅ cos 2 θ  θ sin θ /2  1 
∴ f (cos 4 θ ) =  tan  (1 + sec θ ) = ⋅ 1+

2 cos 2 θ − 1  2  cos θ/2  cos θ 
1 + cos 2 θ (sin θ /2) 2 cos 2 θ /2
= =
cos 2 θ (cos θ/2) cos θ
 1 3 (2 sin θ /2) cos θ /2 sin θ
⇒ f   =1 ± [from Eq. (ii)] = = = tan θ
 3 2 cos θ cos θ
Trigonometrical Ratios and Identities 493

∴ fn (θ ) = (tan θ /2)(1 + sec θ )  π π  π 3π   π 5π  


2
=  cos  −  ⋅ cos  −  ⋅ cos  − 
(1 + sec 2 θ ) (1 + sec 22θ )... (1 + sec 2nθ )   2 14  2 14   2 14  
= (tan θ )(1 + sec 2 θ )(1 + sec 22θ ).... (1 + sec 2nθ ) 2
 3π 2π π
= tan 2 θ ⋅ (1 + sec 22θ )... (1 + sec 2nθ ) =  cos ⋅ cos ⋅ cos 
 7 7 7
= tan (2nθ ) 2
 π 2π 4π 
π  π  π =  − cos ⋅ cos ⋅ cos 
Now, f2  = tan 22 ⋅  = tan   = 1  7 7 7
 16  16  4
2
 sin 23 π / 7 
= − 3 
Therefore, (a) is the answer.  2 ⋅ sin π / 7
π  π  π
f3   = tan 23 ⋅  = tan   = 1  1 sin 8π / 7
2
 8π  π π
 32  32  4 = − ⋅ 
 8 sin π / 7  Q sin 7 = sin  π + 7  = − sin 7 
 
Therefore, (b) is the answer. = 1 / 64
π  π  π
f4   = tan 24 ⋅  = tan   = 1 26. We know that,
 64  64   4
1 − tan 2 θ  1 − tan 2 θ 
Therefore, (c) is the answer. cot θ − tan θ = =2  = 2 cot 2 θ …(i)
 π   5 π   π tan θ  2 tan θ 
f5   = tan 2 ⋅  = tan   = 1
 128  128  4 LHS = tan α + 2 tan 2 α + 4 tan 4 α + 8 cot 8 α
Therefore, (d) is the answer.
= − (cot α − tan α − 2 tan 2 α − 4 tan 4 α )
13 π 14 π 
23. In the interval  ,  , cos 2 α < 0 and sin 3 α > 0. + 8 cot 8 α + cot α
 48 48  = – (2 cot 2 α − 2 tan 2 α − 4 tan 4 α )
sin 3 α + 8 cot 8 α + cot α
⇒ is negative, therefore B → p.
cos 2 α [from Eq. (i)]
 18 π 23 π  = − (2 (cot 2 α − tan 2 α ) − 4 tan 4 α )
Again, in the interval  ,  , both sin 3 α and
 48 48  + 8 cot 8 α + cot α
sin 3 α = − (2 (2 cot 4 α ) − 4 tan 4 α ) + 8 cot 8 α + cot α
cos 2 α are negative, so is positive, therefore A→ r.
cos 2 α
[from Eq. (i)]
24. Using the relation, = − 4 (cot 4 α − tan 4α ) + 8 cot 8 α + cot α
π  π  sin 3 θ = − 8 cot 8 α + 8 cot 8 α + cot α
sin θ sin  − θ sin  + θ = [from Eq. (i)]
3  3  4
= cot α = RHS
π
Taking θ = , we get 2π 4π 8π 16π 
18 27. 16  cos ⋅ cos ⋅ cos ⋅ cos 
π  15 15 15 15 
sin
π 5π 7π 6 =1 = 16 (cos A ⋅ cos 2 A cos 22 A ⋅ cos 23 A)
sin ⋅ sin ⋅ sin =
18 18 18 4 8  2π 
Alternative Method where, A = 15 
Given, k = sin 10° ⋅ sin 50° ⋅ sin 70°  2π 
sin 24  
= cos 80° ⋅ cos 40° ⋅ cos 20°  sin 24 A   15 
= 16  4 =
sin 23 A  2 sin A  2π 
= cos A ⋅ cos 2 A ⋅ cos 22 A = sin  
23 sin A  15 
where, A = 20°  32π   2π 
sin   sin 2π + 
sin 160° sin (180°–20° ) sin 20° 1  15   15 
= = = = = =
8 sin 20° 8 sin 20° 8 sin 20° 8  2π   2π 
sin   sin  
 15   15 
π 3π 5π 7π 9π 11π 13π
25. sin ⋅ sin ⋅ sin ⋅ sin ⋅ sin ⋅ sin ⋅ sin
14 14 14 14 14 14 14  2π 
sin  
π 3π 5π  5π   15 
= sin ⋅ sin ⋅ sin ⋅ sin  π −  = =1
14 14 14  14   2π 
sin  
 15 
 3π   π
⋅ sin  π −  ⋅ sin  π −  1
 14   14 28. sin 12° sin 48° sin 54° = (2 sin 12° sin 48° ) sin 54°
2 2
π 3π 5π  π 3π 5π  1
= sin 2 ⋅ sin 2 ⋅ sin 2 = sin ⋅ sin ⋅ sin  = [cos (36° ) − cos (60° )] sin 54°
14 14 14  14 14 14  2
494 Trigonometrical Ratios and Identities

1 1 31. Given equations can be written as


=  cos 36° −  sin 54°
2 2 cos 3 θ cos 3 θ
x sin 3 θ − − =0 ...(i)
1 y z
= (2 cos 36° sin 54° − sin 54° )
4 2 cos 3 θ 2 sin 3 θ
1 x sin 3 θ − − =0 ...(ii)
= (sin 90° + sin 18° − sin 54° ) y z
4 2 1
1 5 −1 5 + 1 and x sin 3 θ − cos 3 θ − (cos 3 θ + sin 3 θ ) = 0 ...(iii)
= 1 + −  y z
4 4 4  Eqs. (ii) and (iii), implies
1 5 − 1 − 5 − 1 2 sin 3 θ = cos 3 θ + sin 3 θ
= 1 + 
4 4  ⇒ sin 3 θ = cos 3 θ
1 1 1 ∴ tan 3 θ = 1
= 1 −  =
4 2 8 π 5π 9π π 5π 9π
⇒ 3θ = , , or θ = , ,
29. LHS = sin 2 α + sin 2 β − sin 2 γ 4 4 4 12 12 12

= sin 2 α + (sin 2 β − sin 2 γ ) Topic 2 Graph and Conditional Identities


= sin 2 α + sin ( β + γ ) sin ( β − γ )
= sin 2 α + sin ( π − α )sin (β − γ ) [Qα + β + γ = π] 1. Given, α + β = π /2
= sin 2 α + sin α sin ( β − γ ) ⇒ α = (π /2) − β
= sin α [sin α + sin ( β − γ )] ⇒ tan α = tan (π /2 − β )
= sin α [sin (π − ( β + γ )) + sin ( β − γ )] ⇒ tan α = cot β
= sin α [sin ( β + γ ) + sin ( β − γ )] ⇒ tan α tan β = 1
= sin α [2 sin β cos γ ]
Again, β + γ =α [given]
= 2 sin α sin β cos γ = RHS
⇒ γ = (α − β )
π
30. It is given that, for α , β ∈ 0,  ⇒ tan γ = tan (α − β )
 2
tan α − tan β
2 sin α 1 ⇒ tan γ =
= 1 + tan α tan β
1 + cos 2α 7
tan α − tan β
2 sin α 1 ⇒ tan γ =
⇒ = 1+1
2|cos α| 7 ∴ 2 tan γ = tan α − tan β
1  π ⇒ tan α = tan β + 2 tan γ
⇒ tan α = Q α ∈ 0,  …(i)
7  2 α β  γ
2. Since, + = π − 
1 − cos 2 β 1 2 2  2
and =
2 10  α β  γ
∴ tan  +  = tan  π − 
1  2 2  2
⇒ |sin β| =
10 α β
tan + tan
⇒ sin β =
1  π
Qβ ∈ 0,  ⇒ 2 2 = − tan γ
10  2 α β 2
1 − tan tan
1 2 2
∴ tan β = …(ii)
3 α β γ α β γ
⇒ tan + tan + tan = tan tan tan
2 tan β 2 2 2 2 2 2
∴ tan 2 β =
1 − tan 2 β n
3. Given, sin3 x sin 3x = Σ Cm cos nx is an identity in x,
2 m= 0
6 3 where, C 0 , C1 ,... , C n are constants.
= 3 = =
1 8 4 1
1− sin3 x sin 3x = {3 sin x − sin 3x} ⋅ sin 3x
9 4
tan α + tan 2 β 1 3
Now, tan(α + 2 β ) = 
1 − tan α tan 2 β =  ⋅ 2 sin x ⋅ sin 3x − sin 2 3x
4 2 
1 3
+ 1 3 1 
7 4 4 + 21 25 =
 (cos 2x − cos x) − (1 − cos 6x)
= = = =1 4 2 2 
 1 3 28 − 3 25
1− ×  1
 7 4 = (cos 6x + 3 cos 2x − 3 cos x − 1)
8
Hence, answer is 1.
∴ On comparing both sides, we get n = 6
Trigonometrical Ratios and Identities 495

2 sin 2
B ⇒ tan A + tan B = − tan C + tan A tan B tan C
1 − cos B 2 ⇒ tan A + tan B + tan C = tan A tan B tan C
4. Since, tan A = =
sin B B B 4
2 sin cos 9. Since, cos(α + β) =
2 2 5
tan A = tan B / 2 5
⇒ tan 2 A = tan B and sin(α − β ) =
13
Hence, it is a true statement. 3
∴ tan(α + β ) =
5. Since, A+ B+C=π 4
5
A B π C and tan(α − β ) =
⇒ + = − 12
2 2 2 2
Now, tan 2 α = tan[(α + β ) + (α − β )]
 A B  π C
⇒ cot  +  = cot  −  3 5
+
2 2   2 2 tan(α + β ) + tan(α − β ) 56
= = 4 12 =
A B 1 − tan(α + β ) ⋅ tan(α − β ) 1 − 3 ⋅ 5 33
cot ⋅ cot − 1
2 2 C
⇒ = tan 4 12
B A 2
cot + cot Topic 3 Maxima and Minima
2 2
π
A B C C
⇒ cot ⋅ cot ⋅ cot − cot = cot + cot
A B 1. Given expression 3 cos θ + 5 sin θ − 
6 
2 2 2 2 2 2
A B C A B C  π π 
⇒ cot + cot + cot = cot cot cot = 3 cos θ + 5 sin θ cos − sin cos θ
2 2 2 2 2 2  6 6 
 3 1 
6. Given, in ∆ABC , A , B and C are in an AP. = 3 cos θ + 5  sin θ − cos θ
 2 2 
∴ A + C = 2B
Also, A + B + C = 180° ⇒ B = 60° 5
= 3 cos θ −cos θ +
5 3
sin θ
and sin (2 A + B) = sin (C − A ) 2 2
1 1 5 3
= − sin (B + 2C ) = …(i) = cos θ + sin θ
2 2 2
1 Q The maximum value of a cos θ + b sin θ is a 2 + b2
⇒ sin (2 A + 60° ) = sin (C − A ) = − sin (60° + 2C ) =
2 1 5 3
⇒ 2 A + 60° = 30° , 150° [neglecting 30°, as not possible] So, maximum value of cos θ + sin θ is
2 2
⇒ 2 A + 60° = 150° ⇒ A = 45° 2
Again, from Eq. (i),  1
2
5 3 1 75 76
=   +  = + = = 19.
sin (60° + 2C ) = − 1 / 2  2  2  4 4 4
⇒ 60° + 2C = 210° , 330° π
⇒ C = 75° or 135° 2. As when θ ∈ 0,  , tan θ < cot θ
 4
Also, from Eq. (i),
sin (C − A ) = 1 / 2 Since, tan θ < 1 and cot θ > 1
⇒ C − A = 30° , 150° ∴ (tan θ )cot θ < 1 and (cot θ )tan θ > 1
For A = 45° , C = 75° ∴ t4 > t1 which only holds in (b).
and C = 135° [not possible] Therefore, (b) is the answer.
∴ C = 75° π
Hence, A = 45° , B = 60° , C = 75° 3. Since, A + B = and, we know product of term is
3
7. LHS = sin 2 α + sin 2 β − sin 2 γ = sin 2 α + (sin 2 β − sin 2 γ ) maximum, when values are equal.
= sin 2 α + sin (β + γ ) sin (β − γ ) ∴(tan A ⋅ tan B) is maximum.
= sin 2 α + sin (π − α )sin (β − γ ) [Qα + β + γ = π] When A = B = π /6
= sin 2 α + sin α sin (β − γ ) π π 1
i.e. y = tan tan =
= sin α [sin α + sin (β − γ )] 6 6 3
= sin α [sin (π − (β + γ )) + sin (β − γ )] sin x cos 3x tan x
= sin α [sin (β + γ ) + sin (β − γ )] 4. Let y = =
sin 3x cos x tan 3x
= sin α [2 sin β cos γ ] = 2 sin α sin β cos γ = RHS
tan x tan x (1 − 3 tan 2 x)
⇒ y= =
8. Since, A + B = 180° − C tan 3x 3 tan x − tan3 x
∴ tan( A + B) = tan(180°− C )
tan A + tan B 1 − 3 tan 2 x
⇒ = − tan C = [Q x ≠ 0]
1 − tan A tan B 3 − tan 2 x
496 Trigonometrical Ratios and Identities

Put tan x = t 2. Given a ladder of length l = 2m leans against a vertical


1 − 3t 2 wall. Now, the top of ladder begins to slide down the
⇒ y=
3 − t2 wall at the rate 25 cm/s.
Let the rate at which bottom of the ladder slides away
⇒ 3 y − t 2 y = 1 − 3t 2 dx
from the wall on the horizontal ground is cm /s.
⇒ 3 y − 1 = t 2y − 3t 2 dt
wall
⇒ 3 y − 1 = t 2 ( y − 3)
l
y Ladder
+ – +

1/3 3 x
ground
3y − 1 3y − 1
⇒ = t2 ⇒ >0 Q x2 + y 2 = l 2
y−3 y−3
[by Pythagoras theorem]
∴ t2 > 0
NOTE It is a brilliant technique to convert equation into inequation ⇒ x + y =4
2 2
[Ql = 2m]… (i)
and asked in IIT papers frequently. On differentiating both sides of Eq. (i) w.r.t. ‘t’, we get
⇒ y < 1 / 3 or y > 3. This shows that y cannot lie between dx dy
1 / 3 and 3. 2x + 2y =0
dt dt
dx  y dy
Topic 4 Height & Distance ⇒ = −  … (ii)
dt  x  dt
1. According to the question, we have the following figure. From Eq. (i), when y = 1m, then
T x2 + 1 2 = 4 ⇒ x2 = 3 ⇒ x = 3 m [Q x > 0]
On substituting x = 3m and y = 1m in Eq. (ii), we get
xm dx 1  25   dy 
=− −  m /s given = − 25 cm /sec
dt 3  100  dt 
30° B
25
P = cm /s
30 m
3

45° 3. Given ABC is a triangular park with AB = AC = 100 m.


S ym A A vertical tower is situated at the mid-point of BC.
Let the height of the tower is h m.
Now, let distance of foot of the tower from the point A is
Now, according to given information, we have the
y m.
following figure.
Draw BP ⊥ ST such that PT = x m. Q
Then, in ∆TPB, we have
x 1 C
tan 30° = ⇒ x = y …(i)
y 3
x + 30 h
and in ∆TSA, we have tan 45° =
y
⇒ y = x + 30 …(ii) 100 P
On the elimination of quantity x from Eqs. (i) and (ii), we l α
get β
1 B
y= y + 30 1 00
3 A
 1
⇒ y 1 −  = 30 From the figure and given information, we have
 3
β = cot−1 (3 2 ) and α = cosec−1 (2 2 )
30 3 30 3 ( 3 + 1) Now, in ∆QPA,
⇒ y= =
3 −1 3 −1 l
30 cot β =
= 3 ( 3 + 1) = 15 (3 + 3 ) h
2
⇒ l = (3 2 )h …(i)
Trigonometrical Ratios and Identities 497

and in ∆BPQ, tan α =


h and MN = hm is the height of intersection point from
BP the horizontal plane
h 80
BP (100)2 − l2 Q tan α = = [in ∆MNB and ∆PQB] …(i)
⇒ cot α = = x x+ y
h h
[Q p is mid-point of isosceles ∆ABC, AP ⊥ BC] h 20
and tan β = =
y x+ y
⇒ h 2 cot2 α = (100)2 − l2
⇒ h (cosec2α − 1) = (100)2 − (3 2h )2
2
[from Eq. (i)] [in ∆MNQ and ∆ABQ] …(ii)
⇒ h 2(8 − 1) = (100)2 − 18h 2 From Eqs. (i) and (ii), we get
y
⇒ 25h 2 = (100)2 = 4 ⇒ y = 4x …(iii)
2 x
 100
⇒ h2 =   ⇒ h = 20 m From Eqs. (i) and (iii), we get
 5 
h 80 80
= ⇒ h= = 16 m
4. Given heights of two poles are 5 m and 10 m. x x + 4x 5
A
6. According to given information, we have the following
figure,
5m
15° Q Cloud
B E
10 m d xm
30° M
5m P 60°
25 m 25 m
15° Surface
F y
C d D
(25 +x)m
i.e. from figure AC = 10 m, DE = 5 m
∴ AB = AC − DE = 10 − 5 = 5 m R Image of cloud
Let d be the distance between two poles.
Clearly, ∆ABE ~ ∆ACF x
In ∆PQM , tan 30° = …(i)
[by AA- similarity criterion] y
∴ ∠AEB = 15° 25 + (25 + x)
In ∆PRM, tan 60° = …(ii)
In ∆ABE, we have y
AB 3 −1 5  3 − 1 On eliminating ‘y’ from Eqs. (i) and (ii), we get
tan 15° = ⇒ = Q tan 15° =  25 + (25 + x)
BE 3+1 d  3 + 1 3=
3x
5( 3 + 1)
⇒ d= ⇒ 3x = 50 + x ⇒ 2x = 50 ⇒ x = 25 m.
( 3 − 1)
∴ Height of cloud from surface = x + 25 = 50 m.
3+1 3 + 1 5(3 + 2 3 + 1)
⇒ d =5 × = 7. According to given information, we have the following
3 −1 3+1 3 −1
figure.
5(2 3 + 4) 2 × 5( 3 + 2) E
= = = 5(2 + 3 ) m A
2 2
5. Let a first pole AB having height 20 m and second pole
PQ having height 80 m
c=7

b=6
and ∠PBQ = α, ∠AQB = β D
P
30°

B a=5 C
80 m 1
Clearly, length of BD =
2a 2 + 2c2 − b2,
A
2
M (using Appollonius theorem)
20 m where, c = AB = 7, a = BC = 5
h
α β and b = CA = 6
B Q 1
x N y ∴ BD = 2 × 25 + 2 × 49 − 36
2
498 Trigonometrical Ratios and Identities

1 1 AB
= 112 = 4 7 = 2 7 Now, in ∆ABP, tan (α + β ) =
2 2 AP
Now, let ED = h be the height of the lamp post. h 1
= =
E 2h 2
h
AC 1
h Also, in ∆ACP, tan α = = 2 =
AP 2h 4
30° Now, tan β = tan[(α + β ) − α ]
B D
tan(α + β ) − tan α
h =
Then, in ∆BDE , tan 30° = 1 + tan(α + β ) tan α
BD
1 1 1
1 h −
⇒ = 2 4 2
3 2 7 = =4 =
1 1 9 9
2 7 2 1+ ×
⇒ h= = 21 2 4 8
3 3
10. According to given information, we have the following
8. P figure
Now, from ∆ACD and ∆BCD, we have

45° 200 m D
200 m
T
Pillar h
30° 90° 30°
Q M R
30º 60º
Let height of tower TM be h. A x B y C
TM
In ∆PMT , tan 45° =
PM h
tan 30° =
h x+ y
⇒ 1= ⇒ PM = h
PM h
h and tan 60° =
In ∆TQM, tan 30° = ; QM = 3h y
QM
x+ y
In ∆PMQ, PM 2 + QM 2 = PQ 2 ⇒ h= ...(i)
3
h 2 + ( 3h )2 = ( 200)2
and h= 3 y ...(ii)
⇒ 4h 2 = ( 200)2 ⇒ h = 100 m
x+ y
From Eqs. (i) and (ii), = 3 y
9. Let AB = h, then AP = 2h 3
h ⇒ x +y = 3y
and AC = BC =
2 ⇒ x − 2y = 0
Again, let ∠CPA = α ⇒ y=
x
B 2
h/2 Q Speed is uniform
h C and distance x covered in 10 min.
h/2 x
β ∴ Distance will be cover in 5 min.
α 2
A P
2h ∴ Distance y will be cover in 5 min.
21
Trigonometrical Equations
Topic 1 General Solution
Objective Questions I (Only one correct option) π
8. If 0 ≤ x < , then the number of values of x for which
2
1. If the equation cos 4 θ + sin 4 θ + λ = 0 has real solutions
sin x − sin 2x + sin 3x = 0, is (2019 Main, 9 Jan II)
for θ, then λ lies in the interval (2020 Main, 2 Sep II)
(a) 2 (b) 3 (c) 1 (d) 4
(a)  − , − 1 (b)  − 1, − 
5 1
 4   2  9. If sum of all the solutions of the equation
 1  π  π  1

(d) − , − 
1 3 5 8 cos x ⋅  cos + x ⋅ cos − x −  = 1
(c)  − , −
 2 4   2 4   6  6  2
2. Let S be the set of all α ∈ R such that the equation, in [0, π ] is kπ, then k is equal to (2018 Main)
cos 2x + α sin x = 2α − 7 has a solution. Then, S is equal (a)
2
(b)
13
(c)
8
(d)
20
to (2019 Main, 12 April II) 3 9 9 9
(a) R (b) [1, 4] (c) [3, 7] (d) [2, 6] 10. If 5 (tan 2 x − cos 2 x) = 2 cos 2x + 9, then the value of
3. The number of solutions of the equation cos 4x is (2017 Main)
 5π 5π  (a) −
3 1 2
(d) −
7
1 + sin 4 x = cos 2 3x, x ∈ − , is (b) (c)
 2 2  (2019 Main, 12 April I) 5 3 9 9
(a) 3 (b) 5 (c) 7 (d) 4 11. If 0 ≤ x < 2π, then the number of real values of x, which
satisfy the equation
4. Let S = {θ ∈ [−2π , 2π ] : 2 cos 2 θ + 3 sin θ = 0}, then the
cos x + cos 2x + cos 3x + cos 4x = 0, is (2016 Main)
sum of the elements of S is (2019 Main, 9 April I)
5π 13 π (a) 3 (b) 5 (c) 7 (d) 9
(b) π π
12. Let S = x ∈ (− π , π ): x =/ 0, ± . The sum of all
(a) 2π (c) (d)
3 6
 2
5. If sin 4 α + 4 cos 4 β + 2 = 4 2 sin α cos β; distinct solutions of the equation 3 sec x + cosec x
α, β ∈ [0, π ], then cos(α + β ) − cos(α − β ) is equal to + 2(tan x − cot x) = 0 in the set S is equal to (2016 Adv.)
(2019 Main, 12 Jan II) 7π 2π 5π
(a) − (b) − (c) 0 (d)
(a) − 1 (b) 2 (c) − 2 (d) 0 9 9 9
6. Let α and β be the roots of the quadratic equation 13. If P = {θ :sin θ − cos θ = 2 cos θ } and
x2 sin θ − x(sin θ cos θ + 1) + cos θ = 0 (0 < θ < 45º ) and Q = {θ :sin θ + cos θ = 2 sin θ } be two sets. Then, (2011)

 (− 1)n  (a) P ⊂ Q and Q − P ≠ φ (b) Q ⊄ P
α < β. Then, ∑ α n +  is equal to
n = 0 βn  (2019 Main, 11 Jan II)
(c) P ⊄ Q (d) P = Q
n

∑ br sin r θ, for every


1 1 1 1
(a) − (b) + 14. Let n be an odd integer. If sin nθ =
1 − cosθ 1 + sin θ 1 − cosθ 1 + sin θ value of θ, then r=0 (1998, 2M)
1 1 1 1
(c) − (d) + (a) b0 = 1, b1 = 3 (b) b0 = 0, b1 = n
1 + cosθ 1 − sin θ 1 + cosθ 1 − sin θ
(c) b0 = − 1, b1 = n (d) b0 = 0, b1 = n 2 − 3n + 3
π
7. The sum of all values of θ ∈ 0,  satisfying 15. The general value of θ satisfying the equation
 2 2 sin 2 θ − 3 sin θ − 2 = 0, is (1995,2M)
3 π π
sin 2 2θ + cos 4 2θ = is (a) nπ + (−1) n
(b) nπ + (−1) n

4 (2019 Main, 10 Jan I) 6 2


3π 5π π n 5π n 7π
(a) (b) (c) (d) π (c) nπ + (−1) (d) nπ + (−1)
8 4 2 6 6
500 Trigonometrical Equations

16. In a ∆ ABC , angle A is greater than angle B. If the 23. The number of distinct solutions of the equation
5
measures of angles A and B satisfy the equation cos2 2x + cos4 x + sin4 x + cos6 x + sin6 x = 2 in the
3 sinx − 4 sin3 x − k = 0, 0 < k < 1, then the measure of ∠ C 4
is (1990, 2M) interval [0, 2π ] is (2015 Adv.)
π π 2π 5π
(a) (b) (c) (d)
3 2 3 6 Match the Columns
17. The general solution of 24. Let f (x) = sin(π cos x) and g (x) = cos(2π sin x) be two
sin x − 3 sin 2x + sin 3x = cos x − 3 cos 2x + cos 3x is functions defined for x > 0. Define the following sets
π nπ π
(a) nπ + (b) + (1989, 2M) whose elements are written in the increasing order :
8 2 8
n nπ π 3 X = { x : f (x) = 0}, Y = { x : f ′ (x) = 0}
(c) (−1) + (d) 2nπ + cos−1
2 8 2 Z = { x : g (x) = 0}, W = { x : g′ (x) = 0}
18. The general solution of the trigonometric equation List-I contains the sets X , Y , Z and W . List-II contains
sin x + cos x = 1 is given by (1981, 2M) some information regarding these sets. (2019 Adv.)
(a) x = 2nπ ; n = 0, ± 1, ± 2, ...
List-I List-II
(b) x = 2 nπ + π / 2 ; n = 0, ± 1, ± 2, ....
π π  π 3π 
(c) x = nπ + (− 1)n − ; n = 0, ± 1, ± 2 , ... (I) X (P) ⊇ , , 4π , 7π 
4 4 2 2 
(d ) None of the above
(II) Y (Q) an arithmetic progression
π
19. The equation 2 cos 2   sin 2 x = x2 + x−2, x ≤ has
x
 2 9 (III) Z (R) NOT an arithmetic progression
(1980, 1M)
 π 7π 13π 
(a) no real solution (IV) W (S) ⊇ , , 
6 6 6 
(b) one real solution
(c) more than one real solution  π 2π 
(T) ⊇ , , π
(d) None of the above 3 3 

Objective Questions II  π 3π 
(U) ⊇ , 
6 4 
(One or more than one correct option)
20. Let α and β be non zero real numbers such that Which of the following is the only CORRECT combination?
2(cos β − cos α ) + cos α cos β = 1. Then which of the (a) (IV), (P), (R), (S)
following is/are true? (2017 Adv.) (b) (III), (P), (Q), (U)
α β
(a) 3 tan   − tan   = 0
(c) (III), (R), (U)
 2  2 (d) (IV), (Q), (T)
α β
(b) tan   − 3 tan   = 0 25. Let f (x) = sin(π cos x) and g (x) = cos(2π sin x) be two
 2  2
functions defined for x > 0. Define the following sets
α β
(c) tan   + 3 tan   = 0 whose elements are written in the increasing order :
 2  2
X = { x : f (x) = 0}, Y = { x : f ′ (x) = 0}
α β
(d) 3 tan   + tan   = 0 Z = { x : g (x) = 0}, W = { x : g′ (x) = 0}
 2  2
21. The values of θ lying between θ = 0 and θ = π /2 and List-I contains the sets X , Y , Z and W . List-II contains
some information regarding these sets. (2019 Adv.)
satisfying the equation
1 + sin 2 θ cos 2 θ 4 sin 4 θ List-I List-II
sin θ
2
1 + cos θ2
4 sin 4θ = 0, is  π 3π 
(I) X (P) ⊇ , , 4π , 7π 
sin 2 θ cos 2 θ 1 + 4 sin 4θ  2 2 
(1988, 3M)
(II) Y (Q) an arithmetic progression
(a) 7 π / 24 (b) 5 π / 24 (c) 11π / 24 (d) π / 24
(III) Z (R) NOT an arithmetic progression
Integer & Numerical Answer Type Questions  π 7π 13π 
(IV) W (S) ⊇ , , 
22. Let a , b, c be three non-zero real numbers such that the 6 6 6 
 π π
equation 3 a cos x + 2b sin x = c, x ∈ − , , has two  π 2π 
 2 2  (T) ⊇ , , π
π 3 3 
distinct real roots α and β with α + β = . Then, the
3  π 3π 
b (U) ⊇ , 
value of is .................. . (2018 Adv.) 6 4 
a
Trigonometrical Equations 501

Which of the following is the only CORRECT 29. If exp {(sin 2 x + sin 4 x + sin 6 x + ... ∞ ) log e 2}, satisfies
combination? the equation x2 − 9x + 8 = 0, find the value of
(a) (II), (Q), (T) cos n x π
(b) (II), (R), (S) ,0 < x < . (1991, 4M)
cos x + sin x 2
(c) (I), (P), (R)
(d) (I), (Q), (U) 30. Consider the system of linear equations in x, y, z
(sin 3 θ ) x − y + z = 0 ,
Fill in the Blank
(cos 2 θ ) x + 4 y + 3z = 0,
26. General value of θ satisfying the equation
2x + 7 y + 7z = 0
tan 2 θ + sec 2 θ = 1 is…… . (1996, 1M)
Find the values of θ for which this system has
True/False non-trivial solutions. (1986, 4M)

27. There exists a value of θ between 0 and 2π that satisfies 31. Find the values of x (− π , π ) which satisfy the equation
2
the equation sin θ − 2 sin θ + 1 = 0.
4 2
(1984, 1M) 21 + | cos x | + | cos x | + ...
=4 (1984, 2M)

32. Find all the solutions of 4cos x sin x − 2 sin x = 3 sin x.


2 2
Analytical & Descriptive Questions
(1983, 2M)
28. Determine the smallest positive value of x ( in degrees)
for which 33. Solve 2 (cos x + cos 2x) + (1 + 2 cos x)sin 2x
tan (x + 100° ) = tan (x + 50° ) tan (x) tan (x − 50° ). = 2 sin x, − π ≤ x ≤ π (1978, 3M)
(1993, 5M)

Topic 2 Solving Equations with Graph


Objective Question I (Only one correct option) (b) xn + 1 − xn > 2 for every n
1. All x satisfying the inequality (c) x1 < y1
(d) xn ∈  2n , 2n +  for every n
1
(cot− 1 x)2 − 7(cot− 1 x) + 10 > 0, lie in the interval
(2019 Main, 11 Jan II)  2
(a) (− ∞ , cot 5) ∪ (cot 2, ∞ ) (b) (cot 5, cot 4) 4. Let θ, φ ∈ [0, 2π ] be such that 2 cos θ (1 − sin φ ) = sin 2 θ
(c) (cot 2, ∞ )
 θ θ
(d) (− ∞ , cot 5) ∪ (cot 4, cot 2)  tan + cot  cos φ − 1, tan (2π − θ ) > 0
 2 2
2. The set of values of θ satisfying the inequation
3
2 sin 2 θ − 5 sin θ + 2 > 0, where 0 < θ < 2π , is (2006, 3M) and − 1 < sin θ < − . Then, φ cannot satisfy (2012)
2
π 5π π 5π
(a)  0,  ∪  , 2 π  (b)  0,  ∪  , 2π π
 6  6   6   6  (a) 0 < φ <
2
π 2π
(c)  0,  ∪  , 2π (d) None of these (b)
π
< φ<

 3   3  2 3
4π 3π
(c) < φ<
Objective Question II 3 2

(One or more than one correct option) (d) < φ < 2π
2
sin πx
3. Let, f (x) = , x>0
x2 Analytical & Descriptive Question
Let x1 < x2 < x3 < … < xn < … be all the points of local π π
maximum of f and y1 < y2 < y3 < … < yn < … be all the 5. Find all values of θ in the interval  − ,  satisfying
 2 2
points of local minimum of f. 2
θ
Then which of the following options is/are correct? the equation (1 − tan θ )(1 + tan θ )sec2 θ + 2tan = 0.
(a)|xn − yn|> 1for every n (2019 Adv.) (1996, 2M)
502 Trigonometrical Equations

Topic 3 Problems Based on Maximum and Minimum


Objective Questions I (Only one correct option) (a) xyz = xz + y
(b) xyz = xy + z
1. For x ∈ (0, π ), the equation sin x + 2 sin 2x − sin 3x = 3 has
(c) xyz = x + y + z
(a) infinitely many solutions (2014 Adv.) (d) xyz = yz + x
(b) three solutions
(c) one solution Integer & Numerical Answer Type Questions
(d) no solution
12. Let f : [0, 2] → R be the function defined by
2. The number of solutions of the pair of equations
2 sin θ − cos 2θ = 0 and
2
2 cos θ − 3 sin θ = 0
2
in the  π  π
f (x) = (3 − sin(2πx))sin  πx −  − sin 3πx +  .
interval [0, 2π] is (2007, 3M)  4  4
(a) 0 (b) 1 (c) 2 (d) 4 If α , β ∈ [0, 2] are such that { x ∈ [0, 2]: f (x) ≥ 0} = [α , β ],
3. The number of integral values of k for which the then the value of β − α is …… (2020 Adv.)
equation 7 cos x + 5 sin x = 2k + 1 has a solution, is
(2002, 1M) 13. The positive integer value of n > 3 satisfying the
(a) 4 (b) 8 (c) 10 (d) 12 1 1 1
equation = + is …… (2011)
4. The number of values of x in the interval [0, 5π ]  π  2π   3π 
sin   sin   sin  
 n  n  n
satisfying the equation 3 sin 2 x − 7 sin x + 2 = 0 is
(1998, 2M)
π π
(a) 0 (b) 5 (c) 6 (d) 10 14. The number of values of θ in the interval  − ,  such
 2 2
5. Number of solutions of the equation nπ
tan x + sec x = 2 cos x lying in the interval [0, 2π ] is that θ ≠ for n = 0, ±1, ± 2 and tan θ = cot 5θ as well as
5
(1993, 1M)
sin 2 θ = cos 4θ is…… (2010)
(a) 0 (b) 1 (c) 2 (d) 3
6. The number of solutions of the equation Fill in the Blanks
sin (ex ) = 5x + 5− x is (1991, 2M)
15. The set of all x in the interval [0, π ] for which
(a) 0 (b) 1
(c) 2 (d) infinitely many 2 sin 2 x − 3 sin x + 1 ≥ 0, is…… . (1987, 2M)

7. The equation (cos p − 1) x + (cos p) x + sin p = 0 in the


2
16. The solution set of the system of equations
variable x, has real roots. Then, p can take any value in 2π 3
the interval (1990, 2M) x+ y= , cos x + cos y = , where x and y are real,
3 2
π π
(a) (0, 2 π) (b) ( − π , 0) (c)  − ,  (d) (0, π ) is…… .
 2 2
−π π
8. The smallest positive root of the equation tan x − x = 0 17. The larger of cos (log θ ) and log (cos θ ) if <θ < ,
2 2
lies in (1987, 2M) is …… . (1983, 1M)
π π 3π 3π
(a)  0,  (b)  , π  (c)  π ,  (d)  , 2 π 
 2 2   2   2 
Analytical & Descriptive Questions
9. The number of all possible triplets (a1 , a 2, a3 ) such that
18. Find the smallest positive number p for which the
a1 + a 2 cos (2x) + a3 sin 2 (x) = 0, ∀ x is (1987, 2M)
equation cos ( p sin x) = sin ( p cos x) has a solution
(a) 0 (b)1 (c) 3 (d) ∞
x ∈[0, 2π ] . (1995, 5M)
Objective Questions II 19. Show that the equation e sin x − e – sin x − 4 = 0 has no real
(One or more than one correct option) solution. (1982, 2M)
4xy
10. sec θ =2
is true if and only if (1996, 1M) 20. Find the coordinates of the points of intersection of the
(x + y)2 π π
curves y = cos x, y = sin 3x, if − ≤x≤ .
(a) x = y ≠ 0 (b) x = y, x ≠ 0 2 2 (1982, 3M)
(c) x = y (d) x ≠ 0, y ≠ 0 21. For all θ in [0, π / 2], show that cos (sin θ ) ≥ sin(cos θ ).
∞ ∞

∑ cos ∑ sin
(1981, 4M)
11. For 0 < φ < π / 2, if x = 2n
φ, y = 2n
φ,
π
n=0 n=0 22. Prove that 5 cosθ + 3 cos θ +  + 3 lies between − 4 and
∞  3
z= ∑ cos 2n
φ sin 2n
φ , then (1993, 2M) 10. (1979, 3M)
n=0
Trigonometrical Equations 503

Answers
Topic 1 π π π
33. x = − π , − ,− , ,π
1. (b) 2. (d) 3. (b) 4. (a) 2 3 3
5. (c) 6. (b) 7. (c) 8. (a) Topic 2
9. (b) 10. (d) 11. (c) 12. (c) 1. (c) 2. (a) 3. (a,b,d)
13. (d) 14. (b) 15. (d) 16. (c) 4. (a,c,d) 5. θ = ± π / 3
17. (b) 18. (c) 19. (a) 20. (b, c)
21. (a, c) 22. (0.5) 23. (8) 24. (a) Topic 3
π 1. (d) 2. (c) 3. (b) 4. (c)
25. (a) 26. θ = mπ , nπ ± 27. False
3 5. (c) 6. (a) 7. (d) 8. (c)
3 −1 9. (d) 10. (a, b) 11. (b, c) 12. (1)
28. x = 30 ° 29.
2 13. (7) 14. (3)
n π  π 2π 
30. θ = nπ or nπ + ( −1 )   31. ± , ±   π  π  5π 
 6  3 3  15. x ∈ 0, ∪  ∪ , π 16. No solution
 6   2   6 
 π π
32. { x : x = n π } ∪ x : x = n π + ( −1 )n  17. cos (log θ ) 18. Smallest positive value of p =
 10  2 2
  − 3π  
∪ x : x = n π + ( − 1 )n   π π  π π   3π 3π 
  10   20.  , cos   , cos   − , cos 
8 8  4 4  8 8

Hints & Solutions


Topic 1 General Solution Now, as we know −1 ≤ sin x ≤ 1
α −4
1. The expression, cos θ + sin θ4 4
∴ −1 ≤ ≤1 [from Eq. (i)]
2
= (cos θ + sin θ ) − 2 sin θ cos θ
2 2 2 2 2
⇒ − 2 ≤ α − 4 ≤ 2 ⇒ 2 ≤ α ≤ 6 ⇒ α ∈ [2, 6]
1
= 1 − sin 2(2 θ ) 3. Given equation is 1 + sin 4 x = cos 2(3x)
2
Q sin 2(2 θ) ∈ [0, 1] Since, range of (1 + sin 4 x) = [1, 2]
1  1  and range of cos 2(3x) = [0, 1]
⇒ − sin 2(2 θ) ∈ − , 0
2  2 
So, the given equation holds if
1 1 
⇒ 1 − sin 2(2 θ ) ∈ , 1 1 + sin 4 x = 1 = cos 2(3x)
2 2 
⇒ sin 4 x = 0 and cos 2 3x = 1
Now, as cos 4 θ + sin 4 θ + λ = 0
 5π 5π 
⇒ λ = − (cos 4 θ + sin 4 θ) Since, x ∈ − ,
 2 2 
⇒ λ ∈[−1, − 1 / 2] for real solution of the given equation
cos 4 θ + sin 4 θ + λ = 0 for θ. ∴ x = − 2π, − π, 0, π, 2 π.
Hence, option (b) is correct. Thus, there are five different values of x is possible.
2. The given trigonometric equation is 4. We have, θ ∈ [−2π , 2π ]
cos 2x + α sin x = 2α − 7
and 2 cos 2 θ + 3 sin θ = 0
⇒ 1 − 2 sin 2x + α sin x = 2α − 7
⇒ 2 (1 − sin 2 θ ) + 3 sin θ = 0
[Q cos 2x = 1 − 2 sin 2 x]
⇒ 2 sin 2 x − α sin x + 2α − 8 = 0 ⇒ 2 − 2 sin 2 θ + 3 sin θ = 0

⇒ 2(sin 2 x − 4) − α (sin x − 2) = 0 ⇒ 2 sin 2 θ − 3 sin θ − 2 = 0


⇒ 2 (sin x − 2) (sin x + 2) − α (sin x − 2) = 0 ⇒ 2 sin 2 θ − 4 sin θ + sin θ − 2 = 0
⇒ (sin x − 2) (2 sin x + 4 − α ) = 0 ⇒ 2 sin θ (sin θ − 2) + 1(sin θ − 2) = 0
∴ 2 sin x + 4 − α = 0 [Q sin x − 2 ≠ 0] ⇒ (sin θ − 2) (2 sin θ + 1) = 0
α −4 −1
⇒ sin x = …(i) ∴ sin θ = [Q(sin θ − 2) ≠ 0]
2 2
504 Trigonometrical Equations

π π π π 1
∴ θ = 2π –
6
,−π + ,− ,π +
6 6 6
[Q θ ∈ [−2π , 2π ]] ⇒ 2 cos 2 2 θ − 1 = 0 ⇒ cos 2 2 θ =
2
Now, sum of all solutions 1
⇒ cos 2 θ = ±
π π π π 2
= 2π − − π + − + π + = 2π  π
6 6 6 6 If θ ∈ 0,  , then 2 θ ∈ (0, π )
 2
5. By applying AM ≥ GM inequality, on the numbers 1 π 3π
∴ cos 2 θ = ± ⇒ 2θ = , ,
sin 4 α , 4 cos 4β , 1 and 1, we get 2 4 4
sin 4 α + 4 cos 4 β + 2   3π   π π 1 
≥ ((sin 4 α ) (4 cos 4 β ) ⋅ 1 ⋅ 1)1/ 4
4 Q cos  4  = cos  π − 4  = − cos 4 = − 2 
 
⇒ sin 4 α + 4 cos 4 β + 2 ≥ 4 2 sin α cos β π 3π
⇒ θ= ,
But, it is given that 8 8
π 3π π
sin 4 α + 4 cos 4 β + 2 = 4 2 sin α cos β Sum of values of θ = + =
8 8 2
So, sin 4 α = 4 cos 4 β = 1
[Q In AM ≥ GM , equality holds when all given 8. We have, sin x − sin 2x + sin 3x = 0
positive quantities are equal.] ⇒ (sin x + sin 3x) − sin 2x = 0
1  x + 3x  x − 3x
⇒ sin α = 1 and sin β = …(i) ⇒ 2 sin   cos   − sin 2x = 0
2  2   2 
[Q α , β ∈ [0, π ]]  C + D  C − D
[Qsin C + sin D = 2 sin   cos  ]
 2   2 
Now, cos (α + β ) − cos (α − β ) = −2 sin α sin β
⇒ 2 sin 2x cos x − sin 2x = 0 [Qcos (− θ) = cos θ]
 C+D D −C 
Q cos C − cos D = 2 sin sin ⇒ sin 2x(2 cos x − 1) = 0
 2 2  ⇒ sin 2x = 0 or 2 cos x − 1 = 0
1 1
= −2 × 1 × [from Eq. (i)] ⇒ 2x = 0, π, ... or cos x =
2 2
=− 2 π π
⇒ x = 0, ... or x =
2 3
6. Given,
 π
x2 sin θ − x sin θ cos θ − x + cos θ = 0, In the interval 0,  only two values satisfy, namely
 2 
where 0 < θ < 45° π
x = 0 and x = .
⇒ x sin θ (x − cos θ ) − 1(x − cos θ ) = 0 3
⇒ (x − cos θ ) (x sin θ − 1) = 0 9. Key idea Apply the identity
⇒ x = cos θ , x = cosec θ cos(x + y) cos(x − y) = cos 2 x − sin 2 y
⇒ α = cos θ and β = cosec θ and cos 3x = 4 cos3 x − 3 cos x
1  π  π  1
(Q For 0 < θ < 45° , < cos θ < 1 and 2 < cosecθ < ∞ We have, 8 cos x cos  + x cos  − x −  = 1
 6  6  2
2
⇒ cos θ < cos ecθ)  π 1 
⇒ 8 cos x cos 2 − sin 2 x −  = 1

 (−1)n  ∞ ∞
(−1)n  6 2
Now, consider, ∑ α n +  = ∑ α n
+ ∑ n 3 1
n = 0 βn  n = 0 n=0 β ⇒ 8 cos x − sin 2 x −  = 1
4 2
= (1 + α + α 2 + α 3 + .... ∞ )  3 1 
⇒ 8 cos x − − 1 + cos 2 x = 1
 1 1 1  4 2 
+ 1 − + 2 − 3 + .... ∞
 β β β   −3 + 4 cos x
2
⇒ 8 cos x  =1
1 1 1 1  4 
= + = +
1 −α   1 −α 1 + 1
1 ⇒ 2(4 cos3 x − 3 cos x) = 1
1 − − 
 β β 1
⇒ 2 cos 3x = 1 ⇒ cos 3x =
1 1  1  2
= + Q = sin θ  π 5π 7π
1 − cos θ 1 + sin θ  β  ⇒ 3x = , , [0 ≤ 3x ≤ 3π]
3 3 3
3 π 5π 7π
7. Given, sin 2 2 θ + cos 4 2 θ = ⇒ x= , ,
4 9 9 9
3 π 5π 7π 13π 13π
⇒ (1 − cos 2 θ) + cos 2 θ =
2 4
(Q sin 2 x = 1 − cos 2 x) Sum = + + = ⇒ kπ =
4 9 9 9 9 9
⇒ 4 cos 4 2 θ − 4 cos 2 2 θ + 1 = 0 Hence, k=
13
⇒ (2 cos 2 2 θ − 1)2 = 0 9
Trigonometrical Equations 505

10. Given, 5 (tan 2 x − cos 2 x) = 2 cos 2x + 9 ⇒ x=π [Q 0 ≤ x < 2π]


π 3π π 3π 7π 9π
 2 sin 2 x  Hence, x= , , π, , , ,
⇒ 5  − cos 2 x = 2 cos 2x + 9 2 2 5 5 5 5
 2 cos x
2

12. Given, 3 sec x + cosec x + 2(tan x − cot x) = 0,
 1 − cos 2x 1 + cos 2x
⇒ 5 −  = 2 cos 2x + 9
 1 + cos 2x 2  (− π < x < π ) − {0, ± π / 2}
Put cos 2x = y, we have ⇒ 3 sin x + cos x + 2 (sin 2 x − cos 2 x) = 0
 1 − y 1 + y ⇒ 3 sin x + cos x − 2 cos 2x = 0
5 −  = 2y + 9
1 + y 2  Multiplying and dividing by a 2 + b2, i.e. 3 + 1 = 2 ,
we get
⇒ 5 (2 − 2 y − 1 − y2 − 2 y) = 2(1 + y)(2 y + 9)  3 
1
⇒ 5(1 − 4 y − y2) = 2(2 y + 9 + 2 y2 + 9 y) 2 sin x + cos x − 2 cos 2x = 0
 2 2 
⇒ 5 − 20 y − 5 y2 = 22 y + 18 + 4 y2 π π

⇒ 9 y2 + 42 y + 13 = 0 ⇒  cos x ⋅ cos + sin x ⋅ sin  − cos 2x = 0
 3 3
⇒ 9 y2 + 3 y + 39 y + 13 = 0  π
⇒ cos  x −  = cos 2x
⇒ 3 y (3 y + 1) + 13(3 y + 1) = 0  3
⇒ (3 y + 1)(3 y + 13) = 0  π since, cos θ = cos α 
∴ 2 x = 2 nπ ±  x − 
1 13  3  ⇒ θ = 2 nπ ± α 
⇒ y=− ,−
3 3 π
⇒ 2 x = 2 nπ + x −
1 13 3
∴ cos 2x = − , −
3 3 π
or 2 x = 2 nπ − x +
1  13  3
∴ cos 2x = − Q cos 2x ≠ −
3  3  π
⇒ x = 2 nπ −
Now, cos 4x = 2 cos 2 2x − 1 3
2 π
 1 2 7 or 3 x = 2 nπ +
= 2 −  − 1 = − 1 = − 3
 3 9 9
π
⇒ x = 2 nπ −
11. Given equation is cos x + cos 2x + cos 3x + cos 4x = 0 3
⇒ (cos x + cos 3x) + (cos 2x + cos 4x) = 0 2 nπ π
or x= +
⇒ 2 cos 2x cos x + 2 cos 3x cos x = 0 3 9
⇒ 2 cos x (cos 2x + cos 3x) = 0 −π
∴ x=
 5x x 3
⇒ 2 cos x 2 cos cos  = 0
 2 2 π −5 π 7 π
or x= , ,
5x x 9 9 9
⇒ cos x ⋅ cos ⋅ cos = 0
2 2 Now, sum of all distinct solutions
⇒ cos x = 0 −π π 5π 7π
5x = + − + =0
or cos =0 3 9 9 9
2
x 13. P = {θ : sin θ − cos θ = 2 cos θ }
or cos = 0
2 ⇒ cos θ ( 2 + 1) = sin θ
Now, cos x = 0 ⇒ tan θ = 2 + 1
π 3π ⇒ Q = {θ : sin θ + cos θ } = 2 sin θ
⇒ x= , [Q 0 ≤ x < 2π]
2 2 ⇒ sin θ ( 2 − 1) = cos θ
5x 1 2 +1
cos =0 ⇒ tan θ = × = ( 2 + 1)
2 2 −1 2 +1
5x π 3π 5π 7π 9π 11π
⇒ = , , , , ..., ∴ P =Q
2 2 2 2 2 2 2 n
π 3π 7π 9π
⇒ x= , , π, , [Q 0 ≤ x < 2π] 14. Given, sin nθ = ∑ br sin r θ
5 5 5 5 r=0
x
and cos = 0 Now, put θ = 0, we get 0 = b0
2 n
x π 3π 5π
⇒ = , , , ... ∴ sin nθ = ∑ br sin r θ
2 2 2 2 r =1
506 Trigonometrical Equations

sin nθ n
18. Given, sin x + cos x = 1
⇒ = ∑ br (sin θ )r − 1
sin θ r =1 On dividing and multiplying each terms by 2, we get
Taking limit as θ → 0 1 1 1
sin x + cos x =
sin n θ n 2 2 2
⇒ lim = lim ∑ br (sin θ )r − 1 π π 1
θ → 0 sin θ θ→ 0
r =1
⇒ sin x cos = cos x sin =
4 4 2
sin n θ  π  π
nθ⋅ ⇒ sin  x +  = sin  
nθ  4  4
⇒ lim = b1 + 0 + 0 + 0 + K
θ→ 0 sin θ π π
θ⋅ ⇒ x + = nπ + (−1)n
θ 4 4
[Q other values becomes zero for higher powers of sin θ ] n π π
⇒ x = nπ + (−1) − , n ∈I
n ⋅1 4 4
⇒ = b1 ⇒ b1 = n
π
19. Given equation is 2 cos 2   sin 2 x = x2 + x−2, x ≤
1 x
 2 9
15. Given, 2 sin θ − 3 sin θ − 2 = 0
2

 x 1
⇒ ( 2 sin θ + 1)(sin θ − 2) = 0 LHS = 2 cos 2  sin 2 x < 2 and RHS = x2 + 2 ≥ 2
 2 x
⇒ sin θ = − 1 / 2
∴ The equation has no real solution.
[neglecting sin θ = 2, as|sin θ| ≤ 1]
∴ θ = nπ + (−1) (7π / 6)
n 20. We have, 2(cos β − cos α ) + cos α cos β = 1
16. Given, 3 sin x − 4 sin3 x = k , 0 < k < 1 which can also be or 4(cos β − cos α ) + 2 cos α cos β = 2
written as sin 3x = k. ⇒ 1 − cos α + cos β − cos α cos β
It is given that A and B are solutions of this equation. = 3 + 3 cos α − 3 cos β − 3 cos α cos β
Therefore, ⇒ (1 − cos α )(1 + cos β ) = 3(1 + cos α )(1 − cos β )
sin 3 A = k and sin 3B = k, where 0 < k < 1 (1 − cos α ) 3(1 − cos β )
⇒ =
⇒ 0 < 3 A < π and 0 < 3B < π (1 + cos α ) 1 + cos β
Now, sin 3 A = k and sin 3B = k α β
⇒ = 3 tan 2
tan 2
⇒ sin 3 A − sin 3B = 0 2 2
3 3 α β
⇒ 2 cos ( A + B) sin ( A − B) = 0 ∴ tan ± 3 tan = 0
2 2 2 2
 A + B  A − B 1 + sin 2 θ cos 2 θ 4 sin 4 θ
⇒ cos 3   = 0, sin 3   =0
 2   2  21. Given, sin θ 1 + cos θ
2 2
4 sin 4 θ =0
But it is given that, A > B and 0 < 3 A < π, 0 < 3B < π. sin 2 θ cos 2 θ 1 + 4 sin 4 θ
 A − B
Therefore, sin 3   ≠0 Applying R3 → R3 − R1 and R2 → R2 − R1, we get
 2 
1 + sin 2 θ cos 2 θ 4 sin 4 θ
 A + B
Hence, cos 3   =0 −1 1 0 =0
 2 
−1 0 1
 A + B π
⇒ 3  =
 2  2 Applying C1 → C1 + C 2, we get
π 2 cos 2 θ 4 sin 4 θ
⇒ A+ B=
3 0 1 0 =0
2π −1 0 1
⇒ C = π − ( A + B) =
3
−1
17. Given, sin 3x + sin x − 3 sin 2x = cos 3x + cos x − 3 cos 2x ⇒ 2 + 4 sin 4 θ = 0 ⇒ sin 4 θ =
2
⇒ 2 sin 2x cos x − 3 sin 2x = 2 cos 2x cos x − 3 cos 2x  π
⇒ 4 θ = nπ + (−1)n  − 
⇒ sin 2x ( 2 cos x − 3) = cos 2x ( 2 cos x − 3)  6
[Q 2 cos x − 3 ≠ 0] nπ π
⇒ θ= + (−1)n+ 1  
⇒ sin 2x = cos 2x 4  24
⇒ tan 2x = 1 7π 11π
Clearly, θ = , are two values of θ lying between
π nπ π 24 24
⇒ 2 x = nπ + ⇒ x= +
4 2 8 0 and π /2.
Trigonometrical Equations 507

22. We have, α , β are the roots of 3 1 1 3


here values of sin x, − , − , , are in an A.P. but
4 4 4 4
3 a cos x + 2b sin x = c
corresponding values of x are not in an AP so, (iii) → R.
∴ 3 a cos α + 2b sin α = c …(i)
For W = { x : g′ (x) = 0}, x > 0
and 3 a cos β + 2b sin β = c …(ii)
so, g′ (x) = − 2π cos x sin(2π sin x) = 0
On subtracting Eq. (ii) from Eq. (i), we get
⇒ either cos x = 0 or sin(2π sin x) = 0
3a (cos α − cos β ) + 2b(sin α − sin β ) = 0 π
  α + β   α − β ⇒ either x = (2n + 1) or 2π sin x = nπ , n ∈Integers.
⇒ 3 a  − 2 sin   sin   2
  2    2 
Q 2π sin x = nπ
  α + β   α − β n 1 1
+ 2b 2 cos   sin   =0 ⇒ sin x = = − 1, − , 0, , 1 {Q sin x ∈ [− 1, 1)}
  2    2  2 2 2
 α + β  α + β π  π
⇒ 3 a sin   = 2b cos   ∴ x = nπ, (2n + 1) or nπ + (− 1)n  ± 
 2   2  2  6
 α + β 2b ⇒ (iv) → P, R, S
⇒ tan   =
 2  3a Hence, option (a) is correct.
 π 2b  π  25. For, X = { x : f (x) = 0}, x > 0
⇒ MM tan   = Q α + β = , given
 6 3a  3 
Now, f (x) = 0
1 2b b 1 ⇒ sin(π cos x) = 0, x > 0
⇒ = ⇒ =
3 3a a 2 ⇒ π cos x = nπ , n ∈ Integer.
b ⇒ cos x = n
⇒ = 0.5
a ⇒ cos x = − 1, 0, 1 {Qcos x ∈ [− 1, 1]}
5
23. Here, cos 2 2x + (cos 4x + sin 4 x) + (cos 6x + sin 6 x) = 2 π
4 ⇒ x = nπ or (2n + 1) , n is an integer. so, (i) → (P), (Q)
2
5
⇒ cot 2x + [(cos 2 x + sin 2 x)2 − 2 sin 2 x cos 2 x] For, Y = { x : f ′ (x) = 0}, x > 0
4
Now, f ′ (x) = 0
+ (cos 2 x + sin 2 x)[(cos 2 x + sin 2 x)2 − 3 sin 2 x cos 2 x] = 2
5 ⇒ − π sin x cos(π cos x) = 0
⇒ cos 2 2x + (1 − 2 sin 2 x cos 2 x) + (1 − 3 cos 2 x sin 2 x) = 2
4 ⇒ either sin x = 0 ⇒ x = nπ, n is an integer, or
5 cos(π cos x) = 0
⇒ cos 2 2x − 5 sin 2 x cos 2 x = 0 π
4 ⇒ π cos x = (2n + 1) , n is an integer
5 5 2
⇒ cos 2 2x − sin 2 2x = 0 2n + 1
4 4 ⇒ cos x =
5 5 5 2
⇒ cos 2 2x − + cos 2 2x = 0 1
4 4 4 ⇒ cos x = ± , {Q cos x ∈ [− 1, 1]}
5 5 1 2
⇒ cos 2x = ⇒ cos 2 2x =
2
π 2π
2 4 2 ⇒ x = 2nπ ± or 2nπ ± , n is an integer.
⇒ 2 cos 2 2x = 1 3 3
⇒ 1 + cos 4x = 1 So, (ii) → (Q), (T)
⇒ cos 4x = 0, as 0 ≤ x ≤ 2π Hence, option (a) is correct.
 π 3π 5π 7π 9π 11π 13π 15π  26. Given, tan 2 θ + sec 2θ = 1
∴ 4x =  , , , , , , , 
2 2 2 2 2 2 2 2 
1
as 0 ≤ 4x ≤ 8π ⇒ tan 2 θ + =1
cos 2θ
 π 3π 5π 7π 9π 11π 13π 15π 
⇒ x= , , , , , , ,  1 + tan 2 θ
8 8 8 8 8 8 8 8  ⇒ tan 2 θ + =1
Hence, the total number of solutions is 8. 1 − tan 2 θ

24. For Z = { x : g (x) = 0}, x > 0 ⇒ tan 2 θ (1 − tan 2 θ ) + (1 + tan 2 θ ) = 1 − tan 2 θ


⇒ 3 tan 2 θ − tan 4 θ = 0
Q g (x) = cos(2π sin x) = 0
π ⇒ tan 2 θ (3 − tan 2 θ ) = 0
⇒ 2π sin x = (2n + 1) , n ∈Integer
2 ⇒ tan θ = 0
2n + 1 or tan θ = ± 3
⇒ sin x =
4 Now, tan θ = 0
3 1 1 3
⇒ sin x = − , − , , [Q sin x ∈ [− 1, 1]] ⇒ θ = mπ , where m is an integer.
4 4 4 4
508 Trigonometrical Equations

 π  π
2
and tan θ = ± 3 = tan  ±  ⇒ x=nπ and tan 2 x =  tan 
 3  3
π π
⇒ θ = nπ ± ⇒ x = nπ and x = nπ ±
3 3
π π
∴ θ = mπ , nπ ± , where m and n are integers. Neglecting x = nπ as 0 < x <
3 2
27. Given, sin 4 θ − 2 sin 2 θ + 1 = 2 π  π
⇒ x= ∈ 0, 
⇒ (sin θ − 1) = 2
2 2
⇒ sin θ = ± 2 + 1 2 3  2
1
which is not possible. Hence, given statement is false.
cos x 2 1 3 −1
∴ = = ×
28. tan (x + 100° ) = tan (x + 50° ) tan x tan(x − 50° ) cos x + sin x 1 3 1+ 3 3 −1
+
tan(x + 100° ) 2 2
⇒ = tan (x + 50° ) tan(x − 50° ).
tan x cos x 3 −1
⇒ =
sin(x + 100° ) cos x sin(x + 50° ) sin(x − 50° ) cos x + sin x 2
⇒ ⋅ =
cos (x + 100° ) sin x cos(x + 50° ) cos (x − 50° )
30. Since, the given system has non-trivial solution.
sin(2x + 100° ) + sin 100° cos 100° − cos 2x sin 3 θ −1 1
⇒ =
sin(2x + 100° ) − sin 100° cos 100° + cos 2x
∴ cos 2 θ 4 3 =0
⇒ [sin(2x + 100° ) + sin 100° ] [cos 100° + cos 2x] 2 7 7
= [cos 100° − cos 2x] × [sin(2x + 100° ) − sin 100° ] ⇒ sin 3 θ (28 − 21) + 1 (7 cos 2 θ − 6)
⇒ sin(2x + 100° ) ⋅ cos 100° + sin(2x + 100° ) ⋅ cos 2x + 1 (7 cos 2 θ − 8) = 0
+ sin 100° cos 100° + sin 100° cos 2x ⇒ 7 sin 3 θ + 14 cos 2 θ − 14 = 0
= cos 100° sin(2x + 100° ) − cos 100° sin 100° ⇒ sin 3 θ + 2 cos 2 θ − 2 = 0
− cos 2x sin (2x + 100° ) + cos 2x sin 100° ⇒ 3 sin θ − 4 sin3 θ + 2 (1 − 2 sin 2 θ ) − 2 = 0
⇒ 2 sin(2x + 100° ) cos 2x + 2 sin 100° cos 100° = 0 ⇒ sin θ (4 sin 2 θ + 4 sin θ − 3) = 0
⇒ sin(4x + 100° ) + sin 100° + sin 200° = 0 ⇒ sin θ = 0
⇒ sin(4x + 100° ) + 2 sin 150° cos 50° = 0 ⇒ θ = nπ ...(i)
1 or 4 sin 2 θ + 4 sin θ − 3 = 0
⇒ sin(4x + 100° ) + 2 ⋅ sin(90° − 50° ) = 0
2 ⇒ ( 2 sin θ − 1) ( 2 sin θ + 3) = 0
⇒ sin(4x + 100° ) + sin 40° = 0 1 3
⇒ sin θ = [Qsin θ = − is not possible]
⇒ sin(4x + 100° ) = sin(− 40° ) 2 2
⇒ 4x + 100° = nπ + (− 1)n (− 40° )  π 
∴ θ = nπ + (−1)n   ...(ii)
⇒ 4x = n (180° ) + (− 1)n (− 40° ) − 100°  6


1
x = [n (180° ) + (− 1)n (− 40° ) − 100° ] ∴ From Eqs. (i) and (ii), we get
4  π
θ = nπ or nπ + (−1)n  
The smallest positive value of x is obtained  6
when n = 1. 2
x | + |cos 3 x| + ....
31. Given, 2 1+ | cos x | + |cos = 22
1
Therefore, x = (180° + 40° − 100° ) 1
4
⇒ 21 −|cos x| = 22
1
⇒ x = (120° ) = 30° 1
4 ⇒ =2
1 − |cos x|
29. exp {(sin 2 x + sin 4 x + sin 6 x + ... ∞ ) log e 2}
1
sin 2 x sin 2 x ⇒ |cos x| =
⋅log e 2 log e 2 2
2
cos 2 x
= e 1 − sin x
=e 1
⇒ cos x = ±
tan 2 x 2
⇒ 2 satisfies x − 9x + 8 = 0
2

π 2π π 2π
⇒ x = 1, 8 ∴ x=
, ,− ,− [Q x ∈ (−π , π )]
2 2 3 3 3 3
∴ 2tan x
=1 and 2tan x
=8 π π
 2 
⇒ tan 2 x = 0 and tan 2 x = 3 Thus, the solution set is ± , ± .
 3 3 
Trigonometrical Equations 509

32. Given, 4 cos 2 x sin x − 2 sin 2 x = 3 sin x Topic 2 Solving Equations with Graph
⇒ 4 (1 − sin 2 x) sin x − 2 sin 2 x − 3 sin x = 0 1. Given, (cot−1 x)2 − 7(cot−1 x) + 10 > 0
⇒ 4 sin x − 4 sin x − 2 sin x − 3 sin x = 0
3 2
⇒ (cot−1 x − 2)(cot−1 x − 5) > 0 (by factorisation)
⇒ − 4 sin3 x − 2 sin 2 x + sin x = 0 ⇒ cot−1 x < 2 or cot−1 x > 5
⇒ − sin x (4 sin 2 x + 2 sin x − 1) = 0 By wavy curve method,
⇒ sin x = 0 or 4 sin 2 x + 2 sin x − 1 = 0 + – +
−2 ± 4 + 16 cot−1 x =2 cot−1 x =5
⇒ sin x = sin 0 or sin x =
2 (4) ∴ cot−1 x ∈ (−∞ , 2) ∪ (5, ∞ )
−1 ± 5 cot−1 x ∈ (0, 2) [Q Range of cot−1 x is (0, π )]
⇒ x = nπ or sin x = ∴ x ∈ (cot 2, ∞ )
4
π 2. Since, 2 sin 2 θ − 5 sin θ + 2 > 0
⇒ x = nπ or sin x = sin
10 ⇒ ( 2 sin θ − 1) (sin θ − 2) > 0
 3π  [where, (sin θ − 2) < 0, ∀ θ ∈ R]
or sin x = sin  − 
 10 
∴ ( 2 sin θ − 1) < 0
π  − 3π 
⇒ x = nπ , nπ + (−1)n , nπ + (−1)n   Y
10  10 
∴ General solution set is
y=1
2
 π
{ x : x = nπ} ∪ x : x = nπ + (−1)n  X' X
 10  O π 5π π 2π
6 6
  −3 π  
∪ x : x = nπ + (−1)n  
  10   Y'
1
33. Given that, ⇒ sin θ <
2 cos x + 2 cos 2x + sin 2x + sin 3x + sin x − 2 sin x = 0 2
∴ 2 cos x + 2 cos 2x + 2 sin x cos x + (sin 3x − sin x) = 0  π  5π 
∴ From the graph, θ ∈ 0,  ∪  , 2π
 6  6 
⇒ 2 cos x + 2 cos 2x + 2 sin x cos x + 2 cos 2x sin x = 0
⇒ 2 cos x(1 + sin x) + 2 cos 2x(1 + sin x) = 0 sin(πx)
3. (a, b, d) Given, f (x) = ,x>0
⇒ 2 (1 + sin x)(cos x + cos 2x) = 0 x2
 3x x x2π cos(πx) − 2x sin(πx)
⇒ 4 (1 + sin x) cos   cos = 0 ⇒ f ′ (x) =
 2 2 x4
 xπ 
∴ 1 + sin x = 0 2x cos(πx) − tan(πx)
 2 
3x x =
or cos = 0 or cos = 0 x 4
2 2
 xπ 
If 1 + sin x = 0, then sin x = −1 2 cos(πx) − tan(πx)
 2 
3π = 3
∴ x = 2 nπ + ...(i) x
2
Since, for maxima and minima of f (x), f ′ (x) = 0
3x 3x π πx
If cos = 0, then = (2n + 1) ⇒ cos(πx) = 0 or tan(πx) = , (as x > 0)
2 2 2 2
π πx
∴ x = (2 n + 1) ...(ii) Q cos(πx) ≠ 0 ⇒ tan(πx) =
3 2
x x π y=tan (px)
And if cos = 0, then = ( 2 n + 1) Y
2 2 2
∴ x = ( 2 n + 1)π ...(iii) y=px
But given interval is [−π , π ] . 2
π
Put n = −1 in Eq. (i), x = − 3 5 7
X
2 1 1 P1 2 P2 3 P3 4 9
O 2 2 2
2
π π 2
Put n = 0, 1, − 1, − 2 in Eq. (ii), x =,π − ,− π
3 3
π π π
Hence, the solution in [− π , π ] are − π , − , − , , π.
2 3 3
510 Trigonometrical Equations

 3 3π 5π
Q f ′ (P1− ) < 0 and f ′ (P1+ ) > 0 ⇒ x = P1 ∈ 1,  is point of From Eq. (i), <θ <
 2 2 3
local minimum. ⇒ 2 cos θ + 1 ∈ (1, 2)
 5 ∴ 1 < 2 sin (θ + φ ) < 2
Q f′ (P2− ) > 0 and f ′ (P2+ ) < 0 ⇒ x = P2 ∈ 2,  is point of
 2 1
local maximum. ⇒ < sin (θ + φ ) < 1 …(iii)
2
From the graph, for points of maxima x1 , x2, x3 …… it is π 5π
clear that ⇒ <θ+ φ <
6 6
5 9 13 17
− x1 > − x2 > − x3 > − x4 ...... 13π 17π
2 2 2 2 or <θ+ φ <
6 6
⇒ xn + 1 − xn > 2, ∀ n.
π 5π
From the graph for points of minima y1 , y2, y3 ....., it is ∴ −θ < φ < −θ
6 6
clear that
3 5 7 9 13π  17π 
− y1 > − x1 > − y2 > − x2 ...... or −θ < φ <   −θ
2 2 2 2 6  6 
|xn − yn|> 1, ∀ n and x1 > ( y1 + 1)  3π 2π   2π 7π   3π 5π 
⇒ φ ∈− , −  or  ,  , as θ ∈  , 
 5  9  13  2 3  3 6  2 3
And x1 ∈ 2,  , x2 ∈ 4,  , x3 ∈ 6,  ........
 2  2  2 2
θ
5. Given, (1 − tan θ )(1 + tan θ ) sec2 θ + 2tan =0
 1
⇒ xn ∈ 2n , 2n +  , ∀ n. 2
θ
 2 ⇒ (1 − tan 2 θ ) ⋅ (1 + tan 2 θ ) + 2tan =0
tan 2 θ
Hence, options (a), (b) and (d) are correct. ⇒ 1 − tan θ + 2
4
=0
4. PLAN It is based on range of sin x, i.e. Put tan θ = x
2
[− 1,1] and the internal for a < x < b.
∴ 1 − x2 + 2 x = 0
Description of Situation As θ , φ ∈ [0, 2π ] and
⇒ x2 − 1 = 2x
3
tan (2π − θ ) > 0, − 1 < sin θ < − NOTE 2 x and x 2 − 1 are uncompatible functions, therefore we
2
have to consider range of both functions.
tan (2π − θ ) > 0
Curves y = x2 − 1 and y = 2x
⇒ − tan θ > 0
Y
∴ θ ∈II or IV quadrant.
3
Also, − 1 < sin θ < −
2 X′ X
–1 1
Y
–1
2π – π/3
π + π/3

1
Y′
It is clear from the graph that two curves intersect at
π 2π
X
one point at x = 3, y = 8.
y = –√3
O
–1 y = –1 2 Therefore, tan 2 θ = 3
⇒ tanθ = ± 3
4π 5π
⇒ <θ < but θ ∈II or IV quadrant π
3 3 ⇒ θ=±
3
3π 5π
⇒ <θ < …(i)
2 3
 θ θ
Topic 3 Problems Based on
Here, 2 cos θ (1 − sin φ ) = sin 2 θ  tan + cot  cos φ − 1 Maximum and Minimum
 2 2
1. PLAN For solving this type of questions, obtain the LHS and RHS in
 2θ 2 θ
 sin + cos  equation and examine, the two are equal or not for a given
⇒ 2 cos θ − 2 cos θ sin φ = sin θ 
2 2 2  cos φ − 1 interval.
θ
 sin cos θ  Given, trigonometrical equation
 2 2 
(sin x − sin 3x) + 2 sin 2x = 3
 1 
⇒ 2 cos θ − 2 cos θ sin φ = 2 sin θ   cos φ − 1
2
⇒ −2 cos 2x sin x + 4 sin x cos x = 3
 sin θ 
 C + D  C − D
⇒ 2 cos θ + 1 = 2 sin φ cos θ + 2 sin θ cos φ [Qsin C − sin D = 2 cos   sin   and
 2   2 
⇒ 2 cos θ + 1 = 2 sin (θ + φ ) …(ii) sin 2θ = 2 sin θ cos θ ]
Trigonometrical Equations 511

⇒ 2 sin x ( 2 cos x − cos 2x) = 3 ⇒ sin x + 1 = 2 (1 − sin 2 x)


⇒ 2 sin x ( 2 cos x − 2 cos 2 x + 1) = 3 ⇒ 2 sin 2 x + sin x − 1 = 0

3 ⇒ ( 2 sin x − 1) (sin x + 1) = 0
1 
2

⇒ 2 sin x  − 2  cos x −   = 3 1
 2  ⇒ sin x = , sin x = − 1
2 2
 1
2 π 5π
⇒ 3 sin x − 3 = 4  cos x −  sin x ⇒ x= ,
 2 6 6

As x ∈ (0, π ) LHS ≤ 0 and RHS ≥ 0 or x=
2
For solution to exist, LHS = RHS = 0 π
Now, LHS = 0 but x ∉ (2n + 1)
2
⇒ 3 sin x − 3 = 0 π 5π
∴ x= ,
⇒ sin x = 1 6 6
π Hence, number of solutions are two.
⇒ x=
2
6. Given equation is sin (ex ) = 5x + 5−x is
π
For x= , LHS = sin (ex ) < 1 , ∀ x ∈R
2
2 and RHS = 5x + 5− x ≥ 2
 π 1 π  1
RHS = 4  cos −  sin = 4   (1) = 1 ≠ 0 ∴ sin (ex ) = 5x + 5− x has no solution.
 2 2 2  4
∴ No solution of the equation exists. 7. Since, the given quadratic equation
(cos p − 1) x2 + (cos p) x + sin p = 0
2. 2 sin 2 θ − cos 2 θ = 0
has real roots.
1
⇒ sin 2 θ = ∴ Discriminant, cos 2 p − 4 sin p (cos p − 1) ≥ 0
4
⇒ (cos p − 2 sin p)2 − 4 sin 2 p + 4 sin p ≥ 0
Also, 2 cos 2 θ = 3 sin θ
1 ⇒ (cos p − 2 sin p)2 + 4 sin p (1 − sin p) ≥ 0
∴ sin θ = [Q sin θ + 2 ≠ 0] Q 4 sin p (1 − sin p) > 0 for 0 < p < π
2
and (cos p − 2 sin p)2 ≥ 0
⇒ Two solutions exist in the interval [0, 2π].
Thus, (cos p − 2 sin p)2 + 4 sin p (1 − sin p) ≥ 0
3. We know that,
for 0 < p< π .
− a 2 + b2 ≤ a sin x + b cos x ≤ a 2 + b2
Hence, the equation has real roots for 0 < p < π .
∴ − 74 ≤ 7 cos x + 5 sin x ≤ 74 8. Let f (x) = tan x − x
i.e. − 74 ≤ 2k + 1 ≤ 74 π
We know, for 0 < x <
Since, k is integer, − 9 < 2k + 1 < 9 2
⇒ − 10 < 2k < 8 ⇒ − 5 < k < 4 ⇒ tan x > x
⇒ Number of possible integer values of k = 8. ∴ f (x) = tan x − x has no root in (0, π / 2)
4. Given, 3 sin 2 x − 7 sin x + 2 = 0 For π / 2 < x < π , tan x is negative.
⇒ 3 sin 2 x − 6 sin x − sin x + 2 = 0 ∴ f (x) = tan x − x < 0
⇒ 3 sin x (sin x − 2) − 1 (sin x − 2) = 0 π 
So, f (x) = 0 has no root in  , π  .
2 
⇒ (3 sin x − 1) (sin x − 2) = 0
1 3π
⇒ sin x = [Q sin x = 2 is rejected] For < x < 2π , tan x is negative.
3 2
1 ∴ f (x) = tan x − x < 0
⇒ x = nπ + (−1)n sin −1 , n ∈ I
3  3π 
So, f (x) = 0 has no root in  , 2π .
For 0 ≤ n ≤ 5, x ∈ [0, 5π ]  2 
There are six values of x ∈ [0, 5π ] which satisfy the We have, f (π ) = 0 − π < 0
equation 3 sin 2 x − 7 sin x + 2 = 0.  3π  3π 3π
and f   = tan − >0
π  2 2 2
5. tan x + sec x = 2 cos x, x ∉ ( 2n + 1)
2 3π
∴ f (x) = 0 has at least one root between π and .
⇒ sin x + 1 = 2 cos 2 x 2
512 Trigonometrical Equations

9. Given, a1 + a 2 cos 2x + a3 sin 2 x = 0, ∀ x Therefore, (b) is the answer from Eq. (i).
 1 − cos 2x [putting the value of xy]
⇒ a1 + a 2 cos 2x + a3   = 0, ∀ x
 2  ⇒ xyz = x + y + z
 a   a  Therefore, (c) is also the answer.
⇒  a1 + 3  +  a 2 − 3  cos 2x = 0, ∀ x
 2  2 12. The given function f : [0, 2] → R defined by
⇒ a1 +
a3
= 0 and a 2 − 3 = 0
a  π  π
f (x) = (3 − sin(2πx))sin  πx −  − sin 3πx + 
2 2  4  4
k k
⇒ a1 = − , a 2 = , a3 = k, where k ∈ R sin πx cos πx  sin 3πx cos (3πx) 
= (3 − sin(2πx)) − − + 
2 2  2 2   2 2 
 k k  [sin(πx) − cos(πx)]
Hence, the solutions, are  − , , k , where k is any 1
 2 2  = (3 − sin(2πx)) −
2 2
real number.
[3 sin(πx) − 4 sin3 (πx) + 4 cos3 (πx) − 3 cos(πx)]
Thus, the number of triplets is infinite.
sin(πx) − cos(πx)
10. We know that, sec2 θ ≥ 1 = [3 − sin(2πx) − 3
2

4xy
≥1 + 4 {sin 2(πx) + cos 2(πx) + sin(πx) cos(πx)}]
(x + y)2 sin(πx) − cos(πx)
= [4 + sin(2πx)]
⇒ 4xy ≥ (x + y)2 2
As, f (x) ≥ 0 ∀ ∈ [α , β ] , where α , β ∈ [0, 2] , so
⇒ (x + y)2 − 4xy ≤ 0
sin(π x) − cos(π x) ≥ 0 as 4 + sin(2π x) > 0 ∀x ∈ R.
⇒ (x − y)2 ≤ 0
 π 5π 
⇒ x− y=0 ⇒ x= y ⇒ π x∈ ,
 4 4 
Therefore, x + y = 2x [add x both sides] 1 5 
⇒ x∈ ,
But x + y ≠ 0 since it lies in the denominator, 4 4 
⇒ 2x ≠ 0 ⇒ x ≠ 0 1 5
∴ α= and β =
Hence, x = y, x ≠ 0 is the answer. 4 4
Therefore, (a) and (b) are the answers. Therefore the value of (β − α ) = 1

11. For 0 < φ < / π / 2, we have 13. Given, n > 3 ∈ Integer


∞ 1 1 1
and = +
x= ∑ cos2n φ = 1 + cos2 φ + cos4 φ + cos6 φ + K  π  2π   3π 
sin   sin   sin  
n=0  n  n  n
It is clearly a GP with common ratio of cos 2 φ which is 1 1 1
≤ 1. ⇒ − =
π 3π 2π
sin sin sin
1 1  a  n n n
Hence, x = = Q S∞ = , − 1 < r < 1
1 − cos φ sin φ 
2 2
1−r  3π π
sin − sin
1 ⇒ n n = 1
Similarly, y = π 3π 2π
cos 2 φ sin ⋅ sin sin
n n n
1
and z= π 3π
1 − sin 2 φ cos 2 φ sin ⋅ sin
 2π  π n n
⇒ 2 cos   ⋅ sin =
1 1  n n 2π
Now, x + y = + sin
sin 2 φ cos 2 φ n
2π 2π 3π
cos 2 φ + sin 2 φ 1 ⇒ 2 sin ⋅ cos = sin
= = n n n
cos φ sin φ
2 2
cos φ sin 2 φ
2
4π 3π
1 1 ⇒ sin = sin
Again, = 1 − sin 2 φ cos 2 φ = 1 − n n
z xy
4π 3π
1 xy − 1 ⇒ =π−
⇒ = ⇒ xy = xyz − z n n
z xy 7π
⇒ =π ⇒ n=7
⇒ xy + z = xyz …(i) n
Trigonometrical Equations 513

14. Given, tan θ = cot 5 θ 18. Given, cos ( p sin x) = sin ( p cos x) , ∀ x ∈ [0, 2π ]
π  π 
⇒ tan θ = tan  − 5θ ⇒ cos ( p sin x) = cos  − p cos x
2  2 
π π 
⇒ − 5 θ = nπ + θ ⇒ p sin x = 2nπ ±  − p cos x , n ∈ I
2 
2
π [Q cos θ = cos α ⇒ θ = 2nπ ± α , n ∈ I ]
⇒ 6 θ = − nπ
2 ⇒ p sin x + p cos x = 2nπ + π / 2
π nπ or p sin x − p cos x = 2nπ − π / 2, n ∈ I
⇒ θ= −
12 6 ⇒ p (sin x + cos x) = 2nπ + π / 2
π  or p (sin x − cos x) = 2nπ − π / 2, n ∈ I
Also, cos 4 θ = sin 2 θ = cos  − 2 θ
2  π π π
⇒ p 2 (cos sin x + sin cos x) = 2nπ +
π  4 4 2
⇒ 4 θ = 2 nπ ±  − 2 θ
2   π π  π
or p 2  cos sin x − sin cos x = 2nπ − , n ∈ I
 4 4  2
Taking positive sign,
π (4n + 1)π
6 θ = 2 nπ + ⇒ p 2 [sin (x + π / 4)] =
2 2
nπ π π
⇒ θ= + or p 2 [sin (x − π / 4)] = (4n − 1) , n ∈ I
3 12 2
Taking negative sign, Now, − 1 ≤ sin (x ± π / 4) ≤ 1
π π ⇒ − p 2 ≤ p 2 sin (x ± π / 4) ≤ p 2
2 θ = 2 nπ − ⇒ θ = nπ −
2 4 (4n + 1) ⋅ π
⇒ −p 2≤ ≤ p 2, n ∈I
Above values of θ suggest that there are only 3 common 2
solutions. (4n − 1) π
or −p 2≤ ≤ p 2, n ∈I
15. Given, 2 sin 2 x − 3 sin x + 1 ≥ 0 2
Second inequality is always a subset of first, therefore
⇒ 2 sin 2 x − 2 sin x − sin x + 1 ≥ 0
we have to consider only first.
⇒ (2 sin x − 1) (sin x − 1) ≥ 0 It is sufficient to consider n ≥ 0, because for n > 0, the
⇒ 2 sin x − 1 ≤ 0 or sin x ≥ 1 solution will be same for n ≥ 0.
1 If n ≥ 0, − 2 p ≤ (4n + 1) π / 2
⇒ sin x ≤ or sin x = 1
2 ⇒ (4n + 1) π / 2 ≤ 2 p
 π  π  5π 
⇒ x ∈ 0, ∪ ∪ ,π For p to be least, n should be least.
 6   2   6 
⇒ n =0
2π π
16. Given, x+ y= ⇒ 2 p ≥ π /2 ⇒ p≥
3 2 2
3 π
and cos x + cos y = Therefore, least value of p =
2 2 2
 2 π  3
⇒ cos x + cos  − x = 1
 3  2 19. Given, e sin x − =4
e sin x
 1 3  3 ⇒ (e sin x )2 − 4 (e sin x ) − 1 = 0
⇒ cos x +  − cos x + sin x =
 2 2  2 4 ± 16 + 4
⇒ e sin x = =2 ± 5
1 3 3 2
⇒ cos x + sin x =
2 2 2 But since, e ~ 2 .72 and we know, 0 < e sin x < e
π  3 ∴ e sin x = 2 ± 5 is not possible.
⇒ sin  + x = , which is never possible.
6  2 Hence, it does not exist any solution.
Hence, no solution exists. 20. The point of intersection is given by
cos θ ≤ 1 ⇒ log (cos θ ) < 0 π 
17. Since, sin 3x = cos x = sin  − x
2 
and cos (log θ ) > 0
π 
∴ cos (log θ ) > log (cos θ ) ⇒ 3x = nπ + (−1)n  − x
2 
514 Trigonometrical Equations

(i) Let n be even i.e. n = 2 m π


⇒ cos θ + sin θ <
π 2
⇒ 3x = 2 mπ + − x
2 π
Since, cos θ < − sin θ
mπ π 2
⇒ n= + ...(i)
2 8 π 
⇒ sin (cos θ ) < sin  − sin θ
(ii) Let n be odd i.e. n = ( 2 m + 1) 2 
π  ⇒ sin (cos θ ) < cos (sin θ )
∴ 3x = ( 2 m + 1) π −  − x
2  ⇒ cos (sin θ ) > sin (cos θ )
π π
⇒ 3x = 2 mπ + + x 22. Let f (θ ) = 5 cos θ + 3 cos θ +  + 3
2  3
π
⇒ x = mπ + ...(ii)  π π
4 = 5 cos θ + 3  cos θ cos − sin θ sin  + 3
 3 3
π π
Now, − ≤x≤  1  3
2 2 = 5 cos θ + 3   cos θ − 3   sin θ + 3
π π 3π  2  2
⇒ x= , − [from Eqs. (i) and (ii)]
8 4 8 13 3 3
= cos θ − sin θ + 3
Thus, points of intersection are 2 2
π π  π π   3π 3π 

1
f (θ ) = (13 cos θ − 3 3 sin θ ) + 3
 , cos   , cos   − , cos 
8 8   4 4   8 8 2
 1 1  Put r cos α = 13, r sin α = 3 3 , then
21. We have, cos θ + sin θ = 2  cos θ + sin θ
 2 2  r = 169 + 27 = 196 = 14
π π 1
  ∴ f (θ ) = (r cos α cos θ − r sin α sin θ ) + 3
= 2 sin ⋅ cos θ + cos ⋅ sin θ 2
 4 4 
1
π  = r cos (θ + α ) + 3
= 2 sin  + θ 2
4 
= 7 cos (θ + α ) + 3
π
⇒ cos θ + sin θ ≤ 2 < Now, −1 ≤ cos (θ + α ) ≤ 1
2
 π  ⇒ −7 ≤ 7 cos (θ + α ) ≤ 7
as, 2 = 1.4141, = 1.57 (approx)
 2  ⇒ − 4 ≤ f (θ ) ≤ 10
22
Inverse Circular Functions

Topic 1 Domain and Range


Objective Question I (Only one correct option) 2. The number of real solutions of
π
1. The domain of the function tan −1 x (x + 1) + sin −1 x2 + x + 1 = is (1999, 2M)
2
|x|+ 5
f (x) = sin −1 2  is (a) zero (b) one
 x + 1 (c) two (d) infinite
(−∞ ,− a ] ∪ [a , ∞ ). Then a is equal to
(2020 Main, 2 Sep I) Fill in the Blank
17 17 − 1 3. The greater of the two angles A = 2 tan −1 (2 2 − 1) and
(a) +1 (b)
2 2  1  3
1 + 17 17 B = 3 sin −1   + sin −1   is …… .
(c) (d)  3  5 (1989, 2M)
2 2

Topic 2 Properties of Inverse Functions


Objective Question I (Only one correct option)
(a) If α = tan(cos−1 f (6)), then α 2 + 2α − 1 = 0
1. 2π − sin −1 + sin −1
4 5 16
+ sin −1  is equal to 3
 5 13 65 (b) f (4) =
2
(2020 Main, 3 Sep I)
π 5π 3π 7π (c) sin(7 cos−1 f (5)) = 0
(a) (b) (c) (d) 1
2 4 2 4 (d) lim f (n ) =
n→ ∞ 2
Objective Questions II 3. Let f (x) = log e (sin x), (0 < x < π ) and g (x) = sin − 1 (e− x ),
(One or more than one correct option) (x ≥ 0). If α is a positive real number such that
a = ( fog )′ (α ) and b = ( fog )(α ), then (2019 Main, 10 April II)
2. For non-negative integers n, let (a) aα 2 − bα − a = 0 (b) aα 2 − bα − a = 1
n
k+1  k+2  (c) aα 2 + bα − a = − 2α 2 (d) aα 2 + bα + a = 0
∑ sin  n + 2 π sin  n + 2 π  19  n 
k=0
f (n ) = 4. The value of cot  ∑ cot−1 1 + ∑ 2 p  is
n
k+1  
∑ sin 2 n + 2 π n =1  p =1 
k=0 23 21 (2019 Main, 10 Jan II)
(a) (b)
−1
Assuming cos x takes values in [0, π ], which of the 22 19
following options is/are correct? (2019 Adv.) 19 22
(c) (d)
21 23
516 Inverse Circular Functions

π
5. If cos −1 
2 −1  3  3 12. Let (x, y) be such that
 + cos   =  x >  , then x is equal to π
 3x  4x 2  4 sin −1 (ax) + cos −1 ( y) + cos −1 (bxy) = . (2007)
(2019 Main, 9 Jan I) 2
145 146 145 145 Column I Column II
(a) (b) (c) (d)
10 12 12 11
A. If a = 1 and b = 0, then (x, y) p. lies on the circle x 2 + y 2 = 1
6. If α = 3sin −1   and β = 3 cos −1   , where the inverse
6 4
 
11 9   B. If a = 1 and b = 1, then (x, y) q. lies on ( x 2 − 1)( y 2 − 1) = 0
trigonometric functions take only the principal values, C. If a = 1 and b = 2, then (x, y) r. lies on y = x
then the correct option(s) is/are (2015 Adv.)
(a) cos β > 0 (b) sin β < 0 D. If a = 2 and b = 2, then (x, y) s. lies on ( 4x 2 − 1)( y 2 − 1) = 0
(c) cos (α + β ) > 0 (d) cos α < 0
x
7. If 0 < x < 1, then 1 + x2 [{ x cos (cot−1 x) 13. Let E1 = { x ∈ R : x ≠ 1 and > 0} and
x−1
+ sin (cot−1 x)}2 − 1]1/ 2 is equal to (2008, 3M)    x  
x E 2 = x ∈ E1 : sin − 1  log e    is a real number 
(a) (b) x (c) x 1 + x2 (d) 1 + x2   x − 1 
1 + x2
 
(Here, the inverse trigonometric function sin − 1 x
8. The value of x for which sin [cot−1 (1 + x)] = cos (tan −1 x)
 π π
is assumes values in − , ).Let f : E1 → R be the
(2004, 1M)  2 2 
1 1
(a) (b) 1 (c) 0 (d) −
2 2  x 
function defined by f (x) = log e   and g : E 2 → R be
 x2
x 
3  x x  4 6  x − 1
9. If sin −1  x − + − K + cos −1  x2 − + − K
 2 4   2 4    x 
π the function defined by g (x) = sin − 1  log e  .
= , for 0 <| x| < 2, then x equals (2001, 1M)   x −1 
2 (2018 Adv.)
(a) 1/2 (b) 1 (c) –1/2 (d) –1

List I List II
2π 
10. The principal value of sin −1 sin  is (1986, 2M)  1   e 
 3  − ∞, ∪
P. The range of f is 1.  1 − e   e − 1 , ∞
2π 2π π 5π 
(a) − (b) (c) (d)
3 3 3 3 Q. The range of g contains 2. (0, 1)
Match the Columns The domain of f 3.  1 1
− ,
R. contains  2 2 
11. Match List I with List II and select the correct answer
using the code given below the lists.
S. The domain of g is 4. (− ∞ , 0) ∪ (0, ∞ )
List I List II  e 
 − ∞,
e − 1 
1/ 2 5.
P. 
1  cos(tan−1 y) + y sin(tan−1 y)
2  1. 1 5 
 2  + y 
4
−1 −1
takes value 2 3
 y  cot (sin y) + tan(sin y)   1 e 
  (− ∞ , 0) ∪  ,
 2 e − 1 
6.
Q. If cos x + cos y + cos z = 0 = sin x + sin y + sin z, then 2. 2
x − y
possible value of cos is
2 The correct option is
R. If cos  π − x  cos 2 x + sin x sin2 x sec x 3. 1 (a) P → 4; Q → 2; R → 1; S → 1
  (b) P → 3; Q → 3; R → 6; S → 5
4  2
π (c) P → 4; Q → 2; R → 1; S → 6
= cos x sin2 x sec x + cos  + x  cos 2 x, then
4  (d) P → 4; Q → 3; R → 6; S → 5
possible value of sec x is
S. If cot (sin−1 1 − x 2 ) = sin[tan−1( x 6 )], 4. 1 Integer & Numerical Answer Type Questions
x = 0. Then, possible value of x is 14. The value of
Codes  1 10  7 π kπ   7π (k + 1)π  
sec−1  ∑ sec +  sec + 
4  
P Q R S  k = 0  12 2  12 2
(a) 4 3 1 2
(b) 4 3 2 1  π 3π 
in the interval − , equals ………
(c) 3 4 2 1  4 4  (2019 Adv.)
(d) 3 4 1 2
Inverse Circular Functions 517

15. The number of real solutions of the equation 16. If f : [ 0, 4π ] → [ 0, π ] be defined by f (x) = cos −1 (cos x).9
 ∞ ∞
 x
i
Then, the number of points x ∈ [0, 4π ] satisfying the
sin −1  ∑ xi+ 1 − x ∑    10 − x
  2 
i = 1 i =1 equation f (x) = , is (2014 Adv.)
10
π  ∞  x i ∞ 
= − cos −1  ∑  −  − ∑ (− x)i lying in the interval
2    
i = 1 2 i =1  Analytical & Descriptive Questions
 1 1
 − ,  is ................... . x2 + 1
 2 2 17. Prove that cos tan −1 [sin (cot−1 x)] = .
x2 + 2 (2002, 5M)
(Here, the inverse trigonometric functions sin −1 x
 π π 1
and cos −1x assume values in − , and [0, π ], 18. Find the value of cos (2 cos −1 x + sin −1 x) at x = , where
 2 2  5
respectively.) (2018 Adv.) 0 ≤ cos −1 x ≤ π and − π / 2 ≤ sin −1 x ≤ π / 2. (1981, 2M)

Topic 3 Sum and Difference Formulae


Objective Questions I (Only one correct option)  2x  1
6. If tan −1 y = tan −1 x + tan −1   , where |x| < .
 1 − x2
1. The value of sin −1 
12 −1  3  3
 − sin   is equal to
 13  5 Then, the value of y is
(2019 Main, 12 April I) 3x − x3 3x + x 2
(a) (b)
(a) π − sin −1  
63 1 − 3x 2
1 − 3x 2
 65 
3x − x3 3x + x3
π (c) (d)
− sin −1  
56
(b) 1 + 3x 2
1 + 3x 2
2  65 
π  23   
− cos−1  
9 n
(c)
2  65  7. The value of cot  ∑ cot−1 1 + ∑ 2k  is

n = 1 k =1  
(d) π − cos−1  
33 (2013 Main)
 65  23 25 23 24
(a) (b) (c) (d)
y 25 23 24 23
2. If cos − 1 x − cos − 1 = α, where − 1 ≤ x ≤ 1, − 2 ≤ y ≤ 2,
2 8. If x, y and z are in AP and tan − 1 x, tan − 1 y and tan − 1 z are
y
x ≤ , then for all x, y, 4x2 − 4xy cos α + y2 is equal to also in AP, then (2013 Main)
2
(2019 Main, 10 April II)
(a) x = y = z (b) 2x = 3 y = 6z
(c) 6x = 3 y = 2z (d) 6x = 4 y = 3z
(a) 2 sin 2 α
(b) 4 cos2 α + 2x2 y2
 2 
9. The value of tan cos −1   + tan −1    is
(c) 4 sin 2 α 4
(1983, 1M)
(d) 4 sin 2 α − 2x2 y2   5  3 
π
3. If α = cos −1   , β = tan −1   , where 0 < α , β < , then
3 1 6 17
(a) (b)
 5  3 2 17 6
16
α − β is equal to (2019 Main, 8 April I) (c) (d) None of these
7
−1  9  −1  9 
(a) tan   (b) cos  
 5 10   5 10 
Fill in the Blanks
(d) sin −1 
9 
(c) tan −1  
9
  π
 14   5 10 
10. The numerical value of tan 2 tan −1   −  is … .
1
  5  4 (1984, 2M)
4. Considering only the principal values of inverse
 π
functions, the set A = x ≥ 0 : tan −1 (2x) + tan −1 (3x) =  11. If a, b, c are positive real numbers
 4 a (a + b + c) b (a + b + c)
(2019 Main, 12 Jan I) θ = tan −1 + tan −1
bc ca
(a) is an empty set
c (a + b + c)
(b) is a singleton + tan −1 .
(c) contains more than two elements ab
(d) contains two elements Then, tan θ equals …… . (1981, 2M)
5. If x = sin −1 (sin 10) and y = cos −1 (cos 10), then y − x is
12. Solve the following equation for x.
equal to (2019 Main, 9 Jan II)
π
(a) 0 (b) 10 (c) 7π (d) π tan −1 2x + tan −1 3x = (1978, 3M)
4
Answers
Topic 1 2 6
14. (0) 15. (2) 16. (3) 18. −
1. (c) 2. (c) 3. A 6

Topic 2 Topic 3
1. (c) 2. (a,b,c) 3. (b) 4. (b) 1. (b) 2. (c) 3. (d) 4. (b)
5. (c) 6. (c) 7. (c) 8. (d) 5. (d) 6. (a) 7. (b) 8. (a)
9. (b) 10. (c) 11. (b)  7 1
9. (b) 10.  −  11. 0 12.
12. (A → p; B → q; C → p; D → s) 13. (a)  17 6

Hints & Solutions


Topic 1 Domain and Range 3. Given, A = 2 tan −1 ( 2 2 − 1)
|x| + 5  1  3
1. Given function, f (x) = sin −1   and B = 3 sin −1   + sin −1  
 x2 + 1   3  5
For, domain of ‘f ’ Here, A = 2 tan −1 (2 2 − 1)
|x| + 5 |x| + 5
≤ 1, as 2 > 0 ∀ x ∈R = 2 tan −1 (2 × 1.414 − 1) = 2 tan −1 (1.828)
x2 + 1 x +1
π 2π
⇒ |x| + 5 ≤ x2 + 1 as x2 + 1 is positive ∀ x ∈ R ∴ A > 2 tan −1 ( 3 ) = 2 ⋅ =
⇒ x2 − |x| − 4 ≥ 0 3 3
⇒ |x|2 − |x| − 4 ≥ 0 [Q ∀ x ∈ R, x2 = |x|2 ] …(i) To find the value of B, we first say
Q For|x| − |x| − 4 = 0
2 1 1 π
sin −1 < sin −1 =
1 ± 1 + 16 3 2 6
⇒ |x| = 1 π
2 so that 0 < 3 sin −1 <
3 2
1 ± 17 1 + 17
⇒ |x| = ⇒|x| = as|x| ≥ 0 1  1 1  23
2 2 Now, 3 sin −1 = sin −1 3 ⋅ − 4 ⋅  = sin −1  
3  3 27  27
17 + 1
∴ |x| ≥  3 π

2 = sin −1 (0.851) < sin −1   =
17 + 1   17 + 1   2 3
⇒ x ∈  − ∞, − ∪ , ∞
 2   2   3 π
 3
17 + 1 sin −1   = sin −1 (0.6) < sin −1   =
∴ a=  5  2 3
2
π π 2π
2. Given function is ∴ B< + =
3 3 3
π 2π 2π
tan −1 x(x + 1) + sin −1 x2 + x + 1 = Thus, A > and B <
2 3 3
Function is defined, if Hence, greater angle is A.
(i) x (x + 1) ≥ 0, since domain of square root function.
(ii) x2 + x + 1 ≥ 0, since domain of square root function. Topic 2 Properties of Inverse Functions
−1
(iii) x + x + 1 ≤ 1, since domain of sin function.
2
1. We have,
From (ii) and (iii), 0 ≤ x + x + 1 ≤ 1 ∩ x + x ≥ 0
2 2  4 5 16
2π − sin −1 + sin −1 + sin −1 
 5 13 65
⇒ 0 ≤ x + x+ 1 ≤1∩x + x+ 1 ≥1
2 2

⇒ x2 + x + 1 = 1  4 5 16
= 2π −  tan −1 + tan −1 + tan −1 
 3 12 63
⇒ x + x=0
2
 4 5 
⇒ x (x + 1) = 0  + 
−1 3 12 −1 16
= 2π −  tan + tan 
⇒ x = 0, x = − 1  5 63
1−
 9 
Inverse Circular Functions 519

  21 × 3 −1 16  π 
= 2π −  tan −1   + tan  sin  
  4 ×4  63  π   n + 2  π 
(n + 1) cos   + cos  
 63 16  n + 2  π   n + 2
= 2π −  tan −1 + tan −1  sin  
 16 63  n + 2
=
 16 16  π 
= 2π −  cot−1 + tan −1  sin  
 63 63  n + 2
(n + 1) +
π 3π  π 
= 2π − = sin  
2 2  n + 2
Hence, option (c) is correct.  π 
(n + 2 ) cos  
2. It is given, that for non-negative integers ‘n’,  n + 2  π 
= = cos  
n k+1  k+2  (n + 2 )  n + 2
Σ sin  π  sin  π
k=0 n + 2  n + 2   π 
f (n ) = ⇒ f (n ) = cos  
n k+1   n + 2
Σ sin 2 π
k=0 n + 2   π
Now, f (6) = cos  
n  π  2k + 3    8
Σ  cos − cos  π 
k=0 n+2  n + 2  Q cos −1cos x = x
=  π  
n   2k + 2   Qα = tan (cos −1 f ((6))) = tan     π 
Σ 1 − cos  π   8 if x ∈  0 , 
  2  
k=0  n + 2 
[Q2 sin A sin B = cos( A − B) − cos( A + B) and = 2 −1
2 sin 2 A = 1 − cos 2 A] ⇒ (α + 1) = 2
 3π 5π 7π  ⇒ (α + 1)2 = 2
cos + cos + cos
  π  n  n+2 n+2 n+2 
 cos   Σ 1 −  ⇒ α 2 + 2α + 1 = 2
  n + 2  k = 0  2n + 3 
 + ..... + cos  π  ⇒ α 2 + 2α − 1 = 0
  n+2    π 
=  π 3
 2π 4π 6π  Now, f (4) = cos   = cos   = ,
cos + cos + cos +  4 + 2  6  2
n  n+2 n+2 n+2 
Σ 1−    Now,
k=0 2 n + 2
 ..... + cos  π     π 
  n + 2   sin (7 cos −1 f (5 )) = sin  7 cos −1  cos  
   5 + 2  
 nπ 
sin  
 π   n + 2 n + 3    π
(n + 1) cos   − cos  π = sin  7   = sin π = 0
 n + 2  π  n + 2    7 
sin   π
 n + 2 and Now, lim f (x) = lim cos = cos 0 = 1
= n→ ∞ n→ ∞ n+2
 nπ 
sin  
 n + 2 n + 2  Hence, options (a), (b) and (c) are correct.
(n + 1) − cos  π
 π  n + 2  3. Given functions, f (x) = log e (sin x), (0 < x < π ) and
sin  
 n + 2 g (x) = sin − 1 (e− x ), x ≥ 0.
[Q cos(α ) + cos(α + β) + cos(α + 2 β ) + ... Now, fog (x) = f ( g (x)) = f (sin − 1 (e− x ))
 nβ   = log e (sin(sin − 1 (e− x )))
sin  
 2  2 α + (n − 1 ) β  = log e (e− x ) {Qsin(sin − 1 x) = x, if x ∈ [− 1, 1]}
+ cos(α + (n − 1)β ) = cos  
 β  2  =−x …(i)
sin  
 2  d
 and ( fog )′ (x) = (− x) = − 1 …(ii)
π  dx
sin  π − 
 π   n + 2  π  According to the question,
(n + 1) cos   − cos  π + 
 n + 2  π   n + 2 Q a = ( fog )′ (α ) = − 1 [from Eq. (ii)]
sin  
 n + 2 and b = ( fog ) (α ) = − (α ) [from Eq. (i)]
=
 π  for a positive real value ‘α’.
sin  π − 
 n + 2 Since, the value of a = − 1 and b = − α, satisfy
(n + 1) − cos(π ) the quadratic equation (from the given options)
 π 
sin   aα 2 − bα − a = 1 .
 n + 2
520 Inverse Circular Functions

 19
 n  ⇒ x2(144x2 − 145) = 0
4. Consider, cot  ∑ cot−1 1 + Σ 2 p 

 n =1  p =1
 ⇒ x = 0 or x = ±
145

145
144 12
 19   19 n (n + 1) 
= cot  ∑ cot−1(1 + n (n + 1)) Q ∑ p =  But x >
3,
x=
145
  2
 n =1   n = 1  4 12
 19  6. Here, α = 3 sin −1   and β = 3 cos −1   as
6 4 6 1
>
= cot  ∑ cot−1 (1 + n + n 2)  11  9
  11 2
 n =1 
 19  6  1 π
1 ⇒ sin −1   > sin −1   =
= cot  ∑ tan −1   11  2 6
 1 + n (n + 1)
 n =1
6 π
1 ∴ α = 3 sin −1   > ⇒ cos α < 0
[ Q cot−1 x = tan −1 , if x > 0 ]  11 2
x
 4
 19  (n + 1) − n   Now, β = 3 cos −1  
= cot  ∑ tan −1   [put 1 = (n + 1) − n]  9
  1 + n (n + 1) 
 n =1 4 1  4  1 π
As < ⇒ cos −1   > cos −1   =
19
9 2  9  2 3
= cot ∑ (tan −1 (n + 1) − tan −1 n )
 4
n =1
∴ β = 3 cos −1   > π
 9
 −1 x − y −1 −1 
Q tan 1 + xy = tan x − tan y
 ∴ cos β < 0 and sin β < 0
= cot [(tan −1 −1
2 − tan 1) + (tan 3 − tan 2) + −1 −1 3π
Now, α + β is slightly greater than .
......+ (tan −1 20 − tan − 1 19)] 2
= cot (tan 20 − tan −1 1)
−1 ∴ cos (α + β ) > 0
 π  π  7. We have, 0 < x<1
= cot  − cot−1 20 −  − cot− 1 1 
 2  2  Let cot−1 x = θ
[Q tan −1 x + cot−1 x = π / 2 ] C
−1 −1
= cot (cot 1 − cot 20)
cot (cot−1 1) cot (cot−1 20) + 1 √1 + y2
= 1
cot (cot−1 20) − cot (cot−1 1)
cot A cot B + 1  θ
[Q cot ( A − B) =
cot B − cot A  B x A
(1 × 20) + 1 21 ⇒ cot θ = x
= = [Qcot (cot−1 x) = x]
20 − 1 19 1
⇒ sin θ = = sin (cot−1 x)
Key Idea Use the formula, 1+ x 2
5. cos −1 x + cos −1 y = cos −1( xy − 1 − x 2 1 − y 2 )
x
and cos θ = = cos (cot−1 x)
2 −1   3 π 1 + x2
We have, cos   + cos −1   =
 3x  4x 2
Now, 1 + x2 [{ x cos (cot−1 x) + sin (cot−1 x)}2 − 1]1/ 2
 2 3 4 9  π
⇒ cos −1  ⋅
1/ 2
− 1− 2 1− =  2 
 3x 4x 9x 16x2  2 x 1 
= 1 + x  x 2
+ − 1
 1 + x2 1 + x2  
[Q cos −1 x + cos −1 y = cos −1 (xy − 1 − x2 1 − y2 )]  
 1 9x2 − 4 16x2 − 9  π  2 
1/ 2

⇒ cos −1  2 − = 1 + x2 
 2x 12x2  2 = 1 + x2   − 1
   1 + x2  
 
6 − 9x2 − 4 16x2 − 9 π
⇒ = cos =0 = 1 + x2 [1 + x2 − 1]1/ 2 = x 1 + x2
12x2 2
⇒ 9x2 − 4 16x2 − 9 = 6 8. Given, sin [cot−1 (1 + x)] = cos (tan −1 x) … (i)

On squaring both sides, and we know that,


⇒ (9x2 − 4)(16x2 − 9) = 36  1   1 
cot−1 θ = sin −1  and tan −1 θ = cos −1 
⇒ 144x − 81x2 − 64x2 + 36 = 36
4  1 + θ2   1 + θ2 
   
⇒ 144x4 − 145x2 = 0
Inverse Circular Functions 521

From Eq. (i), Q. Given, cos x + cos y = − cos z


 1   1  and sin x + sin y = − sin z
sin sin −1  = cos  cos −1
 2
1 + (1 + x)   1 + x2  On squaring and adding, we get
 
cos 2 x + sin 2 x + cos 2 y + sin 2 y + 2 cos x cos y
1 1
⇒ = + 2 sin x sin y = 1
1 + (1 + x)2 1 + x2
1
⇒ 2 + 2 [cos(x − y)] = 1 ⇒ cos (x − y) = −
⇒ 1 + x2 + 2 x + 1 = x 2 + 1 2
⇒ x=−
1  x − y 1
⇒ 2 cos 2   −1 = −
2  2  2
π x− y 1 x− y 1
9. We know that, sin −1 (α ) + cos −1 (α ) = ⇒ 2 cos 2   = ⇒ cos   =
2  2  2  2  2
Therefore, α should be equal in both functions.
 π  π 
x2 x3 x4 x6 R. cos 2x ⋅  cos  − x − cos  + x  + 2 sin 2 x
∴ x− + − K = x2 − + −K  4  4 
2 4 2 4 = 2 sin x ⋅ cos x
x x2 x x2 2x 2 x2 ⇒ cos 2x ⋅ ( 2 sin x) + 2 sin x = 2 sin x ⋅ cos x
2
⇒ = ⇒ = ⇒ =
x x 2 2+ x 2+ x 2
2 + x 2 + x2 ⇒ 2 sin x [cos 2x + 2 sin x − 2 cos x] = 0
1+ 1+
2 2 2 2 ⇒ sin x = 0, (cos x − sin x) (cos x + sin x − 2 ) = 0
⇒ 2x (2 + x ) = 2x (2 + x)
2 2
⇒ sec x = 1 or tan x = 1 ⇒ sec x = 1 or 2
⇒ 4x + 2x3 = 4x2 + 2x3
S. cot (sin −1 1 − x2 ) = sin(tan −1 (x 6 ))
⇒ x (4 + 2x2 − 4x − 2x2) = 0
⇒ Either x = 0 or 4 − 4x = 0 x x 6
⇒ =
⇒ x=0 or x=1 1 − x2 1 + 6x2
Q 0 < | x|< 2 ⇒ 1 + 6 x2 = 6 − 6 x2
∴ x=1 and x ≠ 0 5 5
⇒ 12x2 = 5 ⇒ x= =
2π  −1  π  π π
10. sin −1 sin  −1 
12 2 3
 = sin sin  π −   = sin sin  =
 3   3   3 3 (P) → 4, (Q) → 3, (R) → 2 or 4, (S) → 1
11. P. Here, innermost function is inverse. 12. A. If a = 1, b = 0, then sin −1 x + cos −1 y = 0
⇒ sin −1 x = − cos −1 y ⇒ x2 + y2 = 1
√1 + y2
B. If a = 1 and b = 1, then
π
y sin −1 x + cos −1 y + cos −1 xy =
2
1 θ ⇒ cos −1 x − cos −1 y = cos −1 xy
2
⇒ xy + 1 − x2 1 − y2 = xy ⇒ (x2 − 1)( y2 − 1) = 0
−1
∴ Put tan y = θ ⇒ tan θ = y C. If a = 1, b = 2 , then
 1  cos (tan −1 y) + y sin(tan −1 y)  
1/ 2 π
sin −1 x + cos −1 y + cos −1 (2 xy) =
2
 2⋅ −1 −1  + y 
4
2
 y  cot (sin y) + tan(sin y)   ⇒ cos −1 x – cos −1 y = cos −1 (2xy)
1/ 2
  
2  ⇒ xy + 1 − x2 1 − y2 = 2xy ⇒ x2 + y2 = 1
  1 y2 
+  D. If a = 2 and b = 2, then
 1  1 + y2 1 + y2   π
= 2  + y  sin −1 (2x) + cos −1 ( y) + cos −1 (2xy) =
4
 y  1 − y2 y   2
  +  cos −1 (2x) − cos −1 ( y) = cos −1 (2xy)
2  ⇒
  y 1− y  
  ⇒ 2xy + 1 − 4x2 1 − y2 = 2xy
1/ 2
1  ⇒ (4x2 − 1) ( y2 − 1) = 0
=  2 ⋅ y2(1 − y4 ) + y4  =1
y  Hence, A → p; B → q; C → p; D → s
522 Inverse Circular Functions

13. We have,  7π (k + 1)π   7π k π  


sin  +  − + 
 x  10
  12 2   12 2  
E1 = x ∈ R : x ≠ 1 and > 0 = Σ
 x−1 
k=0  7 π kπ   7π (k + 1) π 
cos  +  cos  + 
x  12 2  12 2 
∴ E1 = >0
x−1  7π (k + 1) π  7π kπ  π π 
Q 12 + −

+  = and sin = 1

+ – +  2 12 2 2 2 
0 1  7π (k + 1) π   7 π kπ 
sin  +  cos  + 
E1 = x ∈ (− ∞ , 0) ∪ (1, ∞ )  12 2   12 2
 7 π kπ   7π (k + 1) π 
and − sin  +  cos  + 
10  12 2  12 2 
   x   = Σ
E 2 = x ∈ E1 : sin −1  log e    is a real number  k=0  7 π kπ   7π (k + 1) π 
  −   cos  +  cos  + 
 x 1   12 2  12 2 
x x 10 
 7π (k + 1) π   7 π kπ  
E 2 = − 1 ≤ log e ≤ 1 ⇒ e−1 ≤ ≤e = Σ tan  +  − tan  + 
x−1 x−1 k=0   12 2   12 2  
x
≥ e−1 ⇒
x 1
− ≥0  7π π   7π 
Now, = tan  +  − tan  
x−1 x−1 e  12 2   12 
ex − x + 1 x (e − 1) + 1  7π 2π   7π π 
⇒ ≥0⇒ ≥0 + tan  +  − tan  + 
e(x − 1) (x − 1) e  12 2  12 2 
+ – + .
–1 1 .
e–1 .
 7π 11π   7π 10 π 
 1  + tan  +  − tan  + 
⇒ x ∈  − ∞, ∪ (1, ∞ )  12 2   12 2 
 1 − e 
 7π 11 π  7π π π
x = tan  +  − tan = tan + cot
Also, ≤e  12 2  12 12 12
x−1
1 2
(e − 1)x − e = = =4
⇒ ≥0 π π π
x−1 sin cos sin
12 12 6
+ – +  1 10
 7 π kπ   7π (k + 1) π  
So, sec−1  Σ sec  +  sec  + 
1 e 4 k = 0  12 2   12 2  
e–1
= sec−1 (1) = 0
 e 
⇒ x ∈ (− ∞ , 1) ∪  , ∞ 15. We have,
 e − 1 
 ∞ ∞
 x 
i

 1   e  sin −1  Σ xi + 1 − x Σ   
E 2 =  −∞ , ∪ i = 1 i = 1  2 
So,
 1 − e   e − 1 , ∞

π  ∞  − x i ∞ 
∴ The domain of f and g are = − cos −1  Σ   − Σ (− x)i
2 i = 1  2  i = 1 
 1   e 
 −∞ , ∪  e − 1 , ∞  −x
1 − e  
 x  
 x⋅
−1 x2 2  π −1  2 (− x) 
⇒ sin  − = − cos  −
1 − x 1 − x 2 x 1 + x
x
and Range of is R+ − {1}
x−1   1 + 
 2  2 
⇒ Range of f is R − {0} or (−∞ , 0) ∪ (0, ∞ )  ∞ i+1 x2 
 π π  π   π Q i Σ
=1
x = x2 + x3 + x4 + ... =
1 − x
Range of g is − , − {0} or − , 0 ∪ 0,  
 2 2   2   2  using sum of infinite terms of GP 
Now, P → 4, Q → 2, R → 1, S → 1  x2 x2  π −1  x x 
⇒ sin −1  −  = − cos  −
7 π kπ   7π (k + 1) π  1 − x 2 − x 2  1 + x 2 + x 
14. Q Σ sec   
10
+  sec  + 
k=0  12 2   12 2   x2 x2   x x 
10 1 ⇒ sin −1  −  = sin −1  − 
= Σ 1 − x 2 − x   1 + x 2 + x
k=0  7 π kπ   7π (k + 1) π 
cos  +  cos  +   π 
 12 2  12 2  Q sin −1 x = − cos −1 x
 2 
Inverse Circular Functions 523

x2 x2 x x  1  1
⇒ − = − ⇒ f   = − sin sin −1 1 − 2 
1−x 2−x 1+ x 2+ x  5  5 
 2 − x−1 + x (2 + x − 1 − x)  2 6
⇒ x2   =x = − sin sin −1  =−
2 6
 (1 − x) (2 − x) (1 + x) (2 + x)  5  5
x 1
⇒ = or x = 0
2 − 3 x + x2 2 + 3 x + x 2
Topic 3 Sum and Difference Formulae
⇒ x3 + 3x2 + 2x = x2 − 3x + 2
⇒ x3 + 2x2 + 5x − 2 = 0 or x = 0 Key Idea Use formulae
Let f (x) = x3 + 2x2 + 5x − 2 1.
(i) sin− 1 x − sin− 1 y
f ′ (x) = 3x2 + 4x + 5
= sin− 1( x 1 − y 2 − y 1 − x 2 ) if x 2 + y 2 ≤ 1
f ′ (x) > 0, ∀ x ∈ R
or if xy > 0 and x 2 + y 2 > 1 ∀x , y ∈ [ − 1, 1]
∴ x3 + 2x2 + 5x − 2 has only one real roots
Therefore, total number of real solution is 2. (ii) sin− 1 x = cos − 1 1 − x 2 and
π
16. PLAN (iii) sin− 1θ + cos − 1θ =
(i) Using definition of f( x ) = cos −1( x ) , we trace the curve 2
f( x ) = cos −1(cos x ) .
We have,
(ii) The number of solutions of equations involving trigonometric
 12  3
and algebraic functions and involving both functions are sin − 1   − sin − 1  
found using graphs of the curves.  13  5

x, x ∈ [0, π]  12 2 2
if  3 3  12
2π − x, = sin − 1  1−  − 1−  
 if x ∈ [π , 2π]  13  5 5  13 
We know that, cos −1 (cos x) =   
− 2π + x, if x ∈ [2π, 3π]
4π − x, if x ∈ [3π, 4π] [Qsin − 1 x − sin − 1 y = sin − 1 (x 1 − y2 − y 1 − x2),
Y if x2 + y2 ≤ 1 or if xy > 0 and x2 + y2 > 1 ∀x, y ∈ [− 1, 1]]
 12 4 3 5 
π y = cos –1 (cos x) = sin − 1  × − × 
10 – π  13 5 5 13
y= –π
10 π/2 x 2π

–x  48 − 15 − 1  33
4π = sin − 1   = sin  
π–

(0, 1)  65   65
X
π π 3π 2π 5π 3π 10 4π 2
 33
2 2 2 = cos − 1 1 −  
10 – x x  65
y= =1–
10 10
10 − x 3136
From above graph, it is clear that y = and = cos − 1 [Qsin − 1 x = cos − 1 1 − x2]
10 4225
−1
y = cos (cos x) intersect at three distinct points, so  56 π  56
= cos − 1   = − sin − 1  
number of solutions is 3.  65 2  65
17. LHS = cos tan −1 [sin (cot−1 x)]  π
Q sin − 1 θ + cot− 1 θ =
 2 
  1  
= cos tan −1 sin sin −1 
  1 + x2   2. Given equation is
 
y
 cos − 1 x − cos − 1 = α, where − 1 ≤ x ≤ 1,
1  2
= cos  tan −1
 1 + x2  y
 − 2 ≤ y ≤ 2 and x ≤ ∴
2
x +1
2
 y 
= = RHS cos − 1  x + 1 − x2 1 − ( y / 2)2 = α
x2 + 2  2 
[Q cos − 1 x − cos − 1 y = cos − 1 (xy + 1 − x2 1 − y2),
18. Let f (x) = cos ( 2 cos −1 x + sin −1 x)
 π  π | x|,| y| ≤ 1 and x + y ≥ 0]
= cos  cos −1 x +  Q cos −1 x + sin −1 x = xy
 2  2  ⇒ + 1 − x2 1 − ( y / 2)2 = cos α
2
= − sin (cos −1 x)
xy
⇒ f (x) = − sin (sin −1 1 − x2 ) ⇒ 1 − x2 1 − ( y / 2)2 = cos α −
2
524 Inverse Circular Functions

On squaring both sides, we get 1


⇒ 6x (x + 1) − 1 (x + 1) = 0, 0≤x<
 y2  x2 y2 xy 6
(1 − x2) 1 −  = cos 2 α + −2 cos α 1
 4 4 2 ⇒ (6x − 1)(x + 1) = 0, 0≤x<
6
y2 x2y2 x2 y2
⇒ 1 − x2 − + = cos 2 α + − xy cos α 1 1
4 4 4 ⇒ x= , − 1, 0≤x<
6 6
y2
⇒ x2 + − xy cos α = 1 − cos 2 α ⇒
1
x= , [Q 0 ≤ x <
1
]
4 6 6
⇒ 4x2 − 4xy cos α + y2 = 4 sin 2 α
So ‘A’ is a singleton set.
3. Given, α = cos −1   and β = tan −1  
3 1 ∴ The solution of given differential equation represents
 5  3 a circle with centre on the X-axis.
π 5. The graph of y = sin − 1 (sin x) is
where, 0 < α , β <
2

5
52 – 32 = 4

α
3
∴ x = sin − 1 (sin 10) = − 10 + 3π ...(i)
4
Clearly, α = tan −1
and the graph of y = cos − 1 (cos x) is
3
 4 1  Y
 − 

y=
y=

π+
−1 4 −1 1 −1  3 3  x
So, α − β = tan − tan = tan

–x
y=

–2

+
–x
 4 1 

y=


3 3
 1 +  ×  
 3 3  O π 2π 3π 10 4π
X

 −1 −1 −1 x − y 
Q tan x − tan y = tan 1 + xy , if xy > − 1
  ∴ y = cos − 1 (cos 10) = − 10 + 4π ...(ii)
−1 1 −1 9 Now, from Eqs. (i) and (ii),
= tan = tan
1+
4 13 y − x = (− 10 + 4π) − (− 10 + 3π) = π
9  2x 
6. Given, tan −1 y = tan −1 x + tan −1  
250  1 − x2
2 =
3
2 1
9+ 9  2x 
α –β
 x+ 2 
1  1−x 
13 where|x| < ⇒ tan −1 y = tan −1  
3 1 − x  2x  

9 9   1 − x2 
= sin−1 = sin−1 
9 + 13
2 2 250
 −1 −1 −1  x + y
−1 9  Q tan x + tan y = tan  ,
 1 − xy
= sin   
 5 10 

where x > 0, y > 0 and xy < 1
4. Given equation is

π
tan − 1 (2x) + tan −1 (3x) = , x≥0  x − x3 + 2x 
4 = tan −1  
5x π  1 − x2 − 2 x 2 
⇒ tan −1 = , 6 x2 < 1
1 − 6 x2 4  3x − x3 
 x + y tan −1 y = tan −1  
[Q tan − 1 x + tan − 1 y = tan −1   , xy < 1]  1 − 3x2 
 1 − xy
5x 1 3x − x3
⇒ = 1 , x2 < ⇒ y=
1 − 6 x2 6 1 − 3 x2
1 1
⇒ 6x2 + 5x − 1 = 0, 0 ≤ x < [Q x ≥ 0] |x| <
6 3
1 1 1
⇒ 6x + 6x − x − 1 = 0, 0 ≤ x <
2
⇒ − <x<
6 3 3
Inverse Circular Functions 525

Let x = tan θ   3 2 
  +    17  17
π π = tan tan  4 3   = tan tan −1    =
−1
⇒ − <θ < 3 2  6  6
6 6  1 − ⋅   
  4 3 
∴ tan −1 y = θ + tan −1 (tan 2θ ) = θ + 2θ = 3θ
⇒ y = tan 3θ   1  
 −1  1  π  −1
 2⋅  π 
3 tan θ − tan θ 3
10. tan 2 tan   −  = tan tan  5  − 
⇒ y=   5 4  1 − 1  4 
1 − 3 tan 2 θ 
  25  
3x − x3   5  π
⇒ y= = tan tan −1   − 
1 − 3 x2   12 4 
 23  n    5   π
7. We have, cot  ∑ cot−1 1 + tan tan −1    − tan  

∑ 2k    12   4

n = 1 k =1  =
  5  π
 23  1 + tan tan −1    tan
  12 
⇒ cot  ∑ cot−1 (1 + 2 + 4 + 6 + 8 + K + 2n ) 4
n = 1  5
−1
7
 23  = 12 =−
⇒ cot  ∑ cot−1 {1 + n (n + 1)} 1+
5
⋅1 17
n = 1  12
 23 1  11. Given,
⇒ cot  ∑ tan −1  a (a + b + c) b (a + b + c)
n = 1 1 + n (n + 1)  θ = tan −1 + tan −1
 bc ac
 23  (n + 1) − n 
⇒ cot  ∑ tan −1  c (a + b + c)
 + tan −1
n = 1 1 + n (n + 1)  ab
 23   −1 −1 −1 −1  x + y + z − xyz  
⇒ cot  ∑ (tan −1 (n + 1) − tan −1 ln n ) Q tan x + tan y + tan z = tan  1 − xy − yz − zx 
 
n = 1 
  a b c  
⇒ cot [(tan −1 2 − tan −1 1) + (tan −1 3 − tan −1 2) a + b+ c + + 
  bc ca ab  
 
+ (tan −1 4 − tan −1 3)] + K + (tan −1 24 − tan −1 23)]
 − (a + b + c) a + b + c 
⇒ cot (tan −1 24 − tan −1 1) −1  abc 
= tan  
 24 − 1   −1 23
⇒ cot  tan −1  = cot  tan   1 − (a + b + c)  1 + 1 + 1 
 1 + 24 ⋅ (1)  25   a b c 
 25 25
= cot  cot−1  =  a + b+ c a + b+ c
 23 23 (a + b + c) − (a + b + c)
−1
 abc abc 
= tan  
8. Since, x , y and z are in an AP.  (a + b + c) (ab + bc + ca ) 
1−
∴ 2y = x + z  abc 
Also, tan − 1 x, tan − 1 y and tan − 1 z are in an AP. ⇒ θ = tan −1 0
∴ 2 tan − 1 y = tan − 1 x + tan − 1 (z ) ⇒ tan θ = 0
 2y   x+ z −1 π
⇒ tan − 1   = tan − 1   12. Given, tan 2x + tan −1 3x =
1 − y 
2
 1 − xz  4
 2x + 3x π 5x
x+ z x+ z ⇒ tan −1   = ⇒ =1
⇒ = ⇒ y2 = xz  1 − 6x2  4 1 − 6 x2
1− y 2
1 − xz
Since x, y and z are in an AP as well as in a GP. ⇒ 6 x2 + 5 x − 1 = 0
∴ x= y=z ⇒ (x + 1) (6x − 1) = 0
1
 2   2  ⇒ x = −1 or
9. tan cos −1   + tan −1    = tan tan −1   + tan −1   
4 3
 5  3   4  3  6
 
But x = −1 does not satisfy the given equation.
 –1  4 −1  3   1
Q cos  5 = tan  4  ∴ We take x =
  6
23
Properties of Triangles
Topic 1 Applications of Sine, Cosine,
Projection and Half Angle Formulae
Objective Questions I (Only one correct option) 6. In a ∆ ABC, among the following which one is true?
B + C
= a sin 
A
1. The angles A, B and C of a ∆ABC are in AP and (a) (b + c) cos  (2005, 1M)
2  2 
a : b = 1 : 3. If c = 4 cm, then the area (in sq cm) of
B + C
(b) (b + c) cos 
A
this triangle is  = a sin
(2019 Main, 10 April II)  2  2
2 4
(c) (b − c) cos   = a cos  
(a) (b) 4 3 (c) 2 3 (d) B – C A
3 3   
 2   2
b+ c c+ a a+ b B − C
2. Given, = = for a ∆ABC with usual (d) (b − c) cos
A
= a sin  
11 12 13 2  2 
cos A cos B cos C
notation. If = = , then the ordered 7. If the angles of a triangle are in the ratio 4 : 1 : 1, then
α β γ
the ratio of the longest side to the perimeter is
triad (α , β , γ ) has a value (2019 Main, 11 Jan II) (2003, 1M)
(a) (19, 7, 25) (b) (3, 4, 5) (a) 3 : (2 + 3 ) (b) 1 : 3 : 2
(c) (5, 12, 13) (d) (7, 19, 25) (c) 1 : 2 + 3 (d) 2 : 3
3. In a triangle, the sum of lengths of two sides is x and 1 
the product of the lengths of the same two sides is y. 8. In a ∆ABC, 2ac sin  ( A − B + C ) is equal to
If x 2 − c2 = y, where c is the length of the third side of 2  (2000, 2M)
(a) a + b − c
2 2 2
(b) c + a − b2
2 2
the triangle, then the circumradius of the triangle is
(2019 Main, 11 Jan I) (c) b2 − c2 − a 2 (d) c2 − a 2 − b2
(a)
c
(b)
c π  P Q
3 3 9. In a ∆PQR, ∠R = , if tan   and tan   are the
2  2  2
3 y
(c) y (d) roots of the equation ax + bx + c = 0 ( a ≠ 0), then
2
2 3
(a) a + b = c (b) b + c = a (1999, 2M)
4. ABCD is a trapezium such that AB and CD are (c) a + c = b (d) b = c
parallel and BC ⊥ CD, if ∠ ADB = θ, BC = p and
CD = q, then AB is equal to (2013 Main) 10. If in a ∆PQR, sin P, sin Q, sin R are in AP, then
(a) the altitudes are in AP (1998, 2M)
( p + q ) sin θ
2 2
p + q cos θ
2 2
(a) (b) (b) the altitudes are in HP
p cos θ + q sin θ p cos θ + q sin θ
(c) the medians are in GP
p 2 + q2 ( p 2 + q2 ) sin θ (d) the medians are in AP
(c) (d)
p cos θ + q sin θ
2 2
( p cos θ + q sin θ)2 π π
11. In a ∆ABC, ∠B = and ∠C = . Let D divides BC
5. If the angles A, B and C of a triangle are in an 3 4
arithmetic progression and if a, b and c denote the sin ∠BAD
lengths of the sides opposite to A, B and C internally in the ratio 1 : 3, then is equal to
sin ∠CAD
respectively, then the value of the expression
a c 1 1 (1995, 2M)
sin 2 C + sin 2 A is (2010) (a) (b)
c a 6 3
1 3 1 2
(a) (b) (c) 1 (d) 3 (c) (d)
2 2 3 3
Properties of Triangles 527

Objective Questions II Fill in the Blanks


(One or more than one correct option) 17. In a ∆ABC, AD is the altitude from A. Given
1 abc
12. In a ∆ PQR , P is the largest angle and cos P = . b > c, ∠ C = 23° and AD = , then ∠B = … .
3 b − c2
2
(1994, 2M)
Further in circle of the triangle touches the sides 18. If in a ∆ABC,
PQ , QR and RP at N , L and M respectively, such 2 cos A cos B 2 cos C a b
that the lengths of PN , QL and RM are consecutive + + = +
a b c bc ca
even integers. Then, possible length(s) of the side(s)
of the triangle is (are) (2017 Main) Then, the value of the ∠A is …… . (1993, 2M)

(a) 16 (b) 18 (c) 24 (d) 22


Analytical & Descriptive Questions
π
13. Let ABC be a triangle such that ∠ACB = . If a, b 19. Let A1 , A2, …, An be the vertices of an n-sided regular
6 1 1 1
and c denote the lengths of the sides opposite to A, B polygon such that = + . Find the
A1 A2 A1 A3 A1 A4
and C respectively. Then, the value(s) of x for which (1994, 4M)
a = x 2 + x + 1, b = x 2 − 1 and c = 2x + 1 is (are) (2010) value of n.
(a) − (2 + 3 ) (b) 1 + 3 (c) 2 + 3 (d) 4 3 20. The sides of a triangle are three consecutive natural
14. In a ∆ABC with fixed base BC, the vertex A moves numbers and its largest angle is twice the smallest
A one. Determine the sides of the triangle. (1991, 4M)
such that cos B + cos C = 4 sin2 . If a, b and c denote
2 21. In a ∆ABC, the median to the side BC is of length
the lengths of the sides of the triangle opposite to the 1
angles A, B and C respectively, then (2009)
and it divides the ∠A into angles 30° and
11 − 6 3
(a) b + c = 4a (b) b + c = 2a
(c) locus of point A is an ellipse 45°. Find the length of the side BC. (1985, 5M)
(d) locus of point A is a pair of straight line b+ c c+ a
22. With usual notation, if in a ∆ABC =
15. Internal bisector of ∠A of ∆ABC meets side BC at D. 11 12
A line drawn through D perpendicular to AD a+b cos A cos B cos C
= , then prove that = = .
intersects the side AC at E and side AB at F. If a, b, c 13 7 19 25
represent sides of ∆ ABC, then (2006, 5M) (1984, 4M)
2bc A
(a) AE is HM of b and c (b) AD = cos 23. ABC is a triangle. D is the middle point of BC. If AD
b+ c 2
is perpendicular to AC, then prove that
4bc A
(c) EF = sin (d) ∆ AEF is isosceles 2(c2 − a 2)
b+ c 2 cos A cos C = .
3ac (1980, 3M)
16. There exists a ∆ABC satisfying the conditions
π π (1986, 2M) 24. If in a triangle ABC, a = 1 + 3 cm, b = 2 cm and
(a) b sin A = a, A < (b) b sin A > a, A >
2 2 ∠C = 60° , then find the other two angles and the
π π third side.
(c) b sin A > a , A < (d) b sin A < a, A < , b > a (1978, 3M)
2 2

Topic 2 Applications of Area, Napier’s Analogy


and Solution of a Triangle
Objective Questions I (Only one correct option) the triangle opposite to the angles at P , Q and R,
1. With the usual notation, in ∆ABC, if 2 sin P − sin 2P
respectively. Then, equals (2012)
∠A + ∠B = 120°, a = 3 + 1 and b = 3 − 1, then the 2 sin P + sin 2P
ratio ∠A : ∠B, is
2 5
(2019 Main, 10 Jan II) 3 45  3   45 
(a) (b) (c)   (d)  
(a) 7 : 1 (b) 3 : 1 4∆ 4∆  4∆   4∆ 
(c) 9 : 7 (d) 5 : 3
3. In radius of a circle which is inscribed in a isosceles
7 triangle one of whose angle is 2π / 3, is 3, then area
2. If PQR is a triangle of area ∆ with a = 2, b = and
2 of triangle (in sq units) is (2006, 2M)
5 (b) 12 − 7 3
c = , where a, b and c are the lengths of the sides of (a) 4 3
2 (c) 12 + 7 3 (d) None of these
528 Properties of Triangles

4. The sides of a triangle are in the ratio 1 : 3 : 2 , then π


11. Let A, B, C be three angles such that A = and
the angles of the triangle are in the ratio (2004, 1M) 4
(a) 1 : 3 : 5 (b) 2 : 3 : 2 (c) 3 : 2 : 1 (d) 1 : 2 : 3 tan B, tan C = p. Find all positive values of p such
that A, B, C are the angles of triangle. (1997C, 5M)
Objective Questions II 12. Consider the following statements concerning a
(One or more than one correct option) ∆ABC
(i) The sides a , b, c and area of triangle are rational.
5. Let x , y and z be positive real numbers. Suppose x , y
B C
and z are the lengths of the sides of a triangle (ii) a , tan , tan are rational.
opposite to its angles X , Y and Z, respectively. If 2 2
X Z 2y (iii) a , sin A , sin B, sin C are rational.
tan + tan = , Prove that (i) ⇒ (ii) ⇒ (iii) ⇒ (i)
2 2 x+ y+z (1994, 5M)
13. In a triangle of base a, the ratio of the other two sides
then which of the following statements is/are TRUE?
is r(< 1). Show that the altitude of the triangle is less
(a) 2Y = X + Z (b)Y = X + Z (2020 Adv.) ar
X x than or equal to . (1991, 4M)
(c) tan = (d) x + z − y = xz
2 2 2
2 y+ z 1 − r2
14. If in a ∆ABC, cos A cos B + sin A sin B sin C = 1, then
Fill in the Blank show that a : b : c = 1 : 1 : 2. (1986, 5M)
6. If the angle of a triangle are 30° and 45° and the
3
included side is ( 3 + 1) cm, then the area of the 15. For a ∆ABC, it is given that cos A + cos B + cos C = .
2
triangle is … . (1988, 2M)
Prove that the triangle is equilateral. (1984, 4M)
7. The set of all real numbers a such that a 2 + 2a , 2a + 3 16. If p1 , p2 , p3 are the altitudes of a triangle from the
and a 2 + 3a + 8 are the sides of a triangle is …… vertices A, B, C and ∆ is the area of the triangle, then
(1985, 2M) prove that
1 1 1 2ab C
Analytical & Descriptive Questions + − = cos 2
p1 p2 p3 (a + b + c)∆ 2 (1978, 3M)
8. If ∆ is the area of a triangle with side lengths a , b, c,
then show that 17. If p1 , p2 , p3 are the perpendiculars from the vertices
1 of a triangle to the opposite sides, then prove that
∆≤ ( a + b + c) abc
4 a 2b2c2
p1 p2p3 = (1978, 3M)
Also, show that the equality occurs in the above 8R3
inequality if and only if a = b = c. (2001, 6M)

9. Prove that a ∆ABC is equilateral if and only if


Integer & Numerical Answer Type Question
tan A + tan B + tan C = 3 3. (1998, 8M) 18. Let ABC and ABC′ be two non-congruent triangles
with sides AB = 4 , AC = AC ′ = 2 2 and angle
10. Show that for any triangle with sides a , b, c
B = 30° .The absolute value of the difference between
3( ab + bc + ca ) ≤ ( a + b + c)2 ≤ 4( ab + bc + ca ).
(1979, 3M)
the areas of these triangles is (2009)

Topic 3 Circumcircle, Incircle, Escribed, Orthocentre


and Centroid of a Triangle
Objective Questions I (Only one correct option) 2. Let the equations of two sides of a triangle be
3x − 2 y + 6 = 0 and 4x + 5 y − 20 = 0. If the
1. Two vertices of a triangle are (0, 2) and
orthocentre of this triangle is at (1, 1) then the
(4, 3). If its orthocentre is at the origin, then its third
equation of its third side is (2019 Main, 9 Jan II)
vertex lies in which quadrant? (2019 Main, 10 Jan II)
(a) 122 y − 26x − 1675 = 0 (b) 26x − 122 y − 1675 = 0
(a) Fourth (b) Third
(c) 122 y + 26x + 1675 = 0 (d) 26x + 61y + 1675 = 0
(c) Second (d) First
Properties of Triangles 529

3. In a triangle, the sum of two sides is x and the 10. In a ∆ PQR, let ∠PQR = 30° and the sides PQ and QR
product of the same two sides is y. If x 2 − c2 = y, have lengths 10 3 and 10, respectively. Then, which
where c is the third side of the triangle, then the of the following statement(s) is (are) TRUE? (2018 Adv)
ratio of the inradius to the circumradius of the (a) ∠QPR = 45°
triangle is (2014 Adv.) (b) The area of the ∆ PQR is 25 3 and ∠QRP = 120°
(a)
3y
(b)
3y (c) The radius of the incircle of the ∆ PQR is 10 3 − 15
2x (x + c) 2 c (x + c) (d) The area of the circumcircle of the ∆ PQR is 100 π
3y 3y
(c) (d) 11. In a ∆XYZ, let x , y , z be the lengths of sides opposite
4x (x + c) 4 c (x + c) to the angles X , Y , Z respectively and 2s = x + y + z.
s−x s− y s−z
4. Which of the following pieces of data does not If = = and area of incircle of the
uniquely determine an acute angled ∆ABC (R being 4 3 2

the radius of the circumcircle)? (2002, 1M) ∆XYZ is , then (2016 Adv.)
(a) a , sin A , sin B (b) a , b, c 3
(c) a , sin B , R (d) a , sin A , R (a) area of the ∆XYZ is 6 6
35
5. In a ∆ABC, let ∠ C = π / 2. If r is the inradius and R is (b) the radius of circumcircle of the ∆XYZ is 6
6
the circumradius of the triangle, then 2 (r + R ) is X Y Z 4
equal to (2000, 2M)
(c) sin sin sin =
2 2 2 35
(a) a + b (b) b + c (c) c + a (d) a + b + c X +Y 3
(d) sin 2   =
 2  5
Passage Based Problems
12. A straight line through the vertex P of a ∆PQR
Consider the circle x 2 + y 2 = 9 and the parabola intersects the side QR at the point S and the
y 2 = 8x. They intersect at P and Q in the first and the circumcircle of the ∆PQR at the point T . If S is not
fourth quadrants, respectively. Tangents to the the centre of the circumcircle, then (2008, 4M)
1 1 2
circle at P and Q intersect the X-axis at R and (a) + <
tangents to the parabola at P and Q intersect the PS ST QS × SR
X-axis at S. 1 1 2
(2007, 8M) (b) + >
PS ST QS × SR
6. The radius of the incircle of ∆PQR is 1 1 4
8 (c) + <
(a) 4 (b) 3 (c) (d) 2 PS ST QR
3
1 1 4
(d) + >
7. The radius of the circumcircle of the ∆PRS is PS ST QR
(a) 5 (b) 3 3 (c) 3 2 (d) 2 3
8. The ratio of the areas of ∆PQS and ∆PQR is Fill in the Blanks
(a) 1 : 2 (b) 1 : 2 13. In a ∆ABC, a : b : c = 4 : 5 : 6. The ratio of radius of the
(c) 1 : 4 (d) 1 : 8 circumcircle to that of the incircle is… . (1996, 1M)
14. The sides of a triangle inscribed in a given
Objective Questions II circle subtend angles α, β and γ at the centre. The
(One or more than one correct option) minimum value of the arithmetic mean of
 π  π  π
9. In a non-right-angled triangle ∆PQR, let p, q , r cos α +  , cos β +  and cos  γ +  is ….
denote the lengths of the sides opposite to the angles  2   2   2 (1987, 2M)
at P , Q , R respectively. The median from R meets the 15. A polygon of nine sides, each of length 2, is inscribed
side PQ at S, the perpendicular from P meets the in a circle. The radius of the circle is… . (1987, 2M)
side QR at E, and RS and PE intersect at O. If p = 3,
q = 1, and the radius of the circumcircle of the ∆PQR Analytical & Descriptive Questions
equals 1, then which of the following options is/are 16. Circle with radii 3, 4 and 5 touch each other
correct? (2019 Adv.) externally, if P is the point of intersection of tangents
1 to these circles at their points of contact. Find the
(a) Length of OE =
6 distance of P from the point of contact. (2005, 2M)
7
(b) Length of RS = 17. I n is the area of n sided regular polygon inscribed in
2 a circle of unit radius and On be the area of the
3 polygon circumscribing the given circle, prove that
(c) Area of ∆SOE =
12  2
O  2I 
(d) Radius of incircle of ∆PQR =
3
(2 − 3) I n = n 1 + 1 −  n   .
2 2   n   (2003, 5M)
 
530 Properties of Triangles

18. Let ABC be a triangle with incentre I and inradius r. the ∆BIO is a right angled triangle if and only if b is
Let D , E , F be the feet of the perpendiculars from I to the arithmetic mean of a and c. (1999, 10M)
the sides BC, CA and AB, respectively. If r1 , r2 and r3 20. The exradii r1 , r2 , r3 of ∆ABC are in HP, show that its
are the radii of circles inscribed in the quadrilaterals sides a , b, c are in AP. (1983, 3M)
AFIE, BDIF and CEID respectively, then prove that
r1 r2 r3 r1r2r3 Integer & Numerical Answer Type Question
+ + = .
r − r1 r − r2 r − r3 (r − r1 ) (r − r2) (r − r3 ) (2000 3M) 21. Consider a ∆ABC and let a, b and c denote the
lengths of the sides opposite to vertices A, B and C,
19. Let ABC be a triangle having O and I as its
respectively. a = 6, b = 10 and the area of the
circumcentre and incentre, respectively. If R and r
triangle is 15 3. If ∠ACB is obtuse and if r denotes
are the circumradius and the inradius respectively,
then prove that ( IO )2 = R 2 − 2Rr. Further show that the radius of the incircle of the triangle, then r 2 is
equal to……

Answers
Topic 1 11. p ∈ ( − ∞, 0 ) ∪ [3 + 2 2, ∞ )
1. (c) 2. (d) 3. (b) 4. (a) 18. 4 sq units
5. (d) 6. (d) 7. (a) 8. (b)
9. (a) 10. (b) 11. (a) 12. (b, d) Topic 3
13. (b) 14. (b, c) 15. (a, b, c, d) 16. (a, d) 1. (c) 2. (b) 3. (b) 4. (c)
17. 113° 18. 90° 19. n = 7 20. 4, 5, 6 units 5. (a) 6. (d) 7. (b) 8. (c)
21. 2 24. C = 6 , ∠B = 45 ° and ∠A = 75 ° 9. (a,b,d) 10. (b,c,d) 11. (a,c,d) 12. (b,d)
16 3
Topic 2 13. 14. − 15. cosec 20° 16. 5
7 2
1. (a) 2. (c) 3. (c) 4. (d)
21. 3
1+ 3
5. (c) 6. sq cm 7. a > 5
2

Hints & Solutions


Topic 1 Applications of Sine, Cosine, So, ∠C = 90º
Projection and Half Angle ∴ From sine rule,
a b c
Formulae = =
sin A sin B sin C
1. It is given that angles of a ∆ABC are in AP. a b 4
So, ∠A + ∠B + ∠C = 180º ⇒ = = [Q c = 4 cm]
1 3 1
⇒ ∠B − d + ∠B + ∠B + d = 180º 2 2
[if ∠A , ∠B and ∠C are in AP, then it taken as ∠B − d, ⇒ a = 2 cm, b = 2 3 cm
∠B, ∠B + d respectively, where d is common difference 1 1
of AP] ∴ Area of ∆ABC = ab sin C = × 2 × 2 3 × 1
2 2
⇒ 3∠B = 180º ⇒ ∠B = 60º …(i)
= 2 3 sq cm
a 1
and = [given] b+ c c+ a a + b
b 3 2. Given, = = = λ (say)
11 12 13
sin A 1
⇒ =
sin B 3 A
 sin A sin B sin C 
by sine rule a = b = c 
c b
sin A 1  3
⇒ = Q sin B = sin 60° = 
3 3  2 
2 B a C
1
⇒ sin A = ⇒ ∠A = 30º b + c = 11λ ,c + a = 12λ and a + b = 13λ ...(i)
2
Properties of Triangles 531

⇒ 2(a + b + c) = 36λ 4. Applying sine rule in ∆ABD,


⇒ a + b + c = 18λ ...(ii)
q C
From Eqs. (i) and (ii), we get D
a = 7λ, b = 6λ, c = 5λ θ

p
2
Now,

+
q
2
b2 + c2 − a 2 λ2[36 + 25 − 49] 12 1 P
cos A = = = =
2bc 60λ2 60 5
a 2 + c2 − b2 λ2[49 + 25 − 36] 19 π−(θ+α)
cos B = = = A B
2ac 70λ2 35
a 2 + b2 − c2
and cos C = AB p2 + q 2
2ab =
sin θ sin { π − (θ + α )}
λ2[49 + 36 − 25]
= AB p2 + q 2
84λ2 ⇒ =
60 5 sin θ sin(θ + α )
= =
84 7 p2 + q2 sin θ  q 
1 7 ⇒ AB = Q cos α = 
Thus, cos A = = , sin θ cos α + cos θ sin α  p + q 
2 2
5 35
19 25 ( p2 + q2)sin θ p
cos B = , cos C = = and sin α =
35 35 p cos θ + q sin θ p2 + q 2
cos A cos B cos C 1
= = = Alternate Solution
7 19 25 35
Let AB = x
⇒ (α , β , γ ) = (7, 19, 25)
q
a b c C
3. We know that, = = = 2R D α
sin A sin B sin C
θ
and given that, a + b = x, ab = y and x2 − c2 = y

p
2
+
A

q
2
p p

π−(θ+α) α
A B
c O
b x−q q
M
x
R
p
B C In ∆DAM, tan(π − θ − α ) =
a x−q
p
⇒ tan(θ + α ) =
∴ (a + b) − c = ab
2 2 q−x
⇒ a 2 + b2 − c2 = − 2ab + ab ⇒ q − x = p cot(θ + α )
⇒ a + b − c = − ab
2 2 2 ⇒ x = q − p cot(θ + α )
a +b −c
2 2
− ab 2
1  cot θ cot α − 1  q
⇒ = =− = q− p  Q cot α = p 
2ab 2ab 2  cot α + cot θ   
∴ cos C = −
1  q 
 cot θ – 1
2 p  q cot θ − p 
= q− p  = q− p 
⇒ C = 120°  q   q + p cot θ 
 + cot θ 
a 2 + b2 − c2  p 
[using cosine rule, cos C = ]
2ab  q cos θ − p sin θ 
= q− p 
c  q sin θ + p cos θ 
Now, = 2R
sin C q2 sin θ + pq cos θ − pq cos θ + p2 sin θ
1 c c 2 ⇒ x=
⇒ R= = p cos θ + q sin θ
2 sin (120° ) 2 3
( p2 + q2)sin θ
c ⇒ AB =
∴ R= p cos θ + q sin θ
3
532 Properties of Triangles

5. Since, A, B, C are in AP. 8. We know that, A + B + C = 180°


⇒ 2 B = A + C i.e. ∠ B = 60º ⇒ A + C − B = 180 − 2B
a c 1 
∴ (2 sin C cos C ) + (2 sin A cos A ) Now, 2ac sin  ( A − B + C ) = 2ac sin (90° − B)
c a 2 
= 2 k (a cos C + c cos A ) 2 ac ⋅ (a 2 + c2 − b2)
  = 2ac cos B = [by cosine rule]
2ac
using, a = b = c = 1 
 sin A sin B sin C k  = a 2 + c2 − b2
= 2 k (b) 9. It is given that, tan (P / 2) and tan (Q / 2) are the roots of
= 2 sin B [using b = a cos C + c cos A ] the quadratic equation ax2 + bx + c = 0
= 3 and ∠R = π /2
∴ tan (P / 2) + tan (Q / 2) = − b/a
6. Let a , b, c are the sides of ∆ABC.
and tan (P / 2) tan (Q / 2) = c / a
b + c k (sin B + sin C )
Now, = [by sine rule] Since, P + Q + R = 180°
a k sin A
⇒ P + Q = 90°
 B + C  B − C  B − C
2 sin   cos   cos   P+Q
 2   2  b+ c  2  ⇒ = 45°
= ⇒ = 2
A A a A
2 sin cos sin  P + Q
2 2 2 ⇒ tan   = tan 45°
 2 
 B − C
sin   tan (P / 2) + tan (Q / 2)
b−c  2  ⇒ =1
Also, = 1 − tan (P / 2) tan (Q / 2)
a A
cos − b/a
2 ⇒ =1
A  B − C 1 − c/a
or (b − c) cos = a sin  
2  2  −b / a
⇒ =1
a−c
7. Given, ratio of angles are 4 : 1: 1. a
⇒ 4 x + x + x = 180° ⇒ x = 30° −b
⇒ =1
∴ ∠ A = 120°, ∠B = ∠C = 30° a−c
C ⇒ −b = a − c
⇒ a+ b=c
30°
10. By the law of sine rule,
a a b c
= = =k [say]
b sin P sin Q sin R
120 ° P
30°
A c B

a
Thus, ratio of longest side to perimeter = c b
a+ b+ c
p2 p3
Let b=c=x p1
⇒ a 2 = b2 + c2 − 2bc cos A [by cosine rule] Q a R
⇒ a 2 = 2x2 − 2x2 cos A 1
Also, ap1 = ∆
= 2x2(1 − cos A ) 2
⇒ a 2 = 4x2 sin 2 A / 2 2∆
⇒ = p1
a
⇒ a = 2x sin A / 2 ⇒ a = 2x sin 60° = 3x 2∆
Thus, required ratio ⇒ p1 =
k sin P
a 3x 2∆ 2∆
= = Similarly, p2 = and p3 =
a + b + c x + x + 3x k sin Q k sin R
3
= = 3 :2 + 3 Since, sin P, sin Q and sin R are in AP, hence p1 , p2, p3
2+ 3 are in HP.
Properties of Triangles 533

11. In ∆ABD, applying sine rule, we get a 2 + b2 − c2


13. Using, cos C =
A 2ab
β
α A

2x
1

+
π/3 π/4

x 2-

1=
b=

c
B x D 3x C
AD x 30º
=
sin π / 3 sin α C a = x2 + x + 1 B
3
⇒ AD = x sin α …(i)
2 3 (x2 + x + 1)2 + (x2 − 1)2 − (2x + 1)2
⇒ =
and in ∆ACD, applying sine rule, we get 2 2 (x2 + x + 1) (x2 − 1)
AD
=
3x ⇒ (x + 2) (x + 1) (x − 1) x + (x2 − 1)2 = 3 (x2 + x + 1) (x2 − 1)
sin π / 4 sin β ⇒ x2 + 2x + (x2 − 1) = 3 (x2 + x + 1)
3 ⇒ (2 − 3 ) x2 + (2 − 3 ) x − ( 3 + 1) = 0
⇒ AD = x sin β …(ii)
2
⇒ x = − (2 + 3 )
3x 3x x=1+ 3
From Eqs. (i) and (ii), = and
2 sin α 2 sin β
But x = − (2 + 3 )
sin α 1
⇒ = ⇒ c is negative.
sin β 6
∴ x = 1 + 3 is the only solution.
12. PLAN Whenever cosine of angle and sides are given or to find out, we A
should always use Cosine law. 14. Given, cos B + cos C = 4 sin 2
b2 + c 2 − a2 a2 + c 2 − b2 2
i.e. cos A = , cos B =
2 bc 2 ac A
a2 + b2 − c 2
and cosC =
2 ab

P c b

n n
c b
B a C

(n + 2) Fixed base
(n + 4)

 B + C  B −C 2 A
Q R ⇒ 2 cos   cos   = 4 sin
 2   2  2
(n + 2) (n + 4)
a A  B −C A
⇒ 2 sin cos   − 2 sin  = 0
b2 + c2 − a 2 2   2  2
∴ cos P =
2bc  B −C  B + C A
⇒ cos   − 2 cos   = 0 as sin ≠ 0
1 (2n + 4)2 + (2n + 2)2 − (2n + 6)2  2   2  2
⇒ =
3 2(2n + 4) (2n + 2) B C B C
⇒ − cos cos + 3 sin sin = 0
 1  2 2 2 2
Q cos p = 3 , given  B C 1
⇒ tan tan =
4n 2 − 16 1 2 2 3
⇒ =
8(n + 1) (n + 2) 3 (s − a ) (s − c) (s − b) (s − a ) 1
⇒ . =
n2 − 4 1 s (s − b) s (s − c) 3
⇒ =
2(n + 1) (n + 2) 3 s− a 1
⇒ =
(n − 2) 1 s 3
⇒ =
2(n + 1) 3 ⇒ 2s = 3a
⇒ 3n − 6 = 2n + 2 ⇒ n = 8 ⇒ b + c = 2a
∴ Sides are (2n + 2), (2n + 4), (2n + 6), i.e. 18, 20, 22. ∴ Locus of A is an ellipse.
534 Properties of Triangles

15. Since, ∆ABC = ∆ABD + ∆ACD AD


17. In ∆ADC , = sin 23°
1 1 A 1 A b
⇒ bc sin A = c AD sin + b AD sin A
2 2 2 2 2
A
b
c

A A
c 2 2 b 23°
D a C
B
E
⇒ AD = b sin 23°
B abc
D
C But AD = 2 [given]
a b − c2
abc
⇒ = b sin 23°
b2 − c2
a sin 23°
F
⇒ = …(i)
b −c
2 2
c
2bc A
⇒ AD = cos Again, in ∆ABC,
b+ c 2 sin A sin 23°
=
A 2bc a c
Again, AE = AD sec =
2 b+ c sin A a
⇒ = 2 [from Eq. (i)]
⇒ AE is HM of b and c. a b − c2
A a2
EF = ED + DF = 2DE = 2 AD tan ⇒ sin A =
2 b2 − c2
2bc A A 4bc A
=2 cos tan = sin k2 sin 2 A
b+ c 2 2 b+ c 2 ⇒ sin A =
k sin 2 B − k2 sin 2 C
2
Since, AD ⊥ EF and DE = DF and AD is bisector.
sin 2 A
⇒ ∆AEF is isosceles. ⇒ sin A =
sin B − sin 2 C
2
Hence, (a), (b), (c), (d) are correct answers.
sin 2 A
16. The sine formula is ⇒ sin A =
sin (B + C ) sin (B − C )
a b
=
sin A sin B sin 2 A
⇒ sin A =
⇒ a sin B = b sin A sin A ⋅ sin (B − C )
(a) b sin A = a ⇒ a sin B = a ⇒ sin (B − C ) = 1 [Q sin A ≠ 0]
π ⇒ sin (B − 23° ) = sin 90°
⇒ B=
2
⇒ B − 23° = 90°
π
Since, ∠ A < , therefore the triangle is possible. ∴ B = 113°
2
(b) and (c) b sin A > a 2 cos A cos B 2 cos C a b
18. Given, + + = + …(i)
⇒ a sin B > a a b c bc ca
⇒ sin B > 1 b2 + c2 − a 2
We know that, cos A =
∴ ∆ ABC is not possible. 2bc
(d) b sin A < a c2 + a 2 − b2
cos B =
⇒ a sin B < a 2ac
⇒ sin B < 1 a 2 + b2 − c2
and cos C =
⇒ ∠B exists. 2ab
Now, b > a On putting these values in Eq. (i), we get
⇒ B> A
2 (b2 + c2 − a 2) c2 + a 2 − b2
π +
Since, A < 2abc 2abc
2
∴ The triangle is possible. 2 ( a 2 + b2 − c2 ) a b
+ = +
Hence, (a) and (d) are the correct answers. 2abc bc ca
Properties of Triangles 535

2 (b2 + c2 − a 2) + c2 + a 2 − b2 + 2 (a 2 + b2 − c2)  2π   3π   π  3π 
⇒ ⇒ sin   ⋅ sin   = sin   sin  
2abc  n  n  n  n
a 2 + b2  π  2π 
= + sin   ⋅ sin  
abc  n  n
⇒ 3b2 + c2 + a 2 = 2a 2 + 2b2 ⇒ b2 + c2 = a 2  2π    3π   π   π  3π 
⇒ sin   sin  n  − sin  n   = sin  n  ⋅ sin  n 
Hence, the angle A is 90°.  n  
19. Let O be the centre and r be the radius of the circle  2π    3π + π   3 π − π  
⇒ sin   2 cos   sin  
passing through the vertices A1 , A2, …, An.  n    2n   2n  

Then, ∠A1OA2 =  π  3π 
n = sin   . sin  
 n  n
also OA1 = OA2 = r
 2π   2π   π  π  3π 
Again, by cos formula, we know that, ⇒ 2 sin   ⋅ cos   ⋅ sin   = sin   sin  
 n  n  n  n  n
 2π  OA1 + OA2 − A1 A2
2 2 2
cos   =  2π   2π   3π 
 n 2(OA1 )(OA2) ⇒ 2 sin   cos   = sin  
 n  n  n
O  4π   3π 
⇒ sin   = sin  
 n  n
2
4π 3π
r
n
r ⇒ =π−
n n

⇒ =π
A1 A2 n
⇒ n=7
 2π  r + r − A1 A2
2 2 2
⇒ cos   = 20. Let ABC be the triangle such that the lengths of its
 n 2(r )(r ) sides CA , AB and BC are (x − 1), x and (x + 1)
 2π  respectively, where x ∈ N and x > 1. Let ∠B = α be the
⇒ 2r 2 cos   = 2r 2 − A1 A22
 n smallest angle and ∠ A = 2α be the largest angle.
A
 2π 
⇒ A1 A22 = 2r 2 − 2r 2 cos  
 n
2 (x−1)
2  2π  
⇒ A1 A22 = 2r 1 − cos    x
  n 
 π
⇒ A1 A22 = 2r 2 ⋅ 2 sin 2   B C
 n (x +1)

 π
⇒ A1 A22 = 4r 2 sin 2   Then, by sine rule, we have
 n
sin α sin 2 α
 π =
⇒ A1 A2 = 2r sin   x−1 x+1
 n
sin 2α x + 1
 2π  ⇒ =
Similarly, A1 A3 = 2r sin   sin α x−1
 n
x+1
 3π  ⇒ 2 cos α =
and A1 A4 = 2r sin   x−1
 n
x+1
1 1 1 ∴ cos α = ...(i)
Since, = + [given] 2 (x − 1)
A1 A2 A1 A3 A1 A4
x2 + (x + 1)2 − (x − 1)2

1
=
1
+
1 Also, cos α = [using cosine law]
2r sin (π / n ) 2r sin (2π / n ) 2r sin (3π / n ) 2x (x + 1)
x+4
1 1 1 ⇒ cos α = ...(ii)
⇒ = + 2 (x + 1)
sin (π / n ) sin (2π / n ) sin (3π / n )
 3π   2π  From Eqs. (i) and (ii),
sin   + sin   x+1 x+4
1  n  n =
⇒ = 2 (x − 1) 2 (x + 1)
sin(π / n ) sin (2π / n ) sin (3π / n )
536 Properties of Triangles

⇒ (x + 1)2 = (x + 4) (x − 1) On solving Eqs. (i) and (ii), we get


⇒ x + 2x + 1 = x + 3x − 4
2 2
a = 7λ , b = 6λ and c = 5λ
⇒ x=5 b2 + c2 − a 2 36λ 2 + 25λ 2 − 49λ 2 1
cos A = = =
Hence, the lengths of the sides of the triangle are 4, 5 2bc 2 (30) λ 2 5
and 6 units.
a 2 + c2 − b2 49λ 2 + 25λ2 − 36λ 2 19
21. Let AD be the median to the base BC = a of ∆ABC cos B = = =
2ac 70λ 2 35
and let ∠ ADC = θ, then a 2 + b2 − c2 49λ 2 + 36λ 2 − 25λ 2 5
 a a a a cos C = = =
 +  cot θ = cot 30° − cot 45° 2ab 84λ 2 7
 2 2 2 2
1 19 5
3 −1 ∴ cos A : cos B : cos C = : : = 7 : 19 : 25
⇒ cot θ = 5 35 7
2
23. In ∆ADC, we have
A
B
30° 45°
a/2
D

90°–A
c
a/2

B C C
a/2 D a/2 b
A
Applying sine rule in ∆ ADC, we get
AD DC AC
= cos C =
sin (π − θ − 45° ) sin 45° CD
2b
a cos C = …(i)
AD a
⇒ = 2
sin (θ + 45° ) 1 Applying cosine formula in ∆ABC, we have
2 b2 + c2 − a 2
cos A =
a 2bc
⇒ AD = (sin 45° cos θ + cos 45° sin θ )
2 a 2 + b2 − c2
and cos C = ...(ii)
a  cos θ + sin θ  a 2ab
⇒ AD =   = (cos θ + sin θ )
2  2  2 From Eqs. (i) and (ii),
1 a  3 −1 2  a 2 + b2 − c2 2b
⇒ =  +  =
11 − 6 3 2  8 −2 3 8 − 2 3  2ab a

⇒ a 2 + b2 − c2 = 4b2
2 8 −2 3 2 8 −2 3
⇒ a= = ⇒ a 2 − c2 = 3b2 ...(iii)
( 3 + 1) 11 − 6 3 ( 3 + 1 )2 11 − 6 3
b + c − a 2b
2 2 2
Now, cos A cos C = ⋅
2 8 −2 3 2bc a
=
(4 + 2 3 ) (11 − 6 3 ) b2 + c2 − a 2 3b2 + 3 (c2 − a 2)
= =
ac 3ac
2 8 −2 3
=
44 − 24 3 + 22 3 − 36 (a 2 − c2) + 3 (c2 − a 2) 2 (c2 − a 2)
= =
3ac 3ac
8 −2 3
=2 =2 24. Given that,
8 −2 3
a = 1 + 3 , b = 2 and ∠C = 60°
b+ c c+ a a + b We have, c2 = a 2 + b2 − 2ab cos C
22. Let = = =λ
11 12 13 ⇒ c = (1 + 3 )2 + 4 − 2(1 + 3 ) ⋅ 2 cos 60°
2

⇒ (b + c) = 11λ , c + a = 12λ , a + b = 13λ …(i)


⇒ c2 = 1 + 2 3 + 3 + 4 − 2 − 2 3
⇒ 2 (a + b + c) = 36λ
⇒ c2 = 6
⇒ a + b + c = 18λ …(ii)
⇒ c= 6
Properties of Triangles 537

Using sine rule, 2 sin P − sin 2 P 2 sin P (1 − cos P )


∴ =
a b c 2 sin P + sin 2 P 2 sin P (1 + cos P )
= =
sin A sin B sin C 2 sin 2 (P /2)
= = tan 2 (P /2)
1+ 3 2 6 2 cos 2 (P /2)
⇒ = =
sin A sin B sin 60°
P
3

2 sin 60° 2 = 1
∴ sin B = = c = 5/2 b = 7/2
6 6 2
⇒ B = 45° Q R
a=2
∴ A = 180° − (60°+45° ) = 75°
(s − b) (s − c) (s − b) (s − c)
⇒ ×
s (s − a ) (s − b) (s − c)
Topic 2 Applications of Area, Napier’s 2 2
Analogy and Solution of a Triangle  7  5
4 −  4 −  2
[(s − b)2 (s − c)2]  2  2  3
1. For a ∆ABC, it is given that a = 3 + 1 , = = = 
∆2 ∆2  4∆ 
b = 3 − 1 and ∠A + ∠B = 120º
3. Let AB = AC = a and ∠ A = 120° .
A
1 2
∴ Area of triangle =
a sin 120°
2
where, a = AD + BD = 3 tan 30° + 3 cot 15°
c b
3
=1 +
tan (45° − 15° )
B C A
a

Clearly, ∠C = 60º [Q ∠A + ∠B + ∠C = 180º ] D 60° 60°


Now, by tangent law, we have a a
√3
A−B a−b C
tan = cot
2 a+b 2 O

( 3 + 1) − ( 3 − 1)  60º  15°
= cot  15°
( 3 + 1) + ( 3 − 1)  2 
B C
2
= cot (30º )  1 + tan 45° tan 30°
2 3 ⇒ a =1 + 3  
1  tan 45° − tan 30° 
= × 3 =1
3  3 + 1
=1 + 3   =4 + 2 3
 A − B  3 − 1
⇒tan   = 1 = tan 45º
 2 
∴ Area of a triangle
A−B
⇒ = 45º 1  3
2 = (4 + 2 3 )2   = (12 + 7 3 ) sq units
2  2
⇒ ∠A − ∠B = 90º
On solving ∠A − ∠B = 90º and ∠A + ∠B = 120º ,we get 4. Let a : b : c = 1 : 3 : 2 ⇒ c2 = a 2 + b2
∠A = 105º and ∠B = 15º ∴ Triangle is right angled at C.
So, ∠A : ∠B = 7 : 1
B
2. PLAN If ∆ABC has sides a, b, c .
A
c
c b a

B C
a
( s − b) ( s − a) C A
Then, tan ( A / 2) = b
s ( s − a)
7 5 or ∠C = 90°
2+ +
a+ b+ c 2 2 =4 a 1
where, s= ⇒ s= and =
2 2 b 3
538 Properties of Triangles

In ∆BAC , tan A =
a
=
1 ∴ Area of triangle
b 3 1 1 ( 3 + 1)sin 30° sin 45°
= ab sin 45° = ( 3 + 1)
⇒ A = 30° 2 2 sin 105°
and B = 60° [Q A + B = 90°] 1 ( 3 + 1 )2 1 1
= ⋅ ⋅ ⋅
∴ Ratio of angles, A : B : C = 30° : 60° : 90° = 1 : 2 : 3 2 (sin 45° cos 60° + cos 45° sin 60° ) 2 2
5. For a ∆XYZ, it is given that 1 (3 + 1 + 2 3 ) (4 + 2 3 )
X Z 2y = = ⋅2 2
tan + tan = 4 2  1 1 1 3  4 2 (1 + 3 )
2 2 x+ y+ z  ⋅ + ⋅ 
 2 2 2 2
∆ ∆ y
⇒ + = (1 + 3 )2 1 + 3
s(s − x) s(s − z ) s = = sq cm
(s − z + s − x) 2(1 + 3 ) 2
⇒ ∆ =y
(s − x) (s − z ) 7. Since, a 2 + 2a , 2a + 3 and a 2 + 3a + 8 form sides of a
⇒ ∆ = (s − x)(s − z ) triangle.
⇒ s(s − x)(s − y)(s − z ) = (s − x)2 (s − z )2 Now, a 2 + 3a + 8 < (a 2 + 2a ) + (2a + 3)
⇒ s2 − sy = s2 − (x + z )s + xz ⇒ a 2 + 3a + 8 < a 2 + 4a + 3
⇒ s(x + z − y) = xz ⇒ a >5 ...(i)
⇒ (x + z )2 − y2 = 2xz
Also, (a 2 + 3a + 8) + (2a + 3) > a 2 + 2a
⇒ x2 + z 2 = y 2
π ⇒ 3a > −11
∴ y= 11
2 ⇒ a>− ...(ii)
3
Q X +Y + Z = π
π Again, (a 2 + 3a + 8) + (a 2 + 2a ) > 2a + 3
⇒ X + Z = =Y
2 ⇒ 2a 2 + 3a + 5 > 0
⇒ X + Z =Y which is always true.
1−
z ∴ Triangle is formed, if a > 5
X 1 − cos x y 1
Q tan = = 8. Given, ∆≤ (a + b + c) abc
2 1 + cos x 1+
z
4
y 1
⇒ (a + b + c) abc ≥ 1
4∆
X
(a + b + c) abc
⇒ ≥1
16∆2
z y 2s abc
⇒ ≥1
16∆2
sabc
Y x Z ⇒ ≥1
8 ⋅ s (s − a ) (s − b) (s − c)
y−z y2 − z 2 abc
= = ⇒ ≥1
y+ z y+ z 8 (s − a ) (s − b) (s − c)
X x abc
tan = {Q x2 + z 2 = y2} ⇒ ≥ (s − a ) (s − b) (s − c)
2 y+ z 8
a b c Now, put s − a = x ≥ 0, s − b = y ≥ 0, s − c = z ≥ 0
6. By sine rule, = =
sin A sin B sin C s−a + s−b=x+ y
3 +1 b 2s − a − b = x + y
⇒ =
sin (105° ) sin 30° c=x+ y
Similarly, a = y + z, b = x + z
A
(x + y) ( y + z ) (x + z )
c 105° ⇒ ⋅ ⋅ ≥ xyz
b 2 2 2
30° 45° which it true.
B C
a = ( 3+1) Now, equality will hold, if x = y = z
⇒ a=b=c
( 3 + 1) sin 30°
⇒ b= ⇒ Triangle is equilateral.
sin 105°
Properties of Triangles 539

9. If the triangle is equilateral, then cos (B − C ) 1 + p


⇒ =
A = B = C = 60° cos (B + C ) 1 − p
⇒ tan A + tan B + tan C = 3 tan 60° = 3 3 (1 + p)
⇒ cos (B − C ) = − …(ii)
2 (1 − p)
Conversely assume that,
tan A + tan B + tan C = 3 3 [Q B + C = 3π / 4]
But in ∆ABC, A + B = 180° − C Since, B or C can vary from 0 to 3π / 4
Taking tan on both sides, we get ∴ 0 ≤ B − C < 3π / 4
1
tan ( A + B) = tan (180° − C ) ⇒ − < cos (B − C ) ≤ 1 …(iii)
tan A + tan B 2
⇒ = − tan C 1 1+ p
1 − tan A tan B From Eqs. (ii) and (iii), − < ≤1
2 2 ( p − 1)
⇒ tan A + tan B = − tan C+ tan A tan B tan C
1 1+ p 1+ p
⇒ tan A + tan B + tan C = tan A tan B tan C = 3 3 ⇒ − < and ≤1
2 2 ( p − 1) 2 ( p − 1)
⇒ None of the tan A, tan B, tan C can be negative
1+ p 1 + p − 2 p+ 2
So, ∆ABC cannot be obtuse angle triangle. ⇒ + 1 ≥ 0 and ≤0
p−1 2 ( p − 1)
Also, AM ≥ GM
1  1 + 2
⇒ [tan A + tan B + tan C ] ≥ [tan A tan B tan C ]1/3 (1 − 2 )  p − 
3 2p  1 − 2
⇒ ≥ 0 and ≤0
1 p−1 2 ( p − 1)
⇒ (3 3 ) ≥ (3 3 )1/3 ⇒ 3 ≥ 3.
3
2p ( p − ( 2 + 1 )2 )
So, equality can hold if and only if ⇒ > 0 and ≥0
p−1 ( p − 1)
tan A = tan B = tan C + – + + – +
or A = B = C or when the triangle is equilateral. 2
0 1 1 (√2+1)
10. By using triangular inequality,
⇒ ( p < 0 or p > 1)
c<a+b
and ( p < 1 or p > ( 2 + 1)2)
⇒ c2 < ca + ab
On combining above expressions, we get
Similarly, a 2 < ab + ac and b2 < bc + ab
p < 0 or p ≥ ( 2 + 1)2
∴ a 2 + b2 + c2 < 2ab + 2bc + 2ca
i.e. p ∈ (−∞ , 0) ∪ [( 2 + 1)2, ∞ )
⇒ (a 2 + b2 + c2) + 2ab + 2bc + 2ca < 4(ab + bc + ca )
or p ∈ (−∞ , 0) ∪ [3 + 2 2 , ∞ )
⇒ (a + b + c)2 < 4(ab + bc + ca ) ... (i)
12. It is given that a , b, c and area of triangle are rational.
Now, using AM-GM inequality in a , b and c, we get
B (s − c) (s − a )
a 2 + b2 b2 + c2 c2 + a 2 We have, tan =
≥ ab, ≥ bc and ≥ ca 2 s (s − b)
2 2 2
⇒ a 2 + b2 + c2 ≥ ab + bc + ca s (s − a )(s − b)(s − c)
=
⇒ a 2 + b2 + c2 + 2ab + 2bc + 2ca ≥ 3(ab + bc + ca ) s (s − b)
a+ b+ c
⇒ (a + b + c)2 ≥ 3(ab + bc + ca ) ... (ii) Again, a , b, c are rational given, s = are
2
From Eqs. (i) and (ii), we get rational, Also, (s − b) is rational, since triangle is
3(ab + bc + ca ) ≤ (a + b + c)2 < 4(ab + bc + ca ) rational, therefore we get
11. Since, A+ B+C=π  B ∆
tan   = is rational.
 2  s (s − b)
⇒ B + C = π − π / 4 = 3π / 4 …(i)
C ∆
[Q A = π / 4, given] Similarly, tan = is rational.
2 s (s − c)
∴ 0 < B, C < 3π / 4
B C
Also, given tan B ⋅ tan C = p Therefore a , tan , tan are rational.
sin B ⋅ sin C p 2 2
⇒ = which shows that, (i) ⇒ (ii).
cos B ⋅ cos C 1
sin B ⋅ sin C + cos B cos C p+1 Again, it is given that,
⇒ =
sin B ⋅ sin C − cos B ⋅ cos C p − 1 B C
a , tan , tan are rational, then
2 2
540 Properties of Triangles

A  π B + C abc sin A ⋅ sin (B + C ) sin (B − C )


tan = tan  −  =
2 2 2  (b2 sin 2 A − c2 sin 2 A )
 B + C 1  a b c 
= cot   =
 2   B C using sine rule, sin A = sin B = sin C 
tan  +   
2 2
abc sin 2 A ⋅ sin (B − C ) abc sin (B − C )
 B C = =
1 − tan   ⋅ tan   (b2 − c2) ⋅ sin 2 A b2 − c2
 2  2
=
 B C ab2r sin (B − C ) ar sin (B − C )
tan   + tan   = =
 2  2 b2 − b2r 2 1 − r2
Since, tan (B / 2) and tan (C / 2) are rational, hence ar
⇒ p≤ [Q sin (B − C ) ≤ 1]
tan ( A / 2) is a rational. 1 − r2
2 tan A / 2
Now, sin A = as tan ( A / 2) is a rational 14. Given, cos A cos B + sin A sin B sin C = 1
1 + tan 2 A / 2
1 − cos A cos B
number, sin A is a rational . Similarly, sin B and sin C ⇒ sin C = ...(i)
are. Thus, a, sin A, sin B, sin C are rational, therefore sin A sin B
(ii) ⇒ (iii). 1 − cos A cos B
⇒ ≤1 [Q sin C ≤ 1]
Again, a, sin A, sin B, sin C are rational. sin A sin B
By the sine rule, ⇒ 1 − cos A cos B ≤ sin A sin B
a b c
= = ⇒ 1 ≤ cos ( A − B)
sin A sin B sin C
a sin B a sin C ⇒ cos ( A − B) ≥ 1
⇒ b= and c = ⇒ cos ( A − B) = 1 [Q as cos (θ ) ≤ 1]
sin A sin A
⇒ A − B =0
Since a, sin A, sin B and sin C are rational,
On putting A = B in Eq. (i), we get
Hence, b and c are also rational.
1 1 − cos 2 A
Also, ∆ = bc sin A sin C =
2 sin 2 A
As b, c and sin A are rational, so triangle is rational ⇒ sin C = 1
number. Therefore, a , b, c and triangle are rational. ⇒ C = π /2
Therefore, (iii) ⇒ (i). Now, A+ B+C=π
13. Let ABC be a triangle with base BC = a and altitude π π  π
⇒ A+ B= ⇒ A= Q A = B and C = 
AD = p, then 2 4  2
A π π π
∴ sin A : sin B : sin C = sin : sin : sin
4 4 2
1 1
b ⇒ a :b:c= : :1
c p 2 2
= 1 :1 : 2

B C 15. Let a , b, c are the sides of a ∆ABC.


D
a 3
Given, cos A + cos B + cos C =
1 2
Area of ∆ABC = bc sin A
2 b2 + c2 − a 2 a 2 + c2 − b2 a 2 + b2 − c2 3
⇒ + + =
1 2bc 2ac 2ab 2
Also, area of ∆ABC = ap
2 ⇒ ab2 + ac2 − a3 + ba 2 + bc2 − b3
1 1 + ca 2 + cb2 − c3 = 3abc
∴ ap = bc sin A
2 2 ⇒ a (b − c)2 + b (c − a )2 + c (a − b)2
bc sin A (a + b + c)
⇒ p= = [(a − b)2 + (b − c)2 + (c − a )2]
a 2
abc sin A
⇒ p= ⇒ (a + b − c) (a − b)2 + (b + c − a ) (b − c)2
a2
+ ( c + a − b) ( c − a )2 = 0
abc sin A ⋅ (sin 2 B − sin 2 C )
⇒ p= [as we know, a + b − c > 0, b + c − a > 0, c + a − b > 0]
a 2(sin 2 B − sin 2 C )
Properties of Triangles 541

∴ Each term on the left of equation has positive Area of ∆ABC′ =


1
AB × AC sin A
coefficient multiplied by perfect square, each term must 2
be separately zero. 1
⇒ a=b=c = × 4 × 2 2 sin (15° )
2
∴ Triangle is an equilateral.
= 2 ( 3 − 1) sq. units
1 1 a
16. Since, ∆ = ap1 ⇒ = Difference of areas of triangle
2 p1 2∆
=|2 ( 3 + 1) − 2 ( 3 − 1)|= 4 sq units
1 b 1 c
Similarly, = , = Alternate Solution
p2 2∆ p3 2∆
C′
1 1 1 1
∴ + − = (a + b − c)
p1 p2 p3 2∆ D
2
2(s − c) s − c s(s − c) ab 2 2 2 C
= = = ⋅ 2 2
2∆ ∆ ab s∆ 30° B
ab 2C A
= ⋅ cos
 a + b + c 2
  ∆
 2 
2ab C
= cos 2
(a + b + c)∆ 2
1 Here, AD = 2 , DC = 2
17. We know that, ∆ = ap1
2 Difference of areas of ∆ABC and ∆ABC′
2∆ = Area of ∆ACC′
⇒ p1 =
a 1 1
= AD × CC′ = × 2 × 4 = 4 sq units
2∆ 2∆ 2 2
Similarly, p2 = and p3 =
b c
8∆3 Topic 3 Circumcircle, Incircle, Escribed,
Now, p1 p2 p3 =
abc Orthocentre and Centroid of a
Since, ∆=
abc Triangle
4R 1. Let ABC be a given triangle with vertices
8 (abc)3 (abc)2 B(0, 2), C (4, 3) and let third vertex be A (a , b)
∴ p1 p2 p3 = ⋅ =
abc 64R3 8R3 A (a , b )
a 2 2 4
18. In ∆ ABC , by sine rule, = =
sin A sin 30° sin C E
F
⇒ C = 45° , C′ = 135° (0,0)
When, C = 45° ⇒ A = 180° − (45°+30° ) = 105°
When, C′ = 135° ⇒ A = 180° − (135° + 30° ) = 15° (0, 2) B C (4,3)
D
A
Also, let D , E and F are the foot of perpendiculars
15° drawn from A , B and C respectively.
b −0 3 −2
2

2 4
Then, AD⊥ BC ⇒ × = −1
2

2
a −0 4 −0
45° 135° 30°
C B
[if two lines having slopes m1 and m2, are
C′ perpendicular then m1m2 = −1]
1 ⇒ b + 4a = 0 …(i)
Area of ∆ABC = AB × AC sin A
2 and CF⊥ AB
1 b −2 3 −0
= × 4 × 2 2 sin (105° ) ⇒ × = −1
2 a −0 4 −0
⇒ 3 b − 6 = −4 a
3 +1
=4 2 × ⇒ 4a + 3b = 6 …(ii)
2 2
From Eqs. (i) and (ii), we get
= 2 ( 3 + 1) sq. units −b + 3b = 6 ⇒ 2b = 6
542 Properties of Triangles

⇒ b =3 13  1 
∴ Slope of BC is Q mBC = − 
3 61  mAH 
and a=− [from Eq. (i)]
4
Now, equation of line BC is given by y − y1 = m(x − x1 ),
So, the third vertex where (x1 , y1 ) are coordinates of B.
 3  13  35
(a , b) ≡  − , 3 , which lies in II quadrant. ∴ y − (− 10) = x − 
 4  61  2
2. 13
⇒ y + 10 = (2x − 35)
A 61 × 2
⇒ 122 y + 1220 = 26x − 455
E
H
F ⇒ 26x − 122 y − 1675 = 0
(1, 1) a2 + b 2 − c 2 abc ∆
3. PLAN (i) cos C = (ii) R = ,r=
2 ab 4∆ s
C B where, R, r , ∆ denote the circumradius, inradius and
3x–2y+6=0 4x+5y–20=0
area of triangle, respectively.
Let equation of AB be 4x + 5 y − 20 = 0 and AC be Let the sides of triangle be a , b and c.
3x − 2 y + 6 = 0 Given, x=a + b
3
Clearly, slope of AC = y = ab
2 x2 − c2 = y
a ⇒ (a + b)2 − c2 = y
[Q slope of ax + by + c = 0 is − ]
b
∴Slope of altitude BH, which is perpendicular to ⇒ a 2 + b2 + 2ab − c2 = ab
2  1  ⇒ a 2 + b2 − c2 = − ab
AC = − . Q mBH = − 
3  mAC  a 2 + b2 − c2 1
⇒ = − = cos 120°
Equation of BH is given by y − y1 = m(x − x1 ) 2ab 2
2 2π
Here, m = − , x1 = 1 and y1 = 1 ⇒ ∠C =
3 3
2 abc ∆ r 4∆2
∴ y − 1 = − (x − 1) Q R= ,r = ⇒ =
3 4∆ s R s (abc)
⇒ 2x + 3 y − 5 = 0 1  2π  
2

Now, equation of AB is 4x + 5 y − 20 = 0 and 4  ab sin   


 3 
= 
2
equation of BH is 2x + 3 y − 5 = 0 x+ c
⋅ y⋅ c
Solving these, we get point of intersection 2
(i.e. coordinates of B). r 3y
∴ =
4x + 5 y − 20 = 0  R 2c (x + c)
 ⇒ y = − 10
4x + 6 y − 10 = 0
4. First solve each option separately.
35 (a) If a, sin A, sin B are given, then we can determine
On substituting y = − 10 in 2x + 3 y − 5 = 0, we get x = a a
2 b= sin B, c = sin C. So, all the three
 35  sin A sin A
∴ B , − 10
2  sides are unique.
So, option (a) is incorrect.
Solving 4x + 5 y − 20 = 0 and 3x − 2 y + 6 = 0, we get (b) The three sides can uniquely make an acute angled
coordinate of A. triangle. So, option (b) is incorrect.
12x + 15 y − 60 = 0 (c) If a, sin B, R are given, then we can determine
 ⇒ 23 y = 84 a sin B
12x − 8 y + 24 = 0  b = 2R sin B, sin A = . So, sin C can be
b
84 10  10 84
⇒ y= ⇒x= ⇒A ,  determined.
23 23  23 23
Hence, side c can also be uniquely determined.
 84 
 y2 − y1   23 − 1
(d) If a, sin A, R are given, then
61
Now, slope of AH =   = = . b c
 x2 − x1   = = 2R
− 1 − 13
10
sin B sin C
 23 
But this could not determine the exact values of b
Q BC is perpendicular to AH. and c.
Properties of Triangles 543

1 1
5. Here, R2 = MC 2 = (a 2 + b2) [by distance from origin] Area of ∆ PQS = ⋅ 4 2 ⋅ 2 = 4 2 sq units3
4 2
1 2 Y
= c [by Pythagoras theorem]
4

Y P
(1, 2 2)

S (1,0) R
A X' X
(-3, 0) (-1, 0) O
(9, 0)
M (a /2, b /2) Q
b
(1, -2 2)

C X Y'
a B
Ratio of areas of ∆PQS and ∆PQR is 1 : 4.
c
⇒ R= 9. Let a non-right angled ∆PQR.
2
Now, by sine rule
Next, r = (s − c) tan (C / 2) = (s − c) tan π / 4 = s − c
P
∴ 2(r + R) = 2r + 2R = 2s − 2c + c
= a + b + c−c q=1
rS
=a+b O

6. Radius of incircle is, r = Q E R
s p=Ö 3
¾

Since, ∆ = 16 2
P q r
6 2 +6 2 +4 2 = = = 2 × circumradius
Now, s= sin P sin Q sin R
2
=8 2 3 1 r
⇒ = = = 2 ×1 [circumradius = 1 unit]
16 2 sin P sin Q sin R
∴ r= 3 1
8 2 ⇒ sin P = and sin Q =
2 2
=2
⇒ P = 120º and Q = 30º (Q ∆PQR is non-right angled
7. Equation of circumcircle of ∆PRS is triangle)
(x + 1) (x − 9) + y2 + λy = 0 So, R = 30º
It will pass through (1, 2 2 ), then ⇒ r = 1, so ∆PQR is an isosceles triangle. And, since RS
− 16 + 8 + λ ⋅ 2 2 = 0 and PE are the median of ∆PQR, so ‘O’ is centroid of the
∆PQR.
8
⇒ λ= =2 2 Now,
2 2
Option (a),
∴ Equation of circumcircle is
From Apollonius theorem,
x2 + y2 − 8 x + 2 2 y − 9 = 0 2(PE 2 + QE 2) = PQ 2 + PR2
Hence, its radius is 3 3 .  3
⇒ 2  PE 2 +  = 1 + 1
Alternate Solution  4
2 2 3 1 1
Let ∠ PSR = θ ⇒ sin θ = ⇒ PE 2 = 1 − ⇒ PE 2 = ⇒ PE = units
2 3 4 4 2
1 1
∴ sin θ =
PR and OE = PE = units [QO divides PE is 2 : 1]
2R 3 6
Option (b),
⇒ PR = 6 2 = 2R ⋅ sin θ
Again from Apollonius theorem,
⇒ R=3 3 2(PS 2 + RS 2) = PR2 + QR2
8. Coordinates of P and Q are (1, 2 2 ) and (1, − 2 2 ). 1 
⇒ 2 + RS 2 = 1 + 3
4 
Now, PQ = (4 2 )2 + 02 = 4 2
1 7 7
1 ⇒ RS 2 = 2 − ⇒ RS 2 = ⇒ RS = units
Area of ∆ PQR = ⋅ 4 2 ⋅ 8 = 16 2 sq units 4 4 2
2
544 Properties of Triangles

Option (c), 11. Given a ∆XYZ, where 2s = x + y + z


1 s−x s− y s−z
Area of ∆SOE = (OE ) (ST ) and = =
2 4 3 2
1 1
= × [(PS )sin 60º ] X
2 6
1 1 3
= × × z y
12 2 2
3
= square units Z
48 Y
x
Option (d), s−x s− y s−z
1 1 1 ∴ = =
pq sin R ( 3 )(1) 4 3 2

Q Inradius of ∆PQR = = 2 = 2 2 3s − (x + y + z ) s
s 1 ( p + q + r ) 1 ( 3 + 1 + 1) = =
4+3+2 9
2 2
s−x s− y s−z s
3 or = = = = λ (let)
= (2 − 3 ) units 4 3 2 9
2
Hence, options (a), (b) and (d) are correct. ⇒ s = 9λ, s = 4λ + x, s = 3λ + y
and s = 2λ + z
10. We have,
In ∆ PQR
P ∴ s = 9λ, x = 5λ, y = 6λ, z = 7λ
∠ PQR = 30° Now, ∆ = s(s − x)(s − y)(s − z )
PQ = 10 3 10 3 [Heron’s formula]
QR = 10 = 9λ ⋅ 4λ ⋅ 3λ ⋅ 2λ = 6 6λ2 …(i)
30° 8π
Also, πr 2 =
By cosine rule Q 10 R 3
PQ 2 + QR2 − PR2 ⇒ r2 =
8
cos 30° = …(ii)
2PQ ⋅ QR 3
3 300 + 100 − PR2 xyz (5λ )(6λ )(7λ ) 35λ
⇒ = and R= = = …(iii)
2 4∆ 4 ⋅ 6 6λ2 4 6
200 3
⇒ 300 = 300 + 100 − PR2 8 ∆2 216λ4
Now, r2 = = =
⇒ PR = 10 3 S2 81λ2
Since, PR = QR = 10 8 8 2
⇒ = λ [from Eq. (ii)]
∴ ∠QPR = 30° and ∠QRP = 120° 3 3
1 ⇒ λ =1
Area of ∆ PQR = PQ ⋅ QR ⋅ sin 30°
2 (a) ∆XYZ = 6 6λ2 = 6 6
1 1
= × 10 3 × 10 × = 25 3 ∴ Option (a) is correct.
2 2 35 35
Radius of incircle of (b) Radius of circumcircle (R) = λ=
4 6 4 6
Area of ∆ PQR
∆ PQR = ∴ Option (b) is incorrect.
Semi - perimetre of ∆ PQR
X Y Z
∆ 25 3 25 3 (c) Since, r = 4R sin ⋅ sin ⋅ sin
i.e. r= = = 2 2 2
s 10 3 + 10 + 10 5( 3 + 2)
2 2 35 X Y Z
2 ⇒ = 4⋅ sin ⋅ sin ⋅ sin
3 4 6 2 2 2
⇒ r = 5 3 (2 − 3)
4 X Y Z
= 10 3 − 15 ⇒ = sin ⋅ sin ⋅ sin
abc 35 2 2 2
and radius of circumcircle (R) = ∴ Option (c) is correct.
4∆
10 3 × 10 × 10 X +Y 2 Z 
= = 10 (d) sin 2  = cos  
4 × 25 3  2   2
X +Y Z s(s − z ) 9 × 2 3
∴Area of circumcircle of as = 90° − = = =
∆ PQR = πR2 = 100 π 2 2 xy 5 ×6 5

Hence, option (b), (c) and (d) are correct answer. ∴ Option (d) is correct.
Properties of Triangles 545

12. Let a straight line through the vertex P of a given 14. Since, sides of a triangle subtends α , β, γ at the centre.
∆ PQR intersects the side QR at the point S and the A
circumcircle of ∆ PQR at the point T.
Points P , Q , R, T are concyclic, then PS ⋅ ST = QS ⋅ SR
PS + ST
Now, > PS ⋅ ST [Q AM > GM]
2 γ β
P α

B C

∴ α + β + γ = 2π ...(i)
Q Now, arithmetic mean
S R
π  π  π 
cos  + α  + cos  + β + cos  + γ
T 2  2  2 
=
1 1 2 2 3
and + > =
PS ST PS ⋅ ST QS ⋅ SR As we know that, AM ≥ GM, i.e.
SQ + QR π π π
Also, > SQ ⋅ SR AM is minimum, when + α = + β = + γ
2 2 2 2
QR or α = β = γ = 120°
⇒ > SQ ⋅ SR ∴ Minimum value of arithmetic mean
2
1 2 π 
⇒ > = cos  + α 
SQ ⋅ SR QR 2 
2 4 3
⇒ > = cos (210° ) = −
SQ ⋅ SR QR 2
1 1 2 4 360°
∴ + > > 15. Here, central angle = = 40°
PS ST QS ⋅ SR QR 9

abc ∆
13. We have, R = and r =
4∆ s
R abc s abc ⋅ s
= ⋅ = C
r 4∆ ∆ 4∆2 20°
abc
= r
4 (s − a ) (s − b) (s − c)
1 1
But a : b : c = 4 : 5 : 6 [given] A1 M A2
a b c
⇒ = = =k [let] 1
4 5 6 In ∆ACM , = sin 20°
r
⇒ a = 4k, b = 5k, c = 6k
⇒ r = cosec 20°
1
Now, s = (a + b + c) ∴ Radius of circle = cosec 20°
2
1 15k 16. Since, the circles with radii 3, 4 and 5 touch each other
= (4k + 5k + 6x) = externally and P is the point of intersection of tangents.
2 2
A
R (4k) (5k) (6k)
∴ =
r  15k   15k   15k 
4 − 4k  − 5k  − 6k
 2   2   2 
4 5
C2 C1
30k3
= 4 P 5
 15 − 8  15 − 10  15 − 12
k3     
 2   2  2  3 3
30 ⋅ 8 16 C3
= =
7 ⋅5 ⋅3 7
546 Properties of Triangles

⇒ P is incentre of ∆C1 C 2 C3 . Now, in right angled ∆IHJ,


Thus, distance of point P from the points of contact ∠ JIH = π / 2 − A / 2
= inradius (r) of ∆C1C 2 C3 [Q ∠ IEA = 90°, ∠ IAE = A /2 and ∠ JIH = ∠ AIE]
∆ s (s − a )(s − b)(s − c) In ∆JIH,
i.e. r= =
s s  π A r A r
tan  −  = 1 ⇒ cot = 1
where, 2s = 7 + 8 + 9 ⇒ s = 12  2 2  r − r1 2 r − r1
(12 − 7)(12 − 8)(12 − 9) 5 ⋅4 ⋅3 B
=
r2 C
and cot = 3
r
Hence, r = = = 5 Similarly, cot
12 12 2 r − r2 2 r − r3
n 2 2π On adding above results, we get
17. We know that, I n = r sin
2 n cot A / 2 + cot B / 2 + cot C / 2
[since, I n is area of regular polygon] = cot A / 2 cot B / 2 cot C / 2
2I n 2π r1 r2 r3 r1r2 r3
⇒ = sin [Q r = 1] …(i) ⇒ + + =
n n r − r1 r − r2 r − r3 (r − r1) (r − r2) (r − r3 )
π
and On = nr 2 tan 19. It is clear from the figure that, OA = R
2
A
[since, On is area of circumscribing polygon]
On π
= tan …(ii) A/2

90
n n B

°
F G
R
On dividing Eq. (i) by Eq. (ii), we get 90°
2π r
I
sin O
2I n n
=
On π
tan B C
n
2π IF
1 + cos AI =
In π n sin ( A / 2)
⇒ = cos 2 =
On n 2 r
Q ∆AIF is right angled triangle, so =
sin ( A / 2)
I n 1 + 1 − (2I n / n )
2
∴ = [from Eq. (i)]
On 2 But r = 4R sin ( A / 2) sin (B / 2) sin (C / 2)
On ∴ AI = 4R sin (B / 2) sin (C / 2)
⇒ In = (1 + (1 − (2I n / n )2 )
2 Again, ∠GOA = B ⇒ OAG = 90° − B
18. The quadrilateral HEKJ is a square, because all four Therefore, ∠IAO = ∠IAC − ∠OAC
angles are right angles and JK = JH . 1
= A / 2 − (90° − B) =
( A + 2B − 180° )
A 2
1 1
= ( A + 2B − A − B − C ) = (B − C )
2 2
A/2
A/2

In ∆OAI,OI 2 = OA 2 + AI 2 − 2(OA )( AI ) cos (∠IAO )

r1
K = R2 + [4R sin (B / 2) sin (C / 2)]2
90°  B − C
J − 2R ⋅ [4R sin (B / 2) sin (C / 2)] cos  
 2 
F E
r1 90° r1 = [R2 + 16R2 sin 2(B / 2) sin 2(C / 2)
H
 B − C 
I − 8R2 sin (B / 2) sin (C / 2) cos  
r2 r3  2  
= R2[1 + 16 sin 2(B / 2) sin 2(C / 2)
 B − C 
− 8 sin (B / 2) sin (C / 2) cos  
B C  2  
D
90° = R2[1 + 8 sin (B / 2) sin (C / 2)
Therefore, HE = JK = r1 and IE = r [given]   B − C  
2 sin (B / 2) sin (C / 2) − cos  
⇒ IH = r − r1   2  
Properties of Triangles 547

= R2[1 + 8 sin (B / 2) sin (C / 2) 4∆2


⇒ b [b2 − (a − c)2] =
  B − C  B + C  B − C   s
cos   + cos   − cos  
  2   2   2   ⇒ b [b2 − (a − c)2] = 8(s − a )(s − b)(s − c)
⇒ b [{ b − (a − c)}{ b + (a − c)}]
  B + C 
= R21 − 8 sin (B / 2) sin (C / 2) cos   = 8(s − a )(s − b)(s − c)
  2  
⇒ b [(b + c − a )(b + a − c)] = 8(s − a )(s − b)(s − c)
  π A 
= R21 − 8 sin (B / 2) sin (C / 2) cos  −   ⇒ b [(2s − 2a )(2s − 2c)] = 8(s − a )(s − b)(s − c)
 2 2
⇒ b [2 ⋅ 2 (s − a )(s − c)] = 8(s − a )(s − b)(s − c)
 A B C π
Q 2 + 2 + 2 = 2  ⇒ b = 2s − 2b
⇒ 2b = a + c
= R2[1 − 8 sin ( A / 2) sin (B / 2) sin (C / 2)]
which shows that b is arithmetic mean between a
  r 
= R2 1 − 8    = R2 − 2Rr and c.
  4 R 
20. Since, r1 , r2 and r3 are exradii of ∆ABC are in HP.
Now, in right angled ∆BIO, 1 1 1
OB2 = BI 2 + IO 2 ∴ , , are in AP.
r1 r2 r3
⇒ R2 = BI 2 + R2 − 2Rr s−a s−b s−c
⇒ , , are in AP.
⇒ 2Rr = BI 2 ∆ ∆ ∆
⇒ 2 Rr = r 2 /sin 2(B / 2) ⇒ s − a , s − b, s − c are in AP.
⇒ 2R = r /sin 2(B / 2) ⇒ − a , − b, − c are in AP.
⇒ 2R sin B / 2 = r
2
⇒ a , b, c are in AP.
⇒ R (1 − cos B) = r 3
abc ∆ 21. sin C = and C is given to be obtuse.
⇒ (1 − cos B) = 2
4∆ s 2π
⇒ C= = a 2 + b2 − 2ab cos C
4∆2 3
⇒ abc (1 − cos B) =
s 2π
= 62 + 102 − 2 × 6 × 10 × cos = 14
 a +c −b
2 2 2
4∆2 3
⇒ abc 1 − =
2ac s ∆ 225 × 3
  ∴ r= ⇒ r2 = =3
2
2ac − a 2 − c2 + b2  4∆2
s  6 + 10 + 14
⇒  
abc  =  2 
 2ac  s
24
Vectors

Topic 1 Scalar Product of Two Vectors


Objective Questions I (Only one correct option) 7. Two adjacent sides of a parallelogram ABCD are
→ →
1. Let A( 3, 0, −1), B ( 2, 10, 6) and C(1, 2, 1) be the vertices given by AB = 2$i + 10$j + 11k$ and AD = − $i + 2$j + 2k$ .
of a triangle and M be the mid-point of AC. If G The side AD is rotated by an acute angle α in the
divides BM in the ratio 2 : 1, then cos ( ∠GOA) (O plane of the parallelogram so that AD becomes
being the origin) is equal to (2019 Main, 10 April I)
1 1 1 1 AD’. If AD’ makes a right angle with the side AB,
(a) (b) (c) (d)
15 2 15 30 6 10 then the cosine of the angle α is given by (2010)
8 17 1 4 5
π π (a) (b) (c) (d)
2. If a unit vector a makes angles with $i, with $j and 9 9 9 9
3 4
$
θ ∈( 0, π ) with k, then a value of θ is (2019 Main, 9 April II) 8. Let P, Q, R and S be the points on the plane with
5π π 5π 2π position vectors − 2$i − $j , 4$i , 3$i + 3$j and − 3$i + 2$j,
(a) (b) (c) (d)
6 4 12 3 respectively. The quadrilateral PQRS must be a
(2010)
3. Let $ , b = b $i + b $j + 2
a = i$ + $j + 2 k $
k and
1 2 (a) parallelogram, which is neither a rhombus nor a
$ be three vectors such rectangle
c = 5 $i + $j + 2 k that the (b) square
projection vector of b on a is a. If a + b is (c) rectangle, but not a square
perpendicular to c, then|b | is equal to (d) rhombus, but not a square
(2019 Main, 9 Jan II)
9. Let two non-collinear unit vectors a$ and b$ form an
(a) 6 (b) 4
(c) 22 (d) 32 acute angle. A point → P moves, so that at any time t
the position vector OP (where, O is the origin) is
4. If lines x = ay + b, z = cy + d and x = a ′ z + b′, given by a$ cos t + b$ sin t. When→
P is farthest from
y = c′ z + d ′ are perpendicular, then (2019 Main, 9 Jan II) $ be the unit
origin O, let M
→
be the length of OP and u
(a) ab′+ bc′+1 = 0 (b) bb′+ cc′+1 = 0
vector along OP. Then, (2008, 3M)
(c) aa ′+ c + c′ = 0 (d) cc′+ a + a ′ = 0 $b
a$ + $ )1/ 2
(a) u$ = and M = (1 + a$ ⋅ b
5. Let O be the origin and let PQR be an arbitrary | a$ + $b|
triangle. The point S is such that a$ − $b
(b) u$ = and M = (1 + $ )1/ 2
a$ ⋅ b
OP ⋅OQ + OR ⋅ OS = OR ⋅ OP + OQ ⋅ OS | a$ − $b|
= OQ ⋅ OR + OP ⋅ OS (c) u$ =
a$ + $
b
and M = (1 + $ )1/ 2
2 a$ ⋅ b
| a$ + $|
b
Then the triangle PQR has S as its (2017 Adv.)
a$ − $b
(a) centroid (b) orthocentre (d) u$ = and M = (1 + $ )1/ 2
2 a$ ⋅ b
(c) incentre (d) circumcentre | a$ − $b|
→ →
→ $ , →b = i$ − $j + k
$ and → $ be $ →c = $i + $j − k$ .A vector
10. Let, a = $i + 2$j + k$ , b = $i − $j + k,
6. Let a = $i + $j + k c = $i − $j − k
→ → → → → →
three vectors. A vector v in the plane of a and b, coplanar to a and b has a projection along c of
→ 1 1
whose projection on c is , is given by (2011) magnitude , then the vector is (2006, 3M)
3 3
(a) $i − 3$j + 3 k$ (b) − 3 $i − 3$j − k$ (a) 4 $i − $j + 4 k$ (b) 4 $i + $j − 4 k$
(c) 3 $i − $j + 3 k
$ (d) $i + 3 $j − 3 k
$ (c) 2 $i + $j + k$ (d) None of these
Vectors 549

→ → →
11. If a , b , c are three non-zero, non-coplanar vectors Objective Question II
→ →
→ →
b⋅a→ → → b⋅a→
→ → (One or more than one correct option)
and b1 = b − a , b2 = b + a, → → →
→2 →2
| a| | a| 18. Let a = 2$i − $j + k$ , b = $i + 2$j − k$ and c = $i + $j − 2 k$ be
→ →
→ → → → → → → → three vectors. A vector in the plane of b and c, whose
→ → c ⋅ a → c ⋅ b → → → c ⋅ a → c ⋅ b1 → →
c1 = c− a − b , c2 = c − a− b1 , projection on a is of magnitude 2 / 3 , is (1993, 2M)
→ → → →
| a|2 | b|2 | a |2 | b |2
(a) 2 $i + 3$j − 3 k$ (b) 2 $i + 3$j + 3 k$
→ → → → → →
→ → c ⋅ a → c ⋅ b2 → → → c ⋅ a → (c) − 2 i$ − $j + 5 k$ (d) 2 i$ + $j + 5 k$
c3 = c − a− b 2 , c4 = a − a.
→ 2 → 2 →
| a| | b2 | | a |2 Integer & Numerical Answer Type Question
Then, which of the following is a set of mutually 19. Let a and b be two unit vectors such that a ⋅ b = 0. For
orthogonal vectors? (2005, 1M) some x , y ∈ R, let c = xa + yb + (a × b ). If|c| = 2 and
→ → →
(a) {a , b1 , c1 } (b)
→ → →
{a , b1 , c2 } the vector c is inclined at the same angle α to both a
→ → → → → → and b, then the value of 8 cos2 α is ......... . (2018 Adv.)
(c) {a , b2 , a3 } (d) {a , b2 , c4 }
→ → → → Fill in the Blanks
12. If a and b1 are two unit vectors such that a + 2 b and

→ → 20. The components of a vector a along and perpendicular
5 a − 4 b , are perpendicular to each other, then the →
→ → to a non-zero vector b are ……and……respectively.
angle between a and b is (2002, 1M)
(1988, 2M)
(a) 45° (b) 60°
21. A, B, C and D, are four points in a plane with position
(c) cos−1   (d) cos−1  
1 2
→ → → →
 3  7 vectors a , b , c and d respectively such that
→ → → → → → → → → → →
13. If a,b and c are unit vectors, then ( a − d) ⋅ ( b − c ) = ( b − d) ⋅ ( c − a ) = 0
→ → → → → →
|a − b|2 +|b − c|2 +| c − a|2 does not exceed (2001, 2M) The point D, then, is the… of the ∆ ABC. (1984, 2M)
→ → →
(a) 4 (b) 9 22. Let A, B, C be vectors of length 3, 4, 5 respectively.
(c) 8 (d) 6 → → → → → → →
→ → → → → → → Let A be perpendicular to B + C, B to C + A and C to
14. Let u, v and w be vectors such that u + v + w = 0. If → → → → →
→ → → → → → → → → A + B. Then, the length of vector A + B + C is ... .
|u|= 3,|v|= 4 and|w|= 5, then u⋅ v + v⋅ w + w⋅ u is (1981, 2M)
(1995, 2M)
(a) 47 (b) – 25 True/False
(c) 0 (d) 25 → → → →
23. The points with position vectors a + b, a − b and
15. The number of vectors of unit length perpendicular → →
→ →
to vectors a = (1, 1, 0) and b = ( 0, 1, 1) is (1987, 2M) a + k b are collinear for all real values of k. (1984, 1M)

(a) one (b) two Analytical & Descriptive Questions


(c) three (d) infinite
→ → → →
16. A vector a has components 2 p and 1 with respect to a 24. Find 3-dimensional vectors v1 , v 2 , v3 satisfying
rectangular cartesian system. This system is rotated → → → → → →
v1 ⋅ v1 = 4, v1⋅ v2 = − 2, v1 ⋅ v3 = 6,
through a certain angle about the origin in the
→ → → → → →
counter clockwise sense. If, with respect to the new
→ v2⋅ v2 = 2, v2⋅ v3 = − 5, v3 ⋅ v3 = 29 (2001, 5M)
system, a has components p + 1 and 1, then (1986, 2M)
1 25. Show, by vector methods, that the angular bisectors
(a) p = 0 (b) p = 1or p = −
3 of a triangle are concurrent and find an expression
(c) p = − 1or p =
1
(d) p = 1or p = − 1
for the position vector of the point of concurrency in
3 terms of the position vectors of the vertices.
(2001, 5M)
17. The points with position vectors
26. In a ∆ ABC, D and E are points on BC and AC
60i$ + 3$j, 40i$ − 8$j, a$i − 52$j are collinear, if (1983, 1M)
respectively, such that BD = 2DC and AE = 3EC.Let
(a) a = − 40 (b) a = 40 P be the point of intersection of AD and BE. Find
(c) a = 20 (d) None of these BP / PE using vector methods. (1993, 5M)
550 Vectors

27. Determine the value of c, so that for all real x, the → → →


A (1) = 6i$ + 2$j, B ( 0) = 3$i + 2$j and B (1) = 2$j + 6$j.
vector cx $i − 6$j − 3k$ and x $i + 2$j + 2cx k$ make an → →
obtuse angle with each other. (1991, 4M) Then, show that A ( t ) and B ( t ) are parallel for some t.
28. In a ∆ OAB, E is the mid-point of BO and D is a point
on AB such that AD : DB = 2 : 1. If OD and AE Integer & Numerical Answer Type Questions
intersect at P, determine the ratio OP : PD using
31. Suppose that p , q and r are three non-coplanar
methods. (1989, 4M)
vectors in R3 . Let the components of a vector s along
29. Let OACB be a parallelogram with O at the origin p , q and r be 4, 3 and 5, respectively. If the
and OC a diagonal. Let D be the mid-point of OA. components of this vector s along ( − p + q + r ),
Using vector methods prove that BD and CO ( p – q + r ) and ( − p – q + r ) are x , y and z respectively,
intersect in the same ratio. Determine this ratio. then the value of 2x + y + z is (2015 Adv.)
(1988, 3M)
→ → →
→ → 32. If a, b and c are unit vectors satisfying
30. Let A ( t ) = f1( t )i$ + f2( t )$j and B ( t ) = g( t )i$ + g2( t )$j, → → 2 → → 2 → → 2

| a − b| +| b − c| +| c − a| = 9, then
t ∈ [0, 1], f1 , f2 , g1g2 are continuous functions. If A ( t ) → → →
→ →
| 2 a + 5 b + 5 c| is equal to (2012)
and B ( t ) are non-zero vectors for all t and A ( 0) = 2$i,

Topic 2 Vector Product of Two Vectors


r r r r r
Objective Questions I (Only one correct option) $ If β = β − β , where
5. Let α = 3$i + $j and β = 2i$ − $j + 3k.
r r 1 2
r r
1. Let a = i$ − 2 $j + k$ and b = i$ − $j + k$ be two vectors. If c β1 is parallel to α and β 2 is perpendicular to α, then
r r
is a vector such that b × c = b × a and c ⋅ a = 0, then β1 × β 2 is equal to (2019 Main, 9 April I)
c ⋅ b is equal to (2020 Main, 8 Jan II) 1 $ 1
1 3 (a) (3i − 9$j + 5k$) (b) (−3$i + 9$j + 5k
$)
(a) (b) − 2 2
2 2 (c) −3$i + 9$j + 5k
$ (d) 3i$ − 9$j − 5k$)
1
(c) − (d) − 1
2 $ and b = i$ − $j + k
6. Let a = 3$i + 2$j + xk $ , for some real x.
$ and b = i$ + 2$j − 2k
2. Let a = 3i$ + 2$j + 2k $ be two vectors. Then|a × b| = r is possible if (2019 Main, 8 April II)
If a vector perpendicular to both the vectors a + b and (a) 0 < r ≤
3
(b)
3
<r≤3
3
a − b has the magnitude 12, then one such vector is 2 2 2
(2019 Main, 12 April II) 3 3 3
(c) 3 <r<5 (d) r ≥ 5
(a) ± 4 (2i$ + 2$j + k
$) (b) ± 4 (2i$ − 2$j − k$) 2 2 2
(c) ± 4 (2$i + 2$j − k
$) (d) ± 4 (− 2$i − 2$j + k$) 7. A tetrahedron has vertices P(1, 2, 1),
Q( 2, 1, 3), R( − 1, 1, 2) and O( 0, 0, 0). The angle
3. Let α ∈ R and the three vectors
between the faces OPQ and PQR is (2019 Main, 12 Jan I)
$ , b = 2i$ + $j − αk
a = αi$ + $j + 3k $
(a) cos−1   (b) cos−1   (c) cos−1   (d) cos−1  
7 9 19 17
$
and c = αi$ − 2$j + 3k. Then, the set  31  35   35   31
S = { α : a, b and c are coplanar} (2019 Main, 12 April II) 8. Let a = 2$i + $j − 2k,
$ b = i$ + $j and c be a vector such
(a) is singleton that |c − a| = 3, |( a × b ) × c| = 3 and the angle
(b) is empty between c and a × b is 30°. Then, a ⋅ c is equal to
(c) contains exactly two positive numbers (2017 Main)
(d) contains exactly two numbers only one of which is 25 1
(a) (b) 2 (c) 5 (d)
positive 8 8
4. If the length of the perpendicular from the point → → → →
9. If a and b are vectors such that|a + b| = 29 and
x y−1 z +1 3
(β, 0, β ) (β ≠ 0) to the line, = = is , → $ ) = ( 2i$ + 3$j + 4k$)× →
1 0 −1 2 a × ( 2i$ + 3$j + 4k b, then a possible
→ →
then β is equal to (2019 Main, 10 April +I) value of ( a + b ) ⋅ ( − 7$i + 2$j + 3k $ ) is (2012)
(a) 2 (b) − 2
(a) 0 (b) 3
(c) − 1 (d) 1
(c) ± 4 (d) 8
Vectors 551

→ → → → → → →
10. Let a , b, c be unit vectors such that a + b + c = 0 . Integer & Numerical Answer Type Question
Which one of the following is correct? (2007, 3M) 16. In a triangle PQR, let a = QR , b = RP and c = PQ .
→ → → → → → → a .(c − b ) |a|
(a) a × b = b × c = c × a = 0 If|a | = 3,| b| = 4 and = ,
→ → → → → → → c . (a − b ) |a | +| b|
(b) a × b = b × c = c × a ≠ 0
→ → → → → → →
then the value of|a × b|2 is …… . (2020 Adv.)
(c) b × b = b × c = a × c = 0
→ → → → → →
(d) a × b, b × c , c × a are mutually perpendicular Assertion and Reason
→ → → For the following question, choose the correct answer
11. If the vectors a , b and c from the sides from the codes (a), (b), (c) and (d) defined as follows.
BC , CA and AB respectively of a ∆ ABC, then (a) Statement I is true, Statement II is also true;
→ → → → → →
(a) a ⋅ b + b⋅ c + c ⋅ a = 0 (2000, 2M)
Statement II is the correct explanation of
→ → → → → → Statement I
(b) a × b = b × c = c × a (b) Statement I is true, Statement II is also true;
→ → → → → → Statement II is not the correct explanation of
(c) a ⋅ b = b ⋅ c = c ⋅ a
→ → → → → → →
Statement I
(d) a × b+ b× c + c × a = 0 (c) Statement I is true; Statement II is false
(d) Statement I is false; Statement II is true
Objective Questions II → → → → → →
17. Let the vectors PQ, QR , RS , ST , TU and UP represent
(One or more than one correct option) the sides of a regular hexagon.
12. Let a and b be positive real numbers. Suppose → → → →
PQ = a$i + b$j and PS = a$i − b$j are adjacent sides of a Statement I PQ × ( RS + ST ) ≠ 0 .
parallelogram PQRS. Let u and v be the projection because
→ → → → → →
vectors of w = i$ + $j along PQ and PS, respectively. If Statement II PQ × RS = 0 and PQ × ST ≠ 0 . (2007, 3M)
| u| +| v | =| w |and if the area of the parallelogram
PQRS is 8, then which of the following statements Passage Based Problems
is/are TRUE? (2020 Adv.)
Let O be the origin and OX , OY, OZ be three unit
(a) a + b = 4 vectors in the directions of the sides QR , RP, PQ
(b) a − b = 2 respectively, of a triangle PQR. (2017 Adv.)
(c) The length of the diagonal PR of the parallelogram
PQRS is 4 18. If the triangle PQR varies, then the minimum value of
(d) w is an angle bisector of the vectors PQ and PS cos ( P + Q ) + cos (Q + R ) + cos( R + P ) is
3 3 5 5
13. Let ∆PQR be a triangle. Let a = QR , b = RP and (a) − (b) (c) (d) −
2 2 3 3
c = PQ. If |a| = 12 ,| b| = 4 3 and b ⋅ c = 24, then
19. |OX × OY| =
which of the following is/are true? (2015 Adv.)
2 2 (a) sin(P + Q ) (b) sin(P + R )
|c| |c|
(a) − |a| = 12 (b) + |a| = 30 (c) sin (Q + R ) (d) sin 2R
2 2
(c)|a × b + c × a|= 48 3 (d) a ⋅ b = − 72 Fill in the Blanks
→ → → →
14. Let A be vector parallel to line of intersection of 20. If b and c are any two non-collinear unit vectors and a
planes P1 and P2 through origin. P1 is parallel to the is any vector, then
vectors 2$j + 3k$ and 4$j − 3k$ and P2 is parallel to → → →
→ → → → → → a ⋅(b × c) → →
→ (a ⋅ b) b + (a ⋅ c) c + ( b × c ) = ……
and 3$i + 3$j , then the angle between vector A and → →2 (1996, 2M)
| b × c|
2$i + $j − 2k$ is
(2006, 5M)
π π 21. The unit vector perpendicular to the plane determined
(a) (b) by P (1,–1, 2), Q( 2, 0,–1) and R ( 0, 2, 1) is … . (1983, 2M)
2 4
π 3π 22. The area of the triangle whose vertices are
(c) (d)
6 4 A(1,–1, 2), B( 2, 1,–1) C( 3,–1, 2) is … . (1983, 2M)
→ →
15. Let a and b be two non-collinear unit vectors. If True/False
→ → → → → → → → →
u = a − ( a ⋅ b ) b and v = a × b , then|v| is (1999, 3M) → → → → → → →
→ → → →
23. Let A, B and C be unit vectors. If A⋅ B = A⋅ C = 0 and
(a)|u| (b)|u|+|u ⋅ a| → →
→ → → → → → →
that the angle between B and C is π / 6.
(c)|u|+|u ⋅ b| (d)|u|+ u ⋅ (a + b) → → →
Then, A = ± 2 ( B × C). (1981, 2M)
552 Vectors

Analytical & Descriptive Questions 26. If A, B, C , D are any four points in space, then prove
→ → → →
→ → → → → →
24. If a , b , c , d are four distinct vectors satisfying the that|AB × CD + BC × AD + CA × BD|
→ → → → → → → → = 4 (area of ∆ ABC). (1987, 2M)
conditions a × b = c × d and a × c = b × d , then
→ → → → → → → → 27. If A1 , A2 , …, An are the vertices of a regular plane
prove that a⋅ b + c⋅ d ≠ a ⋅ c + b ⋅ d. (2004, 2M)
polygon with n sides and O is its centre. Then, show
→ →
25. For any two vectors u and v, prove that that
n −1
→ → → → → → → → → →
(i)|u ⋅ v|2 +|u × v|2 =|u|2|v|2
→ → → → → → → →
∑ (OA i × OA i + 1) = (1 − n ) (OA 2 × OA 1).
i =1
(ii) (1 +|u|2 )(1 +|v|2 ) =|1 − u ⋅ v|2 +|u + v + ( u × v )|2 (1982, 2M)

(1998, 8M)

Topic 3 Scalar Triple Product/Dot Product/Mixed Product


Objective Questions I (Only one correct option) 6. The value of a, so that the volume of parallelopiped
1. The sum of the distinct real values of µ, for which the formed by $i + a $j + k$ , $j + a k$ and a $i + k$ become
$ i$ + µ$j + k
vectors, µ$i + $j + k, $ , $i + $j + µk$ are coplanar, minimum, is (2003, 1M)
is (2019 Main, 12 Jan I) (a) − 3 (b) 3 (c) 1 / 3 (d) 3
(a) 2 (b) 0 → → → → → → → →
7. If V = 2 i + j − k and W = i + 3 k. If U is a unit
(c) 1 (d) − 1
$, $ vector, then the maximum value of the scalar triple
2. Let a = i$ + 2$j + 4k b = i$ + λ$j + 4k and → → →
product [U V W ] is (2002, 1M)
$ $ $
c = 2i + 4 j + ( λ − 1) k be coplanar vectors. Then, the
2
(a) −1 (b) 10 + 6
non-zero vector a × c is (2019 Main, 11 Jan I)
(c) 59 (d) 60
(a) − 10 $i + 5$j
→ →
(b) − 10 $i − 5$j 8. If a = $i − k$ , b = x $i + $j + (1 − x ) k$ and
→ → → →
(c) − 14 $i − 5$j c = y $i + x$j + (1 + x − y ) k$ . Then, [a b c ] depends on
(d) − 14 $i + 5$j (a) only x (b) only y (2001, 2M)
→ → → → (c) neither x nor y (d) both x and y
3. If a , b , c and d are the unit vectors such that → → →
→ → → → → →
1 9. If a , b and c are unit coplanar vectors, then the
( a × b ) ⋅ ( c × d ) = 1 and a ⋅ c = , then (2009) → → → → → →
2 scalar triple product [ 2 a − b 2 b − c 2c − a ] is
→ → →
(a) a , b, c are non-coplanar (2000, 2M)
→ → → (a) 0 (b) 1
(b) a , b, d are non-coplanar (c) − 3 (d) 3
→ →
(c) b, d are non-parallel → → →
→ → → →
10. For three vectors u , v , w which of the following
(d) a , d are parallel and b, c are parallel expressions is not equal to any of the remaining
three?
4. The edges of a parallelopiped are of unit length and → → → → → →
(a) u ⋅ ( v × w ) (b) ( v × w ) ⋅ u (1998, 2M)
are parallel to non-coplanar unit vector a$ , $b, c$ such
→ → → → → →
$ ⋅ c$ = c$ ⋅ a$ = 1 . Then, the volume of the
$=b (c) v ⋅ (u × w ) (d) (u × v ) ⋅ w
that a$ ⋅ b
2 → → →
parallelopiped is (2008, 3M) 11. If a = $i + $j + k$ , b = 4$i + 3$j + 4 k$ and c = $i + α $j + β k$
1 1 →
(a) cu unit (b) cu unit are linearly dependent vectors and| c| = 3 , then
2 2 2
3 1 (a) α = 1, β = − 1 (b) α = 1, β = ± 1 (1998, 2M)
(c) cu unit (d) cu unit (c) α = − 1, β = ± 1 (d) α = ± 1, β = 1
2 3
→ → →
5. The number of distinct real values of λ, for which the 12. If a , b and c are three non-coplanar vectors, then
→ → → → → → →
vectors − λ 2 $i + $j + k$ , i$ − λ 2 $j + k$ and $i + $j − λ 2 k$ are ( a + b + c ) ⋅ [( a + b ) × ( a + c )] equals (1995, 2M)
coplanar, is (2007, 3M) →→→
(a) 0 (b) [ c b c ]
(a) 0 (b) 1 →→→ →→→
(c) 2 (d) 3 (c) 2⋅ [ c b c ] (d) [a b c ]
Vectors 553

→ → → →
13. Let a = $i − $j , b = $j − k$ , c = k$ − i$ . If d is a unit vector Objective Questions II
→ → →→→ → (One or more than one correct option)
such that a⋅ d = 0 = [b c d ] , then d equals (1995, 2M)
19. Let u = u1$i + u 2$j + u3 k$ be a unit vector in R3 and
$i + $j − 2 k$ $i + $j − k$
(a) ± (b) ± 1 $ $ $ ). Given that there exists a vector v
6 3 w= (i + j + 2k
6
$i + $j + k$
(c) ± (d) ± k$ in R3 , such that|u + v| = 1 and w ⋅ (u + v) = 1.(2016 Adv.)
3
Which of the following statement(s) is/are correct?
14. Let a , b, c be distinct non-negative numbers. If the
(a) There is exactly one choice for such v
vectors a $i + a $j + c k$ , $i + k$ and c $i + c $j + bk$ lie in a (b) There are infinitely many choices for such v
plane, then c is (1993, 1M) (c) If u$ lies in the XY-plane, then|u1| = |u2|
(a) the arithmetic mean of a and b (d) If u$ lies in the XY-plane, then 2|u1| = |u3|
(b) the geometric mean of a and b
20. Which of the following expressions are meaningfull
(c) the harmonic mean of a and b
operations? (1998, 2M)
(d) equal to zero → → →
→ →
(a) u⋅ ( v × w )
15. Let a = a1$i + a2$j + a3 k$ , a = b1$i + b2$j + b3 k$ and → → →
(b) (u ⋅ v )⋅ w

a = c1$i + c2 $j + c3 k$ be three non-zero vectors such → → →
(c) (u ⋅ v ) w

→ → →
that c is a unit vector perpendicular to both the (d) u × ( v ⋅ w )
→ → → → π
vectors c and b. If the angle between a and b is ,
6 Fill in the Blanks
2
a1 a2 a3 → → → → → → →
21. Let OA = a , OB = 10 a + 2 b and OC = b, where O, A
then b1 b2 b3 is equal to
and C are non-collinear points. Let p denotes the
c1 c2 c3 (1986, 2M) area of the quadrilateral OABC and let q denotes,
(a) 0 the area of the parallelogram with OA and OC as
(b) 1 adjacent sides. If p = kq, then k = …… . (1997, 2M)
1 $ $ $ $ $ $
(c) (a12 + a22 + a32 ) (b12 + b22 + b32 ) 22. If the vectors ai + j + k , i + bj + k and
4
3 i$ + $j + ck$ ( a ≠ b ≠ c ≠ 1) are coplanar, then the value
(d) (a12 + a22 + a32 ) (b12 + b22 + b32 ) (c12 + c22 + c32 )
4 1 1 1
of + + = …… . (1987, 2M)
16. The volume of the parallelopiped whose sides are (1 − a ) (1 − b) (1 − c)
→ → → → →
given by OA = 2i$ − 3$j, OB = $i + $j − k$ , 23. If A , B , C are three non-coplanar vectors, then
→ → → → → → →
OC = 3$i − k$ , is (1983, 1M) A ⋅( B × C) B ⋅( A × C )
4 + = …. (1985, 2M)
(a) (b) 4 → → → → → →
13 ( C × A )⋅ B C ⋅( A × B )
2
(c) (d) None of these
7  a a 2 1 + a3 
 b b2 1 + b3 
24. If   = 0 and the vectors
2 3
→ → → → → →
17. For non-zero vectors a , b , c|,( a × b ) ⋅ c| 
c c 1

You might also like